0% found this document useful (0 votes)
38 views1,475 pages

4 Pathology

Copyright
© © All Rights Reserved
We take content rights seriously. If you suspect this is your content, claim it here.
Available Formats
Download as PDF, TXT or read online on Scribd
0% found this document useful (0 votes)
38 views1,475 pages

4 Pathology

Copyright
© © All Rights Reserved
We take content rights seriously. If you suspect this is your content, claim it here.
Available Formats
Download as PDF, TXT or read online on Scribd
You are on page 1/ 1475

Index

Cell Injury - Page 1


Newer Cell Deaths - Page 13
Reversible & Irreversible Cell Injury - Page 32
Apoptosis. - Page 48
Intracellular Accumulations, Autophagy and Free Radical Injury - Page 60
Previous Year Questions (Cell Injury) - Page 83
Acute Inflammation - Page 104
Inflammation - Mediators - Page 131
Chronic Inflammation, Wound Healing and Granuloma - Page 157
Previous Year Questions (Inflammation) - Page 169
Introduction and Mendelian Disorders - Page 182
Non Mendelian and Chromosomal Disorders - Page 212
Previous Year Questions (Genetics) - Page 267
Classification of Neoplasms & Features of Neoplasia - Page 289
Fundamentals of Neoplasia - Page 305
Etiology of Neoplasia - Page 327
Previous Year Questions (Neoplasia) - Page 340
Types of Immunity - Page 362
Types of Hypersensitivity Reactions - Page 382
Tolerance and Autoimmune Disorder - Page 398
Immunodeficiency Disorders - Page 419
Transplant Immunology - Page 431
Amyloidosis - Page 442
Previous Year Questions (Immunity ) - Page 450
Vasculitis - Page 483
Myocardial Infarction,Angina Pectoris,Cardiac Enzymes - Page 525
Cardiomyopathies and Cardiac Tumors. - Page 545
Infective Endocarditis & Rheumatic Fever - Page 565
Previous Year Questions (CVS, Blood Vessels and Vasculitis) - Page 616
Bronchial Asthma,Emphysema,Chronic Bronchitis - Page 633
Pulmonary Infections ( Tuberculosis,Pneumonia) - Page 643
Pneumoconiosis,Sarcoidosis,Asbestosis - Page 651
Previous Year Questions (Respiratory System) - Page 661
Barrett's Esophagus, Plummer Vinson syndrome, Esophageal Tumors - Page 682
Peptic Ulcer Disease,Gastric Polyps & Tumors - Page 701
Malabsorption Syndrome,Inflammatory Bowel Disease,Intestinal Carcinoma - Page 724
Index
Previous Year Questions (Gastrointestinal Tract) - Page 763
Salivary Glands - Page 780
Hyperbilirubinemia - Page 795
Hepatitis,Alcoholic Liver Disease,Hemochromatosis - Page 802
Hepatic Tumors - Page 819
Previous Year Questions (Liver, Biliary System and Pancreas) - Page 822
Cystic Diseases of Kidney - Page 836
Crystals & Casts - Page 851
Renal Tumors - Page 859
Glomerular Diseases - Page 868
Previous Year Questions (Renal System) - Page 901
Female Genital System - Page 917
Male Genital System - Page 943
Breast Tumor - Page 965
Inflammatory Lesions of Breast - Page 987
Basics of CNS and Brain Tumors - Page 1011
Thyroid Gland Pathology - Page 1033
Parathyroid Gland Pathology - Page 1053
Adrenal and Pituitary Gland Pathology - Page 1061
Previous Year Questions (Endocrinology) - Page 1079
Diagnosis of Genetic Disorders & Miscellaneous - Page 1089
Diagnosis of Tumors and Paraneoplastic Syndromes - Page 1117
Pulmonary Hypertension,Lung Tumors - Page 1128
Previous Year Questions (Central Nervous System) - Page 1148
Skin, Bones and Joints - Page 1155
Previous Year Questions (Musculoskeletal Disorders) - Page 1175
Introduction to Hematology, RBC Inclusions and Shapes - Page 1186
Microcytic Hypochromic Anemia - Page 1208
Macrocytic Hypochromic Anemia - Page 1224
Hemolytic Anemias - Page 1235
Sickle Cell Anemia and Thalassemia - Page 1252
Basics of WBC and Flow Cytometry - Page 1267
ALL & AML - Page 1280
CLL & CML - Page 1287
Chronic Myeloproliferative Disorders and Myelodysplastic Syndromes - Page 1299
Plasma Cell Dyscrasia - Page 1307
Index
Hodgkin's & Non- Hodgkin's Lymphoma - Page 1319
Platelets - Page 1336
Previous Year Questions (Platelet and Bleeding Disorders) - Page 1348
Blood Banking and Blood Transfusion Medicine - Page 1368
Tissue Processing - Page 1391
Previous Year Questions (Hematology Red Blood Cells) - Page 1398
Previous Year Questions (Hematology White Blood Cells) - Page 1438
Cell Injury
1. Which of the following is true regarding cell injury induced by hypoxia?
A. Hypoxia is a less common cause of cell injury and cell death
B. It may be caused by ischemia, cardiorespiratory failure, carbon monoxide poisoning and severe
blood loss
C. Increased aerobic respiration
D. Sudden and severe hypoxia causes atrophy
----------------------------------------
2. Which of the following cells are most sensitive to hypoxia?
A. Myocardial cells
B. Neurons
C. Hepatocytes
D. Renal tubular epithelial cells
----------------------------------------
3. Which of the following morphologic changes to partial ischemia is most likely to occur in right renal
artery stenosis?
A. Atrophy
B. Dysplasia
C. Metaplasia
D. Hypertrophy
----------------------------------------
4. Please select the option that has been proven to cause hypoxia.
A. Ischemia
B. Respiratory failure
C. Carbon monoxide poisoning
D. All of the above
----------------------------------------
5. Assertion (A): Ischemia tends to cause more rapid and severe tissue injury than hypoxia in the
absence of ischemia Reason (R): In ischemic tissues, aerobic metabolism is compromised, and
anaerobic energy generation also stops. Which of the following statements is correct?
A. Both A and R are true, and R is the correct explanation of A
B. Both A and R are true, but R is not a correct explanation of A
C. A is true, but R is false
D. A is false, but R is true
----------------------------------------

1
6. In a patient with a suspected neurodegenerative disorder, which organelles are directly involved in
the process of apoptosis, potentially contributing to the progression of the disease?
A. Cytoplasm
B. Golgi complex
C. Mitochondria
D. Nucleus
----------------------------------------
7. A sixty-nine-year-old male patient is diagnosed with Parkinson's disease, which of the following is not
seen in the neurons of this patient?
A. Nuclear fragmentation
B. Cellular shrinkage
C. Inflammation
D. Alteration in cellular membrane structure
----------------------------------------
8. What is the role of the CD95 cell marker?
A. cellular hyperplasia
B. Extrinsic pathway of apoptosis
C. Necrosis of cell
D. Cellular adaption
----------------------------------------
9. Which of the following is a feature of caspases?
A. It is an alanine protease in nature
B. Exists as zymogens and undergo cleavage to get activated
C. Independent of proapoptotic and antiapoptotic proteins
D. Not required for embryogenesis
----------------------------------------
10. Which of the following is an antiapoptotic factor?
A. BCL-2
B. BAX
C. p53
D. Smac
----------------------------------------
11. Match the following types of necrosis with their appropriate description: 1 Coagulative necrosis A
Occurs mostly in the CNS 2 Colliquative necrosis B Associated with antigen-antibody reactions 3 Fat
necrosis C Grossly -chalky white appearance 4 Fibrinoid necrosis D Microscopic findings-Ghost cells
1 Coagulative necrosis A Occurs mostly in the CNS
2 Colliquative necrosis B Associated with antigen-antibody reactions

Page 2

2
3 Fat necrosis C Grossly -chalky white appearance
4 Fibrinoid necrosis D Microscopic findings-Ghost cells

A. 1-D, 2-A, 3-C, 4-B


B. 1-C, 2-D, 3-B, 4-A
C. 1-A, 2-D, 3-B, 4-C
D. 1-B, 2-A, 3-D, 4-C
----------------------------------------
12. A 45-year-old male presents to the emergency department with severe abdominal pain radiating to
his back, accompanied by nausea and vomiting. Laboratory tests reveal elevated serum amylase and
lipase levels. Imaging studies demonstrate swollen and inflamed pancreas. Which of the following is
likely to be seen in the pancreatic tissue?
A. Coagulative necrosis
B. Zenker’s degeneration
C. Liquefactive necrosis
D. Caseous necrosis
----------------------------------------

Correct Answers
Question Correct Answer

Question 1 2
Question 2 2
Question 3 1
Question 4 4
Question 5 1
Question 6 3
Question 7 3
Question 8 2
Question 9 2
Question 10 1
Question 11 1
Question 12 3

Solution for Question 1:


Correct Option B - It may be caused by ischemia, cardiorespiratory failure, carbon monoxide poisoning
and severe blood loss:
• Sudden and severe hypoxia results in cell injury and death, while slow, prolonged hypoxia leads to
atrophy. Example: tissue supplied by a narrowed artery undergoes atrophy/ shrinkage.

Page 3

3
• A deficiency of oxygen causes cell injury by reducing aerobic oxidative respiration.
• An extremely important and common cause of cell injury and cell death.
Causes:
• Reduced blood flow (ischemia)
• Cardiorespiratory failure
• Anaemia or carbon monoxide poisoning or after severe blood loss (Decreased oxygen-carrying
capacity of the blood).
Incorrect Options:
Option A - Hypoxia is a less common cause of cell injury and cell death:
• Hypoxia is the most common cause of cell injury and cell death
• Therefore this option is incorrect
Option C - Increased aerobic respiration:
• In the case of hypoxia, cells go into anaerobic respiration/metabolism, elevating lactate levels. Or
there is decreased aerobic respiration in hypoxia.
• Therefore this option is incorrect
Option D - Sudden and severe hypoxia causes atrophy:
• Chronic hypoxia causes atrophy of the cells.
• Therefore this option is incorrect

Solution for Question 2:


Correct Option B - Neurons:
• Neurons undergo irreversible damage within 3 to 4 minutes following deprivation of blood supply
• Fibroblasts within the myocardium remain viable even after many hours of ischemia
• The most sensitive neurons in the brain are pyramidal neurons in the hippocampus (especially area
CA1, also referred to as the Sommer sector), cerebellar Purkinje cells, and pyramidal neurons in the
cerebral cortex (especially layers III and V)
Incorrect Options:
Option A, C, and D - Myocardial cells, Hepatocytes and Renal tubular epithelial cells are less sensitive
to hypoxia than neurons

Solution for Question 3:


Correct Option A - Atrophy:
• Ischemia is a condition in which blood flow to tissues is disrupted.
• Infarction occurs when there is total ischemia.

Page 4

4
• Partial ischemia occurs when an artery is partially occluded or when collateral circulation is
inadequate. This causes a continuous reduction in oxygen delivery, which is typically compatible with
cell survival.
• Cell atrophy is prevalent in these situations.
• It's common at the insufficiently perfused margins of infarcts in the heart, brain, and kidneys.
• None of the other options refers to a reduction in organ size and function.
Incorrect Options:
Option B - Dysplasia:
• Dysplasia is the abnormal growth or development of cells or organs
• Examples are cervical dysplasia, bronchopulmonary dysplasia, etc.
Option C - Metaplasia:
• Metaplasia is the transformation of one cell type to another cell type.
• Types are intestinal metaplasia and squamous metaplasia.
Option D - Hypertrophy:
• Hypertrophy is an increase in the size of cells that increases the size of the affected organ.
• For example, muscle hypertrophy after weight lifting or exercise.
• Whereas hyperplasia, in which cells remain approximately the same size but increase in number.
• Hyperplasia and hypertrophy can occur together, such as in the case of the hormonally induced
proliferation and enlargement of the uterus during pregnancy.

Solution for Question 4:


Correct Option D - All of the above:
• Hypoxia is an important and common cause of cell injury and death.
Causes of hypoxia include:
• Reduced blood flow (ischemia)
• Inadequate oxygenation of the blood due to cardiorespiratory failure
• The decreased oxygen-carrying capacity of the blood, as in anemia or carbon monoxide poisoning
(producing a stable Carboxyhemoglobin that blocks oxygen carriage) or after severe blood loss.
Incorrect Options:
Option A - Ischemia:
• Ischemia is a condition in which the blood flow is restricted in a part of the body.
• This is the major cause of hypoxia
Option B - Respiratory failure:
• When respiratory failure develops, the lungs can't get enough oxygen into the blood resulting in
hypoxia

Page 5

5
• It is a serious condition that causes carbon dioxide retention inside the body and causes respiratory
acidosis.
Option C - Carbon monoxide poisoning:
• Carbon monoxide poisoning causes blockage of the mitochondrial cytochrome system, which causes
a lack of ATP production and results in hypoxia.
• This is cytotoxic hypoxia.

Solution for Question 5:


Correct Option A - Both A and R are true, and R is the correct explanation of A:
• Ischemia is the most prevalent type of cell injury.
• Restricted blood flow most commonly from arterial obstruction causes hypoxia in cells.
• Reduced venous drainage can also be the cause.
• In contrast to hypoxia, during which anaerobic glycolysis continues to produce energy, ischemia
impairs the delivery of glycolysis substrates.
• As a result, aerobic metabolism is limited in ischemic tissues, and anaerobic energy generation also
ceases when glycolytic substrates are depleted, or glycolysis is blocked due to the accumulation of
metabolites that would otherwise be washed away by circulating blood.
• For this reason, ischemia tends to cause more rapid and severe cell and tissue injury than hypoxia
alone.
Incorrect Options:
Options B, C and D are incorrect.

Solution for Question 6:


Correct Option C- Mitochondria:
• Apoptosis is the programmed cell death that occurs in multicellular organisms.
• It is characterized by the following: Blebbing Cell shrinkage Nuclear fragmentation Chromatin
condensation DNA fragmentation mRNA decay
• Blebbing
• Cell shrinkage
• Nuclear fragmentation
• Chromatin condensation
• DNA fragmentation
• mRNA decay
• Apoptosis is a highly regulated and controlled process.
• It can be initiated through: Intrinsic pathway Extrinsic pathway

Page 6

6
• Intrinsic pathway
• Extrinsic pathway
• The intrinsic pathway is also known as the mitochondrial pathway.
• Cytochrome c is released from the mitochondria by proteins Bax and Bak.
• Cytochrome c binds with apoptotic protease activating factor-1 (Apaf-1) and ATP.
• This combination further binds to pro-caspase-9 to create a protein complex called an apoptosome.
• This apoptosome cleaves the pro-caspase to its active form, which initiates apoptosis.
• Mitochondria play a central role in the intrinsic pathway of apoptosis.
• Blebbing
• Cell shrinkage
• Nuclear fragmentation
• Chromatin condensation
• DNA fragmentation
• mRNA decay
• Intrinsic pathway
• Extrinsic pathway

Incorrect Options:
Option A- Cytoplasm:
• Caspases are present in the cytoplasm of cells.
• Mitochondria play a central role in the intrinsic pathway of apoptosis.
Option B- Golgi complex:
• Mitochondria play a central role in the intrinsic pathway of apoptosis.
• The Golgi apparatus is not involved in this process.

Page 7

7
Option D- Nucleus:
• Mitochondria play a central role in the intrinsic pathway of apoptosis.

Solution for Question 7:


Correct Option C - Inflammation:
• Apoptosis is the programmed cell death that occurs in multicellular organisms.
• Apoptosis is a highly regulated and controlled process.
• It can be initiated through: Intrinsic pathway Extrinsic pathway
• Intrinsic pathway
• Extrinsic pathway
• The following changes are seen in apoptosis: Reduced cell size (shrinkage) Chromatin condensation
Nuclear fragmentation into nucleosome-size segments The plasma membrane remains intact;
however, there is an alteration in the structure of the membrane (changes in orientation of lipids)
Cytoplasmic blebs Formation of apoptotic bodies, which are rapidly ingested by phagocytes and
degraded by the phagocyte lysosomal enzymes.
• Reduced cell size (shrinkage)
• Chromatin condensation
• Nuclear fragmentation into nucleosome-size segments
• The plasma membrane remains intact; however, there is an alteration in the structure of the
membrane (changes in orientation of lipids)
• Cytoplasmic blebs
• Formation of apoptotic bodies, which are rapidly ingested by phagocytes and degraded by the
phagocyte lysosomal enzymes.
• Inflammation is not seen in the vicinity of apoptotic cells
• It is a feature of necrosis
• Intrinsic pathway
• Extrinsic pathway
• Reduced cell size (shrinkage)
• Chromatin condensation
• Nuclear fragmentation into nucleosome-size segments
• The plasma membrane remains intact; however, there is an alteration in the structure of the
membrane (changes in orientation of lipids)
• Cytoplasmic blebs
• Formation of apoptotic bodies, which are rapidly ingested by phagocytes and degraded by the
phagocyte lysosomal enzymes.
Incorrect Options:
Option A - Nuclear fragmentation:

Page 8

8
• Nuclear fragmentation occurs due to the activity of endonucleases.
• This process is seen in apoptosis.
Option B - Cellular shrinkage:
• There is a reduction in the size of the cell going through apoptosis.
• This process is cellular shrinkage.
Option D - Alteration in cellular membrane structure:
• Even though the plasma membrane of the cell remains intact, there is a chance in the orientation of
the lipids present in the membrane.
• This leads to alterations in the cellular membrane structure.

Solution for Question 8:


Correct Option B - Extrinsic pathway of apoptosis:
• The extrinsic pathway is activated by extracellular ligands binding to cell-surface death receptors,
which leads to the formation of the death-inducing signalling complex (DISC)
• The extrinsic pathway can be activated by: TNF (tumor necrosis factor) pathway Fas (first apoptosis
signal) pathway
• TNF (tumor necrosis factor) pathway
• Fas (first apoptosis signal) pathway
• The fas receptor is also known as Apo-1 or CD95
• CD95 is a transmembrane protein of the TNF family which binds the Fas ligand (FasL)
• This complex results in the formation of a death-inducing signalling complex (DISC), which contains
the FADD (fas-associated death domain), caspase-8 and caspase-10
• Formation of DISC leads to the initiation of apoptosis.
• TNF (tumor necrosis factor) pathway
• Fas (first apoptosis signal) pathway
Incorrect Options:
Option A - cellular hyperplasia
it is an increase in the number of cells in response to stress or physiological need. Eg. hyperplasia of ut
erus. CD95 has no role in hyperplasia of cell.
Option C - Necrosis of cell:
• The premature death of cells due to cell injury is called necrosis.
• Necrosis is caused by external factors, such as infection or trauma, resulting in cellular components'
unregulated destruction.
Option D - Cellular adaption:
• Cellular adaptation is the changes made by a cell in response to any change.
• This adaptation may be physiological or pathological.

Page 9

9
Solution for Question 9:
Correct Option B - Exists as zymogens and undergo cleavage to get activated:
• Caspases are a group of enzymes involved in programmed cell death.
• The activation of caspase enzymes causes apoptosis.
• The stages of apoptosis are as follows: Initiation phase in which some caspases become catalytically
active Execution phase in which other caspases trigger the degradation of critical cellular components
• Initiation phase in which some caspases become catalytically active
• Execution phase in which other caspases trigger the degradation of critical cellular components
• Caspases are inactive proenzymes, or zymogens, that must be cleaved by an enzyme to become
active.
• The presence of cleaved, active caspases is a sign that a cell is about to die.
• Initiation phase in which some caspases become catalytically active
• Execution phase in which other caspases trigger the degradation of critical cellular components
Incorrect Options:
Option A - It is an alanine protease in nature:
• Caspases are cysteine proteases in nature.
• They are also called cysteine-aspartic proteases.
Option C - Independent of proapoptotic and antiapoptotic proteins:
• The activation of caspases depends on a finely tuned balance between the production of proapoptotic
and antiapoptotic proteins.
• Apoptosis is a highly controlled process.
Option D - Not required for embryogenesis:
• Caspases are required for embryogenesis.
• They are involved in the following: Implantation Organogenesis Developmental involution
Metamorphosis
• Implantation
• Organogenesis
• Developmental involution
• Metamorphosis
• Implantation
• Organogenesis
• Developmental involution
• Metamorphosis

Page 10

10
Solution for Question 10:
Correct Option A- BCL-2:
• Bcl-2 is an anti-apoptotic protein that is encoded by BCL-2 gene.
• Overexpression of these proteins causes cancers.
• Pro-Apoptotic Anti-Apoptotic p53 BCL-2 BAX MCL1 Smac BCL XL

Pro-Apoptotic
Anti-Apoptotic
p53
BCL-2
BAX
MCL1
BCL XL
Incorrect Options:
Options B, C and D are pro-apoptotic factors, Underexpression of these proteins causes cancer.

Solution for Question 11:


Correct Option D - 1-D, 2-A, 3-C, 4-B:
• (D): Coagulative necrosis typically occurs in solid organs such as the heart, kidney, and liver. It is
characterized by the formation of multiple ghost cells, leading to the typical tombstone appearance on
histology.
• (A): Colliquative necrosis occurs due to increased hydrolytic enzymatic activity. It is commonly seen in
the CNS and pancreas.
• ( C): Fat necrosis often occurs in adipose tissue, and its gross appearance can be chalky white due to
the deposition of calcium salts.
• (B): Fibrinoid necrosis is commonly associated with immune complex deposition due to
antigen-antibody reaction in diseases such as rheumatic heart disease and polyarteritis nodosa.
Incorrect Options:
Options B, C, and D are incorrectly matched

Solution for Question 12:


Correct Option C - Liquefactive necrosis:

Page 11

11
• In acute pancreatitis, inflammatory processes lead to the release of hydrolytic enzymes that can
digest pancreatic tissue, resulting in liquefaction of the affected area. Therefore, liquefactive necrosis is
more likely to be seen in this patient’s pancreatic tissue.
Incorrect Options:
Option A - Coagulative necrosis:
• Coagulative necrosis is more likely to be seen in the heart, kidney, and liver.
Option B - Zenker’s degeneration:
• Zenker's degeneration, also known as Zenker's necrosis, is a type of coagulative necrosis seen in
cases of enteric fever and it affects the diaphragm and the rectus abdominis muscle.
Option D - Caseous necrosis:
• Caseous necrosis is characterized by a cheese-like appearance of the affected tissue and is
commonly associated with granulomatous inflammation.

Page 12

12
Newer Cell Deaths
1. Which of the following transcription factors is not associated with cellular hypertrophy?
A. GATA 4
B. NFAT
C. MEF 2
D. MLL 1
----------------------------------------
2. Which statement regarding the pathological process underlying abnormal uterine bleeding in a
woman taking tamoxifen for hormone receptor-positive breast cancer is false?
A. The process occurs in labile cells
B. There is an increase in the number of cells.
C. The risk of malignancy is present in certain cases.
D. Bone marrow cannot undergo this process.
----------------------------------------
3. At 38 weeks of pregnancy, a 32-year-old lady with poorly controlled diabetes delivered a healthy
child. Pancreatic islets in the neonate would be expected to demonstrate which of the following
morphological responses to injury resulting from maternal hyperglycemia during pregnancy?
A. Atrophy
B. Dysplasia
C. Hyperplasia
D. Metaplasia
----------------------------------------
4. A 54-year-old woman lost her right kidney following an automobile accident. A CT scan of her
abdomen 2 years later shows a marked enlargement of her left kidney. This is an example of which of
the following adaptations?
A. Atrophy
B. Dysplasia
C. Hyperplasia
D. Hypertrophy
----------------------------------------
5. What adaptation occurs in a 42-year-old smoker with COPD, presenting with dyspnea, tachycardia,
and stratified squamous cells with underlying glands on bronchial biopsy?
A. Metaplasia
B. Anaplasia
C. Dysplasia
D. Neoplasia
----------------------------------------

13
6. Which of the following terms best reflects the morphologic reaction seen in Barrett's esophagus due
to prolonged injury?

A. Atypical hyperplasia
B. Complex hyperplasia
C. Glandular metaplasia
D. Simple hyperplasia
----------------------------------------
7. Which of the following represents the physiological adaptation of a female breast at puberty?
A. Hypertrophy
B. Hyperplasia
C. Hypertrophy and hyperplasia
D. Atrophy
----------------------------------------
8. Which of the following statements is incorrect regarding hypertrophy?
A. Occurs due to the synthesis and assembly of additional intracellular components.
B. There is an increase in the size of the cells.
C. Cells incapable of division respond to stress by hypertrophy.
D. There is an increase in the number of cells.
----------------------------------------
9. Which of the following tissues are capable of undergoing both hypertrophy and hyperplasia?
A. Breast enlargement during lactation
B. Uterus during pregnancy
C. Skeletal muscle enlargement during exercise
D. Heart muscle due to hypertension
----------------------------------------
10. In the presence of recombinant erythropoietin, bone marrow cells from an organ donor are grown in
vitro at 37°C. Which of the following physiologic adaptations is seen?
A. Atrophy

Page 2

14
B. Dysplasia
C. Hyperplasia
D. Metaplasia
----------------------------------------
11. Which of the following does not represent the example of smooth muscle hypertrophy as an
adaptive response?
A. Urinary bladder in urine outflow obstruction
B. Gall bladder in chronic cholecystitis
C. Triceps enlargement in body-builders
D. Uterus enlargement during pregnancy
----------------------------------------
12. Which of the following best describes the morphology in a bronchial mucosal lesion with
preneoplastic alterations?
A. Cellular maturation shows an abnormal pattern
B. Increased numbers of otherwise normal cells
C. Invasiveness
D. Presence of poorly differentiated keratin-producing cells
----------------------------------------
13. Which cytokine is released during acute cholecystitis due to inflammasome activation in a
47-year-old female with leukocytosis and pain in the right hypochondrium?
A. TNFα
B. IL-1β
C. IFNα
D. IFNγ
----------------------------------------
14. Which of the following processes leads to inflammasome activation by activating caspase 1 in cells
infected by certain microbes, ultimately leading to cell death?
A. Necrosis
B. Necroptosis
C. Apoptosis
D. Pyroptosis
----------------------------------------
15. Which of the following is not a part of programmed necrosis?
A. TNF binds to TNF receptor
B. RIPK-1, RIPK-3 and procaspase 8 form a complex
C. Procaspase 8 activation
D. MLKL phosphorylation

Page 3

15
----------------------------------------

Correct Answers
Question Correct Answer

Question 1 4
Question 2 4
Question 3 3
Question 4 4
Question 5 1
Question 6 3
Question 7 3
Question 8 4
Question 9 2
Question 10 3
Question 11 3
Question 12 1
Question 13 2
Question 14 4
Question 15 3

Solution for Question 1:


Correct Option D- MLL1:
• The patient presents with dyspnea, diaphoresis, palpitations, and an ejection fraction of 40%, which
are cardinal features of cardiac failure.
• The patient also had a history of uncontrolled hypertension which led to left ventricular hypertrophy
with concomitant heart failure.
• Signalling pathways that trigger hypertrophy activate a set of transcription factors such as: GATA 4
Nuclear factor of activated T-cells (NFAT) Myocyte enhancer factor-2 (MEF2)
• GATA 4
• Nuclear factor of activated T-cells (NFAT)
• Myocyte enhancer factor-2 (MEF2)
• These transcription factors coordinate to increase the synthesis of muscle proteins responsible for
hypertrophy.
• MLL 1 (Mixed lineage leukemia protein-1) is a gene in acute leukemia.
• GATA 4
• Nuclear factor of activated T-cells (NFAT)
• Myocyte enhancer factor-2 (MEF2)

Page 4

16
Incorrect Options:
Option A- GATA 4:
• It is a family of zinc-finger transcription factors that promote cardiomyocyte survival and cardiac
morphogenesis and maintain cardiac function in the adult heart.
Option B- NFAT:
• It is found mostly in immune-system cells, which play a role in the transcription of cytokine and other
genes responsible for the immune response.
• It is involved in cardiac, skeletal, and nerve development.
Option C- Myocyte enhancer factor-2 (MEF2):
• MEF2 codes for a transcription factor involved in the normal development of the following Body's
vascular blood flow Heart Craniofacial Brain Immune systems.
• Body's vascular blood flow
• Heart
• Craniofacial
• Brain
• Immune systems.
• Body's vascular blood flow
• Heart
• Craniofacial
• Brain
• Immune systems.

Solution for Question 2:


Correct Option D- Bone marrow cannot undergo this process:
• The pathological process responsible for abnormal uterine bleeding in this patient, who is taking
tamoxifen for hormone receptor-positive breast cancer, primarily involves the endometrium. Tamoxifen,
a selective estrogen receptor modulator (SERM), can have estrogenic effects on the endometrium,
leading to changes such as endometrial hyperplasia
• Hyperplasia occurs in labile cells (capable of dividing).
• It is an increase in the number of cells of an organ or tissue in response to stimulus or stress.
• Hyperplasia can be physiologic or pathologic.
• Most forms of pathologic hyperplasia are caused by excessive or inappropriate actions of hormones
or growth factors acting on target cells.
• Pathologic hyperplasia may eventually lead to cancerous proliferation.
• Endometrial hyperplasia is an example of abnormal hormone-induced hyperplasia, and patients with
endometrial hyperplasia are at increased risk for developing endometrial cancer.

Page 5

17
• Bone marrow can undergo rapid hyperplasia in response to a deficiency of terminally
differentiated/mature blood cells (anemia, leucopenia, and/or thrombocytopenia).
Incorrect Options:
Option A- The process occurs in labile cells:
• Hyperplasia occurs in labile cells, which replace the cells lost from the body by dividing continuously.
• Examples of labile cells include Epidermis Epithelia of ducts Hematopoietic stem cells.
• Epidermis
• Epithelia of ducts
• Hematopoietic stem cells.
• Epidermis
• Epithelia of ducts
• Hematopoietic stem cells.
Option B- There is an increase in the number of cells:
• The number of cells is increased in cellular hyperplasia.
• It is an increase in the number of cells of an organ or tissue in response to stimulus or stress.
Option C: Risk of malignancy is present in certain cases:
• Pathological hyperplasia can be a risk factor for malignancy.
• An abnormal stressor may lead to uncontrollable divisions, which result in malignancies.

Solution for Question 3:


Correct Option C- Hyperplasia:
• Infants born to diabetic moms have a 6% to 10% chance of developing serious developmental
abnormalities such as heart and great vascular malformations and neural tube problems.
• The islet cells of the pancreas have a proliferative capacity during fetal development, and they
respond to increasing demand for insulin by physiologic hyperplasia.
• A fetus may develop pancreatic hyperplasia, which can produce insulin and induce hypoglycemia at
birth.
Incorrect Options:
Option A- Atrophy:
• Atrophy is a decrease in the size of a cell or organ
• This can be due to the disuse of your muscles in neurological conditions.
Option B- Dysplasia:
• Dysplasia is the abnormal growth or development of cells or organs
• Dysplastic cells may exhibit considerable pleomorphism and often contain large hyperchromatic nuclei
with a high nuclear-to-cytoplasmic ratio. Dysplastic epithelial surfaces also typically show architectural
disarray and a loss of orderly differentiation

Page 6

18
• Examples are cervical dysplasia, bronchopulmonary dysplasia, etc.
Option D- Metaplasia:
• Change in phenotype of differentiated cells in response to chronic irritation, that makes cells better
able to withstand the stress.
• Usually induced by altered differentiation pathway of tissue stem cells
• May result in reduced functions or increased propensity for malignant transformation
• Types are intestinal metaplasia and squamous metaplasia.

Solution for Question 4:


Correct Option D- Hypertrophy:
• Hypertrophy is an increase in the size of cells that increase the size of the affected organ.
• It is caused by increased functional demand or stimulation by hormones and growth factors.
Incorrect Options:
Option A- Atrophy:
• Atrophy: decreased cell and organ size, as a result of decreased nutrient supply or disuse;
• Associated with decreased synthesis of cellular components and increased breakdown of organelles
by increased autophagy.
• For example muscle atrophy due to disuse of muscle
Option B- Dysplasia:
• Dysplasia is an abnormal and potentially reversible process where there is the organized growth and
maturation of cells.
Option C- Hyperplasia:
• Hyperplasia is an increase in the number of cells in response to stress in labile cells.
• There are two types of hyperplasia: Physiologic hyperplasia: Occurs in response to a normal stressor.
Examples include the epidermis, epithelia of ducts, and hematopoietic stem cells. Pathologic
hyperplasia: Occurs in response to an abnormal stressor. Examples include the growth of adrenal
glands due to excessive adrenocorticotropic hormone (ACTH) production by a pituitary adenoma and
the proliferation of endometrium due to prolonged estrogen stimulus.
• Physiologic hyperplasia: Occurs in response to a normal stressor. Examples include the epidermis,
epithelia of ducts, and hematopoietic stem cells.
• Pathologic hyperplasia: Occurs in response to an abnormal stressor. Examples include the growth of
adrenal glands due to excessive adrenocorticotropic hormone (ACTH) production by a pituitary
adenoma and the proliferation of endometrium due to prolonged estrogen stimulus.
• Physiologic hyperplasia: Occurs in response to a normal stressor. Examples include the epidermis,
epithelia of ducts, and hematopoietic stem cells.
• Pathologic hyperplasia: Occurs in response to an abnormal stressor. Examples include the growth of
adrenal glands due to excessive adrenocorticotropic hormone (ACTH) production by a pituitary
adenoma and the proliferation of endometrium due to prolonged estrogen stimulus.

Page 7

19
Solution for Question 5:
Correct Option A- Metaplasia:
Metaplasia
• Normally in bronchi →pseudostratified ciliated columnar epithelium is present.
• In a chronic smoker, this is replaced by →stratified squamous epithelium.
• This type of change in the epithelium is called metaplasia.
• The most common epithelial metaplasia is columnar to squamous, as seen in smokers
• Metaplasia is a benign, reversible condition.
Incorrect Options:
Option B- Anaplasia:
• The loss of mature or specialized features of a cell or tissue, for example, a malignant tumor.
• Grading is done depending on the degree of anaplasia.
• A lack of differentiation is called Anaplasia (a hallmark of malignancy).
• Features of Anaplasia: pleomorphism, tumor giant cells, atypical mitoses, loss of polarity
Option C- Dysplasia:
• Dysplasia means “disordered growth.”
• Dysplastic cells may exhibit considerable pleomorphism and often contain large hyperchromatic nuclei
with a high nuclear-to-cytoplasmic ratio
• These abnormal types of cells are present in a tissue in the pre-cancerous stage.
Option D- Neoplasia:
• Neoplasia is a new uncontrolled growth of cells that is not physiological but pathological.
• When dysplastic changes are marked and involve the full thickness of the epithelium but the lesion
does not penetrate the basement membrane; it is considered a preinvasive neoplasm.

Solution for Question 6:


Correct Option C- Glandular metaplasia:
• The major adaptive responses of cells to sublethal injury are atrophy, hypertrophy, hyperplasia,
metaplasia, dysplasia, and intracellular storage.
• Metaplasia is defined as the conversion of one cell type to another.
• Barrett's oesophagus is the most well-known example of metaplasia, where the oesophagal
squamous epithelium is replaced by intestinal columnar epithelium.
• Histologically it is recognized by the presence of non-keratinized columnar epithelial cells alongside
mucous-producing goblet cells.

Page 8

20
• Barrett's oesophagus is the response of the oesophagal epithelium to continuous acidity caused by
gastroesophageal reflux.

• Choices A, B, and D are preneoplastic changes that are most often described in the uterine
endometrium of postmenopausal women.
• Other examples of metaplasia include: Metaplasia in bronchi from glandular to the squamous
epithelium in response to smoking Schistosoma haematobium infection leads to metaplasia from
transitional to the squamous epithelium in the urinary bladder. Myositis ossificans, which is defined as
mesenchymal metaplasia. In this condition, bone is formed within the muscle tissue.
• Metaplasia in bronchi from glandular to the squamous epithelium in response to smoking
• Schistosoma haematobium infection leads to metaplasia from transitional to the squamous epithelium
in the urinary bladder.
• Myositis ossificans, which is defined as mesenchymal metaplasia. In this condition, bone is formed
within the muscle tissue.
• Metaplasia in bronchi from glandular to the squamous epithelium in response to smoking
• Schistosoma haematobium infection leads to metaplasia from transitional to the squamous epithelium
in the urinary bladder.
• Myositis ossificans, which is defined as mesenchymal metaplasia. In this condition, bone is formed
within the muscle tissue.
Incorrect Options:
Option A-Atypical hyperplasia:
• Atypical hyperplasia is a clonal proliferation having some, but not all, of the histologic features of
carcinoma in situ.
Option B- Complex hyperplasia:
• Complex hyperplasia is a precancerous condition.
• It increases the risk of endometrial cancer if left untreated.
Option D- Simple hyperplasia:

Page 9

21
• Simple hyperplasia is defined as the proliferation of endometrial cells without atypia.
• This condition may improve without treatment.

Solution for Question 7:


Correct Option C- Hypertrophy and hyperplasia:
• Hormonal hyperplasia is well demonstrated by the proliferation of the glandular epithelium of the
female breast at puberty and during pregnancy.
• It is usually accompanied by the enlargement (hypertrophy) of the glandular epithelial cells.
• Simultaneous Hypertrophy and Hyperplasia occur in the female breast at puberty.
Incorrect Options:
Option A- Hypertrophy:
• Hypertrophy is an increase in the size of the cell.
• Hyperplasia and hypertrophy are both involved in breast development.
Option B- Hyperplasia:
• Hyperplasia is an increase in the number of cells.
• Hyperplasia and hypertrophy are both involved in breast development.
Option D- Atrophy:
• Atrophy is a decrease in the size of the cell due to increased long-term stress.
• For example, muscle atrophy due to disuse of the muscle

Solution for Question 8:


Correct Option D- There is an increase in the number of cells:
• Hypertrophy is a condition in which cells grow in size, increasing the size of the affected organ.
• Hyperplasia is a condition in which there is an increase in the number of cells
• Stress can cause hypertrophy in cells with limited ability to divide, while it causes hyperplasia in cells
that can divide.
• Hypertrophy and hyperplasia can occur together, leading to an enlarged organ.
• A limit is reached beyond which the muscle mass cannot enlarge to compensate for the increased
burden.
• The hypertrophy then changes to regressive/degenerative changes leading to ventricular dilation and
heart failure in the heart.
Incorrect Options:
Option A- Occurs due to the synthesis and assembly of additional intracellular components:
• Hypertrophy involves the synthesis of new proteins and cell components

Page 10

22
• The synthesis and assembly of proteins increase the size of the cells and extra structural components
within the cell.
Option B- There is an increase in the size of the cells:
• In hypertrophy, cell size increases in response to stress
• Hypertrophy is a condition in which cells grow in size, resulting in an increase in the size of the
affected organ.
• Hypertrophy can progress to significant functional cell injury if the stress is not relieved.
Option C- Cells incapable of division respond to stress by hypertrophy:
• Cells respond to stress by either hypertrophy (increase in cell size) and/or hyperplasia (increase in cell
number).
• Cells incapable of division respond by hypertrophy.

Solution for Question 9:


Correct Option B- Uterus during pregnancy:
• Hypertrophy describes the growth in the size of cells in an organ, increasing organ's size.
• It occurs in cells that have limited capacity to divide.
• Examples of hypertrophy include: Skeletal muscle during exercise The heart muscle in cases of
hypertension and aortic stenosis (pathologic hypertrophy) Breast enlargement during lactation
• Skeletal muscle during exercise
• The heart muscle in cases of hypertension and aortic stenosis (pathologic hypertrophy)
• Breast enlargement during lactation
• Hyperplasia is an increase in the number of cells.
• It occurs in labile cells, which can divide and produce new cells.
• Examples of labile cells are the epidermis, epithelia of ducts, liver cells, and hematopoietic stem cells.
• The uterus is unique in a way that it is capable of undergoing both hypertrophy and hyperplasia.
Estrogen stimulates smooth muscle hypertrophy and hyperplasia in the uterus during pregnancy.
• Skeletal muscle during exercise
• The heart muscle in cases of hypertension and aortic stenosis (pathologic hypertrophy)
• Breast enlargement during lactation
Incorrect Options:
Option A- Breast enlargement during lactation:
• Prolactin and estrogen stimulate breast growth during breastfeeding.
• The proliferation of the glandular epithelium of the female breast during adolescence and pregnancy
is an example of physiologic hormonal hypertrophy and hyperplasia.
• However, during lactation, the breast size increase is only due to hypertrophy.
Option C- Skeletal muscle enlargement during exercise:

Page 11

23
• During exercise, skeletal muscles undergo hypertrophy due to increased stress on the muscles.
• However, hyperplasia doesn’t take place.
• Therefore this option is incorrect
Option D- Heart muscle due to hypertension:
• In hypertension, cells undergo hypertrophy to compensate for the increased demand of pressure
overload.
• However, hyperplasia doesn’t take place.
• Therefore this option is incorrect

Solution for Question 10:


Correct Option C- Hyperplasia:
• Hyperplasia is an increase in the number of cells in an organ or tissue.
• It is often a normal process and is a reaction to trophic cues or increasing functional demand, similar
to hypertrophy.
• For example, people living at high altitudes are more likely to develop erythroid hyperplasia. Low
oxygen tension stimulates the formation of erythropoietin, which helps erythroid precursors survive and
proliferate in the bone marrow.
• The physiological and molecular mechanisms that cause hyperplasia are linked to cell proliferation
regulation (i.e., cell cycle).
Incorrect Options:
Option A- Atrophy:
• Atrophy is a decrease in the number of cells in a tissue.
• Decreased oxygenation and nutrient supply to a tissue lead to a cascade of events that causes
cellular apoptosis and, ultimately, tissue atrophy.
Option B- Dysplasia:
• Dysplasia is characterized by cellular atypia.
• It entails abnormal tissue morphology and nuclear hyperchromatism.
• Dysplasia can be precancerous.
Option D- Metaplasia:
• Metaplasia is the change of one cell line to another
• The most common example is the change of esophageal squamous epithelium to the columnar
epithelium, as seen in Barrett's esophagus.

Solution for Question 11:


Correct Option C- Triceps enlargement in body-builders:

Page 12

24
• Hypertrophy means excessive growth of the size of individual cells due to the over synthesis of
structural protein.
• It can be both physiological and pathological.
• Examples of physiological hypertrophy include: Uterus during pregnancy Breast during puberty,
pregnancy, and lactation Skeletal muscles during exercise
• Uterus during pregnancy
• Breast during puberty, pregnancy, and lactation
• Skeletal muscles during exercise
• Examples of pathological hypertrophy include: Cardiac hypertrophy in hypertensive patients
• Cardiac hypertrophy in hypertensive patients
• Skeletal, smooth, and cardiac muscles can undergo hypertrophy.
• The enlargement of the triceps is an example of skeletal muscle hypertrophy (not smooth muscle
hypertrophy).
• Uterus during pregnancy
• Breast during puberty, pregnancy, and lactation
• Skeletal muscles during exercise
• Cardiac hypertrophy in hypertensive patients
Incorrect Options:
Option A - Urinary bladder in urine outflow obstruction:
• The urinary bladder has a smooth muscle that undergoes smooth muscle hypertrophy due to volume
overload, secondary to obstruction.
• The question is regarding skeletal muscle therefore this is an incorrect option
Option B- Gall bladder in chronic cholecystitis:
• The gall bladder is lined by smooth muscle, which undergoes hypertrophy in response to chronic
cholecystitis.
• The question is regarding skeletal muscle therefore this is an incorrect option
Option D- Uterus enlargement during pregnancy:
• During pregnancy, uterine smooth muscles undergo hypertrophy resulting in a 20-fold increase in the
size of the uterus.
• The question is regarding skeletal muscle therefore this is an incorrect option

Solution for Question 12:


Correct Option A- Cellular maturation shows an abnormal pattern:
• Dysplasia is the abnormal growth or development of cells, characterized by cytological abnormalities
and disorganized arrangement within their original basement membrane.
• It can be caused by various factors such as chronic inflammation, viral infections, genetic mutations,
or exposure to carcinogens.

Page 13

25
• Dysplastic cells appear cytologically abnormal, with changes in cell size, shape variations, and
hyperchromatism, observed histologically.
• It can be classified into low-grade and high-grade types. An example of low-grade dysplasia is cervical
dysplasia, which is often considered precancerous. High-grade dysplasia can progress to cancer if left
untreated, such as in the case of severe dysplasia in the colon.
• Low-grade dysplasia is typically reversible with appropriate treatment, whereas high-grade dysplasia
is more likely to progress to cancer and is considered irreversible if not treated promptly.
Dysplasia is the abnormal growth or development of cells, characterized by cytological abnormalities a
nd disorganized arrangement within their original basement membrane.
It can be caused by various factors such as chronic inflammation, viral infections, genetic mutations, or
exposure to carcinogens.
Dysplastic cells appear cytologically abnormal, with changes in cell size, shape variations, and hyperch
romatism, observed histologically.
It can be classified into low-grade and high-grade types. An example of low-grade dysplasia is cervical
dysplasia, which is often considered precancerous. High-grade dysplasia can progress to cancer if left
untreated, such as in the case of severe dysplasia in the colon.
Low-grade dysplasia is typically reversible with appropriate treatment, whereas high-grade dysplasia is
more likely to progress to cancer and is considered irreversible if not treated promptly.
Incorrect Options:
Option B- Increased numbers of otherwise normal cells:
• An increased number of otherwise normal cells is also known as hyperplasia.
• Hyperplastic cells are non-cancerous.
• They retain their normal shape, size, and regularity.
Option C- Invasiveness:
• Invasiveness is a key characteristic of cancerous cells.,
• Unlike precancerous dysplastic lesions, cancerous cells invade the basement membrane and other
surrounding tissue leading to metastasis.
Option D- Presence of poorly defined keratin cells:
• The presence of poorly defined keratin indicates squamous cell carcinoma.
• This is an incorrect option as the question is regarding preneoplastic lesion

Solution for Question 13:


Correct Option B- IL-1β:
Inflammasome:
• The inflammasome is a multiprotein complex that is located in the cytoplasm of a cell. This complex is
activated when there is an infection or stress-associated stimuli. It is made of 3 components an NLRP-3
sensor, an adapter, and an inactive Caspase-1 enzyme.
• It is activated by NOD-like receptors (NLRs).

Page 14

26
• When the NLRs signal is triggered, a version of the cytokine interleukin-1 known as Pro IL-1, which is
a precursor, is cleaved to produce the physiologically active form (IL-1).
• This is responsible for fever and the recruitment of leukocytes.
• The inflammasome pathway may be involved in the process of mediating inflammation in a variety of
widespread diseases, such as type 2 diabetes, atherosclerosis, and gout.

Solution for Question 14:


Correct Option D- Pyroptosis:
• Pyroptosis is an inflammatory form of programmed cell death in cells infected by microbes.
• It involves the activation of caspase-1, which cleaves the precursor form of IL-1 to generate
biologically active IL-1.
• Caspase-1 and inflammasome activation causes the death of the infected cell.
• Also called caspase 1-dependent cell death.
• Pyroptosis is thought to be the mechanism by which some microbes cause the death of infected cells
and at the same time trigger local inflammation.

Incorrect Options:
Option A- Necrosis:
• Necrosis is an irreversible tissue injury that results in cellular death and inflammation.
• Typically caused by cellular swelling and damage (oncosis).
• In contrast to apoptosis, necrosis involves the rupture of the cellular membrane and is generally not
considered physiologic.

Page 15

27
Option B- Necroptosis:
• Necroptosis: - It is caspase-independent programmed cell death.
• It is a hybrid that shares aspects of both necrosis and apoptosis.
• It resembles necrosis characterized by loss of ATP, swelling of the cell and organelles, generation of
reactive oxygen species (ROS), release of lysosomal enzymes, and ultimately rupture of the plasma
membrane.
• It is triggered by signal pathways that result in cell death, a feature similar to apoptosis.
• Because of these overlapping features, it is called programmed necrosis.
• Necroptosis involves two kinases called receptor-interacting protein kinases 1 and 3 (RIPK1 and
RIPK3).
• Eg: Growth plate formation in the human body (physiological), Steatohepatosis (pathological).
Option C- Apoptosis:

Page 16

28
• A process of programmed cell death. Characterized by destroying cellular contents with an intact cell
membrane and minimal inflammation. The mitochondrial pathway (Intrinsic) leads to the activation of
the initiator caspase-9, and the death receptor pathway (Extrinsic) to the initiator caspases-8 and 10.
• Characterized by destroying cellular contents with an intact cell membrane and minimal inflammation.
• The mitochondrial pathway (Intrinsic) leads to the activation of the initiator caspase-9, and the death
receptor pathway (Extrinsic) to the initiator caspases-8 and 10.
• Characterized by destroying cellular contents with an intact cell membrane and minimal inflammation.
• The mitochondrial pathway (Intrinsic) leads to the activation of the initiator caspase-9, and the death
receptor pathway (Extrinsic) to the initiator caspases-8 and 10.

• Apoptosis is a type of programmed cell death that is carried out by a set of intracellular protease
enzymes known as caspases, which are activated via two pathways: Extrinsic (Death Receptor)
Pathway: Initiated by the binding of certain ligands (e.g., FasL) to their receptors (e.g., Fas) or by
cytotoxic T cells introducing proteases (e.g., perforin, granzyme B) into cells. Ligand-receptor binding
activates initiator caspase 8 or caspase 10. Intrinsic (Mitochondrial) Pathway: Initiated by DNA damage
(e.g., radiation, toxins) or withdrawal of proliferating cellular factors (e.g., IL-2). Pro-apoptotic factors
(BAX and BAK) increase mitochondrial membrane permeability, leading to the release of cytochrome
C. Cytochrome C binds to APAF-1, forming an apoptosome that activates initiator caspase 9. Common
Pathway: Initiator caspases (8, 9, or 10) activate executioner caspases (3, 6, 7). Executioner caspases
cause nuclear fragmentation and cytoskeletal disintegration, resulting in cell shrinkage and membrane
blebbing. Apoptotic bodies containing disintegrated cellular material are formed and phagocytosed by
macrophages.
• Extrinsic (Death Receptor) Pathway: Initiated by the binding of certain ligands (e.g., FasL) to their
receptors (e.g., Fas) or by cytotoxic T cells introducing proteases (e.g., perforin, granzyme B) into cells.
Ligand-receptor binding activates initiator caspase 8 or caspase 10.
• Initiated by the binding of certain ligands (e.g., FasL) to their receptors (e.g., Fas) or by cytotoxic T
cells introducing proteases (e.g., perforin, granzyme B) into cells.
• Ligand-receptor binding activates initiator caspase 8 or caspase 10.
• Intrinsic (Mitochondrial) Pathway: Initiated by DNA damage (e.g., radiation, toxins) or withdrawal of
proliferating cellular factors (e.g., IL-2). Pro-apoptotic factors (BAX and BAK) increase mitochondrial
membrane permeability, leading to the release of cytochrome C. Cytochrome C binds to APAF-1,

Page 17

29
forming an apoptosome that activates initiator caspase 9.
• Initiated by DNA damage (e.g., radiation, toxins) or withdrawal of proliferating cellular factors (e.g.,
IL-2).
• Pro-apoptotic factors (BAX and BAK) increase mitochondrial membrane permeability, leading to the
release of cytochrome C.
• Cytochrome C binds to APAF-1, forming an apoptosome that activates initiator caspase 9.
• Common Pathway: Initiator caspases (8, 9, or 10) activate executioner caspases (3, 6, 7).
Executioner caspases cause nuclear fragmentation and cytoskeletal disintegration, resulting in cell
shrinkage and membrane blebbing. Apoptotic bodies containing disintegrated cellular material are
formed and phagocytosed by macrophages.
• Initiator caspases (8, 9, or 10) activate executioner caspases (3, 6, 7).
• Executioner caspases cause nuclear fragmentation and cytoskeletal disintegration, resulting in cell
shrinkage and membrane blebbing.
• Apoptotic bodies containing disintegrated cellular material are formed and phagocytosed by
macrophages.
• Extrinsic (Death Receptor) Pathway: Initiated by the binding of certain ligands (e.g., FasL) to their
receptors (e.g., Fas) or by cytotoxic T cells introducing proteases (e.g., perforin, granzyme B) into cells.
Ligand-receptor binding activates initiator caspase 8 or caspase 10.
• Initiated by the binding of certain ligands (e.g., FasL) to their receptors (e.g., Fas) or by cytotoxic T
cells introducing proteases (e.g., perforin, granzyme B) into cells.
• Ligand-receptor binding activates initiator caspase 8 or caspase 10.
• Intrinsic (Mitochondrial) Pathway: Initiated by DNA damage (e.g., radiation, toxins) or withdrawal of
proliferating cellular factors (e.g., IL-2). Pro-apoptotic factors (BAX and BAK) increase mitochondrial
membrane permeability, leading to the release of cytochrome C. Cytochrome C binds to APAF-1,
forming an apoptosome that activates initiator caspase 9.
• Initiated by DNA damage (e.g., radiation, toxins) or withdrawal of proliferating cellular factors (e.g.,
IL-2).
• Pro-apoptotic factors (BAX and BAK) increase mitochondrial membrane permeability, leading to the
release of cytochrome C.
• Cytochrome C binds to APAF-1, forming an apoptosome that activates initiator caspase 9.
• Common Pathway: Initiator caspases (8, 9, or 10) activate executioner caspases (3, 6, 7).
Executioner caspases cause nuclear fragmentation and cytoskeletal disintegration, resulting in cell
shrinkage and membrane blebbing. Apoptotic bodies containing disintegrated cellular material are
formed and phagocytosed by macrophages.
• Initiator caspases (8, 9, or 10) activate executioner caspases (3, 6, 7).
• Executioner caspases cause nuclear fragmentation and cytoskeletal disintegration, resulting in cell
shrinkage and membrane blebbing.
• Apoptotic bodies containing disintegrated cellular material are formed and phagocytosed by
macrophages.
Extrinsic (Death Receptor) Pathway:
• Initiated by the binding of certain ligands (e.g., FasL) to their receptors (e.g., Fas) or by cytotoxic T
cells introducing proteases (e.g., perforin, granzyme B) into cells.

Page 18

30
• Ligand-receptor binding activates initiator caspase 8 or caspase 10.
Intrinsic (Mitochondrial) Pathway:
• Initiated by DNA damage (e.g., radiation, toxins) or withdrawal of proliferating cellular factors (e.g.,
IL-2).
• Pro-apoptotic factors (BAX and BAK) increase mitochondrial membrane permeability, leading to the
release of cytochrome C.
• Cytochrome C binds to APAF-1, forming an apoptosome that activates initiator caspase 9.
Common Pathway:
• Initiator caspases (8, 9, or 10) activate executioner caspases (3, 6, 7).
• Executioner caspases cause nuclear fragmentation and cytoskeletal disintegration, resulting in cell
shrinkage and membrane blebbing.
• Apoptotic bodies containing disintegrated cellular material are formed and phagocytosed by
macrophages.

Solution for Question 15:


Correct Option C- Procaspase 8 activation:
• Programmed necrosis, also known as necroptosis, is a form of cell death that involves features of both
necrosis and apoptosis. Programmed necrosis is characterized by the formation of a complex of
receptor-interacting protein kinases (RIPKs), particularly RIPK-1 and RIPK-3, as well as procaspase 8,
but it does not involve the activation of procaspase 8 to caspase 8.
Incorrect Options:
Option A- TNF binds to TNF receptor:
• TNF (Tumor Necrosis Factor) binding to its receptor is the initial step in initiating programmed
necrosis.
Option B- RIPK-1, RIPK-3, and procaspase 8 form a complex:
• The formation of a complex involving RIPK-1, RIPK-3, and procaspase 8 is the second step in the
programmed necrosis pathway.
Option D- MLKL phosphorylation:
• MLKL (Mixed Lineage Kinase Domain-Like protein) phosphorylation is the final step in programmed
necrosis.

Page 19

31
Reversible & Irreversible Cell Injury
1. Which of the following biological processes is most likely to blame for the hydropic degeneration of
neurons seen on autopsy?
A. DNA synthesis
B. Lipid peroxidation
C. Activation of caspases
D. Na-K ATPase
----------------------------------------
2. What morphologic alteration is likely to be found in a patient with acute pancreatitis?
A. Coagulative necrosis
B. Caseous necrosis
C. Fat necrosis
D. Fibrinoid necrosis
----------------------------------------
3. What type of necrosis is most likely observed in the brain biopsy of a patient with a brain abscess?
A. Coagulative necrosis
B. Liquefactive necrosis
C. Fibrinoid necrosis
D. Fat necrosis
----------------------------------------
4. Which of the following pathologic changes are most likely to be seen in the hilar lymph nodes of a
patient with tuberculosis?
A. Caseous necrosis
B. Coagulative necrosis
C. Fat necrosis
D. Fibrinoid necrosis
----------------------------------------
5. What cause microscopic findings of preserved cellular architecture with eosinophilic cytoplasm and
no visible nuclei in the spleen during the autopsy of a 77-year-old man with hypertension, type 2
diabetes mellitus, and atrial fibrillation who developed aphasia and left-sided paralysis before his death
despite appropriate life-saving measures?

32
A. Coagulative necrosis
B. Nonenzymatic fat necrosis
C. Gangrenous necrosis
D. Liquefactive necrosis
----------------------------------------
6. What histological characteristic confirms necrosis in a 70-year-old man who passed away after a
confirmed acute myocardial infarction following a stroke?
A. Disaggregation of polyribosomes
B. Increased intracellular volume
C. Mitochondrial swelling and calcification
D. Nuclear fragmentation
----------------------------------------
7. What type of necrosis is likely to be seen in the heart of the 63-year-old male chronic alcoholic with
coronary artery disease, chest pain, and ST elevation leads V1 to V6 on ECG, despite treatment with
aspirin, clopidogrel, and atorvastatin, leading to his death?
A. Coagulative necrosis
B. Liquefactive necrosis
C. Fat necrosis
D. Caseous necrosis
----------------------------------------
8. Which of the following biological processes is most likely to blame for the hydropic degeneration of
neurons seen on autopsy?
A. DNA synthesis
B. Lipid peroxidation
C. Activation of caspases
D. Na-K ATPase
----------------------------------------
9. What type of necrosis is indicated by a wedge-shaped infarct in the right adrenal gland with an
invisible nucleus on histopathology but intact cellular outlines?

Page 2

33
A. Coagulative
B. Liquefactive
C. Fibrinoid
D. Caseous
----------------------------------------
10. Which of the following features is more commonly seen in irreversible cell injury than reversible cell
injury?
A. Cytoplasmic blebs
B. Fatty changes
C. Anaerobic glycolysis
D. Concentric lamellations
----------------------------------------
11. What is this stage of nuclear changes(shown below) in cell injury known as?

A. Normal cell
B. Pyknosis
C. Karyorrhexix
D. Karyolysis
----------------------------------------
12. What is the composition of amorphous flocculent densities that are formed in response to severe
mitochondrial damage?
A. Calcium deposits
B. Lipid droplets
C. Protein aggregates
D. Fluoride deposits
----------------------------------------
13. What is the first change to be observed during cell injury?
A. Cellular swelling
B. Decreased protein synthesis

Page 3

34
C. Anaerobic glycolysis
D. Mitochondrial dysfunction
----------------------------------------

Correct Answers
Question Correct Answer

Question 1 4
Question 2 3
Question 3 2
Question 4 1
Question 5 1
Question 6 4
Question 7 1
Question 8 4
Question 9 1
Question 10 4
Question 11 2
Question 12 1
Question 13 4

Solution for Question 1:


Correct Option D - Na-K ATPase:
• The acute and transient change known as cellular swelling results from the body's response to injuries
that are not fatal. Because the cells are unable to properly regulate the ionic and fluid equilibrium in
their environment, an accumulation of water within the cytoplasm occurs. In parenchymal organs, such
as the liver (hepatitis, hypoxia), kidney (shock), and myocardium, it is simple to observe (hypoxia,
phosphate intoxication). It could be localized to a specific area, or it could be diffuse and affect the
entire organ.
• The above-mentioned changes cause hydropic swelling of cells.
• The accumulation of sodium in the cell generates a rise in the cell's water content, which leads the cell
to expand so that isosmotic conditions can be maintained.
Incorrect Options:
Option A - DNA synthesis:
• DNA synthesis is unrelated to volume control, so it cannot be blamed.
Option B - Lipid peroxidation:
• Lipid peroxidation may lead to cell injury, but it is not a direct cause of hydropic degeneration.
Option C - Activation of caspases:

Page 4

35
• Activation of caspases is typically associated with apoptosis, which is a different form of cell death
characterized by specific morphological changes different from hydropic degeneration

Solution for Question 2:


Correct Option C - Fat necrosis:
• Fat necrosis is characterized by the saponification of fat from peripancreatic fat cells exposed to
pancreatic enzymes. Lipase is secreted from pancreatic acinar cells during acute pancreatitis, which
hydrolyzes fat into fatty acids and glycerol.
• Saponification is a process in which free fatty acids bind with calcium to generate white chalky
regions, this leads to hypocalcemia.

• Fat necrosis in acute pancreatitis- areas of white chalky deposits represent foci of fat necrosis with
calcium soap formation (saponification) at sites of lipid breakdown in the mesentery.
Incorrect Options:
Option A - Coagulative necrosis:
• Coagulative necrosis is caused by tissue ischemia in most tissues except the brain.
• Decreased oxygen delivery → ↓ ATP Anaerobic metabolism → ↑ lactic acid production → ↓ pH →
denaturation of proteins (including proteolytic enzymes) → cell death Impaired Na+/K+ -ATPase → ↑
intracellular Na+ → ↑ intracellular H2O → cell swelling
• Anaerobic metabolism → ↑ lactic acid production → ↓ pH → denaturation of proteins (including
proteolytic enzymes) → cell death
• Impaired Na+/K+ -ATPase → ↑ intracellular Na+ → ↑ intracellular H2O → cell swelling
• Preserved, anuclear, eosinophilic cellular architecture
• H&E;: eosinophilic staining due to the binding of eosin stain to denatured intracellular proteins
• Seen in Myocardial, splenic, hepatic, and renal infarction Gangrene (dry) Organ damage caused by
acidic solutions

Page 5

36
• Myocardial, splenic, hepatic, and renal infarction
• Gangrene (dry)
• Organ damage caused by acidic solutions
• Anaerobic metabolism → ↑ lactic acid production → ↓ pH → denaturation of proteins (including
proteolytic enzymes) → cell death
• Impaired Na+/K+ -ATPase → ↑ intracellular Na+ → ↑ intracellular H2O → cell swelling
• Myocardial, splenic, hepatic, and renal infarction
• Gangrene (dry)
• Organ damage caused by acidic solutions
Option B - Caseous necrosis:
• Caseous necrosis is characterized by a soft, cheese-like appearance of necrotic tissue. It is often
seen in tuberculosis and other granulomatous diseases.
Option D - Fibrinoid necrosis:
• A type of necrosis characterized by small blood vessel injury with an accumulation of fibrin and other
plasma proteins.
• Vessel wall damage caused by immune complex deposition (e.g., due to type II hypersensitivity,
reaction) → fragmentation of collagenous and elastic fibers → leakage of fibrin and other plasma
proteins
• Visible damage: thick walls with fragments of embedded cellular debris, serum, and fibrin
• Affected necrotic areas stain intense red.
• Seen in: Rheumatoid arthritis Immune Vasculitis (e.g., polyarteritis nodosa) Vascular reactions
Preeclampsia Hypertensive emergency
• Rheumatoid arthritis
• Immune Vasculitis (e.g., polyarteritis nodosa)
• Vascular reactions
• Preeclampsia
• Hypertensive emergency
• Rheumatoid arthritis
• Immune Vasculitis (e.g., polyarteritis nodosa)
• Vascular reactions
• Preeclampsia
• Hypertensive emergency

Solution for Question 3:


Correct Option B - Liquefactive necrosis:

Page 6

37
• The clinical presentation and radiological findings described in the case are suggestive of a brain
abscess, which is commonly associated with liquefactive necrosis. Liquefactive necrosis is a type of
necrosis where tissue architecture is completely lost, and the affected area transforms into a fluid-filled
space due to the action of hydrolytic enzymes, particularly in the presence of neutrophils. This type of
necrosis is often observed in bacterial or fungal infections, as well as in ischemic infarctions of the
brain.
• In this case, the neutrophilic infiltrate seen in the histopathological examination of the brain biopsy is
characteristic of liquefactive necrosis. The ring-enhancing lesion observed on MRI is a result of the
accumulation of inflammatory cells and fluid at the border between the necrotic tissue and the
surrounding healthy tissue.
Incorrect Options:
Option A - Coagulative necrosis:
• Coagulative necrosis is a type of necrosis commonly seen in ischemic injuries (In all organs except
brain), such as in myocardial infarction. It is characterized by the preservation of the tissue architecture
with coagulation of proteins, and enzymes, so blocks the proteolysis of the dead cells; resulting in a firm
texture.
Option C - Fibrinoid necrosis:
• Associated with the antigen - antibody reactions (immune complex deposition). Some examples are:
Polyarteritis Nodosa (PAN) Rheumatic Heart Disease (RHD) SLE Malignant Hypertension (HTN
Option D - Fat necrosis:
• Fat necrosis is often seen in adipose tissue after trauma or pancreatitis. It is characterized by the
formation of chalky-white areas due to the saponification of fatty acids.

Solution for Question 4:


Correct Option A - Caseous necrosis:
• A type of necrosis characterized by granular debris results from macrophages walling off a pathogen.
• Seen in tubercular, fungal, and Nocardia infections.
• Histopathological examination shows cellular debris, lymphocytes, and macrophages that form
granulomas.
• Caseous necrosis refers to necrosis in tuberculosis that appears granular and cheesy on the surface.
• This necrotic material appears amorphous, structureless, eosinophilic, granular debris under the
microscope, with no cellular features visible (as opposed to coagulative necrosis, in which cell outlines
are preserved).
Incorrect Options:
Option B - Coagulative necrosis:
• Coagulative necrosis occurs due to tissue ischemia in solid organs such as the heart, liver, spleen,
and kidneys (e.g., splenic infarction).
• Ischemia is characterized by decreased oxygen delivery, which reduces the amount of ATP produced
via aerobic glycolysis.

Page 7

38
• The hypoxic cells have to switch to anaerobic metabolism, which increases lactic acid production,
resulting in acidosis. Acidosis causes the denaturation of proteins (including proteolytic and particularly
lysosomal enzymes), preventing the cell from dissolving (autolysis).
• For this reason, coagulative necrosis is characterized by a temporarily preserved cellular architecture
without cell nuclei or nuclei still undergoing karyolysis.
Option C - Fat necrosis:
• A type of necrosis in which adipose cells die off prematurely, caused by an enzymatic reaction or
traumatic injury.
• Enzymatic fat necrosis: release of lipase and triglycerides from the cytoplasm of damaged cells →
breakdown of triglycerides by lipase→ binding of fatty acids to calcium → saponification → chalky-white
appearance
• Combination of fat saponification and calcium → dark blue appearance on H&E; stain
• Enzymatic: acute pancreatitis (due to saponification of peripancreatic fat)
• Non-enzymatic: traumatic breast injury
Option D - Fibrinoid necrosis:
• Fibrinoid necrosis is most commonly seen in blood vessels in patients with vasculitides or malignant
hypertension. It is characterized by an immune complex and fibrin deposition in the blood vessel wall.
• Histologic examination of fibrinoid necrosis would show protein-rich, eosinophilic acellular material in
damaged vessels.

Solution for Question 5:


Correct Option A - Coagulative necrosis:
• Coagulative necrosis occurs due to tissue ischemia in solid organs (except the brain) such as the
heart, liver, spleen, and kidneys (e.g., splenic infarction). Ischemia is characterized by decreased
oxygen delivery, which reduces the amount of ATP produced via aerobic glycolysis.
• The hypoxic cells have to switch to anaerobic metabolism, which increases lactic acid production,
resulting in acidosis.
• Acidosis causes the denaturation of proteins (including proteolytic and particularly lysosomal
enzymes), preventing the cell from dissolving (autolysis).
• For this reason, coagulative necrosis is characterized by a temporarily preserved cellular architecture
without cell nuclei or nuclei still undergoing karyolysis.
Incorrect Options:
Option B - Nonenzymatic fat necrosis:
• Nonenzymatic fat necrosis is most commonly seen in fatty tissue (e.g., breast, subcutaneous tissue)
following blunt trauma or surgery.
• This patient’s ischemic infarct was in the spleen, where nonenzymatic fat necrosis could not occur.
• Histologic examination would show lipid-laden macrophages with foreign body giant cells and fibrosis
and dystrophic calcifications.
Option C - Gangrenous necrosis:

Page 8

39
• Gangrenous necrosis is most commonly seen in patients with chronic limb ischemia (dry gangrene) or
gastrointestinal tract ischemia (wet gangrene).
• A histologic examination would show preservation of cellular architecture and the absence of nuclei,
as seen in this patient. However, this patient's infarct was in the spleen, which is not subject to
gangrenous necrosis.
Option D - Liquefactive necrosis:
• Lysosomal enzymes are released in liquefactive necrosis to degrade and liquefy necrotic material.
• It is classically seen in cerebral infarctions. It can also occur in bacterial abscesses (pyogenic renal
abscesses), but this patient’s ischemic infarct was in the spleen, which is not subject to liquefactive
necrosis
• Histologic examination would show liquified tissue (due to the release of hydrolytic and lysosomal
enzymes) with multiple macrophages and cyst formation.

Solution for Question 6:


Correct Option D - Nuclear fragmentation:
• Nuclear degeneration is a hallmark of cell death. It occurs in the form of the following effects:
Pyknosis: shrinkage of the nucleus due to chromatin condensation Karyorrhexis: fragmentation of the
nucleus (mediated by endonucleases) Karyolysis: disintegration or dissolution of the nucleus.
• Pyknosis: shrinkage of the nucleus due to chromatin condensation
• Karyorrhexis: fragmentation of the nucleus (mediated by endonucleases)
• Karyolysis: disintegration or dissolution of the nucleus.
• Pyknosis: shrinkage of the nucleus due to chromatin condensation
• Karyorrhexis: fragmentation of the nucleus (mediated by endonucleases)
• Karyolysis: disintegration or dissolution of the nucleus.

Page 9

40
• The gross image shows triphenyl tetrazolium chloride in the section of the heart to detect infarction.

• It is possible to highlight the area of necrosis by immersion of tissue slices in a solution of triphenyl
tetrazolium chloride.
• This gross histochemical stain imparts a brick-red color to intact, non-infarcted myocardium with
preserved lactate dehydrogenase activity.
• Because dehydrogenases leak out through the damaged membranes of dead cells, an infarct appears
as an unstained pale zone.
• Pale staining areas in the gross photo indicate myocardial infarction.

Incorrect Options:
Option A - Disaggregation of polyribosomes:
• The early stage of cell injury and ischemia causes the detachment of ribosomes and polysomes,
decreasing protein synthesis.
• It is a feature of the reversible stage of cell injury.
Option B - Increased intracellular volume:
• The reversible stage of cellular swelling is characterized by an increase in the intracellular volume of
the cell because tissue hypoxia leads to decreased ATP production.
• Decreased function of Na+/K+ ATPase and diffusion of Na+ and water into the cell and expansion of
the cell wall with swelling of the mitochondria, cytosol, endoplasmatic reticulum, and golgi apparatus
(earliest morphologic changes)
Option C - Mitochondrial swelling and calcification:
• Mitochondrial swelling is a hallmark feature of necrosis. Due to cellular stress or injury, mitochondria
undergo swelling, which impairs their function. However, calcification of mitochondria is not a typical
feature of necrosis.
• therefore this option is incorrect

Page 10

41
Solution for Question 7:
Correct Option A - Coagulative necrosis:
• Coagulative necrosis occurs due to tissue ischemia in solid organs such as the heart, liver, spleen,
and kidneys (e.g., splenic infarction). Ischemia is characterized by decreased oxygen delivery, which
reduces the amount of ATP produced via aerobic glycolysis.
• The hypoxic cells have to switch to anaerobic metabolism, which increases lactic acid production,
resulting in acidosis.
• Acidosis causes the denaturation of proteins (including proteolytic and particularly lysosomal
enzymes), preventing the cell from dissolving (autolysis).
• For this reason, coagulative necrosis is characterized by a temporarily preserved cellular architecture
without cell nuclei or nuclei still undergoing karyolysis.
Incorrect Options:
Option B - Liquefactive necrosis:
• Lysosomal enzymes are released in liquefactive necrosis to degrade and liquefy necrotic material.
Liquefactive necrosis is classically seen in cerebral infarctions and can also occur in bacterial
abscesses (pyogenic renal abscesses), but this patient’s cardiac infarction is not subjected to
liquefactive necrosis.
• Histologic examination of the tissue affected by Liquefactive necrosis would show liquified tissue (due
to the release of hydrolytic and lysosomal enzymes) with multiple macrophages and cyst formation.
Option C - Fat necrosis:
• Fat necrosis is most commonly seen in patients with acute pancreatitis or traumatic breast lesions.
• This patient's infarct was in the heart, which is not subject to fat necrosis.
Option D: Caseous necrosis
• Caseous necrosis is associated with granulomatous infections such as nocardiosis, tuberculosis
(including renal tuberculosis), and histoplasmosis.
• This patient's ischemic infarct was in the heart, which does not result in caseous necrosis.
Granulomas typically appear as focal, round, or lobulated nodules with surrounding oedema.
• Histologic examination of caseous necrosis would show acellular material that appears granular and
is surrounded by macrophages, T-cells and giant cells.

Solution for Question 8:


Correct Option D - Na-K ATPase:
• The acute and transient change known as cellular swelling results from the body's response to injuries
that are not fatal. Because the cells are unable to properly regulate the ionic and fluid equilibrium in
their environment, an accumulation of water within the cytoplasm occurs. In parenchymal organs, such
as the liver (hepatitis, hypoxia), kidney (shock), and myocardium, it is simple to observe (hypoxia,
phosphate intoxication). It could be localized to a specific area, or it could be diffuse and affect the
entire organ.
• The above-mentioned changes cause hydropic swelling of cells.

Page 11

42
• The accumulation of sodium in the cell generates a rise in the cell's water content, which leads the cell
to expand so that isosmotic conditions can be maintained.
Incorrect Options:
Option A - DNA synthesis:
• DNA synthesis is unrelated to volume control, so it cannot be blamed.
Option B - Lipid peroxidation:
• Lipid peroxidation may lead to cell injury, but it is not a direct cause of hydropic degeneration.
Option C - Activation of caspases:
• Activation of caspases is typically associated with apoptosis, which is a different form of cell death
characterized by specific morphological changes different from hydropic degeneration

Solution for Question 9:


Correct Option A - Coagulative necrosis:
• A type of necrosis caused by tissue ischemia, which denatures proteolytic enzymes. Occurs in most
tissues except the brain.
• Histopathologic examination shows preserved anuclear, eosinophilic cellular architecture.
• Tissue ischemia in solid organs such as the heart, liver, spleen, and kidneys leads to coagulative
necrosis (e.g., splenic infarction).
• Reduced oxygen delivery, which characterizes ischemia, lowers the rate at which ATP is created
through aerobic glycolysis. Acidosis results from the hypoxic cells switching to anaerobic metabolism,
which raises lactic acid generation.
• Acidosis results in the denaturation of proteins, particularly lysosomal and proteolytic enzymes, which
inhibit the cell from dissolving (autolysis).
• The absence of cell nuclei or the presence of nuclei still undergoing karyolysis are hence
characteristics of coagulative necrosis.
• Histologically, this is the most prevalent kind of necrosis. For at least a few days, the architecture of
dead tissue remains retained. An infarct is a localized area of coagulative necrosis.

Page 12

43
Incorrect Options:
Option B - Liquefactive necrosis:
• A type of necrosis characterized by the partial or complete dissolution of dead tissue and
transformation into a liquid, viscous mass.
• Caused by neutrophilic release of lysosomal enzymes that digest tissue.
• Seen in bacterial abscesses and brain infarcts.
• Numerous cystic spaces and disintegration of the normal tissue architecture are visible. There is
abundant cellular debris.

Option C - Fibrinoid necrosis:


• A type of necrosis characterized by small blood vessel injury and accumulation of fibrin and other
plasma proteins caused by malignant hypertension or vasculitis (type III hypersensitivity reaction).

Page 13

44
• Seen on histological examination of vessels in patients with polyarteritis nodosa, preeclampsia, and
hypertensive emergency.

• Microscopic image of arteriole in the lung tissues (H&E; stain)


• The arteriole wall exhibits annular deposition of amorphous and proteinaceous material and
inflammatory infiltrates.
• These are the typical features of fibrinoid necrosis.
Option D - Caseous necrosis:
• A type of necrosis characterized by granular debris results from macrophages walling off a pathogen.
• Seen in tubercular, fungal, and Nocardia infections.
• The image shows a granuloma with necrosis in the center (green outline), a surrounding layer of
immune cells such as macrophages ), and a peripheral layer that consists of epitheloid cells. A giant
cell can also be seen.
• This is the typical histopathological appearance of a caseating granuloma seen in tuberculosis.

Page 14

45
Solution for Question 10:
Correct Option D - Concentric lamellations:
• Concentric lamellations are composed of phospholipids and calcium. They are more commonly seen
in irreversible cell injury than in reversible cell injury. They are formed after the failure of the Na+ K+
ATPase pump in response to toxic or hypoxic stimuli.
Correct Options:
Option A - Cytoplasmic blebs:
• Cytoplasmic blebs are small protrusions or bulges from the cell membrane and are commonly seen in
reversible cell injury or apoptosis.
Option B - Fatty changes:
• Fatty changes also occur commonly in reversible cell injury due to a decrease in protein synthesis.
Option C - Anaerobic glycolysis:
• Anaerobic glycolysis can occur in both reversible and irreversible cell injury scenarios.

Solution for Question 11:


Correct Option B - Pyknosis:
• The given image shows a cell having a pink cytoplasm with a blue, dark, and small nucleus, which
occurs due to condensation of nuclear chromatin. This phenomenon is known as pyknosis.
Incorrect Options:
Option A - Normal cell: A normal cell does not contain condensed nuclear chromatin.
Option C - Karyorrhexix: Karyorrhexis will manifest as nuclear fragmentation.
Option D - Karyolysis: In karyolysis, nuclear material resolves so the nucleus will not be seen.

Solution for Question 12:


Correct Option A - Calcium deposits:
• During irreversible cell injury, when mitochondria are damaged, they can release calcium ions into the
cytoplasm, leading to an accumulation of calcium. This excessive calcium can precipitate and form
deposits known as amorphous flocculent densities.
Incorrect Options:
Options B, C, and D do not form amorphous flocculent densities in the mitochondria.

Page 15

46
Solution for Question 13:
Correct Option D - Mitochondrial dysfunction:
• Mitochondrial dysfunction is the initial event in cell injury. When cells are subjected to injury,
mitochondrial function can be compromised, leading to a decrease in ATP production, which impairs
the Na+ K+ ATPase pump, causing further damage to the cell.
Incorrect Options:
Option A - Cellular swelling:
• While cellular swelling, also known as hydropic change, is the earliest morphological change
observed in cell injury, it typically occurs subsequent to mitochondrial dysfunction.
Option B and C - (Decreased protein synthesis & Anaerobic glycolysis): Decreased protein synthesis a
nd Anaerobic glycolysis occur after mitochondrial dysfunction.

Page 16

47
Apoptosis.
1. In a patient with a scaly erythematous lesion on the dorsal surface of his left hand and the given
histopathological findings, which of the following proteins plays the most critical role in mediating
programmed cell death?

A. Catalase
B. Cytochrome c
C. Cytokeratin
D. Myeloperoxidase
----------------------------------------
2. Which of the following is the correct sequence for the activation of apoptosis?
A. Removal of BCL-2 → release of cytochrome C → activation of APAF-1 → caspase activation
B. Caspase activation → activation of APAF-1 → release of cytochrome C → removal of BCL-2
C. Release of cytochrome C → activation of APAF-1 → caspase activation → removal of BCL-2
D. Removal of BCL-2 → activation of APAF-1 → caspase activation → release of cytochrome C
----------------------------------------
3. Which of the following caspases mediates the execution phase of apoptosis?
A. Caspase 9
B. Caspase 8
C. Caspase 3
D. Caspase 10
----------------------------------------
4. A seventy-five-year-old male presents to your clinic with complaints of decreased memory. He states
he cannot remember the names of the new people he meets. The patient also says he feels no
emotions, even for his close relatives, and sometimes has depressive thoughts. Investigations reveal
that the person is in an early stage of Alzheimer's disease. The greater expression of BAK / BAX genes
is caused by a lack of cellular growth factors, leading to higher mitochondrial permeability, increased
activation of APAF 1, and cell death. Which of the following induces the extrinsic pathway of apoptosis?
A. Withdrawal of growth factor stimulus
B. Radiation injury

48
C. Activation of Fas receptor by FasL
D. Intracellular protein misfolding
----------------------------------------
5. A fifty-five-year-old male presents to the emergency department with complaints of chest pain. The
chest pain is left-sided and radiates to his left arm. The patient is diagnosed with myocardial infarction,
and percutaneous stenting is performed. Light microscopy of myocardial tissue suffering from ischemia
shows extensive necrosis after a few days. Which of the following is a feature of necrosis but not of
apoptosis?
A. Activation of caspase
B. Local inflammatory response
C. Intact plasma membrane
D. Intact cellular contents
----------------------------------------
6. Which of the following cells participates in the process of "efferocytosis"?
A. Macrophages
B. T lymphocytes
C. B lymphocytes
D. NK cells
----------------------------------------
7. A seventy-five-year-old male presents to your clinic with complaints of decreased memory. He states
he cannot remember the names of the new people he meets. The patient also says he feels no
emotions, even for his close relatives, and sometimes has depressive thoughts. Investigations reveal
that the person is in an early stage of Alzheimer's disease. Overactivation of caspases causes the
destruction of neural cells, which is seen in Alzheimer's. Which of the following caspases is involved in
the activation of IL-1?
A. Caspase 1
B. Caspase 3
C. Caspase 5
D. Caspase 8
----------------------------------------
8. Which of the following cellular processes, evident in the significant reduction of tumor size and a
concurrent decline in blood cell counts during chemotherapy, affects both tumor and healthy cells?
A. Necrosis
B. Apoptosis
C. Autophagy
D. Mitosis
----------------------------------------
9. Which molecule activates procaspase 9 into caspase 9 in the intrinsic pathway of apoptosis?
A. CD95

Page 2

49
B. Apoptosome
C. Fas Associated Death Domain
D. FLIP
----------------------------------------
10. Mark the incorrect statement regarding the extrinsic pathway of apoptosis:
A. It involves trimerization of CD95/Fas
B. FADD converts procaspase 8 and 9 to caspase 8 and 9
C. It is called the death receptor pathway
D. It is inhibited by FLIP
----------------------------------------
11. Which of the following factors aid in the survival of cancer cells?
A. C1q
B. Thrombospondin
C. CD47
D. Phosphatidyl serine
----------------------------------------
12. What is the hallmark of apoptosis in neurons?
A. Caspase 9
B. CD95
C. AIF
D. Caspase 8 and 10
----------------------------------------

Correct Answers
Question Correct Answer

Question 1 2
Question 2 1
Question 3 3
Question 4 3
Question 5 2
Question 6 1
Question 7 1
Question 8 2
Question 9 2
Question 10 2

Page 3

50
Question 11 3
Question 12 3

Solution for Question 1:


Correct Option B- Cytochrome C:
• Cytochrome C is a critical regulator of apoptosis present in the mitochondrial membrane.
• It seeps out of mitochondrial pores and activates apoptotic protease activating factor-1 (APAF-1).
• APAF-1 converts procaspase-9 to caspase-9, causing downstream caspases to be activated
(cysteine proteases).
• These effector caspases break various target proteins, such as endonucleases, nuclear proteins, and
cytoskeletal proteins.
Incorrect Options:
Option A- Catalase:
• Catalases through reactive oxygen species can cause cell death but do not cause planned cell death.
• Therefore this option is incorrect
Option C- Cytokeratin:
• Cytokeratin is a protein found in epithelial cells.
• It is not associated with apoptosis.
Option D- Myeloperoxidase:
• Myeloperoxidase, through reactive oxygen species, can cause cell death but does not cause planned
cell death.
• Therefore this option is incorrect

Solution for Question 2:


Correct Option A- Removal of BCL-2 → release of cytochrome C → activation of APAF-1 →
caspase activation:
• Apoptosis is the programmed cell death that occurs in multicellular organisms.
• Apoptosis is a highly regulated and controlled process.
• It can be initiated through: Intrinsic pathway Extrinsic pathway
• Intrinsic pathway
• Extrinsic pathway
• The intrinsic pathway is shown below:
• Intrinsic pathway
• Extrinsic pathway

Page 4

51
Incorrect Options:
Option B- Caspase activation → activation of APAF-1 → release of cytochrome C → removal of BCL-2:
• This is not the correct sequence for the intrinsic pathway.
• The correct sequence is: Removal of BCL-2 → release of cytochrome C → activation of APAF-1 →
caspase activation
• Removal of BCL-2 → release of cytochrome C → activation of APAF-1 → caspase activation
• Removal of BCL-2 → release of cytochrome C → activation of APAF-1 → caspase activation
Option C- Release of cytochrome C → activation of APAF-1 → caspase activation →
removal of BCL-2:
• This is not the correct sequence for the intrinsic pathway.
• The correct sequence is: Removal of BCL-2 → release of cytochrome C → activation of APAF-1 →
caspase activation
• Removal of BCL-2 → release of cytochrome C → activation of APAF-1 → caspase activation
• Removal of BCL-2 → release of cytochrome C → activation of APAF-1 → caspase activation
Option D- Removal of BCL-2 → activation of APAF-1 → caspase activation →
release of cytochrome C:
• This is not the correct sequence for the intrinsic pathway.
• The correct sequence is: Removal of BCL-2 → release of cytochrome C → activation of APAF-1 →
caspase activation
• Removal of BCL-2 → release of cytochrome C → activation of APAF-1 → caspase activation
• Removal of BCL-2 → release of cytochrome C → activation of APAF-1 → caspase activation

Solution for Question 3:

Page 5

52
Correct Option C- Caspase 3:
• Caspases are a group of enzymes involved in programmed cell death.
• The activation of caspase enzymes causes apoptosis.
• The stages of apoptosis are as follows: Initiation phase in which some caspases become catalytically
active Execution phase in which other caspases trigger the degradation of critical cellular components
• Initiation phase in which some caspases become catalytically active
• Execution phase in which other caspases trigger the degradation of critical cellular components
• The intrinsic and extrinsic pathways of apoptosis combine to form a cascade of caspase activation,
which mediate the final phase of apoptosis.
• The death receptor (extrinsic) pathway activates initiator caspases 8 and 10, while the mitochondrial
(intrinsic) pathway activates initiator caspase-9.
• The active forms of these caspases trigger the rapid and sequential activation of the executioner
caspases, such as caspase-3 and caspase-6
• These then act on many cellular components leading to apoptotic changes.
• Caspases that operate as executioners, such as caspase-3 and 6, affect many cellular components.
• They cleave an inhibitor of a cytoplasmic DNase to make it enzymatically active, which results in
nuclear fragmentation.
• Caspases also degrade the nuclear matrix's structural components, promoting nuclei fragmentation.
• Initiation phase in which some caspases become catalytically active
• Execution phase in which other caspases trigger the degradation of critical cellular components
Incorrect Options:
Option A- Caspase 9:
• The death receptor (extrinsic) pathway activates initiator caspases 8 and 10, while the mitochondrial
(intrinsic) pathway activates initiator caspase-9.
Option B- Caspase 8:
• The death receptor (extrinsic) pathway activates initiator caspases 8 and 10, while the mitochondrial
(intrinsic) pathway activates initiator caspase-9.
Option D- Caspase 10:
• The death receptor (extrinsic) pathway activates initiator caspases 8 and 10, while the mitochondrial
(intrinsic) pathway activates initiator caspase-9.

Solution for Question 4:


Correct Option C- Activation of Fas receptor by FasL:
• Apoptosis is also referred to as programmed cell death.
• Apoptosis is a highly regulated and controlled process.
• It can be initiated through: Intrinsic pathway Extrinsic pathway
• Intrinsic pathway

Page 6

53
• Extrinsic pathway
• The death receptor (Extrinsic) pathway of apoptosis is triggered by the activation of Fas and TNF
receptors.
• The ligand for the Fas receptor is called the Fas ligand (FasL).
• FasL is expressed on T cells that recognize self-antigens and some cytotoxic T cells that kill
virus-infected and tumour cells.
• Intrinsic pathway
• Extrinsic pathway
Incorrect Options:
Option A- Withdrawal of growth factor stimulus:
• The Mitochondrial (intrinsic) pathway of apoptosis is triggered due to cellular injury caused by: Growth
factor withdrawal DNA damage (by radiation, toxins, free radicals) Protein misfolding (endoplasmic
reticulum stress)
• Growth factor withdrawal
• DNA damage (by radiation, toxins, free radicals)
• Protein misfolding (endoplasmic reticulum stress)
• Growth factor withdrawal
• DNA damage (by radiation, toxins, free radicals)
• Protein misfolding (endoplasmic reticulum stress)
Option B- Radiation injury:
• The Mitochondrial (intrinsic) pathway of apoptosis is triggered due to cellular injury caused by: Growth
factor withdrawal DNA damage (by radiation, toxins, free radicals) Protein misfolding (endoplasmic
reticulum stress)
• Growth factor withdrawal
• DNA damage (by radiation, toxins, free radicals)
• Protein misfolding (endoplasmic reticulum stress)
• Growth factor withdrawal
• DNA damage (by radiation, toxins, free radicals)
• Protein misfolding (endoplasmic reticulum stress)
Option D- Intracellular protein misfolding:
• The Mitochondrial (intrinsic) pathway of apoptosis is triggered due to cellular injury caused by: Growth
factor withdrawal DNA damage (by radiation, toxins, free radicals) Protein misfolding (endoplasmic
reticulum stress)
• Growth factor withdrawal
• DNA damage (by radiation, toxins, free radicals)
• Protein misfolding (endoplasmic reticulum stress)
• Growth factor withdrawal
• DNA damage (by radiation, toxins, free radicals)

Page 7

54
• Protein misfolding (endoplasmic reticulum stress)

Solution for Question 5:


Correct Option B- Local inflammatory response:
• An inflammatory response is seen in necrosis but not in apoptosis.
• Features of necrosis and apoptosis are shown below:
• Feature Necrosis Apoptosis Physiologic or pathologic Always pathological May be physiological or
pathological Plasma membrane Disrupted Intact; altered structure, especially orientation of lipids
Cellular contents Enzymatic digestion; may leak out of cell Intact; may be released in apoptotic bodies
Adjacent inflammation Frequent No Cell size Cell size is increased Affect single cells
Feature
Necrosis
Apoptosis
Physiologic or pathologic
Always pathological
May be physiological or pathological
Plasma membrane
Disrupted
Intact; altered structure, especially orientation of lipids
Cellular contents
Enzymatic digestion; may leak out of cell
Intact; may be released in apoptotic bodies
Adjacent inflammation
Frequent
No
Cell size
Cell size is increased
Affect single cells
Incorrect Options:
Options A, C and, D are characteristic of apoptosis and not seen in necrosis.

Solution for Question 6:


Correct Option A- Macrophages:
• Efferocytosis is a process of removing apoptotic cells without causing inflammation

Page 8

55
• It is carried about by dendritic cells, macrophages, fibroblasts and epithelial cells, which recognize
“find me” and “eat me” signals from apoptotic cells.
• Production of pro-inflammatory cytokines is reduced in macrophages that have ingested apoptotic
cells.
Incorrect Options:
Options B, C, and D are not involved in this process. Macrophages secrete proteins that bind to apopto
tic cells leading to the engulfment of cells.

Solution for Question 7:


Correct Option A- Caspase 1:
• Caspases are a group of enzymes involved in cell death.
• Pyroptosis is a process in which injury is initiated because of exposure to microbes, and microbial
proteins.
• Whenever exposed to microbes, there is an activation of enzyme caspase 1.
• Caspase-1, along with closely related caspase-11, cause the death of the infected cell.
• Caspase 1 convert inactive form of interleukin 1 (IL 1) to active form of IL1.
• IL1 acts centrally; it enters the CNS and results in the development of fever.
Incorrect Options:
Option B- Caspase 3:
• Caspase 3 is an executioner caspase that cleaves DNA and other substrates to cause cell death.
• It does not activate IL 1.
Option C- Caspase 5:
• Caspase-5, recognized primarily for its role in inflammation within the inflammasome complex, also
participates in apoptosis through distinct pathways
• It does not activate interleukin-1.
Caspase-5, recognized primarily for its role in inflammation within the inflammasome complex, also par
ticipates in apoptosis through distinct pathways
It does not activate interleukin-1.
Option D- Caspase 8:
• Caspase 8 can initiate extrinsic and intrinsic pathways of apoptosis.
• It does not activate IL 1.

Solution for Question 8:


Correct Option B - Apoptosis:

Page 9

56
• Apoptosis is a programmed cell death process that occurs in both physiological and pathological
conditions. It is crucial for maintaining tissue homeostasis, eliminating damaged or infected cells, and
during development.
• In cancer treatment, chemotherapy drugs often induce apoptosis in tumor cells, leading to a reduction
in tumor size, as observed in this patient. However, chemotherapy can also affect healthy cells,
particularly rapidly dividing cells like blood cells in the bone marrow, leading to anemia (decrease in red
blood cells), thrombocytopenia (decrease in platelets), and leukopenia (decrease in white blood cells).
Incorrect Options:
Option A - Necrosis:
• Necrosis is a type of cell death that occurs due to external factors, such as injury, toxins, or lack of
blood supply. It is not a programmed process like apoptosis and is associated with inflammation.
Necrosis is not responsible for the observed effects in the given case.
Option C - Autophagy:
• Autophagy is a cellular process in which cells recycle and degrade their own components to maintain
cellular homeostasis. It can be induced by stress or nutrient deprivation and is often considered a
pro-survival mechanism. While autophagy may play a role in cancer development and treatment, it
does not explain the reduction in blood cells in this patient.
Option D - Mitosis:
• Mitosis is a normal cell division process by which cells replicate and produce two identical daughter
cells. It is not responsible for the observed effects in the given case.

Solution for Question 9:


Correct Option B - Apoptosome:
• In the intrinsic pathway of apoptosis, also known as the mitochondrial pathway, the activation of
procaspase 9 to caspase 9 occurs through the apoptosome. The apoptosome is formed when
cytochrome c, released from the mitochondria in response to cellular stress or damage, binds to
apoptotic protease activating factor 1 (Apaf-1). This complex then activates procaspase 9, converting it
into its active form, caspase 9. Caspase 9 then initiates the cascade of events leading to apoptosis.

Page 10

57
Incorrect Options:
Options A, C, and D do not play a role in the intrinsic pathway of apoptosis.

Solution for Question 10:


Correct Option B - FADD converts procaspase 8 and 9 to caspase 8 and 9:
• The extrinsic pathway of apoptosis involves the activation of procaspase 8 and 10 to caspase 8 and
10 through FADD (Fas-associated death domain). It does not involve the activation of pro-caspase-9.
Activation of procaspase-9 to caspase -9 is a feature of intrinsic pathway.
Incorrect Options:
Options A, C, and D are correct about the extrinsic pathway of apoptosis.

Solution for Question 11:


Correct Option C - CD47:
• The surface of cancer cells has CD47which sends a "don't eat me" signal to macrophages so they not
to engulf or destroy the cancer cell. This interaction effectively aids in cancer cells in promoting their
survival and proliferation within the body.
Incorrect Options:
Options A, B, and D: The apoptotic bodies express eat-me signals through C1q, Thrombospondin, and
PS (Phosphatidyl Serine). They lead to apoptosis and do not help in survival of cancer cells.

Page 11

58
Solution for Question 12:
Correct Option C - AIF:
• Neurons do not have caspases, instead they have apoptotic inducing factors which lead to apoptosis.
Apoptosis inducing factor(AIF) is a hallmark feature of neuronal apoptosis.
Incorrect Options:
Options A, B, and D are not hallmarks of apoptosis in neurons.

Page 12

59
Intracellular Accumulations, Autophagy and Free
Radical Injury
1. What mechanisms are involved in the pathogenesis of cellular aging?
A. Decreased protein synthesis
B. Telomere lengthening
C. Defective degradation of proteins
D. Decreased immune response
----------------------------------------
2. During increased cellular stress, the energy-depleted cell eats its cellular components to compensate
for nutrient depletion. Which of the following is a good marker to detect this process?
A. BAK
B. BAX
C. LC3
D. BID
----------------------------------------
3. Which of the following enzyme protects the brain from free radical injury?
A. Myeloperoxidase
B. Superoxide dismutase
C. NADPH oxidase
D. Hydroxylase
----------------------------------------
4. Free radicals have unpaired electrons. Due to free unpaired electrons, free electrons are such
reactive species. Which of the following is the most reactive free radical?
A. Superoxide
B. Hydroxyl
C. Peroxide
D. Carboxyl
----------------------------------------
5. A 60-year-old man post orthotopic liver transplantation presents with decreased bile production
during the first 3 days. Reperfusion injury is thought to be the cause of Poor graft function in this
patient. Which of the following substances is most likely to cause reperfusion injury in the transplanted
liver?
A. Cationic proteins
B. Free ferric iron
C. Hydrochlorous acid
D. Reactive oxygen species

60
----------------------------------------
6. What genetic mutation is commonly linked to growth failure, hair loss, distinct facial features like a
small face and prominent eyes, and severe cardiovascular issues in a 4-year-old child?
A. ATM gene mutation
B. LMNA gene mutation
C. CFTR gene mutation
D. FBN1 gene mutation
----------------------------------------
7. What genetic mutation is commonly associated with growth retardation, developmental delays,
sensitivity to sunlight, progressive neurologic decline, and premature aging in a 5-year-old child?
(or)
Which gene mutation is commonly associated with Cockayne syndrome?
A. ERCC2 gene mutation
B. LMNA gene mutation
C. ERCC8 gene mutation
D. FBN1 gene mutation
----------------------------------------
8. What stain should be used to detect lipid-containing organelles in blast cells of a suspected case of
acute lymphocytic leukemia?
A. Periodic Acid Schiff (PAS)
B. Myeloperoxidase
C. Sudan black
D. Mucicarmine
----------------------------------------
9. What is the composition of braided bundles of Mallory-Denk's bodies found in a 62-year-old man with
alcoholic hepatitis?
A. α1-Antitrypsin
B. β-Amyloid (Aβ)
C. Intermediate filaments
D. Prion protein (PrP)
----------------------------------------
10. Where does intracellular calcification start among the following organelles?
A. Mitochondria
B. Golgi body
C. Lysosomes
D. Nucleus
----------------------------------------

Page 2

61
11. What leads to peri-nuclear brown pigment observed during autopsy in an 83-year-old male with a
flatline ECG and a Glasgow Coma Scale of 3/15?
A. Melanin
B. Hemosiderin
C. Bilirubin
D. Lipofuscin
----------------------------------------
12. What's the key intracellular iron storage protein in the hepatocytes of a 45-year-old male with
abdominal enlargement, yellow skin, sclera, hepatomegaly, and jaundice and the given liver biopsy
findings on Prussian blue stain?

A. Bilirubin
B. Haptoglobin
C. Hemoglobin
D. Hemosiderin
----------------------------------------
13. What histological finding is most likely in the biopsy of a 62-year-old male with occupational
asbestos exposure, presenting with shortness of breath, pleuritic chest pain, and imaging showing
pleural thickening with calcified nodules and microscopy showing spindle-shaped cells forming papillary
structures with focal areas of calcification?
A. Auer rods
B. Lewy bodies
C. Psammoma bodies
D. Call-Exner bodies
----------------------------------------
14. What best describes the cells with a clear halo around the nucleus in the pituitary gland of a
45-year-old female with weight gain, central obesity, moon facies, hypertension, elevated serum
cortisol levels, and failure to suppress cortisol after a low-dose dexamethasone suppression test?
A. Ghon focus
B. Russell bodies
C. Crooke hyaline change

Page 3

62
D. Caroli disease
----------------------------------------
15. Which of the following is least likely to generate free oxygen radicals within neutrophils to kill
intracellular bacteria?
A. Nitric oxide
B. Fenton's reaction
C. NADPH oxidase
D. Catalase
----------------------------------------
16. Which is incorrect about Reactive oxygen species (ROS) removal from the cells?
A. Superoxide is converted to hydrogen peroxide and oxygen by superoxide dismutase
B. Hydrogen peroxide is converted to water by glutathione peroxidase
C. Hydroxyl radical is converted to water by glutathione peroxidase
D. Peroxynitrite is converted to nitrite by glutathione peroxidase
----------------------------------------
17. Match the following pigments with their corresponding stains: 1 Melanin A Lillie’s stain 2 Copper B
Perl’s stain 3 Fe2+ C Rhodanine stain 4 Fe3+ D Dopaoxidase stain
1 Melanin A Lillie’s stain
2 Copper B Perl’s stain
3 Fe2+ C Rhodanine stain
4 Fe3+ D Dopaoxidase stain

A. 1-C, 2-D, 3-B, 4-A


B. 1-C, 2-D, 3-A, 4-B
C. 1-D, 2-C, 3-A, 4-B
D. 1-D, 2-C, 3-B, 4-A
----------------------------------------
18. Which of the following conditions may lead to the tabby cat effect in the heart?
A. Rheumatic heart disease
B. Infective endocarditis
C. Pericarditis
D. Diphtherial myocarditis
----------------------------------------
19. Which of the following histopathological images corresponds to the findings of Cushing’s disease?
A.
B.
C.

Page 4

63
D.
----------------------------------------
20. Which of the following is not a feature of metastatic calcification?
A. Elevated blood calcium levels
B. Most commonly affects lungs
C. Normal tissue can also be seen
D. Seen in Monckeberg sclerosis
----------------------------------------
21. Which stain is used to differentiate between the mineralized bone and osteoid?
A. Von Kossa stain
B. Alizarin red S stain
C. Calcein stain
D. AZAN stain
----------------------------------------
22. In which type of autophagy are heat shock proteins utilized?
A. Macroautophagy
B. Microautophagy
C. Chaperone-mediated autophagy
D. Mitophagy
----------------------------------------
23. Which mutation can lead to increased chances of Crohn’s disease?
A. ATG1
B. ATG16L
C. ATG5
D. ATG3
----------------------------------------
24. Mark the correct statement: SIRTUINS increase insulin sensitivity Calorie deficit diet can increase
SIRTUINS SIRTUINS are produced by the SIRT gene SIRTUINS are histone deacetylases
A. Only 3
B. 2 and 3
C. 2, 3 and 4
D. 1, 2, 3 and 4
----------------------------------------
25. Which molecule serves as the master regulator in the process of ferroptosis?
A. Iron ions
B. Glutathione peroxidase Type 4

Page 5

64
C. Cytoplasmic SOD
D. Mitochondrial SOD
----------------------------------------

Correct Answers
Question Correct Answer

Question 1 3
Question 2 3
Question 3 2
Question 4 2
Question 5 4
Question 6 2
Question 7 3
Question 8 3
Question 9 3
Question 10 1
Question 11 4
Question 12 4
Question 13 3
Question 14 3
Question 15 4
Question 16 4
Question 17 3
Question 18 4
Question 19 4
Question 20 4
Question 21 4
Question 22 3
Question 23 2
Question 24 4
Question 25 2

Solution for Question 1:


Correct Option C - Defective degradation of proteins:
• Cellular aging is due to a gradual decline in cellular function and viability. This is a result of genetic
abnormalities and the accumulation of cellular and molecular damage due to the effects of exogenous

Page 6

65
influences.
• The following signaling pathways affect the aging of cells: Mutations, DNA damage, and faulty DNA
repair. Replication of cells, the shortening or attrition of telomeres, and the death of cells. Protein
homeostasis declines with age, which causes problems with the regular folding of proteins and the
removal of proteins that have become misfolded (there is decrease in normal proteins & increase in
damaged proteins). Caloric restriction is believed to enhance longevity through two main mechanisms:
reducing the signaling intensity of the IGF-1 pathway and increasing sirtuins..
• Mutations, DNA damage, and faulty DNA repair.
• Replication of cells, the shortening or attrition of telomeres, and the death of cells.
• Protein homeostasis declines with age, which causes problems with the regular folding of proteins
and the removal of proteins that have become misfolded (there is decrease in normal proteins &
increase in damaged proteins).
• Caloric restriction is believed to enhance longevity through two main mechanisms: reducing the
signaling intensity of the IGF-1 pathway and increasing sirtuins..
• Proteins that do not fold correctly have been linked to the following diseases: Retinitis pigmentosa,
which is caused by abnormal folding of rhodopsin. Creutzfeldt-Jakob disease, which is caused by
abnormal folding of PrPsc. Alzheimer's disease, which is caused by the abnormal folding of A peptide.
• Retinitis pigmentosa, which is caused by abnormal folding of rhodopsin.
• Creutzfeldt-Jakob disease, which is caused by abnormal folding of PrPsc.
• Alzheimer's disease, which is caused by the abnormal folding of A peptide.
Cellular aging is due to a gradual decline in cellular function and viability. This is a result of genetic abn
ormalities and the accumulation of cellular and molecular damage due to the effects of exogenous influ
ences.
The following signaling pathways affect the aging of cells:
• Mutations, DNA damage, and faulty DNA repair.
• Replication of cells, the shortening or attrition of telomeres, and the death of cells.
• Protein homeostasis declines with age, which causes problems with the regular folding of proteins
and the removal of proteins that have become misfolded (there is decrease in normal proteins &
increase in damaged proteins).
• Caloric restriction is believed to enhance longevity through two main mechanisms: reducing the
signaling intensity of the IGF-1 pathway and increasing sirtuins..
Mutations, DNA damage, and faulty DNA repair.
Replication of cells, the shortening or attrition of telomeres, and the death of cells.
Protein homeostasis declines with age, which causes problems with the regular folding of proteins and
the removal of proteins that have become misfolded (there is decrease in normal proteins &
increase in damaged proteins).
Caloric restriction is believed to enhance longevity through two main mechanisms: reducing the signali
ng intensity of the IGF-1 pathway and increasing sirtuins..
Proteins that do not fold correctly have been linked to the following diseases:
• Retinitis pigmentosa, which is caused by abnormal folding of rhodopsin.
• Creutzfeldt-Jakob disease, which is caused by abnormal folding of PrPsc.

Page 7

66
• Alzheimer's disease, which is caused by the abnormal folding of A peptide.
Retinitis pigmentosa, which is caused by abnormal folding of rhodopsin.
Creutzfeldt-Jakob disease, which is caused by abnormal folding of PrPsc.
Alzheimer's disease, which is caused by the abnormal folding of A peptide.
Incorrect Options:
Option A - Decreased protein synthesis:
• The pathogenesis of aging does not include decreased protein synthesis.
• Protein homeostasis declines with age, which causes problems with the regular folding of proteins
and the removal of proteins that have become misfolded.
The pathogenesis of aging does not include decreased protein synthesis.
Protein homeostasis declines with age, which causes problems with the regular folding of proteins and
the removal of proteins that have become misfolded.
Option B - Telomere lengthening:
• The pathogenesis of aging includes telomere shortening.
• Therefore this option is incorrect
The pathogenesis of aging includes telomere shortening.
Therefore this option is incorrect
Option D - Decreased immune response:
• The pathogenesis of aging does not include decreased immune response.
• Therefore this option is incorrect
The pathogenesis of aging does not include decreased immune response.

Solution for Question 2:


Correct Option C - LC3:
• Autophagy ("self-eating") is the process in which the starved cell eats its components to reduce
nutrient demand to match the supply.
• Some cell debris within the autophagic vacuoles may resist digestion and persist in the cytoplasm as
membrane-bound residual bodies. An example of residual bodies is lipofuscin granules, as shown in
the below image.

Page 8

67
• Pathological accumulation of lipofuscin is seen in Parkinson’s disease.
• The most important gene that we associate with autophagy is ATG-3.
• The formation of autophagosomes requires ubiquitin-like conjugation systems that result in the
covalent linkage of the lipid phosphatidylethanolamine (PE) to microtubule-associated protein light
chain 3 (LC3). PE-lipidated LC3 is increased during autophagy, and it is a useful marker for identifying
cells in which autophagy occurs. The newly formed autophagosome fuses with lysosomes to form an
autophagolysosome.
• Inhibition of autophagy increases the risk of malignancy.
Incorrect Options:
Option A – BAK:
• BAK, BAX, and BID are involved with cellular apoptosis.
• These are proapoptotic factors.
Option B – BAX:
• BAK, BAX, and BID are involved with cellular apoptosis.
• These are proapoptotic factors.
Option D – BID:
• BAK, BAX, and BID are involved with cellular apoptosis.
• These are proapoptotic factors.

Solution for Question 3:


Correct Option B - Superoxide dismutase:
• Superoxide dismutase is an enzyme that protects the cell from reactive oxygen species.

Page 9

68
• Superoxidase dismutases (SODs) are found in many cell types and converts O2 radicals into
hydrogen peroxide.
This group of enzymes includes:-
• Manganese-SOD localized in mitochondria
• Copper-zinc- SOD found in the cytosol.
One of the earliest discovered hereditary forms of Amyotrophic lateral sclerosis (ALS) has mutations in
the gene encoding copper-zinc superoxide dismutase (SOD1) on chromosome 21; this variant account
s for about 20% of familial cases.
Incorrect Options:
Option A – Myeloperoxidase:
• Myeloperoxidase is a peroxidase enzyme.
• It is mainly present in neutrophil granulocytes
• It produces hypochlorous acid to act as an antimicrobial enzyme.
Option C - NADPH oxidase:
• It produces reactive oxygen species
Option D – Hydroxylase:
• Hydroxylases are enzymes that catalyze the addition of hydroxyl groups.
• It is just involved in hydroxylation reactions.

Solution for Question 4:


Correct Option B - Hydroxyl:
• Reactive oxygen species are essential for the signaling and physiological functions of the cell.
• But an increased amount of reactive oxygen species can alter the oxidation-reduction redox reactions
and disrupt normal biological functions.
• Oxidative stress occurs when there is an imbalance in reactive oxygen species and antioxidant
activity.
• There are many reactive oxygen species; Hydroxyl radical is the most reactive free radical.
• It is highly reactive because it has an unpaired electron in its outermost electron shell, making it very
unstable. This unpaired electron seeks to pair with another electron, leading to rapid and often
damaging chemical reactions with nearby molecules, including DNA, proteins, and lipids. Hydroxyl
radicals are generated during various biological processes, particularly in the presence of reactive
oxygen species (ROS).
Incorrect Options:
Option A – Superoxide:
• Superoxide is an important it is an anion that generates free radicals.
• It is impermeable to the cell membrane and mainly affects enzyme function.

Page 10

69
• Superoxide radical (•O2-) is a reactive free radical, but it is generally less reactive than the hydroxyl
radical.
Option C – Peroxide:
• Peroxide is not a typical free radical, but it can cause damage to the cells even at low concentrations.
• Peroxide is less potent than hydroxyl free radicals.
Option D – Carboxyl:
• Carboxyl is also a free radical form, but it is less reactive than hydroxyl radical.
• Therefore this is not the best option
• Carboxyl radicals are not typically considered highly reactive free radicals in the context of oxidative
stress and cellular damage. They are functional groups found in organic compounds but do not
possess the same level of reactivity as hydroxyl radicals.

Solution for Question 5:


Correct Option D - Reactive oxygen species:
• Ischemia/reperfusion injury is a common clinical problem in occlusive cardiovascular disease,
transplantation, infection, shock, and many other circumstances.
• The beginning of injury caused by ischemia and reperfusion is linked to the interaction between the
temporary lack of blood flow and the restoration of blood flow. When ischemia occurs, it damages the
cells and generates free radical species.
• Subsequently, reperfusion provides abundant molecular oxygen (O2) combined with free radicals to
form reactive oxygen and nitrogen species. Oxygen radicals are formed inside cells through the
xanthine oxidase pathway and cause reperfusion injury.
Incorrect Options:
Option A - Cationic proteins:
• Cationic proteins, which are eosinophilic, are found in the body fluids of patients with allergic and
inflammatory diseases. Their secretions are induced by immunoglobulins, complement factors, and
serum-opsonized particles.
Option B - Free ferric iron:
• Transition metals such as iron and copper donate or accept free electrons during intracellular
reactions and catalyze free radical formation. This is done by ferrous(Fe2+) and not ferric(3+).
Option C - Hydrochlorous acid:
• Hydrochloric acid is a weak acid that forms when chlorine dissolves in water and partially dissociates.
• It is a weak acid that does not have any reactive properties, so it is not involved in reperfusion injury.

Solution for Question 6:


Correct Option B - LMNA gene mutation:

Page 11

70
• Hutchinson-Gilford progeria syndrome (HGPS), a rare genetic disorder causing premature aging in
children, is most commonly associated with a spontaneous mutation in the LMNA gene. This mutation
leads to the production of an abnormal protein, progerin, contributing to the pathogenesis of HGPS.
Incorrect Options:
Option A - ATM gene mutation:
• The ATM gene is associated with Ataxia Telangiectasia (A-T), a rare genetic disorder affecting
multiple body systems. A-T primarily involves neurological problems, such as progressive difficulty with
movement and coordination (ataxia), immune system issues, and an increased risk of certain cancers.
Telangiectasias, which are tiny red "spider" veins, may appear in the corners of the eyes and on the
skin.
Option C - CFTR gene mutation:
• Cystic Fibrosis (CF) is a genetic disorder caused by mutations in the CFTR gene. It primarily affects
the respiratory and digestive systems. CF leads to the production of thick, sticky mucus, causing
breathing difficulties, frequent lung infections, and digestive problems due to the mucus obstructing the
pancreas and intestines.
Option D - FBN1 gene mutation:
• Mutations in the FBN1 gene are associated with Marfan syndrome. Marfan syndrome is a connective
tissue disorder that affects various body systems, particularly the cardiovascular, skeletal, and ocular
systems. Individuals with Marfan syndrome may exhibit features like tall stature, long limbs, joint
hypermobility, and potential complications related to the heart and blood vessels.

Solution for Question 7:


Correct Option C - ERCC8 gene mutation:
• Cockayne syndrome, characterized by growth failure, developmental delays, photosensitivity, and
progressive neurological decline, is primarily associated with mutations in the ERCC8 (CSA) gene.
Mutations in ERCC8 disrupt DNA repair mechanisms, leading to the symptoms observed in affected
individuals.
• Individuals with Cockayne syndrome exhibit growth retardation, photosensitivity, and progressive
neurological decline due to defective DNA repair mechanisms.
Incorrect Option:
Option A - ERCC2 gene mutation:
• Mutations in the ERCC2 gene are associated with Xeroderma Pigmentosum (XP). XP is a genetic
disorder characterized by extreme sensitivity to ultraviolet (UV) rays from sunlight. Individuals with XP
are at a high risk of developing skin cancers due to the inability to repair damage caused by UV light.
Option B - LMNA gene mutation:
• Mutations in the LMNA gene are linked to Hutchinson-Gilford progeria syndrome (HGPS), also known
as progeria. HGPS is a rare genetic disorder causing accelerated aging in children. It features
symptoms such as growth failure, hair loss, distinct facial features, and severe cardiovascular issues.
• The absence of photosensitivity can aid in ruling out progeria
Option D - FBN1 gene mutation:

Page 12

71
• Mutations in the FBN1 gene are associated with Marfan syndrome. Marfan syndrome is a genetic
disorder that affects the body's connective tissue, resulting in abnormalities in the skeletal,
cardiovascular, and ocular systems. People with Marfan syndrome tend to be tall with long limbs and
may have cardiovascular issues, such as aortic enlargement.

Solution for Question 8:


Correct Option C- Sudan black:
• Sudan black is a fat-soluble dye.
• It stains lipids such as sterols, neutral fats, and phospholipids.
• It can be used to detect lipids in a tissue sample.
• Secondary granules of myelocytic and lysosomal granules of monocytic cells can be stained using
this dye.
• On microscopic examination, varying degrees of black-coloured pigments are seen in the positive
reaction.
• In acute lymphocytic leukaemia, less than 3% of blast cells show a positive reaction.
Incorrect Options:
Option A- Periodic Acid Schiff (PAS):
• Periodic acid–Schiff (PAS) is a staining method used to detect polysaccharides like glycogen, and
glycoproteins.
• PAS stain can detect fungi (Candida, Histoplasma, Cryptococcus, and Blastomycosis), as their cell
walls have high polysaccharide content. It doesn't detect lipids
Option B- Myeloperoxidase:
• Myeloperoxidase is a lysosomal enzyme in the azurophilic granules of neutrophils, eosinophils, and
monocytes.
• Myeloperoxidase stain is useful for differentiating the blast cells of acute myeloid leukaemia from
those of acute lymphoblastic leukaemia.
Option D- Mucicarmine:
• Mucicarmine stain is intended for the staining of mucin.
• It is specific for mucin of epithelial origin.

Solution for Question 9:


Correct Option C- Intermediate filaments:
• Mallory bodies (MB), also known as Mallory-Denk bodies (MDB), are cytoplasmic hyaline inclusions of
hepatocytes comprising intermediate cytoskeletal filaments.
• Mallory bodies are seen- Alcoholic hepatitis, nonalcoholic steatohepatitis (NASH), cholestatic liver
diseases, primary biliary cirrhosis (PBC) and hepatocellular carcinoma.

Page 13

72
Incorrect Options:
Option A- α1-Antitrypsin:
• Alpha anti-trypsin is a protein that is produced in the liver.
• It protects the body tissues from being damaged by infection-fighting agents released by the immune
system.
Option B- β-Amyloid (Aβ):
• Beta-amyloid peptide is a 42 amino acid peptide.
• It plays a key role in the pathogenesis of Alzheimer's disease.
Option D- Prion protein (PrP):
• Prions are the type of misfolded proteins.
• They characterize several transmissible neurodegenerative diseases in animals as well as humans
such as CJD (Creutzfeldt-Jacob disease).

Solution for Question 10:


Correct Option A- Mitochondria:
• The cell undergoes calcium buildup regardless of the form of the initial injury, such as ischemia or
direct disruption to the cell membrane.
• This is because the cell will either have an altered energy metabolism or plasmalemma alterations,
which can cause calcium to accumulate. This increased intracellular calcium concentration is
responsible for cytoskeletal alterations, which change the structure of the cell; the activation of
phospholipases, which leads to the maintenance of membrane damage; and ultimately, calcification of
the mitochondria. These changes are brought about as a result of the cell's structure being altered.
• The accumulation of calcium at the sites of cell death can result in pathologic calcification. Intracellular
calcification typically starts in the mitochondria of calcium-accumulating cells that have either died or
are in the process of dying.
Incorrect Options:
Option B - Golgi body:
• Intracellular calcification does not begin in the Golgi body.
Option C- Lysosomes:
• Intracellular calcification does not begin in lysosomes.
Option D- Nucleus:
• Intracellular calcification does not begin in the nucleus.

Solution for Question 11:


Correct Option D: Lipofuscin:

Page 14

73
• Lipofuscin is an insoluble pigment known as Lipochrome or Wear and tear pigment. Derived from the
Latin word meaning brown lipid.
• It's a sign of free radical injury and lipid peroxidation.
• The sectioned tissue appears yellow-brown, finely granular cytoplasmic, often perinuclear pigment.
• It is seen in ageing cells and is particularly prominent in the liver and heart of ageing patients.
• Also seen in patients with severe malnutrition and cancer cachexia.

Incorrect Options:
Option A- Melanin:
• A pigmented polymer is produced by melanocytes, stored in organelles called melanosomes, and
then transferred to keratinocytes. Protects against ultraviolet radiation and determines the colour of the
skin and iris.
Option B- Hemosiderin:
• Hemosiderin, a hemoglobin-derived golden yellow to brown granular, is a storage form of iron.
• It is probably derived from ferritin by lysosomal proteolysis
Option C- Bilirubin:
• A hemoglobin breakdown product and parameter of cholestasis. Serum total bilirubin consists of
unconjugated bilirubin and conjugated bilirubin.
• After iron removal from hemoglobin, the heme moiety is converted to biliverdin (green bile) and then to
bilirubin (red bile).

Solution for Question 12:


Correct Option D- Hemosiderin:
• Hemosiderin is an iron storage complex produced during the breakdown of ferritin and phagocytosed
haemoglobin (e.g., after red blood cell destruction).

Page 15

74
• It may be found in various tissues (e.g., alveolar macrophages, liver parenchyma) or urine, depending
on the underlying pathology (e.g., pulmonary haemorrhage, hemochromatosis, and intravascular
hemolysis).
Incorrect Options:
Option A- Bilirubin:
• A haemoglobin breakdown product and parameter of cholestasis. Serum total bilirubin consists of
unconjugated bilirubin and conjugated bilirubin.
• It does not stain with Prussian blue.
Option B- Haptoglobin:
• A plasma glycoprotein that binds to free haemoglobin, resulting in its clearance. Hemolysis (especially
intravascular hemolysis) results in decreased free, circulating haptoglobin concentrations. It does not
stain with Prussian blue.
Option C: Hemoglobin:
• An iron-containing oxygen transport protein found in red blood cells. Reference ranges vary by
source. Elevated in polycythemia vera and decreased in anaemia. It does not stain with Prussian blue.

Solution for Question 13:


Correct Option C- Psammoma bodies:
• The patient's history of occupational asbestos exposure and the presentation of pleural thickening and
calcified nodules are indicative of mesothelioma, a malignant neoplasm commonly associated with
asbestos exposure. Microscopic examination of mesothelioma often reveals spindle-shaped cells
forming papillary structures and frequently demonstrates psammoma bodies, which are concentric,
lamellated, calcific structures. These structures are commonly observed in serous tumors, such as
papillary carcinoma of the thyroid and meningiomas, and can also be found in malignant mesothelioma.
Their presence helps in the histological diagnosis of mesothelioma.

Incorrect Options:

Page 16

75
Option A- Auer rods: Auer rods are needle-like, azurophilic inclusions found in the cytoplasm of myelob
lasts or immature myeloid cells. They are characteristic of acute myeloid leukemia (AML) and are not ty
pically seen in mesothelioma.
Option B- Lewy bodies: Lewy bodies are abnormal protein aggregates, specifically alpha-synuclein, fou
nd in the cytoplasm of neurons. They are a pathological hallmark of Parkinson's disease and other neu
rodegenerative disorders but are not associated with mesothelioma.
Option D- Call-Exner bodies: Call-Exner bodies are small, eosinophilic, glandular structures found in gr
anulosa cell tumors of the ovary. These structures are not associated with mesothelioma.

Solution for Question 14:


Correct Option C- Crooke hyaline change:
• Crooke hyaline change refers to the alterations in pituitary cells observed in Cushing's disease,
especially due to chronic exposure to elevated adrenocorticotropic hormone (ACTH) levels. It is
characterized by cells with a perinuclear halo or clearing in the cytoplasm, indicative of this pathology.

Incorrect Options:
Option A - Ghon focus:
• Ghon focus is a term used in tuberculosis. It refers to a primary lesion caused by Mycobacterium
tuberculosis seen in the lung. The primary Ghon focus is the initial site of infection and often occurs in
the lower lobes of the lung. An example would be the Ghon focus along with associated lymph node
involvement, which is referred to as the Ghon complex.
Option B - Russell bodies:
• Russell bodies are eosinophilic, spherical inclusions found in plasma cells. They are seen in various
conditions, including multiple myeloma and some autoimmune diseases. In multiple myeloma, these
bodies are an accumulation of immunoglobulins within the plasma cells and are a characteristic
histological feature.
Option D- Caroli disease:

Page 17

76
• Caroli disease is a rare congenital disorder characterized by non-obstructive saccular or fusiform
dilatation of the intrahepatic bile ducts. It can present with recurrent episodes of cholangitis, abscess
formation, and increased risk of bile duct stones.

Solution for Question 15:


Correct Option D - Catalase:
Reactive oxygen species-mediated killing of the microbes:
• ROS are produced by the rapid assembly and activation of a multicomponent oxidase, NADPH
oxidase (also called phagocyte oxidase), which oxidises reduced nicotinamide-adenine dinucleotide
phosphate (NADPH) and, in the process, reduces oxygen to superoxide anion (O2 • ).
• In neutrophils, this oxidative reaction is triggered by activating signals accompanying phagocytosis
and is called the respiratory burst.
• Phagocyte oxidase is an enzyme complex consisting of at least seven proteins. Different enzyme
components are located in the plasma membrane and the cytoplasm in resting neutrophils.
• In response to activating stimuli, the cytosolic protein components translocate to the phagosomal
membrane, where they assemble and form the functional enzyme complex.
• Thus, the ROS are produced within the phagolysosome, where they can act on ingested particles
without damaging the host cell.
• O2 • is converted into hydrogen peroxide (H2O2), mostly by spontaneous dismutation. H2O2 is not
able to kill microbes efficiently by itself.
• However, the azurophilic granules of neutrophils contain the enzyme myeloperoxidase (MPO), which,
in the presence of a halide such as Cl−, converts H2O2 to hypochlorite (HOCl■), the active ingredient
in household bleach.
• The latter is a potent antimicrobial agent that destroys microbes by halogenation (in which the halide
is bound covalently to cellular constituents) or by oxidation of proteins and lipids (lipid peroxidation).
• The H2O2-MPOhalide system is the most potent bactericidal system of neutrophils. Inherited
deficiency of MPO leads to a minimal increase in susceptibility to infection.
• H2O2 is also converted to hydroxyl radical (■OH), another powerful destructive agent
Incorrect Options:
Option A - Nitric oxide:
• In macrophages, NO reacts with superoxide (O2 • ) to generate the highly reactive free radical
peroxynitrite (ONOO− ). These nitrogen-derived free radicals, similar to ROS, attack and damage the
lipids, proteins, and nucleic acids of microbes.
Option B - Fenton's reaction:
• Fenton's reaction is involved in the production of reactive oxygen species.
• It involves a reaction between H2O2 and iron molecules to generate OH free radicals
Option C - NADPH oxidase:
• NADPH oxidase is part of the oxidative burst mechanism that kills the microbes and generates oxygen
free radicals.

Page 18

77
Solution for Question 16:
Correct Option D - Peroxynitrite is converted to nitrite by glutathione peroxidase:
• Cells have mechanisms to remove free radicals and minimize their harmful effects.
• Free radicals are highly unstable and decay spontaneously.
• However, nonenzymatic and enzymatic systems (free radical scavengers) inactivate free radicals.
• These enzymes include: Superoxide dismutase Glutathione peroxidase Catalase Several enzymes in
mitochondria and cytosol
• Superoxide dismutase
• Glutathione peroxidase
• Catalase
• Several enzymes in mitochondria and cytosol
• Endogenous or exogenous oxidants such as Vitamin E, A, C and beta-carotene also have a role in
blocking the formation of free radicals or scavenging them after their formation.
• Peroxynitrite is converted to nitrite by enzymes in the mitochondria and cytosol.
• Superoxide dismutase
• Glutathione peroxidase
• Catalase
• Several enzymes in mitochondria and cytosol
Incorrect Options:
Option A - Superoxide is converted to hydrogen peroxide and oxygen by superoxide dismutase:
• Superoxide is formed by incomplete oxygen reduction in the process of mitochondrial oxidative
phosphorylation and phagocyte oxidase in leukocytes.
• It is converted to hydrogen peroxide and oxygen by superoxide dismutase.
Option B - Hydrogen peroxide is converted to water by glutathione peroxidase:
• Glutathione peroxidases are a family of enzymes responsible for cellular protection from oxidative
damage.
• The most abundant enzyme of this family is glutathione peroxidase 1 (in the cytoplasm of all cells).
• It converts hydrogen peroxide to water.
• Catalase is another enzyme which is responsible for hydrogen peroxide degradation. It degrades
hydrogen peroxide to water and oxygen.
Option C - Hydroxyl radical is converted to water by glutathione peroxidase:
• Hydroxyl radical is formed by hydrogen peroxide, water, and oxygen through various reactions.
• It is converted to water by glutathione peroxidase.

Page 19

78
Solution for Question 17:
Correct Option: C-1-D, 2-C, 3-A, 4-B:

Solution for Question 18:


Correct Option D- Diphtherial myocarditis:
• The tabby cat effect, also known as the tiggered effect, refers to a distinctive pattern of irregular bands
or patches of pale myocardium due to fat deposition interspersed with areas of normal coloration. This
pattern resembles the coat of a tabby cat. It is typically observed in diphtherial myocarditis.
Incorrect Options:
Options A, B, and C- Rheumatic heart disease, Infective endocarditis &
Pericarditis: These are not responsible for causing the tabby cat effect.

Solution for Question 19:


Correct Option D:

• The image given in option D shows Crooke hyaline changes. These changes are seen in the pituitary
gland and are caused by Cushing's disease.
Incorrect Options:
Option A:

Page 20

79
• Shows Mallory Denk bodies. They are commonly seen in patients with alcoholic liver disease.
Option B:

• Shows intracytoplasmic Russel bodies. It is caused by increased antibody production.


Option C:

Page 21

80
• Shows intranuclear Dutcher bodies. It is caused by increased antibody production.

Solution for Question 20:


Correct Option D- Seen in Monckeberg sclerosis:
• Monckeberg's sclerosis is a vascular lesion characterized by calcification of the media of the muscular
arteries. It shows dystrophic calcification rather than metastatic calcification.
Incorrect Options:
Options A, B, and C (Elevated blood calcium levels, Most commonly affects lungs &
Normal tissue can also be seen): These are features of metastatic calcification.

Solution for Question 21:


Correct Option D- AZAN stain:
• The AZAN stain, also known as Azocarmine Aniline Blue Stain, is used to differentiate between
mineralized bone and osteoid. AZAN stain stains the osteoid (unmineralized bone matrix) pink to red
and the mineralized bone (calcified matrix) blue to green.
Incorrect Options:
Option A- Von Kossa stain: The Von Kossa stain is used to detect calcium deposits in tissues.
Option B- Alizarin red S stain: Alizarin red S stain also detects minute amounts of calcium deposits.
Option C- Calcein stain: Calcein is a fluorescent metal indicator for calcium with EDTA titration

Page 22

81
Solution for Question 22:
Correct Option C- Chaperone-mediated autophagy:
• Chaperone-mediated autophagy (CMA) is a selective form of autophagy where specific proteins are
targeted for degradation. Heat shock proteins (HSPs), particularly HSP70, play a crucial role in
facilitating the degradation process in CMA.
Incorrect Options:
Option A- Macroautophagy: Macroautophagy involves the formation of autophagosomes. It does not in
volve heat shock proteins.
Option B- Microautophagy: Microautophagy involves direct uptake by lysosomes via endocytosis.
Option D- Mitophagy: Mitophagy is a type of macro-autophagy targeted towards the mitochondria.

Solution for Question 23:


Correct Option B- ATG16L:
• ATG16L1 is a gene involved in the autophagy process. Mutation in the ATG16L1 gene has been
implicated in Crohn's disease, a type of inflammatory bowel disease.
Incorrect Options:
Option A- ATG1: ATG1 is the most essential gene of autophagy, but its mutation is not associated with
Crohn’s disease.
Option C- ATG5: ATG5 mutation increases the susceptibility of tuberculosis.
Option D- ATG3: ATG3 mutation is not associated with Crohn’s disease.

Solution for Question 24:


Correct Option D-1, 2, 3 and 4:

Solution for Question 25:


Correct Option B- Glutathione peroxidase Type 4:
• Glutathione peroxidase Type 4 (GPX4) is the master regulator in the process of ferroptosis.
Incorrect Options:
Options A, C, and D (Iron ions, Cytoplasmic SOD &
Mitochondrial SOD):These are not the master regulators of ferroptosis.

Page 23

82
Previous Year Questions
1. Differential expression of a gene depending upon the parent of origin is a feature of:
A. Gonadal mosaicism
B. Genomic imprinting
C. Anticipation
D. Incomplete penetrance
----------------------------------------
2. Which of the options listed below is not included as a constituent of Papanicolaou stain?
A. Light green SF
B. Orange G
C. Eosin B
D. Hematoxylin
----------------------------------------
3. What is the mechanism for healing in the case of a 57-year-old male with a gradually developing
swelling over the scalp region, pus discharge, and pain?
A. Granuloma formation
B. Scab formation
C. Granulation tissue
D. Neovascularization
----------------------------------------
4. Choose the correct statement regarding the telomerase theory of ageing:
A. Telomere stability is associated with ageing
B. Abnormal telomerase activation is associated with ageing
C. Decreased telomere length is associated with ageing
D. Increased telomere length is associated with ageing
----------------------------------------
5. Which of the following measures is associated with an increased life span?
A. Regular moderate exercise for 30 min
B. Decrease stress
C. Decreasing calorie intake by 30%
D. Pharmacological intervention with proton pump inhibitors
----------------------------------------
6. Metastatic calcification occurs in all except?
A. Kidney
B. Fundus of the stomach

83
C. Pulmonary veins
D. Atheroma
----------------------------------------
7. Identify the cell marked in the image below.

A. Mast cell
B. Macrophage
C. Plasma cell
D. Fibroblast
----------------------------------------
8. Cyclooxygenase plays a role in which pathway?
A. Kreb’s cycle
B. Prostaglandin
C. Leukotriene
D. Vitamin K
----------------------------------------
9. Cell-in-cell appearance is seen in which of the following?
A. Necrosis
B. Apoptosis
C. Necroptosis
D. Entosis
----------------------------------------
10. Please pair the following stains with their corresponding options: 1. Prussian blue stain a. Iron 2.
PAS stain b. Glycogen 3. Congo red stain c. Leprosy 4. Fite-Faraco stain d. Amyloid
1. Prussian blue stain a. Iron
2. PAS stain b. Glycogen
3. Congo red stain c. Leprosy
4. Fite-Faraco stain d. Amyloid

A. 1-d, 2-c, 3-b, 4-a

Page 2

84
B. 1-c, 2-a, 3-b, 4-d
C. 1-a, 2-b, 3-d, 4-c
D. 1-b, 2-a, 3-c, 4-d
----------------------------------------
11. Which of the following is true regarding sirtuins? Increases insulin sensitivity and, thus, glucose
metabolism Promotes genes that increase longevity They are of 7 types They are one of the types of
histone deacetylases
A. 1, 2, 3
B. 1, 2, 3, 4
C. 2, 3
D. 2, 4
----------------------------------------
12.
(or)
Match the following cells with their locations: 1. Kupffer cells a. Stomach 2. Parietal cells b. Liver 3.
Dust cells c. Alveoli 4. Clara cells d. Bronchioles
A. 1-a, 2-b, 3-c, 4-d
B. 1-c, 2-d, 3-a, 4-b
C. 1-d, 2-a, 3-d, 4-c
D. 1-b, 2-a, 3-c, 4-d
----------------------------------------
13. Which of the following is an anti-apoptotic gene?
A. BAK
B. MCL1
C. BIN
D. NOX-Q
----------------------------------------
14. A 70-year-old patient presented with cough, fatigue, and weight loss. He was diagnosed with
squamous cell carcinoma on bronchoscopy. The resected specimen had a hilar lymph node that was 1
cm in size and showed a black pigment. What is the black pigment likely to be?
A. Anthracotic pigment
B. Melanin
C. Lipochrome
D. Hemosiderin
----------------------------------------
15. A pap smear taken from a patient revealed hyperchromatic nuclei, irregular nuclear borders, and a
low maturation index involving the whole thickness but did not breach the basement membrane. What
is the diagnosis?

Page 3

85
A. Dysplasia
B. Carcinoma
C. Metaplasia
D. Hyperplasia
----------------------------------------
16. Which of the following ions is involved in cell injury?
A. Calcium
B. Copper
C. Selenium
D. Zinc
----------------------------------------
17. Liquefactive necrosis is seen in which of the following?
A. Heart
B. Brain
C. Lungs
D. Kidney
----------------------------------------
18. Premature aging in Werner syndrome is due to:
A. Increased telomerase activity
B. Defect in DNA helicase
C. Increase in length telomere due to decrease in telomerase activity
D. Decreased telomerase activity
----------------------------------------
19. The lining epithelium of the respiratory tract of a middle-aged man who is a chronic smoker shows a
change from columnar to stratified squamous epithelium, as depicted in the image below. The change
is called:

A. Dysplasia
B. Hypertrophy

Page 4

86
C. Atrophy
D. Metaplasia
----------------------------------------
20. Identify the anti-apoptotic factor among the following:
A. p53
B. KRAS
C. Bcl-2
D. BAK
----------------------------------------
21. What is the type of necrosis seen in the vessel wall of a patient with polyarteritis nodosa?
A. Fibrinoid necrosis
B. Gangrenous necrosis
C. Fat necrosis
D. Caseous necrosis
----------------------------------------
22. Which lymphoma is strongly associated with overexpression of Bcl-2?
A. Follicular lymphoma
B. Burkitt lymphoma
C. Diffuse large cell lymphoma
D. Chronic lymphocytic leukemia
----------------------------------------
23. Among the following, which molecule is required for the presentation of the antigens by the APCs to
TH2 cells?
A. NK cells
B. G-protein coupled receptor
C. MHC
D. Lectin receptor
----------------------------------------
24. Which of the following is a true statement about apoptosis?
A. ATP-dependent programmed cell death
B. Intrinsic and extrinsic pathways; both pathways activate interleukins
C. Karyolysis is present
D. Cell membrane do not remain intact
----------------------------------------
25. What type of calcification is likely to be seen in a 58-year-old man who died 24 hours after
experiencing substernal chest pain radiating to the left arm, with an autopsy revealing an obstructing

Page 5

87
atheromatous plaque in the left anterior descending artery?
A. Dystrophic with increased serum calcium
B. Metastatic with increased serum calcium
C. Dystrophic with normal serum calcium
D. Metastatic with normal serum calcium
----------------------------------------
26. If DNA is damaged in the cell cycle, which gene causes cell cycle arrest?
A. RB
B. Myc
C. p53
D. KRAS
----------------------------------------
27. Which of the following is a sign of irreversible cell injury?
A. Amorphous densities in mitochondria
B. Bleb formation
C. Loss of microvilli
D. Cell swelling
----------------------------------------
28. Which of the following caspases binds to Cyt c and the APAF-1 complex?
A. Caspase 1
B. Caspase 8
C. Caspase 9
D. Caspase 4
----------------------------------------
29. The body of an 82-year-old man has been seen for a post-mortem to determine the cause of death.
You are examining a histopathological slide containing a section of his cardiac tissue where you notice
some pigmentation. Which is the substance that is seen in the image given below?

A. Melanin

Page 6

88
B. Lipofuscin
C. Hemosiderin
D. Iron
----------------------------------------
30. All of the following are signs of reversible cell injury, except:
A. Loss of microvilli
B. Cell swelling
C. Bleb formation
D. Dense mitochondrial deposit
----------------------------------------

Correct Answers
Question Correct Answer

Question 1 2
Question 2 3
Question 3 3
Question 4 3
Question 5 3
Question 6 4
Question 7 1
Question 8 2
Question 9 4
Question 10 3
Question 11 2
Question 12 4
Question 13 2
Question 14 1
Question 15 1
Question 16 1
Question 17 2
Question 18 2
Question 19 4
Question 20 3
Question 21 1
Question 22 1

Page 7

89
Question 23 3
Question 24 1
Question 25 3
Question 26 3
Question 27 1
Question 28 3
Question 29 2
Question 30 4

Solution for Question 1:


Correct Option B - Genomic imprinting:
• Genomic imprinting is the regulation of genes whose expression depends on whether they are
maternally or paternally inherited
Incorrect Options:
Option A - Gonadal mosaicism: It refers to the phenomenon where different cells in the gonads of an in
dividual have different genetic makeups. This can result in offspring with genetic abnormalities even if t
he parents do not have them.
Option C - Anticipation: It is a phenomenon where genetic disorders manifest at an earlier age and with
more severe symptoms in successive generations due to the expansion of unstable trinucleotide repe
ats.
Option D - Incomplete penetrance: It refers to the phenomenon where individuals with the same geneti
c mutation do not necessarily express the corresponding phenotype. This can be due to environmental
or other genetic factors that modify the expression of the gene.

Solution for Question 2:


Correct option:
Option C: Eosin B:
• Eosin B is a counterstain that stains the cytoplasmic components of cells pink or red. It provides
additional contrast and helps distinguish cell boundaries.
• Papanicolaou stain, commonly known as the Pap stain, is a special stain used in cytology to
differentiate cells and identify abnormalities, particularly in Pap smears for cervical cancer screening.
• In the context of the Pap stain, it is not used; instead, the stain relies on the specific combination of
light green SF, orange G, and Hematoxylin for cell staining.
Incorrect Option:
Option A: Light green SF: This is a
counterstain used in the Pap stain. It stains the cytoplasm of cells and helps visualize cell morphology.
Option B: Orange G: Orange G is another counterstain used in the Pap stain. It stains the cytoplasm as
well and enhances the contrast of the stained cells.

Page 8

90
Option D: Hematoxylin: Hematoxylin is a component of the Papanicolaou stain. Hematoxylin is a nucle
ar stain that is often used in combination with eosin (Hematoxylin and Eosin stain, or H&E; stain) to stai
n cell nuclei blue-purple.

Solution for Question 3:


Correct option: C
• In this case, the mechanism for healing is granulation tissue formation. Granulation tissue is a type of
connective tissue that forms during the healing process of a wound. It is composed of new blood
vessels (angiogenesis), fibroblasts, and an extracellular matrix. Granulation tissue fills the wound
defect and helps in the formation of new tissue. It appears as red, soft, and granular tissue on the
wound surface.
Incorrect options:
Option A: Granuloma formation: Granuloma formation is a type of chronic inflammation characterized b
y the aggregation of immune cells, typically macrophages, in response to persistent or resistant antige
ns. It is not the primary mechanism involved in the healing of the described wound.
Option B: Scab formation: Scab formation occurs during the initial stages of wound healing when blood
components, such as fibrin and platelets, dry and form a protective crust over the wound. It is not the p
rimary mechanism involved in the healing process after incision and drainage of an infected scalp swell
ing.
Option D: Neovascularization: Neovascularization refers to the formation of new blood vessels. It is an i
mportant step in wound healing, but in the context of the described case, granulation tissue formation i
s a more relevant mechanism for healing.

Solution for Question 4:


Correct Option C - Decreased telomere length is associated with ageing:
• Telomerase theory of ageing: Cellular ageing and the emergence of age-related disorders are linked
to the steady shortening of telomeres.
• The telomerase enzyme may prolong telomeres, but as people age, its activity decreases.
• Telomeres can no longer shield the chromosome from harm as they get shorter and reach a critical
length, which causes cell death.
• Telomere length loss is linked to aging and diseases of the elderly.
Incorrect Options:
Option A - Telomere stability is associated with ageing: Telomere stability is not associated with ageing
as telomere length shortens with age. This progressive shortening of telomeres causes senescence, a
poptosis, or oncogenic transformation of somatic cells, thereby impacting the health and lifespan of an i
ndividual.
Option B - Abnormal telomerase activation is associated with ageing: Telomerase adds nucleotides to t
he telomere of the chromosome and keeps them alive, thereby slowing the ageing process.

Page 9

91
Option D - Increased telomere length is associated with ageing: Increasing telomere length helps in ma
intaining healthy cells, decreases apoptosis and senescence, and slows the ageing process.

Solution for Question 5:


Correct Option C - Decreasing calorie intake by 30%:
• Reducing calorie intake by 30% can increase life expectancy.
Incorrect Options:
Option A, B, and D are incorrect.

Solution for Question 6:


Correct Option D - Atheroma:
• The type of calcification seen in atheroma is dystrophic calcification.
• Metastatic calcification: Metastatic calcification is caused by increased calcium phosphate in the blood
and may result from hypercalcemia, hyperphosphatemia, or both. It is commonly associated with
hyperparathyroidism, sarcoidosis, metastatic disease, and myeloma.
• Metastatic calcification is caused by increased calcium phosphate in the blood and may result from
hypercalcemia, hyperphosphatemia, or both.
• It is commonly associated with hyperparathyroidism, sarcoidosis, metastatic disease, and myeloma.
• Dystrophic calcification: The deposition of calcium salt in degenerated tissues in the absence of a
systemic imbalance is called dystrophic calcification. It is mostly associated with trauma, inflammation,
or infection.
• The deposition of calcium salt in degenerated tissues in the absence of a systemic imbalance is called
dystrophic calcification.
• It is mostly associated with trauma, inflammation, or infection.
• Metastatic calcification is caused by increased calcium phosphate in the blood and may result from
hypercalcemia, hyperphosphatemia, or both.
• It is commonly associated with hyperparathyroidism, sarcoidosis, metastatic disease, and myeloma.
• The deposition of calcium salt in degenerated tissues in the absence of a systemic imbalance is called
dystrophic calcification.
• It is mostly associated with trauma, inflammation, or infection.
Incorrect Options:
• Metastatic calcification occurs in options A, B, and C.

Solution for Question 7:

Page 10

92
Correct Option A - Mast cell:
• The cell marked in the image is a mast cell.
• The mast cells are stained with toluidine blue.
Incorrect Options:
Options B, C, and D are incorrect.

Solution for Question 8:


Correct Option B - Prostaglandin:
• Cyclooxygenase is an enzyme that catalyzes the conversion of arachidonic acid into prostaglandins.
• There are two forms of this enzyme: COX-1 and COX-2.
• Prostaglandins are lipid compounds that play important roles in inflammation, pain, fever, regulation of
blood flow, and other functions.
Incorrect Options:
• Options A, C, and D are incorrect.

Solution for Question 9:


Correct Option D - Entosis:
• Cell-in-cell appearance is a phenomenon where one cell is engulfed by another cell.
• Entosis is a process where a live cell actively engulfs another live cell, resulting in a cell-in-cell
appearance.
• Entosis can occur during normal development and tissue homeostasis, but it has also been implicated
in cancer progression.
• In cancer, entosis can lead to the formation of giant multinucleated cells, which may promote tumor
growth and invasion.
Incorrect Options:
• Options A, B, and C are not associated with cell-in-cell appearance.

Solution for Question 10:


Correct Option C - 1-a, 2-b, 3-d, 4-c:
Incorrect Options:
• Option A, B, and D are incorrect. Refer to the explanation of the correct answer.

Page 11

93
Solution for Question 11:
Correct Option B - 1, 2, 3, 4:
• Sirtuins have been shown to play a role in regulating insulin sensitivity and glucose metabolism.
• They can modulate cellular responses to changes in nutrient availability and energy status, including
the regulation of glucose homeostasis.
• Sirtuins have been linked to the regulation of lifespan and longevity.
• Sirtuins are involved in various cellular processes that impact ageing, such as DNA repair, stress
response, and mitochondrial function.
• In humans, there are seven sirtuin proteins, referred to as SIRT1 to SIRT7.
• Each sirtuin protein has distinct cellular locations and functions.
• Sirtuins are a subgroup of histone deacetylases (HDACs).
• Unlike other HDACs (class I and class II HDACs), which use zinc as a cofactor, sirtuins require
nicotinamide adenine dinucleotide (NAD+) as a cofactor for their enzymatic activity.
Incorrect Options:
Options A, C, and D are incorrect.

Solution for Question 12:


Correct Option D - 1-b, 2-a, 3-c, 4-d:
Incorrect Options:
• Option A, B, and C are incorrect.

Solution for Question 13:


Correct Option B - MCL1:
• MCL1 is an anti-apoptotic gene belonging to the BCL-2 family.
• It plays a crucial role in cell survival by inhibiting apoptosis.
• MCL1 promotes cell survival by preventing the release of cytochrome c from mitochondria and
inhibiting the activation of caspases.
Incorrect Options:
Option A - BAK:
• BAK is a pro-apoptotic gene belonging to the BCL-2 family.
• It promotes apoptosis by promoting mitochondrial outer membrane permeabilization.
Option C - BIN:
• BIN is not directly associated with apoptosis.

Page 12

94
• It is involved in vesicular trafficking and signal transduction but does not have a direct role in
regulating cell death.
Option D - NOX-Q:
• NOX-Q is not a known gene associated with apoptosis.
• The term NOX typically refers to NADPH oxidases, which are enzymes involved in generating reactive
oxygen species (ROS).
• ROS can have both pro-apoptotic and anti-apoptotic effects depending on the context, but NOX-Q is
not specifically implicated in apoptosis.

Solution for Question 14:


Correct Option A - Anthracotic pigment:
• Anthracotic pigment, also known as carbon pigment, is a type of black pigment that is commonly seen
in individuals exposed to environmental pollutants, particularly inhaled particles of carbonaceous
material such as soot or coal dust.
• The pigment accumulates in the lungs and can be transported to the regional lymph nodes, including
the hilar lymph nodes.
• It is typically asymptomatic but can be observed on pathological examination.
Incorrect Options:
Option B - Melanin: Melanin is a
brown-black pigment responsible for determining the color of the skin, hair, and eyes.

Option C - Lipochrome: Lipochrome refers to yellow-brown pigments derived from lipid oxidation.

Page 13

95
Option D - Hemosiderin:
• Hemosiderin is a brown pigment that results from the breakdown of hemoglobin and the accumulation
of iron in tissues.
• While hemosiderin deposition can occur in various organs and tissues, it does not typically manifest
as a black pigment and is not commonly observed in hilar lymph nodes.

Solution for Question 15:


Correct Option A - Dysplasia:
• Dysplasia is characterized by disordered growth and maturation of cells, leading to architectural and
cytological abnormalities.

Page 14

96
• The presence of hyperchromatic nuclei, irregular nuclear borders, and a low maturation index are
indicative of dysplastic changes.
Incorrect Options:
Option B - Carcinoma:
• Carcinoma is a more advanced stage of neoplastic progression characterized by invasion of the
basement membrane and surrounding tissues.
Option C - Metaplasia:
• Metaplasia refers to the reversible change in cell type from one mature cell type to another in
response to certain stimuli or chronic irritation.
• It does not necessarily involve dysplastic changes or progression to malignancy.
Option D - Hyperplasia:
• Hyperplasia refers to an increase in the number of cells in an organ or tissue, resulting in increased
tissue mass.
• It may be a response to physiological or pathological stimuli.

Solution for Question 16:


Correct Option A - Calcium:
• Calcium ions play a significant role in cell injury.
• Increased intracellular calcium levels can trigger several damaging processes within the cell.
• Excessive calcium influx into the cell can lead to mitochondrial dysfunction, activation of enzymes that
degrade cellular components, and disruption of cell membrane integrity.
• These processes can ultimately result in cell death.
Incorrect Options:
• Options B, C, and D are incorrect.

Solution for Question 17:


Correct Option B - Brain:
• Liquefactive necrosis is characterized by the dissolution of affected cells, resulting in a liquid, viscous
appearance.
• It typically occurs in organs with abundant enzyme activity and limited structural support, such as the
brain.
• In the brain, this form of necrosis can be observed in cases of cerebral infarction or abscess
formation.
• The liquefied material is often composed of dead cells, tissue debris, and inflammatory cells.
Incorrect Options:

Page 15

97
Option A - Heart: The most common type of cell death seen in the heart is coagulative necrosis.
Option C - Lungs: The lungs are prone to different types of necrosis, including coagulative necrosis in r
esponse to infarction and caseous necrosis in certain infections like tuberculosis.
Option D
- Kidney: Renal tissue is susceptible to ischemic injury, leading to coagulative necrosis in most cases.

Solution for Question 18:


Correct Option B - Defect in DNA helicase:
• Werner syndrome is a rare genetic disorder characterized by premature ageing.
• It is caused by a mutation in the WRN gene, which encodes a DNA helicase enzyme.
• DNA helicase is involved in unwinding the DNA double helix during DNA replication and repair
processes.
• The defect in DNA helicase function in Werner syndrome results in genomic instability, DNA damage
accumulation, and premature ageing.
Incorrect Options:
Options A, C, and D are incorrect. Refer to the explanation of the correct answer.

Solution for Question 19:


Correct Option D - Metaplasia:
• Metaplasia is a reversible change in which one mature cell type is replaced by another mature cell
type.
• It is often seen as an adaptive response to chronic irritation, inflammation, or other pathological
conditions.
• In chronic smokers, the constant exposure to smoke irritants can lead to the replacement of the
normal columnar respiratory epithelium with stratified squamous epithelium, which is a protective
response.
Incorrect Options:
Option A - Dysplasia:
• Dysplasia refers to the abnormal growth and development of cells or tissues.
• It is characterized by disorganized cell morphology and cellular changes that can be precursors to
cancer.
Option B - Hypertrophy:
• Hypertrophy refers to an increase in the size of cells, resulting in the enlargement of an organ or
tissue.
• It is typically a response to increased workload or stimulation.
Option C - Atrophy:

Page 16

98
• Atrophy refers to a decrease in the size or number of cells, resulting in a reduced organ or tissue size.
• It can occur due to disuse, loss of innervation, or an inadequate blood supply.

Solution for Question 20:


Correct Option C - Bcl-2:
• Bcl-2 (B-cell lymphoma 2) is an anti-apoptotic protein that plays a key role in regulating cell survival.
• It functions by inhibiting apoptosis.
• Bcl-2 prevents the release of cytochrome c from the mitochondria, which is a crucial step in the
initiation of apoptosis.
• By inhibiting cytochrome c release, Bcl-2 helps to maintain cell survival and prevent apoptosis.
Incorrect Options:
Option A - p53:
• p53 is an apoptotic factor.
• p53 is a tumor suppressor protein that plays a role in promoting apoptosis in response to DNA
damage or other cellular stresses.
Option B - KRAS:
• KRAS is an oncogene that is involved in promoting cell proliferation and survival.
Option D - BAK:
• BAK is a pro-apoptotic factor belonging to the Bcl-2 family.
• BAK promotes apoptosis by inducing mitochondrial outer membrane permeabilization, leading to
cytochrome c release and activation of the apoptotic cascade.

Solution for Question 21:


Correct Option A - Fibrinoid necrosis:
• Fibrinoid necrosis is seen in the vessel wall of a patient with polyarteritis nodosa.
• In this condition, immune complexes and inflammatory cells infiltrate the vessel walls, causing
damage and necrosis.
• The deposition of fibrin-like proteinaceous material leads to thickening and an eosinophilic
appearance of the vessel walls.
• Fibrinoid necrosis is a characteristic finding in autoimmune diseases affecting blood vessels.
Incorrect Options:
Option B - Gangrenous necrosis:
• Gangrenous necrosis typically occurs in ischemic or infected tissues.
• It is characterized by dry or wet gangrene, depending on the presence of bacterial infection and tissue
liquefaction.

Page 17

99
Option C - Fat necrosis:
• Fat necrosis occurs in adipose tissue and is often associated with conditions like acute pancreatitis or
trauma.
• It involves the breakdown of fat cells by pancreatic enzymes, leading to the release of fatty acids and
the formation of necrotic areas.
Option D - Caseous necrosis:
• Caseous necrosis is commonly associated with tuberculosis and other granulomatous infections.
• It is characterized by the formation of a granular, cheese-like necrotic material within the affected
tissue.

Solution for Question 22:


Correct Option A - Follicular lymphoma:
• Follicular lymphoma is strongly associated with the overexpression of Bcl-2.
• Bcl-2 is a gene involved in regulating programmed cell death, or apoptosis.
• In follicular lymphoma, there is a genetic translocation [most commonly t(14;18)] that leads to the
overexpression of the Bcl-2 protein.
• This overexpression of Bcl-2 helps the cancer cells evade apoptosis, contributing to their uncontrolled
growth and survival.
Incorrect Options:
Options B, C, and D are not associated with overexpression of Bcl-2.

Solution for Question 23:


Correct Option C - MHC:
• The molecule required for the presentation of antigens by antigen-presenting cells to TH2 cells is
MHC (Major Histocompatibility Complex).
Incorrect Options:
• Options A, B, and D are incorrect.

Solution for Question 24:


Correct Option A - ATP-dependent programmed cell death:
• Apoptosis is a highly regulated process of programmed cell death.
• During apoptosis, specific enzymes, such as caspases, are activated, leading to the enzymatic
digestion of cellular contents, including DNA, proteins, and organelles.
Incorrect Options:

Page 18

100
Option B - Intrinsic and extrinsic pathways; both pathways activate interleukins:
• Intrinsic and extrinsic pathways; both pathways activate caspases.
Option C - Karyolysis is present:
• Karyolysis refers to the dissolution or fading of the cell nucleus, which is more characteristic of
necrosis.
• In apoptosis, the nucleus undergoes condensation and fragmentation.
Option D - Cell membrane do not remain intact:
• The cell membrane typically remains intact without significant inflammation.

Solution for Question 25:


Correct Option C - Dystrophic with normal serum calcium:
• The type of calcification seen in atheroma is dystrophic calcification.
• Metastatic calcification: Metastatic calcification is caused by increased calcium phosphate in the blood
and may result from hypercalcemia, hyperphosphatemia, or both. It is commonly associated with
hyperparathyroidism, sarcoidosis, metastatic disease, and myeloma.
• Metastatic calcification is caused by increased calcium phosphate in the blood and may result from
hypercalcemia, hyperphosphatemia, or both.
• It is commonly associated with hyperparathyroidism, sarcoidosis, metastatic disease, and myeloma.
• Dystrophic calcification: The deposition of calcium salt in degenerated tissues in the absence of a
systemic imbalance is called dystrophic calcification. It is mostly associated with trauma, inflammation,
or infection.
• The deposition of calcium salt in degenerated tissues in the absence of a systemic imbalance is called
dystrophic calcification.
• It is mostly associated with trauma, inflammation, or infection.
• Metastatic calcification is caused by increased calcium phosphate in the blood and may result from
hypercalcemia, hyperphosphatemia, or both.
• It is commonly associated with hyperparathyroidism, sarcoidosis, metastatic disease, and myeloma.
• The deposition of calcium salt in degenerated tissues in the absence of a systemic imbalance is called
dystrophic calcification.
• It is mostly associated with trauma, inflammation, or infection.
Incorrect Options:
• Options A, B, and D are incorrect. Refer to the explanation of the correct answer.

Solution for Question 26:


Correct Option C - p53:
• p53 is a tumor suppressor gene that is often referred to as the guardian of the genome.

Page 19

101
• It plays a crucial role in monitoring DNA integrity and orchestrating cellular responses to DNA
damage.
• When DNA is damaged, p53 is activated and can induce cell cycle arrest, allowing time for DNA repair
or triggering apoptosis if the damage is irreparable.
Incorrect Options:
Option A - RB: The RB gene is involved in regulating the cell cycle by inhibiting the progression from th
e G1 phase to the S phase.
Option B - Myc: The Myc is an oncogene that plays a role in cell growth and proliferation.
Option D - KRAS: The KRAS gene is an oncogene involved in cell signalling pathways that regulate cel
l growth and proliferation.

Solution for Question 27:


Correct Option A - Amorphous densities in mitochondria:
• Mitochondrial changes, such as swelling and dysfunction, are associated with irreversible cell injury.
Incorrect Options:
• Options B, C, and D are associated with reversible cell injury.

Solution for Question 28:


Correct Option C - Caspase 9:
• Caspases are a family of protease enzymes involved in programmed cell death, also known as
apoptosis.
• In the intrinsic apoptotic pathway, the release of cytochrome c (Cyt c) from the mitochondria triggers
the formation of the apoptosome, a complex composed of Cyt c, apoptotic protease-activating factor 1
(APAF-1), and procaspase 9.
• This complex formation is crucial for initiating the caspase cascade and ultimately leading to cell
death.
• Caspase 9 is activated within the apoptosome complex by binding to Cyt c and APAF-1.
• Once activated, caspase 9 cleaves and activates downstream caspases, including caspase 3 and
caspase 7, which then execute the apoptotic process.
Incorrect Options:
• Options A, B, and D do not bind to Cyt c and the APAF-1 complex.

Solution for Question 29:


Correct Option B - Lipofuscin:

Page 20

102
• The histopathological image shows lipofuscin in the section of cardiac tissue.
• Lipofuscin is a yellow to red-brown pigment found in cardiac muscle cells, particularly around the
nucleus in the hearts of elderly, debilitated people.
Incorrect Options:
• Options A, C, and D are incorrect.

Solution for Question 30:


Correct Option D - Dense mitochondrial deposit:
• Mitochondrial changes, such as swelling and dysfunction, are associated with irreversible cell injury.
Incorrect Options:
• Options A, B, and C are associated with reversible cell injury.

Page 21

103
Acute Inflammation
1. Which of the following immune cell assessments is done using a nitroblue tetrazolium test?
A. Polymorphs
B. Complement
C. T cell
D. B cell
----------------------------------------
2. Which of the following locations is most commonly involved in diapedesis?
A. Lymphatic capillaries
B. Postcapillary venules
C. Precapillary arterioles
D. Small dermal arteries
----------------------------------------
3. The electron microscopy image below depicts the rolling of leukocytes on endothelial cells; identify
the protein involved in this process.

A. Selectins
B. Integrins
C. Transferrin
D. PECAM-1
----------------------------------------
4. Among the listed steps in the migration of inflammatory substances, which of the following processes
will occur the earliest ?
A. Chemotaxis
B. Phagocytosis
C. Adhesion
D. Diapedesis
----------------------------------------

104
5. Which of the following is a correct match of enzymes present in their respective neutrophilic
granules?
A. Lactoferrin are present in primary granules
B. Gelatinase is present in secondary granules
C. Acid hydrolase is present in secondary granules
D. Myeloperoxidase is present in primary granules
----------------------------------------
6. What's the earliest transient change in acute inflammation due to bacterial infection in an 8-year-old
with throat pain, fever, hyperemic pharyngeal mucosa, and enlarged exudative tonsils?
A. Eosinophilia
B. Neutrophilia
C. Monocytosis
D. Lymphocytosis
----------------------------------------
7. Which of the following are cardinal signs of inflammation?
A. Redness, heat, swelling, pain
B. Swelling, redness, heat, loss of function
C. Redness, pain, loss of function, swelling
D. Loss of function, heat, pain, swelling
----------------------------------------
8. In a breastfeeding woman with a tender, erythematous area around the left nipple and a thick, yellow
fluid emerging from an open fissure, which inflammatory cell is increased in the fluid sample?
A. B lymphocytes
B. Eosinophils
C. Mast cells
D. Neutrophils
----------------------------------------
9. What is a characteristic feature of the pleural effusion in a 39-year-old woman with pneumococcal
pneumonia and a high specific gravity pleural fluid with numerous polymorphonuclear leukocytes?
A. Fibrinous exudate
B. Chylous exudate
C. Purulent exudate
D. Serosanguineous exudate
----------------------------------------
10. Myocardial biopsy of a patient with viral myocarditis will show increased levels of which of the
following inflammatory cells?
A. Eosinophils

Page 2

105
B. Lymphocytes
C. Macrophages
D. Mast cells
----------------------------------------
11. Why is a 53-year-old woman with metastatic breast cancer susceptible to bacterial pneumonia after
her first chemotherapy dose?
A. Depletion of serum complement
B. Impaired neutrophil respiratory burst
C. Inhibition of clotting factor activation
D. Neutropenia
----------------------------------------
12. What facilitates the firm binding of neutrophils to the endothelium during margination and rolling in
acute inflammation?
A. Tumor necrosis factor
B. Integrin
C. Interleukin
D. Interferons
----------------------------------------
13. Which statement pertains to neutrophil extracellular traps (NETs) in the study of microbe killing
mechanisms by neutrophils?
A. NET is an example of beneficial suicide for the neutrophil
B. Provide a low concentration of antimicrobial at the site of infection
C. It is an intracellular fibrillary network
D. It can be associated with decreased risk of autoimmune disease
----------------------------------------
14. Which of the following is the best statement about cells and their role in inflammation?
A. In acute inflammation, neutrophils are predominant in the inflammatory infiltrate during the first 6 to
24 hours
B. In acute inflammation, neutrophils are predominant in the inflammatory infiltrate during the first 24 to
48 hours
C. In acute inflammation, monocytes are predominant in the inflammatory infiltrate during the first 6 to
24 hours
D. In acute inflammation, monocytes are predominant in the inflammatory infiltrate during the first 6 to
24 hours
----------------------------------------
15. Which endothelial molecule is involved in leukocyte adhesion during acute inflammation?
A. ICAM-1
B. Integrins

Page 3

106
C. CD 31
D. CD 34
----------------------------------------
16. Which of the following amino acids plays an important role in the process shown above?

A. Leucine
B. Methionine
C. Arginine
D. Valine
----------------------------------------
17. What protein-encoding gene mutation is responsible for the defective oxidative burst reaction in an
11-year-old boy with recurrent bacterial infections, fever, and productive cough?
A. Catalase
B. Cytochrome P450
C. Myeloperoxidase
D. NADPH oxidase
----------------------------------------
18. Which of the following is the function of the peroxidase enzyme found in neutrophils?
A. Macrophage activation
B. Chemotactic agent
C. Microbial killing
D. Vasodilatation
----------------------------------------
19. What is a specific form of endocytosis primarily executed by neutrophils, macrophages, and
monocytes to engulf and eliminate solid particles like bacteria, viruses, and apoptotic cells from the
body?
A. Chemotaxis
B. Phagocytosis
C. Leukocyte adhesion

Page 4

107
D. Repair
----------------------------------------
20. Which of the following gives the correct definition of chemotaxis?
A. Increased random movement of neutrophils
B. Increase adhesiveness to intima
C. Engulfment of bacteria
D. Unidirectional locomotion of the neutrophils
----------------------------------------
21. Which of the following is not an opsonin?
A. C3b
B. IgG
C. Mannose binding lectin
D. C5a
----------------------------------------
22. What is the most likely diagnosis in a 5-year-old boy with a 2-day fever, fatigue, loss of appetite,
oculocutaneous albinism, lymphadenopathy, hepatosplenomegaly, Hb of 8.6g/dL, TLC of 22.1x103/uL,
and large purple intracytoplasmic granules in lymphocytes on peripheral smear, along with defective
phagolysosomes on electron microscopy?
A. Chronic Granulomatous Disease
B. Chediak – Higashi – syndrome
C. Ataxia telangiectasia
D. Fanconi Anemia
----------------------------------------
23. What is the likely diagnosis in a 9-month-old child with recurrent respiratory infections, decreased
green fluorescence in the dihydro rhodamine assay, and a positive bronchial lavage for Nocardia?
Temperature: 99°F (37.2°C) Blood pressure is 100/70 mmHg Pulse rate is 102/minute Respiratory rate:
17 /minute
A. Chronic granulomatous disease
B. Chediak Higashi syndrome
C. Hyper IgM Syndrome
D. Ataxia Telanguectasia
----------------------------------------

Correct Answers
Question Correct Answer

Question 1 1
Question 2 2

Page 5

108
Question 3 1
Question 4 3
Question 5 4
Question 6 2
Question 7 1
Question 8 4
Question 9 3
Question 10 2
Question 11 4
Question 12 2
Question 13 1
Question 14 1
Question 15 1
Question 16 3
Question 17 4
Question 18 3
Question 19 2
Question 20 4
Question 21 4
Question 22 2
Question 23 1

Solution for Question 1:


Correct Option A - Polymorphs (Neutrophils):
• The nitroblue testing is done to assess the function of polymorphs
Nitroblue tetrazolium test:
• Nitroblue tetrazolium or NBT test measures ROS generated by spermatozoa and leukocytes.
• NBT becomes reduced by free oxygen radicals forming a blue-black compound, formazan.
• The reaction can be monitored spectro-photometrically, or the stained cells can be scored
histochemically.
• The NBT test is a diagnostic tool used to evaluate the ability of neutrophils to produce superoxide, an
important component of the oxidative burst that helps these immune cells destroy bacteria and other
pathogens. In some inherited immunodeficiency disorders like chronic granulomatous disease (CGD),
neutrophils lack the ability to generate superoxide effectively, making individuals more susceptible to
recurrent bacterial and fungal infections.

Incorrect Options:

Page 6

109
Option B - Complement:
• Complement testing is used for the measurement of complement system protein activity.
• Nitroblue testing is done in the suspected chronic granulomatous disease.
• The NBT test is not used to assess complement levels. Complement is a group of proteins that play a
role in immune responses but is not directly evaluated by the NBT test.
Option C - T cell:
• Nitroblue testing is done to assess neutrophilic function.
• T cell function is not assessed by the NBT test. T cell function can be evaluated using other tests,
such as lymphocyte proliferation assays.
Option D - B cell:
• Nitroblue testing is not done for the B-cells.
• B cell function is typically evaluated through tests that assess antibody production and
immunoglobulin levels.

Solution for Question 2:


Correct Option B - Post-capillary venules:
• The leukocyte migration through the vessel wall occurs mainly through the post-capillary venules.
Leukocyte migration:
• The migration of the leukocytes through intact endothelium is called transmigration or diapedesis
• It occurs mainly in post-capillary venules.
• Chemokines act on the adherent leukocytes and stimulate the cells to migrate through
inter-endothelial gaps toward the chemical concentration gradient
• CD31 or PECAM-1 are present in the intercellular junctions between endothelial cells and involved in
the migration of leukocytes
• After traversing the endothelium, leukocytes pierce the basement membrane, by secreting
collagenases and enter the extravascular space.
Incorrect Options:
Options A, C, and D are incorrect as they are not involved in diapedesis.

Solution for Question 3:


Correct Option A - Selectins:
• Rolling interactions of leucocytes are mediated by a family of proteins called selectins
• 3 types of selectins: L-selectins (on leucocytes), E-selectin ( on endothelium ), and P-selectin (on
platelets, endothelium).

Page 7

110
• TNF and IL-1 act on the endothelial cells of postcapillary venules and induce the expression of many
adhesion molecules.
Incorrect Options:
Option B - Integrins:
• Firm adhesion of the leukocyte to the endothelial cell is mediated by a family of leukocyte surface
proteins called integrins.
Option C - Transferrin:
• Transferrin is an iron-transporting protein and is responsible for ferric-ion delivery.
• It transports iron to various tissues through the blood.
Option D - PECAM-1:
• PECAM-1 is present in the inter-cellular junction between endothelial cells
• It helps in the migration of leucocytes.

Solution for Question 4:


Correct Option C - Adhesion:
• Among the given options, adhesion is the first step of inflammatory process
Steps in the inflammatory process:
A. Step 1-Margination
• Vasodilation slows blood flow in post-capillary venules.
• Cells marginate from the centre of the flow to the periphery.
B. Step 2-Rolling
• Selectin "speed bumps" are upregulated on endothelial cells.
• P-selectin release from Weibel-Palade bodies is mediated by histamine.
• TNF and IL-1 induce E-selectin.
• Selectins bind Sialyl Lewis X on leukocytes.
• Interaction results in the rolling of leukocytes along the vessel wall.
C. Step 3-Adhesion
• 1-Cellular adhesion molecules (ICAM and VCAM) are upregulated on the endothelium by TNF and
IL-l.
• 2. Integrins are upregulated on leukocytes by C5a and LTB4
• 3. Interaction between CAMs and integrins results in firm adhesion of leukocytes to the vessel wall.
• 4. Leukocyte adhesion deficiency is most commonly due to an autosomal recessive defect of integrins
(CD18 subunit):
D. Step 4-Transmigration and Chemotaxis
• l. Leukocytes transmigrate across the endothelium of post-capillary venules (diapedesis) and move
toward chemical attractants (chemotaxis).

Page 8

111
• 2. Neutrophils are attracted by bacterial products, IL-8, C5a, and LTB4•
E. Step 5-Phagocytosis
• l. Consumption of pathogens or necrotic tissue; phagocytosis is enhanced by opsonins (IgG and C3a).
• 2. Pseudopods extend from leukocytes to form phagosomes, which are internalized and merged with
lysosomes to produce phagolysosomes.
Step 6-Destruction of phagocytosed material
• Oxygen-dependent killing is the most effective mechanism.
• HOCl generated by an oxidative burst in phagolysosomes destroys phagocytosed microbes.
Incorrect Options:
Option A - Chemotaxis:
• Chemotaxis is mediated by the mediators released during tissue damage.
• It is a late event in inflammation as compared to adhesion.
Option B - Phagocytosis:
• Phagocytosis occurs after the transmigration and chemotaxis of the inflammatory cells.
Option D - Diapedesis:
• Diapedesis is the movement of inflammatory molecules out of blood vessels.
• This occurs after the adhesion of the cells to the vessel wall.

Solution for Question 5:


Correct Option D - Myeloperoxidase - primary granules:
• Primary (Azurophilic) Granules: Contains acid hydrolase enzymes, including myeloperoxidase (MPO),
collagenase, cathepsin, defensin, and elastase. Myeloperoxidase (MPO) is essential for killing bacteria.
Collagenase and cathepsin contribute to tissue remodeling and degradation. Defensin is involved in
antimicrobial defense. Elastase aids in the breakdown of elastin in connective tissues.
• Contains acid hydrolase enzymes, including myeloperoxidase (MPO), collagenase, cathepsin,
defensin, and elastase.
• Myeloperoxidase (MPO) is essential for killing bacteria.
• Collagenase and cathepsin contribute to tissue remodeling and degradation.
• Defensin is involved in antimicrobial defense.
• Elastase aids in the breakdown of elastin in connective tissues.
Primary (Azurophilic) Granules:
• Contains acid hydrolase enzymes, including myeloperoxidase (MPO), collagenase, cathepsin,
defensin, and elastase.
• Myeloperoxidase (MPO) is essential for killing bacteria.
• Collagenase and cathepsin contribute to tissue remodeling and degradation.
• Defensin is involved in antimicrobial defense.

Page 9

112
• Elastase aids in the breakdown of elastin in connective tissues.
Incorrect Options:
Option A - Lactoferrin are present in primary granules:
• Lactoferrin are present in secondary granules.
• Therefore, this option is incorrect.
Option B -Gelatinase is present in secondary granules
• Gelatinase is tertiary granule
Option C - Acid Hydrolases are present in secondary granules:
• Acid hydrolases are present in primary granules.
• Therefore, this option is incorrect.

Solution for Question 6:


Correct Option B - Neutrophilia:
• The nature of the leukocyte infiltrate varies with the age of the inflammatory response and the type of
stimulus.
• In most forms of acute inflammation, neutrophils predominate in the inflammatory infiltrate during the
first 6 to 24 hours and are replaced by monocytes in 24 to 48 hours.
• There are several reasons for the early preponderance of neutrophils; they are more numerous in the
blood than other leukocytes, they respond more rapidly to chemokines, and they may attach more
firmly to the adhesion molecules that are rapidly induced on endothelial cells, such as P and E-
-selectins.
• After entering tissues, neutrophils are short-lived; they undergo apoptosis and disappear within 24 to
48 hours.

Page 10

113
Incorrect Options:
Option A - Eosinophilia:
• Eosinophils are abundant in immune reactions mediated by IgE and in parasitic infections.
• Eosinophils contain granules containing a major basic protein, a highly cationic protein toxic to
parasites that can also cause lysis of mammalian epithelial cells. This dual function explains why
eosinophils are beneficial in controlling parasitic infections but also contribute to tissue damage in
immune reactions such as allergies.
Option C - Monocytosis:
• Monocytes infiltrate the site of inflammation in 24 to 48 hours.
• Monocytes not only survive longer but may also proliferate in the tissues, and thus they become the
dominant population in prolonged inflammatory reactions.
Option D - Lymphocytosis:
• Microbes and other environmental antigens activate T and B lymphocytes, which amplify and
propagate chronic inflammation.
• While lymphocytes primarily serve as mediators of adaptive immunity, defending against infectious
pathogens, they are frequently found in chronic inflammation.
• When activated, they can exacerbate inflammation, leading to persistent and severe reactions.
• Certain types of chronic inflammation, like granulomatous inflammation, heavily rely on lymphocyte
responses for their development and persistence.
• Lymphocytes may be the dominant population in the chronic inflammation seen in autoimmune and
other hypersensitivity diseases.

Solution for Question 7:


Correct Option A - Redness, heat, swelling, pain:
• Acute inflammation
• Four cardinal signs of inflammation were given by Celsus.
• Rubor (redness) & Calor (heat): Due to vasodilation, which results in increased blood flow. Occurs via
relaxation of arteriolar smooth muscle; key mediators are histamine, prostaglandins, and bradykinin.
• Due to vasodilation, which results in increased blood flow.
• Occurs via relaxation of arteriolar smooth muscle; key mediators are histamine, prostaglandins, and
bradykinin.
• A tumour (swelling): Due to leakage of fluid from post-capillary venules into the interstitial space. Key
mediators are histamine, which causes endothelial cell contraction and tissue damage, resulting in
endothelial cell disruption.
• Due to leakage of fluid from post-capillary venules into the interstitial space.
• Key mediators are histamine, which causes endothelial cell contraction and tissue damage, resulting
in endothelial cell disruption.
• Dolor (pain): Pain is mediated by bradykinin and prostaglandin E2, which sensitize sensory nerve
endings. These signs are hallmarks of acute inflammation.

Page 11

114
• Pain is mediated by bradykinin and prostaglandin E2, which sensitize sensory nerve endings.
• These signs are hallmarks of acute inflammation.
• A fifth clinical sign, loss of function (functio laesa), was added by Rudolf Virchow in the 19th century.
• Due to vasodilation, which results in increased blood flow.
• Occurs via relaxation of arteriolar smooth muscle; key mediators are histamine, prostaglandins, and
bradykinin.
• Due to leakage of fluid from post-capillary venules into the interstitial space.
• Key mediators are histamine, which causes endothelial cell contraction and tissue damage, resulting
in endothelial cell disruption.
• Pain is mediated by bradykinin and prostaglandin E2, which sensitize sensory nerve endings.
• These signs are hallmarks of acute inflammation.
Incorrect Options:
Option B - Swelling, redness, heat, functional loss:
• Redness, heat, swelling, and pain are the cardinal signs of inflammation.
• These are described by Celsus.
• Rudolf Virchow added a fifth clinical sign, loss of function (functio laesa).
Option C - Redness, pain, functional loss, swelling:
• Redness, heat, swelling, and pain are the cardinal signs of inflammation.
• These are described by Celsus.
• Rudolf Virchow added a fifth clinical sign, loss of function (functio laesa).
Option D - Functional loss, heat, pain, swelling:
• Redness, heat, swelling, and pain are the cardinal signs of inflammation.
• These are described by Celsus.
• Rudolf Virchow added a fifth clinical sign, loss of function (functio laesa).

Solution for Question 8:


Correct Option D - Neutrophils:
• The above scenario is acute mastitis.
• The thick, yellow fluid draining from the breast fissure is called a purulent exudate.
• Purulent exudates and effusions are seen in conditions such as pyogenic bacterial infections, in which
the major cell type is the segmented neutrophil (polymorphonuclear leukocyte).
• In most forms of acute inflammation, neutrophils predominate in the inflammatory infiltrate during the
first 6 to 24 hours.
• There are several reasons for the early preponderance of neutrophils; they are more numerous in the
blood than other leukocytes, they respond more rapidly to chemokines, and they may attach more
firmly to the adhesion molecules that are rapidly induced on endothelial cells, such as P and E-

Page 12

115
-selectins.
• After entering tissues, neutrophils are short-lived; they undergo apoptosis and disappear within 24 to
48 hours.

Incorrect Options:
Option A - B lymphocytes:
• B lymphocytes and plasma cells play key roles in chronic inflammation, as they are activated by
microbes and environmental antigens, perpetuating and amplifying the inflammatory response.
• While their primary function is adaptive immunity against pathogens, their activation in chronic
inflammation leads to persistent and severe inflammation.
• Lymphocyte responses, particularly in granulomatous inflammation, drive strong chronic inflammatory
reactions.
• In autoimmune and hypersensitivity diseases, lymphocytes often dominate the chronic inflammatory
response.
Option B - Eosinophils:
• Eosinophils are abundant in immune reactions mediated by IgE and in parasitic infections.
• Eosinophils are recruited to sites of inflammation through adhesion molecules and chemokines,
similar to neutrophils.
• They possess granules containing major basic protein, which is effective against parasites but can
also harm mammalian epithelial cells.
• While eosinophils are beneficial in combating parasitic infections, they also contribute to tissue
damage in immune reactions such as allergies.
Option C - Mast cells:
• Mast cells are granulated cells that contain receptors for IgE on the cell surface.
• Involved in type I hypersensitivity reactions.
• Mediate local tissue allergic reactions and contain basophilic granules.
• They originate from the same precursor as basophils but are not the same cell type.

Page 13

116
• These cells can bind to the Fc portion of IgE on their membrane and become activated by various
stimuli including tissue trauma, C3a and C5a, and surface IgE cross-linking by antigens. Upon
activation, they undergo degranulation, releasing histamine, heparin, tryptase, and eosinophil
chemotactic factors.

Solution for Question 9:


Correct Option C - Purulent exudate:
• The purulent exudate observed in this patient signifies an accumulation of fluid in the pleural cavity,
typically due to capillary leakage caused by inflammation.
• Inflammatory processes trigger increased vascular permeability, allowing protein-rich fluid to leak into
the pleural space.
• Additionally, the presence of inflammatory cells such as polymorphonuclear leukocytes (PMNs)
contributes to the purulent nature of the exudate. In addition to PMNs, other components commonly
found in purulent exudate include bacteria, cellular debris, and fibrin.
Incorrect Options:
Option A - Fibrinous exudate:
• A fibrinous exudate is due to fibrinous inflammation.
• Fibrinous inflammation is a form of inflammation which is characterized by fibrin deposition.
• It may be acute, but more often, it is a chronic response.
• It results from the exudation of a high concentration of fibrin (fibrinogen passes out of the blood, and
fibrin gets deposited in the extracellular space).
• It is seen in rheumatoid arthritis and other auto-immune conditions seen in the lining of body cavities,
such as the meninges, pericardium, and pleura.
• The fibrinous exudate does not contain white blood cells.
Option B - Chylous exudate:
• A chylothorax is the accumulation of chyle in the pleural space.
• Occurs after trauma to the thoracic duct and is typically diagnosed based on the milky appearance of
fluid due to high-fat content.
• Most patients with chylothoraces will require surgical exploration of the thoracic duct.
Option D - Serosanguineous exudate:
• The serosanguineous exudate contains red blood cells.
• The serous exudate is characterized by the lack of a pronounced cellular response and is straw
yellow.

Solution for Question 10:


Correct Option B - Lymphocytes:

Page 14

117
• This patient with viral myocarditis shows an increase in lymphocytes in the affected heart muscle.
• Naive lymphocytes are found with (APC) antigen-presenting cells (macrophages and dendritic cells) in
secondary lymphatic organs.
• Cytotoxic T lymphocytes target viral proteins to destroy infected cells and fight infections, but this
process can also cause tissue damage. This same mechanism is responsible for killing cancer cells
and rejecting transplanted tissues.
Incorrect Options:
Option A - Eosinophils:
• Eosinophils are abundant in immune reactions mediated by IgE and in parasitic infections.
• Their recruitment is driven by adhesion molecules similar to those used by neutrophils and by specific
chemokines (e.g., eotaxin) derived from leukocytes and epithelial cells.
• Eosinophils have granules that contain a major basic protein, a highly cationic protein that is toxic to
parasites but also causes lysis of mammalian epithelial cells.
Option C - Macrophages:
• Macrophages are the primary cells in chronic inflammation, releasing cytokines and growth factors
that affect different cells, eliminate foreign invaders and tissues, and activate T lymphocytes and other
cells.
• Macrophages are professional phagocytes that act as filters for particulate matter, microbes, and
senescent cells.
• They also function as effector cells that eliminate microbes in cellular and humoral immune
responses.
Option D - Mast cells:
• Mast cells are granulated cells that contain receptors for IgE on the cell surface.
• Involved in type I hypersensitivity reactions.
• Mediate local tissue allergic reactions and contain basophilic granules.
• Can bind the Fc portion of IgE to the membrane and are activated by tissue trauma, C3a and C5a,
surface IgE cross-linking by antigen (IgE receptor aggregation) and their degranulation release of
histamine, heparin, tryptase, and eosinophil chemotactic factors.

Solution for Question 11:


Correct Option D - Neutropenia:
• Neutropenia is a reduction in the number of neutrophils in the blood.
• Causes: Inadequate or ineffective granulopoiesis. Increased destruction or sequestration of
neutrophils in the periphery.
• Inadequate or ineffective granulopoiesis.
• Increased destruction or sequestration of neutrophils in the periphery.
• The most common cause of agranulocytosis is drug toxicity.

Page 15

118
• Certain drugs, such as alkylating agents and anti-metabolites used in cancer treatment, produce
neutropenia in a predictable, dose-related fashion.
• Because such drugs cause a generalized suppression of hematopoiesis, the production of red cells
and platelets is also affected.
• Neutropenia can also occur as an idiosyncratic reaction to a large variety of agents such as
aminopyrine, chloramphenicol, sulfonamides, chlorpromazine, thiouracil, and phenylbutazone.
• The most important defect is iatrogenic neutropenia as a result of cancer chemotherapy.
• Inadequate or ineffective granulopoiesis.
• Increased destruction or sequestration of neutrophils in the periphery.
Incorrect Options:
Option A - Depletion of serum complement:
• Depletion of serum complement is seen in persistent activation of C3 convertase.
• Increased formation of immune complexes as seen in systemic lupus erythematosus and
post-streptococcal glomerulonephritis, and membranoproliferative glomerulonephritis results in
complement activation, often leading to hypocomplementemia.
• Decreased serum complement is not seen in chemotherapy.
Option B - Impaired neutrophil respiratory burst:
• Inherited defects in neutrophil respiratory burst cause defects in bacterial killing, making patients
susceptible to recurrent bacterial infection. CGD results from defective phagocyte NADPH oxidase
enzyme. (O−•).
Option C - Inhibition of clotting factor activation:
• Clot formation is the final result of interaction among multiple plasma proteins; after activation, it
results in the conversion of fibrinogen to fibrin and to cross-link of fibrin by activated factor XIII, which
stabilizes the formed clot.
• Natural inhibitors of clotting factors include antithrombin III, protein S, and protein C. When activated,
these proteins inactivate specific clotting factors, providing a regulatory mechanism that serves to
control the coagulation response and limit the extension of the clot.
• Inhibition of clotting factor activation is not related to neutropenia.

Solution for Question 12:


Correct Option B - Integrin:
• The weak rolling interactions slow down the leukocytes and give them the opportunity to bind more
firmly to the endothelium.
• Firm adhesion is mediated by a family of heterodimeric leukocyte surface proteins called integrins.
• Tumour necrosis factor (TNF) and interleukin-1 (IL-1) induce endothelial expression of ligands for
integrins, mainly vascular cell adhesion molecule 1 (VCAM1, the ligand for the β1 integrin VLA4) and
intercellular adhesion molecule1 (ICAM1, the ligand for the β2 integrins LFA1 and Mac1).

Page 16

119
Incorrect Options:
Option A - Tumor necrosis factor:
• TNF and IL-1 serve critical roles in leukocyte recruitment by promoting the adhesion of leukocytes to
endothelium and their migration through vessels.
• These cytokines are produced mainly by activated macrophages and dendritic cells; TNF is also
produced by T lymphocytes and mast cells, and IL-1 is produced by some epithelial cells.
• The secretion of TNF and IL1 can be stimulated by microbial products, immune complexes, foreign
bodies, physical injury, and a variety of other inflammatory stimuli.
• The production of TNF is induced by signals through toll-like receptors (TLRs) and other microbial
sensors, and the synthesis of IL-1 is stimulated by the same signals, but the generation of the biologi-
cally active form of this cytokine is dependent on the inflammasome.
• The most important roles of these cytokines in inflammation are the following.
• Endothelial activation
• Activation of leukocytes and other cells
• Systemic acute-phase response
• Histamine, prostaglandins, leukotrienes, cytokines (TNF, IL-1, IL-6), chemokines, platelet-activating
factors, complements, and kinins are mediators of acute inflammation.
Option C - Interleukin-1:
• TNF and IL-1 serve critical roles in leukocyte recruitment by promoting the adhesion of leukocytes to
endothelium and their migration through vessels.
• These cytokines are produced mainly by activated macrophages and dendritic cells; TNF is also
produced by T lymphocytes and mast cells, and IL-1 is produced by some epithelial cells.
• The secretion of TNF and IL1 can be stimulated by microbial products, immune complexes, foreign
bodies, physical injury, and a variety of other inflammatory stimuli.
• The production of TNF is induced by signals through toll-like receptors (TLRs) and other microbial
sensors, and the synthesis of IL-1 is stimulated by the same signals, but the generation of the biologi-
cally active form of this cytokine is dependent on the inflammasome.

Page 17

120
• The most important roles of these cytokines in inflammation are the following.
• Endothelial activation
• Activation of leukocytes and other cells
• Systemic acute-phase response
• Histamine, prostaglandins, leukotrienes, cytokines (TNF, IL-1, IL-6), chemokines, platelet-activating
factors, complements, and kinins are mediators of acute inflammation.
Option D - Interferons:
• Interferon-γ, produced by T lymphocytes and natural killer cells, cause activation of macrophages
(increased ability to kill microbes and tumour cells)
• They participate in chronic inflammation.
• A part of innate host defence, interferons interferes with both RNA and DNA viruses.
• Cells infected with a virus synthesize these glycoproteins, which act on local cells, priming them for
viral defence by downregulating protein synthesis to resist potential viral replication and by upregulating
MHC expression to facilitate recognition of infected cells. It also plays a major role in activating
antitumor immunity.

Solution for Question 13:


Correct Option A - NET is an example of beneficial suicide for the neutrophil:
• They are produced by neutrophils in response to infectious pathogens (mainly bacteria and fungi) and
inflammatory mediators (e.g. chemokines, cytokines [mainly interferons], complement proteins and
reactive oxygen species).
• During NET formation, the nuclei of the neutrophils are lost, which leads to the death of cells. NETs
were also detected in the blood in sepsis.
Incorrect Options:
Option B - Provide a low concentration of antimicrobial at the site of infection:
• Neutrophil extracellular traps (NET) are extracellular fibrillar networks that provide high concentrations
of antimicrobial substances to infection sites and prevent the spread of microbes by trapping them in
fibrils.
Option C - It is an intracellular fibrillary network:
• The extracellular traps consist of a viscous meshwork of nuclear chromatin that binds and
concentrates granule proteins such as antimicrobial peptides and enzymes.
Option D - It can be associated with decreased risk of autoimmune disease:
• The nuclear chromatin in the NETs, which includes histones and associated DNA, has been
postulated to be a source of nuclear antigens in systemic autoimmune diseases, particularly lupus, in
which individuals react against their own DNA and nucleoproteins.

Solution for Question 14:

Page 18

121
Correct Option A
- In acute inflammation, neutrophils are predominant in the inflammatory infiltrate during the first 6
to 24 hours:
• In most forms of acute inflammation, neutrophils dominate the inflammatory infiltrate for the first 6 to
24 hours and are replaced by monocytes within 24 to 48 hours.
• There are several reasons for the early predominance of neutrophils:
• They are more numerous in the blood than other leukocytes,
• They react faster to chemokines.
• They adhere more firmly to adhesion molecules which are rapidly induced in cells endothelial cells,
such as P-selectins and E-selectins.
• After penetrating tissue, neutrophils are short-lived; they go through apoptosis and disappear within
24 to 48 hours. Monocytes not only survive longer but can also multiply in the tissue and thus become
the dominant population in persistent inflammatory reactions.

Incorrect Options:
Option B - In acute inflammation, neutrophils are predominant in the inflammatory infiltrate during the fi
rst 24 to 48 hours:
• In acute inflammation, neutrophils predominate in the inflammatory infiltrate during the first 6 to 24
hours
Option C
- In acute inflammation, monocytes are predominant in the inflammatory infiltrate during the first 6
to 24 hours:
• In acute inflammation, neutrophils predominate in the inflammatory infiltrate during the first 6 to 24
hours
Option D - Monocytes are predominant in the inflammatory infiltrate during the first 6
to 24 hours and are short-lived:
• Monocytes infiltrate the site of inflammation in 24 to 48 hours.

Page 19

122
• Monocytes not only survive longer but may also proliferate in the tissues, and thus they become the
dominant population in prolonged inflammatory reactions.

Solution for Question 15:


Correct Option A - ICAM-1:
• The weak rolling interactions slow down the leukocytes and allow them to bind more firmly to the
endothelium.
• Firm adhesion is mediated by a leukocyte surface proteins called integrins.
• Tumour necrosis factor (TNF) and interleukin-1 (IL-1) induce endothelial expression of ligands for
integrins, mainly vascular cell adhesion molecule 1 (VCAM-1, the ligand for the β1 integrin VLA4) and
intercellular adhesion molecule1 (ICAM-1, the ligand for the β2 integrins LFA1 and MAC1).
Incorrect Options:
Option B - Integrins:
• Integrins are leukocyte surface proteins which are induced by TNF and IL-1, mediate firm adhesion to
endothelium by binding to vascular cell adhesion molecule 1 (VCAM-1) and intercellular adhesion
molecule1 (ICAM-1).
• These are not endothelial surface molecules.
Option C - CD 31:
• The process of leukocyte recruitment is the migration of leukocytes through the endothelium, called
transmigration or diapedesis.
• Several adhesion molecules present in the intercellular junctions between endothelial cells are
involved in the migration of leukocytes.
• These molecules include a member of the immunoglobulin superfamily called CD31 or PECAM-1
(platelet endothelial cell adhesion molecule).
• After traversing the endothelium, leukocytes pierce the basement membrane, probably by secreting
collagenases and enter the extravascular tissue.
• The cells then migrate toward the chemotactic gradient created by chemokines and other
chemoattractants and accumulate in the extra vascular site.
Option D - CD 34:
• CD34 is a transmembrane glycoprotein expressed on early lymphohematopoietic stem cells,
progenitor cells, and endothelial cells.
• Also, embryonic fibroblasts and some cells in fetal and adult nervous tissue are CD34-positive.
• Almost all hematopoietic pluripotent and committed stem cells in colony-forming assays express
CD34.

Solution for Question 16:


Correct Option C - Arginine:

Page 20

123
Arginine plays a crucial role in NETosis, which is the process by which neutrophils release web-like str
uctures composed of chromatin (DNA) and antimicrobial proteins to capture and neutralize pathogens.

Incorrect Options:
Option A. Leucine, B. Methionine, and D. Valine are important amino acids in protein synthesis and oth
er cellular processes, but they are not specifically known to play a central role in NETosis. Arginine, on
the other hand, is directly involved in the processes critical for NET formation, making it the most impor
tant amino acid in this context.

Solution for Question 17:


Correct Option D - NADPH oxidase:
• Defect in the NADPH oxidase contributes to the chronic granulomatous disease.
Chronic granulomatous disease:
• Chronic granulomatous disease (CGD) is characterised by poor oxygen-dependent killing.
• Due to NADPH oxidase defect (X-linked or autosomal recessive)
• This leads to recurrent infection and granuloma formation with catalase-positive organisms,
particularly Staphylococcus aureus, Pseudomonas cepacia, Serratia marcescens, Nocardia, and
Aspergillus
• Nitroblue tetrazolium test is used to screen for CGD.
• Leukocytes are incubated with NBT dye, which turns blue if NADPH oxidase can convert oxygen to its
free radical form but remains colourless if NADPH oxidase is defective.
• The name of this disease comes from the macrophage-rich chronic inflammatory reaction that tries to
control the infection when the initial neutrophil defence is inadequate.
• This often leads to collections of activated macrophages that wall off the microbes, forming
granulomas.
Incorrect Options:
Option A - Catalase:
• Acatalasemia is characterised by an inherited absence of the enzyme catalase.
• Affected infants cannot degrade endogenous or exogenous hydrogen peroxide, which accumulates,
resulting in oxidation deprivation.
• Oxidative burst is intact.
Option B - Cytochrome P450:
• Cytochrome P450 is involved in detoxifying toxins and drugs in the microsome, mainly in the liver.
• It is not linked with the oxidative burst mechanism.
Option C - Myeloperoxidase:
• MPO deficiency results in defective conversion of H202 to HOCl
• Increased risk for Candida infections; however, most patients are asymptomatic.

Page 21

124
• NBT is normal; respiratory burst (02 to H202) is intact.

Solution for Question 18:


Correct Option C - Microbial killing:
Reactive oxygen species-mediated killing of the microbes:
• Reactive oxygen species (ROS) in neutrophils are produced by NADPH oxidase during the respiratory
burst.
• NADPH oxidase is a multicomponent oxidase comprising at least seven proteins.
• Activation of NADPH oxidase occurs during phagocytosis, triggering the production of superoxide
anion (O2 • ).
• O2 • is converted to hydrogen peroxide (H2O2).
• Neutrophils contain myeloperoxidase (MPO) in azurophilic granules.
• MPO, in the presence of a halide (e.g., chloride), converts H2O2 to hypochlorite (HOCl■).
• HOCl■ is a potent antimicrobial agent, acting through halogenation and oxidation of microbial
components.
• The H2O2-MPO-halide system is a highly effective bactericidal mechanism in neutrophils.
• Inherited deficiency of MPO results in minimal susceptibility to infection, indicating redundancy in
microbicidal mechanisms.
Incorrect Options:
Option A - Macrophage activation:
• Microphage activation is mediated through either classical or alternative processes.
• This is mainly done by interleukins such as IL1, IL4 and IL13.
• The main function of peroxidases is to kill the microbe by generating oxygen-free radicals.
Option B - Chemotactic agent:
• The chemotactic agents are leukotrienes, bacterial products and Interleukins.
• Peroxidases are microbial-killing enzymes, not chemoattractants.
Option D - Vasodilatation:
• The vasodilation is mediated by histamine and prostaglandins during the inflammation.
• This contributes to redness over the inflammatory area due to high blood flow.

Solution for Question 19:


Correct Option B - Phagocytosis:
• Phagocytosis is activated by foreign substances such as bacteria, necrotic products, and several
mediators.

Page 22

125
• Phagocytosis involves:
• Recognition of the particle to be ingested by the leukocyte
• Engulfment and formation of a phagocytic vacuole
• Killing or degradation of the ingested substance
• Recognition of foreign substances is made easier by opsonins (IgG, C3b, mannose-binding lectins).
• Attachment and engulfment of the particle results in the formation of a phagosome which attaches to a
lysosome to form a phagolysosome.
• The particles are killed mainly by reactive oxygen species and lysosomal enzymes within the
phagolysosome.
Incorrect Options:
Option A - Chemotaxis:
• Chemotaxis is the process by which certain mediators attract leukocytes to the point of injury.
• Chemotactic agents include:
• N-formylmethionine terminal of bacterial products
• Cytokines and chemokines (IL-8)
• C5a
• LTB4.
Option C - Leukocyte adhesion:
• Leukocyte adhesion is the process by which leukocytes attach to the vascular endothelium.
• Selectins and integrins mediate leukocyte adhesion and migration.
Option D - Repair:
• The tissue repair mechanism involves regeneration and scar formation after tissue injury and
inflammation.

Solution for Question 20:


Correct Option D - Unidirectional locomotion of neutrophils:
• Chemotaxis in inflammation is the process by which leukocytes migrate to the injury site.
• It is defined as locomotion along a chemical gradient (unidirectional).
• The chemoattracts include bacterial products, cytokines, chemokines, C5a and LTB4.
• The chemoattractants attach to the G-proteins on the surface of leukocytes and mediate their
movement by activating the actin-myosin machinery and filopodia.
• The type of leukocyte infiltrate varies with the time of inflammatory response and stimulus.
• Usually, neutrophils predominate in the first 6-24 hours and later by monocytes and macrophages.
However, in Pseudomonal infections, neutrophils persist for several days. In viral infections,
lymphocytes are the earliest cells, and in allergic reactions, the eosinophils predominate.
Incorrect Options:

Page 23

126
Option A - Increased random movement of neutrophils:
• Chemokinesis is the name given to the increased random movement of cells.
• Chemotaxis is unidirectional and towards a chemotactic agent.
Option B - Increased adhesiveness to the intima:
• Increased adhesiveness to intima is the process of leukocyte adhesion.
• It is a step followed by leukocyte migration in the inflammation process.
• Increased adhesiveness to the endothelium is a step in the process of diapedesis (the movement of
leukocytes out of the bloodstream and towards the site of tissue injury or infection). It is not the
definition of chemotaxis.
Option C - Engulfment of bacteria:
• The engulfment of bacteria is a step in phagocytosis
• Phagocytosis follows chemotaxis in inflammation.
• Engulfment of bacteria is phagocytosis, which occurs after chemotaxis and diapedesis, when immune
cells reach the site of infection or injury. It is a subsequent step but not the definition of chemotaxis.

Solution for Question 21:


Correct Option D - C5a:

• Phagocytosis involves a step called recognition by phagocytic receptors.


• The efficacy of phagocytosis increases when the foreign substances are coated with opsonins for
which the phagocytes express high-affinity receptors.
• The opsonins include: C3b IgG Mannose-binding lectin
• C3b
• IgG

Page 24

127
• Mannose-binding lectin
• Leukocytes recognize these opsonins.
• C5a is a component of the complement system, which is an anaphylatoxin and chemoattractant. It has
no role as an opsonin.
• C3b
• IgG
• Mannose-binding lectin
Incorrect Options:
Option A - C3b: C3b is one of the opsonins with receptors on the leukocytes' surface.
Option B - IgG: IgG is one of the opsonins.
Option C - Mannose-binding lectin: Mannose-binding lectin is one of the opsonins.

Solution for Question 22:


Correct Option B - Chédiak-Higashi syndrome:

• The patient presents with the clinical features of Chédiak-Higashi syndrome.


• Chediak-Higashi syndrome is a rare autosomal recessive disorder characterized by partial albinism,
recurrent pyogenic infections, and neurological abnormalities
• It is an autosomal recessive condition characterized by a defective fusion of phagosomes and
lysosomes, leading to defective phagocytosis and susceptibility to infections.
• Mutation occurs in the gene encoding for the LYST protein, which regulates lysosomal trafficking.
• Neutropenia, defective degranulation and delayed microbial killing are the main leukocyte
abnormalities.
• As a result, large giant granules are seen in the leukocytes (as shown above).

Page 25

128
• Additional abnormalities include: Melanocyte abnormalities leading to albinism Abnormalities in cells
of the nervous system leading to nerve defects Abnormalities in platelets leading to bleeding disorders
• Melanocyte abnormalities leading to albinism
• Abnormalities in cells of the nervous system leading to nerve defects
• Abnormalities in platelets leading to bleeding disorders
• Melanocyte abnormalities leading to albinism
• Abnormalities in cells of the nervous system leading to nerve defects
• Abnormalities in platelets leading to bleeding disorders
Incorrect Options:
Option A - Chronic granulomatous disease:
• Chronic granulomatous disease (CGD) is characterized by defects in the gene encoding for
components of phagocyte oxidase.
• Phagocyte oxidase is the enzyme responsible for generating superoxide radicals.
• The patients present with recurrent bacterial infections but have no abnormalities of melanocytes (as
in this patient), nervous system or platelet function.
Option C - Ataxia telangiectasia:
• Ataxia telangiectasia is characterized by an abnormal gait, vascular malformations, neurologic defects
and increased incidence of lymphoid tumours and immunodeficiency.
• It is an autosomal recessive disorder which affects the gene encoding for the ATM protein. The ATM
protein acts as a sensor of DNA damage that activates cell cycle checkpoints and apoptosis in cells
with damaged DNA.
• The patients have a deficiency of IgA and IgG2, which lead to upper respiratory tract bacterial
infections.
• Giant granules in lymphocytes are not seen.
Option D - Fanconi anaemia:
• Fanconi anaemia consists of short stature, skeletal abnormalities, abnormal skin pigmentation, heart
or kidney malformations, small head or eyes, and small reproductive organs in males.
• It is an autosomal recessive disorder caused by mutations in several genes responsible for DNA
repair.
• Giant granules in lymphocytes are not seen.

Solution for Question 23:


Correct Option A - Chronic granulomatous disease:
• Chronic granulomatous disease is a condition where intracellular killing is impaired due to a defective
respiratory burst, leading to decreased production of reactive oxygen species such as superoxide.
• This disease is mainly found in infant boys infected with catalase-positive organisms, such as
Streptococcus aureus. Screening for chronic granulomatous disease is recommended for individuals
with infections caused by Burkholderia, Serratia, and Nocardia.

Page 26

129
• The investigation includes an abnormal dihydro rhodamine test and a Nitroblue Tetrazolium Test.
Incorrect Options:
Option B - Chediak Higashi syndrome:
• Chediak-Higashi syndrome is a rare autosomal recessive disorder characterized by defects in
lysosomal trafficking. While recurrent infection are common due to impaired immune function
(neutropenia), other features include oculocutaneous albinism (hypopigmented hair and skin due to
abnormal melanosome trafficking), bleeding tendency and neurological symptoms.
Option C - Hyper IgM Syndrome:
• It presents with symptoms like increased respiratory, skin, and sinus infection
• Hyper IgM syndrome is due to defective CD 40L present on the helper t cells
• So, they would be no class switching, e.g., IgG, IgA, and IgE are decreased, but IGM levels are
normal or highly increased.
Option D - Ataxia Telangiectasia:
• Ataxia telangiectasia (A-T) is a rare autosomal recessive disorder caused by mutations in the ATM
gene, which plays a crucial role in DNA repair and cell cycle control.
• The clinical features of A-T include progressive cerebellar ataxia, telangiectasias (small dilated blood
vessels) in the skin and eyes, immune system dysfunction leading to recurrent infections, increased
risk of cancer, particularly lymphomas and leukemias, and sensitivity to ionizing radiation.
• Additionally, individuals with A-T may exhibit slowed growth and development, choreoathetosis
(involuntary jerky movements), and oculomotor apraxia (difficulty in initiating voluntary eye
movements).

Page 27

130
Inflammation - Mediators
1. What is the main role of histamine in inflammation?
A. Platelet aggregation
B. Vasodilation
C. Mediates Pain
D. Kills microbes
----------------------------------------
2. In a 70-year-old male with complicated pneumonia, hypertension, poorly controlled diabetes, fever,
and cough, taking amlodipine, metformin, and insulin, what is the least likely mediator of fever?
A. IL-1
B. TNF-α
C. Nitric Oxide
D. Prostaglandin
----------------------------------------
3. Which complement fragments are called ‘anaphylatoxins’?
A. C3a and C3b
B. C3b and C5b
C. C5a and C3b
D. C3a and C5a
----------------------------------------
4. Which specific leukocyte is increased in a 20-year-old student experiencing frequent sneezing and
"watery" eyes, particularly during spring and summer seasons?
A. Basophils
B. Eosinophils
C. Lymphocytes
D. Monocytes
----------------------------------------
5. Which of the following substances most likely has the greatest affinity for neutrophils, in the context
of chemotaxis?
A. C5a
B. Fucosyl transferase
C. P-selectin
D. TNF-α
----------------------------------------
6. Aspirin is usually effective in relieving symptoms of acute inflammation because it stops which of the
following enzymes?

131
A. Cyclooxygenase
B. Myeloperoxidase
C. Phospholipase A2
D. Superoxide dismutase
----------------------------------------
7. What inflammatory mediator is formed due to phospholipase A2 activation in intra-alveolar cells in a
61-year-old alcoholic man with a pulmonary abscess and subsequent acute bronchopneumonia with
given histological findings?

A. Arachidonic acid
B. cAMP
C. cGMP
D. Inositol trisphosphate
----------------------------------------
8. IFN-■ is produced by which of the following cells?
A. Neutrophills
B. Macrophages
C. T - cells
D. B - cells
----------------------------------------
9. Which vitamin deficiency is likely responsible for deranged liver enzymes, hypoalbuminemia,
markedly elevated PT, and mildly elevated aPTT in a 57-year-old male with chronic liver disease?
A. Vitamin A
B. Vitamin B
C. Vitamin E
D. Vitamin K
----------------------------------------
10. Which clotting factors in the coagulation cascade are inhibited by Protein C?
A. Factor IIa and Factor VIIIa

Page 2

132
B. Factor VIIIa and Factor IXa
C. Factor Va and Factor VIIIa
D. Factor Va and Factor VIIa
----------------------------------------
11. Which of the following represents the function of Vitamin K on coagulation factors?
A. γ - carboxylation of aspartic acid residues to clotting factors 2,7, 9 and 10
B. β- carboxylation of glutamic acid residues to clotting factors 2,7, 9 and 10
C. α-carboxylation of aspartic acid residues to clotting factors 2,7, 9 and 10
D. γ- carboxylation of glutamic acid residues to clotting factors 2,7, 9 and 10
----------------------------------------
12. Which of the following complements constitute MAC?
A. C5a, C6, C7, C8, C9
B. C1q, C3b, C5, C6, C9
C. C1, C4, C2, C3b, C7
D. C5b, C6, C7, C8, C9
----------------------------------------
13. What's the likely cause of infection in a 20-year-old military trainee with fever, nausea, vomiting,
headache, neck rigidity, TLC- 15x109cells/L with neutrophilic predominance, and a previous history of
lack of membrane attack complex (MAC)?
A. Streptococcus pneumoniae
B. Neisseria meningitidis
C. Pseudomonas aeruginosa
D. Staphylococcus aureus
----------------------------------------
14. What's the likely diagnosis for a 26-year-old female with a severe episode of shortness of breath
(for the last 10 mins), wheezing, and swelling of the face, hands, and feet, along with laboratory
findings of deficiency of C1 inhibitor?

A. Chronic granulomatous disease


B. Hereditary angioedema

Page 3

133
C. Myeloperoxidase deficiency
D. Selective IgA deficiency
----------------------------------------
15. What function is not associated with the structure causing the symptoms in an 8-year-old male
presenting with easy fatiguability, dark-colored urine every other morning, and pancytopenia?
(or)
Which of the following is not a function of CD55 and CD59?
A. Blocks activation of C1.
B. Prevents formation of C3 convertase.
C. Inhibits formation of MAC
D. Prevents activation of complement on host cells
----------------------------------------
16. Which of the following eicosanoids is represented by carboprost?
A. Prostaglandin I2
B. Lipoxins
C. Leukotriene B4
D. Prostaglandin F2■
----------------------------------------
17. Which of the following product of arachidonic acid metabolism has anti-inflammatory properties?
A. Prostacyclin
B. TGF-beta
C. Lipoxin
D. Leukotrienes
----------------------------------------
18. Which one of the following most accurately describes the function of PGI2?
A. Vasoconstriction and inhibits platelet aggregation
B. Vasodilator and inhibits platelet aggregation
C. Vasoconstriction and promotes platelet aggregation
D. Vasodilator and promotes platelet aggregation
----------------------------------------
19. Which protein family do ceruloplasmin, fibrinogen, α2-macroglobulin, serum amyloid A protein, and
C-reactive protein belong to?
A. Acute phase proteins
B. Anaphylatoxins
C. Inhibitors of platelet activation
D. Regulators of coagulation

Page 4

134
----------------------------------------
20. Which of the following structures is most representative of lymphotactin?
A. C
B. C-C
C. C-X-C
D. C-X3-C
----------------------------------------
21. Which of the following is an example of group 3 chemokine?

A. Interleukin 8 (IL-8)
B. Lymphotactin
C. Monocyte chemoattractant protein 1 (MCP-1)
D. Fractalkine
----------------------------------------
22. Which statement about the mediator of acute inflammation in a 37-year-old female with significant
laryngeal edema and a past history of recurrent angioneurotic edema is correct?
A. Antibodies and complement products bind to specific mast cell receptors and trigger signalling
pathways that induce degranulation to release histamine
B. It causes constriction of arterioles
C. It decreases the permeability of venules
D. Histamine H2 receptor antagonists are used to treat allergies
----------------------------------------
23. Which of the following enzymes are present in the primary granules of neutrophils?
A. Histaminase
B. Myeloperoxidase
C. Lactoferrin
D. Complement Receptor-1
----------------------------------------

Correct Answers
Page 5

135
Question Correct Answer

Question 1 2
Question 2 3
Question 3 4
Question 4 2
Question 5 1
Question 6 1
Question 7 1
Question 8 3
Question 9 4
Question 10 3
Question 11 4
Question 12 4
Question 13 2
Question 14 2
Question 15 1
Question 16 4
Question 17 3
Question 18 2
Question 19 1
Question 20 1
Question 21 1
Question 22 1
Question 23 2

Solution for Question 1:


Correct Option B – Vasodilation:

• Histamine is a major vasodilator in the inflammatory response.


Mediators of inflammation:
• Inflammatory mediators are the substances that initiate and regulate inflammatory reactions. Many
mediators have been identified and targeted therapeutically to limit inflammation.
• The most important mediators of acute inflammation are vasoactive amines, lipid products
(prostaglandins and leukotrienes), cytokines (including chemokines), and products of complement
activation.

Page 6

136
• These mediators induce various components of the inflammatory response, typically by distinct
mechanisms.
Role of histamine in inflammation:
• Histamine causes dilation of arterioles and increases the permeability of venules.
• Histamine is considered the principal mediator of the immediate transient phase of increased vascular
permeability, producing interendothelial gaps in venules.
• Its vasoactive effects are mediated mainly via binding to receptors on microvascular endothelial cells.
• Histamine also causes the contraction of some smooth muscles.
Incorrect Options:
Options A, C, and D are incorrect and the correct option has been explained above.

Solution for Question 2:


Correct Option C - Nitric Oxide:
• Nitric oxide is not involved in the mediation of fever.
• Rather, it is a major vasodilator and a source of reactive oxygen species.
Mediator
Source
Action
Histamine
Mast cells, basophils, platelets
Vasodilation, increased vascular permeability, endothelial activation
Prostaglandins
Mast cells, leukocytes
Vasodialtion, pain, fever
Leukotrienes
Increased vascular permeability, chemotaxis, leukocyte adhesion and activation
Cytokines (TNF, IL-1, IL-6)
Macrophages, endothelial cells, mast cells
Local: endothelial activation (expression of adhesion molecules) Systemic, fever, metabolic abnormaliti
es, hypotension (shock)
Chemokines
Leukocytes, activated macrophages
Chemotaxis, leukocyte activation
Platelet-activating factor
Leukocytes, mast cells

Page 7

137
Vasodilation, increased vascular permeability, leukocyte adhesion, chemotaxis, degranulation, oxidativ
e burst
Complement
Plasma (produced in liver)
Leukocyte chemotaxis and activation, direct target killing (membrane attack complex), vasodilation (ma
st cell stimulation)
Kinins
Increased vascular permeability, smooth muscle contraction, vasodilation, pain
Incorrect Options:
Option A - IL-1:
• IL-1 is also known as osteoclast activating factor.
• It is one of the major interleukins involved in fever.
Option B - TNF-α:
• TNF-alpha is released from inflammatory cells such as macrophages.
• It is involved in the mediation of fever.
Option D – Prostaglandin:
• Prostaglandins are involved in the mediation of fever.

Solution for Question 3:


Correct Option D - C3a and C5a :
• C3a and C5a are anaphylatoxins which cause histamine release from mast cells.
• Anaphylatoxins are considered pro-inflammatory polypeptides.
• These anaphylatoxins are generated by the cleavage of C3 and C5 in response to complement
activation.
• Anaphylatoxins play an important function in the immune response.
• Anaphylatoxins are important in the activation of mast cells, granulocytes and macrophages and
chemotaxis.
Incorrect
Option A - C3a and C3b:
• C3 is converted into C3a and C3b by the C3 convertase enzyme.
• C3b triggers phagocytosis and is used for opsonization.
Option B - C3b and C5b:
• C3b covalently bonded with the target surface and triggered phagocytosis.
• C5b is a component of MAC.
Option C - C5a and C3b:

Page 8

138
• C5a and C3a are anaphylatoxins, not C3b.
• C3b is used for opsonisation.

Solution for Question 4:


Correct Option B - Eosinophils:
• The symptoms described, like frequent sneezing, "watery" eyes, and a history of similar episodes in
spring and summer, suggest allergic or hypersensitivity reactions, such as hay fever or allergic rhinitis.
• Eosinophils play a key role in allergic reactions, often increasing during the late phase.
• In this phase, leukocytes are recruited to sustain the inflammatory response without additional antigen
exposure, with eosinophils being abundant. IL-5 is crucial in the development and activation of
eosinophils, significant effectors in type I hypersensitivity.
Incorrect Options:
Options A, C and D are incorrect and the correct option has been explained above.

Solution for Question 5:


Correct Option A - C5a:
• Several substances have the chemotactic potential for neutrophils.
• C5a is a prominent example.
Chemotactic Factors
Factor
Description
Chemotactic For
Formylated peptides
Bacterial products of Escherichia coli
Neutrophils
C5a
Activated complement component
HETE, LTB
Leukotrienes
Kallikrein
Product of factor XIIa – mediated conversion, and other cells
PAF
AGEPC; from basophils, mast cells, and other cells
Eosinophils

Page 9

139
PDGF
From platelets, monocytes – macrophages, smooth muscle cells and endothelial cells
Neutrophils and macrophages
TGF-β
From platelets, neutrophils, macrophages, lymphocytes, and fibroblasts
Macrophages and fibroblasts

Leukocyte adhesion deficiency type 2


is caused by the absence of Sialyl-Lewis X, the fucose-containing ligand for E- and P-selectins, as a
result of a defect in a
fucosyltransferase, an enzyme that attaches fucose moieties to protein backbones. This results in a fail
ure of leukocyte adhesion to endothelium, preventing the cells from migrating into tissues and making
patients prone to bacterial infections, which are often recurrent and frequently life-threatening.
Incorrect Options:
Option B - Fucosyl transferase:
• The defect in fucosyltransferase causes leukocyte adhesion deficiency type 2.
• The H antigen is synthesized by fucosyltransferase.
Option C - P-selectin:
• P-selectin is a protein which is produced by activated platelets.
• The absence of P-selectin causes leukocyte adhesion deficiency type 2.
Option D - TNF-α:
• It is an inflammatory cytokine which is produced by macrophages.
• TNF alpha is an inflammatory cytokine involved in scar formation and regression.

Solution for Question 6:


Correct Option A – Cyclooxygenase:
• Arachidonic acid is metabolized by cyclooxygenases (COX-1, COX-2) and lipoxygenases (5-LOX) to
generate prostanoids and leukotrienes, respectively.
• Early inflammatory prostanoid response is COX-1 dependent. COX-2 becomes the major source of
prostanoids as inflammation goes on further.
• Inhibition of COX (cyclooxygenases) is one mechanism by which nonsteroidal anti-inflammatory drugs
(NSAIDs), including aspirin, indomethacin and ibuprofen, show their potent analgesic and
anti-inflammatory effects.
• NSAIDs block both COX–1 & COX–2 productions of prostaglandins, thereby mitigating pain and
inflammation.
Incorrect Options:
Options B, C and D are incorrect and the correct option has been explained above.

Page 10

140
Solution for Question 7:
Correct Option A - Arachidonic acid:

• The IMAGE above shows polymorphonuclear leukocytes responding to bacterial pneumonia.


• The activation of phospholipase A2 results in the release of arachidonic acid.
• Phospholipase A2 Activation: In the context of inflammation, phospholipase A2 is an enzyme that
plays a crucial role. When cells are activated, such as intra-alveolar cells in response to infection,
phospholipase A2 is activated.
• In the context of inflammation, phospholipase A2 is an enzyme that plays a crucial role.
• When cells are activated, such as intra-alveolar cells in response to infection, phospholipase A2 is
activated.
• Release of Arachidonic Acid: Phospholipase A2 acts on membrane phospholipids, specifically
phosphatidylcholine, to release arachidonic acid.
• Phospholipase A2 acts on membrane phospholipids, specifically phosphatidylcholine, to release
arachidonic acid.
• Formation of Inflammatory Mediators: Arachidonic acid can be metabolized by different enzymes to
produce various inflammatory mediators. Two major pathways are the cyclooxygenase (COX) pathway
and the lipoxygenase (LOX) pathway.
• Arachidonic acid can be metabolized by different enzymes to produce various inflammatory
mediators.
• Two major pathways are the cyclooxygenase (COX) pathway and the lipoxygenase (LOX) pathway.
• Cyclooxygenase (COX) Pathway: Arachidonic acid is converted to prostaglandins through the action
of cyclooxygenase (COX) enzymes. Prostaglandins are involved in the mediation of inflammation, pain,
and fever.

Page 11

141
• Arachidonic acid is converted to prostaglandins through the action of cyclooxygenase (COX)
enzymes.
• Prostaglandins are involved in the mediation of inflammation, pain, and fever.
• Lipoxygenase (LOX) Pathway: Arachidonic acid is converted to leukotrienes through the action of
lipoxygenase (LOX) enzymes. Leukotrienes are potent mediators of inflammation and play a role in
allergic and asthmatic responses.
• Arachidonic acid is converted to leukotrienes through the action of lipoxygenase (LOX) enzymes.
• Leukotrienes are potent mediators of inflammation and play a role in allergic and asthmatic
responses.
• In the context of inflammation, phospholipase A2 is an enzyme that plays a crucial role.
• When cells are activated, such as intra-alveolar cells in response to infection, phospholipase A2 is
activated.
• Phospholipase A2 acts on membrane phospholipids, specifically phosphatidylcholine, to release
arachidonic acid.
• Arachidonic acid can be metabolized by different enzymes to produce various inflammatory
mediators.
• Two major pathways are the cyclooxygenase (COX) pathway and the lipoxygenase (LOX) pathway.
• Arachidonic acid is converted to prostaglandins through the action of cyclooxygenase (COX)
enzymes.
• Prostaglandins are involved in the mediation of inflammation, pain, and fever.
• Arachidonic acid is converted to leukotrienes through the action of lipoxygenase (LOX) enzymes.
• Leukotrienes are potent mediators of inflammation and play a role in allergic and asthmatic
responses.
Incorrect Options:
Option B – cAMP:
• cAMP is involved in the regulation of sugar, glycogen and lipid metabolism.
• It is used for intracellular signal transduction.
Option C – cGMP:
• cGMP is derived from guanosine triphosphate.
• cGMP is a key regulator of cell differentiation, proliferation, and apoptosis.
• It reduces the proinflammatory cytokines and oxidative stress.
Option D - Inositol trisphosphate:
• Inositol trisphosphate is generated by phosphatidyl inositol breakdown.
• Inositol trisphosphate mobilizes calcium from storage organelles and regulates cell proliferation.
• Inositol triphosphate (IP3) is a second messenger that is produced by the cleavage of
phosphatidylinositol 4,5-bisphosphate (PIP2) by phospholipase C, not phospholipase A2.

Page 12

142
Solution for Question 8:
Correct Option C - T-cells:
• Activated T-cells (helper) produce IFN-■, the major cytokine of granulomatous inflammation.
• Helper T-cells are activated by IL-2 and produce IFN-■, the major cytokine of granulomatous
inflammation.
• Finally, there is the formation of granuloma induced by IFN-■.
Incorrect Options:
Options A, B and D are incorrect and the correct option has been explained above.

Solution for Question 9:


Correct Option D - Vitamin K:

• Vitamin K is responsible for converting some clotting factors to their active forms.
• It activates the factors by carboxylation of their glutamate residues. This is called the Vitamin K
epoxide cycle.
• The factors which require Vitamin K for activation include: Factor II Factor VII Factor IX Factor X
• Factor II
• Factor VII
• Factor IX
• Factor X
• It also has a role in the formation/activation of antithrombotic factors, which include: Protein C Protein
S Protein Z
• Protein C

Page 13

143
• Protein S
• Protein Z
• Other proteins which require Vitamin K for formation include osteocalcin and transthyretin
(prealbumin).
• As factor VII is Vitamin K dependent, the extrinsic pathway cannot start with Vitamin K deficiency,
leading to an increase in PT.
• Factor IX and X are involved in the intrinsic pathway. The pathway initiates normally but reduces in
speed resulting in mildly elevated aPTT.
• Factor II
• Factor VII
• Factor IX
• Factor X
• Protein C
• Protein S
• Protein Z
Incorrect Options:
Option A - Vitamin A:
• A deficiency of Vitamin A leads to night blindness.
• It is not related to disorders of the coagulation cascade.
Option B - Vitamin B:
• Vitamin B deficiency is not associated with elevated levels of PT and aPTT.
Option C - Vitamin E:
• A deficiency of Vitamin E is associated with muscle pain and weakness, a weak immune system,
chronic diarrhoea, and skin problems.
• It is not associated with elevated levels of PT and aPTT.

Solution for Question 10:


Correct Option C - Factor Va and Factor VIIIa:
• Endothelium normally expresses several factors that oppose coagulation.
• These include thrombomodulin, endothelial protein C receptor, heparin-like molecules, and tissue
factor pathway inhibitor.
• Thrombomodulin and endothelial protein C receptors bind thrombin and protein C in a complex.
• The complex cleaves and activates protein S (a cofactor).
• Activated protein C and S complex potently inhibits factors Va and VIIIa.
• Protein C deficiency presents with skin necrosis when a patient is started on warfarin.

Page 14

144
• Protein C has a short half-life of 8 hours compared to other Vitamin K-dependent factors and is rapidly
depleted with the initiation of warfarin.
• This results in a transient hypercoagulable state.
Incorrect Options:
Option A - Factor IIa and Factor VIIIa:
• Factor IIa is called activated thrombin.
• Thrombin is deactivated by thrombomodulin and antithrombin III.
• Protein C does not affect factor IIa.
Option B - Factor VIIIa and Factor IXa:
• Heparin-like molecules bind and activate antithrombin III, which inhibits factors IXa, Xa, XIa and XIIa.
• Protein C has no role in inhibiting IXa.
Option D - Factor Va and Factor VIIa:
• Factor VIIa is an activated tissue factor.
• It is inhibited by tissue factor pathway inhibitors.

Solution for Question 11:


Correct Option D - γ- carboxylation of glutamic acid residues to clotting factors 2,7, 9 and 10:

• Vitamin K is responsible for converting some clotting factors to their active forms.
• It activates the factors by carboxylation of their glutamate residues. This is called the Vitamin K
epoxide cycle.
• The factors which require Vitamin K for activation include: Factor II Factor VII Factor IX Factor X
• Factor II

Page 15

145
• Factor VII
• Factor IX
• Factor X
• Factor II
• Factor VII
• Factor IX
• Factor X
Incorrect Options:
Options A, B, and C are incorrect and the correct answer has been explained above.

Solution for Question 12:


Correct Option D - C5b, C6, C7, C8, C9:

• The complement system has 3 main functions: Inflammation Opsonization and phagocytosis Cell lysis
• Inflammation
• Opsonization and phagocytosis
• Cell lysis
• The cell lysis is the responsibility of the MAC.
• MAC is formed when C5b attaches to the C6-9 complex.
• MAC forms pores in the cell membrane, making the cell-permeable to water and ions. This leads to
cell lysis.
• MAC is important for killing microbes with thin cell walls, such as Neisseria. Hence, deficiency of this
terminal complex predisposes an individual to disseminated meningococci and gonococci infections

Page 16

146
• Hence, deficiency of this terminal complex predisposes an individual to disseminated meningococci
and gonococci infections
• Hence, deficiency of this terminal complex predisposes an individual to disseminated meningococci
and gonococci infections
Incorrect Options:

• Option A: C5a, C6, C7, C8, C9 C5a is not a part of the MAC. C5a is involved in inflammation and
chemotaxis, but it doesn't contribute directly to the formation of the MAC.
• C5a is not a part of the MAC. C5a is involved in inflammation and chemotaxis, but it doesn't contribute
directly to the formation of the MAC.
• Option B: C1q, C3b, C5, C6, C9 C1q, C3b, and C5 are involved in the activation of the complement
system, but they are not part of the MAC itself. C6, C9 are included in the MAC, but C1q and C3b are
not.
• C1q, C3b, and C5 are involved in the activation of the complement system, but they are not part of the
MAC itself. C6, C9 are included in the MAC, but C1q and C3b are not.
• Option C: C1, C4, C2, C3b, C7 C1, C4, and C2 are involved in the classical pathway of complement
activation, but they are not part of the MAC. C3b is involved in the formation of C5 convertase but is not
a direct part of the MAC. C7 is not part of the MAC.
• C1, C4, and C2 are involved in the classical pathway of complement activation, but they are not part
of the MAC. C3b is involved in the formation of C5 convertase but is not a direct part of the MAC. C7 is
not part of the MAC.
• C5a is not a part of the MAC. C5a is involved in inflammation and chemotaxis, but it doesn't contribute
directly to the formation of the MAC.
• C1q, C3b, and C5 are involved in the activation of the complement system, but they are not part of the
MAC itself. C6, C9 are included in the MAC, but C1q and C3b are not.
• C1, C4, and C2 are involved in the classical pathway of complement activation, but they are not part
of the MAC. C3b is involved in the formation of C5 convertase but is not a direct part of the MAC. C7 is
not part of the MAC.

Solution for Question 13:


Correct Option B - Neisseria meningitidis:
• The patient presents with a deficiency of MAC and symptoms of meningitis.
• MAC is responsible for cell lysis.
• MAC is formed when C5b attaches to the C6-9 complex.
• MAC forms holes in the cell membrane, making the cell-permeable to water and ions. This leads to
cell lysis.
• MAC is important for killing microbes with thin cell walls, such as Neisseria.
• Hence, deficiency of this terminal complex predisposes an individual to disseminated meningococci
and gonococci infections.
Incorrect Options:

Page 17

147
Options A, C and D are incorrect and the correct option has been explained above.

Solution for Question 14:


Correct Option B - Hereditary angioedema:
• The patient presents with the clinical features of hereditary angioedema.
• It is an autosomal dominant inherited deficiency of C1 inhibitor.
• The absence of a C1 inhibitor leads to uncontrollable cleavage of C4 and C2 by C1, which results in
recurrent swelling in: Extremities Genitals Face Lips Larynx Gastrointestinal tract
• Extremities
• Genitals
• Face
• Lips
• Larynx
• Gastrointestinal tract
• Treatment includes intravenous replacement of C1 inhibitor concentrate.
• Fresh frozen plasma may be an alternative as it contains a C1 inhibitor
It is an autosomal dominant inherited deficiency of C1 inhibitor.
• Extremities
• Genitals
• Face
• Lips
• Larynx
• Gastrointestinal tract
Incorrect Options:
Option A - Chronic granulomatous disease:
• Chronic granulomatous disease occurs due to a deficiency of NADPH oxidase.
• Patients present with recurrent bacterial infections, lymphadenopathy, skin rash, and gastrointestinal
symptoms.
• Swelling or oedema is not seen with chronic granulomatous disease
Option C - Myeloperoxidase deficiency:
• Patients with myeloperoxidase deficiency present with severe fungal infections, particularly Candida
albicans.
• No swelling or oedema is seen
Option D - Selective IgA deficiency:

Page 18

148
• Selective IgA deficiency is a congenital immunodeficiency disorder due to defects of lymphocyte
function.
• Patients with IgA deficiency present with recurrent respiratory and gastrointestinal infections.
• No oedema or selling is seen.

Solution for Question 15:


Correct Option A - Blocks activation of C1:
• Paroxysmal nocturnal hemoglobinuria is caused by the absence of GPI anchors (and CD59 with
DAF).
• It presents with hemolytic anemia leading to hemoglobinuria. Cell lysis eventually leads to
pancytopenia
• In paroxysmal nocturnal hemoglobinuria (PNH), the deficiency of CD55 and CD59 leads to
uncontrolled complement activation. CD55 (DAF) inhibits the formation of C3 convertase, and CD59
inhibits the formation of the membrane attack complex (MAC).
• Prevention of activation of C1 is a function of C1 INH (C1 inhibitor), it is not directly related to the
deficiency seen in PNH
Incorrect Options:
Options B, C, and D correspond to the functions of CD55 and CD59 as explained above

Solution for Question 16:


Correct Option D - Prostaglandin F2■:
• Carboprost is a synthetic analog of prostaglandin F2■
• Most tissues of the body produce Prostaglandin F2■.
• It is responsible for vasoconstriction and bronchoconstriction.
• It helps to stimulate uterine contraction to induce labour.
• PGF2■ acts to cause the contraction of smooth muscles.
Incorrect Options:
Option A - Prostaglandin I2:
• Prostaglandin I2 (prostacyclin) is produced in the endothelium of vessels from arachidonic acid under
the action of the cyclooxygenase-2 enzyme.
• It has the following functions: Vasodilation Inhibits platelet aggregation
• Vasodilation
• Inhibits platelet aggregation
• Vasodilation

Page 19

149
• Inhibits platelet aggregation
Option B - Lipoxins:
• Lipoxins are generated from arachidonic acid by the lipoxygenase pathway, but they suppress
inflammation by inhibiting the recruitment of leukocytes.
• They inhibit the chemotaxis of neutrophils and adhesion to endothelium.
• Leukocytes produce intermediates during the synthesis of lipoxins, which are converted to lipoxins by
platelets interacting with leukocytes.
Option C - Leukotriene C4:
• Leukotriene C4 and its metabolites, LTD4 and LTE4, are produced mainly in mast cells.
• It has the following functions that promote inflammation: Contraction of smooth muscles
Bronchoconstriction Intense Vasoconstriction Increased permeability of venules Cysteinyl leukotrienes
make up a slow-reacting substance of anaphylaxis. Involved in the pathophysiology of asthma and
seasonal allergies.
• Contraction of smooth muscles
• Bronchoconstriction
• Intense Vasoconstriction
• Increased permeability of venules
• Cysteinyl leukotrienes make up a slow-reacting substance of anaphylaxis.
• Involved in the pathophysiology of asthma and seasonal allergies.
• Contraction of smooth muscles
• Bronchoconstriction
• Intense Vasoconstriction
• Increased permeability of venules
• Cysteinyl leukotrienes make up a slow-reacting substance of anaphylaxis.
• Involved in the pathophysiology of asthma and seasonal allergies.

Solution for Question 17:


Correct Option C - Lipoxin:
• Lipoxins are arachidonic acid metabolites generated during the lipoxygenase pathway, and they
suppress the inflammatory process by inhibiting the recruitment of leukocytes.
• Their main action is to inhibit: Neutrophil chemotaxis Adhesion of neutrophils to endothelium
• Neutrophil chemotaxis
• Adhesion of neutrophils to endothelium
• Transcellular biosynthesis of these mediators requires two-cell generation.
• Neutrophils produce Intermediates, particularly during lipoxin synthesis, which are converted to
lipoxins by interacting with the platelets.

Page 20

150
• Lipoxins can control the proliferation of immune cells and cancer cells by inhibiting growth-promoting
PI3K and AKT pathways.
• Neutrophil chemotaxis
• Adhesion of neutrophils to endothelium
Arachidonic Acid Pathway:

Incorrect Options:
Option A – Prostacyclin:
• Prostaglandin I2 is an arachidonic acid metabolite that mediates the inflammatory process by causing
vascular smooth muscle relaxation, leading to increased blood flow responsible for erythema observed
with inflammation.
Option B - TGF-β:
• TGF-β is also an anti-inflammatory cytokine that limits and stops inflammatory responses.
• It functions by inhibiting lymphocyte proliferation and the activity of other leukocytes. However, it is not
an arachidonic acid metabolite.
Option D – Leukotrienes:
• Leukotrienes are inflammatory metabolites of arachidonic acid. They cause inflammation by: Vascular
and smooth muscles reactions Leukocytes chemotaxis Increasing vascular permeability Generating
oxygen-reactive species Release of lysosomal enzymes
• Vascular and smooth muscles reactions
• Leukocytes chemotaxis
• Increasing vascular permeability
• Generating oxygen-reactive species
• Release of lysosomal enzymes
• Vascular and smooth muscles reactions
• Leukocytes chemotaxis

Page 21

151
• Increasing vascular permeability
• Generating oxygen-reactive species
• Release of lysosomal enzymes

Solution for Question 18:


Correct Option B - Vasodilator and inhibits platelet aggregation:
• Prostacyclin is a prostaglandin that relaxes vascular smooth muscles leading to vasodilation.
• It binds to ADP receptors on platelets causing its inactivation and inhibiting platelet aggregation, thus
acting as an antiplatelet substance.
Incorrect Options:
Options A, C and D are incorrect and the function of PGI2 has been described above.

Solution for Question 19:


Correct Option A - Acute phase proteins:
• Acute-phase proteins, primarily synthesized in the liver, undergo significant plasma concentration
increases in response to inflammatory stimuli.
• Notable proteins include C-reactive protein (CRP), fibrinogen, and serum amyloid A (SAA), whose
synthesis is triggered by cytokines like interleukin-6 (IL-6) for CRP and fibrinogen, and interleukin-1
(IL-1) or tumor necrosis factor (TNF) for SAA.
• These proteins play roles in various inflammatory processes, including opsonization, complement
fixation, chromatin binding, and erythrocyte sedimentation rate (ESR) elevation, providing clinical
insights into inflammatory disease activity.
Incorrect Options:
Option B - Anaphylatoxins:
• Anaphylatoxins such as (C3a and C5a) have effects similar to those of mast cell mediators that are
involved in anaphylaxis.
Anaphylatoxins such as (C3a and C5a) have effects similar to those of mast cell mediators that are inv
olved in anaphylaxis.
Option C - Inhibitors of platelet activation:
• Prostacyclin PG I-2, nitric oxide, and adenosine diphosphatase are inhibitors of platelet activation
secreted by endothelium.
Prostacyclin PG I-2, nitric oxide, and adenosine diphosphatase are inhibitors of platelet activation secr
eted by endothelium.
Option D: Regulators of coagulation
• The protein C system provides important control of blood coagulation by regulating the activities of
factor VIIIa (FVIIIa) and factor Va (FVa), cofactors in the activation of factor X and prothrombin,
respectively.

Page 22

152
• The system comprises membrane-bound and circulating proteins that assemble into multi-molecular
complexes on cell surfaces.
• Vitamin K-dependent protein C, the key component of the system, circulates in the blood as a
zymogen to an anticoagulant serine protease.
• It is efficiently activated on the surface of endothelial cells by thrombin bound to the membrane protein
thrombomodulin.
• The endothelial protein C receptor (EPCR) further stimulates protein C activation. Activated protein C
(APC), together with its cofactor protein S inhibits coagulation by degrading FVIIIa and FVa on the
surface of negatively charged phospholipid membranes.
The protein C system provides important control of blood coagulation by regulating the activities of fact
or VIIIa (FVIIIa) and factor Va (FVa), cofactors in the activation of factor X
and prothrombin, respectively.
The system comprises membrane-bound and circulating proteins that assemble into multi-molecular co
mplexes on cell surfaces.
Vitamin K-dependent protein C, the key component of the system, circulates in the blood as a
zymogen to an anticoagulant serine protease.
It is efficiently activated on the surface of endothelial cells by thrombin bound to the membrane protein
thrombomodulin.
The endothelial protein C receptor (EPCR) further stimulates protein C activation. Activated protein C
(APC), together with its cofactor protein S inhibits coagulation by degrading FVIIIa and FVa on the surf
ace of negatively charged phospholipid membranes.

Solution for Question 20:


Correct Option A – C:
• Groups of Chemokines based on Cysteine residues number and arrangement

• Lymphotactin is an example of group 1 chemokines.

Page 23

153
• Group 1 chemokines are C chemokines.
• They lack first and third cysteine residues.
• C chemokines are specific for lymphocytes.
• Another example of C chemokines is XCL1.
Incorrect Options:
Option B - C-C:
• MCP-1 is an example of C-C chemokines, which represent group 2 chemokines.
• C-C chemokines have adjacent cysteine residues.
• These act as chemoattractants for eosinophils, basophils, monocytes, and lymphocytes.
• Other examples include CCL2, eotaxin (CCL1 specific for eosinophils) and macrophage inflammatory
protein 1α (MIP-1α, CCL3).
Option C - C-X-C:
• Group 3 chemokines represent C-X-C chemokines.
• C-X-C chemokines have one amino acid between the first two of the four cysteines.
• These chemokines act mainly on neutrophils.
• An example of C-X-C chemokines is IL-8 (CXCL8).
• It is induced by bacterial products, IL-1 and TNF.
• IL-8 is secreted by activated macrophages and endothelial cells mainly.
• Limited activity on monocytes and eosinophils is also seen.
Option D - CX3C:
• Fractalkine is the only example of a CX3C chemokine which represent group 4 chemokines.
• CX3C chemokines have three amino acids between the first two cysteines.
• It can be a cell-surface-bound protein on endothelial cells or a soluble form that causes monocyte and
T-cell adhesion.

Solution for Question 21:


Correct Option A - Interleukin 8 (IL-8):
• Group 3 chemokines represent C-X-C chemokines.
• C-X-C chemokines have one amino acid between the first two of the four cysteines.
• These chemokines act mainly on neutrophils.
• An example of C-X-C chemokines is IL-8 (CXCL8).
• It is induced by bacterial products, IL-1 and TNF.
• IL-8 is secreted by activated macrophages and endothelial cells mainly.
• Limited activity on monocytes and eosinophils is also seen.
Incorrect Options:

Page 24

154
Option B – Lymphotactin:
• Lymphotactin is an example of group 1 chemokines
• Group 1 chemokines are C chemokines
• They lack first and third cysteine residues.
• C chemokines are specific for lymphocytes.
• Another example of C chemokines is XCL1.
Option C - Monocyte chemoattractant protein 1 (MCP-1):
• MCP-1 is an example of C-C chemokines, which represent group 2 chemokines.
• C-C chemokines have adjacent cysteine residues.
• These act as chemoattractants for eosinophils, basophils, monocytes, and lymphocytes.
• Other examples include CCL2, eotaxin (CCL1 specific for eosinophils) and macrophage inflammatory
protein 1α (MIP-1α, CCL3).
Option D – Fractalkine:
• Fractalkine is the only example of a CX3C chemokine which represent group 3 chemokines.
• CX3C chemokines have three amino acids between the first two cysteines.
• It can be a cell-surface-bound protein on endothelial cells or a soluble form that causes monocyte and
T-cell adhesion.

Solution for Question 22:


Correct Option A - Antibodies and complement products bind to specific mast cell receptors and trigger
signaling pathways that induce degranulation to release histamine:
• Mast cell degranulation, induced by the binding of antibodies and complement products to specific
receptors, triggers the release of histamine.
• Histamine serves as a principal mediator in acute inflammation, stored in mast cell granules and
promptly released upon degranulation in response to various stimuli.
• These stimuli include physical injury, antibody binding, anaphylatoxins C3a and C5a, as well as
neuropeptides and cytokines, all contributing to histamine release and subsequent angioedema
manifestations.
Incorrect Options:
Option B - It causes constriction of arterioles:
• It causes dilation of arterioles by relaxation of vascular smooth muscles that increases blood flow to
the site of injury leading to erythema, one of the cardinal signs of inflammation.
• It can cause contraction of some smooth muscles, such as in the lungs leading to bronchospasm.
Option C - It decreases the permeability of venules
• Histamine increases the permeability of venules, producing interendothelial gaps in postcapillary
venules.
Option D - Histamine receptor antagonists are used to treat allergies:

Page 25

155
• Pharmacological significance -H1 receptor antagonists that bind to and block these histamine
receptors are used to treat inflammatory reactions (allergies).
• H2 receptor antagonists are used for the treatment of duodenal ulcers.

Solution for Question 23:


Correct Option B – Myeloperoxidase:
• Myeloperoxidase is present in the primary granules of the neutrophils.
Lysosomal Enzymes and Other Lysosomal Proteins:
• Neutrophils and macrophages contain lysosomal granules that contribute to microbial killing and may
cause tissue damage when released.
• Neutrophils have two main types of granules.
• The smaller specific (or secondary) granules contain lysozyme, collagenase, gelatinase, lactoferrin,
plasminogen activator, histaminase, and alkaline phosphatase.
• The larger azurophil (or primary) granules contain MPO, bactericidal proteins (lysozyme, defensins),
acid hydrolases, and a variety of neutral proteases (elastase, cathepsin G, nonspecific collagenases,
proteinase 3).
Incorrect Options:
Option A – Histaminase:
• This enzyme is involved in the synthesis of histamines.
• It is found in mast cells.
Option C - Lactoferrin:
• Lactoferrin is an iron-binding protein released from activated neutrophils at sites of inflammation and
has antimicrobial and anti-inflammatory properties.
• Found in the secondary granule of the neutrophils
Option D - Complement Receptor 1:
• CR1 is mainly localised to to the cell membrane and is not typically stored in secretory vesicles.

Page 26

156
Chronic Inflammation, Wound Healing and
Granuloma
1. Which of the following cells are expected to be elevated in a patient with a parasitic disease such as
acute schistosomiasis
A. Basophils
B. Eosinophils
C. Macrophages
D. Monocytes
----------------------------------------
2. What complication of wound healing is a 30-year-old firefighter with extensive third-degree burns
over his arms and hands at high risk of developing?
A. Contracture
B. Dehiscence
C. Incisional hernia
D. Keloid
----------------------------------------
3. What are the expected findings at the site of inflammation in a 17-year-old boy presenting with
severe abdominal pain, nausea, vomiting, mild fever, and tenderness at McBurney's point, with a WBC
count of 14,000 / mm3 with 74% segmented and 12% neutrophils?
(or)
Which statements describe the expected results at the site of inflammation in acute appendicitis?
A. Fistula connecting to the abdominal wall
B. Granulomatous inflammation with prominent aggregates of epithelioid cells and multinucleated giant
cells
C. Massive infiltration of lymphocytes and plasma cells
D. The accumulation of neutrophils within the muscularis propria
----------------------------------------
4. From what source are the cells marked in the CT-guided lung biopsy of a cachetic, vitally stable
36-year-old male patient with chest pain, shortness of breath, evening fever, and significant weight loss
derived?

157
A. Basophils
B. Capillary endothelial cells
C. Macrophages
D. Smooth muscle cells
----------------------------------------
5. What is the predominant healing mode in a poorly controlled diabetic patient with a large tissue loss
and an infected wound on the face sustained three days ago after being hit by a car?
A. Primary
B. Quaternary intention
C. Secondary
D. Tertiary
----------------------------------------
6. Which cell type is responsible for scar formation in a 16-year-old boy with a left frontal lobe cerebral
contusion following a concussion during an ice hockey game, who regains consciousness after three
days of unconsciousness?
A. Fibroblasts
B. Astrocytes
C. Neurons
D. Oligodendrocytes
----------------------------------------
7. Which of the following is the most important factor determining whether a wound will heal by primary
or secondary intention?
A. Apposition of edges
B. Depth of wound
C. Metabolic status
D. Skin site affected
----------------------------------------
8. What is the primary element involved in the process of healing by secondary intention in a large
wound left to heal without sutures following the excision of a benign nevus from the lower back of a

Page 2

158
39-year-old female under anesthesia?
A. Cadherins
B. Fibronectins
C. Integrins
D. Laminins
----------------------------------------
9. Which of the following best characterizes the healing of superficial abrasions?
A. Fibrosis
B. Granulation tissue
C. Primary intention
D. Regeneration
----------------------------------------
10. Which nutritional factors are required for proper collagen assembly in the scar tissue in case of a
deep, open lacerations?
A. Folic acid
B. Thiamine
C. Vitamin A
D. Vitamin C
----------------------------------------
11. What are the major cellular components found in the granulation tissue covering the ulcer bed of a
73-year-old woman with type II diabetes mellitus and taking metformin?

A. Fibroblasts
B. Eosinophils
C. Neutrophils
D. Plasma cells
----------------------------------------

Correct Answers

Page 3

159
Question Correct Answer

Question 1 2
Question 2 1
Question 3 4
Question 4 3
Question 5 3
Question 6 2
Question 7 1
Question 8 2
Question 9 4
Question 10 4
Question 11 1

Solution for Question 1:


Correct Option B – Eosinophils:
• Eosinophils are the most common cells to respond to parasitic infections
Eosinophils:
• Eosinophils are white blood cells that play an important role in the body's response to allergic
reactions, asthma, and infection with parasites.
• These cells protect immunity against certain parasites but also contribute to the inflammation that
occurs in allergic disorders.
• Eosinophils usually account for less than 7% of the circulating white blood cells (100 to 500
eosinophils per microliter of blood [0.1 to 0.5 × 109 per litre]).
A low number of eosinophils
• A low number of eosinophils in the blood (eosinopenia) can occur with Cushing syndrome,
bloodstream infections (sepsis), and treatment with corticosteroids.
A high number of eosinophils:
• Allergic disorders, including drug sensitivities, asthma, allergic rhinitis, and atopic dermatitis, often
increase the number of eosinophils. Many parasites, particularly ones that invade tissue, cause
eosinophilia.
• Cancers that cause eosinophilia include Hodgkin lymphoma, leukemia, and certain myeloproliferative
neoplasms.
Incorrect Options:
Option A – Basophils:
• While basophils are also involved in allergic reactions and immune responses, eosinophils are more
prominently associated with parasitic infections.
• Basophils release histamine and other mediators, contributing to allergic reactions, but they are not as
specific to parasitic diseases as eosinophils

Page 4

160
Option C – Macrophages:
• Macrophages are essential components of the immune system and play a role in phagocytosis, which
involves engulfing and digesting foreign particles, including parasites.
• However, eosinophils are particularly known for their efficacy against parasites, and an increase in
eosinophil count is a more specific marker for parasitic infections.
Option D – Monocytes:
• Monocytes are precursors to macrophages, and they also contribute to immune responses by
differentiating into tissue macrophages.
• However, eosinophils are more directly associated with the defense against parasitic infections.

Solution for Question 2:


Correct Option A - Contracture:
• In third-degree burns involving the extremities, there are high chances of forming extensive contractile
tissue that may contribute to the formation of contractures.
• Contraction in the size of a wound is an important part of the normal healing process (because of
collagen crosslinking and remodeling)
• An exaggeration of this process gives rise to contracture and results in deformities of the wound and
the surrounding tissues.
• Contractures are particularly prone to develop on the palms, the soles, and the anterior aspect of the
thorax.
• Contractures are commonly seen after serious burns and can compromise the movement of joints
Incorrect Options:
Option B - Dehiscence:
• Wound dehiscence is a surgical complication where the incision made during a surgical procedure,
reopens.
• It is sometimes called wound breakdown, wound disruption, or wound separation.
• Partial dehiscence means that the edges of an incision have pulled apart in one or more small areas
Option C - Incisional hernia:
• An incisional hernia is seen in a patient who has undergone previous abdominal surgery and whose
scar is weak.
• Burn involving the upper extremity does not lead to an increased risk of incisional hernia.
Option D - Keloid:
• Keloid formation is usually seen on the facial area.
• It has a genetic predisposition.
• After an extensive burn, there are greater chances of wound contracture during the healing process.

Page 5

161
Solution for Question 3:
Correct Option D - The accumulation of neutrophils within the muscularis propria:
• This is a case of acute appendicitis in which the patient presents with complaints of severe abdominal
pain, nausea, vomiting, and mild fever for one day. The pain was initially in the periumbilical region but
has moved to the lower right quadrant of the abdomen.
• On abdominal examination, there is maximum tenderness at McBurney's point. Laboratory tests
reveal a WBC count of 14,000 / mm3, with 74% segmented and 12% neutrophils.
• Because the danger of perforation is great, early appendectomy is the treatment of choice.
• Acute appendicitis is characterized by: Early acute appendicitis: Congested subserosal vessels,
modest perivascular neutrophilic infiltrate across all wall layers. Diagnostic indicator: Neutrophilic
infiltration into the muscularis propria. Severe progression: Prominent neutrophilic exudate, potential
abscess formation, ischemia, gangrenous necrosis, and possible rupture leading to suppurative
peritonitis.
• Early acute appendicitis: Congested subserosal vessels, modest perivascular neutrophilic infiltrate
across all wall layers.
• Diagnostic indicator: Neutrophilic infiltration into the muscularis propria.
• Severe progression: Prominent neutrophilic exudate, potential abscess formation, ischemia,
gangrenous necrosis, and possible rupture leading to suppurative peritonitis.
This is a case of acute appendicitis in which the patient presents with complaints of severe abdominal
pain, nausea, vomiting, and mild fever for one day. The pain was initially in the periumbilical region but
has moved to the lower right quadrant of the abdomen.
On abdominal examination, there is maximum tenderness at McBurney's point. Laboratory tests reveal
a WBC count of 14,000 / mm3, with 74% segmented and 12% neutrophils.
Because the danger of perforation is great, early appendectomy is the treatment of choice.
Acute appendicitis is characterized by:
• Early acute appendicitis: Congested subserosal vessels, modest perivascular neutrophilic infiltrate
across all wall layers.
• Diagnostic indicator: Neutrophilic infiltration into the muscularis propria.
• Severe progression: Prominent neutrophilic exudate, potential abscess formation, ischemia,
gangrenous necrosis, and possible rupture leading to suppurative peritonitis.
Incorrect Options:
Options A, B and C are incorrect.

Solution for Question 4:


Correct Option C - Macrophages:
• The likely diagnosis in this patient with chest pain, shortness of breath, evening rise in temperature,
significant weight loss and cachectic appearance is pulmonary tuberculosis.

Page 6

162
• The biopsy reveals multinucleated giant cells, which suggests tuberculosis.
• Granulomas are collections of epithelioid cells and multinucleated giant cells that are formed by
cytoplasmic fusion of macrophages.
• When the nuclei are arranged around the periphery of the cell in a horseshoe pattern (see
photomicrograph), the cell is termed a Langhans giant cell.
• The other cells do not form multinucleated giant cells in granulomas.
Incorrect Options:
Options A, B and D are incorrect as they do not fuse or form giant cells.

Solution for Question 5:


Correct Option C - Secondary:
Healing by Second Intention
• Healing by second intention (secondary union) is characterized by larger fibrin clots, increased
exudate, and necrotic debris due to large tissue deficits, leading to more intense inflammation.
• Larger amounts of granulation tissue form to fill the larger gap left by tissue deficits, resulting in a
greater mass of scar tissue composed primarily of type I collagen.
• Dermal appendages destroyed in the incision line are permanently lost, and wound contraction,
facilitated by myofibroblasts, helps close the wound by reducing the gap between dermal edges and the
wound surface area. This process is slower than primary intention healing but is suitable for wounds
with tissue loss and infection.
Incorrect Options:
Option A - Primary
Healing by First Intention
• When only the epithelial layer is injured, epithelial regeneration, or primary union, is the primary repair
mechanism. This occurs in clean, uninfected surgical incisions where sutures approximate the wound
edges.
• The process involves minimal disruption to the epithelial basement membrane and limited cell death.
Repair involves inflammation, proliferation of epithelial and other cells, and scar maturation, initiated by
rapid coagulation pathway activation.
• Primary intention healing is preferred for clean, well-approximated surgical wounds, where granulation
tissue is not formed. It's unsuitable for large tissue loss with infection.
Option B - Quaternary intention:
• There is no quaternary intention of healing.
• When tissue loss is large, the healing mode is the secondary mode with large granulation tissue
formation.
• Quaternary intention healing is not a common term used in wound healing. It's possible that it may be
a less conventional or regional term, but it's not widely recognized.
Option D - Tertiary intention:
• Also known as delayed primary closure.

Page 7

163
• Tertiary intention healing is a hybrid approach that combines elements of primary and secondary
intention healing. Initially, the wound is left open (similar to secondary intention) to allow for the
resolution of infection and tissue debridement. Once the wound is deemed clean and healthy, it is
surgically closed (similar to primary intention).

Solution for Question 6:


Correct Option B - Astrocytes:
• Astrocytes play a crucial role in the brain's reaction to injury and in the formation of scar tissue, termed
gliosis, which is the brain's way of repairing damage. After injury, astrocytes become activated and
contribute to the formation of scar tissue, creating a barrier around the injured area to prevent further
damage and facilitate the healing process. Fibroblasts, while significant in scar formation in other
tissues, aren't the primary mediators in brain injury-related scar formation.
Incorrect Options:
Option A - Fibroblasts: Fibroblasts are cells primarily responsible for producing collagen and forming sc
ar tissue in various tissues outside the central nervous system. While they play a significant role in wou
nd healing and scar formation in tissues such as skin or organs, in the brain their role in brain is limited.
Option C - Neurons: They are not directly involved in scar formation, although their function can be affe
cted by scar tissue if the injury occurs in the nervous system.
Option D - Oligodendrocytes: Oligodendrocytes are another type of glial cell found in the CNS. Their m
ain function is to produce myelin, allowing for efficient nerve signal transmission. Like neurons and astr
ocytes, oligodendrocytes are not directly involved in scar formation

Solution for Question 7:


Correct Option A - Apposition of edges:
• Apposition of edges is the primary factor in deciding whether the healing will be primary or secondary.
• When edges are approximated, there are greater chances that minimal scar will form and healing will
be of primary intention.
• This occurs when the wound is approximated with sutures and the wound is clean
Primary intention mode of healing
• Happens when the wound edges are approximated, e.g. by sutures, staples or glue.
• The condition for enabling primary healing is that the wound edges are sharp, completely clean, and
free of microbes, as is the case with a wound produced via a surgical incision (in a sterile environment).
• The advantage of primary healing is that the time to closure is short, reducing the risk of infection and,
furthermore, the scarring is limited.
• If the wound edges cannot be approximated, the wound will need to heal by second intention.
Secondary intention mode of healing
• Occurs when the wound edges cannot be approximated, and the wound needs to heal from the
bottom.

Page 8

164
• This can be the case if there is not enough skin to pull the edges together, Eg: venous leg ulcers, or if
the tissue loss is extensive e.g. in fourth-degree burns, if the wound area is dirty or colonized or the
wound edges are not sharp and clean, Eg: blast trauma wounds, and in wound dehiscence
• Secondary healing will typically be characterized by visible granulation tissue, and the scar will be
bigger than in wounds healed by first intention.
Incorrect Options:
Option B - Depth of the wound:
• Although the death of a wound can decide whether the healing would be via primary or secondary
mode, the most decisive factor is whether the edges are approximated.
• For example, if the edges are not approximated, the healing would most likely be with secondary
intention, no matter if the depth is not too extensive.
Option C - Metabolic status:
• Metabolic status greatly governs the timing of wound healing.
• For example, when edges are approximated, wound healing would occur through primary intention;
however, the healing can be delayed due to defective metabolic status.
Option D - Skin site affected:
• The role of the skin site involved is also a factor in deciding the mode of healing.
• However, it contributes to only a few per cent.
• The major deciding factor is whether the wound edges are approximated or not.

Solution for Question 8:


Correct Option B - Fibronectins:
• Fibronectin is crucial in secondary intention healing due to: Enhanced granulation tissue formation:
Fibronectin promotes the formation of larger amounts of granulation tissue, facilitating wound closure in
cases of extensive tissue deficits. Scar maturation: Its involvement in scar formation ensures the
transition from granulation tissue to a mature, avascular scar, essential for long-term wound stability
and integrity.
• Enhanced granulation tissue formation: Fibronectin promotes the formation of larger amounts of
granulation tissue, facilitating wound closure in cases of extensive tissue deficits.
• Scar maturation: Its involvement in scar formation ensures the transition from granulation tissue to a
mature, avascular scar, essential for long-term wound stability and integrity.
Fibronectin is crucial in secondary intention healing due to:
• Enhanced granulation tissue formation: Fibronectin promotes the formation of larger amounts of
granulation tissue, facilitating wound closure in cases of extensive tissue deficits.
• Scar maturation: Its involvement in scar formation ensures the transition from granulation tissue to a
mature, avascular scar, essential for long-term wound stability and integrity.
Incorrect Options:
Option A - Cadherins:
• Cadherins are cell adhesion molecules.

Page 9

165
• They are found at the cell surface.
• They do not play a part in healing
Option C - Integrins:
• Integrins are cell adhesion molecules.
• They do not play a part in wound healing.
Option D - Laminins:
• Laminins are glycoproteins with both common and specific functions.
• One common and most important function of laminins is interacting with receptors anchored in the
plasma membrane of cells adjacent to basement membranes.
• They are not a part of healing by secondary intention.

Solution for Question 9:


Correct Option D - Regeneration:
For superficial abrasions, there is a complete resolution.
Regeneration relies on cell proliferation stimulated by growth factors, contingent upon the integrity of th
e ECM and the differentiation of tissue stem cells into mature cells.
• In part, the ability of tissues to repair themselves is determined by their intrinsic proliferative capacity
and the presence of tissue stem cells. The body's tissues are divided into three groups based on these
criteria.
• Labile tissues: These are continuously dividing tissues (e.g., skin, gastrointestinal and urinary tract
epithelia) and can readily regenerate as long as the pool of stem cells is preserved.
• Stable tissues: Cells in these tissues are usually in a resting (quiescent) state but can divide in
response to injury. This includes cells like endothelial cells, fibroblasts, and certain solid organ cells.
However, except for the liver, their regenerative capacity is limited.
• Permanent tissues: These cells are considered terminally differentiated and nonproliferative in
postnatal life. Examples are neurons and cardiac muscle cells, leading to scarring after injury, as these
cells cannot regenerate adequately.
Incorrect Options:
Option A - Fibrosis:
• Fibrosis occurs due to excessive deposition of collagen tissue.
• This usually happens when the inflammation is extensive seen in permanent tissues which cannot
regenerate (tissue is replaced by fibrosis).
Option B - Granulation tissue:
• Granulation tissue is formed when the wound is large.
• This usually happens during the healing by secondary intention.
Option C - Primary intention:

Page 10

166
• Healing by primary intention means an approximation of wound edges with a suture.
• The scar tissue is minimum in this case.
• In case of abrasion, the tissue is completely regenerated due to the preservation of stem cells.

Solution for Question 10:


Correct Option D - Vitamin C:
• Vitamin C is required to synthesise collagen fibres, the main substance involved in the repair and
healing process.
Vitamin C is required to synthesise collagen fibres, the main substance involved in the repair and heali
ng process.
Factors influencing tissue repair:
• Infection
• Diabetes.
• Nutritional status and vitamin C deficiency inhibit collagen synthesis and retard healing.
• Glucocorticoids (steroids) due to their anti-inflammatory effects
• Mechanical factors such as increased local pressure or torsion. Poor perfusion due to peripheral
vascular disease, arteriosclerosis, and diabetes or obstructed venous drainage (e.g., in varicose veins)
• Foreign bodies such as steel, glass, or bone fragments impede healing by perpetuating chronic
inflammation.
Infection
Diabetes.
Nutritional status and vitamin C deficiency inhibit collagen synthesis and retard healing.
Incorrect Options:
Options A, B and C are incorrect as these factors do not have role in collagen assembly.

Solution for Question 11:


Correct Option A - Fibroblasts:
• Fibroblasts, macrophages, and myofibroblasts are involved in forming granulation tissue.
Fibroblasts, macrophages, and myofibroblasts are involved in forming granulation tissue.
Granulation tissue:
• Granulation tissue is an important component in the wound healing process. Wounds can heal by
primary intention (wound edges approximate easily) and secondary intention (wound edges do not
approximate).

Page 11

167
• The granulation tissue matrix will fill wounds that heal by the second intention. This tissue type is also
present in chronic wounds with many different causes.
Granulation tissue is an important component in the wound healing process. Wounds can heal by prim
ary intention (wound edges approximate easily) and secondary intention (wound edges do not approxi
mate).
The granulation tissue matrix will fill wounds that heal by the second intention. This tissue type is also p
resent in chronic wounds with many different causes.

Incorrect Options:
Option B - Eosinophils:
• Eosinophils provide an effective mechanism of defence against helminth and parasitic infection.
• They are also involved in the pathogenesis of allergic reactions.
• They do not take part in the formation of granulation tissue.
Eosinophils provide an effective mechanism of defence against helminth and parasitic infection.
They are also involved in the pathogenesis of allergic reactions.
They do not take part in the formation of granulation tissue.
Option C - Neutrophils:
• Neutrophils are involved in acute inflammatory reactions.
• Their main function is to fight against pathogens.
• During the healing, macrophages rather than neutrophils form granulation tissue.
Neutrophils are involved in acute inflammatory reactions.
Their main function is to fight against pathogens.
During the healing, macrophages rather than neutrophils form granulation tissue.
Option D - Plasma cells:
• Plasma cells are types of immune cells that are involved in antibody production.
• They do not enter the tissues and participate in forming granulation tissue.
Plasma cells are types of immune cells that are involved in antibody production.
They do not enter the tissues and participate in forming granulation tissue.

Page 12

168
Previous Year Questions
1. Which type of cells are implicated in the development of contact dermatitis in a 12-year-old boy who
presented to the emergency department?
A. Helper T cells
B. Neutrophils
C. NK cells
D. Cytotoxic T cells
----------------------------------------
2. Both intrinsic and extrinsic pathways are seen in :
A. Apoptosis
B. Necrosis
C. Necroptosis
D. Pyroptosis
----------------------------------------
3. The given inclusion bodies are characteristic of which of the following organisms?

A. Human papillomavirus
B. Cytomegalovirus
C. Ebstein-Barr virus
D. Herpes simplex virus
----------------------------------------
4. A patient who underwent surgery for a lipoma 5 weeks ago now presents with a scar at the incision
site that is limited to the original wound boundaries. What is the most likely diagnosis?
A. Keloid
B. Exuberant granulation
C. Hypertrophic scar
D. Granulation tissue
----------------------------------------

169
5. During wound healing, the cells that appear first, due to chemotaxis, at the site of the wound are:
A. Neutrophils
B. Monocytes
C. Macrophages
D. Lymphocytes
----------------------------------------
6. Which of the following molecules helps in the adhesion of the leucocyte to the endothelium during
inflammation?
A. P-selectin
B. Platelet-activating factor
C. VCAM-1
D. Complements
----------------------------------------
7. Arrange the following cellular events of inflammation in the correct sequence: Rolling
Cytokine-mediated integrin activation Adhesion Migration
A. 1, 2, 3, 4
B. 2, 1, 4, 3
C. 4, 1, 3, 2
D. 3, 4, 1, 2
----------------------------------------
8. IL-1 is activated by:
A. Caspase-1
B. Caspase-3
C. Caspase-9
D. Caspase-8
----------------------------------------
9. Which of the following is true about fever?
A. Aspirin increases fever by inhibiting prostaglandin synthesis
B. PGD2 is responsible for resetting the temperature to a higher level
C. IL-1 is an endogenous pyrogen
D. Fever occurs during inflammation due to release of histamine
----------------------------------------
10. Stellate granuloma is seen in:
A. Sarcoidosis
B. Cat-scratch disease
C. Cryptococcosis

Page 2

170
D. Histoplasmosis
----------------------------------------
11. Fever occurs during inflammation due to all except:
A. IL-1
B. Prostaglandins
C. TNF
D. Histamine
----------------------------------------
12. Which of the following is an opsonin?
A. C3a
B. C3b
C. C5a
D. C6
----------------------------------------
13. Which of the following is involved in the recognition and clearance of necrotic cells?
A. Inflammasome
B. Toll-like receptors (TLRs)
C. P-selectin
D. Manoreceptor
----------------------------------------
14. Which of the following cancers is the most common primary for brain metastasis?
A. Lung cancer
B. Head and neck cancer
C. Prostate cancer
D. Breast cancer
----------------------------------------
15. An acute inflammatory response is not seen in which of the following?
A. Pyroptosis
B. Necroptosis
C. Apoptosis
D. Necrosis
----------------------------------------
16. Which of the following causes vasodilation?
A. Histamine
B. Thromboxane A2

Page 3

171
C. Angiotensin II
D. Leukotriene C4
----------------------------------------
17. Match the following: 1. Howell-Jolly bodies A. Ig inclusions 2. Dohle bodies B. Azurophilic granules
3. Flame cells C. Nuclear material remnants 4. Auer rod cells D. Remnants of ER
1. Howell-Jolly bodies A. Ig inclusions
2. Dohle bodies B. Azurophilic granules
3. Flame cells C. Nuclear material remnants
4. Auer rod cells D. Remnants of ER

A. 1-C, 2-D, 3-A, 4-B


B. 1-A, 2-B, 3-C, 4-D
C. 1-C, 2-D, 3-B, 4-A
D. 1-D, 2-B, 3-A, 4-C
----------------------------------------

Correct Answers
Question Correct Answer

Question 1 1
Question 2 1
Question 3 2
Question 4 3
Question 5 1
Question 6 3
Question 7 3
Question 8 1
Question 9 3
Question 10 2
Question 11 4
Question 12 2
Question 13 1
Question 14 1
Question 15 3
Question 16 1
Question 17 1

Solution for Question 1:

Page 4

172
Correct Option: A
The correct answer is Helper T cells.
Explanation for the options:
Option A: Helper T cells: Helper T cells play a crucial role in the immune response by coordinating and
regulating other immune cells. They release cytokines that activate and recruit other cells to the site of i
nflammation, including in the case of contact dermatitis.
Option B: Neutrophils: Neutrophils are a
type of white blood cell that are typically involved in acute inflammation and play a major role in fighting
bacterial infections. While neutrophils may be recruited to the site of inflammation in some cases of co
ntact dermatitis, they are not the primary cells involved in the pathogenesis of the condition.
Option C: NK cells: Natural Killer (NK) cells are a type of cytotoxic lymphocyte that play a role in the inn
ate immune response. They are primarily involved in the recognition and elimination of virally-infected c
ells and tumor cells. NK cells are not typically the primary cells involved in contact dermatitis.
Option D: Cytotoxic T cells: Cytotoxic T cells are a type of T
lymphocyte that directly kill infected or abnormal cells. They play a critical role in the immune response
against viruses and tumor cells. While they may be involved in certain immune responses, they are not
the primary cells involved in contact dermatitis.
Contact dermatitis is an inflammatory skin condition that occurs when the skin comes into contact with
an allergen or irritant. The pathogenesis involves a complex interplay between various immune cells, in
cluding antigen-presenting cells (such as dendritic cells), T cells (including helper T
cells), and other immune mediators. Helper T cells are activated upon recognition of the allergen, and t
hey release cytokines that recruit and activate other immune cells, leading to the characteristic inflamm
ation and skin reactions seen in contact dermatitis.

Solution for Question 2:


Correct Option A.
Apoptosis: This option is correct. Apoptosis is a programmed cell death process characterized by contr
olled and regulated cell dismantling. It can be initiated through both intrinsic and extrinsic pathways. Th
e intrinsic pathway is triggered by internal signals, such as DNA damage or cellular stress, leading to th
e release of cytochrome c from the mitochondria. The extrinsic pathway is initiated by external signals,
such as binding of death ligands (e.g., Fas ligand) to death receptors on the cell surface.
Incorrect Options:
Option B: Necrosis: This option is incorrect. Necrosis is a form of cell death that occurs due to uncontro
lled injury or damage. It is characterized by cell swelling, membrane rupture, and the release of cellular
contents into the surrounding tissue. Necrosis does not involve the intrinsic or extrinsic pathways seen
in apoptosis, as it is not a programmed process.
Option C: Necroptosis: This option is incorrect. Necroptosis is a programmed form of necrosis, distinct f
rom apoptosis, that occurs under specific conditions. It is triggered by certain death receptors, such as
tumor necrosis factor receptor 1 (TNFR1) and involves a cascade of intracellular events leading to cell
death. However, necroptosis does not involve the intrinsic or extrinsic pathways associated with apopto
sis.

Page 5

173
Option D: Pyroptosis: This option is incorrect. Pyroptosis is a highly inflammatory form of programmed
cell death that occurs in response to infection or cellular stress. It is characterized by the formation of la
rge pores in the plasma membrane, leading to cell lysis and the release of pro-inflammatory cytokines.
Pyroptosis is distinct from apoptosis and does not involve the intrinsic or extrinsic pathways seen in ap
optosis.

Solution for Question 3:


Correct Option B - Cytomegalovirus:
• The histopathological image shows the characteristic owl’s eye appearance of intranuclear inclusion
bodies.
• The inclusion bodies are round with a central eosinophilic-stained core surrounded by a clear halo and
a chromatin ring.

Incorrect Options:
Option A - Human papillomavirus: On histological examination, the lesion shows multinucleated, perinu
clear halo, acanthosis, and koilocytes.

Page 6

174
Option C - Ebstein-Barr virus:
• The peripheral blood smear shows atypical T or Downey cells.
• The nucleus of these cells is kidney-shaped and contains fine chromatin without nucleoli, along with
basophilic, abundant, and finely granular cytoplasm.

Option D - Herpes simplex virus: Multinucleated giant cells with Cowdry type A
intranuclear inclusions are seen on microscopic examination of the vesicular fluids.

Page 7

175
Solution for Question 4:
Correct Option C - Hypertrophic scar:
• The scar is a hypertrophic scar as it is limited to the original wound boundaries, which is due to the
accumulation of collagen and will regress over time.
Incorrect Options:
Option A - Keloid: Keloid grows beyond the boundaries of the wound.
Option B - Exuberant granulation: Exuberant granulation shows granulation in excess that rises above t
he level of the surrounding skin and prevents re-epithelialization.
Option D
- Granulation tissue: The absence of granulations rules out the possibility of granulation tissue.

Solution for Question 5:


Correct Option A - Neutrophils:
• Neutrophils are the first cells to arrive at the site of injury.
• They are brought in response to chemotactic mediators upon vessel injury.
• They appear within 24 hours, and the peak is seen on the second day.
Incorrect Options:
• Options B, C, and D are incorrect.

Page 8

176
Solution for Question 6:
Correct Option C - VCAM-1:
• VCAM-1 helps in the adhesion of the leucocyte to the endothelium during inflammation.
Incorrect Options:
• Options A, B, and D are incorrect.

Solution for Question 7:


Correct Option C - 4, 1, 3, 2:
• The correct sequence of cellular events of inflammation is as follows: Migration: Leukocytes undergo
diapedesis, or transmigration, after firmly adhering to the endothelial cells. They migrate through the
endothelial junctions and move towards tissue injury or infection sites, guided by chemotactic signals.
This migration allows leukocytes to enter the tissue and participate in the inflammatory response.
Rolling: It involves the interaction between selectins on endothelial cells and their ligands on
leukocytes. This interaction causes leukocytes to roll along the endothelial surface. Adhesion: Activated
integrins on leukocytes bind to their ligands on endothelial cells, such as ICAM-1 and VCAM-1. This
adhesion stabilizes the interaction between leukocytes and endothelial cells, allowing leukocytes to
adhere firmly to the endothelial surface. Cytokine-mediated integrin activation: Inflammatory cytokines,
such as TNF-alpha and IL-1, are released at the site of inflammation. These cytokines induce the
activation of integrins in leukocytes. Integrins are cell adhesion molecules that bind to their ligands on
endothelial cells, promoting stronger adhesion.
• Migration: Leukocytes undergo diapedesis, or transmigration, after firmly adhering to the endothelial
cells. They migrate through the endothelial junctions and move towards tissue injury or infection sites,
guided by chemotactic signals. This migration allows leukocytes to enter the tissue and participate in
the inflammatory response.
• Leukocytes undergo diapedesis, or transmigration, after firmly adhering to the endothelial cells.
• They migrate through the endothelial junctions and move towards tissue injury or infection sites,
guided by chemotactic signals.
• This migration allows leukocytes to enter the tissue and participate in the inflammatory response.
• Rolling: It involves the interaction between selectins on endothelial cells and their ligands on
leukocytes. This interaction causes leukocytes to roll along the endothelial surface.
• It involves the interaction between selectins on endothelial cells and their ligands on leukocytes.
• This interaction causes leukocytes to roll along the endothelial surface.
• Adhesion: Activated integrins on leukocytes bind to their ligands on endothelial cells, such as ICAM-1
and VCAM-1. This adhesion stabilizes the interaction between leukocytes and endothelial cells,
allowing leukocytes to adhere firmly to the endothelial surface.
• Activated integrins on leukocytes bind to their ligands on endothelial cells, such as ICAM-1 and
VCAM-1.
• This adhesion stabilizes the interaction between leukocytes and endothelial cells, allowing leukocytes
to adhere firmly to the endothelial surface.
• Cytokine-mediated integrin activation: Inflammatory cytokines, such as TNF-alpha and IL-1, are
released at the site of inflammation. These cytokines induce the activation of integrins in leukocytes.

Page 9

177
Integrins are cell adhesion molecules that bind to their ligands on endothelial cells, promoting stronger
adhesion.
• Inflammatory cytokines, such as TNF-alpha and IL-1, are released at the site of inflammation.
• These cytokines induce the activation of integrins in leukocytes.
• Integrins are cell adhesion molecules that bind to their ligands on endothelial cells, promoting stronger
adhesion.
• Leukocytes undergo diapedesis, or transmigration, after firmly adhering to the endothelial cells.
• They migrate through the endothelial junctions and move towards tissue injury or infection sites,
guided by chemotactic signals.
• This migration allows leukocytes to enter the tissue and participate in the inflammatory response.
• It involves the interaction between selectins on endothelial cells and their ligands on leukocytes.
• This interaction causes leukocytes to roll along the endothelial surface.
• Activated integrins on leukocytes bind to their ligands on endothelial cells, such as ICAM-1 and
VCAM-1.
• This adhesion stabilizes the interaction between leukocytes and endothelial cells, allowing leukocytes
to adhere firmly to the endothelial surface.
• Inflammatory cytokines, such as TNF-alpha and IL-1, are released at the site of inflammation.
• These cytokines induce the activation of integrins in leukocytes.
• Integrins are cell adhesion molecules that bind to their ligands on endothelial cells, promoting stronger
adhesion.
Incorrect Options:
• Options A, B, and D are incorrect.

Solution for Question 8:


Correct Option A - Caspase-1:
• Caspase-1 is an enzyme involved in the activation of interleukin-1 (IL-1) in the innate immune
response.
• It cleaves the inactive precursor form of IL-1 into its active form, allowing it to exert its
pro-inflammatory effects.
• IL-1 plays a crucial role in the initiation and regulation of inflammatory responses.
Incorrect Options:
Options B, C, and D are not involved in the activation of IL-1.

Solution for Question 9:


Correct Option C - IL-1 is an endogenous pyrogen:

Page 10

178
• IL-1 is released by immune cells in response to infection or inflammation and acts on the
hypothalamus to raise the body's temperature set-point, leading to fever.
Incorrect Options:
Option A - Aspirin increases fever by inhibiting prostaglandin synthesis: Aspirin reduces fever by inhibiti
ng prostaglandin synthesis.
Option B - PGD2 is responsible for resetting the temperature to a higher level: PGD2 is involved in slee
p regulation and inflammation but is not responsible for resetting the temperature to a
higher level during fever.
Option D - Fever occurs during inflammation due to release of histamine: Histamine release can contri
bute to the symptoms of inflammation, but it is not the primary cause of fever.

Solution for Question 10:


Correct Option B - Cat-scratch disease:
• Stellate granuloma is a characteristic histopathological finding in cat-scratch disease.
• Cat-scratch disease is caused by the bacteria Bartonella henselae, which is typically transmitted to
humans through scratches or bites from infected cats.
• Stellate granulomas are collections of histiocytes with radiating, slender cytoplasmic projections,
giving them a star-shaped appearance.
• These granulomas are commonly found in the lymph nodes near the site of inoculation.
Incorrect Options:
• Stellate granuloma is not seen in options A, C, and D.

Solution for Question 11:


Correct Option D - Histamine:
• Histamine is released during allergic reactions and is involved in immediate hypersensitivity reactions.
Incorrect Options:
• Fever occurs during inflammation due to options A, B, and C.

Solution for Question 12:


Correct Option B - C3b:
• C3b is a component of the complement system.
• When C3b binds to the surface of pathogens, it acts as an opsonin, facilitating the binding of
phagocytic cells to the pathogen.
• This enhances phagocytosis and promotes the clearance of pathogens.

Page 11

179
Incorrect Options:
Option A - C3a: C3a acts as an inflammatory mediator.
Option C - C5a: C5a acts as a potent chemoattractant for immune cells.
Option D - C6: C6 participates in the formation of the membrane attack complex, which helps destroy p
athogens.

Solution for Question 13:


Correct Option A - Inflammasome:
• Inflammasomes are multiprotein complexes found within cells of the innate immune system.
• When activated by various danger signals, including necrotic cell debris, inflammasomes assemble
and activate caspase-1, which leads to the processing and release of pro-inflammatory cytokines IL-1β
and IL-18.
• While the primary function of inflammasomes is not the direct recognition and clearance of necrotic
cells, their activation in response to necrotic cell debris can contribute to the induction of inflammatory
responses and the clearance of necrotic cells by attracting immune cells.
Incorrect Options:
• Options B, C, and D are not involved in the recognition and clearance of necrotic cells.

Solution for Question 14:


Correct Option A - Lung cancer:
• Lung cancer is the most common primary cancer that metastasizes to the brain.
• Due to their ability to spread through the bloodstream or lymphatic system, lung cancer cells can
reach the brain and form metastatic tumors.
Incorrect Options:
• Options B, C, and D are incorrect.

Solution for Question 15:


Correct Option C - Apoptosis:
• Apoptosis is a controlled and orderly process of programmed cell death.
• It does not induce an acute inflammatory response but rather leads to the removal of dying cells by
phagocytosis without provoking an intense immune response.
Incorrect Options:
• Options A, B, and D can trigger an acute inflammatory response.

Page 12

180
Solution for Question 16:
Correct Option A - Histamine:
• Histamine is a potent vasodilator released during allergic reactions and inflammation.
• It acts on histamine receptors in blood vessel walls, causing relaxation of smooth muscle and resulting
in vasodilation.
Incorrect Options:
Option B - Thromboxane A2:
• Thromboxane A2 is a vasoconstrictor and platelet aggregator.
• It promotes platelet aggregation and constriction of blood vessels, leading to vasoconstriction rather
than vasodilation.
Option C - Angiotensin II:
• Angiotensin II is a vasoconstrictor that can cause the narrowing of blood vessels.
Option D - Leukotriene C4:
• Leukotriene C4 is a pro-inflammatory mediator that is associated with vasoconstriction and
bronchoconstriction.

Solution for Question 17:


Correct Option A - 1-C, 2-D, 3-A, 4-B:
Incorrect Options:
• Options B, C, and D are incorrect.

Page 13

181
Introduction and Mendelian Disorders
1. What is the mode of inheritance in Haemophilia A?
A. Autosomal recessive disorder
B. Autosomal dominant disorder
C. X-linked recessive disorder
D. X-linked dominant disorder
----------------------------------------
2. In the context of studying risk factors for diabetes mellitus, how many genes are estimated to be
present in the human genome?
A. 40,000
B. 20,000
C. 80,000
D. 1,00,000
----------------------------------------
3. What's the likely diagnosis for a 15-year-old boy presenting with fatigue, shortness of breath, tall
stature, exceptionally long extremities, tapering fingers and toes, mitral valve prolapse murmur, and
ectopia lentis in both eyes?
A. Marfan’s syndrome
B. Marchesani’s syndrome
C. Homocystinuria
D. Ehler Danlos syndrome
----------------------------------------
4. What type of transmission is depicted by widespread thickened skin, keratotic lesions, and thickened
nails observed in a 14-year-old boy whose father also has the same condition?
(or)

A. AD Inheritance
B. AR Inheritance
C. X linked recessive

182
D. Holandric inheritance
----------------------------------------
5. In fragile X syndrome, what causes mental impairment across generations in male family members?
A. Mitochondrial DNA mutation
B. Frameshift mutation
C. Y-linked disorder
D. Trinucleotide repeat mutation
----------------------------------------
6. What property characterizes autosomal dominant disorder in a 5-year-old boy presenting with
recurrent joint dislocations, easy bruising, fragile skin, and mitral valve prolapse due to genetic defects
in collagen?
(or)
Which is true of autosomal dominant disorder in Ehlers-Danlos syndrome?
A. Affects males only
B. Vertical inheritance
C. Complete penetrance
D. Constant expressivity
----------------------------------------
7. What is the most common cause of death in patients with Marfan syndrome.?
A. Liver disease
B. Myocardial infarction
C. Recurrent infection
D. Aortic pathology
----------------------------------------
8. Which genetic concept best explains the difference in severity of Marfan syndrome manifestations
between a 22-year-old male presenting with mild skeletal abnormalities and his brother who
experienced severe life-threatening features in the early disease course?
(or)
Which genetic theory accounts for the boy's moderate skeletal defects while having severe,
life-threatening Marfan syndrome symptoms in the same family and his sibling, who died of an aortic
dissection?
A. Complete penetrance
B. Mosaicism
C. Incomplete penetrance
D. Variable expressivity
----------------------------------------
9. What's the mode of inheritance of the disease in a 56-year-old male presenting with abnormal neck
posture, declining intellectual ability, nystagmus, choreiform movements, brain MRI showing caudate

Page 2

183
nucleus atrophy, and a family history of similar symptoms in his father?
(or)
What is the mode of inheritance occurs in Huntington's disease?
A. Autosomal recessive
B. X-linked recessive
C. Autosomal dominant
D. X-linked dominant
----------------------------------------
10. What is the pattern of inheritance in patients with cystic fibrosis?
A. Autosomal recessive
B. X-linked recessive
C. Autosomal dominant
D. X-linked dominant
----------------------------------------
11. Which of the following is true regarding the methylation of cytosine in the genome?
A. Increases the expression of the gene
B. Decreases the expression of the gene
C. No effect on gene expression
D. Mutation
----------------------------------------
12. Among the choices given below, which of the following is least likely to occur due to a defect in the
homologous recombination repair mechanism?
A. Fanconi's anaemia
B. Ehler-Danlos syndrome
C. Bloom syndrome
D. Ataxia telangiectasia
----------------------------------------
13. What type of mutation is associated with a 35-year-old man presenting with personality changes,
jerky limb movements, family history of early-onset dementia, restlessness, irregular movements of
extremities, twitching of face and tongue, and impaired memory?
(or)
What kind of mutations cause Huntington's disease?
A. Point mutation
B. Gene deletion
C. Frameshift mutation
D. Trinucleotide repeat expansion

Page 3

184
----------------------------------------
14. In a 35-year-old male with bone pain, splenomegaly, and pancytopenia, which substance is most
likely to accumulate based on the histological features observed in a spleen biopsy image given below?

A. Glucocerebroside
B. Sphingomyelin
C. Sulfatide
D. Ganglioside
----------------------------------------
15. What are the chances of having a child who is unaffected if both parents have achondroplasia ("Aa"
and "Aa")?
A. 0%
B. 25%
C. 50%
D. 100%
----------------------------------------
16. What could be the mode of inheritance in a family where a 2-year-old male child exhibits intellectual
disabilities, kinky hair, pudgy rosy cheeks, and sagging facial skin, with a family history of similar
features in his two maternal cousins and his mother's elder brother?
(or)
What is the mode of inheritance in Menkes disease?

A. X-linked recessive

Page 4

185
B. X-linked dominant
C. Autosomal recessive
D. X-linked dominant
----------------------------------------
17. What are the odds of a 30-year-old female's children having sickle cell illness and sickle cell trait,
respectively, given her history of low hemoglobin, splenomegaly, and her mother's sickle cell anemia?
A. 0% and 100%
B. 25% and 50%
C. 50% and 50%
D. 100% and 0%
----------------------------------------
18. What's the mode of inheritance for the lack of clotting factor VIII observed in a 13-year-old boy
whose mother and elder brother are affected, but none of his 2 sisters show symptoms despite
presenting with easy bruising and knee swelling after a minor fall?
(or)
What is the mode of inheritance in Hemophilia A?
A. X-linked recessive
B. Autosomal dominant
C. X-linked dominant
D. Autosomal recessive
----------------------------------------
19. What's true about the mode of inheritance of the disease affecting a 10-year-old boy who presents
with difficulty walking, standing from a sitting position, and going upstairs, with a family history of a
similar condition in his mother's brother who died at 23?
(or)
Which of the following is true regarding the Duchenne muscular dystrophy X-linked recessive trait?
A. 50% of boys of carrier mothers are affected
B. 50% of girls with diseased fathers are carrier
C. Father transmits the disease to the son
D. Mother transmits the disease to the daughter
----------------------------------------
20. What's the inheritance pattern of a 15-year-old boy's difficulty distinguishing some shades of red
and green on the Ishihara chart, while both his parents have no such difficulty?
A. Autosomal dominant inheritance
B. Autosomal recessive inheritance
C. X-linked recessive inheritance
D. Mitochondrial inheritance

Page 5

186
----------------------------------------

Correct Answers
Question Correct Answer

Question 1 3
Question 2 2
Question 3 1
Question 4 4
Question 5 4
Question 6 2
Question 7 4
Question 8 4
Question 9 3
Question 10 1
Question 11 2
Question 12 2
Question 13 4
Question 14 1
Question 15 2
Question 16 1
Question 17 1
Question 18 1
Question 19 1
Question 20 3

Solution for Question 1:


Correct Option C - X-linked recessive disorder:
• Haemophilia A, which is an X-linked recessive disorder.
• They manifest only in males.
• A male with an affected allele on his single X chromosome is hemizygous and can not transmit the
disorder to their male offspring, but all his daughters would be carriers.
• Healthy heterozygous carrier females pass the disorder to affected sons.
• So from affected males, it can be transmitted to male grandchildren through carrier daughter
Incorrect Options:
Option A - Autosomal recessive disorder:
• Haemophilia is not an Autosomal recessive disorder.

Page 6

187
• In an autosomal recessive type inheritance pattern, 2 copies of the same gene must be mutated for an
individual to express the disease.
• An affected individual usually has unaffected parents, each of whom carries a single copy of the
mutated gene (carriers) and normally does not exhibit disease symptoms.
Option B - Autosomal dominant disorder:
• Haemophilia is not an Autosomal dominant disorder.
• A single affected parent has a 50% chance of transferring the mutated gene to their child.
• The mutated genes are located on non-sex chromosomes.
Option D - X-linked dominant disorder :
• Haemophilia is not an X-linked dominant disorder.
• The X-linked dominant disorder is a mode of inheritance in which an X chromosome carries the
dominant gene.
• This means that the gene causing the disorder is on the X chromosome.
• Only one copy of the allele is sufficient to express the disease if inherited from a diseased
parent. Females are more affected in number. Affected males can transmit the disorder only to their
daughters, while affected females can transmit the disorder to sons as well as daughters.

Solution for Question 2:


Correct Option B - 20,000:
• The human genome is a comprehensive set of nucleic acid sequences for humans, encoded in DNA
within the 23 pairs of chromosomes and in small DNA molecules found within individual mitochondria.
• There are roughly 3.2 billion DNA base pairs in the human genome.
• Out of the total genome, about 20,000 protein-encoding genes comprise only 1.5% of the genome.
• There are, in fact, many human genome sequences, as every individual has their version. The
variances between individual genomes are mainly due to single nucleotide polymorphisms (SNPs),
positions in the genome where some individuals have one nucleotide and others have a different
nucleotide.
Incorrect Options:
Option A - 40,000:
• The human genome does not contain 40,000 protein-encoding genes
• It only contains 20.000 genes.
Option C - 80,000:
• The human genome does not contain 80,000 protein-encoding genes
• It only contains 20.000 genes.
Option D - 1,00,000:
• The human genome does not contain 1.00,000 protein-encoding genes
• It only contains 20.000 genes.

Page 7

188
Solution for Question 3:
Correct Option A - Marfan syndrome:
• The case mentioned above has clinical features of fatigue and shortness of breath, and examination
findings of long extremities, tapering fingers and toes, and mitral valve prolapse; suggest Marfan's
syndrome.
• Marfan's syndrome is an autosomal dominant connective tissue disorder.
• It is caused by a defect in the FBN1 gene on chromosome 15 that codes for the protein fibrillin-1.
• It affects around 1 in 3,000 people.
• Loss of fibrillin leads to clinical manifestations of Marfan syndrome.
• There is a loss of structural support in microfibril-rich connective tissue and excessive activation of
TGF-β signaling:
• Skeletal and cardiac abnormalities in Marfan syndrome include: Unusually tall patients with Tapering
fingers and toes and exceptionally long extremities Cardiovascular lesions (Mitral valve prolapse, Aortic
Dissection) Subluxation or dislocation (usually outward and upward) of the lens, referred to as ectopia
lentis. Spinal deformities like kyphosis, and scoliosis. The chest is classically deformed, presenting
either pectus excavatum (deeply depressed sternum) or a pigeon chest deformity.
• Unusually tall patients with Tapering fingers and toes and exceptionally long extremities
• Cardiovascular lesions (Mitral valve prolapse, Aortic Dissection)
• Subluxation or dislocation (usually outward and upward) of the lens, referred to as ectopia lentis.
• Spinal deformities like kyphosis, and scoliosis.
• The chest is classically deformed, presenting either pectus excavatum (deeply depressed sternum) or
a pigeon chest deformity.
• Unusually tall patients with Tapering fingers and toes and exceptionally long extremities
• Cardiovascular lesions (Mitral valve prolapse, Aortic Dissection)
• Subluxation or dislocation (usually outward and upward) of the lens, referred to as ectopia lentis.
• Spinal deformities like kyphosis, and scoliosis.
• The chest is classically deformed, presenting either pectus excavatum (deeply depressed sternum) or
a pigeon chest deformity.

Page 8

189
• The life expectancy of patients is around 40-50 years.
• With regular echocardiography monitoring and beta-blocker/ACE-inhibitor therapy, life expectancy
has improved significantly.
• Aortic dissection and other cardiovascular problems remain the leading cause of death in patients with
Marfan syndrome.

Incorrect Options:
Option B - Marchesani’s syndrome:
• Weill-Marchesani syndrome, also known as Spherophakia-Brachymorphia syndrome and
Mesodermal dysmorphodystrophy, is an inherited connective tissue disorder.
• It is characterised by eye lens abnormalities, secondary glaucoma, short stature, brachydactyly, joint
stiffness, and cardiovascular defects.
Option C – Homocystinuria:
• Homocystinuria is a rare autosomal recessive disease caused by cystathionine beta-synthase
deficiency.
• This results in severe elevations in plasma and urine homocysteine concentrations.
Option D - Ehler Danlos syndrome:
• Ehler-Danlos syndrome is an autosomal dominant connective tissue disorder that mostly affects type
III collagen.
• This resulted in the tissue being more elastic than normal, leading to joint hypermobility and increased
skin elasticity.

Solution for Question 4:


Correct Option D - Holandric inheritance:

Page 9

190
• Father only transfers the feature to the male child in the subsequent generation.
• Y-linked inheritance or holandric inheritance.
• Some examples of the holandric genes are: Hypertrichosis on the pinna Porcupine skin Keratoderma
dissipated Webbed toes
• Hypertrichosis on the pinna
• Porcupine skin
• Keratoderma dissipated
• Webbed toes
• Hypertrichosis on the pinna
• Porcupine skin
• Keratoderma dissipated
• Webbed toes

Incorrect Options:
Option A - Autosomal dominant Inheritance:
• One copy of a mutated gene from one parent can cause the genetic condition.

Page 10

191
Option B - Autosomal recessive:
• In autosomal recessive inheritance, a genetic condition occurs when the child inherits one mutated
copy of a gene from each parent.
• The parents usually do not have the condition.

Option C - X –linked dominant:


• The X-linked dominant pattern is rare.
• Both male and female offspring of an affected female can have the trait.

Solution for Question 5:

Page 11

192
Correct Option D - Trinucleotide repeat mutation:
• The likely diagnosis in the patient described in the vignette is Fragile X Syndrome. The child
demonstrates classical features of this condition such as intellectual disability, large face and mandible,
large everted ears and macroorchidism.
• The severity of mental impairment among male family members has increased through generations
due to trinucleotide repeat mutation.
• Fragile X syndrome is a trinucleotide repeat disorder.
• Features in males include: Difficulty in learning May have large, low, set ears, a long thin face, and
high arched palate. Macro-orchidism Hypotonia Autism Mitral valve prolapse
• Difficulty in learning
• May have large, low, set ears, a long thin face, and high arched palate.
• Macro-orchidism
• Hypotonia
• Autism
• Mitral valve prolapse
• Symptoms in females with one fragile chromosome and one normal X chromosome range from
normal to mild.
• Diagnosis can be made antenatally by chorionic villus sampling or amniocentesis and by analysis of
the number of CGG repeats using restriction endonuclease digestion and Southern blot analysis.
• Trinucleotide repeats cause worsening of clinical features with each successive generation.
• The unstable trinucleotide repeats result in defects in a protein encoded by a gene, changed gene
expression regulation, and production of a toxic RNA that may lead to chromosome instability.
• Difficulty in learning
• May have large, low, set ears, a long thin face, and high arched palate.
• Macro-orchidism
• Hypotonia
• Autism
• Mitral valve prolapse
Incorrect Options:
Option A - Mitochondrial DNA mutation:
• Mitochondrial DNA mutations accumulate with age and disease in mitotic tissues, such as the colon,
and post-mitotic cells, such as muscle fibres and neurons.
• At low levels, the mitochondrial DNA mutation will have a little functional impact.
Option B - Frameshift mutation:
• Crohn's disease, cystic fibrosis, and certain types of cancer are due to frameshift mutations.
• Trinucleotide repeats because clinical features worsen with each successive generation of Fragile X
syndrome.
Option C - Y-linked disorder:

Page 12

193
• Examples of Y-linked inheritance are hypertrichosis of the ears, webbed toes, and porcupine.
• Trinucleotide repeats because clinical features worsen with each successive generation of Fragile X
syndrome.

Solution for Question 6:


Correct Option B - Vertical inheritance:
• Based on the features of right shoulder joint dislocation, recurrent episodes of joint dislocation and
easy bruising, examination revealing fragile skin, and auscultation findings suggest mitral valve
prolapse. His features are consistent with Ehlers-Danlos syndrome.
• Genetic defects in collagen cause this disorder.
• One copy of a mutated gene from one parent can cause the genetic condition.
• The autosomal dominant disorder is a heterozygosity expression in which 50% of progeny are
affected with a single affected parent .
• It has vertical inheritance (at least 1 affected member is seen in each generation).
• It affects both females and males equally.
• Autosomal disorder is associated with the involvement of structural protein.
• There is incomplete penetrance, and not complete penetrance is a feature of the autosomal dominant
disorder. (Penetrance is the percentage of individuals who express the phenotype associated with that
underlying genotype. Expressivity refers to the degree to which a particular genotype is expressed as a
phenotype within an individual).
• Autosomal inheritance has variable expressivity (some affected individuals have mild, and some
patients have moderate or severe disease).

Incorrect Options:
Option A - Affects males only:

Page 13

194
• It affects both females and males equally.
Option C - Complete penetrance:
• One copy of a mutated gene from one parent can cause the genetic condition.
• Autosomal dominant disorders are characterized by incomplete penetrance
Option D - Constant expressivity:
• It has variable expressivity (some affected individuals have mild, and some patients have moderate or
severe disease).

Solution for Question 7:


Correct Option D - Aortic pathology:
• Based on fatigue and shortness of breath over the previous 2 months, tall for his age, examination
reveals exceptionally long extremities along with tapering fingers and toes, a murmur suggestive of
mitral valve prolapse. The clinical features confirm a diagnosis of Marfan syndrome
• Aortic root disease, leading to aortic regurgitation, aneurysmal dilatation, and dissection, is the
primary cause of morbidity and mortality in Marfan's syndrome (MFS), in up to 60% to 80% of patients
• Marfan's syndrome is an autosomal dominant connective tissue disorder.
• It is caused by a defect in the FBN1 gene on chromosome 15 that codes for the protein fibrillin-1.
• It affects around 1 in 3,000 people.
• It leads to a defect in an extracellular glycoprotein called fibrillin-1.
• Loss of fibrillin leads to clinical manifestations of Marfan syndrome.
• There is a loss of structural support in microfibril-rich connective tissue and excessive activation of
TGF-β signalling:
• Skeletal abnormalities in Marfan syndrome include. Unusually tall patients Tapering fingers and toes
Exceptionally long extremities and long:
• Unusually tall patients
• Tapering fingers and toes
• Exceptionally long extremities and long:
• Unusually tall patients
• Tapering fingers and toes
• Exceptionally long extremities and long:
Incorrect Options:
Option A - Liver disease:
• Marfan syndrome is an autosomal dominant connective tissue disorder.
• It does not affect the liver.
• Aortic disease and other cardiovascular problems remain the leading cause of death in patients with
Marfan syndrome.

Page 14

195
Option B - Myocardial infarction:
• Marfan syndrome is an autosomal dominant connective tissue disorder.
• It does not lead to myocardial infarction.
• Aortic disease and other cardiovascular problems remain the leading cause of death in patients with
Marfan syndrome.
Option C - Recurrent infection:
• Marfan syndrome is an autosomal dominant connective tissue disorder.
• Recurrent infections do not cause death in these patients.
• Aortic disease and other cardiovascular problems remain the leading cause of death in patients with
Marfan syndrome.

Solution for Question 8:


Correct Option D - Variable expressivity:
• In genetics, expressivity is defined as the degree to which a genetic trait expression differs among
individuals.
• People with the same genotype can also show altered degrees of the same phenotype.
• Expressivity defines individual variability of the phenotypic expression of a gene.
• Regarding Marfan syndrome, some individuals have only mild symptoms, like tall and thin with long,
slender fingers. On the other hand, others will be experiencing life-threatening cardiovascular
complications. Despite variable features of Marfan syndrome among these individuals, all people with
this disorder will have a dominant mutation in the FBN1 gene, encoding the fibrillin 1 protein.
• Variable expressivity occurs due to a change in the position of the mutation in the FBN1 gene.
• A mutation in one FBN1 position is predominant in individuals with severe symptoms, whereas a
mutation in another FBN1 position is predominant in individuals with less severe symptoms.
Incorrect Options:
Option A - Complete penetrance:
• Complete penetrance does not explain variations in the degree or extent of phenotypic expression of
a gene mutation.
• Penetrance is the proportion of individuals in a population who are carrying a disease-causing allele
and expressing the disease phenotype.
• This implies that not all individuals carrying the mutated gene will express the disease.
• Individuals who carry a particular mutated gene and express the disease phenotype display complete
penetrance. In comparison, those with the same mutant gene but do not display phenotype at all are
said to display reduced or incomplete phenotype.
Option B – Mosaicism:
• Mosaicism does not explain variations in the degree or extent of phenotypic expression of a gene
mutation.
• Mosaicism describes more than one cell type in a single individual.

Page 15

196
• For instance, a person having some cells with 46 chromosomes and some cells with 47 chromosomes
will say to have genetic mosaicism. Examples include Down's syndrome.
Option C - Incomplete penetrance:
• Variation in the degree or extent of phenotypic expression of a gene mutation is not explained by
incomplete penetrance.
• Penetrance is the proportion of individuals in a population who are carrying a disease-causing allele
and expressing the disease phenotype.
• That individual who carries a mutant gene but does not display phenotype at all is said to display
reduced or incomplete phenotype.

Solution for Question 9:


Correct Option C - Autosomal dominant:
• Huntington's disease, also known as Huntington's chorea, is a neurodegenerative disease mostly
inherited in an autosomal dominant fashion.
• Huntington’s disease is classically inherited from an affected parent carrying a mutant Huntington
gene. The Huntington gene codes for a specific protein known as the Huntington protein. Expansion of
CAG (cytosine-adenine-guanine) repeats, known as a trinucleotide repeat expansion in the gene
coding for the Huntington protein, produces an abnormal mutant protein, which steadily damages
neurons.
• In an autosomal dominant inheritance pattern, mutated genes are located on non-sex chromosomes,
and a single copy of the mutated gene is enough to express the disease.
• A single affected parent has a 50% chance of transferring mutated genes to their children.
• In autosomal dominant inheritance, males and females are both affected.
• A distinctive feature of autosomal dominant inheritance is penetrance, which renders an individual
variable expression of disease.
Incorrect Options:
Option A - Autosomal recessive:
• Huntington's disease is not an autosomal recessive disorder.
• Examples- Cystic fibrosis (CF) Sickle cell anemia (SC) Tay Sachs disease.
• Cystic fibrosis (CF)
• Sickle cell anemia (SC)
• Tay Sachs disease.
• Cystic fibrosis (CF)
• Sickle cell anemia (SC)
• Tay Sachs disease.
Option B - X-linked recessive:
• Huntington's disease is not an X-linked recessive disorder.
• Examples Duchenne muscular dystrophy Some types of colorblindness Hemophilia A

Page 16

197
• Duchenne muscular dystrophy
• Some types of colorblindness
• Hemophilia A
• Duchenne muscular dystrophy
• Some types of colorblindness
• Hemophilia A
Option D - X-linked dominant:
• Huntington's disease is not an X-linked dominant disorder.
• Families with an X-linked dominant disorder often have both affected males and affected females in
each generation. A striking characteristic of X-linked inheritance is that fathers cannot pass X-linked
traits to their sons; fathers only pass X chromosomes to their daughters and Y chromosomes to their
sons.

Solution for Question 10:


Correct Option A - Autosomal recessive:
• Cystic fibrosis is an autosomal recessive disorder affecting exocrine gland function, most prevalent in
Northern European individuals at a rate of 1 in 3500.
• It is a chronic disease that often leads to chronic sino-pulmonary infections and pancreatic
insufficiency. The most common cause of death is end-stage pulmonary disease.
• It is caused by a mutation in a gene on chromosome 7 that encodes a protein transmembrane
conductance regulator (CFTR) protein. This protein functions as a transmembrane cAMP-activated
chloride channel.
• Being autosomal recessive, both gene copies are mutated in clinical disease.
• Autosomal recessive conditions result from mutations in genes on the autosomal chromosomes.
• Disease outcomes for a child with carrier patients in autosomal recessive disease include: 25%
chance that the child is born with two normal genes and will neither be a carrier nor express the
disease. 50% chance that the child is born with one abnormal and one normal gene, i.e., would be a
carrier without the disease. 25% chance that the child is born with two abnormal genes, i.e., at risk for
the disease.
• ■■■25% chance that the child is born with two normal genes and will neither be a carrier nor express
the disease.
• 50% chance that the child is born with one abnormal and one normal gene, i.e., would be a carrier
without the disease.
• 25% chance that the child is born with two abnormal genes, i.e., at risk for the disease.
• ■■■25% chance that the child is born with two normal genes and will neither be a carrier nor express
the disease.
• 50% chance that the child is born with one abnormal and one normal gene, i.e., would be a carrier
without the disease.
• 25% chance that the child is born with two abnormal genes, i.e., at risk for the disease.

Page 17

198
Incorrect Options:
Option B - X-linked recessive:
• Cystic fibrosis is not an x-linked recessive disorder.
• X-linked recessive conditions result from mutations in genes on the X chromosome.
• Males are much more frequently affected as compared to females.
• Examples include Hemophilia A.
Option C - Autosomal dominant:
• In an autosomal dominant inheritance pattern, mutated genes are located on non-sex chromosomes,
and a single copy of the mutated gene is enough to express the disease.
• Affected parents have a 50% chance of transferring mutated genes to their children.
• In autosomal dominant inheritance, males and females are both affected.
• Examples include Achondroplasia and autosomal dominant polycystic kidney disease.
Option D - X-linked dominant:
• X-linked dominant inheritance is a mode of inheritance in which an X chromosome carries the
dominant gene. This means that the gene causing the disorder is located on the X chromosome, and
only one copy of the allele is sufficient to express the disease if inherited from a diseased parent.
• The affected father, carrying the x-linked dominant disorder, will pass on the disease to all the
daughters but not to any son.
• If the mother is affected, then sons will have a chance of being affected depending on whether a
recessive or dominant X chromosome is passed on.

Solution for Question 11:


Correct Option B - Decreases the expression of the gene:
DNA Methylation:
• DNA methylation regulates gene expression by recruiting proteins involved in gene repression or
inhibiting transcription factor(s) binding to DNA.
• Methylation within gene promoter (CpG islands) typically represses (silences) gene transcription.
Methylation makes DNA mute
• Methylation is essential for normal development and is associated with a number of key processes,
including genomic imprinting, X-chromosome inactivation, repression of transposable elements,
ageing, and carcinogenesis.
Incorrect Options:
Option A - Increases the expression of the gene:
• Acetylation increases the expression of the gene (Histone acetylation).
• When histones are acetylated, their electrostatic interactions with DNA become weaker, resulting in
relaxed chromatin, which upregulates transcription. The opposite happens when histones are
deacetylated

Page 18

199
• Dysregulated acetylation is implicated in Huntington’s disease.
• Acetylation makes DNA active
Option C - No effect on gene expression:
• Methylation decreases the expression of genes.
• Therefore this option is incorrect
Option D - Mutation:
• Methylation does not change the sequence of the genome.
• It only decreases the expression of the DNA segments.
• Therefore this option is incorrect

Solution for Question 12:


Correct Option B - Ehler-Danlos syndrome:
• Ehlers-Danlos syndrome (EDS) comprises a spectrum of genetic connective tissue disorders
characterized by abnormalities in collagen synthesis or structure. Unlike conditions primarily linked to
homologous recombination repair defects, EDS commonly results from mutations affecting genes
encoding collagen
Homologous DNA repair:
• Repair by exchanging homologous segments between two DNA molecules (sister chromatids)
• The error can be repaired using the complementary strand from the intact sister chromatid.
• Requires a (nearly) identical sequence (such as the complementary strand) to serve as a template for
repair
• No DNA is lost in the process.
• The defects in this process are bloom syndrome, Fanconi anaemia, breast cancer and ataxia
telangiectasia.

Incorrect Options:
Option A - Fanconi's anaemia:
• Fanconi anaemia is an autosomal recessive disorder with mutations may cause in more than a dozen
different genes, each encoding a protein that participates in a pathway that repairs DNA cross-links
through homologous recombination.
• It is characterized by developmental abnormalities (short stature, skeletal abnormalities),
hypersensitivity to chemotherapeutic agents that cross-link DNA, and increased risk of bone marrow
failure (aplasia) and leukaemia
Option C - Bloom syndrome:
• Bloom syndrome is an autosomal recessive disorder caused by loss-of-function mutations in a
helicase required for homologous recombination repair.
• Affected individuals have developmental anomalies and an increased risk of developing many
different types of cancer

Page 19

200
Option D - Ataxia telangiectasia:
• Ataxia telangiectasia is an autosomal recessive disorder caused by defects in ATM, a gene encoding
a kinase that acts upstream of p53 or cellular DNA repair, cell cycle control, and cellular response to
external triggers, such as oxidative damage, ionizing radiation, and alkylating agents.
• This syndrome is characterized by neurodegeneration (particularly of the cerebellum, hence the
ataxia), telangectasia, immunodeficiency, hypersensitivity to radiation (due to an inability to repair
double-stranded DNA breaks), and predisposition to cancer, particularly certain forms of leukemia and
lymphoma.
• Somatic driver mutations in ATM are also common in certain lymphoid neoplasms.

Solution for Question 13:


Correct Option D - Trinucleotide repeat expansion:
• Huntington's disease is caused by trinucleotide repeat expansion
Huntington disease:
• HD is an autosomal dominant disease caused by the degeneration of striatal neurons and
characterized by progressive movement disorder and dementia.
• Jerky, hyperkinetic, sometimes dystonic movements involving all parts of the body (chorea) are
characteristic; affected individuals may later develop bradykinesia and rigidity.
• The disease is relentlessly progressive and uniformly fatal, with an average course of about 15 years.
Pathogenesis:
• Huntington's disease (HD) is a polyglutamine trinucleotide repeat expansion disorder caused by the
HTT gene on chromosome 4p16.3.
• Normal genes have 6 to 35 CAG repeats, but exceeding this range leads to the disease.
• Longer repeats correlate with earlier onset.
Clinical features:
• The loss of striatal neurons, which dampen motor activity, results in increased motor output, often
manifested as choreoathetosis.
• The cognitive changes associated with the disease are probably related to neuronal loss in the
cerebral cortex.
• The age at onset is most common in the fourth and fifth decades and is related to the length of the
CAG repeat in the HTT gene.
• Motor symptoms often precede cognitive impairment.
• The movement disorder of HD is choreiform, with increased and involuntary jerky movements of all
body parts; writhing movements of the extremities are typical.
• Early symptoms of higher cortical dysfunction include forgetfulness as well as cognitive and affective
dysfunction, with progression to a severe dementia
Incorrect Options:
Option A - Point mutation:

Page 20

201
• A point mutation is a genetic mutation where a single nucleotide base is changed, inserted or deleted
from a DNA or RNA sequence of an organism's genome.
• Point mutations have various effects on the downstream protein product—consequences that are
moderately predictable based on the specifics of the mutation.
• Different blood disorders such as ß-thalassemia, sickle cell disease, hereditary spherocytosis,
Fanconi anaemia, and Hemophilia A and B are usually caused by point mutations.
Option B - Gene deletion:
• A deletion, as related to genomics, is a type of mutation that involves the loss of one or more
nucleotides from a segment of DNA.
• A deletion can involve the loss of any number of nucleotides, from a single nucleotide to an entire
piece of a chromosome.
• Deletions are responsible for an array of genetic disorders, including some cases of male infertility,
two-thirds of cases of Duchenne muscular dystrophy, and two-thirds of cases of cystic fibrosis
Option C - Frameshift mutation:
• A frameshift mutation is a genetic mutation caused by the indels of several nucleotides in a DNA
sequence that is not divisible by three.
• Due to the triplet nature of gene expression by codons, the insertion or deletion can change the
reading frame, resulting in a completely different translation from the original.
• Crohn's disease, cystic fibrosis, and certain types of cancer are due to frameshift mutations.

Solution for Question 14:


Correct Option A – Glucocerebroside:
• The boy is most likely suffering from Gaucher disease, in which glucocerebrosides accumulate.

Gaucher disease:

Page 21

202
Aetiology:
• Autosomal recessive inherited disease
Epidemiology
• Most common lysosomal lipid storage disease. Increased incidence of type I in the Ashkenazi Jewish
population
Pathophysiology:
• Deficiency of β-glucocerebrosidase → accumulation of glucocerebroside (sphingolipid found in cell
membranes that can accumulate in the lysosome of macrophages) in the brain, liver, spleen, and bone
marrow
Clinical features
• Vary according to the exact subtype of Gaucher disease
• Type I: non-neuronopathic Gaucher disease
• Type II: acute neuronopathic
• Gaucher disease Type III: chronic neuronopathic Gaucher disease
All types
• Hepatosplenomegaly
• Bone: bone crises, osteoporosis, avascular necrosis of the femur
• Blood abnormalities: anemia, thrombocytopenia, pancytopenia
• Pulmonary manifestations, Growth delays
Diagnostics
• Reduced glucocerebrosidase activity in leukocytes or fibroblasts
• Accumulation of glucocerebroside in leukocytes or fibroblasts
• Gaucher cell: lipid-rich macrophages with an enlarged cytoplasm with inclusions that resemble
crumpled tissue paper on microscopy

• This image shows a Gaucher cell in Wright stain.

Page 22

203
Treatment:
• Enzyme replacement therapy with recombinant glucocerebrosidase or substrate reduction therapy
with eliglustat
Incorrect Options:
Option B – Sphingomyelin:
• Sphingomyelin accumulation is seen in Niemann-Pick disease.
• Light microscopy: lipid-laden macrophages (foam cells) in the bone marrow, spleen, and liver
• Electron microscopy: Zebra bodies (abnormal configuration of myeloid membranes into parallel
palisading lamellae in the lysosomal cytoplasm
Option C – Sulfatide:
• Arylsulfatase A is deficient in metachromatic leukodystrophy
• The cerebroside sulfate accumulates
• The presentation includes demyelination in CNS and PNS, resulting in ataxia and dementia
• The inheritance is autosomal recessive.
Option D – Ganglioside:
• Ganglioside accumulation is seen in Tay Sach disease
• Hexosaminidase A deficiency → intracellular accumulation of GM2 ganglioside → progressive
neurodegeneration

Solution for Question 15:


Correct Option B - 25%:
• Achondroplasia is an autosomal dominant disorder in which the disease is caused by only one mutant
allele.
• As a result, “AA” and “Aa” will be affected, while “aa” will be unaffected. [A denotes a mutant allele,
whereas a denotes a normal allele].
• As shown in the diagram, 3 out of 4 children, or 75 per cent, will be afflicted, while 1 out of 4 children,
or 25 per cent, will be unaffected.
• However, babies with the AA genotype usually do not survive clinically.

Page 23

204
Incorrect Options:
Option A - 0%:
• Achondroplasia is an autosomal dominant disorder.
• Therefore, even if a single parent were affected, there would still be a chance of having an affected
child.
Option C - 75%:
• Since both parents have the disease, there is a 75% chance of having an affected child (3 out of 4
affected).
• The chance of having an unaffected child is 25%, not 75%.
Option D - 100%:
• 100% chance of having an unaffected child would be true in a case where there is a family history of
achondroplasia, but none of the parents is affected.
• Since it’s an autosomal dominant disorder, an unaffected person will have both normal alleles (aa),
and none of the children of unaffected parents will have any chance of having the disease.

Solution for Question 16:


Correct Option A - X-linked recessive:
• Based on the symptoms of intellectual disability, kinky or tangled hair, pudgy rosy cheeks with sagging
facial skin since birth, along with a family of kinky hair disease, the patient is most probably suffering
from Menkes syndrome.
• In any situation when the previous generation of the affected individual is not affected, the condition is
recessive.
• It occurs due to mutations in genes encoding for copper-transporting protein ATP7A, leading to
copper deficiency.

Page 24

205
• In X-linked recessive disease, the female won't have the disease, but she will transmit it to her son.
• If there is selective male-to-female transmission, it is most likely an X-linked condition.
• Male-to-male transmission is seen in the autosomal condition.
• Kinky hair disease /Menkes syndrome is: Characteristic findings include: Pudgy, rosy cheeks, sagging
facial skin Feeding difficulties and irritability Lack of muscle tone and floppiness Hypothermia Brittle,
kinky, or tangled hair Intellectual disability and developmental delay Seizures Manifested only in males.
Females are not affected, so they must be carriers.
• Characteristic findings include: Pudgy, rosy cheeks, sagging facial skin Feeding difficulties and
irritability Lack of muscle tone and floppiness Hypothermia Brittle, kinky, or tangled hair Intellectual
disability and developmental delay Seizures
• Pudgy, rosy cheeks, sagging facial skin
• Feeding difficulties and irritability
• Lack of muscle tone and floppiness
• Hypothermia
• Brittle, kinky, or tangled hair
• Intellectual disability and developmental delay
• Seizures
• Manifested only in males.
• Females are not affected, so they must be carriers.
• Characteristic findings include: Pudgy, rosy cheeks, sagging facial skin Feeding difficulties and
irritability Lack of muscle tone and floppiness Hypothermia Brittle, kinky, or tangled hair Intellectual
disability and developmental delay Seizures
• Pudgy, rosy cheeks, sagging facial skin
• Feeding difficulties and irritability
• Lack of muscle tone and floppiness
• Hypothermia
• Brittle, kinky, or tangled hair
• Intellectual disability and developmental delay
• Seizures
• Manifested only in males.
• Females are not affected, so they must be carriers.
• Pudgy, rosy cheeks, sagging facial skin
• Feeding difficulties and irritability
• Lack of muscle tone and floppiness
• Hypothermia
• Brittle, kinky, or tangled hair
• Intellectual disability and developmental delay
• Seizures

Page 25

206
Incorrect Options:
Option B - Autosomal dominant:
• One copy of a mutated gene from one of the parents can cause the genetic condition.
• An example is a Familial hypercholesterolemia.
Option C - Autosomal recessive:
• It occurs when the child receives one mutated gene copy from each parent.
• Parents usually don't have the disease, but are carriers of it.
• An example is G6pD.
Option D - X linked dominant:
• The X-linked dominant pattern is rare.
• Both male and female children of an affected female can have the trait.
• An example is a resistant rickets.

Solution for Question 17:


Correct Option A - 0 and 100%:
• The case, as mentioned above, has clinical features consistent with sickle cell anemia. It is an
autosomal recessive trait, and % of sickle cell disease will be zero, and that of sickle cell trait will be
100%.
• Sickle-cell anemia is an autosomal recessive condition that results in the synthesis of an abnormal
hemoglobin chain termed HbS.
• It is more frequent in people of African descent as the heterozygous condition offers some protection
against malaria (falciparum).

Page 26

207
• HbS homozygous state (SS), where S stands for HbS gene coding. HbS heterozygous state (SA),
where A denotes the absence of HbS (normal).
• A normal person does not have the HbS gene (AA). If the mother has sickle cell disease 'SS' and the
father is normal 'AA', then all the offspring will be 'SA'. Therefore, the percentage of sickle cell disease
(SS) will be zero, whereas the percentage of sickle cell trait (SA) will be 100%.
• Symptoms in homozygotes don't tend to develop until 4-6 months when the abnormal HbSS
molecules take over from fetal haemoglobin.
Incorrect Options:
Option B
- 25% and 50%: If both the mother and father are carriers of the disease (A, S), then the chance of a
child being affected is 25% and the chance of a child being a carrier is 50%.
Option C - 50% and 50%: If the mother has sickle cell disease (S, S) and the father is a
carrier (A, S), the children will have 50% chance of being carriers and a 50% chance of being affected.
Option D - 100% and 0%: In a situation when both the mother and father have sickle cell anaemia, the
n all of their children will have the disease, and none will be carriers.

Solution for Question 18:


Correct Option A - X-linked recessive:
• The case, as mentioned above, has clinical features and laboratory findings consistent with
Hemophilia A.
• Since his mother and elder brother are affected, and both sisters are unaffected, the inheritance mode
is X-linked recessive.
• For the mother to have the disease, she must carry both affected alleles (Xa, Xa).
• The unaffected father's alleles will therefore be X and Y.
• The mother will transfer the affected allele to all of her sons ( Xa Y) and all daughters will be carriers (
Xa X ).
Incorrect Options:
Option B - Autosomal dominant:
• In autosomal dominant diseases, both homozygotes and heterozygotes manifest disease (there is no
carrier state) and both males and females are affected.
• Only affected individuals can pass on the disease to 50% of children. It normally appears in every
generation and the risk remains the same for each successive pregnancy.
Option C - X-linked dominant disease:
• Hemophilia A is not an X-linked dominant disease;
• Fragile X syndrome and Alport syndrome are examples of X-linked dominant diseases.
• X-linked dominant disease is transferred to 50% of daughters and 50% of sons from an affected
mother.
Option D - Autosomal recessive:

Page 27

208
• These diseases are mostly due to enzyme deficiencies and are mostly seen in only 1 generation,
unlike as seen here.
• Hemophilia A is not an autosomal recessive disease.
• An affected mother and unaffected father will have affected children if the father is a carrier for the
defective allele.
• Autosomal recessive conditions typically require both parents to be carriers (heterozygous) of the
mutated gene, and affected individuals can occur in both genders with a 25% chance of being affected
in each pregnancy when both parents are carriers. Hemophilia A does not follow an autosomal
recessive inheritance pattern.

Solution for Question 19:


Correct Option A - 50% of boys of carrier mothers are affected:
• Based on the history of difficulty walking, standing from a sitting position, going upstairs, and positive
Gower's sign.

• This child is diagnosed with a case of Duchenne muscular dystrophy, which is an X-linked recessive
trait.
• In the X-linked recessive trait, the X-chromosome carries the mutant gene and is carried by females to
affect only the males.
• The affected males are homozygous because they contain only one X-chromosome, the mutant gene.
At the same time, carrier females are heterozygous and perfectly healthy as the healthy X-chromosome
balances the defect.
• In this particular situation (where the mother is a carrier), there are 1 in 2 chances (50%) of
transmitting the mutant gene to offspring. So, 50% of the daughters have the chance to become a
carrier, and 50% of the males have the chance to be affected.
• The affected male usually dies at the age of 17-20 due to respiratory failure, heart disease, obstructive
airway disease, or Pneumonia.

Page 28

209
Incorrect Options:
Option B - 50% of girls with diseased fathers are carriers:
• In the context of an X-linked recessive disorder.
• If the father is affected, all daughters (100%) inherit the condition as carriers. When the mother is a
carrier, 50% of her daughters also become carriers
• However, these girls will not have the disease.
Option C - Father transmits the disease to the son:
• Fathers cannot transmit the disease to their sons as they transmit just the "Y" chromosome to their
sons, whereas the disease is "X" linked.
• Only a mother can transmit the disease to the son if she is the carrier having an affected X
chromosome.
• Therefore this statement is incorrect
Option D - Mother transmits the disease to the daughter:
• In X-linked recessive disorders, it's incorrect to say mothers directly pass the disease to daughters.
• If the mother has the disease, 100% boys will be affected since they get either of affected
chromosome.

Solution for Question 20:


Correct Option C - X-linked recessive inheritance:
• Red-green color blindness has an X-linked recessive pattern of inheritance. In the given case his
mother should be a carrier.
Incorrect Options:

Page 29

210
Option A - Autosomal dominant inheritance: Red-green color blindness does not follow an autosomal d
ominant pattern of inheritance

Option B – Autosomal recessive inheritance: Red-green color blindness does not follow an Autosomal r
ecessive pattern of inheritance
Option D - Mitochondrial inheritance: Red-Green color blindness does not follow the Mitochondria patte
rn of inheritance

Page 30

211
Non Mendelian and Chromosomal Disorders
1. What does "q" represent in the translocation t(9;22)(q34.1;q11.2)?
A. Short arm
B. Long arm of chromosomes
C. Centromere
D. Telomere
----------------------------------------
2. What is the mode of inheritance of the condition seen in a 12-year-old boy presenting with left knee
swelling post-trauma, prolonged activated partial thromboplastin time (aPTT), a history of excessive
bleeding during circumcision, and a positive family history of a bleeding disorder in the grandfather?
(or)
What is the mode of inheritance of Hemophilia A?
A. X linked dominant
B. X linked recessive
C. Autosomal dominant
D. Autosomal recessive
----------------------------------------
3. What is the mode of inheritance of the condition in a 12-year-old boy with bony aches, lower
extremity bowing, and hypophosphatemia, given the family history of similar complaints in his mother?

A. Autosomal recessive type


B. Autosomal dominant type
C. X-Linked dominant type
D. X-linked recessive type
----------------------------------------
4. What's the likely diagnosis for a 2-month-old child presenting with pale skin, feeding difficulties, a
history of blood transfusion at birth, hemoglobin level of 3.2gm/dl, and bone marrow study showing
reduced red cell precursors, along with a genetic mutation in the ribosomal protein S19 (RPS19) gene?
(or)

212
What diagnosis is probable in a 2-month-old with severe anemia, reduced red cell precursors, and a
detected RPS19 gene mutation?
A. Schwachman diamond syndrome
B. Diamond blackfan anaemia
C. Dyskeratosis congenita
D. Congenital amegakaryocytic thrombocytopenia
----------------------------------------
5. What is the most likely diagnosis for the newborn boy brought to the neonatology department with a
drooping eyelid, a deep groove between the nose and the mouth, and a depressed nose tip?

A. Noonan syndrome
B. Downs syndrome
C. Edwards syndrome
D. Patau syndrome
----------------------------------------
6. What is the most likely diagnosis in a 2-year-old with delayed development, large face, everted ears,
and macro-orchidism, showing CGG repeat sequence?
A. Fragile X syndrome
B. Huntington's chorea
C. Dentatorubropallidolusyian atrophy
D. Machado Joseph disease
----------------------------------------
7. Which gene mutation is primarily responsible for Noonan syndrome?
A. RASA2
B. BCR
C. c-myc
D. EWSR1
----------------------------------------
8. What is a true characteristic of Fragile X syndrome diagnosed in a 13-year-old boy presenting with
testicular enlargement, developmental delays, autism, and a long, narrow face with distinct facial

Page 2

213
features?
(or)
Which of the following is true about Fragile X syndrome?
A. Triple nucleotide CAG Sequence mutation
B. 10% female carriers have mental disability
C. Affected males have IQ of 20-40
D. Micro-orchidism in affected males
----------------------------------------
9. Which of the following is the most common genetic cause of infertility is associated with an increased
risk of leukemia?
A. Angelman Syndrome
B. Klinefelter syndrome
C. Sturge weber syndrome
D. Turner Syndrome
----------------------------------------
10. What is trisomy 13 called?
A. Edward’s syndrome
B. Patau’s syndrome
C. Down’s syndrome
D. Klinefelter’s syndrome
----------------------------------------
11. What is the most likely diagnosis for a 6-month-old baby with an elongated skull, hypertelorism,
malformed low-set ears, micrognathia, microphallus, flexion deformity of fingers, rocker-bottom feet, big
short toes, and trisomy of chromosome 18 on karyotyping?
(or)
What is the most likely diagnosis when the third chromosome is found on chromosome 18?
A. Patau syndrome
B. Edward syndrome
C. Down syndrome
D. Klinefelter syndrome
----------------------------------------
12. Which trinucleotide sequence expansion underlies the neurological condition characterized by
choreiform movements and caudate nucleus atrophy in a 58-year-old male with a family history of
similar symptoms?
A. CTG
B. CUG
C. CAG

Page 3

214
D. CGG
----------------------------------------
13. Which of the following statements best describes the cause of trisomy 21?
A. 95% of individuals with Down syndrome have a meiotic non-disjunction
B. Mitotic non-disjunction is very common
C. There is no Robertsonian translocation between chromosomes 13 and 19
D. There is no Robertsonian translocation between chromosomes 14 and 21
----------------------------------------
14. What causes obesity in a patient with maternal disomy of chromosome 15?
A. LH
B. FSH
C. Growth hormone
D. Ghrelin
----------------------------------------
15. Which of the following is a manifestation of 22q11 mutation syndrome?
A. Brushfield spots
B. Polydactyly
C. Thymic hypoplasia
D. Cystic hygroma
----------------------------------------
16. What is the most common hematological malignancy in children with trisomy 21?

A. Marginal zone lymphoma


B. Burkitt lymphoma
C. Chronic lymphocytic leukaemia
D. Acute myelogenous leukaemia
----------------------------------------
17. Which subtype of a chromosome is implicated in a disorder with characteristics including open
fontanelle, microcephaly, flat occiput, slanting palpebral fissures, hypertelorism, low set and malformed

Page 4

215
ears, protruding tongue, open mouth, and hypotonia?
(or)
What subtype of chromosome is involved in Down syndrome?
A. Acrocentric chromosome
B. Metacentric chromosome
C. Submetacentric chromosome
D. Telocentric chromosome
----------------------------------------
18. Which of the following is true about NARP syndrome?
A. Mutation of MT-ATP6 gene are seen
B. It is a cytoplasmic disease
C. Neuropathy, ataxia, and optic atrophy are seen
D. It is a Lipid storage disease
----------------------------------------
19. Which of the following is a feature of Down syndrome?
A. Upward-slanting palpebral fishers
B. Upward and outward lens dislocation
C. Cystic Hygroma
D. PostaxialPolydactyly
----------------------------------------
20. What vascular lesion is commonly observed in individuals with the absence of the X chromosome
(45, X)?
A. Nevus flammeus
B. Cavernous lymphangioma
C. Pyogenic granuloma
D. Capillary hemangioma
----------------------------------------
21. A 20-year-old female patient is brought by her mother with complaints of fever and cough for the
last two days. She has been mentally disabled since birth. She is morbidly obese and has narrow
shoulders. Upon further tests, maternal disomy of chromosome 15 is seen. What is the most likely
diagnosis?
(or)
Which condition is characterised by maternal disomy of chromosome 15 inheritance?
A. Prader-Willi syndrome
B. Klinefelter syndrome
C. Angelman syndrome
D. Turner syndrome

Page 5

216
----------------------------------------
22. What is the mode of inheritance for Leber hereditary optic neuropathy?
A. Autosomal dominant
B. Autosomal recessive
C. X-linked recessive
D. Mitochondrial inheritance
----------------------------------------
23. Which chromosomal abnormality describes Down syndrome?
A. Trisomy 21
B. Trisomy 18
C. Trisomy 13
D. 45, X
----------------------------------------
24. What could be causing a 6-year-old to have obesity, developmental delay, hypotonic musculature,
small hands, and feet, while having a height at the 50th percentile and weight at the 98th percentile?
A. Genomic imprinting
B. Mosaicism
C. Anticipation
D. Non-penetrance
----------------------------------------
25. What is the diagnosis for the newborn girl showing the given lip defect, polydactyly, and
microphthalmia, born to a 27-year-old G2 P1 at 39 weeks gestation, with APGAR scores of 7 and 8 at 1
and 5 minutes, respectively?

A. Trisomy 13
B. Trisomy 18
C. Trisomy 21
D. None of the above
----------------------------------------

Page 6

217
26. A 23-year-old male patient with Huntington’s disease is brought to the OPD for evaluation. His
father also had the disease but was symptomatic only after 40 years of age. The patient's mother asks
you. Which of the following best describes the reason behind her son developing a more severe form of
this disease early?
A. Anticipation
B. Pleiotropy
C. Imprinting
D. Mosaicism
----------------------------------------
27. Which of the following is correct about Klinefelter’s syndrome?
A. Intellectual disability is common
B. Increased levels of FSH.
C. Karyotype 45, XO
D. Results due to non-disjunction during mitotic division
----------------------------------------
28. When a chromosome splits along an axis, the axis is perpendicular to the conventional division
axis. What is formed when a chromosome is applied along an axis?
A. Ring chromosome
B. Isochromosome
C. Acrocentric chromosome
D. Subtelocentric chromosome
----------------------------------------
29. What mode of inheritance is exemplified by a genetic condition that passes exclusively through
maternal lineage in which the affected females transmit the condition to all their children, while affected
males do not pass it to their descendants?
A. Autosomal dominant
B. Autosomal recessive
C. Mitochondrial
D. X-linked dominant
----------------------------------------
30. Which of the following is a life-threatening disorder associated with Down syndrome?
A. Berry aneurysm of the circle of Willis
B. Creutzfeldt-Jakob disease
C. Lymphoblastic leukemia
D. Medullary carcinoma of the thyroid
----------------------------------------
31. Which of the following is true about Chromothrypsis?

Page 7

218
A. It is associated with chromosome rearrangements termed chromosomal catastrophes.
B. Atleast 50% of malignancies demonstrate this pattern
C. Associated with the inactivation of oncogenes and activation of tumour suppressors
D. It is associated with a decreased frequency of osteosarcomas and gliomas
----------------------------------------
32. What term best characterizes the situation where two normal parents give birth to two children
affected with Osteogenesis Imperfecta?
A. Incomplete penetrance
B. Uniparental disomy
C. Genomic imprinting
D. Gonadal mosaicism
----------------------------------------

Correct Answers
Question Correct Answer

Question 1 2
Question 2 2
Question 3 3
Question 4 2
Question 5 1
Question 6 1
Question 7 1
Question 8 3
Question 9 2
Question 10 2
Question 11 2
Question 12 3
Question 13 1
Question 14 4
Question 15 3
Question 16 4
Question 17 1
Question 18 1
Question 19 1
Question 20 2

Page 8

219
Question 21 1
Question 22 4
Question 23 1
Question 24 1
Question 25 1
Question 26 1
Question 27 2
Question 28 2
Question 29 3
Question 30 3
Question 31 1
Question 32 4

Solution for Question 1:


Correct Option B - Long arm of chromosomes:
• The chromosomal defect in the case of the Philadelphia chromosome is a reciprocal translocation
involving parts of chromosomes 9 and 22. This results in a fusion gene by juxtaposing the ABL1 gene
on chromosome 9 (region q34) to a part of the BCR (breakpoint cluster region) gene on chromosome
22 (region q11).
• In a sub-metacentric chromosome, the centromere divides each chromosome into two regions: the
larger one, the q region, and the smaller one, the p region. The sister chromatids will be spread to each
daughter cell at the end of the cell division. The p and q system is meaningless in a chromosome where
the centromere is at the centre and arms are of equal length.
• The short arm of a chromosome is represented by ‘p’ (for petit) and the long arm is referred to as ‘q’.
• In a banded karyotype, each arm of the chromosome is divided into two or more regions by prominent
bands

Page 9

220
• Regions are numbered in a direction away from the centromere.
• Each chromosomal region is further subdivided into bands, and sub-bands are numbered numerically.
• Translocation t (9;22) (q34.1;q11.2) is read as breaks happening in the region (3), the band (4),
sub-band (1) of the long arm (q) of chromosome 9, and region (1), band (1), sub-band (2) of the long
arm (q) of chromosome 22
Incorrect Options:
Option A - Short arm:
• Acronym 'q' does not represent the short arm of the chromosome.
• The short arm of the chromosome is represented by the letter 'p'.
• p is for petit, which is French for small.
Option C – Centromere:
• Acronym 'q' does not represent the short arm of the chromosome.
• There is no acronym to represent the centromere in translocation, as it does not occur on the
centromere.
• Depending on the position of the centromere, a chromosome is divided into two arms.
Option D – Telomere:
• Acronym 'q' does not represent the short arm of the chromosome.
• There is no acronym to represent telomere in translocation as it does not occur on telomere.
• A telomere represents a region of repetitive DNA sequences at the end of a chromosome.

Solution for Question 2:


Correct Option B - X-linked recessive:

Page 10

221
• This boy is suffering from the genetic disorder of Hemophilia A, which is an X-linked recessive
disorder.
• X-linked recessive inheritance is characterised by a mutation in a gene on the X chromosome, which
produces a phenotype always in males (who are essentially homozygous for the gene mutation
because they have one X and one Y chromosome).
• The inheritance pattern of X-linked recessive traits is unique.
• Affected fathers can’t pass X-linked recessive traits to their sons as fathers give them Y
chromosomes. This implies that males affected by an X-linked recessive disorder inherit the
responsible X chromosome from their mothers.
• X-linked recessive traits are more commonly expressed in males than females. This happens
because males possess only a single X chromosome and need only one mutated X chromosome to be
affected. Women have two X chromosomes and, therefore, must receive two of the mutated recessive
X chromosomes, one from each parent.
• X-linked recessive traits tend to skip generations, the affected father will not have an affected son.
• All daughters of an affected father will receive his mutated X chromosome and then be either affected
themselves or carriers, depending on the mother. Sons of carrier mothers will have a 50% chance of
being affected, and sons of affected mothers will be surely affected 100%. Therefore X-linked recessive
traits are frequently seen in males compared to females.
Incorrect Options:
Option A - X linked dominant:
• Haemophilia is not an X-linked dominant disorder.
• The X-linked dominant disorder is a mode of inheritance in which an X chromosome carries the
dominant gene.
• This means that the gene causing the disorder is on the X chromosome. Only one copy of the allele is
sufficient to express the disease if inherited from a diseased parent.
Option C - Autosomal dominant:
• Haemophilia is not an Autosomal dominant disorder.
• Affected parents have a 50% chance of transferring the mutated gene to their child.
• The mutated genes are located on nonsex chromosomes.
Option D - Autosomal recessive:
• Haemophilia is not an Autosomal recessive disorder.
• In an autosomal recessive type inheritance pattern, two copies of the gene must be mutated for an
individual to express the disease.
• An affected individual usually has unaffected parents, each of whom carries a single copy of the
mutated gene (carriers) and normally does not exhibit disease symptoms.

Solution for Question 3:


Correct Option C - X-Linked dominant type:

Page 11

222
• X-linked hypophosphatemic rickets is an X-linked dominant disorder resulting from renal phosphate
wasting, causing growth retardation and severe skeletal abnormalities. Defective phosphate
reabsorption is the primary mechanism of this disease.
• X-linked dominant inheritance, is a mode of inheritance in which an X chromosome carries the
dominant gene. This means that the gene causing the disorder is on the X chromosome. Only one copy
of the allele is sufficient to express the disease if inherited from a diseased parent.
• The affected father, carrying the x-linked dominant disorder, will pass on the disease to all the
daughters but not to any son.
• If the mother is affected, then sons will have a chance of being affected depending on whether a
recessive or dominant X chromosome is passed on.
• If the son is affected, the mother will always be affected.
• In the first generation, females are affected by the disease.
• In the second generation, the same disease affects the son
• In the third generation, all the daughters are affected, not the sons.
• The given pedigree chart shows that all daughters of affected males have the disease, while none of
their sons has it.
• This is suggestive of X-linked dominant inheritance.
In the third generation, all the daughters are affected, not the sons.
Incorrect Options:
Option A - Autosomal recessive type:
• The inheritance pattern in the given pedigree does not account for an autosomal recessive trait.
• In an autosomal recessive type inheritance pattern, two copies of the gene must be mutated for an
individual to express the disease.
• An affected individual usually has unaffected parents, each of whom carries a single copy of the
mutated gene (carriers) and normally does not exhibit disease symptoms.
• Two carriers have a 25% risk of transferring disease to their offspring.
Option B - Autosomal dominant type:
• The inheritance pattern in the given pedigree does not account for an autosomal recessive trait.
• Affected parents have a 50% chance of transferring the mutated gene to their child.
• The mutated genes are located on nonsex chromosomes.
Option D - X-linked recessive type:
• X-linked recessive conditions also result from mutations in genes on the X chromosome.
• Males are much more frequently affected as compared to females.
• The sons of an affected father with an X-linked recessive disorder will not be affected (since they
receive their father's Y chromosome), but his daughters will be carriers of one copy of the mutated
gene.
• A woman carrying an X-linked recessive disorder (XRXr) has a 50% chance of having daughters who
are carriers of one copy of the mutated gene and a 50% chance of having sons who are affected.

Page 12

223
Solution for Question 4:
Correct Option B - Diamond blackfan anaemia:
• Diamond blackfan anaemia is an autosomal dominant condition characterised by congenital
abnormalities with severe macrocytic anaemia and reticulocytopenia.
• There is selective depletion of erythroid precursors in the bone marrow.
• RPS19 (ribosomal protein S19) gene mutation is seen in Diamond Blackfan Anemia
• Symptoms include rapid heartbeat, pale skin, presence of triphalangeal, sleepiness, irritability, poor
appetite, and weakness.
• Diamond-Blackfan anemia can be treated by an infusion of blood-forming stem cells from a healthy
donor, called a stem cell transplant, bone marrow transplant, or hematopoietic cell transplant.
Incorrect Options:
Option A - Schwachman diamond syndrome:
• It is an autosomal recessive disease due to a Biallelic mutation in the SBDS gene characterised by
bone marrow failure and exocrine pancreatic insufficiency.
• There is an increased risk of myelodysplasia and leukaemia.
Option C - Dyskeratosis congenita:
• It is an inherited bone marrow failure syndrome with a triad of skin pigmentation, nail dystrophy, and
mucosal leukoplakia.
• It is an X-linked and autosomal condition characterised by bone marrow aplasia, pulmonary fibrosis,
liver disease, and neurologic and eye abnormalities.
• There is an increased predisposition to cancer.
Option D - Congenital amegakaryocytic thrombocytopenia:
• It is an autosomal recessive condition with a mutation in thrombopoietin (TPO) receptor c-mp,
• Characterised by aplastic anaemia by 5 years of age.

Solution for Question 5:


Correct Option A - Noonan syndrome:
• Based on abnormalities in his facial features, such as a drooping eyelid, mouth has a deep groove
between the nose and the mouth, along with a depressed nose tip; the neonate is suffering from
Noonan syndrome.
• Noonan syndrome is an autosomal dominant condition associated with a normal karyotype.
• It is thought to be caused by a defect in a gene on chromosome 12.
• Features similar to Turner's syndrome (webbed neck, widely-spaced nipples, short stature, pectus
carinatum and excavatum), several characteristic clinical signs may also be seen: Unusual facies (i.e.,
hypertelorism, down slanting eyes, webbed neck) Cardiac: pulmonary valve stenosis Ptosis
Triangular-shaped face Low-set ears Coagulation problems: factor XI deficiency
• Unusual facies (i.e., hypertelorism, down slanting eyes, webbed neck)

Page 13

224
• Cardiac: pulmonary valve stenosis
• Ptosis
• Triangular-shaped face
• Low-set ears
• Coagulation problems: factor XI deficiency
• Bleeding diathesis in 50% skeletal, genitourinary, neurologic, lymphatic, skin and eye.
• Findings may be present in varying degrees.
• Unusual facies (i.e., hypertelorism, down slanting eyes, webbed neck)
• Cardiac: pulmonary valve stenosis
• Ptosis
• Triangular-shaped face
• Low-set ears
• Coagulation problems: factor XI deficiency
Incorrect Options:
Option B - Downs syndrome:
• Down syndrome is the most common inheritable cause of mental retardation.
• Symptoms of Down’s syndrome (Trisomy 21): Mental retardation Epicanthic folds Flat facial profile
Umbilical hernia Simian crease The gap between the first and second toe is increased Hypotonia
Congenital heart defects
• Mental retardation
• Epicanthic folds
• Flat facial profile
• Umbilical hernia
• Simian crease
• The gap between the first and second toe is increased
• Hypotonia
• Congenital heart defects
• Mental retardation
• Epicanthic folds
• Flat facial profile
• Umbilical hernia
• Simian crease
• The gap between the first and second toe is increased
• Hypotonia
• Congenital heart defects
Option C - Edwards’s syndrome:

Page 14

225
• Trisomy 18 is Edwards’s syndrome.
• Its symptoms include: Prominent occiput Low-set ears Mental retardation Congenital heart defects
Rocker-bottom feet
• Prominent occiput
• Low-set ears
• Mental retardation
• Congenital heart defects
• Rocker-bottom feet
• Prominent occiput
• Low-set ears
• Mental retardation
• Congenital heart defects
• Rocker-bottom feet
Option D - Patau syndrome:
• Trisomy 13 (Patau syndrome) symptoms: Microphthalmia Microcephaly and mental retardation
Polydactyly Cleft lip and palate Renal defects
• Microphthalmia
• Microcephaly and mental retardation
• Polydactyly
• Cleft lip and palate
• Renal defects
• Microphthalmia
• Microcephaly and mental retardation
• Polydactyly
• Cleft lip and palate
• Renal defects

Solution for Question 6:


Correct Option A - Fragile X syndrome:
• The case mentioned above has clinical features of mental retardation and has examination findings
such as a large face, mandible, large everted ears, and macroorchidism, suggestive of Fragile X
syndrome.
• Fragile X syndrome is a trinucleotide repeat disorder.
• Features in males include: Difficulty in learning May have large, low, set ears, a long thin face, and
high-arched palate. Macroorchidism Hypotonia Autism Mitral valve prolapse
• Difficulty in learning

Page 15

226
• May have large, low, set ears, a long thin face, and high-arched palate.
• Macroorchidism
• Hypotonia
• Autism
• Mitral valve prolapse
• Symptoms in females with one fragile chromosome and one normal X chromosome range from
normal to mild.
• Diagnosis can be made antenatally by chorionic villus sampling or amniocentesis and by analysis of
the number of CGG repeats using restriction endonuclease digestion and Southern blot analysis.
• Difficulty in learning
• May have large, low, set ears, a long thin face, and high-arched palate.
• Macroorchidism
• Hypotonia
• Autism
• Mitral valve prolapse
Incorrect Options:
Option B - Huntington's Disease:
• Huntington's disease (HD) is an autosomal dominant neurodegenerative disorder.
• It manifests between the ages of 30 and 50 years.
• Huntington's disease is caused by a mutation in the HTT gene located on chromosome 4, with the
mutation involving an expansion of the CAG trinucleotide repeat within this gene.
Option C - Dentatorubropallidolusyian atrophy:
• Dentatorubral-pallidoluysian atrophy, commonly known as DRPLA.
• It is described as a progressive neurological disorder causing involuntary movements and decreased
mental abilities.
• The age of onset of DRPLA is usually 30 years.
• DRPLA is caused by an expansion of CAG repeats in the Atrophin-1(ATN1) gene on chromosome 12.
Option D - Machado joseph disease:
• Machado-Joseph disease (MJD) is also known as spinocerebellar ataxia 3/ SCA3.
• It’s the most common dominant ataxia.
• It usually appears between 30 and 40 years of age.

Solution for Question 7:


Correct Option A - RASA2:
• Noonan syndrome is a syndrome due to a defect in the RAS/MAPK pathway.

Page 16

227
• RASopathy describes a phenotypically similar group of genetic syndromes caused by germline
mutations resulting in pathogenic variants in components of the RAS or mitogen-activated protein
kinase (RAS/MAPK) pathway.
• This pathway is vital in regulating many cell functions, including cell maturation, growth, and death.
• Many RASopathy syndromes are marked by unusual facial features, skin abnormalities, heart defects,
and ocular and musculoskeletal abnormalities.
• Examples of RASopathies: Cardiofaciocuatneous syndrome, and Costello syndrome.
• Molecular diagnosis of Noonan syndrome is established with suggestive clinical findings and a
heterozygous pathogenic variant in one of The gene including PTPN11, SOS1, RAF1/CRAF, KRAS,
NRAS, SHOC2, CBL, RRAS, RIT1, RASA2, SOS2, MAP3K8, SPRY1, MYST4/KAT6B, LZTR1, A2ML1,
PPP1CB, MRAS, RALA, RRAS2, and ERK2/MAPK1.
• PTPN11, SOS1, RAF1/CRAF, KRAS, NRAS, SHOC2, CBL, RRAS, RIT1, RASA2, SOS2, MAP3K8,
SPRY1, MYST4/KAT6B, LZTR1, A2ML1, PPP1CB, MRAS, RALA, RRAS2, and ERK2/MAPK1.
• All contributory pathogenic variants are part of the well-studied and complex RAS/MAPK signal
transduction pathway, which is essential for development and oncogenesis.
• RASopathies are unique because these are pathway-based, mechanistic abnormalities in medical
genetic syndromes as opposed to the isolated one gene–one syndrome.
• Various other examples of RASopathies include: Costello syndrome Cardia-facio cutaneous
syndrome. Legius syndrome
• Costello syndrome
• Cardia-facio cutaneous syndrome.
• Legius syndrome
Examples of RASopathies: Cardiofaciocuatneous syndrome, and Costello syndrome.
• PTPN11, SOS1, RAF1/CRAF, KRAS, NRAS, SHOC2, CBL, RRAS, RIT1, RASA2, SOS2, MAP3K8,
SPRY1, MYST4/KAT6B, LZTR1, A2ML1, PPP1CB, MRAS, RALA, RRAS2, and ERK2/MAPK1.
• Costello syndrome
• Cardia-facio cutaneous syndrome.
• Legius syndrome
Option B – BCR:
• Mutation in the BCR gene is not responsible for causing Noonan syndrome.
• BCR mutation is responsible for the Philadelphia chromosome in chronic myelogenous leukaemia.
• A BCR-ABL translocation results in a protein that has intrinsic tyrosine kinase properties and renders
cells uncontrolled growth and cell division.
Option C - C-myc:
• Mutation in the C-myc gene is not responsible for causing Noonan syndrome.
• A translocation involving c-myc is crucial in developing most cases of Burkitt lymphoma.
• C-myc is found on chromosome 8 and is thought to regulate the expression of 15% of all genes by
binding on enhancer box sequences.
Option D - EWSR1:
• Mutation in the EWSR1 gene is not responsible for causing Noonan syndrome.

Page 17

228
• This gene encodes RNA-binding protein EWS protein.
• A gain-of-function mutation of this gene results in the production of an oncogenic protein responsible
for causing Ewing sarcoma.

Solution for Question 8:


Correct Option C - Affected males have an IQ of 20-40:
• Fragile X syndrome is the most common cause of inherited intellectual disability and autism and is the
second most common cause of genetically associated mental deficiencies after Down's syndrome.
• It results indirectly secondary to the expansion of the cytosine-guanine-guanine (CGG) triplet repeat
within the FMR1 (Fragile X mental retardation one) gene located on the X chromosome.
• This trinucleotide expansion silences the expression of FMR1, resulting in diminished expression of
the FMRP (fragile X mental retardation protein), which is necessary for developing neuronal synapsis.
• The lack of fragile X mental retardation protein is the direct cause of Fragile X Syndrome.
• Affected males are mentally disabled, and their IQ range from 20 to 60.
• The characteristic phenotype includes a large mandible, enlarged testicles (macro-orchidism), and
large everted ears.
• The most characteristic feature is macro-orchidism which occurs in at least 90% of affected
post-pubertal males.
• About 30-50% of carrier females have intellectual disabilities.
• Diagnosis is confirmed by molecular genetic testing, which measures the number of CGG repeats in
the FMR1 gene.
Incorrect Options:
Option A - Triple nucleotide CAG Sequence mutation:
• Fragile x syndrome does not involve CAG sequence mutation.
• Trinucleotide repeat mutation in the CAG sequence is characteristic of Huntington's disease.
Option B - 10% of female carriers have a mental disability:
• Fragile x syndrome causes mental disability in more than 10% of carrier individuals.
• At least 30-50% of carrier females suffer from mental disabilities.
• This occurs due to defective neuronal synapsis development and functioning.
Option D - Micro-orchidism in affected males:
• Fragile x syndrome does not cause micro-orchids.
• It results in macro-orchidism characterised by testicular enlargement.
• The usual testicular size is >4 cm in the case of fragile x syndrome.

Solution for Question 9:

Page 18

229
Correct Option B - Klinefelter syndrome:
Klinefelter's syndrome (47XXY):
• Male hypogonadism occurs when there are two or more X chromosomes and one or more
Y-chromosomes.
• It can rarely be diagnosed before puberty.
• In males with this syndrome, testicular tubules are atrophied and replaced by pink, hyaline,
collagenous ghosts.
• The mean IQ is somewhat lower than normal, but intellectual disabilityis uncommon.
• The plasma gonadotropin concentrations, particularly follicle-stimulating hormone (FSH), are
consistently elevated, whereas testosterone levels are variably reduced.
• There is an increased incidence of type 2 diabetes.
• It results from the non-disjunction of chromosomes during meiosis.
• In 25% of cases, patients with the syndrome also have Autism.
• Association of Klinefelter syndrome: Breast cancer Extragonadal germ cell tumours Autoimmune
diseases (SLE) Acute leukaemia.
• Breast cancer
• Extragonadal germ cell tumours
• Autoimmune diseases (SLE)
• Acute leukaemia.
• It is one of the most common genetic causes of infertility
• Breast cancer
• Extragonadal germ cell tumours
• Autoimmune diseases (SLE)
• Acute leukaemia.
Incorrect Options:
Option A - Angelman Syndrome:
• Angelmann is a type of UPD (uniParental Disomy) where the child receives both copies of a gene
from a single parent. Individuals with Angelman syndrome may inherit both copies of chromosome 15
from their father
• Deletion of a segment of the maternal chromosome 15, usually occurs on the long arm (q) of
chromosome 15.
• It is not the common genetic cause of infertility.
Option C - Sturge-Webber syndrome:
• Sturge-Weber syndrome (SWS) is a neurological condition presenting at birth.
• This syndrome causes blood vessels to grow, forming angiomas.
• A port-wine birthmark is formed on the face.
• It is not the common genetic cause of infertility, and there is no risk of leukaemia.
Option D - Turner Syndrome:

Page 19

230
• Turner syndrome is the most common cause of primary amenorrhea

Solution for Question 10:


Correct Option B - Patau’s syndrome:
• Trisomy 13 (Patau syndrome) symptoms: Microphthalmia Microcephaly and mental retardation
Polydactyly Cleft lip and palate Renal defects
• Microphthalmia
• Microcephaly and mental retardation
• Polydactyly
• Cleft lip and palate
• Renal defects
• The diagnosis can be confirmed prenatally by chorionic villus sampling (CVS) or amniocentesis.
• There is no specific treatment for Patau syndrome.
• Management focuses on the treatment of symptoms.
• Microphthalmia
• Microcephaly and mental retardation
• Polydactyly
• Cleft lip and palate
• Renal defects
Incorrect Options:
Option A - Trisomy 18 is Edward's syndrome:
• Its features are: Micrognathia Low-set ears Rocker bottom feet Overlapping of fingers.

Page 20

231
• Micrognathia
• Low-set ears
• Rocker bottom feet
• Overlapping of fingers.
• Micrognathia
• Low-set ears
• Rocker bottom feet
• Overlapping of fingers.
Option C - Trisomy 21:
• Trisomy 21 is Down's syndrome.
• Its features are: Palpebral fissures Low-set ears Providing tongue Hypotonia Single palmar crease.
• Palpebral fissures
• Low-set ears
• Providing tongue
• Hypotonia
• Single palmar crease.
• Palpebral fissures
• Low-set ears
• Providing tongue
• Hypotonia
• Single palmar crease.
Option D - Klinefelter's syndrome:
• Extra X chromosome = Klinefelter syndrome (47 XXY)
• Klinefelter syndrome is a condition in which male hypogonadism occurs when there are two or more X
chromosomes and one or more Y-chromosomes.
• It is the most common genetic cause of infertility

Solution for Question 11:


Correct Option B - Edward syndrome:
• Based on the complaint of a clenched fist and clinical features characterized by the elongated skull,
hypertelorism, low-set malformed ears, micrognathia, microphallus, flexion deformity of fingers,
rocker-bottom, feet and big short toes, the most likely diagnosis is Edward syndrome.
• Edward syndrome is primarily caused by trisomy of chromosome 18, which can lead to multiple
systemic complications.
• Features: Incidence 1:6000 births Low birth weight Closed fists with index finger overlapping the 3rd
digit and the 5th overlapping the 4th digit Narrow hips with limited abduction Rocker-bottom feet

Page 21

232
Microcephaly Micrognathia Cardiac and renal malformations Mental retardation 95% of cases die in the
1st year of life
• Incidence 1:6000 births
• Low birth weight
• Closed fists with index finger overlapping the 3rd digit and the 5th overlapping the 4th digit
• Narrow hips with limited abduction
• Rocker-bottom feet
• Microcephaly
• Micrognathia
• Cardiac and renal malformations
• Mental retardation
• 95% of cases die in the 1st year of life
• Incidence 1:6000 births
• Low birth weight
• Closed fists with index finger overlapping the 3rd digit and the 5th overlapping the 4th digit
• Narrow hips with limited abduction
• Rocker-bottom feet
• Microcephaly
• Micrognathia
• Cardiac and renal malformations
• Mental retardation
• 95% of cases die in the 1st year of life

• Management: A goal-directed treatment plan based on careful risk vs benefit assessment for the
individual patient and developed with the coordination between health professionals and parents is

Page 22

233
recommended. The infections should be treated appropriately. Infections usually occur secondary to
otitis media, upper respiratory tract infections and urinary tract infections. Provide nasogastric and
gastrostomy supplementation for feeding problems. Scoliosis can be managed by the orthopaedic if
needed. Most of these patients need a diuretic and digoxin for congestive heart failure.
• A goal-directed treatment plan based on careful risk vs benefit assessment for the individual patient
and developed with the coordination between health professionals and parents is recommended.
• The infections should be treated appropriately. Infections usually occur secondary to otitis media,
upper respiratory tract infections and urinary tract infections.
• Provide nasogastric and gastrostomy supplementation for feeding problems.
• Scoliosis can be managed by the orthopaedic if needed.
• Most of these patients need a diuretic and digoxin for congestive heart failure.
• A goal-directed treatment plan based on careful risk vs benefit assessment for the individual patient
and developed with the coordination between health professionals and parents is recommended.
• The infections should be treated appropriately. Infections usually occur secondary to otitis media,
upper respiratory tract infections and urinary tract infections.
• Provide nasogastric and gastrostomy supplementation for feeding problems.
• Scoliosis can be managed by the orthopaedic if needed.
• Most of these patients need a diuretic and digoxin for congestive heart failure.
Incorrect Options:
Option A - Patau Syndrome:
• The most likely diagnosis of the clinical scenario mentioned above is Edward syndrome, not Patau
syndrome.
• In Patau syndrome, there is trisomy of chromosome 13, which leads to different abnormalities.
• Features: Polydactyly Cleft lip and palate Microphthalmia Microcephaly Mental retardation Cardiac
and renal defect
• Polydactyly
• Cleft lip and palate
• Microphthalmia
• Microcephaly
• Mental retardation
• Cardiac and renal defect
• Polydactyly
• Cleft lip and palate
• Microphthalmia
• Microcephaly
• Mental retardation
• Cardiac and renal defect
Option C - Down syndrome:

Page 23

234
• The most likely diagnosis of the clinical scenario mentioned above is Edward syndrome, not Down
syndrome.
• In Down syndrome, there is trisomy of chromosome 21, which leads to different abnormalities.
• Features: Most common autosomal trisomy Mental retardation Short stature Hypotonia Depressed
nasal bridge Congenital heart defect Increased risk of AML Alzheimer's disease
• Most common autosomal trisomy
• Mental retardation
• Short stature
• Hypotonia
• Depressed nasal bridge
• Congenital heart defect
• Increased risk of AML
• Alzheimer's disease
• Most common autosomal trisomy
• Mental retardation
• Short stature
• Hypotonia
• Depressed nasal bridge
• Congenital heart defect
• Increased risk of AML
• Alzheimer's disease
Option D - Klinefelter syndrome:
• The most likely diagnosis of the clinical scenario mentioned above is Edward syndrome, not
Klinefelter syndrome.
• The karyotype of Klinefelter syndrome is XXY.
• Features: Testicular atrophy Infertility Gynaecomastia Female distribution of hair Low testosterone
Elevated FSH and LH High-pitched voice
• Testicular atrophy
• Infertility
• Gynaecomastia
• Female distribution of hair
• Low testosterone
• Elevated FSH and LH
• High-pitched voice
• Testicular atrophy
• Infertility
• Gynaecomastia

Page 24

235
• Female distribution of hair
• Low testosterone
• Elevated FSH and LH
• High-pitched voice

Solution for Question 12:


Correct Option C – CAG:
• Huntington's disease, also known as Huntington's chorea, is a neurodegenerative disease mostly
inherited in an autosomal dominant fashion.
• The primary feature is the degeneration of neurons in the caudate nucleus, putamen, and cerebral
cortex.
• Huntington’s disease is classically inherited from an affected parent carrying a mutant Huntington
gene(HTT). The Huntington gene codes for a specific protein known as the Huntington protein.
Expansion of CAG (cytosine-adenine-guanine) repeats, known as a trinucleotide repeat expansion in
the gene coding for the Huntington protein, produces an abnormal mutant protein, which steadily
damages neurons.
• It commonly affects individuals aged 30-50 years. However, the longer the CAG repeats, the earlier
the onset of symptoms.
• In patients with a typical phenotype of Huntington's disease, genetic testing for pathological CAG
repeat expansion in HTT is negative in 1-12%. In such patients, many genetically heterogenous
diseases which are phenotypically identical to Huntington's disease, such as spinocerebellar ataxia 17,
should be considered
Incorrect Options:
Option A – CTG:
• Huntington's disease does not result from the expansion of the CTG trinucleotide sequence.
• CTG sequence expansion is characteristic of Dystrophia myotonica.
• It results in progressive muscle wasting and weakness.
Option B – CUG:
• Huntington's disease is not the result of the expansion of the CUG trinucleotide sequence.
• CUG sequence expansion also occurs in type 1 myotonic dystrophy.
• It results in progressive muscle wasting and weakness.
Option D – CGG:
• Huntington's disease is not the result of the expansion of the CGG trinucleotide sequence.
• CGG sequence expansion is characteristic of Fragile-X syndrome.
• This produces typical facial features, intellectual disabilities, macro-orchidism, and autism.

Page 25

236
Solution for Question 13:
Correct Option A - 95% of individuals with Down syndrome have a meiotic non-disjunction:
• Meiotic non-disjunction is the most common (95%) cause of Down syndrome, which is the correct
statement out of the four options. This meiotic non-disjunction is associated with advanced maternal
age.
• Robertsonian translocation (4%) between chromosomes 14 and 21 is also associated with down
syndrome.
• In all trisomy conditions, non-disjunction occurs in meiosis I, except in trisomy 18, where
non-disjunction occurs in meiosis II.
• The least common cause is mosaicism (mitotic non-disjunction) which is an unequal distribution of
chromosomes at the time of mitosis, leading to the presence of two different types of cells in one
individual.
Incorrect Options:
Option B - Mitotic non-disjunction is very common:
• This option is incorrect as mitotic non-disjunction is not very common in Down syndrome.
• The least common cause is mosaicism (mitotic non-disjunction) which is the unequal distribution of
chromosomes at the time of mitosis, leading to the presence of two different types of cells in one
individual.
Option C - There is no Robertsonian translocation between chromosomes 13 and 19:
• This option is incorrect
Option D - There is no Robertsonian translocation between chromosomes 14 and 21:
• This option is incorrect as Robertsonian translocation is associated with Down syndrome.
• Robertsonian translocation (4%) between chromosomes 14 and 21 is also associated with down
syndrome.

Solution for Question 14:


Correct Option D – Ghrelin:
• The case mentioned above has features suggesting Prader-Willi syndrome and obesity are due to
ghrelin.
• When two copies of a chromosome come from the same parent rather than from the mother and the
father, this condition is referred to as uniparental disomy: When a baby receives duplicate copies of a
certain piece of chromosome 15 from the father, Angelman syndrome may develop (rather than one
from the mother and one from the father). On the other side, PWS (Prader-Willi syndrome) can occur
when a newborn receives both copies of chromosome no.5 from the mother.
• When a baby receives duplicate copies of a certain piece of chromosome 15 from the father,
Angelman syndrome may develop (rather than one from the mother and one from the father).
• On the other side, PWS (Prader-Willi syndrome) can occur when a newborn receives both copies of
chromosome no.5 from the mother.

Page 26

237
• Prader-Willi syndrome (PWS) is a genetic condition occurring in 1 of 10,000–16,000 live births. It is
characterised by excessive appetite, progressive obesity, short stature, and mental retardation.
• The cause of obesity is thought to be increased levels of ghrelin. The majority of individuals have
hypogonadotropic hypogonadism and GH deficiency.
• When a baby receives duplicate copies of a certain piece of chromosome 15 from the father,
Angelman syndrome may develop (rather than one from the mother and one from the father).
• On the other side, PWS (Prader-Willi syndrome) can occur when a newborn receives both copies of
chromosome no.5 from the mother.
Incorrect Options:
Option A – LH:
• There is no role of LH in PWS-induced obesity.
• Obesity in PWS is due to ghrelin leading to hyperphagia.
Option B – FSH:
• FSH has no role in causing obesity.
• PWS causes obesity due to increased ghrelin, a growth hormone secretagogue leading to
hyperphagia.
Option C - Growth hormone:
• Obesity is due to increased ghrelin, a growth hormone secretagogue.
• Growth hormone has no role in inducing obesity in patients with PWS.

Solution for Question 15:


Correct Option C - Thymic hypoplasia:
• Chromosome 22q11.2 deletion syndrome refers to a group of illnesses caused by a modest deletion
of band q11.2 on chromosome 22's long arm.
• It is also called Velocardiofacial syndrome or DiGeorge syndrome
• It is characterised by Dysmorphogenesis of the 3rd and 4th pharyngeal pouches resulting in the
following features: CATCH-22 C: Cardiac anomaly/conotruncal anomaly A: Abnormal facies T: Thymic
hypoplasia/T cell abnormality C: Cleft palate H: Hypocalcemia 22: Chromosome 22
• C: Cardiac anomaly/conotruncal anomaly
• A: Abnormal facies
• T: Thymic hypoplasia/T cell abnormality
• C: Cleft palate
• H: Hypocalcemia
• 22: Chromosome 22
• C: Cardiac anomaly/conotruncal anomaly
• A: Abnormal facies
• T: Thymic hypoplasia/T cell abnormality

Page 27

238
• C: Cleft palate
• H: Hypocalcemia
• 22: Chromosome 22
Incorrect Options:
Option A - Brushfield spots:
• Brushfield spots are a feature of Down syndrome, also called trisomy 21.
• Other features include Epicanthic folds, Upslanting palpebral fissures, Transfer palmar creases
Cardiac defects.
• Epicanthic folds,
• Upslanting palpebral fissures,
• Transfer palmar creases
• Cardiac defects.
• Epicanthic folds,
• Upslanting palpebral fissures,
• Transfer palmar creases
• Cardiac defects.
Option B - polydactyly:
• It is a feature of trisomy 13.
• Other features include Microphthalmia Microcephaly, Holoprosencephaly Rocker bottom feet.
• Microphthalmia
• Microcephaly, Holoprosencephaly
• Rocker bottom feet.
• Microphthalmia
• Microcephaly, Holoprosencephaly
• Rocker bottom feet.
Option D - Cystic hygroma:
• It is seen in Turner syndrome.
• In Turner syndrome, patients have a single X-chromosome.
• Other features include short stature, a webbed neck, shield chest, and primary ovarian insufficiency.

Solution for Question 16:


Correct Option D - Acute myelogenous leukaemia:
• The images show typical features of Down syndrome: simian crease, hypotonia, saddle toe,
upslanting palpebral fissures, and epicanthal folds.

Page 28

239
• Down syndrome, also called trisomy 21, is due to an extra pair of chromosome 21, resulting in a total
of 47 chromosomes.
• In addition to the physical Defects and cardiac anomalies, downstream patients are at high risk of
Leukemias.
• Children with trisomy 21 have a high risk of developing leukaemia.
• There is a 20-fold increased risk of developing acute B lymphoblastic leukaemias.
• There is a 500-fold increased risk of acute myeloid leukaemias.
Incorrect Options:
Option A - Marginal zone lymphoma:
• It is a type of non-Hodgkin lymphoma caused by a translocation between chromosome 11 and
chromosomal 18.
• It is associated with chronic inflammatory disorders, like Sjogren’s syndrome, chronic gastritis etc.
• Not associated with down syndrome.
Option B - Burkitt lymphoma:
• It is a type of non-Hodgkin lymphoma caused by a translocation between chromosomes 8 and 14.
• It is associated with Epstein bar virus infection
• Not associated with down syndrome.
Option C - chronic lymphocytic leukaemia:
• It is more common in older patients
• It is a B-cell neoplasm that progresses slowly and is characterised by smudge cells on peripheral
blood smear.
• Down syndrome increases the chance of acute lymphoblastic leukaemia, not chronic lymphocytic
leukaemia.

Solution for Question 17:


Correct Option A - Acrocentric chromosome:
• Based on the features of open fontanelle, microcephaly, flat occiput, slanting palpebral fissures,
hypertelorism, low set and malformed ears, protruding tongue, open mouth, and hypotonia, and
diagnosis of the most common inheritable cause of mental retardation, the most likely disorder is Down
syndrome.
• Down syndrome is chiefly caused by trisomy of chromosome 21, which can lead to multiple systemic
complications which belong to the acrocentric subtype of chromosomes.
• An acrocentric chromosome is defined as a chromosome whose centromere is not central and is
instead located near the end of the chromosome.
• Humans have five pairs of acrocentric chromosomes (13, 14, 15, 21, 22). The Y chromosome also
belongs to the acrocentric subtype.
• Main features of Down syndrome: Short extremities Short hands along with single transverse palmar
crease Hypotonia Congenital heart defects Flat occiput and a flattened facial appearance Epicanthal

Page 29

240
folds Flat nasal bridge Protruding tongue Low-set, small ears
• Short extremities
• Short hands along with single transverse palmar crease
• Hypotonia
• Congenital heart defects
• Flat occiput and a flattened facial appearance
• Epicanthal folds
• Flat nasal bridge
• Protruding tongue
• Low-set, small ears
• Investigations: Complete blood count Bone marrow biopsy to rule out leukaemia Thyroid-stimulating
hormone and T4 to rule out hypothyroidism Karyotyping for diagnosis of trisomy 21 Echocardiography
to diagnose any congenital heart disease if present
• Complete blood count
• Bone marrow biopsy to rule out leukaemia
• Thyroid-stimulating hormone and T4 to rule out hypothyroidism
• Karyotyping for diagnosis of trisomy 21
• Echocardiography to diagnose any congenital heart disease if present
• Management: No medical treatments are available for intellectual abnormalities associated with Down
syndrome, but good medical care can significantly boost the quality of life. Medical care includes the
following elements: Genetic counselling Standard immunisations Management of specific features of
Down syndrome and associated disorders (e.g., endocrine, infectious, cardiac, respiratory, neurologic
disorders)
• No medical treatments are available for intellectual abnormalities associated with Down syndrome,
but good medical care can significantly boost the quality of life.
• Medical care includes the following elements: Genetic counselling Standard immunisations
Management of specific features of Down syndrome and associated disorders (e.g., endocrine,
infectious, cardiac, respiratory, neurologic disorders)
• Genetic counselling
• Standard immunisations
• Management of specific features of Down syndrome and associated disorders (e.g., endocrine,
infectious, cardiac, respiratory, neurologic disorders)
• Complications: Atrioventricular septal defects Atrioventricular valve malformations Lung infections
Celiac disease is more common in patients with Down syndrome. Chronic constipation Refractive
errors, such as myopia, hyperopia, and astigmatism. Hypothyroidism and type I diabetes mellitus
Haematological malignancies such as transient myeloproliferative disorder, acute myeloid leukaemia,
and acute lymphoblastic leukaemia. Recurrent respiratory and systemic infections secondary to
reduced immunoglobulin levels. Alzheimer-type dementia
• Atrioventricular septal defects
• Atrioventricular valve malformations

Page 30

241
• Lung infections
• Celiac disease is more common in patients with Down syndrome.
• Chronic constipation
• Refractive errors, such as myopia, hyperopia, and astigmatism.
• Hypothyroidism and type I diabetes mellitus
• Haematological malignancies such as transient myeloproliferative disorder, acute myeloid leukaemia,
and acute lymphoblastic leukaemia.
• Recurrent respiratory and systemic infections secondary to reduced immunoglobulin levels.
• Alzheimer-type dementia
• Short extremities
• Short hands along with single transverse palmar crease
• Hypotonia
• Congenital heart defects
• Flat occiput and a flattened facial appearance
• Epicanthal folds
• Flat nasal bridge
• Protruding tongue
• Low-set, small ears
• Complete blood count
• Bone marrow biopsy to rule out leukaemia
• Thyroid-stimulating hormone and T4 to rule out hypothyroidism
• Karyotyping for diagnosis of trisomy 21
• Echocardiography to diagnose any congenital heart disease if present
• No medical treatments are available for intellectual abnormalities associated with Down syndrome,
but good medical care can significantly boost the quality of life.
• Medical care includes the following elements: Genetic counselling Standard immunisations
Management of specific features of Down syndrome and associated disorders (e.g., endocrine,
infectious, cardiac, respiratory, neurologic disorders)
• Genetic counselling
• Standard immunisations
• Management of specific features of Down syndrome and associated disorders (e.g., endocrine,
infectious, cardiac, respiratory, neurologic disorders)
• Genetic counselling
• Standard immunisations
• Management of specific features of Down syndrome and associated disorders (e.g., endocrine,
infectious, cardiac, respiratory, neurologic disorders)
• Atrioventricular septal defects

Page 31

242
• Atrioventricular valve malformations
• Lung infections
• Celiac disease is more common in patients with Down syndrome.
• Chronic constipation
• Refractive errors, such as myopia, hyperopia, and astigmatism.
• Hypothyroidism and type I diabetes mellitus
• Haematological malignancies such as transient myeloproliferative disorder, acute myeloid leukaemia,
and acute lymphoblastic leukaemia.
• Recurrent respiratory and systemic infections secondary to reduced immunoglobulin levels.
• Alzheimer-type dementia

Incorrect Options:
Option B - Metacentric chromosome:
• Down syndrome is mainly caused by trisomy of chromosome 21, which can lead to multiple systemic
complications which belong to the acrocentric subtype of chromosomes, not the metacentric subtype of
chromosomes.
• Metacentric chromosomes have the centromere located in the middle of the ends of the chromosome,
separating the two arms of the chromosome.
Option C - Submetacentric chromosome:
• Down syndrome is chiefly caused by trisomy of chromosome 21, which can lead to multiple systemic
complications which belong to the acrocentric subtype of chromosomes, not to the submetacentric
subtype of chromosomes.
• A submetacentric chromosome is defined as a chromosome whose centromere is located near the
middle resulting in an unequal length of the chromosomal arms.
Option D - Telocentric chromosome:
• Down syndrome is primarily caused by trisomy of chromosome 21, which can lead to multiple
systemic complications which belong to the acrocentric subtype of chromosomes, not the telocentric

Page 32

243
subtype of chromosomes.
• A telocentric chromosome is defined as a chromosome whose centromere is located at one end.
• The centromere is located close to the end of the chromosome in a way the p arms would not, or
slightly visible.

Solution for Question 18:


Correct Option A - Mutation of MT-ATP6 gene are seen:
• Mutations in the MT-ATP6 gene cause NARP syndrome.
• This gene can be found in mitochondrial DNA (mtDNA).
• There is no deletion in the MT-ATP6 gene
Incorrect Options:
Option C - Neuropathy, ataxia, and optic atrophy are seen:
• NARP syndrome: NARP syndrome is a triad of neuropathy, ataxia, and retinitis pigmentosa It is
related to changes in mitochondrial DNA. Mutations in the MT-ATP6 gene cause NARP syndrome. This
gene can be found in mitochondrial DNA (mtDNA).NARP syndrome is inherited from the mother
(maternal inheritance). Only females pass mitochondrial DNA to their children. There is no cure for
NARP syndrome.
• NARP syndrome is a triad of neuropathy, ataxia, and retinitis pigmentosa
• It is related to changes in mitochondrial DNA.
• Mutations in the MT-ATP6 gene cause NARP syndrome. This gene can be found in mitochondrial
DNA (mtDNA).NARP syndrome is inherited from the mother (maternal inheritance).
• Only females pass mitochondrial DNA to their children.
• There is no cure for NARP syndrome.
• NARP syndrome is a triad of neuropathy, ataxia, and retinitis pigmentosa
• It is related to changes in mitochondrial DNA.
• Mutations in the MT-ATP6 gene cause NARP syndrome. This gene can be found in mitochondrial
DNA (mtDNA).NARP syndrome is inherited from the mother (maternal inheritance).
• Only females pass mitochondrial DNA to their children.
• There is no cure for NARP syndrome.

Page 33

244
Option B - It is a cytoplasmic disease:
• NARP syndrome is not a cytoplasmic disease.
• Mutations in the MT-ATP6 gene cause NARP syndrome.
• This gene can be found in mitochondrial DNA. (mtDNA).
Option D - It is a Lipid storage disease:
• NARP syndrome is not a lipid storage disease.
• It is a mitochondrial disease.
• Therefore this option is incorrect

Solution for Question 19:


Correct Option A - Upward-slanting palpebral fishers:
• Down syndrome is a chromosomal abnormality in which a newborn has an extra chromosome 21,
resulting in a total of 47 chromosomes.
• It is also called trisomy 21.
• The risk of Down syndrome increases as the mother's age increases.
• Common clinical features include Upslanting palpebral fissures Epicanthal folds Brushfield spots
High-arched palate Broad and flat nasal bridge Excess skin at the nape of the neck Transverse palmar
crease Short stature Atlanto-axial instability Congenital heart defects Varying levels of intellectual
disability
• Upslanting palpebral fissures
• Epicanthal folds
• Brushfield spots
• High-arched palate

Page 34

245
• Broad and flat nasal bridge
• Excess skin at the nape of the neck
• Transverse palmar crease
• Short stature
• Atlanto-axial instability
• Congenital heart defects
• Varying levels of intellectual disability
• Upslanting palpebral fissures
• Epicanthal folds
• Brushfield spots
• High-arched palate
• Broad and flat nasal bridge
• Excess skin at the nape of the neck
• Transverse palmar crease
• Short stature
• Atlanto-axial instability
• Congenital heart defects
• Varying levels of intellectual disability
Incorrect Options:
Option B - Upward and outward lens dislocation:
• Upward and outward lens dislocation is a feature of Marfan syndrome.
• It is an autosomal dominant connective tissue disorder affecting the microfibrils and elastin in
connective tissue throughout the body.
• Other features include cardiovascular defects, tall stature, long extremities, and joint hypermobility
Option C - Cystic hygroma:
• Cystic hygroma is a feature of Turner syndrome, the most common cause of ovarian dysgenesis and
primary ovarian insufficiency.
• It is a thin-walled sac-like structure that occurs due to damage to the lymphatic system during
development.
• Other features of Turner syndrome include short stature, a shield chest, a webbed neck, lymph
oedema, and delayed puberty.
Option D - Postaxial Polydactyly:
• Postaxial Polydactyly, the presence of extra digits on the ulnar (pinky) side of the hand or the fibular
(outer) side of the foot, is not commonly associated with Down syndrome.
• Polydactyly is a feature of trisomy 13 (Patau).
• It is caused by meiotic non-disjunction resulting in an extra chromosome 13.

Page 35

246
• Clinical features include polydactyly, microphthalmia, microcephaly, holoprosencephaly, congenital
heart defects, and rocker bottom feet.

Solution for Question 20:


Correct Option B - Cavernous lymphangioma:
• Karyotyping reveals this as a case of Turner syndrome; Cavernous lymphangioma is commonly seen
in it.
• Cavernous Lymphangioma: Large, diffuse, poorly defined, fluctuant, soft tissue mass. Found in the
neck/axilla of children, retroperitoneum (rare). May cause gross deformities of the neck or may fill the
axilla. Cavernous lymphangioma of the neck can be commonly found in Turner syndrome.
• Large, diffuse, poorly defined, fluctuant, soft tissue mass.
• Found in the neck/axilla of children, retroperitoneum (rare).
• May cause gross deformities of the neck or may fill the axilla.
• Cavernous lymphangioma of the neck can be commonly found in Turner syndrome.
• Large, diffuse, poorly defined, fluctuant, soft tissue mass.
• Found in the neck/axilla of children, retroperitoneum (rare).
• May cause gross deformities of the neck or may fill the axilla.
• Cavernous lymphangioma of the neck can be commonly found in Turner syndrome.
• Features of cavernous lymphangioma: Lesions are composed of dilated lymphatic spaces lined by
endothelial cells and separated by intervening connective tissue stroma containing lymphoid
aggregates. The tumour margins are indistinct and non-capsulated.
• Lesions are composed of dilated lymphatic spaces lined by endothelial cells and separated by
intervening connective tissue stroma containing lymphoid aggregates.
• The tumour margins are indistinct and non-capsulated.
• Lesions are composed of dilated lymphatic spaces lined by endothelial cells and separated by
intervening connective tissue stroma containing lymphoid aggregates.
• The tumour margins are indistinct and non-capsulated.
Incorrect Options:
Option A - Nevus flammeus:
• It is also called port wine, stain,
• It is a malformation of dermal, capillaries and venules that presents as a unilateral red patch that
blanches when pressed.
• It is associated with Sturge-Webber syndrome,
Option C - Pyogenic granuloma:
• It is a polypoid, lobulated, capillary hemangioma.
• It can ulcerate and bleed.
• It is associated with trauma and pregnancy.

Page 36

247
Option D - Capillary hemangioma:
• These are benign hemangiomas that form due to the abnormal development of capillaries.
• Strawberry hemangiomas are capillary hemangiomas of infancy, whereas cherry hemangiomas are
benign capillary hemangiomas that appear in adults.
• They are not associated with turner syndrome.

Solution for Question 21:


Correct Option A - Prader-Willi syndrome:
• The case mentioned above is about a mentally disabled child whose clinical features of obesity and
narrow shoulders suggest Prader-Willi syndrome.
• Prader-Willi syndrome is an example of genetic imprinting where the phenotype depends on whether
the deletion occurs on a gene inherited from the mother or father.
• Prader-Willi syndrome: if gene deleted from father.
• Angelman syndrome: if gene deleted from mother.
• PWS (Prader-Willi syndrome) could be present because of the following: Deletion of paternal
chromosome 15 Uniparental disomy of maternal chromosome15 SNORP mutation
• Deletion of paternal chromosome 15
• Uniparental disomy of maternal chromosome15
• SNORP mutation
• Prader-Willi syndrome (PWS) is a genetic disorder occurring in 1 of 10,000–16,000 live births. It is
characterised by excessive appetite, massive progressive obesity, short stature, and mental
retardation.
• The majority of individuals have hypogonadotropic hypogonadism and GH deficiency.
• The cause of obesity is thought to be increased levels of ghrelin
• Deletion of paternal chromosome 15
• Uniparental disomy of maternal chromosome15
• SNORP mutation

Page 37

248
Incorrect Options:
Option B - Klinefelter syndrome:
• Extra X chromosome = Klinefelter syndrome (47 XXY).
• Klinefelter syndrome is a condition in which male hypogonadism occurs when there are two or more X
chromosomes and one or more Y-chromosomes.
• It is one of the most common genetic causes of infertility.
Option C - Angelman syndrome:
• When a child receives duplicate copies of chromosome #15 from the father, Angelman syndrome may
develop(rather than one from the mother and one from the father).
• It is not the common genetic cause of infertility.
• Angelman syndrome occurs if the gene is deleted from the mother.
Option D - Turner syndrome:
• Turner syndrome is the most common cause of primary amenorrhea.
• Its Karyotype is XO.

Page 38

249
Solution for Question 22:
Correct Option D - Mitochondrial inheritance:
• It is transmitted only through the mother.
• All offsprings of affected females may show signs of disease.
• Male-to-male transmission does not occur as mitochondria are passed to offspring only by the mother.
• Variable expression in a population or even within a family occurs due to heteroplasmy (combination
of mutant and wild-type genes.
• Mitochondrial DNA (mtDNA): Maternal mitochondrial DNA (mtDNA) is inherited by all offspring,
regardless of gender, from their mothers. However, only daughters, not sons, pass on this mtDNA to
their own children. mtDNA produces enzymes involved in oxidative phosphorylation; mutations
affecting these genes have the greatest impact on organs that rely on it, such as the central nervous
system, skeletal muscle, cardiac muscle, liver, and kidneys. Thousands of copies of mtDNA exist in
each mitochondrion, and detrimental mtDNA mutations often affect some, but not all, of these copies.
As a result, individuals can have both wild-type and mutant mtDNA, a condition known as
heteroplasmy. Leber hereditary optic neuropathy is the prototype Leber hereditary optic neuropathy:
Leber Hereditary Optic Neuropathy (LHON) is the most common inherited mitochondrial disorder and
typically affects young males. It typically begins as a unilateral progressive optic neuropathy with
sequential involvement of the fellow eye months to years later. Vision loss occurs due to cell death of
optic nerve neurons.
• Maternal mitochondrial DNA (mtDNA) is inherited by all offspring, regardless of gender, from their
mothers. However, only daughters, not sons, pass on this mtDNA to their own children.
• mtDNA produces enzymes involved in oxidative phosphorylation; mutations affecting these genes
have the greatest impact on organs that rely on it, such as the central nervous system, skeletal muscle,
cardiac muscle, liver, and kidneys. Thousands of copies of mtDNA exist in each mitochondrion, and
detrimental mtDNA mutations often affect some, but not all, of these copies. As a result, individuals can
have both wild-type and mutant mtDNA, a condition known as heteroplasmy.

Page 39

250
• Leber hereditary optic neuropathy is the prototype
• Leber hereditary optic neuropathy: Leber Hereditary Optic Neuropathy (LHON) is the most common
inherited mitochondrial disorder and typically affects young males. It typically begins as a unilateral
progressive optic neuropathy with sequential involvement of the fellow eye months to years later. Vision
loss occurs due to cell death of optic nerve neurons.
• Another example of mitochondrial inheritance is MELAS syndrome.
• Mitochondrial encephalomyopathy, lactic acidosis, and stroke-like episodes (MELAS) is an extremely
rare genetic condition that begins in childhood.
• Maternal mitochondrial DNA (mtDNA) is inherited by all offspring, regardless of gender, from their
mothers. However, only daughters, not sons, pass on this mtDNA to their own children.
• mtDNA produces enzymes involved in oxidative phosphorylation; mutations affecting these genes
have the greatest impact on organs that rely on it, such as the central nervous system, skeletal muscle,
cardiac muscle, liver, and kidneys. Thousands of copies of mtDNA exist in each mitochondrion, and
detrimental mtDNA mutations often affect some, but not all, of these copies. As a result, individuals can
have both wild-type and mutant mtDNA, a condition known as heteroplasmy.
• Leber hereditary optic neuropathy is the prototype
• Leber hereditary optic neuropathy: Leber Hereditary Optic Neuropathy (LHON) is the most common
inherited mitochondrial disorder and typically affects young males. It typically begins as a unilateral
progressive optic neuropathy with sequential involvement of the fellow eye months to years later. Vision
loss occurs due to cell death of optic nerve neurons.
Maternal mitochondrial DNA (mtDNA) is inherited by all offspring, regardless of gender, from their moth
ers. However, only daughters, not sons, pass on this mtDNA to their own children.
mtDNA produces enzymes involved in oxidative phosphorylation; mutations affecting these genes have
the greatest impact on organs that rely on it, such as the central nervous system, skeletal muscle, car
diac muscle, liver, and kidneys. Thousands of copies of mtDNA exist in each mitochondrion, and detri
mental mtDNA mutations often affect some, but not all, of these copies. As a
result, individuals can have both wild-type and mutant mtDNA, a condition known as heteroplasmy.
Incorrect Options:
Option A - Autosomal dominant:
• In the autosomal dominant pattern of inheritance, a single copy of the mutated gene (from one parent)
is enough to cause the disorder.
• A child of a person affected by an autosomal dominant condition has a 50% chance of being affected
by that condition via inheritance of a dominant allele.
• It is often due to defects in structural genes.
• Many generations, both males and females, are affected.
• Examples: Achondroplasia, Huntington’s disease, Neurofibromatosis type 1 and 2 etc
Option B - Autosomal recessive:
• For a recessive trait or disease to be displayed, two copies of the trait or disorder need to be
presented. The trait or gene will be located on a non-sex chromosome.
• Often due to enzyme deficiencies.
• With 2 carrier (heterozygous) parents, ¼ of the children will be affected, ½ of the children will be
carriers, and ¼ of the children will be unaffected.

Page 40

251
• Examples: Cystic fibrosis, Wilson disease, Sickle cell disease etc
Option C - X-linked recessive:
• X-linked recessive inheritance refers to genetic conditions associated with mutations in genes on the
X chromosome. A male carrying such a mutation will be affected because he carries only one X
chromosome. A female carrying a mutation in one gene, with a normal gene on the other X
chromosome, is generally unaffected.
• Sons of heterozygous mothers have a 50% chance of being affected.
• No male-to-male transmission occurs.
• Example: Haemophilia

Solution for Question 23:


Correct Option A - Trisomy 21:
• Trisomy 21 is down's syndrome.
• Down syndrome is the most common inheritable cause of mental retardation
• Antenatal Screening Tests
• Triple test: β-hCG ↑↑ Maternal Serum Alpha-fetoprotein (MSAFP) ↓↓ Unconjugated Estriol (uE3) ↓↓
• β-hCG ↑↑
• Maternal Serum Alpha-fetoprotein (MSAFP) ↓↓
• Unconjugated Estriol (uE3) ↓↓
• Quadruple Test: β-hCG ↑↑ Maternal Serum Alpha-fetoprotein (MSAFP) ↓↓ Unconjugated Estriol
(uE3) ↓↓ Dimeric Inhibin-A ↑↑
• β-hCG ↑↑
• Maternal Serum Alpha-fetoprotein (MSAFP) ↓↓
• Unconjugated Estriol (uE3) ↓↓
• Dimeric Inhibin-A ↑↑
• Down syndrome is the most common inheritable cause of mental retardation
• There is no definitive treatment for Down syndrome. Early treatment programs can help improve skills
and lifestyle. These include physical, occupational, and educational therapy
• Early treatment programs can help improve skills and lifestyle. These include physical, occupational,
and educational therapy
• β-hCG ↑↑
• Maternal Serum Alpha-fetoprotein (MSAFP) ↓↓
• Unconjugated Estriol (uE3) ↓↓
• β-hCG ↑↑
• Maternal Serum Alpha-fetoprotein (MSAFP) ↓↓
• Unconjugated Estriol (uE3) ↓↓

Page 41

252
• Dimeric Inhibin-A ↑↑
• Early treatment programs can help improve skills and lifestyle. These include physical, occupational,
and educational therapy
Incorrect Options:
Option B - Trisomy 18:
• Trisomy 18 is Edwards’s syndrome.
• Its symptoms include: Prominent occiput Low-set ears Mental retardation Congenital heart defects
Rocker-bottom feet
• Prominent occiput
• Low-set ears
• Mental retardation
• Congenital heart defects
• Rocker-bottom feet
• Prominent occiput
• Low-set ears
• Mental retardation
• Congenital heart defects
• Rocker-bottom feet
Option C - Trisomy 13:
• Trisomy 13 (Patau syndrome) symptoms: Microphthalmia Microcephaly and mental retardation
Polydactyly Cleft lip and palate Renal defects
• Microphthalmia
• Microcephaly and mental retardation
• Polydactyly
• Cleft lip and palate
• Renal defects
• The diagnosis can be confirmed prenatally by chorionic villus sampling (CVS) or amniocentesis.
• There is no specific cure for Patau syndrome.
• Management focuses on the treatment of symptoms.
• Microphthalmia
• Microcephaly and mental retardation
• Polydactyly
• Cleft lip and palate
• Renal defects
Option D - 45, X:
• 45, X is Turner syndrome.

Page 42

253
• Features include: Short stature Coarctation of the aorta Cubitus valgus Streak ovaries, infertility,
amenorrhea Peripheral lymphedema at birth Low posterior hairlin Webbing of neck Broad chest and
widely spaced nipples
• Short stature
• Coarctation of the aorta
• Cubitus valgus
• Streak ovaries, infertility, amenorrhea
• Peripheral lymphedema at birth
• Low posterior hairlin
• Webbing of neck
• Broad chest and widely spaced nipples
• Short stature
• Coarctation of the aorta
• Cubitus valgus
• Streak ovaries, infertility, amenorrhea
• Peripheral lymphedema at birth
• Low posterior hairlin
• Webbing of neck
• Broad chest and widely spaced nipples

Solution for Question 24:


Correct Option A - Genomic imprinting:
• Genomic imprinting: A phenomenon in which one copy of the gene is silenced by methylation and the
other copy is expressed. Example: Prader-Willi and Angelman syndrome.
• Prader-Willi syndrome: The specific genetic alterations leading to Prader-Willi syndrome include:
Deletion of the Paternally Inherited Chromosome Segment: In about 60-70% of cases, Prader-Willi
syndrome results from the deletion of the paternally derived chromosome 15q11-15q13 region. This
leads to the loss of critical genes in that region, contributing to the characteristic features of PWS.
Uniparental Disomy (UPD) of Maternal Chromosome 15: In approximately 30-40% of cases, individuals
with Prader-Willi syndrome exhibit uniparental disomy, meaning they have inherited both copies of
chromosome 15 from their mother, and none from their father. This results in the absence of the
paternally expressed genes on chromosome 15, leading to PWS. Mutation of Genes/Loci Involved in
Imprinting: Specific genes within the 15q11-15q13 region are subject to genomic imprinting, where the
expression of the gene depends on whether it is inherited from the mother or the father. Mutations
affecting these imprinted genes can also lead to Prader-Willi syndrome.
• The specific genetic alterations leading to Prader-Willi syndrome include:
• Deletion of the Paternally Inherited Chromosome Segment: In about 60-70% of cases, Prader-Willi
syndrome results from the deletion of the paternally derived chromosome 15q11-15q13 region. This
leads to the loss of critical genes in that region, contributing to the characteristic features of PWS.

Page 43

254
• In about 60-70% of cases, Prader-Willi syndrome results from the deletion of the paternally derived
chromosome 15q11-15q13 region. This leads to the loss of critical genes in that region, contributing to
the characteristic features of PWS.
• Uniparental Disomy (UPD) of Maternal Chromosome 15: In approximately 30-40% of cases,
individuals with Prader-Willi syndrome exhibit uniparental disomy, meaning they have inherited both
copies of chromosome 15 from their mother, and none from their father. This results in the absence of
the paternally expressed genes on chromosome 15, leading to PWS.
• In approximately 30-40% of cases, individuals with Prader-Willi syndrome exhibit uniparental disomy,
meaning they have inherited both copies of chromosome 15 from their mother, and none from their
father. This results in the absence of the paternally expressed genes on chromosome 15, leading to
PWS.
• Mutation of Genes/Loci Involved in Imprinting: Specific genes within the 15q11-15q13 region are
subject to genomic imprinting, where the expression of the gene depends on whether it is inherited from
the mother or the father. Mutations affecting these imprinted genes can also lead to Prader-Willi
syndrome.
• Specific genes within the 15q11-15q13 region are subject to genomic imprinting, where the
expression of the gene depends on whether it is inherited from the mother or the father. Mutations
affecting these imprinted genes can also lead to Prader-Willi syndrome.
• Signs and symptoms: hyperphagia, obesity, intellectual disability, hypogonadism and hypotonia.
• Angelman syndrome: It is a genetic disorder caused by a loss of function of specific genes on
chromosome 15. Paternally derived genes are silenced. The disease occurs when the maternal allele is
mutated or deleted. Signs and symptoms: Seizures, ataxia, severe intellectual disability, inappropriate
laughter.
• It is a genetic disorder caused by a loss of function of specific genes on chromosome 15.
• Paternally derived genes are silenced.
• The disease occurs when the maternal allele is mutated or deleted.
• Signs and symptoms: Seizures, ataxia, severe intellectual disability, inappropriate laughter.
• The specific genetic alterations leading to Prader-Willi syndrome include:
• Deletion of the Paternally Inherited Chromosome Segment: In about 60-70% of cases, Prader-Willi
syndrome results from the deletion of the paternally derived chromosome 15q11-15q13 region. This
leads to the loss of critical genes in that region, contributing to the characteristic features of PWS.
• In about 60-70% of cases, Prader-Willi syndrome results from the deletion of the paternally derived
chromosome 15q11-15q13 region. This leads to the loss of critical genes in that region, contributing to
the characteristic features of PWS.
• Uniparental Disomy (UPD) of Maternal Chromosome 15: In approximately 30-40% of cases,
individuals with Prader-Willi syndrome exhibit uniparental disomy, meaning they have inherited both
copies of chromosome 15 from their mother, and none from their father. This results in the absence of
the paternally expressed genes on chromosome 15, leading to PWS.
• In approximately 30-40% of cases, individuals with Prader-Willi syndrome exhibit uniparental disomy,
meaning they have inherited both copies of chromosome 15 from their mother, and none from their
father. This results in the absence of the paternally expressed genes on chromosome 15, leading to
PWS.
• Mutation of Genes/Loci Involved in Imprinting: Specific genes within the 15q11-15q13 region are
subject to genomic imprinting, where the expression of the gene depends on whether it is inherited from
the mother or the father. Mutations affecting these imprinted genes can also lead to Prader-Willi

Page 44

255
syndrome.
• Specific genes within the 15q11-15q13 region are subject to genomic imprinting, where the
expression of the gene depends on whether it is inherited from the mother or the father. Mutations
affecting these imprinted genes can also lead to Prader-Willi syndrome.
The specific genetic alterations leading to Prader-Willi syndrome include:
• In about 60-70% of cases, Prader-Willi syndrome results from the deletion of the paternally derived
chromosome 15q11-15q13 region. This leads to the loss of critical genes in that region, contributing to
the characteristic features of PWS.
• In approximately 30-40% of cases, individuals with Prader-Willi syndrome exhibit uniparental disomy,
meaning they have inherited both copies of chromosome 15 from their mother, and none from their
father. This results in the absence of the paternally expressed genes on chromosome 15, leading to
PWS.
• Specific genes within the 15q11-15q13 region are subject to genomic imprinting, where the
expression of the gene depends on whether it is inherited from the mother or the father. Mutations
affecting these imprinted genes can also lead to Prader-Willi syndrome.
• It is a genetic disorder caused by a loss of function of specific genes on chromosome 15.
• Paternally derived genes are silenced.
• The disease occurs when the maternal allele is mutated or deleted.
• Signs and symptoms: Seizures, ataxia, severe intellectual disability, inappropriate laughter.
Incorrect Options:
Option B - Mosaicism:
• It is the presence of genetically distinct cell lines in the same individual.
• Mosaicism occurs when a person has two or more genetically different sets of cells in the body.
• If those abnormal cells begin to outnumber the normal cells, it can lead to disease that can be traced
from the cellular level to affected tissue, like skin, the brain, or other organs.
• Example: McCune-Albright syndrome
Option C - Anticipation:
• A phenomenon in which the signs and symptoms of some genetic conditions tend to become more
severe and/or appear at an earlier age as the disorder is passed from one generation to the next.
• Example: Huntington’s disease
Option D - Non-penetrance:
• A genetic trait (characteristic) that is present in the genome but does not manifest itself in the
individual is known as Non penentrance
• Genomic imprinting: A phenomenon in which one copy of the gene is silenced by methylation and the
other copy is expressed.
• A phenomenon in which one copy of the gene is silenced by methylation and the other copy is
expressed.
• A phenomenon in which one copy of the gene is silenced by methylation and the other copy is
expressed.

Page 45

256
Solution for Question 25:
Correct Option A - Trisomy 13:
• The case mentioned above shows that the newborn suffers from Patau syndrome and has features of
cleft lip, polydactyly, and microphthalmia.
• The image depicts a cleft lip, a typical feature of Patau syndrome.
• Trisomy 13 (Patau syndrome) symptoms: Microphthalmia Microcephaly and mental retardation
Polydactyly Cleft lip and palate Renal defects
• Microphthalmia
• Microcephaly and mental retardation
• Polydactyly
• Cleft lip and palate
• Renal defects
• The diagnosis can be confirmed prenatally by chorionic villus sampling (CVS) or amniocentesis.
• There is no specific cure for Patau syndrome.
• Management focuses on the treatment of symptoms
• Microphthalmia
• Microcephaly and mental retardation
• Polydactyly
• Cleft lip and palate
• Renal defects
Incorrect Options:
Option B - Trisomy 18:
• Trisomy 18 is Edward's syndrome.
• Its features are: Micrognathia Low-set ears Rocker bottom feet Overlapping of fingers
• Micrognathia
• Low-set ears
• Rocker bottom feet
• Overlapping of fingers
• Micrognathia
• Low-set ears
• Rocker bottom feet
• Overlapping of fingers
Option C - Trisomy 21:
• Trisomy 21 is Down's syndrome.
• Its features are: Palpebral fissures Low-set ears Providing tongue Hypotonia Single palmar crease
• Palpebral fissures

Page 46

257
• Low-set ears
• Providing tongue
• Hypotonia
• Single palmar crease
• Palpebral fissures
• Low-set ears
• Providing tongue
• Hypotonia
• Single palmar crease
Option D - None of the above:
• The case mentioned above has features suggestive of Trisomy 13, Patau Syndrome

Solution for Question 26:


Correct Option A - Anticipation:
• A phenomenon in which disease onset occurs earlier and/or the disease manifestation is more severe
in offspring than in parents.
Anticipation:
• In genetics, anticipation is a phenomenon whereby, as a genetic disorder is passed on to the next
generation, the symptoms of the genetic disorder become apparent at an earlier age with each
generation. In most cases, an increase in the severity of symptoms is also noted.
• Anticipation is common in trinucleotide repeat disorders, such as Huntington's disease and myotonic
dystrophy, where a dynamic mutation in DNA occurs.
• All of these diseases have neurological symptoms. Before understanding the genetic mechanism for
anticipation, it was debated whether anticipation was a true biological phenomenon or whether the
earlier age of diagnosis was related to heightened awareness of disease symptoms within a family.
Diseases showing genetic anticipation:
Autosomal Dominant
Autosomal Recessive
Multiple spinocerebellar ataxias
Friedreich’s ataxia – GAA
Huntington’s disease – CAG
X-linked
Myotonic dystrophy – CTG
Fragile X Syndrome – CGG
Congenital dyskeratosis – TTAGGG (telomere repeating sequence)
No expression type

Page 47

258
Crohn’s disease
Behcet’s disease
Incorrect Options:
Option B - Pleiotropy:
• Pleiotropy occurs when one gene influences two or more seemingly unrelated phenotypic traits. Such
a gene that exhibits multiple phenotypic expressions is called a pleiotropic gene.
• One of the most widely cited examples of pleiotropy in humans is phenylketonuria (PKU).
• This disorder is caused by a deficiency of the enzyme phenylalanine hydroxylase, which is necessary
to convert the essential amino acid phenylalanine to tyrosine
Option C - Imprinting:
• Imprinting is a mechanism of gene regulation in which one allele of a gene is silenced and imprinted
while the other allele is expressed depending on which parent it was inherited from (i.e., parent-of-origin
effect)
• Disorders of imprinting are Angelman syndrome and Prader-Willi syndrome.
Option D - Mosaicism:
• Mosaicism or genetic mosaicism is a condition in multicellular organisms in which a single organism
possesses more than one genetic line due to genetic mutation.
• This means that various genetic lines resulted from a single fertilized egg.
• For example, a person may have some cells in their body with 46 chromosomes, while others in their
body have 47 chromosomes.
• An example of mosaicism is Down mosaic syndrome.

Solution for Question 27:


Correct Option B - FSH increase:
• Klinefelter syndrome is a condition in which male hypogonadism occurs when there are two or more X
chromosomes and one or more Y-chromosomes.
• It can rarely be diagnosed before puberty.
• In males with this syndrome, testicular tubules are atrophied and replaced by pink, hyaline,
collagenous ghosts.
• The mean IQ is somewhat lower than normal, but Intellectual disability is uncommon.
• The plasma gonadotropin concentrations, particularly follicle-stimulating hormone (FSH), are
consistently elevated, whereas testosterone levels are variably reduced (<300 ng/dL).
• Normal levels of FSH in adults: 1.5 to 12.4 mIU/mL (1.5 to 12.4 IU/L).
• There is an increased incidence of type 2 diabetes.
• It results from the non-disjunction of chromosomes during meiosis.
• In 25% of cases, patients with the syndrome also have Autism.
• It is the most common genetic cause of infertility

Page 48

259
Incorrect Options:
Option A - Mental retardation is common:
• Individuals with Klinefelter's syndrome generally do not have intellectual disability.
• But may be found to have a lower IQ for age.
Option C - Karyotype 45, XO:
• Klinefelter syndrome is a condition in which male hypogonadism occurs when there are two or more X
chromosomes and one or more Y-chromosomes.
• The karyotype is 47, XXY, not 45, XO.
Option D - Results due to non-disjunction during mitotic division:
• Klinefelter's syndrome results from the non-disjunction of chromosomes during meiosis.

Solution for Question 28:


Correct Option B - Isochromosome:

• Isochromosome formation occurs when one arm of a chromosome is deleted while the other is
duplicated.
• This results in a chromosome with only two short arms or two long arms.
• Genetic information in both limbs is morphologically identical.
• The centromere misdivision is the cause of the creation of an isochromosome
• Instead of dividing longitudinally to separate the two sister chromatids, the centromere undergoes a
transverse split that separates the two arms.
• Examples: Xq, 17q, 12p
Incorrect Options:
Option A - Ring chromosome:
• A ring chromosome is an aberrant whose ends have fused to form a ring.
Option C - Acrocentric chromosome:
• An acrocentric chromosome is where the centromere is not central and is located near the end.
• Humans generally have five pairs of acrocentric autosomes (chromosomes 13, 14, 15, 21, 22).
• The Y chromosome is also acrocentric.
Option D - Subtelocentric chromosome:
• A sub-telocentric chromosome is a centromere between its middle and end but closer to the end.

Solution for Question 29:


Correct Option C - Mitochondrial:

Page 49

260
• Based on the pedigree discussed above, where all the children from sisters are affected, but all the
children from sons are not affected, this is a case of mitochondrial myopathy.
• It has a mitochondrial inheritance pattern, and the symptoms discussed point towards mitochondrial
myopathy.
• In mitochondrial inheritance, transmission is exclusively maternal; affected men do not pass the
characteristics to their children, whereas affected females pass it on to all of their children.
• Genes that code for oxidative phosphorylation enzymes are commonly involved in mitochondrial
inheritance abnormalities.
• Symptoms of mitochondrial myopathy: Muscle weakness or exercise intolerance, deafness,
blindness, droopy eyelids, movement disorders, stroke-like episodes, limited mobility of the eyes, heart
failure or rhythm disturbances, dementia, vomiting, and seizures.
• Treatment: Physical therapy may increase the muscles' range of motion and increase dexterity even
though there is no cure for any of the mitochondrial myopathies.
• Disorders due to Mitochondrial inheritance include: Mitochondrial myopathy Leigh syndrome Leber’s
hereditary optic neuropathy Diabetes mellitus and deafness (DAD) combination
• Mitochondrial myopathy
• Leigh syndrome
• Leber’s hereditary optic neuropathy
• Diabetes mellitus and deafness (DAD) combination
• Mitochondrial myopathy
• Leigh syndrome
• Leber’s hereditary optic neuropathy
• Diabetes mellitus and deafness (DAD) combination

Incorrect Options:
Option A - Autosomal dominant:

Page 50

261
• Based on the pedigree discussed above, where all the children from sisters are affected, but all the
children from sons are not affected, this is a case of the Mitochondrial inheritance pattern.
• In the Autosomal dominant pattern, males and females are equally affected. Examples include:
Achondroplasia Marfan syndrome Ehlers-Danlos syndrome
• Achondroplasia
• Marfan syndrome
• Ehlers-Danlos syndrome
• Achondroplasia
• Marfan syndrome
• Ehlers-Danlos syndrome
Option B - Autosomal recessive:
• Based on the pedigree discussed above, where all the children from sisters are affected, but all the
children from sons are not affected, this is a case of the Mitochondrial inheritance pattern.
• In the Autosomal recessive pattern, males and females are equally affected. Parents are carriers and
are clinically normal. Only homozygous children are affected. Examples include: Beta thalassemia
Glycogen storage disease Cystic fibrosis
• Beta thalassemia
• Glycogen storage disease
• Cystic fibrosis
• Beta thalassemia
• Glycogen storage disease
• Cystic fibrosis
Option D - X-linked dominant:
• Based on the pedigree discussed above, where all the children from sisters are affected, but all the
children from sons are not affected, this is a case of the Mitochondrial inheritance pattern.
• In the X-linked dominant pattern, females are more affected than men. It can manifest in heterozygous
females and homozygous males. Examples include: Vitamin D-resistant rickets.
Pseudohypoparathyroidism
• Vitamin D-resistant rickets.
• Pseudohypoparathyroidism
• Vitamin D-resistant rickets.
• Pseudohypoparathyroidism

Solution for Question 30:


Correct Option C - Lymphoblastic leukaemia:
• Based on the features of open fontanelle, microcephaly, flat occiput, slanting palpebral fissures,
hypertelorism, low set and malformed ears, protruding tongue, open mouth, and hypotonia and

Page 51

262
karyotyping finding of 47, XY with trisomy 21, the most likely diagnosis is Down syndrome.
• The life-threatening disorder that should be discussed with the parents regarding its association with
this syndrome is lymphoblastic leukaemia.
• Down syndrome is primarily caused by trisomy of chromosome 21, which can lead to multiple
systemic complications.
• Main features: Short extremities Short hands along with single transverse palmar crease Hypotonia
Congenital heart defects Flat occiput and a flattened facial appearance Epicanthal folds Flat nasal
bridge Protruding tongue Low-set, small ears
• Short extremities
• Short hands along with single transverse palmar crease
• Hypotonia
• Congenital heart defects
• Flat occiput and a flattened facial appearance
• Epicanthal folds
• Flat nasal bridge
• Protruding tongue
• Low-set, small ears
• Investigations: Complete blood count Bone marrow biopsy to rule out leukaemia Thyroid-stimulating
hormone and T4 to rule out hypothyroidism Karyotyping for diagnosis of trisomy 21 Echocardiography
to diagnose any congenital heart disease if present
• Complete blood count
• Bone marrow biopsy to rule out leukaemia
• Thyroid-stimulating hormone and T4 to rule out hypothyroidism
• Karyotyping for diagnosis of trisomy 21
• Echocardiography to diagnose any congenital heart disease if present
• Management: No medical treatments are available for intellectual abnormalities associated with Down
syndrome, but good medical care can greatly boost the quality of life. Medical care includes the
following elements: Genetic counselling Standard immunisations Management of specific features of
Down syndrome and associated disorders (e.g., endocrine, infectious, cardiac, respiratory, neurologic
disorders)
• No medical treatments are available for intellectual abnormalities associated with Down syndrome,
but good medical care can greatly boost the quality of life.
• Medical care includes the following elements: Genetic counselling Standard immunisations
Management of specific features of Down syndrome and associated disorders (e.g., endocrine,
infectious, cardiac, respiratory, neurologic disorders)
• Genetic counselling
• Standard immunisations
• Management of specific features of Down syndrome and associated disorders (e.g., endocrine,
infectious, cardiac, respiratory, neurologic disorders)
• Short extremities

Page 52

263
• Short hands along with single transverse palmar crease
• Hypotonia
• Congenital heart defects
• Flat occiput and a flattened facial appearance
• Epicanthal folds
• Flat nasal bridge
• Protruding tongue
• Low-set, small ears
• Complete blood count
• Bone marrow biopsy to rule out leukaemia
• Thyroid-stimulating hormone and T4 to rule out hypothyroidism
• Karyotyping for diagnosis of trisomy 21
• Echocardiography to diagnose any congenital heart disease if present
• No medical treatments are available for intellectual abnormalities associated with Down syndrome,
but good medical care can greatly boost the quality of life.
• Medical care includes the following elements: Genetic counselling Standard immunisations
Management of specific features of Down syndrome and associated disorders (e.g., endocrine,
infectious, cardiac, respiratory, neurologic disorders)
• Genetic counselling
• Standard immunisations
• Management of specific features of Down syndrome and associated disorders (e.g., endocrine,
infectious, cardiac, respiratory, neurologic disorders)
• Genetic counselling
• Standard immunisations
• Management of specific features of Down syndrome and associated disorders (e.g., endocrine,
infectious, cardiac, respiratory, neurologic disorders)
Incorrect Options:
Option A - Berry aneurysm of the circle of Willis:
• The clinical scenario point towards the diagnosis of down syndrome.
• The life-threatening disorder that should be discussed with the parents in terms of its association with
this syndrome is lymphoblastic leukaemia, not a berry aneurysm of the circle of Willis.
• Berry aneurysms, which account for 90% of all cerebral aneurysms, are the most prevalent
intracranial aneurysms.
Option B - Creutzfeldt-Jakob disease:
• As the patient is diagnosed with Down syndrome, The life-threatening disorder that should be
discussed with the parents regarding its association with this syndrome is lymphoblastic leukaemia, not
Creutzfeldt-Jakob disease.
• Creutzfeldt-Jakob disease is a rapidly progressive and fatal neurodegenerative disorder caused by an
abnormal isoform of a cellular glycoprotein called the prion protein.

Page 53

264
Option D - Medullary carcinoma of the thyroid:
• As the patient is diagnosed with Down syndrome, The life-threatening disorder that should be
discussed with the parents in terms of its association with this syndrome is lymphoblastic leukaemia,
not medullary carcinoma of the thyroid.
• Medullary carcinoma of the thyroid is a neuroendocrine tumour that arises from the parafollicular C
cells of the thyroid gland

Solution for Question 31:


Correct Option A
- It is associated with chromosome rearrangements termed chromosomal catastrophes:
• Chromothrypsis is a phenomenon by which up to thousands of clustered chromosomal
rearrangements occur in a single event in localised and confined genomic regions
• It is also involved in cancer (1% -2%), osteosarcomas, and gliomas.
• Chromosome breaks occur within part or across the entirety of a single chromosome or several
chromosomes in Chromothrypsis.
• Chromothrypsis activate oncogenes and inactivate tumour suppressors, thereby expediting the
process of carcinogenesis.
Incorrect Options:
Option B - It is observed in 50% of cancers as a whole:
• Chromothrypsis is a phenomenon by which up to thousands of clustered chromosomal
rearrangements occur in a single event in localised and confined genomic regions.
• It is involved in 1% -2% of all cancers.
Option C - Atleast 50% of malignancies demonstrate this pattern:
• Chromothrypsis is a phenomenon which leads to the activation of oncogenes and the inactivation of
tumour suppressors, thereby expediting the process of carcinogenesis.
Option D -It is associated with a decreased frequency of osteosarcomas and gliomas:
• Chromothrypsis is a phenomenon by which up to thousands of clustered chromosomal
rearrangements occur in a single event in localised and confined genomic regions.
• It is associated with a 25 increase in the frequency of osteosarcomas and a high frequency in gliomas.

Solution for Question 32:


Correct Option D - Gonadal mosaicism:
• Osteogenesis imperfecta is a disorder of collagen metabolism resulting in bone fragility and fractures.
The most common and milder form of osteogenesis imperfecta is type 1. Gonadal mosaicism best
describes the family's two afflicted children.
• Although osteogenesis imperfecta is inherited in autosomal dominant manner, in this scenario the
parents are unaffected while both children demonstrate features of the disease, this is likely due to

Page 54

265
gonadal mosaicism.
• This mutation responsible for mosaicsm occurs post zygotically. If the mutation only affect cells that
form gonads, then the gametes carry the mutation.
• A phenotypically normal parent with gonandal mosaicism can trasmit the disease causing mutation to
the offspring through their mutated gametes.
Incorrect Options:
Option A - Incomplete penetrance:
• Incomplete penetrance is a form of gene interaction in which both alleles of a gene are partially
expressed, resulting in an intermediate or different phenotype.
Option B - Uniparental disomy:
• Prader-Willi syndrome can occur due to uniparental disomy (paternal chromosome 15).
• It affects the paternally derived chromosome 15.
Option C - Genomic imprinting:
• Genomic imprinting is characterised by differential gene expression, which depends on the parent of
origin.
• Gene can be silent or imprinted by DNA methylation or Histone deacetylation, making gene
transcriptionally inactive.

Page 55

266
Previous Year Questions
1. In this case, what is the mode of transmission of the medical condition from the boy's grandmother to
his father and subsequently to him?
A. X-linked dominance
B. X-linked recessive
C. Autosomal dominance
D. Autosomal recessive
----------------------------------------
2. Below is a family pedigree chart. Please determine the pattern of inheritance for this condition.

A. Autosomal dominant
B. Autosomal recessive
C. X-linked recessive
D. X-linked dominant
----------------------------------------
3. In an X-linked dominant pattern, which of the following disorders is inherited?
A. Rett syndromes
B. Duchenne muscular dystrophy
C. Huntington’s disease
D. Tuberous sclerosis
----------------------------------------
4. On which chromosome is the SRY gene located?
A. Y-chromosome short arm
B. Y-chromosome long arm
C. X-chromosome short arm
D. X-chromosome long arm
----------------------------------------

267
5. Which of the following disorders does not have a contraindication for administering vaccines to
individuals affected by the disease?
A. Digeorge syndrome
B. Wiskott Aldrich syndrome
C. Complement fixation disorders
D. Ataxia telangiectasia
----------------------------------------
6. Following ANA pattern on IF is seen, identify the pattern.

A. Nucleolar
B. Speckeled
C. Centromeric
D. Diffuse
----------------------------------------
7. How should the provided pedigree chart be interpreted?

A. Incomplete penetrance
B. Autosomal dominant
C. Autosomal recessive
D. Pseudo dominance
----------------------------------------
8. Which of the following options is not used for detecting abnormal gene sequences?

Page 2

268
A. FISH
B. RFLP analysis
C. Pyrosequencing
D. Flow cytometry
----------------------------------------
9. A girl presents with short stature, a webbed neck, thick calves, a shield-shaped chest, swelling of the
ankle, and an increased carrying angle. What is the likely diagnosis?
A. Turner syndrome
B. Down syndrome
C. Patau syndrome
D. Edward syndrome
----------------------------------------
10. The mother of a 12-year-old girl was worried that her daughter is shorter when compared with her
classmates. On examination, the child has a webbed neck, a low hairline, and a shield-shaped chest. A
provisional diagnosis of Turner syndrome was made. Which of the following is a likely finding to be
seen in this patient?
A. Aortic dissection
B. Subluxation of lens
C. Mitral valve prolapse
D. Bilateral small streak ovaries
----------------------------------------
11. All-trans retinoic acid (ATRA) is used in the treatment of tumor that is associated with which of the
following genetic aberrations?
A. PML-RARA
B. BCR-ABL
C. cMYC
D. RUNX1-ETO
----------------------------------------
12. The family pedigree chart is given below. Identify the mode of inheritance of this condition.

Page 3

269
A. Autosomal dominant
B. Autosomal recessive
C. X-linked dominant
D. X-linkedrecessive
----------------------------------------
13. What is the diagnosis of a 14-year-old child who has short stature, widely placed nipples, and
primary amenorrhea?

A. Turner syndrome
B. Down syndrome
C. Edward syndrome
D. Pattau syndrome
----------------------------------------
14. What are the anticipated clinical presentations in patients depicted in the provided karyotyping
image?

A. Gynecomastia and long limbs


B. Flat occiput
C. Webbing of neck and lymphedema
D. Short stature and polydactyly
----------------------------------------
15. Which of the statements below is false regarding stem cells?

Page 4

270
A. They can differentiate
B. They can replicate
C. They can divide without any regulatory stimulus
D. They can regenerate damaged tissues
----------------------------------------
16. In a patient afflicted with color blindness, what is the pattern of inheritance?
A. AD
B. AR
C. XLR
D. XLD
----------------------------------------
17. In myotonic dystrophy, which chromosome is implicated?
A. Chromosome 19
B. Chromosome 20
C. Chromosome 21
D. Chromosome 22
----------------------------------------
18. Which amino acid is necessary for the production of nitric oxide in the blood vessels?
A. Citrulline
B. Arginine
C. Histidine
D. Tryptophan
----------------------------------------
19. t(11,22) is best diagnosed by …………..
A. Conventional karyotyping
B. FISH
C. Bone marrow aspiration
D. Next generation sequencing
----------------------------------------
20. Which disease exhibits an inheritance pattern of X-linked recessive?
A. Hurler Syndrome
B. Thalassemia
C. Hereditary spherocytosis
D. G6PD deficiency
----------------------------------------

Page 5

271
21. With which pattern of disease inheritance is consanguinity associated?
A. Autosomal recessive
B. Autosomal dominant
C. X linked recessive
D. X linked dominant
----------------------------------------
22. An affected male does not have affected children but an affected female always has affected
children. Type of inheritance ?
A. X linked recessive
B. Autosomal recessive
C. X linked dominant
D. Mitochondrial
----------------------------------------
23. Which amongst the following is true about Turner’s syndrome ?
A. Webbing of hands and toes are associated with risk of visceral anomalies
B. Noonan syndrome is more associated with cardiac defects
C. Absence of neck webbing is associated with higher risk of co-arctation of aorta
D. Webbing of neck is associated with higher risk of cardiac anomalies
----------------------------------------
24. Which of the following is true regarding the Mendelian mode of inheritance?
A. 2/3rd of Mendelian disorders are autosomal dominant
B. 1/3rd of Mendelian disorders are autosomal recessive
C. Every 5th patient is X-linked recessive
D. Every 10th patient is Y-linked
----------------------------------------
25. All of the following are features of fragile X syndrome, except:
A. Trinucleotide mutation in FMR-1 gene
B. Mental retardation
C. Microorchidism
D. Large everted ears
----------------------------------------
26. Alpha-1 antitrypsin is the major inhibitor of which enzyme?
A. Catalase
B. Collagenase
C. Elastase
D. Phospholipase

Page 6

272
----------------------------------------
27. Which of the following diseases is most commonly associated with Down syndrome?
A. Parkinson Disease
B. Alzheimer disease
C. Schizophrenia
D. Huntington disease
----------------------------------------

Correct Answers
Question Correct Answer

Question 1 3
Question 2 1
Question 3 1
Question 4 1
Question 5 3
Question 6 3
Question 7 2
Question 8 4
Question 9 1
Question 10 4
Question 11 1
Question 12 1
Question 13 1
Question 14 1
Question 15 3
Question 16 3
Question 17 1
Question 18 2
Question 19 2
Question 20 4
Question 21 1
Question 22 4
Question 23 4
Question 24 4
Question 25 3

Page 7

273
Question 26 3
Question 27 2

Solution for Question 1:


• Autosomal dominance occurs when the gene responsible for the condition is located on one of the
autosomes (non-sex chromosomes), typically on one of the numbered chromosomes (chromosome
pairs 1-22). In this pattern of inheritance, a single copy of the gene is sufficient to express the trait. If the
boy inherited the condition from his grandmother and his father, it indicates that the gene is on one of
the autosomes and is dominant. Therefore, this option seems to be the correct pattern of inheritance
based on the information provided.
Incorrect choices:
• Option a. X-linked dominance: This pattern of inheritance occurs when a gene is located on the X
chromosome, and the presence of a single copy of the gene (in males) or two copies (in females) leads
to the expression of the trait. In this case, the condition is believed to have been passed down from the
grandmother to the father and then to the boy. However, X-linked dominance is more commonly
observed in females who have a higher chance of inheriting the condition because they have two X
chromosomes. Males, who have one X and one Y chromosome, are more likely to express the
condition if they inherit the gene on their X chromosome. Since the boy is affected, it suggests that this
option is not the correct pattern of inheritance.
• Option b. X-linked recessiveness: This pattern of inheritance also occurs when a gene is located on
the X chromosome, but the presence of a single copy of the gene (in males) or two copies (in females)
does not lead to the expression of the trait. Males need to inherit two copies of the recessive gene to
show the condition, while females need to inherit two copies on both X chromosomes. Since the
condition is believed to have been passed down from the grandmother to the father and then to the
boy, it suggests that this could be a possible pattern of inheritance. However, since the boy is affected,
it implies that this option is not the correct pattern of inheritance.
• Option d. Autosomal recessiveness: Autosomal recessiveness occurs when the gene responsible for
the condition is located on one of the autosomes, and the presence of two copies of the gene is
required to express the trait. If this were the correct pattern of inheritance, both the father and mother
would need to be carriers of the gene and pass it on to the boy for him to be affected. However, based
on the information provided, it suggests that the condition was passed down from the grandmother to
the father and then to the boy, indicating that the trait can be expressed in individuals who inherit only
one copy of the gene. Therefore, this option is not the correct pattern of inheritance.

Solution for Question 2:


Answer Option A
• This condition’s inheritance mode in the given family pedigree chart is autosomal dominant.
• This type of inheritance pattern occurs due to one copy of a defective gene which is the mutant gene
in any autosome, and a copy of a normal gene.
• Both males and females have equal chances of inheriting and expressing this trait.
• If one of the parents has an autosomal dominant disorder, there is a 50% chance that the disease
gets passed on to the next generation.

Page 8

274
• Examples are: Adult polycystic kidney Hereditary spherocytosis Marfan’s syndrome
Neurofibromatosis Acute intermittent porphyria Osteogenesis imperfecta Von Willebrand’s disease
• Adult polycystic kidney
• Hereditary spherocytosis
• Marfan’s syndrome
• Neurofibromatosis
• Acute intermittent porphyria
• Osteogenesis imperfecta
• Von Willebrand’s disease
• Adult polycystic kidney
• Hereditary spherocytosis
• Marfan’s syndrome
• Neurofibromatosis
• Acute intermittent porphyria
• Osteogenesis imperfecta
• Von Willebrand’s disease
Incorrect choices:
• Option b. Autosomal recessive: In autosomal recessive inheritance, the individual will inherit the
disease or disorder when both the parents are carriers of the defective gene, while having one normal
gene and one defective gene, hence the individual inherits 2 copies of the defective gene. The chances
of transmission is 25% if both the parents are carriers. A few examples are beta thalassemia, sickle cell
anemia, albinism, wilson’s disease, and xeroderma pigmentosum.
• Option c. X-linked recessive: In X-linked recessive inheritance, the mutated gene present in the X
chromosome is derived from either one of the two in women and/ or from the single X chromosome in
men. The disorder is caused by the recessive gene present in the X chromosome. Since men have only
one X chromosome, they are most commonly affected, whereas females are usually the carriers. A few
examples are hemophilia A, G6PD deficiency, diabetes insipidus, chronic granulomatous disease, color
blindness, and muscular dystrophies.
• Option d. X-linked dominant: In X-linked dominant inheritance, the disease/disorder is caused by a
dominant gene in the X chromosome. A few examples are hypophosphatemic rickets and
iincontinentpigmenti.

Solution for Question 3:


Correct Option A - Rett syndromes:
• Rett syndrome is a rare genetic disorder inherited in an X-linked dominant pattern. This means that
the gene responsible for the disorder is located on the X chromosome, and only one copy of the
mutated gene is required to cause the disorder.
• Rett syndrome primarily affects females, although in rare cases it can affect males as well. It is
caused by mutations in the MECP2 gene, which is responsible for producing a protein that is essential

Page 9

275
for the normal development of the brain. The loss of function of this protein leads to a disruption in the
development of the nervous system, resulting in the characteristic symptoms of the disorder.
• Symptoms of Rett syndrome typically appear around 6-18 months of age and can include a loss of
developmental milestones, a loss of purposeful hand movements, a loss of speech, and the
development of repetitive hand movements. Other features of the syndrome can include breathing
irregularities, seizures, scoliosis, and cognitive impairment.
• Because Rett syndrome is inherited in an X-linked dominant pattern, affected mothers have a 50%
chance of passing the mutated gene to each of their children. However, the severity of the disorder can
vary widely, even among members of the same family. Males who inherit the mutated gene are typically
more severely affected than females, as they have only one X chromosome.
Incorrect Options:
Option B - Duchenne muscular dystrophy: Duchenne muscular dystrophy (DMD) is an inherited disord
er that is caused by mutations in the DMD gene, which is located on the X
chromosome. However, DMD is inherited in an X-linked recessive pattern
Option C - Huntington's disease: This disorder is inherited in an autosomal dominant pattern
Option D - Tuberous sclerosis: Tuberous sclerosis is an inherited disorder that is caused by mutations i
n either the TSC1 or TSC2 gene, which are located on chromosomes 9
and 16, respectively. The disorder is inherited in an autosomal dominant pattern

Solution for Question 4:


Correct Option A: Y-chromosome short arm
• The short arm of the Y-chromosome is referred to as Yp. This is the correct location of the SRY gene.
The SRY gene is found on the Yp11.3 region.
Incorrect options
Option B: Y-chromosome long arm: The long arm of the Y-chromosome is referred to as Yq. The SRY
gene is not located on the long arm of the Y-chromosome.
Option C: X-chromosome short arm: The short arm of the X-chromosome is referred to as Xp. The SR
Y gene is not located on the X-chromosome.
Option D: X-chromosome long arm: The long arm of the X-chromosome is referred to as Xq. The SRY
gene is not located on the X-chromosome.

Solution for Question 5:


Correct Option
Option C: Complement fixation disorders: Complement fixation disorders refer to a group of rare geneti
c disorders that affect the complement system, an important part of the immune system. In individuals
with complement fixation disorders, vaccines are not contraindicated. Vaccines can still be given to pro
vide protection against specific infectious diseases, although the response to vaccines may be diminish
ed in these individuals due to their underlying immune deficiency.
Incorrect options

Page 10

276
Option A: Digeorge syndrome: DiGeorge syndrome is a genetic disorder characterized by the partial or
complete absence of the thymus and impaired immune system function. In individuals with DiGeorge s
yndrome, vaccines may be less effective due to the underlying immune deficiency. However, vaccines
are not contraindicated, and they are still recommended for these individuals to provide whatever level
of protection they can offer.
B: Wiskott-Aldrich syndrome
• Wiskott-Aldrich syndrome is an inherited immunodeficiency disorder characterized by low levels of
platelets, eczema, and recurrent infections. Individuals with Wiskott-Aldrich syndrome have a
compromised immune system, and vaccines may not be as effective in providing immunity.
Option D: Ataxia telangiectasia: Ataxia telangiectasia is a rare genetic disorder characterized by progre
ssive neurodegeneration, immune system dysfunction, and a higher susceptibility to infections. Vaccin
es are generally recommended for individuals with ataxia telangiectasia, as they can help protect again
st vaccine-preventable diseases. However, the immune response to vaccines may be impaired in indivi
duals with ataxia telangiectasia due to their underlying immune deficiency.

Solution for Question 6:


Correct Option C: Centromeric
• The centromeric pattern is characterized by staining around the centromeric region of chromosomes.
This pattern is associated with autoantibodies targeting centromere proteins, particularly centromere
protein B (CENP-B). It is most commonly seen in limited cutaneous systemic sclerosis (lcSSc) or
CREST syndrome (calcinosis, Raynaud's phenomenon, esophageal dysmotility, sclerodactyly, and
telangiectasia).The image shown is a centromeric pattern
Incorrect options
Option A: Nucleolar: The nucleolar pattern refers to the staining of autoantibodies within the nucleoli, w
hich are specialized structures inside the nucleus involved in ribosome biogenesis. This pattern is often
associated with autoantibodies targeting nucleolar antigens, such as fibrillarin, nucleolin, or nucleopho
smin. The nucleolar pattern is frequently seen in autoimmune conditions like systemic sclerosis (sclero
derma) and mixed connective tissue disease.

Page 11

277
Option B: Speckled: The speckled pattern appears as multiple discrete speckles throughout the nucleu
s. It is the most common ANA pattern and can be associated with a variety of autoantibodies targeting
different nuclear antigens. Examples of speckled pattern-related autoantibodies include antibodies agai
nst extractable nuclear antigens (ENA), such as Smith (Sm), ribonucleoprotein (RNP), and Sjögren's sy
ndrome-related antigens (SSA/SSB). The speckled pattern can be found in systemic lupus erythemato
sus (SLE), Sjögren's syndrome, and other autoimmune conditions.

Option D: Diffuse: The diffuse pattern refers to a


uniform distribution of staining throughout the nucleus. It is often a
non-specific finding and does not indicate a specific autoimmune disease or autoantibody. The diffuse
pattern can be seen in various autoimmune diseases, as well as in non-autoimmune conditions.

Solution for Question 7:


Correct Option B: Autosomal dominant
• In this pedigree chart an autosomal dominance pattern is observed, an autosomal dominant trait is
typically observed in every generation, as affected individuals have a 50% chance of passing on the
gene mutation to their offspring. Affected individuals would appear as filled-in or shaded symbols in the
pedigree.
Incorrect options
Option A: Incomplete penetrance: Incomplete penetrance refers to a
situation where an individual carries a gene mutation associated with a
certain trait or condition but does not actually express the trait or condition. In a pedigree chart, this wo
uld be represented by individuals who inherit the gene mutation but do not show the trait or condition. T
hey may appear as unaffected individuals in the pedigree, despite carrying the gene mutation.
Option C: Autosomal recessive: Autosomal recessive inheritance pattern occurs when an individual ne
eds to inherit two copies of a
gene mutation, one from each parent, to express the trait or condition. In a
pedigree chart, autosomal recessive traits often appear in a "skipped generation" pattern. Typically, un
affected parents can have affected offspring if both parents are carriers of the gene mutation. In the pe
digree, unaffected carriers would appear as carriers (heterozygotes) and affected individuals would ap

Page 12

278
pear as filled-in or shaded symbols.
Option D: Pseudo dominance: Pseudo dominance can occur when an autosomal recessive trait appear
s to be inherited in a
dominant pattern due to the presence of consanguinity (intermarriage between close relatives) or a
high frequency of carriers in a population. In a pedigree chart, affected individuals may appear in multip
le generations or have affected siblings, which can resemble an autosomal dominant pattern. However,
if both parents are carriers of the gene mutation, the trait would still follow an autosomal recessive inh
eritance pattern.

Solution for Question 8:


Correct Option D: Flow cytometry: Flow cytometry is a
technique used to analyze and measure certain characteristics of cells or particles in a fluid suspension
. It is commonly used for cell counting, cell sorting, and assessing cell characteristics such as size, gra
nularity, and surface markers. Flow cytometry does not directly detect alterations in DNA sequence.
Option C: Pyrosequencing: Pyrosequencing is a DNA sequencing method that measures the release of
pyrophosphate during DNA synthesis. It allows for the identification of specific nucleotides and can be
used to determine the sequence of DNA.
Option A: FISH (Fluorescence in situ hybridization): FISH is a molecular cytogenetic technique used to
detect and visualize specific DNA sequences on chromosomes or within cells.
Option B: RFLP analysis (Restriction Fragment Length Polymorphism): RFLP analysis is a technique u
sed to detect variations in DNA sequences by digesting DNA samples with specific restriction enzymes
. RFLP analysis can provide information about genetic variations.

Solution for Question 9:


Correct Option A - Turner syndrome:
• Turner syndrome, also known as monosomy X, is a genetic disorder that affects females.
• It occurs when one of the two X chromosomes is partially or completely missing.
• The characteristic features of Turner syndrome include short stature, a webbed neck, thick calves, a
shield-shaped chest, swelling of the ankle, and an increased carrying angle.
Incorrect Options:
Option B - Down syndrome:
• Down syndrome (trisomy 21) is a different genetic disorder caused by the presence of an extra copy
of chromosome 21. It is characterized by distinct facial features and an intellectual disability.
Option C & D - Patau syndrome & Edward syndrome:
• Patau syndrome (trisomy 13) and Edward syndrome (trisomy 18) are both chromosomal disorders
caused by the presence of an extra copy of either chromosome 13 or chromosome 18, respectively.
They are associated with severe developmental abnormalities, intellectual disabilities, and multiple
organ system defects.

Page 13

279
Solution for Question 10:
Correct Option D - Bilateral small streak ovaries:
• This is a hallmark feature of Turner syndrome.
• Due to the absence or underdevelopment of normal ovarian tissue, individuals with Turner syndrome
typically have streak ovaries, which are fibrous structures devoid of functional follicles.
• This leads to ovarian insufficiency and infertility.
Incorrect Options:
Options A, B, and C are not seen in Turner syndrome.

Solution for Question 11:


Correct Option A - PML-RARA:
• ATRA is a medication derived from vitamin A and is used as a differentiation therapy in the treatment
of acute promyelocytic leukemia, a subtype of acute myeloid leukemia (AML).
• APL is characterized by the presence of the PML-RARA fusion gene, resulting from a reciprocal
translocation between chromosomes 15 and 17 [t(15;17)].
• This translocation fuses the promyelocytic leukemia (PML) gene on chromosome 15 with the retinoic
acid receptor alpha (RARA) gene on chromosome 17.
• The PML-RARA fusion protein blocks the normal differentiation of myeloid cells and contributes to
leukemogenesis.
• ATRA acts by binding to the RARA portion of the fusion protein, promoting the differentiation of
leukemic cells into mature granulocytes.
• It helps to restore normal hematopoiesis by inducing the terminal differentiation and apoptosis of
leukemic cells.
Incorrect Options:
• ATRA is not used in the treatment of options B, C, and D.

Solution for Question 12:


Correct Option A - Autosomal dominant:
• There is male-to-male transmission of disease; hence, it should not be X-liked.
• Every generation is affected; hence, it is the dominant pattern.

Page 14

280
Incorrect Options:
Options B, C, and D are incorrect. Refer to the explanation of the correct answer.

Solution for Question 13:


Correct option:
Option A: Turner syndrome
• Turner syndrome, also known as 45,X or monosomy X, is a genetic condition that affects females. It
occurs when one of the two X chromosomes is partially or completely missing. The syndrome can lead
to a variety of physical and developmental features.
The symptoms mentioned in the question are characteristic of Turner syndrome:
Incorrect Options:
Option B: Down syndrome (trisomy 21): Down syndrome is a genetic disorder caused by the presence
of an extra copy of chromosome 21. It is characterized by distinct physical features, intellectual disabilit
y, and various health issues. However, the symptoms mentioned in the question, such as short stature,
widely placed nipples, and primary amenorrhea, are not typical of Down syndrome. Therefore, Down s
yndrome is not the correct diagnosis in this case.
Option C: Edward syndrome (trisomy 18): Edward syndrome is a genetic disorder caused by the prese
nce of an extra copy of chromosome 18. It is associated with multiple congenital abnormalities affectin
g various organ systems. However, the symptoms mentioned in the question, including short stature, w
idely placed nipples, and primary amenorrhea, are not specifically associated with Edward syndrome.
Hence, Edward syndrome is not the correct diagnosis for the presented symptoms.
Option D: Patau syndrome (trisomy 13): Patau syndrome is a genetic disorder caused by the presence
of an extra copy of chromosome 13. It is characterized by severe developmental issues, multiple conge
nital abnormalities, and intellectual disability. However, the symptoms described in the question, such a
s short stature, widely placed nipples, and primary amenorrhea, are not typically associated with Patau
syndrome. Therefore, Patau syndrome is not the correct diagnosis for the given symptoms.

Page 15

281
Solution for Question 14:
The karyotyping image shows an extra copy of the X
chromosome, indicating the presence of Klinefelter syndrome (47, XXY). Klinefelter syndrome is a
chromosomal disorder that occurs in males due to the presence of an additional X
chromosome. The typical karyotype is 47, XXY, but variations can occur. An extra X
chromosome disrupts the normal male sex chromosome complement (46, XY).
Clinical features of Klinefelter syndrome include:
Incorrect Choices:
• Option b. Flat occiput: Flat occiput refers to a flattened back of the head. This characteristic is not
specifically associated with Klinefelter syndrome.
• Option c. Webbing of neck and lymphedema: Webbing of the neck and lymphedema are features
commonly seen in Turner syndrome, a condition characterized by the absence or structural
abnormalities of one of the X chromosomes (45, X).
• Option d. Short stature and polydactyly: Short stature refers to a condition in which an individual's
height is significantly below average for their age and gender. Polydactyly is a congenital condition
characterized by the presence of extra fingers or toes.

Solution for Question 15:


Correct Option C:
Stem cells are undifferentiated cells that have the ability to differentiate into specialized cell types and s
elf-renew through cell division. However, their division and differentiation are tightly regulated by variou
s internal and external signals. Stem cells require specific regulatory stimuli and signals to initiate and c
ontrol their division.
Incorrect Options:
Option A: Stem cells can differentiate: One of the defining characteristics of stem cells is their ability to
differentiate into different cell types. Depending on their potency, stem cells can give rise to a
wide range of specialized cells in the body.
Option B: Stem cells can replicate: Stem cells can undergo self-renewal through replication. This mean
s they can produce identical copies of themselves during cell division, ensuring the maintenance of a
stem cell population.
Option D: Stem cells can regenerate damaged tissues: Stem cells play a crucial role in tissue regenera
tion and repair. They have the ability to differentiate into the specific cell types needed for tissue repair
and can replace damaged or lost cells.

Solution for Question 16:

Page 16

282
Correct Option: C
The mode of inheritance in a
patient suffering from color blindness is typically X-linked recessive (XLR).
Explanation for the options:
Option A: AD (Autosomal Dominant): Autosomal dominant inheritance means that the affected gene is
located on an autosome (non-sex chromosome) and only one copy of the mutated gene is required for
the disorder to be expressed. However, color blindness is primarily inherited in an X-linked recessive m
anner rather than autosomal dominant.
Option B: AR (Autosomal Recessive): Autosomal recessive inheritance means that both copies of the g
ene (one from each parent) must be mutated for the disorder to be expressed. Color blindness is not c
ommonly inherited in an autosomal recessive pattern.
Option C: XLR (X-Linked Recessive): X-linked recessive inheritance means that the affected gene is lo
cated on the X chromosome, and males are more commonly affected than females. In this case, color
blindness is primarily caused by mutations in genes located on the X
chromosome, and the condition is often inherited from carrier mothers to their sons.
Option D: XLD (X-Linked Dominant): X-linked dominant inheritance means that the affected gene is loc
ated on the X chromosome, and both males and females can be affected. However, color blindness is
not typically inherited in an X-linked dominant pattern.
Therefore, the correct answer is XLR (X-Linked Recessive) as it is the most common mode of inheritan
ce for color blindness.

Solution for Question 17:


Correct Option A: Chromosome 19
• Myotonic dystrophy is caused by an expansion of CTG trinucleotide repeats in the DMPK gene, which
is located on chromosome 19.
• This genetic mutation leads to the production of an abnormal form of the myotonic dystrophy protein
kinase (DMPK) and results in the characteristic symptoms of myotonic dystrophy.
Incorrect Options:
Option B: Chromosome 20- Chromosome 20 is not involved in myotonic dystrophy.
Option C: Chromosome 21- Chromosome 21 is not involved in myotonic dystrophy. Chromosome 21 is
associated with conditions such as Down syndrome (trisomy 21) but is not linked to myotonic dystroph
y.
Option D: Chromosome 22- Chromosome 22 is not involved in myotonic dystrophy.

Solution for Question 18:


Correct Option B: Arginine

Page 17

283
• Nitric oxide (NO) is a critical signaling molecule involved in various physiological processes, including
vasodilation in blood vessels. Arginine serves as the precursor for nitric oxide synthesis in the body.
• When arginine is converted to nitric oxide, it undergoes enzymatic conversion by the enzyme nitric
oxide synthase (NOS). This conversion process involves the oxidation of arginine and the release of
nitric oxide.
Incorrect options:
Option A: Citrulline: Citrulline is an amino acid that can be converted to arginine in the body through a
metabolic pathway known as the citrulline-arginine cycle. While citrulline can indirectly contribute to the
production of nitric oxide by being converted to arginine, it is not directly required for the formation of n
itric oxide in blood vessels.
Option C: Histidine: Histidine is an amino acid involved in various physiological processes, including pr
otein synthesis. However, histidine is not directly involved in the synthesis of nitric oxide in blood vesse
ls.
Option D: Tryptophan: Tryptophan is an essential amino acid that serves as a precursor for the synthes
is of various molecules in the body, including serotonin and melatonin. However, it is not directly involv
ed in the formation of nitric oxide in blood vessels.
In summary, arginine is the amino acid required for the formation of nitric oxide in the blood vessels.

Solution for Question 19:


Correct Option: B
• FISH is a molecular cytogenetic technique that uses fluorescent probes to visualize specific DNA
sequences. In the case of t(11,22) translocation, FISH can identify the rearrangement by detecting the
fusion of the genes involved in the translocation. FISH is highly sensitive and specific for detecting
chromosomal abnormalities, even at the level of individual genes.
Incorrect Options:
Option A. Conventional karyotyping: Conventional karyotyping involves examining the chromosomes' s
tructure and number under a microscope. However, it may not provide sufficient resolution to detect sm
all translocations like t(11,22).
Option C. Bone marrow aspiration: Bone marrow aspiration is a procedure to obtain a
sample of bone marrow for examination under a microscope. While it can help diagnose certain conditi
ons, it is not the primary method for detecting chromosomal translocations like t(11,22).
Option D. Next-generation sequencing (NGS): NGS is a powerful sequencing technology that allows th
e analysis of multiple genes and genetic variants simultaneously. It is primarily used for genetic testing,
including the identification of specific mutations and genetic alterations. While NGS can detect genetic
abnormalities, it may not be the most efficient method for diagnosing t(11,22) translocation specifically.

Solution for Question 20:


Correct option: D

Page 18

284
• G6PD deficiency, also known as glucose-6-phosphate dehydrogenase deficiency, is a genetic
disorder characterized by impaired function of the enzyme glucose-6-phosphate dehydrogenase. It
follows an X-linked recessive pattern of inheritance, meaning that the gene mutation is located on the X
chromosome. As a result, males are more commonly affected, while females are carriers of the
condition.
Incorrect options:
Option A: Hurler Syndrome: Hurler Syndrome is an autosomal recessive disorder caused by a deficien
cy of the enzyme alpha-L-iduronidase. It does not follow an X-linked recessive pattern of inheritance.
Option B: Thalassemia: Thalassemia is a group of inherited blood disorders that affect the production o
f hemoglobin. It can be inherited in an autosomal recessive manner, not X-linked recessive.
Option C: Hereditary spherocytosis: Hereditary spherocytosis is an autosomal dominant disorder chara
cterized by abnormalities in red blood cell shape and function. It does not follow an X-linked recessive
pattern of inheritance.

Solution for Question 21:


Correct option: A
• Consanguinity refers to a close blood relationship between the parents, such as being first cousins or
more closely related. Autosomal recessive diseases are more likely to occur in offspring of
consanguineous couples. This is because the parents may both carry a single copy of a recessive gene
mutation, and when they have children together, there is an increased chance of both parents passing
on the mutated gene, resulting in the expression of the disease phenotype in the offspring.
Incorrect options:
Option B: Autosomal dominant: Autosomal dominant diseases can occur in the offspring of consanguin
eous couples, but it is not specifically associated with consanguinity. Autosomal dominant diseases ca
n also arise from de novo mutations or occur in non-consanguineous families.
Option C: X-linked recessive: X-linked recessive diseases are inherited through the X chromosome. Alt
hough consanguinity can increase the likelihood of inheriting X-linked recessive diseases, it is not the c
haracteristic pattern associated with consanguinity.
Option D: X-linked dominant: X-linked dominant diseases can also occur in the offspring of consanguin
eous couples, but they are less common and not specifically associated with consanguinity.

Solution for Question 22:


Correct Option D.
Mitochondrial: mitochondrial inheritance, is the correct answer. Mitochondrial inheritance involves the tr
ansmission of genetic material exclusively through the maternal line via mitochondria. Mitochondrial D
NA is inherited from the mother, and therefore, affected females always pass the mitochondrial gene to
all of their children, resulting in all of their children being affected. However, affected males do not pas
s the mitochondrial gene to their children, and thus they do not have affected children. This inheritance
pattern matches the scenario described.
Incorrect option:

Page 19

285
Option A: X-linked recessive: X-linked recessive inheritance occurs when the gene responsible for the
condition is located on the X chromosome, and the trait is expressed predominantly in males. In X-linke
d recessive inheritance, affected males typically pass the gene to all their daughters (who become carri
ers) but not to their sons. However, affected females have a 50% chance of passing the gene to both t
heir sons and daughters. This inheritance pattern does not match the scenario described, as affected
males do not have affected children.
Option B: Autosomal recessive: Autosomal recessive inheritance occurs when the gene responsible for
the condition is located on an autosome (a non-sex chromosome). In autosomal recessive inheritance,
both parents must carry a copy of the recessive gene to have an affected child. The condition typically
occurs equally in males and females, and unaffected individuals can be carriers of the gene. This inheri
tance pattern does not match the scenario described, as an affected female always has affected childr
en.
Option C: X-linked dominant: X-linked dominant inheritance occurs when the gene responsible for the c
ondition is located on the X chromosome, and the trait is expressed in both males and females. In X-lin
ked dominant inheritance, affected males pass the gene to all their daughters and none of their sons. A
ffected females have a 50% chance of passing the gene to both their sons and daughters. This inherita
nce pattern does not match the scenario described, as affected males do not have affected children.

Solution for Question 23:


Correct Option -D
• This statement is true. In Turner's syndrome, the presence of webbing of the neck, also known as a
"webbed neck," is associated with a higher risk of cardiac anomalies. Webbed neck refers to extra folds
of skin on the sides of the neck, and it can be a characteristic physical feature in individuals with
Turner's syndrome. The presence of a webbed neck may indicate the need for closer monitoring and
evaluation of the heart for potential cardiac abnormalities.
Incorrect Options:
Option A. Webbing of hands and toes are associated with the risk of visceral anomalies:
• This statement is not true for Turner's syndrome. Turner's syndrome is a genetic condition that affects
females and is typically characterized by the absence or underdevelopment of the second sex
chromosome (X chromosome). While individuals with Turner's syndrome may have certain physical
features like short stature and lymphedema (swelling), webbing of hands and toes is not commonly
associated with the condition.
Option B. Noonan syndrome is more associated with cardiac defects:
• Noonan syndrome is a separate genetic disorder characterized by distinct features such as facial
abnormalities, short stature, and developmental delays. While both Turner's syndrome and Noonan
syndrome can have cardiac manifestations, Turner's syndrome is specifically associated with a higher
risk of cardiac anomalies.
Option C. Absence of neck webbing is associated with a higher risk of coarctation of the aorta:
• This statement is not accurate. Neck webbing refers to the presence of excess skin on the neck,
which can be a characteristic feature of Turner's syndrome. However, the presence or absence of neck
webbing is not directly associated with a higher or lower risk of coarctation of the aorta, a congenital
heart defect commonly found in individuals with Turner's syndrome.

Page 20

286
Solution for Question 24:
Correct Option D - Every 10th patient is Y-linked
Incorrect Options: Options A, B, and C are incorrect.

Solution for Question 25:


Correct Option C - Microorchidism:
• Fragile X syndrome is not associated with microorchidism.
• However, it is associated with macroorchidism.
• Macroorchidism becomes more evident during puberty and is a characteristic finding in males with
fragile X syndrome.
Incorrect Options:
Option A - Trinucleotide mutation in FMR-1 gene:
• Fragile X syndrome is caused by an expansion of CGG trinucleotide repeats in the FMR-1 gene.
• This expansion leads to a decreased production of fragile X mental retardation protein (FMRP),
resulting in the characteristic features of the syndrome.
Option B - Mental retardation:
• Intellectual disability is a hallmark feature of fragile X syndrome.
• It can range from mild to severe and is often associated with learning difficulties, speech and
language delays, and behavioural challenges.
Option D - Large everted ears:
• Individuals with fragile X syndrome commonly exhibit large, prominent ears that are described as
everted.

Solution for Question 26:


Correct Option C - Elastase:
• Elastase is an enzyme that breaks down elastin, a protein found in elastic tissues such as the lungs
and blood vessels.
• Alpha-1 antitrypsin is a potent inhibitor of elastase, preventing its excessive activity and protecting
tissues from damage.
Incorrect Options:
Options A, B, and D are incorrect.

Solution for Question 27:

Page 21

287
Correct Option B - Alzheimer disease:
• Alzheimer disease is the most commonly observed in individuals with Down syndrome.
• The accumulation of beta-amyloid plaques and neurofibrillary tangles in the brain, characteristic of
Alzheimer disease, occurs at an earlier age and progresses more rapidly in individuals with Down
syndrome.
Incorrect Options:
Options A, C, and D are incorrect.

Page 22

288
Classification of Neoplasms & Features of Neoplasia
1. Which of the following statements accurately describes a sarcoma?
A. Tumor cells resembling stratified squamous epithelium.
B. Malignant tumors arising in solid mesenchymal tissue.
C. Neoplastic epithelial cells growing in glandular pattern
D. Tumors that contain recognizable mature and immature cells or tissues belonging to more than one
germ layer.
----------------------------------------
2. What is a choristoma in the context of neoplastic pathology?
A. A benign tumor composed of cells from more than one germ cell layer
B. A malignant tumor arising from the salivary glands
C. A tumor containing islands of cartilage or bone within a myxoid stroma
D. A heterotopic rest of cells indigenous to a tissue, often misplaced but considered benign
----------------------------------------
3. Which of the following describes a uterine leiomyoma?
A. Benign tumour of mesenchymal tissue
B. Benign tumour of surface epithelium
C. Malignant tumour of epithelial tissue
D. Malignant tumour of mesenchymal tissue
----------------------------------------
4. Which of the following statements accurately describes an adenoma?
A. Benign epithelial neoplasms arise to form numerous tightly clustered small glands.
B. Benign epithelial neoplasms producing visible fingerlike or warty projections.
C. Tumors that produce papillary patterns that protrude into cystic spaces.
D. Benign or malignant neoplasm produces a macroscopically visible projection above a mucosal
surface and projects into a lumen.
----------------------------------------
5. A 5-year-old boy has a a well-defined non-infiltrating mass within the ventricular wall. Further
investigation demonstrates the presence of myocytes with a characteristic "spider cell" appearance.
Which of the following is the diagnosis of this lesion?
A. Aneurysm
B. Myxoma
C. Rhabdomyoma
D. Lipoma
----------------------------------------

289
6. Pathological examination of an ovarian mass shows a 10cm cystic mass with black hair and
sebaceous substances are discovered in the cystic cavity. A range of benign differentiated tissues,
including skin, cartilage, brain, and mucinous glandular epithelium, are found on histologic inspection of
the cyst wall. What is the diagnosis?
A. Adenoma
B. Chondroma
C. Hamartoma
D. Teratoma
----------------------------------------
7. Which of the following best describes the histologic features of a ‘hamartoma’?
A. Malignant neoplasm of epithelial origin
B. Disorganized proliferation of normal tissue
C. Ectopic islands of normal tissue
D. Granulation tissue
----------------------------------------
8. A 4-year-old child is diagnosed with bilateral Wilm’s tumor. Which of the following accurately
describes the histological features of a Wilm’s tumor?
A. Squamous, fibrous, glandular
B. Blastemal, stromal, epithelial
C. Neuroendocrine, lymphoid, myeloid
D. Hematopoietic, adipose, vascular
----------------------------------------
9. Which of the following statements accurately describes the term desmoplasia?
A. An irregular accumulation of blood vessels
B. Abnormal tissue placed within the normal anatomical location
C. Normal tissue misplaced to abnormal anatomical location
D. The proliferation of non-neoplastic fibrous connective tissue
----------------------------------------
10. Excisional biopsy of a 4cm breast lump shows malignant cells forming solid nests and gland-like
structures surrounded by a dense collagenous stroma are found. A connective tissue stain (trichrome)
of the biopsy is shown in the figure below. Which of the following terms best describes the blue areas
observed in this specimen?

Page 2

290
A. Inflammatory change
B. Anaplastic change
C. Desmoplastic change
D. Dysplastic change
----------------------------------------
11. What is a sentinel lymph node?
A. The first node in a regional lymphatic basin that receives flow from a primary tumour
B. First lymph node to be biopsied in the tumour
C. The lymph node that is positive for malignancy in a tumour
D. The lymph node that is positive for malignancy and more than 2 cm in diameter
----------------------------------------
12. Which of the following is a pre-malignant lesion to esophageal carcinoma?
A. Barrett's esophagus
B. Mallory-Weiss syndrome
C. Boerhaave syndrome
D. Eosinophilic esophagitis
----------------------------------------
13. Which of the following options best describes anaplasia?
A. Reversible replacement of one differentiated cell type with another
B. Presence of morphological and functional resemblance to normal cells
C. Disordered growth of cells and tissues, often observed in epithelial tissues
D. Lack of differentiation characterized by marked pleomorphism, abnormal nuclear morphology, and
high mitotic activity
----------------------------------------
14. An autopsy of a patient treated for serous ovarian cystadenocarcinoma reveals that the peritoneum
is riddled with tiny tumours, as shown in the image below. Which of the following tumour metastatic
routes can explain these autopsy findings?

Page 3

291
A. Seeding of body cavity
B. Hematogenous spread
C. Lymphatic spread
D. None of the above.
----------------------------------------
15. Which of the following is the most common pathway for the initial dissemination of carcinomas?
A. Direct Seeding of Body Cavities
B. Lymphatic Spread
C. Arterial Spread
D. Iatrogenic Spread
----------------------------------------
16. Which of the following is not a common site for a metastatic bone tumor?
A. Skull
B. Hand and feet bones
C. Proximal limb bones
D. Vertebrae
----------------------------------------

Correct Answers
Question Correct Answer

Question 1 2
Question 2 4
Question 3 1
Question 4 1
Question 5 3
Question 6 4
Question 7 2

Page 4

292
Question 8 2
Question 9 4
Question 10 3
Question 11 1
Question 12 1
Question 13 4
Question 14 1
Question 15 2
Question 16 2

Solution for Question 1:


Correct Option B – Malignant tumors arising in solid mesenchymal tissue:
• Sarcomas are malignant tumors that originate from mesenchymal tissues such as bone, cartilage,
muscle, fat, or connective tissues.
Incorrect Options:
Option A – Tumor cells resembling stratified squamous epithelium:
• This description is more indicative of a squamous cell carcinoma rather than a sarcoma. Sarcomas
arise from mesenchymal tissues, such as bone or muscle, and do not typically resemble squamous
epithelium.
Option C – Neoplastic epithelial cells growing in a glandular pattern:
• This description aligns more closely with adenocarcinomas rather than sarcomas. Adenocarcinomas
are a subtype of carcinoma that arise from glandular tissues and exhibit glandular growth patterns,
which are not characteristic of sarcomas.
Option D – Tumors that contain recognizable mature and immature cells or tissues belonging to more t
han one germ layer:
• This description refers to teratomas, which are a type of germ cell tumor containing tissues derived
from more than one germ layer (ectoderm, mesoderm, endoderm).

Solution for Question 2:


Correct Option D –A heterotopic rest of cells indigenous to a
tissue, often misplaced but considered benign:
• Choristomas are characterized as heterotopic rests of cells indigenous to a tissue, often found in
abnormal locations, and considered benign despite their misplaced nature.
Incorrect Options:
Option A – A benign tumor composed of cells from more than one germ cell layer:
• This option describes a teratoma rather than a choristoma. Teratomas are tumors that contain
recognizable cells or tissues from more than one germ cell layer (mesoderm, endoderm, or ectoderm)

Page 5

293
Option B - A malignant tumor arising from the salivary glands:
• This option describes a mixed tumor or pleomorphic adenoma, not a choristoma. Choristomas are
benign lesions, whereas mixed tumors may exhibit both benign and malignant characteristics but are
often considered benign.
Option C - A tumor containing islands of cartilage or bone within a myxoid stroma■■■■■■■:
• This option describes features of a mixed tumor.

Solution for Question 3:


Correct Option A - Benign tumor of mesenchymal tissue:
• A fibroid (leiomyoma) is a benign tumor of the uterus's smooth musculature and is of mesenchymal
origin.
• Painful, Profuse menses and infertility are major complications of this most common tumor of the
female genital tract.
Incorrect Options:
Options B, C, and D are incorrect and the correct option has been explained above.

Solution for Question 4:


Correct Option A –Benign epithelial neoplasms arise to form numerous tightly clustered small glands:
• An adenoma is a benign epithelial neoplasm that typically arises from glandular tissues.
• The key characteristic of an adenoma is the formation of numerous tightly clustered small glands.
• These glands may resemble the normal glandular architecture of the tissue from which the adenoma
arises. The term "adenoma" is derived from "adeno-" meaning gland and "-oma" indicating a tumor or
mass.
Incorrect Options:
Option B – Benign epithelial neoplasms producing visible fingerlike or warty projections:
• This description is more characteristic of papillomas rather than adenomas.
• Papillomas are benign tumors that often grow outward in a finger-like or warty manner and may have
a papillary structure.
Option C - Tumors that produce papillary patterns that protrude into cystic spaces.■■■■■■:
• This description is indicative of papillary neoplasms, which commonly form projections or finger-like
structures extending into cystic spaces.
• Adenomas, while they may exhibit some papillary growth patterns, typically form glandular structures
rather than papillae.
Option D - Benign or malignant neoplasm produces a macroscopically visible projection above a
mucosal surface and projects into a lumen:

Page 6

294
• This description is more consistent with polyps rather than adenomas. Polyps can be benign or
malignant and may project from mucosal surfaces into a lumen, such as in the gastrointestinal tract or
urinary bladder.

Solution for Question 5:


Correct Option C - Rhabdomyoma:
• Rhabdomyomas are benign cardiac tumors composed of myocytes, the muscle cells responsible for
heart contraction.
• They exhibit a characteristic histological appearance known as the "spider cell", where the cells have
a web-like or spider-like appearance under a microscope.

• Rhabdomyomas are often associated with tuberous sclerosis ( TSC1 or TSC2 tumor suppressor
genes, a genetic disorder affecting various organ systems.
• Commonly found in young children, cardiac rhabdomyomas are one of the manifestations of tuberous
sclerosis.
• While they are benign and do not metastasize, their presence in the heart can lead to various cardiac
issues.
• Management may involve medical treatment or surgical intervention, depending on the size and
location of the tumor.
Incorrect Options:
Option A - Aneurysm: Aneurysms are not typically characterized by "spider cell" appearance and are a
ssociated with dilatation of blood vessels.
Option B - Myxoma: Myxomas are benign tumors but originate from connective tissue, not myocytes.
Option D - Lipoma: Lipomas are composed of adipose tissue, not myocytes.

Page 7

295
Solution for Question 6:
Correct Option D - Teratoma:
• Teratomas are benign tumors composed of tissues derived from all three primary germ layers:
ectoderm, mesoderm, and endoderm.
• Teratoma originates from totipotential germ cells that are normally present in the ovary and testis and
sometimes also found in abnormal midline embryonic rests.
• Such cells can differentiate into any of the cell types found in the adult body and so, not surprisingly,
may give rise to neoplasms that contain, in a helter-skelter fashion, bone, epithelium, muscle, fat,
nerve, and other tissues.
• A particularly common pattern is seen in the ovarian cystic teratoma (dermoid cyst), which
differentiates principally along ectodermal lines to create a cystic tumor lined by skin replete with hair,
sebaceous glands, and tooth structures.

Incorrect Options:
Option A - Adenoma:
• An adenoma is a benign tumour of epithelial origin.
• The term adenoma is applied to benign epithelial neoplasms derived from glands, although they may
or may not form glandular structures.
Option B - Chondroma:
• Chondroma is a benign cartilaginous tumour.
• Chondromas are benign tumours of hyaline cartilage that usually occur in bones of enchondral origin.
• They can arise within the medullary cavity, where they are known as enchondromas, or on the surface
of the bone, where they are called juxtacortical chondromas.
• Enchondromas are the most common intraosseous cartilage tumours and are usually diagnosed in
individuals 20 to 50 years of age.
• Typically, they appear as solitary metaphyseal lesions of tubular bones of the hands and feet. The
radiographic features consist of circumscribed lucencies with central irregular calcifications, a sclerotic

Page 8

296
rim and an intact cortex.
Option C - Hamartoma:
• Hamartoma is disorganized normal tissue.
• Hamartomas are disorganized but benign masses composed of cells indigenous to the involved site.
• The term hamartoma refers to an excessive, focal over-growth of cells and tissues native to the organ
in which it occurs.

Solution for Question 7:


Correct Option B - Disorganized normal tissue:
• Hamartomas are disorganized normal tissues.
• The term hamartoma refers to an excessive, focal overgrowth of cells and tissues native to the organ
in which it occurs. Although the cellular elements are mature and identical to those found in the
remainder of the organ, they do not reproduce the normal architecture of the surrounding tissue. The
line of demarcation between a hamartoma and a benign neoplasm is often unclear because both
lesions can be clonal.
Incorrect Options:
Option A - Malignant neoplasm of epithelial origin:
• Hamartoma is a benign tumor.
Option C - Ectopic islands of normal tissue:
• Choriostoma is a heterotropic (misplaced) tissue. Pancreatic tissue in the stomach submucosa.
• Pancreatic tissue in the stomach submucosa.
• Pancreatic tissue in the stomach submucosa.
Option D - Granulation tissue:
• Granulation tissue is made up of proliferating fibroblasts and blood vessels.
• It forms on the wound's surface and aids in the healing process.

Solution for Question 8:


Correct Option B - Blastemal, stromal, epithelial:
• Wilms tumor, the most common renal malignancy in children, presents as a triphasic neoplasm with
blastemal (primitive, undifferentiated cells), stromal (tissue-supporting cells), and epithelial (resembling
kidney tubule lining) elements, as observed on microscopic examination. This histological pattern is
characteristic of Wilms tumor in pediatric patients presenting with renal masses.
• Wilms tumor (Nephroblastoma): As previously discussed, Wilms tumor in the kidneys is characterized
by blastemal (undifferentiated cells), stromal (supportive tissue), and epithelial (resembling kidney
tubule cells) elements.
Incorrect Options:

Page 9

297
Options A, C, and D are incorrect. Refer to the explanation of the correct answer above

Solution for Question 9:


Correct Option D - Proliferation of non-neoplastic fibrous connective tissue:
• Desmoplasia refers to the proliferation of non-neoplastic fibrous connective tissue within a tumor.
• Parenchymal cells stimulate the formation of abundant collagenous stroma, referred to as
desmoplasia.
• Desmoplasia is a process seen in breast cancer, neuroendocrine tumors, lung cancer and pancreatic
ductal adenocarcinoma (PDAC).
Incorrect Options:
Option A - An irregular accumulation of blood vessels:
• An asymmetrical collection of blood vessels is known as a hemangioma.
Option B - Abnormal tissue placed within the normal anatomic location:
• Hemartoma is an abnormal tissue located in its normal anatomical region.
Option C - Normal tissue misplaced to abnormal anatomical location:
• A Choristoma is a misplaced region of tissue within another organ.

Solution for Question 10:


Correct Option C - Desmoplastic change:
• Desmoplasia refers to the growth of benign fibrous tissue secondary to tissue injury in cancer.
• It is the proliferation of non-neoplastic fibrous connective tissue within a tumor.
• Desmoplasia is a process seen in breast cancer, neuroendocrine tumors, lung cancer, and pancreatic
ductal adenocarcinoma (PDAC)
• The blue areas seen in the specimen are best described by desmoplasia.

• A trichrome stain is a histological staining technique used to stain collagen fibers.
• In desmoplastic tissue, the collagenous stroma appears prominently stained with blue dye, contrasting
with the other tissue components. This results in a distinct visualization of the fibrous connective tissue
surrounding the tumor cells.
• The image presented above illustrates how the specimens are observed through H&E; and trichrome
staining..

Page 10

298
Incorrect Options:
Option A - Inflammatory change:
• Inflammatory change refers to the presence of inflammation within the tissue.
• This can involve the infiltration of immune cells, such as lymphocytes and macrophages, as part of the
body's response to injury or infection.
• Inflammatory changes can occur in tumors, but they are characterized by the presence of
inflammatory cells rather than fibrous connective tissue proliferation.
Option B - Anaplastic change:
• Anaplastic change refers to the loss of cellular differentiation and organization, leading to the
development of more primitive and undifferentiated cells within a tumor.
• This can result in features such as pleomorphism, increased nuclear size, and abnormal mitotic
activity.
• Anaplastic change is characteristic of malignant tumors and signifies dedifferentiation.
Option D - Dysplastic change:
• Dysplastic change refers to the abnormal growth, differentiation, or development of cells within a
tissue.
• Dysplasia can range from mild to severe and is often considered a precursor to cancer.
• It is characterized by cellular atypia, disordered growth patterns, and alterations in cell morphology.

Solution for Question 11:


Correct Option A - First node in a regional lymphatic basin that receives flow from a primary tumor:
• The sentinel lymph node is the first regional lymph node to drain lymph from the affected area.
• The sentinel lymph node in breast cancer are the axillary lymph nodes.

Page 11

299
• To make therapeutic decisions, sentinel lymph node biopsies are performed. The malignancies where
SNLB is performed- breast, melanoma, penis, etc
• A sentinel lymph node biopsy can help determine the presence or absence of metastatic disease.
Incorrect Options:
Options B, C, and D are incorrect. Refer to the explanation of Option A.

Solution for Question 12:


Correct Option A - Barrett's esophagus:
• Barrett's esophagus is a chronic complication of gastroesophageal reflux disease(GERD).
• It increases the risk of carcinoma.
• It is a metaplasia caused by chronic acid exposure.
• The normal stratified squamous epithelium of the esophagus is replaced by columnar epithelium with
intestinal characteristics (goblet cells are diagnostic).
• In response to the chronic insult from acid exposure, the esophageal epithelial cells undergo
metaplastic changes, transforming into a cell type better adapted to withstand the acidic environment of
the lower esophagus.
• Morphological features of Barrett's esophagus are Red velvety mucosa extending from the
gastroesophageal junction. Metaplastic mucosa alternate with smooth, pale mucosa.
• Red velvety mucosa extending from the gastroesophageal junction.
• Metaplastic mucosa alternate with smooth, pale mucosa.
• It can be classified as Long segment (>3cm) Short segment (<3cm)
• Long segment (>3cm)
• Short segment (<3cm)
• Patients with short-segment disease have a lower risk of developing carcinoma.
• GERD is more common in people over the age of 40.
• Symptoms of GERD are Sour taste in the mouth Regurgitation Heartburn Dysphagia
• Sour taste in the mouth
• Regurgitation
• Heartburn
• Dysphagia
• Endoscopy is required for diagnosis.
• A proton pump inhibitor is the treatment option for both GERD and Barrett's esophagus.
• Red velvety mucosa extending from the gastroesophageal junction.
• Metaplastic mucosa alternate with smooth, pale mucosa.
• Long segment (>3cm)
• Short segment (<3cm)

Page 12

300
• Sour taste in the mouth
• Regurgitation
• Heartburn
• Dysphagia
Incorrect Options:
Option B - Mallory-Weiss syndrome:
• Mallory Weiss syndrome is caused by forced vomiting and results in linear ulcers on the lower
esophageal sphincter.
• It can be found in alcoholics and bulimics
Option C - Boerhaave syndrome:
• Boerhaave syndrome is a complication of Mallory Weiss syndrome.
• It causes complete oesophageal perforation
Option D - Eosinophilic esophagitis:
• Acute inflammation caused by eosinophil infiltration is known as eosinophilic oesophagitis.
• It has been linked to atopic disease.
• It does not increase the risk of developing carcinoma.

Solution for Question 13:


Correct Option D - Lack of differentiation characterized by marked pleomorphism, abnormal nuclear m
orphology, and high mitotic activity:
• Anaplasia refers to the lack of differentiation observed in malignant tumors.
• It is characterized by marked pleomorphism (variation in cell size and shape), abnormal nuclear
morphology (disproportionately large nuclei, irregular nuclear shape, coarse chromatin), and high
mitotic activity.
• These features indicate a loss of morphological and functional resemblance to normal cells and are
hallmarks of malignancy.
Differentiation VS anaplasia
Differentiation vs. Anaplasia
Differentiation
Refers to how closely neoplastic parenchymal cells resemble normal parenchymal cells, both morpholo
gically and functionally.
Benign tumors are typically well-differentiated.
Examples like lipomas consist of benign adipocytes that closely resemble normal adipocytes, making t
he tumor nearly indistinguishable under microscopic examination.
Well-differentiated benign tumors often have rare mitoses, which are of normal configuration.
In some cases, well-differentiated tumors can exhibit subtle features. For instance, well-differentiated t
hyroid adenocarcinomas form normal-appearing follicles.

Page 13

301
The malignant nature of well-differentiated tumors becomes evident through tissue invasion and the abi
lity to metastasize.
Anaplasia
Represents a
lack of differentiation where neoplastic cells exhibit little to no resemblance to normal tissue cells.
Highly anaplastic, poorly differentiated tumors have a
morphologic appearance strongly predictive of malignant behavior.
Tumors that are moderately well-differentiated fall in between these extremes.

Incorrect Options:
Option A - Reversible replacement of one differentiated cell type with another:
• This option describes metaplasia, not anaplasia. Metaplasia refers to the reversible replacement of
one differentiated cell type with another in response to chronic irritation, inflammation, or injury. It is a
physiological adaptation aimed at better suiting the tissue to its altered environment.
Option B - Presence of morphological and functional resemblance to normal cells:
• This option describes differentiation, not anaplasia. Differentiation refers to the extent to which
neoplastic cells resemble normal cells morphologically and functionally. Well-differentiated tumors
closely resemble their normal counterparts, while anaplastic tumors lack this resemblance.
Option C - Disordered growth of cells and tissues, often observed in epithelial tissues:
• This option describes dysplasia, not anaplasia. Dysplasia is characterized by disordered growth of
cells and tissues, often observed in epithelial tissues. It is recognized by various morphological
changes, including pleomorphism, hyperchromatic nuclei, increased mitotic activity, and architectural
disarray. Dysplasia can be a precursor to invasive cancer.
This option describes dysplasia, not anaplasia. Dysplasia is characterized by disordered growth of cells
and tissues, often observed in epithelial tissues. It is recognized by various morphological changes, in
cluding pleomorphism, hyperchromatic nuclei, increased mitotic activity, and architectural disarray. Dys
plasia can be a precursor to invasive cancer.

Solution for Question 14:


Correct Option A - Seeding of the body cavity:
• The illustration depicts a metastatic carcinoma on the peritoneal surface.
• The presence of tiny tumors riddling the peritoneum at autopsy suggests that seeding of the body
cavity is the most likely explanation for these findings.
• Seeding of the body cavity, also known as peritoneal dissemination, involves the shedding of tumor
cells from the primary site (in this case, the ovary) into the peritoneal cavity, leading to the widespread
dissemination and implantation of tumor cells on the peritoneal surfaces. This route of metastasis is
commonly observed in ovarian cancer, particularly serous ovarian cystadenocarcinoma.
• Appendiceal carcinoma (mucous secreting) or ovarian carcinoma seeding can cause the peritoneal
cavity to fill with gelatinous mass.
• Seeding into the peritoneal cavity can produce symptoms like Ascites Infection Pressure effects

Page 14

302
• Ascites
• Infection
• Pressure effects
• Ascites
• Infection
• Pressure effects
Incorrect Options:
Option B - Hematogenous spread:
• Hematogenous spread involves the dissemination of tumor cells through the bloodstream to distant
organs or tissues.
• In the case of ovarian cancer, hematogenous spread could lead to the formation of metastases in
organs with a rich blood supply, such as the liver, lungs, or bones.
• However, the scenario describes the peritoneum being riddled with tiny tumors, which is more
indicative of seeding of the body cavity rather than hematogenous spread.
Option C - Lymphatic spread:
• The lymphatic spread refers to carcinoma metastasis through lymphatic vessels.
• The lymphatic system is where the majority of sarcoma spreads.
Option D - None of the above:
• The correct option among the given choices is seeding of the body cavity.

Solution for Question 15:


Correct Option B - Lymphatic Spread:
• Transport through lymphatic vessels is the most common pathway for the initial dissemination of
carcinomas.
• Tumors do not contain functional lymphatics, but lymphatic vessels located at the tumor margins are
sufficient for the lymphatic spread of tumor cells. Sentinal Lymph Node (gatekeeper/caretaker) Lymph
node → Tumor cells spread first to these lymph nodes.
Incorrect Options:
Option A - Direct Seeding Of Body Cavities:
• This option refers to the dissemination of tumor cells into natural "open fields" lacking physical
barriers, such as the peritoneal, pleural, or other body cavities.
• While direct seeding of body cavities is a significant mode of spread for certain cancers, such as
ovarian carcinomas or pseudomyxoma peritonei, it is not the most common pathway for the initial
dissemination of carcinomas.
Option C - Arterial Spread:
• Arterial spread involves the dissemination of tumor cells through the bloodstream to distant organs or
tissues.

Page 15

303
• While hematogenous spread is common in some cancers, particularly sarcomas, it is not the most
common pathway for the initial dissemination of carcinomas.
• Carcinomas tend to spread primarily via the lymphatic system, with hematogenous spread occurring
later in the disease process.
Option D - Iatrogenic Spread:
• Iatrogenic spread refers to the unintentional spread of tumor cells during medical procedures, such as
surgery or biopsies.
• While iatrogenic spread can occur, especially if proper precautions are not taken during surgical
procedures, it is not a primary pathway for the initial dissemination of carcinomas.
• Lymphatic spread is the most common mode of initial dissemination for carcinomas.

Solution for Question 16:


Correct Option B - Hand and feet bones:
Hand and feet bone metastases
• Metastasis may occur in any bone but most commonly involve the axial skeleton (e.g. vertebra, pelvis,
ribs, skull, sternum). In the order of frequency, the most common locations include:
• Spine
• Pelvis
• Ribs
• Proximal limb girdles
• Metastases distal to the knee and elbow are extremely uncommon.
• Metastasis in the small bones of the hand and feet is thus uncommon and usually originates in cancer
of the lung, kidney, and colon
Incorrect Options:
Option A - Skull; Option C - Proximal limb bones; Option D: Vertebrae:
• These options are incorrect and the correct answer has been explained above

Page 16

304
Fundamentals of Neoplasia
1. Which protein plays an integral part in mediating cell-to-matrix adhesion?
A. Cadherins
B. Integrins
C. Selectins
D. Calmodulin
----------------------------------------
2. Which of the following phases in a cell cycle does the tumor suppressor gene TP53 normally induce
cell cycle arrest?
A. G2-M phase
B. S-G2 phase
C. G1-S phase
D. G0 phase
----------------------------------------
3. Which of the following cyclins is majorly responsible for G2 to M phase transition
A. Cyclin D
B. Cyclin A
C. Cyclin E
D. Cyclin B
----------------------------------------
4. A 30-year-old male died soon after presenting to the hospital with ARDS. The figure depicts the
vertebral column after the autopsy. What is the most likely diagnosis?

A. Chondrosarcoma
B. Melanoma
C. Multiple myeloma
D. Osteosarcoma
----------------------------------------

305
5. BRAF V600E mutation is commonly found in which of the following malignancies?
A. Hairy cell leukaemia
B. Burkitt lymphoma
C. Chronic myeloid leukaemia
D. Acute myeloid leukaemia
----------------------------------------
6. Which of the following statements regarding the fluorescence in situ hybridization (FISH) method for
detecting gene mutations is correct?
A. FISH uses DNA probes that recognize sequences specific to particular chromosomal regions
B. In the process, clones are labeled with fluorescent dyes and applied to the prophase of the cell cycle
C. Single chromosomes can be detected simultaneously by the introduction of spectral karyotyping
D. Cannot be performed on prenatal samples
----------------------------------------
7. Which of the following causes symptoms such as loss of appetite, fatigue, and loss of weight in
patients with metastatic cancer.
A. Platelet-derived growth factor
B. Fibroblast growth factor
C. Interleukin-2
D. Tumor necrosis factor-α
----------------------------------------
8. Which of the following describes the function of the protein mutated in patients with
Neurofibromatosis type I??
A. Activates the GTPase activity of Ras
B. Cleaves cellular proteins during apoptosis
C. Functions as a cell cycle checkpoint regulator
D. Promotes angiogenesis
----------------------------------------
9. How does the t(9;22) chromosomal translocation play a role in the pathogenesis of chronic
myelogenous leukaemia (CML)?
A. Altered DNA methylation status
B. Enhanced expression of telomerase gene
C. Expansion of a trinucleotide repeat
D. Protooncogene activation
----------------------------------------
10. Which of the following tumor suppressor genes act by inhibiting the Hedgehog signaling pathway?
A. PTCH
B. SDHB

Page 2

306
C. BRCA1
D. CDKN2A
----------------------------------------
11. Which of the following incorrectly describes the purpose of epithelial-mesenchymal transition (EMT)
in cancer cells? Controlled by the transcription factors SNAIL and TWIST Upregulation of E-cadherin
expression Integral to the metastasis of carcinomas Homotypic intercellular adhesion is lost
A. 2 and 3
B. 2 Only
C. 3 Only
D. 1 & 3
----------------------------------------
12. Compared to the normal epithelial cells of the breast lobule, which of the following proteins would
exhibit decreased expression in epithelial cells in a biopsy specimen of lobular carcinoma in situ?
A. Desmin
B. E-cadherin
C. Lysyl hydroxylase
D. P-selectin
----------------------------------------
13. Which of the following methods does a neoplasm use to generate the components required for
proliferation?
A. Warburg effect
B. Glycolysis
C. Kreb Cycle
D. Oxidative phosphorylation
----------------------------------------
14. A 23-year-old female patient is diagnosed with Lynch syndrome. Which of the following
malignancies is this patient at increased risk of?
A. Breast and ovary
B. Pituitary, endometrium and ovary
C. Cardiac and Skin
D. Endometrium and ovary
----------------------------------------
15. Which of the following genetic events best describes the carcinogenesis mechanism in a patient
with retinoblastoma?
A. Balanced translocation
B. Expansion of trinucleotide repeat
C. Gene amplification

Page 3

307
D. Loss of heterozygosity
----------------------------------------
16. Several years after enucleation for retinoblastoma, a patient presents with headaches, nausea, and
ataxia. Imaging studies show a mass in the pineal region of the brain. A biopsy reveals small, blue,
undifferentiated cells arranged around areas of neuropil forming rosette-like structures. What is the
most likely diagnosis based on these findings?
A. Neuroblastoma
B. Medulloblastoma
C. Ependymoma
D. Trilateral retinoblastoma
----------------------------------------
17. A 28-year-old woman came with the following paternal family history. Father had breast cancer at
the age of 35. Aunt had soft tissue sarcoma at 30. Grandmother had a brain cancer at 40. Her cousin
recently passed away from leukemia at the age of 25. Which of the following genetic conditions is likely
responsible for the strong family history of cancer described here?
A. Lynch syndrome
B. BRCA1 mutation
C. Li-Fraumeni syndrome
D. Cowden syndrome
----------------------------------------
18. Which of the following translocations is present in chronic myelogenous leukemia?
A. t(8;14)
B. t(9;22)
C. t(11;22)
D. t(14;18)
----------------------------------------
19. BRCA1 protein promotes which cellular activities in addition to cell cycle control?
A. Apoptosis
B. Cell Adhesion
C. DNA Repair
D. Gene Transcription
----------------------------------------

Correct Answers
Question Correct Answer

Question 1 2
Question 2 3

Page 4

308
Question 3 4
Question 4 2
Question 5 1
Question 6 1
Question 7 4
Question 8 1
Question 9 4
Question 10 1
Question 11 2
Question 12 2
Question 13 1
Question 14 4
Question 15 4
Question 16 4
Question 17 3
Question 18 2
Question 19 3

Solution for Question 1:


Correct Option B- Integrins:
• Integrins mediate both cell-to-cell and cell-to-extracellular matrix adhesion.

• Cell adhesion molecules (CAMs) are proteins located on the cell surface that mediate interactions
between cells and their surrounding environment. CAMs can be broadly classified into several
subtypes: Cadherins Integrins Selectins
• Cadherins
• Integrins
• Selectins
• Integrins: Integrins are heterodimeric transmembrane receptors composed of α and β subunits. They
mediate both cell-cell and cell-matrix interactions by binding to specific ligands, such as ECM proteins
(e.g., fibronectin, collagen) or cell surface receptors (e.g., ICAMs). Integrins are involved in a wide
range of cellular processes, including cell adhesion, migration, proliferation, and signal transduction.
• Cadherins: Cadherins are calcium-dependent transmembrane proteins. They play a crucial role in
cell-to-cell adhesion and tissue organization. Cadherins are categorized into different subtypes,
including E-cadherin, N-cadherin, and P-cadherin, each with specific tissue distribution and functions.
• Selectins: Selectins are a family of calcium-dependent transmembrane proteins that mediate
interactions between leukocytes and endothelial cells during inflammation and immune responses.
Selectins recognize and bind to carbohydrate ligands on cell surfaces, facilitating leukocyte rolling and
adhesion to endothelial cells.

Page 5

309
• Cadherins
• Integrins
• Selectins
Incorrect Options:
Option A, Option C are explained above.
Option D- Calmodulin:
• Calmodulin is a multi-functional intermediate calcium-binding messenger protein exhibited in all
eukaryotic cells.
• The binding of Ca+2 is required to activate calmodulin.

Solution for Question 2:


Correct Option C- G1-S phase:
• The sequence of events that cause cell division is called the cell cycle; it consists of G1 (pre-synthetic
growth), S (DNA synthesis), G2 (pre-mitotic growth), and M (mitotic) phases.
• Cell cycle progression is driven by proteins called cyclins and cyclin-associated enzymes called
cyclin-dependent kinases.
• Surveillance mechanisms primed to sense DNA or chromosomal damage are embedded in the cell
cycle. These quality control checkpoints ensure that cells with genetic imperfections do not complete
replication.
• These quality control checkpoints ensure that cells with genetic imperfections do not complete
replication.
• The G1-S checkpoint monitors the integrity of DNA before irreversibly committing cellular resources to
DNA replication.

Page 6

310
• The p53 protein (guardian of the genome) is the central recorder of stress in the cell and can be
triggered by anoxia, inappropriate signaling by mutated oncoproteins, or DNA damage. p53 controls the
expression and activity of proteins involved in cell cycle arrest, cellular senescence, DNA arrest and
apoptosis. Rapid onset, p53-mediated cell cycle arrest may be considered a primordial response to
DNA damage. It occurs late in the G1 phase and is partly caused by p53-dependent transcription of the
CDKN1A gene, which encodes the CDK inhibitor p21. p21 → inhibition of CDK4/d→ RB gene is not
phosphorylated → block of progression from G1 to S. p53 also helps the process by introducing certain
proteins, such as GDD45 (growth arrest and DNA damage), that enhance DNA repair
• p53 controls the expression and activity of proteins involved in cell cycle arrest, cellular senescence,
DNA arrest and apoptosis.
• Rapid onset, p53-mediated cell cycle arrest may be considered a primordial response to DNA
damage.
• It occurs late in the G1 phase and is partly caused by p53-dependent transcription of the CDKN1A
gene, which encodes the CDK inhibitor p21.
• p21 → inhibition of CDK4/d→ RB gene is not phosphorylated → block of progression from G1 to S.
• p53 also helps the process by introducing certain proteins, such as GDD45 (growth arrest and DNA
damage), that enhance DNA repair
• These quality control checkpoints ensure that cells with genetic imperfections do not complete
replication.
• p53 controls the expression and activity of proteins involved in cell cycle arrest, cellular senescence,
DNA arrest and apoptosis.
• Rapid onset, p53-mediated cell cycle arrest may be considered a primordial response to DNA
damage.
• It occurs late in the G1 phase and is partly caused by p53-dependent transcription of the CDKN1A
gene, which encodes the CDK inhibitor p21.
• p21 → inhibition of CDK4/d→ RB gene is not phosphorylated → block of progression from G1 to S.
• p53 also helps the process by introducing certain proteins, such as GDD45 (growth arrest and DNA
damage), that enhance DNA repair
Incorrect Options:
Option A- G2-M phase:
• The G2 phase is the pre-mitotic growth phase, while M is the mitotic phase.
• The G2-M phase is the checkpoint that prevents cells from entering mitosis when DNA is damaged. It
allows these cells to repair the damaged DNA before propagating genetic defects to the daughter cells.
• The arrest in G2 is mediated by the inhibition of the M-phase promoting factor (MPF).
Option B- S-G2 phase:
• S is the synthesis phase, and G2 is the phase of pre-mitotic growth
• If damage occurs during S-phase or has evaded the G1 checkpoint, the intra-S-phase checkpoint is
activated, blocking further replication.
• The remaining damage is detected by the G2 checkpoint, which prevents cells from entering mitosis.
These last checkpoints are triggered by ATR/Chk1 signalling.
Option D- G0 phase:
• Quiescent cells not actively cycling are said to be in the G0 state.

Page 7

311
• It is the form of a resting state, or quiescence, in which cells reside until they receive appropriate
signals -for example, from growth factors-stimulating them to reenter and progress through the cell
cycle

Solution for Question 3:


Correct Option D- Cyclin B:
• Cell proliferation is fundamental to developing, maintaining steady-state tissue homeostasis, and
replacing dead or damaged cells.
• Cyclin B-CDK1 is essential for the G2-M phase transition
• Cell cycle progression is driven by proteins called cyclins and cyclin-associated enzymes called
cyclin-dependent kinases (CDKs).
• CDKs acquire the ability to phosphorylate protein substrates by forming complexes with the relevant
cyclins.
• Transiently increased synthesis of a particular cyclin leads to increased kinase activity of the
appropriate CDK binding partner; as the CDK completes its round of phosphorylation, the associated
cyclin is degraded, and the CDK activity abates.
• More than 15 cyclins have been identified; cyclins D, E, A, and B appear sequentially during the cell
cycle and bind to one or more CDKs.
Incorrect Options:
Option A- Cyclin D:
• Cyclin D-CDK4 and cyclin D-CDK6 regulate the G1 to S phase.
Option B- Cyclin A:
• Cyclin A-CDK2 and cyclin A-CDK1 are active in the S phase.
Option C - Cyclin E:
• Cyclin E-CDK2 regulate the G1 to S phase along with cyclin D.

Solution for Question 4:


Correct Option B- Melanoma:
• This patient likely has melanoma that has metastasized throughout his body.
• The cause of death may be pulmonary emboli secondary to metastatic melanoma.
• Melanoma is a type of skin cancer caused by UV radiation exposure (UVA & UVB).
• UV radiation causes DNA damage, and mutations accumulate over time.
• The biopsy image depicts melanoma metastasis to the vertebrae.
• Melanocytes produce black pigment.
• Melanoma can also develop in the following locations: Mucosal surface (GI tract, oropharynx)
Oesophagus Meninges Uvea of eye

Page 8

312
• Mucosal surface (GI tract, oropharynx)
• Oesophagus
• Meninges
• Uvea of eye
• Mucosal surface (GI tract, oropharynx)
• Oesophagus
• Meninges
• Uvea of eye
• The common mutations that lead to melanoma are CDKN2A gene mutation P16 is common, which
inhibits CDK4 and helps in uncontrolled melanocyte proliferation. PI3K/AKT pathway mutation
Activating BRAF mutation Loss of PTEN tumour suppressor gene Neurofibromin 1(NF1) gene mutation
TERT gene mutation (reactivation of telomerase)
• CDKN2A gene mutation P16 is common, which inhibits CDK4 and helps in uncontrolled melanocyte
proliferation.
• P16 is common, which inhibits CDK4 and helps in uncontrolled melanocyte proliferation.
• PI3K/AKT pathway mutation
• Activating BRAF mutation
• Loss of PTEN tumour suppressor gene
• Neurofibromin 1(NF1) gene mutation
• TERT gene mutation (reactivation of telomerase)
• CDKN2A gene mutation P16 is common, which inhibits CDK4 and helps in uncontrolled melanocyte
proliferation.
• P16 is common, which inhibits CDK4 and helps in uncontrolled melanocyte proliferation.
• PI3K/AKT pathway mutation
• Activating BRAF mutation
• Loss of PTEN tumour suppressor gene
• Neurofibromin 1(NF1) gene mutation
• TERT gene mutation (reactivation of telomerase)
• P16 is common, which inhibits CDK4 and helps in uncontrolled melanocyte proliferation.
Incorrect Options:
Option A- Chondrosarcoma:
• Chondrosarcoma is a bone tumour.
• It may appear in the spine, but it is not a pigmented tumour.
Option C- Multiple myeloma:
• Multiple myeloma is a type of cancer involving plasma cell proliferation.
• It does not produce pigments that can be seen on a biopsy.
Option D- Osteosarcoma:

Page 9

313
• It is a malignancy that involves long bones.
• Morphologically, it is a grey-white mass with hemorrhagic and cystic degeneration.
• It doesn't appear as a pigmented tumor.

Solution for Question 5:


Correct Option A- Hairy cell leukaemia:
• BRAF is a serine/threonine protein kinase that sits at the top of a cascade of other serine/threonine
kinases of the MAPK family.
• Like activating RAS mutations, activating mutations in BRAF stimulate each of these downstream
kinases and ultimately activate transcription factors.
• Mutations in other MAPK family members downstream of BRAF are uncommon in cancer, suggesting
only mutations affecting factors near the top of the RAS/MAPK cascade produce significant pro-growth
signals in most cell types.
• It is a member of the RAF family and has been detected in close to 100% of hairy cell leukaemias,
more than 60% of melanomas, 80% of benign nevi, and a smaller percentage of a wide variety of other
neoplasms, including colon carcinomas and dendritic cell tumours.

Incorrect Options:
Option B- Burkitt lymphoma:
• The MYC proto-oncogene is expressed in virtually all eukaryotic cells. It belongs to the immediate
early response genes, which are rapidly and transiently induced by RAS/MAPK signaling following
growth factor stimulation of quiescent cells.
• MYC can be considered a master transcriptional regulator of cell growth.
• MYC transcriptions can even reprogram somatic cells to pluripotent stem cells.
Option C- Chronic myelogenous leukemia:

Page 10

314
• In chronic myelogenous leukemia (CML) and some acute lymphoblastic leukemias, the ABL gene is
translocated from its normal abode on chromosome 9 to chromosome 22, where it fuses with the BCR
gene.
• The resultant chimeric gene is oncogenic BCR-ABL tyrosine kinase.
Option D- Acute myeloid leukemia:
• Certain mutated transcription factors, such as a PML-RARA fusion protein, are associated with acute
promyelocytic leukemia.

Solution for Question 6:


Correct Option A- FISH uses DNA probes that recognize sequences specific to particular chromosomal
regions:
• Fluorescence in Situ Hybridization (FISH) uses DNA to detect specific chromosomal regions.
The process of FISH is

• Cloning of 200kb DNA genome


• Labeling with fluorescent dyes
• Applied at metaphase or interphase nuclei (pre-treated)
• The probe that is homologous to a specific genetic sequence at a chromosome can help in
visualization with the help of a fluorescent microscope.
• It can be performed on Prenatal samples Peripheral blood cells Cancer cells Fixed archival tissue
sections
• Prenatal samples
• Peripheral blood cells
• Cancer cells
• Fixed archival tissue sections

Page 11

315
• It helps in the detection of Numeric abnormalities in chromosomes Subtle microdeletions Complex
translocations Gene amplification (HER 2 and NMYC gene)
• Numeric abnormalities in chromosomes
• Subtle microdeletions
• Complex translocations
• Gene amplification (HER 2 and NMYC gene)
• It can be helpful in certain conditions where rapid diagnosis is necessary to make therapeutic
decisions.
• FISH uses DNA probes that recognize sequences specific to particular chromosomal regions.
• Prenatal samples
• Peripheral blood cells
• Cancer cells
• Fixed archival tissue sections
• Numeric abnormalities in chromosomes
• Subtle microdeletions
• Complex translocations
• Gene amplification (HER 2 and NMYC gene)
Incorrect Options:
Option B- In the process, clones are labeled with fluorescent dyes and applied to the prophase of the c
ell cycle:
• Florescent dye labeling is used to detect abnormal genetic sequences.
• It is applied at metaphase, not prophase of the cell cycle.
• This statement is not accurate. In FISH, DNA probes are used to bind to specific sequences of DNA
within cells. It is not dependent on the cell cycle phase, and it does not involve labeling clones.
Option C- Single chromosomes can be detected simultaneously by the introduction of spectral karyotyp
ing:
• FISH can be used to detect single as well as multiple chromosomal mutations.
• It can be used to detect complex translocations.
• Spectral karyotyping (SKY) is a different technique that allows for the visualization of all chromosomes
in a single cell simultaneously. FISH, while it can be used for specific chromosomal regions or genes,
does not inherently detect all chromosomes in one cell.
Option D- Cannot be performed on prenatal samples:
• It can be done on prenatal samples.
• FISH can help in the detection of aneuploidy when done on prenatal samples
• It is commonly used in prenatal testing to detect chromosomal abnormalities, such as Down
syndrome.

Page 12

316
Solution for Question 7:
Correct Option D- Tumor necrosis factor-α:
• Cachexia, or cancer-related wasting, is characterized by weakness, weight loss, anorexia, anaemia,
and infection. Tumour necrosis factor- (TNF-) is the main cytokine that causes these alterations.
• Tumour Necrosis Factor alpha (TNF alpha) is an inflammatory cytokine produced by
macrophages/monocytes during acute inflammation. It is responsible for various cell signaling events,
leading to necrosis or apoptosis. The protein is also important for resistance to infection and cancers.
• TNF alpha is shown to be a common cytokine of chronic infections, cancers, and Hep C
Incorrect Options:
Option A- Platelet-derived growth factor:
• Both fibroblast growth factor and platelet-derived growth factor are involved in wound healing.
• Platelet-derived growth factor help heal wounds and repair damage to blood vessel walls.
• PDGF released by tumor cells can induce the proliferation and migration of vascular endothelial cells
(angiogenesis), smooth muscle cells and tumour cells and inhibit their apoptosis.
Option B- Fibroblast growth factor:
• Both fibroblast growth factor and platelet-derived growth factor are involved in wound healing.
• Fibroblast growth factors (FGFs), a family of cell signaling proteins made by macrophages.
• The fibroblast growth factor (FGF) signaling network plays an important role in normal cell growth,
survival, differentiation, and angiogenesis but has also been implicated in tumor development.
Option C- Interleukin-2:
• Interleukin-2 (IL-2) is an immuno-stimulating cytokine produced by activated T cells.
• IL-2 has an immunoregulatory role; it promotes the growth and development of peripheral immune
cells in initiating the (defensive) immune response and keeps them alive as effector cells.

Solution for Question 8:


Correct Option A- Activates the GTPase activity of Ras:
• The protein mutated in patients with Neurofibromatosis type I (NF1) is neurofibromin.
• Neurofibromin is a tumor suppressor protein that functions as a negative regulator of the Ras
signaling pathway.
• Specifically, it acts as a GTPase-activating protein (GAP) for Ras proteins, promoting the hydrolysis of
GTP to GDP and thereby inactivating Ras.
• Mutations in the NF1 gene lead to loss of neurofibromin function, resulting in dysregulated Ras
signaling, which contributes to the development of neurofibromas and other manifestations of NF1.
Incorrect Options:
Options B, C, and D are incorrect and the correct answer has been explained above

Page 13

317
Solution for Question 9:
Correct Option D- Protooncogene activation:
• The Philadelphia chromosome is the most well-known example of an acquired chromosomal
translocation in human malignancy, appearing in 95% of CML patients.
• The c-abl protooncogene on chromosome 9 is translocated to chromosome 22.
• The c-abl gene and the bcr area form a hybrid oncogene that codes for an abnormal protein with
extremely high tyrosine kinase activity (nonreceptor tyrosine kinase activity) that generates mitogenic
and antiapoptotic signals.
Incorrect Options:
Option A- Altered DNA methylation status:
• DNA methylation is a mechanism for changing the base sequence of DNA without altering its coding
function.
• As a heritable yet reversible epigenetic change, it has the potential to alter gene expression and has
profound developmental and genetic consequences.
• DNA methylation is a frequent method of transcriptional repression that silences genes.
• X-chromosome inactivation, the suppression of repetitive sequences, or genomic imprinting are some
processes strongly associated with DNA methylation.
Option B- Enhanced expression of telomerase gene:
• Cancerous cells divide more often, and their telomeres shorten as a result.
• The cell could perish if its telomeres shorten too much.
• Often, these cells escape death by making more telomerase enzymes, which prevents the telomeres
from getting even shorter.
• Expression of telomerase is usually required for cell immortalization and long-term tumour growth. In
humans, telomerase activity is tightly regulated during development and oncogenesis.
Option C- Expansion of a trinucleotide repeat:
• The mutation, referred to as "trinucleotide repeat (TNR) expansion," occurs when the number of
triplets in a mutated gene exceeds the number found in a normal gene.
• Additionally, the number of triplets in the disease gene continues to increase as the disease gene is
inherited.
• Slippage in DNA replication can cause trinucleotide repeats to grow.
• Therefore, with each cell division, the number of repeats can rise. Expanded repeats can lengthen
and are also inherited.

Solution for Question 10:


Correct Option A- PTCH:
• PTCH gene is responsible for the production of the ‘patched’ protein. It is an inhibitor of hedgehog
signaling and is associated with Gorlin syndrome, basal cell carcinoma, and medulloblastoma.

Page 14

318
• The sonic hedgehog(SHH) signaling pathway controls polarity and the development of the central
nervous system during embryogenesis.
• PTCH (Patched) is a tumor suppressor gene that acts by inhibiting the Hedgehog signaling pathway.
When activated by Hedgehog ligands, PTCH relieves its inhibition on the Smoothened (SMO) receptor,
allowing SMO to initiate downstream signaling events. In its inactive state, PTCH suppresses the
activity of SMO, thereby regulating the Hedgehog pathway. Mutations in PTCH can lead to
dysregulated Hedgehog signaling, contributing to the development of various cancers, including basal
cell carcinoma and medulloblastoma. Therefore, PTCH functions as a tumor suppressor by negatively
regulating the Hedgehog pathway.
Incorrect Options:
Option B- SDHB:
• SDHB mutations are seen in patients with Familial paraganglioma and familial pheochromocytoma.
They are not associated with the Sonic Hedgehog (SHH) signaling pathway.
• They are Inhibitors of cell cycle progression and cause Familial melanoma.
Option C- BRCA1:
• BRCA1 is a well-known tumor suppressor gene that plays a critical role in DNA repair and
maintenance of genomic stability. Mutations in BRCA1 increase the risk of breast, ovarian, and other
cancers.
Option D- CDKN2A:
• CDKN2A (also known as p16INK4a) is another tumor suppressor gene that regulates the cell cycle by
inhibiting cyclin-dependent kinases (CDKs), thus preventing progression from G1 to S phase. Mutations
in CDKN2A are associated with increased susceptibility to melanoma, pancreatic cancer, and other
malignancies.
• However, CDKN2A does not directly inhibit the Hedgehog signaling pathway.

Solution for Question 11:


Correct Option B- 2 Only:
The statement that incorrectly describes the purpose of epithelial-mesenchymal transition (EMT) in can
cer cells is:
b. Upregulation of E-cadherin expression
Explanation of the correct and incorrect options:
• Controlled by the transcription factors SNAIL and TWIST: This is correct. EMT is controlled by
transcription factors like SNAIL, TWIST, and others that suppress epithelial markers and promote the
mesenchymal phenotype.
• Upregulation of E-cadherin expression: This is incorrect. EMT typically involves the downregulation of
E-cadherin expression, which is a hallmark of EMT. E-cadherin is a key epithelial cell adhesion
molecule.
• Integral to the metastasis of carcinomas: This is correct. EMT is indeed integral to the process of
metastasis in carcinomas. It allows cancer cells to acquire a more invasive and migratory phenotype,
which is crucial for metastatic spread.

Page 15

319
• Homotypic intercellular adhesion is lost: This is correct. EMT often results in the loss of homotypic
(like-to-like) intercellular adhesion, as epithelial cells transition to a more individual and migratory
mesenchymal phenotype.

Solution for Question 12:


Correct Option B- E-cadherin:
• Cadherins are transmembrane glycoproteins dependent on Ca2+ that mediate cell-cell adhesion.
• A decreased expression of E-cadherin would aid in loosening the cell adhesions, leading to the
spread of the malignant cells to distant sites.
Incorrect Options:
Option A- Desmin:
• Desmin is an intermediate filament protein found in cells of mesenchymal origin.
Option C- Lysyl hydroxylase:
• Lysyl hydroxylase is involved in the post-translational modification of collagen.
Option D- P-selectin:
• P-selectin is a cell adhesion molecule that mediates the margination of neutrophils during acute
inflammation.

Solution for Question 13:


Correct Option A- Warburg Effect:
• The neoplasm uses the Warburg effect to generate the components required for proliferation. This
metabolic phenomenon involves a shift from oxidative phosphorylation to glycolysis, even in the
presence of oxygen, allowing cancer cells to meet their increased energy demands and biosynthetic
requirements for rapid proliferation
• Noncancerous cells use oxidative phosphorylation and the formation of ATP through it for energy
whereas cancerous cells use glycolysis even in the presence of oxygen.
• The function of PET scan follows the Warburg effect and areas of glycolysis:
• PET Scan Imaging: Positron Emission Tomography (PET) scans utilize radiolabeled glucose (such as
FDG - fluorodeoxyglucose), which is taken up by cells and trapped when phosphorylated. Cancer cells,
with their increased glycolytic activity, uptake more FDG compared to normal cells. Therefore, PET
scans can effectively detect and localize areas of increased glucose metabolism, which often correlate
with cancerous tissues.
Incorrect Options:
Option B- Anaerobic Glycolysis:
• While glycolysis is indeed involved in the Warburg effect, it is not the method exclusively used by
neoplastic cells to generate the components required for proliferation. Glycolysis alone does not fully
account for the metabolic adaptations observed in cancer cells, as they also utilize other metabolic
pathways to support rapid proliferation.

Page 16

320
Option C- Kreb Cycle:
• The Krebs cycle, or citric acid cycle, is a central metabolic pathway that generates energy in the form
of ATP and produces metabolic intermediates. While it is essential for cellular metabolism, neoplastic
cells do not primarily rely on the Krebs cycle to generate components required for proliferation.
Option D - Oxidative phosphorylation:
• Oxidative phosphorylation is the process by which ATP is synthesized using energy derived from the
oxidation of nutrients. While it is an efficient way to produce ATP, neoplastic cells typically exhibit a
preference for glycolysis over oxidative phosphorylation, even in the presence of oxygen, as described
by the Warburg effect.

Solution for Question 14:


Correct Option D- Endometrium and ovary:
• Patients with lynch syndrome are at increased risk of endometrial and ovarian cancers.
Lynch Syndrome:
• Etiology Autosomal dominant disorder caused by a germline mutation in DNA mismatch repair (MMR)
genes (MLH1, MSH2, MSH6, PMS2) or loss of MSH2 expression due to EPCAM gene deletion.
• Autosomal dominant disorder caused by a germline mutation in DNA mismatch repair (MMR) genes
(MLH1, MSH2, MSH6, PMS2) or loss of MSH2 expression due to EPCAM gene deletion.
• Prevalence Most common inherited colorectal cancer (CRC) susceptibility syndrome, accounting for
about 3% of newly diagnosed CRC cases and 2-3% of endometrial cancers.
• Most common inherited colorectal cancer (CRC) susceptibility syndrome, accounting for about 3% of
newly diagnosed CRC cases and 2-3% of endometrial cancers.
• Clinical Clues for Suspecting Lynch Syndrome Suspected in patients with synchronous or
metachronous CRC, CRC or endometrial cancer before 50 years of age, multiple Lynch-associated
cancers, and familial clustering of Lynch-associated cancers.
• Suspected in patients with synchronous or metachronous CRC, CRC or endometrial cancer before 50
years of age, multiple Lynch-associated cancers, and familial clustering of Lynch-associated cancers.
• Tumor Characteristics Tumors in Lynch syndrome typically exhibit microsatellite instability (MSI) and
loss of staining of MMR proteins on immunohistochemistry (IHC) testing. Compared to sporadic CRCs,
Lynch syndrome-associated tumors are more often mucinous, signet ring cell, or medullary histologic
type, poorly differentiated, and may have a brisk lymphocytic infiltrate or are rimmed by a Crohn-like,
germinal center-producing lymphoid reaction.
• Tumors in Lynch syndrome typically exhibit microsatellite instability (MSI) and loss of staining of MMR
proteins on immunohistochemistry (IHC) testing.
• Compared to sporadic CRCs, Lynch syndrome-associated tumors are more often mucinous, signet
ring cell, or medullary histologic type, poorly differentiated, and may have a brisk lymphocytic infiltrate
or are rimmed by a Crohn-like, germinal center-producing lymphoid reaction.
• Evaluation Approach The evaluation for Lynch syndrome varies based on individual and family
history, type of cancer, and age of onset. Ideally, genetic evaluation for Lynch syndrome should begin
with patients who have Lynch syndrome-associated cancer.

Page 17

321
• The evaluation for Lynch syndrome varies based on individual and family history, type of cancer, and
age of onset. Ideally, genetic evaluation for Lynch syndrome should begin with patients who have
Lynch syndrome-associated cancer.
• Autosomal dominant disorder caused by a germline mutation in DNA mismatch repair (MMR) genes
(MLH1, MSH2, MSH6, PMS2) or loss of MSH2 expression due to EPCAM gene deletion.
• Most common inherited colorectal cancer (CRC) susceptibility syndrome, accounting for about 3% of
newly diagnosed CRC cases and 2-3% of endometrial cancers.
• Suspected in patients with synchronous or metachronous CRC, CRC or endometrial cancer before 50
years of age, multiple Lynch-associated cancers, and familial clustering of Lynch-associated cancers.
• Tumors in Lynch syndrome typically exhibit microsatellite instability (MSI) and loss of staining of MMR
proteins on immunohistochemistry (IHC) testing.
• Compared to sporadic CRCs, Lynch syndrome-associated tumors are more often mucinous, signet
ring cell, or medullary histologic type, poorly differentiated, and may have a brisk lymphocytic infiltrate
or are rimmed by a Crohn-like, germinal center-producing lymphoid reaction.
• The evaluation for Lynch syndrome varies based on individual and family history, type of cancer, and
age of onset. Ideally, genetic evaluation for Lynch syndrome should begin with patients who have
Lynch syndrome-associated cancer.
Incorrect Options:
Option A- Breast and ovary:
• Patients with Lynch syndrome are not at increased risk of breast carcinoma.
• Breast carcinoma is common in Le-Fraumeni syndrome (activates p53 gene.
Option B- Pituitary, endometrium and ovary:
• Pituitary carcinoma is not associated with Lynch syndrome.
• However, endometrial and ovarian carcinoma are associated with Lynch syndrome.
Option C- Cardiac and Skin:
• Although a patient with lynch syndrome is at increased risk of skin cancer, cardiac cancers are not
associated with Lynch syndrome.

Solution for Question 15:


Correct Option D- Loss of heterozygosity:
• Retinoblastomas are malignant ocular tumors of young children.
• In hereditary retinoblastoma, an affected child inherits one defective Rb allele and one normal gene;
this heterozygous state is sufficient to prevent the development of retinoblastoma.
• If the remaining normal Rb allele is inactivated by deletion or mutation, the loss of its suppressor
function results in the appearance of a neoplasm.
• Loss of heterozygosity is the term for this genetic process.
Incorrect Options:
Option A- Balanced translocation:

Page 18

322
• Sometimes a section from one chromosome of a particular pair changes places with a section from a
chromosome of another pair.
• When the two breaks do not pass through a gene, and there is no gain or loss of material when the
chromosomes are looked at under a microscope, it is called a balanced translocation.
• When one couple member carries a balanced chromosome translocation, the risk of miscarriage is
approximately doubled.
• One partner has a balanced translocation in 3-5% of couples with recurrent miscarriages.
Option B- Expansion of trinucleotide repeat:
• The mutation, referred to as “trinucleotide repeat (TNR) expansion," occurs when the number of
triplets in a mutated gene exceeds the number found in a normal gene.
• Additionally, the number of triplets in the disease gene continues to increase as the disease gene is
inherited.
• Slippage in DNA replication can cause trinucleotide repeats to grow.
• Therefore, with each cell division, the number of repeats can rise. Expanded repeats can lengthen
and are also inherited.
Option C- Gene amplification:
• Gene amplification refers to various organic and synthetic mechanisms that increase a gene's copy
number "without a commensurate rise in other genes."
• A rise in the gene's copy number. Additionally, there can be an increase in the RNA and protein
produced by that gene.
• An increase in the number of copies of a gene in a genome is called gene amplification.
• In response to signals from other cells or the environment, cancer cells, for instance, occasionally
make extra copies of a gene or genes.

Solution for Question 16:


Correct Option D- Trilateral retinoblastoma:
• Trilateral RB: When a person has bilateral RB and occurs with Pinealoblastoma, then they are said to
have trilateral RB. It is a form of familial RB. ·
• Microscopically: Homer Wright PseudoRosette. Flexner wintersteiner Rosette
Explaining the clinical scenario:
• Leukocoria (white pupil) and retinal detachment indicate retinoblastoma.
• Histopathologic confirmation of retinoblastoma with subsequent enucleation supports the diagnosis.
• Headaches, nausea, and ataxia years later suggest an intracranial tumor.
• Imaging reveals a pineal region mass, common in pineoblastoma (PNET).
• Microscopic exam shows undifferentiated cells around neuropil (Neuropil refers to the complex
network of interwoven nerve fibers, dendrites, and synapses within the central nervous system), typical
of pineoblastoma.
• Therefore, the most likely diagnosis is Trilateral retinoblastoma.

Page 19

323
Incorrect Options:
Option A- Neuroblastoma:
• Neuroblastoma is a type of childhood cancer that typically arises in the sympathetic nervous system,
often in the adrenal glands.
• It typically presents with Abdominal mass, Watery diarrhea syndrome, Horner's syndrome, Skin
bruising and raccoon eyes (periorbital ecchymosis and proptosis) Opsoclonus-myoclonus-ataxia (OMA)
syndrome: Characterized by rapid eye movements, ataxia, and irregular muscle movements.
• Abdominal mass,
• Watery diarrhea syndrome,
• Horner's syndrome,
• Skin bruising and raccoon eyes (periorbital ecchymosis and proptosis)
• Opsoclonus-myoclonus-ataxia (OMA) syndrome: Characterized by rapid eye movements, ataxia, and
irregular muscle movements.
• Histologically: Undifferentiated neuroblasts, Homer-Wright pseudorosettes similar to the findings
described in the case, However, Neuroblastoma typically occurs without a prior history of
retinoblastoma.
• Abdominal mass,
• Watery diarrhea syndrome,
• Horner's syndrome,
• Skin bruising and raccoon eyes (periorbital ecchymosis and proptosis)
• Opsoclonus-myoclonus-ataxia (OMA) syndrome: Characterized by rapid eye movements, ataxia, and
irregular muscle movements.
Option B- Medulloblastoma:
• Medulloblastoma is a malignant brain tumor that predominantly occurs in the cerebellum.
• Common clinical features include symptoms of increased intracranial pressure, such as headaches,
nausea, and ataxia.
• On microscopic examination, medulloblastomas often exhibit small, undifferentiated cells forming
Homer-Wright rosettes, similar to the findings described in the case. However, medulloblastoma
typically occurs without a prior history of retinoblastoma.
Option C- Ependymoma:
• Ependymomas are tumors that arise from the ependymal cells lining the ventricles of the brain and the
central canal of the spinal cord.
• Clinical features can vary depending on the location but may include headaches, nausea, and ataxia,
similar to those described in the case.
• On histopathological examination, ependymomas can display perivascular pseudorosettes, which are
different from Homer-Wright pseudorosettes seen in medulloblastoma but can be confused due to their
rosette-like appearance. Ependymomas can occur in the pineal region, but are not associated with
retinoblastomas.

Page 20

324
Solution for Question 17:
Correct Option C- Li-Fraumeni syndrome:
• Li-Fraumeni syndrome is an autosomal dominant genetic condition caused by TP53 gene mutation.
• It is characterized by a diverse spectrum of cancers, often occurring at a young age, including breast
cancer, sarcomas, brain tumors, leukemia, and others.
• It often presents with a strong family history of multiple early-onset cancers.
Incorrect Options:
Option A- Lynch syndrome:
• Lynch syndrome, also known as hereditary nonpolyposis colorectal cancer (HNPCC), is an autosomal
dominant genetic condition.
• It is associated with a high risk of colorectal cancer and other cancers, such as endometrial, ovarian,
gastric, and urinary tract cancers.
• The age of onset for these cancers is typically earlier than in the general population.
• It is caused by mutations in DNA mismatch repair genes (e.g., MLH1, MSH2) and is characterized by
microsatellite instability in tumors.
Option B- BRCA1 mutation:
• Mutations in the BRCA1 gene are associated with an increased risk of breast, ovarian, and other
cancers.
• Individuals with BRCA1 mutations often have a family history of breast and ovarian cancers.
• Breast cancer risk is significantly elevated, and it often occurs at a younger age. Ovarian cancer risk is
also increased.
• The inheritance is autosomal dominant.
Option D- Cowden syndrome:
• Cowden syndrome is an autosomal dominant genetic condition caused by mutations in the PTEN
tumor suppressor gene.
• It is associated with a higher risk of breast, thyroid, endometrial, and other cancers.
• The syndrome can also manifest with non-cancerous features, such as multiple hamartomas and
macrocephaly.

Solution for Question 18:


Correct Option B- t (9;22):
• The so-called "Philadelphia chromosome" is formed by the translocation t(9;22). This translocation is
seen in patients with chronic myelogenous leukemia.
Incorrect Options:
Option A- t (8;14):
• The translocation t (8;14) is seen in Burkitt lymphoma.
Option C- t (11;22):

Page 21

325
• The translocation t (11;22) is seen in Ewing sarcoma, a relatively uncommon tumour of the bone.
Option D- t (14;18):
• The translocation t (14;18) is seen in follicular lymphoma.

Solution for Question 19:


Correct Option C- DNA Repair:
• The BRCA1 and BRCA2 breast cancer susceptibility genes (BRCA1 and BRCA2) encode tumour
suppressor proteins that play a role in checkpoint functions associated with cell cycle progression into
the S phase.
• The other choices may be aberrant in neoplasia, but they are not predominantly influenced by
BRCA1.
Incorrect Options:
Option A - Apoptosis:
• The BCL-2 protein family is important in controlling apoptosis. It governs apoptosis induction by
controlling caspase activation via a process involving the release of mitochondrial cytochrome c.
Option B- Cell Adhesion:
• Cadherins, selectins, and integrins are all cell adhesion proteins that rely on extracellular Ca2+ (or
Mg2+ for certain integrins). BRCA1 and 2 play no role in this process.
Option D- Gene Transcription:
• A transcription factor (TF) (or sequence-specific DNA-binding factor) is a protein in molecular biology
that regulates the transcription rate of genetic information from DNA to messenger RNA by binding to a
specific DNA sequence. BRCA1 and 2 play no role in this process

Page 22

326
Etiology of Neoplasia
1. A 9-year-old African boy presents with the findings shown below. A biopsy is consistent with a
malignancy that has infiltrated the bone marrow of the jaw. This neoplasm is linked to a virus that
prefers which of the following cells?

A. Chondrocytes
B. Fibroblasts
C. Lymphocytes
D. Macrophages
----------------------------------------
2. An excisional lymph node biopsy reveals Reed-Sternberg cells and the presence of a particular viral
genome. Which of the following viruses is most likely associated with this patient's condition?
A. Herpes simplex
B. EBV
C. CMV
D. Influenza virus
----------------------------------------
3. Which of the following best describes HPV's role in the pathogenesis of cervical neoplasia?
A. Activation of cellular oncogenes
B. Enhanced transcription of telomerase gene
C. Episomal viral replication
D. Inactivation of tumor suppressor proteins
----------------------------------------
4. Which of the following is associated with an increased risk of malignancy?
A. Fibroadenoma of breast
B. Bronchial asthma
C. Chronic ulcerative colitis
D. Leiomyoma of the uterus
----------------------------------------

327
5. Which of the following carcinogens is not associated with angiosarcoma of the liver?
A. Arsenic
B. Vinyl chloride
C. Thorotrast
D. Arylamine
----------------------------------------
6. Which of the following carcinogens is associated with transitional cell carcinoma of the bladder?
A. Aniline dyes
B. Arsenic
C. Benzene
D. Cisplatinum
----------------------------------------
7. A 69-year-old shipyard worker presented to the physician's office complaining of chest discomfort,
malaise, fever, night sweats, and weight loss for the past three months. A chest X-ray showed pleural
effusion. The patient died 6 months later. The lung at autopsy is shown in the image given below.
Which environmental carcinogen is this malignant neoplasm associated with?
(or)
Mesothelioma is associated with which environmental carcinogens?

A. Aflatoxin B1
B. Asbestos
C. Beryllium
D. Radon
----------------------------------------
8. Which of the following is strongly associated with an increased risk of developing skin cancer?
A. Aflatoxin B1
B. Divalent metal cations
C. Aromatic amines and azo dyes
D. Sunlight

Page 2

328
----------------------------------------
9. Which of the following are the high-risk HPV types?
A. 16 and 18
B. 6 and 11
C. 42 and 44
D. 70 and 72
----------------------------------------
10. Which of the following is an incorrect statement regarding HTLV-1?
A. Leukemia develops in only 3 to 5% of infected individuals
B. It has a tropism for CD8+ T cell
C. It has a long latent period of about 40 to 60 years
D. It causes adult T-cell lymphoma/leukaemia
----------------------------------------
11. A patient had a direct smear of urine show the presence of a characteristic egg, as shown in the
image. What can histological type of bladder malignancy is he at risk of?
(or)
Schistosoma hematobium infection is associated with which of the following histological types of
bladder malignancies?

A. Squamous cell carcinoma


B. Adenocarcinoma
C. Small cell carcinoma
D. Transitional cell carcinoma
----------------------------------------

Correct Answers
Question Correct Answer

Question 1 3
Question 2 2

Page 3

329
Question 3 4
Question 4 3
Question 5 4
Question 6 1
Question 7 2
Question 8 4
Question 9 1
Question 10 2
Question 11 1

Solution for Question 1:


Correct Option C - Lymphocytes:
Burkitt's lymphoma:
• Burkitt's lymphoma/leukemia is a rare disease in adults but more common in children.
• It is one of the fastest-growing neoplasms, with a doubling time of <24 h. In general, it is a pediatric
tumor that has three major clinical presentations.
• It is associated with the Ebstein-Barr virus infection.
• The endemic (African) form presents as a jaw or facial bone tumor that spreads to extranodal sites,
including the ovary, testis, kidney, breast, and especially to the bone marrow and meninges.
• The non-endemic form has an abdominal presentation with massive tumor, ascites, renal, testis,
and/or ovarian involvement. Like the endemic form, it also spreads to the bone marrow and CNS.
• Immunodeficiency-related cases more often involve lymph nodes and may present as acute leukemia.
• The gp350/220 glycoprotein, which is the most abundant glycoprotein on the envelope of EBV virions,
binds specifically to CD21/CD35 receptors on the surface of B cells. This interaction mediates the
attachment of EBV to B cells.
• Once bound to CD21/CD35, gp350/220 facilitates the capping of CD21 and subsequent endocytosis
of virions into B cells. This process allows EBV to gain entry into B cells and establish infection.
Incorrect Options:
Option A - Chondrocytes:
• The most likely cause of this patient's presentation is Burkitt's lymphoma.
• Most cases of Burkitt's lymphoma are associated with EBV infection.
• The chondrocytes are not affected; rather, the virus has a preference for the B-lymphocytes.
Option B - Fibroblasts:
• EBV does not affect fibroblast.
• Rather, the cells affected are lymphocytes.
Option D - Macrophages:
• The histology of Burkitt's lymphoma does show tangible body macrophages.

Page 4

330
• However, these cells are not affected by the EBV.
• Rather B-lymphocytes which are affected in Burkitt's lymphoma.

Solution for Question 2:


Correct Option B - EBV:
• The patient's biopsy is concerning for Hodgkin's lymphoma, a lymphoid malignancy characterized by
the presence of Reed-Sternberg cells.
The patient's biopsy is concerning for Hodgkin's lymphoma, a
lymphoid malignancy characterized by the presence of Reed-Sternberg cells.

• It is known that Epstein-Barr virus (EBV) is associated with the development of Hodgkin's lymphoma.
• EBV is a member of the herpesvirus family and has been implicated in the pathogenesis of various
lymphoid malignancies, including a subset of Hodgkin's lymphoma cases. This virus can be detected in
the Reed-Sternberg cells in these cases.
• Other viruses listed in the options are not typically associated with the development of Hodgkin's
lymphoma
It is known that Epstein-Barr virus (EBV) is associated with the development of Hodgkin's lymphoma.
EBV is a member of the herpesvirus family and has been implicated in the pathogenesis of various lym
phoid malignancies, including a subset of Hodgkin's lymphoma cases. This virus can be detected in the
Reed-Sternberg cells in these cases.
Other viruses listed in the options are not typically associated with the development of Hodgkin's lymph
oma
Incorrect Options:
Options A, C, and D are incorrect. Refer to the explanation of the correct answer.

Page 5

331
Solution for Question 3:
Correct Option D - Inactivation of tumor suppressor proteins:
• HPV (Human Papillomavirus) plays a crucial role in the pathogenesis of cervical neoplasia primarily
through the inactivation of tumor suppressor proteins, particularly the p53 and retinoblastoma (Rb)
proteins.
• HPV encodes viral proteins, such as E6 and E7, which interact with and degrade these tumor
suppressor proteins, leading to uncontrolled cell proliferation and neoplastic transformation.
• This disruption of normal cell cycle regulation is a key mechanism by which HPV contributes to the
development of cervical cancer.
Incorrect Options:
Options A, B, and C are incorrect. Refer to the explanation of Option D

Solution for Question 4:


Correct Option C - Chronic ulcerative colitis:
• Certain non-neoplastic disorders are associated with an increased risk of malignancy.
Examples of such lesions are:-
• Chronic atrophic gastritis of pernicious anemia
• Solar keratosis of the skin
• Chronic ulcerative colitis
• Leukoplakia of the oral cavity, vulva, and penis
Chronic atrophic gastritis of pernicious anemia
Solar keratosis of the skin
Chronic ulcerative colitis
Leukoplakia of the oral cavity, vulva, and penis
These conditions have a well-defined association with cancer and are termed precancerous conditions.
• Ulcerative colitis is prone to colitis-associated neoplasia, the most feared long-term complication.
Ulcerative colitis is prone to colitis-associated neoplasia, the most feared long-term complication.
Incorrect Options:
Option A - Fibroadenoma of the breast:
• Fibroadenoma is a well-defined, benign mass and is not considered precancerous
Fibroadenoma is a well-defined, benign mass and is not considered precancerous
Option B - Bronchial asthma:

Page 6

332
• It is a type I hypersensitivity-mediated allergic reaction of the body to foreign environmental
substances triggering an immune response. Such patients show a mixed pattern of acute and chronic
inflammation and repeated bouts of inflammation and fibrosis in later stages.
It is a type I hypersensitivity-mediated allergic reaction of the body to foreign environmental substances
triggering an immune response. Such patients show a mixed pattern of acute and chronic inflammatio
n and repeated bouts of inflammation and fibrosis in later stages.
Option D - Leiomyoma of the uterus:
• Leiomyoma is a benign smooth muscle tumor of the uterus. It is a well-demarcated smooth muscle
mass from the other part of the uterus. Even though non-encapsulated, leiomyoma is not a malignant
tumor
Leiomyoma is a benign smooth muscle tumor of the uterus. It is a well-demarcated smooth muscle ma
ss from the other part of the uterus. Even though non-encapsulated, leiomyoma is not a
malignant tumor

Solution for Question 5:


Correct Option D - Arylamine:
• Angiosarcoma of the liver is strongly associated with exposure to certain carcinogens.
• Arsenic, vinyl chloride, and thorotrast are well-established carcinogens linked to the development of
angiosarcoma of the liver.
• However, arylamines are not typically associated with this particular type of cancer.
• Arylamines are a group of chemicals found in various substances such as tobacco smoke, hair dyes,
and certain industrial chemicals, but they are not known to be causative agents for angiosarcoma of the
liver.
Angiosarcoma of the liver is strongly associated with exposure to certain carcinogens.
Arsenic, vinyl chloride, and thorotrast are well-established carcinogens linked to the development of an
giosarcoma of the liver.
However, arylamines are not typically associated with this particular type of cancer.
Arylamines are a group of chemicals found in various substances such as tobacco smoke, hair dyes, a
nd certain industrial chemicals, but they are not known to be causative agents for angiosarcoma of the
liver.
Incorrect Options:
Option A - Arsenic:
• Arsenic is a well-known carcinogen associated with various cancers, including skin, lung, bladder, and
liver cancers.
• Chronic exposure to arsenic, often through contaminated drinking water or occupational exposure,
has been linked to an increased risk of developing angiosarcoma of the liver.
• Arsenic induces DNA damage and disrupts cellular processes, contributing to carcinogenesis.
Arsenic is a well-known carcinogen associated with various cancers, including skin, lung, bladder, and l
iver cancers.

Page 7

333
Chronic exposure to arsenic, often through contaminated drinking water or occupational exposure, has
been linked to an increased risk of developing angiosarcoma of the liver.
Arsenic induces DNA damage and disrupts cellular processes, contributing to carcinogenesis.
Option B - Vinyl chloride:
• Vinyl chloride is a chemical used in the production of polyvinyl chloride (PVC) plastics.
• Occupational exposure to vinyl chloride, particularly among workers in the plastics industry, has been
strongly associated with an elevated risk of angiosarcoma of the liver.
• Vinyl chloride undergoes metabolic activation in the liver, leading to the formation of reactive
intermediates that can cause DNA damage and promote tumor development.
Vinyl chloride is a chemical used in the production of polyvinyl chloride (PVC) plastics.
Occupational exposure to vinyl chloride, particularly among workers in the plastics industry, has been s
trongly associated with an elevated risk of angiosarcoma of the liver.
Vinyl chloride undergoes metabolic activation in the liver, leading to the formation of reactive intermedi
ates that can cause DNA damage and promote tumor development.
Option C - Thorotrast:
• Thorotrast is a radiographic contrast agent that was previously used for imaging purposes.
• Exposure to thorotrast, particularly through medical procedures such as angiography, has been linked
to an increased risk of angiosarcoma of the liver.
• Thorotrast contains thorium dioxide particles that deposit in liver tissue, where they emit alpha
radiation, leading to chronic tissue damage and the development of liver tumors.
Thorotrast is a radiographic contrast agent that was previously used for imaging purposes.
Exposure to thorotrast, particularly through medical procedures such as angiography, has been linked t
o an increased risk of angiosarcoma of the liver.
Thorotrast contains thorium dioxide particles that deposit in liver tissue, where they emit alpha radiation
, leading to chronic tissue damage and the development of liver tumors.

Solution for Question 6:


Correct Option A - Aniline dyes:
Bladder cancer
Transitional cell carcinoma
• Transitional carcinoma is the most frequent cause of bladder cancer caused by environmental factors
such as smoking, radiation exposure, cyclophosphamide and occupational exposures such as
phenacetin, aniline dyes, and arylamine dyes.
• Bladder cancer often involves: the tumor-suppressor genes on 9p and 9q are deleted, and papillary
tumors are created, which may later develop TP53 mutations and turn into an invasive illness. This
process is known as an acquired genetic mutation.
• These carcinogens are eliminated in the urine and predominantly affect the bladder's transitional
epithelium. The most common symptom of carcinoma bladder is painless hematuria.
• In the liver, these azo dyes are transformed into water-soluble carcinogens.

Page 8

334
Incorrect Options:
Option B - Arsenic:
• Arsenic toxicity is associated with several cancers, including skin, lung, urinary bladder, kidney, liver,
and prostate; the most common cancer is skin cancer, followed by lung cancer.
• The toxicity occurs after long-term ingestion of food (fish, seafood, cereals), air (coal-fired power
generation, smelting), herbicides, and water.
Arsenic toxicity is associated with several cancers, including skin, lung, urinary bladder, kidney, liver, a
nd prostate; the most common cancer is skin cancer, followed by lung cancer.
The toxicity occurs after long-term ingestion of food (fish, seafood, cereals), air (coal-fired power gener
ation, smelting), herbicides, and water.
Option C - Benzene:
• Benzene is a well-known carcinogen associated with various hematological malignancies such as
leukemia, particularly acute myeloid leukemia (AML).
• While exposure to benzene has been linked to bladder cancer in some studies, it is not primarily
associated with transitional cell carcinoma (TCC) of the bladder.
• Benzene exposure occurs primarily in occupational settings such as the chemical industry, petroleum
refining, and rubber manufacturing.
Benzene is a well-known carcinogen associated with various hematological malignancies such as leuk
emia, particularly acute myeloid leukemia (AML).
While exposure to benzene has been linked to bladder cancer in some studies, it is not primarily associ
ated with transitional cell carcinoma (TCC) of the bladder.
Benzene exposure occurs primarily in occupational settings such as the chemical industry, petroleum r
efining, and rubber manufacturing.
Option D - Cisplatinum:
• Cisplatinum, or cisplatin, is a chemotherapy drug used in the treatment of various cancers, including
bladder cancer.
• While cisplatin exposure is associated with potential side effects and toxicities, including kidney
damage and nerve damage, it is not a primary carcinogen linked to the development of transitional cell
carcinoma (TCC) of the bladder.
• Instead, cisplatin is used as a therapeutic agent in the management of bladder cancer and other
malignancies.
Cisplatinum, or cisplatin, is a
chemotherapy drug used in the treatment of various cancers, including bladder cancer.
While cisplatin exposure is associated with potential side effects and toxicities, including kidney damag
e and nerve damage, it is not a
primary carcinogen linked to the development of transitional cell carcinoma (TCC) of the bladder.
Instead, cisplatin is used as a
therapeutic agent in the management of bladder cancer and other malignancies.

Solution for Question 7:

Page 9

335
Correct Option B - Asbestos:
Mesothelioma
• The presentation of the patient and the given image align with the diagnosis of mesothelioma.
• Mesothelioma typically arises from the mesothelial cells lining the pleural cavity, peritoneum, or
pericardium. As the tumor grows, it can disrupt normal fluid dynamics within the pleural space, leading
to the accumulation of fluid, known as a pleural effusion.
• Malignant mesothelioma occurs in association with exposure to airborne asbestos. It is a rare cancer
of mesothelial cells, usually arising in the parietal or visceral pleura and less commonly in the
peritoneum and pericardium.
• Direct asbestos exposure occurs in shipyard workers, miners, and insulators.
• The latency duration for developing malignant mesothelioma is after 25 to 40 years of exposure.
• Once inhaled, asbestos fibres remain in the body and cause a lifetime risk after exposure. These
fibres gather near the mesothelial cell layer, generating reactive oxygen species that cause DNA
damage and mutations. Mutations in the genes involved in DNA repair lead to loss of DNA repair, cell
cycle control and growth factor signalling.
• The patient presents with weight loss, chest discomfort, shortness of breath and fatigue.
The presentation of the patient and the given image align with the diagnosis of mesothelioma.
Mesothelioma typically arises from the mesothelial cells lining the pleural cavity, peritoneum, or pericar
dium. As the tumor grows, it can disrupt normal fluid dynamics within the pleural space, leading to the
accumulation of fluid, known as a pleural effusion.
Malignant mesothelioma occurs in association with exposure to airborne asbestos. It is a rare cancer of
mesothelial cells, usually arising in the parietal or visceral pleura and less commonly in the peritoneum
and pericardium.
Direct asbestos exposure occurs in shipyard workers, miners, and insulators.
The latency duration for developing malignant mesothelioma is after 25 to 40 years of exposure.
Once inhaled, asbestos fibres remain in the body and cause a lifetime risk after exposure. These fibres
gather near the mesothelial cell layer, generating reactive oxygen species that cause DNA damage an
d mutations. Mutations in the genes involved in DNA repair lead to loss of DNA repair, cell cycle control
and growth factor signalling.
The patient presents with weight loss, chest discomfort, shortness of breath and fatigue.
Incorrect Options:
Option A - Aflatoxin B1:
• Hepatocellular carcinoma occurs due to exposure to aflatoxin B1. It causes characteristic mutations in
TP53.
Hepatocellular carcinoma occurs due to exposure to aflatoxin B1. It causes characteristic mutations in
TP53.
Option C - Beryllium:
• Beryllium causes lung cancer due to exposure to missile fuel, space vehicles, and hardeners for
lightweight metal alloys, particularly in aerospace applications and nuclear reactors.
Beryllium causes lung cancer due to exposure to missile fuel, space vehicles, and hardeners for lightw
eight metal alloys, particularly in aerospace applications and nuclear reactors.

Page 10

336
Option D - Radon:
• The exposure to Radon occurs via decaying minerals containing uranium in quarries and underground
mines.
• Exposure to Radon and its decaying products leads to lung carcinoma.
The exposure to Radon occurs via decaying minerals containing uranium in quarries and underground
mines.
Exposure to Radon and its decaying products leads to lung carcinoma.

Solution for Question 8:


Correct Option D - Sunlight:
• Exposure to sunlight, specifically ultraviolet (UV) radiation, is strongly associated with an increased
risk of developing skin cancer. UV radiation from the sun can damage the DNA in skin cells, leading to
mutations that can result in the development of skin cancer over time.
• There is a direct association between total hours of exposure to sunlight and the incidence of Basal
cell carcinoma, as well as melanoma and squamous cell carcinoma.
• The molecular hallmark of basal cell carcinoma is loss-of-function mutations in PTCH1, a tumor
suppressor gene that negatively regulates Hedgehog signaling; therefore, the tumor exhibits the
Hedgehog pathway activation. Increased activation of this pathway leads to activation of genes
downstream that are involved in cell growth and survival leading to malignant transformation of the
lesion.
• PCTH1 mutations occur due to UV-light-induced DNA damage. Mutations in TP53 caused by UV
light-induced damage can also lead to cancer.
Incorrect Options:
Option A - Aflatoxin B1:
• Hepatocellular carcinoma is associated with exposure to aflatoxin B1. It causes characteristic
mutations in TP53.
Option B - Divalent metal cations:
• Divalent metal ions such as Cadmium, Chromium, and Nickel are involved in occupational cancers.
Cadmium and its compounds cause prostate cancer due to exposure to solders, batteries metal
platings, and coatings. Chromium causes lung cancer due to exposure to metal alloys, paints, and
pigments. Nickel compounds cause lung and oropharyngeal cancers due to exposure to nickel plating,
ferrous alloys, ceramics, and batteries.
Option C - Aromatic amines and azodyes:
• The most common form of bladder cancer is transitional carcinoma which is associated with exposure
to aromatic amines, aniline, and arylamine azo dyes.

Solution for Question 9:


Correct Option A - 16 and 18:

Page 11

337
Human papilloma virus (HPV)
• The human papillomavirus is a DNA virus. Some types (e.g. 1, 2, 4, and 7) cause benign squamous
papillomas (warts) in humans.
• Genital warts have low malignant potential and are associated with low-risk HPVs, predominantly
HPV-6 and HPV-11.
• High-risk HPVs (e.g. types 16 and 18) cause several cancers, particularly squamous cell carcinoma of
the cervix and anogenital region. This is due to the oncogenic potential of HPV, which is related to the
products of two viral genes, E6 and E7, each of which promotes oncogenesis.
• E6 binds to and mediates the degradation of p53 and promotes cellular proliferation. E7 has similar
effects and, in addition, speeds up the cell cycle through G1 to the S phase.
Incorrect Options:
Option B - 6 and 11:
• The HPV 6 and 11 variants are low-risk subtypes and mainly cause LSIL and condylomas.
Option C - 42 and 44:
• The HPV 42 and 44 subtypes are low-risk variants of the HPV virus and mainly cause LSIL and
condylomas.
Option D - 70 and 72:
• The HPV 70 and 72 subtypes are low-risk variants mainly involved in condylomas and low-grade
squamous intra-epithelial lesions (LSIL).

Solution for Question 10:


Correct Option B - It has a tropism for CD8+ T-cells:
Human T-cell leukemia virus (HTLV-1)
• Similar to the human immunodeficiency virus, which causes AIDS, HTLV-1 has a tropism for CD4+ T
cells; hence, this subset of T cells is the major target for neoplastic transformation. CD8+ T-cells are
cytotoxic killer cells, and HTLV-1 has more tropism for CD4+ T-cells.
• Leukemia develops in only 3 to 5 % of the infected individuals, typically after a long latent period of 40
to 60 years.
• The virus is transmitted by transferring infected T-cells via sexual intercourse, blood products, or
breastfeeding.
Incorrect Options:
Options A, C, and D
are correct statements regarding HTLV-1. Refer to the explanation of the correct answer

Solution for Question 11:


Correct Option A - Squamous cell carcinoma:
• Schistosoma hematobium infections in endemic areas are an established risk for bladder cancer.

Page 12

338
• 70% of cancers with a history of S. hematobium infection are squamous cell carcinoma.
• Schistosomiasis causes chronic granulomatous cystitis leading to squamous metaplasia of
transitional epithelium and, subsequently, the development of squamous cell carcinoma.
• Diagnosis of carcinoma bladder is made by biopsy. Transitional epithelium has multiple layers and
poor cohesion between cells which causes the shedding of these cells in urine.
Incorrect Options:
Options B, C and D are incorrect

Page 13

339
Previous Year Questions
1. What could be a potential cause of scrotal swelling and ataxia in a 60-year-old patient?
A. Glioblastoma multiforme
B. Hodgkin lymphoma
C. Non – Hodgkin lymphoma
D. Non seminomatous germ cell tumor
----------------------------------------
2. The type of cancer that rarely spreads through the lymphatic system is:
A. Squamous cell carcinoma
B. Merkel cell carcinoma
C. Basal cell carcinoma
D. Malignant melanoma
----------------------------------------
3. Why do neoplastic cells utilise Warburg metabolism?
A. It decreases glucose utilisation by neoplastic cells
B. It forms metabolic intermediates which are needed for cell growth and multiplication
C. It provides more energy in the form of increased ATP production
D. In prevents apoptosis and makes the cancer immortal
----------------------------------------
4. A male patient with hemoptysis. On examination, a hilar mass was present. The histopathological
image is shown below. Immunohistochemistry revealed that the cells were positive for p40. What is the
diagnosis?

A. Squamous cell carcinoma of lung


B. Adenocarcinoma lung
C. Small cell cancer lung
D. Large cell carcinoma lung
----------------------------------------

340
5. Which of the following is true about proto-oncogenes? KIT has decreased expression via the tyrosine
kinase signalling pathway in GIST KRAS mutation is more commonly associated with small-cell lung
cancer ERBB2 is overexpressed in association with breast carcinomas PDGFR-β has suppressed
expression in colorectal cancer
A. 2 only
B. 3 only
C. 1 and 4
D. 2 and 3
----------------------------------------
6. A 7-year-old child presents with fever, erythematous rash, and bone pain. An X-ray of the skull
shows multiple lytic lesions. A skin biopsy shows prominent nuclear grooves with eosinophils, as shown
in the image below. Immunohistochemistry for S100 and CD1a is also shown in the images below.
What would be the diagnosis?

Page 2

341
A. Rosai-Dorfman disease
B. Kimura disease
C. Langerhans cell histiocytosis
D. Juvenile xanthogranuloma
----------------------------------------
7. Match the following cell types with their associated malignancy: 1. Popcorn cell a. Acute
promyelocytic leukemia 2. Faggot cell b. Lymphocyte-predominant Hodgkin’s lymphoma 3. Cerebriform
nuclei c. Burkitt lymphoma 4. Starry sky appearance d. Sezary syndrome
1. Popcorn cell a. Acute promyelocytic leukemia
2. Faggot cell b. Lymphocyte-predominant Hodgkin’s lymphoma
3. Cerebriform nuclei c. Burkitt lymphoma
4. Starry sky appearance d. Sezary syndrome

A. 1-b, 2-a, 3-d, 4-c


B. 1-a, 2-d, 3-c, 4-b
C. 1-b, 2-c, 3-a, 4-c
D. 1-a, 2-b, 3-d, 4-c
----------------------------------------
8. Match the malignancies in column A with the genes in column B: Column A Column B 1.
Pleomorphic adenoma a. Rb 2. Burkitt lymphoma b. nMyc 3. Neuroblastoma c. cMyc 4. Retinoblastoma
d. PLAG 1
Column A Column B
1. Pleomorphic adenoma a. Rb
2. Burkitt lymphoma b. nMyc
3. Neuroblastoma c. cMyc
4. Retinoblastoma d. PLAG 1

A. 1-c, 2-b, 3-d, 4-a


B. 1-b, 2-c, 3-d, 4-a
C. 1-d, 2-c, 3-b, 4-a
D. 1-b, 2-d, 3—c, 4-a
----------------------------------------
9. Which of the following is the most common brain tumor seen in children?
A. Medulloblastoma
B. Ependymal tumors
C. Meningioma
D. Glioma
----------------------------------------
10. Which is the cell of origin of chronic lymphocytic leukaemia / small lymphocytic lymphoma?

Page 3

342
A. Mature B cells
B. Mature T cells
C. Progenitor T cells
D. Naive B cell
----------------------------------------
11. A 5-year-old child who presented with proptosis of one of the eyes was found to have a
desmin-positive tumour. What is the probable diagnosis?
A. Ewing’s sarcoma
B. Embryonal rhabdomyosaroma
C. Leukemia
D. Retinoblastoma
----------------------------------------
12. Which of the following is true? BRCA1 is an oncogene HER2neu is amplified only in a fraction of
breast cancer EGFR+ is seen in non-small cell lung cancer N-MYC is a tumor suppressor gene
A. 1, 2
B. 1, 3
C. 2, 3
D. All of the above
----------------------------------------
13. Which of the following is associated with the cell given below?

A. NHL
B. Kaposi sarcoma
C. HL
D. Infectious mononucleosis
----------------------------------------
14. A 65-year-old male patient with chronic fatigue and lymphadenopathy presents to the outpatient
department. His peripheral smear shows the presence of smudge cells. Which of the following
additional investigations will you do to arrive at a diagnosis?
A. Polymerase chain reaction

Page 4

343
B. Fluorescent in site hybridization
C. Flow cytometry
D. Cytogenetics
----------------------------------------
15. A boy presents with a fever, night sweats, and neck swelling. The biopsy of the swelling showed a
starry sky appearance. What is the most likely genetic abnormality seen in this case?
A. MYC
B. RAS
C. BCR-ABL
D. p53
----------------------------------------
16. The human papillomavirus inactivates the Rb gene by which of the following proteins?
A. E6
B. E7
C. L1
D. L2
----------------------------------------
17. Predominantly osteoblastic secondaries are seen in which of the following?
A. Prostate carcinoma
B. Breast carcinoma
C. Stomach carcinoma
D. Bone carcinoma
----------------------------------------
18. Which of the following enzymes is responsible for the immortality of cancer cells?
A. Telomerase
B. Topoisomerase
C. Helicase
D. RNA polymerase
----------------------------------------
19. Anaplasia refers to:
A. Lack of differentiation
B. Variation in cell size and shape
C. Replacement of one type of cell by other
D. Disordered arrangement of tumor cells
----------------------------------------

Page 5

344
20. Which of the following components is typically not included in immunohistochemistry for acute
lymphoblastic leukemia?
A. Tdt
B. CD1a
C. CD3
D. Cytokeratin
----------------------------------------
21. Which of the following does not fall under the provisional entity in the WHO 2016 classification of
hematolymphoid tumours?
A. Natural Killer cell lymphoblastic leukemia / lymphoma
B. B-lymphoblastic leukemia / lymphoma with hypodipioidy
C. Acute myeloid leukemia with RUNX1 mutation
D. Splenic B-cell lymphoma / leukemia
----------------------------------------
22. Which of the following proto-oncogenes is not implicated in carcinoma of the thyroid??
A. RAS
B. RET
C. BRAF
D. p53
----------------------------------------
23. Which of the following is true about anaplasia?
A. Loss of cohesion between cells
B. Loss of differentiation
C. Change of epithelium type
D. Benign and fully reversible
----------------------------------------
24. Oil Red O stain is used in which of the following?
A. Formalin fixation
B. Frozen section
C. Alcohol fixation
D. Gluteraldehyde fixation
----------------------------------------
25. The mutations / changes that convert proto-oncogene to oncogene include all except:
A. Promoter insertion
B. Enhancer deletion
C. Gene amplification

Page 6

345
D. Point mutation
----------------------------------------
26. NARP syndrome belongs to:
A. Lysosomal storage disorder
B. Mitochondrial function disorder
C. Glycogen storage disorder
D. Golgi body transport disorder
----------------------------------------
27. How would you distinguish between a thymoma and ALL when presented with a mediastinal mass?
A. CD1a
B. CD3
C. S100
D. Cytokeratin
----------------------------------------
28. A 3-year-old boy was brought by his parents with concerns about being increasingly clumsy for the
past 6 months. On examination, there is leukocoria with an absence of the red reflex in the left eye. As
further workup reveals a tumor, the eye is enucleated, and the microscopic appearance is as shown in
the image. Molecular analysis of the tumor indicates a loss of cell cycle control in the tumor cells. What
is the most likely diagnosis?

A. Wilms tumor
B. Retinoblastoma
C. Ependymoma
D. Neuroblastoma
----------------------------------------

Correct Answers
Question Correct Answer

Question 1 3

Page 7

346
Question 2 3
Question 3 2
Question 4 1
Question 5 2
Question 6 3
Question 7 1
Question 8 3
Question 9 1
Question 10 4
Question 11 2
Question 12 3
Question 13 3
Question 14 3
Question 15 1
Question 16 2
Question 17 1
Question 18 1
Question 19 1
Question 20 4
Question 21 4
Question 22 4
Question 23 2
Question 24 2
Question 25 2
Question 26 2
Question 27 4
Question 28 2

Solution for Question 1:


Correct Option C:
Non-Hodgkin lymphoma (NHL) can present with scrotal swelling and ataxia in some cases. NHL is a
type of cancer that originates from lymphocytes, which are a
type of white blood cell involved in the immune system.
The involvement of lymph nodes and other lymphoid tissues by NHL can lead to swelling of the scrotu
m due to lymphatic obstruction or infiltration. This can result in the accumulation of fluid in the scrotum,
causing swelling.

Page 8

347
Ataxia, which refers to a lack of coordination and unsteady movements, can occur as a result of NHL in
volving the central nervous system. NHL can infiltrate the brain or spinal cord, leading to neurological s
ymptoms such as ataxia.
Incorrect Option:
Option A. Glioblastoma multiforme: Glioblastoma multiforme is a type of brain tumor that arises from gli
al cells. It typically does not cause scrotal swelling or ataxia. The symptoms associated with glioblasto
ma multiforme are typically related to the location of the tumor within the brain.
Option B. Hodgkin lymphoma: Hodgkin lymphoma is a specific type of lymphoma characterized by the
presence of Reed-Sternberg cells. While it can involve lymph nodes and other organs, it is less commo
nly associated with scrotal swelling or ataxia.
Option D. Non-seminomatous germ cell tumor: Non-seminomatous germ cell tumors are a type of testi
cular cancer that arise from germ cells in the testicles. While they can cause scrotal swelling, ataxia is
not a typical presentation of this type of tumor.

Solution for Question 2:


Correct Option C.
• Basal cell carcinoma is the most common form of skin cancer, typically originating in the basal cells of
the skin. BCC is usually localized to the site of origin and tends to grow slowly. While it can invade and
damage nearby tissues if left untreated, it has a low potential for metastasis. BCC rarely spreads to
lymph nodes or distant organs through the lymphatic or bloodstream routes, making it less likely to
disseminate through the lymphatic system compared to SCC, MCC, or malignant melanoma.
Incorrect Option:
Option A. Squamous cell carcinoma (SCC):
• Squamous cell carcinoma is a type of skin cancer that arises from squamous cells, which are flat cells
found in the outermost layer of the skin. SCC can occur in various areas of the body, including the skin,
lips, mouth, throat, esophagus, lungs, and other internal organs. It has the potential to metastasize and
spread to nearby lymph nodes and distant sites, making it more likely to spread through the lymphatic
route compared to basal cell carcinoma.
Option B. Merkel cell carcinoma (MCC):
• Merkel cell carcinoma is a rare and aggressive type of skin cancer that develops from Merkel cells,
which are involved in the sensation of touch. MCC typically occurs on sun-exposed areas of the skin,
such as the head, neck, and arms. It has a higher tendency to spread through the lymphatic system,
involving nearby lymph nodes and potentially metastasizing to distant sites.
Option D. Malignant melanoma:
• Malignant melanoma is a type of skin cancer that arises from melanocytes, the pigment-producing
cells of the skin. It is known for its ability to metastasize and spread rapidly. Malignant melanoma has a
high potential to invade lymphatic vessels and lymph nodes, leading to lymphatic metastasis. It can
also spread through the bloodstream to distant organs, such as the liver, lungs, brain, and bones. Early
detection and treatment are crucial for improving outcomes in cases of malignant melanoma.

Page 9

348
Solution for Question 3:
Correct Option B - It forms metabolic intermediates which are needed for cell growth and multiplication:
• Neoplastic cells utilise a metabolic pathway known as the Warburg effect, which involves the
conversion of glucose to lactate even in the presence of sufficient oxygen, resulting in lower ATP
production than in normal cells.
• This phenomenon is also known as aerobic glycolysis.
• Neoplastic cells use this pathway as it generates metabolic intermediates such as pyruvate and
lactate, which are required for the biosynthesis of nucleotides, amino acids, and lipids, which are
essential for cell growth and division.
Incorrect Options:
Option A - It decreases glucose utilisation by neoplastic cells:
• Warburg metabolism increases glucose utilisation by neoplastic cells.
Option C - It provides more energy in the form of increased ATP production:
• Warburg metabolism produces less ATP than oxidative phosphorylation.
Option D - In prevents apoptosis and makes the cancer immortal:
• Warburg metabolism is not the direct cause of preventing apoptosis.
• While altered metabolism in cancer cells can contribute to their survival and resistance to apoptosis,
there are other factors involved in these processes, including genetic mutations, dysregulation of
signalling pathways, and interactions with the tumor microenvironment.

Solution for Question 4:


Correct Option A - Squamous cell carcinoma of lung:
• The patient's presentation with hemoptysis and a hilar mass showing keratin pearls on histopathology
and cells showing p40 positivity on immunohistochemistry is suggestive of squamous cell carcinoma of
lung.
Incorrect Options:
• Options B, C, and D are incorrect.

Solution for Question 5:


Correct Option B - 3 only : ERBB2 is overexpressed in association with breast carcinomas:
• ERBB2, also known as HER2/neu, is a proto-oncogene that encodes a receptor tyrosine kinase.
• Amplification or overexpression of the ERBB2 gene is observed in approximately 20-30% of breast
carcinomas, leading to increased ERBB2 protein levels.
• Overexpression of ERBB2 is associated with more aggressive tumor behavior and is a target for
specific therapies, such as trastuzumab.
Incorrect Options:

Page 10

349
Option A - 2 only:
• KRAS mutation is more commonly associated with small-cell lung cancer: KRAS mutations are more
commonly associated with non-small cell lung cancer.
Option C - 1 and 4:
• KIT has decreased expression via the tyrosine kinase signalling pathway in GIST: Gastrointestinal
stromal tumors (GIST) are associated with activating mutations in the KIT gene, which encodes a
receptor tyrosine kinase. These mutations lead to increased expression and constitutive activation of
KIT, contributing to the development of GIST.
• Gastrointestinal stromal tumors (GIST) are associated with activating mutations in the KIT gene,
which encodes a receptor tyrosine kinase.
• These mutations lead to increased expression and constitutive activation of KIT, contributing to the
development of GIST.
• PDGFR-β has suppressed expression in colorectal cancer: In colorectal cancer, overexpression or
activation of PDGFR-β has been observed.
• Gastrointestinal stromal tumors (GIST) are associated with activating mutations in the KIT gene,
which encodes a receptor tyrosine kinase.
• These mutations lead to increased expression and constitutive activation of KIT, contributing to the
development of GIST.
Option D - 2 and 3:
• ERBB2 is overexpressed in association with breast carcinomas: ERBB2, also known as HER2/neu, is
a proto-oncogene that encodes a receptor tyrosine kinase. Amplification or overexpression of the
ERBB2 gene is observed in approximately 20-30% of breast carcinomas, leading to increased ERBB2
protein levels. Overexpression of ERBB2 is associated with more aggressive tumor behavior and is a
target for specific therapies, such as trastuzumab.
• ERBB2, also known as HER2/neu, is a proto-oncogene that encodes a receptor tyrosine kinase.
• Amplification or overexpression of the ERBB2 gene is observed in approximately 20-30% of breast
carcinomas, leading to increased ERBB2 protein levels.
• Overexpression of ERBB2 is associated with more aggressive tumor behavior and is a target for
specific therapies, such as trastuzumab.
• KRAS mutation is more commonly associated with small-cell lung cancer: KRAS mutations are more
commonly associated with non-small cell lung cancer
• ERBB2, also known as HER2/neu, is a proto-oncogene that encodes a receptor tyrosine kinase.
• Amplification or overexpression of the ERBB2 gene is observed in approximately 20-30% of breast
carcinomas, leading to increased ERBB2 protein levels.
• Overexpression of ERBB2 is associated with more aggressive tumor behavior and is a target for
specific therapies, such as trastuzumab.

Solution for Question 6:


Correct Option C - Langerhans cell histiocytosis:

Page 11

350
• The clinical presentation of fever, erythematous rash, and bone pain in a 7-year-old child with multiple
lytic lesions on X-ray and skin biopsy showing prominent nuclear grooves and eosinophils and
immunohistochemistry for S100 and CD1a suggests Langerhans cell histiocytosis.
• Langerhans cell histiocytosis is a rare disorder characterized by the accumulation and proliferation of
Langerhans cells, a type of dendritic cell.
• The disease can affect any age group but is most commonly seen in children between the ages of 1
and 15 years.
• The most common sites of involvement are bone, skin, lymph nodes, and lungs.
Incorrect Options:
Option A - Rosai-Dorfman disease:
• Rosai-Dorfman disease is a rare disorder characterized by the overproduction of non-Langerhans
histiocytes.
• The disease most commonly affects lymph nodes.
• The clinical presentation of Rosai-Dorfman disease includes painless lymphadenopathy, fever, and
night sweats.
Option B - Kimura disease:
• Kimura disease is a rare inflammatory disorder that affects the lymph nodes and soft tissues of the
head and neck.
• The disease is most commonly seen in young Asian males.
• The clinical presentation of Kimura disease includes painless swelling of the lymph nodes and soft
tissues, along with elevated levels of eosinophils in the blood.
Option D - Juvenile xanthogranuloma:
• Juvenile xanthogranuloma is a rare disorder characterized by the overproduction of histiocytes.
• The disease most commonly affects children under the age of 5.
• The clinical presentation of juvenile xanthogranuloma includes firm, dome-shaped papules or nodules
on the skin.

Solution for Question 7:


Correct Option A - 1-b, 2-a, 3-d, 4-c:
Incorrect Options:
• Options B, C, and D are incorrect.

Solution for Question 8:


Correct Option C - 1-d, 2-c, 3-b, 4-a:
Incorrect Options:
• Options A, B, and D are incorrect.

Page 12

351
Solution for Question 9:
Correct option A: Medulloblastoma:
• Medulloblastoma is the most common brain tumors seen in children.
• It typically occurs in the cerebellum, which is the part of the brain responsible for coordination and
balance.
• Medulloblastomas are fast-growing tumors that often present with symptoms such as headaches,
vomiting, unsteadiness, and problems with motor coordination.
• Treatment typically involves a combination of surgery, radiation therapy, and chemotherapy.
Incorrect options: Options B, C, and D are incorrect.

Solution for Question 10:


Correct Option D - Naive B cell:
• CLL/SLL is characterized by the accumulation of abnormal, mature B lymphocytes in the peripheral
blood, bone marrow, and lymphoid tissues.
• These abnormal B cells are derived from naive B cells, which are B cells that have not encountered
an antigen and have not undergone differentiation into plasma cells or memory B cells.
• The abnormal B cells in CLL/SLL typically express CD5, CD19, CD20, and CD23 markers.
• The accumulation of these abnormal B cells leads to the clinical manifestations of CLL/SLL.
Incorrect Options:
• Options A, B, and C are incorrect.

Solution for Question 11:


Correct Option B - Embryonal rhabdomyosaroma:
• The probable diagnosis for a 5-year-old child presenting with proptosis and a desmin-positive tumor is
embryonal rhabdomyosarcoma.
• Rhabdomyosarcoma is a malignant tumor derived from skeletal muscle cells.
• Proptosis is a common presentation in orbital rhabdomyosarcoma due to the space-occupying effect
of the tumor.
• Desmin is an intermediate filament protein that is expressed in muscle cells, including
rhabdomyosarcoma cells.
Incorrect Options:
• Options A, C, and D are incorrect.

Page 13

352
Solution for Question 12:
Correct Option C - 2, 3:
• HER2neu is amplified only in a fraction of breast cancer: In breast cancer, amplification of the HER2
gene and overexpression of the HER2 protein can occur. HER2 amplification is not observed in all
breast cancers but only in a fraction of cases. Breast cancers that show HER2 amplification are
referred to as HER2-positive breast cancers and can be targeted with specific therapies like
trastuzumab.
• In breast cancer, amplification of the HER2 gene and overexpression of the HER2 protein can occur.
• HER2 amplification is not observed in all breast cancers but only in a fraction of cases.
• Breast cancers that show HER2 amplification are referred to as HER2-positive breast cancers and
can be targeted with specific therapies like trastuzumab.
• EGFR+ is seen in non-small cell lung cancer: In non-small cell lung cancer, mutations or
overexpression of the EGFR gene can occur. EGFR-targeted therapies, such as tyrosine kinase
inhibitors, are used in the treatment of EGFR-positive NSCLC.
• In non-small cell lung cancer, mutations or overexpression of the EGFR gene can occur.
• EGFR-targeted therapies, such as tyrosine kinase inhibitors, are used in the treatment of
EGFR-positive NSCLC.
• In breast cancer, amplification of the HER2 gene and overexpression of the HER2 protein can occur.
• HER2 amplification is not observed in all breast cancers but only in a fraction of cases.
• Breast cancers that show HER2 amplification are referred to as HER2-positive breast cancers and
can be targeted with specific therapies like trastuzumab.
• In non-small cell lung cancer, mutations or overexpression of the EGFR gene can occur.
• EGFR-targeted therapies, such as tyrosine kinase inhibitors, are used in the treatment of
EGFR-positive NSCLC.
Incorrect Options:
Options A, B, and D are incorrect.
• BRCA1 is an oncogene: BRCA1 is a tumor suppressor gene.
• N-MYC is a tumor suppressor gene: N-MYC is an oncogene.

Solution for Question 13:


Correct Option C - HL:
• The image shows a Reed-Sternberg cell.
• They have distinct morphological features, including a bilobed or multinucleated appearance,
prominent nucleoli, and abundant cytoplasm.
• Hodgkin lymphoma is a type of lymphoma characterized by the presence of Reed-Sternberg cells
within the lymph nodes.

Page 14

353
• It commonly affects young adults and presents with painless lymphadenopathy, often in the cervical
region.
Incorrect Options:
• Options A, B, and D are not associated with Reed-Sternberg cells.

Solution for Question 14:


Correct Option C - Flow cytometry:
• The presence of smudge cells in the peripheral smear suggests chronic lymphocytic leukemia (CLL).
• Flow cytometry is a technique that analyzes the physical and chemical characteristics of cells using
fluorescently-labelled antibodies.
• It allows for the identification and quantification of different cell populations based on their surface
markers.
• In CLL, flow cytometry can provide important diagnostic information by analyzing the
immunophenotype of the abnormal lymphocytes.
Incorrect Options:
Option A - Polymerase chain reaction:
• PCR is a molecular technique used to amplify specific DNA sequences.
• While PCR-based assays may have a role in the diagnosis and monitoring of certain hematological
malignancies, they are not the initial investigation of choice for CLL.
Option B - Fluorescent in situ hybridization:
• FISH is a cytogenetic technique used to detect and visualize specific DNA sequences within cells.
• It is commonly used to detect genetic abnormalities, including chromosomal translocations, deletions,
and amplifications.
• While FISH can be helpful in the diagnosis and prognostication of CLL, it is not the initial investigation
of choice.
Option D - Cytogenetics:
• Cytogenetics involves the study of chromosomes and their abnormalities using various techniques,
such as karyotyping and fluorescent microscopy.
• While cytogenetic analysis can be useful in CLL, it is not the initial investigation of choice.

Solution for Question 15:


Correct Option A - MYC:
• The presence of a starry sky appearance on biopsy suggests Burkitt lymphoma.
• The starry sky appearance in the biopsy is due to the presence of macrophages that have engulfed
apoptotic tumor cells, creating a pattern reminiscent of stars in the night sky.
• Burkitt lymphoma is characterized by the translocation of the MYC gene.

Page 15

354
• The MYC gene is located on chromosome 8 and encodes a transcription factor involved in the
regulation of cell growth and proliferation.
• In Burkitt lymphoma, there is a characteristic translocation involving the MYC gene, most commonly
the t(8;14)(q24;q32) translocation.
• This translocation juxtaposes the MYC gene with the enhancer region of the immunoglobulin heavy
chain gene (IgH), resulting in dysregulated MYC expression.
• The dysregulated MYC expression leads to uncontrolled cell growth and division, contributing to the
development of Burkitt lymphoma.
Incorrect Options:
• Options B, C, and D are incorrect.

Solution for Question 16:


Correct Option B - E7:
• The human papillomavirus (HPV) inactivates the retinoblastoma (RB) gene primarily through the
action of the E7 protein.
• The E7 protein of high-risk HPV types, such as HPV-16 and HPV-18, is crucial in promoting cell cycle
progression and inhibiting cell cycle checkpoints.
• E7 interacts with the p105Rb (retinoblastoma protein), encoded by the Rb gene, leading to its
inactivation.
• By binding to p105RB, E7 disrupts the normal interaction between p105RB and E2F, a transcription
factor involved in cell cycle control.
• This disruption results in the release of E2F, allowing it to activate genes required for cell cycle
progression, including genes involved in DNA replication.
• Consequently, the cell cycle becomes dysregulated, and uncontrolled cell proliferation can occur, a
hallmark of HPV-associated cancers.
Incorrect Options:
Option A - E6: The E6 protein of HPV primarily targets and degrades the tumor suppressor protein p53
, rather than inactivating the RB gene.
Option C - L1: The L1 protein is the major capsid protein of HPV and plays a
role in the virus particle assembly.
Option D - L2: The L2 protein is a
capsid protein of HPV, which assists in entering the viral DNA into the host cell during infection.

Solution for Question 17:


Correct Option A - Prostate carcinoma:
• Predominantly osteoblastic secondaries refer to the presence of bone metastases in which there is an
excessive production of bone by osteoblasts, leading to increased bone density.

Page 16

355
• Prostate carcinoma commonly metastasizes to bone, and the resulting bone metastases are typically
osteoblastic in nature.
Incorrect Options:
Option B - Breast carcinoma: Breast carcinoma can also metastasize to bone, but the bone lesions are
often mixed with areas of osteolytic and osteoblastic activity.
Option C - Stomach carcinoma: Stomach carcinoma rarely metastasizes to bone, and when it does, th
e bone lesions are usually osteolytic rather than predominantly osteoblastic.
Option D - Bone carcinoma: Bone metastases can arise from various primary cancers, but the osteobla
stic or osteolytic nature of the bone lesions depends on the primary tumor type.

Solution for Question 18:


Correct Option A - Telomerase:
• Telomerase is an enzyme that is responsible for maintaining the length of telomeres, which are
protective structures at the ends of chromosomes.
• Telomeres shorten with each cell division, and their shortening is associated with cellular aging and
eventual cell death.
• However, telomerase can extend the length of telomeres, allowing cells to divide indefinitely.
• In cancer cells, telomerase is often overexpressed, leading to continuous cell division and immortality.
Incorrect Options:
Option B - Topoisomerase:
• Topoisomerases are responsible for manipulating the structure of DNA by inducing and resolving
DNA strand breaks.
Option C - Helicase:
• Helicases unwind the DNA double helix, allowing other enzymes to access and replicate the DNA.
Option D - RNA polymerase:
• RNA polymerase is an enzyme responsible for synthesizing RNA from a DNA template during
transcription.
• It is essential for gene expression and protein synthesis.
• While dysregulation of RNA polymerase activity can occur in cancer, it is not directly associated with
the immortality of cancer cells.

Solution for Question 19:


Correct Option A - Lack of differentiation:
• Anaplasia is characterized by the loss of normal cellular differentiation, i.e., the tumor cells do not
resemble the parent tissue cells they originated from.
Incorrect Options:

Page 17

356
Option B - Variation in cell size and shape: Pleomorphism refers to the presence of cells with different
sizes and shapes within a tumor.
Option C - Replacement of one type of cell by another: Metaplasia refers to the reversible change of on
e mature cell type to another mature cell type in response to certain stimuli or chronic irritation.
Option D - Disordered arrangement of tumor cells: The disordered arrangement of tumor cells is known
as architectural disorder or loss of normal tissue architecture, which is often observed in malignant tu
mors.

Solution for Question 20:


Correct Option D - Cytokeratin:
• Cytokeratins are a group of proteins that are primarily found in epithelial cells, which line the surfaces
and cavities of the body.
• They are not specific markers for lymphoid cells and are not typically used in the diagnosis of acute
lymphoblastic leukemia.
Incorrect Options:
Option A - TdT:
• Tdt is an enzyme that is typically present in the early stages of lymphoid cell development.
• It is not found in mature lymphocytes but is expressed in lymphoblasts, which are the immature
precursor cells in ALL.
• Tdt is commonly used as a marker to identify and confirm the presence of lymphoblasts in ALL.
• Tdt-positive staining indicates the presence of immature lymphoid cells, which is a characteristic
feature of ALL.
Option B - CD1a:
• CD1a is a cell surface marker that is predominantly expressed on Langerhans cells, which are
specialized immune cells found in the skin and mucosal tissues.
• In the context of immunohistochemistry for ALL, CD1a is used to differentiate T-cell ALL (which is
CD1a-positive) from B-cell ALL (which is CD1a-negative).
Option C - CD3:
• CD3 is a complex of proteins that is present on the surface of mature T lymphocytes.
• In the context of immunohistochemistry for ALL, CD3 is used to detect T-cell ALL, as the malignant
cells in this subtype express CD3 on their surface.
• CD3-positive staining helps confirm the T-cell lineage of the leukemia cells.

Solution for Question 21:


Correct Option D - Splenic B-cell lymphoma / leukemia:
• In the WHO 2016 classification of hematolymphoid tumors, provisional entity refers to a category that
includes certain rare or less well-characterized entities where further research and clinical experience

Page 18

357
are needed to determine their precise classification and clinical significance.
• This is not included as a provisional entity. It has its own classification within the WHO classification of
lymphoid neoplasms.
Incorrect Options:
• Option A, B, and C are included as a provisional entity in the WHO 2016 classification of
hematolymphoid tumors.

Solution for Question 22:


Correct Option D - p53:
• p53 is not implicated in carcinoma of the thyroid.
Incorrect Options:
Option A - RAS: Mutations in the RAS proto-oncogene, particularly the HRAS and KRAS isoforms, are
found in thyroid cancer.
Option B - RET: Mutations in RET are associated with multiple endocrine neoplasia type 2
(MEN2) syndromes, which include medullary thyroid carcinoma.
Option C - BRAF: Mutations in the BRAF proto-oncogene, particularly the V600E mutation, are found i
n papillary thyroid carcinoma.

Solution for Question 23:


Correct Option B - Loss of differentiation:
• Anaplasia refers to the loss of differentiation of cells.
• It is a characteristic feature of malignant tumors and is associated with the loss of normal tissue
organization and cellular features.
• Anaplastic cells often exhibit pleomorphism, hyperchromatic nuclei, an increased
nuclear-to-cytoplasmic ratio, and increased mitotic activity.
Incorrect Options:
Option A - Loss of cohesion between cells: Loss of cohesion between cells is a
characteristic feature of malignancy.
Option C - Change of epithelium type: Metaplasia refers to the reversible change of one mature cell typ
e to another mature cell type in response to certain stimuli or chronic irritation.
Option D
- Benign and fully reversible: Anaplasia is typically seen in malignant tumors and is not fully reversible.

Solution for Question 24:


Correct Option B - Frozen Section:

Page 19

358
• Oil Red O stain is commonly used in frozen sections to detect the presence of lipids, particularly
neutral lipids such as triglycerides.
• Frozen sections are typically used for rapid intraoperative diagnosis or for research purposes.
• Oil Red O is a lipophilic dye that selectively stains lipids within cells or tissues.
• It binds to the neutral lipid droplets and produces a bright red color when examined under a
microscope.
• This stain is particularly useful in identifying lipid accumulation in various tissues and organs, including
adipose tissue, liver, muscle, and atherosclerotic plaques.
Incorrect Options:
Option A - Formalin fixation:
• Formalin fixation refers to the preservation of tissue samples using formaldehyde solutions.
• Formalin fixation leads to the cross-linking of proteins and modification of lipid structures, which may
interfere with the staining properties of Oil Red O.
Option C - Alcohol fixation:
• Alcohol fixation involves the preservation of tissues using alcohol-based solutions, such as ethanol or
methanol.
• Alcohol fixation can also alter the structure of lipids, affecting the staining properties of Oil Red O.
Option D - Gluteraldehyde fixation:
• Gluteraldehyde fixation is typically used for electron microscopy studies.
• Gluteraldehyde fixation may lead to changes in lipid composition and affect the staining properties of
Oil Red O.

Solution for Question 25:


Correct Option B - Enhancer deletion:
• Enhancer deletion is not a mechanism that directly converts a proto-oncogene to an oncogene.
• Enhancers are regulatory DNA sequences that enhance the transcription of genes.
• While enhancer mutations can contribute to gene dysregulation, they do not directly convert a
proto-oncogene to an oncogene.
Incorrect Options:
Option A - Promoter insertion:
• Promoter insertion is a genetic alteration that can convert a proto-oncogene to an oncogene.
• When a promoter sequence is inserted near a proto-oncogene, it can lead to increased and
unregulated expression of the gene, promoting cell growth and division.
Option C - Gene amplification:
• Gene amplification refers to an increase in the number of copies of a specific gene.
• If a proto-oncogene undergoes amplification, it results in an increased dosage of the gene product,
leading to abnormal cell proliferation and potentially converting it to an oncogene.

Page 20

359
Option D - Point mutation:
• Point mutations, which are single nucleotide changes in the DNA sequence, can convert a
proto-oncogene to an oncogene.
• Point mutations can result in altered protein function, increased protein stability, or constitutive
activation of signalling pathways, promoting uncontrolled cell growth and division.

Solution for Question 26:


Correct Option B - Mitochondrial function disorder:
• NARP syndrome is caused by mutations in the mitochondrial DNA, specifically in the ATPase 6 gene.
• Mitochondrial function disorders involve defects in the mitochondria, which are responsible for
producing energy in the form of ATP through oxidative phosphorylation.
Incorrect Options:
Options A, C, and D are incorrect.

Solution for Question 27:


Correct Option D - Cytokeratin:
• Cytokeratins are intermediate filaments found in epithelial cells.
• Thymomas are tumors of thymic epithelial cells and express cytokeratin markers.
• ALL is a lymphoid malignancy and does not express cytokeratins.
Incorrect Options:
• Options A, B, and C are incorrect.

Solution for Question 28:


Correct Option B - Retinoblastoma:
• Retinoblastoma is the most common ocular neoplasm in children.
• The usual age for clinical presentation in non-familial cases is 1 year, and the usual clinical
appearance is leukocoria.
• Histopathologically, retinoblastoma is composed of small blue cells intermingled with anastomosing
pools of pink necrosis.
• More differentiated retinoblastomas often exhibit Flexner-Wintersteiner rosettes.

Page 21

360
Incorrect Options:
Option A - Wilms tumor:
• Wilms tumor is a pediatric renal tumor.
• Patients usually present with an abdominal mass or abdominal tenderness.
• It classically shows a triphasic pattern consisting of blastemal, stromal, and epithelial components.
Option C - Ependymoma:
• Ependymoma arises in the fourth ventricle and spinal cord.
• Patients often present with symptoms of hydrocephalus, including nausea, vomiting, and headaches.
• Perivascular pseudo-rosettes and true rosettes are seen.
Option D - Neuroblastoma:
• Neuroblastoma is most commonly found in the adrenal gland.
• Homer-Wright pseudo-rosettes are often seen.
• Positive for chromogranin, synaptophysin, and NSE.

Page 22

361
Types of Immunity
1. Which of the following is secreted by classically activated macrophages?
A. IL-1
B. IL-2
C. IL-4
D. IL-10
----------------------------------------
2. What is the first line of defense against tumor or viral infection?
A. Natural Killer cells
B. T cell
C. Histiocyte
D. Macrophage
----------------------------------------
3. A patient has a positive Mantoux test. Which of the following glycoproteins was directly engaged in
antigen presentation?
A. Class III HLA molecules
B. Non-HLA-dependent antigen presentation
C. Class I HLA molecules
D. Class II HLA molecules
----------------------------------------
4. Toll-like receptors are seen on which of the following cells?
A. Macrophages
B. Endothelial cells
C. Natural killer cells
D. All of the above
----------------------------------------
5. Which of the following is a secondary lymphoid organ?
A. Liver
B. Spleen
C. Bone marrow
D. Thymus
----------------------------------------
6. Which class of immunoglobulins can cross the placenta?
A. IgM
B. IgD

362
C. IgG
D. IgA
----------------------------------------
7. HLA is located on which of the following chromosome?
A. Short arm of chromosome 6
B. Long arm of chromosome 6
C. Short arm of chromosome 3
D. Long arm of chromosome 3
----------------------------------------
8. Which is the most important antigen-presenting cell (APC) for initiating T-cell response against
protein antigens?
A. NK cell
B. Dendritic cell
C. Macrophage
D. B-lymphocyte
----------------------------------------
9. The image below depicts a certain type of major histocompatibility complex. Which of the following
components of this complex functions as the antigen binding site?

A. Beta microglobulin
B. Distal domain alpha 1 and 2
C. Alpha and beta domains
D. Alpha 1 and 3 domain
----------------------------------------
10. Which of the following is not true regarding natural killer cells?
A. Derived from large granular cells
B. Comprise about 5% of peripheral blood lymphocytes
C. Do not secrete interferon-γ
D. Express IgG Fc receptors

Page 2

363
----------------------------------------
11. Which of the following characteristics best describes an essential feature of innate immunity?
A. Immediate and highly pathogen-specific response
B. Exclusively based on T-cell and B-cell receptors
C. Recognition through pattern recognition receptors
D. Demonstrates a lag phase between exposure and maximal response
----------------------------------------
12. Which of the following cell types lacks expression of HLA antigen?
A. B cells
B. Thrombocyte
C. Macrophage
D. Red blood cell
----------------------------------------
13. Which of the following is the most potent stimulator of naive T cells?
A. Dendritic cell
B. Plasma cell
C. Macrophage
D. B cell
----------------------------------------
14. In the thymus, which of the following genes is responsible for the recognition of self-antigens?
A. NOTCH 1
B. AIRE
C. Rb
D. PTEN
----------------------------------------
15. Which of the following is not an immune-privileged site?
A. Brain
B. Seminiferous tubule
C. Cornea
D. Heart
----------------------------------------
16. A 24-year-old woman receives a blood transfusion after a previously unremarkable transfusion.
Shortly after, itching, widespread urticaria, laryngeal edema, and dyspnea with wheezing develops. She
has a history of recurrent upper respiratory infections and regular bouts of diarrhea. She likely has
deficieny of which of these immunoglobulins?
A. IgA

Page 3

364
B. IgD
C. IgE
D. IgG
----------------------------------------

Correct Answers
Question Correct Answer

Question 1 2
Question 2 1
Question 3 4
Question 4 4
Question 5 2
Question 6 3
Question 7 1
Question 8 2
Question 9 2
Question 10 3
Question 11 3
Question 12 4
Question 13 1
Question 14 2
Question 15 4
Question 16 1

Solution for Question 1:


Correct
Option B - IL-2:
• M1 (classically activated macrophages): Activated by interferon-gamma (IFN-γ). Release interleukin-2
(IL-2). Proinflammatory phenotype.
• Activated by interferon-gamma (IFN-γ).
• Release interleukin-2 (IL-2).
• Proinflammatory phenotype.
• Activated by interferon-gamma (IFN-γ).
• Release interleukin-2 (IL-2).
• Proinflammatory phenotype.

Page 4

365
Incorrect Options:
Option A - IL-1:
• IL-1 has a wide range of biological functions, which include acting as a leukocyte pyrogen, a mediator
of fever and an endogenous leukocyte mediator, and an inducer of several components of the
acute-phase response and lymphocyte-activating factor.

Option C
- IL-4: Il-4 is secreted by (Th2) T-helper cells to activate eosinophils and increase IgE production.
Option D
- IL-10: IL-10 is secreted by regulatory T-cells to prevent autoimmunity and maintain tolerance.

Solution for Question 2:

Page 5

366
Correct Option A - Natural Killer cells:
NK cells
• Ability to kill virus-infected cells and tumor cells
• NK cells act as the first line of defense against viral infections and some tumors.
• Two cell surface molecules, CD16 and CD56, are most commonly used to identify NK cells.
• CD16 is an Fc receptor for IgG, and it confers on NK cells the ability to lyse IgG-coated target cells.
This phenomenon is known as antibody-dependent cell-mediated cytotoxicity (ADCC).
• The function of NK cells is regulated by a balance between signals from activating and inhibitory
receptors.
• NK cell inhibitory receptors recognize self-class I MHC molecules, which are expressed in all healthy
cells. The inhibitory receptors prevent NK cells from killing the normal cells.
Incorrect Options:
Option B - T cell:
• Cell-mediated immunity is a type of acquired immunity.
• CD4+ T cells help B cells make antibodies and produce cytokines to recruit phagocytes and activate
other leukocytes.
• CD8+ T cells directly kill virus-infected and tumor cells via perforin and granzymes (similar to NK
cells).
• T cells are responsible for delayed cell-mediated hypersensitivity (type IV).
• Therefore, these are not the first line of defense against viral infections.
Option C - Histiocyte:
• A histiocyte is a morphological term referring to tissue-resident macrophages.
• However, in viral infections, natural killer cells are the first line of defence.
Option D - Macrophage:
• The dominant cells in most chronic inflammatory reactions are macrophages, which contribute to the
reaction by secreting cytokines and growth factors that act on various cells, destroying foreign invaders
and tissues, and by activating other cells, notably T lymphocytes.
• Th1 cells secrete IFN-γ, which enhances the ability of monocytes and macrophages to kill microbes
they ingest. This function is also enhanced by the interaction of T cell CD40L with CD40 on
macrophages. Macrophages also activate lymphocytes via antigen presentation.
• Macrophage receptors are CD14 (receptor for pathogen-associated molecular patterns [e.g.,
lipopolysacharides]), CD40, CCR5, major histocompatibility (MHC) II, B7, Fc and C3b receptors
(enhanced phagocytosis)
• Macrophages are part of innate immunity; however, these are not the first line of defence against viral
infections.

Solution for Question 3:


Correct Option D - Class II HLA molecules:

Page 6

367
• In delayed hypersensitivity reactions, such as the one described in the scenario following injection
with purified protein derivative (PPD), antigen presentation primarily occurs through Class II HLA
molecules.
• Class II HLA molecules are expressed on the surface of antigen-presenting cells (APCs) such as
dendritic cells, macrophages, and B cells.
• These cells process exogenous antigens, like those derived from Mycobacterium tuberculosis, and
present them to CD4+ T cells, initiating the immune response characteristic of delayed-type
hypersensitivity.
• Therefore, the engagement of Class II HLA molecules in antigen presentation is directly involved in
the delayed hypersensitivity reaction observed in the patient.

Incorrect Options:
Option A - Class III HLA molecules:
• Class III HLA molecules, also known as complement proteins, are not directly involved in antigen
presentation. Instead, they encode proteins involved in the immune response, such as components of
the complement system (C2, C4, and others), tumor necrosis factor (TNF), and heat shock proteins.
These molecules play roles in inflammation, cell signaling, and immunity, but they do not directly
present antigens to T cells.
Option B - Non-HLA-dependent antigen presentation:
• While there are non-HLA-dependent mechanisms of antigen presentation, such as presentation by
CD1 molecules for lipid antigens, these mechanisms are not typically involved in delayed
hypersensitivity reactions like the one described in the scenario.
• Delayed hypersensitivity reactions primarily involve the presentation of protein antigens by HLA
molecules, particularly Class II HLA molecules on antigen-presenting cells
Option C - Class I HLA molecules:
• Class I HLA molecules are involved in presenting endogenous antigens, typically peptides derived
from intracellular pathogens or self-proteins (viral or tumor), to CD8+ cytotoxic T cells.
• While Class I HLA molecules play a crucial role in cell-mediated immunity and cytotoxic T-cell
responses, they are not primarily involved in delayed hypersensitivity reactions, which primarily rely on

Page 7

368
the presentation of exogenous antigens by Class II HLA molecules.

Solution for Question 4:


Correct Option D - All of the above:
Toll-like receptors:
• Toll-like receptors are family pattern recognition receptors essential for host defense against
microbes.
• Toll-like receptors (TLRs) are a class of receptors that play a crucial role in detecting microbial
infections and initiating immune responses.
• They recognize specific molecular patterns associated with various pathogens, including bacteria,
viruses, fungi, and protozoa.
• TLRs are not only expressed on immune cells like macrophages, dendritic cells, and natural killer cells
but also on non-immune cells like endothelial cells.
• Mammals have 10 TLRs, each recognizing a different set of microbial molecules.
• The TLRs are present in the plasma membrane and endosomal vesicles.
• All TLRs signal by a common pathway that culminates in the activation of two sets of transcription
factors: NF-κB, which stimulates the synthesis and secretion of cytokines and the expression of
adhesion molecules, both of which are critical for the recruitment and activation of leukocytes Interferon
regulatory factors (IRFs), which stimulate the production of the antiviral cytokines, type I interferons.
• NF-κB, which stimulates the synthesis and secretion of cytokines and the expression of adhesion
molecules, both of which are critical for the recruitment and activation of leukocytes
• Interferon regulatory factors (IRFs), which stimulate the production of the antiviral cytokines, type I
interferons.
• Inherited loss-of-function mutations affecting TLRs and their signaling pathways are associated with
rare but serious immunodeficiency syndromes.

Page 8

369
Incorrect Options:
Option A - Macrophages:
• These are key players in the innate immune response and are involved in engulfing and destroying
pathogens.
• Macrophages express TLRs on their surface, allowing them to detect the presence of pathogens and
trigger immune responses.
Option B - Endothelial cells:
• These cells line the interior surface of blood vessels and play important roles in regulating vascular
function and inflammation.
• Endothelial cells also express TLRs, which enable them to detect microbial components and initiate
immune responses, particularly in the context of vascular inflammation and infection.
Option C - Natural killer cells:
• NK cells are a type of lymphocyte that plays a critical role in the innate immune response against
infected or abnormal cells, such as cancer cells and virus-infected cells.
• While NK cells primarily function in cytotoxicity and cytokine production, they also express TLRs,
which can modulate their activation and function in response to microbial challenges.

Solution for Question 5:


Correct Option B - Spleen:
• The spleen is a secondary lymphoid organ.
Lymphoid organs:
• The organs of the body which comprise the immune system and/or contribute to immune function
include the bone marrow, spleen, thymus, lymph nodes, and a network of lymphoid tissue along
secretory surfaces (i.e., the so-called mucosa-associated lymphoid tissue, MALT), and the skin.

Page 9

370
• Lymphoid organs can be classified in two ways.
• The first classification is based on the role that organs play in developing the immune system and/or
its ability to elicit a response.
Primary and Secondary Lymphoid Organs:
• Primary lymphoid organs are those organs in which the production of the cells of the immune system
takes place.
• For example, bone marrow is a primary organ. It contains a pluripotent stem cell which serves as the
precursor to red blood cells (i.e., erythrocytes) and myeloid progenitors (which ultimately differentiate
into granulocytes, mast cells, monocytes, and platelets), in addition to lymphoid progenitors (which
ultimately differentiate into the various types of lymphocytes).
• Secondary (Peripheral) Lymphoid Organs (in which adaptive immune responses to microbes are
initiated) Lymph nodes Spleen Mucosal & Cutaneous lymphoid tissues
• Lymph nodes
• Spleen
• Mucosal & Cutaneous lymphoid tissues
• Lymph nodes
• Spleen
• Mucosal & Cutaneous lymphoid tissues
Incorrect Options:
Option A - Liver:
• The liver is not a lymphoid organ.
• Rather it is an extra-medullary site of hematopoiesis.
Option C - Bone marrow:
• Bone marrow is a primary, not secondary lymphoid organ
• It contains a pluripotent stem cell that serves as the precursor to red blood cells (i.e., erythrocytes)
and myeloid progenitors (which ultimately differentiate into granulocytes, mast cells, monocytes, and
platelets), in addition to lymphoid progenitors (which ultimately differentiate into the various types of
lymphocytes).
Option D - Thymus:
• The thymus is a primary lymphoid organ.
• Maturation of T cells occurs in the thymus
• Lymphoid progenitors which mature the thymus are called thymus-derived lymphocytes or T cells.

Solution for Question 6:


Correct Option C - IgG:
• IgG is the only class of immunoglobulins capable of crossing the placenta and reaching the fetal
circulation.

Page 10

371
• This ability is facilitated by active transplacental transfer mechanisms, primarily mediated by
syncytiotrophoblast cells expressing neonatal Fc receptors (FcRn). These receptors interact with the Fc
domain of IgG, allowing for efficient transport of IgG from the maternal bloodstream to the fetal
circulation.
• IgG is thus the most abundant antibody present in newborns.
Incorrect Options:
Option A - IgM:
• IgM is a large immunoglobulin typically found in the bloodstream.
• However, unlike IgG, IgM molecules are too large to efficiently cross the placenta due to their size and
structural complexity.
• IgM molecules are pentameric in structure and are unable to pass through the placental barrier
formed by syncytiotrophoblast cells.
• Therefore, IgM does not participate in transplacental transfer to the fetus.
Option B - IgD:
• IgD is primarily involved in activating B cells in the immune response and is found mainly on the
surface of mature B cells.
• Unlike IgG, IgD is not known to be involved in transplacental transfer or fetal immunity.
• Its role is more focused on antigen recognition and B cell activation within the immune system, rather
than systemic distribution across the placenta to provide passive immunity to the fetus.
Option D - IgA:
• IgA is the primary immunoglobulin found in mucosal secretions such as saliva, tears, and breast milk,
where it plays a crucial role in protecting mucosal surfaces from infections.
• While IgA can be transferred to the newborn through breast milk postnatally, it does not cross the
placenta during pregnancy to provide fetal immunity.
• The placental barrier is not permeable to IgA molecules, and thus, IgA does not participate in the
transplacental transfer process described in the passage.

Solution for Question 7:


Correct Option A - Short arm of chromosome 6:
• The HLA locus is located on the short arm of chromosome number 6
MHC Class I:
• Class I MHC molecules display peptides derived from cytoplasmic proteins, including normal proteins
and virus- and tumour-specific antigens.
• These are all recognized and bound to class I MHC molecules by CD8+ T cells.
• Because important functions of CD8+ CTLs include eliminating viruses, which may infect any
nucleated cell, and killing tumour cells, which may arise from any nucleated cell, it makes good sense
that all nucleated cells express class I MHC molecules and can be surveyed by CD8+ T cells.
MHC Class II

Page 11

372
• Class II MHC molecules present antigens derived from extracellular microbes and proteins following
their internalization into endosomes or lysosomes.
• Here, the internalized proteins are proteolytically digested, producing peptides that associate with
class II heterodimers in the vesicles.
• Because CD4+ T cells can recognize antigens only in the context of self-class II molecules, they are
referred to as class II MHC restricted.
• In contrast to class I molecules, class II molecules are mainly expressed in cells that present ingested
antigens and respond to T-cell help (macrophages, B lymphocytes, and DCs).
Incorrect Options:
Option B - Long arm of chromosome 6:
• The HLA locus is present on chromosome number 6
• However, it is located on the short arm, not on the long arm of this chromosome.
Option C - Short arm of chromosome 3:
• Chromosome 6 has genes on the short arm that codes for the human leukocyte antigen.
• This option is incorrect
Option D - Long arm of chromosome 3:
• The location of the MHC molecule is on the short arm of chromosome 6

Solution for Question 8:


Correct Option B - Dendritic cell:
• The most important antigen-presenting cell is a dendritic cell.
Dendritic cells:
• DCs (sometimes called interdigitating DCs) are the most important antigen-presenting cells for
initiating T-cell responses against protein antigens.
• These cells have numerous fine cytoplasmic processes that resemble dendrites, from which they
derive their name.
• Several features of DCs account for their key role in antigen presentation.
• First, these cells are located at the right place to capture antigens— under epithelia, the common site
of entry of microbes and foreign antigens, and in the interstitial cells of all tissues, where antigens may
be produced.
• Immature DCs within the epidermis are called Langerhans cells.
• Second, DCs express many receptors for capturing and responding to microbes (and other antigens),
including TLRs and lectins.
• Third, in response to microbes, DCs are recruited to the T-cell zones of lymphoid organs, where they
are ideally located to present antigens to naïve T cells.
• Fourth, DCs express high levels of MHC and other molecules needed for antigen presentation and
T-cell activation.

Page 12

373
• They play a role in humoral immune responses by presenting antigens to B cells in the germinal
center, part of a process through which only B cells that express antibodies with high affinity for antigen
survive and mature into plasma or memory cells.
Incorrect Options:
Option A - NK cell:
• Natural killer (NK) cells are effector lymphocytes of the innate immune system that control several
types of tumours and microbial infections by limiting their spread and subsequent tissue damage.
• They do not play a role in the antigen presentation
Option C - Macrophage:
• The main function of macrophages are:
• Macrophages that have phagocytosed microbes and protein antigens process the antigens and
present peptide fragments to T cells. Thus, macrophages function as antigen-presenting cells in T-cell
activation.
• Macrophages are key effector cells in certain forms of cell-mediated immunity, the reaction that
serves to eliminate intracellular microbes. In this response, T cells activate macrophages and enhance
their ability to kill ingested microbes (discussed later).
• Macrophages also participate in the effector phase of humoral immunity.
• Macrophages efficiently phagocytose and destroy opsonized (coated) microbes by IgG or C3b.
Option D - B-lymphocyte:
• B lymphocytes are the only cells in the body capable of producing antibodies, the mediators of
humoral immunity.
• B cells recognize antigens via the B-cell antigen receptor complex.
• Membrane-bound antibodies of the IgM and IgD isotypes, present on the surface of all mature, naïve
B cells, are the antigen-binding component of the B-cell receptor (BCR) complex.
• They are not the most important cells involved in antigen presentation to T cells.

Solution for Question 9:


Correct Option B - Distal domain alpha 1 and 2:
• The above-shown Image is of MHC I
• The peptide binding site is formed by the distal domain of alpha 1 and 2.
MHC Class I:
• Class I MHC molecules are expressed on all nucleated cells and platelets.
• They are heterodimers consisting of a polymorphic α, or heavy, chain (44-kD) linked non-covalently to
a smaller (12-kD) nonpolymorphic protein called β2-microglobulin.
• The α chains are encoded by three genes, designated HLA-A, HLA-B, and HLA-C, that lie close to
one another in the MHC locus.
• Class I MHC molecules display peptides derived from cytoplasmic proteins, including normal proteins
and virus- and tumour-specific antigens. These are all recognized and bound to class I MHC molecules

Page 13

374
by CD8+ T cells.
• Peptide-loaded MHC molecules then associate with β2-microglobulin to form a stable complex
transported to the cell surface.
Incorrect Options:
Option A - Beta macroglobulin:
• The beta microglobulin does not act as a primary site for antigen binding.
• Rather, once the antigen is bound to alpha chains, there is an association of this peptide load MHC
with B2-microglobulin
• The main function of beta-2-microglobulin is to stabilize the complex.
Option C - Alpha and beta domains:
• The above Image is of MHC class I, not MHC class II.
• The antigen binding between alpha and beta domains occurs in the MHC Class II, not Class I.
Option D - Alpha 1 and 3 domain:
• The antigen binding in the MHC class I is between the alpha 1 and alpha 2 domains.
• The alpha 3 domain attaches the alpha domain to the membranes.

Solution for Question 10:


Correct Option C - Do not secrete interferon-γ:
• NK cells do secrete IFN- γ
Natural killer cells:
• The function of NK cells is to recognize and destroy severely stressed or abnormal cells, such as
virus-infected cells and tumour cells.
• There are many types of activating receptors that recognize surface molecules that are induced by
various kinds of stress, such as infection and DNA damage.
• These receptors enable NK cells to recognize damaged or infected cells. NK cell inhibitory receptors
recognize self-class I MHC molecules expressed in all healthy cells.
• The inhibitory receptors prevent NK cells from killing normal cells. Virus infection or neoplastic
transformation often enhances the expression of ligands for activating receptors and simultaneously
reduces the expression of class I MHC molecules.
• As a result, when NK cells engage these abnormal cells, the balance is tilted toward activation, and
the infected or tumour cell is killed.
• NK cells also secrete cytokines such as interferon-γ (IFN-γ), which activates macrophages to destroy
ingested microbes, and thus NK cells provide an early defence against intracellular microbial infections.
• The activity of NK cells is regulated by many cytokines, including the interleukins IL-2, IL-15, and
IL-12. IL-2 and IL-15 stimulate the proliferation of NK cells, whereas IL-12 activates the killing of target
cells and the secretion of IFN-γ.
Incorrect Options:
Option A - Derived from large granular cells:

Page 14

375
• NK cells are derived from large granular lymphocyte progenitors.
• Therefore this statement is true
Option B - Comprise about 5% of peripheral blood lymphocytes:
• About 5% of peripheral blood lymphocytes are composed of natural killer cells
• Therefore this statement is true
Option D - Express IgG Fc receptors:
• NK cells express CD16, a receptor for IgG Fc.
• This helps them in antibody-dependent cytotoxicity.
• Therefore this statement is true

Solution for Question 11:


Correct Option C - Recognition through pattern recognition receptors:
• "Recognition through pattern recognition receptors," accurately represents an essential feature of
innate immunity. Pattern recognition receptors (PRRs) are a critical component of the innate immune
system. These receptors can recognize conserved structures shared by various pathogens, known as
pathogen-associated molecular patterns (PAMPs). The recognition of PAMPs by PRRs triggers an
immediate and non-specific response, playing a pivotal role in the body's defense against a wide range
of pathogens.
Incorrect Options:
Option A - Immediate and highly pathogen-specific response: Innate immunity is immediate in nature, p
roviding the first line of defense against invading pathogens. However, it is non-specific in its response,
meaning it doesn't specifically target a particular pathogen. Instead, it responds to general patterns sh
ared by various pathogens through pattern recognition receptors.
Option B - Exclusively based on T-cell and B-cell receptors: This statement does not accurately describ
e innate immunity. While T-cells and B-cells are components of adaptive immunity, they possess specif
ic receptors (T-cell receptor and B-cell receptor, respectively) that enable a highly specific response to
particular antigens. Innate immunity, however, primarily employs pattern recognition receptors and doe
s not rely on T-cell or B-cell receptors
Option D - Demonstrates a lag phase between exposure and maximal response: Innate immunity provi
des an immediate response to pathogens upon exposure. There is no lag phase; its response is rapid
and occurs without the need for prior exposure to a
specific pathogen. This contrasts with adaptive immunity, which does have a
lag phase between initial exposure and the development of a
maximal response due to the time required to generate specific T-cell and B-cell responses.

Solution for Question 12:


Correct Option D - Red blood cells:
• MHC 1 loci include HLA-A, HLA-B, HLA-C, etc.

Page 15

376
• They bind T cell receptors and CD 8 cells.
• They have one long chain and one short chain.
• They are expressed on all nucleated cells, antigen-presenting cells, and platelets. They are not
expressed in red blood cells.
• So mature red blood cells which lack nuclei are the only cells that do not express MHC-1.
• Class I MHC molecules are present on the membrane of almost every cell in an organism, while class
II molecules are restricted to cells of the immune system.
• MHC-1 presents endogenous antigens (viral and cytosolic proteins) to CD 8 cytotoxic T cells.
• Antigen peptides are loaded onto the MHC in RER after delivery via TAP. MHC 1 is associated with
beta 2 microglobulin.

Incorrect Options:
Option A - B cells:
• B cells act as antigen-presenting cells.
• They express MHC I and MHC II on their surface.
Option B - Thrombocyte:
• Thrombocytes express MHC class 1.
• They play a role in blood coagulation.
Option C - Macrophages:
• Macrophage acts as antigen-presenting cells.
• They express MHC I and MHC II on their surface.

Solution for Question 13:


Correct Option A - Dendritic cell:

Page 16

377
• T cells are stimulated by the interaction of a T-cell receptor (TCR) with MHC-peptide complexes
(MHCps) on the surface of antigen-presenting cells (APCs). The T-cell stimulation is carried by many
APCs, including macrophages, B-cells and dendritic cells. Of these, the dendritic cells are the most
potent APCs for stimulating the naive T cells.
• The reversible antigen-independent binding mechanism allows naive T-cells to survey the dendritic
cells for antigens and respond by forming clusters with dendritic cells.
• As a result of this contact between the dendritic cells and T-cells, stimulatory signals are delivered that
allow the release of cytokines and chemokines.
• In response to the microbes, the dendritic cells are recruited to the T-cell zones of the lymphoid
organs, where they are ideally positioned to present antigens to the T-cells.
• Dendritic cells (sometimes called interdigitating dendritic cells) are the most important
antigen-presenting cells for initiating T-cell responses against protein antigens. The following contribute
to this: First, these cells are located at the ideal site to capture the antigens: under epithelium, i.e. the
common site of entry of microbes and foreign antigens) and in the interstitium of all tissues, where
antigens may be produced. Second, dendritic cells express high levels of MHC and other molecules
needed for presenting antigens to and activating T cells. Also, dendritic cells express many receptors
for capturing and responding to microbes (and other antigens), including TLRs and receptors for C-type
lectins.
• First, these cells are located at the ideal site to capture the antigens: under epithelium, i.e. the
common site of entry of microbes and foreign antigens) and in the interstitium of all tissues, where
antigens may be produced.
• Second, dendritic cells express high levels of MHC and other molecules needed for presenting
antigens to and activating T cells.
• Also, dendritic cells express many receptors for capturing and responding to microbes (and other
antigens), including TLRs and receptors for C-type lectins.
• First, these cells are located at the ideal site to capture the antigens: under epithelium, i.e. the
common site of entry of microbes and foreign antigens) and in the interstitium of all tissues, where
antigens may be produced.
• Second, dendritic cells express high levels of MHC and other molecules needed for presenting
antigens to and activating T cells.

Page 17

378
• Also, dendritic cells express many receptors for capturing and responding to microbes (and other
antigens), including TLRs and receptors for C-type lectins.
Incorrect Options:
Option B - Plasma cells:
• Plasma cells are the activated form of the B-cells responsible for producing antibodies.
• However, they cannot act as APCs since they cannot switch antibody class and no longer display
MHC-II molecules and therefore do not take up the antigen (as they no longer display a significant
quantity of immunoglobulin on the cell surface).
Option C - Macrophages:
• Macrophages ingest the microbes and display peptides for recognition by the T-cells, which activate
the macrophages to kill microbes and mediate the central reaction of cell-mediated immunity.
• However, the dendritic cells actively participate through the rearrangement of their actin cytoskeleton
in the stimulation of T-cells and induce an antigen-independent calcium signal and the polarization of
signalling molecules. Therefore, they are superior to the other APCs.
Option D - B-cells:
• B-cells present peptides to helper T-cells and receive signals that stimulate antibody responses to
protein antigens, critical steps in humoral immune responses.
• However, the dendritic cells actively contribute to the activation of T-cells by rearranging their actin
cytoskeleton, resulting in an antigen-independent calcium signal and the polarization of signalling
molecules.
• They are, therefore, better than the other APCs.

Solution for Question 14:


Correct Option B - AIRE:
AIRE (Autoimmune Regulator)
• The central tolerance is induced by the antigen-induced deletion of self-reactive T lymphocytes and B
lymphocytes during their maturation in central lymphoid organs (thymus and bone marrow). In the
thymus, the self-protein antigens are processed and presented by thymic APCs. When immature T
cells with T cell receptors for self antigen are identified, they immediately undergo apoptosis.
• A protein called AIRE (autoimmune regulator) causes the expression of some "peripheral
tissue-restricted" self-antigens in the thymus. It is necessary for the deletion or negative selection of
immature T cells specific to these antigens. In the CD4+ T-cell lineage, some of these cells,
self-antigens in the thymus, do not die but develop into regulatory T cells ( T-reg).
• Mutations in the AIRE gene are the cause of an autoimmune polyendocrinopathy.
Incorrect Options:
Option A - NOTCH 1:
• The notch 1 gene is important in T-cell differentiation. It has an oncogenic role in developing T-cell
leukaemia due to its ability to turn on pro-growth genes in the T-cell progenitors.
Option C - Rb:

Page 18

379
• Rb is a retinoblastoma gene and is a negative regulator (i.e. tumour suppressor) of the cell cycle and
is directly or indirectly activated in human cancer, most commonly seen in retinoblastoma.
Option D - PTEN:
• PTEN is a tumour suppressor gene that signals the inhibition of growth factors (PI3K) and thus
prevents altered gene expression, increased growth and metastasis of cancer. An example is its
involvement in melanoma, where the loss of PTEN causes metastases.

Solution for Question 15:


Correct Option D - Heart:
Immune privilege sites
• Some self-antigens are hidden (sequestered) from the immune system because the tissues in which
they are hidden have no communication with blood and lymph. As a result, the immune response
cannot be generated unless released into circulation and gets ignored by the immune system.
• Such tissues are the testis, eye, and brain. These tissues are known as immune-privileged sites
because the antigens inside are hidden from the immune system.
• The heart is an organ that is surrounded by pericardium and is richly supplied by blood and lymph
vessels. Therefore, it is not an immune-privileged site.
Incorrect Options:
Option A - Brain:
• The blood-brain barrier is an anatomical barrier that provides a specialized micro-environment for the
cellular constituents of the brain.
• The brain has all parts within the blood-brain barrier except for the area postrema (which lies outside),
thus forming a barrier from blood and lymphatics and creating an immune-privileged site.
Option B - Seminiferous tubules:
• The blood-testis barrier is an anatomical barrier that prevents the communication of the
spermatocytes formed inside the seminiferous tubules. As the cells are haploid, the difference in
chromosome count from the other cells in blood and lymph will create an immune response, and thus
the testes' seminiferous tubules are protected, and the barrier helps to conceal the contents.
Option C - Cornea:
• In the eye, the cornea contains no blood or lymph vessels. Furthermore, the endothelial cells, corneal
epithelial cells and keratocytes do not express MHC class II molecules but only low levels of MHC class
I molecules. Also, there is immunological tolerance, immune deviation and immune suppressive
intra-ocular environment, which makes the cornea an immune-privileged site.

Solution for Question 16:


Correct Option A - IgA:
• The given clinical scenario is a case of Isolated IgA deficiency.
• It is most often asymptomatic but can be characterized by anaphylactic reactions to transfused blood..

Page 19

380
• It is critical to notify the transfusion service about a patient with possible IgA deficiency prior to giving
blood products so that products can be washed to remove any immunoglobulins prior to administration.
• Associated with frequent episodes of diarrhea and recurrent sinopulmonary infections.
• This inherited B-cell defect is due to the inability of B cells to mature into IgA-producing plasma cells.
• Patients lacking IgA can develop IgE antibodies against the IgA antibodies present in transfused
blood as the body doesn't recognize it as a self-antigen.
• The patient in the vignette received a transfusion earlier which was unremarkable because she was
not sensitised to ‘foreign’ IgA.
• This sensitization can result in susceptibility to anaphylaxis on subsequent transfusion.
Incorrect Options:
Option B - IgD:
• IgD deficiency is not associated with anaphylaxis.
Option C - IgE:
• Patients with IgA deficiency can develop IgE antibodies against the IgA antibodies present in
transfused blood.
• Given the scenario of anaphylaxis IgE is elevated and not decreased.
Option D - IgG:
• IgG deficiency is not associated with anaphylaxis.
• IgG levels are normal In selective IgA deficiency

Page 20

381
Types of Hypersensitivity Reactions
1. A 27-year-old patient presents to the emergency with a complaint of generalized oedema, sweating,
and flushing after a bee sting. Blood pressure is 88/79mm Hg, and heart rate is 126 bpm. The patient
experienced which type of hypersensitivity?
A. T-cell mediated cytotoxicity
B. IgE-mediated reaction
C. IgG-mediated reaction
D. IgA-mediated reaction
----------------------------------------
2. An elderly patient is screened for tuberculosis with a tuberculin test. Which of the following is true
regarding this test?
A. Indicates the cell-mediated immunity against the bacteria
B. Detects the presence of tuberculosis bacteria in the bloodstream.
C. Can distinguish between infection and vaccination
D. May be falsely positive in immunosuppression
----------------------------------------
3. Which of the following is an example of an immune complex-mediated hypersensitivity reaction?
A. Asthma
B. Arthus reaction
C. Contact dermatitis
D. Rheumatic fever
----------------------------------------
4. Which of the following is a type II Hypersensitivity reaction?
A. Chronic kidney rejection
B. Autoimmune Hemolytic anemia
C. Arthus reaction
D. Mantoux test
----------------------------------------
5. A 47-years-old male, an active smoker with a known case of bronchial asthma, now presents with
difficulty breathing with the breath sounds being audible externally. Which of the following feature will
you expect to find in this patient?
A. Chronic, necrotizing infection of bronchi
B. Increase in the number of airway goblet cells
C. Fibroblastic foci
D. Hypertrophy and hyperplasia of mucus-secreting glands in bronchi
----------------------------------------

382
6. A 38-year-old male presents with complaints of an itchy rash around his waistline. The rash is
localised to the area where his skin is in contact with the nickel buckle of his new belt. Which of the
following is the likely pathophysiology of this patient’s presentation?
(or)
Pathophysiology involved in nickel allergic contact dermatitis?
A. IgE
B. IgG
C. Immune complex deposition
D. T-cells
----------------------------------------
7. A 20-year-old gardener presents to his family physician with a pruritic rash as shown in the image
below. He discloses a history of exposure to poison ivy. Which of the following best describes the
pathogenesis of these skin lesions?
(or)
Which of the following describes the pathogenesis of Contact dermatitis?

A. Cytotoxic antibody production


B. Delayed-type hypersensitivity
C. Deposition of antigluten antibodies
D. Deposition of circulating immune complexes
----------------------------------------
8. Which of the following is/are an example of type IV hypersensitivity?
A. Anaphylaxis
B. Rheumatic fever
C. Myasthenia gravis
D. Contact dermatitis
----------------------------------------
9. A 15-year-old boy presents to the emergency department complaining of shortness of breath after
consuming peanuts. On examination, hives are found all over his body. He is given epinephrine, and
recovers. Which of the following antibodies plays a role in the pathogenesis of his condition?

Page 2

383
(or)
Which of the following antibodies plays a role in the pathogenesis of Type 1 hypersensitivity?
A. IgA
B. IgM
C. IgE
D. IgG
----------------------------------------
10. Which of the following accurately describes the underlying disease process in Guillain-Barré
syndrome?
A. Type 1 Hypersensitivity reaction
B. Type 2 Hypersensitivity reaction
C. Type 3 Hypersensitivity reaction
D. Type 4 Hypersensitivity reaction
----------------------------------------
11. Which of the following would most likely confirm the diagnosis of Myasthenia gravis?
A. Single fibre electromyography (SFEMG)
B. CT scan of the thorax
C. Edrophonium (Tensilon) test
D. Muscle biopsy
----------------------------------------
12. Identify the correct statement regarding the Cellular process shown below:

A. A type of mechanism of spread of malignant tumors


B. A type of mechanism of HIV dissemination
C. A mechanism for the persistence and evolution of autoimmune disease
D. One of the mechanisms of apoptosis
----------------------------------------
13. Which of the following mechanisms best describes the pathophysiology of Goodpasture Syndrome?
A. Opsonization and phagocytosis of renal tubular cells

Page 3

384
B. Antibody-mediated activation of proteases, leading to disruption of intercellular adhesions
C. Neutrophil degranulation and inflammation within the glomerular capillaries
D. Complement- and Fc receptor-mediated inflammation in the kidney glomeruli
----------------------------------------

Correct Answers
Question Correct Answer

Question 1 2
Question 2 1
Question 3 2
Question 4 2
Question 5 2
Question 6 4
Question 7 2
Question 8 4
Question 9 3
Question 10 2
Question 11 1
Question 12 3
Question 13 4

Solution for Question 1:


Correct Option B - IgE-mediated reaction:
• Type I hypersensitivity reaction is an IgE-mediated reaction.
• Systemic anaphylaxis is characterized by vascular shock, widespread edema, and difficulty in
breathing.
• It may occur in sensitized individuals after administration of foreign proteins (e.g., antisera),
hormones, enzymes, polysaccharides, and drugs (e.g., the antibiotic penicillin), following exposure to
food allergens (e.g., peanuts, shell sh) or insect toxins (e.g., those in bee venom).
• The symptoms range from rashes to anaphylactic shock with vasodilation hypotension and
bronchiolar spasm.
Incorrect Options:
Option A - T-cell mediated cytotoxicity:
• T-cell mediated cytotoxicity is a type IV hypersensitivity.
• Two mechanisms, each involving T cells: Direct cell cytotoxicity: CD8+ cytotoxic T-cells kill targeted
cells. Inflammatory reaction: effector CD4+
• Direct cell cytotoxicity: CD8+ cytotoxic T-cells kill targeted cells.

Page 4

385
• Inflammatory reaction: effector CD4+
• T-cells recognize antigens and release inflammation-inducing cytokines.
• The response does not involve antibodies (vs. types I, II, and III).
• Examples are contact dermatitis (e.g., poison ivy, nickel, allergy) & graft-versus-host disease.
• The principle of type IV hypersensitivity is used for tests such as purified protein derivatives (PPD) for
tuberculosis infection, patch test for contact dermatitis & Candida skin test for T cell immune function.
Option C - IgG-mediated reaction:
• IgG hypersensitivity reaction is a type II hypersensitivity reaction.
• Antibodies bind to cell-surface antigens causing inflammation, and cellular dysfunction.
• Cellular destruction occurs when a cell is opsonized (coated) by antibodies, leading to either
phagocytosis and/or activation of the complement system & natural cell killing (antibody-dependent
cellular cytotoxicity).
• Examples are autoimmune hemolytic anemia (including drug-induced form), immune
thrombocytopenia, transfusion reactions & hemolytic disease of the newborn.
• Inflammation is mediated by the binding of antibodies to cell surfaces, activation of the complement
system, and Fc receptor-mediated inflammation.
• Examples are Goodpasture syndrome, rheumatic fever & hyperacute transplant rejection.
• Cellular dysfunction by antibodies binds to cell-surface receptors, abnormal blockade or activation of
the downstream process.
• Examples are myasthenia gravis, graves disease & pemphigus vulgaris.
Option D - IgA-mediated reaction:
• IgA antibodies are not involved in type I hypersensitivity reactions.
• Therefore, this option is incorrect.

Solution for Question 2:


Correct Option A - Indicates the cell-mediated immunity against the bacteria:
• The purified protein derivative (PPD) test reveals whether or not someone has established an immune
response to the tuberculosis-causing bacteria.
• The Mantoux test is done by injecting 0.1 mL of liquid containing 5 tuberculin units of PPD (purified
protein derivative) intradermally.
• It is read 48-72 hours apart
• The basis of the reading of the skin test is the presence or absence and the amount of the induration (
i.e. localized hardening).
• A negative (no swelling) test does not always mean that a person is free of tuberculosis. A person
who has received a bacillus Calmette Guerin (BCG) vaccine against tuberculosis may also have a
positive skin reaction to the TB test (false positive).
Incorrect Options:

Page 5

386
Option B - Detects the presence of tuberculosis bacteria in the bloodstream: Involves injecting a purifie
d protein derivative (PPD) of the tuberculosis bacterium into the skin and then observing the immune re
sponse at the injection site. It does not directly detect the presence of tuberculosis bacteria in the blood
stream but rather assesses the individual's immune response to the bacteria..
Option C - Can distinguish between infection and vaccination: A positive tuberculin skin test result signi
fies cell-mediated hypersensitivity to tubercular antigens but does not differentiate between infection an
d vaccination
Option D - Maybe a false positive in immunosuppression: False-negative reactions (or skin test anergy
) may be produced by certain viral infections, sarcoidosis, malnutrition, Hodgkin lymphoma, immunosu
ppression, and overwhelming active tuberculous disease.

Solution for Question 3:


Correct Option B - Arthus reaction:
• Deposition of circulating immune complexes occurs in type III hypersensitivity.
• The antigen-antibody (mostly IgG) complexes activates complement, which attracts neutrophils;
neutrophils release lysosomal enzymes.
• It can be associated with vasculitis and systemic manifestations.
• Arthus reaction is a local subacute immune complex-mediated hypersensitivity reaction. Intradermal
injection of antigen into a pre-sensitized (has circulating IgG) individual leads to the immune complex
formation in the skin (e.g., enhanced local reaction to a booster vaccination).
• Serum sickness is the prototypic immune complex disease. Antibodies to foreign proteins are
produced and 1–2 weeks later, antibody-antigen complexes form and deposit in tissues, complement
activation inflammation and tissue damage (serum C3, C4).
Incorrect Options:
Option A - Asthma:
• Asthma is a classic example of a type I hypersensitivity reaction.
• IgE are already produced and fixed to mast cells- mast cell degranulation causes the release of
mediators.
• This leads to the later recruitment of inflammatory cells.
• Resulting in vascular dilation, oedema, smooth muscle contraction, mucus production, tissue injury,
and inflammation.
• Examples are anaphylaxis; allergies; bronchial asthma (atopic forms)
Option C - Contact dermatitis:
• Contact dermatitis is an example of a cell-mediated (type IV) hypersensitivity reaction.
• Two mechanisms, each involving T cells: Direct cell cytotoxicity: CD8+ cytotoxic T cells kill targeted
cells. Inflammatory reaction: effector CD4+
• Direct cell cytotoxicity: CD8+ cytotoxic T cells kill targeted cells.
• Inflammatory reaction: effector CD4+
• T cells recognize antigens and release inflammation-inducing cytokines.

Page 6

387
• The response does not involve antibodies (vs types I, II, and III).
• Examples are contact dermatitis (e.g., poison ivy, nickel allergy) & graft-versus-host disease.
• This principle is used in clinical tests such as purified protein derivatives for tuberculosis infection,
patch tests for contact dermatitis & candida skin test for T cell immune function.
Option D - Rheumatic fever:
• Rheumatic fever is an example of type II hypersensitivity.
• IgG hypersensitivity reaction is a type II hypersensitivity reaction.
• Antibodies to the M protein of group A streptococcus cross-react with self-antigens, and cross-react
with myocardial antigen, due to molecular mimicry.
• Antibodies bind to cell-surface antigens or extracellular matrix and causecellular destruction,
inflammation, and cellular dysfunction.
• Cellular destruction occurs when a cell is opsonized (coated) by antibodies, leading to either
phagocytosis and/or activation of the complement system & natural cell killing (antibody-dependent
cellular cytotoxicity).
• Examples are autoimmune hemolytic anaemia (including drug-induced form), immune
thrombocytopenia, transfusion reactions & hemolytic disease of the newborn.
• Inflammation is mediated by the binding of antibodies to cell surfaces, activation of the complement
system, and Fc receptor-mediated inflammation.
• Examples are Goodpasture syndrome, rheumatic fever & hyperacute transplant rejection.
• Cellular dysfunction by antibodies binds to cell-surface receptors, abnormal blockade or activation of
the downstream process.
• Examples are myasthenia gravis, graves disease & pemphigus Vulgaris.

Solution for Question 4:


Correct Option B - Autoimmune Hemolytic anemia:
• Normocytic anemia is usually idiopathic and Coombs ⊕.
• It has two major types:
• Warm Autoimmune hemolytic anemia (AIHA): It is chronic anemia in which primarily IgG causes
extravascular hemolysis. It is seen in systemic lupus erythematosus (SLE) and chronic lymphocytic
leukemia (CLL) and with certain drugs (e.g., β-lactams, α-methyldopa).
• Cold Autoimmune hemolytic anemia (AIHA): Acute anemia in which primarily IgM + complement
causes red blood cells (RBC) agglutination and extravascular hemolysis upon exposure to cold, painful,
blue fingers and toes are observed.
• Seen in chronic lymphocytic leukemia (CLL), Mycoplasma pneumoniae infections, and infectious
mononucleosis.
Incorrect Options:
Option A - Chronic kidney rejection:
• Chronic kidney rejection typically involves Type IV hypersensitivity reactions mediated by T cells
rather than Type II hypersensitivity reactions.

Page 7

388
• In chronic rejection, T cells recognize foreign antigens on the transplanted kidney and initiate an
immune response that leads to inflammation and tissue damage.
• This process is more closely associated with cellular immune responses rather than
antibody-mediated destruction, which is characteristic of Type II hypersensitivity reactions.
• CD4+ T cells respond to recipient antigen-presenting cells (APCs) presenting donor peptides,
including allogeneic major histocompatibility complex (MHC)
• Recipient T cells react and secrete cytokines that lead to the proliferation of vascular smooth muscle,
parenchymal atrophy, and interstitial fibrosis.
• Thus, chronic rejection has a major type IV hypersensitivity component.
Option C - Arthus reaction:
• Arthus reaction is a classic example of type III hypersensitivity.
• Arthus reaction is a local subacute immune complex-mediated hypersensitivity reaction. Intradermal
injection of antigen into a pre-sensitized (has circulating IgG) individual leads to the immune complex
formation in the skin (e.g., enhanced local reaction to a booster vaccination).
Option D - Mantoux test:
• Tuberculosis Infection typically leads to the development of delayed hypersensitivity to M. tuberculosis
antigens, which can be detected by the tuberculin (PPD, or Mantoux) skin test.
• About 2 to 4 weeks after infection, intracutaneous injection of purified protein derivative of M.
tuberculosis induces a visible and palpable induration that peaks in 48 to 72 hours.
• A positive tuberculin test signifies T-cell–mediated immunity to mycobacterial antigens but does not
differentiate between infection and active disease.

Solution for Question 5:


Correct Option B - Increase in number of airway goblet cells:
• Bronchial asthma is an example of a Type I Hypersensitivity reaction and is characterized by airway
remodeling, which includes: Thickening of the bronchial wall with hypertrophy and/or hyperplasia of
bronchial smooth muscles Increase in size of submucosal glands and number of goblet cells Increased
vascularity Sub-basement membrane fibrosis (due to deposition of type I and III collagens)
• Thickening of the bronchial wall with hypertrophy and/or hyperplasia of bronchial smooth muscles
• Increase in size of submucosal glands and number of goblet cells
• Increased vascularity
• Sub-basement membrane fibrosis (due to deposition of type I and III collagens)
• Histological findings in bronchial asthma: Curschmann spirals due to impacted mucin plugs. Charcot
Leyden crystals due to eosinophil crushing artifact Creola bodies are damaged respiratory epithelial
cells.
• Curschmann spirals due to impacted mucin plugs.
• Charcot Leyden crystals due to eosinophil crushing artifact
• Creola bodies are damaged respiratory epithelial cells.
• Thickening of the bronchial wall with hypertrophy and/or hyperplasia of bronchial smooth muscles

Page 8

389
• Increase in size of submucosal glands and number of goblet cells
• Increased vascularity
• Sub-basement membrane fibrosis (due to deposition of type I and III collagens)
• Curschmann spirals due to impacted mucin plugs.
• Charcot Leyden crystals due to eosinophil crushing artifact
• Creola bodies are damaged respiratory epithelial cells.
Incorrect Options:
Option A - Chronic, necrotizing infection of bronchi:
• Chronic, necrotizing infection of the bronchi can be seen in patients with bronchiectasis.
• In bronchiectasis, airways are permanently dilated.
• It is associated with bronchial obstruction, poor ciliary motility (e.g., tobacco smoking, Kartagener
syndrome), cystic fibrosis, allergic bronchopulmonary aspergillosis, and pulmonary infections (e.g.,
Mycobacterium avium complex).
Option C - Fibroblastic foci:
• Fibroblastic foci (exuberant proliferation of fibroblasts) are a feature of idiopathic pulmonary fibrosis
(IPF).
• Progressive fibrotic lung disease of unknown etiology. It may involve multiple cycles of lung injury,
inflammation, and fibrosis.
• Associated with cigarette smoking, environmental pollutants, and genetic defects.
Option D - Hypertrophy and hyperplasia of mucus-secreting glands in bronchi:
• Reid index is the thickness of the mucosal gland layer to the thickness of the wall between epithelium
and cartilage > 50% is a hallmark of chronic bronchitis.
• Increased Reid index is due to the hypertrophy and hyperplasia of mucus-secreting glands in the
bronchi

Solution for Question 6:


Correct Option D - T-cells:
• The clinical presentation described is consistent with allergic contact dermatitis, specifically due to
nickel exposure from the belt buckle.
• Allergic contact dermatitis is a type IV delayed hypersensitivity reaction, which is mediated by T-cells.
• In this scenario, the nickel ions from the belt buckle penetrate the skin and bind to proteins, forming
haptens. Antigen-presenting cells then present these haptens to T-cells, leading to their activation and
recruitment to the skin. The activated T-cells release inflammatory mediators, resulting in the
characteristic erythematous rash with vesicles and papules seen in allergic contact dermatitis.
Incorrect Options:
Option A - IgE:
• IgE is primarily associated with Type I hypersensitivity reactions, not Type IV hypersensitivity
reactions like allergic contact dermatitis.

Page 9

390
• Type I hypersensitivity reactions involve the release of histamine and other inflammatory mediators
from mast cells and basophils, leading to immediate allergic responses such as urticaria (hives) and
anaphylaxis.
• In allergic contact dermatitis, the immune response is primarily mediated by IgE, not T-cells.
Option B - IgG :
• IgG antibodies are more commonly associated with Type II hypersensitivity reactions (such as
autoimmune hemolytic anemia). In allergic contact dermatitis, the immune response is primarily
mediated by T-cells rather than IgG antibodies.
• In type II hypersensitivity antibodies bind to cell-surface antigens or extracellular matrix causing
cellular destruction, inflammation, and cellular dysfunction.
Option C - Immune complex deposition :
• Immune complex deposition is characteristic of Type III hypersensitivity reactions, not Type IV
hypersensitivity reactions like allergic contact dermatitis.
• In Type III hypersensitivity reactions, antigen-antibody complexes deposit in tissues, leading to
complement activation and inflammation.

Solution for Question 7:


Correct Option B - Delayed-type hypersensitivity:
• Contact dermatitis is a type IV hypersensitivity reaction.
• Common triggers are poison ivy/oak, nickel dyes, topical medication, and skin care products.
• Patients with pruritic, erythematous plaque, vesicles, and bullae developed a few days after contact,
which triggers develop allergic contact dermatitis.
• In sensitized individuals, the rash develops within 4-96 hours of exposure.
• In previously unexposed individuals, it may not be seen for 3 weeks.
• The typical rash, as seen in this patient, is usually limited to exposed skin, frequently forming linear
streaks where the skin has brushed against plant leaves.
Incorrect Options:
Option A - Cytotoxic antibody production:
• Cytotoxic antibody production is seen in type II hypersensitivity reactions.
• Antibodies bind to cell-surface antigens or extracellular matrix cellular destruction, inflammation, and
cellular dysfunction.
Option C - Deposition of antigluten antibodies:
• Deposition of anti-gluten antibodies occurs in patients with dermatitis herpetiformis.
• Dermatitis herpetiformis is a type III (immune complex) hypersensitivity reaction. It relies on the
formation of immune complexes through the binding of antibodies (IgG and IgM) to antigens.
• These complexes activate the complement cascade and induce neutrophils to secrete lysosomal
enzymes.

Page 10

391
• In dermatitis herpetiformis, there is the deposition of antigen-antibody complexes along the
dermal-epidermal junction.
Option D - Deposition of circulating immune complexes:
• Deposition of circulating immune complexes occurs in type III hypersensitivity.
• The immune complex of antigen-antibody (mostly IgG) complexes activates complement, which
attracts neutrophils; neutrophils release lysosomal enzymes.
• It can be associated with vasculitis and systemic manifestations.
• Eg., Serum sickness and Arthus reaction

Solution for Question 8:


Correct Option D - Contact dermatitis:
• Contact dermatitis is a type IV hypersensitivity reaction.
• Common triggers are poison ivy/oak, nickel dyes, topical medication, and skin care products.
• Patients develop pruritic, erythematous plaques, vesicles, and bullae a few days after contact with
triggering allergens, indicative of allergic contact dermatitis.
• In sensitized individuals, the rash develops within 4-96 hours of exposure.
• In previously unexposed individuals, it may not be seen for 3 weeks.
• The typical rash, as seen in this patient, is usually limited to exposed skin, frequently forming linear
streaks where the skin has brushed against plant leaves.
Incorrect Options:
Option A – Anaphylaxis:
• Anaphylaxis is a type I hypersensitivity reaction.
• Type I is mediated by IgE, and it flares up within minutes.
• The symptoms range from rashes to anaphylactic shock with vasodilation hypotension and
bronchiolar spasm.
• Foreign proteins (e.g., antisera), hormones, enzymes, polysaccharides, drugs (e.g., the antibiotic
penicillin), food allergens (e.g., peanuts, shell), or insect toxins can trigger in sensitized individuals.
Option B - Rheumatic fever:
• It is an immune-mediated type II hypersensitivity reaction.
• Antibodies to M protein cross-react with self-antigens, often myosin, due to molecular mimicry.
Option C - Myasthenia gravis:
• Myasthenia gravis is an example of a type II hypersensitivity reaction
• Caused by antibodies against the acetylcholine receptor.

Solution for Question 9:

Page 11

392
Correct Option C – IgE:
• Type 1 hypersensitivity is a rapid immunologic reaction to an allergen in already sensitised patients.
• IgE is the primary antibody associated with type 1 hypersensitivity.
• IgE antibodies cause mast cell degranulation, which results in Vascular dilation Oedema Smooth
muscle contraction Inflammation
• Vascular dilation
• Oedema
• Smooth muscle contraction
• Inflammation
• Pathogenesis - Pathogenesis begins with the activation of Th2 helper T cells.
• Dendritic cells detect allergens and present them to Th2 helper T-cells.
• Th2 helper T-cells produce IL-4, IL-5, and IL-13. IL - 4 (Act on B -cells to produce IgE) IL-5 (Activation
of eosinophils) IL-13 (Mucous production)
• IL - 4 (Act on B -cells to produce IgE)
• IL-5 (Activation of eosinophils)
• IL-13 (Mucous production)
• Th2 cells also produce chemokines, which attract leukocytes to the reaction site.
• Vascular dilation
• Oedema
• Smooth muscle contraction
• Inflammation
• IL - 4 (Act on B -cells to produce IgE)
• IL-5 (Activation of eosinophils)
• IL-13 (Mucous production)
Incorrect Options:
Option A – IgA:
• IgA protects from mucosal infections.
• IgA deficiency can lead to GI infections.
• It has no role in type 1 hypersensitivity
Option B – IgM:
• IgM is the first antibody secreted in case of acute infection
• Antigen, with the help of CD40L and helper T cells, helps produce different antibodies (IgG, IgA, IgE).
Option D – IgG:
• In the case of chronic infections, IgG antibodies are secreted.
• Plasma cells secrete these.
• It plays a role in Type 2 and 3 hypersensitivities.

Page 12

393
Solution for Question 10:
Correct Option B - Type 2 Hypersensitivity reaction:
• Guillain-Barré syndrome is primarily considered a Type 2 Hypersensitivity reaction.
• In GBS, the immune system mistakenly targets the peripheral nerves, leading to demyelination and
damage to the nerve fibers. The elevated protein in the cerebrospinal fluid (CSF) without an increase in
white blood cells is a characteristic finding in GBS. It is termed albuminocytologic dissociation. The
treatment with plasma exchange and intravenous immunoglobulin aims to modulate the immune
response and improve the patient's condition.
Incorrect Options:
Option A - Type 1 Hypersensitivity reaction:
• Type 1 hypersensitivity reactions involve IgE-mediated responses, typically associated with allergic
reactions (e.g., hay fever, asthma, anaphylaxis). Guillain-Barré syndrome is not a Type 1
hypersensitivity reaction.
Option C - Type 3 Hypersensitivity reaction:
• Type 3 hypersensitivity reactions involve immune complex-mediated diseases, where immune
complexes deposit in tissues and cause inflammation. Guillain-Barré syndrome is not primarily
mediated by immune complexes.
Option D - Type 4 Hypersensitivity reaction:
• Type 4 hypersensitivity reactions are cell-mediated immune responses, commonly associated with
delayed-type hypersensitivity (e.g., contact dermatitis, some autoimmune diseases). Guillain-Barré
syndrome is not a Type 4 hypersensitivity reaction; it involves antibodies attacking peripheral nerves

Solution for Question 11:


Correct Option A - Single electromyography (SFEMG):
• Myasthenia gravis is often diagnosed using the Tensilon test. However, it is most definitively
diagnosed with single-fibre electromyography, which shows a decremental response to motor nerve
stimulation
• SFEMG is a highly sensitive and specific test for diagnosing MG. It involves inserting a fine needle
electrode into a muscle to record the electrical activity at the neuromuscular junction. In MG, there is
typically an abnormal increase in jitter (variability in the time it takes for nerve impulses to reach the
muscle) seen on SFEMG. This test can be used to support the diagnosis, but it is not the primary test
for confirmation.
• Patients with myasthenia gravis present with features such as:
• Extraocular muscle weakness - ptosis and diplopia in the patient described in the vignette.
• Bulbar muscle weakness - Presents as difficulty chewing, choking, dysphagia, hoarse voice and
dysarthria.
• Limb weakness - Proximal muscles are more involved than distal muscles.

Page 13

394
• Myasthenic crisis - Weakness of intercostal muscles and diaphragm → worsening respiratory
function.
Patients with myasthenia gravis present with features such as:
Incorrect Options:
Option B - CT scan of the thorax:
• A CT scan of the thorax can rule out a thymoma. Only 10-20% of patients with myasthenia gravis
have a thymoma.
• A CT scan of the thorax may be used to evaluate the thymus gland in cases of suspected MG.
Thymoma (a tumor of the thymus gland) can be associated with MG, and imaging may help identify
thymic abnormalities. However, it is not a direct confirmatory test for MG.
Option C - Edrophonium (Tensilon) test:
• The use of Edrophonium inhibits AChE at the synapse, leading to increased concentration of
acetylcholine and, thereby, a marked improvement of symptoms. This test is limited by a high
false-positive rate.
• The Edrophonium test involves the administration of the acetylcholinesterase inhibitor edrophonium,
which temporarily increases the concentration of acetylcholine at the neuromuscular junction. If the
patient has MG, there is often a significant and rapid improvement in muscle strength and ptosis
following the administration of edrophonium, which confirms the diagnosis.
Option D - Muscle biopsy:
• Muscle biopsies are typically not performed for myasthenia gravis. Biopsy results are often dependent
on technique and less specific than single fibre electromyography (SFEMG).
• Muscle biopsy is not a routine test for diagnosing MG. In MG, the characteristic defect is at the
neuromuscular junction, where the nerve signal fails to transmit to the muscle properly. Muscle biopsy
is more useful for evaluating other muscle disorders but is not a primary test for MG diagnosis.

Solution for Question 12:


Correct Option C - A mechanism for the persistence and evolution of autoimmune disease:
Mechanism of Autoimmunity
The diagram shows a B-cell picking up antigens from damaged tissue and presenting it to the T-cell. N
ormally the T-cell binding to such an antigen causes cell death to prevent immune response against th
e self-tissues of the body. However, in some cases, autoimmunity occurs due to a
loss of tolerance towards the self-tissues.
• Epitope spreading is defined as when an immune response against one self-antigen causes tissue
damage. This causes the release of other antigens and results in the activation of the lymphocytes by
the newly encountered epitopes. This causes autoimmune diseases.
• The microbial infections result in tissue necrosis and inflammation, leading to the stimulation of the
expression of costimulatory molecules on APCs in the tissue. As a result, the breakdown of T cell
tolerance occurs with subsequent T cell activation.
• A variety of microbes, including bacteria, mycoplasmas, and viruses, trigger autoimmunity. One of the
mechanisms of causing autoimmunity is by sharing cross-reacting epitopes with self-antigens, and as a
result, responses induced by the microbes may extend to self-tissues by a phenomenon called

Page 14

395
molecular mimicry. The best example is immunologic cross-reaction in rheumatic heart disease, in
which antibody response against streptococci cross-targets cardiac antigens.
Incorrect Options:
Option A - A type of mechanism of spread of malignant tumors:
• A carcinoma spreads by breaching the underlying basement membrane, then traverses the interstitial
connective tissues and ultimately gains access to the circulation by penetrating the vascular basement
membrane.
• The process shown in the diagram of question is of autoimmunity as it involves activation of T-cell
mediated response to the self-antigens causing further damage to the local tissues
Option B - A type of mechanism of HIV dissemination:
• HIV dissemination follows the mucosal infection. Dendritic cells (DCs) in epithelia at sites of virus
entry capture the virus and then migrate into the lymph nodes. Here the DCs pass HIV on to CD4+ T
cells. The replication occurs and leads to viremia, during which high numbers are present in the blood,
and the virus disseminates throughout the body and infects helper T cells, macrophages, and DCs.
• The image above shows a mechanism of autoimmunity that involves the B-cells causing the activation
of T-cells against self-antigens.
Option D - One of the mechanisms of apoptosis:
• Apoptosis occurs in an extrinsic or intrinsic manner. The extrinsic or death receptor pathway involves
the interaction of the Fas ligand found on activated T lymphocytes with its receptor (Fas L death
receptor). As a result, caspases are recruited and activated, causing cell death. Such a pathway is
used in the elimination of self-reactive lymphocytes and in killing target cells by cytotoxic T cells.
• The process shown by the diagram is an immune response to self-tissues, i.e. autoimmunity, by
activating T-helper cells to induce B cell proliferation and subsequent antibody-mediated damage to
self-tissues.

Solution for Question 13:


Correct Option D - Complement- and Fc receptor-mediated inflammation in the kidney glomeruli:
• Goodpasture syndrome is characterized by the presence of autoantibodies directed against
noncollagenous protein in the basement membranes of kidney glomeruli and lung alveoli.
• These autoantibodies lead to complement activation and subsequent inflammation, as well as Fc
receptor-mediated responses in the affected tissues.
• In the kidney, the complement system is activated, leading to inflammation and damage to the
glomerular basement membrane, resulting in glomerulonephritis.
• Therefore, the correct mechanism underlying the pathophysiology of Goodpasture syndrome involves
complement- and Fc receptor-mediated inflammation in the kidney glomeruli.
Incorrect Options:
Option A – Opsonization and phagocytosis of renal tubular cells:
• This option describes a mechanism more commonly associated with autoimmune hemolytic anemia
(AIHA), where autoantibodies target red blood cells leading to their opsonization and phagocytosis,
resulting in hemolysis.

Page 15

396
• In Goodpasture syndrome, the target antigen is found in the basement membrane of the kidney
glomeruli and lung alveoli, not in renal tubular cells.
Option B - Antibody-mediated activation of proteases, leading to disruption of intercellular adhesions:
• This option describes a mechanism more characteristic of Pemphigus Vulgaris, where autoantibodies
against proteins in intercellular junctions of epidermal cells lead to disruption of cell adhesions, resulting
in skin vesicles (bullae).
• In Goodpasture syndrome, autoantibodies target the basement membrane proteins in the kidneys and
lungs, leading to complement and Fc receptor-mediated inflammation, not antibody-mediated activation
of proteases.
Option C - Neutrophil degranulation and inflammation within the glomerular capillaries:
• This option describes a mechanism more typical of vasculitis caused by antineutrophil cytoplasmic
antibodies (ANCA), where neutrophil granule proteins, presumably released from activated neutrophils,
lead to inflammation in the blood vessel walls.
• In Goodpasture syndrome, inflammation occurs in the kidney glomeruli due to complement and Fc
receptor-mediated responses, rather than neutrophil degranulation within glomerular capillaries.

Page 16

397
Tolerance and Autoimmune Disorder
1. A 19-year-old woman comes in complaining of malaise, joint pain, weight loss, and intermittent
fever(101°F)for three months. Physixal examination shows malar rash and mouth ulcers. She has
significant proteinuria and a decline in GFR. Which of the following antibodies are likely present in this
patient's serum?
A. C-ANCA (anti-proteinase-3)
B. anti Double-stranded DNA
C. P-ANCA (anti-myeloperoxidase)
D. Rheumatoid factor
----------------------------------------
2. A 45-year-old woman has been complaining of severe headaches, swallowing difficulty, and small,
red lesions around her mouth for the past six months. Examination shows her skin is hardened and
thickened over the fingers and appear shiny and smooth. She also complains of bluish fingernails when
exposed to cold. Which of the following antigens is associated with this patient’s disease?
(or)
Which antigens are used to diagnose scleroderma most frequently and specifically?
A. C-ANCA (anti-proteinase-3)
B. Double-stranded DNA
C. P-ANCA (anti-myeloperoxidase)
D. Scl-70 (topoisomerase I)
----------------------------------------
3. A 45-year-old woman with systemic lupus erythematosus complains of red, frothy urine for two
months despite being on treatment. Urinalysis shows RBC's and protein. An image from the renal
biopsy is given below. Which of the following accurately describes the histopathological findings?
(or)
Which of the following accurately describes the histopathological findings shown below in a patient with
Lupus Nephritis?

A. Capillary wall thickening


B. Basement membrane thickening
C. Sub-epithelial deposits

398
D. Sclerosis of mesangium
----------------------------------------
4. Which of the following most accurately describes the progression of Grave's Disease?
A. Antibody-dependent cellular cytotoxicity
B. Immune Complex–Mediated disease
C. Delayed-type hypersensitivity
D. Immediate hypersensitivity
----------------------------------------
5. A 50-year-old man complains of shortness of breath with exertion. Blood pressure is 110/90mmHg,
pulse is 90/minute, and SpO2 is 93%. He has history of multiple joint pains and a facial rash. His lab
tests are positive for anti-ds DNA. Chest X-rays show reduced lung volume. What is the most likely
cause of the patient's symptoms?
A. SLE
B. Wegner Granulomatosis
C. Scleroderma
D. Sarcoidosis
----------------------------------------
6. Which antibodies are more specific for Sjogren's syndrome?
A. Anti-Ro
B. Anti La
C. Anti-Scl 70
D. Anti-U1 RNP
----------------------------------------
7. A 30-year-old woman presents with sun-exposed rash, joint pain, shortness of breath, oral ulcers,
and hair thinning. On examination: facial rash, painless oral ulcers, conjunctival pallor, and hair thinning
are seen. Which antibodies are most specific for the disease?
(or)
Which of the following is the most specific antibody in Systemic Lupus Erythematosus?

A. Anti-dsDNA antibodies

Page 2

399
B. Anti-U1-RNP
C. Anti-CCP antibodies
D. Anti-DNA topoisomerase I antibody
----------------------------------------
8. Which of the following statements regarding polymyositis is most accurate?
A. Polymyositis is characterized by CD4+ T cell infiltration in affected muscle tissues.
B. The pathogenesis of polymyositis involves significant vascular injury.
C. Polymyositis is typically associated with distinct cutaneous features.
D. Inflammatory infiltrates in polymyositis are predominantly endomysial in location.
----------------------------------------
9. A 32-year-old woman presented to the physician's office complaining of low-grade fever, loss of
appetite, fatigue, and myalgias for the past 3 months. On physical examination findings are shown
below. Which of the following is not included in the criteria for diagnosis of the most likely disease?

A. Oral ulcers
B. Psychosis
C. Discoid rash
D. Leukocytosis
----------------------------------------
10. A 52-year-old female patient came with dysphagia, heartburn, and pain in finger joints. Her fingers
and toes turn blue on exposure to cold. Examination reveals thickened skin of her fingers and tender,
painful small lumps under the skin of the flexor surface of her hand, some of which drain white chalky
substance. What is the diagnosis?
A. Systemic lupus erythematosus
B. Sjögren Syndrome
C. CREST syndrome
D. Mikulicz syndrome
----------------------------------------
11. A 26-year-old woman prsents with fatigue, weight loss, and bilateral joint pain in her distal
extremities. She has an erythematous rash over her cheeks that worsens with sun exposure. ANA and
anti-dsDNA are positive. Which of the following is the most likely cause of the patient’s condition?

Page 3

400
A. Autoimmune mediated deposition of collagen
B. Disordered B cell maturation
C. Mutation of CD40L on helper T cells
D. Type III hypersensitivity reaction
----------------------------------------
12. A 45-year-old woman presented with a 4-month history of severe headache, pain, and blanching of
hands when exposed to the cold and increasing dysphagia to solids. She does not smoke. Physical
examination shows smooth and firm skin over the face and fingers. Anti-Scl 70 (anti-topoisomerase) is
positive. Which of the following terms best describes this patient's painful hands?

A. Chilblains
B. Homans’ sign
C. Intermittent claudication
D. Raynaud phenomenon
----------------------------------------

Correct Answers
Question Correct Answer

Question 1 2
Question 2 4
Question 3 1
Question 4 1
Question 5 1
Question 6 2
Question 7 1
Question 8 4
Question 9 4
Question 10 3
Question 11 4

Page 4

401
Question 12 4

Solution for Question 1:


Correct Option B - anti Double-stranded DNA:
• SLE is an autoimmune disease that involves various organs. This disease is characterized by an array
of antinuclear antibodies. The characteristic findings in this disease are due to cellular injury by these
antibodies and the deposition of various immune complexes. The hallmark autoantibodies in patients
with this disease are antibodies to double-stranded DNA and the Smith antigen. Presentation is
diverse and patients present with different features depending on the severity of the disease. Some of
the classical clinical features are:
• Constitutional symptoms such as low-grade fever, malaise, fatigue, weight loss
• Joint pains
• Malar rash, Discoid rash
• Alopecia
• Oral or nasopharyngeal ulcers
• Joint disease
• Findings of pleuritis and pericarditis. ( An easy way to remember the systemic manifestations of SLE
is to think of Synovitis, Pleuritis, Pericarditis)
• Persistent proteinuria with decreasing renal function
• Hemolytic anemia, Leukopenia/Lymphopenia and Thrombocytopenia.
SLE is an autoimmune disease that involves various organs. This disease is characterized by an array
of antinuclear antibodies. The characteristic findings in this disease are due to cellular injury by these a
ntibodies and the deposition of various immune complexes.
The hallmark autoantibodies in patients with this disease are antibodies to double-stranded DNA and t
he Smith antigen.
Presentation is diverse and patients present with different features depending on the severity of the dis
ease. Some of the classical clinical features are:
Incorrect Options:
Option A - C-ANCA (anti-proteinase-3):
• c-ANCA, also known as anti-proteinase-3, is a cytoplasmic granule.
• It is associated with granulomatosis with polyangiitis.
c-ANCA, also known as anti-proteinase-3, is a cytoplasmic granule.
It is associated with granulomatosis with polyangiitis.
Option C - P-ANCA (anti-myeloperoxidase):
• P-ANCA is a lysosomal granule also known as anti-myeloperoxidase.
• It has been linked to Churg-Strauss syndrome and microscopic polyangiitis.
P-ANCA is a lysosomal granule also known as anti-myeloperoxidase.
It has been linked to Churg-Strauss syndrome and microscopic polyangiitis.
Option D - Rheumatoid factor:

Page 5

402
• Antibodies against rheumatic factors are a defining feature of rheumatic disease.
• Rheumatoid arthritis is an autoimmune disease that primarily affects the joints.
Antibodies against rheumatic factors are a defining feature of rheumatic disease.
Rheumatoid arthritis is an autoimmune disease that primarily affects the joints.

Solution for Question 2:


Correct Option D - Scl-70 (topoisomerase I):
• The likely diagnosis in this patient is limited scleroderma. The patient is exhibiting features of CREST
syndrome.
• Scl - 70 (topoisomerase I) is a common antigen associated with scleroderma.
• It is characterised by: Chronic inflammation Skin contains Th2 cells that produce cytokines. TGF-beta
and IL 13 induce the synthesis of collagen and other proteins13. Additionally, cytokines attract
neutrophils and induce inflammation. Damage to the blood vessel Endothelial cell injury causes the
release of von Willebrand factor and PDGF, which causes platelets to clump together. Digital arteries
show intimal proliferation. Capillary dilation and leakage Perivascular fibrosis develops as a result of
repeated cycles of injury. Fibrosis of interstitial and perivascular connective tissue Intrinsic fibroblast
abnormalities cause excessive collagen production. Scarring after an injury is caused by the activation
of macrophages, cytokines, leukocytes, and fibroblasts.
• Chronic inflammation Skin contains Th2 cells that produce cytokines. TGF-beta and IL 13 induce the
synthesis of collagen and other proteins13. Additionally, cytokines attract neutrophils and induce
inflammation.
• Skin contains Th2 cells that produce cytokines.
• TGF-beta and IL 13 induce the synthesis of collagen and other proteins13.
• Additionally, cytokines attract neutrophils and induce inflammation.
• Damage to the blood vessel Endothelial cell injury causes the release of von Willebrand factor and
PDGF, which causes platelets to clump together. Digital arteries show intimal proliferation. Capillary
dilation and leakage Perivascular fibrosis develops as a result of repeated cycles of injury.
• Endothelial cell injury causes the release of von Willebrand factor and PDGF, which causes platelets
to clump together.
• Digital arteries show intimal proliferation.
• Capillary dilation and leakage
• Perivascular fibrosis develops as a result of repeated cycles of injury.
• Fibrosis of interstitial and perivascular connective tissue Intrinsic fibroblast abnormalities cause
excessive collagen production. Scarring after an injury is caused by the activation of macrophages,
cytokines, leukocytes, and fibroblasts.
• Intrinsic fibroblast abnormalities cause excessive collagen production.
• Scarring after an injury is caused by the activation of macrophages, cytokines, leukocytes, and
fibroblasts.
• Anti-nuclear antibodies (ANA) associated with systemic sclerosis are anti-Scl -70 ((antitopoisomerase
I) - highly specific 10 - 20% of patients with diffuse systemic sclerosis Anticentromere antibody 20 -

Page 6

403
30% of patients with CREST syndrome
• anti-Scl -70 ((antitopoisomerase I) - highly specific 10 - 20% of patients with diffuse systemic sclerosis
• 10 - 20% of patients with diffuse systemic sclerosis
• Anticentromere antibody 20 - 30% of patients with CREST syndrome
• 20 - 30% of patients with CREST syndrome
• The condition can be divided into two categories. Diffuse systemic sclerosis CREST syndrome
• Diffuse systemic sclerosis
• CREST syndrome
• Diffuse systemic sclerosis Widespread involvement of multiple organs and skin Early visceral
involvement
• Widespread involvement of multiple organs and skin
• Early visceral involvement
• Limited Scleroderma - It is also known as CREST syndrome Calcinosis Raynaud phenomenon
esophageal dysmotility Sclerodactyly Telangiectasia
• Calcinosis
• Raynaud phenomenon
• esophageal dysmotility
• Sclerodactyly
• Telangiectasia
The likely diagnosis in this patient is limited scleroderma. The patient is exhibiting features of CREST s
yndrome.
Scl - 70 (topoisomerase I) is a common antigen associated with scleroderma.
It is characterised by:
• Chronic inflammation Skin contains Th2 cells that produce cytokines. TGF-beta and IL 13 induce the
synthesis of collagen and other proteins13. Additionally, cytokines attract neutrophils and induce
inflammation.
• Skin contains Th2 cells that produce cytokines.
• TGF-beta and IL 13 induce the synthesis of collagen and other proteins13.
• Additionally, cytokines attract neutrophils and induce inflammation.
• Damage to the blood vessel Endothelial cell injury causes the release of von Willebrand factor and
PDGF, which causes platelets to clump together. Digital arteries show intimal proliferation. Capillary
dilation and leakage Perivascular fibrosis develops as a result of repeated cycles of injury.
• Endothelial cell injury causes the release of von Willebrand factor and PDGF, which causes platelets
to clump together.
• Digital arteries show intimal proliferation.
• Capillary dilation and leakage
• Perivascular fibrosis develops as a result of repeated cycles of injury.

Page 7

404
• Fibrosis of interstitial and perivascular connective tissue Intrinsic fibroblast abnormalities cause
excessive collagen production. Scarring after an injury is caused by the activation of macrophages,
cytokines, leukocytes, and fibroblasts.
• Intrinsic fibroblast abnormalities cause excessive collagen production.
• Scarring after an injury is caused by the activation of macrophages, cytokines, leukocytes, and
fibroblasts.
Chronic inflammation
• Skin contains Th2 cells that produce cytokines.
• TGF-beta and IL 13 induce the synthesis of collagen and other proteins13.
• Additionally, cytokines attract neutrophils and induce inflammation.
Skin contains Th2 cells that produce cytokines.
TGF-beta and IL 13 induce the synthesis of collagen and other proteins13.
Additionally, cytokines attract neutrophils and induce inflammation.
Damage to the blood vessel
• Endothelial cell injury causes the release of von Willebrand factor and PDGF, which causes platelets
to clump together.
• Digital arteries show intimal proliferation.
• Capillary dilation and leakage
• Perivascular fibrosis develops as a result of repeated cycles of injury.
Endothelial cell injury causes the release of von Willebrand factor and PDGF, which causes platelets to
clump together.
Digital arteries show intimal proliferation.
Capillary dilation and leakage
Perivascular fibrosis develops as a result of repeated cycles of injury.
Fibrosis of interstitial and perivascular connective tissue
• Intrinsic fibroblast abnormalities cause excessive collagen production.
• Scarring after an injury is caused by the activation of macrophages, cytokines, leukocytes, and
fibroblasts.
Intrinsic fibroblast abnormalities cause excessive collagen production.
Scarring after an injury is caused by the activation of macrophages, cytokines, leukocytes, and fibrobla
sts.
Anti-nuclear antibodies (ANA) associated with systemic sclerosis are
• anti-Scl -70 ((antitopoisomerase I) - highly specific 10 - 20% of patients with diffuse systemic sclerosis
• 10 - 20% of patients with diffuse systemic sclerosis
• Anticentromere antibody 20 - 30% of patients with CREST syndrome
• 20 - 30% of patients with CREST syndrome
anti-Scl -70 ((antitopoisomerase I) - highly specific
• 10 - 20% of patients with diffuse systemic sclerosis

Page 8

405
10 - 20% of patients with diffuse systemic sclerosis
Anticentromere antibody
• 20 - 30% of patients with CREST syndrome
20 - 30% of patients with CREST syndrome
The condition can be divided into two categories.
• Diffuse systemic sclerosis
• CREST syndrome
Diffuse systemic sclerosis
CREST syndrome
Diffuse systemic sclerosis
• Widespread involvement of multiple organs and skin
• Early visceral involvement
Widespread involvement of multiple organs and skin
Early visceral involvement
Limited Scleroderma - It is also known as CREST syndrome
• Calcinosis
• Raynaud phenomenon
• esophageal dysmotility
• Sclerodactyly
• Telangiectasia
Calcinosis
Raynaud phenomenon
esophageal dysmotility
Sclerodactyly
Telangiectasia
Incorrect Options:
Option A - C-ANCA (anti-proteinase-3):
• C-ANCA is a granular protein present in the cytoplasm.
• It is associated with granulomatosis with polyangiitis.
C-ANCA is a granular protein present in the cytoplasm.
It is associated with granulomatosis with polyangiitis.
Option B - Double-stranded DNA:
• Anti-double-stranded DNA is the hallmark of SLE.
Anti-double-stranded DNA is the hallmark of SLE.
Option C - P-ANCA (anti-myeloperoxidase):

Page 9

406
• The lysosomal granule p-ANCA is also known as anti-myeloperoxidase.
• It is linked to Churg-Strauss syndrome and microscopic polyangiitis and contributes to forming free
oxygen radicals.
The lysosomal granule p-ANCA is also known as anti-myeloperoxidase.
It is linked to Churg-Strauss syndrome and microscopic polyangiitis and contributes to forming free oxy
gen radicals.

Solution for Question 3:


Correct Option A - Capillary wall thickening:
Lupus Nephritis

• Biopsy histological findings show glomeruli with proliferation of endothelial, mesangial, and epithelial
cells forming crescents filling the Bowman’s space. Subendothelial immune complex deposits create a
circumferential thickening of the capillary wall forming a wire-loop pattern seen in lupus nephritis.
• 50% of SLE patients have clinically significant renal involvement
• Currently, there are six patterns of glomerular disease associated with SLE, where Class 1 (Minimal
mesangial lupus nephritis) is the least common, and class 4 (Diffuse lupus nephritis) is the most
common.
• Minimal mesangial lupus nephritis (class I): Immune complex deposition in the mesangium
• Mesangial proliferative lupus nephritis (class II): Mesangial cell proliferation accompanied by
accumulation of mesangial matrix and granular mesangial deposits of immunoglobulin and complement
without the involvement of glomerular capillaries
• Focal lupus nephritis (class III): Involves fewer than 50% of all glomeruli and may be segmental (part
of glomerulus) or global (entire glomerulus). Glomeruli exhibit swelling and proliferation of the
endothelial and mesangial cells associated with leukocyte accumulation, capillary necrosis and hyaline
thrombi; also, there are extra capillary proliferations associated with focal necrosis and crescent
formation.

Page 10

407
• Diffuse lupus nephritis (class IV): identical to III but differ in extent. The glomeruli show proliferation of
endothelial, mesangial and epithelial cells showing crescents filling the Bowman’s space.
Subendothelial immune complex deposits create a circumferential thickening of the capillary wall
forming a wire-loop pattern. This type is most commonly seen in patients with SLE.

Incorrect Options:
Option B - Basement membrane thickening: Membranous lupus nephritis (class V): Diffuse thickening
of the capillary walls due to sub-epithelial immune complex deposits. In addition to immune complex de
position, basement membrane-like structure is increasingly deposited, leading to the thickening of the c
apillary walls.
Option C - Sub-epithelial deposits: Membranous lupus nephritis (class V): Subepithelial immune compl
ex deposits cause diffuse thickening of the capillary walls. Along with immune complex formation, a
structure resembling a foundation membrane is steadily formed, thickening the capillary walls.
Option D - Sclerosis of mesangium: Advanced sclerosing lupus nephritis (class VI): characterised by sc
lerosis of more than 90% of the glomeruli representing end-stage disease.

Solution for Question 4:


Correct Option A - Antibody-dependent cellular cytotoxicity:
• Graves disease is a Type II hypersensitivity reaction.
• Antibody-dependent cellular cytotoxicity is a mechanism of type II hypersensitivity.
• Antibodies bind to TSH receptors and stimulate thyroid hormone production.
• Labs of patients with graves disease are High T3 and T4 Low TSH
• High T3 and T4
• Low TSH
• Symptoms of graves disease are Sweating Weight loss Tachycardia Exophthalmos
• Sweating

Page 11

408
• Weight loss
• Tachycardia
• Exophthalmos
• TYPE II hypersensitivity is caused by an antigen-antibody reaction in the extracellular matrix.
Antibody-mediated cellular dysfunction Antibodies bind to surface receptors, causing them to
malfunction. Graves disease is an example of TSH receptor antibodies stimulating T3 and T4
production.
• Antibodies bind to surface receptors, causing them to malfunction.
• Graves disease is an example of TSH receptor antibodies stimulating T3 and T4 production.
Graves disease is a Type II hypersensitivity reaction.
Antibody-dependent cellular cytotoxicity is a mechanism of type II hypersensitivity.
Antibodies bind to TSH receptors and stimulate thyroid hormone production.
Labs of patients with graves disease are
• High T3 and T4
• Low TSH
High T3 and T4
Low TSH
Symptoms of graves disease are
• Sweating
• Weight loss
• Tachycardia
• Exophthalmos
Sweating
Weight loss
Tachycardia
Exophthalmos
TYPE II hypersensitivity is caused by an antigen-antibody reaction in the extracellular matrix.
• Antibodies bind to surface receptors, causing them to malfunction.
• Graves disease is an example of TSH receptor antibodies stimulating T3 and T4 production.
Antibodies bind to surface receptors, causing them to malfunction.
Graves disease is an example of TSH receptor antibodies stimulating T3 and T4 production.
Incorrect Options:
Option B - Immune Complex–Mediated disease:
• Type III hypersensitivity is characterised by immune complex-mediated disease.
• It results in the accumulation of antigen-antibody complexes in tissues.
Type III hypersensitivity is characterised by immune complex-mediated disease.

Page 12

409
It results in the accumulation of antigen-antibody complexes in tissues.
Option C - Delayed-type hypersensitivity:
• Type IV hypersensitivity is also delayed hypersensitivity.
• It is mediated by CD4 and CD8 T lymphocytes.
Type IV hypersensitivity is also delayed hypersensitivity.
It is mediated by CD4 and CD8 T lymphocytes.
Option D - Immediate hypersensitivity:
• Type I hypersensitivity is immediate hypersensitivity.
• It causes the release of IgE antibodies, which results in mast cell degranulation.
Type I hypersensitivity is immediate hypersensitivity.
It causes the release of IgE antibodies, which results in mast cell degranulation.

Solution for Question 5:


Correct Option A - SLE:
• It is an autoimmune disease that affects several organs in the body.
• Although 50% of SLE patients have lung involvement, these changes are not unique to SLE.
Interstitial fibrosis Secondary hypertension Pleuritis Pleural effusion
• Interstitial fibrosis
• Secondary hypertension
• Pleuritis
• Pleural effusion
• Other organs affected by SLE are Blood vessels (necrotising vasculitis) Kidney (lupus nephritis) Skin
(butterfly rash, urticaria, maculopapular lesions) Cytopenia Joints (non-erosive synovitis)
Cardiovascular (pericarditis, endocarditis) Spleen (thickening of the capsule and follicular hyperplasia)
• Blood vessels (necrotising vasculitis)
• Kidney (lupus nephritis)
• Skin (butterfly rash, urticaria, maculopapular lesions)
• Cytopenia
• Joints (non-erosive synovitis)
• Cardiovascular (pericarditis, endocarditis)
• Spleen (thickening of the capsule and follicular hyperplasia)
• Antibodies associated with SLE: Anti-double-stranded DNA DNA - anti DNA antibodies complexes
deposits in different organs (type III hypersensitivity) Antibodies directed at blood cells lead to
cytopenia Antiphospholipid antibody leads to thrombosis and multiple miscarriages in females.
• Anti-double-stranded DNA DNA - anti DNA antibodies complexes deposits in different organs (type III
hypersensitivity)

Page 13

410
• DNA - anti DNA antibodies complexes deposits in different organs (type III hypersensitivity)
• Antibodies directed at blood cells lead to cytopenia
• Antiphospholipid antibody leads to thrombosis and multiple miscarriages in females.
It is an autoimmune disease that affects several organs in the body.
Although 50% of SLE patients have lung involvement, these changes are not unique to SLE.
• Interstitial fibrosis
• Secondary hypertension
• Pleuritis
• Pleural effusion
Interstitial fibrosis
Secondary hypertension
Pleuritis
Pleural effusion
Other organs affected by SLE are
• Blood vessels (necrotising vasculitis)
• Kidney (lupus nephritis)
• Skin (butterfly rash, urticaria, maculopapular lesions)
• Cytopenia
• Joints (non-erosive synovitis)
• Cardiovascular (pericarditis, endocarditis)
• Spleen (thickening of the capsule and follicular hyperplasia)
Blood vessels (necrotising vasculitis)
Kidney (lupus nephritis)
Skin (butterfly rash, urticaria, maculopapular lesions)
Cytopenia
Joints (non-erosive synovitis)
Cardiovascular (pericarditis, endocarditis)
Spleen (thickening of the capsule and follicular hyperplasia)
Antibodies associated with SLE:
• Anti-double-stranded DNA DNA - anti DNA antibodies complexes deposits in different organs (type III
hypersensitivity)
• DNA - anti DNA antibodies complexes deposits in different organs (type III hypersensitivity)
• Antibodies directed at blood cells lead to cytopenia
• Antiphospholipid antibody leads to thrombosis and multiple miscarriages in females.
Anti-double-stranded DNA
• DNA - anti DNA antibodies complexes deposits in different organs (type III hypersensitivity)

Page 14

411
DNA - anti DNA antibodies complexes deposits in different organs (type III hypersensitivity)
Antibodies directed at blood cells lead to cytopenia
Antiphospholipid antibody leads to thrombosis and multiple miscarriages in females.
Incorrect Options:
Option B - Wegner Granulomatosis:
• It is a vasculitis of the small blood vessels that affect the ENT, lungs, and kidneys.
• It causes a granulomatous lung reaction, cavitation and ground glass opacities.
It is a vasculitis of the small blood vessels that affect the ENT, lungs, and kidneys.
It causes a granulomatous lung reaction, cavitation and ground glass opacities.
Option C - Scleroderma:
• Systemic sclerosis is also known as Scleroderma.
• Limited scleroderma is the most common type of scleroderma.The symptoms include calcinosis,
Raynaud's phenomenon, esophageal dysmotility, sclerodactyly, and telangiectasia.
Systemic sclerosis is also known as Scleroderma.
Limited scleroderma is the most common type of scleroderma.The symptoms include calcinosis, Rayn
aud's phenomenon, esophageal dysmotility, sclerodactyly, and telangiectasia.
Option D - Tuberculosis:
• Tuberculosis is a caseating granulomatous infection caused by mycobacteria.
• It causes cavitary pulmonary lesions
Tuberculosis is a caseating granulomatous infection caused by mycobacteria.
It causes cavitary pulmonary lesions

Solution for Question 6:


Correct Option B - Anti La:
• Among the options provided, anti-La antibodies (also known as anti-SS-B antibodies) are more
specific for the diagnosis of Sjögren's syndrome. These antibodies are commonly detected in patients
with Sjögren's syndrome and help confirm the diagnosis.
Incorrect Options:
Option A - Anti-Ro:
• While anti-Ro antibodies are commonly associated with Sjögren's syndrome, they are more
sensitive compared to anti-La antibodies.
Option C - Anti-Scl 70:
• Anti-Scl 70 antibodies are associated with diffuse systemic sclerosis (scleroderma) and are not
specific for Sjögren's syndrome.
Option D - Anti-U1 RNP:
• Anti-U1 RNP antibodies are associated with mixed connective tissue disease (MCTD),

Page 15

412
Solution for Question 7:
Correct Option A- Anti-dsDNA antibodies:
SLE
• SLE is an autoimmune disease characterized by both type 2 (antibody-mediated cytotoxicity) and type
3 (antibody-antigen complex-mediated) hypersensitivity reactions.
• It affects women more commonly than men and the age group 17 to 55 years
• The patient presents with multiple system involvement, and "SOAP BRAIN MD" is a mnemonic used
to remember the clinical features of systemic lupus erythematosus (SLE) : S: Serositis (e.g., pleuritis,
pericarditis) O: Oral ulcers A: Arthritis P: Photosensitivity B: Blood disorders (e.g., anemia, leukopenia)
R: Renal involvement (e.g., lupus nephritis) A: Antinuclear antibodies (ANA) positive I: Immunologic
phenomena (e.g., positive anti-dsDNA, anti-Sm antibodies) N: Neurologic disorders (e.g., seizures,
psychosis) MD: Malar rash (butterfly rash) and Discoid rash
• S: Serositis (e.g., pleuritis, pericarditis)
• O: Oral ulcers
• A: Arthritis
• P: Photosensitivity
• B: Blood disorders (e.g., anemia, leukopenia)
• R: Renal involvement (e.g., lupus nephritis)
• A: Antinuclear antibodies (ANA) positive
• I: Immunologic phenomena (e.g., positive anti-dsDNA, anti-Sm antibodies)
• N: Neurologic disorders (e.g., seizures, psychosis)
• MD: Malar rash (butterfly rash) and Discoid rash
• Auto-antibodies in Systemic Lupus Erythematosus (SLE) AntidsDNA - (Diagnositic and most specific)
Anti-U1-RNP - Mainly found in all mixed connective tissue diseases. Anti-smith (SM) - Specific Anti-Ro
(SS-A) nucleoprotein - Seen in Congenital heart block and neonatal lupus Antiphospholipid protein
complexes - Antiphospholipid syndrome (seen in ~ 10% SLE patients)
• AntidsDNA - (Diagnositic and most specific)
• Anti-U1-RNP - Mainly found in all mixed connective tissue diseases.
• Anti-smith (SM) - Specific
• Anti-Ro (SS-A) nucleoprotein - Seen in Congenital heart block and neonatal lupus
• Antiphospholipid protein complexes - Antiphospholipid syndrome (seen in ~ 10% SLE patients)
• S: Serositis (e.g., pleuritis, pericarditis)
• O: Oral ulcers
• A: Arthritis
• P: Photosensitivity
• B: Blood disorders (e.g., anemia, leukopenia)

Page 16

413
• R: Renal involvement (e.g., lupus nephritis)
• A: Antinuclear antibodies (ANA) positive
• I: Immunologic phenomena (e.g., positive anti-dsDNA, anti-Sm antibodies)
• N: Neurologic disorders (e.g., seizures, psychosis)
• MD: Malar rash (butterfly rash) and Discoid rash
• AntidsDNA - (Diagnositic and most specific)
• Anti-U1-RNP - Mainly found in all mixed connective tissue diseases.
• Anti-smith (SM) - Specific
• Anti-Ro (SS-A) nucleoprotein - Seen in Congenital heart block and neonatal lupus
• Antiphospholipid protein complexes - Antiphospholipid syndrome (seen in ~ 10% SLE patients)
Incorrect Options:
Option B- Anti-U1-RNP:
• Anti-RNP is mainly found in all mixed connective tissue disorders.
• They are not specific to SLE but are found in SLE and other conditions such as systemic sclerosis.
Option C- Anti-CCP antibodies:
• Anti-CCP antibodies are antibodies against citrullinated peptides and are found in rheumatoid arthritis.
• They are considered as most specific for the diagnosis of Rheumatoid arthritis.
Option D- Anti-DNA topoisomerase I antibody:
• Anti-Scl-70 (anti-topoisomerase I antibody): associated with severe and rapidly progressive diffuse
Systemic Sclerosis, limited Systemic Sclerosis, Interstitial lung disease, digital ulcers.
• This patient has SLE, defined by the presence of malar rash, joint involvement and photosensitivity,
diffuse hair loss and DNA topoisomerase antibodies are not seen in SLE.

Solution for Question 8:


Correct Option D - Inflammatory infiltrates in polymyositis are predominantly endomysial in location:
• This statement accurately reflects the histological findings in polymyositis, where mononuclear
inflammatory cell infiltrates are typically located in the endomysium of affected muscle tissues.
• This distinguishes polymyositis from dermatomyositis, where the inflammatory infiltrates are
perifascicular in distribution.
Incorrect Options:
Option A - Polymyositis is characterized by CD4+ T cell infiltration in affected muscle tissues:
• This statement is incorrect. In polymyositis, the predominant inflammatory infiltrates consist of CD8+
cytotoxic T cells, not CD4+ T cells.
• CD8+ T cells are believed to be the mediators of tissue damage in polymyositis.
Option B - The pathogenesis of polymyositis involves significant vascular injury:

Page 17

414
• This statement is incorrect. Unlike dermatomyositis, vascular injury does not play a major role in the
pathogenesis of polymyositis.
Option C - Polymyositis is typically associated with distinct cutaneous features:
• This statement is incorrect. Unlike dermatomyositis, which is characterized by distinctive cutaneous
manifestations such as heliotrope rash and Gottron's papules, polymyositis lacks specific cutaneous
features.
• It primarily presents with myalgia and weakness, making it a diagnosis of exclusion when
dermatomyositis features are absent.

Solution for Question 9:


Correct Option D- Leukocytosis:
SLE
• SLE is an autoimmune disease involving multiple organs and is characterized by a vast array of
auto-antibodies which cause injury mainly by deposition of immune complexes.
• Leukocytosis is defined as an elevation in the leukocyte levels in the blood. It is a feature of
inflammatory reactions, especially in response to bacterial infections.
• However, in SLE, instead of leukocytosis, leukopenia or lymphopenia is observed. Therefore,
leukocytosis is not a diagnostic criterion for SLE.
• The diagnosis of SLE is made by the following clinical and immunological criteria:
Criteria (2012) for diagnosis of SLE:
Clinical manifestations
• Skin Acute, subacute cutaneous lupus (photosensitive, malar, maculopapular, bullous rash) Chronic
cutaneous lupus (discoid lupus, panniculitis, lichen planus-like, hypertrophic verrucous, chilblains)
• Acute, subacute cutaneous lupus (photosensitive, malar, maculopapular, bullous rash)
• Chronic cutaneous lupus (discoid lupus, panniculitis, lichen planus-like, hypertrophic verrucous,
chilblains)
• Oral or nasal ulcers
• Nonscarring Alopecia
• Synovitis involving ≥ 2 joints characterized by tenderness, swelling and effusion
• Serositis (pleurisy characterized by pleural rub or pain, pericarditis characterized by pericardial friction
rub)
• Renal Persistent proteinuria ≥ 0.5 g/ 24 hours or RBC casts
• Persistent proteinuria ≥ 0.5 g/ 24 hours or RBC casts
• Neurologic Seizures, psychosis, mononeuritis, myelitis, peripheral or cranial neuropathies, acute
confusional state
• Seizures, psychosis, mononeuritis, myelitis, peripheral or cranial neuropathies, acute confusional
state
• Hemolytic anaemia

Page 18

415
• Leukopenia (< 4.0 x 109 cells/L) or Lymphopenia (< 1.5 x 109cells/L) on two or more occasions
• Thrombocytopenia (< 100 x 109 cells/L)
• Acute, subacute cutaneous lupus (photosensitive, malar, maculopapular, bullous rash)
• Chronic cutaneous lupus (discoid lupus, panniculitis, lichen planus-like, hypertrophic verrucous,
chilblains)
• Persistent proteinuria ≥ 0.5 g/ 24 hours or RBC casts
• Seizures, psychosis, mononeuritis, myelitis, peripheral or cranial neuropathies, acute confusional
state
Immunological manifestations
• Anti-dsDNA antibody
• Anti-Sm antibody
• Antiphospholipid (any of lupus anticoagulant, anti-cardiolipin, anti-β glycoprotein I)
• Low serum complement (C3, C4 or CH50)
• Positive direct Coombs test in the absence of hemolytic anemia
Incorrect Options:
Option A- Oral ulcers:
• Oral ulcers and nasopharyngeal ulcers are common clinical findings seen in SLE patients and are part
of the diagnostic criteria of SLE.
Option B- Psychosis:
• Psychosis is an important neurological complication of SLE and is a diagnostic criterion of SLE.
Option C- Discoid rash:
• Chronic lupus demonstrates discoid rash, panniculitis and other types of rashes which are important
diagnostic criteria of SLE.

Solution for Question 10:


Correct Option C - CREST syndrome:
The most likely diagnosis for the above given clinical scenario is CREST Syndrome.

CREST syndrome is a
subtype of systemic sclerosis (scleroderma) characterized by the following clinical features:
• Calcinosis: Formation of calcium deposits in the skin, which may lead to the formation of tender,
painful lumps that can drain a white chalky substance.
• Raynaud's phenomenon: Fingers and toes turning blue or white in response to cold temperatures or
stress.
• Esophageal dysfunction: Dysphagia and heartburn due to involvement of the esophagus.
• Sclerodactyly: Thickening and tightening of the skin of the fingers.

Page 19

416
• Telangiectasia: Small dilated blood vessels visible on the skin, particularly on the face and hands.
Incorrect Options:
Option A - Systemic lupus erythematosus:
• Systemic lupus erythematosus (SLE) is an autoimmune disease that can affect multiple organs and
systems in the body, including the skin, joints, kidneys, and cardiovascular system.
• While SLE can present with joint pain and skin manifestations, such as a malar rash, it typically does
not cause the specific findings described in the patient's presentation, such as calcinosis, sclerodactyly,
and telangiectasia, which are more characteristic of systemic sclerosis (scleroderma) and CREST
syndrome.
Option B - Sjögren Syndrome:
• Sjögren syndrome is an autoimmune disorder characterized by dry eyes and mouth due to
inflammation and dysfunction of the salivary and lacrimal glands.
• While Sjögren syndrome can present with joint pain and Raynaud's phenomenon, it does not typically
cause the specific findings described in the patient's presentation, such as calcinosis, sclerodactyly,
and telangiectasia, which are more characteristic of systemic sclerosis (scleroderma) and CREST
syndrome.
Option D - Mikulicz syndrome:
• Mikulicz syndrome, also known as Mikulicz disease, is a rare condition characterized by symmetrical
swelling of the lacrimal and salivary glands, leading to dry eyes and mouth.
• While Mikulicz syndrome shares some features with Sjögren syndrome, such as glandular
involvement, it is not typically associated with the specific findings described in the patient's
presentation, such as calcinosis, sclerodactyly, and telangiectasia, which are more characteristic of
systemic sclerosis (scleroderma) and CREST syndrome.

Solution for Question 11:


Correct Option D- Type III hypersensitivity reaction:
• Systemic lupus erythematosus (SLE ) involves self-reactive lymphocytes producing antibodies to host
nuclear antigens, which form immune complexes that accumulate in tissues causing symptoms like a
malar rash. Thus the majority of the associated symptoms are categorized as type III hypersensitivity
reactions.
Incorrect Options:
Option A- Autoimmune mediated deposition of collagen:
• While systemic lupus erythematosus is an autoimmune disorder, it is not associated with the
deposition of collagen. Systemic sclerosis ( scleroderma ) is associated with autoimmune damage of
mesenchyme, leading to fibroblast activation and subsequent collagen deposition.
Option B- Disordered B cell maturation:
• Disordered B cell maturation is characteristic of X-linked agammaglobulinemia ( Bruton
agammaglobulinemia ), not systemic lupus erythematosus. Patients with this syndrome are commonly
characterized as infants with characteristic bacterial, enterovirus and Giardia lamblia infections after 6
months of life.
Option C- Mutation of CD40L on helper T cells:

Page 20

417
• CD40L on helper T cells binds to CD40 B cells, assisting in immunoglobulin class switching from IgM
to IgA, IgE and IgG. Mutation of CD40L or CD40 prevents immunoglobulin class switching, resulting in
excessive IgM production characteristic of hyper IgM syndrome. Hyper IgM Syndrome results in
recurrent pyogenic infections.

Solution for Question 12:


Correct Option D - Raynaud phenomenon:
• This affected person has scleroderma complicated by the Raynaud phenomenon.
• It refers to intermittent, bilateral assaults of vasospasm of the palms or toes (now and again affecting
the ears or nose), resulting in pallor, paresthesias, and pain from ischemia.
• The signs are precipitated by cold or emotional stimuli and relieved via way of means of heat.
• Raynaud phenomenon may also arise as an isolated disorder or a function of some systemic
diseases, together with systemic lupus erythematosus and scleroderma.
Incorrect Options:
Option A - Chilblains:
• Chilblains are tiny, irritating swellings on the skin caused by cold temperatures.
• They most commonly affect the body's extremities, such as the toes, fingers, heels, ears, and nose.
• Chilblains can be painful, but they seldom result in permanent harm.
Option B - Homans’ sign:
• Homan’s sign is a calf pain screening test to detect deep vein thrombosis (DVT). It is also known as
the dorsiflexion sign.
• The Homans test is carried out with the subject supine.
• The examiner raises the afflicted leg and swiftly dorsiflexes the patient's foot while extending the
knee.
• This procedure is done with the patient's knee flexing and the examiner palpating the calf
simultaneously.
Option C - Intermittent claudication:
• Intermittent claudication develops due to muscle ischemia produced by a restriction in arterial flow
during activity usually due to peripheral arterial disease.

Page 21

418
Immunodeficiency Disorders
1. Which of the following genetic defects is most commonly associated with X-linked severe combined
immunodeficiency (SCID)?
A. Mutation in adenosine deaminase (ADA) gene
B. Mutation in recombinase-activating genes (RAG)
C. Mutation in the common γ-chain (γc) subunit of cytokine receptors
D. Mutation in Janus kinase 3 (JAK3) gene
----------------------------------------
2. An 8-month-old infant has a history of recurring otitis media and pneumonia. Serum electrophoresis
reveals absent IgG and normal levels of T cells. His brother had a similar history of recurrent infections
and did not survive beyond early childhood. His parents and sisters all have normal IgG levels in their
blood. What is the most likely diagnosis?
A. DiGeorge syndrome
B. Isolated IgA deficiency
C. Wiskott Aldrich syndrome
D. X-linked agammaglobulinemia of Bruton
----------------------------------------
3. A 31-year-old male IV drug abuser presents to a clinic with difficulty swallowing. On examination,
most of his tongue and palate are covered in yellowish membranes. Several white, ulcerated lesions in
the esophagus are seen on endoscopy. The primary cause of these findings is the loss of which of the
following immune cells?
A. B lymphocytes
B. Helper T lymphocytes
C. Killer T lymphocytes
D. Monocytes/macrophages
----------------------------------------
4. Which of the following statements best describes the mechanism underlying severe combined
immunodeficiency (SCID) due to adenosine deaminase (ADA) deficiency?
A. Deficiency of ADA leads to toxic accumulation of deoxyadenosine and its derivatives, resulting in
impaired T-cell development and function
B. Mutations in the common γ-chain (γc) subunit of cytokine receptors lead to defective signaling
through interleukin receptors, resulting in reduced T-cell numbers and impaired B-cell function.
C. ADA deficiency results in the accumulation of adenosine and deoxyadenosine, causing oxidative
stress and hemolytic anemia
D. Mutations in recombinase-activating genes (RAG) prevent somatic gene rearrangements essential
for T and B cell development, leading to SCID
----------------------------------------
5. An 8-month-old child is brought to the pediatric outpatient department with complaints of recurrent
upper respiratory tract infections. Laboratory investigations reveal low levels of serum calcium along

419
with low levels of functional T cells. What is the likely diagnosis?
A. Adenosine deaminase deficiency
B. Common variable immunodeficiency
C. DiGeorge syndrome
D. Wiskott-Aldrich Syndrome.
----------------------------------------
6. Which of the following is predominantly a marker of T cells?
A. CD 3
B. CD 14
C. CD 34
D. CD 16
----------------------------------------
7. Which of the following is not a feature of Hyper IgE syndrome?
A. Autosomal dominantinheritance
B. Kyphoscoliosis
C. Recurrent cutaneous abscesses
D. Low serum IgG, IgA and IgM levels
----------------------------------------
8. In a patient with acquired immunodeficiency syndrome, the physical findings shown in the image
below are seen. What is the likely causative organism?

A. Kaposi sarcoma
B. B cell lymphoma
C. Leukemia of myeloid origin
D. Burkitt's lymphoma
----------------------------------------
9. Which of the following is not an AIDS defining malignancy?
A. Non-Hodgkins lymphoma
B. B cell Lymphoma

Page 2

420
C. Pleomorphic adenoma
D. Kaposi's sarcoma
----------------------------------------
10. Which of the following is a characteristic feature of Wiskott-Aldrich syndrome?
A. Autosomal recessive disorder
B. High serum IgM level
C. Thrombocytosis
D. Eczema
----------------------------------------

Correct Answers
Question Correct Answer

Question 1 3
Question 2 4
Question 3 2
Question 4 1
Question 5 3
Question 6 1
Question 7 4
Question 8 1
Question 9 3
Question 10 4

Solution for Question 1:


Correct Option C- Mutation in the common γ-chain (γc) subunit of cytokine receptors:
• X-linked severe combined immunodeficiency (SCID) is the most common form of SCID, accounting
for 50% to 60% of cases.
• It results from a mutation in the common γ-chain (γc) subunit of cytokine receptors.
• This subunit is essential for signaling pathways involved in lymphocyte development, particularly
T-cell development.
• Defective signaling through the IL-7 receptor, which requires the γc subunit, leads to a profound defect
in the earliest stages of lymphocyte development, resulting in reduced T-cell numbers and impaired
B-cell function due to deficient Th cells.
Incorrect Options:
Option A- Mutation in adenosine deaminase (ADA) gene:
• Mutation in adenosine deaminase (ADA) gene is associated with autosomal recessive SCID, not
X-linked SCID.

Page 3

421
Option B- Mutation in recombinase-activating genes (RAG):
• Mutation in recombinase-activating genes (RAG), is another cause of autosomal recessive SCID but
is not specific to X-linked SCID.
Option D- Mutation in Janus kinase 3 (JAK3) gene:
• Mutation in Janus kinase 3 (JAK3) gene, is also associated with autosomal recessive SCID, not
X-linked SCID.

Solution for Question 2:


Correct Option D- X-linked agammaglobulinemia of Bruton:
• X-linked agammaglobulinemia is caused by the failure of B-cell precursors to mature.
• It is a common primary immunodeficiency.
• It is caused by a mutation in the BTK (Bruton tyrosine kinase) gene found on chromosome Xq21.22.
• The BTK gene encodes a protein required for the rearrangement of immunoglobulin heavy chain
genes in precursor B cells.
• The rearrangement of genes results in a signal that promotes B-cell differentiation.
• A mutation in the BTK gene causes a signal loss, failing B-cell maturation.
• There are no antibodies in the body. (Severe pan-hypogammaglobulinemia, Deficient B cells)
• Symptoms appear after 6 months due to the presence of maternal antibodies till that age.
• Common symptoms (recurrent infections, typically) are: Respiratory tract infection Chronic pharyngitis
Sinusitis Otitis media
• Respiratory tract infection
• Chronic pharyngitis
• Sinusitis
• Otitis media
• Common causative organisms are Hemophilusus influenza Streptococcus pneumoniae
Staphylococcus aureus
• Hemophilusus influenza
• Streptococcus pneumoniae
• Staphylococcus aureus
• Cell-mediated immunity (T-cells dependent) is normal
• Inhibitors of BTK genes are effective in the treatment of these patients.
• Respiratory tract infection
• Chronic pharyngitis
• Sinusitis
• Otitis media
• Hemophilusus influenza

Page 4

422
• Streptococcus pneumoniae
• Staphylococcus aureus
Incorrect Options:
Option A- DiGeorge syndrome:
• Digeorge syndrome occurs due to the deletion of chromosome 22q11.
• It is a T-cell immunodeficiency.
• The defect is the lack of third and fourth pharyngeal pouches, which leads to Absent thymus Absent
parathyroids (Decrease calcium) Congenital heart defects, Conotruncal defects
• Absent thymus
• Absent parathyroids (Decrease calcium)
• Congenital heart defects, Conotruncal defects
• Absent thymus
• Absent parathyroids (Decrease calcium)
• Congenital heart defects, Conotruncal defects
Option B- Isolated IgA deficiency:
• It is a specific IgA deficiency that leads to recurrent mucosal infections.
• Impairment of B-cell differentiation. Ig A levels are low but not absent. The levels of other
immunoglobulins are within the normal range.

Option C- Wiskott Aldrich syndrome:
• It is an X-linked recessive disease. The mutation is at the WAS located on the short arm of the X
chromosome, Xp13.23
• IgM levels are low, but IgG levels are normal. Patients present with a triad of immunodeficiency,
thrombocytopenia, and eczema.
• Levels of IgA and IgE are elevated

Solution for Question 3:


Correct Option B- Helper T lymphocytes:
• Given the patient's history of Intravenous Drug Use with esophageal candidiasis raise concerns of
HIV. HIV targets and infects CD4+ T cells (helper T cells), leading to their depletion and subsequent
immune dysfunction.
• The patient's presentation with difficulty swallowing, yellowish membranes covering the tongue and
palate, and white ulcerated lesions in the esophagus suggest a condition known as oral candidiasis
(thrush) and esophageal candidiasis. These fungal infections are commonly seen in individuals with
compromised immune function, particularly a deficiency in helper T lymphocytes (CD4+ T cells).
• Helper T cells play a crucial role in coordinating the immune response against fungal infections by
activating other immune cells, such as macrophages and B cells, to eliminate pathogens like Candida
albicans (Th1 cells mediated by IL12, IFN gamma are important, antibodies are needed to opsonize).

Page 5

423
Therefore, the loss of helper T lymphocytes in this patient would lead to impaired immune responses
against Candida, resulting in the observed pathological findings.

Incorrect Options:
Option A- B lymphocytes:
• B - Lymphocytes are not primarily affected by HIV
• They form plasma cells that produce antibodies.
Option C- Killer T lymphocytes:
• Natural killer T lymphocytes are part of innate immunity.
• They Protect against many viruses and intracellular bacteria.
• While they contribute to the immune response against viral infections, their loss alone would not
typically lead to the development of oral or esophageal lesions as described. Additionally, Killer T cells
are not the primary target of HIV infection, and their loss is not a hallmark feature of HIV/AIDS.
Option D- Monocytes/macrophages:
• They are phagocytes and can rapidly act in case of infection or inflammation.
• Monocytes, when entering into tissues, converted into macrophages.
• While they are susceptible to HIV infection and can serve as reservoirs for the virus, they are not the
primary cells responsible for coordinating the immune response against opportunistic infections seen in
HIV/AIDS.

Solution for Question 4:


Correct Option A -:Deficiency of ADA leads to toxic accumulation of deoxyadenosine and its derivative
s, resulting in impaired T-cell development and function:
• ADA deficiency is the most common cause of autosomal recessive SCID. ADA is an enzyme involved
in the purine salvage pathway, that converts adenosine and deoxyadenosine to inosine and
deoxyinosine, respectively.
• Deficiency of ADA leads to toxic accumulation of deoxyadenosine and its derivatives, particularly in
rapidly dividing immature lymphocytes, resulting in impaired T-cell development and function.
• This toxic accumulation preferentially affects T cells, leading to a greater reduction in T-cell numbers
compared to B cells, resulting in the severe combined immunodeficiency (SCID) phenotype.
Incorrect Options:
Option B-: Mutations in the common γ-chain (γc) subunit of cytokine receptors lead to defective signalin
g through interleukin receptors, resulting in reduced T-cell numbers and impaired B-cell function:
• This statement describes the mechanism underlying X-linked severe combined immunodeficiency
(SCID with IL2 receptor mutation), not ADA deficiency.
• X-linked SCID is caused by mutations in the gene encoding the common γ-chain (γc) subunit of
cytokine receptors, leading to defective signaling through interleukin receptors.
• Consequently, patients with X-linked SCID exhibit reduced T-cell numbers and impaired B-cell
function.

Page 6

424
Option C -: ADA deficiency results in the accumulation of adenosine and deoxyadenosine, causing oxi
dative stress and hemolytic anemia:
• While ADA deficiency leads to the toxic accumulation of adenosine and deoxyadenosine, resulting in
impaired lymphocyte development and function, it does not cause oxidative stress and hemolytic
anemia.
• Instead, the toxic metabolites of ADA deficiency primarily affect lymphocytes, leading to severe
combined immunodeficiency (SCID) characterized by impaired T-cell and B-cell function.
Option D- : Mutations in recombinase-activating genes (RAG) prevent somatic gene rearrangements e
ssential for T and B cell development, leading to SCID:
• Mutations in recombinase-activating genes (RAG) indeed lead to severe combined immunodeficiency
(SCID) by preventing somatic gene rearrangements essential for T and B cell development.
• However, RAG deficiency is distinct from ADA deficiency, which involves the toxic accumulation of
adenosine and deoxyadenosine due to the deficiency of the enzyme adenosine deaminase.

Solution for Question 5:


Correct Option C- DiGeorge syndrome:
• DiGeorge syndrome, also known as 22q11.2 deletion syndrome, is characterized by congenital thymic
hypoplasia or aplasia, resulting in T-cell deficiency. This causes impaired cellular immunity. There is
also reduced B cell activation
• This deficiency in functional T cells predisposes individuals to recurrent infections, particularly
respiratory tract infections.
• Additionally, DiGeorge syndrome commonly presents with hypocalcemia due to hypoparathyroidism
or parathyroid hypoplasia, contributing to the low levels of serum calcium observed in this case.
• DiGeorge syndrome is characterized by the failure of the third and fourth pharyngeal pouches to
develop.
• It prevents the development of the thymus and parathyroid glands → immunodeficiency and
hypocalcemia.
Incorrect Options:
Option A- Adenosine deaminase deficiency:
• Adenosine deaminase deficiency causes severe combined immunodeficiency (SCID)
• It leads to a deficiency of both B and T-cells.
• Hypocalcemia is not seen
Option B - Common variable immunodeficiency:
• CVID occurs due to B-cell differentiation defect, the B cell number is normal, but immunoglobulins are
absent Manifests in adulthood/late teens with recurrent infections.
Option D- Wiskott-Aldrich syndrome:
• Wiskott-Aldrich syndrome is an X-linked immunodeficiency disorder characterized by eczema,
thrombocytopenia, and recurrent infections.
• While it can present with immunodeficiency, hypocalcemia is not a typical feature of this syndrome.

Page 7

425
Solution for Question 6:
Correct Option A - CD 3:
• The markers of T cells include T cell receptors (Binds antigen-MHC complex), CD 3(associated with T
cell receptor for signal transduction, and CD 28 (Binds B7 on antigen-presenting cells.)
• Markers of helper T cells include CD 4, CD 40L, and CXCR 4/ CCR 5 (co-receptor for HIV).
• Cytotoxic T cells express CD 8
• Regulatory T cells express CD 4 and CD 25.

Incorrect Options:
Option B- CD 14:
• CD 14 is the marker of macrophages.
• It is the receptor for PAMPs (Lipopolysaccharide)
Option C- CD 20:
• CD 20 is the marker of B cells.
• Other surface proteins of B cells include CD 19, CD 21, Ig, CD 40, MHC-II, etc.
Option D- CD 16:
• CD 16 is the receptor for natural killer cells.
• It binds the Fc portion of IgG.

Solution for Question 7:


Correct Option D- Low serum IgG, IgA, and IgM levels:

Page 8

426
Hyper IgE syndrome / Job’s syndrome:
• Job Syndrome, also known as Hyper-IgE syndrome, is a rare primary immunodeficiency disorder
characterized by a clinical triad comprising: atopic dermatitis recurrent skin staphylococcal infections
recurrent pulmonary infections.
• atopic dermatitis
• recurrent skin staphylococcal infections
• recurrent pulmonary infections.
• Inheritance: Autosomal dominant
• Gene Mutation: STAT3 (Signal Transducer and Activator of Transcription 3)
• Effect: Inhibition of normal STAT signaling
• Consequence: Deficient TH17 cells
• Result: It leads to impaired migration of neutrophils to the site of infection.
• Patients have characteristic facies: Broad nose and retained baby teeth.
• atopic dermatitis
• recurrent skin staphylococcal infections
• recurrent pulmonary infections.

• Bony abnormalities: Kyphoscoliosis, Osteoporosis ( Fractures with minimal trauma).


• Serum IgE level is significantly elevated, whereas IgM, IgG, and IgA levels are normal
• The eosinophil count is also raised. Other counts are normal.
• In vivo assessment of inflammation is possible with a Rebuck skin window test or a skin blister assay,
which measures the ability of leukocytes and inflammatory mediators to accumulate locally in the skin.
• In vitro tests of phagocyte aggregation, adherence, chemotaxis, phagocytosis, degranulation, and
microbicidal activity (for S. aureus) are used for laboratory diagnosis.
Incorrect Options:

Page 9

427
Option A - Autosomal dominant:
• Job syndrome is usually inherited in an autosomal dominant pattern.
• IgM, IgG, and IgA levels are normal in Job syndrome.
• However, IgE levels are elevated.
Option B- Kyphoscoliosis:
• Kyphoscoliosis is also seen in job syndrome.
• Other skeletal problems include osteopenia and fracture of bone after minimal trauma.
Option C- Recurrent cutaneous abscesses:
• Recurrent Staphylococcus aureus cutaneous abscesses are seen in the job syndrome.
• They are referred to as cold abscesses.

Solution for Question 8:


Correct Option A- Kaposi sarcoma:
• Kaposi sarcoma (neoplasm of endothelial cells) caused by Human herpes virus-8 (HHV-8) seen in
HIV/AIDS and transplant patients.
• It is estimated that 25% to 40% of untreated HIV-infected individuals will eventually develop a
malignancy.
• Clinical features include dark/violaceous plaques or nodules that represent vascular proliferations.
• Endothelial malignancy most commonly affects the skin, mouth, GI tract, and respiratory tract.
• Perivascular spindle cells invading and forming vascular tumors on histology.
• There is also a profusion of slit-like vascular spaces, suggesting that the lesions may arise from
primitive mesenchymal precursors of vascular channels.

Incorrect Options:
Option B- B cell lymphoma:
• Lymphoma occurs at a markedly increased rate in individuals with AIDS, making it one of several
AIDS-defining conditions.
• Roughly 5% of AIDS patients present with lymphoma, and approximately another 5% develop
lymphoma during their subsequent course.
• As a result, AIDS patients are at high risk of developing aggressive B cell lymphomas composed of
tumor cells infected by oncogenic viruses, particularly the Epstein-Barr virus (EBV).
• The skin involvement in B-cell lymphoma would often present as nodules or plaques rather than the
progressive, violaceous lesions characteristic of Kaposi sarcoma.
Option C- Leukemia of myeloid origin:
• Leukemia of myeloid origin is not an AIDS-defining illness.
Option D- Burkitt's lymphoma:

Page 10

428
• Patients with AIDS risk developing malignancies due to the Epstein-Barr virus, such as B-cell
lymphoma and Burkitt's lymphoma. However, the most common tumor associated with AIDS is
Kaposi's sarcoma.
• While Burkitt's lymphoma can occur in individuals with AIDS, it typically presents as rapidly growing
masses, often involving the jaw or abdomen and would not typically present as the progressive,
violaceous lesions characteristic of Kaposi sarcoma.

Solution for Question 9:


Correct Option C- Pleomorphic adenoma: Pleomorphic adenoma is not an AIDS defining malignancy.
• A pleomorphic adenoma is a benign (noncancerous) tumor typically of the parotid gland. .
Incorrect Options:
Option A- Non-Hodgkin's lymphoma: Non-Hodgkin's lymphoma is an AIDS-defining malignancy. It occu
rs more frequently in individuals with HIV/AIDS and is associated with severe immunosuppression.
Option B- B cell lymphoma: B cell lymphomas, including diffuse large B-cell lymphoma, are also AIDS-
defining malignancies. They occur more commonly in people with HIV/AIDS, particularly those with low
CD4 cell counts.
Option D- Kaposi's sarcoma: Kaposi's sarcoma is an AIDS-defining malignancy. It is strongly associate
d with HIV/AIDS, especially in individuals with low CD4 cell counts. It is caused by Human Herpesvirus
8 (HHV-8) and can present as skin lesions or affect internal organs.

Solution for Question 10:


Correct Option D- Eczema:
• Wiskott-Aldrich syndrome is an X-linked recessive disease.
• It is characterized by thrombocytopenia and small platelets, eczema, and marked vulnerability to
recurrent infections (especially encapsulated organisms eg: Streptococcus pneumoniae, Haemophilus
influenzae, Neisseria meningitidis).
• It occurs due to a mutation in the gene encoding the Wiskott-Aldrich syndrome protein (WASP),
located at Xp11.23 (causes impaired cytoskeleton remodeling - impaired cell migration).
• In this disease, IgM levels in the serum are low, while levels of IgA and IgE are paradoxically elevated.
• The only treatment is HSC transplantation.
Incorrect Options:
Option A- Autosomal recessive disorder:
• Wiskott-Aldrich syndrome is an X-linked recessive disease.
Option B - High serum IgM level:
• Wiskott-Aldrich syndrome occurs due to a mutation in the gene encoding the Wiskott-Aldrich
syndrome protein (WASP), located at Xp11.23.
• IgM levels in the serum are low, but levels of IgG are usually normal.

Page 11

429
• Paradoxically, the levels of IgA and IgE are often elevated.
Option C- Thrombocytosis:
• Wiskott-Aldrich syndrome is an X-linked recessive disease characterized by Thrombocytopenia
Eczema Marked vulnerability to recurrent infection.
• Thrombocytopenia
• Eczema
• Marked vulnerability to recurrent infection.
• Thrombocytopenia
• Eczema
• Marked vulnerability to recurrent infection.

Page 12

430
Transplant Immunology
1. Which of these complement factors is a marker of acute humoral rejection?
A. C3d
B. C5a
C. C3b
D. C4d
----------------------------------------
2. Which of the following statements regarding acute cellular rejection after liver transplantation is
incorrect?
A. CD8+ cytotoxic T lymphocytes (CTLs) may directly destroy graft cells.
B. CD4+ T cells primarily secrete cytokines and induce inflammation, leading to graft damage.
C. Acute cellular rejection may present with two different patterns of injury: tubulointerstitial pattern and
vascular pattern.
D. Humoral rejection is the main cause of acute cellular rejection, leading to graft failure.
----------------------------------------
3. Which organ has the lowest rejection rate for tissue transplants?
A. Hematopoietic stem cells
B. Cornea
C. Kidney
D. Heart
----------------------------------------
4. Transplantation of a graft from an individual of a different species is denoted by which of the
following terms?
A. Isograft
B. Allograft
C. Autograft
D. Xenograft
----------------------------------------
5. Which of the following measures can reduce the incidence of graft-versus-host disease ?
A. T-cell removal
B. Prior immune suppression
C. Post-procedure immune suppression
D. All of the above
----------------------------------------
6. Which of the following statements is not true regarding hematopoietic stem cell transplantation?
A. Used in the treatment of leukaemia

431
B. Stem cells can only be collected directly from the bone marrow
C. G-CSF is given to increase the number of stem cells
D. Immunodeficiency is a frequent complication
----------------------------------------
7. A 48-year-old male patient presents to the hospital with sudden onset of fever and chills. He
underwent a liver transplant 2 weeks ago. His heart rate is 110/min, respirations are 25/min, and SpO2
is 87%. All of the following regarding the condition described above is true, except?
A. CD8+ T-cells directly destroy the graft
B. Donor APCs may be implicated in the response
C. Preformed antibodies against graft are the main causative factor
D. Cytokines secreted by CD4+ T-cells cause inflammation which destroys the graft
----------------------------------------
8. A woman received an allogeneic bone marrow transplant 3 weeks ago for acute myeloid leukemia.
At present, she complains of a skin rash and diarrhea for three days. A skin biopsy reveals a sparse
lymphocytic infiltrate in the dermis and epidermis and apoptotic cells in the basal cell layer of the
epidermis. AST and ALT levels are increased. Which of the following cells is the primary cause of this
patient's skin rash and diarrhea?
A. Donor lymphocytes
B. Donor plasma cells
C. Fixed tissue macrophages
D. Recipient lymphocytes
----------------------------------------
9. Three weeks post renal transplant for end-stage renal disease, a 62-year-old woman develops fever,
decreased urine output, and pain in the graft region. Creatinine and BUN are markedly raised. Which of
the following pathologic findings should not be expected on a kidney biopsy?
A. Necrosis of vessel walls
B. Extensive interstitial and tubular inflammation
C. Vessel wall intimal thickening
D. Focal tubular injury
----------------------------------------
10. Which of the following is the most appropriate term for a graft when the donor and recipient are
identical twins?
A. Xenograft
B. Allograft
C. Isograft
D. Autograft
----------------------------------------

Correct Answers

Page 2

432
Question Correct Answer

Question 1 4
Question 2 4
Question 3 2
Question 4 4
Question 5 4
Question 6 2
Question 7 3
Question 8 1
Question 9 3
Question 10 3

Solution for Question 1:


Correct Option D - C4d:
• Humoral rejection is hyperacute rejection and is an acute antibody-mediated reaction
• Humoral rejection occurs in individuals who are sensitized or who develop optimal levels of de
novo-specific antibodies after the transplantation of the organ.
• Humoral rejection is associated with the deposition of the complement breakdown product C4d
• C4d has a role in visualizing the direct link between anti-donor antibodies and tissue injury at antibody
binding sites in a graft.
• The activation of the complement-dependent classical pathway produces it.
• It may occur after kidney, liver, or lung transplantation. Evidence of endothelial injury and deposition of
C4d should be considered as it's a reliable marker of humoral rejection. C4d deposition in the lungs is
nonspecific and patchy. The diagnosis of humoral rejection should be made when there is a diffuse
expression of C4d along the alveolar microvasculature.

Page 3

433
Incorrect Options:
Option A - C3d:
• C3d is the final degradation product of C3.
• It enhances the immune response to fused antigens.
Option B - C5a:
• C5a is involved in chemotaxis.
• It has a role in the recruitment of neutrophils, eosinophils, lymphocytes, etc.,
Option C - C3b:
• The breakdown of complement component 3 produces C3b.
• It is involved in opsonization.

Solution for Question 2:


Correct Option D -
Humoral rejection is the main cause of acute cellular rejection, leading to graft failure:
• This option states that humoral rejection is the main cause of acute cellular rejection, which is
incorrect.
• Acute cellular rejection primarily involves T cell-mediated immune responses, where CD8+ cytotoxic T
lymphocytes (CTLs) directly destroy graft cells or CD4+ T cells secrete cytokines and induce
inflammation, leading to graft damage.
• The different patterns of injury seen in acute cellular rejection include the tubulointerstitial pattern and
the vascular pattern.
• Humoral rejection, on the other hand, involves antibody-mediated responses targeting vascular
endothelium, leading to complement activation and endothelial damage.
• It is not the main cause of acute cellular rejection

Page 4

434
Incorrect Options:
Options A, B, and C
are correct regarding acute cellular rejection and is explained under the correct option.

Solution for Question 3:


Correct Option B - Cornea:
• Graft rejection occurs when the immune system of the recipient recognizes the graft as foreign and
triggers an immune response against it, which can destroy the graft.
• Amongst the organs described above, the transplantation of the cornea (keratoplasty) has the least
chance of rejection. This phenomenon is attributable to the “immune privilege”.
• Immune privilege is a phenomenon in which the introduction of new (foreign) antigens does not trigger
the immune system of the recipient, unlike what happens in the other usual sites. Other than the
cornea, the immune-privileged sites are the central nervous system, placenta, and testes.
• Keratoplasty is the most common form of human solid tissue transplantation.
• Unlike other tissues and organs, corneal allotransplantation usually does not require systemic and
permanent immunosuppression. Nevertheless, allograft rejection is the leading cause of graft failure in
corneal transplantation. It is estimated that approximately 10% of all grafts fail because of immunologic
rejection.
Incorrect Options:
Option A - Hematopoietic stem cells:
• The transplantation of hematopoietic stem cells does not have the immune privilege, and severe
immune reactions can potentially happen. For example, the transplantation of allogeneic hematopoietic
stem cells are the most prone to causing graft-versus-host reaction.
Option C - Kidney:
• The kidney is a highly vascular organ and does not have an immune-privileged status. Graft rejections
are common in renal transplantation and require the proper use of immunosuppressants.
Option D - Heart:
• The transplantation of the heart is not free of complications attributable to transplant rejection. The
heart is a highly perfused organ, and an immune response can be triggered against the transplanted
heart.

Solution for Question 4:


Correct Option D - Xenograft:
• The transplantation of grafts extracted from the animals is indicated by the term xenograft.
• There are four types of graft based on histocompatibility between donor and recipient: Donor and
recipient are of the same species and different individuals: Allograft Donor and recipient are of the
same individual: Autograft Graft between different individuals but of similar genetic makeup as in

Page 5

435
identical twins: Isograft Graft between different species: Xenograft
• Donor and recipient are of the same species and different individuals: Allograft
• Donor and recipient are of the same individual: Autograft
• Graft between different individuals but of similar genetic makeup as in identical twins: Isograft
• Graft between different species: Xenograft
• Donor and recipient are of the same species and different individuals: Allograft
• Donor and recipient are of the same individual: Autograft
• Graft between different individuals but of similar genetic makeup as in identical twins: Isograft
• Graft between different species: Xenograft
Incorrect Options:
The correct answer is Xenograft and the incorrect options have been explained above.

Solution for Question 5:


Correct Option D - All of the above:
• All the options mentioned (T-cell removal, prior immune suppression, and post-procedure immune
suppression) can reduce the incidence of graft-versus-host disease. In many cases, a combination of
these strategies is used to minimize the risk and severity of GVHD after a bone marrow transplant.
Incorrect Options:
Option A - T-cell removal: Removing T-cells from the donor graft can significantly reduce the incidence
and severity of graft-versus-host disease (GVHD). T-cells are primarily responsible for the immune rea
ction in GVHD, so their removal decreases the risk of this complication.
Option B - Prior immune suppression: Immunosuppressive therapy given to the recipient before the tra
nsplant can reduce the recipient's immune response, making them less prone to GVHD. This approach
is particularly useful in haploidentical (half-matched) or mismatched transplants, where complete matc
hing between donor and recipient is not possible.
Option C - Post-procedure immune suppression: After the transplant, immunosuppressive medications
are often given to the recipient to prevent or treat GVHD. These drugs suppress the immune response,
reducing the likelihood of the transplanted immune cells attacking the recipient's tissues.

Solution for Question 6:


Correct Option B - Stem cells can only be collected directly from the bone marrow:
• In hematopoietic stem cell transplantation, hematopoietic stem cells are taken from the donor and
transfused to the recipient.
• In the past, hematopoietic stem cells were extracted from the bone marrow of the recipient. But
nowadays, these stem cells can be collected from peripheral blood after they are mobilized from the
bone marrow by administering hematopoietic growth factors.

Page 6

436
• Hematopoietic stem cell transplantation can be done in the management of the following disease:
Hematologic malignancies (for example, leukaemia) Bone marrow failure syndromes (as in this patient
who has suffered severe aplastic anaemia), Disorders caused by inherited HSC defects (such as sickle
cell anaemia, thalassemia,and immunodeficiency states)
• Hematologic malignancies (for example, leukaemia)
• Bone marrow failure syndromes (as in this patient who has suffered severe aplastic anaemia),
• Disorders caused by inherited HSC defects (such as sickle cell anaemia, thalassemia,and
immunodeficiency states)
• The two potential complications of hematopoietic stem cell transplantation are graft-versus-host
disease and immunodeficiency. Graft-versus-host-disease occurs when immunologically competent
cells or their precursors are transplanted into recipients, and the transferred cells recognize alloantigen
in the host tissues and attack them, leading to destruction.
• Graft-versus-host-disease occurs when immunologically competent cells or their precursors are
transplanted into recipients, and the transferred cells recognize alloantigen in the host tissues and
attack them, leading to destruction.
• Hematologic malignancies (for example, leukaemia)
• Bone marrow failure syndromes (as in this patient who has suffered severe aplastic anaemia),
• Disorders caused by inherited HSC defects (such as sickle cell anaemia, thalassemia,and
immunodeficiency states)
• Graft-versus-host-disease occurs when immunologically competent cells or their precursors are
transplanted into recipients, and the transferred cells recognize alloantigen in the host tissues and
attack them, leading to destruction.
Incorrect Options:
Option A - Used in the treatment of leukaemia:
• Allogenic hematopoietic stem cell transplantation can trigger an immune reaction against the
recipient's body.
• This phenomenon is employed therapeutically to target tumour cells, for example, in leukaemia.
Option C - G-CSF is given to increase the number of stem cells:
• Donors are usually given hematopoietic growth factors for four or five days before the collection of the
stem cells.
• In this way, the hematopoietic stem cells are mobilized from the bone marrow.
Option D - Immunodeficiency is a frequent complication:
• The immunodeficiency is a result of prior treatment (e.g., for leukaemia), myeloablation prior to
hematopoietic stem cell transplantation, a delay in the repopulation of the recipient's immune system,
and an attack on the host's immune cells by grafted lymphocytes

Solution for Question 7:


Correct Option C - Preformed antibodies against graft are the main causative factor:
• This patient’s presentation with fever, and chills, 2 weeks after the hepatic transplant is most likely due
to acute graft rejection.

Page 7

437
• This option is incorrect as preformed antibodies against the graft can cause hyperacute rejection, they
are not the main causative factor in acute graft rejection, which typically occurs within days or
weeks after transplantation.
• There are two types of divisions: CD8+ CTLs may directly destroy graft cells in acute cellular rejection,
or CD4+ cells produce cytokines and cause inflammation, damaging the graft.
Incorrect Options:
Option A - CD8+ T-cells directly destroy the graft:
• Acute graft rejection is a type IV hypersensitivity reaction.
• Acute graft rejection can occur when CD8+ Cytotoxic T-Lymphocytes directly destroy graft cells in
acute cellular rejection.
Option B - Donor APCs may be implicated in the response:
• In the direct allorecognition pathway, donor antigen-presenting cells (APCs) are recognized by
recipient T cells, leading to T cell activation and subsequent immune response against the graft.
Therefore, this statement accurately reflects the involvement of donor APCs in the rejection response
Option D - Cytokines secreted by CD4+ T-cells cause inflammation which destroys the graft:
• Cytokines by CD4+ T-cells cause inflammation.
• It is a mechanism involved in the pathway of acute graft reaction.
• Biopsy shows vasculitis of the graft vessels.

Solution for Question 8:


Correct Option A - Donor lymphocytes:
• This patient’s presentation with rash and diarrhoea after three weeks of bone marrow transplant is
most likely due to graft-versus-host disease (GVHD)
• GVHD is characterised by the following: Diarrhoea The diarrhoea is usually secretory and is often
associated with cramping abdominal pain and vomiting. Some patients also report bloody diarrhea It is
not related to eating and occurs at night Dermatitis It is often a diffuse, painful maculopapular rash.
Jaundice Hepatosplenomegaly
• Diarrhoea The diarrhoea is usually secretory and is often associated with cramping abdominal pain
and vomiting. Some patients also report bloody diarrhea It is not related to eating and occurs at night
• The diarrhoea is usually secretory and is often associated with cramping abdominal pain and
vomiting.
• Some patients also report bloody diarrhea
• It is not related to eating and occurs at night
• Dermatitis It is often a diffuse, painful maculopapular rash.
• It is often a diffuse, painful maculopapular rash.
• Jaundice
• Hepatosplenomegaly
• It can occur weeks to months after an organ transplant.

Page 8

438
• When lymphocytes in the transplanted tissue recognize and react to the recipient’s cells, GVHD
develops.
• GVHD might ensue when an immunocompromised patient is transfused with blood containing
HLA-incompatible cells.
• Diarrhoea The diarrhoea is usually secretory and is often associated with cramping abdominal pain
and vomiting. Some patients also report bloody diarrhea It is not related to eating and occurs at night
• The diarrhoea is usually secretory and is often associated with cramping abdominal pain and
vomiting.
• Some patients also report bloody diarrhea
• It is not related to eating and occurs at night
• Dermatitis It is often a diffuse, painful maculopapular rash.
• It is often a diffuse, painful maculopapular rash.
• Jaundice
• Hepatosplenomegaly
• The diarrhoea is usually secretory and is often associated with cramping abdominal pain and
vomiting.
• Some patients also report bloody diarrhea
• It is not related to eating and occurs at night
• It is often a diffuse, painful maculopapular rash.
Incorrect Options:
Option B - Donor plasma cells:
• Donor plasma cells are not involved in the clinical manifestations of GVHD.
• It is the donor’s T cells that cause this.
Option C - Fixed tissue macrophages:
• Fixed tissue macrophages in the recipient can be involved in antigen presentation in acute and
chronic rejection of grafts.
• However, they are not involved in GVHD.
Option D - Recipient lymphocytes:
• Recipient lymphocytes cause acute and chronic rejection of the graft.
• It occurs due to cell-mediated immunity and recipient T lymphocyte reaction against donor major
histocompatibility complexes (MHCs).

Solution for Question 9:


Correct Option C -Vessel wall intimal thickening:
• The patient presents with fever and pain in the graft region along with the sudden deterioration of
renal function three weeks after a history of renal transplant. These features are most suggestive of
acute graft rejection.

Page 9

439
• Graft rejection is a common complication that develops after the transplantation of the organs from
donor to recipient.
• In acute rejection following kidney transplantation, the pathologic findings typically include extensive
interstitial and tubular inflammation, focal tubular injury, and sometimes necrosis of vessel walls.
However, vessel wall intimal thickening is more characteristic of chronic rejection rather than acute
rejection. Therefore, vessel wall intimal thickening would not be expected on a kidney biopsy in the
acute phase of rejection.
In acute rejection following kidney transplantation, the pathologic findings typically include extensive int
erstitial and tubular inflammation, focal tubular injury, and sometimes necrosis of vessel walls. Howeve
r, vessel wall intimal thickening is more characteristic of chronic rejection rather than acute rejection. T
herefore, vessel wall intimal thickening would not be expected on a
kidney biopsy in the acute phase of rejection.

Incorrect Options:

Pathologic findings on a kidney biopsy in acute rejection typically include:


• Extensive interstitial inflammation and tubular inflammation (tubulitis)
• Focal tubular injury
• Inflammation of vessels
• Sometimes necrosis of vessel walls
Hence OPTIONS A, B, and D are features seen in acute renal graft rejection.

Solution for Question 10:


Correct Option C - Isograft:
• To be able to carry out a successful transplantation that has the least chance of rejection, a thorough
and comprehensive workup of donor and recipient compatibility is essential.

Page 10

440
• The histocompatibility of donor and recipient is based on HLA / MHC matching, mainly at the HLA-DR,
HLA-A, and HLA-B loci.
• There are four types of graft based on histocompatibility between donor and recipient: Autograft
Isograft Allograft Xenograft
• Autograft
• Isograft
• Allograft
• Xenograft
• Autograft implies that the donor tissue is taken from the recipient’s own body. For example, hair
transplants and skin grafts.
• Isograft means taking the graft from a person who is genetically identical. This is true in the case of an
identical twin.
• Allograft means taking the graft from a genetically different person of the same species.
• Xenograft means that the graft originates from an organism of different species.
• Autograft
• Isograft
• Allograft
• Xenograft
Incorrect Options:
Option A - Xenograft:
• A xenograft is a type of graft that is taken from an organism of different species.
• Bovine heart valves are an example of xenograft.
Option B - Allograft:
• The word “allo” means different. So the allograft implies that the graft is taken from a genetically
different person (i.e., not an identical twin) of the same species.
Option D - Autograft:
• The word “auto” means self. So, autograft implies that the graft is taken from the same individual.
• For example, hair transplant.

Page 11

441
Amyloidosis
1. Which of the following statements about Familial Mediterranean Fever is incorrect?
A. Form of hereditary amyloidosis
B. It is associated with decreased production of the cytokine IL-1
C. Autosomal recessive
D. Mutation affecting pyrin protein
----------------------------------------
2. A 56-year-old man is being evaluated for motor-dominant sensorimotor polyneuropathy and erectile
dysfunction. Transthyretin gene mutation was noted on genetic evaluation. Sural nerve biopsy showed
focal edema and amyloid deposit in the sub-perineural tissue. Which of the following diseases is being
described here?
A. Familial Mediterranean fever
B. Familial amyloidosis polyneuropathy
C. Dialysis associated amyloidosis
D. Prion protein associated amyloidosis
----------------------------------------
3. Serum amyloid-associated protein is found in which of the following?
A. Alzheimer’s disease
B. Chronic inflammatory states
C. Chronic renal failure
D. Malignant hypertension
----------------------------------------
4. Amyloid deposits are stained with which of the following?
A. Congo-Red
B. Alizarin Red
C. Von Kossa
D. Gram stain
----------------------------------------
5. Which of the following is not correctly matched:
A. Chronic Inflammation - AA
B. Hemodialysis associated amyloidosis - A beta 2
C. Alzheimer’s disease - A Cal
D. Multiple myeloma - AL
----------------------------------------
6. Assertion: Alzheimer's disease is associated with Down syndrome. Reason: Beta-amyloid gene is
located on chromosome 21 .Which of the following is the correct option?

442
A. Both Assertion and Reason are independently true/correct statements, and the Reason is the correct
explanation for the Assertion.
B. Assertion and Reason are independently true/correct statements, but Reason is not the correct
explanation for the Assertion.
C. The Assertion is independently a true/correct statement, but the Reason is independently a
false/incorrect statement.
D. The Assertion is independently a false/incorrect statement, but the Reason is independently a
true/correct statement.
----------------------------------------
7. Familial Mediterranean fever is associated with which of the following characteristics?
A. Autosomal dominant inheritance pattern
B. Excessive production of IL-1 leading to autoinflammatory syndrome
C. Deposition of amyloid predominantly in peripheral and autonomic nerves
D. Commonly found in individuals of Japanese and Chinese origins
----------------------------------------
8. Which of the following complications is commonly associated with long-term hemodialysis in patients
with renal failure?
A. The abnormal protein deposited in extracellular tissue are immunoglobulin light chain.
B. In patients with dialysis-associated amyloidosis, transthyretin is the predominant deposit.
C. Carpal tunnel syndrome resulting from deposition of β2-microglobulin
D. Patients with generalized amyloidosis have an excellent prognosis.
----------------------------------------

Correct Answers
Question Correct Answer

Question 1 2
Question 2 2
Question 3 2
Question 4 1
Question 5 3
Question 6 1
Question 7 2
Question 8 3

Solution for Question 1:


Correct Option B - It is associated with decreased production of the cytokine IL-1:

Page 2

443
• Familial Mediterranean Fever (FMF) is associated with excessive production of the cytokine IL-1 in
response to inflammatory stimuli, not decreased production.
Familial Mediterranean Fever (FMF) is associated with excessive production of the cytokine IL-1 in res
ponse to inflammatory stimuli, not decreased production.
Incorrect Options:
Option A - Form of hereditary amyloidosis:
• It is the most common and well-studied form of hereditary amyloidosis
• Familial Mediterranean fever (FMF) also known as 'periodic peritonitis,' 'familial paroxysmal
polyserositis,' 'periodic disease,' 'Siegal-Cattan-Mamou disease,' 'Wolff periodic disease' or 'Reimann
syndrome' is an autoinflammatory genetic disorder that causes recurrent fevers and serosal
inflammation of the abdomen, lungs, and joints leading to severe pain
Option C - Autosomal recessive:
• FMF is an autosomal recessive disorder characterized by increased cytokine IL-1 production in
response to inflammatory stimuli. The pyrin protein is encoded by the FMF gene. This protein plays a
crucial role in reducing the innate immune cells' reactivity to inflammatory stimuli, especially neutrophils
FMF is an autosomal recessive disorder characterized by increased cytokine IL-1 production in respon
se to inflammatory stimuli. The pyrin protein is encoded by the FMF gene. This protein plays a
crucial role in reducing the innate immune cells' reactivity to inflammatory stimuli, especially neutrophils
Option D - Mutation affecting pyrin protein:
• FMF is an Autosomal recessive condition and the most common and best-studied form of hereditary
amyloidosis. It is an auto-inflammatory diseaseassociated with excessive production of the cytokine
IL-1 in response to inflammatory stimuli. The gene for familial Mediterranean fever encodes a protein
called pyrin. This protein is important in dampening the response of innate immune cells, particularly
neutrophils, to inflammatory mediators.
• The patient presents with fever accompanied by inflammation of serosal surfaces, including the
peritoneum, pleura, and synovial membrane and is sometimes associated with widespread amyloidosis
in the body.
• The amyloid fibril proteins are made up of AA proteins, reflecting the increased production of serum
amyloid A (SAA) in response to excessive IL-1 secretion.
FMF is an Autosomal recessive condition and the most common and best-studied form of hereditary a
myloidosis. It is an auto-inflammatory diseaseassociated with excessive production of the cytokine IL-1
in response to inflammatory stimuli. The gene for familial Mediterranean fever encodes a protein calle
d pyrin. This protein is important in dampening the response of innate immune cells, particularly neutro
phils, to inflammatory mediators.
The patient presents with fever accompanied by inflammation of serosal surfaces, including the periton
eum, pleura, and synovial membrane and is sometimes associated with widespread amyloidosis in the
body.
The amyloid fibril proteins are made up of AA proteins, reflecting the increased production of serum am
yloid A (SAA) in response to excessive IL-1 secretion.

Solution for Question 2:


Correct Option B - Familial amyloidosis polyneuropathy:

Page 3

444
• Transthyretin (TTR) is a normal serum protein that binds and transports thyroxine and retinol.
• Several distinct mutant forms of TTR (and its fragments) are deposited in a group of genetically
determined disorders referred to as Familial Amyloid Polyneuropathies.
• Normal TTR is also deposited (due to overproduction) in the heart of aged individuals (Senile
Systemic Amyloidosis).
• This disease is caused by mutations in transthyretin, apolipoprotein A1, or gelsolin genes. Amyloid
depositions can be detected in biopsies of the abdominal fat pad, rectum, and nerves. In the vignette,
the patient’s sural nerve biopsy demonstrates the presence of amyloid. The nervous dysfunction seen
in patients with this disease are generalized or multifocal, predominantly axonal and occasionally
demyelinating sensorimotor polyneuropathy.
• Patients can also present with features such as: numbness and painful paresthesias in the distal lower
limbs Carpal tunnel syndrome Autonomic involvement —> postural hypotension Constipation or
persistent diarrhea Erectile dysfunction.
• numbness and painful paresthesias in the distal lower limbs
• Carpal tunnel syndrome
• Autonomic involvement —> postural hypotension
• Constipation or persistent diarrhea
• Erectile dysfunction.
• Patients usually die 10-15 years after the onset of symptoms, from cardiac failure or complications
from malnutrition.
• numbness and painful paresthesias in the distal lower limbs
• Carpal tunnel syndrome
• Autonomic involvement —> postural hypotension
• Constipation or persistent diarrhea
• Erectile dysfunction.
Incorrect Options:
Option A - Familial Mediterranean fever:
• Familial Mediterranean fever is a genetic disorder that causes recurrent episodes of fever with
Abdominal pain Chest discomfort Joint pains.
• Abdominal pain
• Chest discomfort
• Joint pains.
• Abdominal pain
• Chest discomfort
• Joint pains.
Option C - Dialysis-associated amyloidosis:
• Age-related amyloidosis is mainly seen in chronic kidney disease and dialysis patients.
• This patient has no chronic kidney disease, so this option is eliminated.
• It is the main type of systemic amyloidosis (beta 2 microglobulin implicated)

Page 4

445
Option D - Prion protein-associated amyloidosis:
• Prion diseases are group of infectious, fatal, sporadic and genetically determined diseases in which
prion protein PrP is accumulated in the nervous system of humans and mammals.
Prion diseases are group of infectious, fatal, sporadic and genetically determined diseases in which pri
on protein PrP is accumulated in the nervous system of humans and mammals.

Solution for Question 3:


Correct Option B - Chronic inflammatory states:
• The AA (amyloid-associated) type of amyloid fibril protein is derived from a unique non-Ig protein
made by the liver.
• Fibrils are derived by proteolysis from a larger precursor in the serum called SAA (serum
amyloid-associated)protein.
• The production of SAA protein is increased in inflammatory states as part of the acute phase
response.
• Therefore, this form of amyloidosis is associated with chronic inflammation and is often called
secondary amyloidosis.
Incorrect Options:
Option A - Alzheimer’s disease:
• Alzheimer’s disease exhibits a form of localized amyloidosis due to the deposition of Beta-amyloid
protein, not serum amyloid a protein. Therefore this option is incorrect
Option C - Chronic renal failure:
• Dialysis-related amyloidosis is systemic amyloidosis due to the deposition of Beta 2 microglobulins.
• This condition is seen in patients with end-stage renal disease and/or long-term dialysis.
Option D - Malignant hypertension:
• Malignant hypertension is not a characteristic of amyloidosis, so this option is wrong.

Solution for Question 4:


Correct Option A - Congo-Red:
• Amyloid deposits are typically stained with Congo-Red, which results in characteristic apple-green
birefringence under polarized light microscopy.
• This staining property is a key feature used in the histopathological diagnosis of amyloidosis.
Incorrect Options:
Option B - Alizarin Red:
• Alizarin Red is commonly used to stain calcium deposits, such as those seen in bone tissues, but it is
not specific for amyloid.
Option C - Von Kossa:

Page 5

446
• Von Kossa stain is also used to detect calcium deposits and is not specific for amyloid.
Option D - Gram stain:
• Gram stain is an important differential staining technique in differentiating gram-positive and
gram-negative organisms.
• It is not used to stain amyloid deposits.

Solution for Question 5:


Correct Option C - Alzheimer’s disease- A Cal :
• Alzheimer’s disease - Aβ is the type of amyloidosis seen. A Cal is seen in Medullary carcinoma thyroid
Alzheimer’s disease - Aβ is the type of amyloidosis seen. A Cal is seen in Medullary carcinoma thyroid
Incorrect Options:
Option A, B and D are correctly matched

Solution for Question 6:


Correct Option A - Both Assertion and Reason are independently true/correct statements, and the Rea
son is the correct explanation for the Assertion:
• The amyloid precursor protein (APP) gene is located on chromosome 21.
• Amyloid protein is divided into smaller segments called peptides.
• Some of these peptides are released outside the cell
• Two of these peptides are called soluble amyloid peptide and Amyloid beta peptide.
• APP gene mutation causes increased generation of Aβ.
• Alzheimer's disease is associated with trisomy 21 (Down syndrome).
• This is related to a gene dosage effect with increased production of APP and, subsequently, Aβ.
INCORRECT OPTIONS:
Options B, C and D:
The correct answer has been explained above.

Solution for Question 7:


Correct Option B - Excessive production of IL-1 leading to autoinflammatory syndrome:

Page 6

447
• Familial Mediterranean fever is associated with excessive production of the cytokine IL-1 in response
to inflammatory stimuli, leading to an autoinflammatory syndrome characterized by attacks of fever and
inflammation of serosal surfaces.
Familial Mediterranean fever is associated with excessive production of the cytokine IL-1 in response t
o inflammatory stimuli, leading to an autoinflammatory syndrome characterized by attacks of fever and
inflammation of serosal surfaces.
Incorrect Options:
Option A) Autosomal dominant inheritance pattern :
• Familial Mediterranean fever (FMF) is inherited in an autosomal recessive pattern, not autosomal
dominant.
• Autosomal recessive disorders require both copies of the gene to be mutated for the disease to
manifest.
• In the case of FMF, an individual must inherit a mutated copy of the gene from both parents to
develop the condition.
Familial Mediterranean fever (FMF) is inherited in an autosomal recessive pattern, not autosomal domi
nant.
Autosomal recessive disorders require both copies of the gene to be mutated for the disease to manife
st.
In the case of FMF, an individual must inherit a
mutated copy of the gene from both parents to develop the condition.
Option C) Deposition of amyloid predominantly in peripheral and autonomic nerves :
• This description corresponds more closely to familial amyloidotic polyneuropathies (FAPs), which are
a group of autosomal dominant genetic disorders characterized by the deposition of amyloid primarily in
peripheral and autonomic nerves.
• FMF, on the other hand, is associated with inflammation of serosal surfaces and is not primarily
characterized by amyloid deposition in nerves
This description corresponds more closely to familial amyloidotic polyneuropathies (FAPs), which are a
group of autosomal dominant genetic disorders characterized by the deposition of amyloid primarily in
peripheral and autonomic nerves.
FMF, on the other hand, is associated with inflammation of serosal surfaces and is not primarily charac
terized by amyloid deposition in nerves
Option D) Commonly found in individuals of Japanese and Chinese origins :
• While familial amyloidotic polyneuropathies have been described in different parts of the world,
including Japan and China, FMF is more commonly found in individuals of Mediterranean descent,
including those of Armenian, Sephardic Jewish, and Arabic origins.
• These populations have a higher prevalence of FMF compared to other ethnic groups.
While familial amyloidotic polyneuropathies have been described in different parts of the world, includin
g Japan and China, FMF is more commonly found in individuals of Mediterranean descent, including th
ose of Armenian, Sephardic Jewish, and Arabic origins.
These populations have a higher prevalence of FMF compared to other ethnic groups.

Page 7

448
Solution for Question 8:
Correct Option C - Carpal tunnel syndrome resulting from deposition of β2-microglobulin:
• Carpal tunnel syndrome is a common complication associated with long-term hemodialysis in patients
with renal failure.
• It results from the deposition of β2-microglobulin in joints, muscles, tendons, or ligaments.
• This accumulation can lead to compression of the median nerve as it passes through the carpal
tunnel, causing symptoms such as numbness, tingling, and weakness in the hand and fingers.
Carpal tunnel syndrome is a
common complication associated with long-term hemodialysis in patients with renal failure.
It results from the deposition of β2-microglobulin in joints, muscles, tendons, or ligaments.
This accumulation can lead to compression of the median nerve as it passes through the carpal tunnel,
causing symptoms such as numbness, tingling, and weakness in the hand and fingers.
Incorrect Options:
Option A) The abnormal protein deposited in extracellular tissue are immunoglobulin light chain:
• In long-term hemodialysis patients with renal failure, the abnormal protein deposited in extracellular
tissue is primarily β2-microglobulin, not the immunoglobulin light chain.
In long-term hemodialysis patients with renal failure, the abnormal protein deposited in extracellular tiss
ue is primarily β2-microglobulin, not the immunoglobulin light chain.
Option B) In patients with dialysis-associated amyloidosis, transthyretin is the predominant deposit:
• While transthyretin amyloidosis exists, it is not typically associated with dialysis.
• Dialysis-associated amyloidosis is primarily characterized by the deposition of β2-microglobulin as
amyloid fibrils, leading to complications such as carpal tunnel syndrome
While transthyretin amyloidosis exists, it is not typically associated with dialysis.
Dialysis-associated amyloidosis is primarily characterized by the deposition of β2-microglobulin as amy
loid fibrils, leading to complications such as carpal tunnel syndrome
Option D) Patients with generalized amyloidosis have an excellent prognosis:
• Patients with generalized amyloidosis, regardless of the underlying cause, typically have a guarded
prognosis.
• The prognosis depends on various factors, including the extent of organ involvement and the
underlying disease process.
Patients with generalized amyloidosis, regardless of the underlying cause, typically have a
guarded prognosis.
The prognosis depends on various factors, including the extent of organ involvement and the underlyin
g disease process.

Page 8

449
Previous Year Questions
1. Which immunoglobulin is responsible for the facial swelling, itching, and hypotension experienced by
a person brought to the emergency department after being stung by a bee?
A. IgA
B. IgE
C. IgM
D. IgG
----------------------------------------
2. Identify the given condition.

A. Pemphigus vulgaris
B. Leprosy
C. Mycosis fungoides
D. Visceral leishmaniasis
----------------------------------------
3. A patient presents with abdominal distention and intermittent fever. Peripheral smear findings are
normal. A tumor is resected and the histopathology image of the resected lymph node is shown below.
What is the most probable diagnosis ?

A. Angiolymphoma
B. Hodgkin’s lymphoma
C. IgG-4 related disease

450
D. Castleman disease
----------------------------------------
4. Acute graft rejection occurs within:
A. Few minutes
B. Week after the transplant to 3 months afterward
C. < 6 months
D. 6-12 months
----------------------------------------
5. In the context of Goodpasture syndrome, which organ is affected?
A. Liver
B. Adrenals
C. Kidney
D. Brain
----------------------------------------
6. Acute graft rejection occurs within?
A. Seconds to minutes
B. Days to weeks
C. Weeks to months
D. Months to years
----------------------------------------
7. A 3-year-old boy presents with recurrent bacterial infections and Pneumocystis Jiroveci pneumonia.
The flowcytometry analysis of the boy (Boxes 3 and 4) is shown below (Boxes 1 and 2 are normal).
What is the most likely diagnosis?

Page 2

451
A. Bare lymphocytes
B. Chronic granulocyte deficiency
C. Hyper IgM syndrome
D. DiGeorge syndrome
----------------------------------------
8. Which of the following cells is responsible for graft rejection?
A. Macrophages
B. T cells
C. B cells
D. Plasma cells
----------------------------------------
9. The X-ray of a middle-aged man showed multiple lytic lesions, following which a urine examination
was done. The sample revealed Bence Jones proteins. Which is the most common immunoglobulin
associated with this condition?
A. IgA
B. IgD
C. IgG
D. IgM
----------------------------------------
10. A young person with recurrent seasonal conjunctivitis presents with itchiness and eye-watering.
What is the most likely cell involved in the immediate phase of this condition?
A. Eosinophils
B. Mast cells
C. Neutrophils
D. Lymphocyte
----------------------------------------
11. Which of the following is not a part of innate immunity?
A. Alpha defensins

Page 3

452
B. B lymphocytes
C. Gamma delta T lymphocytes
D. Lectin pathway
----------------------------------------
12. Which of the following cytokines is not involved in the Th1 response?
A. IFN gamma
B. IL-2
C. IL-12
D. IL-4
----------------------------------------
13. The JAK-STAT pathway is seen in which of the following?
A. Vasopressin
B. Calcitonin
C. Leptin
D. Aldosterone
----------------------------------------
14. What does CD40 deficiency in a person signify?
A. B cell absent
B. T cell absent
C. IgM increase
D. IgG increase
----------------------------------------
15. Which type of amyloidosis is seen in the patients going through dialysis?
A. A-beta
B. β2-microglobulin
C. aTTR
D. AL
----------------------------------------
16. Identify the correct statements regarding amyloidosis. Amyloidosis is called so because of the
deposition of starch. Beta-pleated sheets are seen. AL is a type of secondary amyloidosis It is an
extracellular protein.
A. 1, 2 and 3
B. 1, 4
C. 2, 3 and 4
D. 1, 2 and 4
----------------------------------------

Page 4

453
17. A child is presenting with palpable blanching purpura, abdominal pain, and pain in the knee joint.
Urine examination showed hematuria and proteinuria. What is the most likely diagnosis?
A. Immune thrombocytopenic purpura
B. Dengue
C. Churg-Strauss syndrome
D. Henoch-Schonlein purpura
----------------------------------------
18. B-cells are induced to produce IgE by which of the following?
A. IL-2
B. IL-4
C. IL-1
D. IL-6
----------------------------------------
19. Graft from an identical twin is called:
A. Allograft
B. Autograft
C. Isograft
D. Xenograft
----------------------------------------
20. A person was brought to the emergency department with facial swelling, itching, and hypotension
following a bee sting. The mentioned clinical features are due to the increase in which of the following
immunoglobulins?
A. IgA
B. IgE
C. IgM
D. IgG
----------------------------------------
21. A Rh-negative mother delivered an Rh-positive baby, and Coomb’s test was positive for the baby.
What is the mechanism of hypersensitivity seen in this scenario?
A. Antigen-dependent cellular cytotoxicity
B. Antibody-dependent cellular cytotoxicity
C. Antigen-antibody immune complex deposition
D. CD4-T cell-mediated cytokine release
----------------------------------------
22. A 24-year-old female patient presented with malar rash, recurrent oral ulcers, and multiple joint
pains. She has no relevant past history and does not take any medication. On further investigations,
thrombocytopenia was seen, and the anti-Sm antibody was positive. Which of the following is the most
likely diagnosis for this patient?

Page 5

454
A. Scleroderma
B. Sjogren’s syndrome
C. Rheumatoid arthritis
D. Systemic lupus erythematosus
----------------------------------------
23. Which of the following conditions is associated with hypocalcemia, thymic hypoplasia, cardiac
abnormalities, and deletion of chromosome 22q11.2?
A. Williams syndrome
B. Pitt syndrome
C. Digeorge syndrome
D. Cri-du-chat syndrome
----------------------------------------
24. A 42-year-old female patient presents with a cough, a low-grade fever, and hemoptysis.
Investigations reveal a cavitary lesion on her right lung apex, which, on biopsy, reveals caseous
necrosis. The underlying pathophysiology is:
A. Enzyme degradation
B. Type-4 hypersensitivity reaction
C. Fibrinoid deposition
D. Sudden cut off of blood supply
----------------------------------------
25. Which of the following mediators is involved in the early phase of type 1 hypersensitivity? Mast cell
Histamine Neutrophils Prostaglandins Leukotrienes
A. 1, 5
B. 1, 2, 3, 4
C. 1, 2, 4, 5
D. 2, 3, 4
----------------------------------------
26. Given below is an electron microscopic image of the lung tissue. A patient infected with
SARS-CoV2 presents with ARDS. The infection would affect and expand which of the labeled layers?

Page 6

455
A. B
B. A
C. D
D. None of the above
----------------------------------------
27. All of the following conditions are related to the same core pathology, except:
A. Parkinson’s disease
B. Alzheimer's disease
C. Lewy body dementia
D. Multisystem atrophy
----------------------------------------
28. A child, upon disturbing a bee hive, had several stings. After a few minutes, the child developed
symptoms of shock, respiratory failure, and vascular collapse. Which of the following is the principal
mediator of this reaction?
A. Adrenaline
B. Histamine
C. Dopamine
D. Substance P
----------------------------------------
29. A 78-year-old male presented with complaints of fatigue, breathlessness, and palpitations. He has
no co-morbidities and does not smoke or drink. On examination, there was pedal edema, and vitals
were: PR-55 bpm, BP-100/70 mmHg, Spo2-94%. An ECG was taken, which showed no abnormalities,
after which he was given oxygen, and an ECHO revealed restrictive cardiomyopathy. The physician, in
doubt of a disease, did a cardiac biopsy, which showed apple-green birefringence when viewed under
polarized light after staining with Congo red stain. What is the diagnosis?
A. Heart failure
B. Cardiac amyloidosis
C. Hemosiderosis
D. Dystrophic calcification
----------------------------------------
30. What is the single most important long-term limitation of cardiac transplantation?
A. Allograft rejection
B. Allograft arteriopathy
C. Infection
D. Malignancy
----------------------------------------
31. Toluidine blue staining is used in the identification of which of the following cells?
A. Lymphocyte

Page 7

456
B. Fibroblast
C. Macrophage
D. Mast cell
----------------------------------------
32. Gamma delta T cell (yδ) receptors recognize:
A. Lipids without MHC proteins
B. Peptides with MHC proteins
C. CD20
D. CD3
----------------------------------------
33. Antibody-dependent cellular cytotoxicity (ADCC) is seen with :
A. NK cells only
B. Macrophages only
C. NK cells, neutrophils, and eosinophils
D. Neutrophils and eosinophils only
----------------------------------------
34. Type II Hypersensitivity is seen in:
A. Pernicious anaemia
B. Serum sickness
C. Arthus phenomenon
D. Pathergy phenomenon
----------------------------------------
35. Anti-centromere antibodies are seen in which of the following conditions?
A. SLE
B. Drug induced lupus
C. Sjogren syndrome
D. Scleroderma
----------------------------------------
36. Isotype switching is seen in:
A. Helper T cells
B. Activated B cells
C. Cytotoxic T cells
D. B cells
----------------------------------------
37. Xenograft is:

Page 8

457
A. Graft across species
B. Graft from same species
C. Graft from same individual
D. Graft from same organ
----------------------------------------
38. Which of the following is a type-2 hypersensitivity reaction?
A. Chronic kidney rejection
B. Autoimmune hemolytic anaemia
C. Arthus reaction
D. Mantoux test
----------------------------------------
39. True about anaphylactoid reactions are all except:
A. Involves degranulation of mast cells
B. IgE-mediated
C. Can occur on first exposure
D. Can be caused by NSAIDs
----------------------------------------
40. Which of the following about γδ T-cells is incorrect?
A. Kills extracellular bacteria by granulysin and perforin
B. Need antigen processing and presentation for it recognition
C. Direct killing of infected macrophages
D. Provide protection against mycobacteria
----------------------------------------
41. Basophils are activated by?
A. IL-5
B. INF-gamma
C. TNF-alpha
D. IL-2
----------------------------------------
42. Which gene is not involved in SCID?
A. BTK
B. JAK-3
C. RAG1
D. IL-7Rα
----------------------------------------

Page 9

458
43. Identify the correct statement regarding immune surveillance.
A. Anti CTLA-4 Ab is a new modality in the treatment of advanced cancers
B. Mutation of tumor suppressor genes decreases antigen toxicity
C. PD1/PDL1 activation leading to tumor suppression
D. Binding of CD28 leads to immune tolerance
----------------------------------------
44. Immune privilege site is:
A. Liver
B. Seminiferous tubule
C. Heart
D. Spinal cord
----------------------------------------
45. Graft from an identical twin is called:
A. Allograft
B. Autograft
C. Isograft
D. Xenograft
----------------------------------------
46. A teenager developed swelling of the face, lips, and tongue and also found it difficult to breathe
within a few minutes of consuming prawns. What is the type of hypersensitivity seen here?
A. Type 1
B. Type 2
C. Type 3
D. Type 4
----------------------------------------
47. Which of the following statements is true regarding amyloidosis?
A. SAA forms amyloid in multiple myeloma
B. On Congo red staining, amyloid shows apple green birefringence
C. In AL type amyloid is most commonly formed from kappa chain
D. β2 - macroglobulin forms amyloid in systemic senile amyloidosis
----------------------------------------
48. Match the following: Column A Column B 1. Omalizumab A. Anti-CD6 2. Itolizumab B. Anti-CD25 3.
Daclizumab C. Anti-IgE 4. Belimumab D. Anti-IgG1 E. Anti-B lymphocyte stimulator
Column A Column B
1. Omalizumab A. Anti-CD6
2. Itolizumab B. Anti-CD25

Page 10

459
3. Daclizumab C. Anti-IgE
4. Belimumab D. Anti-IgG1
E. Anti-B lymphocyte stimulator

A. 1-C; 2-A; 3-B; 4-D


B. 1-C; 2-D; 3-B; 4-E
C. 1-C; 2-A; 3-D; 4-B
D. 1-C; 2-A; 3-B; 4-E
----------------------------------------

Correct Answers
Question Correct Answer

Question 1 2
Question 2 1
Question 3 4
Question 4 2
Question 5 3
Question 6 3
Question 7 3
Question 8 2
Question 9 3
Question 10 2
Question 11 2
Question 12 4
Question 13 3
Question 14 1
Question 15 2
Question 16 4
Question 17 4
Question 18 2
Question 19 3
Question 20 2
Question 21 2
Question 22 4
Question 23 3
Question 24 2

Page 11

460
Question 25 3
Question 26 2
Question 27 2
Question 28 2
Question 29 2
Question 30 2
Question 31 4
Question 32 1
Question 33 3
Question 34 1
Question 35 4
Question 36 2
Question 37 1
Question 38 2
Question 39 2
Question 40 2
Question 41 1
Question 42 1
Question 43 1
Question 44 2
Question 45 3
Question 46 1
Question 47 2
Question 48 4

Solution for Question 1:


• The clinical features described following a bee sting, including facial swelling, itching, and
hypotension, are due to an allergic reaction. Allergic reactions are mediated by the immune system,
specifically involving the immunoglobulin IgE.
• IgE (Immunoglobulin E) is an antibody that plays a key role in allergic responses. When a person is
exposed to an allergen, such as bee venom, their immune system produces specific IgE antibodies
against that allergen. These IgE antibodies bind to specific receptors on mast cells and basophils,
which are cells involved in the immune response.
• Upon subsequent exposure to the same allergen, the allergen binds to the IgE antibodies attached to
mast cells and basophils. This triggers the release of various chemicals, including histamine,
leukotrienes, and prostaglandins. These chemicals are responsible for the characteristic symptoms of
an allergic reaction, such as facial swelling (angioedema), itching, and hypotension.

Page 12

461
• Histamine causes blood vessels to dilate, leading to increased permeability and fluid leakage,
resulting in facial swelling. It also stimulates nerve endings, leading to itching. In severe cases,
histamine-induced vasodilation can cause hypotension and even anaphylaxis, a life-threatening allergic
reaction.
Incorrect Choices:
• Option a. IgA: IgA is primarily involved in mucosal immunity, protecting the mucous membranes of the
respiratory, digestive, and genitourinary tracts. It is not typically associated with immediate
hypersensitivity reactions like those seen in response to bee stings.
• Option c. IgM: IgM is the first antibody produced during an immune response. It is involved in the
primary immune response and is not specifically associated with allergic reactions or the symptoms
described in the scenario.
• Option d. IgG: IgG is the most abundant antibody in the bloodstream and is involved in long-term
immunity. While IgG can play a role in some types of allergic reactions, it is not the primary
immunoglobulin responsible for the immediate hypersensitivity reactions seen in response to a bee
sting.

Solution for Question 2:


Correct Option A:
Pemphigus vulgaris is an autoimmune blistering disorder that affects the skin and mucous membranes.
It is characterized by the presence of intraepidermal blisters, which are caused by the loss of cell-to-ce
ll adhesion between keratinocytes.
Histopathological features:
Histopathological examination of a biopsy from a pemphigus vulgaris lesion shows intraepidermal blist
er formation with acantholysis, which is the separation of keratinocytes within the epidermis as seen in
the given image. This acantholysis is caused by the production of autoantibodies, specifically anti-des
moglein antibodies, which target proteins involved in cell adhesion.
Incorrect Option:
Option B. Leprosy: Leprosy, also known as Hansen's disease, is a chronic infectious disease caused b
y the bacteria Mycobacterium leprae. It primarily affects the skin, peripheral nerves, and mucous memb
ranes. Leprosy does not present with the characteristic intraepidermal blisters seen in the given image.
Option C. Mycosis fungoides: Mycosis fungoides is the most common type of cutaneous T-cell lympho
ma. It presents with a variety of skin lesions, including patches, plaques, and tumors. The characteristi
c features of mycosis fungoides are not seen in the given image, which shows intraepidermal blistering
.
Option D. Visceral leishmaniasis: Visceral leishmaniasis, also known as kala-azar, is a systemic parasit
ic infection caused by Leishmania parasites. Granulomatous inflammation, Phagocytosed parasites, H
yperplasia of reticuloendothelial system, Necrosis and fibrosis and Hemophagocytosis, the engulfment
of red blood cells, white blood cells, or platelets by macrophages are the features of it.

Page 13

462
Solution for Question 3:
Correct Option D.
• Castleman disease, also known as angiofollicular lymph node hyperplasia, is a rare
lymphoproliferative disorder that can involve a single lymph node or multiple lymph nodes. It is
characterized by abnormal lymph node architecture and an increased number of blood vessels within
the affected lymph nodes. The histopathology image of the resected lymph node is not provided, so we
cannot directly confirm the presence of these features. However, based on the information given
(abdominal distention, intermittent fever, and a resected lymph node), Castleman disease is a
reasonable differential diagnosis.
Incorrect Option
Option A. Angiolymphoma:
• Angiolymphoma, also known as epithelioid hemangioma, is a rare vascular tumor that can occur in the
skin, soft tissues, and other organs. It typically presents as a solitary skin lesion rather than a lymph
node involvement. Therefore, it is less likely to be the correct diagnosis in this case.
Option B. Hodgkin's lymphoma:
• Hodgkin's lymphoma is a type of lymphoma characterized by the presence of Reed-Sternberg cells,
which are large abnormal cells found in the lymph nodes. The peripheral smear findings in this patient
are reported as normal, which is not consistent with Hodgkin's lymphoma. Additionally, the
histopathology image of the resected lymph node is not provided, so it is difficult to evaluate the
presence of Reed-Sternberg cells.
Option C. IgG-4 related disease:
• IgG4-related disease is a relatively newly recognized fibroinflammatory condition that can affect
various organs, including the lymph nodes. It is characterized by lymphoplasmacytic infiltration and
fibrosis. However, without the histopathology image, it is challenging to determine if the lymph node in
question shows the typical histological features associated with IgG4-related disease. Additionally, the
clinical presentation of abdominal distention and intermittent fever is not specific to IgG4-related
disease.

Solution for Question 4:


Correct Option B - Week after the transplant to 3 months afterward:
• Acute graft rejection can occur within the first week after transplantation and typically continues to be
a concern for the first few months post-transplantation.
• This timeframe is crucial because the recipient's immune system is actively monitoring the
transplanted organ, and acute rejection reactions are most common during this period.
• Intense immunosuppressive therapy is usually given during these initial months to prevent or minimize
the risk of acute rejection.
Incorrect Options:
• Options A, C, and D are incorrect.

Page 14

463
Solution for Question 5:
Correct option: C
• Goodpasture syndrome is an autoimmune disorder characterized by the presence of autoantibodies
directed against the basement membrane of the kidneys and lungs. It primarily affects the kidneys and
lungs, but renal involvement is more prominent. The autoantibodies target the type IV collagen in the
glomerular basement membrane, leading to inflammation, damage, and ultimately, kidney dysfunction.
Incorrect options:
Option A: Liver: Goodpasture syndrome does not primarily involve the liver. The main organs affected
are the kidneys and lungs.
Option B: Adrenals: Adrenal involvement is not a characteristic feature of Goodpasture syndrome. The
adrenal glands are not typically targeted by the autoimmune response in this disorder.
Option D:. Brain: Goodpasture syndrome primarily affects the kidneys and lungs and does not involve t
he brain.

Solution for Question 6:


Correct Option: Option C: Weeks to months: This option is correct because weeks to months is typicall
y associated with acute graft rejection.
Incorrect options:
Option A: Seconds to minutes: Rejection occurring within seconds to minutes after transplantation is ex
tremely rare and not characteristic of acute graft rejection.
Option B: Days to weeks: This option is incorrect.
Option D: Months to years: This chronic graft rejection.

Solution for Question 7:


Correct option C: Hyper IgM syndrome:
• Hyper IgM syndrome is a primary immunodeficiency disorder characterized by a deficiency in the
production of immunoglobulin class-switched antibodies (IgG, IgA, and IgE).
• Patients with hyper IgM syndrome have normal or elevated levels of IgM but low levels or absence of
other antibody classes.
• The flow cytometry analysis showing elevated IgM levels and low IgG levels supports the diagnosis of
hyper IgM syndrome.
Incorrect options: Options A, B, and D are incorrect. Refer to the explanation of the correct answer.

Solution for Question 8:

Page 15

464
Correct Option B - T cells:
• When a graft is transplanted into a recipient, the immune system recognizes the graft as foreign and
triggers an immune response.
• This response is primarily mediated by T cells, which can recognize and respond to specific foreign
antigens.
• The T cells recognize and bind to the foreign antigens expressed by the graft.
• This triggers the activation of other immune cells, such as macrophages, which can help to engulf and
destroy the foreign tissue.
Incorrect Options:
• Options A, C, and D are incorrect.

Solution for Question 9:


Correct Option C - IgG:
• The patient's presentation with multiple lytic lesions on X-ray and Bence-Jones proteins in urine
examination is suggestive of multiple myeloma.
• In multiple myeloma, abnormal plasma cells accumulate in the bone marrow and produce excessive
amounts of a single type of antibody, or immunoglobulin.
• Bence Jones proteins are fragments of immunoglobulin light chains that are filtered by the kidneys
and excreted in the urine.
• The most common immunoglobulin associated with multiple myeloma is IgG, which accounts for
about 50% of cases.
• IgA is the second most common type, accounting for about 20-25% of cases.
• IgM is rare in multiple myeloma, and IgD is not typically associated with this condition.
Incorrect Options:
• Options A, B, and D are incorrect. Refer to the explanation of the correct answer.

Solution for Question 10:


Correct Option B - Mast cells:
• Re-exposure to any environmental allergens by inhalation or ingestion in an already sensitized
individual causes mast cell activation.
• The allergen binds to an IgE antibody in the mast cells and can cause degranulation of the mast cells
and the release of primary mediators stored inside them.
• There are two responses: immediate and late-phase responses.
• An immediate response occurs within 5-30 minutes of exposure and is characterized by
vasodilatation, vascular leakage, and glandular secretions.

Page 16

465
• The late-phase response occurs 2-8 hours after exposure and is characterized by eosinophilic,
neutrophilic, basophilic, and monocytic infiltration of tissues, resulting in damage to the epithelial cell.
Incorrect Options:
• Options A, C, and D are not involved in the immediate phase response.

Solution for Question 11:


Correct Option B - B lymphocytes:
• B lymphocytes are involved in adaptive immunity.
• Adaptive immunity is a more specific and targeted response to pathogens and involves the production
of antibodies by B lymphocytes and the activation of T lymphocytes.
Incorrect Options:
• Options A, C, and D are a part of innate immunity.
Option A - Alpha defensins:
• They are small peptides that are produced by neutrophils, Paneth cells, and other cells of the innate
immune system.
• They have antimicrobial properties and can directly kill bacteria, viruses, and fungi.
Option C - Gamma delta T lymphocytes:
• They are a subset of T cells that have innate-like properties.
• They can recognize and respond to a broad range of antigens, including those that are not processed
and are presented by the major histocompatibility complex.
Option D - Lectin pathway:
• It is a component of the complement system, which is part of innate immunity.
• It is activated by the binding of specific lectin proteins to carbohydrates on the surface of pathogens,
leading to the activation of complement proteins and the formation of membrane attack complexes that
can lyse the pathogen.

Solution for Question 12:


Correct Option D - IL-4:
• IL-4 is primarily associated with the Th2 response.
• It is produced by activated Th2 cells and plays a critical role in promoting antibody production, class
switching to IgE, and allergic responses.
• IL-4 also inhibits the production of IFN gamma, which is a characteristic cytokine of the Th1 response.
Incorrect Options:
• Option A, B, and C are involved in the Th1 response.

Page 17

466
Solution for Question 13:
Correct Option C - Leptin:
• Leptin is a hormone secreted by adipose tissue and plays a key role in regulating energy balance and
appetite.
• Leptin signalling involves the binding of leptin to its receptor, leading to the activation of the JAK-STAT
pathway.
Incorrect Options:
Option A - Vasopressin:
• The vasopressin hormone, also known as antidiuretic hormone (ADH), primarily acts through the V2
receptor and regulates water reabsorption in the kidneys.
• The signalling pathway of vasopressin involves the activation of the V2 receptor, which primarily
activates the G-protein-coupled receptor signalling pathway, leading to an increase in cyclic adenosine
monophosphate (cAMP) levels.
Option B - Calcitonin:
• The signalling pathway of calcitonin involves the activation of the calcitonin receptor, which activates
the adenylyl cyclase pathway and inhibits osteoclast activity.
Option D - Aldosterone:
• The signalling pathway of aldosterone primarily involves the mineralocorticoid receptor.

Solution for Question 14:


Correct Option A - B cell absent:
• CD40 signalling is essential for B cell activation, maturation, and the production of antibodies.
• In the absence of CD40, B cells would not receive the necessary signals for proper activation and
differentiation.
• This would result in impaired B cell function and reduced production of antibodies.
Incorrect Options:
• Options B, C, and D are incorrect.

Solution for Question 15:


Correct Option B - β2-microglobulin:
• Beta-2 microglobulin amyloidosis is a type of amyloidosis that occurs as a result of long-term
hemodialysis in patients with end-stage renal disease.

Page 18

467
• It is characterized by the deposition of beta-2 microglobulin protein fibrils in various tissues,
particularly in joints and periarticular structures, leading to destructive arthropathy.
Incorrect Options:
Option A - A-beta:
• A-beta amyloidosis refers to the accumulation of amyloid-beta (A-beta) peptide in the brain, which is
commonly associated with Alzheimer's disease.
Option C - aTTR:
• aTTR stands for amyloidogenic transthyretin amyloidosis.
• It is caused by the deposition of misfolded transthyretin protein in various organs, leading to organ
dysfunction.
Option D - AL:
• AL amyloidosis, also known as primary amyloidosis, is associated with the deposition of abnormal
immunoglobulin light chains produced by clonal plasma cells.

Solution for Question 16:


Correct Option D - 1, 2, and 4:
1. Amyloidosis is called so because of the deposition of starch:
• Amyloidosis is the deposition of abnormal proteins in tissues, leading to organ dysfunction.
• The term amyloid originates from the early mistaken identification of the deposits as starch-like
material, although they are composed of misfolded proteins.
2. Beta-pleated sheets are seen:
• The hallmark of amyloid deposits is the presence of beta-pleated sheets formed by the misfolding of
proteins into a specific structure.
• This abnormal conformation makes the proteins insoluble and resistant to degradation, leading to their
accumulation in tissues.
4. It is an extracellular protein:
• Amyloid deposits accumulate extracellularly in various tissues and organs throughout the body.
• By interfering with normal cellular function and structure, these deposits can lead to tissue damage
and organ dysfunction.
Incorrect Options:
• Options A, B, and C are incorrect.
3. AL is a type of secondary amyloidosis: AL amyloidosis is a primary form of amyloidosis.

Solution for Question 17:


Correct Option D - Henoch-Schonlein purpura:

Page 19

468
• Henoch-Schonlein purpura (HSP) is a systemic vasculitis that primarily affects small blood vessels,
commonly seen in children.
• The characteristic clinical features include palpable purpura, abdominal pain, joint pain, and renal
involvement.
• Hematuria and proteinuria are common findings in urine examination due to kidney involvement.
• The symptoms may be preceded by an upper respiratory tract infection.
Incorrect Options:
Option A - Immune thrombocytopenic purpura:
• While ITP can present with purpura and thrombocytopenia, it does not typically involve joint pain,
abdominal pain, or renal manifestations such as hematuria and proteinuria.
Option B - Dengue:
• Dengue is a viral infection that can cause fever, rash, and joint pain, but it does not typically present
with palpable purpura or renal involvement.
Option C - Churg-Strauss syndrome:
• Churg-Strauss syndrome, also known as eosinophilic granulomatosis with polyangiitis, is a rare
systemic vasculitis that primarily affects medium and small blood vessels.
• It is characterized by asthma, eosinophilia, and systemic vasculitis.

Solution for Question 18:


Correct Option B - IL-4:
• IL-4 is a cytokine that plays a crucial role in the differentiation and activation of B-cells.
• IL-4 stimulates B-cells to undergo class switching from IgM to IgE production and promotes the growth
and survival of IgE-producing B-cells.
Incorrect Options:
Options A, C, and D are incorrect.

Solution for Question 19:


Correct Option C - Isograft:
• An isograft refers to a graft or transplant of tissue between genetically identical individuals, such as
identical twins.
• Since identical twins share the same genetic makeup, an isograft has the highest chance of success
with minimal risk of rejection.
• The immune system recognizes the transplanted tissue as its own and does not mount an immune
response against it.
Incorrect Options:
Option A - Allograft:

Page 20

469
• An allograft refers to a graft or transplant of tissue between individuals of the same species but with
different genetic makeup, such as between two non-identical humans.
• It involves matching certain tissue compatibility factors (such as blood type and human leukocyte
antigens) to minimize the risk of rejection.
• In an allograft, there is a possibility of immune rejection due to genetic differences between the donor
and recipient.
Option B - Autograft:
• An autograft is a graft or transplant of tissue from one part of an individual's body to another part.
• In an autograft, the donor and recipient are the same person.
• Autografts are often used because they have a very low risk of rejection since the tissue is taken from
the individual's own body.
Option D - Xenograft:
• A xenograft refers to a graft or transplant of tissue between individuals of different species.
• For example, if tissue from an animal is transplanted into a human, it would be considered a
xenograft.
• Xenografts pose significant challenges due to immune rejection and compatibility issues between
different species.

Solution for Question 20:


Correct Option B - IgE:
• The patient's presentation with facial swelling, itching, and hypotension following a bee sting is a type
1 hypersensitivity reaction and is mediated by IgE.
Incorrect Options:
• Options A, C, and D are not involved in type 1 hypersensitivity reactions.

Solution for Question 21:


Correct Option B - Antibody-dependent cellular cytotoxicity:
• The above case is that of a type-2 hypersensitivity reaction, which is due to antibody-dependent
cellular cytotoxicity.
Incorrect Options:
Option A - Antigen-dependent cellular cytotoxicity: Antigen-dependent cellular cytotoxicity is not an app
ropriate term as cytotoxicity is driven by antibodies in the above case.
Option C - Antigen-antibody immune complex deposition: Antigen-antibody immune complex depositio
n is seen in immune complex-mediated or type-3 hypersensitivity reactions.
Option D
- CD4-T cell-mediated cytokine release: CD4-T cell-mediated cytokine release is responsible for T
cell-mediated or type IV hypersensitivity reactions.

Page 21

470
Solution for Question 22:
Correct Option D - Systemic lupus erythematosus:
• Systemic lupus erythematosus is a connective tissue disorder characterized by a malar rash,
recurrent oral ulcers, and multiple joint pains. Thrombocytopenia and anti-Sm antibodies also support
the diagnosis of systemic lupus erythematosus.
Incorrect Options:
• Options A, B, and C are incorrect.

Solution for Question 23:


Correct Option C - Digeorge syndrome:
• In DiGeorge syndrome, there is a 22q11.2 deletion, hypocalcemia, thymic hypoplasia, and cardiac
abnormalities. The deficits can be memorized by the following mnemonic:
CATCH-22:
• Cardiac anomalies
• Abnormal facies
• Thymic aplasia
• Cleft palate
• Hypocalcemia
• Deletion 22q11.2
Incorrect Options:
Option A - Williams syndrome: Williams syndrome occurs due to deletion at chromosome 7.
Option B - Pitt syndrome: Pitt syndrome occurs due to a
partial deletion or monosomy of chromosome 4.
Option D - Cri-du-chat syndrome: Cri-du-chat syndrome is due to a
partial deletion of the short arm of chromosome 5.

Solution for Question 24:


Correct Option B - Type-4 Hypersensitivity Reaction:
• The presentation of a cavitary lesion on the right lung apex with caseous necrosis suggests
pulmonary tuberculosis.
• In pulmonary tuberculosis, the underlying pathophysiology involves a type-4 hypersensitivity reaction,
also known as a delayed-type hypersensitivity reaction.

Page 22

471
• In the case of tuberculosis, the immune response is characterized by the activation of T lymphocytes,
particularly CD4+ T cells.
• These T cells recognize antigens presented by macrophages infected with Mycobacterium
tuberculosis.
• The activated CD4+ T cells release cytokines, such as interferon-gamma (IFN-γ), which leads to the
recruitment of macrophages to the site of infection.
• The activated macrophages attempt to eliminate the bacteria through phagocytosis.
• However, Mycobacterium tuberculosis has the ability to survive within the phagosomes of
macrophages and evade complete destruction.
• This results in the formation of granulomas, which are organized collections of macrophages
surrounded by T cells.
• Within the granulomas, there is central caseous necrosis, characterized by a cheese-like appearance.
• The type 4 hypersensitivity reaction is responsible for the tissue damage seen in tuberculosis.
Incorrect Options:
Option A - Enzyme degradation:
• While enzymes may be involved in the breakdown of tissue during the course of infection, the primary
mechanism of tissue damage in tuberculosis is the type-4 hypersensitivity reaction.
Option C - Fibrinoid deposition:
• Fibrinoid deposition refers to the accumulation of fibrin-like material in the walls of blood vessels.
• It is typically associated with conditions such as vasculitis.
Option D - Sudden cutoff of blood supply:
• The sudden cutoff of blood supply, also known as infarction, occurs when the blood supply to a tissue
is abruptly interrupted, leading to ischemia and subsequent tissue death.
• In the case of tuberculosis, the tissue damage is primarily caused by the immune response and the
formation of granulomas.

Solution for Question 25:


Correct Option C - 1, 2, 4, 5:
• The mediators involved in the early phase of type 1 hypersensitivity include:
1. Mast cells:
• Mast cells are the primary effector cells in type 1 hypersensitivity.
• They are activated by allergens cross-linking with IgE antibodies bound to their surface.
2. Histamine:
• Histamine is one of the key mediators released by mast cells.
• It plays a major role in the early phase of type 1 hypersensitivity.
• Histamine causes vasodilation, increased vascular permeability, smooth muscle contraction, and
mucus production.

Page 23

472
4. Prostaglandins:
• Prostaglandins are lipid mediators released by mast cells during the early phase of hypersensitivity
reactions.
• They contribute to vasodilation, vascular permeability, and bronchoconstriction.
5. Leukotrienes:
• Leukotrienes are a group of lipid mediators released by mast cells during the early phase of type 1
hypersensitivity.
• They promote bronchoconstriction, increased vascular permeability, and the recruitment of other
inflammatory cells.
Incorrect Options:
• Options A, B, and D are incorrect.

Solution for Question 26:


Correct Option B - A:
• In the electron microscopic image of the lung tissue, the layer labeled "A" represents the endothelium.
• SARS-CoV-2, the virus causing COVID-19, primarily affects the respiratory system and can lead to
acute respiratory distress syndrome (ARDS).
• In ARDS, the virus can directly invade the endothelial cells lining the blood vessels in the lungs.
• The viral invasion of endothelial cells can disrupt their normal function and integrity, leading to
increased vascular permeability, inflammation, and the development of pulmonary oedema.
Incorrect Options:
Option A - B:
• In the electron microscopic image of the lung tissue, the layer labeled "B" represents the basal
laminae.
• The basal laminae provide structural support to the epithelial cells and separate them from the
underlying connective tissue.
• While basal laminae may be involved in certain lung diseases, they are not the primary site of
infection in SARS-CoV-2-related ARDS.
Option C - D:
• In the electron microscopic image of the lung tissue, the layer labeled "D" represents the epithelium.
• The epithelium lines the airways and is involved in the defense mechanisms of the respiratory system.
• While it can be affected by SARS-CoV-2, the primary target and primary pathological changes occur
in the endothelium.
Option D - None of the above:
• This option is incorrect.

Page 24

473
Solution for Question 27:
Correct Option B - Alzheimer's disease:
• Alzheimer's disease is not directly related to the same core pathology as the other conditions listed.
• While Parkinson's disease, Lewy body dementia, and multisystem atrophy are all neurodegenerative
disorders that share a common core pathology involving the accumulation of abnormal protein
aggregates called Lewy bodies, Alzheimer's disease is characterized by different pathological features.
• In Alzheimer's disease, the main pathological hallmarks are the accumulation of beta-amyloid plaques
and neurofibrillary tangles composed of hyperphosphorylated tau protein.
Incorrect Options:
• Options A, C, and D are incorrect. Refer to the explanation of the correct answer.

Solution for Question 28:


Correct Option B - Histamine:
• Histamine is the principal mediator released in the skin during an immediate bee venom allergy.
• Histamine causes various symptoms like vasodilation, increased capillary permeability, and smooth
muscle contraction.
Incorrect Options:
• Options A, C, and D are incorrect.

Solution for Question 29:


Correct Option B - Cardiac amyloidosis:
• The patient's presentation of fatigue, breathlessness, palpitations, and pedal edema, along with the
findings of restrictive cardiomyopathy on the echocardiogram and cardiac biopsy results, specifically
the presence of apple-green birefringence when viewed under polarized light after staining with Congo
red stain, are characteristic of amyloid deposition in the heart.
Incorrect Options:
• Options A, C, and D do not show apple-green birefringence when viewed under polarized light.

Solution for Question 30:


Correct Option B - Allograft Arteriopathy:
• The single most important long-term limitation of cardiac transplantation is allograft arteriopathy, also
known as cardiac allograft vasculopathy (CAV).

Page 25

474
• Allograft arteriopathy refers to the progressive narrowing and occlusion of the coronary arteries of the
transplanted heart.
Incorrect Options:
• Options A, C, and D are incorrect.

Solution for Question 31:


Correct Option D - Mast cell:
• Toluidine blue staining is primarily used for the identification of mast cells.
• Mast cells contain metachromatic granules that can be selectively stained by toluidine blue.
• The dye binds to the granules, causing them to appear purple or blue.
• This staining technique helps visualize and identify mast cells in histological sections, particularly in
the assessment of mast cell-related diseases such as mastocytosis or allergic reactions.
Incorrect Options:
• Toluidine blue staining is not used for options A, B, and C.

Solution for Question 32:


Correct Option A - Lipids without MHC proteins:
• Gamma delta T cell (γδ) receptors are capable of recognizing lipids without the need for major
histocompatibility complex (MHC) proteins.
• This means that γδ T cells can directly interact with certain lipids, glycolipids, and phospholipids,
allowing them to respond to lipid antigens independently of MHC presentation.
Incorrect Options:
Option B - Peptides with MHC proteins:
• Alpha beta T cell receptors (αβ TCRs) are predominantly responsible for recognizing peptide
antigens, which are typically presented by MHC molecules on the surface of antigen-presenting cells
(APCs).
• Peptides derived from pathogens or abnormal cellular processes are presented by MHC molecules,
and αβ TCRs bind to these peptide-MHC complexes to initiate an immune response.
Option C - CD20:
• CD20 is a cell surface protein expressed primarily on the surface of B cells.
• It is not directly related to the recognition of antigens by T-cell receptors.
• CD20 is mainly targeted by therapeutic antibodies in the treatment of certain B-cell lymphomas and
autoimmune diseases, such as rheumatoid arthritis and multiple sclerosis.
Option D - CD3:
• CD3 is a complex of proteins associated with the T cell receptor (TCR) on the surface of T cells.

Page 26

475
• It is involved in the signal transduction process upon TCR engagement with peptide-MHC complexes.
• CD3 proteins transmit activation signals to the T cell when the TCR recognizes its specific antigen,
leading to T cell activation and an immune response.
• CD3 is not directly responsible for antigen recognition but is crucial for T-cell signalling.

Solution for Question 33:


Correct Option C - NK cells, Neutrophils, and Eosinophils:
• NK cells, neutrophils, and eosinophils are all involved in ADCC.
• NK cells have Fc receptors that enable them to recognize and destroy antibody-bound target cells.
• Neutrophils and eosinophils also possess Fc receptors and can participate in ADCC by releasing
cytotoxic substances or producing reactive oxygen species to eliminate antibody-coated cells.
Incorrect Options:
• Options A, B, and D are incorrect. Refer to the explanation of the correct answer.

Solution for Question 34:


Correct Option A - Pernicious Anaemia:
• In pernicious anemia, the immune system produces antibodies against intrinsic factors or parietal cells
in the stomach.
• These antibodies interfere with the absorption of vitamin B12, leading to its deficiency.
• It is an example of type II hypersensitivity.
Incorrect Options:
Option B - Serum sickness:
• Serum sickness is a type III hypersensitivity reaction that occurs in response to the administration of
certain medications or the injection of foreign proteins, such as antivenoms or serum.
• It involves the formation of immune complexes that deposit in various tissues and activate the
complement system.
Option C - Arthus phenomenon:
• Arthus phenomenon is a localized immune complex-mediated hypersensitivity reaction.
• It occurs when there is an excessive immune response at the site of antigen exposure, usually due to
repeated exposure to an antigen.
Option D - Pathergy phenomenon:
• Pathergy phenomenon refers to the development of a skin lesion in response to minor trauma or
injury.
• It is commonly seen in certain conditions, like Behçet's disease.

Page 27

476
Solution for Question 35:
Correct Option D - Scleroderma:
• Anti-centromere antibodies are strongly associated with systemic sclerosis (scleroderma), specifically
the limited cutaneous form of the disease known as limited systemic sclerosis or CREST syndrome.
• These antibodies are found in approximately 60-80% of patients with limited systemic
sclerosis/CREST syndrome.
• Anti-centromere antibodies are considered a specific marker for this subtype of systemic sclerosis.
Incorrect Options:
• Anti-centromere antibodies are not seen in options A, B, and C.

Solution for Question 36:


Correct Option B - Activated B cells:
• Isotype switching, also known as class switching or immunoglobulin class switch recombination
(CSR), is a process that occurs in activated B cells.
• During this process, B cells change the class of antibodies they produce from IgM to other antibody
isotypes such as IgG, IgA, or IgE.
• Isotype switching allows B cells to produce antibodies with different effector functions while retaining
the specificity of the antigen-binding region.
Incorrect Options:
• Options A, C, and D are incorrect. Refer to the explanation of the correct answer.

Solution for Question 37:


Correct Option A - Graft across species:
• Xenograft refers to a type of graft or transplantation that involves transferring tissues or organs from
one species to another.
Incorrect Options:
• Options B, C, and D are incorrect. Refer to the explanation of the correct answer.

Solution for Question 38:


Correct Option B - Autoimmune hemolytic anaemia:

Page 28

477
• Type-2 hypersensitivity reactions are mediated by antibodies, particularly IgG or IgM, binding to
antigens on the surface of target cells, leading to their destruction or dysfunction.
• Autoimmune hemolytic anemia is an example of a type-2 hypersensitivity reaction.
• In autoimmune hemolytic anemia, autoantibodies are produced against the antigens on the surface of
red blood cells, resulting in their premature destruction by phagocytes in the spleen and liver.
Incorrect Options:
Option A - Chronic kidney rejection:
• Chronic kidney rejection involves a complex immune response, including cellular and humoral
components.
Option C - Arthus reaction:
• Arthus reaction is a localized immune complex-mediated hypersensitivity reaction that occurs when
preformed antibodies react with antigens in the blood vessel walls, leading to inflammation and tissue
damage.
• It is considered a type-3 hypersensitivity reaction.
Option D - Mantoux test:
• Mantoux test is a type-4 hypersensitivity reaction.

Solution for Question 39:


Correct Option B - IgE-mediated:
• Anaphylactoid reactions are not mediated by IgE antibodies.
• Unlike true anaphylaxis, anaphylactoid reactions are triggered by non-immunologic mechanisms,
such as the direct activation of mast cells or the complement system.
Incorrect Options:
Option A - Involves degranulation of mast cells:
• Anaphylactoid reactions involve the rapid release of mediators, such as histamine, from mast cells
and basophils.
• These mediators cause symptoms similar to those seen in anaphylaxis, but anaphylactoid reactions
are not IgE-mediated.
Option C - Can occur on first exposure:
• Anaphylactoid reactions can occur on the first exposure to a triggering agent.
• Unlike some allergic reactions that require sensitization and previous exposure, anaphylactoid
reactions can happen without prior exposure or sensitization.
Option D - Can be caused by NSAIDs:
• NSAIDs (non-steroidal anti-inflammatory drugs) can potentially trigger anaphylactoid reactions, but
they are not the most common cause.
• Anaphylactoid reactions can be caused by various substances, such as certain drugs (e.g., contrast
media), radiocontrast dyes, intravenous fluids, and some food additives.

Page 29

478
• It is important to note that anaphylactoid reactions to NSAIDs are not IgE-mediated but rather due to
direct activation of mast cells.

Solution for Question 40:


Correct Option B - Need antigen processing and presentation for it recognition:
• αβ T-cells and γδ T-cells do not require antigen processing and presentation by major
histocompatibility complex (MHC) molecules for recognition.
• They can recognize and respond to antigens directly, including certain stress-induced molecules and
microbial products.
Incorrect Options:
Option A - Kills extracellular bacteria by granulysin and perforin:
• γδ T-cells can kill extracellular bacteria through the release of granulysin and perforin, which are
cytotoxic molecules.
Option C - Direct killing of infected macrophages:
• γδ T-cells can directly kill infected macrophages, which helps in the control of intracellular infections.
Option D - Provide protection against mycobacteria:
• γδ T-cells have been shown to provide protection against mycobacterial infections, including
Mycobacterium tuberculosis, the causative agent of tuberculosis.
• They contribute to the immune response by producing cytokines and exerting cytotoxic effects.

Solution for Question 41:


Correct Option A - IL-5:
• Basophils, along with eosinophils and mast cells, are activated by IL-5, which is an interleukin
involved in the regulation of immune responses, particularly in allergic reactions and parasitic
infections.
Incorrect Options:
Option B - INF-gamma: INF-gamma primarily activates macrophages and plays a
role in cellular immune responses.
Option C - TNF-alpha: TNF-alpha is a
cytokine produced by activated macrophages and other immune cells.
Option D - IL-2: IL-2 stimulates the proliferation and activation of T-cells and natural killer (NK) cells.

Page 30

479
Solution for Question 42:
Correct Option A - BTK:
• BTK is involved in X-linked agammaglobulinemia (XLA), which is a primary immunodeficiency
disorder characterized by the absence or significantly reduced levels of mature B cells.
• Mutations in the BTK gene impair B cell development and function, leading to a lack of antibody
production.
• XLA is not considered severe combined immunodeficiency (SCID).
Incorrect Options:
Option B - JAK-3:
• Mutations in the JAK-3 gene can cause severe combined immunodeficiency (SCID).
• JAK-3 is involved in the signalling pathway of certain cytokines important for immune cell
development and function.
• Loss-of-function mutations in JAK-3 disrupt the development of T cells, B cells, and natural killer (NK)
cells, resulting in SCID.
Option C - RAG1:
• Mutations in the RAG1 gene can cause severe combined immunodeficiency (SCID).
• RAG1 is crucial for the rearrangement of immunoglobulin and T cell receptor genes during
lymphocyte development.
• Loss-of-function mutations in RAG1 impair the development of functional T and B cells, leading to
SCID.
Option D - IL-7Rα:
• Mutations in the IL-7Rα gene can cause severe combined immunodeficiency (SCID).
• IL-7Rα is a subunit of the receptor for interleukin-7, a cytokine involved in the development and
survival of T cells.
• Defects in IL-7Rα lead to impaired T cell development, resulting in SCID.

Solution for Question 43:


Correct Option A - Anti CTLA-4 Ab is a new modality in the treatment of advanced cancers:
• Immune surveillance refers to the process by which the immune system detects and eliminates
cancerous cells in the body.
• It plays a crucial role in protecting against the development and progression of cancer.
• CTLA-4 (cytotoxic T-lymphocyte-associated protein 4) is a checkpoint protein that negatively
regulates T-cell activation.
• Antibodies targeting CTLA-4, such as ipilimumab, have been developed as immunotherapeutic agents
and have shown promising results in the treatment of advanced cancers by enhancing the anti-tumor
immune response.
Incorrect Options:

Page 31

480
Option B - Mutation of tumor suppressor genes decreases antigen toxicity:
• Tumor suppressor genes play a critical role in regulating cell growth and preventing the development
of cancer.
• Mutations in tumor suppressor genes can lead to the loss of their normal functions, resulting in
increased susceptibility to tumor formation, not decreased antigen toxicity.
Option C - PD1/PDL1 activation leading to tumor suppression:
• PD-1 (programmed cell death protein 1) and PD-L1 (programmed death-ligand 1) are immune
checkpoint proteins involved in regulating immune responses.
• Activation of the PD-1/PD-L1 pathway can inhibit T-cell activity and promote immune tolerance, which
can actually facilitate tumor growth and progression rather than tumor suppression.
Option D - Binding of CD28 leads to immune tolerance:
• CD28 is a co-stimulatory molecule expressed on T cells that plays a crucial role in T-cell activation
and proliferation.
• The binding of CD28 to its ligands on antigen-presenting cells provides a co-stimulatory signal
necessary for the activation of T cells.
• It does not lead to immune tolerance but rather promotes immune activation and response against
antigens.

Solution for Question 44:


Correct Option B - Seminiferous tubule:
• The seminiferous tubules in the testes are considered immune privilege sites.
• These tubules play a crucial role in sperm production.
• The blood-testis barrier, formed by specialized cells in the walls of the seminiferous tubules, limits the
immune response within the testes.
• This barrier helps protect developing sperm from the immune system, preventing potential
autoimmune reactions against sperm antigens.
Incorrect Options:
• Options A, C, and D are not immune privilege sites.

Solution for Question 45:


Correct Option C - Isograft:
• Isograft refers to tissue transplanted between genetically identical twins.
Incorrect Options:
• Options A, B, and D are incorrect. Refer to the explanation of the correct answer.

Page 32

481
Solution for Question 46:
Correct Option A - Type-1:
• The above clinical case is suggestive of an anaphylactic reaction to prawns, which is a type-1
hypersensitivity reaction.
• Type I hypersensitivity is also known as an immediate reaction and involves the IgE-mediated release
of antibodies against the soluble antigen.
• This results in mast cell degranulation and the release of histamine and other inflammatory mediators.
Incorrect Options:
• Options B, C, and D are incorrect.

Solution for Question 47:


Correct Option B - On Congo red staining, amyloid shows apple green birefringence:
• Congo red, when used in conjunction with polarized light microscopy, produces a characteristic apple
green birefringence in amyloid deposits.
• This staining property is a key diagnostic feature of amyloidosis and helps differentiate amyloid
deposits from other substances.
Incorrect Options:
Option A - SAA forms amyloid in multiple myeloma: In multiple myeloma, amyloidosis is usually associ
ated with the deposition of immunoglobulin light chains, specifically the lambda chain, known as AL am
yloidosis.
Option C - In AL type amyloid is most commonly formed from kappa chain: In AL amyloidosis, the amyl
oid fibrils are most commonly formed from the lambda chain.
Option D - β2-macroglobulin forms amyloid in systemic senile amyloidosis: Systemic senile amyloidosi
s is most commonly associated with the deposition of amyloid fibrils derived from transthyretin.

Solution for Question 48:


Correct Option D - 1-C; 2-A; 3-B; 4-E:
Incorrect Options:
• Options A, B, and C are incorrect. Refer to the explanation of the correct answer.

Page 33

482
Vasculitis
1. A 19-year-old male, a known HIV Positive patient, developed cutaneous lesions for the past 1 week,
as shown in the picture below. On examination, purplish papules were noted, not blanching on the
diascopy. Which of the following is the tissue of origin of this lesion?
(or)
Which of the following is the given lesion's tissue of origin?

A. Lymphoid
B. Vascular
C. Neural
D. Muscular
----------------------------------------
2. Which of the following is associated with Kaposi's sarcoma?
A. Caused by human herpes virus 8
B. Seen in immune-competent patients
C. Proliferative tumour of the lymphoid origin
D. Surgery is the treatment of choice
----------------------------------------
3. Which of the following organs are most commonly involved in a patient with Granulomatosis with
polyangiitis?
A. Cutaneous
B. Lung and kidney
C. Heart and kidney
D. Nervous system
----------------------------------------
4. What's the likely diagnosis for a 23-year-old man presenting with nasal obstruction, serosanguinous
discharge, cough with bloody sputum for 1 month, and cavitated lesions and nodules in both lungs,
maxillary sinus obliteration, hematuria and RBC casts, elevated c-ANCA levels and given open lung
biopsy?
(or)

483
What is the most likely diagnosis for a male with a nasal obstruction, hemoptysis, hematuria, RBC
casts, increased c-ANCA, and lung biopsy results?

A. Adenocarcinoma of lung
B. Churg-Strauss syndrome
C. Necrotizing sarcoid granulomatosis
D. Wegener granulomatosis
----------------------------------------
5. What's the likely diagnosis for a 39-year-old female with headache, fever, upper extremity pain for 3
weeks, suspected of autoimmune vasculitis, and arteriography showing thoracic aorta involvement?
A. Takayasu’s arteritis
B. Churg-Strauss syndrome
C. Wegener’s granulomatosis
D. Microscopic polyangiitis
----------------------------------------
6. What is true regarding the condition of a 73-year-old male presenting with fever, headache localized
to the right temporal area, jaw claudication, elevated ESR, and treated with corticosteroids?
(or)
Which of the following statements about giant cell arteritis is true?
A. Involves small arteries
B. Granulomatous inflammation
C. Most commonly involved artery is the abdominal aorta
D. Continuous nature of involvement
----------------------------------------
7. A 55-yar-old male known case of hepatitis C and chronic liver disease came to the emergency
department with complaints of 3 episodes of hematemesis and an altered level of consciousness. Upon
arriving, the patient was given terlipressin and managed for oesophagal bleeding. Which of the
following vasculitis is associated with hepatitis C infection?
A. Takayasu’s arteritis
B. Churg-Strauss syndrome

Page 2

484
C. Cryoglobulinemia
D. Microscopic polyangiitis
----------------------------------------
8. A 27-year-old male presents to the outpatient department with complaints of fever, weight loss,
abdominal pain and myalgias for 2 weeks. Laboratory investigations revealed autoimmune
inflammatory vasculitis, and a probable diagnosis of polyarteritis nodosa was made. Which of the
following conditions is associated with this patient's suspected diagnosis?
A. Arsenic exposure
B. Chlamydia pneumonia
C. Hepatitis B virus
D. Asthma
----------------------------------------
9. A 42-year-old male patient complained of fever and myalgias for 2 weeks, cough, shortness of breath
for 5 days, and paresthesias with weakness of limbs for 1 week. He has a history of chronic sinusitis
and asthma with severe acute exacerbations. Laboratory investigations are positive for P-ANCA. Which
of the following is the most likely diagnosis?
(or)
What is the most likely diagnosis in an individual with chronic sinusitis and asthma with a history of
significant acute exacerbations and a positive P-ANCA?
A. Buerger disease
B. Churg-Strauss disease
C. Wegener granulomatosis
D. Takayasu arteritis
----------------------------------------
10. A 70-year-old male known case of hypertension and diabetes for the past 25 years comes to the
emergency department with a complaint of chest pain and shortness of breath. Despite acute
management, the patient dies soon of congestive heart failure. The cause of this condition is an
atheromatous plaque that ruptured, leading to acute thrombosis. Which among the following is a
component of such plaque?
A. Macrophages
B. Eosinophils
C. Magnesium
D. IgA
----------------------------------------
11. What pathology is most likely associated with the thoracic aorta shown in the autopsy image of a
53-year-old man who collapsed and diedafter having a tearing chest pain, hypotension, bradycardia,
diastolic murmur, negative troponin I and T?
(or)
Which of the following conditions is most likely responsible for aortic dissection?

Page 3

485
A. Connective tissue weakness of the aortic wall
B. Fibrinoid necrosis and smooth muscle hyperplasia
C. Immune complex-mediated vasculitis
D. Sub-intimal lipid deposition and smooth muscle necrosis
----------------------------------------
12. What is the most likely diagnosis for a person who has pain in his legs when walking and
blackening of the tips of the fingers, 80-pack-year history of smoking, hemoglobin of 15 g/dL, WBC of
8,300/µL, total cholesterol of 223 mg/dL, and fasting blood sugar of 97 mg/dL, negative ANA?
(or)
What is the most likely diagnosis for a person who has lower limb pain when walking, blackening of the
tips of the fingers, biopsy revealing intraluminal thrombi in medium-sized arteries and inflammation
extending from the arteries to nearby veins and nerves?

Page 4

486
A. Buerger disease
B. Churg-Strauss disease
C. Wegener granulomatosis
D. Takayasu arteritis
----------------------------------------
13. Which of the following viruses is responsible for the pathogenesis of a 34-year-old man with AIDS
who presents with several purple skin nodules on his hands and feet, varying from 1 mm to 1 cm in
diameter, and the given skin biopsy?
(or)
Which of the following viruses is implicated in Kaposi's sarcoma in a patient with AIDS?

A. Human Herpesvirus-8 (HHV-8)


B. Herpes Simplex virus-1 (HSV-1)
C. Cytomegalovirus (CMV)
D. Epstein Barr virus (EBV)
----------------------------------------
14. What is the most likely diagnosis in a 70-year-old woman who complains of unilateral throbbing
headaches, visual disturbances, a history of weight loss, jaw pain while eating, and recurring attacks of
fever with malaise and muscle pain; the physical examination shows a nodular enlargement of the
temporal artery?
(or)
What diagnosis is most likely for person who has unilateral throbbing headaches, visual problems,
nodular enlargement of the temporal artery, and the biopsy shown below?

Page 5

487
A. Giant cell arteritis
B. Buerger disease
C. Kawasaki disease
D. Wegener granulomatosis
----------------------------------------
15. What is the most likely diagnosis in a 48-year-old man with a raised, red, and exquisitely painful
lesion on the dorsal surface of his left hand and a skin biopsy revealing nests of regular round cells
within connective tissue associated with branching vascular spaces?
(or)
What is the most likely diagnosis when a skin biopsy of a painful, red lesion on the dorsal surface of the
hand reveals nests of regular round cells within connective tissue and branching vascular spaces?
A. Angiosarcoma
B. Dermatofibroma
C. Glomus tumor
D. Lipoma
----------------------------------------
16. Which of the following is correct regarding bacillary angiomatosis?
A. Vascular proliferative lesions are seen mainly in immunocompetent hosts
B. Bartonella henselae and Bartonella quintana are the main cause of it
C. It is a disease of the skin only
D. There is a proliferation of neurons lined by prominent epithelioid cells.
----------------------------------------
17. What diagnosis is most likely for a 20-year-old woman with episodes of fainting, double vision,
tingling in left hand, numbness in right hand, diminished pulse in the right arm, increased ESR,
thrombocytosis, aortogram indicating arterial narrowing and occlusion (including right subclavian
artery), subsequent heart failure, and autopsy revealing aorta with thickened wall, vasculitis, and
fragmented elastic fibers?
A. Buerger disease
B. Churg-Strauss disease

Page 6

488
C. Kawasaki disease
D. Takayasu arteritis
----------------------------------------
18. A 19-year-old man with asthma has chest pain, intermittent claudication, and shortness of breath
that are unresponsive to bronchodilators and antibiotics. On examination, B.P. - 150/100 mmHg,
bilateral wheezing, and numerous purple skin lesions. Laboratory studies show leukocytosis with
eosinophilia and platelet count of 450,000/µL, BUN - 30 mg/dL, S. creatinine of 3.5 mg/dL, positive
cytoplasmic antibody test and 3+ proteinuria with hematuria. A kidney biopsy shows vasculitis of
medium-sized arteries accompanied by eosinophilia. Which of the following diagnoses is most likely?
(or)
What is the likely diagnosis for a 19-year-old man with asthma, experiencing chest pain, intermittent
claudication, unresponsive shortness of breath, bilateral wheezing, purple skin lesions, elevated blood
pressure, eosinophilia, thrombocytosis, BUN and S.creatinine, positive cytoplasmic antibody test, 3+
proteinuria, red blood cells in urine, and vasculitis on kidney biopsy?
A. Churg-Strauss disease
B. Henoch-Schönlein purpura
C. Loeffler syndrome
D. Wegener granulomatosis
----------------------------------------
19. What immunoglobulin deposition is likely to occur around vessels in a 17-year-old male with diffuse
purpuric rash, colicky abdominal pain, multiple joint pain, and acute glomerulonephritis?
(or)
Which of the following immunoglobulin deposits surrounding the vessels defines Henoch-Schönlein
purpura?
A. IgM
B. IgG
C. IgA
D. IgE
----------------------------------------
20. A 36-year-old male patient presents to the OPD with complaints of weight loss, fever, abdominal
discomfort, and muscle pain for 3 weeks. Laboratory investigations revealed autoimmune inflammatory
vasculitis, and a probable diagnosis of polyarteritis nodosa was made. Which of the following vessels is
spared in this patient’s diagnosis?
A. Pulmonary artery
B. Coronary Artery
C. Renal artery
D. Cerebral artery
----------------------------------------
21. What's the likely diagnosis for a 28-year-old man with abdominal pain, myalgias, fever, weight loss
for 2 weeks, and lab findings indicating autoimmune inflammatory vasculitis with negative serum

Page 7

489
antineutrophilic cytoplasmic antibody?
(or)
If a patient has autoimmune inflammatory vasculitis with negative antinuclear anti-neutrophil
cytoplasmic antibodies, what is the most likely diagnosis?
A. Wegner granulomatosis
B. Churg strauss syndrome
C. Polyarteritis nodosa
D. Microscopic polyangiitis
----------------------------------------
22. What's the most feared complication in a 3-year-old with fever for seven days, bilateral
conjunctivitis, cracked lips, cervical lymphadenopathy, and erythema and desquamation of palms and
soles?
(or)
Which of the following is the most dreaded complication of Kawasaki’s disease?
A. Renal failure
B. Pulmonary Embolism
C. Coronary artery aneurysms
D. Thrombocytosis
----------------------------------------
23. A 57-year-old male presents to the clinic with complaints of headache and intermittent jaw
claudication. He also complained of scalp tenderness, fever and malaise. He has been diagnosed with
large vessel vasculitis. Which among the following conditions falls under large vessel vasculitis?
A. Polyarteritis nodusa
B. Temporal arteritis
C. Buerger disease
D. Wegener granulomatosis
----------------------------------------
24. A 27-year-old male presents to the outpatient department with complaints of fever, weight loss,
abdominal pain and myalgias for 2 weeks. Laboratory investigations revealed autoimmune
inflammatory vasculitis, and a probable diagnosis of polyarteritis nodosa was made. What is the most
common cause of death in polyarteritis nodosa?
A. Renal failure
B. Diabetes mellitus
C. Mononeuritis multiplex
D. None of the above
----------------------------------------
25. A 7-year-old male presents to the clinic. He has a diffuse purpuric rash, colicky abdominalpain, pain
in multiple joints of his body, and acute glomerulonephritis. Which among the following is the most
common vasculitis in children?

Page 8

490
A. Churg-Strauss syndrome
B. Bechet's disease
C. Henoch-Schonlein purpura
D. Microscopic polyangiitis
----------------------------------------
26. What's the likely diagnosis for a 64-year-old hypertensive male with sudden tearing chest pain
radiating to his back, history of hypertension but non-compliant with medications, BMI 21kg/m2, normal
ECG, no cardiac enzyme elevation, and widened mediastinum on a chest x-ray?
(or)
The most likely diagnosis for a male with sudden onset of tearing chest pain radiating to his back,
hypertension, non-compliance with medications, and a chest radiograph showing a widening of the
mediastinum is which of the following?
A. Arteriovenous fistula
B. Atherosclerotic aneurysm
C. Syphilitic aneurysm
D. Aortic dissection
----------------------------------------
27. Which is a special stain used for the visualization of elastic tissue?
A. Wright's stain
B. Hematoxylin and eosin (H&E;) stain
C. Verhoeff-Van Gieson (VVG) stain
D. Periodic acid-Schiff (PAS) stain
----------------------------------------
28. Which of the following components is primarily found within Weibel Palade Bodies?
A. Factor IX
B. P-selectin
C. CD34
D. CD31
----------------------------------------
29. Which of the following is the most dangerous symptom associated with giant cell arteritis?
A. Meningitis
B. Loss of vision
C. Stroke
D. Jaw claudication
----------------------------------------
30. Which artery is most commonly affected by pulseless disease?
A. Carotid artery

Page 9

491
B. Brachial artery
C. Subclavian artery
D. Femoral artery
----------------------------------------
31. Which condition is not associated with P-ANCA/MPO-ANCA?
A. Wegener's granulomatosis
B. Microscopic polyangiitis
C. Churg-Strauss syndrome
D. Takayasu arteritis
----------------------------------------
32. What is the most common central nervous system (CNS) manifestation of Behçet's disease?
A. Ischemic stroke
B. Dural sinus thrombosis
C. Meningitis
D. Brain abscess
----------------------------------------
33. Match the following: 1 Hyaline Arteriolosclerosis A 2 Hyperplastic Arteriolosclerosis B 3
Monckeberg's Sclerosis C

1 Hyaline Arteriolosclerosis A

2 Hyperplastic Arteriolosclerosis B

Page 10

492
3 Monckeberg's Sclerosis C

A. 1-A, 2-C, 3-B


B. 1-C, 2-B, 3-A
C. 1-B, 2-C, 3-A
D. 1-C, 2-A, 3-B
----------------------------------------
34. Which of the following vessels is least affected by atherosclerosis?
A. Popliteal Artery
B. Descending Thoracic Aorta
C. Internal Carotid Artery
D. Circle of Willis
----------------------------------------
35. Which syndrome is commonly associated with the most common benign tumor in the liver?
A. Down syndrome
B. Turner's syndrome
C. Marfan syndrome
D. Von Hippel-Lindau syndrome
----------------------------------------
36. Cystic Hygroma is a manifestation of which type of vascular tumor?
A. Capillary Lymphangioma
B. Cavernous Lymphangioma
C. Pyogenic granuloma
D. Cavernous Hemangioma
----------------------------------------
37. Which of the following substances is not associated with the development of angiosarcoma?
A. Vinyl Chloride
B. Arsenic
C. Thorotrast

Page 11

493
D. Formaldehyde
----------------------------------------
38. Which of the following statements about Hemangiopericytoma is true?
A. It is a benign condition.
B. It originates from endothelial cells.
C. Roughet cells are not involved in its pathogenesis.
D. It typically shows staghorn blood vessels on microscopy.
----------------------------------------

Correct Answers
Question Correct Answer

Question 1 2
Question 2 1
Question 3 2
Question 4 4
Question 5 1
Question 6 2
Question 7 3
Question 8 3
Question 9 2
Question 10 1
Question 11 1
Question 12 1
Question 13 1
Question 14 1
Question 15 3
Question 16 2
Question 17 4
Question 18 1
Question 19 3
Question 20 1
Question 21 3
Question 22 3
Question 23 2
Question 24 1

Page 12

494
Question 25 3
Question 26 4
Question 27 3
Question 28 2
Question 29 2
Question 30 3
Question 31 4
Question 32 2
Question 33 4
Question 34 4
Question 35 4
Question 36 2
Question 37 4
Question 38 4

Solution for Question 1:


Correct Option B - Vascular:
• The given clinical history and image points towards the diagnosis of Kaposi's sarcoma
• Kaposi sarcoma, a vascular tumour induced by the human herpes virus (HHV 8), also called KS
herpesvirus (KSHV)
• Most common neoplasm in patients with AIDS.
• The incidence of Kaposi sarcoma increased dramatically during the AIDS epidemic, now decreased
10-fold with highly active antiretroviral therapy.
• The lesions of KS are characterized by the proliferation of spindle-shaped cells that express markers
of both endothelial cells and smooth muscle cells.
• KS lesions display chronic inflammatory cell infiltrates.
• Solid organ transplant (SOT) patients are highly susceptible to the development of KS.
Incorrect Options:
Option A - Lymphoid:
• Non-Hodgkin lymphomas (NHL) are considered an AIDS-defining condition and currently represent
the most common type of cancer in individuals with HIV infection in developed countries.
Option C - Neural:
• Kaposi sarcoma is a tumor of vascular origin;
• Therefore, this option is incorrect.
Option D - Muscular:
• Examples of tumors of muscle origin in HIV - multiple leiomyomata, leiomyosarcomata, Epstein-Barr
virus-associated smooth muscle tumor

Page 13

495
Solution for Question 2:
Correct Option A - Caused by human herpes virus 8:
• Kaposi sarcoma is a vascular tumour induced by the human herpes virus (HHV 8).
• Dark/violaceous plaques or nodules represent vascular proliferations.

• Endothelial malignancy most commonly affects the skin, mouth, gastrointestinal tract, and respiratory
tract.
• The lesions of KS are characterized by the proliferation of spindle-shaped cells that express markers
of both endothelial cells and smooth muscle cells.
• There is also a profusion of slit-like vascular spaces, suggesting that the lesions may arise from
primitive mesenchymal precursors of vascular channels.
• KS lesions display chronic inflammatory cell infiltrates.
• Solid organ transplant (SOT) patients are highly susceptible to the development of Kaposi's Sarcoma.

Incorrect Options:
Option B - Seen in immune-competent patients:
• It is estimated that 25% to 40% of untreated HIV-infected individuals will eventually develop a
malignancy.
• Kaposi's sarcoma is 550-1000 times more common among solid organ transplant recipients than in
the normal population
• Therefore, this option is incorrect.
Option C - Proliferative tumour of the lymphoid origin:
• Kaposi sarcoma is a tumour of vascular origin.
• Therefore, this option is incorrect.

Page 14

496
Option D - Surgery is the treatment of choice:
• Mainstay of treatment is anti-retroviral therapy.
• Therefore, this option is incorrect.

Solution for Question 3:


• Granulomatosis with polyangiitis (GPA), formerly known as Wegener granulomatosis, is a necrotizing
vasculitis characterized by a triad of: Acute necrotizing granulomas of the upper respiratory tract (ear,
nose, sinuses, throat) or the lower respiratory tract (lung) or both Necrotizing or granulomatous
vasculitis affecting small- to medium-sized vessels (e.g., capillaries, venules, arterioles, and arteries),
most prominent in the lungs and upper airways but affecting other sites as well Focal necrotizing, often
crescentic, glomerulonephritis
• Acute necrotizing granulomas of the upper respiratory tract (ear, nose, sinuses, throat) or the lower
respiratory tract (lung) or both
• Necrotizing or granulomatous vasculitis affecting small- to medium-sized vessels (e.g., capillaries,
venules, arterioles, and arteries), most prominent in the lungs and upper airways but affecting other
sites as well
• Focal necrotizing, often crescentic, glomerulonephritis
• Anti-neutrophilic cytoplasmic antibodies (ANCA) have been implicated in the etiopathogenesis of
GPA.
• Clinical features: hemoptysis, hematuria and proteinuria, arthralgia, myalgia, and palpable cutaneous
purpura.
• GPA is treated with several immunosuppressive medications in various combinations.
• Acute necrotizing granulomas of the upper respiratory tract (ear, nose, sinuses, throat) or the lower
respiratory tract (lung) or both
• Necrotizing or granulomatous vasculitis affecting small- to medium-sized vessels (e.g., capillaries,
venules, arterioles, and arteries), most prominent in the lungs and upper airways but affecting other
sites as well
• Focal necrotizing, often crescentic, glomerulonephritis
A. Option A - Cutaneous:
• While Wegener's granulomatosis primarily affects the respiratory tract and kidneys, it can occasionally
involve the skin (purpura affecting the lower limbs, tender nodules in the olecranon areas). However,
cutaneous involvement is less common compared to lung and kidney involvement.
C. Option C - Musculoskeletal:
• Arthralgia and myalgia are frequently present in GPA. Although joint complaints are frequently
observed, a diagnosis cannot usually be made without considering additional signs and symptoms.
D. Option D - Nervous system:
• About 30%–40% of individuals experience neurological system involvement, with peripheral
neuropathies being the most frequent type.
• Mononeuritis multiplex may result from neuropathic pain. There have also been reports of cerebritis,
pachymeningitis, convulsions, and cranial neuropathies.

Page 15

497
Solution for Question 4:
Correct Option D - Wegener granulomatosis:
• The clinical presentation and investigation result findings of the patient mentioned above suggest
Wegener granulomatosis (WG).
• Wegener granulomatosis is a disease of unknown cause characterised by aseptic, necrotising,
granulomatous inflammation and vasculitis.
• Granulomatosis with polyangiitis is now the preferred term for Wegener's granulomatosis.'
• It is an autoimmune condition associated with necrotising granulomatous vasculitis, affecting the
upper and lower respiratory tract and the kidneys.
Features:
• Eye involvement (e.g. proptosis),
• Upper respiratory tract: saddle-shaped nose deformity, epistaxis, sinusitis, and nasal crusting.
• Lower respiratory tract: dyspnoea, hemoptysis
• Renal: rapidly progressive glomerulonephritis ('pauci-immune', 80% of patients)
Work up:
• c- ANCA +
• Renal biopsy- crescentic glomerulonephritis
Treatment = Cyclophosphamide.
Incorrect Options:
Option A - Adenocarcinoma of the lung:
• Adenocarcinoma of the lung is on histology: Invasive mucinous adenocarcinoma is characterized by
goblet or columnar cells with abundant mucin, invasive nonmucinous adenocarcinoma demonstrates
glandular differentiation
Option B - Churg-Strauss syndrome:
• Churg Strauss syndrome is a disease of unknown cause associated with vasculitis dyspnea and
asthma.
• Necrotizing vasculitis and extravascular eosinophil-rich granulomatous inflammation are
histopathological features commonly associated with conditions such as Churg-Strauss syndrome
• It is positive for p-Anca
Option C - Necrotizing sarcoid granulomatosis:
• It is characterized by sarcoid-like granulomas along with necrotic areas and vasculitis.

Solution for Question 5:

Page 16

498
• Takayasu arteritis, also known as pulseless disease, is a chronic inflammatory illness that damages
the medium and large arteries and their branches.
• Young Asian women are the main demographic affected.
• It typically affects the aorta and its major branches, especially the carotid, subclavian, and renal
arteries, and causes stenosis, occlusions, or aneurysmal degeneration of these large arteries.
• Its basic etiology is described as an inflammatory granulomatous vasculitis of the medium and large
arteries, which results in transmural fibrous thickening of the artery walls, causing numerous vascular
blockages and, ultimately, ischemic changes.
• Degeneration of elastic fibers is another hallmark, where inflammation-induced loss of medial smooth
muscle cells can lead to aneurysm development.
• The vasa vasora and the medio-adventitial junction are the focus of inflammation. Giant cell
granulomatous response and mononuclear cell infiltration are both frequently seen.
• Corticosteroids are the first line of treatment for Takayasu arteritis.
B. Option B - Churg-Strauss syndrome:
• It is also known as eosinophilic granulomatosis with polyangiitis (EGPA).
• The histology of EGPA demonstrates tissue eosinophilia, necrotising vasculitis, and granulomatous
inflammation that is eosinophil-rich.
C. Option C - Wegener’s granulomatosis:
• Granulomatosis with polyangiitis, formerly known as Wegener granulomatosis (WG), is a
small-to-medium vessel necrotising vasculitis that is part of a wide range of conditions known as
anti-neutrophil-cytoplasmic-antibody (ANCA) linked vasculitides (AAV).
• Hallmark features include necrotising granulomas and pauci-immune vasculitis that most commonly
affects the upper respiratory tract, lungs, and kidneys.
D. Option D -Microscopicpolyangiitis:
• Microscopic polyangiitis is an idiopathic autoimmune illness characterised by systemic vasculitis
affecting primarily small-calibre blood vessels and is related to developing anti-neutrophil cytoplasmic
autoantibodies (ANCA).
• Granulomatous inflammation is typically not seen in MPA.

Solution for Question 6:


• Giant cell arteritis is the most prevalent vasculitis in adults (often older than 50 years old).
• Granulomatous arteritis of the aorta and its major branches, particularly the extracranial branches of
the carotid artery, defines this vasculitis.
• Giant cell arteritis is also known as temporal arteritis since the superficial temporal artery is the most
often impacted blood vessel.
• Clinical symptoms include regional headaches (particularly severe along the anatomical course of the
superficial temporal artery), fever, exhaustion, weight loss, jaw and face pain, and the rapid onset of
blindness (due to opthalmic involvement).
• The preferred test is a temporal artery biopsy, which reveals granulomatous inflammation with
multinucleated large cells and fragmentation of the inner elastic lamina under a microscope.

Page 17

499
A. Option A - Involves small arteries:
• The temporal artery and other cranial arteries (now referred to as cranial-GCA) are the arteries most
commonly impacted by giant cell arteritis; however, inflammation of the aorta and other big arteries in
the body may also occur differently (now called large vessel-GCA).
C. Option C - Most commonly involvedarteryis theabdominalaorta:
• The temporal artery and other cranial arteries (now referred to as cranial-GCA) are the arteries most
commonly impacted by giant cell arteritis.
• The abdominal aorta is not the most commonly involved artery in GCA.
D. Option D - Continuous nature of involvement:
• GCA is characterised by segmental granulomatous inflammation, confirmed on temporal biopsy.
• Although many doctors regard temporal artery biopsy as the gold standard for diagnosing GCA, the
localised or segmental inflammatory nature of involvement and procedural hazards make it challenging
to confirm this illness in most patients.

Solution for Question 7:


• Cryoglobulins are proteins that precipitate when temperatures fall below 37°C.
• They can be just immunoglobulins (Ig) or a combination of immunoglobulin (Ig) and complement
elements.
• Cryoglobulinemia, which is caused by the accumulation of these cryoglobulins in the body's medium-
and large-sized blood arteries, damages endothelial cells and internal organs.
• Patients who present with skin ulcers, arthralgia, glomerulonephritis, neuropathy, and purpura should
have this entity suspected as a diagnosis.
• Small amounts of cryoglobulin are typically found in healthy persons.
• As 90% of occurrences of cryoglobulinemic vasculitis are linked to Hepatitis C infections, drug use is
the main risk factor for the condition.
• Blood vessel inflammation is brought on by the immune complex formation and complement activation
by producing hepatitis C IgG and IgM rheumatoid factor.
• Hepatitis C most frequently causes mixed cryoglobulinemia.
• During the management of cryoglobulinemia, hepatitis C should be evaluated.
A. Option A - Takayasu’s arteritis:
• Takayasu arteritis, also known as pulseless disease, is a chronic inflammatory illness that damages
the medium and large arteries and their branches.
• It primarily affects girls and women younger than 40.
• It is not associated with hepatitis.
B. Option B - Churg-Strauss syndrome:
• Churg-Strauss syndrome is a small vessel vasculitis characterised by granulomatous inflammation.
• It is associated with asthma.
D. Option D -Microscopicpolyangiitis:

Page 18

500
• Microscopic polyangiitis is an idiopathic autoimmune illness characterised by systemic vasculitis
affecting primarily small-calibre blood vessels and is related to developing anti-neutrophil cytoplasmic
autoantibodies (ANCA).
• The disease is said to be associated with pulmonary fibrosis.

Solution for Question 8:


Correct Option C - Hepatitis B virus:
• Polyarteritis nodosa (PAN) is a systemic necrotising vasculitis that mainly affects medium-sized
arterial vessels.
• It is ANCA negative.
• Even in cases where the condition is mild, there is still sizable morbidity due to digital ulcerations,
ischemia, and painful skin nodules. Patients with CPAN may develop the uncommon condition
systemic polyarteritis nodosa (PAN).
• The majority of PAN cases are idiopathic. Less frequently, secondary PAN is seen in hepatitis B,
hepatitis C, and cancers such as hairy cell leukaemia.
• Nearly 30% of patients with polyarteritis nodosa have blood antibodies against the hepatitis B virus,
although the significance is still unclear.
• laboratory investigations show evaluated renal function. Serologies for hepatitis B and hepatitis C are
necessary to exclude secondary causes of PAN.
Incorrect Options:
Option A - Arsenic exposure:
• Arsenic exposure is not associated with PAN.
• Long-term drinking water containing arsenic consumption is linked to an increased risk of skin cancer
and bladder cancer and the development of hepatic angiosarcoma.
Option B - Chlamydia pneumonia:
• Chlamydia pneumoniae is a Chlamydia species, an obligate intracellular bacteria that causes
pneumonia in humans.
• There is a possible association of Chlamydia pneumonia with the development of atherosclerosis;
however, evidence is controversial.
Option D - Asthma:
• Asthma is a chronic lung disease that causes airway inflammation, intermittent airflow obstruction,
and bronchial hyperresponsiveness.
• Among vasculitis, it is associated with Churg-Strauss syndrome.

Solution for Question 9:


Correct Option B - Churg-Strauss disease:

Page 19

501
• Churg-Strauss syndrome, also known as eosinophilic granulomatosis with polyangiitis, is
characterised by necrotising vasculitis of small and medium-sized systemic blood vessels.
• The coexistence of asthma, rhinosinusitis, and the presence of peripheral eosinophilia
• Features: myalgia, migrating polyarthralgia, weight loss, mono neuritis multiplex, and renal
involvement in the form of crescentic or necrotizing glomerulonephritis.
• The pathophysiology and clinical phenotype are either eosinophil-mediated damage or ANCA-induced
endothelial injury.
• Eosinophilic infiltrates in the walls of tiny and medium-sized blood arteries and extravascular tissue
spaces are characteristic of the histological findings in EGPA.
• Elevated eosinophils in (BAL)bronchoalveolar lavage fluid.
• p-ANCA or perinuclear antibodies are positive.
• It is treated with steroids.
Incorrect Options:
Option A - Buerger disease:
• Buerger's disease, an occlusive inflammatory condition, primarily affects the small and medium
arteries in the distal hands and legs, resulting in symptoms such as rest pain in the hands, feet, and
fingers, Raynaud phenomenon or livedo reticularis.
Buerger's disease, an occlusive inflammatory condition, primarily affects the small and medium arteries
in the distal hands and legs, resulting in symptoms such as rest pain in the hands, feet, and fingers, R
aynaud phenomenon or livedo reticularis.
Option C - Wegener granulomatosis:
• Granulomatosis with polyangiitis, formerly known as Wegener granulomatosis (WG), is a
small-to-medium vessel necrotising vasculitis that is a part of a wide range of conditions known as
anti-neutrophil-cytoplasmic-antibody (ANCA) linked vasculitides (AAV).
• Hallmark features include necrotising granulomas and pauci-immune vasculitis that most commonly
affects the upper respiratory tract, lungs, and kidneys.
Option D - Takayasu arteritis:
• Takayasu arteritis, also known as pulseless disease, is a chronic inflammatory illness that damages
the medium and large arteries and their branches.
• It primarily affects girls and women younger than 40.
• It typically affects the aorta and its major branches, especially the carotid, subclavian, and renal
arteries, and causes stenosis, occlusions, or aneurysmal degeneration of these large arteries.

Solution for Question 10:


Correct Option A - Macrophages:
• Macrophages play a key role in the development and destabilization of atherosclerotic plaques.
• They contribute to plaque inflammation and are involved in the process of plaque rupture.
• The release of enzymes and other substances by activated macrophages can weaken the fibrous cap
of the plaque, making it more prone to rupture.

Page 20

502
Incorrect Options:
Option B - Eosinophils:
• Eosinophils are a type of white blood cell that is not a prominent component of atheromatous plaques.
Atherosclerosis is primarily characterized by the accumulation of cholesterol, lipids, and inflammatory
cells such as macrophages within the arterial wall. Eosinophils are more commonly associated with
allergic and parasitic conditions.
• A unique type of vasculitis called cutaneous necrotizing eosinophilic vasculitis with
hypocomplementemia and eosinophilia affects small dermal arteries that are heavily infiltrated with
eosinophils. This type of vasculitis may occur in connective tissue disease patients.
• However, eosinophils are not a component of atherosclerosis plaques.
Option C - Magnesium:
• Atherosclerosis is the gradual buildup of plaque in and on the walls of arteries, which can cause blood
flow restriction. Soft plaques change and eventually calcify due to calcium deposition, turning into hard
plaques.
• Magnesium is not involved in the formation of atherosclerosis plaques.
Option D - IgA:
• Abnormal immune response leads to IgA vasculitis, characterized by deposition of IgA in blood
vessels and inflammation in skin's tiny blood vessels.
• Additionally, impacted arteries maybe those in the bowels, kidneys, or joints. It's not obvious why this
happens.

Solution for Question 11:


Correct Option A - Connective tissue weakness of the aortic wall:
• Blood entering the artery wall and expanding along the vessel results in a dissecting aneurysm
connected to aortic media thinning and degeneration (it presents with tearing chest pain).
• Aortic dissection occurs principally in two groups of patients: (1) men aged 40 to 60 years, with
antecedent hypertension (more than 90% of cases) and (2) younger patients with syndromic diseases
affecting the aorta (e.g., Marfan syndrome).
• Young or middle-aged males are typically affected by dissecting aneurysms, which have a high
rebleeding rate and high morbidity.
• While fusiform dilations of the parent vessel are typical in dissecting aneurysms, alternative patterns
such as lateral outpouching and localized stenosis can also occur.
Incorrect Options:
Option B - Fibrinoid necrosis and smooth muscle hyperplasia:
• These histology findings point towards the Hyperplastic arteriolosclerosis.
• Arteriolar damage caused by the sudden onset of malignant hypertension may cause fibrinoid
necrosis.
• It is characterised by the thickening of the arteriolar wall due to the concentric proliferation of smooth
muscle cells, giving the arterioles an "onion skin" appearance.

Page 21

503
Option C - Immune complex-mediated vasculitis:
• Henoch-Schönlein purpura (HSP), cryoglobulinemic vasculitis, and adverse medication responses are
the main manifestations of immune-complex vasculitis.
• Inflammatory bowel disease, systemic lupus erythematosus (SLE), paraneoplastic syndromes, and
infections are additional causes of immune-complex vasculitis.
Option D - Subintimal lipid deposition and smooth muscle necrosis:
• This biopsy finding relates to the atherosclerotic plaque. Atherosclerosis steadily takes hold when
plaque is formed in your blood by cholesterol, fat, blood cells, and other elements.
• Atherosclerosis is the development of alterations in the intima of bigger vessels, such as the medium
and large arteries.
• Lipid deposition in the intima is frequently linked to fibrosis and calcification, impairing the lumen and
making the artery more susceptible to thrombosis.

Solution for Question 12:


Correct Option A - Buerger disease:
• The clinical presentation described, including pain in the lower limbs upon walking, blackening of the
tips of fingers (suggestive of ischemia), and the histopathological findings of intraluminal thrombi in
medium-sized arteries with inflammation extending to neighboring veins and nerves, is characteristic of
Buerger disease, also known as thromboangiitis obliterans.
• Buerger disease is a non-atherosclerotic, segmental inflammatory condition that primarily affects
small and medium-sized arteries and veins, particularly in the extremities. It is strongly associated with
tobacco use, and the symptoms often include claudication (pain upon walking), as well as ischemic
changes in the fingers and toes. The disease is more common in young to middle-aged male smokers.

Incorrect Options:
Option B - Churg-Strauss disease:
• Churg-Strauss disease is a type of eosinophilic granulomatosis with polyangiitis (EGPA) and is
characterized by asthma, eosinophilia, and systemic vasculitis.
• It is characterized by nonspecific findings, which frequently include flu-like symptoms such as fever,
malaise, generalized weakness and exhaustion, anorexia, weight loss, and muscle discomfort
(myalgia).
• The histology of EGPA demonstrates tissue eosinophilia, necrotizing vasculitis, and granulomatous
inflammation that is eosinophil-rich.
Option C - Wegener granulomatosis:
• Wegener granulomatosis, now known as granulomatosis with polyangiitis (GPA), typically involves the
upper respiratory tract, lungs, and kidneys.
• It is associated with c-ANCA (antineutrophil cytoplasmic antibody) positivity.
• The condition affects blood vessels that are small to medium in size.
Option D - Takayasu arteritis:

Page 22

504
• Takayasu arteritis, also known as pulseless disease, is a chronic inflammatory illness that damages
the medium and large arteries and their branches.
• It primarily affects girls and women younger than 40.
• It typically affects the aorta and its major branches, especially the carotid, subclavian, and renal
arteries, and causes stenosis, occlusions, or aneurysmal degeneration of these large arteries.

Solution for Question 13:


Correct Option A - Human herpesvirus-8 (HHV-8):
• Patients with AIDS may develop Kaposi sarcoma.
• Microscopic examination of the skin lesion shows numerous ill-defined differentiated spindle-shaped
neoplastic cells and vascular lesions filled with red blood cells, which is characteristic of Kaposi
sarcoma.
• HHV-8 is implicated in the pathogenesis of Kaposi sarcoma in HIV-infected patients.
• HIV by itself is not a cause of Kaposi sarcoma.
Incorrect Options:
Option B - Herpes Simplex virus-1 (HSV-1):
• HSV-1 causes primary and recurring vesicular eruptions, particularly in the orolabial and vaginal
mucosa.
• Orolabial herpes, herpetic sycosis (HSV folliculitis), herpes gladiatorum, herpetic whitlow, ocular HSV
infection, herpes encephalitis, Kaposi varicelliform eruption (eczema herpeticum), and severe or
chronic HSV infection are all possible manifestations of HSV-1 infection.
• HSV infection is curtailed by antiviral medication.
Option C - Cytomegalovirus (CMV):
• Cytomegalovirus (CMV) is a common virus that can cause mild to severe end-organ failure in
immunocompromised people with congenital CMV illness.
• Cytomegalovirus is a member of the herpesviruses, Herpesviridae, or human herpesvirus-5 viral
family (HHV-5).
• CMV infections are frequently linked to the salivary glands.
• In healthy persons, CMV infection is asymptomatic, but it can be fatal in immunocompromised
patients. Cytomegalovirus infection at birth can result in morbidity and even death.
• CMV is frequently dormant after infection, although it can reactivate anytime. It eventually leads to
mucoepidermoid carcinoma and may cause prostate cancer.
Option D - Epstein Barr virus (EBV):
• Epstein Barr Virus, also known as (human herpes virus-4) causes infectious mononucleosis, also
known as kissing disease.
• Early symptoms include fever, sore throat, fatigue, malaise, anorexia, and myalgia typically occur in
the early phase of the illness.
• Physical findings include

Page 23

505
• Lymphadenopathy (discrete, non-suppurative, slightly painful, especially along the posterior cervical
chain)
• Transient bilateral upper lid oedema (Hoagland sign)
• Splenomegaly (in up to 50% of patients and sometimes massive)
• Conjunctival haemorrhage, exudative pharyngitis, uvular oedema, tonsillitis, or gingivitis may occur
• Lymphocytosis with atypical lymphocytes
• Soft palatal petechiae may be noted

Solution for Question 14:


Correct Option A - Giant cell arteritis:
• Giant cell (temporal) arteritis is the most common vasculitis.
• The disease is a local, chronic granulomatous inflammation of the temporal arteries. The average age
at onset is 70 years (more than 50yrs).
• Microscopic examination shows granulomatous inflammation with giant cells, which destroys the
media of the temporal artery and predisposes it to thrombosis.

• Throbbing headaches in the temporal region and visual problems may appear.
• A tortuous, palpable, and swollen temporal artery may be the only finding on physical examination.

• Wegener granulomatosis causes granulomatous inflammation but not in the temporal artery. The
other choices are not associated with multinucleated giant cells.

Page 24

506
Incorrect Options:
Option B - Buerger disease:
• Thromboangiitis obliterans, commonly known as Buerger disease, is a persistent, progressive
inflammation and thrombosis (clotting) of small to medium sized arteries and veins in the hands and
feet.
• It is significantly related to the use of tobacco products, particularly smoking, but also smokeless
tobacco.
Option C - Kawasaki disease:
• Kawasaki disease is a febrile sickness that occurs in infancy and youth (80% of patients are 4 years
old or less).
• Its clinical importance stems mostly from a propensity for coronary artery involvement, which can
result in burst aneurysms or thrombi, leading to acute myocardial infarction.
• A strawberry tongue is a pathognomic sign of Kawasaki disease.
Option D - Wegener’s granulomatosis:
• Granulomatosis with polyangiitis, formerly known as Wegener granulomatosis (WG), is a
small-to-medium vessel necrotising vasculitis that is a part of a wide range of conditions known as
anti-neutrophil-cytoplasmic-antibody (ANCA) linked vasculitides (AAV).
• Hallmark features include necrotising granulomas and pauci-immune vasculitis that most commonly
affects the upper respiratory tract, lungs, and kidneys.

Solution for Question 15:


Correct Option C - Glomus tumour:
• A glomus tumour is a benign, extremely painful tumour of the glomus body. Glomus bodies are normal
neuro myo-arterial receptors sensitive to temperature and regulate arteriolar blood flow.

Page 25

507
• The lesions are mostly smaller than 1 cm in diameter. The two main histologic components are
aggregates of specialised glomus cells and branching vascular channels in connective tissue stroma.
• The other choices tend not to be painful.
Incorrect Options:
Option A - Angiosarcoma:
• Angiosarcoma is a rare cancer that occurs in the inner lining of blood and lymphatic vessels. This
cancer can develop everywhere in the body, although it most commonly affects the skin, breast, liver,
and spleen.
• All angiosarcomas have comparable microscopic features, with vascular gaps that are more or less
evident and bordered by atypical tumor cells.
• The vascular gaps in low-grade lesions are bordered by big plump endothelial cells that pierce the
stroma and papillary fronds of cells that protrude into the lumen.
• Higher-grade lesions are more cellular, with aberrant cells and abnormal mitoses.
Option B - Dermatofibroma:
• Dermatofibroma, a common cutaneous condition found in the dermis, is also known as skin fibrous
histiocytoma or cutaneous fibrous histiocytoma.
• Histologically, it often presents with acanthotic epidermis, pseudoepitheliomatous hyperplasia, and
basaloid growth, influenced by fibroblast activity.
• Additionally, increased pigmentation, potentially from iron or melanin, may be observed, with many
lesions exhibiting a grenz zone of normal papillary dermis overlying the tumor.
Option D - Lipoma:
• Lipomas are benign tumours of fat cells (adipocytes) that appear as soft, painless lumps on the trunk
but can occur anywhere on the body.
• Histology reveals mature, normal-appearing adipocytes with a tiny eccentric nucleus.
• Liposarcomas, a malignant lipomatous tumour including lipoblasts and characterised by coarse
vacuoles and one or more scalloped, hyperchromatic nuclei, must be recognised from common lipomas
and their variations.

Solution for Question 16:


Correct Option B - Bartonella henselae and Bartonella quintana are the main cause of it:
• It is caused by opportunistic gram-negative bacilli of the Bartonella family.
• Two species have been implicated: Bartonella henselae Bartonella quintana
• Bartonella henselae
• Bartonella quintana
• Frequently mistaken for Kaposi sarcoma but has neutrophilic infiltrate.
• It can involve the skin, bone, and brain.
• Bacteria induce host tissues to produce hypoxia-inducible factor-1α (HIF-1α), which causes the
proliferation of capillaries lined by prominent epithelioid cells, which exhibit nuclear atypia and mitoses

Page 26

508
by VEGF.
• Bartonella henselae
• Bartonella quintana
Incorrect Options:
Option A - Vascular proliferative lesions seen mainly in immunocompetent hosts:
• Bacillary angiomatosis is characterized by neovascular proliferative lesions, mainly in
immunocompromised hosts (e.g., patients with AIDS)
Option C - It is a disease of the skin only:
• Bacillary angiomatosis can involve the skin, bone, and brain.
• Skin lesions take the form of red papules and nodules or rounded subcutaneous masses.

• Therefore, this option is incorrect.


Option D - There is a proliferation of neurons lined by prominent epithelioid cells.:
• Skin lesions are red papules and nodules or rounded subcutaneous masses. Histologically, there is
capillary proliferation with prominent epithelioid endothelial cells exhibiting nuclear atypia and mitoses.
Lesions contain stromal neutrophils, nuclear dust, and causal bacteria.
• The bacteria induce host tissues to produce hypoxia-inducible factor-1α (HIF-1α), which drives
vascular endothelial growth factor (VEGF) production and vascular proliferation.
• The infections (and lesions) are cured by antibiotic treatment.

Solution for Question 17:


• The clinical presentation of fainting, double vision, tingling, and numbness in the hands, along with
diminished pulse in the right arm and laboratory findings of increased sedimentation rate and
thrombocytosis, suggests a systemic vasculitis affecting the aorta and its branches. In this case, the
aortogram shows narrowing and occlusion of the right subclavian artery, and autopsy findings reveal

Page 27

509
vasculitis and fragmentation of elastic fibers in the aorta.
• The mentioned features suggest that Takayasu arteritis is the probable diagnosis.
• Takayasu arteritis is a type of large vessel vasculitis that primarily affects the aorta and its major
branches. It often occurs in young women and can lead to stenosis, occlusion, and aneurysmal dilation
of the affected arteries. The presentation may include limb claudication, diminished pulses, and
symptoms related to ischemia in various organs.
A. Option A - Buerger disease:
• Buerger disease primarily affects small and medium-sized arteries, particularly in the extremities. It is
strongly associated with tobacco use.
• In the provided case, the involvement of large vessels such as the subclavian artery is not typical for
Buerger disease, which mainly affects smaller vessels.
B. Option B - Churg-Strauss disease:
• Churg-Strauss disease is a small to medium vessel vasculitis associated with eosinophilic infiltration.
It often involves the respiratory tract, and cardiac involvement can occur.
• The clinical presentation in the case, including symptoms in the extremities, diminished pulse in the
right arm, and aortogram findings, is not characteristic of Churg-Strauss disease.
C. Option C - Kawasaki disease:
• Kawasaki disease typically affects infants and young children.
• It primarily affects medium-sized arteries, with a particular focus on the coronary arteries.
• Common symptoms include fever, conjunctivitis, rash, changes in the mucous membranes, and
extremity changes (e.g., swelling, redness).
• Therefore, it is unlikely that the diagnosis for this case is Kawasaki disease.

Solution for Question 18:


Correct Option A - Churg-Strauss disease:
• Churg-Strauss disease is an idiopathic, systemic, granulomatous disease of small- and medium-sized
arteries characterised by vasculitis of many organs, fluctuating eosinophilia and late-onset asthma.
• Most commonly MPO-ANCA or perinuclear antibodies are positive.
• The disease is also known as allergic granulomatosis and angiitis.
• Transbronchial lung biopsy shows granulomatous lesions in vascular and extravascular sites,
accompanied by intense eosinophilia.
Incorrect Options:
Option B - Henoch-Schönlein purpura:
• Henoch-Schönlein purpura is a systemic immune disorder of unknown cause.
• It is characterised by purpuric rash, colicky abdominal pain, polyarthralgia, and acute
glomerulonephritis.
• IgA is deposited in glomerular mesangium

Page 28

510
• All these changes result from the deposition of circulating immune complexes within blood vessels
throughout the body and within the glomerular mesangial regions.
• It is found most commonly in children 3-8 years old.
• It occurs in adults with severe renal manifestations.
Option C - Loeffler syndrome:
• Löffler's syndrome is a condition in which eosinophils build up in the lungs due to a parasite infection..
• The parasite can enter the body by soil contact like Ascaris, Strongyloides stercoralis, or Dirofilaria
immitis.
• It can cause difficulty in breathing, wheezing, coughing, and a fever.
Option D - Wegener granulomatosis:
• Granulomatosis with polyangiitis (GPA), formerly known as Wegener's granulomatosis (WG), is a rare
long-term systemic illness characterised by granuloma development and vasculitis.
• It is a kind of vasculitis that affects tiny and medium-sized arteries in numerous organs, although the
upper respiratory tract, lungs, and kidneys are the most typically affected.
• Nosebleeds, stuffy nose, the crustiness of nasal secretions, and inflammation of the uveal layer of the
eye are common indications and symptoms.

Solution for Question 19:


Correct Option C - IgA:

• Henoch-Schönlein purpura is a systemic immune disorder of unknown cause.


• It is characterised by purpuric rash, colicky abdominal pain, polyarthralgia, and acute
glomerulonephritis.
• All these changes result from the deposition of circulating immune complexes within blood vessels
throughout the body and within the glomerular mesangial regions.

Page 29

511
• It is found most commonly in children 3-8 years old.
• It occurs in adults with severe renal manifestations
• IgA is deposited in glomerular mesangium
Incorrect Options:
Option A - IgM:
• IgM is the biggest antibody and the first antibody to emerge in response to antigen exposure.
• HSP does not involve the deposition of IgM.
Option B - IgG:
• IgG is the most prevalent form of antibody detected in blood circulation, accounting for roughly 75% of
serum antibodies in people.
• Every IgG antibody has two paratopes.
• HSP does not involve the deposition of IgG
Option D - IgE:
• It is a significant immune response component against parasitic worms and protozoans.
• It is also required for type I hypersensitivity, which presents as an allergic disorders and anaphylaxis.
• It is usually the least prevalent isotype.
• HSP does not involve deposition of IgE

Solution for Question 20:


Correct Option A - Pulmonary artery:
• Polyarteritis nodosa (PAN) is a systemic vasculitis of the small or medium-sized muscle arteries that
usually affects the kidney and intestinal vessels but not the pulmonary circulation.
• Anti-neutrophil cytoplasmic antibodies are not frequently linked to polyarteritis nodosa.
• Even in cases where the condition is mild, there is still sizable morbidity due to digital ulcerations,
ischemia, and painful skin nodules.
• The majority of PAN cases are idiopathic. Less frequently, secondary PAN is seen in hepatitis B,
hepatitis C, and cancers such as hairy cell leukaemia.
• Nearly 30% of patients with polyarteritis nodosa have blood antibodies against the hepatitis B virus,
although the significance is still unclear.
• Most of the diagnosis is clinical. A liver panel, creatinine kinase, serum creatinine, and urinalysis
should all be performed in the laboratory to evaluate renal function. Serologies for hepatitis B and
hepatitis C are necessary to exclude secondary causes of PAN.
• The most noticeable morphological feature of the affected artery is an area of fibrinoid necrosis.

Page 30

512
Incorrect Options:
Option B - Mesenteric artery:
• Mesenteric artery is involved in polyarteritis nodosa
Option C - Renal artery:
• The renal artery is involved in polyarteritis nodosa
Option D - Cerebral artery:
• The cerebral artery is involved in polyarteritis nodosa

Solution for Question 21:


Correct Option C - Polyarteritis nodosa:
ANCA 2 types [Anti Neutrophilic Cytoplasmic ab]
P-ANCA
↓aka
MPO ANCA
C-ANCA
↓ aka
PR3 ANCA
Associated with:
• Churg strauss Syndrome
• Microscopic Polyangiitis
• Wegner’s Granulomatosis

Page 31

513
• Polyarteritis nodosa (PAN) is a systemic necrotising vasculitis that mainly affects medium-sized
arterial vessels.
• Anti-neutrophil cytoplasmic antibodies are not frequently linked to polyarteritis nodosa, in contrast to
other small-sized arterial vessel vasculitides (ANCA).
• The majority of PAN cases are idiopathic. Less frequently, secondary PAN is seen in hepatitis B,
hepatitis C, and cancers such as hairy cell leukaemia.
• Nearly 30% of patients with polyarteritis nodosa have blood antibodies against the hepatitis B virus,
although the significance is still unclear.
• Most of the diagnosis is clinical. A liver panel, creatinine kinase, serum creatinine, and urinalysis
should all be performed in the laboratory to evaluate renal function. Serologies for hepatitis B and
hepatitis C are necessary to exclude secondary causes of PAN.
A. Option A - Wegener’s granulomatosis:
• Granulomatosis with polyangiitis, previously termed Wegener granulomatosis (WG), is classified as a
necrotizing vasculitis affecting small to medium-sized vessels and is one of the anti-neutrophil
cytoplasmic antibody (ANCA)-associated vasculitides (AAV).
• Hallmark features include necrotising granulomas and pauci-immune vasculitis that most commonly
affects the upper respiratory tract, lungs, and kidneys.
B. Option B - Churg-Strauss syndrome:
• Churg-Strauss syndrome is a small-vessel necrotising vasculitis classically associated with asthma,
allergic rhinitis, lung infiltrates, peripheral hypereosinophilia, and extravascular necrotising granulomas.
• Vascular lesions can histologically resemble polyarteritis nodosa or microscopic polyangiitis but are
also characteristically associated with granulomas and eosinophils.
D. Option D -Microscopicpolyangiitis:
• Microscopic polyangiitis is an idiopathic autoimmune illness characterised by systemic vasculitis
affecting primarily small-calibre blood vessels and is related to developing anti-neutrophil cytoplasmic
autoantibodies (ANCA).
• Granulomatous inflammation is typically not seen in MPA, and its presence would point to another
diagnosis, like Wegener's granulomatosis.

Solution for Question 22:


Correct Option C -Coronary artery aneurysms:
• Based on the clinical scenario presented, it is highly likely that the patient has Kawasaki disease.
• Kawasaki disease can lead to inflammation of the coronary arteries, and in severe cases, it may result
in the formation of aneurysms or weakened areas in the walls of these arteries.
• Coronary artery involvement is a significant concern as it can lead to long-term cardiovascular
complications, including myocardial infarction, heart failure, and in extreme cases, sudden death.
Based on the clinical scenario presented, it is highly likely that the patient has Kawasaki disease.
Kawasaki disease can lead to inflammation of the coronary arteries, and in severe cases, it may result i
n the formation of aneurysms or weakened areas in the walls of these arteries.

Page 32

514
Coronary artery involvement is a significant concern as it can lead to long-term cardiovascular complic
ations, including myocardial infarction, heart failure, and in extreme cases, sudden death.
Incorrect Options:
Option A - Renal failure:
• Renal failure is not a typical or most dreaded complication of Kawasaki disease.
• The primary concern in Kawasaki disease is the involvement of the coronary arteries, leading to
aneurysms and potential long-term cardiovascular complications.
Renal failure is not a typical or most dreaded complication of Kawasaki disease.
The primary concern in Kawasaki disease is the involvement of the coronary arteries, leading to aneury
sms and potential long-term cardiovascular complications.
Option B - Pulmonary Embolism:
• Kawasaki disease primarily affects the cardiovascular system, with a focus on the coronary arteries.
• Pulmonary embolism involves blood clots in the pulmonary arteries, which is not a prominent feature
of Kawasaki disease.
Kawasaki disease primarily affects the cardiovascular system, with a focus on the coronary arteries.
Pulmonary embolism involves blood clots in the pulmonary arteries, which is not a
prominent feature of Kawasaki disease.
Option D - Thrombocytosis:
• Thrombocytosis (an elevated platelet count) can occur in Kawasaki disease, but it is not the most
dreaded complication.
• The primary concern is coronary artery involvement and the potential formation of aneurysms, which
can lead to severe cardiovascular consequences.
Thrombocytosis (an elevated platelet count) can occur in Kawasaki disease, but it is not the most drea
ded complication.
The primary concern is coronary artery involvement and the potential formation of aneurysms, which c
an lead to severe cardiovascular consequences.

Solution for Question 23:


Correct Option B - Temporal arteritis:
• Temporal arteritis is the most common vasculitis in adults more than 50 years old.
• T-cell-mediated granulomas are present, and the carotid artery, vertebral artery, and ophthalmic
artery are involved.
• The superficial temporal artery, the terminal branch of the external carotid artery, is also involved,
known as temporal arteritis. It is also the vessel from where a biopsy is taken for diagnosis.
• Clinical features The most common symptom is a headache due to the involvement of the superficial
temporal artery. Jaw claudication is the most specific symptom. There is fever, malaise, and joint and
muscle pain known as polymyalgia rheumatica. The shoulder is involved more than the pelvic girdle.
Involvement of the ophthalmic artery results in the sudden onset of blindness. There is scalp
tenderness along with temporal tenderness Visual symptoms such as monocular blindness and weight

Page 33

515
loss may also be present.
• The most common symptom is a headache due to the involvement of the superficial temporal artery.
• Jaw claudication is the most specific symptom.
• There is fever, malaise, and joint and muscle pain known as polymyalgia rheumatica. The shoulder is
involved more than the pelvic girdle.
• Involvement of the ophthalmic artery results in the sudden onset of blindness.
• There is scalp tenderness along with temporal tenderness
• Visual symptoms such as monocular blindness and weight loss may also be present.
• Investigations: Increased TLC Increased ESR (>50 mm/ 1hr) Temporal artery biopsy: investigation of
choice The Verhoeff-Van Gieson (VVG) stain may show a fragmentation of internal elastic lamina
• Increased TLC
• Increased ESR (>50 mm/ 1hr)
• Temporal artery biopsy: investigation of choice
• The Verhoeff-Van Gieson (VVG) stain may show a fragmentation of internal elastic lamina
• The mainstay of treatment is steroids.
• The most common symptom is a headache due to the involvement of the superficial temporal artery.
• Jaw claudication is the most specific symptom.
• There is fever, malaise, and joint and muscle pain known as polymyalgia rheumatica. The shoulder is
involved more than the pelvic girdle.
• Involvement of the ophthalmic artery results in the sudden onset of blindness.
• There is scalp tenderness along with temporal tenderness
• Visual symptoms such as monocular blindness and weight loss may also be present.
• Increased TLC
• Increased ESR (>50 mm/ 1hr)
• Temporal artery biopsy: investigation of choice
• The Verhoeff-Van Gieson (VVG) stain may show a fragmentation of internal elastic lamina
Incorrect Options:
Option A - Polyarteritis nodosa:
• Polyarteritis nodosa (PAN) is a systemic vasculitis of the small or medium-sized muscle arteries that
usually affects the kidney and intestinal vessels but not the pulmonary circulation.
• The most noticeable morphological feature of the affected artery is an area of fibrinoid necrosis.
Option C - Buerger disease:
• Thromboangiitis obliterans, commonly known as Buerger disease, is a persistent, progressive
inflammation and thrombosis (clotting) of small to medium sized arteries and veins in the hands and
feet
• It is significantly related to the use of tobacco products, particularly smoking, but also smokeless
tobacco.
Option D - Wegener’s granulomatosis:

Page 34

516
• Granulomatosis with polyangiitis, formerly known as Wegener granulomatosis (WG), is a
small-to-medium vessel necrotising vasculitis that is a part of a wide range of conditions known as
anti-neutrophil-cytoplasmic-antibody (ANCA) linked vasculitides (AAV).
• Hallmark features include necrotising granulomas and pauci-immune vasculitis that most commonly
affects the upper respiratory tract, lungs, and kidneys.

Solution for Question 24:


Correct Option A - Renal failure:
• Polyarteritis nodosa (PAN) is a systemic vasculitis of the small or medium-sized arteries that usually
affects the kidney and intestinal vessels but not the pulmonary circulation.
• It is a type III hypersensitivity reaction, i.e., caused by immune complexes deposition leading to
fibrinoid necrosis.
• It occurs commonly in young adults.
• It is associated with hepatitis B surface antigen and antibody immune complex deposition in different
organs.
• Multiple organ systems are involved, but the lungs are spared. Kidney: involvement of the renal artery
leads to activation of the renin-angiotensin-aldosterone system (RAAS), resulting in hypertension. GIT:
abdominal pain and melena Skin: nodules, papules, ulcers Nerve: PAN is the most common vasculitis
that causes mono neuritis multiplex Joints: arthralgia CNS: seizures
• Kidney: involvement of the renal artery leads to activation of the renin-angiotensin-aldosterone system
(RAAS), resulting in hypertension.
• GIT: abdominal pain and melena
• Skin: nodules, papules, ulcers
• Nerve: PAN is the most common vasculitis that causes mono neuritis multiplex
• Joints: arthralgia
• CNS: seizures
• The most common cause of death in PAN is renal failure.
• Diagnosis is made via histopathological biopsy, which reveals a transmural inflammation.
• The early phase is characterised by fibrinoid necrosis and neutrophilic infiltration.
• The late phase is characterised by fibrosis and a string of pearl appearance.
• The mainstay of treatment is steroids and cyclophosphamide for immunosuppression.
• Kidney: involvement of the renal artery leads to activation of the renin-angiotensin-aldosterone system
(RAAS), resulting in hypertension.
• GIT: abdominal pain and melena
• Skin: nodules, papules, ulcers
• Nerve: PAN is the most common vasculitis that causes mono neuritis multiplex
• Joints: arthralgia
• CNS: seizures

Page 35

517
Incorrect Options:
Option B - Diabetes mellitus:
• Diabetes mellitus does not occur as a complication of polyarteritis nodosa.
Option C - Mononeuritis multiplex:
• Mononeuritis multiplex is a kind of peripheral neuropathy known as mononeuropathy multiplex or
multifocal neuropathy. It occurs when two or more distinct nerve regions are damaged. Mononeuritis
multiplex is a collection of symptoms rather than a separate illness.
• PAN is the most common vasculitis that causes mono neuritis multiplex
Option D - None of the above:
• The most common cause of death in PAN is renal failure.

Solution for Question 25:


• Henoch-Schonlein purpura is an acute leukocytoclastic vasculitis that predominantly affects the small
blood vessels.
• It is the most common form of vasculitis in children.
• Clinical features include; Renal – glomerulonephritis, microscopic hematuria, proteinuria
Dermatologic– purpura, rash on extensor surface, back, and buttocks Gastrointestinal – abdominal
pain, and melena Rheumatologic – joint pain
• Renal – glomerulonephritis, microscopic hematuria, proteinuria
• Dermatologic– purpura, rash on extensor surface, back, and buttocks
• Gastrointestinal – abdominal pain, and melena
• Rheumatologic – joint pain
• The mainstay of treatment is steroids.
• Renal – glomerulonephritis, microscopic hematuria, proteinuria
• Dermatologic– purpura, rash on extensor surface, back, and buttocks
• Gastrointestinal – abdominal pain, and melena
• Rheumatologic – joint pain
Options A, B and D are incorrect.

Solution for Question 26:


Correct Option D - Aortic dissection:
• Aortic dissection is a critical medical emergency initiated by an intimal tear, leading to blood flow
dissection through the media and hematoma progression.
• Risk factors include hypertension, Marfan's syndrome, Ehlers-Danlos syndrome, and iatrogenic
causes such as arterial cannulations during procedures.

Page 36

518
• Pathogenesis involves medial hypertrophy of vasa vasorum due to hypertension, while connective
tissue disorders affect the extracellular matrix, predisposing to dissection.
• Histologically, cystic medial degeneration is observed in the aortic wall.
• CT angiography is the gold standard for diagnosis of aortic dissection.
• Treatment depends on the type of dissection. Type A dissections should involve a rapid diagnosis and
immediate management with intensive antihypertensive therapy, and surgical plication of the aortic
intimal tear.
• In contrast, Type B dissections can be managed conservatively with antihypertensive therapy or
treated with surgery.

Types of aortic dissection (Stanford classification).


Incorrect Options:
Option A - Arteriovenous fistula:
• An arteriovenous (AV) fistula is an abnormal connection between an artery and a vein. Blood flow
avoids capillaries and directly goes from an artery into a vein. It doesn't present with tearing chest pain
radiating to the back.
Option B - Atherosclerotic aneurysm:
• Atherosclerotic aneurysm involves the presence of significant atherosclerosis, which makes it unusual
for dissection to occur. Moreover, it doesn't present with complaints of tearing chest pain radiating to
the back.
Option C - Syphilitic aneurysm:
• Syphilis is associated with medial scarring due to medial fibrosis. Therefore, it inhibits the progression
of the dissection. In addition, it doesn't present with tearing chest pain with radiation to the back.

Solution for Question 27:


Correct Option C- Verhoeff-Van Gieson (VVG) stain:

Page 37

519
• Verhoeff-Van Gieson (VVG) stain is a special stain commonly used for the visualization of elastic
tissue. This staining method stains elastic fibers black or dark brown, making them easily
distinguishable under a microscope.
Incorrect Options:
Options A, B, and D: They are not special stains used for the visualization of elastic tissue.

Solution for Question 28:


Correct Option B- P-selectin:
• Weibel-Palade bodies are present in the endothelial cells. They contain P-selectin and Von Willebrand
Factor.
Incorrect Options:
Options A, C, and D: They are not components of Weibel Palade bodies.

Solution for Question 29:


Correct Option B- Loss of vision:
• Loss of vision is the most dangerous symptom associated with giant cell arteritis. It indicates
involvement of the ophthalmic artery, which can lead to irreversible blindness if not promptly treated.
Incorrect Options:
Options A, C, and D: They are not the most dangerous symptoms of giant cell arteritis.

Solution for Question 30:


Correct Option C- Subclavian artery:
• Pulseless disease, also known as Takayasu arteritis, predominantly affects the large arteries,
particularly the aorta and its main branches. The subclavian artery is the most commonly involved
artery in pulseless disease.
Incorrect Options:
Options A, B, and D: They are not the most commonly involved arteries in pulseless disease.

Solution for Question 31:


Correct Option D- Takayasu arteritis:
• P-ANCA/MPO-ANCA is associated with Wegener's granulomatosis, microscopic polyangiitis,
Churg-Strauss syndrome, ulcerative colitis, and primary sclerosing cholangitis but not with Takayasu
arteritis.

Page 38

520
Incorrect Options:
Options A, B, and C: They are incorrect, as explained above.

Solution for Question 32:


Correct Option B- Dural sinus thrombosis:
• Behçet's disease is characterized by small and medium vessel involvement, with dural sinus
thrombosis being the most frequently observed CNS manifestation. The Triad of Behcet disease
includes oral ulcer, genital ulcer, and uveitis.
Incorrect Options:
Options A, C, and D are incorrect: They are not the common manifestations of Behcet’s disease.

Solution for Question 33:


Correct Option D-1-C, 2-A, 3-B:
1: Hyaline Arteriolosclerosis

• It shows a pink glassy appearance (hyaline deposition).


• It is seen in benign hypertension and diabetes mellitus.
2: Hyperplastic Arteriolosclerosis

Page 39

521
• It shows the appearance of onion skinning.
• It causes an increase in the smooth muscles and fibroblasts.
• It is seen in malignant HTN.
3: Monckeberg's Sclerosis

• It is also called calcific medial degeneration.


• Calcific: Dystrophic degeneration
• Medial: Tunica media involvement
• Degeneration: Old age
Incorrect Options: Options A, B, and C: They are incorrectly matched.

Page 40

522
Solution for Question 34:
Correct Option D- Circle of Willis:
• Atherosclerosis is a condition characterized by the buildup of plaque in the arteries, leading to
narrowing and decreased blood flow. While atherosclerosis commonly affects large and medium-sized
arteries, the Circle of Willis is least commonly affected by this condition.
Incorrect Options:
Options A, B, and C: They are more commonly involved than Willis's circle.

Solution for Question 35:


Correct Option D- Von Hippel-Lindau syndrome:
• Von Hippel-Lindau syndrome is commonly associated with the most common benign tumor in the
liver, which is cavernous hemangioma. Von Hippel-Lindau syndrome is a hereditary condition
characterized by the development of multiple benign and malignant tumors in various organs, including
the liver. Hepatic hemangiomas are frequently observed in individuals with this syndrome.
Incorrect Options: A, B, and C: They are not associated with cavernous hemangioma.

Solution for Question 36:


Correct Option B- Cavernous Lymphangioma:
• Cavernous Lymphangioma is characterized by the presence of dilated lymphatic channels in deeper
tissues, such as the neck. When Cavernous Lymphangioma occurs in the neck region, it forms large
cystic masses, termed Cystic Hygroma. This condition is often associated with syndromes like Turner's
syndrome.
Incorrect Options: A, C, and D: They do not cause cystic hygroma.

Solution for Question 37:


Correct Option D- Formaldehyde:
• Angiosarcoma can be caused by exposure to Vinyl Chloride (PVC), Arsenic (found in pesticides), and
Thorotrast (thorium-containing contrast dye). However, formaldehyde is not typically associated with
the development of angiosarcoma.
Incorrect Options A, B & C: They are explained above.

Solution for Question 38:


Correct Option D- It typically shows staghorn or fishhook blood vessels on microscopy:

Page 41

523
• Hemangiopericytoma is a malignant condition characterized by the proliferation of pericytes or
Roughet cells around blood vessels. On microscopy, it typically presents with staghorn or fishhook
blood vessels.
Incorrect Options:
Option A- It is a benign condition: Hemangiopericytoma is a malignant tumor, not benign.
Option B- It originates from endothelial cells: Hemangiopericytoma originates from pericytes or Roughe
t cells, not endothelial cells.
Option C- Roughet cells are not involved in its pathogenesis: Pericytes or Roughet cells are indeed inv
olved in the pathogenesis of Hemangiopericytoma.

Page 42

524
Myocardial Infarction,Angina Pectoris,Cardiac
Enzymes
1. A 57-year-old male presents to the emergency department due to chest pain for 20 minutes. The
pain occurred while he was exercising. It radiates to the left arm and neck. It is associated with
sweating and nausea. A clinical diagnosis of the acute coronary syndrome is made, and the patient is
given nitroglycerin immediately to increase blood flow to the heart. At times, reperfusion leads to
ischemia-reperfusion injury. Which of the following is most likely to cause ischemia-reperfusion injury?
A. Neutrophils
B. Monocytes
C. Eosinophils
D. Free radicals
----------------------------------------
2. What's the likely diagnosis for a 55-year-old female with chest pain lasting 5-6 minutes during
physical exertion, relieved by rest, radiating to the left jaw, intensity of 8/10, with a history of
hypertension and is on amlodipine, and normal ECG and cardiac enzymes?
A. Arrhythmia
B. Myocardial infarction
C. Prinzmetal angina
D. Stable angina pectoris
----------------------------------------
3. Which of the following complications of myocardial infarction is expected between 3-7 days after the
infarction?
A. Arrhythmia
B. Mural thrombosis
C. Pericarditis
D. Myocardial rupture
----------------------------------------
4. Which of the following arteries is most likely to be the source of MI when histology reveals occlusion
of an artery supplying the apex of the heart?
A. Left anterior descending artery
B. Left circumflex artery
C. Right coronary artery
D. Right marginal artery
----------------------------------------
5. What is the most likely complication of heart pathology in a 68-year-old female patient with a 20-year
history of hypertension, a 10-year history of type 2 diabetes, and a recent myocardial infarction 1 year
ago, who presented unconscious to the emergency department and subsequently died despite
resuscitative efforts?

525
(or)
Which of the following is the most likely complication of the pathological condition of the heart shown in
the image above?

A. Aortic stenosis
B. Coronary artery aneurysm
C. Hypertrophic cardiomyopathy
D. Stroke
----------------------------------------
6. A 48-year-old male presents to the emergency department due to sudden, severe chest pain for 15
minutes. The pain radiates to the neck and is associated with sweating. ECG shows ST-segment
elevation in leads II, III, and avF. Cardiac biomarkers are increased. A diagnosis of acute inferior
myocardial infarction is made, and the patient is given fibrinolytic therapy. However, the patient
deteriorates and dies after 12 hours. Autopsy and histological investigations reveal
ischemia-reperfusion injury. Which of the following factors does not contribute to ischemia-reperfusion
injury in myocardial infarction?
A. Complement activation
B. Leukocyte aggregation
C. Free radicals
D. Eosinophils
----------------------------------------
7. What histological change is depicted in the myocardium of a 56-year-old male who died two days
after presenting with sudden, severe chest pain consistent with anterior wall myocardial infarction, as
confirmed by ECG and laboratory studies?
(or)
Which of the following histological changes can be seen in this image?

Page 2

526
A. Formation of collagen by fibroblasts
B. Formation of granulation tissue
C. Neutrophilic infiltration surrounding coagulative necrosis
D. Granulomatous inflammation
----------------------------------------
8. A 57-year-old male presents to the emergency department due to severe chest pain for 20 minutes.
It radiates to the left arm and is associated with nausea and sweating. ECG shows acute anterolateral
myocardial infarction (MI). The patient is given fibrinolytic therapy immediately. However, the patient’s
condition deteriorates, and he expires. An autopsy is carried out to find out the cause of death. The
transverse slices of the heart are stained with triphenyl tetrazolium chloride dye to differentiate between
viable and dead myocardium. What colour will be represented by the viable myocardium in the
presence of the stain?
A. Blue
B. White
C. Yellow
D. Brick-red
----------------------------------------
9. A 59-year-old male presents to the emergency department due to severe substernal chest pain for
30 minutes. The pain radiates to the jaw and is associated with nausea and sweating. ECG shows
ST-segment elevation in the leads II, III, and avF. Cardiac enzymes are sent for evaluation. Which of
the following is not true about cardiac enzymes in myocardial infarction?
A. LDH levels can start to rise within the first 24 hours after an MI
B. Myoglobin is one of the earliest enzymes to increase
C. CK-MB returns to normal after 7 days
D. Troponin is highly specific for the diagnosis of infarction
----------------------------------------
10. What histological finding is most likely to be seen in cardiac reperfusion injury during autopsy?
A. Coagulative necrosis with neutrophilic infiltration
B. Granulation tissue
C. Contraction band necrosis

Page 3

527
D. Collagenous scar
----------------------------------------
11. Which of the following shows the correct sequence of coronary vessel involvement in myocardial
infarction?
A. Left anterior descending artery> Right coronary artery,> Left circumflex artery
B. Left anterior descending artery,>Left circumflex artery>Right coronary artery
C. Right coronary artery>Left anterior descending artery>Left circumflex artery
D. Right coronary artery>Left circumflex artery>Left anterior descending artery
----------------------------------------
12. Which of the following is responsible for causing anterior wall myocardial infarction and the given
histology of the left main coronary artery?

A. Ehlers-Danlos syndrome
B. Familial hypercholesterolemia
C. Kawasaki disease
D. Systemic hypertension
----------------------------------------
13. Which of the following is least likely associated with cortical venous sinus thrombosis in a
38-year-old male with severe headaches, no medical history, but a positive family history of bilateral
deep venous thrombosis in his mother, and no history of smoking or drug abuse?
(or)
Which of the following is least likely to be associated with cortical venous sinus thrombosis with a
positive family history of hypercoagulability?
A. Antiphospholipid antibodies
B. Factor V Leiden mutation
C. High plasma homocysteine level
D. Low level of plasma antithrombin
----------------------------------------
14. Which of the following corresponds to the right heart failure? Venous pooling Pulmonary congestion
Nutmeg liver Wet and boggy lungs Heart failure cells

Page 4

528
A. 1, 2 and 4
B. 1 and 3
C. 2, 4 and 5
D. 1, 2, 3, 4 and 5
----------------------------------------
15. Which of the following types of angina best responds to vasodilators?
A. Stable angina
B. Unstable angina
C. Prizmental angin
D. Both A and B
----------------------------------------

Correct Answers
Question Correct Answer

Question 1 4
Question 2 4
Question 3 4
Question 4 1
Question 5 4
Question 6 4
Question 7 3
Question 8 4
Question 9 3
Question 10 3
Question 11 1
Question 12 2
Question 13 1
Question 14 2
Question 15 3

Solution for Question 1:


Correct Option D - Free radicals:
• Sometimes, restoration of blood flow causes ischemia-reperfusion injury instead of the recovery of
reversibly injured cells.
• This occurs especially after myocardial and cerebral ischemia.

Page 5

529
• The mechanisms responsible for ischemia-reperfusion injury include: Reactive oxygen species
(ROS)/free radicals: These are generated by the injured cells (injured mitochondria cannot completely
reduce the oxygen). The cellular antioxidant defense mechanisms are compromised by ischemia and
cannot neutralize the free radicals. Increased inflammation due to influx of leukocytes and plasma
proteins by reperfusion. Complement-mediated damage results in injury.
• Reactive oxygen species (ROS)/free radicals: These are generated by the injured cells (injured
mitochondria cannot completely reduce the oxygen). The cellular antioxidant defense mechanisms are
compromised by ischemia and cannot neutralize the free radicals.
• Increased inflammation due to influx of leukocytes and plasma proteins by reperfusion.
Complement-mediated damage results in injury.
• Free radicals are chemicals with a single unpaired electron in their outer orbit.
• Examples of free radicals include superoxide, hydrogen peroxide, hydroxyl radical and peroxynitrite.
• Free radicals are highly unstable and readily react with inorganic and organic molecules causing
damage to nucleic acids, cellular proteins, and lipids.
• Free radicals produce more free radicals when they attack a molecule resulting in a chain of damage.
• Reactive oxygen species (ROS)/free radicals: These are generated by the injured cells (injured
mitochondria cannot completely reduce the oxygen). The cellular antioxidant defense mechanisms are
compromised by ischemia and cannot neutralize the free radicals.
• Increased inflammation due to influx of leukocytes and plasma proteins by reperfusion.
Complement-mediated damage results in injury.
Incorrect Options:
Option A - Neutrophils:
• Neutrophils and macrophages generate free radicals through the process of the respiratory burst.
• Neutrophils do not directly cause damage in ischemia-reperfusion injury but do by means of free
radicals.
Option B - Monocytes:
• Monocytes have no role in ischemia-reperfusion injury.
• They are precursors of macrophages which form free radicals.
Option C - Eosinophils:
• Eosinophils are associated with allergic and parasitic reactions. They have no role in
ischemia-reperfusion injury.

Solution for Question 2:


Correct Option D - Stable angina pectoris:
• The patient presents with the clinical features of stable angina pectoris.
• Angina pectoris refers to intermittent chest pain caused by transient and reversible myocardial
ischemia.
• The release of adenosine, bradykinin, and other molecules during ischemia stimulates autonomic
nerves, resulting in pain.

Page 6

530
• There are three variants of angina pectoris: Typical/Stable angina Prinzmetal/Variant angina
Unstable/Crescendo angina
• Typical/Stable angina
• Prinzmetal/Variant angina
• Unstable/Crescendo angina
• Stable angina is characterized by episodic chest pain associated with exertion or conditions with
increased oxygen demand (tachycardia).
• The pain is substernal with radiation to the left arm or jaw.
• The pain relieves with rest or vasodilators such as nitroglycerin (increases coronary perfusion).
• Risk factors for stable angina include: Hypertension Diabetes mellitus Hyperlipidemia Cigarette
smoking Sedentary lifestyle Family history of ischemic heart disease Lack of exercise
• Hypertension
• Diabetes mellitus
• Hyperlipidemia
• Cigarette smoking
• Sedentary lifestyle
• Family history of ischemic heart disease
• Lack of exercise
• Typical/Stable angina
• Prinzmetal/Variant angina
• Unstable/Crescendo angina
• Hypertension
• Diabetes mellitus
• Hyperlipidemia
• Cigarette smoking
• Sedentary lifestyle
• Family history of ischemic heart disease
• Lack of exercise
Incorrect Options:
Option A - Arrhythmia:
• The primary symptom in arrhythmias is palpitation rather than chest pain.
• Patients report racing of heart and skipped beats.
• ECG shows changes in rate and rhythm in arrhythmias.
Option B - Myocardial infarction:
• Myocardial infarction presents with similar pain to angina pectoris.
• However, the pain is not relieved by rest or nitroglycerin.

Page 7

531
• ECG shows ST-segment elevation or depression.
Option C - Prinzmetal angina:
• Coronary artery spasms characterize prinzmetal angina. The anginal attacks are unrelated to physical
activity, heart rate, or blood pressure, and can occur at rest.
• It presents with chest pain at rest and usually at night time.
• The pain is relieved by calcium-channel blockers and nitrates.

Solution for Question 3:


Correct Option D - Myocardial rupture:
• Complications of Myocardial Infarction

• Myocardial rupture occurs in 1-5% of cases and is usually fatal.


• The most common site of rupture is the left ventricular wall. The free wall rupture leads to
hemopericardium and cardiac tamponade, which is rapidly fatal.
• Ventricular septal rupture may cause ventricular septal defect with a left-to-right shunt.
• Papillary muscle rupture causes severe mitral regurgitation.
• It occurs between 3-7 days after infarction when the necrotic myocardium is being lysed, and the
infarct is converted to soft, friable granulation tissue.
• Risk factors for myocardial rupture include age greater than 60 years, anterior or lateral wall infarction,
female gender, absence of left ventricular hypertrophy, and first myocardial infarction.
Incorrect Options:
Option A - Arrhythmia:
• MI-associated arrhythmia includes asystole, bradycardia, supraventricular tachyarrhythmias, and
ventricular fibrillation.

Page 8

532
• The risk of arrhythmias is greatest after 1 hour of MI and decreases after that.
Option B - Mural thrombosis:
• Any infarct combined with decreased contractility, chamber dilation, and endocardial damage can
cause the formation of a mural thrombus.
• This can lead to left-sided thromboembolism.
• The risk of mural thrombus formation is high in the first 3 months after MI.
Option C - Pericarditis:
• Pericarditis usually occurs after 2-3 days of MI.
• The risk decreases afterwards.

Solution for Question 4:


Correct Option A - Left anterior descending artery:

• The patient presents with infarction of the apex.


• The left coronary artery has a short stem that divides into the left circumflex artery and the left anterior
descending artery.
• The left anterior descending artery passes anteriorly into the anterior interventricular groove to supply
the following: Anterior septum The anterior wall of the left ventricle The apex of the heart Left bundle
branch
• Anterior septum
• The anterior wall of the left ventricle
• The apex of the heart
• Left bundle branch
• Anterior septum

Page 9

533
• The anterior wall of the left ventricle
• The apex of the heart
• Left bundle branch
Incorrect Options:
Option B - Left circumflex artery:
• The left circumflex artery is a branch of the left coronary artery.
• It runs in the left atrioventricular groove and supplies the left atrium and posterolateral aspect of the
left ventricle.
• It may supply SA and AV nodes in some individuals.
Option C - Right coronary artery:
• The right coronary artery supplies the right ventricle's lateral and posterior wall and the left ventricle's
inferior wall
• The right coronary artery also supplies the SA node, AV node, the bundle of His, and the right bundle
branch.
Option D - Right marginal artery:
• The right marginal artery is a branch of the right coronary artery.
• It supplies the right margin of the heart anterior and inferior surfaces of the right ventricle.
• It terminates by anastomosing with the branches of the anterior interventricular artery.

Solution for Question 5:


Correct Option D - Stroke:
• The image above shows a ventricular aneurysm.
• Ventricular aneurysms are a late complication of myocardial infarction.
• It is the result of a large transmural anteroseptal infarct.
• The infarct heals with a thin-walled scar.
• As myocardial infarcts heal, a newly deposited collagenous matrix is susceptible to stretching and
may become dilated.
• Ventricular aneurysms do not rupture.
• Ventricular aneurysms can lead to the following: Mural thrombi Arrhythmias Heart failure
• Mural thrombi
• Arrhythmias
• Heart failure
• The blood flow in the aneurysm is sluggish and causes the formation of a mural thrombus.
• The mural thrombus can then embolize and block vessels elsewhere.
• The mural thrombus causes left-sided thromboembolism. Hence, the blocking of vessels in the brain
leads to stroke.

Page 10

534
• Mural thrombi
• Arrhythmias
• Heart failure
Incorrect Options:
Option A - Aortic stenosis:
• Ventricular aneurysms cannot cause aortic stenosis.
• Aortic stenosis is a valvular disease that requires stiffening of the valves, usually by calcium
deposition.
• Ageing is one of the common causes of aortic stenosis.
Option B - Coronary artery aneurysm:
• A ventricular aneurysm cannot lead to a coronary artery aneurysm.
• Coronary artery aneurysms are caused by atherosclerosis in adults and Kawasaki disease in children.
Option C - Hypertrophic cardiomyopathy:
• Hypertrophic cardiomyopathy requires thickening of the ventricular wall, whereas ventricular
aneurysm is formed by dilation and thinning of the ventricular wall.
• The causes of hypertrophic cardiomyopathy include: Hypertension Ageing Genetic causes Idiopathic
• Hypertension
• Ageing
• Genetic causes
• Idiopathic
• Hypertension
• Ageing
• Genetic causes
• Idiopathic

Solution for Question 6:


Correct Option D - Eosinophils:
• Sometimes, restoration of blood flow causes ischemia-reperfusion injury instead of the recovery of
reversibly injured cells.
• This occurs especially after myocardial and cerebral ischemia.
• The mechanisms responsible for ischemia-reperfusion injury include: Reactive oxygen species
(ROS)/free radicals: These are generated by the injured cells (injured mitochondria cannot completely
reduce the oxygen). The cellular antioxidant defence mechanisms are compromised by ischemia and
cannot neutralize the free radicals. Increased inflammation due to influx of leukocytes and plasma
proteins by reperfusion. Complement-mediated damage results in injury.
• Reactive oxygen species (ROS)/free radicals: These are generated by the injured cells (injured
mitochondria cannot completely reduce the oxygen). The cellular antioxidant defence mechanisms are

Page 11

535
compromised by ischemia and cannot neutralize the free radicals.
• Increased inflammation due to influx of leukocytes and plasma proteins by reperfusion.
Complement-mediated damage results in injury.
• Free radicals are chemicals with a single unpaired electron in their outer orbit.
• Examples of free radicals include superoxide, hydrogen peroxide, hydroxyl radical, and peroxynitrite.
• Free radicals are highly unstable and readily react with inorganic and organic molecules causing
damage to nucleic acids, cellular proteins, and lipids.
• In myocardial infarction, the factors contributing to reperfusion injury include: Mitochondrial
dysfunction (swelling and rupture of outer membrane releases mitochondrial contents and leads to
apoptosis) Myocyte hypercontracture (increased intracellular calcium levels causes augmented and
uncontrolled contraction of myofibrils) Free radicals Platelet and complement activation Leukocyte
aggregation - may occlude the microvasculature and contribute to the “no-reflow” phenomenon.
• Mitochondrial dysfunction (swelling and rupture of outer membrane releases mitochondrial contents
and leads to apoptosis)
• Myocyte hypercontracture (increased intracellular calcium levels causes augmented and uncontrolled
contraction of myofibrils)
• Free radicals
• Platelet and complement activation
• Leukocyte aggregation - may occlude the microvasculature and contribute to the “no-reflow”
phenomenon.
• Reactive oxygen species (ROS)/free radicals: These are generated by the injured cells (injured
mitochondria cannot completely reduce the oxygen). The cellular antioxidant defence mechanisms are
compromised by ischemia and cannot neutralize the free radicals.
• Increased inflammation due to influx of leukocytes and plasma proteins by reperfusion.
Complement-mediated damage results in injury.
• Mitochondrial dysfunction (swelling and rupture of outer membrane releases mitochondrial contents
and leads to apoptosis)
• Myocyte hypercontracture (increased intracellular calcium levels causes augmented and uncontrolled
contraction of myofibrils)
• Free radicals
• Platelet and complement activation
• Leukocyte aggregation - may occlude the microvasculature and contribute to the “no-reflow”
phenomenon.
• Eosinophils are the mediators of allergic reactions and parasitic infections. They have no role in
ischemia-reperfusion injury.
Incorrect Options:
Option A - Complement activation:
• Ischemia-reperfusion injury is mediated by the complement system and reactive oxygen species.
• Complement activation plays a role in the no-reflow phenomenon by causing an injury to the
endothelium.
Option B - Leukocyte aggregation:

Page 12

536
• Leukocytes aggregate in a vessel and occlude it, resulting in a no-reflow phenomenon.
• It also potentiates the activity of proteases and elastases that cause cell death.
Option C - Free radicals:
• Free radicals or reactive oxygen species are the main mediators of ischemia-reperfusion injury.
• They damage membrane proteins and phospholipids.

Solution for Question 7:


• The image above shows coagulative necrosis with neutrophilic infiltrate.
• Little or no changes are seen in the first four hours following a MI.
• After 4 hours, coagulative necrosis starts, surrounded by oedema and haemorrhage.
• After 12 hours, the necrosis continues, nuclei become pyknotic, eosinophils accumulate in the
myocytes, and the neutrophils infiltrate the interstitial spaces.
• Within 1-3 days, well-developed coagulative necrosis and interstitial neutrophilic infiltrates are seen in
the myocardium.
Incorrect Options:
A. Option A - Formation ofcollagenby fibroblasts:
• Well-healed myocardial infarct with replacement of the necrotic fibers by dense collagenous scar

• Collagen deposition starts after 10 days of MI.


• A dense collagenous scar forms after 2 months of MI.
Option B - Formation of granulation tissue:

Page 13

537
• Granulation tissue forms after 7 days of infarction.
• It consists of fibroblasts, keratinocytes, endothelial cells, new thin-walled capillaries, and inflammatory
cell infiltration of the extracellular matrix.
Option D - Granulomatous inflammation:

• Granulomatous inflammation is a chronic inflammation consisting of granuloma formation by


macrophages, epithelioid cells, and multinucleated giant cells.
• Granulomatous inflammation is associated with sarcoidosis, lymphomas, and inflammatory bowel
disease.
• It is not seen in myocardial infarction.

Solution for Question 8:

Page 14

538
Correct Option D - Brick-red:
• Myocardial infarction demonstrated histochemically by the lack of staining with triphenyl tetrazolium
chloride in the areas of necrosis

• triphenyltetrazolium chloride is a colourless, water-soluble dye.


• It is reduced by lactate dehydrogenase enzyme in the mitochondria of the living cells to formazan.
• Formazan is an insoluble, light-sensitive compound that turns healthy or living tissue to a brick-red
colour.
• As the dehydrogenase leaks out of the damaged membranes of the dead cells, the infarcted area
remains unstained. It is seen as a pale zone.
Incorrect Options:
Option A - Blue:
• Viable myocardium appears brick-red rather than blue.
Option B - White:
• Dead and necrotic myocardium appears as pale/yellow-white areas.
Option C - Yellow:
• Dead and necrotic myocardium appears as pale/yellow-white areas.

Solution for Question 9:


Correct Option C - CK-MB returns to normal after 7 days:

Page 15

539
• The laboratory investigations of MI involve the measurement of macromolecules in the blood that leak
from the injured myocardial cells.
• These include: Cardiac troponins I and T CK-MB Myoglobin Lactate dehydrogenase (LDH)
• Cardiac troponins I and T
• CK-MB
• Myoglobin
• Lactate dehydrogenase (LDH)
• The first enzyme to increase is myoglobin which falls earlier as well.
• CK-MB rises within 2-4 hours of MI, peaks at 24-28 hours, and returns within 72 hours (not 7 days).
• Troponins are detectable within 2-4 hours of MI, peak at 48 hours, and stay elevated for 7-10 days.
• With reperfusion, both troponins and CK-MB levels peak earlier due to a more rapid washout of the
enzyme from the necrotic tissue.
• LDH is the last enzyme to rise. It rises after 8-12 hours of MI, peaks at 72-144 hours, and remains
elevated for 8-14 days.
• Cardiac troponins I and T
• CK-MB
• Myoglobin
• Lactate dehydrogenase (LDH)
Incorrect Options:
Option A - LDH levels can start to rise within the first 24 hours after an MI:
• LDH is the last enzyme to increase.
• It rises after 8 hours of MI .
Option B - Myoglobin is one of the earliest enzymes to increase:
• Myoglobin is the first enzyme to rise after MI.

Page 16

540
• It is the first enzyme to fall back to normal.
Option D - Troponin is highly specific for the diagnosis of infarction:
• Cardiac troponins are the most specific enzymes for myocardial infarction.

Solution for Question 10:


Correct Option C - Contraction band necrosis:

• Reperfusion salvages reversibly injured cells but alters the morphology of irreversibly injured cells.
• Myocytes are irreversibly damaged after reperfusion and undergo contraction band necrosis
• Intense eosinophilic bands of hypercontracted sarcomeres are formed due to an influx of calcium ions
through the plasma membranes, which potentiate the actin-myosin interactions.
• The sarcomeres cannot relax and get stuck in a tetanic state due to the absence of ATP molecules.
• The typical appearance of reperfused myocardium after an acute MI consists of haemorrhage and
contraction bands, visible as prominent hypereosinophilic cross-striations spanning myofibres (as
shown above).
Incorrect Options:
Option A - Coagulative necrosis with neutrophilic infiltrates:
• Coagulative necrosis with neutrophilic infiltrates is seen after 12 hours of MI.
• It is not a characteristic feature of reperfusion injury.
Option B - Granulation tissue:
• Granulation tissue starts forming after 7 days of MI during recovery.
• It has no link to reperfusion injury.
Option D - Collagenous scar:
• A dense collagenous scar marks the end of recovery and repair after MI.

Page 17

541
• It takes more than 2 months to form following an acute MI attack.
• It has no role in reperfusion injury.

Solution for Question 11:


Correct Option A - Left anterior descending artery> Right coronary artery> Left circumflex artery:
• The location, size, and morphological features of myocardial infarct depend on the size and
distribution of the involved vessels, duration and rate of formation of vessel occlusion, and presence
and extent of collateral supply.
• 40-50% of MIs are due to occlusion of the proximal left anterior descending artery.
• 30-40% of MIs are due to proximal right coronary artery occlusion.
• 15-20% of MIs are caused by occlusion of the left circumflex artery.
Incorrect Options:
Options B, C and D are incorrect.

Solution for Question 12:


Correct Option B - Familial hypercholesterolemia:
• The image above shows the deposition of lipid-laden foam cells in the atheroma.
• Lipid-laden foam cells are characteristic of Familial hypercholesterolemia.
• Familial hypercholesterolemia is an autosomal disorder caused by mutations in the LDL receptor
protein that impair the intracellular transport and catabolism of LDL.
• This results in the accumulation of LDL cholesterol in the plasma.
• This, in turn, leads to a marked increase in cholesterol uptake by the macrophages and vascular wall.
• Heterozygotes have elevated serum cholesterol, increasing the risk of atherosclerosis and resultant
coronary artery disease.
• Homozygous individuals have an even greater serum cholesterol level and a high frequency of
ischemic heart disease, premature atherosclerosis (as seen in this patient), and skin xanthomas.
Incorrect Options:
Option A - Ehlers-Danlos syndrome:
• Ehlers-Danlos is a group of inherited disorders that affect connective tissues.
• The vascular type of Ehlers-Danlos syndrome is associated with bruising of the skin, thin skin, fragile
blood vessels, bowel tearing, and hypermobile fingers and toes.
• It is not associated with premature atherosclerosis or lipid-laden foam cells in the vessels.
Option C - Kawasaki disease:
• Kawasaki disease presents in children under 5 years of age with conjunctivitis, rash, oedema,
erythema of hands, lymphadenopathy, and strawberry tongue.

Page 18

542
• Untreated cases lead to myocardial infarction and coronary artery aneurysms.
• Premature atherosclerosis is not a feature.
• However, lipid-laden foam cells are not seen in Kawasaki disease.
Option D - Systemic hypertension:
• Systemic hypertension can accelerate atherosclerosis but cannot lead to premature or early MI.
• Therefore this option is not correct

Solution for Question 13:


Correct Option A - Antiphospholipid antibodies:
• The patient presents with cortical venous sinus thrombosis with a positive family history of
hypercoagulability.
• Hence, the patient has a primary (inherited) hypercoagulability.
• Antiphospholipid antibody syndrome is the cause of secondary (acquired) hypercoagulable states.
• Antiphospholipid antibody syndrome/ lupus anti-coagulant syndrome presents with the following:
Recurrent thrombosis Repeated miscarriages Valvular vegetations Thrombocytopenia
• Recurrent thrombosis
• Repeated miscarriages
• Valvular vegetations
• Thrombocytopenia
• Depending on the vascular bed involved, it may result in the following:
• Pulmonary hypertension
• Pulmonary embolism
• Stroke
• Bowel infarction
• Renovascular hypertension
• The patients have positive antiphospholipid antibodies..
• Recurrent thrombosis
• Repeated miscarriages
• Valvular vegetations
• Thrombocytopenia
Incorrect Options:
Option B - Factor V Leiden mutation:
• Primary hypercoagulability is most commonly caused by factor V Leiden mutation and mutation in the
prothrombin gene.
• The mutation makes factor V resistant to degradation by protein C.

Page 19

543
• Patients have an increased risk of venous thrombosis.
Option C - High plasma homocysteine level:
• High levels of homocysteine levels cause arterial and venous thrombosis along with the development
of atherosclerosis.
• An inherited deficiency of cystathione beta-synthetase causes it.
Option D - Low level of plasma antithrombin:
• Antithrombin deficiency contributes to rare cases of primary hypercoagulable states.
• It causes venous thrombosis and recurrent thromboembolism in adolescence or in early adult life.

Solution for Question 14:


Correct Option B - 1 and 3:
1: Venous pooling: Right heart failure can lead to systemic venous congestion, causing venous pooling
, particularly in the peripheral tissues. This can result in chronic venous congestion of the liver and sple
en.
3: Nutmeg liver: Nutmeg liver, also known as congestive hepatopathy, is a
characteristic finding in right heart failure. It occurs due to hepatic congestion, resulting in a
mottled appearance resembling a nutmeg seed.
Incorrect Options:
Options A, C & D are incorrect
2: Pulmonary congestion is a characteristic feature of left heart failure rather than right heart failure. In l
eft heart failure, blood backs up into the pulmonary circulation, leading to pulmonary congestion and sy
mptoms such as dyspnea and orthopnea.
4: Wet and boggy lungs are also typical of left heart failure due to pulmonary congestion.
5: Heart failure cells are hemosiderin-laden macrophages, typically seen in left heart failure and not rig
ht heart failure.

Solution for Question 15:


Correct Option C - Prizmental angina:
• Prinzmetal angina, also known as variant angina, is caused by transient coronary artery spasm,
leading to myocardial ischemia. This type of angina best responds to vasodilators because the
underlying mechanism involves smooth muscle contraction and spasms of the coronary arteries.
Vasodilators help to relax the smooth muscle and relieve the spasm, thereby improving blood flow to
the myocardium and alleviating symptoms of angina.
Incorrect Options:
• Options A, B, and D are incorrect do not have the best response when treated with vasodilators.

Page 20

544
Cardiomyopathies and Cardiac Tumors.
1. What is the diagnosis for a 68-year-old female with intermittent dull chest pain, palpitations, B.P.-
136/86 mm Hg, a pulse of 65/min, history of heart failure (EF 40%), mitral regurgitation, and a large
mass in the left atrium attached to the interatrial septum?
(or)
What is the most common benign cardiac tumour in adults?
A. Rhabdomyoma
B. Hemangioma
C. Lipoma
D. Myxoma
----------------------------------------
2. What is the most likely diagnosis for a 50-year-old woman presenting with fatigue, dyspnea, left atrial
enlargement, mitral valve calcification, and a CT scan revealing a large left atrial mass, who
subsequently dies of an ischemic stroke before open heart surgery, with the given autopsy findings?
(or)
An autopsy was performed on a woman whose symptoms included shortness of breath, pulmonary
oedema, enlargement of the left atrium, calcification of the mitral valve, and a significant mass in the left
atrium. What is the diagnosis?

A. Calcific aortic stenosis


B. Cardiac Myxoma
C. Fibroelastoma
D. Mural thrombus
----------------------------------------
3. A 4-year-old boy with a history of intellectual disability and seizures comes to the doctor with
shortness of breath for a month. His BP was 138/82 mm Hg, pulse was 75/min, and BMI was 21
kg/m2.On physical examination, the doctor notices numerous papules on the boy's face. In addition,
echocardiography shows tumor-like growth in the left ventricle. Which of the following is the most likely
diagnosis?
(or)

545
Which of the following is the most likely diagnosis for a boy who has an intellectual disability and
seizures and whose left ventricle has a growth that resembles a tumour?
A. Lipoma
B. Myxoma
C. Rhabdomyoma
D. Dilated cardiomyopathy
----------------------------------------
4. A 61-year-old male patient presents to the hospital with complaints of dyspnea associated with
congestive features. He has no significant past medical history. He smokes one pack per day and has
been smoking for 15 years. His BP is 136/82, His pulse is 68/min, and his BMI is 23 kg/m2. His ejection
fraction was <30% on echocardiography, with decreased left ventricular wall thickness and increased
atrial size. A diagnosis of heart failure with dilated cardiomyopathy is confirmed. Which protein is most
commonly defective in dilated cardiomyopathy?
A. Dystrophin
B. Tropomyosin
C. Titin
D. Troponin
----------------------------------------
5. What's the likely diagnosis for a 19-year-old basketball player who collapses due to ventricular
tachycardia and then ventricular fibrillation, has a family history of the sudden death of his father,
echocardiogram showing thickened left ventricular wall with a small slit-like chamber, and dies
suddenly 5 years later, with histological examination of heart given below?

A. Amyloidosis
B. Cardiac myxoma
C. Dilated cardiomyopathy
D. Hypertrophic cardiomyopathy
----------------------------------------
6. A 51-year-old man underwent heart transplantation for low-output cardiac failure and was
unresponsive to medical treatment. The affected heart is shown in the image. It weighs 955g (normal is
up to 350 g) and shows no evidence of coronary artery atherosclerosis. Histologically, the myocardium
demonstrates hypertrophic myocytes and foci of myocardial fibrosis, but there is no evidence of
inflammation or myofiber disarray. Which of the following is the most likely diagnosis?

Page 2

546
(or)
Histologically, the myocardium demonstrates hypertrophic myocytes and foci of myocardial fibrosis, but
there is no evidence of inflammation or myofiber disarray. Which of the following is the most likely
diagnosis?

A. Cardiac amyloidosis
B. Dilated cardiomyopathy
C. Hypertrophic cardiomyopathy
D. Ventricular aneurysm
----------------------------------------
7. Match the following: 1 Rheumatic heart disease A Large and bulky vegetation 2 Infective
endocarditis B Seen in SLE 3 Marantic Endocarditis C Embolization ++ 4 Libman-Sacks Endocarditis D
Small and warty vegetation
1 Rheumatic heart disease A Large and bulky vegetation
2 Infective endocarditis B Seen in SLE
3 Marantic Endocarditis C Embolization ++
4 Libman-Sacks Endocarditis D Small and warty vegetation

A. 1-B, 2-C, 3-D, 4-A


B. 1-C, 2-D, 3-A, 4-B
C. 1-A, 2-D, 3-B, 4-C
D. 1-D, 2-A, 3-C, 4-B
----------------------------------------
8. In Takotsubo Cardiomyopathy, which chamber of the heart commonly experiences dilation due to the
presence of maximum catecholamine receptors?
A. Right ventricle
B. Left ventricle
C. Right atria
D. Left atria
----------------------------------------

Page 3

547
9. Which of the following syndromes is associated with Arrhythmogenic Right Ventricular
Cardiomyopathy (ARVC)?
A. Marfan syndrome
B. Turner syndrome
C. NAXOS syndrome
D. Down syndrome
----------------------------------------
10. Which genetic defect is associated with Hypertrophic Obstructive Cardiomyopathy (HOCM)?
A. Defect in the troponin gene
B. Defect in the alpha-myosin heavy chain gene
C. Defect in the beta-myosin heavy chain gene
D. Defect in the titin gene
----------------------------------------
11. Which of the following is the primary characteristic of restrictive cardiomyopathy?
A. Systolic dysfunction
B. Diastolic dysfunction
C. Increased cardiac output
D. Both A and C
----------------------------------------

Correct Answers
Question Correct Answer

Question 1 4
Question 2 2
Question 3 3
Question 4 3
Question 5 4
Question 6 2
Question 7 4
Question 8 2
Question 9 3
Question 10 3
Question 11 2

Solution for Question 1:


Correct Option D - Myxoma:

Page 4

548
• Myxomas are the most common primary tumour of the heart. Most arise in the left atrium but can
occur in any chamber or valve.
• Myxomas range from small (<1 cm) to large (≥10 cm) and can be sessile or pedunculated lesions.
• They vary from hard globular masses, mottled with hemorrhage, to soft, translucent, papillary, or
villous lesions having a gelatinous appearance
• The causes of Myxoma can be genetic or sporadic.
• Most myomas are sporadic; GNAS 1 is occasionally seen, the major genetic association is Carney
complex. Genetic causes are due to activating mutations in the GNAS1 gene that encodes the alpha
unit of the G protein.
• Clinical manifestations of Myxoma are due to ball valve obstruction of the valve. It can also present
with a syndrome of constitutional symptoms such as fever and malaise. Although the tumour does not
metastasise, it may embolise.
• The constitutional symptoms are due to the release of a cytokine, Interleukin – 6, from the Myxoma.
• More than half the patients have clinical evidence of mitral valve dysfunction, which can progress to
left heart failure.
• Histopathology of Myxoma shows "LEPIDIC CELLS," spindle-shaped and embedded in a myxoid
matrix.
• Diagnosis of the tumour can be made using an echocardiogram. Further imaging, such as CT or MRI,
can help better characterise and identify the tumour.
• Most myxomas are benign, and Surgery is usually the treatment of choice.
Myxomas are the most common primary tumour of the heart. Most arise in the left atrium but can occur
in any chamber or valve.
Myxomas range from small (<1 cm) to large (≥10 cm) and can be sessile or pedunculated lesions.
They vary from hard globular masses, mottled with hemorrhage, to soft, translucent, papillary, or villous
lesions having a gelatinous appearance
The causes of Myxoma can be genetic or sporadic.
Most myomas are sporadic; GNAS 1 is occasionally seen, the major genetic association is Carney com
plex. Genetic causes are due to activating mutations in the GNAS1 gene that encodes the alpha unit of
the G protein.
Clinical manifestations of Myxoma are due to ball valve obstruction of the valve. It can also present wit
h a syndrome of constitutional symptoms such as fever and malaise. Although the tumour does not me
tastasise, it may embolise.
The constitutional symptoms are due to the release of a cytokine, Interleukin – 6, from the Myxoma.
More than half the patients have clinical evidence of mitral valve dysfunction, which can progress to left
heart failure.
Histopathology of Myxoma shows "LEPIDIC CELLS," spindle-shaped and embedded in a
myxoid matrix.
Diagnosis of the tumour can be made using an echocardiogram. Further imaging, such as CT or MRI,
can help better characterise and identify the tumour.
Most myxomas are benign, and Surgery is usually the treatment of choice.

Page 5

549
• Histopathology of Myxoma showing spindle cells
Histopathology of Myxoma showing spindle cells

Incorrect Options:
Option A - Rhabdomyoma:
• Rhabdomyoma is the most common primary benign and malignant tumour in children. The case
presentation above points toward diagnosing Myxoma, the most common primary tumour of the heart
in adults.
Rhabdomyoma is the most common primary benign and malignant tumour in children. The case prese
ntation above points toward diagnosing Myxoma, the most common primary tumour of the heart in adul
ts.
Option B - Hemangioma:
• Hemangioma is a rare primary cardiac tumour. Additionally, a tumour from the Left atrial wall that can
be misdiagnosed as a myxoma is extremely rare. Therefore, the diagnosis is atrial Myxoma, given the

Page 6

550
case presented above.
Hemangioma is a rare primary cardiac tumour. Additionally, a
tumour from the Left atrial wall that can be misdiagnosed as a
myxoma is extremely rare. Therefore, the diagnosis is atrial Myxoma, given the case presented above.
Option C - Lipoma:
• Lipoma is a tumour containing a very rare, encapsulated mass of fat cells. However, the case
presented above points toward the diagnosis of Myxoma, which is the most common primary tumour of
the heart in adults
Lipoma is a tumour containing a very rare, encapsulated mass of fat cells. However, the case presente
d above points toward the diagnosis of Myxoma, which is the most common primary tumour of the hear
t in adults

Solution for Question 2:


Correct Option B - Cardiac Myxoma:
• Myxomas are the most common primary tumour of the heart. Most myxomas occur in the left atrium
but can happen in any chamber or valve.
• These tumours appear as a glistening, gelatinous polypoid mass, usually 6 to 7 cm in diameter, Unlike
a mural thrombus.
• The causes of Myxoma can be genetic or sporadic. Genetic causes are due to activating mutations in
the GNAS1 gene that encodes the alpha unit of the G protein.
• Clinical manifestations of the Myxoma are due to ball valve obstruction of the valve. It can also
present with a syndrome of constitutional symptoms such as fever and malaise. Although the tumour
does not metastasize, it may embolize.
• The constitutional symptoms are due to the release of a cytokine, Interleukin – 6, from the Myxoma.
• More than half the patients have evidence of mitral valve dysfunction, which can progress to left heart
failure.
• Microscopically, Myxoma has a loose myxoid stroma containing abundant proteoglycans.
• Diagnosis of the tumour can be made using an echocardiogram. Further imaging, such as CT or MRI,
can help to characterize better and identify cancer.
• Most myxomas are benign, and Surgery is usually the treatment of choice.
Myxomas are the most common primary tumour of the heart. Most myxomas occur in the left atrium bu
t can happen in any chamber or valve.
These tumours appear as a glistening, gelatinous polypoid mass, usually 6 to 7
cm in diameter, Unlike a mural thrombus.
The causes of Myxoma can be genetic or sporadic. Genetic causes are due to activating mutations in t
he GNAS1 gene that encodes the alpha unit of the G protein.
Clinical manifestations of the Myxoma are due to ball valve obstruction of the valve. It can also present
with a syndrome of constitutional symptoms such as fever and malaise. Although the tumour does not
metastasize, it may embolize.
The constitutional symptoms are due to the release of a cytokine, Interleukin – 6, from the Myxoma.

Page 7

551
More than half the patients have evidence of mitral valve dysfunction, which can progress to left heart f
ailure.
Microscopically, Myxoma has a loose myxoid stroma containing abundant proteoglycans.
Diagnosis of the tumour can be made using an echocardiogram. Further imaging, such as CT or MRI,
can help to characterize better and identify cancer.
Most myxomas are benign, and Surgery is usually the treatment of choice.
Incorrect Options:
Option A - Calcific aortic stenosis:
• Calcific aortic stenosis involves calcifying the aortic valve due to wear and tear or congenital bicuspid
aortic valve. The image given above demonstrated atrial Myxoma and not calcific aortic stenosis.
Calcific aortic stenosis involves calcifying the aortic valve due to wear and tear or congenital bicuspid a
ortic valve. The image given above demonstrated atrial Myxoma and not calcific aortic stenosis.
Option C - Fibroelastoma:
• Fibroelastoma is a benign primary tumour of the heart. However, the image given above
demonstrated atrial Myxoma.
Fibroelastoma is a
benign primary tumour of the heart. However, the image given above demonstrated atrial Myxoma.
Option D - Mural thrombus:
• The image demonstrates a glistening, gelatinous polypoid mass seen in an atrial myxoma, unlike a
mural thrombus.
The image demonstrates a glistening, gelatinous polypoid mass seen in an atrial myxoma, unlike a
mural thrombus.

Solution for Question 3:


Correct Option C - Rhabdomyoma:
• Tuberous sclerosis is an autosomal dominant condition characterized by developing hamartomas and
benign neoplasms involving the brain and other tissues.
• The most frequent clinical manifestations are seizures, autism, and mental retardation.
• Hamartomas within the CNS take the form of cortical tubers, subependymal nodules, and
subependymal giant-cell astrocytomas.
• Half of the patients with tuberous sclerosis develop rhabdomyomas, primary tumours of cardiac
muscle that, although benign, may compromise cardiac function.
• They may be found on atrioventricular valves or ventricle walls, causing outflow obstruction.
• The nuclei of the rhabdomyoma cells are central or eccentrically located. In some cells, eosinophilic
septa stretch from the cell membrane to a centrally placed nucleus, imparting a spiderlike appearance
to the cell known as 'spider cells'
Tuberous sclerosis is an autosomal dominant condition characterized by developing hamartomas and
benign neoplasms involving the brain and other tissues.
The most frequent clinical manifestations are seizures, autism, and mental retardation.

Page 8

552
Hamartomas within the CNS take the form of cortical tubers, subependymal nodules, and subependym
al giant-cell astrocytomas.
Half of the patients with tuberous sclerosis develop rhabdomyomas, primary tumours of cardiac muscle
that, although benign, may compromise cardiac function.
They may be found on atrioventricular valves or ventricle walls, causing outflow obstruction.
The nuclei of the rhabdomyoma cells are central or eccentrically located. In some cells, eosinophilic se
pta stretch from the cell membrane to a centrally placed nucleus, imparting a
spiderlike appearance to the cell known as 'spider cells'

• Spider cells are pathognomonic for cardiac rhabdomyomas and represent degenerating
rhabdomyocytes.
Spider cells are pathognomonic for cardiac rhabdomyomas and represent degenerating rhabdomyocyt
es.
Incorrect Options:
Option A - Lipoma:
• Lipoma can create ball valve obstruction, and with a history suggesting tuberous sclerosis, a lipoma is
less likely.
Lipoma can create ball valve obstruction, and with a history suggesting tuberous sclerosis, a
lipoma is less likely.
Option B - Myxomas:
• Myxomas are almost exclusively seen in adults and are usually located in the atria, making this a less
likely diagnosis.
Myxomas are almost exclusively seen in adults and are usually located in the atria, making this a
less likely diagnosis.
Option D - Dilated cardiomyopathy:
• Dilated cardiomyopathy involves hypertrophy, dilation, and eventually heart failure and is not
associated with tuberous sclerosis.

Page 9

553
Solution for Question 4:
Correct Option C - Titin:
• Dilated cardiomyopathy (DCM): DCM involves progressive morphological and functional cardiac
dilation and contractile dysfunction(systolic). In addition, it is usually associated with eccentric
hypertrophy. It is the most common type of cardiomyopathy, characterized by biventricular dilation and
impaired contractility that eventually progresses to congestive heart failure. It can occur at any age,
including in childhood but is most common in ages 20-50. About 20-50 % of DCM cases are genetic
that involve mutations in Various proteins, predominantly related to the cytoskeleton (from the nucleus
to sarcomere to the cell membrane to adjacent myocytes) or mitochondria. Mutations in TTN, a gene
that encodes titin (so-called because it is the most significant protein expressed in humans), may
account for approximately 20% of all cases of DCM. Additionally, mutations affecting the
membrane-associated dystrophin protein that couples the cytoskeleton to the extracellular matrix are
also reported. The other causes of DCM include Alcohol, chemotherapy drugs (Doxorubicin),
hemochromatosis, coxsackie virus, peripartum cardiomyopathy, and takotsubo cardiomyopathy. DCM
presents slow and progressive signs and symptoms of congestive heart failure, such as dyspnea,
fatigue, and poor exertional capacity. Treatment is similar to the management of heart failure,
according to the patient's presentation. A heart transplant may be necessary for severe
decompensated heart failure. Complications include secondary mitral regurgitation, intracardiac emboli
and arrhythmias, and sudden cardiac death from progressive heart failure or arrhythmiasHistologically,
the myocardium demonstrates hypertrophic myocytes and foci of myocardial fibrosis.
• DCM involves progressive morphological and functional cardiac dilation and contractile
dysfunction(systolic). In addition, it is usually associated with eccentric hypertrophy.
• It is the most common type of cardiomyopathy, characterized by biventricular dilation and impaired
contractility that eventually progresses to congestive heart failure.
• It can occur at any age, including in childhood but is most common in ages 20-50.
• About 20-50 % of DCM cases are genetic that involve mutations in Various proteins, predominantly
related to the cytoskeleton (from the nucleus to sarcomere to the cell membrane to adjacent myocytes)
or mitochondria.
• Mutations in TTN, a gene that encodes titin (so-called because it is the most significant protein
expressed in humans), may account for approximately 20% of all cases of DCM.
• Additionally, mutations affecting the membrane-associated dystrophin protein that couples the
cytoskeleton to the extracellular matrix are also reported.
• The other causes of DCM include Alcohol, chemotherapy drugs (Doxorubicin), hemochromatosis,
coxsackie virus, peripartum cardiomyopathy, and takotsubo cardiomyopathy.
• DCM presents slow and progressive signs and symptoms of congestive heart failure, such as
dyspnea, fatigue, and poor exertional capacity.
• Treatment is similar to the management of heart failure, according to the patient's presentation. A
heart transplant may be necessary for severe decompensated heart failure.
• Complications include secondary mitral regurgitation, intracardiac emboli and arrhythmias, and
sudden cardiac death from progressive heart failure or arrhythmiasHistologically, the myocardium
demonstrates hypertrophic myocytes and foci of myocardial fibrosis.
Dilated cardiomyopathy (DCM):

Page 10

554
• DCM involves progressive morphological and functional cardiac dilation and contractile
dysfunction(systolic). In addition, it is usually associated with eccentric hypertrophy.
• It is the most common type of cardiomyopathy, characterized by biventricular dilation and impaired
contractility that eventually progresses to congestive heart failure.
• It can occur at any age, including in childhood but is most common in ages 20-50.
• About 20-50 % of DCM cases are genetic that involve mutations in Various proteins, predominantly
related to the cytoskeleton (from the nucleus to sarcomere to the cell membrane to adjacent myocytes)
or mitochondria.
• Mutations in TTN, a gene that encodes titin (so-called because it is the most significant protein
expressed in humans), may account for approximately 20% of all cases of DCM.
• Additionally, mutations affecting the membrane-associated dystrophin protein that couples the
cytoskeleton to the extracellular matrix are also reported.
• The other causes of DCM include Alcohol, chemotherapy drugs (Doxorubicin), hemochromatosis,
coxsackie virus, peripartum cardiomyopathy, and takotsubo cardiomyopathy.
• DCM presents slow and progressive signs and symptoms of congestive heart failure, such as
dyspnea, fatigue, and poor exertional capacity.
• Treatment is similar to the management of heart failure, according to the patient's presentation. A
heart transplant may be necessary for severe decompensated heart failure.
• Complications include secondary mitral regurgitation, intracardiac emboli and arrhythmias, and
sudden cardiac death from progressive heart failure or arrhythmiasHistologically, the myocardium
demonstrates hypertrophic myocytes and foci of myocardial fibrosis.
DCM involves progressive morphological and functional cardiac dilation and contractile dysfunction(sys
tolic). In addition, it is usually associated with eccentric hypertrophy.
It is the most common type of cardiomyopathy, characterized by biventricular dilation and impaired con
tractility that eventually progresses to congestive heart failure.
It can occur at any age, including in childhood but is most common in ages 20-50.
About 20-50 % of DCM cases are genetic that involve mutations in Various proteins, predominantly rel
ated to the cytoskeleton (from the nucleus to sarcomere to the cell membrane to adjacent myocytes) or
mitochondria.
Mutations in TTN, a gene that encodes titin (so-called because it is the most significant protein express
ed in humans), may account for approximately 20% of all cases of DCM.
Additionally, mutations affecting the membrane-associated dystrophin protein that couples the cytoskel
eton to the extracellular matrix are also reported.
The other causes of DCM include Alcohol, chemotherapy drugs (Doxorubicin), hemochromatosis, coxs
ackie virus, peripartum cardiomyopathy, and takotsubo cardiomyopathy.
DCM presents slow and progressive signs and symptoms of congestive heart failure, such as dyspnea,
fatigue, and poor exertional capacity.
Treatment is similar to the management of heart failure, according to the patient's presentation. A
heart transplant may be necessary for severe decompensated heart failure.
Complications include secondary mitral regurgitation, intracardiac emboli and arrhythmias, and sudden
cardiac death from progressive heart failure or arrhythmiasHistologically, the myocardium demonstrate
s hypertrophic myocytes and foci of myocardial fibrosis.

Page 11

555
• The image demonstrates ventricular dilation, and the white arrow points to the mural thrombus
• This image shows variable myocyte hypertrophy and interstitial fibrosis, which is characteristic of
DCM
The image demonstrates ventricular dilation, and the white arrow points to the mural thrombus
Incorrect Options:
Option A - Dystrophin:
• Although mutations involving the membrane-associated dystrophin protein, as seen in DCM,
Mutations affecting the Titin gene are the most common in DCM.
Option B - Tropomyosin:
• Mutations in the Tropomyosin (TPM) gene that encodes the regulatory protein α-tropomyosin can be
associated with hereditary cardiomyopathies such as DCM. However, the mutations involving the titin
gene are the most common.
Option D - Troponin:
• Mutations in the troponin gene that encodes troponin T are associated with some cases of DCM and
hereditary cardiomyopathy. However, the mutations involving the titin gene are most commonly seen in
cases of DCM.

Solution for Question 5:


Option D: Hypertrophic cardiomyopathy
• Hypertrophic cardiomyopathy (HCM): Hypertrophic cardiomyopathy (HCM) is a common genetic
disorder with an incidence of 1 in 500. The pattern of transmission in HCM is autosomal dominant with
variable penetrance. It is characterized by myocardial hypertrophy, poorly compliant left ventricular
myocardium leading to abnormal diastolic filling, and thickening of the interventricular septum that can
occasionally result in intermittent ventricular outflow obstruction. The pathogenesis of HCM involves
mutations in any one of several genes that encode sarcomeric proteins. Mutations that HCM are found
most commonly in the β-myosin heavy chain (β-MHC) encoding gene. It may also involve the genes

Page 12

556
coding for cardiac TnT, α-tropomyosin, and myosin-binding protein C (MYBP-C). overall, these gene
mutations account for 70% to 80% of all cases. HCM primarily causes diastolic dysfunction. Systolic
function is usually preserved. The primary abnormality in HCM is reduced stroke volume due to
impaired diastolic filling. This resulted from reduced ventricular chamber size and decreased left
ventricle compliance due to hypertrophy. Additionally, approximately 25% of patients with HCM may
present with dynamic obstruction to the left ventricular outflow due to movement of the anterior mitral
leaflet moves toward the ventricular septum during systole. Decreased cardiac output and increased
pulmonary venous pressure cause exertional dyspnea. Despite the absence of symptoms, persons with
HCM are at risk of sudden death, particularly during vigorous exercise. On auscultation, a harsh
systolic ejection murmur can be heard due to left ventricular outflow tract obstruction The most notable
histologic characteristic of this disorder is myofiber disarray, which is most extensive in the
interventricular septum. Major complications of HCM are atrial fibrillation, formation of mural thrombus
leading to embolization and stroke, cardiac failure, ventricular arrhythmias, and sudden death,
especially in HCM associated with certain specific mutations. HCM is one of the most common causes
of sudden, otherwise unexplained death in young athletes.
• Hypertrophic cardiomyopathy (HCM) is a common genetic disorder with an incidence of 1 in 500. The
pattern of transmission in HCM is autosomal dominant with variable penetrance.
• It is characterized by myocardial hypertrophy, poorly compliant left ventricular myocardium leading to
abnormal diastolic filling, and thickening of the interventricular septum that can occasionally result in
intermittent ventricular outflow obstruction.
• The pathogenesis of HCM involves mutations in any one of several genes that encode sarcomeric
proteins. Mutations that HCM are found most commonly in the β-myosin heavy chain (β-MHC)
encoding gene. It may also involve the genes coding for cardiac TnT, α-tropomyosin, and
myosin-binding protein C (MYBP-C). overall, these gene mutations account for 70% to 80% of all
cases.
• HCM primarily causes diastolic dysfunction. Systolic function is usually preserved. The primary
abnormality in HCM is reduced stroke volume due to impaired diastolic filling. This resulted from
reduced ventricular chamber size and decreased left ventricle compliance due to hypertrophy.
Additionally, approximately 25% of patients with HCM may present with dynamic obstruction to the left
ventricular outflow due to movement of the anterior mitral leaflet moves toward the ventricular septum
during systole.
• Decreased cardiac output and increased pulmonary venous pressure cause exertional dyspnea.
Despite the absence of symptoms, persons with HCM are at risk of sudden death, particularly during
vigorous exercise.
• On auscultation, a harsh systolic ejection murmur can be heard due to left ventricular outflow tract
obstruction
• The most notable histologic characteristic of this disorder is myofiber disarray, which is most
extensive in the interventricular septum.
• Major complications of HCM are atrial fibrillation, formation of mural thrombus leading to embolization
and stroke, cardiac failure, ventricular arrhythmias, and sudden death, especially in HCM associated
with certain specific mutations. HCM is one of the most common causes of sudden, otherwise
unexplained death in young athletes.
Hypertrophic cardiomyopathy (HCM):
• Hypertrophic cardiomyopathy (HCM) is a common genetic disorder with an incidence of 1 in 500. The
pattern of transmission in HCM is autosomal dominant with variable penetrance.
• It is characterized by myocardial hypertrophy, poorly compliant left ventricular myocardium leading to
abnormal diastolic filling, and thickening of the interventricular septum that can occasionally result in

Page 13

557
intermittent ventricular outflow obstruction.
• The pathogenesis of HCM involves mutations in any one of several genes that encode sarcomeric
proteins. Mutations that HCM are found most commonly in the β-myosin heavy chain (β-MHC)
encoding gene. It may also involve the genes coding for cardiac TnT, α-tropomyosin, and
myosin-binding protein C (MYBP-C). overall, these gene mutations account for 70% to 80% of all
cases.
• HCM primarily causes diastolic dysfunction. Systolic function is usually preserved. The primary
abnormality in HCM is reduced stroke volume due to impaired diastolic filling. This resulted from
reduced ventricular chamber size and decreased left ventricle compliance due to hypertrophy.
Additionally, approximately 25% of patients with HCM may present with dynamic obstruction to the left
ventricular outflow due to movement of the anterior mitral leaflet moves toward the ventricular septum
during systole.
• Decreased cardiac output and increased pulmonary venous pressure cause exertional dyspnea.
Despite the absence of symptoms, persons with HCM are at risk of sudden death, particularly during
vigorous exercise.
• On auscultation, a harsh systolic ejection murmur can be heard due to left ventricular outflow tract
obstruction
• The most notable histologic characteristic of this disorder is myofiber disarray, which is most
extensive in the interventricular septum.
• Major complications of HCM are atrial fibrillation, formation of mural thrombus leading to embolization
and stroke, cardiac failure, ventricular arrhythmias, and sudden death, especially in HCM associated
with certain specific mutations. HCM is one of the most common causes of sudden, otherwise
unexplained death in young athletes.
Hypertrophic cardiomyopathy (HCM) is a common genetic disorder with an incidence of 1
in 500. The pattern of transmission in HCM is autosomal dominant with variable penetrance.
It is characterized by myocardial hypertrophy, poorly compliant left ventricular myocardium leading to a
bnormal diastolic filling, and thickening of the interventricular septum that can occasionally result in inte
rmittent ventricular outflow obstruction.
The pathogenesis of HCM involves mutations in any one of several genes that encode sarcomeric prot
eins. Mutations that HCM are found most commonly in the β-myosin heavy chain (β-MHC) encoding ge
ne. It may also involve the genes coding for cardiac TnT, α-tropomyosin, and myosin-binding protein C
(MYBP-C). overall, these gene mutations account for 70% to 80% of all cases.
HCM primarily causes diastolic dysfunction. Systolic function is usually preserved. The primary abnorm
ality in HCM is reduced stroke volume due to impaired diastolic filling. This resulted from reduced ventri
cular chamber size and decreased left ventricle compliance due to hypertrophy. Additionally, approxim
ately 25% of patients with HCM may present with dynamic obstruction to the left ventricular outflow due
to movement of the anterior mitral leaflet moves toward the ventricular septum during systole.
Decreased cardiac output and increased pulmonary venous pressure cause exertional dyspnea. Despit
e the absence of symptoms, persons with HCM are at risk of sudden death, particularly during vigorous
exercise.
On auscultation, a
harsh systolic ejection murmur can be heard due to left ventricular outflow tract obstruction
The most notable histologic characteristic of this disorder is myofiber disarray, which is most extensive
in the interventricular septum.
Major complications of HCM are atrial fibrillation, formation of mural thrombus leading to embolization a
nd stroke, cardiac failure, ventricular arrhythmias, and sudden death, especially in HCM associated wit

Page 14

558
h certain specific mutations. HCM is one of the most common causes of sudden, otherwise unexplaine
d death in young athletes.

• Gross image demonstrates a septal muscle that bulges into the left ventricular outflow tract and
enlarged left atrium.
• Histologic appearance demonstrates myocyte disarray, hypertrophy, and the characteristic interstitial
fibrosis.
Gross image demonstrates a
septal muscle that bulges into the left ventricular outflow tract and enlarged left atrium.
Histologic appearance demonstrates myocyte disarray, hypertrophy, and the characteristic interstitial fi
brosis.
Incorrect Options:
Option A - Amyloidosis:
• Amyloidosis involves amyloid deposition in the heart muscle, which may be primary or secondary
(inflammation, myeloma, etc). However, it doesn't result in hypertrophic myofiber disarray, which is
characteristic of HCM.
Amyloidosis involves amyloid deposition in the heart muscle, which may be primary or secondary (infla
mmation, myeloma, etc). However, it doesn't result in hypertrophic myofiber disarray, which is characte
ristic of HCM.
Option B - Cardiac myxoma:
• Cardiac myxoma is a primary tumour of the heart involving the left atrium most commonly. The
histological pattern in the image above is characteristic of HCM, not cardiac myoma.
Cardiac myxoma is a primary tumour of the heart involving the left atrium most commonly. The histolog
ical pattern in the image above is characteristic of HCM, not cardiac myoma.
Option C - Dilated cardiomyopathy:
• The heart in HCM is thick-walled, heavy, and hyper-contracting, in contrast to DCM's flabby,
hypo-contracting heart. Additionally, the histological pattern in the image above is characteristic of HCM
and not dilated cardiomyopathy.

Page 15

559
The heart in HCM is thick-walled, heavy, and hyper-contracting, in contrast to DCM's flabby, hypo-contr
acting heart. Additionally, the histological pattern in the image above is characteristic of HCM and not d
ilated cardiomyopathy.

Solution for Question 6:


Correct Option B - Dilated cardiomyopathy:
• Dilated cardiomyopathy (DCM): DCM involves progressive morphological and functional cardiac
dilation and contractile dysfunction(systolic). It is usually associated with eccentric hypertrophy. It is the
most common type of cardiomyopathy characterized by biventricular dilation and impaired contractility
that eventually progresses to congestive heart failure. It can occur at any age, including in childhood
but is most common in ages 20-50. About 20-50 % of DCM cases are genetic that involve mutations in
Various proteins, predominantly related to the cytoskeleton (from the nucleus to sarcomere to the cell
membrane to adjacent myocytes) or mitochondria. Mutations in TTN, a gene that encodes titin
(so-called because it is the largest protein expressed in humans), may account for approximately 20%
of all cases of DCM. Additionally, mutations affecting the membrane-associated dystrophin protein that
couples the cytoskeleton to the extracellular matrix are also reported. The other causes of DCM include
Alcohol, chemotherapy drugs (Doxorubicin), hemochromatosis, coxsackie virus, peripartum
cardiomyopathy, and takotsubo cardiomyopathy. DCM presents slow and progressive with signs and
symptoms of congestive heart failure, such as dyspnea, fatigue, and poor exertional capacity.
Histologically, the myocardium demonstrates hypertrophic myocytes and foci of myocardial fibrosis.
Treatment is similar to the management of heart failure, according to the patient's presentation. A heart
transplant may be necessary for severe decompensated heart failure. Complications include secondary
mitral regurgitation, intracardiac emboli and arrhythmias and sudden cardiac death from progressive
heart failure or arrhythmias.
• DCM involves progressive morphological and functional cardiac dilation and contractile
dysfunction(systolic). It is usually associated with eccentric hypertrophy.
• It is the most common type of cardiomyopathy characterized by biventricular dilation and impaired
contractility that eventually progresses to congestive heart failure.
• It can occur at any age, including in childhood but is most common in ages 20-50.
• About 20-50 % of DCM cases are genetic that involve mutations in Various proteins, predominantly
related to the cytoskeleton (from the nucleus to sarcomere to the cell membrane to adjacent myocytes)
or mitochondria.
• Mutations in TTN, a gene that encodes titin (so-called because it is the largest protein expressed in
humans), may account for approximately 20% of all cases of DCM.
• Additionally, mutations affecting the membrane-associated dystrophin protein that couples the
cytoskeleton to the extracellular matrix are also reported.
• The other causes of DCM include Alcohol, chemotherapy drugs (Doxorubicin), hemochromatosis,
coxsackie virus, peripartum cardiomyopathy, and takotsubo cardiomyopathy.
• DCM presents slow and progressive with signs and symptoms of congestive heart failure, such as
dyspnea, fatigue, and poor exertional capacity.
• Histologically, the myocardium demonstrates hypertrophic myocytes and foci of myocardial fibrosis.
• Treatment is similar to the management of heart failure, according to the patient's presentation. A
heart transplant may be necessary for severe decompensated heart failure.

Page 16

560
• Complications include secondary mitral regurgitation, intracardiac emboli and arrhythmias and sudden
cardiac death from progressive heart failure or arrhythmias.
Dilated cardiomyopathy (DCM):
• DCM involves progressive morphological and functional cardiac dilation and contractile
dysfunction(systolic). It is usually associated with eccentric hypertrophy.
• It is the most common type of cardiomyopathy characterized by biventricular dilation and impaired
contractility that eventually progresses to congestive heart failure.
• It can occur at any age, including in childhood but is most common in ages 20-50.
• About 20-50 % of DCM cases are genetic that involve mutations in Various proteins, predominantly
related to the cytoskeleton (from the nucleus to sarcomere to the cell membrane to adjacent myocytes)
or mitochondria.
• Mutations in TTN, a gene that encodes titin (so-called because it is the largest protein expressed in
humans), may account for approximately 20% of all cases of DCM.
• Additionally, mutations affecting the membrane-associated dystrophin protein that couples the
cytoskeleton to the extracellular matrix are also reported.
• The other causes of DCM include Alcohol, chemotherapy drugs (Doxorubicin), hemochromatosis,
coxsackie virus, peripartum cardiomyopathy, and takotsubo cardiomyopathy.
• DCM presents slow and progressive with signs and symptoms of congestive heart failure, such as
dyspnea, fatigue, and poor exertional capacity.
• Histologically, the myocardium demonstrates hypertrophic myocytes and foci of myocardial fibrosis.
• Treatment is similar to the management of heart failure, according to the patient's presentation. A
heart transplant may be necessary for severe decompensated heart failure.
• Complications include secondary mitral regurgitation, intracardiac emboli and arrhythmias and sudden
cardiac death from progressive heart failure or arrhythmias.
DCM involves progressive morphological and functional cardiac dilation and contractile dysfunction(sys
tolic). It is usually associated with eccentric hypertrophy.
It is the most common type of cardiomyopathy characterized by biventricular dilation and impaired cont
ractility that eventually progresses to congestive heart failure.
It can occur at any age, including in childhood but is most common in ages 20-50.
About 20-50 % of DCM cases are genetic that involve mutations in Various proteins, predominantly rel
ated to the cytoskeleton (from the nucleus to sarcomere to the cell membrane to adjacent myocytes) or
mitochondria.
Mutations in TTN, a gene that encodes titin (so-called because it is the largest protein expressed in hu
mans), may account for approximately 20% of all cases of DCM.
Additionally, mutations affecting the membrane-associated dystrophin protein that couples the cytoskel
eton to the extracellular matrix are also reported.
The other causes of DCM include Alcohol, chemotherapy drugs (Doxorubicin), hemochromatosis, coxs
ackie virus, peripartum cardiomyopathy, and takotsubo cardiomyopathy.
DCM presents slow and progressive with signs and symptoms of congestive heart failure, such as dys
pnea, fatigue, and poor exertional capacity.
Histologically, the myocardium demonstrates hypertrophic myocytes and foci of myocardial fibrosis.

Page 17

561
Treatment is similar to the management of heart failure, according to the patient's presentation. A
heart transplant may be necessary for severe decompensated heart failure.
Complications include secondary mitral regurgitation, intracardiac emboli and arrhythmias and sudden
cardiac death from progressive heart failure or arrhythmias.

• The image demonstrates ventricular dilation, and the white arrow points to the mural thrombus
• This image shows variable myocyte hypertrophy and interstitial fibrosis, which is characteristic of
DCM
The image demonstrates ventricular dilation, and the white arrow points to the mural thrombus
This image shows variable myocyte hypertrophy and interstitial fibrosis, which is characteristic of DCM
Incorrect Options:
Option A - Cardiac amyloidosis:
• Amyloidosis involves depositing amyloid protein in the heart muscle, usually seen in chronic
inflammatory conditions. However, it doesn't result in myocyte hypertrophy and interstitial fibrosis,
which is characteristic of DCM.
Option C - Hypertrophic cardiomyopathy (HCM):
• HCM presents with hypertrophied disarray of the myofibers histologically and concentric hypertrophy
of the ventricles in contrast to myocyte hypertrophy and interstitial fibrosis, and eccentric hypertrophy,
which is seen in DCM.
Option D - Ventricular aneurysm:
• A ventricular aneurysm is an outpouching of the weakened area in the ventricle's wall. It most
commonly occurs in the left ventricle after an event of myocardial infarction. The image mentioned
above demonstrated eccentric hypertrophy seen in DCM.

Solution for Question 7:


Correct Option D - 1-D, 2-A, 3-C, 4-B:

Page 18

562
Incorrect Options:
• Options A, B, and C are incorrectly matched, as explained above.

Solution for Question 8:


Correct Option B - Left ventricle.
• In Takotsubo Cardiomyopathy, the left ventricle commonly experiences dilation due to the presence of
maximum catecholamine receptors, leading to the characteristic octopus trap appearance.
Incorrect Options:
• Options A, C, and D are incorrect, as explained above.

Solution for Question 9:


Correct Option C - NAXOS syndrome:
• NAXOS syndrome is a rare autosomal recessive genetic disorder characterized by the triad of
Arrhythmogenic Right Ventricular Cardiomyopathy (ARVC), woolly hair, and palmoplantar keratoderma.
It is associated with a defect in the Plakoglobin gene.
Incorrect Options:
• Options A, B, and D are not associated with ARVC.

Solution for Question 10:


Correct Option C - Defect in the beta-myosin heavy chain gene:
• Hypertrophic Obstructive Cardiomyopathy (HOCM) is an autosomal dominant disorder caused by a
defect in the beta-myosin heavy chain gene, which leads to hypertrophy in the interventricular septum
Incorrect Options:
• Options A, B, and D are incorrect as they are not associated with HOCM.

Solution for Question 11:


Correct Option B - Diastolic dysfunction:
• Restrictive cardiomyopathy is characterized by impaired diastolic function of the heart, leading to
difficulty in filling during the relaxation phase of the cardiac cycle.
Incorrect Options:
• Options A, C, and D are incorrect, as restrictive cardiomyopathy occurs due to diastolic dysfunction.

Page 19

563
Page 20

564
Infective Endocarditis & Rheumatic Fever
1. A 70-year-old male patient with advanced pancreatic cancer complained of chest pain for 30
minutes. ECG shows inferior wall myocardial infarction. Appropriate measures are taken, but the
patient does not survive. Autopsy reveals sterile non-destructive vegetations along the mitral leaflet
edges. Which of the following has a similar pathogenesis to this patient's vegetation?
A. Hypercalcemia of malignancy
B. Distant metastasis
C. Trousseau syndrome
D. Raynaud’s phenomenon
----------------------------------------
2. A 55-year-old female complained of chest pain for 1 hour associated with shortness of breath and
low-grade fever. She has a history of anterior wall myocardial infarction 6 weeks ago, which was
treated with fibrinolytic therapy. On examination, elevated JVP and a friction rub in the 5th intercostal
space are noted. ECG shows diffuse ST elevation with PR depression. Which of the following is the
most likely diagnosis in this case?
(or)
What is the likely diagnosis in an individual with recent MI, fever, and shortness of breath, with
examination showing friction rub in the fifth intercostal area, and elevated jugular venous pressure
(JVP)?
A. Myocardial rupture
B. Papillary muscle dysfunction
C. Dressler syndrome
D. Ventricular aneurysm
----------------------------------------
3. What is the underlying mechanism responsible for small warty lesions on the cardiac valves found in
the autopsy of a 15-year-old girl presenting with chest pain, respiratory distress, and a pan systolic
murmur, leading to her death despite medical intervention?
(or)
Which of the following mechanisms best explains the development of Rheumatic heart disease?

A. Autoimmune destruction of the myocardium

565
B. Immune complex deposition in the cardiac valves:
C. Direct cytotoxic effects of Group A streptococcal toxins
D. Immune-mediated damage to the cardiac valves
----------------------------------------
4. Which of the following valves is most likely involved in rheumatic heart disease?
A. Mitral
B. Aortic
C. Pulmonary
D. Tricuspid
----------------------------------------
5. A 10-year-old boy with a 2-week history of upper respiratory tract infections is hospitalised with a
fever, malaise, diffuse rash, and joint swelling. The patient is treated and discharged, but he develops a
heart murmur a few years later. Which of the following pathogens is most likely to cause the heart
murmur in this child?
(or)
Which of the following pathogens is responsible for the valvular damage seen in patients with acute
rheumatic fever?
A. Beta-hemolytic streptococcus
B. Candida albicans
C. Epstein-Barr virus
D. Streptococcus viridans
----------------------------------------
6. A 40-year-old woman with a history of rheumatic fever presents with shortness of breath, weight loss,
fatigue, and abdominal distension. Physical examination reveals lung rattle, hepatosplenomegaly, and
2+ pitting oedema in the legs. A chest x-ray shows only left atrial enlargement and pulmonary oedema.
What is the most likely cause of pulmonary oedema in this patient?
A. Aortic insufficiency
B. Aortic stenosis
C. Mitral stenosis
D. Tricuspid insufficiency
----------------------------------------
7. An 18-year-old female presents to the emergency department with a 3-months history of sore throat,
back and joint pains. Vital signs are within normal range. On auscultatory findings, a murmur is also
noted. Gross findings of the heart have been shown below. Which of the following is most likely the
diagnosis?
(or)
What is the diagnosis for a patient who has a sore throat, back pain, joint pain, and cardiac murmur with
the gross heart findings given below?

Page 2

566
A. Libman-Sack's endocarditis
B. Infective endocarditis
C. Rheumatic carditis
D. Viral myocarditis
----------------------------------------
8. Which of the following is most likely related to Aschoff bodies in rheumatic heart disease?
A. Liquefactive necrosis
B. Hurthle cells
C. Anitschkowcells
D. Anucleated cells
----------------------------------------
9. In rheumatic fever, which of the following is most likely included in Jones criteria?
A. Subcutaneous nodules
B. Janeway’s lesion
C. Osler nodes
D. Erythema migrans
----------------------------------------
10. Which of the following conditions is also associated with Anitschkow cells?
A. Infective endocarditis
B. Viral myocarditis
C. Iron Deficiency Anemia
D. Infectious mononucleosis
----------------------------------------
11. A 45-year-old patient comes to the outpatient department to evaluate the difficulty in breathing. He
feels fatigued after walking some miles. The patient has no history of allergy, weight loss and
neurological symptoms. Blood pressure is 130/90 mm Hg, and pulse is 70/min. On physical
examination, the S■ sound is loud. The patient is diagnosed with mitral stenosis. Which of the following
is the most common cause of mitral stenosis?

Page 3

567
A. Rheumatic heart disease
B. Infective-endocarditis
C. Diabetes mellitus
D. Congenital
----------------------------------------
12. A 32-year-old patient comes to the emergency department with chest pain radiating to the left arm.
The patient is undergoing myocardial infarction and expires in the hospital. There is a history of
systemic lupus erythematous and diffuse glomerulonephritis. The autopsy report is consistent with
Libman-Sacks endocarditis. Which of the following is most likely the vegetation features in
Libman-Sacks endocarditis?
A. Infective
B. Flat appearance
C. Small and firm
D. Embolism is not common
----------------------------------------
13. What is the likely causative organism for a 32-year-old man with high-grade fever, dyspnea, fatigue,
a history of drug abuse, and a new pan systolic murmur, along with the given skin findings and
petechiae?

A. Staphylococcus epidermidis
B. Streptococcus viridans
C. Staphylococcus aureus

Page 4

568
D. Enterococcus faecalis
----------------------------------------
14. Which of the following is most likely to cause stroke in a comatose 50-year-old man with pancreatic
adenocarcinomas who regains consciousness but dies after 2 days, his brain CT reveals a left temporal
lobe infarction, and the given autopsy findings?
(or)
Which of the following is most likely to cause a stroke in a patient who has known case of pancreatic
adenocarcinomas, left temporal lobe infarction, and cardiac findings from an autopsy?

A. Calcific aortic stenosis


B. Carcinoid heart disease
C. Cardiac metastases
D. Nonbacterial thrombotic endocarditis
----------------------------------------
15. Which of the following is the most likely condition associated with infective endocarditis in a patient
with a history of IV drug abuse?
A. Aortic insufficiency
B. Mitral regurgitation
C. Ruptured chordae tendineae
D. Tricuspid insufficiency
----------------------------------------
16. What is the most appropriate diagnosis for a 54-year-old woman presenting with fever, weight loss,
fatigue, petechiae, clubbing of fingers, and subsequent massive stroke leading to death, with an
autopsy revealing the given mitral valve pathology?

Page 5

569
A. Bacterial endocarditis
B. Carcinoid heart disease
C. Libman-Sacks endocarditis
D. Mitral valve prolapse
----------------------------------------
17. What is the likely cause of a mid-diastolic murmur heard at the apex in a 30-year-old woman with
increasing fatigue and shortness of breath a history of systemic lupus erythematosus (SLE) and
vegetation on the mitral valve leaflets detected by an echocardiogram?
(or)
What is the most likely cause of a mid-diastolic murmur heard at the apex in a patient with Systemic
Lupus Erythematosus?
A. Libman-Sacks endocarditis
B. Mitral valve prolapse
C. Myocardial infarct
D. Rheumatic fever
----------------------------------------
18. Which type of endocarditis is caused by Streptococcus viridans?
A. Acute infective endocarditis
B. Sub-acute infective endocarditis
C. Prosthetic valve infective endocarditis
D. Libman-sacks endocarditis
----------------------------------------
19. A 45-year-old male patient presents with a 3-week history of shortness of breath, chest pain, and
abdominal discomfort. He gave a history of tuberculosis 2 years ago. His BP is 135/75 mm Hg, and his
JVP is elevated, and bilateral pitting edema in his legs. On auscultation, cardiac sounds are muffled,
along with frictional rub and crackling at the base of both lungs. A chest x-ray shows an enlarged heart
silhouette with no visible borders. The patient eventually dies, and the heart during autopsy is shown.
What is the most likely cause of death in this patient?

Page 6

570
A. Constrictive pericarditis
B. Infective endocarditis
C. Restrictive cardiomyopathy
D. Viral myocarditis
----------------------------------------
20. A 35-year-old female patient came for a routine examination. Her BP is 130/85, her pulse is 65/min,
and her BMI is 23 kg/m2. On auscultation, a mid-systolic click is present. There is no history of
rheumatic heart disease. The histopathological examination is most likely to show which of the
following?
(or)
In patients with mitral valve prolapse, the histopathological examination is most likely to show which of
the following?
A. Myxomatous degeneration and prolapse of the mitral valve
B. Fibrinous deposition on the tip of papillary muscle
C. Rupture of chordae tendineae
D. Aschoff nodule on the mitral valve
----------------------------------------

Correct Answers
Question Correct Answer

Question 1 3
Question 2 3
Question 3 4
Question 4 1
Question 5 1
Question 6 3
Question 7 3
Question 8 3

Page 7

571
Question 9 1
Question 10 3
Question 11 1
Question 12 3
Question 13 3
Question 14 4
Question 15 4
Question 16 1
Question 17 1
Question 18 2
Question 19 1
Question 20 1

Solution for Question 1:


Correct Option C - Trousseau syndrome:
• The patient presents with an example of nonbacterial thrombotic endocarditis (NBTE).
• The pathophysiology of NBTE is similar to that of Trousseau's syndrome (migratory thrombophlebitis).
• Nonbacterial thrombotic endocarditis (NBTE) is the deposition of sterile thrombi on cardiac valves.
• It is most likely seen with an underlying hypercoagulable state such as chronic disseminated
intravascular coagulation, hyperoestrogenic states, and those associated with underlying malignancy
especially mucinous adenocarcinoma of the pancreas and adenocarcinoma of the lungs (procoagulant
effect of circulating mucin and tissue factor by tumours
• It is also seen in conditions with general debility or wasting, known as marantic endocarditis.
• The sterile valvular lesions of NBTE occur on previously normal valves and are non-destructive and
small (1-5mm in diameter).
• NBTE lesions give rise to emboli that can cause infarcts in the brain, heart, and other organs.
• NBTE can also serve as a potential nidus for bacterial colonization and the consequent development
of infective endocarditis.
The patient presents with an example of nonbacterial thrombotic endocarditis (NBTE).
The pathophysiology of NBTE is similar to that of Trousseau's syndrome (migratory thrombophlebitis).
Nonbacterial thrombotic endocarditis (NBTE) is the deposition of sterile thrombi on cardiac valves.
It is most likely seen with an underlying hypercoagulable state such as chronic disseminated intravascu
lar coagulation, hyperoestrogenic states, and those associated with underlying malignancy especially
mucinous adenocarcinoma of the pancreas and adenocarcinoma of the lungs (procoagulant effect of ci
rculating mucin and tissue factor by tumours
It is also seen in conditions with general debility or wasting, known as marantic endocarditis.
The sterile valvular lesions of NBTE occur on previously normal valves and are non-destructive and sm
all (1-5mm in diameter).

Page 8

572
NBTE lesions give rise to emboli that can cause infarcts in the brain, heart, and other organs.
NBTE can also serve as a
potential nidus for bacterial colonization and the consequent development of infective endocarditis.
Incorrect Options:
Option A - Hypercalcemia of malignancy:
• Hypercalcemia of malignancy is a common finding in patients with advanced-stage cancers.
• It presents with neuropsychiatric, gastrointestinal, renal, and bony abnormalities.
• It does not form sterile vegetation in the valves.
Hypercalcemia of malignancy is a common finding in patients with advanced-stage cancers.
It presents with neuropsychiatric, gastrointestinal, renal, and bony abnormalities.
It does not form sterile vegetation in the valves.
Option B - Distant metastasis:
• Cancer metastasis affects the pericardium and myocardium. It is less likely to affect cardiac valves.
• Invasion of valves may rarely be seen in metastatic involvement
Cancer metastasis affects the pericardium and myocardium. It is less likely to affect cardiac valves.
Invasion of valves may rarely be seen in metastatic involvement
Option D - Raynaud’s phenomenon:
• Raynaud’s phenomenon occurs due to spasm of small blood vessels in the fingers and toes.
• It presents with skin colour changes, numbness, and sensations of pins and needles.
• It has no link to valvular vegetations.
Raynaud’s phenomenon occurs due to spasm of small blood vessels in the fingers and toes.
It presents with skin colour changes, numbness, and sensations of pins and needles.
It has no link to valvular vegetations.

Solution for Question 2:


Correct Option C - Dressler syndrome:
• Dressler syndrome, also known as postmyocardial infarction syndrome, is a complication of
myocardial infarction or cardiac surgery.
• It results from an autoimmune inflammatory reaction to antigens of the myocardium.
• It is a fibrinous pericarditis
• It presents 2-3 weeks after myocardial infarction with the following clinical features: Low-grade fever
Pleuritic chest pain Friction rub Diffuse ST elevation with PR depression
• Low-grade fever
• Pleuritic chest pain
• Friction rub

Page 9

573
• Diffuse ST elevation with PR depression
• Treatment includes corticosteroids and aspirin.
• Dressler syndrome is an inflammatory condition involving the pericardium (the membrane around the
heart) that occurs after myocardial infarction or cardiac surgery. It typically presents weeks to months
after the initial event. The symptoms often include chest pain, fever, pleuritic pain, and sometimes
pericardial friction rub. Elevated JVP can occur due to impaired filling of the heart caused by pericardial
effusion (fluid accumulation in the pericardial sac).
Dressler syndrome, also known as postmyocardial infarction syndrome, is a
complication of myocardial infarction or cardiac surgery.
It results from an autoimmune inflammatory reaction to antigens of the myocardium.
It is a fibrinous pericarditis
It presents 2-3 weeks after myocardial infarction with the following clinical features:
• Low-grade fever
• Pleuritic chest pain
• Friction rub
• Diffuse ST elevation with PR depression
Low-grade fever
Pleuritic chest pain
Friction rub
Diffuse ST elevation with PR depression
Treatment includes corticosteroids and aspirin.
Dressler syndrome is an inflammatory condition involving the pericardium (the membrane around the h
eart) that occurs after myocardial infarction or cardiac surgery. It typically presents weeks to months aft
er the initial event. The symptoms often include chest pain, fever, pleuritic pain, and sometimes pericar
dial friction rub. Elevated JVP can occur due to impaired filling of the heart caused by pericardial effusi
on (fluid accumulation in the pericardial sac).
Incorrect Options:
Option A - Myocardial rupture:
• Myocardial rupture usually occurs as left ventricular wall rupture, leading to cardiac tamponade and
hemopericardium.
• It occurs 3-7 days after MI and presents with chest pain, elevated JVP, and reduced heart sounds.
Myocardial rupture usually occurs as left ventricular wall rupture, leading to cardiac tamponade and he
mopericardium.
It occurs 3-7 days after MI and presents with chest pain, elevated JVP, and reduced heart sounds.
Option B - Papillary muscle dysfunction:
• Papillary muscle dysfunction leads to mitral valve regurgitation and insufficiency.
• Patients would present with palpitations and cardiac murmurs.
• Friction rub and elevated JVP are not signs of mitral valve regurgitation.
Papillary muscle dysfunction leads to mitral valve regurgitation and insufficiency.

Page 10

574
Patients would present with palpitations and cardiac murmurs.
Friction rub and elevated JVP are not signs of mitral valve regurgitation.
Option D - Ventricular aneurysm:
• Ventricular aneurysms are a late complication of MI.
• They lead to the formation of mural thrombi, arrhythmias, and heart failure but do not rupture.
• Friction rub and elevated JVP are not seen.
Ventricular aneurysms are a late complication of MI.
They lead to the formation of mural thrombi, arrhythmias, and heart failure but do not rupture.
Friction rub and elevated JVP are not seen.

Solution for Question 3:


Correct Option D - Immune-mediated damage to the cardiac valves :
• This patient is affected by rheumatic heart disease, and the image shows “Aschoff body”.
• It results from the immune response to group A streptococci (Strep. Pyogenes) cross-react with the
host tissue.
• The immune system cross-reacts with host tissues, leading to immune-mediated damage to the
cardiac valves, particularly the mitral valve, resulting in valvular dysfunction and subsequent heart
disease.
• Antibodies directed against streptococcal M proteins cross-react with cardiac autoantigens. In
addition, CD4+T cells, which are specific for streptococcal peptides, also react with self-proteins in the
heart and produce cytokines that activate macrophages. Therefore, cardiac tissue damage combines
antibody and T-cell-mediated responses.
• In addition, CD4+T cells, which are specific for streptococcal peptides, also react with self-proteins in
the heart and produce cytokines that activate macrophages.
• Therefore, cardiac tissue damage combines antibody and T-cell-mediated responses.
• These inflammatory lesions in the heart, are called the "Aschoff body".
• They consist of foci of eosinophilic material surrounded by T lymphocytes, occasionally plasma cells,
and bulging macrophages called “Anitschkow cells".
• These characteristic cells have abundant cytoplasm and round-to-void central nuclei in which the
chromatin is arranged in a central, thin, wavy band (caterpillar cells).
• These lesions are pathognomonic for rheumatic fever.
This patient is affected by rheumatic heart disease, and the image shows “Aschoff body”.
It results from the immune response to group A
streptococci (Strep. Pyogenes) cross-react with the host tissue.
The immune system cross-reacts with host tissues, leading to immune-mediated damage to the cardia
c valves, particularly the mitral valve, resulting in valvular dysfunction and subsequent heart disease.
Antibodies directed against streptococcal M proteins cross-react with cardiac autoantigens.

Page 11

575
• In addition, CD4+T cells, which are specific for streptococcal peptides, also react with self-proteins in
the heart and produce cytokines that activate macrophages.
• Therefore, cardiac tissue damage combines antibody and T-cell-mediated responses.
In addition, CD4+T cells, which are specific for streptococcal peptides, also react with self-proteins in t
he heart and produce cytokines that activate macrophages.
Therefore, cardiac tissue damage combines antibody and T-cell-mediated responses.
These inflammatory lesions in the heart, are called the "Aschoff body".
They consist of foci of eosinophilic material surrounded by T
lymphocytes, occasionally plasma cells, and bulging macrophages called “Anitschkow cells".
These characteristic cells have abundant cytoplasm and round-to-void central nuclei in which the chro
matin is arranged in a central, thin, wavy band (caterpillar cells).
These lesions are pathognomonic for rheumatic fever.
Incorrect Options:
Option A - Autoimmune destruction of the myocardium :
• While rheumatic heart disease involves an autoimmune response, it causes pancarditis rather than
targeting the myocardium alone. The characteristic feature of rheumatic heart disease is valvular
damage, leading to conditions like mitral stenosis and mitral regurgitation
Option B - Immune complex deposition in the cardiac valves :
• While immune complexes may play a role in certain immune-mediated diseases, such as systemic
lupus erythematosus, they are not the primary mechanism involved in the pathogenesis of rheumatic
heart disease.
• Instead, the immune response primarily targets specific components of the cardiac valves, resulting in
inflammation and scarring.
While immune complexes may play a role in certain immune-mediated diseases, such as systemic lup
us erythematosus, they are not the primary mechanism involved in the pathogenesis of rheumatic hear
t disease.
Instead, the immune response primarily targets specific components of the cardiac valves, resulting in i
nflammation and scarring.
Option C - Direct cytotoxic effects of Group A streptococcal toxins:
• In rheumatic heart disease, the primary mechanism is an abnormal immune response triggered by the
streptococcal infection.
• The immune system cross-reacts with host tissues, particularly the cardiac valves, leading to
inflammation and damage, rather than direct cytotoxic effects of the toxins.
In rheumatic heart disease, the primary mechanism is an abnormal immune response triggered by the
streptococcal infection.
The immune system cross-reacts with host tissues, particularly the cardiac valves, leading to inflammat
ion and damage, rather than direct cytotoxic effects of the toxins.

Solution for Question 4:

Page 12

576
Correct Option A - Mitral valve:
• Rheumatic heart disease results from untreated or improperly treated acute group A streptococcal
pharyngitis.
• Anti-streptococcal antibodies attack the cardiac and neuronal antigens.
• It most commonly involves the mitral valve > aortic valve.
• The early lesion is mitral valve regurgitation, and the late lesion is mitral stenosis.
Rheumatic heart disease results from untreated or improperly treated acute group A
streptococcal pharyngitis.
Anti-streptococcal antibodies attack the cardiac and neuronal antigens.
It most commonly involves the mitral valve > aortic valve.
The early lesion is mitral valve regurgitation, and the late lesion is mitral stenosis.
Incorrect Options:
Option B - Aortic valve:
• In acute rheumatic fever, high-pressure valves are affected.
• The Mitral valve is affected more than the aortic valve.
In acute rheumatic fever, high-pressure valves are affected.
The Mitral valve is affected more than the aortic valve.
Option C - Pulmonary valve:
• It is rarely affected in acute rheumatic fever.
It is rarely affected in acute rheumatic fever.
Option D - Tricuspid valve:
• The tricuspid valve involves in the following order: Mitral >aortic > tricuspid
• Mitral >aortic > tricuspid
The tricuspid valve involves in the following order:
• Mitral >aortic > tricuspid
Mitral >aortic > tricuspid

Solution for Question 5:


Correct Option A - Beta-hemolytic streptococcus:
• Rheumatic fever develops after antibodies to surface antigens of group A beta-hemolytic streptococci
cross-react with similar antigens found in the heart, the connective tissue of the skin and joints.
• Acute rheumatic fever causes cardiac lesions like endocarditis, myocarditis and pericarditis, or all
three.
• Chronic rheumatic fever endocarditis causes fibrous scarring and deformity of cardiac valves, which
leads to heart murmurs and functional defects.
• None of the other pathogens causes rheumatic heart disease.

Page 13

577
Rheumatic fever develops after antibodies to surface antigens of group A beta-hemolytic streptococci c
ross-react with similar antigens found in the heart, the connective tissue of the skin and joints.
Acute rheumatic fever causes cardiac lesions like endocarditis, myocarditis and pericarditis, or all three
.
Chronic rheumatic fever endocarditis causes fibrous scarring and deformity of cardiac valves, which le
ads to heart murmurs and functional defects.
None of the other pathogens causes rheumatic heart disease.
Incorrect Options:
Option B - Candida albicans:
• It is a naturally occurring fungus on the human body.
• Mainly lives on the skin and within our body (GIT, mouth, vagina).
• The overproduction of this fungus can lead to infection- candidiasis.
• It causes the following signs/symptoms: Skin rash Itching Blisters Lumpy white patches Burning
sensation Vaginal discharge
• Skin rash
• Itching
• Blisters
• Lumpy white patches
• Burning sensation
• Vaginal discharge
• It mainly caused infection in the following cases due to an imbalance between pathogenic and
beneficial organisms: Antibiotics Pregnancy Diabetes HIV
• Antibiotics
• Pregnancy
• Diabetes
• HIV
It is a naturally occurring fungus on the human body.
Mainly lives on the skin and within our body (GIT, mouth, vagina).
The overproduction of this fungus can lead to infection- candidiasis.
It causes the following signs/symptoms:
• Skin rash
• Itching
• Blisters
• Lumpy white patches
• Burning sensation
• Vaginal discharge
Skin rash

Page 14

578
Itching
Blisters
Lumpy white patches
Burning sensation
Vaginal discharge
It mainly caused infection in the following cases due to an imbalance between pathogenic and benefici
al organisms:
• Antibiotics
• Pregnancy
• Diabetes
• HIV
Antibiotics
Pregnancy
Diabetes
HIV
Option C - Epstein-Barr virus:
• It is a type of herpes virus called herpes virus 4.
• It mainly spreads through saliva and body fluids.
• Mostly asymptomatic and causes infectious mononucleosis in young adults.
• It causes the following signs/symptoms: Sore throat and swelling Fatigue Fever Swollen lymph nodes
Rash on skin Enlarged spleen and liver
• Sore throat and swelling
• Fatigue
• Fever
• Swollen lymph nodes
• Rash on skin
• Enlarged spleen and liver
• It affects the body and remains inactive. The activation is caused by the following: Stress Weak
immune system Menopause or hormone changes
• Stress
• Weak immune system
• Menopause or hormone changes
It is a type of herpes virus called herpes virus 4.
It mainly spreads through saliva and body fluids.
Mostly asymptomatic and causes infectious mononucleosis in young adults.
• Sore throat and swelling

Page 15

579
• Fatigue
• Fever
• Swollen lymph nodes
• Rash on skin
• Enlarged spleen and liver
Sore throat and swelling
Fatigue
Fever
Swollen lymph nodes
Rash on skin
Enlarged spleen and liver
It affects the body and remains inactive. The activation is caused by the following:
• Stress
• Weak immune system
• Menopause or hormone changes
Stress
Weak immune system
Menopause or hormone changes
Option D - Streptococcus viridans:
• Sub-Acute Bacterial endocarditis is an infection of the endocardial surface of the heart.
• The following pathogens mostly cause it: Viridans streptococci (sub-acute)
• Viridans streptococci (sub-acute)
• It causes the following signs/symptoms: Fever New murmur Roth spots Osler nodes Splinter
haemorrhages Janeway lesions
• Fever
• New murmur
• Roth spots
• Osler nodes
• Splinter haemorrhages
• Janeway lesions
• It has a gradual onset.
• The Mitral valve is frequently involved.
• It causes smaller vegetation on congenitally abnormal or disease valves.
Sub-Acute Bacterial endocarditis is an infection of the endocardial surface of the heart.
The following pathogens mostly cause it:
• Viridans streptococci (sub-acute)

Page 16

580
Viridans streptococci (sub-acute)
• Fever
• New murmur
• Roth spots
• Osler nodes
• Splinter haemorrhages
• Janeway lesions
New murmur
Roth spots
Osler nodes
Splinter haemorrhages
Janeway lesions
It has a gradual onset.
The Mitral valve is frequently involved.
It causes smaller vegetation on congenitally abnormal or disease valves.

Solution for Question 6:


Correct Option C - Mitral stenosis:
• The mitral valve is the most commonly and severely affected valve in chronic rheumatic disease.
• Chronic rheumatic valvulitis is characterised by calcification and irregular thickening of the leaflets,
with the fusion of the commissures and chordae tendineae. As a result, the valve orifice becomes
reduced to a “fish mouth” or “buttonhole” stenosis.
• The pressure in the left atrium rises and is transmitted via the pulmonary veins to the pulmonary
vasculature.
• The image shows rheumatic vegetation on the mitral valve in chronic rheumatic disease.
The mitral valve is the most commonly and severely affected valve in chronic rheumatic disease.
Chronic rheumatic valvulitis is characterised by calcification and irregular thickening of the leaflets, with
the fusion of the commissures and chordae tendineae. As a
result, the valve orifice becomes reduced to a “fish mouth” or “buttonhole” stenosis.
The pressure in the left atrium rises and is transmitted via the pulmonary veins to the pulmonary vascul
ature.
The image shows rheumatic vegetation on the mitral valve in chronic rheumatic disease.

Page 17

581
Incorrect Options:
Option A - Aortic insufficiency:
• In aortic insufficiency, the aortic valve cannot control blood flow from the left ventricle into the aorta,
resulting in the leaking of blood backwards.
• It causes the following signs/symptoms: Shortness of breath upon exertion or lying flat Coughing and
shortness of breath at night Palpitations Angina Oedema in legs
• Shortness of breath upon exertion or lying flat
• Coughing and shortness of breath at night
• Palpitations
• Angina
• Oedema in legs
• In cases of aortic insufficiency or stenosis, the left atrium is initially protected by the closure of the
mitral valve.
• In acute rheumatic fever, high-pressure valves are affected.
• The Mitral valve is affected more than the aortic valve.
In aortic insufficiency, the aortic valve cannot control blood flow from the left ventricle into the aorta, res
ulting in the leaking of blood backwards.
It causes the following signs/symptoms:
• Shortness of breath upon exertion or lying flat
• Coughing and shortness of breath at night
• Palpitations
• Angina
• Oedema in legs
Shortness of breath upon exertion or lying flat

Page 18

582
Coughing and shortness of breath at night
Palpitations
Angina
Oedema in legs
In cases of aortic insufficiency or stenosis, the left atrium is initially protected by the closure of the mitra
l valve.
In acute rheumatic fever, high-pressure valves are affected.
The Mitral valve is affected more than the aortic valve.
Option B - Aortic stenosis:
• In cases of aortic insufficiency or stenosis, the left atrium is initially protected by the closure of the
mitral valve.
• In acute rheumatic fever, high-pressure valves are affected.
• The Mitral valve is affected more than the aortic valve.
Option D - Tricuspid insufficiency:
• The tricuspid valve is involved in the following order in case of acute rheumatic fever: Mitral >aortic >
tricuspid
• Mitral >aortic > tricuspid
• It is not associated with atrial enlargement or pulmonary oedema.
The tricuspid valve is involved in the following order in case of acute rheumatic fever:
• Mitral >aortic > tricuspid
Mitral >aortic > tricuspid
It is not associated with atrial enlargement or pulmonary oedema.

Solution for Question 7:


Correct Option C - Rheumatic carditis:
• Given the presence of a cardiac murmur along with a history of sore throat and joint pains, the image
shows small warty vegetation along the line of closure of the valve and the absence of chordae
tendineae involvement, Rheumatic Carditis is the most likely diagnosis in this scenario.
• Rheumatic Carditis is a complication of rheumatic fever, which usually occurs when streptococcal
throat infection is left untreated or not treated adequately.
• It leads to inflammation of the heart, which affects all the layers, including the endocardium,
myocardium, and pericardium.
• The vegetations observed in rheumatic carditis typically display Aschoff bodies, which are
characteristic histological features specific to rheumatic carditis.
Given the presence of a cardiac murmur along with a history of sore throat and joint pains, the image s
hows small warty vegetation along the line of closure of the valve and the absence of chordae tendinea
e involvement, Rheumatic Carditis is the most likely diagnosis in this scenario.

Page 19

583
Rheumatic Carditis is a complication of rheumatic fever, which usually occurs when streptococcal throa
t infection is left untreated or not treated adequately.
It leads to inflammation of the heart, which affects all the layers, including the endocardium, myocardiu
m, and pericardium.
The vegetations observed in rheumatic carditis typically display Aschoff bodies, which are characteristi
c histological features specific to rheumatic carditis.
Incorrect Options:
Option A - Libman-Sack's endocarditis:
• Libman-Sacks Endocarditis is typically associated with systemic lupus erythematosus (SLE) and is
characterized by nonbacterial verrucous vegetations on the valve leaflets. It is less commonly seen in
young patients without a known history of autoimmune diseases like SLE. The presentation of sore
throat and joint pains is not typical of SLE-associated Libman-Sacks Endocarditis.

• The vegetation has the following characteristics: Small or medium-sized Friable Tan masses Broad
base
• Small or medium-sized
• Friable
• Tan masses
• Broad base
• The lesions are small (1 to 4 mm in diameter), single or multiple, sterile, pink vegetations with a warty
(verrucous) appearance.
• They may be located on the undersurfaces of the atrioventricular valves, on the valvular endocardium,
on the chords, or on the mural endocardium of the atria or ventricles.
• Vegetations are small, sterile, and firm and commonly affect the valve's lower surface.
Libman-Sacks Endocarditis is typically associated with systemic lupus erythematosus (SLE) and is cha
racterized by nonbacterial verrucous vegetations on the valve leaflets. It is less commonly seen in youn
g patients without a known history of autoimmune diseases like SLE. The presentation of sore throat a
nd joint pains is not typical of SLE-associated Libman-Sacks Endocarditis.

Page 20

584
The vegetation has the following characteristics:
• Small or medium-sized
• Friable
• Tan masses
• Broad base
Small or medium-sized
Friable
Tan masses
Broad base
The lesions are small (1 to 4 mm in diameter), single or multiple, sterile, pink vegetations with a
warty (verrucous) appearance.
They may be located on the undersurfaces of the atrioventricular valves, on the valvular endocardium,
on the chords, or on the mural endocardium of the atria or ventricles.
Vegetations are small, sterile, and firm and commonly affect the valve's lower surface.
Option B - Infective endocarditis:
• While infective endocarditis can present with vegetation on the heart valves, typically involving the
chordae tendineae, the absence of chordae tendineae involvement makes it less likely in this scenario.
Additionally, the presentation of sore throat and joint pains over 3 months is not typical of acute
infective endocarditis.

• Bacterial endocarditis is an infection of the endocardial surface of the heart.
• The following pathogens mostly cause it: S. Aures (acute) Viridans streptococci (sub-acute)
• S. Aures (acute)
• Viridans streptococci (sub-acute)

Page 21

585
• It causes the following signs/symptoms: Fever New murmur Roth spots Osler nodes Splinter
hemorrhages Janeway lesions
• Fever
• New murmur
• Roth spots
• Osler nodes
• Splinter hemorrhages
• Janeway lesions
• The Mitral valve is frequently involved.
• Tricuspid valve endocarditis is associated with IV drug abuse.
• IV drug abuse is more commonly associated with infective endocarditis (IE); S. aureus is the major
offender in IE among intravenous drug abusers.
While infective endocarditis can present with vegetation on the heart valves, typically involving the chor
dae tendineae, the absence of chordae tendineae involvement makes it less likely in this scenario. Add
itionally, the presentation of sore throat and joint pains over 3
months is not typical of acute infective endocarditis.

Bacterial endocarditis is an infection of the endocardial surface of the heart.


The following pathogens mostly cause it:
• S. Aures (acute)
• Viridans streptococci (sub-acute)
S. Aures (acute)
Viridans streptococci (sub-acute)
It causes the following signs/symptoms:
• Fever
• New murmur

Page 22

586
• Roth spots
• Osler nodes
• Splinter hemorrhages
• Janeway lesions
Fever
New murmur
Roth spots
Osler nodes
Splinter hemorrhages
Janeway lesions
The Mitral valve is frequently involved.
Tricuspid valve endocarditis is associated with IV drug abuse.
IV drug abuse is more commonly associated with infective endocarditis (IE); S. aureus is the major offe
nder in IE among intravenous drug abusers.
Option D - Viral myocarditis:
• Viral myocarditis primarily involves inflammation of the myocardium rather than the endocardium.
While it can cause symptoms such as chest pain and shortness of breath, it is less likely to present with
vegetation on the heart valves as shown in the gross findings.
• It is mostly caused by the following: Coxsackie virus Parvovirus B-19 Human herpes virus 6.
• Coxsackie virus
• Parvovirus B-19
• Human herpes virus 6.
Viral myocarditis primarily involves inflammation of the myocardium rather than the endocardium. Whil
e it can cause symptoms such as chest pain and shortness of breath, it is less likely to present with ve
getation on the heart valves as shown in the gross findings.
It is mostly caused by the following:
• Coxsackie virus
• Parvovirus B-19
• Human herpes virus 6.
Coxsackie virus
Parvovirus B-19
Human herpes virus 6.

Solution for Question 8:


Correct Option C - Anitschkow cells:
• Anitschkow cells in rheumatic heart disease are present within the Aschoff bodies (granulomas).

Page 23

587
• These are enlarged macrophages known as caterpillar cells due to the oval nucleus and chromatin
condensing towards the center in the wavy and ribbon-like pattern.
• Many large anitschkow cells join to form giant Aschoff bodies.
• These lesions are pathognomonic for rheumatic fever
Anitschkow cells in rheumatic heart disease are present within the Aschoff bodies (granulomas).
These are enlarged macrophages known as caterpillar cells due to the oval nucleus and chromatin con
densing towards the center in the wavy and ribbon-like pattern.
Many large anitschkow cells join to form giant Aschoff bodies.
These lesions are pathognomonic for rheumatic fever

Incorrect Options:
Option A - Liquefactive necrosis:
• It is a type of necrosis in which all the tissue material is converted into a viscous liquid form.
• It mainly occurs in bacterial or fungal infections and cases of chemical burns.
• Fibrinoid necrosis occurs in case of rheumatic heart disease.
It is a type of necrosis in which all the tissue material is converted into a viscous liquid form.
It mainly occurs in bacterial or fungal infections and cases of chemical burns.
Fibrinoid necrosis occurs in case of rheumatic heart disease.
Option B - Hurthle cells:
• These special cells in the thyroid arise from the follicular epithelium.
• These are larger than follicles and stain the thyroid eosinophilic.
• These are pathognomic for hurthle cell cancer.
These special cells in the thyroid arise from the follicular epithelium.
These are larger than follicles and stain the thyroid eosinophilic.
These are pathognomic for hurthle cell cancer.

Page 24

588
Option D - Anucleated cells:
• In Aschoff bodies, cells are mainly multinucleated.
• In humans, only anucleated cells are mature RBCs.
In Aschoff bodies, cells are mainly multinucleated.
In humans, only anucleated cells are mature RBCs.

Solution for Question 9:


Correct Option A - Subcutaneous nodules:
• These are small lumps under the skin.
• These are usually uncommon and invariably related to carditis.
• These are included in Jones criteria and are painless, even unnoticed by patients.
• The most common location is joints, the back of the scalp, and the backbone.
These are small lumps under the skin.
These are usually uncommon and invariably related to carditis.
These are included in Jones criteria and are painless, even unnoticed by patients.
The most common location is joints, the back of the scalp, and the backbone.

• The following are also components of Jones criteria: Joints pain Pancarditis Erythema marginatum
Chorea
• Joints pain
• Pancarditis
• Erythema marginatum
• Chorea

Page 25

589
The following are also components of Jones criteria:
• Joints pain
• Pancarditis
• Erythema marginatum
• Chorea
Joints pain
Pancarditis
Erythema marginatum
Chorea
Incorrect Options:
Option B - Janeway’s lesion:
• Janeway's lesions are non-tender, macular, and erythematous lesions on the palms and soles of the
patient, as given in the image.
• These are mostly present in the hand's thenar and hypothenar surface and the toes' plantar surface.
• The duration is usually from days to weeks.
• It is associated with acute infective endocarditis.
Janeway's lesions are non-tender, macular, and erythematous lesions on the palms and soles of the p
atient, as given in the image.
These are mostly present in the hand's thenar and hypothenar surface and the toes' plantar surface.
The duration is usually from days to weeks.
It is associated with acute infective endocarditis.

Option C - Osler nodes:


• Osler nodes are tender, violaceous nodules typically located in the pulp of fingers and toes seen in the
patient, as given in the image.

Page 26

590
• These are usually transient and removed within days.
• It is associated with acute infective endocarditis.
Osler nodes are tender, violaceous nodules typically located in the pulp of fingers and toes seen in the
patient, as given in the image.
These are usually transient and removed within days.

Option D - Erythema migrans:


• It is a Bullseye red rash with central clearing:
• It is present in the early stage of Lyme disease caused by borrelia burgdorferi, transmitted by an
infected black-legged tick.
• Lyme disease can cause heart block but is not associated with MS.
It is a Bullseye red rash with central clearing:
It is present in the early stage of Lyme disease caused by borrelia burgdorferi, transmitted by an infect
ed black-legged tick.
Lyme disease can cause heart block but is not associated with MS.

Page 27

591
Solution for Question 10:
Correct Option C - Iron Deficiency Anemia:
• The most common type of anaemia occurs due to a lack of iron.
• Iron is needed to make haemoglobin that carries oxygen.
• The followings present it: Fatigue Weakness Pale skin Brittle nail Shortness of breath Dizziness
• Fatigue
• Weakness
• Pale skin
• Brittle nail
• Shortness of breath
• Dizziness
• Anitschkow cells are called caterpillar cells because their nuclear material is clumped in a wavy form.
Conditions it can be seen in are: Rheumatic Fever Recurrent aphthous stomatitis Iron deficiency
anaemia Megaloblastic anaemia Sickle cell anaemia
• Rheumatic Fever
• Recurrent aphthous stomatitis
• Iron deficiency anaemia
• Megaloblastic anaemia
• Sickle cell anaemia
The most common type of anaemia occurs due to a lack of iron.
Iron is needed to make haemoglobin that carries oxygen.

Page 28

592
The followings present it:
• Fatigue
• Weakness
• Pale skin
• Brittle nail
• Shortness of breath
• Dizziness
Fatigue
Weakness
Pale skin
Brittle nail
Shortness of breath
Dizziness
Anitschkow cells are called caterpillar cells because their nuclear material is clumped in a
wavy form. Conditions it can be seen in are:
• Rheumatic Fever
• Recurrent aphthous stomatitis
• Iron deficiency anaemia
• Megaloblastic anaemia
• Sickle cell anaemia
Rheumatic Fever
Recurrent aphthous stomatitis
Iron deficiency anaemia
Megaloblastic anaemia
Sickle cell anaemia
Incorrect Options:
Option A - Infective endocarditis:
• Bacterial endocarditis is an infection of the endocardial surface of the heart.
• The following pathogens mostly cause it: S. Aures (acute) Viridans streptococci (sub-acute)
• S. Aures (acute)
• Viridans streptococci (sub-acute)
• It causes the following signs/symptoms: Fever New murmur Roth spots Osler nodes Splinter
haemorrhages Janeway lesions
• Fever
• New murmur
• Roth spots

Page 29

593
• Osler nodes
• Splinter haemorrhages
• Janeway lesions
• The Mitral valve is frequently involved.
• Tricuspid valve endocarditis is associated with IV drug abuse.
• IV drug abuse is more commonly associated with infective endocarditis (IE); S. aureus is the major
offender in infective endocarditis among intravenous drug abusers.
Bacterial endocarditis is an infection of the endocardial surface of the heart.
The following pathogens mostly cause it:
• S. Aures (acute)
• Viridans streptococci (sub-acute)
S. Aures (acute)
Viridans streptococci (sub-acute)
It causes the following signs/symptoms:
• Fever
• New murmur
• Roth spots
• Osler nodes
• Splinter haemorrhages
• Janeway lesions
Fever
New murmur
Roth spots
Osler nodes
Splinter haemorrhages
Janeway lesions
The Mitral valve is frequently involved.
Tricuspid valve endocarditis is associated with IV drug abuse.
IV drug abuse is more commonly associated with infective endocarditis (IE); S. aureus is the major offe
nder in infective endocarditis among intravenous drug abusers.
Option B - Viral myocarditis:
• Inflammation of the heart muscle is called myocarditis.
• It causes a decrease in the heart's ability to pump blood and results in the following: Chest pain
Shortness of breath Irregular heart rhythms
• Chest pain
• Shortness of breath

Page 30

594
• Irregular heart rhythms
• It is mostly caused by the following: Coxsackie virus Parvovirus B-19 Human herpes virus 6
• Coxsackie virus
• Parvovirus B-19
• Human herpes virus 6
• Biopsy reveals cellular infiltrates, which are mononuclear and histiocytic, with or without associated
myocyte damage.
• Variants include eosinophilic, granulomatous and giant-cell myocarditis.
Inflammation of the heart muscle is called myocarditis.
It causes a decrease in the heart's ability to pump blood and results in the following:
• Chest pain
• Shortness of breath
• Irregular heart rhythms
Chest pain
Irregular heart rhythms
It is mostly caused by the following:
• Coxsackie virus
• Parvovirus B-19
• Human herpes virus 6
Coxsackie virus
Parvovirus B-19
Human herpes virus 6
Biopsy reveals cellular infiltrates, which are mononuclear and histiocytic, with or without associated my
ocyte damage.
Variants include eosinophilic, granulomatous and giant-cell myocarditis.
Option D - Infectious mononucleosis:
• The Epstein-Barr virus is a type of herpes virus called herpes virus 4.
• It mainly spreads through saliva and body fluids.
• Mostly asymptomatic and causes infectious mononucleosis in young adults.
• It causes the following signs/symptoms: Sore throat and swelling Fatigue Fever Swollen lymph nodes
Rash on skin Enlarged spleen and liver
• Sore throat and swelling
• Fatigue
• Fever
• Swollen lymph nodes
• Rash on skin

Page 31

595
• Enlarged spleen and liver
• It affects the body and remains inactive. The activation is caused by the following: Stress Weak
immune system Menopause or hormone changes
• Stress
• Weak immune system
• Menopause or hormone changes
The Epstein-Barr virus is a type of herpes virus called herpes virus 4.
It mainly spreads through saliva and body fluids.
Mostly asymptomatic and causes infectious mononucleosis in young adults.
• Sore throat and swelling
• Fatigue
• Fever
• Swollen lymph nodes
• Rash on skin
• Enlarged spleen and liver
Sore throat and swelling
Swollen lymph nodes
Rash on skin
Enlarged spleen and liver
It affects the body and remains inactive. The activation is caused by the following:
• Stress
• Weak immune system
• Menopause or hormone changes
Stress
Weak immune system
Menopause or hormone changes

Solution for Question 11:


Correct Option A - Rheumatic heart disease:
• Rheumatic heart disease is a condition that can result from untreated or inadequately treated
streptococcal throat infections (strep throat).
• The inflammatory response triggered by the bacterial infection can lead to damage and scarring of the
heart valves, particularly the mitral valve, causing stenosis over time.
Rheumatic heart disease is a condition that can result from untreated or inadequately treated streptoco
ccal throat infections (strep throat).

Page 32

596
The inflammatory response triggered by the bacterial infection can lead to damage and scarring of the
heart valves, particularly the mitral valve, causing stenosis over time.
Incorrect Options:
Option B - Infective-endocarditis:
• While infective endocarditis can affect heart valves, including the mitral valve, it is not the most
common cause of mitral stenosis.
• Infective endocarditis is often associated with valve regurgitation rather than stenosis.
While infective endocarditis can affect heart valves, including the mitral valve, it is not the most commo
n cause of mitral stenosis.
Infective endocarditis is often associated with valve regurgitation rather than stenosis.
Option C - Diabetes mellitus:
• Diabetes mellitus shows an increased risk of left-sided heart valve diseases.
Diabetes mellitus shows an increased risk of left-sided heart valve diseases.
Option D - Congenital:
• Congenital mitral valve stenosis occurs in patients at birth with stiffening leaflets and narrowing of the
mitral orifice.
• It is not the most common cause of mitral valve stenosis.
Congenital mitral valve stenosis occurs in patients at birth with stiffening leaflets and narrowing of the
mitral orifice.
It is not the most common cause of mitral valve stenosis.

Solution for Question 12:


Correct Option C - Small and firm:
• Libman-sacks endocarditis is a form of non-bacterial thrombotic endocarditis.
• It most commonly affects the mitral and aortic valves.
• The vegetation has the following characteristics: Small and firm Friable Tan masses Broad base
• Small and firm
• Friable
• Tan masses
• Broad base
Libman-sacks endocarditis is a form of non-bacterial thrombotic endocarditis.
It most commonly affects the mitral and aortic valves.
The vegetation has the following characteristics:
• Small and firm
• Friable
• Tan masses

Page 33

597
• Broad base
Small and firm
Friable
Tan masses
Broad base

Incorrect Options:
Option A - Infective:
• These vegetations are sterile.
• Infective vegetation is commonly seen in bacterial endocarditis.
These vegetations are sterile.
Infective vegetation is commonly seen in bacterial endocarditis.
Option B - Flat appearance:
• These vegetations give a warty (verrucous) appearance.
• It is also known as verrucous endocarditis.
These vegetations give a warty (verrucous) appearance.
It is also known as verrucous endocarditis.
Option D - Embolism is not common:
• Embolism is common in the case of Libman-sacks endocarditis (NBTE)
• Therefore this option is incorrect
Embolism is common in the case of Libman-sacks endocarditis (NBTE)
Therefore this option is incorrect

Page 34

598
Solution for Question 13:
Correct Option C- Staphylococcus aureus :
• Given the patient's history of intravenous drug abuse, high-grade fever, dyspnea, and the presence of
new pan systolic murmur best heard over the cardiac apex, along with characteristic skin findings such
as Osler's nodes and Janeway lesions, the most likely diagnosis is Infective Endocarditis.
• Staphylococcus aureus is commonly associated with infective endocarditis in intravenous drug
abusers
Given the patient's history of intravenous drug abuse, high-grade fever, dyspnea, and the presence of
new pan systolic murmur best heard over the cardiac apex, along with characteristic skin findings such
as Osler's nodes and Janeway lesions, the most likely diagnosis is Infective Endocarditis.
Staphylococcus aureus is commonly associated with infective endocarditis in intravenous drug abusers
Incorrect Options:
Option A - Staphylococcus epidermidis:
• While Staphylococcus epidermidis is a common cause of infective endocarditis, it is typically
associated with healthcare-associated infections, particularly prosthetic valve endocarditis or other
indwelling devices.
• In this case, the patient's history of intravenous drug abuse suggests a higher likelihood of
community-acquired infective endocarditis caused by Staphylococcus aureus.
While Staphylococcus epidermidis is a common cause of infective endocarditis, it is typically associate
d with healthcare-associated infections, particularly prosthetic valve endocarditis or other indwelling de
vices.
In this case, the patient's history of intravenous drug abuse suggests a
higher likelihood of community-acquired infective endocarditis caused by Staphylococcus aureus.
Option B -Streptococcus viridans:
• Streptococcus viridans is a common cause of subacute infective endocarditis, often associated with
dental procedures or underlying dental pathology.
• It is less commonly implicated in acute IE or intravenous drug abuse-related endocarditis
Streptococcus viridans is a common cause of subacute infective endocarditis, often associated with de
ntal procedures or underlying dental pathology.
It is less commonly implicated in acute IE or intravenous drug abuse-related endocarditis
Option D - Enterococcus faecalis:
• Enterococcus faecalis is a less common cause of infective endocarditis compared to Staphylococcus
aureus, particularly in the context of intravenous drug abuse.
• Enterococcal endocarditis is more commonly associated with underlying gastrointestinal or
genitourinary tract pathology.
Enterococcus faecalis is a less common cause of infective endocarditis compared to Staphylococcus a
ureus, particularly in the context of intravenous drug abuse.
Enterococcal endocarditis is more commonly associated with underlying gastrointestinal or genitourinar
y tract pathology.

Page 35

599
Solution for Question 14:
Correct Option D - Nonbacterial thrombotic endocarditis:
• Non-bacterial thrombotic endocarditis, also known as marantic endocarditis, refers to sterile
vegetation in seemingly normal heart valves.
• The image shows the sterile vegetation with a warty appearance.
• Almost always associated with cancer or some other debilitating disease.
• The cause of endocarditis marantic has been little researched but is attributed to increased blood
coagulation and deposition of immune complexes.
• The main danger is embolization in distant organs.
Non-bacterial thrombotic endocarditis, also known as marantic endocarditis, refers to sterile vegetation
in seemingly normal heart valves.
The image shows the sterile vegetation with a warty appearance.
Almost always associated with cancer or some other debilitating disease.
The cause of endocarditis marantic has been little researched but is attributed to increased blood coag
ulation and deposition of immune complexes.
The main danger is embolization in distant organs.
Incorrect Options:
Option A - Calcific aortic stenosis:
• Calcific aortic stenosis is the most prevalent heart valve disorder in developed countries.
• It is characterized by progressive fibro-calcific remodelling and thickening of the aortic valve leaflets
that, over the years, have evolved to cause severe obstruction to cardiac outflow.
• The image is consistent with Non-bacterial thrombotic endocarditis
Calcific aortic stenosis is the most prevalent heart valve disorder in developed countries.
It is characterized by progressive fibro-calcific remodelling and thickening of the aortic valve leaflets tha
t, over the years, have evolved to cause severe obstruction to cardiac outflow.
The image is consistent with Non-bacterial thrombotic endocarditis
Option B - Carcinoid heart disease:
• It is a form of neuroendocrine tumour and carcinoid syndrome.
• It occurs in advanced form.
• It involves the right-sided heart walls and eventually leads to right heart failure.
• The image is consistent with Non-bacterial thrombotic endocarditis.
It is a form of neuroendocrine tumour and carcinoid syndrome.
It occurs in advanced form.
It involves the right-sided heart walls and eventually leads to right heart failure.
The image is consistent with Non-bacterial thrombotic endocarditis.
Option C - Cardiac metastases:
• Tumours can reach the heart via blood, lymph, transvenous, or directly.

Page 36

600
• These typically involve the pericardium, and can cause arrhythmias and heart failure.
• The image is consistent with Non-bacterial thrombotic endocarditis.
Tumours can reach the heart via blood, lymph, transvenous, or directly.
These typically involve the pericardium, and can cause arrhythmias and heart failure.

Solution for Question 15:


Correct Option D - Tricuspid insufficiency:
• In patients with a history of intravenous (IV) drug abuse, the tricuspid valve is the most common site of
involvement in cases of infective endocarditis. This is primarily due to the direct introduction of bacteria
into the bloodstream through contaminated needles or drug paraphernalia during IV drug use. The
tricuspid valve is particularly susceptible to infection in this population because the right-sided heart
structures, including the tricuspid valve, are exposed to a higher concentration of contaminants in the
drugs from venous circulation. (Contaminants in the drugs damage the endocardium, leading to
seeding of these damaged valves
• Bacterial endocarditis is an infection of the endocardial surface of the heart.
• The following pathogens mostly cause it: S. Aures (acute) Viridans streptococci (sub-acute)
• S. Aures (acute)
• Viridans streptococci (sub-acute)
• It causes the following signs/symptoms: Fever New murmur Roth spots Osler nodes Splinter
hemorrhages Janeway lesions
• Fever
• New murmur
• Roth spots
• Osler nodes
• Splinter hemorrhages
• Janeway lesions
In patients with a history of intravenous (IV) drug abuse, the tricuspid valve is the most common site of
involvement in cases of infective endocarditis. This is primarily due to the direct introduction of bacteria
into the bloodstream through contaminated needles or drug paraphernalia during IV drug use. The tric
uspid valve is particularly susceptible to infection in this population because the right-sided heart struct
ures, including the tricuspid valve, are exposed to a higher concentration of contaminants in the drugs f
rom venous circulation. (Contaminants in the drugs damage the endocardium, leading to seeding of th
ese damaged valves
Bacterial endocarditis is an infection of the endocardial surface of the heart.
The following pathogens mostly cause it:
• S. Aures (acute)
• Viridans streptococci (sub-acute)
S. Aures (acute)

Page 37

601
Viridans streptococci (sub-acute)
It causes the following signs/symptoms:
• Fever
• New murmur
• Roth spots
• Osler nodes
• Splinter hemorrhages
• Janeway lesions
Fever
New murmur
Roth spots
Osler nodes
Splinter hemorrhages
Janeway lesions
• IV drug abuse is more commonly associated with infective endocarditis (IE); S. aureus is the major
offender in infective endocarditis among intravenous drug abusers.
IV drug abuse is more commonly associated with infective endocarditis (IE); S. aureus is the major offe
nder in infective endocarditis among intravenous drug abusers.
• Options A, B and C are incorrect.

Solution for Question 16:


Correct Option A - Bacterial endocarditis:
• In this case, the mitral valve has destructive vegetation that has been eroded through the free edges
of the valve leaflets.
• Vegetations in bacterial endocarditis form on the atrial side of the atrioventricular valves and the
ventricular side of the semilunar valves, often at the occlusion points of the cusps or cusps.

• They are made up of platelets, fibrin, cell debris, and masses of organisms.
• The underlying valve tissue is damaged, so the leaflet is perforated and regurgitated.
• The disease begins with nonspecific symptoms such as low fever, fatigue, anorexia, and weight loss.
In this case, the mitral valve has destructive vegetation that has been eroded through the free edges of
the valve leaflets.
Vegetations in bacterial endocarditis form on the atrial side of the atrioventricular valves and the ventric
ular side of the semilunar valves, often at the occlusion points of the cusps or cusps.

Page 38

602
They are made up of platelets, fibrin, cell debris, and masses of organisms.
The underlying valve tissue is damaged, so the leaflet is perforated and regurgitated.
The disease begins with nonspecific symptoms such as low fever, fatigue, anorexia, and weight loss.
Incorrect Options:
Option B - Carcinoid heart disease:
• Carcinoid heart disease affects the right side of the heart and causes tricuspid regurgitation and
pulmonary stenosis.
• It occurs in the advanced form of carcinoid syndrome.
• This syndrome occurs due to a carcinoid tumor that secretes serotonin and other chemicals into the
bloodstream.
• It involves the right-sided heart walls and eventually leads to right heart failure.
Carcinoid heart disease affects the right side of the heart and causes tricuspid regurgitation and pulmo
nary stenosis.
It occurs in the advanced form of carcinoid syndrome.
This syndrome occurs due to a
carcinoid tumor that secretes serotonin and other chemicals into the bloodstream.
It involves the right-sided heart walls and eventually leads to right heart failure.
Option C - Libman-Sacks endocarditis:
• Libman-sacks endocarditis is a form of non-bacterial thrombotic endocarditis.
• It most commonly affects the mitral and aortic valves.
• The vegetation has the following characteristics: Small and firm Friable Tan masses Broad base
• Small and firm
• Friable
• Tan masses

Page 39

603
• Broad base
• The lesions are small (1 to 4 mm in diameter), single or multiple, sterile, pink vegetations with a warty
(verrucous) appearance.
• They may be located on the undersurfaces of the atrioventricular valves, on the valvular endocardium,
on the chords, or on the mural endocardium of the atria or ventricles.
• Vegetations are small, sterile, and firm and commonly affect the valve's lower surface.
• Hypercoagulable states are the usual precursor to NBTE.
• These include APLA syndrome, SLE, chronic disseminated intravascular coagulation,
hyperestrogenic states, and those associated with underlying malignancy, particularly mucinous
adenocarcinomas.
Libman-sacks endocarditis is a form of non-bacterial thrombotic endocarditis.
It most commonly affects the mitral and aortic valves.
The vegetation has the following characteristics:
• Small and firm
• Friable
• Tan masses
• Broad base
Small and firm
Friable
Tan masses
Broad base
The lesions are small (1 to 4 mm in diameter), single or multiple, sterile, pink vegetations with a
warty (verrucous) appearance.
They may be located on the undersurfaces of the atrioventricular valves, on the valvular endocardium,
on the chords, or on the mural endocardium of the atria or ventricles.
Vegetations are small, sterile, and firm and commonly affect the valve's lower surface.
Hypercoagulable states are the usual precursor to NBTE.
These include APLA syndrome, SLE, chronic disseminated intravascular coagulation, hyperestrogenic
states, and those associated with underlying malignancy, particularly mucinous adenocarcinomas.
Option D - Mitral valve prolapse:
• Mitral valve prolapse and congenital heart defects are the most common causes of bacterial
endocarditis in adults today.
• It is a type of valvular heart disease in which the valve between the left atrium and left ventricle does
not close completely, resulting in a backward blood flow.
• Mitral valve prolapse is the most common cause of mitral regurgitation.
• It results in the following: Arrhythmia Heart murmur Dyspnea Palpitations edema
• Arrhythmia
• Heart murmur

Page 40

604
• Dyspnea
• Palpitations
• edema
Mitral valve prolapse and congenital heart defects are the most common causes of bacterial endocardit
is in adults today.
It is a type of valvular heart disease in which the valve between the left atrium and left ventricle does n
ot close completely, resulting in a backward blood flow.
Mitral valve prolapse is the most common cause of mitral regurgitation.
It results in the following:
• Arrhythmia
• Heart murmur
• Dyspnea
• Palpitations
• edema
Arrhythmia
Heart murmur
Dyspnea
Palpitations
edema

Solution for Question 17:


Correct Option A - Libman-Sacks endocarditis:
• Libman-Sacks endocarditis is a condition often seen in patients with systemic lupus erythematosus
(SLE). It involves the development of sterile vegetation on the heart valves, particularly affecting the
mitral and aortic valves. These vegetations can lead to valvular dysfunction and the occurrence of
murmurs. In the scenario described, the patient has a known history of SLE and presents with
symptoms suggestive of cardiac involvement, such as fatigue and shortness of breath. The presence of
a mid-diastolic murmur heard best at the apex, along with echocardiographic evidence of vegetations
on the mitral valve leaflets, strongly indicates Libman-Sacks endocarditis as the cause of the murmur.
Libman-Sacks endocarditis is a condition often seen in patients with systemic lupus erythematosus (SL
E). It involves the development of sterile vegetation on the heart valves, particularly affecting the mitral
and aortic valves. These vegetations can lead to valvular dysfunction and the occurrence of murmurs.
In the scenario described, the patient has a known history of SLE and presents with symptoms suggest
ive of cardiac involvement, such as fatigue and shortness of breath. The presence of a mid-diastolic m
urmur heard best at the apex, along with echocardiographic evidence of vegetations on the mitral valve
leaflets, strongly indicates Libman-Sacks endocarditis as the cause of the murmur.
Incorrect Options:
Option B - Mitral valve prolapse:

Page 41

605
• Mitral valve prolapse is a condition characterized by the abnormal movement (ballooning of the mitral
valve leaflets during systole, often resulting in a systolic click and murmur. However, in this case, the
murmur is described as mid-diastolic, which is not characteristic of mitral valve prolapse.
Mitral valve prolapse is a condition characterized by the abnormal movement (ballooning of the mitral v
alve leaflets during systole, often resulting in a systolic click and murmur. However, in this case, the mu
rmur is described as mid-diastolic, which is not characteristic of mitral valve prolapse.
Option C - Myocardial infarct:
• Myocardial infarction typically presents with symptoms such as chest pain, and any associated
murmurs would depend on the extent and location of the infarct, which is not specified in this scenario.
Myocardial infarction typically presents with symptoms such as chest pain, and any associated murmur
s would depend on the extent and location of the infarct, which is not specified in this scenario.
Option D - Rheumatic fever:
• Rheumatic fever can lead to valvular damage, particularly affecting the mitral valve, but the clinical
presentation described, including the mid-diastolic murmur and echocardiographic findings of
vegetation more consistent with Libman-Sacks endocarditis in the setting of SLE.
Rheumatic fever can lead to valvular damage, particularly affecting the mitral valve, but the clinical pre
sentation described, including the mid-diastolic murmur and echocardiographic findings of vegetation
more consistent with Libman-Sacks endocarditis in the setting of SLE.

Solution for Question 18:


Correct Option B - Sub-acute infective endocarditis:
• Sub-acute bacterial endocarditis is an infection of the endocardial surface of the heart.
• The following pathogens mostly cause it: Viridans streptococci (sub-acute)
• Viridans streptococci (sub-acute)
• It causes the following signs/symptoms: Fever New murmur Roth spots Osler nodes Splinter
haemorrhages Janeway lesions
• Fever
• New murmur
• Roth spots
• Osler nodes
• Splinter haemorrhages
• Janeway lesions

• It has a gradual onset.
• The Mitral valve is frequently involved.
• It causes smaller vegetation on congenitally abnormal or disease valves.
Sub-acute bacterial endocarditis is an infection of the endocardial surface of the heart.
The following pathogens mostly cause it:

Page 42

606
• Viridans streptococci (sub-acute)
Viridans streptococci (sub-acute)
It causes the following signs/symptoms:
• Fever
• New murmur
• Roth spots
• Osler nodes
• Splinter haemorrhages
• Janeway lesions

Fever
New murmur
Roth spots
Osler nodes
Splinter haemorrhages
Janeway lesions

Page 43

607
It has a gradual onset.
The Mitral valve is frequently involved.
It causes smaller vegetation on congenitally abnormal or disease valves.
Incorrect Options:
Option A - Acute infective endocarditis:
• Acute Bacterial endocarditis is an infection of the endocardial surface of the heart.
• The following pathogens mostly cause it: S. Aureus (acute)
• S. Aureus (acute)
• It causes the following signs/symptoms: Fever New murmur Roth spots Osler nodes Splinter
haemorrhages Janeway lesions
• Fever
• New murmur
• Roth spots
• Osler nodes
• Splinter haemorrhages
• Janeway lesions
• It has a rapid onset.
• The Mitral valve is frequently involved.
• It has large vegetation on previously normal valves.
Acute Bacterial endocarditis is an infection of the endocardial surface of the heart.
• S. Aureus (acute)
S. Aureus (acute)
• Fever

Page 44

608
• New murmur
• Roth spots
• Osler nodes
• Splinter haemorrhages
• Janeway lesions
It has a rapid onset.
It has large vegetation on previously normal valves.
Option C - Prosthetic valve infective endocarditis:
• Prosthetic valve Endocarditis is an infection of the endocardial surface of the heart.
• The following pathogens mostly cause it: S. Epidermis
• S. Epidermis
• It causes the following signs/symptoms: Fever New murmur Roth spots Osler nodes Splinter
haemorrhages Janeway lesions
• Fever
• New murmur
• Roth spots
• Osler nodes
• Splinter haemorrhages
• Janeway lesions
• Endocarditis occurs within 60 days of valve implantation.
• Mainly occurs due to the contamination of the surgery site.
Prosthetic valve Endocarditis is an infection of the endocardial surface of the heart.
• S. Epidermis
S. Epidermis
• Fever
• New murmur
• Roth spots
• Osler nodes
• Splinter haemorrhages
• Janeway lesions
Endocarditis occurs within 60 days of valve implantation.
Mainly occurs due to the contamination of the surgery site.
Option D - Libman-sacks endocarditis:
• Libman-sacks endocarditis is a form of non-bacterial thrombotic endocarditis.
• It most commonly affects the mitral and aortic valves.
• The vegetation has the following characteristics: Small and firm Friable Tan masses Broad base

Page 45

609
• Small and firm
• Friable
• Tan masses
• Broad base
• The lesions are small (1 to 4 mm in diameter), single or multiple, sterile, pink vegetations with a warty
(verrucous) appearance.
• They may be located on the undersurfaces of the atrioventricular valves, on the valvular endocardium,
on the chords, or the mural endocardium of the atria or ventricles.
• Vegetations are small, sterile, and firm and commonly affect the valve's lower surface.
• Hypercoagulable states are the usual precursor to NBTE.
• These include APLA syndrome, SLE, chronic disseminated intravascular coagulation,
hyperestrogenic states, and those associated with underlying malignancy, particularly mucinous
adenocarcinomas.
Libman-sacks endocarditis is a form of non-bacterial thrombotic endocarditis.
It most commonly affects the mitral and aortic valves.
The vegetation has the following characteristics:
• Small and firm
• Friable
• Tan masses
• Broad base
Small and firm
Friable
Tan masses
Broad base
The lesions are small (1 to 4 mm in diameter), single or multiple, sterile, pink vegetations with a
warty (verrucous) appearance.
They may be located on the undersurfaces of the atrioventricular valves, on the valvular endocardium,
on the chords, or the mural endocardium of the atria or ventricles.
Vegetations are small, sterile, and firm and commonly affect the valve's lower surface.
Hypercoagulable states are the usual precursor to NBTE.
These include APLA syndrome, SLE, chronic disseminated intravascular coagulation, hyperestrogenic
states, and those associated with underlying malignancy, particularly mucinous adenocarcinomas.

Solution for Question 19:


Correct Option A - Constrictive pericarditis:
• Constrictive pericarditis is a chronic fibrosing disease of the pericardium that compresses the heart
and restricts its flow.

Page 46

610
• It is the result of an exuberant healing response after an acute pericardial injury in which the
pericardial space is cleared and the visceral and parietal layers of the pericardium fuse into a dense
mass of fibrous tissue.
• Bacteria, viruses or fungi can cause pericarditis.
• Active tuberculosis (as in this case) is a leading cause of this disease in developing countries.
• The image shows the heart is encased in dense fibrous tissue.
• Radiation therapy in front of the mediastinum and heart surgery make up one-third of the cases. In
other instances, constrictive pericarditis is the result of a chronic infection.
• Signs of constrictive pericarditis include dyspnea, fatigue, chest pain, swelling of the feet due to fluid
buildup and dizziness.
• Beck's triad includes distant or muffled heart sounds, elevated jugular venous pressure, and
peripheral edema.
• Treatment involves surgical resection of the constricting fibrous tissue, the pericardiectomy. Medical
management can be done depending on the cause with non-steroidal anti-inflammatory drugs
(NSAIDs), steroids are necessary, diuretics for fluid overload and antibiotics for bacterial infections
Constrictive pericarditis is a
chronic fibrosing disease of the pericardium that compresses the heart and restricts its flow.
It is the result of an exuberant healing response after an acute pericardial injury in which the pericardial
space is cleared and the visceral and parietal layers of the pericardium fuse into a
dense mass of fibrous tissue.
Bacteria, viruses or fungi can cause pericarditis.
Active tuberculosis (as in this case) is a leading cause of this disease in developing countries.
The image shows the heart is encased in dense fibrous tissue.
Radiation therapy in front of the mediastinum and heart surgery make up one-third of the cases. In oth
er instances, constrictive pericarditis is the result of a chronic infection.
Signs of constrictive pericarditis include dyspnea, fatigue, chest pain, swelling of the feet due to fluid bu
ildup and dizziness.
Beck's triad includes distant or muffled heart sounds, elevated jugular venous pressure, and peripheral
edema.
Treatment involves surgical resection of the constricting fibrous tissue, the pericardiectomy. Medical m
anagement can be done depending on the cause with non-steroidal anti-inflammatory drugs (NSAIDs),
steroids are necessary, diuretics for fluid overload and antibiotics for bacterial infections
Incorrect Options:
Option B - Infective endocarditis:
• The image above demonstrates constrictive pericarditis in which dense fibrous tissue surrounding the
heart affects the pericardium, not the endocardium.
The image above demonstrates constrictive pericarditis in which dense fibrous tissue surrounding the h
eart affects the pericardium, not the endocardium.
Option C - Restrictive cardiomyopathy:
• It is a type of cardiomyopathy in which there is systolic dysfunction causing impaired contraction; the
above image demonstrates constrictive pericarditis.

Page 47

611
It is a type of cardiomyopathy in which there is systolic dysfunction causing impaired contraction; the a
bove image demonstrates constrictive pericarditis.
Option D - Viral myocarditis:
• Viral myocarditis involves the myocardium and not the pericardium. The above image demonstrates
constrictive pericarditis.
Viral myocarditis involves the myocardium and not the pericardium. The above image demonstrates co
nstrictive pericarditis.

Solution for Question 20:


Correct Option A - Myxomatous degeneration and prolapse of the mitral valve:
• Mitral valve prolapse (MVP): MVP involves one or both leaflets of the mitral valve, floppy and prolapse
into the left atrium during the systole. It is most common in females with Male to female ratio of 7:1. It is
usually an incidental finding on examination. Although the exact mechanism causing the weakening of
valve leaflets is unknown, sometimes it is associated with hereditary connective tissue disorders such
as Marfan's syndrome. It can also occur as a complication of rheumatic fever. Most individuals with
MVP are asymptomatic, and it is usually an incidental finding on examination. Auscultation reveals
mid-systolic clicks, followed by a mid to late systolic murmur occasionally. A small percentage of
individuals may have dyspnea due to valvular insufficiency. The fundamental histologic change in the
tissue is marked thickening of the leaflets with the deposition of mucoid (myxomatous) material, called
myxomatous degeneration The echocardiogram and the clinical presentation confirm the diagnosis of
MVP. Treatment of MVP in symptomatic patients involves Valve repair or replacement surgery.
Although the complications from MVP are not seen in most cases, A minority of individuals may suffer
from severe complications such as infective endocarditis, mitral insufficiency, stroke and arrhythmias
• MVP involves one or both leaflets of the mitral valve, floppy and prolapse into the left atrium during the
systole.
• It is most common in females with Male to female ratio of 7:1. It is usually an incidental finding on
examination.
• Although the exact mechanism causing the weakening of valve leaflets is unknown, sometimes it is
associated with hereditary connective tissue disorders such as Marfan's syndrome. It can also occur as
a complication of rheumatic fever.
• Most individuals with MVP are asymptomatic, and it is usually an incidental finding on examination.
Auscultation reveals mid-systolic clicks, followed by a mid to late systolic murmur occasionally. A small
percentage of individuals may have dyspnea due to valvular insufficiency.
• The fundamental histologic change in the tissue is marked thickening of the leaflets with the
deposition of mucoid (myxomatous) material, called myxomatous degeneration
• The echocardiogram and the clinical presentation confirm the diagnosis of MVP.
• Treatment of MVP in symptomatic patients involves Valve repair or replacement surgery.
• Although the complications from MVP are not seen in most cases, A minority of individuals may suffer
from severe complications such as infective endocarditis, mitral insufficiency, stroke and arrhythmias
Mitral valve prolapse (MVP):
• MVP involves one or both leaflets of the mitral valve, floppy and prolapse into the left atrium during the
systole.

Page 48

612
• It is most common in females with Male to female ratio of 7:1. It is usually an incidental finding on
examination.
• Although the exact mechanism causing the weakening of valve leaflets is unknown, sometimes it is
associated with hereditary connective tissue disorders such as Marfan's syndrome. It can also occur as
a complication of rheumatic fever.
• Most individuals with MVP are asymptomatic, and it is usually an incidental finding on examination.
Auscultation reveals mid-systolic clicks, followed by a mid to late systolic murmur occasionally. A small
percentage of individuals may have dyspnea due to valvular insufficiency.
• The fundamental histologic change in the tissue is marked thickening of the leaflets with the
deposition of mucoid (myxomatous) material, called myxomatous degeneration
• The echocardiogram and the clinical presentation confirm the diagnosis of MVP.
• Treatment of MVP in symptomatic patients involves Valve repair or replacement surgery.
• Although the complications from MVP are not seen in most cases, A minority of individuals may suffer
from severe complications such as infective endocarditis, mitral insufficiency, stroke and arrhythmias
MVP involves one or both leaflets of the mitral valve, floppy and prolapse into the left atrium during the
systole.
It is most common in females with Male to female ratio of 7:1. It is usually an incidental finding on exam
ination.
Although the exact mechanism causing the weakening of valve leaflets is unknown, sometimes it is ass
ociated with hereditary connective tissue disorders such as Marfan's syndrome. It can also occur as a
complication of rheumatic fever.
Most individuals with MVP are asymptomatic, and it is usually an incidental finding on examination. Au
scultation reveals mid-systolic clicks, followed by a mid to late systolic murmur occasionally. A
small percentage of individuals may have dyspnea due to valvular insufficiency.
The fundamental histologic change in the tissue is marked thickening of the leaflets with the deposition
of mucoid (myxomatous) material, called myxomatous degeneration
The echocardiogram and the clinical presentation confirm the diagnosis of MVP.
Treatment of MVP in symptomatic patients involves Valve repair or replacement surgery.
Although the complications from MVP are not seen in most cases, A minority of individuals may suffer f
rom severe complications such as infective endocarditis, mitral insufficiency, stroke and arrhythmias

Page 49

613
• A. The arrow shows the prolapsed leaflet of the mitral valve into the left atrium
A. The arrow shows the prolapsed leaflet of the mitral valve into the left atrium

Incorrect Options:
Option B - Fibrinous deposition on the tip of papillary muscle:
• Mid-systolic click heard on auscultation is specific for MVP. Myxomatous degeneration of mitral valve
leaflets is seen in MVP. It does not affect the papillary muscle.
Mid-systolic click heard on auscultation is specific for MVP. Myxomatous degeneration of mitral valve l
eaflets is seen in MVP. It does not affect the papillary muscle.
Option C - Rupture of chordae tendinae:
• Rupture of chordae tendineae is seen in severe cases of mitral regurgitation. MVP causes
Myxomatous degeneration of the mitral valve leaflets.

Page 50

614
Rupture of chordae tendineae is seen in severe cases of mitral regurgitation. MVP causes Myxomatou
s degeneration of the mitral valve leaflets.
Option D - Aschoff nodule on the mitral valve:
• Aschoff nodules on the mitral valve are seen in rheumatic heart disease. MVP causes Myxomatous
degeneration of the mitral valve leaflets.
Aschoff nodules on the mitral valve are seen in rheumatic heart disease. MVP causes Myxomatous de
generation of the mitral valve leaflets.

Page 51

615
Previous Year Questions
1. During an autopsy, which staining techniques can be employed to visualize the infarct in the heart of
a 65-year-old male who passed away as a result of myocardial infarction?
A. Oil red O
B. Triphenyl tetrazolium chloride
C. Sudan black B
D. Masson trichrome
----------------------------------------
2. A 34-year-old female patient presented to the outpatient department with migratory arthritis, and
examination revealed a pan-systolic murmur. An echocardiography was performed, which was
suggestive of mitral regurgitation. A biopsy specimen was obtained, and the image is provided below.
What is the most likely lesion shown in the histopathology slide?

A. Aschoff’s bodies
B. Epithelioid granuloma
C. Granulomatous vasculitis
D. Granuloma inguinale
----------------------------------------
3. Large, friable, irregular vegetation on heart valves is seen in which condition?
A. Infective endocarditis
B. Rheumatic heart disease
C. Libman-Sacks endocarditis
D. Non-bacterial thrombotic carditis
----------------------------------------
4. A 3-year-old child has been brought to the hospital with complaints of fever, rash, and swelling of her
hands and feet for the past 5 days. There is desquamative skin rash, oedema of hands and feet,
erythema of palms and soles, oral erythema, and cervical lymphadenopathy. What is the likely
condition?
A. Kawasaki disease
B. Measles

616
C. Scarlet fever
D. Henoch-Schonlein purpura
----------------------------------------
5. How is thrombosis initiated?
A. Platelet activation
B. Endothelial injury
C. Coagulation cascade
D. Vasoconstriction of vessels
----------------------------------------
6. The histology of the reperfused myocardium shows which of the following features?
A. Wavy fibres
B. Neutrophils in cardiac muscle
C. Swelling of cells
D. Eosinophilc contraction band
----------------------------------------
7. What is the most reliable diagnostic test for identifying myocarditis?
A. Endo-myocardial biopsy
B. Magnetic Resonance Imaging
C. Brain natriuretic Peptide
D. LDH1 to LDH2 ratio
----------------------------------------
8. What is the most probable reason for the sudden death of a 20-year-old athlete who collapsed during
a football match and was later pronounced dead at the hospital, considering the provided autopsy
image?

A. Valvular heart disease due to RHD


B. Sudden cardiac death due to arrhythmia
C. Sudden cardiac death due to Hypertrophic cardiomyopathy
D. Sudden cardiac death due to atherosclerosis

Page 2

617
----------------------------------------
9. The patient reported experiencing difficulty breathing during physical activity for a period of one
month. An echocardiogram revealed the presence of mitral stenosis and left ventricular hypertrophy.
Please identify the histopathological finding mentioned and provide the likely diagnosis.

A. Aschoff body; Rheumatic heart disease


B. Calcification of valve; Mitral stenosis
C. Granuloma; Sarcoidosis
D. Non – caseating granuloma; Tuberculosis
----------------------------------------
10. What is the most common cause of berry aneurysm?
(or)
What is the most common cause of berry aneurysm?
A. Endothelial injury of vessel due to HTN
B. Muscle intimal elastic lamina layer defect
C. Muscle and adventitial layer defect
D. Adventitia defect
----------------------------------------
11. Which of the following is not observed in Tumor lysis Syndrome?
A. Hypophosphatemia
B. Hypocalcemia
C. Hyperuricemia
D. Hyperkalemia
----------------------------------------
12. What is the most accurate indicator for myocardial infarction?
A. Troponin
B. Cytokeratin
C. Myoglobin
D. CPK-MM

Page 3

618
----------------------------------------
13. How can the diagnosis of Group A streptococcal infection be determined in cases of Acute
Rheumatic Fever?
A. ASO
B. CRP
C. ESR elevation
D. Low C3 levels
----------------------------------------
14. What is the histopathological finding not observed in the heart 12 hours after an ischemic injury?
A. Karyorrhexis of myocytes
B. Neocapillary invasion of myocytes
C. Hyper-eosinophilia of myocytes
D. Coagulation necrosis of myocytes
----------------------------------------
15. Which of the following is not a small round blue cell tumor?
A. Hemangioblastoma
B. Ewing’s sarcoma
C. Retinoblastoma
D. Neuroblastoma
----------------------------------------
16. Berry aneurysm occurs due to?
A. Endothelial injury of vessel due to HTN
B. Arterial internal elastic lamina defect
C. Endothelial layer defect
D. Adventitia defect
----------------------------------------
17. Match the following about the endothelial contraction : Column A Column B Sunburn Bee sting
Bacterial enzymes Tumor necrosis factor A. Immediate sustained B. Delayed prolonged C. Endothelial
cell retraction D. Immediate transient
A Column B
Bee stingBacterial enzymesTumor necrosis factor A. Immediate sustainedB. Delayed prolongedC. Endothelial cell retractionD. Immediate

A. 1-B, 2-D, 3-A, 4-C


B. 1-B, 2-D, 3-C, 4-A
C. 1-D, 2-B, 3-A, 4-C
D. 1-B, 2-A, 3-D, 4-C
----------------------------------------

Page 4

619
18. Match the following: Column A Column B 1. Boxcar nuclei 2. Myocyte hypertrophy 3. Myocyte
vacuolization 4. Myocyte disarray A. HOCM B. Hpertension C. DCMP D. Subendocardial ischemia E.
Hypersensitivity myocarditis
Column B
yocyte hypertrophy3. Myocyte vacuolization4. Myocyte disarray A. HOCMB. HpertensionC. DCMPD. Subendocardial ischemiaE. Hyperse

A. 1-C, 2-B; 3-A; 4-D


B. 1-C, 2-B; 3-E; 4-A
C. 1-E, 2-B; 3-C; 4-A
D. 1-C, 2-D; 3-E; 4-A
----------------------------------------
19. A 5-year-old child is brought in with fever, fatigue, and left ventricular dysfunction. An
endomyocardial biopsy is shown below. What is the probable diagnosis?

A. Acute rheumatic fever


B. Chagas cardiomyopathy
C. Pyogenic myocarditis
D. Lymphocytic myocarditis
----------------------------------------
20. Which of the following statements is true regarding the modified Duke’s criteria?
A. Single positive blood culture for HACEK group is a major criterion
B. Single positive blood culture for Coxiella burnetti is a major criterion
C. Complete dehiscence of prosthetic valve
D. Documentation of four minor criteria allows a clinical diagnosis
----------------------------------------
21. Alcohol abuse is strongly associated with the development of which of the following condition?
A. Pericarditis
B. Hypertrophic cardiomyopathy
C. Dilated cardiomyopathy
D. Myocarditis

Page 5

620
----------------------------------------
22. A 47-year-old female patient with a long-standing history of rheumatic heart disease was admitted
to the hospital due to severe breathlessness. She died on the third day following admission. The mitral
valve shows the following findings on autopsy. What is the cause of these findings?

A. Rupture of valve
B. Calcification and fibrosis
C. Hypertrophy of ventricular wall
D. Hypertrophy of the atrial wall
----------------------------------------
23. Which of the following myocardial infarcts causes aneurysm as a post-MI complication?
A. Subendocardial
B. Anterior transmural
C. Posterior transmural
D. Inferior wall
----------------------------------------

Correct Answers
Question Correct Answer

Question 1 2
Question 2 1
Question 3 1
Question 4 1
Question 5 2
Question 6 4
Question 7 1
Question 8 3
Question 9 1
Question 10 2

Page 6

621
Question 11 1
Question 12 1
Question 13 1
Question 14 3
Question 15 1
Question 16 2
Question 17 1
Question 18 3
Question 19 4
Question 20 2
Question 21 3
Question 22 2
Question 23 2

Solution for Question 1:


• Triphenyl tetrazolium chloride is a histochemical stain that can identify an infarct within 2-3 hours after
the onset.
• Non-infarcted myocardium appears brick-red. Infarcted areas are unstained and pale, whereas old
infarcts appear white and glistening.
Incorrect Choices:
• Option a. Oil red O: Helps demonstrate lipids in organs like the liver, skeletal muscle, and heart. In the
liver, it is used in nonalcoholic fatty liver disease or alcoholic liver disease. In skeletal muscle, it is
useful in identifying lipid droplets which are seen in obesity and insulin resistance.
• Option c. Sudan black B: Sudan black B stain is used to identify lipid accumulation in tissues like the
liver, muscle, and brain. It helps in the diagnosis of lipid-containing tumors and fatty liver diseases.
• Option d. Masson trichrome: It is used to identify collagen, smooth muscle, and extracellular matrix. It
is also used to assess the degree of fibrosis that takes place in chronic liver disease. Three dyes are
used in this: hematoxylin, acid fuchsin, and light green. Collagen is stained blue, cell nuclei are stained
dark brown; smooth muscle is stained red, and cytoplasm is stained pink.

Solution for Question 2:


Correct Option A - Aschoff’s bodies:
• The patient's presentation with migratory arthritis, a pan-systolic murmur, and mitral regurgitation is
suggestive of rheumatic heart disease.
• Aschoff's bodies are characteristic histopathological features seen in rheumatic heart disease.
• Aschoff's bodies are focal areas of chronic inflammation within the myocardium.
• They consist of a central focus of fibrinoid necrosis, surrounded by Anitschkow cells.

Page 7

622
Incorrect Options:
Option B - Epithelioid granuloma: Epithelioid granulomas are associated with granulomatous diseases
such as tuberculosis and sarcoidosis.
Option C - Granulomatous vasculitis: Granulomatous vasculitis typically involves inflammation and dam
age to blood vessels.
Option D - Granuloma inguinale: Granuloma inguinale, also known as Donovanosis, is a
sexually transmitted infection characterized by genital ulcers.

Solution for Question 3:


Correct Option A - Infective endocarditis:
• Infective endocarditis is an infection of the endocardium, typically involving heart valves.
• It is characterized by the formation of vegetations, which are friable, irregular growths composed of
fibrin, platelets, and bacteria.
• These vegetations can be large in size and have a tendency to embolize, leading to systemic
complications.
Incorrect Options:
Option B - Rheumatic heart disease:
• Rheumatic heart disease is a condition resulting from rheumatic fever, which is caused by an
autoimmune response to a previous group A Streptococcus infection.
• While rheumatic heart disease can cause valvular abnormalities, such as stenosis or regurgitation, the
findings of large, friable vegetation are more characteristic of infective endocarditis.
Option C - Libman-Sacks endocarditis:
• Libman-Sacks endocarditis, also known as non-bacterial thrombotic endocarditis, is associated with
systemic lupus erythematosus (SLE) or other autoimmune disorders.
• It involves the formation of sterile fibrin-platelet vegetation on heart valves.
• These vegetations are typically smaller and less friable compared to those seen in infective
endocarditis.
Option D - Non-bacterial thrombotic carditis:
• Non-bacterial thrombotic carditis is a condition characterized by the deposition of sterile thrombi on
heart valves.
• It is typically associated with hypercoagulable states, malignancies, or chronic wasting illnesses.
• The thrombi in non-bacterial thrombotic carditis are composed of fibrin and platelets but lack the
inflammatory features and bacterial colonization seen in infective endocarditis.

Solution for Question 4:


Correct Option A - Kawasaki disease:

Page 8

623
• Kawasaki disease, also known as mucocutaneous lymph node syndrome, is an acute systemic
vasculitis that primarily affects children, typically under the age of 5.
• It is characterized by inflammation of medium-sized blood vessels throughout the body.
• It presents with a persistent fever lasting for more than five days, along with other clinical features
such as a desquamative skin rash, edema of the hands and feet, erythema of the palms and soles, oral
erythema, and cervical lymphadenopathy.
Incorrect Options:
• Options B, C, and D do not present with the specific clinical features mentioned in the question.
Option B - Measles: Measles, also known as rubeola, is a highly contagious viral infection.
Option C- Scarlet fever: Scarlet fever is a bacterial infection caused by group A Streptococcus.
Option D - Henoch-Schonlein purpura: Henoch-Schonlein purpura is a
type of vasculitis that affects small blood vessels, primarily seen in children.

Solution for Question 5:


Correct Option B - Endothelial injury:
• Thrombosis is the formation of a thrombus within a blood vessel.
• It is a complex process involving multiple factors, but it is initiated by endothelial injury.
• When the endothelium, the inner lining of blood vessels, is damaged or disrupted, it exposes the
underlying subendothelial components, such as collagen and tissue factor.
• This exposure triggers a series of events leading to platelet adhesion, activation, and the initiation of
the coagulation cascade.
Incorrect Options:
Options A, C, and D do not initiate thrombosis.

Solution for Question 6:


Correct Option D - Eosinophilic contraction band:
• Reperfusion refers to the restoration of blood flow to previously ischemic tissue.
• Eosinophilic contraction bands are bands of densely eosinophilic material that can be seen within
cardiac muscle fibers under microscopic examination.
• These bands are formed due to the sudden influx of calcium ions into the myocardial cells during
reperfusion.
• The increased intracellular calcium levels trigger the contraction of the myofibrils, resulting in the
formation of eosinophilic contraction bands.
• These bands are indicative of cellular injury and are commonly observed in the setting of reperfusion
injuries.
Incorrect Options:

Page 9

624
• Options A, B, and C are not features of repurfused myocardium.
Option A - Wavy fibers:
• They are typically seen in conditions such as hypertrophic cardiomyopathy.
Option B - Neutrophils in cardiac muscle:
• Neutrophils infiltrating cardiac muscle are a feature seen in acute inflammation, such as in acute
myocardial infarction.
Option C - Swelling of cells:
• Cellular swelling can occur in various pathological conditions, including ischemia-reperfusion injury.
• However, it is a nonspecific finding and can be observed in other conditions as well.

Solution for Question 7:


Endo-myocardial biopsy is considered the gold standard investigation for the diagnosis of myocarditis. I
t involves obtaining a small tissue sample from the inner lining of the heart (endocardium) or the heart
muscle (myocardium) to examine it under a microscope. This biopsy allows for direct visualization of th
e cardiac tissue and assessment of cellular inflammation, presence of viral particles, and other histopat
hological changes characteristic of myocarditis.
Myocarditis is an inflammatory condition of the heart muscle, often caused by viral infections but can al
so result from autoimmune reactions or toxic exposures. The clinical presentation of myocarditis can va
ry, ranging from mild symptoms to severe heart failure and life-threatening arrhythmias. Due to the dive
rse clinical manifestations and the need for accurate diagnosis, endo-myocardial biopsy provides valua
ble information for the definitive diagnosis of myocarditis.
Incorrect Choices:
Option b. Magnetic Resonance Imaging (MRI):
• While magnetic resonance imaging (MRI) can provide supportive evidence for the diagnosis of
myocarditis, it is not considered the gold standard investigation. Cardiac MRI can demonstrate
myocardial edema, inflammation, and tissue characterization using specific sequences such as
T2-weighted imaging and late gadolinium enhancement. These findings, along with clinical features,
can support the diagnosis of myocarditis. However, MRI findings alone may not provide definitive
confirmation of the diagnosis, as other conditions can produce similar imaging features.
Option c. Brain natriuretic Peptide (BNP):
• Brain natriuretic peptide (BNP) is a hormone released by the heart in response to increased stretching
of cardiac chambers, typically seen in conditions such as heart failure. While BNP levels can be
elevated in myocarditis due to associated cardiac dysfunction, it is not a specific diagnostic test for
myocarditis. BNP levels can be helpful in assessing cardiac function and determining the severity of
heart failure but do not provide direct evidence of myocarditis.
Option d. LDH1 to LDH2 ratio:
• The LDH1 to LDH2 ratio is not a specific diagnostic test for myocarditis. Lactate dehydrogenase
(LDH) is an enzyme found in various tissues, including the heart. In myocarditis, there may be an
elevation of LDH levels due to myocardial injury. However, the LDH1 to LDH2 ratio is not a specific
marker for myocarditis and does not provide definitive diagnostic information.

Page 10

625
Solution for Question 8:
Correct Option:
Option C. Sudden cardiac death due to Hypertrophic cardiomyopathy (HCM): Hypertrophic cardiomyop
athy is characterized by abnormal heart muscle thickening, particularly in the left ventricle. The autopsy
image shows signs of cardiac hypertrophy, comes out HCM as the cause of death. HCM is a
common cause of sudden cardiac death in young athletes.

Incorrect Options:
Option A. Valvular heart disease due to RHD (Rheumatic Heart Disease): The autopsy image shows n
o significant abnormalities in the heart valves, ruling out valvular heart disease as the cause of death.
Rheumatic heart disease is a condition that results from untreated or inadequately treated streptococca
l infections, leading to inflammation and scarring of the heart valves. Typical findings in RHD include thi
ckened or damaged heart valves, which are not evident in the image.
Option B: Sudden cardiac death due to arrhythmia:
Sudden cardiac death (SCD) is an unexpected and abrupt loss of heart function, usually resulting from
a life-threatening arrhythmia. In athletes, the most common cause of SCD is hypertrophic cardiomyopa
thy (HCM), characterized by abnormal heart muscle thickening.
Option D. Sudden cardiac death due to atherosclerosis: Atherosclerosis is the buildup of plaques in the
arteries, narrowing, and impaired blood flow. While atherosclerosis can lead to heart attacks and sudd
en cardiac death, the given autopsy image does not provide any evidence of significant
plaque formation or obstruction in the coronary arteries.

Solution for Question 9:


Correct Option A:
The histopathological finding shown in the image is an Aschoff body. Aschoff bodies are characteristic
histological features of rheumatic heart disease (RHD), a
condition caused by an autoimmune reaction following Group A Streptococcus infection. Aschoff bodie
s are focal inflammatory lesions typically found in the myocardium, particularly in the left atrium and atri
al appendages.
Histopathological presentation of Aschoff body in rheumatic heart disease:
• Fibrinoid necrosis: Aschoff bodies show central areas of necrosis characterized by fibrinoid material
surrounded by infiltrating immune cells.
• Anitschkow cells: Within the central necrotic area, there are large, plump, activated histiocytes called
Anitschkow cells or Aschoff giant cells. These cells have characteristic elongated, wavy nuclei and are
pathognomonic for rheumatic fever.
• Lymphocytes and plasma cells: Surrounding the necrotic areas, there is an infiltrate of lymphocytes
and plasma cells, indicating an ongoing immune response.
Probable diagnosis:

Page 11

626
Based on the presence of mitral stenosis, left ventricular hypertrophy, and the histopathological finding
of an Aschoff body, the probable diagnosis in this case is rheumatic heart disease. Rheumatic heart dis
ease is a
consequence of rheumatic fever, which typically follows an untreated or inadequately treated Group A
Streptococcus infection, particularly pharyngitis.
Incorrect Option:
Option B. Calcification of valve; Mitral stenosis:
Calcification of the valve is a common finding in mitral stenosis. However, the histopathological image
provided does not show calcification.
Option C. Granuloma; Sarcoidosis:
Sarcoidosis is a
systemic inflammatory disease characterized by the formation of non-caseating granulomas.
Option D. Non-caseating granuloma; Tuberculosis:
Tuberculosis is a granulomatous disease characterized by the presence of non-caseating granulomas.

Solution for Question 10:


Correct Option B.
• Berry aneurysms, also known as saccular aneurysms or intracranial aneurysms, most commonly
occur due to a defect in the muscle intimal elastic lamina layer of the blood vessel wall.
• This defect weakens the vessel wall, leading to the formation of a small outpouching or sac-like
dilation.
• These aneurysms are frequently found in the arteries at the base of the brain, particularly at the
branching points of the major cerebral arteries.
Incorrect options:
Option A. Endothelial injury of vessel due to HTN: While hypertension (HTN) can contribute to the deve
lopment and progression of aneurysms, it is not the most common cause of berry aneurysms. The defe
ct in the muscle intimal elastic lamina layer is the primary factor leading to their formation.
Option C. Muscle and adventitial layer defect: Berry aneurysms primarily involve the defect in the musc
le intimal elastic lamina layer, not the muscle and adventitial layers.
Option D. Adventitia defect: Similar to option C, the defect in the adventitia layer alone is not the main c
ause of berry aneurysms. The defect in the muscle intimal elastic lamina layer is the primary factor.

Solution for Question 11:


Correct Option A:
• Hypophosphatemia, which refers to low levels of phosphate in the blood, is not typically associated
with tumor lysis syndrome (TLS).

Page 12

627
• Tumor lysis syndrome occurs when large numbers of tumor cells are rapidly destroyed, leading to the
release of intracellular contents into the bloodstream.
• The characteristic features of TLS include hyperuricemia (high levels of uric acid), hyperkalemia (high
levels of potassium), and hyperphosphatemia (high levels of phosphate).
• Additionally, TLS can cause hypocalcemia (low levels of calcium) due to the deposition of calcium
phosphate crystals in tissues.
Incorrect Options:
Option B. Hypocalcemia: Hypocalcemia is commonly seen in tumor lysis syndrome due to the complex
ation of calcium with phosphate, resulting in decreased free ionized calcium levels.
Option C. Hyperuricemia: Hyperuricemia, an elevation in uric acid levels, is a hallmark feature of tumor
lysis syndrome. The rapid breakdown of tumor cells leads to the release of intracellular purines, which
are metabolized into uric acid.
Option D. Hyperkalemia: Hyperkalemia, an elevation in potassium levels, is a common finding in tumor
lysis syndrome. The release of intracellular potassium from lysed tumor cells overwhelms the body's ab
ility to excrete it, leading to elevated blood potassium levels

Solution for Question 12:


Correct Option A
• Troponin is the most sensitive and specific marker used in diagnosing myocardial infarction.
• Troponin is a regulatory protein found in cardiac muscle cells, and its release into the bloodstream is a
specific indicator of cardiac muscle damage.
• Elevated troponin levels in the blood indicate cardiac cell death, which occurs during a heart attack.
Incorrect options:
Option B. Cytokeratin: Cytokeratin is a
protein found in epithelial cells and is not specific to cardiac muscle. It is not used as a
marker for myocardial infarction.
Option C. Myoglobin: Myoglobin is an oxygen-binding protein found in muscle tissues, including cardia
c muscle. While it can be released into the bloodstream during a
heart attack, it is not as specific or sensitive as troponin in diagnosing myocardial infarction.
Option D. CPK-MM: CPK-MM, also known as creatine kinase-MB, is an enzyme found predominantly i
n cardiac muscle cells. Elevated CPK-MM levels can indicate cardiac muscle damage, including myoca
rdial infarction. However, troponin is considered more sensitive and specific for diagnosing myocardial i
nfarction than CPK-MM.

Solution for Question 13:


Correct Option A

Page 13

628
• The antecedent diagnosis of Group A streptococcal infection in acute rheumatic fever can be made by
testing for ASO (antistreptolysin O) titers.
• ASO antibodies are produced by the immune system in response to a Group A streptococcal
infection.
• Testing the blood for ASO titers can help determine if the individual had a recent streptococcal
infection, which is a prerequisite for the development of acute rheumatic fever.
Incorrect options:
Option B. CRP (C-reactive protein): CRP is an acute-phase reactant and a
marker of inflammation. While it can be elevated in acute rheumatic fever, it is not specific to Group A
streptococcal infection and cannot be used to make an antecedent diagnosis.
Option C. ESR (Erythrocyte sedimentation rate) elevation: ESR is a nonspecific marker of inflammation
and can be elevated in various conditions, including acute rheumatic fever. However, it does not specif
ically indicate a Group A streptococcal infection and cannot be used as a
definitive antecedent diagnosis.
Option D. Low C3 levels: C3 is a component of the complement system, and its levels can be reduced
during acute rheumatic fever due to immune system activation. However, low C3 levels are not specific
to Group A streptococcal infection and cannot be used alone for antecedent diagnosis.

Solution for Question 14:


Correct option C
• Hyper-eosinophilia invasion of myocytes is seen 1-3 days after injury.
Incorrect options:
Option A: Karyorrhexis of myocytes: Karyorrhexis refers to the fragmentation or dissolution of the nucle
us of a cell. It is a
later histopathological finding and is typically observed within 12 hours of ischemic injury.
Option B: Neocapillary invasion of myocytes: Neocapillary invasion refers to the formation of new blood
vessels within tissues. It is a
characteristic histopathological finding in the early stages of ischemic injury to the heart.
Option D: Coagulation necrosis of myocytes: Coagulation necrosis is a later stage of tissue injury chara
cterized by denaturation and coagulation of cellular proteins. It is typically observed within 12 hours of i
schemic injury.

Solution for Question 15:


Correct Option A.
Hemangioblastoma Hemangioblastoma is not a small round blue cell tumor. It is a rare, benign tumor t
hat arises from the blood vessels in the central nervous system. Histologically, it is characterized by ab
undant blood vessels and stromal cells. It is commonly associated with von Hippel-Lindau disease, an i
nherited condition that predisposes individuals to the development of various tumors.

Page 14

629
Incorrect Option:
Option B: Ewing's sarcoma Ewing's sarcoma is a malignant tumor that predominantly affects children a
nd young adults. It is characterized by small round blue cells with a high nuclear-to-cytoplasmic ratio. It
commonly arises in the bones, but it can also occur in soft tissues. Ewing's sarcoma often presents wit
h localized pain and swelling and can metastasize to other sites.
Option C: Retinoblastoma Retinoblastoma is a malignant tumor that develops in the retina of the eye. It
primarily affects children and can occur unilaterally or bilaterally. Retinoblastoma is characterized by s
mall round blue cells and can cause leukocoria (white pupillary reflex) and visual disturbances. It is ass
ociated with mutations in the RB1 gene.
Option D: Neuroblastoma Neuroblastoma is a malignant tumor that arises from primitive nerve cells, ty
pically in the adrenal glands or along the sympathetic nervous system. It is the most common extracran
ial solid tumor in children. Neuroblastoma is composed of small round blue cells and can present with v
arious signs and symptoms, including abdominal mass, bone pain, and neurologic changes.

Solution for Question 16:


Correct Option:
Option B.
• Arterial internal elastic lamina defect: This option is correct. Berry aneurysms, also known as
intracranial or saccular aneurysms, are localized outpouchings or dilations of blood vessels in the brain.
The primary underlying cause is a defect in the arterial internal elastic lamina, which weakens the
vessel wall and predisposes it to aneurysm formation.
Incorrect Option:
Option A. Endothelial injury of vessel due to HTN: This option is incorrect. While hypertension (HTN) c
an contribute to the weakening of blood vessels over time, it is not the primary cause of berry aneurys
ms. HTN can increase the risk of rupture in a
pre-existing aneurysm, but it does not directly lead to the formation of berry aneurysms.
Option C. Endothelial layer defect: This option is incorrect. While defects in the endothelial layer can co
ntribute to vascular diseases, they are not the primary cause of berry aneurysms. The structural defect
in the arterial internal elastic lamina is the main factor involved in the development of berry aneurysms.
Option D. Adventitia defect: This option is incorrect. The adventitia is the outermost layer of blood vess
els and plays a role in providing structural support. However, a defect in the adventitia is not the primar
y cause of berry aneurysms. The defect in the arterial internal elastic lamina is the main factor contribut
ing to the formation of these aneurysms.

Solution for Question 17:


Correct Option:
Option A. 1-B, 2-D, 3-A, 4-C
The correct match for the endothelial contraction is as follows:

Page 15

630
Incorrect options:
Option B. 1-B, 2-D, 3-C, 4-A: This option incorrectly matches the response of sunburn (1) with immedia
te transient (D) instead of delayed prolonged (B).
Option C. 1-D, 2-B, 3-A, 4-C: This option incorrectly matches the response of bee sting (2) with delaye
d prolonged (B) instead of immediate transient (D).
Option D. 1-B, 2-A, 3-D, 4-C: This option incorrectly matches the response of bacterial enzymes (3) wit
h immediate sustained (B) instead of immediate sustained (A).

Solution for Question 18:


Correct Option C: 1-E, 2-B; 3-C; 4-A
Explanation:
• Boxcar nuclei are a characteristic feature of hypersensitivity myocarditis, which is an inflammatory
condition involving the heart muscle.
• Myocyte hypertrophy is a common response to chronic pressure overload, such as in hypertension,
where the heart has to work harder to pump blood against increased resistance.
• Myocyte vacuolization is often seen in dilated cardiomyopathy (DCMP), a condition where the heart
chambers become enlarged and weakened.
• Myocyte disarray is a prominent feature of hypertrophic obstructive cardiomyopathy (HOCM), a
genetic condition characterized by thickening of the heart muscle, especially the interventricular
septum.

Solution for Question 19:


Correct Option D - Lymphocytic myocarditis:
• The patient's presentation with fever, fatigue, and left ventricular dysfunction with endomyocardial
biopsy showing lymphocytic infiltration of the myocardium is suggestive lymphocytic myocarditis.
Incorrect Options:
Options A, B, and C are incorrect.

Solution for Question 20:


Correct Option B - Single positive blood culture for Coxiella burnetti is a major criterion:
• A single positive blood culture for Coxiella burnetii is a major criterion in the modified Duke's criteria
for infective endocarditis.
Incorrect Options:

Page 16

631
Option A - Single positive blood culture for HACEK group is a major criterion: A single positive blood cu
lture for HACEK group organisms (Haemophilus species, Aggregatibacter actinomycetemcomitans, Ca
rdiobacterium hominis, Eikenella corrodens, and Kingella) is not considered a
major criterion in the modified Duke's criteria.
Option C - Complete dehiscence of prosthetic valve: Complete dehiscence of a
prosthetic valve is not a criterion in the modified Duke's criteria.
Option D - Documentation of four minor criteria allows a clinical diagnosis: A definitive diagnosis of end
ocarditis can be made in patients with two major criteria, one major and three minor criteria, or five min
or criteria.

Solution for Question 21:


Correct Option C - Dilated cardiomyopathy:
• Alcohol abuse has been strongly associated with the development of dilated cardiomyopathy.
• Dilated cardiomyopathy is a condition characterized by the enlargement and weakening of the heart's
left ventricle, leading to reduced pumping ability and potential heart failure.
Incorrect Options:
Options A, B, and D are not associated with alcohol abuse.

Solution for Question 22:


Correct Option B - Calcification and fibrosis:
• The autopsy finding on the mitral valve shows the fish mouth or buttonhole appearance, indicating
calcification and fibrosis, which is a characteristic feature of rheumatic heart disease.
Incorrect Options:
Options A, C, and D are incorrect.

Solution for Question 23:


Correct Option B - Anterior transmural:
• An aneurysm is a potential post-MI complication characterized by the formation of a localized,
abnormal dilation or bulge in the weakened area of the myocardium.
• It occurs as a result of the loss of contractile function and the thinning of the infarcted myocardial wall.
• Anterior transmural myocardial infarction is the most likely to cause an aneurysm.
Incorrect Options:
Options A, C, and D do not cause aneurysm as a post-MI complication.

Page 17

632
Bronchial Asthma,Emphysema,Chronic Bronchitis
1. In a 60-year-old chronic smoker with a barrel-shaped chest, showing hyperinflation of the lungs on
chest X-ray, and the given histology on biopsy: Which of the following cellular mechanisms likely leads
to the development of this underlying disease?

A. Increased activity of alpha-1 antitrypsin


B. Upregulation of antiproteases
C. Activation of neutrophil elastase
D. Inhibition of proteases
----------------------------------------
2. A 20-year-old female complained of recurrent episodes of wheezing, chest tightness, and shortness
of breath, during early mornings and when exposed to her neighbor's pet. Which of the following
mechanisms is responsible for this patient's presentation?
A. Activation of the complement system
B. Activation of T lymphocytes
C. IgE-mediated activation of mast cells
D. IgG-mediated activation of neutrophils
----------------------------------------
3. What cytokine secreted by Th2 cells promotes IgE class switching in patients with atopic asthma?
A. IL 5
B. IL 13
C. IL 17
D. IL 4
----------------------------------------
4. What's the likely etiopathogenesis in a 32-year-old male with complaints of infertility, chronic
sinusitis, recurrent pulmonary infections, showing poor sperm motility on sperm analysis and the
followinf chest X-ray findings?

633
A. X-linked mutation in the WASP gene
B. Adenosine Deaminase Deficiency
C. Inherited defect of the dynein arm
D. Elevated levels of anti-histone antibodies
----------------------------------------
5. Which of the following genetic mutations is not associated with idiopathic pulmonary fibrosis?
A. TERT
B. ABCA3
C. TERC
D. MUC5B
----------------------------------------
6. Which of the following statements describes the Reid Index?
A. Ratio of epithelial cell height to the overall bronchial wall thickness
B. Ratio of thickness of submucosal gland to the overall bronchial wall thickness
C. Ratio of smooth muscle thickness to the overall bronchial wall thickness
D. Ratio of bronchial wall thickness to the thickness of submucosal gland
----------------------------------------
7. Which of the following is the second most common infectious cause of exacerbation of chronic
bronchitis?
A. Hemophilus influenza
B. Diphtheria
C. Aspergillus
D. Moraxella
----------------------------------------
8. Mark the incorrect statement regarding chronic bronchitis:
A. Blue bloaters
B. Decreased Reid’s index

Page 2

634
C. Increased pulmonary resistance
D. Pulmonary vasoconstriction
----------------------------------------
9. Which of the following is not a histopathological feature of asthma?
A.
B.
C.
D.
----------------------------------------
10. Identify the condition depicted in the histopathological slide from lung biopsy given below:

A. Idiopathic Pulmonary Fibrosis


B. Hypersensitivity pneumonia
C. Nonspecific Interstitial Pneumonia
D. Cryptogenic Organizing Pneumonia
----------------------------------------

Correct Answers
Question Correct Answer

Question 1 3
Question 2 3
Question 3 4
Question 4 3
Question 5 2
Question 6 2
Question 7 4
Question 8 2
Question 9 2

Page 3

635
Question 10 4

Solution for Question 1:


Correct Option C - Activation of neutrophil elastase:
Given the presenting features, radiological findings, and biopsy, the patient likely has emphysema.
Neutrophil elastase, released by inflammatory cells like neutrophils, plays a key role in this process. Ne
utrophil elastase breaks down elastin fibers in the alveolar walls, contributing to airspace enlargement
and loss of lung elasticity. This destruction of lung tissue is a hallmark of emphysema.
Pathogenesis of emphysema:
Mechanism
Description
Toxic Injury and Inflammation
• Inhaled cigarette smoke and other harmful particles damage the respiratory epithelium, leading to
inflammation and variable degrees of parenchymal destruction.
• Increased levels of inflammatory mediators such as leukotriene B4, interleukin (IL)-8, and TNF are
observed in the affected lung areas.
Protease-Antiprotease Imbalance
• Inflammatory cells and epithelial cells release several proteases that degrade connective tissue
components in the lungs.
• Patients who develop emphysema often have a relative deficiency of protective antiproteases, which
can have a genetic basis.
Oxidative Stress
• Substances in tobacco smoke, alveolar damage, and inflammatory cells generate oxidants, leading to
tissue damage, endothelial dysfunction, and inflammation.
• NRF2, a transcription factor that senses oxidants in various cell types, plays a role in protecting cells
from oxidant damage.
Infection
• While infection is not considered an initiating factor in tissue destruction, bacterial and/or viral
infections can exacerbate existing emphysema.
Incorrect Options:
Options A, B, and D
are incorrect, As they are protective mechanisms against the development of emphysema.
Option A - Increased activity of alpha-1 antitrypsin: Alpha-1 antitrypsin deficiency predisposes individua
ls to early-onset emphysema. Increased activity of alpha-1 antitrypsin would not lead to the developme
nt of emphysema.
Option B - Upregulation of antiproteases: Antiproteases such as alpha-1 antitrypsin serve to counteract
the activity of proteases like neutrophil elastase. Upregulating antiproteases could potentially provide a
protective effect against the development of emphysema
Option D - Inhibition of proteases: Inhibition of proteases, including neutrophil elastase, would likely pro
tect against the development of emphysema rather than contribute to it.

Page 4

636
Solution for Question 2:
Correct Option C - IgE-mediated activation of mast cells:
Considering the clinical scenario:
• Recurrent episodes of wheezing, chest tightness, and shortness of breath: These symptoms are
classic manifestations of asthma, indicating airway obstruction and inflammation.
• The timing of symptoms, particularly worsened in the early morning and triggered by exposure to
specific allergens like pet dander, suggests an allergic component to asthma.
The mechanism responsible for the patient's presentation is the IgE-mediated activation of mast cells. I
n allergic asthma, exposure to specific allergens leads to the production of allergen-specific IgE antibo
dies. These IgE antibodies bind to high-affinity receptors on mast cells, sensitizing them. Upon subseq
uent exposure to the same allergen, mast cell degranulation and release of inflammatory mediators, co
ntribute to the symptoms of asthma.
Atopic Asthma:
Features
Explanation
Onset
Typically begins in childhood
Triggers
Environmental allergens such as dusts, pollens, cockroach or animal dander, and foods
Family History
A positive family history of asthma is common
Skin Testing Reaction
Positive immediate wheal-and-flare reaction to skin testing with offending antigen
Serum IgE Levels
Elevated total serum IgE levels
Serum RASTs (Radioallergosorbent tests)
Positive for allergen-specific IgE antibodies
Incorrect Options:
Options A, B, and D are incorrect.

Solution for Question 3:


Correct Option D - IL 4:
• In atopic asthma, Th2 cells are activated in response to allergens.

Page 5

637
• Upon activation, Th2 cells secrete IL-4, which is a key cytokine involved in promoting the
differentiation of B cells into plasma cells.
• IL-4 acts on B cells, stimulating them to undergo class switching to produce IgE antibodies instead of
other immunoglobulin classes.
• IgE antibodies produced as a result of IL-4 stimulation bind to high-affinity IgE receptors (FcεRI) on
the surface of mast cells and basophils, sensitizing them to subsequent exposure to allergens.
• Upon re-exposure to allergens, inflammatory mediators such as histamine, leukotrienes, and
cytokines, leading to the characteristic symptoms of asthma, including bronchoconstriction, airway
inflammation, and mucus production.
Incorrect Options:
Option A - IL 5: Plays a role in the activation of locally recruited eosinophils.
Option B - IL 13: It stimulates mucus secretion from bronchial submucosal glands and also promotes Ig
E production by B cells, it does not mainly affect class switching IL4.
Option C - IL 17: They recruit neutrophils

Solution for Question 4:


Correct Option C - Inherited defect of the dynein arm:
• This patient's clinical presentation of chronic sinusitis, recurrent pulmonary infections, infertility with
poor sperm motility, and dextrocardia on the X-ray is suggestive of Kartagener syndrome. It is typically
caused by an inherited defect in the dynein arm, which impairs ciliary function. This defect affects the
movement of cilia in the respiratory tract, leading to impaired mucociliary clearance and resulting in
chronic sinusitis and recurrent pulmonary infections. Additionally, impaired ciliary function in individuals
with Kartagener syndrome often causes decreased sperm motility.
• Kartagener's Syndrome Dynein arm defect in cilia BSS triad: Bronchiectasis Sinusitis Situs Inversus
(dextrocardia) Infertility in both males and females
• Bronchiectasis
• Sinusitis
• Situs Inversus (dextrocardia)
Kartagener's Syndrome
Dynein arm defect in cilia
BSS triad:
• Bronchiectasis
• Sinusitis
• Situs Inversus (dextrocardia)
Infertility in both males and females
Incorrect Options:

Page 6

638
Option A - X-linked mutation in the WASP gene: is associated with Wiskott-Aldrich syndrome, a primar
y immunodeficiency disorder characterized by eczema, thrombocytopenia, and recurrent infections.
Option B - Adenosine Deaminase Deficiency: is associated with severe combined immunodeficiency (S
CID), which typically presents with recurrent infections but does not typically present with chronic sinusi
tis, poor sperm motility, or dextrocardia.
Option D - Elevated levels of anti-histone antibodies: are associated with autoimmune conditions such
as systemic lupus erythematosus (SLE).

Solution for Question 5:


Correct Option B - ABCA3:
• ABCA3: is not associated with idiopathic pulmonary fibrosis. Mutations in the ABCA3 gene, encoding
the ATP-binding cassette transporter A3 protein, disrupt normal surfactant production and cause
Pulmonary Alveolar Proteinosis (PAP). It is a lung disorder characterized by the accumulation of
surfactant-derived lipoproteinaceous material within the alveoli.
• Feature Idiopathic Pulmonary Fibrosis (IPF) Genetic Mutations Telomerase reverse transcriptase
(TERT). Telomerase RNA component (TERC). MUC5B gene. Pathology Chronic and progressive
interstitial lung disease is characterized by fibrosis and scarring of lung tissue. Clinical Features
Progressive dyspnea on exertion. Chronic, dry cough. Inspiratory crackles. Digital clubbing. Imaging
Findings Temporal heterogeneity. Subpleural and interlobar fibrosis. Honeycomb appearance
Treatment Options Pirfenidone (TGF-beta inhibitor). Nintedanib (Tyrosine kinase receptor inhibitor).
Lung transplantation
• Telomerase reverse transcriptase (TERT).
• Telomerase RNA component (TERC).
• MUC5B gene.
• Progressive dyspnea on exertion.
• Chronic, dry cough.
• Inspiratory crackles.
• Digital clubbing.
• Temporal heterogeneity.
• Subpleural and interlobar fibrosis.
• Honeycomb appearance
• Pirfenidone (TGF-beta inhibitor).
• Nintedanib (Tyrosine kinase receptor inhibitor).
• Lung transplantation
Feature
Idiopathic Pulmonary Fibrosis (IPF)
Genetic Mutations
• Telomerase reverse transcriptase (TERT).

Page 7

639
• Telomerase RNA component (TERC).
• MUC5B gene.
Pathology
Chronic and progressive interstitial lung disease is characterized by fibrosis and scarring of lung tissue.
Clinical Features
• Progressive dyspnea on exertion.
• Chronic, dry cough.
• Inspiratory crackles.
• Digital clubbing.
Imaging Findings
• Temporal heterogeneity.
• Subpleural and interlobar fibrosis.
• Honeycomb appearance
Treatment Options
• Pirfenidone (TGF-beta inhibitor).
• Nintedanib (Tyrosine kinase receptor inhibitor).
• Lung transplantation
Incorrect Options:
Options A, C and D are associated with Idiopathic Pulmonary Fibrosis.

Solution for Question 6:


Correct Option B - Ratio of thickness of submucosal gland to the overall bronchial wall thickness:
The Reid Index is a histopathological measure used to assess the degree of hypertrophy of submucos
al glands in the bronchial wall, particularly in the context of chronic bronchitis. It is defined as the ratio o
f the thickness of the submucosal glands to the thickness of the overall bronchial wall.
• Smoking irritates the lungs, leading to an increase in mucous secretion.
• This increase in mucous secretion predisposes the individual to respiratory infections. Among the
most common pathogens involved in these infections are Haemophilus influenzae and Moraxella
species.
• Over time, chronic irritation and infections contribute to hypertrophy of the submucosal glands in the
bronchial walls.
• This hypertrophy results in an increase in Reid's index, which is a ratio of the thickness of the
submucosal glands to the overall thickness of the bronchial wall.
Therefore, an increase in Reid's index indicates a
histological change associated with chronic smoking and its sequelae.
Incorrect Options:

Page 8

640
Options A, C, and D are incorrect.

Solution for Question 7:


Correct Option D – Moraxella:
• Moraxella catarrhalis is the second most commonly associated microorganism responsible for causing
exacerbations of chronic bronchitis, particularly in elderly patients.
Incorrect Options:
Option A - Hemophilus influenza: Hemophilus influenza is the most common infectious cause of acute
exacerbation in patients of chronic bronchitis.
Options B and C: Diphtheria and Aspergillus are not commonly associated with chronic bronchitis.

Solution for Question 8:


Correct option b) Decreased Reid’s index
• Reid's index is a measure of the relative size of the submucosal glands to the bronchial wall thickness.
Reid's index actually increases in chronic bronchitis.
Ried’s Index

• Let submucosa be ‘a’ and the total layers be ‘b’ then, RI = a ÷ b


• RI = a ÷ b
• Normal Reid’s index:0.4
• RI = a ÷ b
Incorrect Options:

Page 9

641
Option A - Blue bloaters: Blue bloaters refer to patients with chronic bronchitis. These individuals often
present with cyanosis (blue coloration of the skin due to low oxygen levels) and may exhibit signs of rig
ht heart failure, such as peripheral edema (hence the term "bloaters").
Option C: Increased pulmonary resistance: The pulmonary resistance increases in response to pulmon
ary vasoconstriction.
Option D - Pulmonary vasoconstriction: Pulmonary vasoconstriction occurs in response to hypoxia in p
atients with chronic bronchitis.

Solution for Question 9:


Correct Option B
• The image in option B shows Schaumann bodies in multinucleated giant cells. They are not a
histopathological feature of asthma. These bodies are actually seen in sarcoidosis.
Incorrect Options:
Option A - The image in option A shows Charcot Leyden Crystal. They are needle-shaped crystals com
posed of Galectin-10. They are seen in sputum samples of patients with asthma.
Option C - The image in option C shows Creola bodies. They are made of epithelial cells. They are see
n in sputum sample of patients with asthma.
Option D - The image in option D shows the Curschmann spiral, which is a
histopathological feature of asthma.

Solution for Question 10:


Correct Option D - Cryptogenic Organizing Pneumonia:
• The given image shows Masson bodies, which are characteristic findings of Cryptogenic organizing
pneumonia (COP), also known as bronchiolitis obliterans organizing pneumonia (BOOP). It is a type of
fibrosing lung disease.
Incorrect Options:
Options A, B, and C do not exhibit Masson bodies on histopathology.

Page 10

642
Pulmonary Infections ( Tuberculosis,Pneumonia)
1. What is the most likely causative organism seen in a chronic alcoholic patient presenting with fever,
cough, and red currant jelly sputum?
A. Streptococcus pneumoniae
B. Mycobacterium tuberculosis
C. Klebsiella pneumoniae
D. Haemophilus influenzae
----------------------------------------
2. Which of the following foci of TB affect the infraclavicular region of the lungs?
A. Ghon focus
B. Weighert focus
C. Simon focus
D. Assman focus
----------------------------------------
3. Match the following organism with their corresponding histopathological image: 1 Histoplasma
capsulatum A 2 Aspergillus B 3 Mucormycosis C 4 Pneumocystis Carinii D

1 Histoplasma capsulatum A

2 Aspergillus B

643
3 Mucormycosis C

4 Pneumocystis Carinii D

A. 1-A, 2-B, 3-D, 4-C


B. 1-B, 2-D, 3-A, 4-C
C. 1-D, 2-C, 3-B, 4-A
D. 1-D, 2-A, 3-B, 4-C
----------------------------------------
4. Which of the following histological findings is identified in the image provided?

A. Rugby ball appearance


B. Owl’s eye appearance
C. Warthin's Finkeldey Giant cells
D. Shaumann bodies

Page 2

644
----------------------------------------
5. Which of the following is incorrectly matched?
A. Puhl’s focus – Supraclavicular region of lung
B. Weighert focus – Blood vessels
C. Assman focus- Infraclavicular region of lung
D. Simon focus- Liver
----------------------------------------
6. Which area is most commonly affected by secondary TB?
A. Lower part of upper lobe and upper part of lower lobe
B. Subpleural region
C. Apex of lungs
D. Spine
----------------------------------------

Correct Answers
Question Correct Answer

Question 1 3
Question 2 4
Question 3 4
Question 4 3
Question 5 4
Question 6 3

Solution for Question 1:


Correct Option C - Klebsiella pneumoniae:
The patient in the scenario is likely suffering from pneumonia caused by Klebsiella pneumoniae.
Risk factor:
Chronic alcoholism is a
significant risk factor for developing Klebsiella pneumoniae pneumonia. Klebsiella pneumoniae is a
part of the normal flora of the gut. Klebsiella pneumoniae-associated pneumonia is a
frequent occurrence among individuals who have a
predisposition to aspiration (malnutrition, debilitation, chronic alcoholism, and diabetes).
Presentation:
The thick mucoid capsule of Klebsiella pneumoniae contributes to the formation of gelatinous sputum,
often described as resembling currant jelly. The sputum has a
characteristic red or rust-colored appearance due to the presence of blood.
An overview of other organisms causing pneumonia:

Page 3

645
Condition
Causative Organism
Clinical Associations
Community-acquired pneumonia (CAP)
Streptococcus pneumoniae
Common cause; typical pneumonia
Alcohol-associated pneumonia
Klebsiella pneumoniae
Red currant jelly sputum; chronic alcoholism (3 Ks: Klebsiella, Kurrant jelly sputum, Chronic alcoholism
)
COPD exacerbations
Haemophilus influenzae

Burns patient pneumonia


Pseudomonas aeruginosa (Blue-green coloration)
Often associated with burn injuries
ICU-acquired pneumonia
Legionella pneumophila, Pseudomonas aeruginosa
Seen in patients admitted to intensive care units
Fungal pneumonia
• Dimorphic fungi (Histoplasma, Blastomycosis, Aspergillus, Mucormycosis)
• Opportunistic infections
• Pneumocystis jiroveci pneumonia (PJP)
• Associated with systemic fungal infections
• Common in immunocompromised individuals; formerly known as PCP
Viral pneumonia
• Influenza virus, Respiratory Syncytial Virus
• (RSV), Measles virus (Hecht's pneumonia), Cytomegalovirus (CMV)
Common viral causes of pneumonia
Incorrect Options:
Option A - Streptococcus pneumoniae:
• Streptococcus pneumoniae is a common cause of community-acquired pneumonia, particularly in
individuals with risk factors such as advanced age, and underlying lung disease.
• It typically does not produce the "red currant jelly" appearance of sputum.
Option B - Mycobacterium tuberculosis:

Page 4

646
• Mycobacterium tuberculosis is a common cause of tuberculosis (TB), which typically presents with
symptoms such as chronic cough, fever, weight loss, and acid fast bacilli in sputum.
• While hemoptysis can occur in TB, particularly in advanced or cavitary disease, the characteristic "red
currant jelly" appearance of sputum is not typically associated with TB.
• TB sputum may instead appear purulent and sometime blood-tinged.
Option D - Haemophilus influenzae:
• While it can lead to respiratory symptoms such as cough and fever, it does not typically produce the
characteristic "red currant jelly" appearance of sputum.

Solution for Question 2:


Correct Option D - Assman focus:
• Assman's focus is foci of TB affect the infraclavicular region of the lungs.
• Foci of tuberculosis include:
Focus

Ghon focus
Lung
Ghon complex
Lung + Lymph node
Ranke's complex
Calcification in Ghon complex
Simon focus
Apex of lung
Puhl's focus
Supraclavicular region of the lung
Assman focus
Infraclavicular region of the lung
Simmond focus
Liver
Weighert focus
Rich's focus
Brain
Incorrect Options:
Refer to explanation of Option D.

Page 5

647
Solution for Question 3:
Correct Option D - 1-D, 2-A, 3-B, 4-C:

Solution for Question 4:


Correct Option C - Warthin's Finkeldey Giant cells:
• Warthin's Finkeldey Giant cells are multinucleated giant cells observed in cases of measles infection.
They are typically found in the lymph nodes of individuals with measles and are characterized by large,
irregularly shaped nuclei surrounded by a clear halo. These cells are formed due to the fusion of
histiocytes (macrophages).
Incorrect Options:
Option A - Rugby ball appearance: It is seen in meconium aspiration pneumonia.

Option B - Owl’s eye appearance: Owl's eye appearance is commonly associated with cytomegalovirus
(CMV) pneumonia.

Page 6

648
Option D - Schaumann bodies: Schaumann bodies are seen in sarcoidosis.

Solution for Question 5:


Correct Option D - Simon focus- Liver:
• Simon focus is incorrectly matched with the liver. The Simon focus is seen at the apex of lungs, not in
liver. The Simmond focus are seen in the liver.
Incorrect Options:
Options A, B, and C are correctly matched.

Page 7

649
Solution for Question 6:
Correct Option C - Apex of lungs:
• Secondary tuberculosis (TB) refers to the reactivation of latent TB infection in individuals who have
previously been infected with Mycobacterium tuberculosis. The apex of the lungs is the most common
site affected by secondary TB.
Incorrect Options:
Option A and B: The lower part of the upper lobe and upper part of the lower lobe, and Subpleural regi
on are affected by primary tuberculosis infection
Option D - Spine: Involvement of the spine is seen in miliary/disseminated tuberculosis infection.

Page 8

650
Pneumoconiosis,Sarcoidosis,Asbestosis
1. Which of the following laboratory findings is indicative of sarcoidosis?
A. Decreased CD4/CD8 ratio in the bronchoalveolar lavage
B. Decreased serum levels of calcium
C. Elevated serum levels of angiotensin-converting enzyme
D. All of the above
----------------------------------------
2. Which of the following is a characteristic histological feature seen in patients with sarcoidosis?
A. Masson bodies
B. Extracellular asteroid body
C. Intracellular asteroid body
D. All of the above
----------------------------------------
3. What substance is responsible for reticular opacities, honeycombing in the lung parenchyma, and
pleural plaques on chest HRCT scan and the following finding from bronchoalveolar lavage?

A. Sugarcane dust
B. Beryllium
C. Asbestos
D. Lead
----------------------------------------
4. What condition is a 63-year-old male construction worker at increased risk for, given bilateral
interstitial opacities on chest X-ray, eggshell calcifications in hilar lymph nodes, and the following
findings on BAL polarizing microscopy?

651
A. Mucormycosis
B. Aspergillosis
C. Tuberculosis
D. Blastomycosis
----------------------------------------
5. Which of the following is the most common tumor associated with asbestosis?
A. Small Cell Carcinoma Lung
B. Laryngeal squamous papilloma
C. Mesothelioma
D. Adenocarcinoma lung
----------------------------------------
6. Which of the following organisms is responsible for causing moldy maple bark hypersensitivity
pneumonitis?
A. Micropoly spora faeni
B. Mycobacterium avium
C. Thermoactinomyces sacchari
D. Cryptostroma corticale
----------------------------------------
7. Which of the following is not likely to be seen in a patient with Sarcoidosis?
A. Mikulicz disease
B. Lofgren syndrome
C. Lupus pernio
D. Viral hepatitis
----------------------------------------
8. Which of the following histopathological findings are likely to be seen in a biopsy sample taken from
a patient with Sarcoidosis? Extra-cellular asteroid bodies Caseating granuloma Granuloma with
lymphocytic collar Shaumann bodies Lamellar bodies
A. 1, 2 and 3

Page 2

652
B. 3, 4 and 5
C. 2, 4 and 5
D. All of the above
----------------------------------------
9. What blood findings are unlikely to be observed in a 45-year-old female presenting with persistent
dry cough, shortness of breath, fatigue, and bilateral hilar lymphadenopathy, which on biopsy reveals
the given findings?

A. Increased ACE levels


B. Decreased CD4/CD8 ratio
C. Increased calcium levels
D. Increased Vitamin D production
----------------------------------------
10. Which of the following conditions is characterized by the combination of pneumoconiosis and
rheumatoid arthritis?
A. Caplan syndrome
B. Erasmus syndrome
C. Wegener's granulomatosis
D. Goodpasture syndrome
----------------------------------------
11. In asbestosis, which of the area of lung is most commonly involved?
A. The upper lobe is predominantly affected
B. The middle lobe is predominantly affected
C. The lower lobe is predominantly affected
D. Both upper and lower lobes are equally affected
----------------------------------------
12. Which of the following is the most pathogenic variant of asbestos?
A. Crocidolite
B. Serpentine

Page 3

653
C. Erionite
D. Crysolite
----------------------------------------
13. Which of the following occupational lung diseases is associated with Monday chest syndrome?
A. Baritosis
B. Berylliosis
C. Byssinosis
D. Bagassosis
----------------------------------------

Correct Answers
Question Correct Answer

Question 1 3
Question 2 3
Question 3 3
Question 4 3
Question 5 4
Question 6 4
Question 7 4
Question 8 3
Question 9 2
Question 10 1
Question 11 3
Question 12 1
Question 13 3

Solution for Question 1:


Correct Option C - Elevated serum levels of angiotensin-converting enzyme:
The likely diagnosis in this scenario is sarcoidosis.
• Clinical features include unintentional weight loss and occasional night sweats due to systemic
inflammation, peripheral lymph adenopathy, and erythema nodosum (erythematous nodules on the
shins).
• Chest X-ray: Bilateral hilar lymphadenopathy
• Microscopic findings: well-formed non-caseating granulomas
• In cases of sarcoidosis: C) Elevated serum levels of angiotensin-converting enzyme is considered a
diagnostic feature. In the granulomas of sarcoidosis, angiotensin-converting enzyme (ACE) is produced
by epithelioid cells that are derived from recently activated macrophages.

Page 4

654
Incorrect Options:
Option A - Decreased CD4/CD8 ratio: In sarcoidosis, there is often an increased CD4/CD8 ratio in the
bronchoalveolar lavage (BAL) fluid due to an accumulation of CD4+ T
cells within the affected tissues. This ratio is commonly used as a marker of disease activity (ranges fro
m 5:1 to 15:1) and is typically elevated in sarcoidosis rather than decreased.
Option B - Decreased serum levels of calcium: Hypercalcemia is a
common finding in sarcoidosis due to increased production of 1,25-dihydroxyvitamin D by activated ma
crophages within granulomas. This leads to increased absorption of calcium from the intestines and re
sorption from bone, resulting in elevated serum calcium levels rather than decreased levels.
Option D - All of the above: This option is incorrect because only option C
(Elevated serum levels of angiotensin-converting enzyme) is typically associated with sarcoidosis.

Solution for Question 2:


Correct Option C - Intracellular asteroid body:
• Sarcoidosis is characterized by the presence of well-formed non-necrotizing granulomas. These
granulomas consist of aggregates of tightly clustered epithelioid macrophages, often with giant cells.
Central necrosis is uncommon. Majority of these granulomas contain laminated concretions composed
of calcium and proteins known as Schaumann bodies and stellate inclusions known as asteroid bodies
within giant cells.
Asteroid body
Schaumann body

Page 5

655
Incorrect Options:
• Sporothrix is associated with extracellular asteroid bodies.
• Options A, B, and D are incorrect and the correct option is explained above.

Solution for Question 3:


Correct Option C - Asbestos:
• Given the presence of pleural plaques on HRCT and ferruginous bodies on microscopy this is a case
of asbestosis.

Page 6

656
• The image shows ferruginous bodies: Ferruginous bodies formed by the deposition of iron-containing
proteins around asbestos fibers.

Incorrect Options:
Options A, B, and D do not typically show pleural plaques and ferruginous bodies. Though ferruginous
bodies can be found in silicosis, anthracosis, etc, pleural plaques are exclusive to asbestosis.

Solution for Question 4:


Correct Option C - Tuberculosis:
• Given the patient's occupation and findings described in the clinical scenario, the presence of
interstitial opacities on chest imaging is consistent with that of pneumoconiosis. Eggshell calcification
(calcium surrounding a zone lacking calcium) on hilar nodes is a characteristic feature seen in silicosis.

• The image shows bright white collections of polarizable crystals, (silica crystals). Based on the
collective data, the findings strongly indicate that the diagnosis for the patient is likely to be silicosis.
Hence, the patient is at increased risk for developing complications related to silicosis, such as
progressive massive fibrosis, lung cancer, and tuberculosis due to impaired immune function.
Silicosis and Tuberculosis:
• Crystalline silica inhibits the ability of pulmonary macrophages to kill phagocytosed mycobacteria.
Incorrect Options:
Option A - Mucormycosis:
• Mucormycosis is a rare but serious fungal infection caused by a group of molds known as
mucormycetes. It typically affects individuals with weakened immune systems, such as those with
uncontrolled diabetes, cancer, or immunosuppression. It is not usually associated with silicosis.
Option B - Aspergillosis:

Page 7

657
• Aspergillosis is a fungal infection caused by the Aspergillus species. It commonly affects individuals
with pre-existing cavitary lesions (pulmonary tuberculosis), but is not usually associated with silicosis.
Aspergillosis can manifest as allergic bronchopulmonary aspergillosis (ABPA), aspergilloma, and
invasive aspergillosis.
Option D - Blastomycosis:
• Blastomycosis is a fungal infection caused by Blastomyces dermatitidis, manifesting as cough, fever,
chest pain, and fatigue, which can mimic other respiratory infections. In severe cases, it can lead to
acute respiratory distress syndrome (ARDS). Blastomycosis is not usually associated with silicosis.

Solution for Question 5:


Correct Option D - Adenocarcinoma lung:
• The most common carcinoma associated with asbestosis exposure is adenocarcinoma of the lung.
• The most specific carcinoma associated with asbestosis exposure isMesothelioma of the lung.
Incorrect Options:
Options A,B, and C are incorrect.

Solution for Question 6:


Correct Option D - Cryptostroma corticale:
• Cryptostroma corticale is a fungus that commonly grows on maple trees, particularly on the bark.
Exposure to its spores can lead to moldy maple bark hypersensitivity pneumonitis, a type of lung
inflammation caused by a combination of type 3 and 4 hypersensitivity.
Incorrect Options:
Option A - Micropoly spora faeni: Micropolyspora faeni causes moldy hay hypersensitivity pneumonitis.
Option B
- Mycobacterium avium: Mycobacterium avium causes hot tub lung hypersensitivity pneumonitis.
Option C - Thermoactinomyces sacchari: Thermoactinomyces sacchari causes moldy sugarcane hyper
sensitivity pneumonitis.

Solution for Question 7:


Correct Option D - Viral hepatitis:
• Sarcoidosis is a systemic inflammatory disease that can affect multiple organs, most commonly the
lungs. Sarcoidosis can present with non-infectious hepatitis, not viral hepatitis.
Incorrect Options:
Option A - Mikulicz disease: Mikulicz disease is characterized by enlargement of the lacrimal and saliv
ary glands. It is associated with Sarcoidosis.

Page 8

658
Option B - Lofgren syndrome: Lofgren syndrome is a classic presentation of acute sarcoidosis, consisti
ng of fever, bilateral hilar lymphadenopathy, erythema nodosum, granulomas, and joint pain.
Option C - Lupus pernio: Lupus pernio is a chronic skin manifestation of sarcoidosis. It signifies a
bad prognosis.

Solution for Question 8:


Correct Option C - 2, 4 and 5:

Solution for Question 9:


Correct Option B - Decreased CD4/CD8 ratio:
• In a 45-year-old female presenting with persistent dry cough, shortness of breath, fatigue, and
bilateral hilar lymphadenopathy with intracellular asteroid bodies revealed on biopsy, sarcoidosis is a
likely diagnosis. In sarcoidosis, there is typically an increase in CD4/CD8 ratio in the blood due to the
accumulation of CD4+ T-helper cells at the sites of granuloma formation.
Incorrect Options:
Option A - Increased ACE levels: Elevated angiotensin-converting enzyme (ACE) levels are commonly
associated with sarcoidosis due to its increased production by activated macrophages within granulom
as.
Option C - Increased calcium levels: Hypercalcemia can occur in sarcoidosis due to increased producti
on of vitamin D within granulomas.
Option D - Increased Vitamin D production: Sarcoidosis can lead to increased activation of vitamin D
within granulomas due to the expression of 1-alpha hydroxylase enzyme by activated macrophages.

Solution for Question 10:


Correct Option A - Caplan syndrome:
• Caplan syndrome is a condition characterized by the combination of pneumoconiosis (Silicosis) and
rheumatoid arthritis.
Incorrect Options:
Option B
- Erasmus syndrome: Erasmus syndrome is combination of pneumoconiosis and scleroderma.
Option C
- Wegener's granulomatosis: Wegener's granulomatosis is not associated with pneumoconiosis.
Option D - Goodpasture syndrome: Goodpasture syndrome does not involve pneumoconiosis or rheum
atoid arthritis.

Page 9

659
Solution for Question 11:
Correct Option C - The lower lobe is predominantly affected:
• Asbestosis is a chronic lung disease caused by the inhalation of asbestos fibers. Over time, these
fibers can cause scarring of the lung tissue, leading to symptoms such as shortness of breath and
coughing. In asbestosis, the lower lobes of the lungs are typically most affected by the fibrotic changes.
Incorrect Options:
Options A, B, and D are not commonly involved in asbestosis.

Solution for Question 12:


Correct Option A - Crocidolite:
• Crocidolite is one of the most pathogenic types of asbestos. Crocidolite fibers are thin, straight, and
rigid, making them more likely to penetrate deep into the lung tissue when inhaled, leading to greater
tissue damage and inflammation.
Incorrect Options:
Options B, C, and D are less pathogenic than crocidolite.

Solution for Question 13:


Correct Option C – Byssinosis:
• Byssinosis, also known as "Monday chest syndrome," is an occupational lung disease caused by
exposure to cotton dust or other organic dust in the workplace. Patients with byssinosis typically
experience worsening respiratory symptoms, such as cough, chest tightness, and shortness of breath,
on the first day of the workweek (usually Monday) after a period of rest (weekend break). These
symptoms gradually improve as the week progresses but recur with subsequent exposure, leading to a
cyclic pattern reminiscent of "Monday morning blues."
Incorrect Options:
Options A, B, and D are not associated with Monday chest syndrome.

Page 10

660
Previous Year Questions
1. Which of the following malignancies may be linked to exposure to asbestos?
A. Mesothelioma
B. Acute myeloid leukemia
C. Hepatic angiosarcoma
D. Skin carcinoma
----------------------------------------
2. What is the diagnosis for a patient who has a history of smoking 24 packs and is currently
experiencing breathlessness and chest pain, and is found to have a lung tumor with positive
chromogranin staining?
A. Adenocarcinoma
B. Squamous cell carcinoma
C. Mesothelioma
D. Small cell carcinoma
----------------------------------------
3. A 12-year-old child presented with 15 days of cough. On examination, cervical lymphadenopathy
was noted, and the lymph node biopsy showed the following features. What is the most probable
diagnosis?

A. Tuberculosis
B. Leprosy
C. Sarcoidosis
D. Syphilis
----------------------------------------
4. A 30-year-old woman with a long history of dyspnea, chronic cough with sputum, and wheezing died
of respiratory failure following a bout of lobar pneumonia. She was not a smoker or an alcoholic. The
lung at autopsy is shown in the image below. Which of the following is most likely to cause the
pathological changes shown?

661
A. Alpha-1 antitrypsin deficiency
B. Cystic fibrosis
C. Mutation in dynein arms
D. Antibodies against type-IV collagen
----------------------------------------
5. Which of the following is a possible association with a middle-aged man working in sandblasting who
developed hemoptysis, a productive cough, and a chest x-ray suggestive of tuberculosis?
A. Asbestosis
B. Silicosis
C. Anthracosis
D. Siderosis
----------------------------------------
6. Which structure in the pancreas is impacted in individuals with cystic fibrosis?

A. A
B. B
C. C
D. D
----------------------------------------
7. What is the probable industry in which a person, who experienced gradually worsening shortness of
breath after working there for an extended period, was employed based on the provided

Page 2

662
histopathological image of their lung biopsy?

A. Silica
B. Asbestos
C. Coal dust
D. Cotton
----------------------------------------
8. Which malignancy is most commonly associated with asbestosis?
A. Mesothelioma
B. Anthracosis
C. Hepatocellular carcinoma
D. Renal cell carcinoma
----------------------------------------
9. What is the likely diagnosis for a 12-year-old child who has had a cough for 15 days and is found to
have cervical lymphadenopathy on examination, based on the features observed in the lymph node
biopsy?

A. Tuberculosis
B. Leprosy
C. Sarcoidosis
D. Syphilis
----------------------------------------

Page 3

663
10. Please identify the medical condition depicted in the provided image.

A. Emphysema
B. Miliary tuberculosis
C. Sarcoidosis
D. Asbestosis
----------------------------------------
11. A patient working in a factory presented with lung fibrosis and thickening of pleura. Biopsy was
done. Diagnosis from the histopathology picture is :

A. Asbestosis
B. Silicosis
C. Tuberculosis
D. Mesothelioma
----------------------------------------
12. A 65 year old HIV positive man presented with fever and breathlessness for 2 weeks. Imaging
shows dense infiltration in the lower lobe and the lung biopsy showed the following finding. What is the
diagnosis ?

Page 4

664
A. Small cell lung cancer
B. CMV associated organizing pneumonia
C. Cryptococcal pneumonia
D. Tuberculosis
----------------------------------------
13. What is the diagnosis of a 45-year-old patient, who has been working in a factory for the last two
decades and presents with breathlessness, based on HRCT chest findings showing pleural thickening
and fibrosis?
A. Asbestosis
B. Coal worker pneumoconiosis
C. Silicosis
D. Berylliosis
----------------------------------------
14. Which of the following is the causative agent for the disease observed in a stable 25-year-old
farmer who was admitted with sudden fever, cough, and difficulty breathing? The patient initially
responded well to antibiotics for pneumonia but later presented with hypoxemia and worsening
dyspnea. CT chest scan revealed diffuse ground glass changes, while transbronchial biopsies showed
multiple small granulomas and significant lymphocytosis.
A. Thermophilic Actinomycetes
B. Aspergillus Fumigatus
C. Actinobacter
D. Aspergillus Flavus
----------------------------------------
15. Most common anterior mediastinal tumour?
A. Thymoma
B. Neurofibroma
C. Pericardial cyst
D. Bronchogenic cyst
----------------------------------------

Page 5

665
16. MC cause of atypical pneumonia?
A. Mycoplasma pneumoniae
B. Klebsiella pneumoniae
C. Hemophilus influenzae
D. Chlamydia
----------------------------------------
17. Which of the following features would you not expect to find in a 47-year-old male patient who is an
active smoker, diagnosed with Bronchial Asthma, and currently experiencing difficulty in breathing with
externally audible breath sounds?
A. Thickening of bronchial wall
B. Increase in number of airway goblet cells
C. Fibroblastic foci
D. Sub-basement membrane fibrosis
----------------------------------------
18. Ciliocytophthoria is most commonly seen in:
A. Acute respiratory infection
B. Kartagener syndrome
C. Situs inversus
D. Cystic fibrosis
----------------------------------------
19. A 55-year-old male presents with a chronic cough, shortness of breath, and chest pain. He has a
history of working in construction for several decades, where he was exposed to asbestos. Physical
examination reveals decreased breath sounds over the lower lung fields. A chest X-ray shows bilateral,
diffuse interstitial opacities. Pulmonary function tests reveal a restrictive pattern. High-resolution
computed tomography (HRCT) of the chest demonstrates "honeycombing" in the lung parenchyma.
what is diagnosis?
A. Tuberculosis
B. Chronic Obstructive Pulmonary Disease (COPD)
C. Asbestosis
D. Pneumonia
----------------------------------------
20. What is the most frequently observed lung abnormality in individuals with COVID-19 infection?
A. Pulmonary infarction
B. Diffuse alveolar damage
C. Endothelial injury
D. Fibrin clots
----------------------------------------

Page 6

666
21. Which of the following is a post-mortem finding seen in the lung after the death of a patient from
COVID-19 after a week?
A. ■■■Fresh and old intra-alveolar hemorrhage■■■
B. Thick layer of fibrin
C. Perivascular cuffing
D. Pulmonary artery hypertrophy with increased resistance
----------------------------------------
22. An elderly male smoker presents with a chronic cough, significant weight loss, and fatigue. The
serum calcium level is raised. A lung biopsy was done, and it showed large atypical cells with
hyperchromasia. What is the probable diagnosis?
A. Large cell neuroendocrine tumor
B. Small cell carcinoma
C. Adenocarcinoma
D. Squamous cell carcinoma
----------------------------------------
23. A chronic smoker who worked in a cement factory for 20 years developed mesothelioma. It is due
to?
A. Silicosis
B. Asbestosis
C. Coal worker pneumoconiosis
D. Bagassosis
----------------------------------------
24. The following cells seen in the lymph node are indicative of:

A. Infectious mononucleosis
B. Measles
C. CMV infection
D. Influenza
----------------------------------------

Page 7

667
Correct Answers
Question Correct Answer

Question 1 1
Question 2 4
Question 3 1
Question 4 1
Question 5 2
Question 6 2
Question 7 2
Question 8 1
Question 9 1
Question 10 2
Question 11 1
Question 12 2
Question 13 1
Question 14 1
Question 15 1
Question 16 1
Question 17 3
Question 18 1
Question 19 3
Question 20 2
Question 21 2
Question 22 4
Question 23 2
Question 24 2

Solution for Question 1:


• Asbestos exposure is associated with mesothelioma, asbestosis, pleural plaques, and carcinoma of
the lungs, stomach, colon, and larynx.
• Usually seen workers working in mining, fabrication, and installing and removing insulation.
• Patients usually present with productive cough, and shortness of breath during exertion, but later it
occurs even at rest.
• Chest X-ray shows irregular linear densities over both the lower lobes of the lung.
• A honeycomb pattern is also observed as the disease progresses.

Page 8

668
Incorrect Choices:
• Option b. Acute Myeloid Leukemia: AML is associated with chronic exposure to ionizing radiation,
benzene (Found in paints and plastic glues), and alkylating agents (Chlorambucil, nitrogen mustard).
• Option c. Hepatic angiosarcoma: Hepatic angiosarcoma occurs due to exposure to polycyclic
aromatic hydrocarbons. Polyvinyl chloride, which is used in the plastic industry, gets metabolized to an
epoxide which results in hepatic angiosarcoma.
• Option d. Skin carcinoma: Skin cancer occurs due to chronic exposure to ultraviolet radiation, which is
derived from sunlight and ionizing radiation (X-rays/ gamma rays). It causes squamous cell carcinoma,
basal cell carcinoma, and malignant melanoma.

Solution for Question 2:


Correct Option D - Small cell carcinoma:
• Small cell lung cancer (SCLC) is a highly aggressive type of lung cancer that is strongly associated
with smoking. It tends to grow rapidly and is often diagnosed at an advanced stage. SCLC is
characterized by positive staining for neuroendocrine markers, including chromogranin, which confirms
its neuroendocrine origin. This cancer typically arises centrally in the lung, often in the bronchi, and can
cause symptoms such as breathlessness and chest pain.
Incorrect Options:
• Option A, B and C are not associated with positive chromogranin staining.

Solution for Question 3:


Correct Option A - Tuberculosis:
• The patient's presentation with cough, cervical lymphadenopathy, and the histopathological features
of caseating granuloma with Langhans giant cells is suggestive of tuberculosis.
• Tuberculosis is a chronic infectious disease caused by the bacterium Mycobacterium tuberculosis.
• The presence of caseating granulomas, which are central necrotic areas surrounded by a rim of
epithelioid cells and Langhans giant cells, is a characteristic histopathological finding in tuberculosis.
Incorrect Options:
Option B - Leprosy:
• Leprosy is caused by Mycobacterium leprae and typically presents with skin lesions, peripheral nerve
involvement, and granulomatous inflammation.
• The histopathological features of leprosy include granulomas, but with a different pattern compared to
tuberculosis.
Option C - Sarcoidosis:
• Sarcoidosis is a systemic disorder characterized by the formation of non-caseating granulomas,
predominantly involving the lungs, lymph nodes, and other organs.
• The absence of caseation and the presence of non-caseating granulomas differentiate sarcoidosis
from tuberculosis.

Page 9

669
Option D - Syphilis:
• Syphilis is a sexually transmitted infection caused by the bacterium Treponema pallidum.
• It typically presents with various stages and manifestations, including primary and secondary skin
lesions, mucosal involvement, and systemic features.

Solution for Question 4:


Correct Option A - Alpha-1 antitrypsin deficiency:
• Alpha-1 antitrypsin is a protein that plays a crucial role in protecting the lungs from damage caused by
enzymes called proteases.
• In individuals with alpha-1 antitrypsin deficiency, there is a lack or abnormality of this protein, leading
to a loss of its protective function.
• As a result, there is an imbalance of protease activity in the lungs, with increased activity of enzymes
such as neutrophil elastase.
• Unopposed protease activity can lead to the destruction of lung tissue and the development of
emphysema, which is characterized by the enlargement of airspaces and the destruction of alveolar
walls.
Incorrect Options:
• The lung pathology seen in the image is more indicative of emphysema; hence, options B, C, and D
are incorrect.

Solution for Question 5:


Correct Option B:
In the above case, the patient gives a
history of working in sandblasting. He presents with hemoptysis and a productive cough and an x-ray is
suggestive of TB. The most probable association is that of silicosis. Silica leads to the dysfunction of p
ulmonary alveoli and results in the development of TB.
Incorrect Options:
Option A: Asbestosis is unlikely as the deposition is peripheral and in deep parts of the respiratory tree.
Clinical picture suggestive of dry cough, and other complaints.
Option C: Anthracosis is unlikely as there is no coal exposure.
Option D: Siderosis is unlikely as there is no iron exposure

Solution for Question 6:


• B is acini. The acini are the functional units of the exocrine pancreas responsible for producing and
secreting digestive enzymes into the pancreatic ducts. In cystic fibrosis, a genetic disorder caused by
mutations in the cystic fibrosis transmembrane conductance regulator (CFTR) gene, there is a defect in

Page 10

670
the production or function of the CFTR protein. This leads to thick, sticky mucus forming in various
organs, including the pancreas.
• In the pancreas, the defective CFTR protein affects the secretion of chloride ions and water,
dehydrating pancreatic secretions. This thickened mucus obstructs the pancreatic ducts, preventing the
proper flow of digestive enzymes from the acini into the duodenum. Consequently, there is a reduction
in the production and release of pancreatic enzymes, leading to impaired digestion and malabsorption
of nutrients.
Incorrect Choices:
• Option a. Islets of Langerhans: The islets of Langerhans are responsible for the production and
secretion of hormones such as insulin and glucagon. In cystic fibrosis, the islets are generally not
affected, and the primary pancreatic pathology lies in the exocrine portion.
• Option c. Intralobular Duct: The intralobular ducts are part of the pancreatic duct system and play a
role in transporting pancreatic enzymes. While cystic fibrosis can cause ductal abnormalities and
obstruction, the acini are the primary site of damage and dysfunction.
• Option d. Interlobular Duct: Similar to the intralobular ducts, the interlobular ducts transport pancreatic
secretions. However, they are not the main focus of damage in cystic fibrosis-related pancreatic
dysfunction.

Solution for Question 7:


Option b. Asbestos: Based on the histopathological image provided, the most likely industry the person
was working in is asbestos. Asbestos exposure is associated with the development of a
specific lung disease known as asbestosis.
• Asbestosis is a chronic lung condition caused by the inhalation of asbestos fibers over a prolonged
period. These fibers accumulate in the lung tissue, leading to inflammation, fibrosis, and scarring. The
histopathological features seen in asbestosis include: Interstitial fibrosis: Asbestos fibers cause injury to
the lung parenchyma, resulting in the deposition of collagen and fibrous tissue in the interstitium. This
fibrosis leads to thickening of the alveolar walls and restrictive lung function. Ferruginous bodies:
Asbestos fibers can be recognized in the lung tissue as long, golden-brown, rod-shaped structures
called ferruginous bodies. These bodies are composed of an asbestos fiber coated with iron and
protein. Their presence is highly suggestive of asbestos exposure. Honeycomb appearance: Severe
fibrosis can result in the formation of honeycomb cysts in the lung tissue. These cysts are characterized
by dilated airspaces surrounded by fibrotic walls. Alveolar macrophages: The presence of asbestos
fibers can stimulate an inflammatory response, leading to an accumulation of alveolar macrophages.
These macrophages can be seen within the lung tissue and bronchoalveolar lavage specimens.
• Interstitial fibrosis: Asbestos fibers cause injury to the lung parenchyma, resulting in the deposition of
collagen and fibrous tissue in the interstitium. This fibrosis leads to thickening of the alveolar walls and
restrictive lung function.
• Ferruginous bodies: Asbestos fibers can be recognized in the lung tissue as long, golden-brown,
rod-shaped structures called ferruginous bodies. These bodies are composed of an asbestos fiber
coated with iron and protein. Their presence is highly suggestive of asbestos exposure.
• Honeycomb appearance: Severe fibrosis can result in the formation of honeycomb cysts in the lung
tissue. These cysts are characterized by dilated airspaces surrounded by fibrotic walls.

Page 11

671
• Alveolar macrophages: The presence of asbestos fibers can stimulate an inflammatory response,
leading to an accumulation of alveolar macrophages. These macrophages can be seen within the lung
tissue and bronchoalveolar lavage specimens.
• Interstitial fibrosis: Asbestos fibers cause injury to the lung parenchyma, resulting in the deposition of
collagen and fibrous tissue in the interstitium. This fibrosis leads to thickening of the alveolar walls and
restrictive lung function.
• Ferruginous bodies: Asbestos fibers can be recognized in the lung tissue as long, golden-brown,
rod-shaped structures called ferruginous bodies. These bodies are composed of an asbestos fiber
coated with iron and protein. Their presence is highly suggestive of asbestos exposure.
• Honeycomb appearance: Severe fibrosis can result in the formation of honeycomb cysts in the lung
tissue. These cysts are characterized by dilated airspaces surrounded by fibrotic walls.
• Alveolar macrophages: The presence of asbestos fibers can stimulate an inflammatory response,
leading to an accumulation of alveolar macrophages. These macrophages can be seen within the lung
tissue and bronchoalveolar lavage specimens.
Incorrect Choices:
• Option a. Silica: Silica exposure is associated with another lung disease called silicosis.
Histopathological features of silicosis include the presence of nodular lesions composed of silica
particles, collagenous fibrosis, and characteristic concentric layers known as "silicotic nodules." These
features differ from the histopathological findings observed in the provided image, which are more
consistent with asbestosis.
• Option c. Coal dust: Coal dust exposure is associated with a lung disease known as coal worker's
pneumoconiosis (CWP). Histopathological features of CWP include the presence of coal macules
(collections of carbon-laden macrophages) and coal nodules (fibrotic lesions with central cores of coal
dust). These features differ from the histopathological findings observed in the provided image, which
are more indicative of asbestosis.
• Option d. Cotton: Exposure to cotton dust can lead to a lung disease known as byssinosis, also called
"brown lung disease." Histopathological findings in byssinosis include bronchiolitis and chronic
bronchitis with peribronchiolar fibrosis. The provided histopathological image does not show these
specific findings associated with cotton dust exposure, making cotton dust an unlikely cause in this
scenario.

Solution for Question 8:


Correct Answer: A
• Mesothelioma is a type of carcinoma that occurs in the outer lining of several organs. As the question
suggests it is linked primarily with asbestos exposure. Asbestos is a conjugate of minerals that are
composed of microscopic fibers. This substance was mainly used in construction. These tiny fibers
travel to the lungs and other body parts during inspiration. However, symptoms take some time to
develop. In the case of mesothelioma, it needs a minimum of 20 years after exposure to develop into a
malignancy. As it gets deposited on the mesothelium of the organs, it causes inflammation. The most
common sites of occurrence are the mesothelium of the lungs, stomach, heart, and testicles.
Incorrect Choices:

Page 12

672
Option B. Anthracosis: Associated with bronchial arthrofibrosis (BAF), anthracosis of the lungs is a blac
k staining of the bronchial mucosa that can obstruct the bronchial lumen. This illness typically manifest
s in an elderly non-smoker lady or man as a chronic course of dyspnea and/or cough. Carbon, silica, a
nd quartz particles accumulate in macrophages, mucosa, and submucosa, leading to anthracosis.
Option C. Hepatocellular carcinoma: HCC is the most common primary liver cancer. As this is a type of
primary liver cancer, where the liver is compromised i.e., they happen in patients with chronic liver dis
eases. This can be cirrhosis due to Hep B or Hep C infection. Thus the main cause of HCC is long-ter
m liver damage and scarring. Other causes of cirrhosis are alcohol abuse, autoimmune diseases, chro
nic liver inflammation, and iron overload in the body (hemochromatosis).
Option D. Renal cell carcinoma: As the name suggests it is the cancer of the kidney that originated in t
he lining of the PCTs. RCC is the most occurring adult kidney carcinoma (~ 90–95% of cases). The initi
al symptoms are blood in the urine, flank pain, abdominal mass or in the flank, weight loss, fever, high
BP, night sweats, and malaise. When RCC metastasizes, it commonly spreads to the LNs, lungs, liver,
adrenal glands, brain, or bones.

Solution for Question 9:


Correct Choice: A
Explanation:
• The most probable diagnosis is tuberculosis. It is a bacterial disease occurring primarily in the lungs. It
is a droplet disease which spreads via cough, sneeze, or spit. It is caused by Mycobacterium
tuberculosis. Apart from the lungs it can happen in the kidneys, spine, bones, and brain. Symptoms are
dependent on the site of infection. TB bacteria usually grow in the lungs (pulmonary TB). In the lungs it
shows as continuous cough lasting 3 weeks or longer, chest pain, blood in the sputum (phlegm from
deep inside the lungs).
• Granulomas are the body's response to chronic antigenic stimulants. They represent the body's first
line of response to Mtb infection and thus serve as a histopathological hallmark of TB. In one of the
studies to categorize TB granulomas three different types of granulomas are found: non-necrotizing,
necrotizing gummatous and necrotizing abscess-type granulomas.
Incorrect Choices:
Option B. Leprosy: It is a chronic infectious disease caused by Mycobacterium leprae. It primarily affect
s the foot, skin, peripheral nerves, mucosa of the upper respiratory tract, and eyes. If untreated it will le
ad to permanent disabilities. In this many severe disfiguring skin sores are formed and nerve damage o
ccurs in arms, legs, and skin.
Option C. Sarcoidosis: It is characterized by granulomas that can grow in any part of the body. Howeve
r, the most common sites are the lungs and LNs. This condition mostly affects young adults and charac
teristically presents with reticular opacities in the lungs and bilateral hilar lymphadenopathy. Sarcoidosi
s most often affects the lungs and may cause lung problems, such as:
• Persistent dry cough
• Shortness of breath
• Wheezing
• Chest pain

Page 13

673
• Sarcoidosis is an inflammatory disease that can affect multiple organs, including the lungs and lymph
nodes. It can cause enlarged lymph nodes, but it typically presents with respiratory symptoms and
constitutional symptoms like fatigue and weight loss.
Option D. Syphilis: It is an STD that is caused by bacteria. It starts with painless sore in the genital are
a, rectum, or mouth. It spreads via skin or mucous membrane contact. A single chancre marks the ons
et of the primary (first) stage of syphilis, but there may be multiple sores. The chancre is usually (but no
t always) firm, round, and painless. It appears at the location where syphilis enters the body. Rashes d
uring the second stage:
• Can appear when the primary chancre is healing or several weeks after the chancre heals.
• Usually, it does not cause itching.
• It can be rough, red, or reddish-brown spots on the palm and ventral side of the feet. However,
sometimes they resemble rashes caused by other diseases.
• Maybe so faint they are hard to notice

Solution for Question 10:


Correct Option B: Miliary tuberculosis
• The image showing lung specimen represents miliary tuberculosis.
• Miliary tuberculosis is a form of tuberculosis characterized by the widespread dissemination of
Mycobacterium tuberculosis throughout the body.
• In this condition, the bacteria spread through the bloodstream and affect multiple organs, including the
lungs.
• The term "miliary" refers to the appearance of small millet seed-like lesions, which can be seen on
imaging or in pathological specimens, giving the characteristic appearance seen in the image.
Incorrect Options:
Option A: Emphysema- Emphysema is a chronic lung condition characterized by the destruction of the
lung's air sacs, leading to the inability to effectively exchange oxygen and carbon dioxide. Emphysema
does not present with the characteristic appearance of millet seed-like lesions seen in the image.
Option C: Sarcoidosis- Sarcoidosis is a systemic inflammatory condition that can affect multiple organs
, including the lungs. It is characterized by the formation of granulomas, which are clusters of immune c
ells. While sarcoidosis can cause changes in the lung tissue, it does not typically present with the wide
spread millet seed-like lesions seen in miliary tuberculosis.
Option D: Asbestosis- Asbestosis is a lung disease caused by long-term exposure to asbestos fibers. It
is characterized by the scarring and fibrosis of lung tissue. Asbestosis does not present with the chara
cteristic appearance of millet seed-like lesions seen in the image.

Solution for Question 11:


Correct Option A.
• Asbestosis is a lung disease caused by long-term exposure to asbestos fibers. Inhalation of these
fibers leads to the formation of scar tissue and fibrosis in the lungs. It is characterized by the presence

Page 14

674
of asbestos bodies, which are asbestos fibers surrounded by iron-containing proteins, within lung
tissue. The disease can cause symptoms such as shortness of breath, coughing, and chest tightness.
In the histopathology picture, the presence of lung fibrosis and thickening of the pleura is consistent
with the diagnosis of asbestosis.
Incorrect Option:
Option B. Silicosis:
• Silicosis is a lung disease caused by the inhalation of crystalline silica dust. It leads to the
development of fibrous tissue in the lungs, similar to asbestosis. However, silicosis is characterized by
the presence of silica particles within the lung tissue rather than asbestos bodies. Occupational
exposure to industries such as mining, sandblasting, or stone cutting is commonly associated with
silicosis.
Option C. Tuberculosis:
• Tuberculosis (TB) is an infectious disease caused by Mycobacterium tuberculosis. It primarily affects
the lungs but can also involve other organs. TB is characterized by the formation of granulomas, which
are nodular inflammatory lesions, in the affected tissues. In the lungs, TB granulomas typically contain
caseous necrosis, which appears as cheesy or crumbly material on histopathology. While TB can
cause lung fibrosis in advanced cases, the thickening of the pleura mentioned in the question is not a
typical finding of tuberculosis.
Option D. Mesothelioma:
• Mesothelioma is a type of cancer that affects the mesothelium, the thin membrane that lines the chest
cavity and covers the lungs. It is primarily caused by exposure to asbestos fibers. Histopathologically,
mesothelioma is characterized by malignant cells forming a disorganized pattern within the
mesothelium. While mesothelioma can cause lung fibrosis in some cases, thickening of the pleura is a
more prominent feature associated with asbestosis rather than mesothelioma.

Solution for Question 12:


Correct Option B.
• CMV (Cytomegalovirus) associated organizing pneumonia is a lung condition caused by infection with
the CMV virus.
• It is characterized by organizing pneumonia, which refers to the accumulation of granulation tissue
within the lung's airspaces.
• This condition often presents with fever, cough, and breathlessness.
• To diagnose CMV-associated organizing pneumonia, the presence of CMV infection would need to be
confirmed through specific testing methods such as viral culture, polymerase chain reaction (PCR), or
immunohistochemical staining.
Incorrect Option:
Option A. Small cell lung cancer:
• Small cell lung cancer is a type of lung cancer characterized by the presence of small cells with high
nuclear-to-cytoplasmic ratios. These cancer cells often grow rapidly and can infiltrate the lung tissue.
However, without specific information from the lung biopsy, it is difficult to definitively diagnose small
cell lung cancer in this case. Clinical features, additional investigations, and specific

Page 15

675
immunohistochemical markers are typically required for an accurate diagnosis of small cell lung cancer.
Option C. Cryptococcal pneumonia:
• Cryptococcal pneumonia is a fungal infection caused by Cryptococcus neoformans. It commonly
affects immunocompromised individuals, particularly those with HIV/AIDS. Cryptococcus is a yeast-like
fungus, and in lung biopsies, the presence of encapsulated yeasts within the lung tissue is
characteristic of cryptococcal pneumonia. Special staining techniques, such as mucicarmine or PAS
stains, can be used to visualize the yeasts and confirm the diagnosis.
Option D. Tuberculosis:
• Tuberculosis (TB) is a bacterial infection caused by Mycobacterium tuberculosis. It primarily affects
the lungs but can also involve other organs. TB typically presents with symptoms such as fever, cough,
weight loss, and breathlessness. Lung biopsy findings consistent with TB would show granulomas,
which are nodular inflammatory lesions containing immune cells such as lymphocytes and
macrophages. Acid-fast staining can be used to detect the presence of acid-fast bacilli (AFB), the
characteristic staining pattern of M. tuberculosis.

Solution for Question 13:


Correct Option A.
• The patient's history of working in a factory for a prolonged period, along with the HRCT chest findings
of pleural thickening and fibrosis, points towards the diagnosis of asbestosis.
• Asbestosis is a lung disease caused by the inhalation of asbestos fibers, commonly encountered in
occupational settings.
• Prolonged exposure to asbestos can lead to the deposition of fibers in the lungs, triggering an
inflammatory response that results in fibrosis and scarring of lung tissue, including the pleura.
Incorrect options: Option B. Coal worker pneumoconiosis: Coal worker pneumoconiosis, also known as
black lung disease, is caused by the inhalation of coal dust. It typically presents with coal macules and
nodules in the lungs, but it does not cause pleural thickening and fibrosis as seen in the patient's case.
Option C. Silicosis: Silicosis is a lung disease caused by the inhalation of crystalline silica dust. It result
s in the formation of fibrotic nodules in the lungs but does not commonly cause pleural thickening and fi
brosis. Option D. Berylliosis: Berylliosis is a lung disease caused by exposure to beryllium, typically en
countered in certain industrial occupations. It primarily affects the lung parenchyma and does not com
monly involve pleural thickening and fibrosis as described in the patient's case

Solution for Question 14:


Correct Option A
• Based on the symptoms, radiographic findings, and biopsy results described, the most likely causative
agent for the disease in this case is thermophilic actinomycetes.
• These microorganisms, also known as thermophilic actinomycetes lung disease or farmer's lung, are
bacteria that commonly colonize organic materials such as hay, straw, or compost.

Page 16

676
• When the person inhales the spores or particles containing these bacteria, they can cause an
inflammatory response in the lungs.
• The symptoms of fever, cough, and dyspnea, along with the radiographic findings of diffuse ground
glass changes, are consistent with hypersensitivity pneumonitis, which is an immune-mediated lung
disease caused by repeated exposure to specific environmental antigens.
• In this case, the exposure to the thermophilic actinomycetes in the farming environment triggers the
immune response and leads to lung inflammation.
Incorrect options:
Option B. Aspergillus fumigatus: While Aspergillus species can cause lung infections, including allergic
bronchopulmonary aspergillosis (ABPA) and invasive aspergillosis, the described clinical presentation
and biopsy findings in the case do not match those typically associated with Aspergillus infections.
Option C. Actinobacter: Actinobacter is not a recognized pathogen associated with lung diseases in hu
mans. The term "Actinobacter" is not commonly used in medical literature.
Option D. Aspergillus flavus: Aspergillus flavus is a species of fungus known for producing toxic compo
unds called aflatoxins, which can contaminate food and cause health issues. However, it is not typically
associated with the clinical presentation and biopsy findings described in the case

Solution for Question 15:


Correct Option: A.
• Thymoma: Thymoma is the most common anterior mediastinal tumor. It arises from the thymus gland
and is typically slow-growing. Thymomas can be associated with autoimmune disorders and
myasthenia gravis.
Incorrect Option
Options: B Neurofibroma: Neurofibromas are benign tumors that arise from nerve tissue. They are mor
e commonly found in peripheral nerves and not specifically associated with the anterior mediastinum.
Option C Pericardial cyst: Pericardial cysts are fluid-filled sacs that develop in the pericardium, the sac
surrounding the heart. They are typically asymptomatic and do not represent a
common anterior mediastinal tumor.
Option D Bronchogenic cyst: Bronchogenic cysts are congenital abnormalities that arise from abnormal
development of the lung bud during embryogenesis. They are usually found in the mediastinum but ar
e not the most common tumor in this region

Solution for Question 16:


Correct Option A. Mycoplasma pneumoniae:
• Mycoplasma pneumoniae is the most common cause of atypical pneumonia.

Page 17

677
• It is a small bacterium that lacks a cell wall and is typically associated with mild respiratory symptoms
such as cough, sore throat, and low-grade fever.
• Atypical pneumonia refers to a milder form of pneumonia that often presents with non-specific
symptoms.
Incorrect options:
Option B. Klebsiella pneumoniae: Klebsiella pneumoniae is a bacterial species that can cause severe p
neumonia, particularly in individuals with underlying conditions such as alcoholism or compromised im
mune systems. It is associated with classic pneumonia symptoms rather than atypical pneumonia.
Option C. Hemophilus influenzae: Hemophilus influenzae can cause both typical and atypical pneumon
ia. However, Mycoplasma pneumoniae is more commonly implicated as the cause of atypical pneumon
ia.
Option D. Chlamydia: Chlamydia pneumoniae is a bacterial species that can cause atypical pneumonia
. However, Mycoplasma pneumoniae is more frequently identified as the primary cause of atypical pne
umonia.

Solution for Question 17:


Correct option C
• Fibroblastic foci are not a characteristic feature of bronchial asthma. Fibroblastic foci are commonly
seen in idiopathic pulmonary fibrosis (IPF), a distinct disease characterized by progressive lung
scarring and fibrosis.
Incorrect options:
• In a patient with a known case of bronchial asthma and difficulty in breathing, audible breath sounds
externally suggest significant airway obstruction. The other features commonly associated with
bronchial asthma include:
Option A: Thickening of bronchial wall: In bronchial asthma, chronic inflammation of the airways leads t
o hypertrophy and hyperplasia of smooth muscle cells, along with edema and infiltration of inflammator
y cells. This contributes to the thickening of the bronchial wall.
Option B: Increase in the number of airway goblet cells: Goblet cells produce and secrete mucus in the
respiratory tract. In asthma, there is an increase in goblet cell hyperplasia, leading to excessive mucus
production and airway obstruction.
Option D: Sub-basement membrane fibrosis: Chronic inflammation in asthma can lead to remodeling of
the airway wall, including thickening of the sub-basement membrane due to deposition of collagen and
other extracellular matrix components. This contributes to airway narrowing.

Solution for Question 18:


Correct Option:
Option A.

Page 18

678
• Acute respiratory infection: This option is correct. Ciliocytophthoria refers to the presence of ciliated
respiratory epithelial cells with cytoplasmic vacuoles containing inhaled particles, such as bacteria,
viruses, or debris. It is commonly observed in respiratory infections, particularly those affecting the
lower respiratory tract.
Incorrect Options:
Option B. Kartagener syndrome: This option is incorrect. Kartagener syndrome is a
genetic disorder characterized by a triad of symptoms: situs inversus (reversal of the normal organ posi
tioning), chronic sinusitis or bronchiectasis, and immotile cilia. While ciliocytophthoria can be observed
in Kartagener syndrome due to the presence of immotile cilia, it is not specific to this condition.
Option C. Situs inversus: This option is incorrect. Situs inversus refers to the mirror-image reversal of t
he normal organ positioning, where organs are located on the opposite side of the body. While ciliocyto
phthoria can be seen in situs inversus, it is not specific to this condition and can be observed in various
respiratory infections as well.
Option D. Cystic fibrosis: This option is incorrect. Cystic fibrosis is a genetic disorder characterized by
defective chloride transport, resulting in thickened secretions. While cystic fibrosis can predispose indiv
iduals to respiratory infections, ciliocytophthoria is not a specific feature of this condition.

Solution for Question 19:


The correct option is 3. Asbestosis.
Explanation of options:
Tuberculosis: Tuberculosis can cause chronic cough and chest pain, but it typically presents with caviti
es or nodules on chest X-ray, not diffuse interstitial opacities as seen in this patient.
Chronic Obstructive Pulmonary Disease (COPD): COPD is characterized by airflow limitation and typic
ally presents with symptoms such as chronic cough, shortness of breath, and decreased breath sound
s. However, COPD is associated with a different pattern on pulmonary function tests (obstructive patter
n) and is not caused by asbestos exposure.
Asbestosis: Asbestosis is a type of pneumoconiosis caused by prolonged exposure to asbestos fibers.
It presents with symptoms like chronic cough, shortness of breath, and chest pain. Chest X-ray shows
diffuse bilateral interstitial opacities, and HRCT may show "honeycombing" due to fibrosis. The restricti
ve pattern on pulmonary function tests is a characteristic finding in asbestosis.
Pneumonia: Pneumonia can cause acute symptoms like cough, shortness of breath, and chest pain. H
owever, on chest X-ray and HRCT, it typically presents with localized opacities or consolidation, not the
diffuse and bilateral pattern seen in this case.

Solution for Question 20:


Correct Option B:
• The most common lung finding in COVID-19 infection is diffuse alveolar damage (DAD). COVID-19 is
caused by the SARS-CoV-2 virus, which primarily affects the respiratory system. When the virus enters
the lungs, it can cause inflammation and damage to the alveoli, which are the tiny air sacs responsible

Page 19

679
for gas exchange.
• Diffuse alveolar damage is a histopathological pattern observed in the lungs of patients with severe
respiratory diseases, including COVID-19. It is characterized by injury to the alveolar epithelium,
inflammation, and the accumulation of fluid, protein, and inflammatory cells within the alveolar spaces.
This leads to impaired oxygenation and respiratory distress.
Incorrect Options:
Option A. Pulmonary infarction: Pulmonary infarction refers to the obstruction of the blood supply to a p
ortion of the lung, resulting in tissue death. While COVID-19 can cause vascular complications and blo
od clotting disorders, pulmonary infarction is not the most common lung finding associated with COVID
-19. The predominant lung pathology in COVID-19 is diffuse alveolar damage rather than localized infa
rction.
Option C. Endothelial injury: COVID-19 has been found to cause endothelial dysfunction and injury, lea
ding to vascular complications and the formation of blood clots. However, endothelial injury is not the m
ost common lung finding specifically associated with COVID-19. The primary lung pathology in COVID-
19 is diffuse alveolar damage.
Option D. Fibrin clots: COVID-19 can lead to a hypercoagulable state and an increased risk of blood cl
ot formation. While fibrin clots can be observed in the lungs of COVID-19 patients, they are not the mo
st common lung finding associated with the infection. The predominant lung pathology in COVID-19 is
diffuse alveolar damage.

Solution for Question 21:


Correct Option B - Thick layer of fibrin:
• In severe COVID-19 cases, fibrin and inflammatory cells accumulate in the alveoli, leading to diffuse
alveolar damage.
• Fibrin deposits and hyaline membranes can form within the alveoli, impairing gas exchange.
Incorrect Options:
Options A, C, and D are not a post-mortem finding seen in the lung after the death of a
patient from COVID-19.

Solution for Question 22:


Correct Option D - Squamous cell carcinoma:
• The elderly male smoker presenting with a chronic cough, significant weight loss, fatigue, and a high
serum calcium level is suggestive of squamous cell carcinoma of the lung.
• The raised serum calcium level is due to paraneoplastic syndrome.
Incorrect Options:
Option A - Large cell neuroendocrine tumor: It does not present with raised serum calcium levels.

Page 20

680
Option B - Small cell carcinoma: It does not present with raised serum calcium levels.
Option C - Adenocarcinoma: It is more common in non-smokers.

Solution for Question 23:


Correct Option B - Asbestosis:
• Chronic asbestos exposure is a risk factor for mesothelioma.
• Smoking has a synergistic effect with asbestos exposure.
Incorrect Options:
Options A, C, and D do not cause mesothelioma.

Solution for Question 24:


Correct Option B - Measles:
• The presence of Warthin-Finkeldey giant cells with large grapelike clusters of inclusion bodies is
suggestive of measles infection.
• The Warthin-Finkeldey giant cells are multinucleated cells that contain viral inclusion bodies and are
commonly seen in measles infections.

Incorrect Options:
Options A, C, and D are not associated with the presence of Warthin-Finkeldey giant cells.

Page 21

681
Barrett's Esophagus, Plummer Vinson syndrome,
Esophageal Tumors
1. Which of the following is a risk factor for esophageal squamous cell carcinoma?
(or)
Which of the following is a risk factor for esophageal squamous cell carcinoma?
A. Mediastinal fibrosis
B. Barrett's Esophagus
C. Caustic alkali burn
D. CMV
----------------------------------------
2. Which of the following viruses is involved in causing punched-out ulcers in a a 45-year-old chronic
smoker, and a biopsy taken from the edge of the ulcer showed multinucleated squamous epithelial cells
and eosinophilic Cowdry 'A' inclusions?
(or)
Which virus is involved in the viral oesophagitis showing eosinophilic Cowdry 'A' inclusions?
A. Cytomegalovirus (CMV)
B. Herpes simplex virus (HSV)
C. Adenovirus
D. Coxsackie virus
----------------------------------------
3. What's the likely diagnosis for a 53-year-old woman with dysphagia, burning tongue, worsening
exertional dyspnea, palpitations, and mucosal webs in the upper third of the esophagus on barium
swallow?
(or)
What disorder is most likely to be the cause of a female patient's dysphagia, burning sensation, and
mucosal webs in the upper third of the esophagus?

A. Plummer Vinson syndrome


B. Achalasia

682
C. Esophageal adenocarcinoma
D. Zenker's diverticulum
----------------------------------------
4. Which of the following is the most common cause of Esophageal perforation?
(or)
What is the most common cause of Esophageal perforation?
A. Mallory Weiss syndrome
B. Boerhaave syndrome
C. Iatrogenic
D. Esophageal varices
----------------------------------------
5. Which of the following conditions is associated with Barrett's oesophagus?
(or)
Which of the following conditions is associated with Barrett's esophagus?
A. Adenocarcinoma
B. Squamous cell carcinoma
C. Adenosquamous carcinoma
D. Achalasia
----------------------------------------
6. What is the likely cause of dysphagia in a 40-year-old woman with a 2-year history of difficulty
swallowing, increasing fatigue, iron deficiency anemia, and an upper endoscopy revealing annular
narrowing in the upper third of the esophagus with no inflammation or neoplasia evident on mucosal
biopsy?
(or)
Which of the following is the likely cause of dysphagia in a woman with, iron deficiency anemia, and the
upper third of the esophagus shows annular narrowing on endoscopy?
A. Achalasia
B. Barrett's esophagus
C. Zenker’s Diverticulum
D. Esophageal web
----------------------------------------
7. What is true about the condition of a 30-year-old male with severe pain during swallowing, cachexia,
CD4 count of 300 cells/mm3, and elevated white plaques on hyperemic, edematous esophageal
mucosa on upper GI endoscopy?
(or)
Which of the given statements is true regarding a 30-year-old male patient with severe pain during
swallowing, cachexia, a recent CD4 count of 300 cells/mm3, and upper GI endoscopy revealing
elevated, white plaques on a hyperemic and edematous esophageal mucosa?

Page 2

683
A. The plaques may progress into punched out lesions.
B. Can occur in immunocompetent patient also.
C. A biopsy will reveal the presence of both intranuclear and intracytoplasmic inclusions
D. The plaques are pseudomembranes formed as a result of infective esophagitis
----------------------------------------
8. What is the histological hallmark of the condition of a 44-year-old male with a 2-year history of
burning epigastric pain, dyspepsia, and a metallic taste in his mouth, who responded partially to
omeprazole and over-the-counter antacids and the following GI endoscopic findings?
(or)
What is the histological hallmark for the diagnosis in a 44-year-old with a burning epigastrium,
dyspepsia, and a metallic taste, partially responding to omeprazole, who underwent upper GI
endoscopy (image shown) with biopsy?

A. Signet ring cells seen on histology


B. Presence of keratin pearls and intercellular bridges
C. Goblet cells
D. Squamous to columnar metaplasia
----------------------------------------
9. Which of the following layers is not present in the esophagus?
A. Submucosa
B. Serosa
C. Muscularis propria
D. Mucosa
----------------------------------------
10. Which is the main presenting complaint associated with aganglionic megacolon?
A. Chronic diarrhea
B. Abdominal distension
C. Failure to pass stool
D. Hematochezia
----------------------------------------

Page 3

684
11. Which is a characteristic feature of dysphagia in patients with megaesophagus?
A. More to solids
B. More to liquids
C. Both liquids and solids
D. No dysphagia
----------------------------------------
12. A 55-year-old male presents with complaints of dysphagia, particularly for liquids, for the past
several months. He denies any significant weight loss or gastrointestinal bleeding. His radiological
findings show: Which of the following is the most appropriate management for this patient's condition?

A. Proton pump inhibitors


B. Esophageal manometry
C. Steroids
D. Heller’s myotomy
----------------------------------------
13. Which of the following medications is commonly associated with the risk of esophagitis?
A. Aspirin
B. Bisphosphonates
C. Acetaminophen
D. Ibuprofen
----------------------------------------
14. Match the following: 1 Candidal esophagitis A 2 CMV esophagitis B 3 Herpes esophagitis C

Page 4

685
1 Candidal esophagitis A

2 CMV esophagitis B

3 Herpes esophagitis C

A. 1-A, 1-C, 3-B


B. 1-C, 2-A, 3-B
C. 1-B, 2-A, 3-C
D. 1-C, 2-B, 3-A
----------------------------------------
15. Which of the following statements is true regarding the condition depicted in the esophagoscopy
image and biopsy from the esophagus?

Page 5

686
A. Its primary cause is Helicobacter pylori
B. Interleukin 5 plays a major its pathogenesis
C. Clinical findings involve hematemesis
D. Treatment typically involves triple therapy
----------------------------------------
16. Which of the following is a special stain used in the diagnosis of Barret’s esophagus?
A. Toluidine blue
B. Prussian blue
C. Alcian blue
D. H and E stain
----------------------------------------

Correct Answers
Question Correct Answer

Question 1 3
Question 2 2
Question 3 1
Question 4 3
Question 5 1
Question 6 4
Question 7 4
Question 8 3
Question 9 2
Question 10 3
Question 11 2
Question 12 4
Question 13 2

Page 6

687
Question 14 4
Question 15 2
Question 16 3

Solution for Question 1:


• Esophageal carcinoma:
• There are two sub-types: Squamous cell carcinoma is in the upper 2/3rd of the Esophagus
Adenocarcinoma is in the lower 1/3rd of the Esophagus
• Squamous cell carcinoma is in the upper 2/3rd of the Esophagus
• Adenocarcinoma is in the lower 1/3rd of the Esophagus
• Squamous cell carcinoma is in the upper 2/3rd of the Esophagus
• Adenocarcinoma is in the lower 1/3rd of the Esophagus
• Squamous Cell Carcinoma: Alcohol & tobacco use Intake of nitrosamines [smoked/ grilled food]
Caustic esophageal injury Deficiency of Zn/Mo/Vit A/ Vit C Achalasia Plummer-Vinson syndrome
Frequent consumption of very hot beverages Previous radiation therapy to the mediastinum
Esophageal diverticula Tylosis (hyperkeratosis palmaris et plantaris) is characterized by focal
thickening of the skin of the hands and feet and is associated with a very high lifetime risk of developing
squamous cell carcinoma of the esophagus
• Alcohol & tobacco use
• Intake of nitrosamines [smoked/ grilled food]
• Caustic esophageal injury
• Deficiency of Zn/Mo/Vit A/ Vit C
• Achalasia
• Plummer-Vinson syndrome
• Frequent consumption of very hot beverages
• Previous radiation therapy to the mediastinum
• Esophageal diverticula
• Tylosis (hyperkeratosis palmaris et plantaris) is characterized by focal thickening of the skin of the
hands and feet and is associated with a very high lifetime risk of developing squamous cell carcinoma
of the esophagus
• Alcohol & tobacco use
• Intake of nitrosamines [smoked/ grilled food]
• Caustic esophageal injury
• Deficiency of Zn/Mo/Vit A/ Vit C
• Achalasia
• Plummer-Vinson syndrome
• Frequent consumption of very hot beverages
• Previous radiation therapy to the mediastinum

Page 7

688
• Esophageal diverticula
• Tylosis (hyperkeratosis palmaris et plantaris) is characterized by focal thickening of the skin of the
hands and feet and is associated with a very high lifetime risk of developing squamous cell carcinoma
of the esophagus
Adenocarcinoma:
• Barrett's Esophagus [d/t GERD/ obesity]
Incorrect Options:
A. Option A -Mediastinalfibrosis:
• External beam irradiation but not mediastinal fibrosis is a risk factor for esophageal cancer.
• Therefore this option is incorrect
B. Option B - Barrett's Esophagus:
• Barrett's Esophagus is a risk factor for adenocarcinoma but not squamous cell carcinoma.
• Therefore this option is incorrect
D. Option D - CMV:
• It is not involved in squamous cell carcinoma.
• HPV DNA is found frequently in esophageal squamous cell carcinoma in high incidence regions
• Therefore, this option is incorrect

Solution for Question 2:


Correct Option B - Herpes simplex virus (HSV):
• The patient likely has viral oesophagitis caused by the herpes simplex virus.
• It is characterized by the following:- Punched-out ulcers on upper GI endoscopy The biopsy
shows cowdry A inclusions and multinucleated cells with nuclear molding and ground glass
appearance.
• Punched-out ulcers on upper GI endoscopy
• The biopsy shows cowdry A inclusions and multinucleated cells with nuclear molding and ground
glass appearance.

• Punched-out ulcers on upper GI endoscopy
• The biopsy shows cowdry A inclusions and multinucleated cells with nuclear molding and ground
glass appearance.

Page 8

689
Incorrect Options:
Option A - Cytomegalovirus (CMV):
• Shallow ulcerations
• Cytomegalic cells, typically 2-4 fold larger than normal, contain basophilic cytoplasmic & intranuclear
inclusion bodies surrounded by a clear halo, giving the appearance of an owl's eye within capillary
endothelium & stromal cells.
Option C - Adenovirus:
• Adenovirus is unlikely to cause esophagitis. Therefore this option is incorrect
Option D - Coxsackie virus:
• It rarely causes esophagitis; if it occurs, it is generally mild and does not cause ulcers.
• Therefore this option is incorrect

Solution for Question 3:


Correct Option A - Plummer Vinson syndrome:
Paterson-Brown-Kelly syndrome:
• The patient in the clinical vignette presents with dysphagia, esophageal webs, exertional dyspnea,
and palpitation. These symptoms suggest Paterson-Brown-Kelly syndrome, also known as
Plummer-Vinson syndrome. This syndrome is characterized by anemia and oesophageal webs. Iron
deficiency anemia Atrophic glossitis, cheilosis Webs in upper 1/3rd of Esophagus Most commonly seen
in middle-aged females Risk factor for hypopharyngeal squamous cell carcinoma
• Iron deficiency anemia
• Atrophic glossitis, cheilosis
• Webs in upper 1/3rd of Esophagus
• Most commonly seen in middle-aged females

Page 9

690
• Risk factor for hypopharyngeal squamous cell carcinoma
The patient in the clinical vignette presents with dysphagia, esophageal webs, exertional dyspnea, and
palpitation. These symptoms suggest Paterson-Brown-Kelly syndrome, also known as Plummer-Vinso
n syndrome. This syndrome is characterized by anemia and oesophageal webs.
• Iron deficiency anemia
• Atrophic glossitis, cheilosis
• Webs in upper 1/3rd of Esophagus
• Most commonly seen in middle-aged females
• Risk factor for hypopharyngeal squamous cell carcinoma
Iron deficiency anemia
Atrophic glossitis, cheilosis
Webs in upper 1/3rd of Esophagus
Most commonly seen in middle-aged females
Risk factor for hypopharyngeal squamous cell carcinoma
Incorrect Options:
Option B - Achalasia:
• Achalasia can cause dysphagia and anemia, however, barium swallow will show a bird's beak
appearance, not the esophageal webs.
Achalasia can cause dysphagia and anemia, however, barium swallow will show a
bird's beak appearance, not the esophageal webs.
Option C - Esophageal adenocarcinoma:
• Esophageal adenocarcinoma can cause anemia and dysphagia, however, an endoscopy will show
esophageal mass.
Esophageal adenocarcinoma can cause anemia and dysphagia, however, an endoscopy will show eso
phageal mass.
Option D - Zenker's diverticulum:
• It can present with halitosis, regurgitation, and dysphagia
• However, anemia and the presence of esophageal webs are unlikely in Zenker diverticulum
It can present with halitosis, regurgitation, and dysphagia
However, anemia and the presence of esophageal webs are unlikely in Zenker diverticulum

Solution for Question 4:


Correct Option C - Iatrogenic:
• The most common cause of oesophageal perforation is Iatrogenic.
• Signs and symptoms of esophageal perforation Chest and abdominal pain Fever Vomiting
Hematemesis Shock Subcutaneous emphysema

Page 10

691
• Chest and abdominal pain
• Fever
• Vomiting
• Hematemesis
• Shock
• Subcutaneous emphysema
• Chest and abdominal pain
• Fever
• Vomiting
• Hematemesis
• Shock
• Subcutaneous emphysema
Causes:
• The most common is iatrogenic, usually caused during endoscopy or balloon dilation of the
esophagus.
• Other causes include acid/alkali ingestion, trauma to the chest, and violent vomiting.
Incorrect Options:
Option A - Mallory Weiss syndrome:
• The most common esophageal lacerations are Mallory- Weiss tears, often induced by severe retching
or vomiting.
• It does not cause perforation.
Option B - Boerhaave syndrome:
• It can cause lacerations extending from the mucosa to the muscularis layer.
• It occurs after violent vomiting.
• It can sometimes cause oesophageal perforation but is not the commonest cause.
Option D - Esophageal varices:
• Varices do not cause esophageal perforation.

Solution for Question 5:


Correct Option A - Adenocarcinoma:
Barrett's esophagus:
• Intestinal metaplasia of squamous cells of the esophagus due to reflux of gastric acid
• Stratified squamous epithelium of the esophagus is replaced by nonciliated columnar epithelium with
goblet cells.

Page 11

692
• The intestinal-like columnar cell has goblet cells which secrete mucin → identified with a stain called
Alcian blue
• Increases risk of esophageal adenocarcinoma

Incorrect Options:
Option B - Squamous cell carcinoma:
• It is not associated with Barrett's esophagus.
• It is characterized by keratin pearls and intercellular bridges on histopathology.
Option C - Adenosquamous carcinoma:
• An uncommon type of esophageal cancer contains adenocarcinoma and squamous cell carcinoma
elements.
• Adenosquamous carcinoma is not associated with Barret's esophagus
Option D - Achalasia:
• Barret's esophagus is not associated with achalasia.

Solution for Question 6:


Correct Option D - Esophageal web:
• The patient described in the clinical vignette has dysphagia, annular narrowing in the upper third of
the esophagus, and iron deficiency anemia.
• This is a classic presentation of Plummer-Vinson syndrome, also known as Paterson-Brown-Kelly
syndrome.
• Esophageal webs are thin, membranous structures that can cause narrowing of the esophagus,
leading to dysphagia. This aligns with the findings of an annular narrowing in the upper esophagus
observed during the endoscopy.
Features:

Page 12

693
• Iron deficiency anaemia
• Atrophic glossitis, cheilosis
• Webs in the upper 1/3rd of Esophagus
• Most commonly seen in middle-aged females
• Risk factor for hypopharyngeal squamous cell carcinoma
Incorrect Options:
Option A - Achalasia:
• It is characterized by narrowing at the lower end of the esophagus, not the upper end
• Therefore this option is incorrect
Option B - Barrett's esophagus:
• It is characterized by intestinal metaplasia at the lower end of the esophagus.
• Therefore this option is incorrect
Option C - Zenker’s Diverticulum:
• Zenker's diverticulum presents as a defect in the mucosa and not annular narrowing

Solution for Question 7:


Correct Option D - The plaques are pseudomembranes formed as a result of infective esophagitis:
• The clinical scenario points to the diagnosis of candidal esophagitis. Candidiasis is characterized by
adherent, gray-white pseudomembranes composed of densely matted fungal hyphae and inflammatory
cells covering the esophageal mucosa.
Incorrect Options:
Option A - The plaques may progress into punched out lesions:
• This is not correct because punched out lesions are more characteristic of conditions like herpes
esophagitis, not esophageal candidiasis.
Option B - Can occur in immunocompetent patient also:
• This is not correct because esophageal candidiasis is more commonly seen in immunocompromised
individuals, such as those with HIV/AIDS, rather than in immunocompetent patients.
Option C - A biopsy will reveal the presence of both intranuclear and intracytoplasmic inclusions:
• The description of both intranuclear and intracytoplasmic inclusions is more characteristic of
cytomegalovirus (CMV) esophagitis rather than esophageal candidiasis.

Solution for Question 8:


Correct Option C - Goblet cells:
Barrett's esophagus:

Page 13

694
• The upper GI endoscopy shows patchy mucosa, the pale pink area is the normal esophagus and the
Red velvety areas are the areas of Barrett's transformation.
• Characterised by intestinal metaplasia within esophageal squamous mucosa. [i.e. squamous to
intestinal columnar metaplasia]
• There are goblet cells in the intestinal epithelium. Goblet cells contain acidic mucin. o Special stain:
Alcian blue stain · Hallmark of Barrett's Esophagus - Goblet cells
• Barrett's esophagus is a complication of chronic GERD
• Increased risk of esophageal adenocarcinoma.
o Special stain: Alcian blue stain
· Hallmark of Barrett's Esophagus - Goblet cells

Incorrect Options:
Option A - Signet ring cells seen on histology:

Page 14

695
• Signet ring cells can be seen in Gastric adenocarcinoma, not Barrett’s esophagus.
• Therefore, this option is incorrect
Signet ring cells can be seen in Gastric adenocarcinoma, not Barrett’s esophagus.
Therefore, this option is incorrect
Option B - Presence of keratin pearls and intercellular bridges:
• Keratin pearls and intercellular bridges are the hallmarks of squamous cell carcinoma.
• Therefore, this option is incorrect
Keratin pearls and intercellular bridges are the hallmarks of squamous cell carcinoma.
Option C - Squamous to columnar metaplasia:
• In Barrett's esophagus, there is a change in epithelium from squamous to columnar but presence of
goblet cells in histology is a hallmark feature.
In Barrett's esophagus, there is a
change in epithelium from squamous to columnar but presence of goblet cells in histology is a
hallmark feature.

Solution for Question 9:


Correct Option B - Serosa:
• The serosa is a layer of connective tissue covered by a mesothelium. The esophagus lacks a serosa
layer.
Incorrect Options:
Option A - Submucosa: The submucosa is a layer of loose connective tissue containing blood vessels,
lymphatics, and nerves. It provides support and nourishment to the mucosa.
Option C - Muscularis propria: The muscularis propria consists of two layers of smooth muscle: an inne
r circular layer and an outer longitudinal layer. These muscles are responsible for peristalsis, the rhyth
mic contraction that propels food through the esophagus into the stomach.
Option D - Mucosa: The mucosa is the innermost layer of the esophagus. It functions in secretion, abs
orption, and protection of the underlying tissues.

Solution for Question 10:


Correct Option C - Failure to pass stool:
• Hirschsprung's disease, also known as congenital aganglionic megacolon, is characterized by the
absence of ganglion cells in a segment of the intestinal wall, typically the rectum and sigmoid colon.
This absence of ganglion cells results in a constricted segment of the intestine, leading to functional
obstruction. Failure to pass meconium (the first stool passed by a newborn) or stool is a classic
presenting complaint of Hirschsprung's disease.
Incorrect Options:
Options A, B, and D are not the primary presenting complaints associated with Hirschsprung disease.

Page 15

696
Solution for Question 11:
Correct Option B - More to liquids:
• The megaesophagus is also known as achalasia cardia. The patient presents with dysphagia, which
is more in for liquids as compared to solids because solids create pressure on the esophagus, forcing it
to open.
Incorrect Options:
Option A
- More to solids: If dysphagia is more to the solids than to the liquids, it is due to esophageal cancer.
Options C and D - (Both liquids and solids and No dysphagia): Dysphagia in the megaesophagus is not
equal for both solids and liquids; it is more for liquids.

Solution for Question 12:


Correct Option D - Heller’s myotomy:
• The patient's symptoms of dysphagia, particularly for liquids, along with the absence of significant
weight loss or gastrointestinal bleeding, are suggestive of achalasia cardia. It is a motility disorder
characterized by loss of inhibitory neurons. Heller’s myomectomy is the preferred surgical management
of achalasia cardia.
Incorrect Options:
Option A - Proton pump inhibitors (PPIs):
• PPIs are not the appropriate management of achalasia cardia.
Option B - Esophageal manometry:
• Esophageal manometry is a diagnostic test used to evaluate achalasia cardia and is not a part of the
management.
Option C - Steroids:
• Steroids are not indicated in the management of achalasia cardia.

Solution for Question 13:


Correct Option B - Bisphosphonates:
• Bisphosphonates, commonly prescribed for the treatment of osteoporosis, can cause esophagitis by
eroding the esophageal lining if not taken properly. Patients are advised not to lie down for at least 30
minutes after taking bisphosphonates to prevent the medication from sticking to the esophagus, which
can lead to irritation and inflammation.
Incorrect Options:
Options A, C, and D are not common causes of esophagitis

Page 16

697
Solution for Question 14:
Correct Option D - 1-C, 2-B, 3-A:
• Pseudomembrane formation

• Candida has long pseudohyphae.


• Appearance: curdy white lesions
2. CMV esophagitis
• Shallow ulcers in the esophagus

• Owl eye appearance


• Blue color eye - Blue intranuclear inclusion

Page 17

698
• It also shows intracytoplasmic inclusions.
3. Herpes esophagitis
• Punched out ulcers in the esophagus.

• Three M's: Multinucleated, Molding, Margination - margins of the nucleus are darker.
• Can occur in immunocompetent people.

Solution for Question 15:


Correct Option B - Interleukin 5 plays a major its pathogenesis:
The given image of esophagoscopy shows stripes (just like a cat's) known as feline esophagus, and th
e histopathology shows an abundance of eosinophils(orange color). They are a
characteristic features of eosinophilic esophagitis. It is caused by allergy and interleukin-5 plays a
major role in its pathogenesis.
Incorrect Options:
Options A, C, and D are incorrect, as explained above.

Solution for Question 16:


Correct Option C - Alcian blue:
• Alcian blue is a special stain commonly used in histology to detect mucin-containing cells, such as
goblet cells. In Barrett's esophagus, which is characterized by the replacement of normal squamous
epithelium with metaplastic columnar epithelium containing goblet cells (hallmark feature), Alcian blue
staining can help to confirm the diagnosis.
Incorrect Options:
Options A, B, and D are not special stains used in the diagnosis of Barret’s esophagus.

Page 18

699
Page 19

700
Peptic Ulcer Disease,Gastric Polyps & Tumors
1. What tumor marker is most likely to be detected in a 50-year-old female patient with abdominal
discomfort, distension, a history of anemia treated with transfusions, and a mass in the fundus of the
stomach?
(or)
What tumor marker is most likely to be elevated in the case of a 50-year-old female patient with
abdominal discomfort, distension, a history of anemia treated with transfusions, and a stomach mass
located in the fundus on abdominal imaging?
A. CD20
B. CD117
C. CD15
D. CD3
----------------------------------------
2. What condition is associated with a 47-year-old male patient presenting with abdominal discomfort,
distension, and a mass in stomach with c-KIT mutation?
A. Duodenum is the most common site
B. The high propensity of malignant change
C. Arises from intestinal cells of Cajal
D. Histology shows squamous-shaped cells
----------------------------------------
3. In a 62-year-old male with a large gastric carcinoma staged as T1b, indicating early gastric
carcinoma, which specific structure is the tumor confined to according to AJCC TNM staging?
(or)
What structure is a T1b early gastric carcinoma (according to the AJCC TNM staging system) confined
to?
A. Mucosa
B. Mucosa and submucosa
C. Gastric wall without lymph node metastasis
D. Gastric glands
----------------------------------------
4. Which of the following statements is incorrect regarding the diagnosis of a 10-day-old infant with
forceful vomiting, failure to thrive, a non-tender, mobile olive-shaped epigastric mass, and signs of
dehydration?
(or)
Which of the following statements is incorrect regarding the cause of an infant's forceful vomiting, lack
of weight gain, dehydration, lethargy, and "olive-shaped" epigastric mass?
A. It is often associated with Edwards syndrome
B. This can be caused due to exposure to Azithromycin

701
C. Pyloromyotomy is the definitive treatment
D. None of the above
----------------------------------------
5. What is the probable diagnosis for a 69-year-old man experiencing nonspecific abdominal
discomfort, maintaining a stable weight, and having a medical history of rheumatoid arthritis while
undergoing medication, alongside the identification of the following gastric lesion during the endoscopic
examination?
(or)
The gastric lesion shown in the figure is resected following initial endoscopic discovery. Which of the
following is the most likely diagnosis?

A. Chronic gastric peptic ulcer


B. Gastric carcinoma
C. Ménétrier disease
D. Gastroesophageal reflux disease
----------------------------------------
6. Which gene is most likely mutated in a 35-year-old man with diffuse gastric thickening, signet-ring
cell infiltration on microscopic examination, and a family history of early-onset gastric carcinoma in his
father?
(or)
Which gene is most likely mutated in familial gastric cancer?
A. APC
B. CDH1
C. PMS2
D. p53
----------------------------------------
7. What's the key factor in the pathogenesis of a 40-year-old woman's condition, characterized by
burning epigastric pain relieved by antacids or food, recent tarry stools, microcytic hypochromic
anemia, and a bleeding mucosal defect (1.5cm diameter) in the antrum on gastroscopy with biopsy
revealing a lesion lacking mucosal lining cells and composed of amorphous cellular debris and
neutrophils?
(or)

Page 2

702
What is the most important factor in pathogenesis of peptic ulcer disease?
A. Achlorhydria
B. Acute ischemia
C. Gastrinoma
D. Helicobacter pylori infection
----------------------------------------
8. A 42-year-old man presents with long-standing abdominal pain after meals, which is relieved by
over-the-counter antacids. The patient has lost 9 kg (20 pounds) in the past year. Physical examination
reveals peripheral edema and ascites. Laboratory studies show decreased serum albumin but normal
serum levels of transaminases and gastrin. The patient is suspected to have a hyper-secretory gastric
disease. The gross and microscopic examination of this patient’s stomach would most likely show
which of the following pathologic changes? What gross and microscopic stomach changes are most
likely in a 42-year-old man with long-standing abdominal pain after meals, relieved by antacids,
peripheral edema, ascites, 9kg weight loss in the past 1 year, decreased serum albumin, normal
transaminases, and gastrin levels?
(or)
The gross and microscopic examination of the stomach of a patient with hyper-secretory gastric
disease would most likely show which of the following pathologic changes?
A. Atrophic gastritis
B. Enlarged rugal folds
C. Intestinal metaplasia
D. Multiple hemorrhagic ulcers
----------------------------------------
9. What is the most likely diagnosis for a 56-year-old female patient with a sore throat for 5 days,
dysphagia to solids, intermittent hiccups for 3 months, weight loss, no history of abdominal pain,
constipation, diarrhea, nausea, vomiting, fever, night sweats, smoking or alcohol use, no family history
of malignancy, and gastric biopsy showing the given image?

A. Adenocarcinoma
B. Gastric lymphoma
C. Leiomyosarcoma
D. Linitis plastic

Page 3

703
----------------------------------------
10. The presence of H.pylori can most likely predispose a patient to which of the following conditions?
(or)
The presence of H.pylori can most likely predispose a patient to which of the following conditions?
A. Gastric Lymphoma
B. Stromal tumors of stomach
C. Squamous metaplasia of esophagus
D. Mantle cell Lymphoma
----------------------------------------
11. What special histology stain should be used for a gastric biopsy in a 28-year-old woman presenting
with epigastric pain, water-brash salivation, occasional spicy food intake, mild tenderness in the
epigastric region, occasional melena, and advised gastric biopsy?
(or)
Which special histology stain is useful for identifying the pathogen associated with epigastric pain,
water-brash salivation, mild tenderness, and occasional melena in a 28-year-old woman undergoing a
gastric biopsy?
A. Sudan black stain
B. Warthin Starry stain
C. Congo red stain
D. Fite's stain
----------------------------------------
12. What is true about the diagnosis for a 57-year-old woman with sudden onset sharp chest pain in the
mid-sternal area, a history of on and off NSAID use, and endoscopic findings of punctate hemorrhagic
areas in the gastric mucosa without ulceration, confirmed by biopsy as mucosal erosions with edema
and hemorrhage?
A. It is likely associated with H-Pylori infection
B. It may cause malabsorption and hence lead to anemia
C. Itis often stress-induced
D. It can progress into adenocarcinoma of stomach
----------------------------------------
13. Which anatomical location may harbor metastatic gastric adenocarcinoma, presenting as a
Krukenberg tumor?
A. Ovary
B. Liver
C. Lung
D. Breast
----------------------------------------
14. Which clinical feature is the earliest manifestation of gastric adenocarcinoma?

Page 4

704
A. Dysphagia
B. Early satiety
C. Weight loss
D. Abdominal pain
----------------------------------------
15. Which of the following is the most common malignant gastric tumor?
A. Gastrointestinal stromal tumor
B. Diffuse large B cell lymphoma
C. Gastric adenocarcinoma
D. Maltoma
----------------------------------------
16. Which type of stress ulcer is associated with increased intracranial tension and occurs in the cardia
of the stomach?
A. Curling ulcer
B. Cushing ulcer
C. Duodenal ulcer
D. Gastric ulcer
----------------------------------------
17. In familial Zollinger-Ellison Syndrome (ZES), which syndrome is commonly associated with multiple
tumors and a genetic predisposition?
A. MEN-2 syndrome
B. Turner syndrome
C. MEN-1 syndrome
D. Cushing syndrome
----------------------------------------
18. Which of the following is a characteristic feature of infantile hypertrophic pyloric stenosis?
A. Bilious vomiting
B. Decreased feeding demands
C. Olive-shaped lump in the abdomen
D. Associated with Trisomy 17
----------------------------------------
19. What is the most common cause of gastric outlet obstruction in adults?
A. Hypertrophic pyloric stenosis
B. Gastric volvulus
C. Gastric cancer
D. NSAIDs use

Page 5

705
----------------------------------------
20. Which of the following is not a feature of type A chronic gastritis?
A. It is an autoimmune phenomenon
B. It most commonly involves the antrum
C. It results in pernicious anemia
D. It results in decreased HCl production
----------------------------------------
21. Which complication of peptic ulcer disease is most commonly associated with death?
A. Perforation
B. Hemorrhage
C. Obstruction
D. Malignancy
----------------------------------------
22. Which of the following statements regarding Helicobacter pylori is true?
A. It is a non-flagellated organism.
B. It is urease-negative.
C. It is not associated with the development of gastric adenocarcinoma.
D. Warthin starry silver stain gives a black color
----------------------------------------
23. Which of the following acts as a protective factor against Helicobacter pylori?
A. Mucus
B. Bicarbonate
C. COX-1
D. Pepsin
----------------------------------------
24. Which is the most common source of bleeding as a complication of duodenal ulcer?
A. Left gastric artery
B. Inferior mesenteric artery
C. Gastroduodenal artery
D. Splenic artery
----------------------------------------
25. Which of the following statements corresponds to a gastric ulcer? It is more common than duodenal
ulcers It occurs along the lesser curvature of the stomach It is not associated with the risk of cancer Its
most common complication is bleeding from left gastric artery
A. 1 and 2
B. 1, 2 and 3

Page 6

706
C. 2 and 4
D. 1 and 4
----------------------------------------

Correct Answers
Question Correct Answer

Question 1 2
Question 2 3
Question 3 2
Question 4 4
Question 5 1
Question 6 2
Question 7 4
Question 8 2
Question 9 2
Question 10 1
Question 11 2
Question 12 3
Question 13 1
Question 14 2
Question 15 3
Question 16 2
Question 17 3
Question 18 3
Question 19 3
Question 20 2
Question 21 1
Question 22 4
Question 23 3
Question 24 3
Question 25 3

Solution for Question 1:


Correct Option B - CD117:
• This patient, with a history of abdominal discomfort, distension, and a stomach mass with GIST
features, is likely to have a positive CD117 expression in this tumor.

Page 7

707
• Gastrointestinal Stromal Tumors mainly consist of interstitial cells of Cajal and lead to abnormal
function and disturbances in the regulation of gastrointestinal (GI) motility. Resulting in changes in
bowel habits, abdominal pain, bloating, or symptoms of obstruction.
• Ways in which a GIST or tumor of the interstitial cells of Cajal can affect normal function:
• The most important immunohistochemical marker of GISTs is the expression of KIT (CD117), which is
found in the vast majority (>90%) of GISTs.
• Most (>80%) gastric GISTs also express CD34.
Incorrect Options:
Option A - CD20:
• It is not associated with GIST but is found in many other tumors, such as CLL.
Option C - CD15:
• CD15 and CD30 are Hodgkin lymphoma markers but have no association with GIST.
Option D - CD3:
• It is a marker of acute leukemias having T-cell origin.

Solution for Question 2:


Correct Option C - Arises from intestinal cells of Cajal:
• This patient, with a history of abdominal discomfort, distension, and a mass having the c-kit mutation,
most likely has a Gastrointestinal stromal tumour (GIST).
• GIST is the most common mesenchymal tumour of the abdomen.
• It arises from benign pacemaker cells called intestinal cells of Cajal.
• The majority of tumours have activating mutations of c-KIT / PDGFRA tyrosine kinases which respond
to kinase inhibitors.
Incorrect Options:
Option A - Duodenum is the most common site:
• The stomach is the most common site of GIST, not the Duodenum.
• Therefore this option is incorrect
Option B - High propensity of malignant change:
• GIST has a very low propensity for malignant change, and 90% of stomach GISTs are found to be
benign.
• Therefore this option is incorrect
Option D - Histology shows squamous-shaped cells:
• The histology of GIST shows elongated spindle-shaped cells/ plumper epithelioid cells.
• Therefore this option is incorrect

Page 8

708
Solution for Question 3:
Correct Option B - Mucosa and submucosa:
• According to the AJCC TNM staging of the tumour, T1b is the stage when the tumour invades the
submucosa.
• So, the tumour is confined to the mucosa and submucosa at this stage.
Incorrect Options:
Option A - Mucosa:
• At the T1b stage, the tumour has invaded the submucosa, so it is not confined to mucosa only.
Option C - Muscularis propria:
• The tumour invading the muscularis propria is at stage T2, not T1b.
Option D - Sub-serosal connective tissue:
• Involvement of the sub-serosal connective tissue is seen in stage T3.
• The tumour is proposed to be at stage T3 if it penetrates sub-serosal connective tissue without
invasion of the visceral peritoneum or adjacent structures.

Solution for Question 4:


Correct Option D - None of the above:
Congenital pyloric stenosis:
• This patient with forceful vomiting after feedings, failure to thrive, weight loss, dehydration, lethargy,
and a firm, non-tender, mobile, "olive-shaped" epigastric mass most likely has Congenital pyloric
stenosis.
• It is caused by hypertrophy of the circular muscle layer of the pylorus.
• Hypertrophy of pyloric musculature → Obstruction, non-bilious projectile vomiting.
• It typically presents in male infants within the first several days to weeks of life and is generally not
seen at the time of birth.
• It is often associated with Edwards syndrome and Down syndrome.
• This can be caused due to exposure to Azithromycin or Erythromycin
• Pyloromyotomy is the definitive treatment.

Solution for Question 5:


Correct Option A - Chronic gastric peptic ulcer:
• This patient with abdominal discomfort, a long-standing history of analgesic use for symptomatic
relief, and the image showing characteristic radiating folds of gastric mucosa starting at the ulcer
margins, most likely has a chronic gastric peptic ulcer.

Page 9

709
• The incidence of peptic ulcer disease (PUD) is falling along with the reduced prevalence of H. pylori
infection (owing to successful H. pylori eradication therapy).
• However, PUD in patients older than 60 years has increased due to growing NSAID use (esp.
selective COX-2 inhibitors).
• The most common form of PUD occurs within the gastric antrum or duodenum as a result of chronic
H. pylori-induced antral gastritis, which is associated with increased gastric acid secretion and
decreased duodenal bicarbonate secretion.
Risk Factors for Peptic Ulcer Disease
• Helicobacter pylori infection
• Cigarette use (synergizes with H. pylori for gastric PUD)
• Chronic obstructive pulmonary disease
• Illicit drugs, e.g., cocaine, that reduce mucosal blood flow
• NSAIDs (potentiated by corticosteroids)
• Alcoholic cirrhosis (primarily duodenal PUD)
• Psychological stress (can increase gastric acid secretion)
• Endocrine cell hyperplasia (can stimulate parietal cell growth and gastric acid secretion)
• Zollinger-Ellison syndrome (PUD of stomach, duodenum, and jejunum)
• Viral infection (CMV, herpes simplex virus)
Incorrect Options:
Option B - Gastric carcinoma:
• It presents with features of: Upper GI bleeding Gastric outlet obstruction Significant weight loss
• Upper GI bleeding
• Gastric outlet obstruction
• Significant weight loss
• Upper GI bleeding
• Gastric outlet obstruction
• Significant weight loss
Option C - Ménétrier disease:
• It is characterized by foveolar hyperplasia leading to non-specific GIT-related symptoms and
protein-losing gastropathy.
• Endoscopy findings include prominent rugae involving the gastric fundus.
Option D - Gastroesophageal reflux disease:
• It is characterized by epigastric discomfort, most commonly occurring in adults with a sedentary
lifestyle.
• It is most commonly reported after the intake of a meal in obese individuals.

Page 10

710
Solution for Question 6:
Correct Option B - CDH1:
• This patient with familial gastric cancer most likely has a mutation in CDH1, the gene encoding the
cellular adhesion protein E-cadherin.
• E-cadherin mutations account for a significant proportion of familial gastric cancers and Lobular
carcinoma of the breast.
Incorrect Options:
Option A - APC:
• APC is mutated in the majority of colorectal adenocarcinomas and intestinal types of gastric cancer.
Option C -PMS2:
• MSH2 and PMS2 are both mismatch repair genes that are mutated in Lynch syndrome.
Option D - p53:
• p53 is mutated in Li Fraumeni syndrome.

Solution for Question 7:


Correct Option D - Helicobacter Pylori infection:
• H. pylori is a bacterium that most commonly causes peptic ulcer disease and gastritis.
• H. pylori multiply in the mucus layer of the stomach lining and duodenum. The bacteria secrete an
enzyme called urease that converts urea to ammonia. This ammonia protects the bacteria from
stomach acid.
• As it multiplies, it erodes the stomach tissue, which leads to gastritis and gastric ulcer.
• The most common symptoms are: Dull or burning pain in your stomach more often a few hours after
eating and at night Bloating Nausea and vomiting Indigestion Burping Loss of appetite Dark stools
• Dull or burning pain in your stomach more often a few hours after eating and at night
• Bloating
• Nausea and vomiting
• Indigestion
• Burping
• Loss of appetite
• Dark stools
• H. Pylori is diagnosed using the stool antigen test, and endoscopy, if required, is done.
• It is treated most commonly using triple therapy, which includes two antibiotics and a proton pump
inhibitor for 14 days.
• Anti – H. pylori Regimens Proton pump inhibitor Amoxicillin Clarithromycin Metronidazole One week
regimens* Omeprazole (20 mg BD) or 1.0 g BD 500 mg BD - Esomeprazole (20 mg BD) or - 250 mg
BD 400 mg BD Lansoprazole (30 mg BD) or 1.0 g BD - 400 mg BD Pantoprazole (40 mg BD) or
Rabeprazole (20 mg BD) Omeprazole (20 mg BD) 500 mg TDS - 400 mg TDS Two weeks – twice

Page 11

711
daily regimens Omeprazole (20 mg) or 750 mg - 400 mg Lansoprazole (30 mg) or - 250 mg 400 mg
Pantoprazole (40 mg) 750 mg 500 mg -
• Dull or burning pain in your stomach more often a few hours after eating and at night
• Bloating
• Nausea and vomiting
• Indigestion
• Burping
• Loss of appetite
• Dark stools
Anti – H. pylori Regimens
Proton pump inhibitor
Amoxicillin
Clarithromycin
Metronidazole
One week regimens*
Omeprazole (20 mg BD) or
1.0 g BD
500 mg BD
-
Esomeprazole (20 mg BD) or
250 mg BD
400 mg BD
Lansoprazole (30 mg BD) or
Pantoprazole (40 mg BD) or

Rabeprazole (20 mg BD)


Omeprazole (20 mg BD)
500 mg TDS
400 mg TDS
Two weeks – twice daily regimens
Omeprazole (20 mg) or
750 mg
400 mg
Lansoprazole (30 mg) or
250 mg
Pantoprazole (40 mg)

Page 12

712
500 mg
Incorrect Options:
Option A - Achlorhydria:
• Achlorhydria cannot cause peptic ulcers.
• Achlorhydria is a state in which Hydrochloric acid is not produced in the stomach.
• Hydrochloric acid plays an integral role in the digestion of food.
• It is caused in cases of Pernicious anaemia, hypothyroidism and gastric cancer.
Option B - Gastric ischemia:
• Ischemic ulcers often occur at gastric sites unusual for a peptic ulcer, and in some cases, they look
macroscopically different from peptic ulcers.
• The most common symptom is severe gastrointestinal bleeding.
• Pain is typical for peptic ulcers and has rarely been noted by patients.
Option C - Gastrinoma:
• Gastrinoma is a rare cause of peptic ulcers.
• Gastrinomas are gastrin-expressing well-differentiated pancreatic or duodenal Neuroendocrine
tumours that often have malignant behaviour and are associated with the Zollinger-Ellison syndrome.
• It is characterized by gastric acid hypersecretion resulting in peptic ulcer disease, usually in the
duodenum, gastroesophageal reflux, and diarrhoea.

Solution for Question 8:


Correct Option B - Enlarged rugal folds:
• Ménétrier disease or Hyperplastic hypersecretory gastropathy is a rare disorder.
• It is associated with excessive secretion of transforming growth factor (TGF)-α, an EGF receptor
(EGFR) ligand, and hyperactivation of the epidermal growth factor receptor on gastric epithelial cells.
• Excessive EGFR activation causes diffuse hyperplasia of the foveolar epithelium within the body and
fundus.
• Hypoproteinemia occurs due to albumin loss across the gastric mucosa
• Secondary symptoms such as weight loss, diarrhoea, and peripheral oedema are common.
• Enlarged rugal folds are present in the body and fundus, but the antrum is generally spared
• The risk of gastric adenocarcinoma is increased.

Page 13

713
Histology shows the following characteristic features:
• Hyperplasia of surface and glandular mucous cells
• Marked intraepithelial lymphocytosis.
• Diffuse or patchy glandular atrophy
• The glands are elongated with a corkscrew-like appearance and cystic dilation present.
• Treatment of Ménétrier disease is supportive, with intravenous albumin and parenteral nutritional
supplementation. In cases associated with herpesvirus, CMV, or H. pylori, treatment of the infection
may be helpful. Antibodies that block epidermal growth factor receptor activation are effective in many
cases. In severe cases gastrectomy remains a therapeutic option.
Incorrect Options:
Option A - Atrophic gastritis:
• Gastric mucosal atrophy is the loss of appropriate glands.
• It occurs when glands damaged by inflammation are replaced by connective tissue (scarring) or
glandular structures inappropriate for the location.
• Atrophic gastritis is the main consequence of long-standing Helicobacter pylori infection and is related
to the development of gastric cancer.
• Common symptoms include upper abdominal pain, nausea and vomiting, loss of appetite, peptic
ulcers and weight loss.
• It is not associated with this case scenario.
Option C - Intestinal metaplasia:
• Intestinal metaplasia occurs when gastric cells are replaced by intestinal cells.
• Risk factors for Intestinal metaplasia include Helicobacter pylori infection, high salt intake, smoking,
alcohol consumption, and chronic bile reflux.
• Intestinal metaplasia is more common in people with chronic acid reflux or gastroesophageal reflux
disease.
• People will often have no noticeable symptoms.

Page 14

714
Option D - Multiple hemorrhagic ulcers:
• It will often present with blood in vomitus or stool.
• Mostly seen in patients with long-standing peptic ulcer disease.
• It can only be seen on endoscopy.

Solution for Question 9:


Correct Option B - Gastric Lymphoma:
• Extranodal lymphomas can arise in any tissue, most commonly in the GI tract and the stomach.
• Nearly 5% of all gastric malignancies are primary lymphomas.
• The most common are indolent extranodal marginal zone B-cell lymphomas.
• In the gut, these tumours are often referred to as lymphomas of MALT (Mucosa-associated lymphoid
tissue) or MALTomas.
• Histologically, gastric lymphomas form a dense lymphocytic infiltrate in the lamina propria.
• GI lymphomas may disseminate as discrete small nodules or diffusely infiltrate the wall in other
regions.
• Other choices do not demonstrate the lymphoid lesion depicted in the image.
• Also, the epithelial lining is intact and shows normal epithelial cells.
Incorrect Options:
Option A - Tubular adenocarcinoma:
• Adenocarcinomas are cancers that develop in gland cells.
• Tubular adenocarcinoma is the most common histologic type of early gastric carcinoma.
• It tends to form polypoid or fungating masses grossly and histologically demonstrates irregularly
distended, fused, or branching tubules of various sizes, often with intraluminal mucus, nuclear, and
inflammatory debris.

Page 15

715
Option C - Mucinous Adenocarcinoma::
• Mucinous adenocarcinoma accounts for 10% of gastric carcinoma.
• Histologically it is characterized by extracellular mucinous pools that constitute at least 50% of tumour
volume.
• The tumour cells can form glandular architecture and irregular cell clusters, with occasional scattered
signet ring cells floating in the mucinous pools.

Option D - Linitis Plastica:


• Linitis plastica is a rare type of adenocarcinoma.
• It spreads to the muscles of the stomach wall and thickens it.
• Histopathologically, linitis plastica is characterized by the diffuse growth of malignant cells with signet
ring features and is usually associated with marked submucosal fibrosis and gastric wall thickening.

Page 16

716
Solution for Question 10:
Correct Option A - Gastric Lymphoma:
• H. pylori infection is the primary pathologic cause of the development of the stomach's low-grade,
mucosa-associated lymphoid tissue (MALT) lymphoma.
• Gastric tissue normally does not contain MALT but may acquire it in response to chronic H. pylori
infection.
• Chronic inflammation causes the proliferation of T-cells and B-cells due to antigen presentation.
• Malignant transformation occurs in a small percentage of B-cells and results in lymphoma.
• The malignant process appears to be driven to a large degree by chronic H. pylori infection because
H. pylori eradication causes lymphoma regression in most cases.
B. Option B - Stromal tumors of stomach:
Cell of Origin:
• GISTs originate from the interstitial cells of Cajal (ICC), which are pacemaker cells in the
gastrointestinal tract responsible for regulating gut motility.
Mutations:
• The most common mutation in GISTs is in the CKIT gene (CD117), leading to activation of tyrosine
kinase. Other mutations include PDGFRβ mutations and succinate dehydrogenase (SDH) mutations,
which are more common in pediatric GIST.
Mechanism:
• CKIT mutation results in increased tyrosine kinase activity, contributing to the development of GISTs.
Treatment:
• The primary treatment for GISTs is surgery, with the goal of complete resection.

Page 17

717
• Tyrosine kinase inhibitors, such as imatinib, are used in the treatment of GISTs. Imatinib targets the
abnormal tyrosine kinase activity associated with CKIT mutations.
C. Option C -Squamousmetaplasiaof esophagus:
• It is associated with carcinoma of esophagus.
• It has no association with H. pylori infection.
D. Option D -Mantlecell lymphoma:
• Mantle cell lymphoma is a B-cell lymphoma that develops from malignant B-lymphocytes within a
region of the lymph node known as the mantle zone.
• The signs and symptoms depend on the extent of the affected region of the body.
• H. pylori has no predisposition to it.

Solution for Question 11:


Correct Option B - Warthin starry stain:
• Warthin-Starry stain is a stain based on silver nitrate used in histology.
• This type of stain is commonly called silver staining.
• It is used for the visualisation of spirochetes but also the detection of Helicobacter pylori and
microsperediates.
Incorrect Options:
Option A - Sudan black stain:
• Sudan Black stain is used for the visualisation of lipids.
• It is used for staining various lipids, such as phospholipids, sterols and neutral triglycerides.
• Microscopic examination of stool using Sudan stain to detect fat is the best screening test for fat
malabsorption.
• It is not used for H.pylori detection.
Option C - Congo red stain:
• Congo red stain is used to visualise amyloid in muscle and nerve fresh frozen sections in patients with
amyloidosis.
• It is not used for H.Pylori detection.
Option D - Fite's stain:
• Fite's Acid-fast stain is used to detect Lepra bacilli (M. leprae).
• It is not used for H.Pylori detection

Solution for Question 12:


Correct Option C - It is often stress-induced:

Page 18

718
The scenario describes acute gastritis:
• Gastritis is inflammation of the gastric mucosa.
• It results from mucosal injury and when neutrophils are present in the lesion.
• Acute gastritis refers to the clinical situation of gastric mucosal erosions but not mucosal ulcers.
• Acute gastritis is also known as hemorrhagic gastritis or acute erosive gastritis.
• The most common causes of acute gastritis are infections.
• Acute gastritis can be associated with using non-steroidal anti-inflammatory drugs, alcohol,
corticosteroids, chemotherapy, and severe stress.
• The gross appearance of acute gastritis is multiple, scattered, punctate, hemorrhagic areas in the
gastric mucosa.
• It helps differentiate acute gastritis from peptic ulcers, which tend to be solitary and larger.
• Microscopically, the gastric mucosa of a patient with acute gastritis is likely to reveal mucosal
erosions, scattered neutrophils, edema, and possibly hemorrhage.
Incorrect Options:
Option A - It is likely associated with H-Pylori infection:
• H.Pylori usually causes chronic gastritis and hence mostly associated with the same
Option B - It may cause malabsorption and hence lead to anemia:
• Malabsorption and anemia are often associated with chronic gastritis.
Option D - It can progress into adenocarcinoma of stomach:
• A progression into adenocarcinoma is often associated with chronic gastritis

Solution for Question 13:


Correct Option A - Ovary:
• Gastric adenocarcinoma can spread to distant sites, and one common site of metastasis is the ovary,
where it can manifest as Krukenberg tumor. This tumor is characterized by bilateral ovarian
involvement and the presence of signet ring cells, which are indicative of gastric origin.
Incorrect Options:
• Options B, C, and D are not the location of the Krukenberg tumor.

Solution for Question 14:


Correct Option B - Early satiety:
• Early satiety, or feeling full soon after beginning to eat, is the earliest clinical manifestation of gastric
adenocarcinoma. This symptom occurs due to the tumor obstructing the passage of food through the
stomach, leading to reduced stomach capacity and a sensation of fullness even with small amounts of
food.

Page 19

719
Incorrect Options:
• Options A, C, and D are not the earliest manifestations of gastric adenocarcinoma.

Solution for Question 15:


Correct Option C - Gastric adenocarcinoma:
• Gastric adenocarcinoma is the most common malignant gastric tumor. It originates from the glandular
epithelial cells lining the stomach. Adenocarcinomas account for the majority of all stomach cancers.
Incorrect Options:
• Options A, B, and D are not the common malignant gastric tumors.

Solution for Question 16:


Correct Option B - Cushing ulcer:
• Cushing ulcer is a type of stress ulcer that occurs as a result of increased intracranial pressure,
typically due to head injury, brain surgery, or other intracranial pathologies. Increased intracranial
pressure leads to activation of the Vagus nerve and an increase in parasympathetic activity, resulting in
excessive gastric acid secretion. Cushing ulcers typically occur in the cardia of the stomach.
Incorrect Options:
Option A - Curling ulcer:
• Curling ulcers are stress ulcers that occur in the duodenum rather than the stomach. They are
associated with conditions such as severe burns, hypovolemia, and ischemia.
Options C and D - Duodenal ulcer & Gastric ulcer:
• Gastric and duodenal ulcers are a type of peptic ulcer that is not associated with a rise in intracranial
pressure.

Solution for Question 17:


Correct Option C - MEN-1 syndrome:
• Familial Zollinger-Ellison Syndrome (ZES) is associated with Multiple Endocrine Neoplasia type 1
(MEN-1) syndrome, also known as Wermer syndrome. MEN-1 syndrome is a rare autosomal dominant
disorder characterized by the development of multiple tumors involving endocrine glands.

Incorrect Options:
• Options A, B, and D are not associated with Zollinger-Ellison syndrome.

Page 20

720
Solution for Question 18:
Correct Option C - Presence of an olive-shaped lump in the abdomen:
• Infantile hypertrophic pyloric stenosis (IHPS) is a condition characterized by hypertrophy of the pyloric
muscle, leading to obstruction at the pyloric sphincter. One of the classic signs of IHPS is the presence
of an olive-shaped mass in the upper abdomen, typically palpable after feeding when the stomach is
full.

Incorrect Options:
Option A - Bilious vomiting: Vomiting in infantile hypertrophic pyloric stenosis is typically non-bilious, as
the obstruction occurs proximal to the bile duct.
Option B - Decreased feeding demands: Infants with IHPS often exhibit increased feeding demands an
d may appear hungry shortly after feeding.
Option D - Associated with Trisomy 17: Congenital hypertrophic pyloric stenosis is associated with chro
mosomal abnormalities such as Trisomy 17 and 21.

Solution for Question 19:


Correct Option C - Gastric cancer:
• Gastric outlet obstruction occurs when a blockage in the pathway from the stomach impedes the
passage of food. Gastric cancer is the most common cause of gastric outlet obstruction in adults.

Incorrect Options:
• Options A, B, and D are not the most common cause of gastric obstruction.

Solution for Question 20:


Correct Option B - It most commonly involves the antrum:
• Type A chronic gastritis primarily affects the fundus and body of the stomach, not the antrum. It is
characterized by an autoimmune phenomenon in which antibodies are produced against the parietal
cells of the stomach. This autoimmune response results in decreased production of hydrochloric acid
(HCl) and intrinsic factors. Consequently, impaired absorption of vitamin B12 occurs, leading to
pernicious anemia or megaloblastic anemia.
Incorrect Options:
Options A, C, and D are true regarding type A chronic gastritis, as explained above.

Solution for Question 21:


Correct Option A -Perforation:

Page 21

721
• Perforation of a peptic ulcer is the most severe complication and is often associated with the highest
mortality rate. When a peptic ulcer perforates, it can lead to peritonitis, which can further lead to death.
Incorrect Options:
Option B - Hemorrhage: While hemorrhage is the most common complication of peptic ulcer disease, it
is not the most commonly associated with death.
Option C and D are not common causes of death in peptic ulcer disease.

Solution for Question 22:


Correct Option D - Warthin starry silver stain gives a black color:
• Helicobacter pylori is a flagellated bacterium known for its ability to colonize the human stomach.
When stained with Warthin starry silver stain, it gives a black color.
Incorrect Options:
Option A - It is a non-flagellated organism:
• Helicobacter pylori is a flagellated bacterium, which contributes to its ability to move and colonize the
gastric mucosa.
Option B - It is urease-negative:
• Helicobacter pylori is urease-positive, meaning it produces the enzyme urease, which allows it to
hydrolyze urea to ammonia and carbon dioxide.
Option C - It is not associated with the development of gastric adenocarcinoma:
• Helicobacter pylori infection is a well-established risk factor for the development of gastric
adenocarcinoma.

Solution for Question 23:


Correct Option C - COX-1:
• COX-1 (cyclooxygenase-1) acts as a protective factor against Helicobacter pylori infection.
Incorrect Options:
Option A - Mucus:
• Mucus secretion is important for protecting the gastric mucosa from hydrochloric acid, not H. pylori.
Option B - Bicarbonate:
• Bicarbonate secretion is protective against pepsin, not H. pylori.
Option D - Pepsin:
• Pepsin is not a protective factor; rather, it is a damaging factor.

Page 22

722
Solution for Question 24:
Correct Option C - Gastroduodenal artery:
• The gastroduodenal artery, a branch of the common hepatic artery, supplies blood to the stomach and
duodenum. In cases of duodenal ulcer, erosion of the ulcer into the gastroduodenal artery or one of its
branches can lead to significant bleeding, resulting in gastrointestinal hemorrhage. The gastroduodenal
artery is particularly vulnerable to erosion due to its proximity to the duodenum.
Incorrect Options:
Options A, B, and D are not commonly involved in bleeding in duodenal ulcers.

Solution for Question 25:


Correct Option C - 2 and 4:
2: It occurs along the lesser curvature of the stomach: Gastric ulcers are more commonly found along t
he lesser curvature of the stomach, as compared to duodenal ulcers, which are typically located in the
duodenum.
4: Its most common complication is bleeding from the left gastric artery: Gastric ulcers have a higher te
ndency to erode into blood vessels, leading to bleeding. The left gastric artery, which runs along the les
ser curvature of the stomach, is particularly vulnerable to erosion from gastric ulcers.
Incorrect Options:
1: It is more common than duodenal ulcers: Gastric ulcers are less common than duodenal ulcers.
3: It is not associated with the risk of cancer: Gastric ulcers are associated with an increased risk of ca
ncer.

Page 23

723
Malabsorption Syndrome,Inflammatory Bowel
Disease,Intestinal Carcinoma
1. A 31-year-old male patient presented with abdominal pain, weight loss, and steatorrhea. Duodenal
biopsy showed villous atrophy with crypt hyperplasia. The given disease is associated with which of the
following?
(or)
Which of the following is associated with villous atrophy and crypt hyperplasia found on duodenal
biopsy?
A. Blood group B
B. HLA-DQ3
C. HLA-DR4
D. HLA-DQ2
----------------------------------------
2. A 37-year-old male patient presented with diarrhoea, weight loss, and joint pain. HPE showed foamy
macrophages in lamina propria, PAS-positive and diastase-resistant granules inside macrophages. The
foam cells are ZN stain negative. Given the symptoms what must be the causative agent?
(or)
What is the likely causative agent in a 37-year-old male presenting with diarrhea, weight loss, and joint
pain, whose histopathological examination reveals foamy macrophages in lamina propria with
PAS-positive, diastase-resistant granules, and ZN stain negativity?
A. Tuberculosis
B. Acinetobacter
C. H. pylori
D. Tropheryma whipplei
----------------------------------------
3. Consider the following statements regarding Whipple's disease and mark the correct statement:
Periodic-Acid Schiff shows positive granular staining within Macrophages. Optimal treatment regimen
involves an initial phase followed by a maintenance phaseInitial phase (2 - 4 weeks): ceftriaxone or
penicillin G Maintenance phase (one year): trimethoprim-sulfamethoxazole
(or)
Consider the following statements regarding Whipple's disease and mark the correct statement:
Periodic-Acid Schiff shows positive granular staining within Macrophages. Optimal treatment regimen
involves an initial phase followed by a maintenance phaseInitial phase (2 - 4 weeks): ceftriaxone or
penicillin G Maintenance phase (one year): trimethoprim-sulfamethoxazole
A. 1 and 2 are true and 2 is the correct explanation of 1
B. 1 is true and 2 is false
C. 1 and 2 are true and 2 is not the correct explanation of 1
D. 2 is true and 1 is false
----------------------------------------

724
4. Which of the following antibodies is likely to present a 30-year-old female with malabsorption and
iron deficiency anemia revealed complete villous atrophy on duodenal histopathological biopsy?
(or)
Which of the following antibodies is likely to be present in celiac disease?
A. Anti-endomysial antibodies
B. Anti-goblet cell antibodies
C. Anti-saccharomyces cerevisiae antibodies
D. Antineutrophil cytoplasmic antibodies
----------------------------------------
5. What's the most likely mutation in a 27-year-old male with recurrent abdominal pain, dark, tarry
stools, and hyperpigmented macules on lips, buccal mucosa, and perioral region, with a family history
of similar skin findings in the father, and a jejunal polyp on endoscopy with the following histological
findings?
(or)
What is the most probable mutation in a 27-year-old male with recurrent abdominal pain, dark tarry
stools, and hyperpigmented macules on lips and buccal mucosa, who presents with a jejunal polyp on
endoscopy, given a family history of similar skin findings in the father and histology of the polyp is given
below?

A. TSC 1 gene
B. PTEN gene
C. STK11 gene
D. APC gene
----------------------------------------
6. What's the likelihood of malignancy in a 32-year-old male with bleeding per rectum, a family history
of colon cancer (father and brother affected), and numerous polyps seen on colonoscopy?
(or)
A 32-year-old male patient has bleeding per rectum and a family history of colon cancer affecting both
his father and only sibling. On colonoscopy, numerous polyps were seen. What are the chances of
malignancy in this disease?
A. 100%
B. 75%

Page 2

725
C. 50%
D. 25%
----------------------------------------
7. What type of ulcer is typically seen in a 40-year-old female patient presenting with abdominal pain,
diarrhea, anorexia, weight loss, markedly increased ESR, and exhibiting the following on a sagittal
ultrasound?
(or)
Which type of ulcer is indicated by an elevated ESR and "Club Sandwich appearance of bowel walls"
on abdominal ultrasound?

A. Longitudinal ulcers
B. Transverse ulcers
C. Punched out ulcers
D. None of the above
----------------------------------------
8. What is the most appropriate statement regarding a 5-year-old child with diarrhea, failure to thrive,
positive anti-tTG antibodies, and insignificant abdominal examination?
(or)
What is true regarding the pathology of a child with a positive anti-tTG serum?
A. Increase in villous: crypt ratio
B. Decrease in intraepithelial lymphocytes
C. Elongated hyperplastic and tortuous crypts
D. Distended macrophages with PAS-positive granules in lamina propria
----------------------------------------
9. Where does the base of the ulcer lie in a 27-year-old male presenting with abdominal pain,
tenesmus, bloody diarrhea, and weight loss, with a colonoscopy showing an ulcer with a wide base and
a narrow mouth and a positive stool sample for trophozoites?
(or)
In a 27-year-old male with abdominal pain, tenesmus, bloody diarrhea, and weight loss, a positive stool
sample for trophozoites, where does the base of the ulcer observed during colonoscopy predominantly
lie?

Page 3

726
A. Mucosa
B. Submucosa
C. Muscularis layer
D. Serosa
----------------------------------------
10. A 35-year-old female patient on oral contraceptive pills complaints of intermittent abdominal pain
and diarrhea for the last few months. colonoscopy reveals rectal sparing and the gross appearance of
the ileum is shown in the image below. Which of the following is a specific feature related to this
patient's underlying condition?
(or)
A 35-year-old female patient on oral contraceptive pills visits the clinic with complaints of intermittent
abdominal pain and diarrhea for the last few months. A colonoscopy reveals rectal sparing and the
gross appearance of the ileum is shown in the image below. Which of the following is a specific feature
related to this patient's underlying condition?

A. Transmural involvement
B. Presence of polyps
C. Limited to mucosa
D. Superficial broad-based ulcers
----------------------------------------
11. A 40-year-old patient presented with intermittent diarrhea, weight loss, and joint pain. Upon
examination, mouth ulcers and rashes on her back and abdomen are seen. Her barium enema image is
shown below. What antibody findings and endoscopy results would be expected in her case?
(or)
In a 40-year-old patient with intermittent diarrhoea, weight loss, joint pain, mouth ulcers, skin rashes,
and characteristic barium enema findings, what antibody and endoscopy results would be most
appropriate for her suspected diagnosis?

Page 4

727
A. ASCA positive with superficial broad-based ulcers
B. Anti tTG positive with decreased villous: crypt ratio
C. P-ANCA positive with pseudo polyp
D. ASCA-positive and Transmural ulcers
----------------------------------------
12. What is the characteristic feature of the underlying condition in a 25-year-old female presenting with
low-grade fever, weight loss, fatigue, crampy abdominal pain, episodic diarrhea, postprandial bloating,
right lower quadrant tenderness, and findings of thickening, edema, luminal narrowing, and
cobblestone appearance of the terminal ileum on capsule endoscopy?
(or)
Which is a characteristic feature of the underlying condition in a patient with features such as thickening
of the terminal ileum, edema, and cobblestone mucosa?
A. Additional typical findings include crypt abscesses and pseudopolyps
B. Inflammation and ulceration are limited to mucosa and submucosa with sparing of deeper layers
C. It can affect any portion of the gastrointestinal tract, but the proximal jejunum is the most common
site of involvement
D. It can cause fistula formation between loops of the affected bowel
----------------------------------------
13. What gastrointestinal disease is most likely causing joint pain and diarrhea in a 25-year-old female
patient affecting the large joints of the legs?
(or)
Which of the following gastrointestinal diseases is most likely to be implicated as the cause of joint
problems accompanied by diarrhoea?
A. Amoebic colitis
B. Acute appendicitis
C. Diverticulosis
D. Ulcerative colitis
----------------------------------------
14. A 45-year-old male patient was diagnosed with Familial adenomatous polyposis (FAP) syndrome
via genetic studies of the APC gene. Which of the following statements is appropriate regarding this

Page 5

728
condition?
(or)
Which of the following statements is appropriate regarding Familial adenomatous polyposis (FAP)
syndrome?
A. Prophylactic colectomy only reduces the risk of cancer
B. If not treated will lead to cancer in 10 % cases
C. Females are more affected than males
D. Gardner FAP is often associated with Medulloblastoma
----------------------------------------
15. What is the most likely diagnosis for the 41-year-old male with advanced, poorly differentiated
colorectal cancer, a family history of early-onset colon cancer in multiple relatives, and the presence of
multiple osteomas, desmoid tumors, and supernumerary teeth?
(or)
What is the diagnosis for a patient who has undergone left colectomy and also presents with multiple
osteomas, desmoid tumors, and supernumerary teeth?
A. Classical FAP
B. Gardner syndrome
C. Peutz jeghers syndrome
D. Cowden syndrome
----------------------------------------
16. What is true regarding the diagnosis of a 19-year-old male with generalized tonic-clonic seizures
but no previous history of seizures, a family history of colectomy in the father, and a brain MRI
revealing an infratentorial cerebellar mass?
(or)
In a 19-year-old male presenting with generalized tonic-clonic seizures with no prior seizure history, his
father has a history of colectomy, and an infratentorial cerebellar mass on MRI, what diagnostic
consideration aligns with the presentation?
A. He may have supernumerary and overcrowding of teeth
B. A colonoscopy may not be indicated
C. PTEN mutation may be observed
D. Histology of the cerebellar mass will be consistent with a glioma
----------------------------------------
17. A 20-year-old girl with abdominal pain, diarrhea, and occasional constipation underwent an
endoscopy. A jejunal lesion as shown in the image was found, a biopsy was done, and the observed
histology is given below. What could be the probable diagnosis?
(or)
What is the most likely diagnosis for a 20-year-old female with these endoscopic and biopsy findings?

Page 6

729
A. Peutz-Jeghers syndrome
B. Cronkhite Canada syndrome
C. Familial adenomatous polyposis
D. Hereditary non-polyposis colonic cancer
----------------------------------------
18. What is the most associated carcinoma in a 41-year-old female patient who underwent right
hemicolectomy for colorectal cancer, which on biopsy turned out to be poorly differentiated
adenocarcinoma, also has a family history of early-onset colorectal cancer on her maternal side of the
family?
(or)
In a 41-year-old female with colorectal carcinoma, a right hemicolectomy revealed poorly differentiated
adenocarcinoma. With a family history of early-onset malignancies in her maternal aunt, grandmother,
and uncle, which is the most associated carcinoma for the syndrome diagnosed for this patient?
A. Testicular carcinoma
B. Leukemia
C. Ovarian cancer
D. Sarcoma
----------------------------------------
19. Which of the following tumors is not associated with BRAF mutation?
A. Adenocarcinoma Colon
B. Medullary thyroid carcinoma

Page 7

730
C. Papillary thyroid carcinoma
D. Hepatocellular carcinoma
----------------------------------------
20. Which of the following is the most likely predisposing lesion or disorder that leads to
adenocarcinoma of the colon, in a patient with a negative family history?
(or)
Which of the following is the most likely predisposing lesion or disorder that leads to adenocarcinoma of
the colon, in a patient with a negative family history?
A. Tubular adenoma
B. FAP syndrome
C. Hyperplastic polyp
D. HNPCC syndrome
----------------------------------------
21. What would a biopsy from the region with a prominent fold in the colon of a 60-year-old woman with
right-sided colonic adenocarcinoma likely reveal?
A. Tubular adenoma
B. Hyperplastic polyp
C. Peutz-Jeghers polyp
D. Sessile serrated adenoma
----------------------------------------
22. What is the probable histological finding in the appendix of a 58-year-old chronic smoker diagnosed
with small cell carcinoma of the lung, who presented with acute right-sided abdominal pain, underwent
an appendectomy, and had "coat button" lesions in the gross specimen?
(or)
What is the most likely histological finding of appendix in a 58-year-old chronic smoker diagnosed with
small cell carcinoma of the lung, presenting with acute right-sided abdominal pain and 'coat button'
lesions in the appendix?
A. Hematopoieticc cells
B. Kulchitsky cells
C. Neuroglial cells
D. Chromaffin cells
----------------------------------------
23. Which statement is incorrect regarding the conditions of two patients: 1. A 35-year-old woman with
thickened skin in certain areas, a family history of thyroid and endometrial cancer, and small polyps
found during endoscopy, and 2. A 10-year-old boy with developmental delay, learning difficulties, and
multiple polyps in the jejunum detected during endoscopy.
(or)
Which statement is incorrect regarding the presented patients: the 35-year-old woman with a family
history of thyroid and endometrial cancer, skin thickening, and small polyps, and the 10-year-old boy

Page 8

731
with developmental delay, learning difficulties, and multiple polyps in the jejunum revealed by
endoscopy?
A. Macrocephaly is a common feature
B. Lipomas and Hemangiomas are common
C. Loss of function mutation of PTEN is seen
D. Both patients have a defect on chromosome 10
----------------------------------------
24. A patient presents with the following: Consider the following statements regarding the diagnosis
and, choose the incorrect statement regarding the same:
(or)
Consider the following statements regarding Cronkhite-Canada Syndrome and, choose the incorrect
statement regarding the same:

A. It is of unknown etiology
B. Onychodystrophy and Alopecia are seen
C. Age at presentation ranges from 25yrs to 45 yrs
D. The polyps are hamartomatous and respond to systemic steroid and antiplasmin therapy
----------------------------------------
25. Which of the following statements about Abetalipoproteinemia is correct?
A. It is caused by a mutation in the APOE gene.
B. It primarily affects liver function.

Page 9

732
C. Intestinal manifestations include the presence of fat.
D. Red blood cells exhibit a smooth, round shape.
----------------------------------------
26. Which of the following microscopic findings is associated with MYO5B gene mutation?
A. Presence of Gastric Hyperplasia
B. Microvillus Inclusions
C. Dysplastic Changes in Intestinal Epithelium
D. Absence of Crypts of Lieberkühn
----------------------------------------
27. Which of the following statements is true regarding Hyperplastic Polyps?
A. Characterized by an increase in the size of cells.
B. Serrated appearance only in the lower part of the mucosa.
C. Predominantly found in the rectosigmoid region.
D. Affects young individuals.
----------------------------------------
28. Which of the following is a characteristic feature of carcinoid tumors when observed under light
microscopy?
A. Tubular pattern
B. Glandular pattern
C. Nesting pattern
D. Papillary pattern
----------------------------------------

Correct Answers
Question Correct Answer

Question 1 4
Question 2 4
Question 3 1
Question 4 1
Question 5 3
Question 6 1
Question 7 2
Question 8 3
Question 9 3
Question 10 1

Page 10

733
Question 11 4
Question 12 4
Question 13 4
Question 14 1
Question 15 2
Question 16 4
Question 17 1
Question 18 3
Question 19 4
Question 20 1
Question 21 4
Question 22 2
Question 23 2
Question 24 3
Question 25 3
Question 26 2
Question 27 3
Question 28 3

Solution for Question 1:


Correct Option D - HLA-DQ2:
• Gluten-sensitive enteropathy, also known as celiac sprue, has a definitive association with HLA
DQ2/DQ8.
• It is also associated with other immune diseases, including type 1 diabetes, thyroiditis, and Sjögren
syndrome.
Diagnosis of celiac sprue is made by:
• Multiple duodenal biopsies, at least two, show villous atrophy, crypt hyperplasia, and lymphocytic
infiltration.
• Noninvasive serologic tests are generally performed prior to biopsy. The most sensitive test is the
measurement of IgA antibodies against tissue transglutaminase. IgA anti-endomysial antibodies can
also be present.
• Skin biopsy shows destruction to dermal papillae by IgA resulting in Dermatitis herpetiformis.

Page 11

734
Page 12

735
Incorrect Options:
Option A - Blood group B:
• Celiac disease is not associated with blood group B
Option B - HLA-DQ3:
• Diabetes mellitus type I, Graves disease, and idiopathic guttate hypomelanosis are associated with
HLA-DQ3
Option C - HLA-DR4:
• Rheumatoid arthritis and spondyloarthropathies are associated with HLA-DR4

Solution for Question 2:


Correct Option D - Tropheryma whipplei:
• According to the given clinical scenario, the most probable diagnosis is whipple's disease
• Tropheryma whipplei, a gram-positive bacteria, is the cause of Whipple's illness.
• It is a systemic condition that affects the cardiovascular, central neurological system, joints, and
vascular systems in addition to the gastrointestinal tract's malabsorption.
• The inflammatory response to the organism is less in the diseased population. It comprises a type 1
T-cell response that is compromised and alters macrophage activation and activity. The disease is
linked to the HLA B27 haplotype.
• Weight loss, diarrhoea, arthralgias, fever, and stomach pain are the main symptoms.
• Periodic acid-Schiff (PAS) staining, electron microscopy, or polymerase chain reaction (PCR) on
intestinal biopsy samples can be used to identify Whipple's illness.
• The presence of T. whipplei inside macrophages is a more significant indication of active disease than
their presence outside the macrophages. The presence of PAS +ve, diastase-resistant granules inside
foamy macrophages harboring rod-shaped bacilli.

Page 13

736
• Antibiotic therapy is the cornerstone of Whipple's illness treatment. Antibiotics that cross the
blood-brain barrier should be used since the CNS may be affected even without symptoms.
Incorrect Options:
Option A - Mycobacterium Tuberculosis:
• The gold standard for diagnosing tuberculosis remains culture; histology is frequently inconclusive.
• Granulomas with caseating necrosis, conglomerate epithelioid histiocytes, and excessive submucosal
inflammation are some of the histological signs of TB.
Option B - Acinetobacter:
• Multidrug-resistant Acinetobacter baumannii is known to colonize the gastrointestinal tract but rarely
causes GI infection.
Option C - H.pylori:
• H. pylori appears as short, curved, or spiral-shaped bacilli sitting on the stomach mucosa's epithelial
surface or mucus layer. Further down in the stomach pits, it can be detected. An irregular mucosal
layer, loss of apical mucin, and lymphoid aggregates in response to reactive atypia can be seen as
morphological alterations.

Solution for Question 3:


Correct Option A - 1 and 2 are true and 2 is the correct explanation of 1:
• It is the correct choice as both statements are true, and statement 2 appropriately explains the
treatment regimen for Whipple's disease based on the diagnosis provided in statement 1.
Incorrect Options:
Option B - 1 is true and 2 is false:
• This option is incorrect because both statements 1 and 2 are true. Statement 1 accurately describes
the histopathological findings of Whipple's disease, and statement 2 provides the correct treatment
regimen.
Option C - 1 and 2 are true, and 2:
• This option is incorrect because statement 2 is, in fact, a correct explanation of statement 1. The
positive granular staining within macrophages observed on Periodic-Acid Schiff staining is indicative of
Whipple's disease, and the treatment regimen described in statement 2 is appropriate for this condition.
Option D - 2 is true and 1 is false:
• This option is incorrect because both statements 1 and 2 are true. Statement 1 accurately describes
the histopathological findings of Whipple's disease, and statement 2 provides the correct treatment
regimen.

Solution for Question 4:


Correct Option A - Anti-endomysial antibodies:

Page 14

737
• From the given case scenario suggesting features of malabsorption, iron deficiency anemia and
complete villous atrophy, a diagnosis of celiac disease can be made.
• Serologic studies of celiac disease include: Anti-endomysial antibodies Tissue transglutaminase [tTG]
antibodies: most specific Antigliadin antibodies
• Anti-endomysial antibodies
• Tissue transglutaminase [tTG] antibodies: most specific
• Antigliadin antibodies
• Anti-endomysial antibodies
• Tissue transglutaminase [tTG] antibodies: most specific
• Antigliadin antibodies
• Complications of celiac disease include: Increased risk for enteropathic-associated T-cell lymphoma
IDA malabsorption and skin complications
• Increased risk for enteropathic-associated T-cell lymphoma
• IDA
• malabsorption and skin complications
• Treatment: Gluten-free diet Cereal substitution with maize and rice Dapsone for dermatitis
herpetiformis Corticosteroids
• Gluten-free diet
• Cereal substitution with maize and rice
• Dapsone for dermatitis herpetiformis
• Corticosteroids
• Increased risk for enteropathic-associated T-cell lymphoma
• IDA
• malabsorption and skin complications
• Gluten-free diet
• Cereal substitution with maize and rice
• Dapsone for dermatitis herpetiformis
• Corticosteroids
Incorrect Options:
Option B - Anti-goblet cell antibodies:
• Autoimmune enteropathy, which is most common in children, is caused by anti-enterocyte antibodies,
also known as anti-goblet cell antibodies, and culminates in persistent diarrhoea with small intestinal
villous atrophy.
Option C - Anti-saccharomyces cerevisiae antibodies:
• Anti-Saccharomyces cerevisiae antibodies (ASCA) are immune proteins often found in persons with
inflammatory bowel disease (IBD), a set of chronic illnesses caused by an autoimmune process and
characterised by swollen and damaged intestinal lining tissues.
Option D - Antineutrophil cytoplasmic antibodies:

Page 15

738
• Antineutrophil cytoplasmic antibodies (ANCA) are autoantibodies directed against antigens identified
in neutrophil and monocyte cytoplasmic granules. ANCA testing is often used to diagnose or exclude
Wegener's granulomatosis and microscopic polyangiitis.

Solution for Question 5:


Correct Option - Option- C) STK11 gene:

• This image is a histology of the jejunal polyp and it shows arborizing bundles of smooth muscle
surrounded by small intestinal glandular epithelium. The clinical presentation of recurrent abdominal
pain, dark tarry stools, and the presence of hamartomatous polyps in the jejunum with characteristic
histology, along with the mucocutaneous hyperpigmentation, is consistent with Peutz-Jeghers
syndrome. Peutz-Jeghers syndrome is caused by mutations in the serine/threonine kinase 11 (STK11)
gene, also known as LKB1. This gene is a tumor suppressor gene, and mutations in STK11 are
associated with the development of hamartomatous polyps in the gastrointestinal tract, as well as
mucocutaneous pigmentation.

This image is a histology of the jejunal polyp and it shows arborizing bundles of smooth muscle surrou
nded by small intestinal glandular epithelium.
The clinical presentation of recurrent abdominal pain, dark tarry stools, and the presence of hamartom
atous polyps in the jejunum with characteristic histology, along with the mucocutaneous hyperpigmenta
tion, is consistent with Peutz-Jeghers syndrome.
Peutz-Jeghers syndrome is caused by mutations in the serine/threonine kinase 11 (STK11) gene, also
known as LKB1. This gene is a tumor suppressor gene, and mutations in STK11 are associated with th
e development of hamartomatous polyps in the gastrointestinal tract, as well as mucocutaneous pigme
ntation.
Incorrect Options:
The other options (A) TSC1 gene, (B) PTEN gene, and (D) APC gene are associated with different gen
etic syndromes:

Page 16

739
• TSC1 gene is associated with tuberous sclerosis.
• PTEN gene is associated with Cowden syndrome.
• APC gene is associated with familial adenomatous polyposis (FAP)

Solution for Question 6:


Correct Option A - 100%:
• Colorectal cancer develops in 100% of untreated FAP patients, often before age 30.
Colorectal cancer develops in 100% of untreated FAP patients, often before age 30.
Familial adenomatous polyposis (FAP)
• Autosomal dominant
• Numerous colorectal adenomas by teenage years.
• Mutation of adenomatous polyposis coli gene (APC).
• At least 100 polyps are necessary for the diagnosis of classic FAP,
• High risk of causing colorectal cancer,
• Prophylactic colectomy is standard therapy
Autosomal dominant
Numerous colorectal adenomas by teenage years.
Mutation of adenomatous polyposis coli gene (APC).
At least 100 polyps are necessary for the diagnosis of classic FAP,
High risk of causing colorectal cancer,
Prophylactic colectomy is standard therapy
INCORRECT OPTIONS:
Options B, C and D are incorrect and the correct option is explained above.

Solution for Question 7:


Correct Option B - Transverse ulcers:
• Intestinal tuberculosis occurs as a result of drinking contaminated milk
• It is a complication of protracted advanced secondary tuberculosis, secondary to swallowing
coughed-up infective material.
• Organisms are trapped in mucosal lymphoid aggregates of the small and large intestines. Resulting in
inflammatory enlargement with ulceration of the overlying mucosa in the ileum.
• Multiple transverse small intestinal ulcers with undermined edges can be seen.

Page 17

740
• Sonographic appearance of a small amount of ascitic fluid between the radially oriented bowel loops
in patients with tuberculous peritonitis is described as a “Club Sandwich appearance of bowel walls”.
Incorrect Options:
Option A - Longitudinal ulcers:
• Longitudinal ulcers are seen in typhoid or enteric fever.
Option C - Punched out ulcers:
• Punched-out ulcers are seen in herpes simplex esophagitis.
Option D - None of the above:
• Multiple transverse small intestinal ulcers with undermined edges can be seen in intestinal
tuberculosis.

Solution for Question 8:


Correct Option C - Elongated hyperplastic and tortuous crypts:
• Gluten-sensitive enteropathy, another name for celiac disease, is an autoimmune condition affecting
the small intestine.
• The consumption of gluten by those who are vulnerable to it causes the manifestation of this disease.
• Patients' genetics determines their susceptibility. Some celiac disease patients may not exhibit
symptoms, while others may experience diarrhoea and failure to thrive.
• Disease typically affects the mucosa of the small bowel. Anti-endomysial antibodies and anti-tissue
transglutaminase antibodies, often reported as negative, weakly positive, or positive, are the two
antibodies evaluated (by ELISA quantified numerically).
• A duodenal mucosal biopsy is typically the following step and the gold standard for the diagnosis,
which shows crypt elongation and hyperplasia, intraepithelial lymphocytosis, and villous atrophy.
• All celiac disease patients are advised to adhere to a tight gluten-free diet.
Gluten-sensitive enteropathy, another name for celiac disease, is an autoimmune condition affecting th
e small intestine.
The consumption of gluten by those who are vulnerable to it causes the manifestation of this disease.
Patients' genetics determines their susceptibility. Some celiac disease patients may not exhibit sympto
ms, while others may experience diarrhoea and failure to thrive.
Disease typically affects the mucosa of the small bowel. Anti-endomysial antibodies and anti-tissue tra
nsglutaminase antibodies, often reported as negative, weakly positive, or positive, are the two antibodi
es evaluated (by ELISA quantified numerically).
A duodenal mucosal biopsy is typically the following step and the gold standard for the diagnosis, whic
h shows crypt elongation and hyperplasia, intraepithelial lymphocytosis, and villous atrophy.
All celiac disease patients are advised to adhere to a tight gluten-free diet.
Incorrect Options:
Option A - Increase in villous: crypt ratio:
• Patients with celiac disease have an increased ratio of crypts to villi.

Page 18

741
• Crypt hyperplasia is characterised by extended grooves instead of short crypts in a healthy intestinal
lining.
• Due to repeated gluten exposure, the villi have shrunk and become flat.
Patients with celiac disease have an increased ratio of crypts to villi.
Crypt hyperplasia is characterised by extended grooves instead of short crypts in a
healthy intestinal lining.
Due to repeated gluten exposure, the villi have shrunk and become flat.
Option B - Decrease in intraepithelial lymphocytes:
• A feature of the jejunal lesion in patients with untreated celiac disease is increased intraepithelial (IE)
lymphocytes.
• The villous structure returns to normal after a protracted gluten-free diet, and the IE lymphocyte count
decreases to normal.
A feature of the jejunal lesion in patients with untreated celiac disease is increased intraepithelial (IE) ly
mphocytes.
The villous structure returns to normal after a
protracted gluten-free diet, and the IE lymphocyte count decreases to normal.
Option D - Distended macrophages with PAS-positive granules in lamina propria:
• Whipple's disease, unlike celiac disease, is characterised by distended macrophages with
PAS-positive granules in lamina propria.
• Therefore, this option is incorrect
Whipple's disease, unlike celiac disease, is characterised by distended macrophages with PAS-positiv
e granules in lamina propria.
Therefore, this option is incorrect

Solution for Question 9:


Correct Option C - Muscularis layer:
• The base of the ulcer lies over the muscularis layer (option C). This feature is typical of amoebic colitis
and helps distinguish it from other causes of colitis. The identification of trophozoites in the stool
sample further supports the diagnosis of Entamoeba histolytica infection.
The base of the ulcer lies over the muscularis layer (option C). This feature is typical of amoebic colitis
and helps distinguish it from other causes of colitis. The identification of trophozoites in the stool sampl
e further supports the diagnosis of Entamoeba histolytica infection.
Incorrect Options:
Option A - Mucosa: This option is incorrect because, in the context of amoebic colitis, the ulcer typically
extends beyond the mucosa into the submucosa and muscularis layers.
Option B
- Submucosa: This option is only partially correct, as the ulcer does extend into the submucosa.
Option D - Serosa: This option is incorrect because the involvement of the serosa is not a typical featur
e of amoebic colitis. Amoebic colitis primarily affects the mucosal and submucosal layers of the intestin

Page 19

742
al wall.

Solution for Question 10:


Correct Option A - Transmural involvement:
• The description in the vignette is typical of Crohn's disease, which often involves the ileum and is
characterized by the presence of superficial, broad-based ulcers.
• Crohn's disease is a type of inflammatory bowel disease (IBD) that can affect any part of the
gastrointestinal tract but commonly involves the ileum and colon.
• Rectal sparing (the rectum is typically not affected) is a distinguishing feature of Crohn's disease, as
opposed to ulcerative colitis, another form of IBD that primarily affects the colon and rectum.
• Crohn's disease is characterized by transmural inflammation, which means it affects the entire
thickness of the bowel wall.
• The cobblestone appearance, as shown in the image in question, is a result of the transmural
inflammation and the formation of skip lesions (areas of diseased bowel interspersed with normal
bowel). This is a distinctive feature of Crohn's disease and distinguishes it from ulcerative colitis, which
typically involves continuous mucosal inflammation without transmural involvement.
The description in the vignette is typical of Crohn's disease, which often involves the ileum and is chara
cterized by the presence of superficial, broad-based ulcers.
Crohn's disease is a type of inflammatory bowel disease (IBD) that can affect any part of the gastrointe
stinal tract but commonly involves the ileum and colon.
Rectal sparing (the rectum is typically not affected) is a distinguishing feature of Crohn's disease, as op
posed to ulcerative colitis, another form of IBD that primarily affects the colon and rectum.
Crohn's disease is characterized by transmural inflammation, which means it affects the entire thicknes
s of the bowel wall.
The cobblestone appearance, as shown in the image in question, is a result of the transmural inflamma
tion and the formation of skip lesions (areas of diseased bowel interspersed with normal bowel). This is
a distinctive feature of Crohn's disease and distinguishes it from ulcerative colitis, which typically involv
es continuous mucosal inflammation without transmural involvement.
Incorrect Options:

Option B - Presence of polyps: Polyps are not a


defining feature of Crohn's disease but can occur in some cases.
Option C - Mucosal edema: While inflammation is a
hallmark of Crohn's disease, the presence of superficial ulcers is a
more specific finding. Mucosal edema may be present but is not as distinctive as the ulcers.
Option D - Superficial broad-based ulcers: Superficial broad-based ulcers are more characteristic of ulc
erative colitis rather than Crohn's disease.

Solution for Question 11:

Page 20

743
Correct Option D - ASCA-positive and Transmural ulcers:
• The given Barium enema shows both String sign and rose thorn sign which are characteristics of
Crohn’s disease.
• In Crohn's disease, Anti-Saccharomyces cerevisiae antibodies (ASCA) are commonly associated, and
the disease often presents with transmural inflammation, leading to deep ulcers. Endoscopic findings in
Crohn's disease may include longitudinal ulcers, skip lesions, and a cobblestone appearance due to the
transmural involvement of the intestinal wall.

The given Barium enema shows both String sign and rose thorn sign which are characteristics of Croh
n’s disease.
In Crohn's disease, Anti-Saccharomyces cerevisiae antibodies (ASCA) are commonly associated, and
the disease often presents with transmural inflammation, leading to deep ulcers. Endoscopic findings in
Crohn's disease may include longitudinal ulcers, skip lesions, and a
cobblestone appearance due to the transmural involvement of the intestinal wall.

Incorrect Options:
Option A - ASCA positive with superficial broad-based ulcers: This option is incorrect because Crohn's
disease typically presents with deep, transmural ulcers rather than superficial broad-based ulcers. Addi
tionally, ASCA positivity is associated with Crohn's disease, but the type of ulcers described here is not
consistent with Crohn's.
Option B - Anti-tTG positive with decreased villous: crypt ratio: This option is more indicative of celiac d
isease rather than Crohn's disease. Anti-tissue transglutaminase antibodies (anti-tTG) are associated
with celiac disease, and the characteristic endoscopic finding in celiac disease is a
decreased villous-to-crypt ratio in the small intestine.
Option C - P-ANCA positive with pseudopolyp: This option is more suggestive of ulcerative colitis rathe
r than Crohn's disease. Perinuclear anti-neutrophil cytoplasmic antibodies (P-ANCA) are associated wit
h certain types of inflammatory bowel disease, including ulcerative colitis. Pseudopolyps are more com
monly seen in ulcerative colitis than in Crohn's disease.

Page 21

744
Solution for Question 12:
Correct Option D - It can cause fistula formation between loops of the affected bowel:
• This patient with low-grade fever, weight loss, fatigue, crampy abdominal pain, episodic diarrhea,
postprandial bloating, and endoscopy findings of thickening of the terminal ileum, edema, marked
luminal narrowing, and a cobblestone appearance of the mucosa, most likely has Crohn's disease.
• Fistula formation between loops of the affected bowel is a characteristic feature of Crohn's disease.
This patient with low-grade fever, weight loss, fatigue, crampy abdominal pain, episodic diarrhea, postp
randial bloating, and endoscopy findings of thickening of the terminal ileum, edema, marked luminal na
rrowing, and a cobblestone appearance of the mucosa, most likely has Crohn's disease.
Fistula formation between loops of the affected bowel is a characteristic feature of Crohn's disease.
Incorrect Options:
Option A - Additional typical findings include crypt abscesses and Pseudopolyps:
• Crypt abscesses and pseudopolyps are seen in ulcerative colitis, not in Crohn's disease.
• Therefore this option is incorrect
Crypt abscesses and pseudopolyps are seen in ulcerative colitis, not in Crohn's disease.
Therefore this option is incorrect
Option B
- Inflammation and ulceration limited to mucosa and submucosa with sparing of deeper layers:
• Transmural involvement of the intestine is seen in Crohn's disease
• Mucosal and submucosal inflammation is a feature of ulcerative colitis.
Transmural involvement of the intestine is seen in Crohn's disease
Mucosal and submucosal inflammation is a feature of ulcerative colitis.
Option C - It can affect any portion of the gastrointestinal tract, but the proximal jejunum is the most co
mmon site of involvement:
• Crohn's disease can involve any part of GIT, but the most commonly involved site is the ileum.
• Therefore this option is incorrect
Crohn's disease can involve any part of GIT, but the most commonly involved site is the ileum.

Solution for Question 13:


Correct Option D - Ulcerative colitis:
• This patient with joint pains involving large joints of legs associated with frequent exacerbations and a
history of diarrhoea most likely has joint problems linked to the skeletal implications of ulcerative colitis.
• Other GI diseases associated with arthropathy are as follows: Whipple's disease Bechet's syndrome
• Whipple's disease
• Bechet's syndrome

Page 22

745
This patient with joint pains involving large joints of legs associated with frequent exacerbations and a
history of diarrhoea most likely has joint problems linked to the skeletal implications of ulcerative colitis.
Other GI diseases associated with arthropathy are as follows:
• Whipple's disease
• Bechet's syndrome
Whipple's disease
Bechet's syndrome
Incorrect Options:
Option A - Colonic carcinoma:
• It is most commonly seen in elderly people and is characterized by a variety of symptoms, including
weight loss, fatigue, abdominal pain, blood in stool, and anemia.
• It has no direct association with any joint disorders.
It is most commonly seen in elderly people and is characterized by a
variety of symptoms, including weight loss, fatigue, abdominal pain, blood in stool, and anemia.
It has no direct association with any joint disorders.
Option B - Acute appendicitis:
• It is an acute condition characterized by abdominal pain that starts from the umbilical region and most
commonly localizes to the right lower quadrant of the abdomen.
• It has no documented association with any joint-related disorder.
It is an acute condition characterized by abdominal pain that starts from the umbilical region and most
commonly localizes to the right lower quadrant of the abdomen.
It has no documented association with any joint-related disorder.
Option C - Diverticulosis:
• It is most commonly seen in elderly people and is characterized by an outpouching of the intestine in
areas of weakness.
It is most commonly seen in elderly people and is characterized by an outpouching of the intestine in ar
eas of weakness.
No direct association with any joint disorder is seen.

Solution for Question 14:


Correct Option A - Prophylactic colectomy only reduces the risk of cancer:
• Prophylactic colectomy does not completely eliminate the risk of cancer because FAP can also lead to
the development of polyps in other parts of the gastrointestinal tract, such as the duodenum. Therefore,
even after colectomy, individuals with FAP are still at risk for other gastrointestinal cancers, and
ongoing surveillance and management are necessary.
Prophylactic colectomy does not completely eliminate the risk of cancer because FAP can also lead to
the development of polyps in other parts of the gastrointestinal tract, such as the duodenum. Therefore
, even after colectomy, individuals with FAP are still at risk for other gastrointestinal cancers, and ongoi

Page 23

746
ng surveillance and management are necessary.
Incorrect Options:
Option B - If not treated will lead to cancer in 10 % of cases:
• The autosomal dominant polyposis syndrome (FPC) has various degrees of penetrance.
• If left untreated, patients will grow hundreds to thousands of polyps throughout the colon and rectum.
• Polyps commonly appear in the early adolescent years and, if left untreated, result in a nearly 100%
lifetime risk of colorectal cancer by the age of forty.
The autosomal dominant polyposis syndrome (FPC) has various degrees of penetrance.
If left untreated, patients will grow hundreds to thousands of polyps throughout the colon and rectum.
Polyps commonly appear in the early adolescent years and, if left untreated, result in a
nearly 100% lifetime risk of colorectal cancer by the age of forty.
Option C - Females are more affected than males:
• FAP affects men and women equally and occurs in around 1 in 10,000 people.
FAP affects men and women equally and occurs in around 1 in 10,000 people.
Option D - Gardner FAP is often associated with Medulloblastoma:
• Familial adenomatous polyposis (FAP) Turcot type is often associated with brain tumors. Gardners
FAP is often associated with Fibroma and osteoma.
Familial adenomatous polyposis (FAP) Turcot type is often associated with brain tumors. Gardners FA
P is often associated with Fibroma and osteoma.

Solution for Question 15:


Correct Option B - Gardner syndrome:
• Gardner syndrome is an unusual variant of familial adenomatous polyposis (FAP).
• It is an autosomal dominant condition with a high risk of malignancy that is characterized by many
adenomatous polyps lining the intestinal mucosal surface.
• Its main features include the development of intestinal polyposis and colorectal cancer.
• Intestinal polyposis, desmoids, osteomas, epidermoid cysts and dental abnormalities (supernumerary
teeth) are extracolonic presentations. Gardner syndrome is characterized by osteomas of the jaw and
skull, epidermal cysts, and fibromatosis.
• These symptoms are frequently asymptomatic, although they can cause pruritus, irritation, and
rupture.
• Gardner syndrome has been linked to the adenomatous polyposis coli (APC) gene, the colon cancer
gene (DCC) deletion, and the TP53 gene.
• Prevention is the most prevalent strategy for treatment for those who are aware of their familial
inheritance or who discover it.
Gardner syndrome is an unusual variant of familial adenomatous polyposis (FAP).
It is an autosomal dominant condition with a high risk of malignancy that is characterized by many ade
nomatous polyps lining the intestinal mucosal surface.

Page 24

747
Its main features include the development of intestinal polyposis and colorectal cancer.
Intestinal polyposis, desmoids, osteomas, epidermoid cysts and dental abnormalities (supernumerary t
eeth) are extracolonic presentations. Gardner syndrome is characterized by osteomas of the jaw and s
kull, epidermal cysts, and fibromatosis.
These symptoms are frequently asymptomatic, although they can cause pruritus, irritation, and rupture.
Gardner syndrome has been linked to the adenomatous polyposis coli (APC) gene, the colon cancer g
ene (DCC) deletion, and the TP53 gene.
Prevention is the most prevalent strategy for treatment for those who are aware of their familial inherita
nce or who discover it.
Incorrect Options:
Option A - Classical FAP:
• Familial adenomatous polyposis (FAP) is an autosomal dominant polyposis condition with different
degrees of penetrance.
• A mutation causes FAP in the APC gene on chromosome 5. The APC gene functions as a tumour
suppressor.
• APC protein mutations limit apoptosis and allow for unregulated cell proliferation, resulting in the
development of adenomas.
Familial adenomatous polyposis (FAP) is an autosomal dominant polyposis condition with different deg
rees of penetrance.
A mutation causes FAP in the APC gene on chromosome 5. The APC gene functions as a
tumour suppressor.
APC protein mutations limit apoptosis and allow for unregulated cell proliferation, resulting in the devel
opment of adenomas.
Option C - Peutz jeghers syndrome:
• Peutz-Jeghers syndrome (PJS) is a hereditary condition distinguished by GI polyposis,
mucocutaneous pigmented macules, and a propensity for specific malignancies.
• Mutation in the STK11 gene, also known as the LKB1 gene, causes PJS.
Peutz-Jeghers syndrome (PJS) is a
hereditary condition distinguished by GI polyposis, mucocutaneous pigmented macules, and a
propensity for specific malignancies.
Mutation in the STK11 gene, also known as the LKB1 gene, causes PJS.
Option D - Cowden syndrome:
• Cowden disease, commonly known as Cowden syndrome or multiple hamartoma syndrome, is an
unusual autosomal dominant genodermatosis caused by mutations in the phosphatase and tensin
homolog (PTEN) gene.
• The majority of Cowden syndrome patients develop a malignant tumour of the thyroid, endometrium,
or breast
Cowden disease, commonly known as Cowden syndrome or multiple hamartoma syndrome, is an unu
sual autosomal dominant genodermatosis caused by mutations in the phosphatase and tensin homolo
g (PTEN) gene.
The majority of Cowden syndrome patients develop a
malignant tumour of the thyroid, endometrium, or breast

Page 25

748
Solution for Question 16:
Correct Option D - Histology of the cerebellar mass will be consistent with a glioma:
• The given clinical scenario with GTCS and familial history of colectomy and MRI findings points
towards the diagnosis of Turcot syndrome
• This statement is accurate. In Turcot syndrome, the primary brain tumors are often glioblastomas or
medulloblastomas.
The given clinical scenario with GTCS and familial history of colectomy and MRI findings points toward
s the diagnosis of Turcot syndrome
This statement is accurate. In Turcot syndrome, the primary brain tumors are often glioblastomas or m
edulloblastomas.
Incorrect Options:
Option A - He may have supernumerary and overcrowding of teeth:
• This statement is not typically associated with Turcot syndrome. Dental abnormalities are more
commonly seen in conditions like Gardner syndrome.
This statement is not typically associated with Turcot syndrome. Dental abnormalities are more commo
nly seen in conditions like Gardner syndrome.
Option B - A colonoscopy may not be indicated:
• This statement is not accurate. In Turcot syndrome, individuals are at an increased risk of colorectal
polyps or colorectal cancer. Therefore, a colonoscopy would be indicated for screening and
surveillance.
This statement is not accurate. In Turcot syndrome, individuals are at an increased risk of colorectal po
lyps or colorectal cancer. Therefore, a colonoscopy would be indicated for screening and surveillance.
Option C - PTEN mutation may be observed:
• This statement is not usually associated with Turcot syndrome. PTEN mutations are more commonly
linked to Cowden syndrome.
This statement is not usually associated with Turcot syndrome. PTEN mutations are more commonly li
nked to Cowden syndrome.

Solution for Question 17:


Correct Option A - Peutz-Jeghers syndrome:
Feature
Peutz-Jeghers Syndrome
Age of Onset
• 11 years
11 years

Page 26

749
Mutation
• Loss of function in STK11/LKB1 gene
Loss of function in STK11/LKB1 gene
Mnemonic - P
• Pigmentation, Polyp
• Arborizing Polyp (tree-like structure)
• Branches with smooth muscles
Pigmentation, Polyp
Arborizing Polyp (tree-like structure)
Branches with smooth muscles
Mnemonic - JE
Most common location in Jejunum

Incorrect Options:
Option B - Cronkhite Canada syndrome:
• Cronkhite-Canada syndrome (CCS) is a rare disease characterised by intestinal polyps, loss of taste,
hair loss, and nail growth issues. It is difficult to treat because of the malabsorption caused by the
polyps.
• Cronkhite-Canada syndrome symptoms are caused by numerous polyps in the stomach, small
intestine, colon, and, less frequently, the oesophagus.
• CCS is more common in older people (average age 59) and is more common in men. It is considered
an acquired, rather than a hereditary, condition.
Cronkhite-Canada syndrome (CCS) is a rare disease characterised by intestinal polyps, loss of taste, h
air loss, and nail growth issues. It is difficult to treat because of the malabsorption caused by the polyps
.
Cronkhite-Canada syndrome symptoms are caused by numerous polyps in the stomach, small intestin
e, colon, and, less frequently, the oesophagus.
CCS is more common in older people (average age 59) and is more common in men. It is considered a
n acquired, rather than a hereditary, condition.
Option C - Familial adenomatous polyposis:
• Familial adenomatous polyposis (FAP) is an autosomal dominant polyposis condition with different
degrees of penetrance.
• A mutation causes FAP in the APC gene on chromosome 5. The APC gene functions as a tumour
suppressor. APC protein mutations limit apoptosis and allow for unregulated cell proliferation, resulting
in the development of adenomas.
Familial adenomatous polyposis (FAP) is an autosomal dominant polyposis condition with different deg
rees of penetrance.
A mutation causes FAP in the APC gene on chromosome 5. The APC gene functions as a tumour sup
pressor. APC protein mutations limit apoptosis and allow for unregulated cell proliferation, resulting in t
he development of adenomas.

Page 27

750
Option D - Hereditary non-polyposis colonic cancer:
• Lynch syndrome, also known as hereditary non-polyposis colorectal cancer syndrome, is the most
frequent genetic form of colorectal cancer (HNPCC).
• The most common cancers involved in Lynch syndrome are colorectal, endometrial and ovarian
carcinoma. Other primary malignancies in these people include gastric, ovarian, small bowel, urothelial
(ureter, renal pelvis), biliary tract, pancreatic, brain (glioblastoma), sebaceous gland adenomas, and
keratoacanthomas.
• A germline mutation causes Lynch syndrome in one of four MMR genes: MLH1, MSH2, MSH6, and
PMS2.
Lynch syndrome, also known as hereditary non-polyposis colorectal cancer syndrome, is the most freq
uent genetic form of colorectal cancer (HNPCC).
The most common cancers involved in Lynch syndrome are colorectal, endometrial and ovarian carcin
oma. Other primary malignancies in these people include gastric, ovarian, small bowel, urothelial (urete
r, renal pelvis), biliary tract, pancreatic, brain (glioblastoma), sebaceous gland adenomas, and keratoac
anthomas.
A germline mutation causes Lynch syndrome in one of four MMR genes: MLH1, MSH2, MSH6, and P
MS2.

Solution for Question 18:


Correct Option C - Ovarian cancer:
• Lynch syndrome, also known as hereditary non-polyposis colorectal cancer syndrome, is the most
frequent genetic form of colorectal cancer (HNPCC).
• The most common cancers involved in Lynch syndrome are colorectal, endometrial, and ovarian
carcinoma. Other primary malignancies in these people include gastric, small bowel, urothelial (ureter,
renal pelvis), biliary tract, pancreatic, brain (glioblastoma), sebaceous gland adenomas, and
keratoacanthomas.
• A germline mutation causes Lynch syndrome in one of four MMR genes: MLH1, MSH2, MSH6, and
PMS2.
• Mismatch repair genes are required for fixing erroneous nucleotide base pairing during DNA
replication. Lynch syndrome is responsible for 2% to 4% of all colorectal cancer cases and 2.5% of
endometrial cancer cases.
• Immunohistochemical staining and microsatellite instability testing are used to screen for Lynch
syndrome.
• Patients who meet the Amsterdam II criteria, the revised Bethesda guidelines, have endometrial
cancer diagnosed before 50 or have a family history of Lynch syndrome should be examined for
genetic testing.
Lynch syndrome, also known as hereditary non-polyposis colorectal cancer syndrome, is the most freq
uent genetic form of colorectal cancer (HNPCC).
The most common cancers involved in Lynch syndrome are colorectal, endometrial, and ovarian carcin
oma. Other primary malignancies in these people include gastric, small bowel, urothelial (ureter, renal
pelvis), biliary tract, pancreatic, brain (glioblastoma), sebaceous gland adenomas, and keratoacanthom
as.

Page 28

751
A germline mutation causes Lynch syndrome in one of four MMR genes: MLH1, MSH2, MSH6, and P
MS2.
Mismatch repair genes are required for fixing erroneous nucleotide base pairing during DNA replication
. Lynch syndrome is responsible for 2% to 4% of all colorectal cancer cases and 2.5% of endometrial c
ancer cases.
Immunohistochemical staining and microsatellite instability testing are used to screen for Lynch syndro
me.
Patients who meet the Amsterdam II criteria, the revised Bethesda guidelines, have endometrial cance
r diagnosed before 50 or have a
family history of Lynch syndrome should be examined for genetic testing.
Incorrect Options:
Option A -Testicular cancer:
• They are not typically associated with lynch syndrome.
They are not typically associated with lynch syndrome.
Option B - Leukemia:
• Ataxia-telangiectasia (A-T) is an autosomal recessive condition caused by mutations in the
ataxia-telangiectasia mutated (ATM) gene, which encodes a serine-threonine protein kinase implicated
in DNA repair pathways.
• They are also more likely to acquire cancer, including leukaemia, lymphoma, and ovarian and breast
carcinoma.
Ataxia-telangiectasia (A-T) is an autosomal recessive condition caused by mutations in the ataxia-telan
giectasia mutated (ATM) gene, which encodes a
serine-threonine protein kinase implicated in DNA repair pathways.
They are also more likely to acquire cancer, including leukaemia, lymphoma, and ovarian and breast c
arcinoma.
Option D - Sarcoma:
• Li-Fraumeni syndrome (also known as the sarcoma, breast, leukaemia, and adrenal gland [SBLA]
cancer syndrome) is a rare inherited syndrome that increases the risk of a variety of cancers, including
sarcoma (including osteosarcoma and soft-tissue sarcomas), leukaemia, brain (central nervous
system) cancers, adrenal cortex cancer, and breast cancer.
• These malignancies frequently arise in young adults or even children.
• Hereditary mutations commonly cause this condition in the tumour suppressor gene TP53.
Li-Fraumeni syndrome (also known as the sarcoma, breast, leukaemia, and adrenal gland [SBLA] canc
er syndrome) is a rare inherited syndrome that increases the risk of a variety of cancers, including sarc
oma (including osteosarcoma and soft-tissue sarcomas), leukaemia, brain (central nervous system) ca
ncers, adrenal cortex cancer, and breast cancer.
These malignancies frequently arise in young adults or even children.
Hereditary mutations commonly cause this condition in the tumour suppressor gene TP53.

Solution for Question 19:

Page 29

752
Correct Option D - Hepatocellular carcinoma:
• Tumours associated with BRAF mutation are: Melanoma Adenocarcinoma of colon Papillary thyroid
carcinoma Hairy cell leukaemia Langerhans cell histiocytosis
• Melanoma
• Adenocarcinoma of colon
• Papillary thyroid carcinoma
• Hairy cell leukaemia
• Langerhans cell histiocytosis
• BRAF inhibitors dabrafenib and vemurafenib have demonstrated efficacy in patients with lung cancer
harboring BRAF mutations
Tumours associated with BRAF mutation are:
• Melanoma
• Adenocarcinoma of colon
• Papillary thyroid carcinoma
• Hairy cell leukaemia
• Langerhans cell histiocytosis
Melanoma
Adenocarcinoma of colon
Papillary thyroid carcinoma
Hairy cell leukaemia
Langerhans cell histiocytosis
BRAF inhibitors dabrafenib and vemurafenib have demonstrated efficacy in patients with lung cancer h
arboring BRAF mutations
Option A - Adenocarcinoma Colon:
• Adenocarcinoma Colon is associated with BRAF mutation.
Adenocarcinoma Colon is associated with BRAF mutation.
Option B - Medullary thyroid carcinoma:
• Medullary thyroid carcinoma is associated with BRAF mutation.
Medullary thyroid carcinoma is associated with BRAF mutation.
Option C - Papillary thyroid carcinoma:
• Papillary thyroid carcinoma is associated with BRAF mutation.
Papillary thyroid carcinoma is associated with BRAF mutation.

Solution for Question 20:


Correct Option A - Tubular adenoma:

Page 30

753
• Adenocarcinoma of the colon develops through a progression of mutations in oncogenes and tumor
suppressor genes in a multi-step process.
• Normal mucosa evolves into tubular adenoma with malignant potential resulting in carcinoma
(Adenoma-Carcinoma sequence)
• Most sporadic colon cancers (70-80%) follow this sequence and usually have left colon cancers.
• The rest of the sporadic cases have defects in DNA mismatch repair or DNA hypermethylation (arise
from serrated adenoma and develop more commonly in the right colon).
• Carcinoma of the rectosigmoid (left-sided) colon causes early obstruction, with a change in bowel
habits & decreased calibre of stool.
• Carcinoma of the right colon (right-sided) can present late, with Iron deficiency anaemia due to
chronic blood loss from the lesion.
Adenocarcinoma of the colon develops through a
progression of mutations in oncogenes and tumor suppressor genes in a multi-step process.
Normal mucosa evolves into tubular adenoma with malignant potential resulting in carcinoma (Adenom
a-Carcinoma sequence)
Most sporadic colon cancers (70-80%) follow this sequence and usually have left colon cancers.
The rest of the sporadic cases have defects in DNA mismatch repair or DNA hypermethylation (arise fr
om serrated adenoma and develop more commonly in the right colon).
Carcinoma of the rectosigmoid (left-sided) colon causes early obstruction, with a
change in bowel habits & decreased calibre of stool.
Carcinoma of the right colon (right-sided) can present late, with Iron deficiency anaemia due to chronic
blood loss from the lesion.
Incorrect Options:
Option B - FAP syndrome:
• Colorectal adenocarcinoma develops in 100% of untreated Familial Adenomatous Polyposis (FAP)
syndrome patients, often before age 30 and nearly always by age 50.
Colorectal adenocarcinoma develops in 100% of untreated Familial Adenomatous Polyposis (FAP) syn
drome patients, often before age 30 and nearly always by age 50.
Option C - Hyperplastic polyp:
• Hyperplastic polyps are without malignant potential.
Hyperplastic polyps are without malignant potential.
Option D - HNPCC syndrome:
• HNPCC syndrome is an autosomal dominant genetic condition characterized by a predisposition to
colorectal cancer and other cancers, including endometrial, ovarian, stomach, small intestine,
hepatobiliary tract, urinary tract, brain, and skin cancers.
• Individuals with HNPCC syndrome have a significantly increased risk of developing colorectal cancer,
typically at a younger age than the general population.
• Similar to FAP syndrome, HNPCC syndrome usually presents with a positive family history of
colorectal or other associated cancers.

Page 31

754
Solution for Question 21:
Correct Option D - Sessile serrated adenoma:
• The likely diagnosis in this patient is sessile serrated adenoma
• Sessile serrated adenomas Easily missed on endoscopy, which may subsequently result in colonic
adenocarcinoma. On M/E resemble hyperplastic polyps but show more complicated glands, often with
“boot-shaped” configurations at gland bases. Lack the overtly dysplastic epithelium & often do not form
polypoid lesions. Hyperplastic polyps, Peutz-Jeghers polyps & inflammatory polyps are not malignant
precursors.
• Easily missed on endoscopy, which may subsequently result in colonic adenocarcinoma.
• On M/E resemble hyperplastic polyps but show more complicated glands, often with “boot-shaped”
configurations at gland bases.
• Lack the overtly dysplastic epithelium & often do not form polypoid lesions.
• Hyperplastic polyps, Peutz-Jeghers polyps & inflammatory polyps are not malignant precursors.
The likely diagnosis in this patient is sessile serrated adenoma
Sessile serrated adenomas
• Easily missed on endoscopy, which may subsequently result in colonic adenocarcinoma.
• On M/E resemble hyperplastic polyps but show more complicated glands, often with “boot-shaped”
configurations at gland bases.
• Lack the overtly dysplastic epithelium & often do not form polypoid lesions.
• Hyperplastic polyps, Peutz-Jeghers polyps & inflammatory polyps are not malignant precursors.
Easily missed on endoscopy, which may subsequently result in colonic adenocarcinoma.
On M/E resemble hyperplastic polyps but show more complicated glands, often with “boot-shaped” con
figurations at gland bases.
Lack the overtly dysplastic epithelium & often do not form polypoid lesions.
Hyperplastic polyps, Peutz-Jeghers polyps & inflammatory polyps are not malignant precursors.
Incorrect Options:
Option A - Tubular adenoma:
• Colonic adenomas are polyps created by glandular tissue that are elevated protrusions of the colonic
mucosa.
• Despite being benign tumors, adenomas are typically regarded as precancerous and can evolve into
malignant structures, as opposed to hyperplastic polyps, which have no malignant potential.
• Colonic tubular adenomas are distinguished by nuclear hyperchromatism (dark purple nuclei), nuclear
crowding (bunched-up nuclei), elliptical/cigar-shaped nuclei, and a loss or decrease in the number of
goblet cells.
• They may not exhibit dysplasia initially but will progressively develop dysplastic characteristics and
may eventually develop colorectal cancer.
Colonic adenomas are polyps created by glandular tissue that are elevated protrusions of the colonic
mucosa.
Despite being benign tumors, adenomas are typically regarded as precancerous and can evolve into m
alignant structures, as opposed to hyperplastic polyps, which have no malignant potential.

Page 32

755
Colonic tubular adenomas are distinguished by nuclear hyperchromatism (dark purple nuclei), nuclear
crowding (bunched-up nuclei), elliptical/cigar-shaped nuclei, and a
loss or decrease in the number of goblet cells.
They may not exhibit dysplasia initially but will progressively develop dysplastic characteristics and ma
y eventually develop colorectal cancer.
Option B - Hyperplastic polyp:
• Serrated architecture has a sawtooth look, with serrations restricted to the top half of the crypts.
• Mild nuclear expansion, stratification, and hyperchromasia of the cryptbases
• There are no crypts that are dilated, branching , or spread horizontally.
• There is no indication of conventional dysplasia, such as that observed in colorectal adenomas.
• The microvesicular variation, which is more frequent, consists of epithelial cells with tiny mucin
vacuoles.
• Goblet cell-rich variation is rare and has prominent goblet cells.
• Because the serrations in the goblet cell-rich variety are more subtle than in the microvesicular form,
comparison with neighboring nonlesional large intestine mucosa may improve detection and minimize
the risk of underdiagnosis.
Serrated architecture has a sawtooth look, with serrations restricted to the top half of the crypts.
Mild nuclear expansion, stratification, and hyperchromasia of the cryptbases
There are no crypts that are dilated, branching , or spread horizontally.
There is no indication of conventional dysplasia, such as that observed in colorectal adenomas.
The microvesicular variation, which is more frequent, consists of epithelial cells with tiny mucin vacuole
s.
Goblet cell-rich variation is rare and has prominent goblet cells.
Because the serrations in the goblet cell-rich variety are more subtle than in the microvesicular form, c
omparison with neighboring nonlesional large intestine mucosa may improve detection and minimize th
e risk of underdiagnosis.
Option C - Peutz-Jeghers polyp:
• Peutz-Jeghers polyp, characterized as a hamartomatous polyp, is distinguished by its structural
support from wide bands of muscularis mucosa smooth muscle, with a notable thickness at the center.
• When viewed at low power, these polyps exhibit a distinctive Christmas tree appearance.
• On the surface, one can observe columnar and goblet cells, while the base of the polyp is home to
Paneth cells and endocrine cells.
• Additionally, approximately 10% of Peutz-Jeghers polyps may present with epithelial misplacement, a
phenomenon known as pseudoinvasion.
Peutz-Jeghers polyp, characterized as a hamartomatous polyp, is distinguished by its structural suppor
t from wide bands of muscularis mucosa smooth muscle, with a notable thickness at the center.
When viewed at low power, these polyps exhibit a distinctive Christmas tree appearance.
On the surface, one can observe columnar and goblet cells, while the base of the polyp is home to Pan
eth cells and endocrine cells.

Page 33

756
Additionally, approximately 10% of Peutz-Jeghers polyps may present with epithelial misplacement, a
phenomenon known as pseudoinvasion.

Solution for Question 22:


Correct Option B - Kulchitsky cells:
• The "coat button" lesions in the appendix refer to metastatic involvement, likely from small cell
carcinoma. Small cell lung carcinoma is known for its aggressive nature and propensity for early
metastasis. In this case, the neuroendocrine nature of the tumor cells can lead to the formation of these
metastatic lesions in the appendix.
• Carcinoid tumor/neuroendocrine tumor arises from neuroendocrine cells called Kulchitsky
cells/Enterochromaffin (EC) cells present in the gastrointestinal tract & respiratory tract:
• CARCINOID TUMOR Carcinoid tumors, arising from neuroendocrine cells, most commonly affect the
gastrointestinal tract, with overproduction of serotonin. Clinical features include hepatomegaly,
intestinal symptoms like diarrhea, vasomotor symptoms such as flushing, asthma-like symptoms, and
systemic fibrosis. In some cases, carcinoid tumors may lead to isolated right heart involvement, causing
tricuspid valve regurgitation and pulmonary valve stenosis. The diagnosis of carcinoid tumor is
supported by the following diagnostic steps: Screening Test: 24-hour HIAA (5-Hydroxyindoleacetic
acid) levels in urine are measured. Elevated levels of HIAA indicate increased production of serotonin,
supporting the diagnosis. Plasma Chromogranin-A Levels: Elevated levels of plasma Chromogranin-A
are observed. Chromogranin-A is a neuroendocrine marker commonly elevated in carcinoid tumors,
further supporting the diagnosis. Biopsy and Electron Microscopy: A biopsy of the tumor tissue is
performed. Histological examination reveals characteristic granules and immunohistochemical staining
shows positivity for Chromogranin A and NSE (Neuron-Specific Enolase). Electron microscopy may be
utilized to further characterize the ultrastructural features of the tumor.
• Carcinoid tumors, arising from neuroendocrine cells, most commonly affect the gastrointestinal tract,
with overproduction of serotonin.
• Clinical features include hepatomegaly, intestinal symptoms like diarrhea, vasomotor symptoms such
as flushing, asthma-like symptoms, and systemic fibrosis.
• In some cases, carcinoid tumors may lead to isolated right heart involvement, causing tricuspid valve
regurgitation and pulmonary valve stenosis.
• The diagnosis of carcinoid tumor is supported by the following diagnostic steps:
• Screening Test: 24-hour HIAA (5-Hydroxyindoleacetic acid) levels in urine are measured. Elevated
levels of HIAA indicate increased production of serotonin, supporting the diagnosis.
• 24-hour HIAA (5-Hydroxyindoleacetic acid) levels in urine are measured. Elevated levels of HIAA
indicate increased production of serotonin, supporting the diagnosis.
• Plasma Chromogranin-A Levels: Elevated levels of plasma Chromogranin-A are observed.
Chromogranin-A is a neuroendocrine marker commonly elevated in carcinoid tumors, further supporting
the diagnosis.
• Elevated levels of plasma Chromogranin-A are observed. Chromogranin-A is a neuroendocrine
marker commonly elevated in carcinoid tumors, further supporting the diagnosis.
• Biopsy and Electron Microscopy: A biopsy of the tumor tissue is performed. Histological examination
reveals characteristic granules and immunohistochemical staining shows positivity for Chromogranin A
and NSE (Neuron-Specific Enolase). Electron microscopy may be utilized to further characterize the

Page 34

757
ultrastructural features of the tumor.
• A biopsy of the tumor tissue is performed. Histological examination reveals characteristic
granules and immunohistochemical staining shows positivity for Chromogranin A and NSE
(Neuron-Specific Enolase). Electron microscopy may be utilized to further characterize the
ultrastructural features of the tumor.
The "coat button" lesions in the appendix refer to metastatic involvement, likely from small cell carcino
ma. Small cell lung carcinoma is known for its aggressive nature and propensity for early metastasis. In
this case, the neuroendocrine nature of the tumor cells can lead to the formation of these metastatic le
sions in the appendix.
Carcinoid tumor/neuroendocrine tumor arises from neuroendocrine cells called Kulchitsky cells/Enteroc
hromaffin (EC) cells present in the gastrointestinal tract & respiratory tract:
CARCINOID TUMOR
• Carcinoid tumors, arising from neuroendocrine cells, most commonly affect the gastrointestinal tract,
with overproduction of serotonin.
• Clinical features include hepatomegaly, intestinal symptoms like diarrhea, vasomotor symptoms such
as flushing, asthma-like symptoms, and systemic fibrosis.
• In some cases, carcinoid tumors may lead to isolated right heart involvement, causing tricuspid valve
regurgitation and pulmonary valve stenosis.
• The diagnosis of carcinoid tumor is supported by the following diagnostic steps:
• Screening Test: 24-hour HIAA (5-Hydroxyindoleacetic acid) levels in urine are measured. Elevated
levels of HIAA indicate increased production of serotonin, supporting the diagnosis.
• 24-hour HIAA (5-Hydroxyindoleacetic acid) levels in urine are measured. Elevated levels of HIAA
indicate increased production of serotonin, supporting the diagnosis.
• Plasma Chromogranin-A Levels: Elevated levels of plasma Chromogranin-A are observed.
Chromogranin-A is a neuroendocrine marker commonly elevated in carcinoid tumors, further supporting
the diagnosis.
• Elevated levels of plasma Chromogranin-A are observed. Chromogranin-A is a neuroendocrine
marker commonly elevated in carcinoid tumors, further supporting the diagnosis.
• Biopsy and Electron Microscopy: A biopsy of the tumor tissue is performed. Histological examination
reveals characteristic granules and immunohistochemical staining shows positivity for Chromogranin A
and NSE (Neuron-Specific Enolase). Electron microscopy may be utilized to further characterize the
ultrastructural features of the tumor.
• A biopsy of the tumor tissue is performed. Histological examination reveals characteristic
granules and immunohistochemical staining shows positivity for Chromogranin A and NSE
(Neuron-Specific Enolase). Electron microscopy may be utilized to further characterize the
ultrastructural features of the tumor.
Carcinoid tumors, arising from neuroendocrine cells, most commonly affect the gastrointestinal tract, wi
th overproduction of serotonin.
Clinical features include hepatomegaly, intestinal symptoms like diarrhea, vasomotor symptoms such a
s flushing, asthma-like symptoms, and systemic fibrosis.
In some cases, carcinoid tumors may lead to isolated right heart involvement, causing tricuspid valve r
egurgitation and pulmonary valve stenosis.
The diagnosis of carcinoid tumor is supported by the following diagnostic steps:

Page 35

758
Screening Test:
• 24-hour HIAA (5-Hydroxyindoleacetic acid) levels in urine are measured. Elevated levels of HIAA
indicate increased production of serotonin, supporting the diagnosis.
Plasma Chromogranin-A Levels:
• Elevated levels of plasma Chromogranin-A are observed. Chromogranin-A is a neuroendocrine
marker commonly elevated in carcinoid tumors, further supporting the diagnosis.
Biopsy and Electron Microscopy:
• A biopsy of the tumor tissue is performed. Histological examination reveals characteristic
granules and immunohistochemical staining shows positivity for Chromogranin A and NSE
(Neuron-Specific Enolase). Electron microscopy may be utilized to further characterize the
ultrastructural features of the tumor.
Incorrect Options
• Options A, C and D are incorrect.

Solution for Question 23:


Correct Option B - Lipomas and Hemangiomas are common:
• In the given scenario, the woman is suffering from Cowden syndrome, and the boy from
Bannayan-Riley-Ruvalcaba Syndrome (BRRS)
• Though both conditions are associated with Skin Abnormalities Lipomas and hemangiomas are a
feature of Bannayan-Riley-Ruvalcaba Syndrome (BRRS), Cowden is associated with Trichilemmomas
and papillomatous papules.
In the given scenario, the woman is suffering from Cowden syndrome, and the boy from Bannayan-Rile
y-Ruvalcaba Syndrome (BRRS)
Though both conditions are associated with Skin Abnormalities Lipomas and hemangiomas are a featu
re of Bannayan-Riley-Ruvalcaba Syndrome (BRRS), Cowden is associated with Trichilemmomas and
papillomatous papules.
Incorrect Options:
Option A - Macrocephaly is a common feature: Both CS and BRRS are associated with macrocephaly.
Option C - Loss of function mutation of PTEN is seen: Both CS and BRRS are caused by mutations in
the PTEN gene, leading to a loss of function.
Option D - Both patients have a defect on chromosome 10: True, as both syndromes involve mutations
in the PTEN gene located on chromosome 10.

Solution for Question 24:


Correct Option C - Age at presentation ranges from 25yrs to 45 yrs:
• The clinical images point toward the diagnosis of Cronkite Canada syndrome, Cronkhite-Canada
syndrome (CCS) is a noninherited condition associated with high morbidity and characterized by
gastrointestinal hamartomatous polyposis, alopecia, onychodystrophy, hyperpigmentation, and

Page 36

759
diarrhea
• The correct information is that Cronkhite-Canada Syndrome (CCS) typically presents later in life,
usually after the age of 50. The age range provided in option C is not consistent with the usual
presentation of CCS.
The clinical images point toward the diagnosis of Cronkite Canada syndrome,
Cronkhite-Canada syndrome (CCS) is a noninherited condition associated with high morbidity and char
acterized by gastrointestinal hamartomatous polyposis, alopecia, onychodystrophy, hyperpigmentation,
and diarrhea
The correct information is that Cronkhite-Canada Syndrome (CCS) typically presents later in life, usuall
y after the age of 50. The age range provided in option C
is not consistent with the usual presentation of CCS.
Incorrect Options:
Option A - It is of unknown etiology:
• This statement is correct. The exact cause of Cronkhite-Canada Syndrome (CCS) is not well
understood. It is considered idiopathic, and the etiology is unknown.
This statement is correct. The exact cause of Cronkhite-Canada Syndrome (CCS) is not well understoo
d. It is considered idiopathic, and the etiology is unknown.
Option B - Onychodystrophy and Alopecia are seen:
• This statement is correct. Onychodystrophy (nail abnormalities) and alopecia (hair loss) are
cutaneous manifestations commonly associated with CCS
This statement is correct. Onychodystrophy (nail abnormalities) and alopecia (hair loss) are cutaneous
manifestations commonly associated with CCS
Option D - The polyps are hamartomatous and respond to systemic steroid and antiplasmin therapy:
• This statement is correct. The polyps in Cronkhite-Canada Syndrome are indeed hamartomatous.
Additionally, systemic steroid therapy has been used in the management of CCS, and antiplasmin
therapy may also be considered in some cases.
This statement is correct. The polyps in Cronkhite-Canada Syndrome are indeed hamartomatous. Addi
tionally, systemic steroid therapy has been used in the management of CCS, and antiplasmin therapy
may also be considered in some cases.

Solution for Question 25:


Correct Option C- Intestinal manifestations include the presence of fat:
• Abetalipoproteinemia is a genetic disorder due to a defect in the microsomal triglyceride transfer
protein (MTTP) gene. It primarily affects the intestines and red blood cells. The intestines show the
presence of fats.
Incorrect Options:
Option A- It is caused by a mutation in the APOE gene:
• Abetalipoproteinemia is caused by a mutation in the microsomal triglyceride transfer protein (MTTP)
gene, not the apolipoprotein E (APOE) gene.
Option B- It primarily affects liver function:

Page 37

760
• Abetalipoproteinemia primarily affects the intestines and red blood cells, not the liver.
Option D- Red blood cells exhibit a smooth, round shape:
• In abetalipoproteinemia, red blood cells may exhibit an abnormal shape known as acanthocytosis,
characterized by spiky or irregularly shaped cells rather than smooth, round cells.

Solution for Question 26:


Correct Option B- Microvillus Inclusions:
• MYO5B gene mutation can lead to defective microvilli formation and result in intractable diarrhea.
Microvillus inclusions are characteristic microscopic findings associated with MYO5B gene mutation. It
is known as Davidson disease.
Incorrect Options:
Options A, C, and D: They are not associated with MYO5B gene mutation.

Solution for Question 27:


Correct Option C- Predominantly found in the rectosigmoid region:
• Hyperplastic polyps are typically found in the rectosigmoid region of the gastrointestinal tract.
Incorrect Options:
Option A- Characterized by an increase in the size of cells:
• Hyperplastic polyps are characterized by an increase in the number of cells, not the size of cells.
Option B- Serrated appearance only in the lower part of the mucosa:
• Hyperplastic polyps exhibit a serrated appearance confined only to the upper part, not the lower part,
not the lower part.
Option D- Affects young individuals:
• Hyperplastic polyps are more commonly associated with individuals in their 50s to 60s, not young
individuals.

Solution for Question 28:


Correct Option C- Nesting pattern:
• Carcinoid tumors typically exhibit a nesting pattern when observed under light microscopy. This
pattern refers to the arrangement of tumor cells in small nests or clusters, often surrounded by
fibrovascular stroma. It also exhibits salt and pepper.
Incorrect Options:
Options A, B, and D: Carcinoid tumors do not demonstrate tubular, glandular, or papillary patterns.

Page 38

761
Page 39

762
Previous Year Questions
1. What is a reliable indicator of intestinal inflammation that is both sensitive and specific?
A. Procalcitonin
B. Fecal lactoferrin
C. Tissue transglutaminase IgA
D. High-sensitivity C-reactive protein
----------------------------------------
2. Which of the following statements are true about familial adenomatous polyposis ? It is autosomal
recessive If not treated, 100% of the cases progress to adenocarcinoma colon. Polyps do not arise
after puberty It is associated with congenital hypertrophy of the retinal pigment epithelium.
A. 2 and 4
B. 1 and 4
C. 2 and 3
D. 3 and 4
----------------------------------------
3. Which of the following medications/treatments is suitable for a patient with celiac disease?
A. Rye
B. Wheat
C. Quinoa
D. Barley
----------------------------------------
4. Which antibody is found in individuals with Celiac disease?
A. Tissue transglutaminase IgA (tlgA)
B. Endomysial IgA
C. Reticulin IgA
D. All above
----------------------------------------
5. Most specific antibodies in systemic sclerosis
A. Anti-DNA
B. Anti-topoisomerase
C. ANA
D. Anti-Ro
----------------------------------------
6. A 5-year-old child was admitted to the hospital with a history of prolapsing rectal mass and painless
rectal bleeding. The histopathological image of the mass is shown below. What is the most likely
diagnosis?

763
A. Peutz-Jegher syndrome
B. Villous adenoma
C. Juvenile polyp
D. Tubular adenoma
----------------------------------------
7. A 20-year-old patient presented to the clinic with a history of malabsorption, diarrhea, and poor
appetite. A duodenal biopsy showed crypt hyperplasia, villi atrophy, and CD8+ cells in the lamina
propria. What is the most probable diagnosis?
A. Environmental enteropathy
B. Celiac disease
C. Whipple’s disease
D. Chronic pancreatitis
----------------------------------------
8. Which of the following is true about the pathogenesis of Hirschsprung disease?
A. Failure of migration of neural crest cells
B. Increase ganglion cells
C. Visceral myopathy
D. Obstruction by inflammatory strictures
----------------------------------------
9. A 17-year-old male patient came with intussusception and underwent surgery. During the procedure,
multiple polyps were discovered. The section of the tissue that was removed was sent for
histopathological examination, and the entire image of the specimen is provided below. Please identify
the specific abnormality present.

Page 2

764
A. Tubulovillous adenoma
B. Hamartomatous polyp
C. Adenocarcinoma
D. Hyperplastic polyp
----------------------------------------
10. What statement regarding celiac disease is correct?
A. The most common cause of malnutrition in developing countries
B. Can be diagnosed with CECT
C. Improves with food rich in carbohydrates
D. Increased risk of GI lymphoma
----------------------------------------
11. What is a distinguishing feature of Barrett's esophagus among the options provided?
A. Squamous metaplasia
B. Severe dysplasia
C. Intestinal metaplasia
D. Cuboidal metaplasia
----------------------------------------
12. Which one of the subsequent options pertains to a stomach tumor present with Acanthosis
nigricans?
A. Carcinoid tumor
B. Lymphoma
C. Gastrointestinal stromal tumor
D. Gastric adenocarcinoma
----------------------------------------
13. What is the gene mutation associated with Cowden syndrome?
A. PTEN
B. WT 1

Page 3

765
C. p53
D. Ras
----------------------------------------
14. Whipple's disease is caused by.
A. Bacteria
B. Virus
C. Protozoa
D. Helminths
----------------------------------------
15. Cells lining the isthmus of the gastric pits
A. Chief cells
B. Stem cells
C. Parietal cells
D. Mucous cells
----------------------------------------
16. Hepcidin inhibits iron transfer from the enterocyte to the plasma by binding to?
A. Ferroportin
B. Transferrin
C. Divalent metal transporter 1 (DMT 1)
D. Ferritin
----------------------------------------
17. What is the transporter needed for iron transportation into the enterocyte?
A. Ferroportin
B. Divalent metal transporter 1
C. Divalent metal transporter 2
D. Haptocorrin
----------------------------------------
18. Hepcidin acts on which of the following?
A. Transferrin
B. Ferroportin
C. Ferritin
D. Divalent metal transporter 1
----------------------------------------
19. The absence of the structure marked as A will present with all except :

Page 4

766
A. Pernicious anemia
B. Autoimmune Metaplastic Atrophic Gastritis
C. Vit B12 Deficiency
D. Zollinger ellison syndrome
----------------------------------------
20. Which of the following statements is true about the pathology of achalasia cardia?
A. Relaxation of lower esophageal sphincter
B. Increased peristalsis
C. Decreased release of NO and VIP in the distal part of esophagus
D. Decreased tone of the lower esophageal sphincter
----------------------------------------
21. Punched-out ulcers in the esophagus are caused by which of the following?
A. Herpes simplex virus
B. Cytomegalovirus
C. Candida
D. Gastric acid
----------------------------------------
22. All is true about the Barrett oesophagus, except?
A. Complication of chronic GERD
B. Seen in 10% of individuals with symptomatic GERD
C. Increases risk of squamous cell carcinoma
D. Intestinal metaplasia
----------------------------------------

Correct Answers
Question Correct Answer

Question 1 2

Page 5

767
Question 2 1
Question 3 3
Question 4 4
Question 5 2
Question 6 3
Question 7 2
Question 8 1
Question 9 2
Question 10 4
Question 11 3
Question 12 4
Question 13 1
Question 14 1
Question 15 2
Question 16 1
Question 17 2
Question 18 2
Question 19 4
Question 20 3
Question 21 1
Question 22 3

Solution for Question 1:


• Lactoferrin is an enzyme that is synthesized from neutrophil granulocytes and epithelial cells.
• Fecal lactoferrin and fecal calprotectin is raised in active intestinal diseases like ulcerative colitis.
• Fecal lactoferrin is a marker of IBD and is used to rule out Irritable bowel syndrome.
• Fecal lactoferrin also helps in monitoring response to treatment.
Incorrect choices:
• Option a: Procalcitonin: Procalcitonin is synthesized by C cells of the thyroid gland. It increases in
response to severe sepsis or any bacterial infection. However, it is not a specific and sensitive marker
for intestinal infection. It increases in response to infection in other organs like the lungs, and liver.
• It increases in response to severe sepsis or any bacterial infection.
• However, it is not a specific and sensitive marker for intestinal infection.
• It increases in response to infection in other organs like the lungs, and liver.
• Option c. Tissue transglutaminase IgA: It is an antibody that helps in the diagnosis of celiac
disease.

Page 6

768
• It is an antibody that helps in the diagnosis of celiac disease.
• Option d. High-sensitivity C-reactive protein: High-sensitivity CRP helps in assessing the risk of
getting coronary artery disease. It is more sensitive than the normal C-reactive protein. Normal levels
are less than 1 mg/L. Levels of more than 3 mg/L indicate a high risk of developing coronary artery
disease.
• It increases in response to severe sepsis or any bacterial infection.
• However, it is not a specific and sensitive marker for intestinal infection.
• It increases in response to infection in other organs like the lungs, and liver.
• It is an antibody that helps in the diagnosis of celiac disease.

Solution for Question 2:


Correct Option A - 2 and 4:
• If not treated, 100% of the cases progress to adenocarcinoma colon: FAP is characterized by the
development of numerous adenomatous polyps in the colon. If left untreated, these polyps have a high
likelihood of progressing to colon cancer (adenocarcinoma). Therefore, it is crucial to detect and
manage FAP early to prevent the development of cancer.
• It is associated with congenital hypertrophy of the retinal pigment epithelium (CHRPE): CHRPE is a
benign finding in FAP patients. It is characterized by flat, pigmented lesions in the retina. Although
CHRPE is not specific to FAP, its presence can raise suspicion about the condition when combined
with other clinical features.
Incorrect Options:
Option B - 1 and 4: FAP is an autosomal dominant inherited disorder, meaning that only one copy of th
e mutated gene is necessary to develop the condition. If a parent carries the mutated gene, there is a
50% chance of passing it on to their children. CHRPE is a
benign finding in FAP patients. It is characterized by flat, pigmented lesions in the retina.
Option C - 2 and 3: Polyps do not arise after puberty is incorrect ,
Thousands of polyps arise starting after puberty.
Option D - 3 and 4: Polyps do not arise after puberty is incorrect , Thousands of polyps arise starting af
ter puberty. It is associated with congenital hypertrophy of the retinal pigment epithelium. is correct stat
ement.

Solution for Question 3:


Correct option:
Option. C
Quinoa: Quinoa is a gluten-free grain and can be safely consumed by individuals with celiac disease.
Quinoa does not contain gluten and is considered a suitable alternative for people following a
gluten-free diet.
Incorrect options :

Page 7

769
Option A. Rye: Rye contains gluten and should be avoided by individuals with celiac disease. Consum
ption of rye can trigger an immune response and further damage the small intestine.
Option B. Wheat: Wheat is a major source of gluten and should be strictly avoided by individuals with c
eliac disease. This includes all varieties of wheat, such as durum, spelt, and kamut. Consuming wheat
products can cause severe symptoms and intestinal damage in people with celiac disease.
Option D. Barley: Barley contains gluten and should be strictly avoided by individuals with celiac diseas
e. Similar to wheat and rye, consuming barley can trigger an immune response and worsen the sympto
ms and intestinal damage in people with celiac disease.

Solution for Question 4:


Correct option:
Options. D
Celiac disease is an autoimmune condition characterized by an immune reaction to gluten, a protein fo
und in wheat, barley, and rye. When gluten is ingested by individuals with celiac disease, it triggers an i
mmune response that leads to damage in the small intestine. Antibodies play a
crucial role in the diagnosis of celiac disease.
In the context of celiac disease, all three antibodies mentioned (tissue transglutaminase IgA, endomysi
al IgA, and reticulin IgA) can be present, although the tTG IgA and endomysial IgA antibodies are more
commonly tested and considered more specific for celiac disease.
Incorrect Options:
Option A : It is present in celiac disease
Option B : It is present in celiac disease
Option C : It is present in celiac disease

Solution for Question 5:


Correct option:
Option. B
Anti-topoisomerase (Anti-Scl-70): Anti-topoisomerase antibodies are highly specific for systemic sclero
sis, particularly its diffuse cutaneous subtype. These antibodies target an enzyme called topoisomeras
e I, which is involved in DNA replication and repair. The presence of anti-topoisomerase antibodies is s
trongly associated with skin involvement and internal organ complications in systemic sclerosis.
Incorrect options:
Option A. Anti-DNA: Anti-DNA antibodies are typically associated with autoimmune diseases such as s
ystemic lupus erythematosus (SLE) rather than systemic sclerosis. While anti-DNA antibodies may be
present in some cases of systemic sclerosis, they are not considered specific to the disease.
Option C. ANA (Antinuclear Antibodies): ANA is a general term for antibodies that target components w
ithin the cell nucleus. While ANA can be detected in various autoimmune diseases, including systemic
sclerosis, it is not specific to the condition. ANA testing is often used as a
screening tool to indicate the presence of an autoimmune process but does not provide a

Page 8

770
definitive diagnosis on its own.
Option D. Anti-Ro (SS-A): Anti-Ro antibodies are associated with various autoimmune conditions, inclu
ding systemic lupus erythematosus (SLE) and Sjögren's syndrome. While they may occasionally be det
ected in systemic sclerosis, they are not specific to the disease.

Solution for Question 6:


Correct Option C - Juvenile polyp:
• Juvenile polyps are benign growths that commonly occur in children, typically under the age of 5.
• They are characterized by a prolapsing rectal mass and painless rectal bleeding.
• The given histopathological image shows juvenile polyps showing dilated crypts and dilated glands
filled with mucin.
Incorrect Options:
Option A - Peutz-Jeghers syndrome:
• Peutz-Jeghers syndrome is an autosomal dominant condition characterized by the presence of
hamartomatous polyps throughout the gastrointestinal tract.
• While it can manifest with rectal polyps, the histopathological image is more characteristic of juvenile
polyps.
Option B - Villous adenoma:
• Villous adenoma is a type of adenomatous polyp that is more commonly seen in adults.
• It is characterized by finger-like projections with abundant mucin production.
• The histopathological image is more characteristic of juvenile polyps.
Option D - Tubular adenoma:
• Tubular adenoma is a common type of adenomatous polyp that typically occurs in adults.
• It is characterized by glandular structures composed of tubular architecture.
• The histopathological image is more characteristic of juvenile polyps.

Solution for Question 7:


Correct Option B - Celiac disease:
• Celiac disease, also known as gluten-sensitive enteropathy, is an autoimmune disorder triggered by
the ingestion of gluten-containing grains, such as wheat, barley, and rye, in genetically predisposed
individuals.
• It is characterized by an immune-mediated response to gluten, leading to damage in the small
intestine.
• The duodenal biopsy findings of crypt hyperplasia, villi atrophy, and the presence of CD8+ cells in the
lamina propria are characteristic features of celiac disease.
Incorrect Options:

Page 9

771
Option A - Environmental enteropathy: Environmental enteropathy is a condition characterized by chro
nic intestinal inflammation and structural changes in the small intestine, typically seen in individuals fro
m developing countries with poor sanitation and hygiene.
Option C - Whipple's disease: Whipple's disease is a rare bacterial infection caused by Tropheryma wh
ipplei and primarily affects the small intestine but can involve other organs.
Option D - Chronic pancreatitis: Chronic pancreatitis is characterized by inflammation and fibrosis of th
e pancreas, leading to impaired pancreatic function.

Solution for Question 8:


Correct Option A - Failure of migration of neural crest cells:
• Hirschsprung disease, also known as congenital aganglionic megacolon, is a developmental disorder
of the gastrointestinal tract characterized by the absence of ganglion cells in the myenteric and
submucosal plexuses of the distal bowel, leading to functional obstruction.
• Hirschsprung disease occurs due to the failure of the migration of neural crest cells, which are
responsible for the development of the enteric nervous system.
• In normal development, neural crest cells migrate along the gastrointestinal tract and populate the
entire length, including the distal bowel.
• In Hirschsprung disease, there is a segment of the bowel where the ganglion cells fail to migrate,
resulting in an aganglionic segment leading to functional obstruction and impaired motility.
Incorrect Options:
Option B - Increase ganglion cells:
• Hirschsprung disease is characterized by a lack of ganglion cells.
Option C - Visceral myopathy:
• Visceral myopathy refers to a group of disorders characterized by abnormal smooth muscle function
in the gastrointestinal tract.
• While Hirschsprung disease involves abnormalities in the enteric nervous system, it does not involve
primary defects in the smooth muscle layer.
Option D - Obstruction by inflammatory strictures:
• Inflammatory strictures are not associated with the pathogenesis of Hirschsprung disease.

Solution for Question 9:


Correct Option B:
Based on the given whole mount image of the specimen, the identified lesion is a hamartomatous poly
pas it shows arborizing pattern of connective tissue, smooth muscle,glands and lamina propria. Hamart
omatous polyps are non-neoplastic growths that result from disorganized overgrowth of normal tissues
in an organ. These polyps can occur in various organs, including the gastrointestinal tract.
Hamartomatous polyps are commonly associated with genetic syndromes such as Peutz-Jeghers synd
rome (PJS) and juvenile polyposis syndrome (JPS). In these syndromes, individuals have an increased

Page 10

772
risk of developing multiple hamartomatous polyps throughout the gastrointestinal tract.
Incorrect Option:
Option A. Tubulovillous adenoma: Tubulovillous adenoma is a premalignant lesion characterized by th
e presence of both tubular and villous components. It has the potential to progress to adenocarcinoma.
However, the given image does not show the characteristic features of a
tubulovillous adenoma, such as dysplasia or the presence of dysplastic cells within the epithelial lining.
Option C. Adenocarcinoma: Adenocarcinoma is a malignant tumor that arises from the glandular cells
of an organ. It is characterized by invasive growth and the presence of dysplastic cells. The given imag
e does not show features consistent with adenocarcinoma, such as infiltration into the surrounding tiss
ue or cytological atypia.
Option D. Hyperplastic polyp: Hyperplastic polyps are non-neoplastic growths characterized by an incr
ease in the number of cells in the epithelial layer of the organ. They are typically small, sessile, and ha
ve a smooth surface. The given image does not match the typical appearance
of hyperplastic polyps, which are usually smaller and more uniform in size.

Solution for Question 10:


Correct Option D:
Celiac disease is an autoimmune disorder characterized by a hypersensitivity reaction to gluten, a prot
ein found in wheat, barley, and rye. It primarily affects the small intestine and leads to chronic inflamma
tion and damage to the intestinal lining. This chronic inflammation and damage increase the risk of dev
eloping gastrointestinal (GI) lymphoma, which is a
type of cancer that affects the lymphatic system in the GI tract.
Incorrect Options:
Option A: Celiac disease is more commonly seen in developed countries, particularly in populations of
European descent. It is not the most common cause of malnutrition in developing countries.
Option B: CECT (contrast-enhanced computed tomography) is not a
diagnostic test for celiac disease. The diagnosis of celiac disease is typically made through a combinati
on of clinical evaluation, serologic testing (e.g., anti-tissue transglutaminase antibodies), and confirmati
on by small bowel biopsy.
Option C: Celiac disease is not improved by consuming a
diet rich in carbohydrates. In fact, the primary treatment for celiac disease is a
strict gluten-free diet, which involves avoiding foods that contain gluten.

Solution for Question 11:


Correct Option: C
The correct answer is Intestinal metaplasia.
Explanation for the options:
Option A: Squamous metaplasia: Squamous metaplasia is a reversible change in which one type of epi
thelial cell is replaced by another type of epithelial cell. However, in Barrett's esophagus, the characteri
stic feature is not squamous metaplasia.

Page 11

773
Option B: Severe dysplasia: Dysplasia refers to the abnormal growth and organization of cells. While d
ysplasia can occur in Barrett's esophagus, it is not a characteristic feature.
Option C: Intestinal metaplasia: Barrett's esophagus is characterized by the presence of intestinal meta
plasia. Intestinal metaplasia is the replacement of the normal squamous epithelium of the esophagus w
ith columnar epithelium similar to that found in the intestine. This change is considered a
precursor to the development of esophageal adenocarcinoma.
Option D: Cuboidal metaplasia: Cuboidal metaplasia refers to the replacement of normal epithelium wit
h cuboidal epithelium. This change is not specifically associated with Barrett's esophagus.
Therefore, the characteristic feature of Barrett's esophagus is the presence of intestinal metaplasia in t
he esophageal epithelium. It is important because it increases the risk of developing esophageal adeno
carcinoma, a type of cancer. Regular monitoring and surveillance are recommended for individuals wit
h Barrett's esophagus to detect any signs of dysplasia or cancer.

Solution for Question 12:


Correct Option D: Gastric adenocarcinoma
• Gastric adenocarcinoma is an epithelial tumor that presents with Acanthosis nigricans.
Incorrect Options:
Option A: Carcinoid tumor- Carcinoid tumors are neuroendocrine tumors that can arise in various orga
ns, including the gastrointestinal tract. While they arise from neuroendocrine cells in the stomach wall o
r elsewhere.
Option B: Lymphoma- Lymphoma is a malignancy that arises from lymphocytes, a
type of immune cell. Stomach lymphomas can be of B-cell or T-cell origin.
Option C: Gastrointestinal stromal tumor (GIST)- GISTs are mesenchymal tumors that arise from the s
pecialized cells of the gastrointestinal tract called interstitial cells of Cajal.

Solution for Question 13:


Correct Option A: PTEN
• Cowden syndrome is associated with a mutation in the PTEN (phosphatase and tensin homolog)
gene.
• PTEN is a tumor suppressor gene that regulates cell growth and division.
• Mutations in the PTEN gene can lead to the development of Cowden syndrome, which is an inherited
condition characterized by multiple benign and malignant tumors, particularly in the breast, thyroid, and
other organs.
Incorrect Options:
Option B: WT1- WT1 (Wilms tumor 1) is a
tumor suppressor gene that is associated with Wilms tumor, a childhood kidney cancer.

Page 12

774
Option C: p53- p53 is another tumor suppressor gene that plays a crucial role in preventing the develo
pment of cancer. Mutations in the p53 gene are associated with Li-Fraumeni syndrome, which predispo
ses individuals to various types of cancers.
Option D: Ras- Ras refers to a group of genes that encode proteins involved in cell signaling pathways.
Mutations in Ras genes can lead to the development of various types of cancers.

Solution for Question 14:


Correct Option A
• Whipple's disease is a rare systemic infectious disease caused by a bacterium called Tropheryma
whipplei. It primarily affects the small intestine but can also involve various other organs such as the
joints, heart, brain, and eyes. The bacterium infects the lining of the small intestine, leading to
malabsorption of nutrients and subsequent systemic symptoms.
Incorrect options:
Option B. Virus: Whipple's disease is not caused by a virus. It is caused by a
bacterium, Tropheryma whipplei.
Option C. Protozoa: Whipple's disease is not caused by a protozoan infection. It is caused by a
bacterium.
Option D. Helminths: Whipple's disease is not caused by a helminth infection. It is caused by a
bacterium.

Solution for Question 15:


Correct Option B:
The cells lining the isthmus of the gastric pits are known as stem cells. These stem cells are undifferent
iated cells that have the capacity to divide and differentiate into different cell types. In the gastric pits, t
he stem cells give rise to the various specialized cells that make up the gastric glands.
These stem cells play a crucial role in maintaining the continuous renewal and repair of the gastric epit
helium. They give rise to several cell types, including mucous cells, chief cells, and parietal cells, which
are responsible for the production and secretion of various components of gastric juice.
Incorrect Options:
Option A. Chief cells are found in the base of the gastric glands and are responsible for the production
and secretion of pepsinogen, an inactive form of the enzyme pepsin. Chief cells are not specifically loc
ated in the isthmus region of the gastric pits.
Option C. Parietal cells are found in the upper region of the gastric glands and are responsible for the p
roduction and secretion of hydrochloric acid (HCl) and intrinsic factor. Parietal cells are not specifically l
ocated in the isthmus region of the gastric pits.
Option D. Mucous cells are found in the neck and surface regions of the gastric pits and are responsibl
e for producing and secreting mucus, which helps protect the gastric mucosa from the acidic environm
ent of the stomach. Mucous cells are not specifically located in the isthmus region of the gastric pits.

Page 13

775
Solution for Question 16:
Correct Option A.
Ferroportin is a transmembrane protein found on the surface of enterocytes (cells in the small intestine)
and macrophages. It is responsible for exporting iron from these cells into the bloodstream. Hepcidin,
a peptide hormone produced by the liver, binds to ferroportin and triggers its internalization and degrad
ation. This inhibits iron transfer from the enterocyte to the plasma, reducing iron absorption.
Incorrect Option:
Option B: Transferrin is a plasma protein that binds to iron and transports it in the bloodstream. Hepcidi
n does not directly bind to transferrin.
Option C: Divalent metal transporter 1 (DMT1) is a transmembrane protein involved in the uptake of va
rious divalent metals, including iron, into the enterocyte. Hepcidin primarily acts on ferroportin, not DMT
1.
Option D: Ferritin is an intracellular protein that serves as the primary storage form of iron in cells. Hep
cidin does not directly bind to ferritin.

Solution for Question 17:


Correct option:
Option B.
• Divalent metal transporter 1: Divalent metal transporter 1 (DMT1) is the transporter responsible for the
uptake of various divalent metals, including iron, across the apical membrane of enterocytes. It plays a
crucial role in the absorption of dietary iron from the intestinal lumen into the enterocyte.
Incorrect Options:
Option A. Ferroportin: Ferroportin is a protein involved in the export of iron from cells, including the exp
ort of iron from enterocytes into the bloodstream. It is not directly responsible for the transport of iron in
to the enterocyte.
Option C. Divalent metal transporter 2 : Divalent metal transporter 2
(DMT2) is primarily involved in the transport of manganese and other divalent metals. It does not play a
significant role in the transport of iron into the enterocyte.
Option D. Haptocorrin : Haptocorrin (also known as transcobalamin I) is a protein involved in the transp
ort of vitamin B12 in the body. It is not directly involved in the transport of iron into the enterocyte.

Solution for Question 18:


Correct option:
Option B.

Page 14

776
• Ferroportin: Ferroportin is an iron exporter protein found on the surface of cells, including enterocytes
in the intestine. Hepcidin binds to ferroportin and induces its internalization and degradation. This leads
to a decrease in iron export from the enterocyte into the plasma, thereby reducing iron absorption and
availability.
Incorrect Options:
Option A. Transferrin Transferrin is a protein that binds to iron in the plasma and facilitates its transport
. Hepcidin does not directly bind to transferrin. Instead, hepcidin regulates iron levels by binding to ferr
oportin.
Option C. Ferritin Ferritin is an intracellular protein that stores iron. Hepcidin does not directly bind to fe
rritin. Instead, hepcidin regulates iron levels by modulating the activity of ferroportin, which in turn affect
s iron release from ferritin.
Option D. Divalent metal transporter 1 Divalent metal transporter 1 (DMT1) is a transmembrane protein
responsible for the uptake of iron from the lumen of the intestine into enterocytes. Hepcidin does not bi
nd to DMT1. Its main target is ferroportin, which is involved in the release of iron from the enterocyte int
o the bloodstream.

Solution for Question 19:


Correct Option D:
• Zollinger-Ellison syndrome is a rare condition characterized by the presence of gastrin-secreting
tumors, called gastrinoma, typically found in the pancreas or duodenum. These tumors lead to
excessive production of gastrin, a hormone that stimulates gastric acid secretion. In Zollinger-Ellison
syndrome, parietal cells, which are responsible for acid production in the stomach, are intact and
functional. Therefore, the absence of parietal cells would not present with Zollinger-Ellison syndrome.
Incorrect Options:
Option A: Pernicious anemia: Pernicious anemia is an autoimmune condition characterized by the lack
of intrinsic factor, a protein necessary for the absorption of vitamin B12 in the intestines. In pernicious a
nemia, the parietal cells of the stomach are affected, leading to their reduced function or destruction. T
he absence of parietal cells is a characteristic feature of pernicious anemia.
Option B: Autoimmune Metaplastic atrophic Gastritis: It is an inherited autoimmune disease that attack
s parietal cells, resulting in hypochlorhydria & decreased production of Intrinsic factors.
Option C: Vitamin B12 deficiency: Vitamin B12 deficiency can occur due to various reasons, including
pernicious anemia or inadequate dietary intake. As mentioned earlier, pernicious anemia involves the a
bsence or dysfunction of parietal cells, which are responsible for the secretion of intrinsic factor needed
for vitamin B12 absorption. Therefore, the absence of parietal cells is associated with vitamin B12 defi
ciency.

Solution for Question 20:


Correct Option C - Decreased release of NO and VIP in the distal part of esophagus:
• Achalasia cardia is a disorder characterized by the dysfunction of the lower esophageal sphincter
(LES) and impaired esophageal peristalsis.

Page 15

777
• The pathology of achalasia cardia involves a decrease in the release of nitric oxide (NO) and
vasoactive intestinal peptide (VIP) in the distal part of the esophagus.
• These neurotransmitters are responsible for relaxing the smooth muscles of the esophagus, allowing
for proper movement and passage of food.
Incorrect Options:
Option A - Relaxation of lower esophageal sphincter:
• In achalasia cardia, there is impaired relaxation of the lower esophageal sphincter (LES).
• The LES fails to relax properly during swallowing, leading to difficulty in food passage into the
stomach.
Option B - Increased peristalsis:
• Achalasia cardia is characterized by ineffective esophageal peristalsis.
Option D - Decreased tone of the lower esophageal sphincter:
• Achalasia cardia is characterized by increased tone of the lower esophageal sphincter (LES).
• The LES fails to relax adequately, causing a functional obstruction at the gastroesophageal junction.

Solution for Question 21:


Correct Option A - Herpes simplex virus:
• Punched-out ulcers in the esophagus can be caused by a herpes simplex virus (HSV) infection.
• HSV esophagitis typically presents with shallow, well-demarcated ulcers with a punched-out
appearance.
• The infection is commonly seen in immunocompromised individuals, such as those with HIV/AIDS or
undergoing immunosuppressive therapy.
Incorrect Options:
Option B - Cytomegalovirus: CMV esophagitis typically presents with linear or linearly arranged shallow
ulcers in the esophagus.
Option C - Candida: Candida esophagitis can cause raised, white plaques or pseudomembranes in the
esophagus.
Option D - Gastric acid: Acid reflux or gastroesophageal reflux disease can cause esophagitis, but it ty
pically results in erosions or linear ulcers.

Solution for Question 22:


Correct Option C - Increases the risk of squamous cell carcinoma:
• Barrett esophagus increases the risk of developing adenocarcinoma of the esophagus.
Incorrect Options:
Option A - Complication of chronic GERD:

Page 16

778
• Barrett esophagus is considered a complication of chronic gastroesophageal reflux disease (GERD).
• Prolonged exposure to stomach acid can cause changes in the lining of the esophagus, leading to the
development of Barrett's esophagus.
Option B - Seen in 10% of individuals with symptomatic GERD:
• Barrett esophagus is estimated to occur in about 10% of individuals with symptomatic GERD.
Option D - Intestinal metaplasia:
• Barrett esophagus is characterized by the presence of intestinal metaplasia, where the normal
squamous epithelium of the esophagus is replaced by columnar epithelium resembling the lining of the
intestine.

Page 17

779
Salivary Glands
1. What is true regarding the condition seen in a 3-year-old with fever, puffy cheeks for 3 days, missed
immunizations since the 14th month, and histology showing the following features?
(or)
In a 3-year-old child presenting with fever and puffy cheeks, and a history of missed immunization
doses since 14 months of age, Which of the following statements regarding the affected gland is true?

A. 90% of tumors of this gland are malignant.


B. With an increase in age, the intraglandular adipocytes are found to decrease.
C. Pleomorphic Adenoma is the most common tumor of this gland.
D. This gland is mostly mucinous.
----------------------------------------
2. Which of the following is most likely associated with the observed histological feature in the image
shown?

A. It is associated with chronic smoking


B. It is associated with blockage of the duct
C. It is a normal histological feature mostly associated with the submandibular gland
D. It happens to be a stress adaptation
----------------------------------------
3. Please match each tumor with its corresponding slide: A. Pleomorphic adenoma 1. B. Papillary
cystadenoma Lymphomatosum 2. C. Mucoepidermoid Carcinoma 3. D. Adenoid Cystic Carcinoma 4.

780
A. Pleomorphic adenoma 1.

B. Papillary cystadenoma Lymphomatosum 2.

C. Mucoepidermoid Carcinoma 3.

D. Adenoid Cystic Carcinoma 4.

A. A-1, B-3, C-2, D-4


B. A-4, B-3, C-2, D-1

Page 2

781
C. A-4, B-1, C-2, D-3
D. A-2, B-3, C-4, D-1
----------------------------------------
4. What is the most relevant feature for the diagnosis in a 67-year-old male patient with painless,
slow-growing bilateral parotid gland swelling for 1.5 years, a 35-year history of smoking, and murky fluid
upon aspiration?
(or)
In a 67-year-old male with painless, bilateral parotid gland swellings for 1.5 years, and a history of
smoking, on aspiration murky fluid is obtained. Choose the most relevant feature regarding this
patient's diagnosis.
A. It is the most common benign tumor of the salivary gland in children
B. This tumor shows perineural invasion
C. It is the most common malignant tumor of the salivary gland.
D. The tumor shows a hot spot on the 99mTc scan.
----------------------------------------
5. What is the most relevant statement regarding a 49-year-old male patient with a mass on his left
cheek anterior to his ear, showing increased size over a year, MRI revealing a well-circumscribed
homogeneous mass within the left parotid gland, and biopsy displaying anastomosing strands of
stellate and fusiform epithelial cells mixed with myxoid stroma?
A. It is also called adenolymphoma or papillary lymphomatous cystadenoma.
B. It is most often localized to the submandibular gland
C. It is the most common malignant salivary gland tumor.
D. Recurrence often takes place after surgical resection
----------------------------------------
6. What is the most likely associated with the diagnosis for a 56-year-old female with a slowly
progressive swelling on her right cheek, exhibiting well-defined borders and a positive curtain sign,
assuming the most common benign tumor of the affected gland?
(or)
Which of the following is most likely associated with pleomorphic adenoma?
A. MAML2 gene defect
B. (6:9) translocation
C. HMGA1 Fusion
D. PLAG1 Fusion
----------------------------------------
7. How the most commonly occurring malignant tumor of the submandibular gland is usually graded?
A. Based on the number of Mucinous cells
B. Based on the nuclear: cytoplasmic ratio
C. It depends on the variation in epidermoid cells

Page 3

782
D. Depends on the Perineural invasion
----------------------------------------
8. Consider the following statements regarding Adenocystic Carcinoma. 1. Assertion: Perineural
invasion is a common histopathological finding in adenoid cystic carcinoma of the salivary glands. 2.
Reason: The presence of perineural invasion in ACC can be challenging to identify by FNAC alone.
A. 1 and 2 are true and 2 is the correct explanation of 1
B. 1 is true and 2 is false
C. 1 and 2 are true and 2 is not the correct explanation of 1
D. 2 is true and 1 is false
----------------------------------------
9. What is the most likely underlying cause for a 43-year-old woman presenting with difficulty
swallowing solid foods for 5 months, multiple joint pains for 7 months (relieved by rest), occasional
grittiness in her eyes, and bilateral parotid enlargement?
(or)
In a 43-year-old woman with difficulty swallowing solid food, joint pains, bilateral parotid gland
enlargement, and a sand-like feeling in her eyes, what is the most likely underlying cause?
A. Autoimmune
B. Infectious
C. Metabolic
D. Metastatic
----------------------------------------
10. What is the likely diagnosis of the painless mass located in front of the right ear, characterized by a
well-circumscribed, mobile mass at the angle of the jaw, with myoepithelial cells intermixed with
mucoid, myxoid, and cartilaginous areas?
(or)
A person who presented with a painless mass in front of her right ear showed a well-circumscribed,
mobile mass that can be seen at the angle of the jaw. Biopsy of this mass shows myoepithelial cells
intermixed with mucoid, myxoid, and cartilaginous areas. Considering the gross image shown, what is
the probable diagnosis?

A. Sialolithiasis
B. Tonsillitis

Page 4

783
C. Pleomorphic adenoma
D. Warthin’s tumor
----------------------------------------
11. In a chronic smoker with bilateral, slow-growing, fluctuant parotid gland enlargement, what is the
most likely diagnosis?
A. Warthin tumour
B. Pleomorphic adenoma
C. Mucoepidermoid carcinoma
D. Adenoid cystic carcinoma
----------------------------------------

Correct Answers
Question Correct Answer

Question 1 3
Question 2 3
Question 3 2
Question 4 4
Question 5 4
Question 6 4
Question 7 3
Question 8 3
Question 9 1
Question 10 3
Question 11 1

Solution for Question 1:


Correct Option C - Pleomorphic Adenoma is the most common tumor of this gland:
• The given slide shows the normal histology of parotid gland. Serous exocrine gland. Mumps mostly
affects the parotid gland and hence backs the histology being that of parotid.
• Pleomorphic Adenoma is the most common benign tumor of the parotid gland.
Incorrect Options:
Option A - 90% of tumors of this gland are malignant: This statement is false. Tumors of parotid are mo
stly benign (pleomorphic adenoma).
90% of tumors of minor salivary glands are malignant.
Option B - With an increase in age, the intraglandular adipocytes are found to decrease: This is not tru
e as the opposite is right, “With an increase in age the adipocyte count increases”.

Page 5

784
Option D - This gland is mostly mucinous: Parotid is mostly a serous gland, the given histology is a
justification all by itself.

Solution for Question 2:


Correct Option C - It is a normal histological feature mostly associated with the submandibular gland:
• The structure shown in the image is a demilune and it is a normal histological feature of a salivary
gland, mainly the submandibular gland. The serous acini is present on the mucinous acini like a mixed
structure.
Incorrect Options:
Option A - It is associated with chronic smoking:
• Chronic smoking may reduce salivary flow, contributing to a dry mouth (xerostomia). This reduction in
saliva can increase the risk of dental caries and other oral health issues.
Option B - It is associated with blockage of the duct:
• Smoking is a risk factor for the development of salivary gland stones (sialolithiasis), particularly in the
submandibular glands. This can lead to recurrent episodes of gland swelling and pain.
Option D - It happens to be a stress adaptation:
These are some basic stress adaptations in the salivary gland.
• Reduced Salivary Flow
• Altered Saliva Composition
• Increased Stress Hormones
• Changes in Immunoglobulins

Solution for Question 3:


Correct Option B – A-4, B-3, C-2, D-1:
The given microscopic features are correctly matched in option B
Pleomorphic Adenoma (Benign Mixed Tumor):
• Components: Epithelial and Mesenchymal. Epithelial Component: Comprising epithelial cells and
myoepithelial cells. Mesenchymal Component: Characterized by chondromyxoid stroma
• Epithelial Component: Comprising epithelial cells and myoepithelial cells.
• Mesenchymal Component: Characterized by chondromyxoid stroma

• Papillary Cystadenoma Lymphomatosum: Microscopic Features: Papillary projections are seen.
Papillae are lined by two rows of oncocytic cells. High lymphocyte count. Tumor exists as a cyst filled
with fluid.

Page 6

785
• Microscopic Features: Papillary projections are seen. Papillae are lined by two rows of oncocytic cells.
High lymphocyte count. Tumor exists as a cyst filled with fluid.
• Papillary projections are seen.
• Papillae are lined by two rows of oncocytic cells.
• High lymphocyte count.

• Tumor exists as a cyst filled with fluid.
• Mucoepidermoid Carcinoma: Microscopic Features: Mucin-containing cells. Squamous cells with a
pink appearance. Higher grade tumors have more epidermoid/squamous cells.
• Microscopic Features: Mucin-containing cells. Squamous cells with a pink appearance. Higher grade
tumors have more epidermoid/squamous cells.
• Mucin-containing cells.
• Squamous cells with a pink appearance.
• Higher grade tumors have more epidermoid/squamous cells.

• Epithelial Component: Comprising epithelial cells and myoepithelial cells.
• Mesenchymal Component: Characterized by chondromyxoid stroma

• Microscopic Features: Papillary projections are seen. Papillae are lined by two rows of oncocytic cells.
High lymphocyte count. Tumor exists as a cyst filled with fluid.
• Papillary projections are seen.
• Papillae are lined by two rows of oncocytic cells.
• High lymphocyte count.

• Tumor exists as a cyst filled with fluid.

Page 7

786
• Papillary projections are seen.
• Papillae are lined by two rows of oncocytic cells.
• High lymphocyte count.

• Tumor exists as a cyst filled with fluid.

• Microscopic Features: Mucin-containing cells. Squamous cells with a pink appearance. Higher grade
tumors have more epidermoid/squamous cells.
• Mucin-containing cells.
• Squamous cells with a pink appearance.
• Higher grade tumors have more epidermoid/squamous cells.

• Mucin-containing cells.
• Squamous cells with a pink appearance.
• Higher grade tumors have more epidermoid/squamous cells.

Page 8

787
• Adenoid Cystic Carcinoma (ACC): Microscopic Features: Cribriform pattern: Sieve-like pattern with
basement membrane-like material.
• Microscopic Features: Cribriform pattern: Sieve-like pattern with basement membrane-like material.
• Cribriform pattern: Sieve-like pattern with basement membrane-like material.

• Microscopic Features: Cribriform pattern: Sieve-like pattern with basement membrane-like material.
• Cribriform pattern: Sieve-like pattern with basement membrane-like material.

• Cribriform pattern: Sieve-like pattern with basement membrane-like material.

Page 9

788
Incorrect Options:
Option A, C, and D do not correctly match the tumors with their microscopic features.

Solution for Question 4:


Correct Option D - The tumor shows a hot spot on the 99mTc scan:
• The given description of symptoms and clinical features describe Warthin’s tumor.
• Warthin tumor is a benign neoplasm primarily found in the parotid gland.
• Of the given options, “The tumor shows a hot spot on the 99mTc scan” is consistent with features of
Warthin’s tumor.
Etiology:
• Its exact cause remains unclear, with proposed factors including Epstein Barr virus infection, tobacco
use, autoimmune disease, ionizing radiation, and chronic inflammation.
Malignant Transformation:
• Despite its benign nature in the majority of cases, Warthin tumors can rarely undergo malignant
changes, leading to the development of various types of carcinomas.
• Common histological types of malignant transformation include mucoepidermoid carcinoma,
squamous cell carcinoma, undifferentiated carcinoma, oncocytic adenocarcinoma, and
adenocarcinoma.
Incorrect Options:
Option A - It is the most common benign tumor of the salivary gland in children: Hemangioma is the mo
st common benign tumor of the salivary gland in children, not Warthin’s tumor.
Option B - This tumor shows perineural invasion: Adenoid cystic carcinoma shows perineural invasion
and not Warthin’s tumor.
Option C - It is the most common malignant tumor of the salivary gland: Mucoepidermoid carcinoma is
the most common malignant tumor of the salivary gland.

Solution for Question 5:


Correct Option D - Recurrence often takes place after surgical resection:
• Based on the scenario, it is likely that the patient has pleomorphic adenoma.
• Pleomorphic adenoma, the most common salivary gland tumor, is also known as a benign mixed
tumor due to its dual origin from the epithelium and myoepithelial parts.
• The pathogenesis of pleomorphic adenoma is unknown, radiation exposure is found to increase the
risk of development.
• Pleomorphic adenoma primarily affects the superficial lobe of the parotid gland.
• Pleomorphic adenoma has a highly varied appearance, hence the name. It is distinguished by the
coexistence of polygonal epithelial and spindle-shaped myoepithelial cells in various stroma matrices of
mucoid, myxoid, cartilaginous, or hyaline origin.

Page 10

789
• Currently, pleomorphic adenoma of the parotid gland is treated with either a superficial (Patey's
surgery) or whole parotidectomy, with the latter being the more commonly used procedure due to a
lower recurrence rate.
• A meticulous approach is essential to maintain the facial nerve.
Incorrect Options:
Option A - It is also called adenolymphoma or papillary lymphomatous cystadenoma:
• The benign salivary gland tumor papillary cystadenoma lymphomatosum (Warthin tumor,
adenolymphoma) arises almost exclusively in the parotid gland.
• It is the second most prevalent benign parotid neoplasm, accounting for 5 to 6% of all salivary gland
tumors.
• A rich lymphoid stroma and a double layer of oncocytic epithelium with a papillary and cystic
architectural pattern identify it histologically.
Option B - It is most often localized to the submandibular gland:
• Pleomorphic adenomas, notably abundant in the parotid gland, account for 70-80% of benign salivary
gland malignancies.
• Pleomorphic adenoma primarily affects the parotid gland's superficial lobe.
• The distribution of salivary glands occurs as follows: parotid gland: 84%, 8% for the submandibular
gland, and 6.5% for the small salivary glands.
Option C - It is the most common malignant salivary gland tumor:
• Pleomorphic adenoma is the most common benign salivary gland tumor.
• The most frequent type of salivary gland cancer is mucoepidermoid carcinoma.
• They appear less frequently in the mouth's submandibular glands or minor salivary glands.

Solution for Question 6:


Correct Option D - PLAG1 Fusion:
• In 90% of pleomorphic adenoma either PLAG1 or HMGA2 fusion is present .
Incorrect Options:
Option A - MAML2 gene defect: This is seen in association with Mucoepidermoid carcinoma. It is not a
ssociated with pleomorphic adenoma
Option B - (6:9) translocation: Adenoid Cystic Carcinoma occurs due to (6:9) translocation. It is not ass
ociated with pleomorphic adenoma.
Option C - HMGA1 Fusion: HMGA2 fusion is associated with pleomorphic adenoma and not HMGA1.

Solution for Question 7:


Correct Option C - It depends on the variation in epidermoid cells:
• The most common malignant submandibular gland tumor happens to be a mucoepidermoid tumor.

Page 11

790
• It has 2 components Mucinous cells and squamoid or epidermoid cells.
• Of the two components, the more the epidermoid component in the tumor the higher the grade of the
tumor.
Incorrect Options:
Option A - Based on the number of Mucinous cells: The more the number of mucinous cells the less ca
ncerous and more the tendency of the gland to keep its functionality.
Option B - Based on the nuclear: cytoplasmic ratio: This is mostly used to describe leukemias, rather th
an Mucoepidermoid carcinoma.
Option D - Depends on the Perineural invasion: Perineural invasion is essentially a
feature of Adenoid cystic carcinoma and it involves the painful progression of the tumor.

Solution for Question 8:


Correct Option C - 1 and 2 are true and 2 is not the correct explanation of 1:
• Assertion (Statement 1): Perineural invasion is a common histopathological finding in adenoid cystic
carcinoma (ACC) of the salivary glands. This statement is generally true, as ACC is known for its
propensity to infiltrate along nerves, resulting in perineural invasion.
• Reason (Statement 2): The presence of perineural invasion in ACC can be challenging to identify by
fine-needle aspiration cytology (FNAC) alone. This statement is also true because FNAC primarily
samples cells from the tumor but may not capture the full extent of perineural invasion, which is better
assessed in the excised tissue during histopathological examination.
• Explanation of the Connection (Correct Explanation): While both statements are true, the reason
(Statement 2) is not a direct explanation of the assertion (Statement 1). The first statement asserts a
histopathological finding in ACC, and the second statement provides an additional context related to
the limitations of FNAC in identifying perineural invasion. The two statements are related but not in a
direct explanatory relationship.
• The image given below is a diagrammatic representation of perineural invasion involving ACC.

Page 12

791
Option A - 1 and 2 are true and 2 is the correct explanation of 1: This option is incorrect because, while
both statements are true, the reason (Statement 2) is not a correct explanation of the assertion (State
ment 1). The reason provides additional context about the limitations of FNAC but doesn't directly expl
ain why perineural invasion is a common histopathological finding in ACC.
Options A, B and D are incorrect.

Solution for Question 9:


Correct Option A – Autoimmune:
• The clinical scenario describes a patient with Sjogren syndrome
• It is a systemic autoimmune disease that manifests as sicca symptoms.
• Sicca is a term used to describe dryness that typically affects the lips and eyes due to lacrimal and
salivary gland disease brought on by inflammation.
• Affected people can also experience extra-glandular involvement, such as the joints, skin, lungs,
gastrointestinal (GI) tract, nervous system, and kidneys.
• Sjogren syndrome often co-occurs with rheumatoid arthritis (RA) or systemic lupus erythematosus
(SLE) in up to 50% of cases.
• Genetic predisposition involving the MHC locus is believed to increase relative risk in persons sharing
haplotypes in the HLA-DQA DQB_ region.
• Mononuclear cells with lymphocyte infiltration are concentrated near blood arteries and excretory
ducts in the exocrine glandular tissue. The foci mainly consist of T cells.
Incorrect Options:
Option B – Infectious:
• Disorders produced by organisms, such as bacteria, viruses, fungi, or parasites, are known as
infectious diseases.

Page 13

792
• The mumps (a paramyxovirus) is the most common cause of epidemic parotitis among the numerous
viruses that cause parotitis.
• Coxsackie A virus, cytomegalovirus, echovirus, enterovirus, influenza, and parainfluenza viruses are
more viral culprits.
Option C – Metabolic:
• Sjogren disease is an autoimmune, not metabolic disorder.
Option D – Metastatic:
• Salivary gland tumors can be benign (noncancerous) or malignant (cancerous) (malignant).
• The majority of salivary gland tumors are benign.
• Tumors do not cause systemic effects except for metastatic and paraneoplastic diseases.

Solution for Question 10:


Correct Option C - Pleomorphic adenoma:
• Pleomorphic adenoma, the most common salivary gland tumor, is also known as a benign mixed
tumor due to its dual origin from the epithelium and myoepithelial parts.
• Pleomorphic adenoma typically manifests as a solitary movable, slow-growing, painless lump that
might be present for years.
• The pathogenesis of pleomorphic adenoma is unknown
• It is a tumor that displays a wide range of appearances due to the presence of both polygonal
epithelial and spindle-shaped myoepithelial cells in various stroma matrices, including mucoid, myxoid,
cartilaginous, or hyaline origin.
Incorrect Options:
Option A – Sialolithiasis:
• Results from impedance of salivary flow with stasis as a result of obstruction from a sialolith
• Affects the submandibular gland (80%) unilaterally without a side predilection
Option B – Tonsillitis:
• Tonsils are large non-encapsulated (or partially encapsulated) masses of lymphoid tissue that lie in
the walls of the pharynx and nasopharynx and at the base of the tongue.
Option D - Warthin’s tumor:
• Most are well circumscribed oval masses.
• Solid areas and multiple cysts with papillary projections are apparent on the cut surface

Solution for Question 11:


Correct Option A - Warthin tumor:
• Bilateral fluctuant parotid swelling in a chronic smoker is most likely indicative of Warthin's tumor.

Page 14

793
• Warthin tumor is the only benign salivary gland neoplasm connected with smoking.
• In some circumstances, it causes little or no pain, ear ringing, ear pain, and hearing loss.
• Warthin's tumor is a well-defined, spherical mass with solid sections and numerous cysts containing
mucoid fluid and papillary projections. It is made up of papillary-cystic structures lined by oncocytic
epithelial cells and a lymphoid stroma with germinal centers. The epithelial component has inner
columnar and outer cuboidal cells.
Incorrect Options:
Option B - Pleomorphic adenoma:
• Pleomorphic adenoma, the most common salivary gland tumour, is also known as a benign mixed
tumour due to its dual origin from the epithelium and myoepithelial parts.
• Pleomorphic adenoma has a highly varied appearance, hence the name.
• It is distinguished by the coexistence of polygonal epithelial and spindle-shaped myoepithelial cells in
various stroma matrices of mucoid, myxoid, cartilaginous, or hyaline origin.
Option C - Mucoepidermoid carcinoma:
• Mucoepidermoid carcinoma (MEC) is a malignant, locally invasive salivary gland tumour.
• It accounts for roughly 35% of all cancers of the major and minor salivary glands, with the parotid
gland being the most common.
• MECs can be found in the submandibular and minor salivary glands as well.
Option D - Adenoid cystic carcinoma:
• Adenoid cystic carcinoma (ACC) is a rare cancer of the secretory glands that most usually affects the
salivary glands.
• The most common manifestation of ACC is a slow-growing, hard, painless swelling in the head and
neck region. However, pain and paresthesia are not uncommon as the tumour is notorious for
perineural infiltration.

Page 15

794
Hyperbilirubinemia
1. What is the diagnosis for a 21-year-old male presenting with yellowing of the skin and sclera for three
days, with a history of binge drinking? Metabolic Panel Result Normal Range Total Bilirubin 3.0 mg/dl
0.1-1.2 mg/dl Direct Bilirubin 0.3 mg/dl 0.1-0.3 mg/dl Indirect Bilirubin 2.7 mg/dl 0.2-0.8 mg/dl
Metabolic Panel Result Normal Range
Total Bilirubin 3.0 mg/dl 0.1-1.2 mg/dl
Direct Bilirubin 0.3 mg/dl 0.1-0.3 mg/dl
Indirect Bilirubin 2.7 mg/dl 0.2-0.8 mg/dl

A. Gilbert syndrome
B. Crigler Najjar syndrome type II
C. Rotor syndrome
D. Dubin Johnson syndrome
----------------------------------------
2. What is the likely cause of jaundice in a five-day-old breastfed neonate born at 36 weeks gestation,
presenting with yellowing of skin, sclera, and mucous membranes, with the given metabolic panel?
Metabolic Panel Result Normal Range Total Bilirubin 4.6 mg/dl 0.1-1.2 mg/dl Direct Bilirubin 0.4 mg/dl
0.1-0.3 mg/dl Indirect Bilirubin 4.2 mg/dl 0.2-0.8 mg/dl
Metabolic Panel Result Normal Range
Total Bilirubin 4.6 mg/dl 0.1-1.2 mg/dl
Direct Bilirubin 0.4 mg/dl 0.1-0.3 mg/dl
Indirect Bilirubin 4.2 mg/dl 0.2-0.8 mg/dl

A. Immature UDP-glucuronosyltransferase
B. Mutation in UGT1A1 gene
C. Defective multi-drug resistance-associated protein 2
D. Defective organic anion transport protein
----------------------------------------
3. What is the likely etiology of the disease in a seventeen-year-old boy whose parents are blood
relatives, presenting with intermittent jaundice since birth, dark urine, yellowing of the skin, sclera, and
mucous membranes, normal liver function tests, no evidence of hemolysis, normal biopsy, and the
given metabolic panel? Metabolic Panel Result Normal Range Total Bilirubin 5.6 mg/dl 0.1-1.2 mg/dl
Direct Bilirubin 4.1 mg/dl 0.1-0.3 mg/dl Indirect Bilirubin 1.5 mg/dl 0.2-0.8 mg/dl
(or)
What is the Rotor syndrome's etiology?
Metabolic Panel Result Normal Range
Total Bilirubin 5.6 mg/dl 0.1-1.2 mg/dl
Direct Bilirubin 4.1 mg/dl 0.1-0.3 mg/dl
Indirect Bilirubin 1.5 mg/dl 0.2-0.8 mg/dl

A. Immature UDP-glucuronosyltransferase

795
B. Mutation in UGT1A1 gene
C. Defective multi-drug resistance-associated protein 2
D. Defective organic anion transport protein
----------------------------------------
4. What's the etiology of the disease in an 18-year-old girl with dark urine, yellowing of skin, and taking
oral contraceptives, with parents being blood relatives, no evidence of hemolysis, mild
hepatosplenomegaly on ultrasound, conjugated hyperbilirubinemia on metabolic panel, and liver biopsy
is given below?

A. Immature UDP-glucuronosyltransferase
B. Mutation in UGT1A1 gene
C. Defective multi-drug resistance-associated protein 2
D. Defective organic anion transport protein
----------------------------------------
5. Consider the following statements regarding the regeneration capabilities of the liver and choose the
right option.
A. The regeneration is attributed to stem cells in the canal of herring.
B. These stem cells of the liver are oval cells.
C. These stem cells span between the hepatocytes and Cholangiocytes.
D. All the statements are true.
----------------------------------------
6. What is the accurate statement about the diagnosis of an 8-year-old male presenting with persistent
jaundice since birth, laboratory findings showing unconjugated hyperbilirubinemia, and confirmed
complete deficiency of uridine diphosphate glucuronosyltransferase (UGT)?
(or)
In an 8-year-old male with persistent jaundice since birth and confirmed complete deficiency of uridine
diphosphate glucuronosyltransferase (UGT), which statement accurately describes his diagnosis?
A. The use of phenobarbitones has been proven to be a curative measure.
B. The underlying defect is complete abscence of the UGT1A1 gene.
C. Phototherapy is not effective in managing this condition.
D. The underlying defect is associated with a mild increase in bilirubin levels.

Page 2

796
----------------------------------------

Correct Answers
Question Correct Answer

Question 1 1
Question 2 1
Question 3 4
Question 4 3
Question 5 4
Question 6 2

Solution for Question 1:


Correct Option A- Gilbert Syndrome:
• In this case, the patient presents with jaundice and mild indirect hyperbilirubinemia..
• The blood report shows increased indirect bilirubin but less than three mg/dl. This transient rise was
also preceded by a stressor (binge drinking in this patient)
• Based on these clinical findings, Gilbert syndrome can be diagnosed.
Gilbert Syndrome:
• Most common inherited hyperbilirubinemia
• Mutation in UGT1A1 gene
• A mild reduction in UDP-glucuronosyltransferase enzyme activity causes an increase in indirect
bilirubin
• Alcohol consumption, stress, and fasting periods may trigger transient jaundice
• Mutation can be detected by using PCR.
• Normal liver function/ slightly deranged
• Slightly increased indirect bilirubin but less than 3mg/dl
Incorrect Options:
Option B- Crigler Najjar syndrome type II:
• In type II Crigler-Najjar syndrome, there is a partial deficiency of the enzyme, The patient may not
present with the symptoms as mentioned in the scenario.
Option C- Rotor syndrome:
• Rotor syndrome is another rare benign condition characterized by conjugated hyperbilirubinemia. It is
not associated with unconjugated hyperbilirubinemia, a key feature in the patient's presentation.
Option D- Dubin Johnson syndrome:
• Dubin-Johnson syndrome is characterized by conjugated hyperbilirubinemia due to impaired bilirubin
secretion. It does not typically present with unconjugated hyperbilirubinemia as seen in the patient's
case.

Page 3

797

Solution for Question 2:


Correct Option A- Immature UDP-glucuronosyltransferase:
• In this case, a five-day neonate presents with yellowing of skin and sclera.
• His indirect bilirubin is 4.2 mg/dl.
• All other laboratory findings are normal.
• Neonatal jaundice is caused due to immature UDP-glucuronosyltransferase enzymes in neonates.
Neonatal jaundice:
• The most common condition that occurs in newborn infants.
• It results from the hemolysis of fetal hemoglobin and immature hepatic bilirubin metabolism.
• Typically asymptomatic except for intermittent jaundice.
• Jaundice manifests after the first day of life.
• It is a diagnosis of exclusion.
• Laboratory tests should rule out all pathological causes.
• Phototherapy is the primary treatment for unconjugated hyperbilirubinemia.
• Exchange transfusion is the most rapid method for lowering serum bilirubin.
Incorrect Options:
Option B- Mutation in UGT1A1 gene:
• It causes Crigler-Najjar syndrome
• It presents as excessive, persistent neonatal jaundice
• Indirect bilirubin level rises to 20-50 mg/dl.
• Without treatment, it is incompatible with life.
Option C- Defective multi-drug resistance-associated protein 2:
• It causes Dubin-Johnson syndrome.
• Impaired excretion of conjugated bilirubin into the bile canaliculi.
• It causes an increase in direct serum bilirubin levels.
• A liver biopsy shows dark granular pigmentation
Option D- Defective organic anion transport protein:
• It causes Rotor syndrome.
• Impaired transport and reduced storage capacity of conjugated bilirubin, lead to increased levels in
plasma.
• It is predominantly associated with conjugated hyperbilirubinemia unlike in this neonate.
• Treatment is generally not required.

Page 4

798
Solution for Question 3:
Correct Option D- Defective organic anion transport protein:
• In this case, the patient presents with mild jaundice and a history of intermittent jaundice since birth.
• The metabolic panel shows moderate direct hyperbilirubinemia and mild, indirect hyperbilirubinemia.
• The liver biopsy is normal.
• These findings help in the diagnosis of rotor syndrome.
• Rotor syndrome is an autosomal recessive conjugated hyperbilirubinemia disorder (consanguinity in
the vignette)
• Rotor syndrome is caused by defective organic anion transport protein (OATP) in hepatocytes.
• It causes impaired transport and reduces the storage capacity of direct bilirubin in hepatocytes.
• In rotor syndrome, the liver has no dark pigmentation.
• It has a milder clinical presentation as compared to Dubin-Johnson syndrome.
Incorrect Options:
Option A- Immature UDP-glucuronosyltransferase:
• It causes neonatal jaundice.
• UDP-glucuronosyltransferase is an important conjugation enzyme and catalyses glucuronidation
reactions.
• Impaired UDP-glucuronosyltransferase causes an increase indirect serum bilirubin.
Option B- Mutation in UGT1A1 gene:
• It causes Crigler-Najjar syndrome
• While mutations in the UGT1A1 gene can present like Gilbert syndrome, which is characterized by
mild unconjugated hyperbilirubinemia, it typically presents later in life.
• Indirect bilirubin level rises to 20-50 mg/dl.
• Without treatment, it is incompatible with life.
Option C- Defective multi-drug resistance-associated protein 2:
• It causes Dubin-johnson syndrome.
• Impaired excretion of conjugated bilirubin into the bile canaliculi.
• It causes an increase in direct serum bilirubin levels.
• A liver biopsy would have demonstrated dark granular pigmentation unlike in this patient.

Solution for Question 4:


Correct Option C- Defective multi-drug resistance-associated protein 2:

Page 5

799
• In this case, the patient presented with jaundice, and the laboratory report showed increased direct
serum bilirubin.
• In the picture, the liver has a grossly black appearance. Microscopically, deep-brown, pigmented
granules can be seen.
• These features help in diagnosing Dubin-Johnson syndrome.
• MRP2 is genetically deficient in Dubin-Johnson syndrome and is associated with conjugated
hyperbilirubinemia.
Dubin-Johnson Syndrome:
• A defect in a canalicular protein called MRP2 (multi-drug resistant protein 2) leads to increased
conjugated bilirubin (CB).
• Autosomal recessive
• Mild to moderate jaundice
• May worsen because of contraceptive pills or pregnancy
• A liver biopsy shows dark, granular pigmentation due to the accumulation of epinephrine metabolites.
• Treatment is often not required.
Incorrect Options:
Option A- Immature UDP-glucuronosyltransferase:
• It causes neonatal jaundice.
• UDP-glucuronosyltransferase is an important conjugation enzyme and catalyses glucuronidation
reactions.
• Impaired UDP-glucuronosyltransferase causes an increase indirect serum bilirubin.
Option B- Mutation in UGT1A1 gene:
• It causes Crigler-Najjar syndrome
• It presents as excessive, persistent neonatal jaundice
• Indirect bilirubin level rises to 20-50 mg/dl.
• Without treatment, it is incompatible with life.
Option D- Defective organic anion transport protein:
• It causes Rotor syndrome.
• Impaired transport and reduced storage capacity of conjugated bilirubin
• It causes an increase in serum direct bilirubin concentration similar to Dubin-Johnson syndrome. The
liver does not appear black or contain pigmented granules in this disease unlike the patient described in
the vignette..
• Treatment is generally not required.

Solution for Question 5:


Correct Option D- All the statements are true:

Page 6

800
• The term "canal of Hering" typically refers to a small ductule located at the interface of the portal triad
and the liver lobule.
• The canal of Hering serves as a potential niche for hepatic progenitor cells, also known as oval cells.
These cells have the capability to differentiate into both hepatocytes (functional liver cells) and
cholangiocytes (cells lining the bile ducts). The presence of oval cells in the canal of Hering suggests a
location where regeneration and repair can occur in response to liver injury.
Option A- The regeneration is attributed to stem cells in the canal of herring:
• This statement is true.
Option B- These stem cells of the liver are oval cells:
• This statement is true .
Option C- These stem cells span between the hepatocytes and Cholangiocytes:
• This statement is true .

Solution for Question 6:


Correct Option B- The underlying defect is the complete absence of the UGT1A1 gene:
• Crigler-Najjar syndrome type 1 is characterized by a complete deficiency of uridine diphosphate
glucuronosyltransferase (UGT), leading to impaired conjugation of bilirubin. This results in
unconjugated hyperbilirubinemia. The underlying genetic defect in Crigler-Najjar syndrome type 1 is a
mutation in the UGT1A1 gene. The condition is associated with high levels of unconjugated bilirubin
and carries a significant risk of bilirubin encephalopathy, making it a challenging and potentially
life-threatening disorder.
Incorrect Options:
Option A- The use of phenobarbitones has been proven to be a curative measure:
• The only curative option available presently for Crigler-Najjar type I syndrome is liver transplantation.
Whereas the use of phenobarbitones is effective in type 2 Crigler-Najjar.
Option C- Phototherapy is not effective in managing this condition:
• The main target of treatment is a reduction in the level of unconjugated bilirubin using phototherapy
and plasmapheresis. Most patients have survival rates beyond puberty without significant brain
damage but, eventually, develop kernicterus later in life
Option D- The underlying defect is associated with a mild increase in bilirubin levels:
• Crigler-Najjar syndrome type 1 is associated with severe unconjugated hyperbilirubinemia. The
deficiency of uridine diphosphate glucuronosyltransferase (UGT) results in impaired conjugation of
bilirubin, leading to high levels of unconjugated bilirubin

Page 7

801
Hepatitis,Alcoholic Liver Disease,Hemochromatosis
1. Which of the followings tests can be used to screen for hereditary hemochromatosis?
A. Antimitochondrial antibodies
B. Ceruloplasmin
C. Heterophil antibodies
D. Transferrin saturation
----------------------------------------
2. What is the most likely diagnosis for a 42-year-old man with abdominal pain, hyperpigmented skin,
hepatomegaly, elevated blood glucose, increased liver enzymes, and a family history of hepatocellular
carcinoma, along with elevated serum ferritin of 1100 ng/ml?
A. Hemochromatosis
B. Thalassemia
C. Wilson's disease
D. Gibert syndrome
----------------------------------------
3. What is the primary gene associated with Alpha1 Antitrypsin Deficiency?
A. TP53 gene
B. SERPINA1 gene
C. BRCA1 gene
D. APC gene
----------------------------------------
4. What's the most likely cause of fever, malaise, icterus, and elevated AST/ALT in a 26-year-old
woman who ate raw oysters 3 weeks ago, with an enlarged liver and increased IgM?
(or)
Which of the following is the likely cause for a woman who reports yellowish discoloration in her eyes,
admits eating a dozen raw oysters, has icterus, a mildly enlarged liver, raised AST and ALT, and
elevated IgM levels?
A. Scombroid poisoning
B. Hepatitis A
C. Non-typhoidal salmonella infection
D. Hepatitis D
----------------------------------------
5. What is the likely diagnosis for a 24-year-old female with high-grade fever, yellowing of skin, loss of
appetite, vomiting, elevated liver enzymes, and the following histopathology findings of liver biopsy?

802
A. Wilson disease
B. Alcoholic hepatitis
C. Acute viral hepatitis
D. Autoimmune hepatitis
----------------------------------------
6. Which of the following is a characteristic feature of autoimmune hepatitis type 1?
(or)
Which of the following is a characteristic feature of Autoimmune hepatitis type 1?
A. ANA
B. Anti-mitochondrial
C. Anti- LKM-1
D. Anti SSA
----------------------------------------
7. Which of the following is a characteristic histologic finding seen in patients with chronic hepatitits B
infection?
A. Ground glass hepatocytes.
B. Portal tract expansion by lymphoid follicle
C. Prominent plasma cells
D. Mallory-Denk bodies
----------------------------------------
8. Which statement is not true regarding the underlying condition of a 14-year-old girl presenting with
deteriorating school performance, fatigue, lethargy, elevated urinary copper excretion, decreased
serum ceruloplasmin, and the following ophthalmic findings?

Page 2

803
A. Bronze skin pigmentation may be observed.
B. It involves mutation of ATP7B
C. It is an autosomal recessive disorder.
D. Treatment with D-Penicillamine or Trientine is effective.
----------------------------------------
9. What is the composition of the characteristic finding observed in the liver biopsy of a 45-year-old man
with chronic alcohol abuse, presenting with abdominal pain, jaundice, malaise, elevated liver enzymes,
centrilobular necrosis, and neutrophilic infiltration?

A. CK 4
B. CK 5/6
C. CK 7
D. CK 8/18
----------------------------------------
10. Which of the following special stains can be used to detect iron in the liver biopsy specimen of a
patient with hemochromatosis?
A. Von Kossa stain
B. Alcian blue stain
C. Perls' Prussian blue stain
D. Crystal violet stain
----------------------------------------

Page 3

804
11. What is true regarding a 23-year-old man presenting with abdominal pain, tremor, cognitive
impairment, yellowish skin discoloration of skin and sclera, Parkinsonian gait, and a wing-beating
tremor?

A. Councilman bodies are seen in chronic cases


B. The stain used is Prussian blue
C. This condition is commonly associated with diabetes.
D. Treatment may involve Trientine or penicillamines
----------------------------------------

Correct Answers
Question Correct Answer

Question 1 4
Question 2 1
Question 3 2
Question 4 2
Question 5 3
Question 6 1
Question 7 1
Question 8 1
Question 9 4
Question 10 3
Question 11 4

Solution for Question 1:


Correct Option D- Transferrin saturation:

• The screening test recommended for hemochromatosis is serum transferrin saturation.


• Molecular testing can be performed to look for mutations in the HFE gene on chromosome 6.

Page 4

805
• Its early detection and occasional phlebotomy can prevent multi-organ failure caused by iron
accumulation in tissues.
Incorrect Options:
Option A - Antimitochondrial antibodies:
• Antimitochondrial antibodies have no role in the diagnosis of hemochromatosis.
• It is helpful in the diagnosis of primary biliary cirrhosis.
Option B- Ceruloplasmin:
• Ceruloplasmin is a protein in the liver that stores and carries copper.
• Ceruloplasmin levels are decreased in Wilson's disease.
Option C- Heterophil antibodies:
• Heterophil antibodies are produced against poorly defined antigens.
• The testing for heterophil antibodies has a high sensitivity for primary acute EBV infection.

Solution for Question 2:


Correct Option A- Hemochromatosis:
• In this case, the patient has hyper-pigmented skin with hepatomegaly and a positive family history of
liver cancer.
• The patient had high serum iron with serum ferritin of 1100 ng/ml.
• Diabetes mellitus (elevated blood glucose levels in the vignette) is seen in patients with this disease
due to the destruction of pancreatic islets.
• Hepatocellular carcinoma is the most common cause of death in patients with Hereditary
hemochromatosis.
• These clinical findings help in the diagnosis of hemochromatosis.
Hemochromatosis
• Hemochromatosis is characterized by excessive iron accumulation in the parenchymal cells of various
organs, particularly the liver and pancreas.
• Hereditary hemochromatosis (primary hemochromatosis) is a homozygous recessive heritable
disorder caused by excessive iron absorption.
• Hemosiderosis (secondary hemochromatosis) denotes disorders associated with parenteral iron
administration (e.g., repetitive transfusions, ineffective erythropoiesis, increased iron intake, or chronic
liver disease).
Pathogenesis
• The adult form of hemochromatosis is caused by mutations of the HFE gene on the short arm of
chromosome 6.
• The most common mutation (more than 70% of patients) is a cysteine-to-tyrosine substitution at
amino acid 282 (C282Y) that inactivates HFE and reduces hepcidin expression.
Morphology

Page 5

806
• Iron demonstrated by Prussian blue accumulates as hemosiderin in various tissues—in decreasing
order of severity: liver, pancreas, myocardium, endocrine glands, joints, and skin.
• Cirrhosis and pancreatic fibrosis are the chief additional morphologic changes.
Clinical Features
• Fully developed cases of hemochromatosis uniformly exhibit micronodular cirrhosis; diabetes mellitus
and skin pigmentation occur in 75% to 80% of these individuals.
• The classic tetrad of features seen in this disease are hepatomegaly, abnormal skin pigmentation,
diabetes mellitus, and cardiac dysfunction.
• Iron accumulation is lifelong, but the injury caused by excessive iron is gradual, so symptoms usually
appear after age 40.
• Death can result from cirrhosis (and hepatocellular carcinoma) and cardiac involvement.
Treatment
• Regular phlebotomy is sufficient treatment; early diagnosis can enable average life expectancy, and
screening of genetic probands is therefore essential.
Iron is deposited in different tissues in the form of hemosiderin.
• This deposition and increased epidermal melanin production → Slate → grey colour to the skin.
• The development of diabetes in these patients is therefore termed bronze diabetes.
• In some patients, the presenting complaint is Hypogonadotropic hypogonadism.
Diagnosis:
• Clinical features of organ dysfunction:
• Heart
• Liver
• Pancreas
• Elevated serum ferritin and transferrin saturation
• Genetic testing confirms hereditary hemochromatosis
Incorrect Options:
Option B- Thalassemia:
• It is a hereditary haemoglobin disorder in alpha or beta globin chains.
• The critical feature in all forms is microcytic hypochromic anaemia.
• Patients have mild to severe hemolytic anaemia that often requires transfusions.
Option C- Wilson's Disease:
• It is an autosomal recessive metabolic disorder.
• Impaired copper excretion results in copper accumulating in the body.
• Mutation in ATP7B gene
• It should be suspected in nonspecific, non-infectious liver disease.
Option D- Gilbert Syndrome:
• It is the most common inherited hyperbilirubinemia disorder

Page 6

807
• Mutation in UGT1A1 gene
• Causes increased indirect bilirubin

Solution for Question 3:


Correct Option B- SERPINA1 gene:
• Alpha1 Antitrypsin Deficiency is primarily associated with the SERPINA1 gene located on the long
arm of chromosome 14.
• The various alleles of this gene, such as M, and Z, determine the levels of alpha-1 antitrypsin.
• PiMM: Represents the normal genotype. PiZZ: Represents the diseased genotype. PiMZ: Represents
the carrier genotype.
• Deficiency of an α1-Antitrypsin (α1-AT) enzyme is an autosomal recessive disorder marked by
deficient levels of serum protease inhibitor (Pi); Its deficiency leads to emphysema and hepatic disease
(cholestasis or cirrhosis).
Alpha1 Antitrypsin Deficiency is primarily associated with the SERPINA1 gene located on the long arm
of chromosome 14.
The various alleles of this gene, such as M, and Z, determine the levels of alpha-1 antitrypsin.
PiMM: Represents the normal genotype.
PiZZ: Represents the diseased genotype.
PiMZ: Represents the carrier genotype.

Incorrect Options:
Options B, C and D are not associated with α1-Antitrypsin, and is explained under correct option.

Solution for Question 4:


Correct Option B- Hepatitis A:
• In this case, a 26-year-old woman presents with fever, malaise, and yellowish discolouration of her
eyes for 5 days. The patient admitted to having a dozen raw oysters 3 weeks ago, and a physical
examination revealed a mildly enlarged liver with tenderness to palpation. Laboratory studies show a
markedly increased AST and ALT and increased IgM. This scenario indicates that the patient has a
hepatitis A infection.
• Hepatitis A, which is also known as infectious hepatitis, is a benign self-limited disease with an
incubation period of 2 to 6 weeks. Hepatitis A virus does not cause chronic hepatitis or a carrier state
and only rarely causes fulminant hepatitis.
• HAV has a very low fatality rate and occurs worldwide due to substandard hygiene and sanitation
conditions. It spreads by ingestion of contaminated water and foods, and it sheds in the stool. The
clinical disease tends to be mild in nature.

Page 7

808
• In developed countries, sporadic infections may be contracted by consuming raw or steamed shellfish
(oysters, mussels, calms), which concentrate the virus from seawater contaminated with human
sewage.
• Faecal shedding of the virus ends as the IgM titer rises. The IgM response declines in a few months,
accompanied by the appearance of IgG anti-HAV. The latter persists for life, providing immunity against
reinfection by all strains of HAV.
• Hepatitis A is a viral liver infection caused by the hepatitis A virus. It is commonly transmitted through
contaminated food or water. Symptoms include fever, malaise, jaundice (yellowish discoloration of the
eyes and skin), and hepatomegaly (enlarged liver). Laboratory findings often reveal elevated liver
enzymes (AST and ALT) and increased IgM antibodies specific to hepatitis A virus.
Incorrect Options:
Option A- Scombroid poisoning:
• It is also a type of poisoning caused by the ingestion of a certain fish. It usually resembles an allergic
reaction. This is due to high levels of histamines in the consumed seafood.
• Symptoms can present within a few minutes to several hours after fish consumption.
• Scombroid poisoning occurs due to the consumption of spoiled fish (not oysters) and can cause
symptoms such as flushing, palpitations, and abdominal cramps, but it does not typically present with
jaundice, elevated liver enzymes (AST and ALT), or increased IgM levels.
Option C- Non-typhoidal salmonella infection:
• This infection is also called salmonellosis, caused by the bacteria known as salmonella. It is a
gram-negative bacteria.
• The onset of the infection occurs after 6 to 72 hours of ingestion of salmonella bacteria. It presents
severe abdominal pain, vomiting, acute diarrhoea, and fever. It can last up to 2 to 7 days.
• Salmonella infections can cause gastroenteritis with symptoms such as diarrhea, abdominal cramps,
and fever. While salmonella infection can lead to liver involvement, it is less likely to cause jaundice and
markedly increased liver enzymes.
Option D- Hepatitis D:
• It is an infection of the liver that is caused by the delta virus. It is associated with people who have
hepatitis B infection already.
• Hepatitis D is caused by the hepatitis D virus and can only occur in individuals who are already
infected with hepatitis B. Hepatitis D infection can exacerbate the severity of hepatitis B, but it does not
typically present with a history of raw oyster consumption or markedly increased liver enzymes and
jaundice on its own.

Solution for Question 5:


Correct Option C- Acute viral hepatitis:
• The clinical features described in the vignette and histological findings of the liver biopsy showing
councilman bodies suggest a diagnosis of acute viral hepatitis
• Acute viral hepatitis is inflammation of the liver due to a viral infection..
• The inflammation is caused by infection with one of the five hepatitis viruses.

Page 8

809
• In most people, the inflammation begins suddenly and lasts only a few weeks.
• Symptoms range from mild to severe.
• Councilman bodies are apoptotic hepatocytes seen in acute viral hepatitis:
• Hepatocyte apoptosis is an active form of “programmed” cell death resulting in hepatocyte shrinkage.

Incorrect Options:
Option A- Wilson disease:
• Wilson's disease is an autosomal recessive disorder.
• It is characterised by excessive copper deposition in the tissues.
• Investigations include slit lamp examination for Kayser-Fleischer rings and reduced serum
caeruloplasmin.

Page 9

810
Option B - Alcoholic hepatitis:
• Alcoholic liver disease covers a spectrum of conditions: Alcoholic fatty liver disease Alcoholic hepatitis
Cirrhosis
• Alcoholic fatty liver disease
• Alcoholic hepatitis
• Cirrhosis
• Gamma-GT is characteristically raised.
• AST:ALT is normally > 2, and a ratio of > 3 strongly indicates acute alcoholic hepatitis.
• Alcoholic fatty liver disease
• Alcoholic hepatitis
• Cirrhosis
Option D- Autoimmune hepatitis:
• Autoimmune hepatitis is a chronic, progressive hepatitis. It has three types.
• ANA/SMA/LKM1 antibodies are seen, and there are raised IgG levels
• Liver biopsy shows piecemeal necroses or bridging necrosis.

Solution for Question 6:


Correct Option A- ANA:
• ANA is a characteristic feature of autoimmune hepatitis type 1.
• Autoimmune hepatitis is a chronic, progressive hepatitis. Autoimmune hepatitis is a condition of
unknown etiology.
• It is most commonly seen in young female patients.

Page 10

811
• It strongly correlates with other autoimmune disorders like hypergammaglobulinaemia and HLA B8,
DR3.
• There are three types of autoimmune hepatitis, according to the types of circulating antibodies
present.
Autoimmune hepatitis is a chronic, progressive hepatitis.
Type 1:
Type 2:
• Middle-aged to older individuals
• Antinuclear antibodies(ANA),
• Anti–smooth muscle actin antibodies (SMA).
• Children and teenagers
• Anti–liver kidney microsome-1 (anti-LKM-1) antibodies, directed against CYP2D6, and anti-liver
cytosol-1 (ACL-1) antibodies.
• It may present with signs of chronic liver disease, acute hepatitis: fever, jaundice and amenorrhoea.
• Liver biopsy reveals characteristic 'piecemeal necrosis', also called bridging necrosis
• Management includes steroids and other immunosuppressants, e.g. azathioprine.
Incorrect Options:
Option B- Anti mitochondrial:
• Anti-mitochondrial antibodies are seen in primary biliary cirrhosis.
• It is not a feature of autoimmune hepatitis type 1.
Option C- Anti- LKM-1:
• Anti-liver/kidney microsomal type 1 antibodies (LKM1) are seen in autoimmune hepatitis type 2.
• It affects children only.
Option D- Anti SSA:
• Anti-SSA is seen in Sjogren's syndrome.
• It is not a feature of autoimmune hepatitis type 1.

Solution for Question 7:


Correct Option A- Ground glass hepatocytes. :
• In chronic hepatitis B: “Ground-glass” hepatocytes—cells with endoplasmic reticulum swollen by
HBsAg—is a diagnostic hallmark. Immuno-staining can confirm the presence of viral antigens.
• “Ground-glass” hepatocytes—cells with endoplasmic reticulum swollen by HBsAg—is a diagnostic
hallmark.

• Immuno-staining can confirm the presence of viral antigens.

Page 11

812
• There is necrosis and inflammation of the liver with hepatitis B infection lasting for more than 6
months: A biopsy of the liver shows chronically inflamed fibrous septa and Hepatocellular nodules.
Surface antigen (HBsAg) is present in patients with chronic hepatitis B serum. If HBeAg is present, it is
often associated with the progression of the disease.
• A biopsy of the liver shows chronically inflamed fibrous septa and Hepatocellular nodules.
• Surface antigen (HBsAg) is present in patients with chronic hepatitis B serum.
• If HBeAg is present, it is often associated with the progression of the disease.
• Mode of transmission of HBV: Unprotected sex and intravenous drug abuse (sharing needles and
syringes) are the dominant modes of spread. Mother to child during birth
• Unprotected sex and intravenous drug abuse (sharing needles and syringes) are the dominant modes
of spread.
• Mother to child during birth
• The HBV-Prolonged incubation period (2 to 26 weeks).
• “Ground-glass” hepatocytes—cells with endoplasmic reticulum swollen by HBsAg—is a diagnostic
hallmark.

• Immuno-staining can confirm the presence of viral antigens.

• A biopsy of the liver shows chronically inflamed fibrous septa and Hepatocellular nodules.
• Surface antigen (HBsAg) is present in patients with chronic hepatitis B serum.
• If HBeAg is present, it is often associated with the progression of the disease.
• Unprotected sex and intravenous drug abuse (sharing needles and syringes) are the dominant modes
of spread.
• Mother to child during birth
Incorrect Options:
Option B- Portal tract expansion by lymphoid follicle:

Page 12

813
• This finding is not typically associated with chronic hepatitis B infection. Portal tract expansion by
lymphoid follicles is more commonly seen in chronic hepatitis C infection, where lymphoid aggregates
may form within the portal areas.
Option C- Prominent presence of plasma cells :
• While chronic hepatitis B infection may involve an inflammatory infiltrate, the prominent presence of
plasma cells is not a characteristic histologic finding. Plasma cells are more commonly associated with
conditions such as autoimmune hepatitis.
Option D- Mallory-Denk bodies :
• Mallory-Denk bodies are eosinophilic cytoplasmic inclusions in hepatocytes and are often associated
with alcoholic liver disease. They are not a specific feature of chronic hepatitis B infection

Solution for Question 8:


Correct Option A- Bronze skin pigmentation may be observed :
• The image shows Kayser-Fleischer rings seen in Wilson's disease (Also known as hepato-lenticular
degeneration).
• Kayser-Fleischer rings are golden to greenish-brown annular copper depositions located in the limbus
of the cornea.
• It is a rare, genetically inherited autosomal recessive liver disorder that results in the improper
metabolism of copper.
• This leads to the accumulation of excessive amounts of copper in the liver, brain, eyes, and other
organs and results in liver, neurologic or psychiatric symptoms.
• It is also known as hepatolenticular degeneration.
• Firstly, it attacks the liver, the central nervous system or both.
• Affect on liver symptoms: yellowing of the skin or eyes, abdominal swelling, vomiting of blood, and
abdominal pain.
• Neurological symptoms: changes in handwriting, drooling, tremors, and difficulty walking, talking, and
swallowing.
• Psychological symptoms: behavioural changes, depression, anxiety, impulsivity, and aggression.
• The most characteristic symptom of Wilson's disease is a rusty brownish ring around the eye's
cornea.
• In Wilson disease, liver damage begins by the age of 6 years but usually presents clinically in the
teenage or early twenties.
• Common signs of associated liver disease: Jaundice and yellowing in the skin, mucous membranes
and the membranes (sclera) that line the eye. Oedema of the legs Ascites The tendency for bruising
and prolonged bleeding Excessive tiredness (fatigue).
• Jaundice and yellowing in the skin, mucous membranes and the membranes (sclera) that line the eye.
• Oedema of the legs
• Ascites
• The tendency for bruising and prolonged bleeding

Page 13

814
• Excessive tiredness (fatigue).
• Treatment is aimed at reducing copper, which includes: Chelation with penicillamine or trientine, Oral
zinc. Low copper diet. Liver transplant in acute liver failure related to Wilson disease
• Chelation with penicillamine or trientine,
• Oral zinc.
• Low copper diet.
• Liver transplant in acute liver failure related to Wilson disease
• Bronze skin pigmentation is not a typical feature of Wilson's disease. The skin usually does not exhibit
bronze pigmentation in Wilson's disease.
• Jaundice and yellowing in the skin, mucous membranes and the membranes (sclera) that line the eye.
• Oedema of the legs
• Ascites
• The tendency for bruising and prolonged bleeding
• Excessive tiredness (fatigue).
• Chelation with penicillamine or trientine,
• Oral zinc.
• Low copper diet.
• Liver transplant in acute liver failure related to Wilson disease

Incorrect Options:
Options B, C and ,D are true regarding Wilson's Disease and is explained under the correct option.

Solution for Question 9:

Page 14

815
Correct Option D CK 8/18:
• The clinical scenario indicates Alcoholic hepatitis, and the histology shows dense eosinophilic
intracytoplasmic material which is consistent with Mallory- Denk bodies.
• Mallory-Denk bodies are known to contain cytokeratins, specifically CK8 and CK18. These
cytokeratins are part of the intermediate filament network in hepatocytes and are involved in the
structural integrity of the cell. In the context of alcoholic hepatitis, the accumulation of Mallory-Denk
bodies is a characteristic histopathological finding.

Mallory-Denk bodies are known to contain cytokeratins, specifically CK8 and CK18. These cytokeratins
are part of the intermediate filament network in hepatocytes and are involved in the structural integrity
of the cell. In the context of alcoholic hepatitis, the accumulation of Mallory-Denk bodies is a
characteristic histopathological finding.
Incorrect Options:
Option A- CK 4:
• Cytokeratin 4 is typically associated with stratified epithelia, such as the skin, and is not a component
of Mallory-Denk bodies found in the liver in the context of alcoholic hepatitis
Cytokeratin 4 is typically associated with stratified epithelia, such as the skin, and is not a
component of Mallory-Denk bodies found in the liver in the context of alcoholic hepatitis
Option B CK 5/6:
• Cytokeratin 5/6 is also not a component of Mallory-Denk bodies. CK5/6 is often used as a marker for
basal cells in stratified epithelia and is not typically associated with hepatocytes.
Cytokeratin 5/6 is also not a component of Mallory-Denk bodies. CK5/6 is often used as a
marker for basal cells in stratified epithelia and is not typically associated with hepatocytes.
Option C CK 7:
• Cytokeratin 7 is commonly found in glandular epithelia, including the bile ducts, but it is not a
component of Mallory-Denk bodies. In alcoholic hepatitis, the characteristic finding involves
cytokeratins 8 and 18.
Cytokeratin 7 is commonly found in glandular epithelia, including the bile ducts, but it is not a compone
nt of Mallory-Denk bodies. In alcoholic hepatitis, the characteristic finding involves cytokeratins 8
and 18.

Solution for Question 10:


Correct Option C- Perls' Prussian blue stain:
• Perls Prussian blue is commonly used in histopathology to detect iron in tissue or cell samples.
• It indicates "non-heme" iron in tissues, which is present in the form of hemosiderin and ferritin. It
doesn't stain the iron usually present in hemoglobin and myoglobin.
Incorrect Options:
Option A- Von Kossa stain:
• The von kossa stain can be used for the detection of the presence of abnormal calcium deposits in the
body.

Page 15

816
• It is used to diagnose calcinosis in all types of tissues.
• It is used to detect the following: Chondrocalcinosis Nephrocalcinosis Cutaneous calcinosis
Calcifications in tumors
• Chondrocalcinosis
• Nephrocalcinosis
• Cutaneous calcinosis
• Calcifications in tumors
• Chondrocalcinosis
• Nephrocalcinosis
• Cutaneous calcinosis
• Calcifications in tumors
Option B- Alcian blue stain:
• Alcian blue is a basic polyvalent dye.
• It is used to stain acidic polysaccharides, for example, glycosaminoglycans, in cartilages.
• It is used to visualise acidic epithelial and connective tissue mucins.
• Acidic mucins will be stained blue, while neutral ones will remain unstained.
Option D- Crystal violet stain:
• Crystal violet stain, also known as hexamethyl pararosaniline chloride, is a triarylmethane dye.
• It is used as a histological stain and classifies bacteria in Gram's staining method.

Solution for Question 11:


Correct Option D - Treatment may involve Trientine or penicillamines:
• The given image shows aKayser-Fleisher ring in a fully dilated eye. Given the symptoms and the
image, the diagnosis is Wilson’s disease. The treatment of choice for Wilson’s is chelating agents like
Trientine or penicillamines.
Incorrect Options:
Option A - Councilman bodies are seen in chronic cases:
• Councilman bodies are not a characteristic feature of Wilson's disease.
• Councilman bodies are apoptotic hepatocytes seen in conditions like viral hepatitis.
Option B - The stain used is Prussian blue:
• Prusian blue is used in Hemochromatosis.
• The stain typically used to visualize copper deposits in Wilson's disease is the rhodanine stain.
Option C - This condition is commonly associated with diabetes.:
• While copper buildup can occur in the pancreas, it does not typically lead to diabetes mellitus as a
primary manifestation of Wilson's disease.

Page 16

817
• Diabetes mellitus is commonly seen in patients with hemochromatosis due to the destruction of
pancreatic islets.

Page 17

818
Hepatic Tumors
1. Which of the following is associated with the development of a liver adenoma?
A. Hepatitis B
B. Polyvinyl chloride
C. Oral contraceptives
D. Polycythemia vera
----------------------------------------
2. What condition is associated with a 36-year-old male presenting with right upper quadrant abdominal
pain, jaundice, palpably enlarged liver, elevated liver enzymes, a single large liver mass on CT, and
markedly elevated serum α-fetoprotein?
A. Aflatoxin B1
B. C. Sinensis
C. Tetracyclines
D. Polyvinyl chloride
----------------------------------------
3. What is the most likely diagnosis for a 2-year-old male child presenting with mild self-limiting right
upper quadrant pain, weight loss, a large abdominal mass, hepatomegaly, normochromic normocytic
anemia, thrombocytopenia, moderately elevated LFTs, AFP levels at 150,000 mcg/mL, and
radiographic findings of a hyperechoic malignant liver tumor with patchy involvement?
A. Angiosarcoma
B. Hepatoblastoma
C. Hepatocellular carcinoma
D. Hepatic adenoma
----------------------------------------

Correct Answers
Question Correct Answer

Question 1 3
Question 2 1
Question 3 2

Solution for Question 1:


Correct Option C- Oral contraceptives:

• Benign liver tumours such as hepatic adenomas are commonly associated with the use of oral
contraceptives in young women.
• These tumours usually resemble hepatocellular carcinoma.

819
• It is usually subcapsular.
• It is a benign tumour and usually doesn’t cause any serious symptoms.
• In rare cases it can rupture and can cause intra-abdominal haemorrhage.
Incorrect Options:
Option A- Hepatitis B:
• Hepatitis B is a virus which can cause inflammation of the hepatocytes.
• It can lead to hepatocellular carcinoma.
Option B- Polyvinyl chloride:
• Polyvinyl chloride is used in IV bags, urine bags, oxygen masks, blood bags and tubing.
• Polyvinyl chloride predisposes to angiosarcoma of the liver.
Option D- Polycythemia vera:
• Polycythemia vera is a chronic proliferative neoplasm in which there is an irreversible increase in
erythrocytes and platelet count.
• It is associated with thrombosis of the hepatic veins.

Solution for Question 2:


Correct Option A- Aflatoxin B1:
• This is a case of hepatocellular carcinoma in which the patient presents with complaints of pain in the
right upper quadrant of the abdomen for one month.
• On examination, the patient has jaundice, and the liver is palpably enlarged.
• Laboratory studies demonstrate an increased level of liver enzymes. Computed tomography
demonstrates a single large mass in the right lobe of the liver, and serum α-fetoprotein is markedly
elevated, which is suggestive of hepatocellular carcinoma.
• Aflatoxin B1 can cause hepatocellular carcinoma.
• It is a fungal metabolite which is found on mouldy nuts and grain.
• It is a co-carcinogen with hepatitis B,
• It increases the incidence of hepatocellular carcinoma.
Incorrect Options:
Option B- C. Sinensis:
• Clonorchis. sinensis is a parasite.
• It is usually associated with cholangiocarcinoma.
Option C- Tetracyclines:
• Tetracyclines are a group of antibiotics that can cause a microvesicular fatty change in the liver.
• Ethanol, valproate and amiodarone are the two other drugs which can also cause fatty liver.
Option D- Polyvinyl chloride:

Page 2

820
• Polyvinyl chloride is used in IV bags, urine bags, oxygen masks, blood bags and tubing.
• It is usually associated with angiosarcoma of the liver.

Solution for Question 3:


Correct Option B- Hepatoblastoma:
• This is a case of hepatoblastoma in which the patient presents with complaints of weight loss and mild
right upper quadrant abdominal pain for two months that is self-limiting.
• Examination reveals a large abdominal mass with signs of hepatomegaly.
• Lab investigations reveal normochromic normocytic anaemia, thrombocytopenia, elevated LFTs, and
AFP levels. The signs and symptoms in the presence of lab findings give strong evidence of a liver
tumour.
• It is confirmed by radiography which shows a hyperechoic malignant liver tumour in the right upper
quadrant with patchy involvement.
• Hepatoblastoma is the most common malignant liver carcinoma in children and is usually seen in
children less than three years of age.
• It is characterized by the activation of the WNT pathway, β-Catenin mutations, loss of heterozygosity
of chromosome markers, and germline mutations in the APC tumour suppressor gene, which supports
the hypothesis that hepatoblastoma usually arises from an error in hepatogenesis.
Incorrect Options:
Option A- Angiosarcoma:
• It is a rare type of malignant neoplasm characterized by rapidly developing anaplastic cells which
begin from blood vessels of the liver.
• They are mostly seen in the liver, breast, spleen, bone, or heart and are often diagnosed in older
people.
Option C- Hepatocellular carcinoma:
• It is commonly seen in patients with pre-existing cirrhosis due to damage from chronic hepatitis B or C
or any drug or alcohol abuse.
• Radiographic findings often include mass in the liver along with cirrhosis and portal hypertension.
Option D- Hepatic adenoma:
• Hepatic adenomas are benign tumours commonly found in young women taking oral contraceptives.
• They usually develop from the proliferation of hepatic cells in an otherwise normal-appearing liver and
can transform into hepatocellular carcinoma.

Page 3

821
Previous Year Questions
1. Based on the physical examination, a 55-year-old male patient is diagnosed with hepatomegaly. In
addition, he reports having a dark tan despite not being exposed to the sun. The microscopic image of
his liver is provided on the slide below. The most probable condition affecting this patient is

A. Wilson’s disease
B. Autoimmune hepatitis
C. Alpha-1 antitrypsin deficiency
D. Hemochromatosis
----------------------------------------
2. 12. In a healthcare worker, the presence of HBsAg, HBeAg, and Anti-HBc: IgM indicates an acute
HBV infection with high infectivity. Which of the following is unlikely to be observed in liver biopsy in
such cases?
A. Ballooning change of hepatocytes
B. Ground glass hepatocytes
C. Focal or spotty necrosis
D. Confluent necrosis of hepatocytes
----------------------------------------
3. Which type of cells is responsible for the destruction of bacterial foreign bodies in the liver sinusoids?
A. Hepatocytes
B. Ito cells
C. Sinusoidal endothelial cells
D. Kupffer cells
----------------------------------------
4. Which marker is indicative of alcohol-related illness?
A. SGPT
B. SGOT
C. GGT
D. ANP

822
----------------------------------------
5. A chronic alcoholic patient presented with increasing abdominal girth. On examination, shifting
dullness was present, and the liver span was less than 7cm. Histopathological examination revealed
the presence of intracytoplasmic eosinophilic inclusions in hepatocytes. What is the content of these
inclusions?
A. Intermediate filaments
B. Actin
C. Microtubules
D. Fibronectin
----------------------------------------
6. Which gene mutation is observed in individuals with hemochromatosis?
A. HFE
B. PTEN
C. ATP7B gene
D. MRP2 protein
----------------------------------------
7. A patient suffering from cirrhosis and ascites has come to you. The external appearance of his liver
is depicted in the image below. Which of the following statements regarding it is correct?

A. Dark areas represent necrosis, white areas represent viable liver


B. Dark areas represent viable liver tissue, white areas represent necrosis
C. White areas represent central hepatocytes
D. Dark areas represent peri-portal hepatocytes
----------------------------------------
8. Which of the following is associated with Hepatic angiosarcoma?
A. Iron
B. Copper
C. Zinc
D. Arsenic
----------------------------------------

Page 2

823
9. Which cells in the liver carry out the same function as macrophages, playing a crucial role in
phagocytosis and chronic inflammation?
A. Hepatocytes
B. Merkel cells
C. Sinusoid all cells
D. Kupffer cells
----------------------------------------
10. In a 30-year-old woman, the clinical presentation includes an unstable walking pattern, memory
loss, unpredictable mood swings, and the presence of Kayser-Fleischer rings. The diagnosis made is
Wilson's disease. Among the following choices, which accurately identifies the specific gene mutation
responsible for this condition and its location?
A. ATP7B gene on chromosome 13
B. ATP7A gene on chromosome 13
C. ATP7B gene on chromosome 17
D. ATP7A gene on chromosome 17
----------------------------------------
11. Which of the following is not a characteristic feature of pancreatic carcinoma?
A. Cigarette smoking is a risk factor
B. Carcinoma of body pancreas often present with jaundice
C. They elicit intense desmoplastic response
D. Perineural invasion is common
----------------------------------------
12. What is the diagnosis for a patient with a positive serological status for HbsAg and HbeAg?
A. Acute viral hepatitis
B. Chronic viral hepatitis
C. Acute viral hepatitis with infectivity
D. Remote infection
----------------------------------------
13. Which of the following viruses is not associated with chronic viral hepatitis?
A. HBV
B. HCV
C. HDV
D. HEV
----------------------------------------
14. What is the underlying reason for the repeated occurrence of malaria in this 25-year-old patient who
is experiencing severe headache, body aches, irregular fever, nausea and vomiting, loss of taste,
hepatosplenomegaly, jaundice, decreased platelet count, and abnormal liver function tests?

Page 3

824
A. P. falciparum
B. P. vivax
C. Heterozygous sickle cell anaemia
D. Babesia
----------------------------------------
15. Which of the following is the most common cause of liver failure ?
(or)
Which of the following is the most common cause of liver failure ?
A. Alcohol
B. Paracetamol
C. CCI4
D. Pesticides
----------------------------------------
16. A 55-year-old man is found to have hepatomegaly on physical examination. He also complains of a
dark tan despite avoiding sun exposure. His liver microscopy is given on the slide below. This patient
most likely suffers from which condition?

A. Wilson’s disease
B. Autoimmune hepatitis
C. Alpha-1 antitrypsin deficiency
D. Hemochromatosis
----------------------------------------
17. Flask shaped ulcers in the colon are caused by:
A. Entamoeba histolytica
B. Giardia lamblia
C. Helicobacter pylori
D. Entamoeba vermicularis
----------------------------------------

Correct Answers
Page 4

825
Question Correct Answer

Question 1 4
Question 2 2
Question 3 4
Question 4 3
Question 5 1
Question 6 1
Question 7 1
Question 8 4
Question 9 4
Question 10 1
Question 11 2
Question 12 3
Question 13 4
Question 14 2
Question 15 2
Question 16 4
Question 17 1

Solution for Question 1:


• Based on the description given, the most likely diagnosis for this patient is hemochromatosis.
Hemochromatosis is a hereditary disorder characterized by the accumulation of iron in the body, which
can lead to damage to various organs, including the liver, heart, and pancreas. It can also cause skin
pigmentation changes, which may explain the patient's dark tan despite avoiding sun exposure.
• The hepatomegaly seen in this patient is consistent with liver damage, which is a common finding in
hemochromatosis. Microscopy of the liver would show increased iron deposition in the liver cells.
Incorrect choices:
• Option a. Wilson's disease: This is a rare inherited disorder that causes copper to accumulate in
various organs, including the liver, brain, and eyes. The symptoms of Wilson's disease usually begin in
childhood or early adulthood and can include neurological symptoms such as tremors, dystonia, and
difficulty with coordination. The liver damage associated with Wilson's disease can lead to
hepatomegaly, as well as other signs of liver disease such as jaundice, ascites, and elevated liver
enzymes.
• Option b. Autoimmune hepatitis: This is a chronic autoimmune disorder that causes inflammation and
damage to the liver. The symptoms of autoimmune hepatitis can include fatigue, joint pain, skin rashes,
and abdominal pain. Liver damage can lead to hepatomegaly, as well as other signs of liver disease
such as jaundice, ascites, and elevated liver enzymes.
• Option c. Alpha-1 antitrypsin deficiency: This is a genetic disorder that affects the production of a
protein called alpha-1 antitrypsin, which plays a role in protecting the lungs and liver from damage. The

Page 5

826
symptoms of alpha-1 antitrypsin deficiency can include respiratory symptoms such as shortness of
breath and coughing, as well as liver diseases such as hepatomegaly and elevated liver enzymes.

Solution for Question 2:


• Histologic analysis is likely to reveal ground-glass cells on liver biopsy in a chronic hepatitis B virus
(HBV) infection. Acute infection is with high infectivity and seropositive for HBsAg, HBeAg, and IgM
anti-HBc. Subsequently, the right solution to the inquiry is "Ground-glass cells".
Incorrect choices:
• Option a. Ballooning change of hepatocytes is a term used in histopathology to describe the swelling
and rounding up of hepatocytes, which are the liver cells responsible for producing and secreting bile,
filtering toxins, and performing many other essential functions. Ballooning change of hepatocytes is
often seen in liver diseases such as steatohepatitis, which is a type of nonalcoholic fatty liver disease
(NAFLD).
• Option c. The term "focal" or "spotty" necrosis refers to the occurrence of cell death in minute or small
clusters of cells. Various liver diseases, such as viral hepatitis, autoimmune liver diseases, and
drug-induced liver injury, can exhibit this kind of necrosis.
• Option d. Confluent necrosis of hepatocytes -This type of necrosis can be seen in conditions such as
hepatitis and drug-induced liver injury, among others. It is typically diagnosed based on histopathology,
which involves examining liver tissue under a microscope.

Solution for Question 3:


• Kupffer cells are scattered flat cells belonging to the reticuloendothelial system and function as
macrophages in the liver.
• They are located within the liver sinusoids and prevent liver injury or infection by breaking down and
engulfing toxic substances from the blood.
• They also help in the regulation of iron metabolism within the liver.
• Kupffer cells play a major role in maintaining the structural integrity of the liver by releasing certain
growth factors and enzymes that build the extracellular matrix of the liver.
Incorrect Choices:
• Option a. Hepatocytes: Hepatocytes are the functional cells of the liver that help in the secretion and
excretion of bile, detoxification of harmful substances like drugs or alcohol, metabolism of lipids,
proteins, and carbohydrates, and the manufacture of important plasma proteins like albumin,
fibrinogen, and prothrombin.
• Option b. Ito cells: Ito cells are fat-storing cells located within the space of Disse, which is the space
between hepatocytes and sinusoidal lining endothelial cells. Following liver injury, the ito cells get
activated and convert to myofibroblasts, which help in collagen deposition and result in hepatic fibrosis.
This, in turn, results in scar tissue formation within the liver.
• Option c. Sinusoidal endothelial cells: They are specialized cells that line the sinusoids. They have
small pores called fenestrations that help in the exchange of nutrients and other various small
molecules between the liver and the blood. They also release cytokines that help regulate the liver's

Page 6

827
immune system.

Solution for Question 4:


Correct Option C: GGT
• GGT is an enzyme found in various tissues, including the liver. Elevated levels of GGT in the blood
are often associated with liver dysfunction and can be indicative of alcohol-related liver disease.
Alcohol consumption increases the production and release of GGT from liver cells into the bloodstream.
Therefore, elevated GGT levels can be used as a specific marker for alcohol abuse or alcohol-related
liver damage.
Incorrect options
Option A and B: (SGPT, also known as ALT) and option b (SGOT, also known as AST) are liver enzym
es that can be elevated in various liver diseases, including alcohol-related liver damage. However, they
are not specific to alcohol-related disease and can be elevated in other liver conditions as well.
Option D: (ANP, Atrial Natriuretic Peptide) is a hormone released by the heart in response to increased
blood volume and pressure. It is not specifically associated with alcohol-related disease.

Solution for Question 5:


Correct Option A - Intermediate filaments:
• Chronic alcoholism can lead to various liver complications, including alcoholic liver disease.
• One of the histopathological features seen in alcoholic liver disease is the presence of
intracytoplasmic eosinophilic inclusions in hepatocytes, known as Mallory-Denk bodies or Mallory
bodies.
• Mallory bodies are composed of abnormal aggregates of intermediate filaments within hepatocytes.

Page 7

828
Incorrect Options:
Options B, C, and D are incorrect.

Solution for Question 6:


Correct Option A:
The gene mutation responsible for hemochromatosis is the HFE gene present on chromosome 6.
Causes of hemochromatosis:
• Genetic causes: HFE gene, present on chromosome 6. HJV gene (HemoJuvelin gene) - Causes
juvenile hemochromatosis.
• HFE gene, present on chromosome 6.
• HJV gene (HemoJuvelin gene) - Causes juvenile hemochromatosis.
• Acquired causes: Frequent blood transfusions Bantu Siderosis
• Frequent blood transfusions
• Bantu Siderosis
• HFE gene, present on chromosome 6.
• HJV gene (HemoJuvelin gene) - Causes juvenile hemochromatosis.
• Frequent blood transfusions
• Bantu Siderosis
Incorrect Options:
Option B: PTEN is a tumor suppressor gene and its mutation is associated with Cowden syndrome.
Option C: ATP7B gene mutation causes Wilson’s disease.
Option D: MRP2 is a protein associated with Dubin-Johnson syndrome.

Solution for Question 7:


Correct Option A:
In the given gross appearance of the liver with cirrhosis and ascites, the dark areas represent necrosis,
while the white areas represent viable liver tissue. This observation is consistent with the typical featur
es of cirrhosis.
Cirrhosis is a chronic liver disease characterized by fibrosis and nodular regeneration of the liver paren
chyma. It results from various causes, such as chronic alcohol abuse, viral hepatitis, or metabolic disor
ders. In cirrhosis, the liver undergoes extensive remodeling, leading to the destruction of normal liver ar
chitecture and the formation of fibrous bands.
The dark areas in the liver represent areas of necrosis. Necrosis refers to the death of liver cells due to
insufficient blood supply or impaired liver function. These areas may appear darker due to the loss of c
ellular integrity and blood congestion.

Page 8

829
The white areas in the liver represent viable liver tissue. These areas contain functional hepatocytes th
at are responsible for the normal liver functions, such as detoxification, metabolism, and synthesis of pr
oteins. The presence of viable liver tissue is essential for the overall liver function despite the structural
abnormalities caused by cirrhosis.
Incorrect Option:
Option B. Dark areas represent viable liver tissue, white areas represent necrosis:
This statement is incorrect. In cirrhosis, the dark areas represent necrosis, not viable liver tissue. The l
oss of viable liver tissue and its replacement by fibrous tissue are characteristic features of cirrhosis.
Option C. White areas represent central hepatocytes:
This statement is also incorrect. In cirrhosis, the white areas do not specifically represent central hepat
ocytes. The nodular regeneration of the liver parenchyma in cirrhosis can lead to the formation of rege
nerative nodules, which can vary in size and location throughout the liver.
Option D. Dark areas represent peri-portal hepatocytes:
This statement is not accurate. In cirrhosis, the dark areas do not specifically represent
peri-portal hepatocytes. The destruction of liver architecture in cirrhosis can affect
hepatocytes in both peri-portal and centrilobular regions, leading to diffuse
abnormalities in liver structure and function.

Solution for Question 8:


Correct Option: D
Hepatic angiosarcoma is associated with exposure to Arsenic.
Explanation of the options:
Option A: Iron: Iron overload conditions, such as hereditary hemochromatosis, can lead to liver damag
e and increase the risk of hepatocellular carcinoma (the most common type of primary liver cancer). Ho
wever, iron is not specifically associated with hepatic angiosarcoma.
Option B: Copper: Copper metabolism disorders, such as Wilson's disease, can cause copper accumul
ation in the liver, leading to liver damage. However, copper is not specifically associated with hepatic a
ngiosarcoma.
Option C: Zinc: Zinc deficiency or excess can affect liver function and contribute to liver disease. Howe
ver, zinc is not specifically associated with hepatic angiosarcoma.
Option D: Arsenic: Exposure to arsenic, either through environmental sources (such as contaminated
water or air) or occupational exposure, is strongly associated with the development of hepatic angiosar
coma. Arsenic is a known carcinogen and has been linked to various types of cancers, including liver c
ancer. Hepatic angiosarcoma is one of the rare types of liver cancer that has a
significant association with arsenic exposure.
Therefore, the correct answer is Arsenic.

Solution for Question 9:

Page 9

830
Correct Option: D
The correct answer is Kupffer cells.
Explanation for the options:
Option A: Hepatocytes: Hepatocytes are the main functional cells of the liver responsible for various m
etabolic functions, such as detoxification, synthesis of proteins, and storage of nutrients. While hepatoc
ytes are involved in many liver functions, they do not primarily perform phagocytosis and chronic inflam
mation.
Option B: Merkel cells: Merkel cells are specialized cells found in the skin involved in the sensation of t
ouch and tactile discrimination. They are not present in the liver and are not involved in phagocytosis o
r chronic inflammation.
Option C: Sinusoidal cells: Sinusoidal cells refer to the cells lining the sinusoids in the liver, which are s
pecialized blood vessels. These cells include endothelial cells, Kupffer cells, and stellate cells. Among t
hese, Kupffer cells are the specific cells involved in phagocytosis and chronic inflammation.
Option D: Kupffer cells: Kupffer cells are specialized macrophages located within the liver sinusoids. T
hey play a crucial role in phagocytosis, removing pathogens, damaged cells, and cellular debris from th
e bloodstream. They are also involved in the regulation of immune responses and the promotion of chr
onic inflammation when necessary.
Therefore, Kupffer cells in the liver perform similar functions to macrophages, including phagocytosis a
nd participation in chronic inflammation.

Solution for Question 10:


Correct Answer: A
• Wilson disease is an autosomal recessive disorder and is the result of pathogenic variants in ATP7B,
a gene encoding a copper transport protein, ATP7B, on chromosome 13. Wilson disease or
hepatolenticular degeneration is an autosomal recessive disease that results in an excess copper
build-up in the body. It primarily affects the liver and basal ganglia of the brain, but it can affect other
organ systems too. Symptoms usually are related to the brain and liver. Liver-related symptoms include
vomiting, weakness, ascites, swelling of the legs, yellowish skin, and itchiness. Brain or neurological
symptoms include tremors, muscle stiffness, trouble speaking, personality changes, anxiety, and
auditory or visual hallucinations.
Incorrect Choices:
Option B. ATP7A gene on chromosome 13: The ATP7A gene is located on chromosome 13 and it is a
code for a protein that helps the body to absorb copper from the intestines. Mutation of the ATP7A gen
e can cause a deficiency of copper in the intestine and cause Menkes disease. Symptoms of Menkes d
isease include Lack of weight gain, Lack of normal growth (failure to thrive), Seizures, Lack of muscle d
evelopment, Poor head control, Reduced muscle tone (hypotonia), Sagging cheeks
Abnormal development of the sternum and rib cage (pectus excavatum), Low body temperature (hypot
hermia) in the newborn period, Intellectual disability in older children, etc.
Option C. ATP7B gene on chromosome 17: It is specifically found on the long arm of chromosome 17,
at position q21.31. The ATP7B gene provides instructions for making a
protein called copper-transporting ATPase 2.

Page 10

831
Option D. ATP7A gene on chromosome 17: The ATP7A gene is located on chromosome 13q14. and e
ncodes a protein called copper-transporting ATPase 1
(ATP7A). The ATP7A gene provides instructions for making a
protein that is important for regulating copper levels in the body.

Solution for Question 11:


Correct Option B: Carcinoma of body pancreas often present with jaundice
• Carcinoma of the body of the pancreas typically does not cause early jaundice.
• Jaundice is more commonly associated with tumors located in the head of the pancreas, where they
can obstruct the common bile duct and lead to the accumulation of bilirubin in the bloodstream.
• In contrast, tumors in the body of the pancreas may cause symptoms such as abdominal pain, weight
loss, and digestive issues but are less likely to cause jaundice.
Incorrect Options:
Option A: Cigarette smoking is a risk factor- Cigarette smoking is a well-established risk factor for panc
reatic carcinoma. It is one of the most significant environmental risk factors for developing pancreatic c
ancer, increasing the likelihood of developing the disease.
Option C: They elicit intense desmoplastic response- Pancreatic carcinoma is known for its intense des
moplastic response. The tumor microenvironment in pancreatic cancer is characterized by the prolifera
tion of fibrous connective tissue, which creates a dense stromal reaction around the tumor cells. This d
esmoplastic response contributes to the poor prognosis and therapeutic challenges associated with pa
ncreatic carcinoma.
Option D: Perineural invasion is common- Perineural invasion is a
common feature of pancreatic carcinoma. The tumor cells have a propensity to invade and infiltrate alo
ng the nerves in the vicinity of the pancreas, which can contribute to the spread and metastasis of the
disease.

Solution for Question 12:


Correct Option C. Acute viral hepatitis with infectivity.
• The serological status of HbsAg (hepatitis B surface antigen) positive and HbeAg (hepatitis B e
antigen) positive indicates an active infection with hepatitis B virus (HBV).
• This combination suggests that the patient is in the acute phase of hepatitis B infection and is highly
infectious.
• HbsAg is the earliest and most reliable marker of HBV infection.
• HbeAg is a marker of active viral replication and high infectivity.
• When both HbsAg and HbeAg are positive, it indicates a high level of viral replication and an
increased risk of transmission to others.
Incorrect options:
Option A. Acute viral hepatitis: While the patient's serological status indicates an acute infection, the pr
esence of HbeAg suggests a higher infectivity level. Therefore, "acute viral hepatitis with infectivity" is a

Page 11

832
more appropriate diagnosis.
Option B. Chronic viral hepatitis: Chronic viral hepatitis is characterized by persistent HBV infection last
ing for at least 6
months. The patient's serological status indicates an acute infection rather than chronic.
Option D. Remote infection: The patient's serological status indicates an active infection, not a
remote or past infection.

Solution for Question 13:


Correct Option D.
• HEV: Hepatitis E virus usually causes an acute self-limiting infection, and chronic infection is rare.
HEV is mainly transmitted through the fecal-oral route and is commonly associated with acute hepatitis.
Incorrect options
Option A. HBV: Chronic infection with hepatitis B
virus can lead to chronic hepatitis, which may progress to liver cirrhosis and hepatocellular carcinoma.
Option B. HCV: Chronic infection with hepatitis C virus is a
major cause of chronic liver disease, leading to cirrhosis and hepatocellular carcinoma.
Option C. HDV: Hepatitis D virus causes co-infection or superinfection in individuals already infected wi
th HBV. Chronic HDV infection can lead to severe liver disease.

Solution for Question 14:


Correct Option B.
• The most common cause of recurrent malaria in this patient is P. vivax.
• P. vivax can cause relapses due to the formation of dormant liver-stage parasites called hypnozoites.
• These hypnozoites can reactivate weeks to months after the initial infection, leading to recurrent
episodes of malaria.
Incorrect options:
Option A. P. falciparum: While P. falciparum is the most deadly species of malaria and can cause seve
re symptoms, it is not the most common cause of recurrent malaria in this case.
Option C. Heterozygous sickle cell anemia: Heterozygous sickle cell trait provides some degree of prot
ection against severe malaria caused by P. falciparum but does not directly cause recurrent malaria.
Option D. Babesia: Babesia is a parasitic infection transmitted by ticks and can cause symptoms simila
r to malaria. However, it is not the most common cause of recurrent malaria in this patient.

Page 12

833
Solution for Question 15:
Correct Option:
Option B.
• Paracetamol: Paracetamol overdose is the most common cause of acute liver failure, particularly in
cases of intentional or accidental overdose. Paracetamol is metabolized in the liver, and high doses or
overdose can overwhelm the liver's detoxification mechanisms, leading to liver damage.
Incorrect Options:
Option A. Alcohol: While alcohol abuse can lead to liver damage and cirrhosis, it is not the most comm
on cause of acute liver failure. Chronic alcohol abuse can contribute to liver diseases such as alcoholic
hepatitis and alcoholic cirrhosis.
Option C. CCI4 (carbon tetrachloride): CCI4 is a toxic chemical that was historically used as a solvent
and industrial agent. Exposure to CCI4 can cause severe liver damage and hepatotoxicity, but it is not t
he most common cause of liver failure.
Option D. Pesticides: Pesticides can have hepatotoxic effects and contribute to liver damage in certain
circumstances, such as occupational exposure or accidental ingestion. However, they are not the most
common cause of liver failure.

Solution for Question 16:


Correct Option D - Hemochromatosis:
• Hemochromatosis is a genetic disorder characterized by excessive iron absorption and accumulation
in various organs, including the liver.
• Hepatomegaly is a common finding in hemochromatosis due to iron deposition in hepatocytes.
• The liver microscopy image showing hepatocellular iron staining supports the diagnosis of
hemochromatosis in this patient.
• Darkening of the skin, known as bronze diabetes or dark tan, is another characteristic feature of
hemochromatosis resulting from iron deposition in the skin.
Incorrect Options:
Option A - Wilson's disease:
• Wilson's disease is a genetic disorder characterized by impaired copper transport, leading to copper
accumulation in various organs, including the liver and brain.
• It typically presents with hepatic and neurological symptoms, such as hepatomegaly, jaundice, and
neurologic manifestations.
Option B - Autoimmune hepatitis:
• Autoimmune hepatitis is an autoimmune disorder characterized by immune-mediated inflammation of
the liver.
• It can present with hepatomegaly, jaundice, and elevated liver enzymes.
Option C - Alpha-1 antitrypsin deficiency:
• Alpha-1 antitrypsin deficiency is a genetic disorder that predisposes individuals to liver and lung
diseases.

Page 13

834
• Liver involvement in alpha-1 antitrypsin deficiency typically presents as neonatal cholestasis or
later-onset liver disease.

Solution for Question 17:


Correct Option A - Entamoeba histolytica:
• Entamoeba histolytica is a protozoan parasite that can cause intestinal infections, specifically amoebic
dysentery.
• One of the characteristic features of amoebic dysentery is the presence of flask-shaped ulcers in the
colon.
• These ulcers are caused by the invasion of the intestinal wall by the parasite.
Incorrect Options:
Options B, C, and D do not cause flask-shaped ulcers in the colon.

Page 14

835
Cystic Diseases of Kidney
1. What is true regarding the condition of a 32-year-old male with PKD1 gene mutation, hypertension,
recurrent UTIs, and multiple kidney cysts?
A. Cysts primarily affect the distal portions of the kidneys in PKD1 mutations
B. PKD2 mutations are associated with a less severe form of ADPKD
C. Adult cystic kidney disease is linked to mutations in genes located on chromosome 6
D. The affected protein in this case is cystatin
----------------------------------------
2. What protein defect is likely present in a 5-year-old child with abdominal distension, hypertension,
and cysts on the inner surface of the kidneys since birth?
A. Cystin
B. Polycystin
C. Fibrocystin
D. Alpha-galactosidase A
----------------------------------------
3. Choose the correct option regarding the condition depicted in the image below:

A. Treatment approaches may include: BP control, Management of renal insufficiency, and treatment of
complications
B. Complications include anemia
C. Autosomal Recessive (AR) form is seen in children
D. All the above
----------------------------------------
4. Match the gross images to the respective relevant factors: A. 1. Medullary sponge kidney B. 2.
ADPKD C. 3. Medullary cystic kidney D. 4. ARPKD

836
A. 1. Medullary sponge kidney

B. 2. ADPKD

C. 3. Medullary cystic kidney

D. 4. ARPKD

A. A-3 ,B-1, C-4, D-2


B. A-2 ,B-1, C-4, D-3

Page 2

837
C. A-2 ,B-4, C-1, D-3
D. A-2 ,B-3, C-4, D-1
----------------------------------------
5. What is the most likely diagnosis for a 36-year-old man presenting with flank pain, red-colored urine,
a history of hypertension, recent treatment for a urinary tract infection, and a family history of his father
dying at the age of 40 from a cerebral hemorrhage?
A. Autosomal recessive polycystic kidney disease
B. Autosomal dominant polycystic kidney disease
C. Transitional cell carcinoma of the bladder
D. Renal cell carcinoma
----------------------------------------
6. What's causing pulmonary distress in a newborn with club feet, severe respiratory distress, facial
abnormalities, and multiple cysts in the kidney on ultrasonography, born at 30 weeks from an
uncomplicated pregnancy?
A. Neonatal respiratory distress syndrome
B. Diffuse Pulmonary hypoplasia
C. Congenital diaphragmatic hernia
D. Meconium Aspiration syndrome
----------------------------------------
7. Which of the following statements is not true regarding the condition depicted by the gross and
microscopic appearance of the urinary bladder of an immunocompromised patient shown below?

A. Phagolysosome defect
B. The slide demonstrates a Von Hansemann Cell
C. Calcium deposition in the macrophage are calledMichaelis-Gutmann Body
D. It is a premalignant condition
----------------------------------------
8. Which protein is the most abundant protein found in the glomerular basement membrane?
A. Type 1 collagen
B. Laminin

Page 3

838
C. Type 3 collagen
D. Heparan sulfate
----------------------------------------
9. Identify the stain used in the histopathological image given below:

A. Prussian blue stain


B. Gomori Methenamine Silver stain
C. Masson trichome stain
D. Toluidine blue stain
----------------------------------------
10. Match the following: 1 Putty kidney A Malignant hypertension 2 Leather grain kidney B Amyloidosis
3 Flea bitten kidney C Benign hypertension 4 Waxy kidney D Tuberculosis
1 Putty kidney A Malignant hypertension
2 Leather grain kidney B Amyloidosis
3 Flea bitten kidney C Benign hypertension
4 Waxy kidney D Tuberculosis

A. 1-D, 2-B, 3-A, 4-C


B. 1-D, 2-A, 3-C, 4-B
C. 1-C, 2-B, 3-D, 4-A
D. 1-D, 2-C, 3-A, 4-B
----------------------------------------
11. Which of the following is a characteristic feature of chronic pyelonephritis (CPN)?
A. Enlarged, smooth kidney
B. Symmetrically scarred kidney
C. Glomerular hyperplasia
D. Thyroidization of Tubules
----------------------------------------
12. Which of the following causes an asymmetrical granular contracted kidney?
A. Acute glomerulonephritis

Page 4

839
B. Chronic glomerulonephritis
C. Benign nephrosclerosis
D. Chronic pyelonephritis
----------------------------------------

Correct Answers
Question Correct Answer

Question 1 2
Question 2 3
Question 3 4
Question 4 2
Question 5 2
Question 6 2
Question 7 4
Question 8 2
Question 9 2
Question 10 4
Question 11 4
Question 12 4

Solution for Question 1:


Correct Option B - PKD2 mutations are associated with a less severe form of ADPKD:
• ADPKD is commonly associated with mutations in two genes: PKD1 and PKD2.
• PKD1 mutations generally result in a more severe form of ADPKD, while PKD2 mutations are
associated with a milder course.
• The genes linked to ADPKD are located on chromosomes 16 (PKD1) and 4 (PKD2).
• The affected proteins are polycystin-1 (encoded by PKD1) and polycystin-2 (encoded by PKD2)
Incorrect Options:
Option A - Cysts primarily affect the distal portions of the kidneys in PKD1 mutations:
• This statement is incorrect. In ADPKD, cysts primarily affect the proximal and distal portions of the
kidneys. PKD1 mutations are associated with a more severe form of the disease, and cysts are
widespread throughout the kidneys, not limited to the distal portions
Option C - ADPKD is linked to mutations in genes located on chromosome 6:
• This statement is incorrect. ADPKD is linked to mutations in two genes: PKD1 on chromosome 16 and
PKD2 on chromosome 4. Chromosome 6 is not associated with ADPKD, It is associated with
Autosomal Recessive Kidney disease
Option D - The affected protein in this case is cystatin:

Page 5

840
• This statement is incorrect. The affected proteins in ADPKD are polycystin-1 (encoded by PKD1) and
polycystin-2 (encoded by PKD2). Cystatin is not associated with ADPKD. Polycystins play a crucial role
in the regulation of cell proliferation and differentiation.

Solution for Question 2:


Correct Option C – Fibrocystin:
• In Autosomal Recessive Polycystic Kidney Disease (ARPKD), the underlying genetic defect involves
mutations in the PKHD1 gene on chromosome 6, which codes for the protein fibrocystin. Fibrocystin is
primarily expressed in the renal tubules, bile ducts, and pancreas.
• Mutations in PKHD1 result in abnormal fibrocystin, contributing to the pathogenesis of ARPKD.
• Patients with ARPKD are usually children and in some cases, the enlarged cystic kidneys are picked
up at birth itself.
Incorrect Options:
Option A - Cystin:
• Cystin is a different protein, and defects in cystin are associated with conditions like cystinuria, a
genetic disorder characterized by the formation of kidney stones due to impaired cystine reabsorption
in the kidneys. However, the clinical presentation in the scenario described is more indicative of
ARPKD rather than cystin-related disorders.
Option B - Polycystin:
• Polycystin refers to two proteins, polycystin-1 and polycystin-2, associated with Autosomal Dominant
Polycystic Kidney Disease (ADPKD). The mutations in ADPKD involve defects in polycystin-1 and
polycystin-2.
Option D - Alpha-galactosidase A:
• Alpha-galactosidase A is associated with Fabry disease, an X-linked lysosomal storage disorder. In
Fabry disease, a deficiency of alpha-galactosidase A leads to the accumulation of
globotriaosylceramide (GL-3) in various tissues. However, the clinical presentation in the scenario
described (abdominal distension and kidney cysts) is not consistent with Fabry disease.

Solution for Question 3:


Correct Option D - All the above:
• The image shows cysts in corticomedullary regions of the kidney.
• All the statements mentioned are true regarding the Medullary cystic kidney.

Page 6

841
Option A - Treatment approaches may include: BP control, Management of renal insufficiency, and trea
tment of complications:
The management of MCKD is generally focused on addressing the symptoms and complications assoc
iated with the disease. Treatment approaches may include:
• Blood Pressure Control: Hypertension is a common complication of MCKD. Blood pressure control
through lifestyle modifications and medications (e.g., angiotensin-converting enzyme inhibitors or
angiotensin II receptor blockers) is essential to slow the progression of kidney damage.
• Hypertension is a common complication of MCKD. Blood pressure control through lifestyle
modifications and medications (e.g., angiotensin-converting enzyme inhibitors or angiotensin II receptor
blockers) is essential to slow the progression of kidney damage.
• Management of Renal Insufficiency: As MCKD progresses, renal insufficiency may occur. Monitoring
and managing complications related to kidney function, such as anemia and electrolyte imbalances, are
important aspects of care.
• As MCKD progresses, renal insufficiency may occur. Monitoring and managing complications related
to kidney function, such as anemia and electrolyte imbalances, are important aspects of care.
• Treatment of Complications: Anemia: Erythropoiesis-stimulating agents (ESA) or iron
supplementation may be used to manage anemia associated with kidney disease. Electrolyte
Imbalances: Correction of imbalances in potassium, phosphate, and other electrolytes may be
necessary.
• Anemia: Erythropoiesis-stimulating agents (ESA) or iron supplementation may be used to manage
anemia associated with kidney disease.
• Electrolyte Imbalances: Correction of imbalances in potassium, phosphate, and other electrolytes may
be necessary.
• Genetic Counseling: Since MCKD is often genetically inherited, genetic counseling may be offered to
affected individuals and their families. Understanding the genetic basis can help in family planning
decisions and risk assessment.
• Since MCKD is often genetically inherited, genetic counseling may be offered to affected individuals
and their families. Understanding the genetic basis can help in family planning decisions and risk
assessment.

Page 7

842
• Regular Monitoring and Follow-Up: Regular monitoring of kidney function, blood pressure, and overall
health is crucial. This allows for timely intervention and management of complications.
• Regular monitoring of kidney function, blood pressure, and overall health is crucial. This allows for
timely intervention and management of complications.
• Hypertension is a common complication of MCKD. Blood pressure control through lifestyle
modifications and medications (e.g., angiotensin-converting enzyme inhibitors or angiotensin II receptor
blockers) is essential to slow the progression of kidney damage.
• As MCKD progresses, renal insufficiency may occur. Monitoring and managing complications related
to kidney function, such as anemia and electrolyte imbalances, are important aspects of care.
• Anemia: Erythropoiesis-stimulating agents (ESA) or iron supplementation may be used to manage
anemia associated with kidney disease.
• Electrolyte Imbalances: Correction of imbalances in potassium, phosphate, and other electrolytes may
be necessary.
• Since MCKD is often genetically inherited, genetic counseling may be offered to affected individuals
and their families. Understanding the genetic basis can help in family planning decisions and risk
assessment.
• Regular monitoring of kidney function, blood pressure, and overall health is crucial. This allows for
timely intervention and management of complications.
Option B - Complications include anemia:
• This is true regarding MCKD, Due to defective renal parenchyma and decreased production of
erythropoetin.
Option C - Autosomal Recessive (AR) form is seen in children:
• Familial Juvenile Nephronophthisis is an autosomal recessive disorder characterized by
corticomedullary cysts and usually presents in childhood.

Solution for Question 4:


Correct Option B - A-2 ,B-1, C-4, D-3:
A.

Page 8

843
2. ADPKD.
Image shows Cysts involving all poles of the kidney .

1. Medullary sponge kidney


The image shows exclusively medullary cysts.

Page 9

844
4. ARPKD
The image shows cysts involving only the inner surface. The outer surface is almost normal

3) Medullary cystic kidney


The image shows multiple cysts in the corticomedullary region of the kidney.

Solution for Question 5:


Correct Option B - Autosomal dominant polycystic kidney disease (ADPKD):
• This is a case of autosomal dominant polycystic kidney disease that presents with flank pain, red
colour urine, a history of hypertension and urinary tract infection.

Page 10

845
• ADPKD is a renal cystic disorder caused by a mutation in PKD1 or PKD2 (encodes polycystin
protein).
• ADPKD presents with cysts in the cortex and medulla, which causes bilateral enlargement of the
kidneys.
• Complications of ADPKD:
Chronic kidney disease, hypertension (due to increase in renin).
• Association of ADPKD:
Berry aneurysms (leads to subarachnoid haemorrhage), hepatic cysts (benign), diverticulosis, and mitr
al valve prolapse.
• Diagnosis: Ultrasonography shows enlarged kidneys with multiple parenchymal anechoic masses of
varying sizes. Genetic testing may be performed in atypical cases (e.g., no family history, early and
severe disease) or if imaging results are inconclusive.
• Ultrasonography shows enlarged kidneys with multiple parenchymal anechoic masses of varying
sizes.
• Genetic testing may be performed in atypical cases (e.g., no family history, early and severe disease)
or if imaging results are inconclusive.
• Ultrasonography shows enlarged kidneys with multiple parenchymal anechoic masses of varying
sizes.
• Genetic testing may be performed in atypical cases (e.g., no family history, early and severe disease)
or if imaging results are inconclusive.
Incorrect Options:
Option A - Autosomal recessive polycystic kidney disease (ARPKD):
• ARPKD is most common in infants, not adults.
• ARPKD is associated with congenital hepatic fibrosis.
• It can lead to systemic hypertension and progressive renal insufficiency beyond the neonatal period.
Option C - Transitional cell carcinoma of the bladder:
• It is also called urothelial cell cancer of the bladder.
• It presents with painless hematuria, voiding (urgency, frequency), and suprapubic pain.
• Risk factors for bladder transitional cell cancer are smoking, phenacetin, and aniline dyes.
Option D - Renal cell carcinoma:
• Renal cell carcinoma (RCC) most commonly originates from the proximal convoluted tubule.
• RCC presents with hematuria, palpable mass, flank pain, secondary polycythemia, and weight loss.
• Most commonly affect men aged 50-70 years old.

Solution for Question 6:


Correct Option B - Diffuse Pulmonary hypoplasia:

Page 11

846
• This is a case of autosomal recessive polycystic kidney disease (ARPKD) which presents with
multiple bilateral cysts on ultrasonography.
• In-utero presentation of kidney failure will lead to oligohydramnios that cause compression of the
fetus. This intrauterine change leads to limb deformities (club feet as seen in this patient) and potter
sequence.
• Oligohydramnios leads to compression of the chest and this along with the lack of amniotic fluid
aspiration into the lungs ultimately leads to immature lung or pulmonary hypoplasia (potter sequence).
• Ultrasound is the first diagnostic evaluation that shows multiple cysts of the same size.
• Genetic analysis shows a mutation in PKHD1 that codes for fibrocystin.
Incorrect Options:
Option A - Neonatal respiratory distress syndrome:
• Neonatal respiratory distress syndrome is due to surfactant deficiency.
• Risk factors: prematurity, diabetic mother, C-section.
• Screening test: lecithin-sphingomyelin ratio in amniotic fluid.
Option C - Congenital diaphragmatic hernia:
• A congenital diaphragmatic hernia occurs due to a congenital defect in the pleuroperitoneal
membrane, which also presents with respiratory distress.
• Also, leads to pulmonary hypoplasia but mostly on the left side
• An X-ray will show herniation of gastric contents in the thorax.
Option D - Meconium aspiration Syndrome:
• Meconium aspiration syndrome occurs when a baby aspirates amniotic fluid and meconium during
delivery or just after the delivery.
• Presents with rapid breathing, retractions, and grunting sounds.
• This disease usually resolves in two to four days

Solution for Question 7:


Correct Option D - It is a premalignant condition
• The likely diagnosis in this patient is Malakoplakia.
• It mimics malignancy but it is not a premalignant/malignant condition
• The gross appearance of the bladder demonstrates yellowish-white mucosal lesions that resemble
cancer.
• On histopathology, the following findings are seen: Large foamy macrophages with multinucleate
giant cells and lymphocytes. This disease is believed to be due to a phagolysosome dysfunction. The
macrophages have abundant granular cytoplasm due to phagosomes being stuffed with bacterial
debris. There is also the presence of laminated, mineralized concretions due to calcium deposition in
the enlarged lysosomes known as Michaelis-Gutmann bodies.
• Large foamy macrophages with multinucleate giant cells and lymphocytes.
• This disease is believed to be due to a phagolysosome dysfunction.

Page 12

847
• The macrophages have abundant granular cytoplasm due to phagosomes being stuffed with bacterial
debris.
• There is also the presence of laminated, mineralized concretions due to calcium deposition in the
enlarged lysosomes known as Michaelis-Gutmann bodies.

• Large foamy macrophages with multinucleate giant cells and lymphocytes.
• This disease is believed to be due to a phagolysosome dysfunction.
• The macrophages have abundant granular cytoplasm due to phagosomes being stuffed with bacterial
debris.
• There is also the presence of laminated, mineralized concretions due to calcium deposition in the
enlarged lysosomes known as Michaelis-Gutmann bodies.

Incorrect Options:
Option A, B and C are true

Solution for Question 8:


Correct Option B – Laminin:
• Laminin is a key structural protein found in the glomerular basement membrane (GBM). It is an
integral part of the GBM's structure, contributing to its stability and integrity. Laminin is essential for
maintaining the glomerulus's barrier function and plays a crucial role in the filtration process.
Incorrect Options:
• Options A, C, and D are not the most abundant proteins found in the glomerular basement membrane.

Page 13

848
Solution for Question 9:
Correct Option B - Gomori Methenamine Silver stain:
• The stain used in the given histopathological image is the Gomori methenamine silver stain.

Incorrect Options:
Option A - Prussian blue stain: Prussian blue stain is used to detect the presence of iron deposits, typic
ally seen in conditions such as hemosiderosis or hemochromatosis.
Option C - Masson trichome stain: The Masson trichrome stain is used to differentiate collagen fibers fr
om other tissue components, highlighting connective tissue and fibrosis.
Option D - Toluidine blue stain: Toluidine blue stain is commonly used to stain acidic components such
as mast cells.

Solution for Question 10:


Correct Option D - 1-D, 2-C, 3-A, 4-B:

Solution for Question 11:


Correct Option D - Thyroidization of Tubules:
• Chronic pyelonephritis (CPN) is a condition characterized by chronic inflammation and scarring of the
renal parenchyma. The kidneys are asymmetrically contracted and have pinpoint hemorrhages on their
surface. The tubules undergo atrophy and are filled with pink material known as Thyroidization of
Tubules.

Incorrect Options:
Option A - Enlarged, smooth kidney: CPN typically leads to a
granular contracted (asymmetric) kidney rather than an enlarged, smooth kidney.
Option B
- Symmetrically scarred kidney: CPN results in an asymmetrically scarred kidney rather than a
symmetrically scarred one.
Option C - Glomerular hyperplasia: Glomerular hyperplasia is not a characteristic feature of CPN.

Solution for Question 12:


Correct Option D - Chronic pyelonephritis:

Page 14

849
• Chronic pyelonephritis (CPN) is characterized by chronic inflammation of the renal parenchyma,
particularly involving the tubules. It results in a granular and asymmetrically contracted kidney. The
asymmetry is due to scarring, affecting one or more areas of the kidney.

Incorrect Options:
Option A - Acute glomerulonephritis: Acute glomerulonephritis typically presents with diffuse glomerular
inflammation and swelling and is not associated with asymmetric and granular contracted kidney.
Option B - Chronic glomerulonephritis: Chronic glomerulonephritis can result in a
symmetric granular contracted appearance of the kidney.
Option C - Benign nephrosclerosis: Benign nephrosclerosis can result in a
symmetric granular contracted appearance of the kidney.

Page 15

850
Crystals & Casts
1. Which of the following is the most common type of kidney stone?
A. Calcium oxalate
B. Triple stone
C. Uric acid stone
D. Cystine stone
----------------------------------------
2. Which of the following is a characteristic of nephrotic syndrome?
A. WBC cast
B. Broad cast
C. Lipid cast
D. RBC cast
----------------------------------------
3. Identify the given crystal observed on urine microscopy:

A. Calcium carbonate crystal


B. Ammonium biurate crystal
C. Calcium oxalate crystal
D. Cysteine crystal
----------------------------------------
4. Which of the following crystals show a coffin-lid appearance?
A. Triple phosphate crystal
B. Uric acid crystal
C. Cholesterol crystal
D. Calcium oxalate dihydrate crystal
----------------------------------------
5. Which of the following casts are seen in acute tubular necrosis?
A.

851
B.
C.
D.
----------------------------------------

Correct Answers
Question Correct Answer

Question 1 1
Question 2 3
Question 3 2
Question 4 1
Question 5 2

Solution for Question 1:


Correct Option A - Calcium oxalate:
• There are 4 main types of calculi: Calcium stones (~70%) mainly, calcium oxalate or calcium oxalate
mixed with calcium phosphate. Triple stones or struvite stones make up about 15%. These stones are
made up of magnesium ammonium phosphate. 5-10% are uric acid stones 1-2% are cystine stones.
• Calcium stones (~70%) mainly, calcium oxalate or calcium oxalate mixed with calcium phosphate.
• Triple stones or struvite stones make up about 15%. These stones are made up of magnesium
ammonium phosphate.
• 5-10% are uric acid stones
• 1-2% are cystine stones.
There are 4 main types of calculi:
• Calcium stones (~70%) mainly, calcium oxalate or calcium oxalate mixed with calcium phosphate.
• Triple stones or struvite stones make up about 15%. These stones are made up of magnesium
ammonium phosphate.
• 5-10% are uric acid stones
• 1-2% are cystine stones.

Incorrect Options:
Options B, C, and D are incorrect and the correct option has been explained above.

Solution for Question 2:


Correct Option C - Lipid cast:

Page 2

852
Casts
Importance
WBC cast

• Differentiate b/w pyelonephritis & cystitis.


• It is not seen in cystitis.
LIPID CAST

• Nephrotic syndrome
• Hypothyroidism
Hypothyroidism
RBC Cast/ Dysmorphic RBCs

Page 3

853
Nephritic syndrome
Broad cast

Chronic renal failure


Incorrect Options:
Options A, B and D are incorrect and the correct option has been explained above.

Solution for Question 3:


Correct Option B - Ammonium biurate crystal:

Page 4

854
• The crystal shown in the given image is an ammonium biuret crystal. It has an apple thorn
appearance.

Incorrect Options:
Option A - Calcium carbonate crystal: They show round wheel appearance.

Option C - Calcium oxalate crystal: They are dumbbell-shaped.

Page 5

855
Option D - Cysteine crystal: They are hexagonal in shape.

Solution for Question 4:


Correct Option A - Triple phosphate crystal:
• Triple phosphate crystals, also known as struvite crystals, often exhibit a coffin-lid appearance when
viewed under a microscope. This appearance is characterized by a rectangular or prism-like shape with
one end pointed, and the other end squared off, resembling the lid of a coffin.

Page 6

856
Incorrect Options:
Option B - Uric acid crystal: Uric acid crystals typically exhibit a
variety of shapes, including rhomboid or triangle-shaped crystals, rather than a coffin-lid appearance.
Option C - Cholesterol crystal: Cholesterol crystals show a broken chip appearance, not a
coffin lid appearance.
Option D
- Calcium oxalate dihydrate crystal: Calcium oxalate crystals may appear as dumbbell-shaped.

Solution for Question 5:


Correct Option B:

Page 7

857
• The image in option B shows muddy brown casts, which are seen in cases of acute tubular necrosis.
The proximal convoluted tubule is most commonly affected.
Incorrect Options:
• The image in option A shows a broad waxy cast. It is seen in chronic renal failure, not in acute tubular
necrosis.
• The image in option C shows a granular cast or degenerated cast. It is not seen in acute tubular
necrosis.
• The image in option D shows a WBC cast. It is seen in chronic pyelonephritis, not i acute tubular
necrosis.

Page 8

858
Renal Tumors
1. From which structure does the benign tumor, characterized by a plethora of mitochondria and
granular eosinophilic cytoplasm, arise in a seventy-year-old male with left flank pain and blood in urine?

A. Loop of henle
B. Collecting duct
C. Proximal tubule
D. Bowman's capsule
----------------------------------------
2. Xp11 translocation carcinoma is associated with which of the following genes?
A. VHL gene
B. TFE3 gene
C. MET gene
D. TSC1 gene
----------------------------------------
3. Which of the following is not a marker of Wilms tumor?
A. Desmin
B. Vimentin
C. TTF 1
D. Cytokeratin
----------------------------------------
4. Which syndrome is associated with defects in the WT2 gene on chromosome 11p15 in a 4-year-old
male presenting with hematuria, fever, and biopsy revealing a triphasic tumor with an epithelial
component containing abortive glomeruli, a mesenchymal component, and a blastema component?
A. Beckwith-Wiedemann Syndrome
B. WAGR Syndrome
C. Denys-Drash Syndrome
D. Li-Fraumeni Syndrome
----------------------------------------

859
5. Which of the following conditions does not exhibit a central stellate scar?
A. Chromophobe RCC
B. Fibrolamellar carcinoma of liver
C. Oncocytoma
D. Renal papillary adenoma
----------------------------------------
6. Which of the following is the most associated risk factor of Grawitz tumor?
A. Alcohol consumption
B. Genetic mutation
C. Recurrent urinary tract infection
D. Tobacco consumption
----------------------------------------
7. Which is the most common type of renal cell carcinoma?
A. Papillary RCC
B. Chromophobe RCC
C. Clear cell RCC
D. Medullary RCC
----------------------------------------
8. What genetic alterations are commonly observed in Chromophobe Renal Cell Carcinoma (RCC)?
A. Gain of chromosomes 1, 5, 7
B. Loss of chromosomes 1, 5, 7
C. Loss of chromosomes 1 and 3
D. Loss of chromosomes 1, 3, 7
----------------------------------------
9. What is the hallmark feature of the type of RCC that has the worst prognosis?
A. Psammoma bodies
B. Plant cells
C. Hobnail cells
D. Clear cells
----------------------------------------
10. What is the most common site of origin of Papillary Renal Cell Carcinoma (PRCC)?
A. Proximal Convoluted Tubule (PCT)
B. Distal Convoluted Tubule (DCT)
C. Loop of Henle
D. Collecting Duct

Page 2

860
----------------------------------------
11. Which syndrome is characterized by the combination of Wilms tumor, aniridia, genital
abnormalities, and mental retardation?
A. WAGR syndrome
B. Beckwith-Wiedemann syndrome
C. Denys-Drash syndrome
D. Li-Fraumeni syndrome
----------------------------------------

Correct Answers
Question Correct Answer

Question 1 2
Question 2 2
Question 3 3
Question 4 1
Question 5 4
Question 6 4
Question 7 3
Question 8 2
Question 9 3
Question 10 1
Question 11 1

Solution for Question 1:


Correct Option B - Collecting duct:
• Renal oncocytoma is a benign epithelial neoplasm.
• It arises from the intercalated cells of collecting ducts.
• The gross appearance of the tumour is a tan-brown-coloured mass which is relatively homogeneous
and well encapsulated with a central stellate scar.

• Microscopically the tumour is characterized by a plethora of mitochondria which gives it a tan colour
and granular eosinophilic cytoplasm.
• Chromophobe carcinoma also arises from the collecting duct, but it is a malignant tumour and a type
of renal cell carcinoma.

Page 3

861
Incorrect Options:
Option A - Loop of Henle:
• Renal oncocytoma is a benign epithelial neoplasm. It does not arise from the loop of Henle.
Option C - Proximal tubule:
• Renal cell carcinoma (RCC), the most common type of kidney cancer in adults, typically arises from
the cells of the renal tubules, including the proximal convoluted.
Option D - Bowman's capsule:
• Renal oncocytoma is a benign epithelial neoplasm. It does not arise from the Bowman’s capsule.

Solution for Question 2:


Correct Option B - TFE3 gene:
• Xp11 translocation carcinoma is a rare genetically distinct subtype of renal cell carcinoma (RCC).
• It is more common in children.
• The TFE3 gene, located at the Xp11.2 locus, is involved in Xp11 translocation carcinoma.
• The neoplastic cells consist of clear cytoplasm with papillary architecture.

Page 4

862
Incorrect Options:
Option A - VHL gene:
• VHL is associated with clear cell RCC.
• Xp11 translocation carcinoma is associated with TFE3.
Option C - MET gene:
• MET is associated with hereditary papillary RCC.
• Xp11 translocation carcinoma is associated with TFE3.
Option D - TSC1 gene:
• TSC1 is associated with renal angiomyolipoma.
• Xp11 translocation carcinoma is associated with TFE3.

Solution for Question 3:


Correct Option C - TTF 1:
• Wilms tumour is the most common renal cancer in children.
• It usually occurs in children aged 2-5.
• It is also called nephroblastoma.
• Symptoms of Wilms tumour include: Painless, palpable abdominal mass Abdominal pain Fever
Nausea and vomiting Hematuria
• Painless, palpable abdominal mass
• Abdominal pain
• Fever
• Nausea and vomiting

Page 5

863
• Hematuria
• Wilms tumour is associated with multiple syndromes: WAGR syndrome Denys-Drash syndrome
Beckwith-Wiedemann syndrome
• WAGR syndrome
• Denys-Drash syndrome
• Beckwith-Wiedemann syndrome
• Wilms tumour is associated with various markers: Desmin Vimentin Cytokeratin
• Desmin
• Vimentin
• Cytokeratin
• TTF-1 (thyroid transcription factor) is a marker for: Lung cancer Thyroid cancer Adenocarcinoma
Small cell carcinoma of the lung
• Lung cancer
• Thyroid cancer
• Adenocarcinoma
• Small cell carcinoma of the lung
• Painless, palpable abdominal mass
• Abdominal pain
• Fever
• Nausea and vomiting
• Hematuria
• WAGR syndrome
• Denys-Drash syndrome
• Beckwith-Wiedemann syndrome
• Desmin
• Vimentin
• Cytokeratin
• Lung cancer
• Thyroid cancer
• Adenocarcinoma
• Small cell carcinoma of the lung
Incorrect Options:
Refer to the explanation of the correct answer above

Solution for Question 4:

Page 6

864
Correct Option A - Beckwith-Wiedemann Syndrome:
• Beckwith-Wiedemann Syndrome (BWS) is associated with defects in the WT2 gene located on
chromosome 11p15. In the scenario described, a 4-year-old male child with an abdominal mass,
hematuria, and fever, along with a triphasic tumor on biopsy (epithelial, mesenchymal, and blastema
components), the genetic analysis indicating defects in the WT2 gene strongly suggests an association
with Beckwith-Wiedemann Syndrome.
Incorrect Options:
Option B - WAGR Syndrome:
• WAGR Syndrome is associated with defects in the WT1 gene on chromosome 11p13, not the WT2
gene. The clinical features of WAGR Syndrome include Wilms tumor, Aniridia (absence of the iris),
Genital abnormalities, and intellectual disabilities
Option C - Denys-Drash Syndrome:
• Denys-Drash Syndrome is associated with defects in the WT1 gene on chromosome 11p13. The
syndrome is characterized by diffuse mesangial sclerosis and dysgenesis of the gonads. It was not the
correct answer in this case because the information provided indicated a defect in the WT2 gene.
Option D - Li-Fraumeni Syndrome:
• Li-Fraumeni Syndrome is a hereditary cancer syndrome caused by mutations in the TP53 tumor
suppressor gene. It is not directly associated with the triphasic tumor described in the question or the
specific genetic defect mentioned (WT2 gene on chromosome 11p15).

Solution for Question 5:


Correct Options D- Renal papillary adenoma:
• Renal papillary adenomas do not present with a central stellate scar. Renal papillary adenomas are
benign tumors of the kidney and are often small, well-circumscribed lesions composed of closely
packed tubules and papillae.
Incorrect Options:
Options A, B, and C (Chromophobe RCC, Fibrolamellar carcinoma of liver & Oncocytoma): Chromoph
obe RCC, Fibrolamellar carcinoma of the liver, and Oncocytoma present with a central stellate scar.

Solution for Question 6:


Correct Option D- Tobacco consumption:
• Grawitz tumor, also known as renal cell carcinoma (RCC), is strongly associated with tobacco
consumption. Cigarette smoking is one of the most significant risk factors for developing RCC.
Incorrect Options:
Options A, B, and C (Alcohol consumption, Genetic mutation &
Recurrent urinary tract infection): They are not a significant risk factor for renal cell carcinoma.

Page 7

865
Solution for Question 7:
Correct Options C- Clear cell RCC:
• Clear cell renal cell carcinoma (RCC) is the most common type of renal cell carcinoma. It is
characterized by its histological appearance of clear or pale cytoplasm due to lipid and glycogen
accumulation. Clear cell RCC is typically associated with mutations in the von Hippel-Lindau (VHL)
tumor suppressor gene.
Incorrect Options:
Option A- Papillary RCC: Papillary RCC is the second most common type of renal cell carcinoma.
Option B- Chromophobe RCC: Chromophobe RCC is the third most common type of renal cell carcino
ma.
Option D- Medullary RCC: Medullary RCC is a rare subtype of renal cell carcinoma.

Solution for Question 8:


Correct Option B- Loss of chromosomes 1, 5, 7:
• Chromophobe RCC typically demonstrates plant cells, dark irregular nuclei with raisinoid appearance,
peripheries containing cytoplasm, and perinuclear halos around the nucleus on microscopy. It is
caused by multiple chromosomal losses, typically in chromosomes 1, 5, and 7.
Incorrect Options:
Options A, C, and D (Gain of chromosomes 1, 5, 7, Loss of chromosomes 1 and 3
& Loss of chromosomes 1, 3, 7): They are not associated with chromophobe renal cell carcinoma.

Solution for Question 9:


Correct Option C- Hobnail cells:
• Bellini RCC, or Collecting duct RCC, has the worst prognosis among all types of renal cell carcinoma.
Hobnail cells are a hallmark feature of this type of renal cell carcinoma.
Incorrect Options:
Option A- Psammoma bodies: Psammoma bodies are seen in papillary RCC, and papillary RCC does
not have the worst prognosis.
Option B- Plant cells: Plant cells are seen in chromophobe RCC, which has the best prognosis.
Option D- Clear cells: Clear cell renal cell carcinoma is the most common subtype of RCC, but it does
not have the worst prognosis.

Solution for Question 10:


Correct Option A- Proximal Convoluted Tubule (PCT):

Page 8

866
• Papillary Renal Cell Carcinoma (PRCC) primarily originates from the cells of the proximal convoluted
tubule (PCT) in the kidney. It is characterized by psammoma bodies.
Incorrect Options:
Options B, C, and D (Distal Convoluted Tubule (DCT), Loop of Henle &
Collecting Duct): They are not the most common site of origin of papillary RCC.

Solution for Question 11:


Correct Option A- WAGR syndrome:
• WAGR syndrome stands for Wilms tumor, Aniridia, Genitourinary anomalies, and mental Retardation.
It is a rare genetic condition caused by a deletion of genetic material on chromosome 11p13, which
includes the WT1 gene.
Incorrect Options:
Option B- Beckwith-Wiedemann syndrome: Beckwith-Wiedemann syndrome is characterized by Wilms
tumor, adrenal cortical tumor, hepatoblastoma, hemihypertrophy, and macroglossia.
Option C- Denys-Drash syndrome: Denys-Drash syndrome is characterized by Wilms tumor, diffuse m
esangial sclerosis, and dysgenesis of the gonads.
Option D- Li-Fraumeni syndrome: Li-Fraumeni syndrome is not associated with Wilms tumor, aniridia,
or mental retardation.

Page 9

867
Glomerular Diseases
1. What renal biopsy findings are most likely in a 17-year-old boy with recurrent sore throat,
cola-colored urine, fever, throat pain, and difficulty swallowing?
(or)
Which of the following renal biopsy results will be present in a 17-year-old boy presenting with recurrent
bouts of cola-colored urine during infections?
A. Predominantly deposition of IgA in the mesangium
B. Enlarged and hypercellular glomeruli
C. Diffuse capillary and glomerular basement membrane thickening
D. Normal glomeruli
----------------------------------------
2. What are the pathological changes seen in diabetic nephropathy?
(or)
What are pathological changes seen in diabetic nephropathy?
A. Kimmelstiel-Wilson nodules
B. Immune complex deposition
C. Normal basement membrane
D. Focal glomerular sclerosis
----------------------------------------
3. Which of the following 24-hour albumin levels corresponds with microalbuminuria?
A. 8-10 mg/day
B. 30-300 mg/day
C. 600-900 mg/day
D. 301-600 mg/day
----------------------------------------
4. Goodpasture syndrome is an example of which of the following types of hypersensitivity?
(or)
Goodpasture syndrome is an example of which of the following types of hypersensitivity?
A. Type I hypersensitivity
B. Type II hypersensitivity
C. Type III hypersensitivity
D. Type IV hypersensitivity
----------------------------------------
5. What are the likely causes of pitting edema, periorbital edema, 3+ proteinuria, 3+ glucosuria, and the
given kidney biopsy (stained with PAS) findings in a 52-year-old obese woman? Which of the following
is the most likely diagnosis for this patient?

868
A. Acute glomerulonephritis
B. Amyloid nephropathy
C. Diabetic glomerulosclerosis
D. Malignant nephrosclerosis
----------------------------------------
6. Which of the following statements is true regarding renal amyloidosis associated with multiple
myeloma?
A. Typical involves kidneys only
B. Hematuria is the early sign
C. Amyloid deposits are heterogenous, cellular and amorphous on H &E; stain
D. Mesangial deposits are of monoclonal kappa/lambda light chains
----------------------------------------
7. In a 7-year-old boy presents with generalized edema, albuminuria, and hyperlipidemia . An electron
microscopic of the glomerular basement membrane, in this case, is shown below. What is the
diagnosis?

A. Minimal change in disease


B. Crescentic glomerulonephritis
C. Acute proliferative glomerulonephritis
D. Henoch-Schonlein purpura
----------------------------------------

Page 2

869
8. What is the underlying pathology in a 35-year-old male patient with proteinuria and the given
immunofluorescence staining pattern from a kidney biopsy?

A. Focal segmental glomerulosclerosis


B. Post-strep glomerulonephritis
C. Lupus nephritis
D. Goodpasture syndrome
----------------------------------------
9. A 28-year-old man presents with end-stage renal disease. On examination, right eye lens dislocation
is found. His maternal uncle also died of the same illness. On electron microscopy of renal biopsy, a
basket-weave appearance of glomerular basement membrane (GBM) is seen. Which of the following is
the most likely working diagnosis?
(or)
If a basket-weave appearance of glomerular basement membrane (GBM) is found on a renal biopsy,
which of the following is a diagnosis?
A. Autosomal dominant polycystic kidney disease
B. Autosomal recessive polycystic kidney disease
C. Marfan's syndrome
D. Alport syndrome
----------------------------------------
10. What is the likely diagnosis for a 3-year-old with skin lesions given below, abdominal pain, intestinal
bleeding, arthralgias, gross hematuria, and in immunofluorescence studies, IgA deposition in the
mesangium are observed?

Page 3

870
A. Focal segmental glomerulosclerosis (FSGS)
B. Minimal change disease (MCD)
C. Alport syndrome
D. Henoch Schönlein purpura (HSP)
----------------------------------------
11. What is likely to be observed in immunofluorescence studies in a 55-year-old man with type 2
diabetes mellitus for 20 years of poor medical compliance, frothy urine, and facial edema?
(or)
A patient who had facial edema, frothy urine, and uncontrolled diabetes. What results could other renal
tests on this patient indicate?
A. Nodular glomerulosclerosis
B. Apple-green birefringence under polarised light
C. Segmental sclerosis and hyalinosis
D. Normal glomeruli with effacement of the foot processes
----------------------------------------
12. What is the diagnosis in a 33-year-old female patient presents pulmonary hemorrhage and features
of glomerulonephritis mediated by the anti-GBM antibodies?
(or)
What disease is mediated by pulmonary haemorrhage, renal abnormalities, and anti-GBM antibodies?
A. Goodpasture syndrome
B. Alport syndrome
C. Focal segmental glomerulosclerosis
D. IgA nephropathy
----------------------------------------
13. Which of the following statements is true for a patient with hematuria and the given renal
histopathology slide? 1. It is a Type III Hypersensitivity reaction. 2. Streptococcus pyogenes: strains 1,
2, and 4 are responsible. 3. Gross findings on the kidney show flea-bitten appearance. 4. Linear
immunofluorescence is seen.
(or)
Which of the following statements is true for a patient with hematuria and the given renal histopathology
slide? 1. It is a Type III Hypersensitivity reaction. 2. Streptococcus pyogenes: strains 1, 2, and 4 are
responsible. 3. Gross findings on the kidney show a flea-bitten appearance. 4. Linear
immunofluorescence is seen.

Page 4

871
A. 1,2 and 3 are true
B. only 4 is true
C. Only 3 is true
D. 2,3 and 4 are true
----------------------------------------
14. What's the probable diagnosis for a 15-year-old with red-colored urine who previously had an upper
respiratory tract infection and IgA deposition in the mesangium?
(or)
Which diagnosis is most likely for this patient, who has hematuria that is occuring at the same time as a
URTI and IgA deposition in the mesangium?
A. Berger disease
B. Alport syndrome
C. Minimal change disease
D. Post-streptococcal glomerulonephritis
----------------------------------------
15. What's the likely diagnosis for a 30-year-old man with progressive ankle and abdominal swelling, a
history of intravenous drug addiction, heavy proteinuria, negative ANCA, normal serum creatinine, and
resistance to steroids, and the following renal biopsy findings?

A. Post-streptococcal glomerulonephritis
B. Minimal change disease

Page 5

872
C. IgA Nephropathy
D. Focal segmental glomerulosclerosis
----------------------------------------
16. In a 45-year-old female with rheumatoid arthritis treated with penicillamine, presenting with
proteinuria and a renal biopsy revealing a spike and dome pattern, which of the following is likely
associated with her condition?
(or)
In a 45-year-old female with rheumatoid arthritis treated with penicillamine, presenting with proteinuria
and a renal biopsy revealing a spike and dome pattern, which of the following is likely associated with
her condition?
A. Antibodies against phospholipase A2 receptor (PLA2Rc)
B. Thrombospondin THSD7A
C. CD 10
D. Immune complex deposition in subepithelium
----------------------------------------
17. Which of the following is the characteristic histological feature of HIV-associated collapsing
glomerulopathy?
A. Crescent formation
B. Mesangial deposition of IgA
C. Subepithelial deposition with endothelial cell proliferation
D. Hypertrophy and proliferation of visceral epithelial cells
----------------------------------------
18. What is the likely diagnosis of a 7-year-old boy presenting with facial and body swelling, foamy
urine, fatigue, loss of appetite, and a protein content of 300 mg/dL in urine analysis, and the following
findings on serum protein electrophoresis?
(or)
A child with 300 mg/dL of proteinuria, generalised body edema, frothy urine, fatigue, and appetite loss.
What is the most likely diagnosis?

A. Tuberculosis
B. Multiple myeloma

Page 6

873
C. Nephrotic syndrome
D. Liver cirrhosis
----------------------------------------
19. What is least likely to be seen in a 35-year-old female with recurrent urinary tract infections, flank
pain, fever, frequent urination, leukocytosis, positive urine cultures for Escherichia coli, imaging findings
of renal scarring and atrophy, and the given renal histology findings?

A. Interstitial fibrosis is observed.


B. Thyroidization of tubules is seen
C. It is not commonly associated with colonization of distal urethra and introitus.
D. Blunted calyces may be observed
----------------------------------------
20. Which of the following corresponds to the stage of focal lupus nephritis affecting less than 50% of
glomeruli?
A. Class 2
B. Class 3
C. Class 4
D. Class 5
----------------------------------------
21. Which of the following is the most common form of rapidly progressive glomerulonephritis and
includes microscopic polyangiitis and Wegener's granulomatosis?
A. RPGN 1
B. RPGN 2
C. RPGN 3
D. RPGN 4
----------------------------------------
22. Match the following: 1 Rapidly Progressive Glomerulonephritis A 2 Membranoproliferative
Glomerulonephritis B 3 IgA nephropathy C 4 Focal segmental glomerulonephritis D

Page 7

874
1 Rapidly Progressive Glomerulonephritis A

2 Membranoproliferative Glomerulonephritis B

3 IgA nephropathy C

4 Focal segmental glomerulonephritis D

A. 1-C, 2-D, 3-B, 4-A


B. 1-B, 2-C, 3-D, 4-A

Page 8

875
C. 1-D, 2-C, 3-A, 4-B
D. 1-C, 2-A, 3-D, 4-B
----------------------------------------

Correct Answers
Question Correct Answer

Question 1 1
Question 2 1
Question 3 2
Question 4 2
Question 5 3
Question 6 4
Question 7 1
Question 8 3
Question 9 4
Question 10 4
Question 11 1
Question 12 1
Question 13 1
Question 14 1
Question 15 4
Question 16 4
Question 17 4
Question 18 3
Question 19 3
Question 20 2
Question 21 3
Question 22 3

Solution for Question 1:


Correct Option A - Predominantly deposition of IgA in the mesangium:
• This is a case of IgA nephropathy, which presents with cola-colored urine (during active infection).
• IgA nephropathy is also known as Berger disease.
• Clinical features: Diffuse hematuria or microscopic hematuria Fever (low grade) Pain in the flank
region It usually occurs during, immediately following, a respiratory or gastrointestinal infection.
Hypertension

Page 9

876
• Diffuse hematuria or microscopic hematuria
• Fever (low grade)
• Pain in the flank region
• It usually occurs during, immediately following, a respiratory or gastrointestinal infection.
• Hypertension
• IgA can also progress to rapidly progressive glomerulonephritis
• IgA can also present with features of nephrotic syndrome (in < 10% of patients)
• Light microscopy (LM) findings: LM reveals mesangial proliferation.
• LM reveals mesangial proliferation.
• Immunofluorescence (IF) finding: Immune complex (IgA-based) deposits in the mesangium.
• Immune complex (IgA-based) deposits in the mesangium.
• Electron microscopy (EM): EM reveals IC deposits in the mesangium.
• EM reveals IC deposits in the mesangium.
• Diffuse hematuria or microscopic hematuria
• Fever (low grade)
• Pain in the flank region
• It usually occurs during, immediately following, a respiratory or gastrointestinal infection.
• Hypertension
• LM reveals mesangial proliferation.
• Immune complex (IgA-based) deposits in the mesangium.
• EM reveals IC deposits in the mesangium.
Incorrect Options:
Option B - Enlarged and hypercellular glomeruli:
• Glomeruli are enlarged and hypercellular in acute post-streptococcal glomerulonephritis (APSGN).
• APSGN is a nephritic syndrome that most commonly affects children.
• It occurs 2-4 weeks after the pharynx and skin infection of streptococcus pyrogens (group A
streptococcus). Unlike the scenario where the patient presents with cola-colored urine during active
infection.
Option C - Diffuse capillary and glomerular basement membrane thickening:
• Diffuse capillary and glomerular basement membrane thickening are seen in the case of membranous
nephropathy.
• Membranous nephropathy presents with nephrotic syndrome.
• The primary disease occurs secondary to phospholipase A2 receptors, and secondary disease occurs
due to penicillamine, gold, hepatitis B, and hepatitis C.
Option D - Normal glomeruli:
• Normal glomeruli are seen in cases of minimal change disease (MCD).
• MCD is the most common cause of nephrotic syndrome in children.

Page 10

877
• MCD may be triggered by immunization and immune stimulus.

Solution for Question 2:


Correct Option A - Kimmelstiel-Wilson nodules:
• Diabetic nephropathy is a kidney disease occurring in patients who have diabetes.
• Diabetes is one of the most common causes of end-stage kidney disease.
• In patients with diabetes mellitus, there is non-enzymatic glycosylation due to increased blood glucose
levels.
• This results in the deposition of advanced glycation end products on the efferent arteriole.
• There is increased filtration injury which causes nodular glomerulosclerosis (most characteristic)
• Increase in the number of mesangial cells and mesangial matrix results in the invasion of the
glomerular capillaries.
• This results in the formation of deposits called Kimmelstiel-Wilson nodules.

• Further expansion results in diffused glomerulosclerosis.
• The most important glomerular lesions seen in diabetic nephropathy are: Capillary basement
membrane thickening Diffuse mesangial sclerosis Nodular glomerulosclerosis.
• Capillary basement membrane thickening
• Diffuse mesangial sclerosis
• Nodular glomerulosclerosis.
• Nodular glomerulosclerosis or Kimmelstiel-Wilson lesions are also called intercapillary
glomerulosclerosis.
• These nodules are PAS-positive.

Page 11

878
• Capillary basement membrane thickening
• Diffuse mesangial sclerosis
• Nodular glomerulosclerosis.
Incorrect Options:
Options B, C and D are incorrect and the correct option has been explained above.

Solution for Question 3:


Correct Option B - 30-300 mg/day:

24-hour albumin values in the range of 30-300 mg/day correspond with microalbuminuria. 24-hour albu
min >300 mg is suggestive of frank proteinuria. Normal range of 24 hour albumin is 8-10 mg/24 hours.
Microalbuminuria can also be described as
• urinary albumin excretion rate of ≥ 20 µg/min or albumin to creatine ratio ≥30mg/g.
Microalbuminuria is a hallmark of early diabetic nephropathy.
Incorrect Options:
Options A, C and D are incorrect and the correct option has been explained above.

Solution for Question 4:


Correct Option B - Type II hypersensitivity:
• Good pasture syndrome is an example of a type 2 hypersensitivity reaction.
• Type II hypersensitivity reaction begins when antibodies bind to surface antigens and lead to cellular
destruction, dysfunction, and inflammation.
• Type II hypersensitivity reaction is either due to cellular destruction (transfusion reaction), Cellular
dysfunction (myasthenia gravis) or inflammation (Goodpasture syndrome).

Goodpasture syndrome
• It is an autoimmune disease
• Certain domains of type IV collagen are intrinsic to the basement membranes of renal glomeruli and
pulmonary alveoli.
• Auto antibodies directed towards the NC1 domain of the α3 chain of type IV collagen in the basement
membranes cause inflammatory damage.
• Resulting in necrotizing hemorrhagic interstitial pneumonitis and rapidly progressive
glomerulonephritis.
• The characteristic linear pattern of immunoglobulin deposition (usually IgG) is the hallmark diagnostic
of immunofluorescence.

Page 12

879
Incorrect Options:
Option A - Type I hypersensitivity reaction:
• Type I hypersensitivity reaction involves anaphylaxis and allergic asthma.
• It involves cross-linking of preformed IgE on mast cells which causes degranulation and leads to
histamine and leukotrienes release.
Option C - Type III hypersensitivity:
• Type III hypersensitivity is immune complex-mediated, which activates the complement and presents
with fever, urticaria, arthralgia, and proteinuria.
• Serum sickness and Arthus reaction are prototypic examples of a Type III hypersensitivity reaction.
Option D - Type IV hypersensitivity:
• Type IV hypersensitivity involves T cells either due to CD8+ (direct cell cytotoxicity) or CD4+
(inflammatory reaction).
• Contact dermatitis and graft-versus-host disease are classic examples.
• PPD test for TB and candida patch skin test also involves Type IV hypersensitivity reaction.

Solution for Question 5:


Correct Option C - Diabetic glomerulosclerosis:
• Diabetic nephropathy is a kidney disease occurring in patients who have diabetes.
• Patients typically present with features of renal disease such as edema and proteinuria, In patients
with uncontrolled diabetes, glucosuria may be seen.
• It is the most common cause of end-stage kidney disease.
• The most important glomerular lesions seen in diabetic nephropathy are: Capillary basement
membrane thickening Diffuse mesangial sclerosis Nodular glomerulosclerosis
• Capillary basement membrane thickening
• Diffuse mesangial sclerosis
• Nodular glomerulosclerosis
• Nodular glomerulosclerosis or Kimmelstiel-Wilson lesions are also called intercapillary
glomerulosclerosis.
• These nodules are PAS-positive.
• Accumulation of extracellular matrix (ECM) proteins in the mesangial interstitial space causes
glomerulosclerosis.
• The most common proteins are collagen types I, III, and IV and fibronectin.
• Nodular widening of the mesangial areas is associated with the hyalinization of arterioles and focal
hyaline changes of the Bowman capsule.
• Microvascular disease is the major cause of renal failure & blindness in people with diabetes.

• Capillary basement membrane thickening

Page 13

880
• Diffuse mesangial sclerosis
• Nodular glomerulosclerosis

Incorrect Options:
Options A, B and D are incorrect and the correct option has been explained above.

Solution for Question 6:


Correct Option D - Mesangial deposits are of monoclonal kappa/lambda light chains:
• The mesangial deposits of monoclonal kappa/lambda light chains are often associated with plasma
cell dyscrasias such as multiple myeloma
• Amyloid deposition begins from mesangium and involves other parts later.
• The associated clinical features are generalized edema, proteinuria, low protein in the blood, a urinary
cast present, and increased kidney size.
• In primary Amyloidosis, AL protein is deposited in the organs, which are made of light chains (usually
lambda type) of immunoglobulins.
• Amyloid proteins are formed due to the malignant growth of plasma cells.
• AL amyloidosis usually affects the heart, kidney, and liver. AA amyloidosis is secondary, usually due
to inflammatory disease, e.g. rheumatoid arthritis.
Incorrect Options:
Option A - Typical involves kidneys only:
• Amyloidosis usually involves the heart, liver, kidneys, spleen, and digestive tract (enlargement of the
tongue).
Option B - Hematuria is the early sign:
• Amyloidosis usually presents with proteinuria and generalised body edema.
• Hematuria is not a typical sign of this disease.

Page 14

881
Option C - Amyloid deposits are heterogenous, cellular and amorphous on H &E; stain:
• The amyloid deposits are homogenous, a-cellular and amorphous by H&E; staining on microscopy.

Solution for Question 7:


Correct Option A - Minimal change in disease:
• It is a case of a nephrotic syndrome characterized by Massive proteinuria ( > 3.5g/ day)
Hypoalbuminemia ( 3g/dl) Generalized edema hyperlipidemia
• Massive proteinuria ( > 3.5g/ day)
• Hypoalbuminemia ( 3g/dl)
• Generalized edema
• hyperlipidemia
• The fusion of foot processes of the glomerular epithelial cells, also known as "foot process
effacement," is a characteristic finding in a condition called "minimal change disease" (MCD)
• Minimal change disease is the most common cause of nephrotic syndrome in children.
• The common presenting age is 2 to 6 years.
• It may occur following a respiratory infection or routine immunization, in response to corticosteroids, or
associated with other atopic disorders such as eczema.
• On light microscopy, the glomeruli appear normal. On electron microscopy, the visceral epithelial cells
show diffuse effacement of foot processes.
• Foot process effacement is also present in membranous glomerulopathy and diabetic nephropathy.
• Most children respond rapidly to corticosteroid therapy.
• Massive proteinuria ( > 3.5g/ day)
• Hypoalbuminemia ( 3g/dl)
• Generalized edema
• hyperlipidemia
Incorrect Options:
Option B - Crescentic glomerulonephritis:
• It is a nephritic syndrome characterized by Hematuria, red cells cast in urine, hypertension, and
proteinuria.
• It is mostly seen in RPGN.
• Patients typically present with a combined nephrotic nephritic clinical picture.
• The glomeruli show focal or segmental necrosis and the crescents are formed in the urinary spaces
by the proliferation of parietal cells.

• Composition of the crescents - Fibrin, Leukocytes, and Parietal epithelial (most common) and visceral
epithelial cells (least common)

Page 15

882
Option C - Acute proliferative glomerulonephritis:
• It is a type of nephritic syndrome typically characterized by Hematuria, hypertension, and proteinuria.
• The electron microscopy in this case will show characteristic subepithelial humps.

• This option is incorrect since the child has presented with nephrotic range proteinuria.

Option D - Henoch-Schonlein purpura:


• It is a type of IgA nephropathy that shows IgA deposits along the mesangium detected through
immunofluorescence

Solution for Question 8:

Page 16

883
Correct Option C - Lupus nephritis:
• This patient, with proteinuria, no history of recent illness, and immunofluorescence staining showing
the deposition of immune complexes and complement, most likely has these findings secondary to
underlying lupus nephritis.
• Lupus nephritis shows the Full House Effect (due to the presence of various immune complexes and
complement deposits) seen on immunofluorescence.
Incorrect Options:
Option A - Focal segmental glomerulosclerosis:
• It is associated with the focal involvement of kidneys and deposition of IgM, C3, and C1q. It is usually
seen in HIV and heroin abuse.

• Lupus nephritis shows the Full House Effect (due to the presence of various immune complexes and
complement deposits) seen on immunofluorescence.

Option B - Post-strep glomerulonephritis:


• It most commonly occurs a few weeks after a strep pyogenes throat infection.
• It is an immunocomplex-related disease .
• Immunofluorescence shows (Granular/lumpy-bumpy/Starry-Sky appearance)

Page 17

884
Option D - Goodpasture syndrome:
• Goodpasture syndrome is characterized by the formation of auto-antibodies against the basement
membrane.
• A linear pattern is seen on immunofluorescence.

Solution for Question 9:


Correct Option D - Alport syndrome:
• Presentation of a male patient with lens dislocation, end-stage renal disease with a family history of
renal disease, and a distinctive "basket-weave" appearance of glomerular basement membrane (GBM)
is highly suggestive of Alport syndrome.

Page 18

885
• The following are characteristics of Alport syndrome:
• Clinical signs consist of abnormalities related to the eyes, ears, and kidneys.
• It results from mutations affecting the gene encoding for the type IV collagen alpha-5 chain.
• The BM's distinctive basket-weave appearance.

Incorrect Options:
Option A - Autosomal dominant polycystic kidney disease:
• AD polycystic kidney disease is ruled out because there is no association of ADPKD with lens
dislocation, as is mentioned in this case.
Option B - Autosomal recessive polycystic kidney disease:
• AR polycystic kidney disease is ruled out because the age of presentation in ARPKD is childhood, and
most affected children do not survive beyond childhood.
Option C - Marfan's syndrome:
• Lens dislocation is also a feature of Marfan's syndrome, but it has no documented association with
renal disease or renal disease-related death.

Solution for Question 10:


Correct Option D - Henoch Schönlein purpura (HSP):

The likely diagnosis in this child is Henoch Schönlein's purpura.


• It is a childhood syndrome and is most common between 3-8 years of age.
• It can also occur in adults, in whom renal manifestations are more severe.

Page 19

886
• Consists of purpuric skin lesions, abdominal pain and intestinal bleeding, arthralgias, and renal
abnormalities.
• Renal abnormalities are usually microscopic or gross hematuria.
• Characteristic findings on fluorescence microscopy are the deposition of IgA(predominantly), IgG, and
C3 in the mesangial region.
Incorrect Options:
Option A - Focal segmental glomerulosclerosis (FSGS):
• Nephrotic syndrome and concomitant sclerosis of some localized segments of the glomeruli are its
defining features.
• It can be idiopathic or develop as a result of the following conditions:
• Sickle cell disease, heroin usage, obesity, and HIV.
Option B - Minimal change disease (MCD):
• In children, it is the most frequent cause of nephrotic syndrome.
• The condition may be idiopathic or have a trigger, such as: Immunization Infection
• Immunization
• Infection
• Immunization
• Infection
Option C - Alport syndrome:
• The following are characteristics of Alport syndrome:
• Errors in the gene that codes for the type IV collagen α5 chain.
• The clinical signs include kidney, ear, and eye-related abnormalities.
• The BM's distinctive basket-weave appearance.

Solution for Question 11:


Correct Option A - Nodular glomerulosclerosis:
• This patient, with frothy urine, facial oedema, and uncontrolled type 2 diabetes mellitus, most likely
has nephrotic syndrome secondary to diabetic nephropathy.
• Eosinophilic nodular glomerulosclerosis is a characteristic finding in diabetic nephropathy.
• It is known as Kimmelstiel-Wilson nodules.
• It takes about 15-20 years the development of nephrotic syndrome secondary to diabetes mellitus.
Incorrect Options:
Option B - Apple-green birefringence under polarised light:
• It is a characteristic finding of amyloidosis.
• It presents with non-specific symptoms related to multiple organs.

Page 20

887
Option C - Segmental sclerosis and hyalinosis:
It is seen in cases of FSGS. FSGS can develop secondary to the following conditions:
• FSGS can develop secondary to the following conditions:
• HIV
• Obesity
• SCD
• Interferon treatment
Option D - Normal glomeruli with effacement of the foot processes:
• It is a characteristic finding of Minimal change disease (MCD).
• MCD is the most common cause of nephrotic syndrome in children.

Solution for Question 12:


Correct Option A - Goodpasture syndrome:
• This patient, with pulmonary haemorrhage, renal abnormalities, and Anti-GBM antibodies, most likely
has Goodpasture syndrome.
The following are the features of Goodpasture syndrome:
• Uncommon autoimmune disease in which kidney and lung injury are caused by circulating
autoantibodies.
• Most common in the 2nd & 3rd decades of life.
• Anti-GBM antibody-mediated disease
• Patients typically present with a rapidly progressive glomerulonephritis and necrotizing hemorrhagic
interstitial pneumonitis.
• Characterized by linear deposits of IgG and, in many cases, C3 in the GBM.

Page 21

888
(Linear pattern in observed in the image above)
Incorrect Options:
Option B - Alport syndrome:
• Alport syndrome is characterized by the following:
• Mutations involving the gene encoding for the α5 chain of type IV collagen.
• The clinical manifestations are comprised of findings associated with the eyes, ears, and kidneys.
• The characteristic basket-weave appearance of the BM.
Option C - Focal segmental glomerulosclerosis:
• Its distinguishing characteristics include nephrotic syndrome and concurrent sclerosis of a few
isolated glomerular segments.
• It may be idiopathic or result from one of the following illnesses:
• HIV
• Heroin use
• Obesity
• Sickle cell disease.
Option D - IgA nephropathy:
It can be identified by the following findings:
• Episodic hematuria
• Hematuria happens at the same time as a URTI or GI illness.

Solution for Question 13:


Correct Option A - 1,2 and 3 are true:

Page 22

889
The image shows hypercellular glomerulus, increased inflammatory (neutrophilic) infiltrate, proliferative
changes in endothelium and mesangial cells
The likely diagnosis in this patient is post-streptococcal glomerulonephritis.
1. It is a Type III Hypersensitivity reaction (Immune complex mediated)
2. Streptococcus pyogenes: strains 1,2 and 4 are responsible.
3. Gross findings on the kidney show a flea-bitten appearance.
These statements are true regarding PSGN.
Incorrect Options:
4. Linear immunofluorescence is seen.
This statement is not true, linear immunofluorescence is seen in good pasture syndrome. In PSGN im
munofluorescence shows starry sky appearance.

Solution for Question 14:


Correct Option A - Berger disease:
• This patient, with hematuria occurring concurrently with a URTI and IgA deposition in the mesangium,
most likely has Berger disease.
• Berger's disease is characterized by the following: IgA-mediated nephropathy Episodic hematuria
(Hematuria occurring at the same time as that of some URTI or GI infection)
• IgA-mediated nephropathy
• Episodic hematuria (Hematuria occurring at the same time as that of some URTI or GI infection)
• IgA-mediated nephropathy
• Episodic hematuria (Hematuria occurring at the same time as that of some URTI or GI infection)
Incorrect Options:
Option B - Alport syndrome:
• This patient has no findings related to the eyes or ears, effectively ruling it out.
• Moreover, a basket-weave appearance of the BM is a characteristic feature, rather than IgA
deposition in the mesangium.
Option C - Minimal change of disease:
• It is characterized by nephrotic-range proteinuria, particularly in children, not hematuria.
• It can be idiopathic or secondary to some triggering factors such as infection or immunization.
Option D - Post-streptococcal glomerulonephritis:
• It is characterized by hematuria that occurs a few weeks after a streptococcus pyogenes infection, not
concurrently.
• It is basically a post-pharyngitic pathology.
• Deposition of IgG, IgM, and C3 is found along the BM and mesangium on immunofluorescence.

Page 23

890
Solution for Question 15:
Correct Option D - Focal segmental glomerulosclerosis:
• This patient, with nephrotic-range proteinuria and a history of intravenous drug addiction, most likely
has Focal segmental glomerulosclerosis (FSGS).
• FSGS has the following features: Characterized by glomerular scarring (sclerosis) that affects some
(focal), but not all, glomeruli. Initially involves only part of an affected glomerular tuft (segmental). The
most common renal complication of intravenous drug abuse. Proteinuria, which occasionally may be so
massive as to produce nephrotic syndrome. Associated with HIV infection is a severe and rapidly
progressive, collapsing form of FSGS.
• Characterized by glomerular scarring (sclerosis) that affects some (focal), but not all, glomeruli.
• Initially involves only part of an affected glomerular tuft (segmental).
• The most common renal complication of intravenous drug abuse.
• Proteinuria, which occasionally may be so massive as to produce nephrotic syndrome.
• Associated with HIV infection is a severe and rapidly progressive, collapsing form of FSGS.
• Characterized by glomerular scarring (sclerosis) that affects some (focal), but not all, glomeruli.
• Initially involves only part of an affected glomerular tuft (segmental).
• The most common renal complication of intravenous drug abuse.
• Proteinuria, which occasionally may be so massive as to produce nephrotic syndrome.
• Associated with HIV infection is a severe and rapidly progressive, collapsing form of FSGS.
Incorrect Options:
Option A - Post-streptococcal glomerulonephritis:
• It is characterized by a nephritic syndrome occurring secondary to streptococcus pyogenes infection.
• It typically occurs 1-2 weeks after the strep. Infection (Post-pharyngitis).
• It occurs due to skin infections too and does not recur again after one occurrence

Page 24

891
The image shows hypercellular glomerulus, increased inflammatory (neutrophilic) infiltrate, proliferative
changes in endothelium and mesangial cells
Option B - Minimal change of disease:
• It is the most common cause of nephrotic syndrome in children.
• It can be idiopathic or occur secondary to the following: Immunization Infection
• Immunization
• Infection
• It is also frequently associated with Hodgkin lymphoma.
• Immunization
• Infection

Option C - IgA Nephropathy:


• It is characterized by the findings of episodic hematuria occurring concurrently with upper respiratory
tract infection or GI infection (Syn-pharyngitic).

Page 25

892
Solution for Question 16:
Correct Option D - Immune complex deposition in subepithelium:
• Penicillamine has been associated with the development of membranous nephropathy, and the
immune complexes deposit in the subepithelial space, leading to the characteristic spike and dome
pattern observed on electron microscopy.
Incorrect Options:
Option A - Antibodies against phospholipase A2 receptor (PLA2Rc): Associated with primary (idiopathi
c) membranous glomerulonephropathy, not specifically with penicillamine-induced membranous glome
rulonephropathy.
Option B - Thrombospondin THSD7A: It is associated with primary (idiopathic) membranous glomerulo
nephropathy, not the secondary form caused by penicillamine.
Option C - CD 10: CD 10 is mentioned in the context of primary membranous glomerulonephropathy, it
is not associated with MGN induced by penicillamine

Solution for Question 17:


Correct Option D - Hypertrophy and proliferation of visceral epithelial cells:
• HIV-associated nephropathy leads to the development of FSGS (focal segmental glomerulosclerosis).
• The most common type associated with HIV is collapsing glomerulopathy.
• It is characterized by segmental collapse of the glomerular capillaries
• Histologically, it is associated with hypertrophy & proliferation of visceral epithelial cells.
• These patients have rapid kidney deterioration.

Page 26

893
• The image below is a micrograph of collapsing glomerulopathy.

Incorrect Options:
Option A - Crescent formation:
• The crescent formation is seen in crescentic glomerulonephritis.
• It is not seen in collapsing glomerulopathy.
Option B - Mesangial deposition of IgA:
• Mesangial deposition is seen in Ig A nephropathy and HSP.
• It is not seen in collapsing glomerulopathy.
Option C - Subepithelial deposition with endothelial cell proliferation:
• Subepithelial deposits are not seen in collapsing glomerulopathy, it is seen in membranoproliferative
glomerulonephritis (MPGN)
Subepithelial deposits are not seen in collapsing glomerulopathy, it is seen in membranoproliferative gl
omerulonephritis (MPGN)

Solution for Question 18:


Correct Option C - Nephrotic syndrome:
• The child is seven years old and presented with complaints of peri-orbital and generalized swelling
and has proteinuria in the range of nephrotic syndrome.

• In the given graph, there is ↓ albumin, ↑ alpha-2 globulin, ↓ gamma globulin.
• In nephrotic syndrome, albumin appears in the urine. Thus, albumin decreases in blood.
• The two major types of protein present in the serum are albumin and globulin proteins. Albumin is the
major protein component of serum and has the largest peak lying closest to the positive electrode.

Page 27

894
• However, globulins are a smaller fraction of the total serum protein but are the primary focus of
interpreting serum protein electrophoresis.

Incorrect Options:
Option A - Tuberculosis:
• In tuberculosis, gamma globulins level increase.
Option B - Multiple myeloma:
• In multiple myeloma, there is M-spike.
• It is due to a rise in gamma globulins.
Option D - Liver cirrhosis:
• In liver cirrhosis, there is a decrease in the function of the liver, due to which all proteins will decrease.

Solution for Question 19:


Correct Option C -
It is not commonly associated with colonization of the distal urethra and introitus:
The clinical presentation and histological findings described in the scenario are suggestive of chronic p
yelonephritis.
• This statement is incorrect. Recurrent urinary tract infections are often associated with colonization of
the distal urethra and introitus, and they may contribute to the ascending infections that lead to chronic
pyelonephritis.

Page 28

895
• The term "thyroidization of tubules" is often associated with chronic pyelonephritis, where the tubules
take on a histological appearance resembling thyroid tissue (as in the middle section of the slide). This
finding is suggestive of long-standing chronic damage to the renal tubules, and it aligns with the clinical
presentation of recurrent urinary tract infections, flank pain, and imaging showing scarring and atrophy.
Incorrect Options:
Option A - Interstitial fibrosis is observed:

• Interstitial fibrosis is a characteristic feature of chronic pyelonephritis, and it is consistent with the
scarring and atrophy of the renal parenchyma described in the scenario
Option B - Thyroidization of tubules is seen:
• This is also a characteristic finding in chronic pyelonephritis, where the tubules exhibit
eosinophilic-staining cells resembling thyroid tissue
Option D - Blunted calyces may be observed:
• Blunted calyces are commonly observed in chronic pyelonephritis, and they result from scarring and
atrophy of the renal parenchyma.

Page 29

896
Solution for Question 20:
Correct Option B- Class 3:
• In the classification system for lupus nephritis, Class 3 corresponds to focal lupus nephritis, where
focal lesions involve less than 50% of glomeruli.
Incorrect Options:
Option A- Class 2: Class 2 lupus nephritis refers to mesangial proliferative lupus nephritis.
Option C- Class 4: Class 4
lupus nephritis refers to diffuse lupus nephritis, where more than 50% of glomeruli are involved.
Option D- Class 5: Class 5 lupus nephritis refers to membranous lupus nephritis.

Solution for Question 21:


Correct Option C- RPGN 3:
• Rapidly progressive glomerulonephritis (RPGN) 3 is the most common form of RPGN and includes
conditions such as microscopic polyangiitis (MPA) and Wegener's granulomatosis (Pauci Immune
Vasculitis).
Incorrect Options:
Option A- RPGN 1: RPGN 1 refers to anti-glomerular basement membrane (anti-GBM) antibody-media
ted glomerulonephritis and includes Goodpasture syndrome.
Option B- RPGN 2: RPGN 2 refers to immune complex-mediated glomerulonephritis, which includes co
nditions such as lupus nephritis and post-infectious glomerulonephritis, HSP and IgA nephropathy.
Option D- RPGN 4: RPGN 4 is not a type of rapidly progressive glomerulonephritis.

Page 30

897
Solution for Question 22:
Correct Option C- 1-D, 2-C, 3-A, 4-B:

IgA Nephropathy/ Berger's Disease


• M/C in Adults
• M/C worldwide
• Associated with infection, galactose-deficient IgA1 antibodies are produced due to an underlying
genetic condition
• Further antibodies are produced against Ig A1, and the Ag-Ab complex gets deposited in the kidney;
Mesangial deposition takes place.

Page 31

898
Focal segmental glomerulonephritis
• Immune complexes deposit in the subepithelial region, forming subepithelial deposits.
• This leads to the formation of spikes from the basement membrane. The spikes and deposits show a
Spike and Dome Pattern.
• These spikes fuse together, forming a Thickened Basement Membrane.

MPGN (Membranoproliferative Glomerulonephritis)/ Mesangiocapillary Glomerulonephritis


• Shows features of both MGN (thickened membrane) and PSGN (proliferation)
• Tram tracking appearance is seen due to the splitting of the basement membrane, which occurs when
sub-endothelial deposits are removed by the mesangial cells.

Rapidly Progressive Glomerulonephritis/ Crescentic glomerulonephritis


• Composition- Fibrin, Leukocytes, and Parietal epithelial (most common) and visceral epithelial cells
(least common)

Page 32

899
• Mnemonics: FLaP
• Prognosis: Increased number of crescent- poor prognosis.

Page 33

900
Previous Year Questions
1. What is the most likely reason for a child to have hematuria and RBC casts in the urine based on
urine microscopy and examination findings?
A. Focal segmental glomerulosclerosis (FSGS)
B. Post-streptococcal glomerulonephritis (PSGN)
C. Membranoproliferative glomerulonephritis (MPGN)
D. Minimal change disease
----------------------------------------
2. Which of the following is a primary constituent of Tamm-Horsfall mucoprotein?
A. Mural thrombi
B. Curschmann spirals
C. Renal casts
D. Bence-Jones proteins
----------------------------------------
3. Which of the following statements is/are true regarding clear cell carcinoma of the kidney? Most
common 1° renal malignancy Associated with loss of long arm of chromosome 3 Not Associated with
paraneoplastic syndrome Classic triad: flank pain, palpable mass, hematuria is seen
A. 1,2,4
B. 2,3
C. 1,3
D. 1,4
----------------------------------------
4. What is the underlying reason for the development of edema in individuals with nephrotic syndrome?
A. Increase colloid osmotic pressure
B. Increase hydrostatic pressure
C. Decreased colloid osmotic pressure
D. Decreased hydrostatic pressure
----------------------------------------
5. A 51-year-old male patient came with complaints of hematuria to the clinic. On examination, he was
normotensive and had pedal edema. Investigations revealed the patient had no glycosuria and had a
creatinine of 9. His renal biopsy is shown below. Which of the following investigations should be done
to identify the disease?

901
A. ANA
B. Anti-GBM antibodies
C. HIV RNA
D. Urine immunoelectrophoresis
----------------------------------------
6. Effacement of podocyte foot processes is seen in which of the following diseases?
A. Steroid-resistant glomerulonephritis
B. Minimal change disease
C. Rapidly progressive glomerulonephritis
D. Chronic glomerulonephritis
----------------------------------------
7. What could be the probable reason behind the occurrence of significant proteinuria in a 40-year-old
male, who was diagnosed with colorectal cancer a few months ago?
A. Membranous nephropathy
B. Focal segmental glomerulosclerosis
C. Membranoproliferative glomerulonephritis
D. Dense deposit disease
----------------------------------------
8. What is the most reliable and enduring characteristic of bladder carcinoma?
A. Painless hematuria
B. Painful hematuria
C. Dysuria
D. Suprapubic mass
----------------------------------------
9. In the papillary type of renal tumor, what is the primary factor that determines its malignant potential?
A. Immunohistology
B. Size

Page 2

902
C. Cytogenetics
D. Part of kidney involved
----------------------------------------
10. Which of these is Nephritic syndrome?
A. Minimal change disease
B. Membranous Glomerulopathy
C. Post infectious Glomerulonephritis
D. Focal segmental glomerulosclerosis
----------------------------------------
11. Most characteristic finding of diabetic nephropathy?
A. Kimmelstein Wilson disease
B. Diffuse glomerulosclerosis
C. Focal segmental glomerulosclerosis
D. Armani Ebstein change
----------------------------------------
12. What are the expected types of urinary casts that may be observed in the urinalysis report of a
30-year-old male with Fabry's disease, encountered during rounds?
A. Waxy cast
B. RBC cast
C. WBC cast
D. Maltese cross cast
----------------------------------------
13. What is the most notable characteristic feature of diabetic nephropathy as demonstrated below?

A. Kimmelstein Wilson nodules


B. Armani Ebstein change
C. Focal segmental glomerulosclerosis
D. Membrano proliferative glomerulonephritis
----------------------------------------

Page 3

903
14. What is the condition in which membranoproliferative glomerulonephritis is observed? HIV SLE CLL
Hepatitis B
A. 1,2,3
B. 1,2,3,4
C. 2,3,4
D. 1,2
----------------------------------------
15. What is the most likely diagnosis based on the given serum electrophoresis image?

A. Multiple myeloma
B. Tuberculosis
C. Hepatic cirrhosis
D. Nephrotic syndrome
----------------------------------------
16. In addition to nephrotic syndrome, which diseases are associated with the presence of Maltese
cross crystals in the urine?
A. Friedrich’s ataxia
B. Felty syndrome
C. Fanconi anaemia
D. Fabry’s disease
----------------------------------------
17. A middle – aged male patient has proteinuria. Looking at the immunofluorescence image below,
what is the diagnosis ?

Page 4

904
A. Focal segmental glomerulosclerosis (FSGS)
B. Post – streptococcal glomerulonephritis (PSGN)
C. Good pasture syndrome
D. Lupus nephritis
----------------------------------------
18. A patient underwent a cystoscopy, which showed multiple yellow-white plaques in the trigone of the
bladder. The histopathology image is given below. What is the diagnosis?

A. Interstitial cystitis
B. Malakoplakia
C. Polypoid cystitis
D. Acute cystitis
----------------------------------------
19. A pediatric patient arrived at the hospital with cola-coloured urine, high blood pressure, and swelling
around the eyes. Laboratory tests were conducted, revealing a creatinine level of 2.5 mg/dL. After
commencing the treatment, the patient exhibited no signs of improvement over a period of 3 weeks.
Furthermore, their creatinine levels rose to 4.5 mg/dL. Consequently, which of the below electron
microscopic observations is expected in this patient?
A. Subendothelial deposits
B. Subepithelial deposits
C. Crescent formation
D. Mesangial deposits

Page 5

905
----------------------------------------
20. What is the diagnosis of a 48-year-old man with facial puffiness, frothy urine, hypertension, a history
of hepatitis B infection, microscopic hematuria, and a kidney biopsy showing a spike and dome pattern?
A. Minimal change disease
B. Membranous nephropathy
C. Focal segmental glomerulosclerosis
D. PSGN-associated disease
----------------------------------------

Correct Answers
Question Correct Answer

Question 1 2
Question 2 3
Question 3 1
Question 4 3
Question 5 2
Question 6 2
Question 7 1
Question 8 1
Question 9 2
Question 10 3
Question 11 1
Question 12 4
Question 13 1
Question 14 2
Question 15 4
Question 16 4
Question 17 4
Question 18 2
Question 19 2
Question 20 2

Solution for Question 1:


• The probable cause is post-streptococcal glomerulonephritis.
• It generally occurs following group A beta-hemolytic streptococcal throat or skin infection within a
period of 1-4 weeks.

Page 6

906
• It affects children between 6-10 years of age but can also occur in adults.
• The child may present with edema around the eyes, high blood pressure, fever, reduced urine output,
nausea, and passage of cola-colored urine.
• There is the formation of immune complexes within the glomeruli, which in turn initiates inflammation,
stimulates activation of neutrophils, and causes the proliferation of mesangial and endothelial cells
resulting in a hypercellular glomerulus.
Incorrect Choices:
• Option a. Focal segmental glomerulosclerosis: Sclerosis involving a part of the capillary tuft of some
glomeruli. The majority are idiopathic. Although it can occur secondary to sickle cell disease, drugs, or
infections like HIV. It can also be congenital or due to inherited mutations. They present as nephrotic
syndrome or with excessive proteinuria. However, hematuria and RBC casts in urine are not
characteristic features. On light microscopy, the glomerulus shows sclerosis and hyalinosis.
• Option c. Membranoproliferative glomerulonephritis is characterized by leukocyte infiltration,
thickened glomerular basement membrane, and proliferation of mesangial cells. It causes nephrotic
syndrome in children and young adults but is mostly seen among adolescents. They may present with a
combination of nephrotic and nephritic syndrome. On electron microscopy, the tram track appearance
of GBM is seen due to the subendothelial deposition of immune complexes.
• Option d. Minimal change disease is the most common cause of nephrotic syndrome in children. They
present with features of nephrotic syndrome, with highly selective proteinuria (more albumin-causing
hypoalbuminemia), and hyperlipidemia. The renal function generally remains unaffected. On light
microscopy, glomeruli appear normal and glomerular changes are minimal or absent. They have an
excellent prognosis.

Solution for Question 2:


• Tamm-Horsfall mucoprotein is synthesized by ascending limb of the loop of Henle in the kidney.
• It is found in abundance in urine.
• It is a major component of renal casts.
• This protein prevents bacteria from getting attached to the urinary epithelium. It also helps in the
regulation of salt and water in the body. It helps in preventing the formation of calcium stones within the
kidney. Elevated levels may indicate impaired kidney function.
Incorrect Choices:
• Option a. Mural thrombi: Mural thrombi are formed within the inner lining of the heart’s chambers.
They are typically composed of fibrin and platelets along with RBCs, WBC, and endothelial cells.
• Option b. Curschmann spirals: Curschmann spirals are cork-screw-shaped twists of condensed
mucus and are surrounded by a clear material. They are typical microscopic features that are observed
in the sputum of patients suffering from bronchial asthma and COPD. They are composed of cellular
debris and certain eosinophil-derived proteins.
• Option d. Bence-Jones proteins: These proteins are found in the urine of patients diagnosed with
multiple myeloma.

Page 7

907
Solution for Question 3:
Correct Option A - 1,2,4:
The correct statements regarding clear cell carcinoma of the kidney are:
Most common 1° renal malignancy- True
Associated with loss of long arm of chromosome 3 - True
Classic triad: flank pain, palpable mass, hematuria is seen- True
So, statements 1, 2, and 4 are true regarding clear cell carcinoma of the kidney. Statement 3 is false.

Solution for Question 4:


Correct option:
Option. C
• "Decreased colloid osmotic pressure." In nephrotic syndrome, there is an excessive loss of proteins,
particularly albumin, in the urine. This loss of protein leads to a decrease in the concentration of
proteins in the blood vessels, resulting in decreased colloid osmotic pressure (oncotic pressure).
• Colloid osmotic pressure is the force generated by the presence of proteins in the blood vessels. It
helps to retain fluid within the vessels by attracting water molecules. When the colloid osmotic pressure
decreases in nephrotic syndrome, there is a reduced ability of the blood vessels to retain fluid, leading
to fluid leakage into the surrounding tissues and the formation of edema.
Incorrect options:
Option A. "Increase colloid osmotic pressure" is not the cause of edema in nephrotic syndrome. In fact,
there is a decrease in colloid osmotic pressure due to protein loss in the urine.
Option B. "Increase hydrostatic pressure" is also incorrect. Hydrostatic pressure refers to the pressure
exerted by the fluid within the blood vessels. While an increase in hydrostatic pressure can lead to ede
ma in conditions such as heart failure or kidney disease, it is not the primary cause of edema in nephro
tic syndrome.
Option D. "Decreased hydrostatic pressure" is not the cause of edema in nephrotic syndrome. Decreas
ed hydrostatic pressure would actually promote fluid reabsorption into the blood vessels, reducing the li
kelihood of edema formation.

Solution for Question 5:


Correct Option B - Anti-GBM antibodies:
• Crescents on renal biopsy indicate glomerular involvement and suggest the possibility of rapidly
progressive glomerulonephritis (RPGN).
• The most relevant test to perform is the detection of anti-glomerular basement membrane (anti-GBM)
antibodies.
• Anti-glomerular basement membrane antibodies are specific markers for Goodpasture syndrome, a
rare autoimmune disease characterized by the presence of autoantibodies targeting the glomerular

Page 8

908
basement membrane and lung alveolar basement membrane.
Incorrect Options:
Option A - ANA:
• Antinuclear antibodies are associated with systemic autoimmune conditions, such as systemic lupus
erythematosus.
• While SLE can involve the kidneys and cause glomerulonephritis, it is not the most likely cause, and
the presence of crescents suggests a more acute and rapidly progressive disease.
Option C - HIV RNA:
• Human immunodeficiency virus infection can cause renal complications, including HIV-associated
nephropathy.
• The patient's presentation of hematuria, pedal edema, and the presence of crescents on renal biopsy
make Goodpasture syndrome a more likely diagnosis than HIV-associated nephropathy.
Option D - Urine immunoelectrophoresis: Urine immunoelectrophoresis is a useful test in evaluating va
rious renal diseases, including the detection of monoclonal immunoglobulins in conditions such as mult
iple myeloma and amyloidosis.

Solution for Question 6:


Correct Option B - Minimal change disease:
• Effacement of podocyte foot processes refers to the flattening or loss of the foot processes of
podocytes, which are specialized cells in the glomerulus of the kidney.
• This structural alteration is a characteristic finding in minimal change disease, also known as minimal
change nephrotic syndrome.
• Minimal change disease is a type of nephrotic syndrome characterized by massive proteinuria,
hypoalbuminemia, edema, and lipiduria.
• In minimal change disease, the glomeruli appear normal under light microscopy, and the
characteristic findings are only visible under electron microscopy, i.e., effacement of podocyte foot
processes.
Incorrect Options:
• Effacement of podocyte foot processes is not seen in Options A, C, and D.

Solution for Question 7:


Correct Option: A
The likely cause of the development of urinary findings in this case is Membranous nephropathy.
Explanation of the options:
Option A: Membranous nephropathy: Membranous nephropathy is a glomerular disease characterized
by the thickening of the glomerular basement membrane. It is commonly associated with solid tumors, i
ncluding cancer of the bowel. The immune complexes formed in the presence of cancer can deposit in

Page 9

909
the glomerular basement membrane, leading to proteinuria.
Option B: Focal segmental glomerulosclerosis: Focal segmental glomerulosclerosis (FSGS) is a kidney
disorder characterized by scarring and sclerosis of the glomeruli. While FSGS can occur in the setting
of cancer, it is not commonly associated with solid cancer of the bowel. Therefore, FSGS is less likely t
o be the cause of the urinary findings in this case.
Option C: Membranoproliferative glomerulonephritis: Membranoproliferative glomerulonephritis (MPGN
) is a type of glomerular disease characterized by thickening of the glomerular basement membrane an
d proliferation of cells within the glomeruli. While MPGN can be associated with certain underlying con
ditions, it is not commonly associated with solid cancer of the bowel. Therefore, MPGN is less likely to
be the cause of the urinary findings in this case.
Option D: Dense deposit disease: Dense deposit disease (DDD), also known as C3 glomerulopathy, is
a rare kidney disorder characterized by the abnormal accumulation of complement component C3 withi
n the glomeruli. While DDD can be associated with certain underlying conditions, it is not commonly as
sociated with solid cancer of the bowel. Therefore, DDD is less likely to be the cause of the urinary findi
ngs in this case.
Based on the association between solid cancer of the bowel and the development of proteinuria, the m
ost likely cause of the urinary findings in this case is Membranous nephropathy.

Solution for Question 8:


Correct Option A: Painless hematuria
• Painless hematuria: Painless hematuria is the most consistent and persistent feature of bladder
carcinoma.
• Hematuria refers to the presence of blood in the urine, and when it occurs without pain, it is often
associated with bladder carcinoma.
• Painless hematuria may occur intermittently or be persistent, and it is an important warning sign that
should prompt further evaluation for bladder cancer.
Incorrect Options:
Option B: Painful hematuria- While hematuria can sometimes be associated with pain, especially in co
nditions such as urinary tract infections or kidney stones, painful hematuria is less commonly seen in bl
adder carcinoma. Bladder carcinoma typically presents with painless hematuria as the primary sympto
m.
Option C: Dysuria- Dysuria refers to painful or difficult urination and is not a consistent feature of bladd
er carcinoma. Dysuria is more commonly associated with urinary tract infections or other conditions aff
ecting the urinary system.
Option D: Suprapubic mass- The presence of a suprapubic mass is not a specific or consistent feature
of bladder carcinoma. Bladder carcinoma may lead to the development of a bladder tumor or a
thickened bladder wall, but the presence of a palpable mass is less common.

Solution for Question 9:

Page 10

910
Correct Option B: Size
• In the papillary type of renal tumor, the most important determinant of malignant potential is the size of
the tumor.
• One of the most important predictors is size: Size<1.5cm ⇒ Adenoma Size >=1.5cm ⇒ Papillary RCC
• Size<1.5cm ⇒ Adenoma
• Size >=1.5cm ⇒ Papillary RCC
• Size<1.5cm ⇒ Adenoma
• Size >=1.5cm ⇒ Papillary RCC
Incorrect Options:
Option A: Immunohistology- Immunohistology can provide valuable information about the molecular ch
aracteristics of a tumor, it is not the most important determinant of malignant potential in the papillary ty
pe of renal tumor.
Option C: Cytogenetics- Cytogenetics can contribute to the assessment of malignant potential in tumor
s, it is not the most important determinant in the papillary type of renal tumor.
Option D: Part of kidney involved- The location of the tumor within the kidney is not the most significant
factor in determining malignant potential in the papillary type of renal tumor.

Solution for Question 10:


Correct Option C.
• Nephritic syndrome is characterized by glomerular inflammation and is typically associated with the
following clinical features:
• Hematuria (red blood cells in the urine)
• Proteinuria (protein in the urine, usually less severe than in nephrotic syndrome)
• Hypertension (elevated blood pressure)
• Edema (swelling due to fluid retention)
Incorrect options:
Option A. Minimal change disease: Minimal change disease is a form of nephrotic syndrome characteri
zed by massive proteinuria but typically lacks the characteristic features of nephritic syndrome such as
hematuria and hypertension.
Option B. Membranous glomerulopathy: Membranous glomerulopathy is another form of nephrotic syn
drome, not nephritic syndrome. It is characterized by thickening of the glomerular basement membrane
, leading to proteinuria, but without significant hematuria or hypertension.
Option D. Focal segmental glomerulosclerosis: Focal segmental glomerulosclerosis is also a form of ne
phrotic syndrome and is characterized by scarring and sclerosis of certain segments of glomeruli. It is n
ot typically associated with the clinical features of nephritic syndrome

Page 11

911
Solution for Question 11:
CorrectOption A.
Kimmelstein Wilson disease: Kimmelstein Wilson disease, also known as Wilson's disease, is a geneti
c disorder that causes copper accumulation in the body, primarily affecting the liver and brain. It is the
most characteristic finding of diabetic nephropathy.
Incorrect options:
Option B: Diffuse glomerulosclerosis: Diffuse glomerulosclerosis refers to the widespread scarring and
thickening of the glomeruli in the kidneys due to long-standing uncontrolled diabetes mellitus. This con
dition is a major cause of end-stage renal disease.
Option C. Focal segmental glomerulosclerosis: Focal segmental glomerulosclerosis is a distinct kidney
disease characterized by scarring in specific areas of the glomeruli. While it can be associated with so
me forms of kidney damage, it is not the most characteristic finding of diabetic nephropathy.
Option D. Armani Ebstein change: Armani Ebstein change is not a
recognized term or characteristic finding related to diabetic nephropathy

Solution for Question 12:


Correct Option D:
Fabry's disease is a rare genetic disorder characterized by the deficiency or dysfunction of the enzyme
alpha-galactosidase A. This leads to the accumulation of a
lipid called globotriaosylceramide (Gb3) within various cells and tissues, including the kidneys.
One of the characteristic findings in the urinalysis of individuals with Fabry's disease is the presence of
Maltese cross casts, also known as "corneal-shaped" or "glycolipid" casts. These casts are formed by t
he precipitation of Gb3 within the renal tubules. They appear as refractile, yellowish-brown structures w
ith a cross-like pattern when viewed under polarized light.
Incorrect Options:
Option A. Waxy cast: Waxy casts are broad, homogeneous casts that indicate chronic renal disease. T
hey are composed of degenerated renal tubular epithelial cells and often associated with severe protei
nuria. While kidney involvement is a
feature of Fabry's disease, the presence of waxy casts is not specific to this condition.
Option B. RBC cast: RBC casts are cylindrical structures formed by the entrapment of red blood cells w
ithin the renal tubules. They are typically seen in glomerulonephritis, vasculitis, or other conditions caus
ing hematuria. While kidney involvement in Fabry's disease may lead to hematuria, RBC casts are not
specific to this condition.
Option C. WBC cast: WBC casts, also known as pyuria, are formed by the entrapment of white blood c
ells within the renal tubules. They are usually seen in urinary tract infections or interstitial nephritis. Alth
ough kidney involvement in Fabry's disease may lead to inflammation, the presence of WBC casts is n
ot specific to this condition.

Page 12

912
Solution for Question 13:
Correct option: A
• The most characteristic feature of diabetic nephropathy is Kimmelstein-Wilson nodules, also known as
nodular glomerulosclerosis or Kimmelstein-Wilson nodules. These are pathognomonic lesions seen in
the kidneys of patients with diabetic nephropathy. They are characterized by the presence of nodular
mesangial expansion, thickening of the glomerular basement membrane, and hyaline deposits. These
changes are indicative of advanced diabetic nephropathy and are associated with the development of
proteinuria and progressive renal dysfunction.
Incorrect options:
Option B: Armani Ebstein change: Armani Ebstein change is not associated with diabetic nephropathy.
It refers to the finding of glycogen accumulation within the tubular cells of the kidney .
Option C: Focal segmental glomerulosclerosis: Focal segmental glomerulosclerosis (FSGS) is a
distinct renal pathology and is not the most characteristic feature of diabetic nephropathy.
Option D: Membrano proliferative glomerulonephritis: Membrano proliferative glomerulonephritis (MPG
N) is a
different type of glomerular disease and is not the most characteristic feature of diabetic nephropathy.

Solution for Question 14:


Correct Option B:
Membranoproliferative glomerulonephritis (MPGN) is a type of glomerulonephritis characterized by thic
kening and proliferation of the glomerular basement membrane. It can be caused by various underlying
conditions. Let's discuss each option:
Incorrect Options:
Option A: 1, 2, 3
This option is incorrect because it excludes Hepatitis B as a
cause of membranoproliferative glomerulonephritis (MPGN). In reality, Hepatitis B
infection can lead to MPGN, as mentioned in the previous explanation.
Option C: 2, 3, 4
This option is incorrect because it excludes HIV as a cause of membranoproliferative glomerulonephriti
s (MPGN). MPGN can be seen in individuals with HIV infection, particularly as a
manifestation of HIV-associated nephropathy.
Option D: 1, 2
This option is incorrect because it excludes CLL and Hepatitis B as causes of membranoproliferative gl
omerulonephritis (MPGN). MPGN has been reported in association with CLL, although it is relatively ra
re. Additionally, Hepatitis B
infection can also lead to MPGN through immune complex deposition in the glomeruli.

Solution for Question 15:

Page 13

913
Correct Option D.
Nephrotic syndrome: Nephrotic syndrome is a kidney disorder characterized by the abnormal leakage
of protein, particularly albumin, into the urine. This results in low levels of albumin in the blood (hypoalb
uminemia). On serum electrophoresis, nephrotic syndrome can present with a
characteristic pattern of decreased albumin levels and a
prominent alpha-2 band. The decreased albumin leads to a decrease in the albumin peak, and the alph
a-2 band becomes more prominent relative to the albumin peak.

Incorrect Options:
Option A: Multiple myeloma: Multiple myeloma is a cancer of plasma cells, a
type of white blood cell. On serum electrophoresis, multiple myeloma typically shows a distinct M
spike or monoclonal spike, representing the excessive production of a
single abnormal immunoglobulin.
Option B: Tuberculosis: Tuberculosis is a bacterial infection that primarily affects the lungs but can invo
lve other organs as well. Serum electrophoresis is not a specific test for diagnosing tuberculosis. Diagn
osis typically relies on other methods such as sputum culture or imaging studies.
Option C: Hepatic cirrhosis: Hepatic cirrhosis refers to severe scarring of the liver, usually due to chroni
c liver disease. Serum electrophoresis is not a specific test for diagnosing cirrhosis. Liver function tests
, imaging studies, and other specific tests are used to assess liver function and diagnose cirrhosis.

Solution for Question 16:


Correct Option:
Option D.
• Fabry's disease Fabry's disease is a genetic disorder caused by the deficiency of an enzyme called
alpha-galactosidase A. It leads to the accumulation of a fatty substance called globotriaosylceramide in
various organs, including the kidneys. Maltese cross crystals, which are lipid droplets, can be seen in
the urine of individuals with Fabry's disease.
Incorrect Option:

Page 14

914
Option A. Friedrich's ataxia Friedrich's ataxia is a genetic disorder characterized by progressive damag
e to the nervous system. It is not associated with the presence of Maltese cross crystals in the urine. T
herefore, this option is incorrect.
Option B. Felty syndrome Felty syndrome is a rare complication of rheumatoid arthritis characterized b
y the triad of rheumatoid arthritis, an enlarged spleen, and a low white blood cell count. It is not associa
ted with the presence of Maltese cross crystals in the urine. Therefore, this option is incorrect.
Option C. Fanconi anaemia Fanconi anaemia is a rare inherited disorder that affects bone marrow and
is associated with various physical abnormalities. It is not associated with the presence of Maltese cros
s crystals in the urine. Therefore, this option is incorrect.

Solution for Question 17:


Correct Option -D
• Lupus nephritis is kidney inflammation that occurs as a result of systemic lupus erythematosus (SLE),
an autoimmune disease. The immunofluorescence findings in lupus nephritis demonstrate a variety of
patterns, including the presence of a "full house" pattern. This pattern shows granular deposits of IgG,
IgA, IgM, C3, and C1q throughout the glomeruli. The "full house" pattern indicates the deposition of
immune complexes containing multiple types of antibodies and complement components.
Incorrect Options:
Option A.
• In FSGS, the immunofluorescence findings typically show nonspecific or minimal changes. FSGS is
characterized by the scarring and sclerosis of certain segments (or portions) of the glomeruli in the
kidney. Immunofluorescence may not reveal any specific immune deposits or show only mild or
nonspecific deposits.
Option B.
• PSGN is a type of glomerulonephritis that occurs after an infection with certain strains of
Streptococcus bacteria, such as strep throat or skin infections. In PSGN, the immunofluorescence
findings show granular deposits of IgG, IgM, and C3 along the glomerular basement membrane (GBM).
These immune deposits represent the immune complex deposition caused by the immune response to
the streptococcal infection.
Option C.
• Goodpasture syndrome is an autoimmune disorder characterized by the presence of antibodies
against the glomerular basement membrane (anti-GBM antibodies). The immunofluorescence findings
in Goodpasture syndrome reveal linear deposits of IgG along the GBM. These linear deposits
correspond to the binding of anti-GBM antibodies to the GBM.

Solution for Question 18:


Correct Option B - Malakoplakia:
• The histopathology image shows Michaelis-Gutmann bodies, which are pathognomonic for
malakoplakia.

Page 15

915
• The histiocyte accumulations in malakoplakia that are seen on cystoscopy are filled with germs and
indigestible detritus.
• The increase of macrophages with an aberrant bacterial metabolism characterizes the rare chronic
inflammatory condition known as malakoplakia.
• It can result in symptoms including dysuria, frequency, and hematuria and is most frequently detected
in the urinary tract.
Incorrect Options:
Options A, C, and D are incorrect.

Solution for Question 19:


Correct Option B - Subepithelial deposits:
• The child presented with complaints of cola-coloured urine, puffiness of the eyes, and raised
creatinine, which is suggestive of post-streptococcal glomerulonephritis.
• In post-streptococcal glomerulonephritis, on electron microscopy, subepithelial deposits are seen.
Incorrect Options:
Option A - Subendothelial deposits: Subendothelial deposits are seen on electron microscopy in memb
ranoproliferative glomerulonephritis.
Option C - Crescent formation: Crescent formation is seen on light microscopy in rapidly progressive gl
omerulonephritis.
Option D - Mesangial deposits: Mesangial deposits are seen in IgA nephropathy and IgA vasculitis.

Solution for Question 20:


Correct Option B - Membranous nephropathy:
• The patient's presentation with facial puffiness, frothy urine, hypertension, a history of hepatitis B
infection, microscopic hematuria, and a kidney biopsy showing a spike and dome pattern is suggestive
of membranous glomerulonephropathy.
Incorrect Options:
Options A, C, and D are incorrect.

Page 16

916
Female Genital System
1. Which of the options listed below is not included as a constituent of Papanicolaou stain?
A. Light green SF
B. Orange G
C. Eosin B
D. Hematoxylin
----------------------------------------
2. What's the probable causative organism for flesh-colored papillary lesions on the vulva with the given
histopathological findings in a 28-year-old female with itching and discomfort?

A. HPV-16
B. HPV-33
C. HPV-18
D. HPV-11
----------------------------------------
3. Which of the following statements is true regarding a 79-year-old postmenopausal woman who
presents with vulvar pruritus and discomfort and an areas of leukoplakia on the vulva?

A. Often seen in individuals with longstanding lichen sclerosus.


B. It is not related to HPV.
C. There is a high frequency of TP53 mutations in this case

917
D. All the above.
----------------------------------------
4. Which of the following markers is expected to turn positive in a 62-year-old postmenopausal woman
who presents with persistent pruritus and erythematous changes on her vulva and well-demarcated,
red, velvety plaques?.

A. Cytokeratin 20
B. Cytokeratin 7
C. Cytokeratin 18
D. Cytokeratin 5
----------------------------------------
5. A 4-year-old girl presents with a visible mass in the vaginal canal. On examination, a polypoid mass
is noted protruding from the vaginal introitus. The mass is removed and its gross appearance is shown
in the image below. What are the characteristic histopathological findings in a patient with this
condition?

A. Tumor cells are small and have oval nuclei, cells resemble tennis rackets.
B. Cells have dark blue “Raisinoid nuclei” with perinuclear halos.
C. Will show polyhedral vesicular tumor cells with round nuclei and clear cytoplasm.
D. Tumor cells arranged in sheets punctuated by small follicle-like structures.
----------------------------------------
6. What part of the cervix is affected in a 28-year-old woman with multiple sexual partners who takes
oral contraceptives with high-grade squamous epithelial dysplasia and subsequent CIN grade II
confirmed by colposcopy biopsy?

Page 2

918
(or)
Which of the following best describes the portion of the cervix that is affected in Cervical intraepithelial
neoplasia (CIN) grade II?
A. Near full-thickness epithelial involvement
B. Apical 33% of the epithelium
C. Basal 67% of the epithelium
D. Basal 33% of the epithelium
----------------------------------------
7. What microorganism is likely increased in a 26-year-old woman with a thin, homogenous vaginal
discharge, fishy odor, and "clue cells" on Pap smear?
A. Staphylococcus aureus
B. Neisseria gonorrhoeae
C. Candida albicans
D. Gardnerella vaginalis
----------------------------------------
8. What is the normal histology of the cervical region where a Pap smear shows cells with enlarged
nuclei and perinuclear halos in a sexually active 30-year-old woman with no complaints and multiple
partners, but no barrier contraception use?
(or)
Which of the following best describes the normal histology of the region of the cervix from where the
sample for PAP smear is obtained?
A. Cuboidal epithelial cells
B. Transition from squamous to columnar epithelial cells
C. Squamous epithelial cells
D. Transition from squamous to cuboidal epithelial cell
----------------------------------------
9. What's the likely cause of a thick whitish vaginal discharge and significant vaginal itching in a
56-year-old diabetic woman recently treated with a 2-week course of antibiotics for a skin infection?
A. C. granulomatis
B. C. albicans
C. Chancroid
D. Neisseria meningitidis
----------------------------------------
10. What's the probable diagnosis for a 26-year-old sexually active woman (with two male partners)
who has regular menstrual cycles, uses oral contraceptives, has no complaints, and the given Pap
smear results on a routine health checkup?

Page 3

919
A. Low-grade squamous intraepithelial lesion
B. High-grade squamous intraepithelial lesion
C. Squamous cell carcinoma
D. Adenocarcinoma
----------------------------------------
11. Which of the following is not a risk factor for endometrial carcinoma?
A. Obesity
B. Increase parity
C. Increasing age
D. Tamoxifen
----------------------------------------
12. The chromosome number of partial hydatidiform mole is?
A. 46XX
B. 46XY
C. 69XXX
D. None of the above
----------------------------------------
13. What's the likely diagnosis for a 58-year-old woman with postmenopausal bleeding, a large pelvic
tumor with multiple locules, and a histopathological examination showing Call-Exner bodies?
(or)
When histology reveals Call-Exner bodies, what is the diagnosis?
A. Granulosa cell tumor
B. Leydig cell tumor
C. Yolk-sac tumor
D. Embryonal rhabdomyosarcoma
----------------------------------------
14. What is the likely diagnosis for a 27-year-old female presenting with inability to conceive despite an
active effort for 2 years, regular but increasingly painful menstrual cycles, pain during intercourse, focal

Page 4

920
tenderness on vaginal examination, immobility of the cervix, and a left adnexal mass, 6cm nodule on
the surface of the ovary in transvaginal ultrasound and laparoscopy?
A. Adenomyosis
B. Endometrial hyperplasia
C. Endometriosis
D. Leiomyoma
----------------------------------------
15. What's the likely diagnosis for a 21-year-old female with vague abdominal pain, a 12-week size firm
mass on the left side, not freely mobile, with transvaginal ultrasound showing a 10 x 12 cm ovarian
mass with uniform echogenicity, scattered calcifications, fat, fluid, and no flow on color Doppler?
A. Granulosa cell tumour
B. Brenner tumour
C. Serous cystadenoma
D. Dermoid cyst
----------------------------------------
16. Which of the following is the most common primary tumor of the ovary?
A. Dysgerminoma
B. Surface epithelial tumour
C. Teratoma
D. Endodermal sinus tumor
----------------------------------------

Correct Answers
Question Correct Answer

Question 1 3
Question 2 4
Question 3 4
Question 4 2
Question 5 1
Question 6 3
Question 7 4
Question 8 2
Question 9 2
Question 10 1
Question 11 2
Question 12 3

Page 5

921
Question 13 1
Question 14 3
Question 15 4
Question 16 2

Solution for Question 1:


Correct option:
Option C: Eosin B:
• Eosin B is a counterstain that stains the cytoplasmic components of cells pink or red. It provides
additional contrast and helps distinguish cell boundaries.
• Papanicolaou stain, commonly known as the Pap stain, is a special stain used in cytology to
differentiate cells and identify abnormalities, particularly in Pap smears for cervical cancer screening.
• In the context of the Pap stain, it is not used; instead, the stain relies on the specific combination of
light green SF, orange G, and Hematoxylin for cell staining.
Incorrect Option:
Option A: Light green SF: This is a
counterstain used in the Pap stain. It stains the cytoplasm of cells and helps visualize cell morphology.
Option B: Orange G: Orange G is another counterstain used in the Pap stain. It stains the cytoplasm as
well and enhances the contrast of the stained cells.
Option D: Hematoxylin: Hematoxylin is a component of the Papanicolaou stain. Hematoxylin is a nucle
ar stain that is often used in combination with eosin (Hematoxylin and Eosin stain, or H&E; stain) to stai
n cell nuclei blue-purple.

Solution for Question 2:


Correct Option D - HPV-11:
• Given the clinical presentation of multiple flesh-colored papillary lesions on the vulva, the most likely
causative organism associated with such lesions is Human Papillomavirus (HPV).

• The given slide shows features of condyloma acuminatum (ie hyperkeratosis, acanthosis, and
koilocytosis) the strains associated with genital warts include HPV-6 and HPV-11. Therefore, the most
appropriate answer would be D) HPV-11

Page 6

922
Incorrect Options:
Option A - HPV-16:
• HPV-16 is a high-risk strain associated with an increased risk of cervical cancer and other
malignancies. It is not typically associated with the development of genital warts, as described in the
clinical scenario.
Option B - HPV-33:
• HPV-33 is also a high-risk strain associated with an increased risk of cervical cancer. It is not
commonly associated with the development of genital warts, which are usually caused by low-risk HPV
types.
Option C - HPV-18:
• Similar to HPV-16 and HPV-33, HPV-18 is a high-risk strain associated with an increased risk of
cervical and other anogenital cancers. It is not a common cause of genital warts.

Solution for Question 3:


Correct Option D - All the above:
• The given scenario and the image shows an invasive keratinizing squamous cell carcinoma,
containing nests of malignant squamous epithelium with prominent keratin pearls.

• All the given statements are true regarding Differentiated VIN.

Page 7

923
Option A - Often seen in individuals with longstanding lichen sclerosus:
• The depicted lesion is consistent with Invasive keratinising squamous cell carcinoma that arise in
differentiated vulvar intraepithelial neoplasia (VIN), which is often associated with long-standing lichen
sclerosus.
Option B - It is not related to HPV:
• Keratinising squamous cell carcinoma occurs more often in individuals with long standing lichen
sclerosus or squamous cell hyperplasia and is not related to HPV. The peak occurrence is in the eighth
decade.It arises from precursor lesion referred to as Differentiated VIN or or VIN simplex.
Option C - There is a high frequency of TP53 mutations in this case:
• It is postulated that chronic epithelial irritation in lichen sclerosus or squamous cell hyperplasia may
contribute to a gradual evolution to a malignant phenotype, presumably through acquisition of driver
mutations in oncogenes and tumor suppressors. In line with this idea, some investigators have reported
a high frequency of TP53 mutations in differentiated VIN.

Solution for Question 4:


Correct Option B - Cytokeratin 7:
• Vulvar Paget's disease is a rare neoplastic condition affecting the vulvar skin, characterized by
pruritus and erythematous plaques or eczema-like chronic skin lesions associated with itchiness,
tenderness, burning sensation, and occasionally pain. Immunohistochemical staining for certain
cytokeratins helps in confirming the diagnosis. In the case of Vulvar Paget's disease, the neoplastic
cells typically express Cytokeratin 7 (CK7).
• Positive staining for CK7 is a characteristic feature of Paget's cells in the epidermis, and it aids in
differentiating Paget's disease from other skin conditions. This marker is often used in pathology to
identify the presence of glandular or Paget cells in various tissues. As in the image below
immunostaining for Ck7 highlights the intraepidermal Paget cells.

Page 8

924
Incorrect Options:
Option A - Cytokeratin 20:
• CK20 is more commonly associated with gastrointestinal and urothelial epithelia. It is not typically
expressed in Paget's cells of the vulva.
Option C - Cytokeratin 18:
• CK18 is a low molecular weight cytokeratin often found in simple epithelia. While it may be expressed
in various tissues, it is not specific to Paget's disease.
Option D - Cytokeratin 5:
• CK5 is commonly expressed in basal and myoepithelial cells, but it is not typically associated with
Paget's cells.

Solution for Question 5:


Correct Option A - Tumor cells are small and have oval nuclei, cells resemble tennis rackets:
• The clinical presentation and image given suggest the diagnosis of embryonal Rhabdomyosarcoma.
• As in the given scenario, presentation at an age younger than 5 years, the mass appearing polypoid
rounded and bulky giving the impression of a cluster of grapes is further supportive of the diagnosis.
• Histology is shown below:

Page 9

925
• Rhabdomyoblasts with Cross-striations in an Embryonal Rhabdomyosarcoma (RMS): Cross-striations
are usually seen in better-differentiated embryonal RMS with a prominent spindle cell component. Most
elongated cells in this image have cross-striations.
• Generally, there is an admixture of small undifferentiated cells and better-differentiated
rhabdomyoblasts. The rhabdomyoblasts range from delicate spindle-shaped cells to large straps,
ribbon, tadpoles, or racket-shaped cells with deeply eosinophilic cytoplasm. The nuclei are large,
round, and vesicular with a prominent nucleolus. The cross-striations in neoplastic cells are arranged
irregularly and traverse only part of the cell in contrast to those seen in entrapped normal muscle cells.
Incorrect Options:
Option B - Cells have dark blue “Raisinoid nuclei” with perinuclear halos:
• This describes the Histology of cells in Condyloma acuminata. Typically described as Koilocytic
Atypia.
Option C - Will show polyhedral vesicular tumor cells with round nuclei and clear cytoplasm:
• This describes the histology of a dysgerminoma. Presence of large vesicular cells having a clear
cytoplasm, well-defined boundaries, and centrally placed regular nuclei.
Option D - Tumor cells arranged in sheets punctuated by small follicle-like structures:
• This is a characteristic histology seen in GRANULOSA CELL TUMOR, describing a Call-Exner body.

Solution for Question 6:


Correct Option C - Basal 67% of the epithelium:
• When assessing a cervical lesion, it is important to determine the degree of dysplasia and the depth of
involvement.
• A Pap smear can determine the former, whereas a colposcopy with biopsy can determine the latter.
• There are two main grades of dysplasia: low-grade squamous intraepithelial lesions (LSIL )
correspond to low-grade dysplasia, whereas high-grade squamous intraepithelial lesions (HSIL )

Page 10

926
correspond to moderate or high-grade dysplasia.
• low-grade squamous intraepithelial lesions (LSIL ) correspond to low-grade dysplasia,
• whereas high-grade squamous intraepithelial lesions (HSIL ) correspond to moderate or high-grade
dysplasia.
• If a Pap smear shows abnormal results, a biopsy can be subsequently performed to determine the
depth of involvement.
• low-grade squamous intraepithelial lesions (LSIL ) correspond to low-grade dysplasia,
• whereas high-grade squamous intraepithelial lesions (HSIL ) correspond to moderate or high-grade
dysplasia.
Incorrect Options:
Option A - Near full-thickness epithelial involvement:
• Near full-thickness epithelial involvement is seen in CIN III.
• However, this patient has CIN II, and only two-thirds of the epithelium is involved.
Option B - Apical 33% of the epithelium:
• Patients with CIN II have a disease that affects only the basal two-thirds of the cervical epithelium
because the lesion originates from the basal side (i.e., the portion facing the basement membrane ),
thus sparing the apical one-third of the epithelium.
Option D - Basal 33% of the epithelium:
• Patients with cervical intraepithelial neoplasia grade 1 ( CIN I) have a disease that affects only the
basal one-third of the cervical epithelium.
• However, this patient has CIN II, which affects the basal two-thirds of the epithelium.

Solution for Question 7:


Correct Option D - Gardnerella vaginalis:
• The most frequent cause of vaginal discharge is bacterial vaginosis.
• In bacterial vaginosis, there's a decrease in the number of normal hydrogen peroxide-producing
Lactobacilli, which are typically found in a healthy vaginal environment, and an overgrowth of anaerobic
bacteria.
• A vaginal fluid pH value of 4–4.5 or less typically indicates a healthy, acidic environment, which is
maintained by the presence of Lactobacilli bacteria. When the pH value rises above 4.5, it can be
associated with various types of vaginitis, including bacterial vaginosis (BV).
• A fishy amine odor is produced when 10% potassium hydroxide is added (Whiff test).
• On the Pap smear, there are usually more Gardnerella vaginalis organisms and a significant number
of "clue cells" (cells attached to the vaginal epithelial cells).
• Another source of vaginosis is T. vaginalis, although a wet mount reveals flagellated motile
organisms.
Incorrect Options:
Option A - Staphylococcus aureus:

Page 11

927
• It has long been known that S. aureus is one of the most significant germs that harm humans. It
mainly contributes to skin and soft tissue infections such as cellulitis, furuncles, and abscesses (boils).
However, the majority of staph infections are not dangerous.
• It mainly contributes to skin and soft tissue infections such as cellulitis, furuncles, and abscesses
(boils).
• S. aureus can cause serious infections such as bloodstream infections, pneumonia, or bone and joint
infections, even though most staph infections are not serious.
• Staphylococcus is generally not a common causative agent of vaginitis. Vaginitis is most commonly
associated with bacterial vaginosis, candidiasis (yeast infection), and trichomoniasis. Staphylococcus
aureus, a type of Staphylococcus bacteria, can be part of the normal vaginal flora but is not typically
considered a primary pathogen in causing vaginitis.
Option B - Neisseria gonorrhea:
• Neisseria gonorrhea is a common etiological agent of pelvic inflammatory disease
• The Neisseria gonorrhea bacteria is the source of the sexually transmitted illness (STD) known as
gonorrhea.
• N. gonorrhea infection impacts the urethra in both men and women, and in women, it also affects the
cervix, uterus, and fallopian tubes.
• A common regimen applied is Ceftriaxone 500 mg IM given plus azithromycin 1 g given orally as pills
(uncomplicated gonorrhea)
• Alternative regimens are available when ceftriaxone cannot treat urogenital or rectal gonorrhea.
Option C - Candida albicans:
• Candida-induced vulvovaginitis is marked by dense white vaginal discharge.
• Candida albicans, an opportunistic yeast prevalent in the gut flora, is detected in the gastrointestinal
tract and oral cavity of 40–60% of healthy adults.
• This fungus, responsible for most vaginal yeast infections, disrupts the natural balance between yeast
(including candida) and bacteria in the vagina, where lactobacillus typically prevents yeast overgrowth.

Solution for Question 8:


Correct Option B - Transition from squamous to columnar epithelial cells:
• It is at the transformation zone where cellular metaplasia and dysplasia most often take place, and
therefore a Pap smear specifically samples tissue from the squamocolumnar junction.
• The cervical canal can be divided into three regions. The endocervix is closer to the uterus and is
lined by columnar epithelial cells. The ectocervix is continuous with the vagina, and it is lined by mature
squamous epithelial cells. Where the squamous epithelium of the ectocervix and the columnar
epithelium of the endocervix meet, there is a line called the squamocolumnar junction (also termed the
transformation zone).
• The endocervix is closer to the uterus and is lined by columnar epithelial cells.
• The ectocervix is continuous with the vagina, and it is lined by mature squamous epithelial cells.
• Where the squamous epithelium of the ectocervix and the columnar epithelium of the endocervix
meet, there is a line called the squamocolumnar junction (also termed the transformation zone).

Page 12

928
• Metaplasia occurs when a stressor, such as the acidity of the vaginal canal during reproductive years,
causes the replacement of one cell type by another. In the cervix, the columnar epithelium is replaced
by squamous epithelium.
• Metaplasia is reversible once the stressor is removed.
• In contrast, dysplasia occurs when fully differentiated cells are replaced by immature cells with varying
shapes and nuclear morphology, as demonstrated by the koilocytes in this question stem, with their
enlarged nuclei and perinuclear halos. Mild dysplasia is reversible. However, moderate or severe
dysplasia is irreversible, and if not monitored, it can progress to cervical cancer.

• The endocervix is closer to the uterus and is lined by columnar epithelial cells.
• The ectocervix is continuous with the vagina, and it is lined by mature squamous epithelial cells.
• Where the squamous epithelium of the ectocervix and the columnar epithelium of the endocervix
meet, there is a line called the squamocolumnar junction (also termed the transformation zone).
Incorrect Options:
Option A - Cuboidal epithelial cells:
• Cuboidal epithelial cells are often found in organs with secretory, absorptive, or excretory functions.
• These include the pancreas, kidneys, and parotid glands.
• Simple cuboidal epithelium is also present on the outer surface of the ovaries.
• However, cuboidal epithelial cells are not present in the cervix.
Option C - Squamous epithelial cells:
• Squamous epithelial cells are found in the ectocervix, which is continuous with the vagina.
• A Pap smear is obtained at the transformation zone, where dysplasia most frequently happens.
Option D - Transition from squamous to cuboidal epithelial cell:
• Cuboidal epithelial cells are not present in the cervix.
• Therefore, there would be no region in the cervix wherein squamous epithelial cells are replaced by
cuboidal epithelial cells.

Solution for Question 9:


Correct Option B - C. albicans:
• C. albicans is a major cause of vulvovaginitis. The opportunistic pathogenic yeast Candida albicans is
a common component of the human gut flora. It is detected in the oral cavity and gastrointestinal tract
of approximately 40% to 60% of individuals in a state of good health.
• The patient presents with thick white vaginal discharge.
• The majority of vaginal yeast infections are caused by Candida albicans.
• A healthy balance of yeast, including candida, and bacteria naturally exists in the vagina.
• Several bacteria, including Lactobacillus, inhibit the overgrowth of yeast. Nevertheless, this delicate
equilibrium may be disrupted.

Page 13

929
Incorrect Options:
Option A - C. granulomatis:
• C. granulomatis causes granuloma inguinale. Chancroid results in painful genital ulcers caused by the
organism H. ducreyi.
• The genital ulcerative disease granuloma inguinale (donovanosis) is brought on by the intracellular
gram-negative bacterium Klebsiella granulomatous (formerly known as Calymmatobacterium
granulomatis).
• One or more painless lumps, commonly located in the genitalia, anal region, or groin, are among the
signs and symptoms of donovanosis. The lump or lumps gradually enlarge and eventually ulcerate.
• These sores are often beefy-red in color, quickly bleed, and have a rolled edge. The sore may
occasionally smell unpleasant.
Option C - Chancroid:
• Chancroid gives rise to lesions on the genitalia, presenting as a soft chancre or an ulcer characterized
by irregular edges. These sores may induce discomfort and exhibit purulent discharge.
• Chancroid is caused by the Haemophilus ducreyi, resulting in painful, superficial ulcers, often with
regional lymphadenopathy.
• It is characterized by painful necrotizing genital ulcers that may accompany inguinal
lymphadenopathy. It is a highly contagious but curable disease.
• No laboratory testing can immediately confirm the diagnosis of chancroid. Chancroid can only be
diagnosed with certainty by isolating H ducreyi on a specialized medium. However, these assays are
only commonly available in some centres.
Option D - Neisseria meningitidis:
• N. meningitidis can cause meningitis.
• Neisseria meningitidis bacteria are the causative agents of meningococcal disease. Approximately
one in ten individuals harbor these microorganisms in the posterior nasopharynx without exhibiting any
signs of illness.
• Neisseria meningitidis septicemia triggers disseminated intravascular coagulation (DIC), causing
compromised blood vessel integrity and subsequent hemorrhage in dermal and visceral tissues.
Clinical symptoms may include chills and fever.

Solution for Question 10:


Correct Option A - Low-grade squamous intraepithelial lesion:
• Low-grade squamous intraepithelial lesion (LSIL) is a pre-malignant cervical lesion.
• Human papillomavirus (HPV) is a risk factor for LSIL and cervical carcinoma.
• The diagnosis of LSIL is based on the identification of nuclear atypia characterised by nuclear
enlargement, hyperchromasia (dark staining), coarse chromatin granules, and variation in nuclear size
and shape.
• Nuclear alterations with an associated perinuclear halo are termed koilocytic atypia.

Page 14

930
• The image above is a marked representation of features found in koilocytes.
• The risk of HSIL is 6.9 %, and the risk of cervical cancer is 1%

Incorrect Options:
Option B - High-grade squamous intraepithelial lesion:
• High-grade squamous intraepithelial lesion (HSIL) is characterised by a high nuclear-to-cytoplasm
ratio and hyperchromatic nuclei without nucleoli.
Option C - Squamous cell carcinoma:
• The basement membrane is invaded by the tumour cells in cervical carcinoma. Whereas in the
cervical intraepithelial lesions, the basement membrane is intact.
Option D - Adenocarcinoma:
• Adenocarcinoma presents with glandular cells. Cervical adenocarcinoma is uncommon. This patient’s
PAP smear result is consistent with LSIN.

Page 15

931
Solution for Question 11:
Correct Option B - Increase parity:
• Increase parity, is associated with a decreased risk of endometrial carcinoma. Women who have had
children (especially multiple pregnancies) have a lower risk, possibly due to the changes in hormonal
exposure during pregnancy.
Incorrect Options:
Option A - Obesity:
• Obesity is a well-established risk factor for endometrial carcinoma. Adipose tissue produces estrogen,
and excess estrogen exposure without the protective effect of progesterone can lead to endometrial
hyperplasia and, eventually, carcinoma.
Option C - Increasing age:
• It is a well-known risk factor for endometrial carcinoma.
Option D - Tamoxifen:
• Tamoxifen, a selective estrogen receptor modulator (SERM) used in the treatment of breast cancer, is
associated with an increased risk of endometrial carcinoma if used for more than 5 years. Tamoxifen
acts as an estrogen agonist on the endometrium, leading to increased proliferation and a higher risk of
developing endometrial hyperplasia and carcinoma.

Solution for Question 12:


Correct Option C - 69XXX:
• Complete mole results from the fertilization of an egg that has lost its female chromosomes, and as a
result, the genetic material is derived from the paternal gamete. 99%: 46 XX karyotypes, 10%: 46 XY
karyotype.
• Partial moles result from the fertilization of an egg with two sperms.
• In Partial moles, the karyotype is triploid (e.g., 69, XXY) or occasionally tetraploid (92, XXXY).

Page 16

932
Incorrect Options:
Option A - 46XX:
• Complete mole has karyotype 46XX, and both chromosomes are of paternal origin.
Option B - 46XY:
• The 46XY karyotype is rare. However, it is present in complete molar pregnancy, not a partial mole.
Option D - None of the above

Solution for Question 13:


Correct Option A - Granulosa cell tumor:
• The patient’s clinical symptoms and pathological findings of the lesion are diagnostic of granulosa cell
tumors of the ovary.
• Granulosa cell tumors are the most common malignant sex cord-stromal neoplasms of the ovary.
• Granulosa cell tumors usually occur in women of the age group 50 to 55 years (like this patient).
• The clinical symptoms of granulosa cell tumors are associated with excessive secretion of estrogen
and/or progesterone. These hormones induce endometrial hyperplasia and result in menstrual
irregularities such as heavy menstrual bleeding and post-menopausal bleeding (as in this patient).
• These tumors are also associated with an increased risk of endometrial cancer of the uterus.
• The histopathological hallmark of the diagnosis of granulosa cell tumors is Call-Exner bodies.
• Call-Exner bodies are a cluster of granulosa cells that surround a central cavity with eosinophilic
secretions, as shown below:

Page 17

933
• The presence of the Call-Exner bodies on the biopsy specimen of this patient confirms granulosa cell
tumours of the ovary.
Incorrect Options:
Option B - Leydig cell tumour:
• Leydig cell tumours secrete androgens which can cause virilization and acne.
• These tumours can also cause estrogenic symptoms like heavy menstrual bleeding.
• The histopathology of Leydig cell tumours is characterized by the presence of seminiferous-like
tubules lined by Sertoli cells and Reinke crystals.
• Call-Exner bodies are not seen in Leydig cell tumours.
Option C - Yolk-sac tumour:
• The yolk-sac tumours of the ovary usually occur in children and adolescent age groups (rather than
this patient of the middle-aged group).
• The histopathology of yolk-sac tumours is significant for Schiller-Duval bodies (structures resembling
primitive glomeruli) rather than the Call-Exner bodies.
Option D - Embryonal rhabdomyosarcoma:
• Embryonal rhabdomyosarcoma occurs in the pediatric age group, unlike this patient.
• These tumors can contain Tennis-Racket cells, but Call-Exner bodies are not characteristic of these
tumors.

Solution for Question 14:


Correct Option C - Endometriosis:
• Endometriosis is characterized by endometrial glands and stroma outside the uterine cavity.
• Ovaries and uterine ligaments are one of the most common sites for endometriosis. Endometrial
tissue in the ovary bleeds cyclically and results in hematoma surrounded by ovarian parenchyma.

Page 18

934
Endometriomas have fibrotic walls and surface adhesions filled with syrup-like chocolate-colored
material. These findings are consistent with the diagnostic laparoscopy given in this scenario.
• Endometrial tissue in the ovary bleeds cyclically and results in hematoma surrounded by ovarian
parenchyma.
• Endometriomas have fibrotic walls and surface adhesions filled with syrup-like chocolate-colored
material.
• These findings are consistent with the diagnostic laparoscopy given in this scenario.
• Usual signs and symptoms of endometriosis include Pelvic pain Severe dysmenorrhea (Painful
menses) Dyspareunia (Painful intercourse) Infertility
• Pelvic pain
• Severe dysmenorrhea (Painful menses)
• Dyspareunia (Painful intercourse)
• Infertility
• Endometrial tissue in the ovary bleeds cyclically and results in hematoma surrounded by ovarian
parenchyma.
• Endometriomas have fibrotic walls and surface adhesions filled with syrup-like chocolate-colored
material.
• These findings are consistent with the diagnostic laparoscopy given in this scenario.
• Pelvic pain
• Severe dysmenorrhea (Painful menses)
• Dyspareunia (Painful intercourse)
• Infertility

• Transvaginal ultrasonogram of a 6 cm (diameter) left ovarian endometrioma and its corresponding


laparoscopic image.
Incorrect Options:

Page 19

935
Option A - Adenomyosis:
• Adenomyosis is the presence of the endometrial glands within the uterine cavity's myometrium,
characterized by very painful menstrual periods and heavy menstrual bleeding. It is not associated with
painful intercourse or subfertility.
• It is not associated with painful intercourse or subfertility.
• It is not associated with painful intercourse or subfertility.
Option B - Endometrial hyperplasia:
• The proliferation of endometrial glands is termed endometrial hyperplasia and is characterized by
abnormal uterine bleeding out of regular periods and less often by post-coital bleeding.
Option D - Leiomyoma:
• The presence of whorls of smooth muscles and fibroblasts in the uterine myometrium is termed
leiomyoma or uterine fibroids.
• Uterine fibroids are characterized by heavy menstrual bleeding and subfertility but are not associated
with dysmenorrhea or dyspareunia.

Solution for Question 15:


Correct Option D - Dermoid Cyst:
• Radiographic evidence of an ovarian mass with mixed echogenicity, calcifications and no flow on
colour dopplers is consistent with a mature teratoma (also known as a dermoid cyst).
• Mature cystic teratomas are composed of mature differentiated elements, including tissue ectodermal
(e.g., skin, hair follicles, sebaceous glands), mesodermal (e.g., muscle, urinary), and endodermal (e.g.,
lung, gastrointestinal) origin. Most patients with teratoma are asymptomatic. However, the symptoms
depend on the size of the teratoma and include Abdominal fullness and pressure Gastrointestinal
discomfort Symptoms related to rupture of the dermoid cyst
• Most patients with teratoma are asymptomatic. However, the symptoms depend on the size of the
teratoma and include
• Abdominal fullness and pressure
• Gastrointestinal discomfort
• Symptoms related to rupture of the dermoid cyst
• Radiographic evidence of dermoid cyst includes Markedly hyperechoic nodule within the mass
Uniformly hyperechoic echoes A fluid-fluid level Calcifications may be present Absence of dopplers flow
• Markedly hyperechoic nodule within the mass
• Uniformly hyperechoic echoes
• A fluid-fluid level
• Calcifications may be present
• Absence of dopplers flow

Page 20

936
• Transvaginal ultrasound of a mature teratoma in the ovary containing heterogeneous contents, which
often shadow. It is round and well-circumscribed.
• This image contains the "ovarian crescent" sign, a rim of a normal-appearing ovary with follicles.
• These features suggest a benign cyst.
• Most patients with teratoma are asymptomatic. However, the symptoms depend on the size of the
teratoma and include
• Abdominal fullness and pressure
• Gastrointestinal discomfort
• Symptoms related to rupture of the dermoid cyst
• Markedly hyperechoic nodule within the mass
• Uniformly hyperechoic echoes
• A fluid-fluid level
• Calcifications may be present
• Absence of dopplers flow

Incorrect Options:
Option A - Granulosa Cell Tumor:
• Granulosa cell tumours grow in the tissues around the ovary and produce high levels of estrogen,
which may lead to abnormal bleeding and irregular periods in addition to the mass effects of the
tumour.
Option B - Brenner Tumor:
• Brenner tumours are benign, large, asymptomatic borderline transitional cell tumours of the ovary.
Brenner tumours usually occur in postmenopausal females and are associated with abnormal vaginal
bleeding.
• Brenner tumours usually occur in postmenopausal females and are associated with abnormal vaginal
bleeding.

Page 21

937
• Brenner tumours usually occur in postmenopausal females and are associated with abnormal vaginal
bleeding.
Option C - Serous Cystadenoma:
• Serous cystadenomas are serous ovarian tumours with a mean age of presentation between 40-60
years. These are commonly bilateral and associated with ovarian torsion, otherwise are asymptomatic.
USG reveals cysts with solid-appearing areas and irregularly thick septations and walls or septal/mural
nodules.
• These are commonly bilateral and associated with ovarian torsion, otherwise are asymptomatic.
• USG reveals cysts with solid-appearing areas and irregularly thick septations and walls or
septal/mural nodules.
• These are commonly bilateral and associated with ovarian torsion, otherwise are asymptomatic.
• USG reveals cysts with solid-appearing areas and irregularly thick septations and walls or
septal/mural nodules.

Solution for Question 16:


Correct Option B - Surface Epithelial Tumor:
• The surface epithelial tumour is the most common ovarian malignancy (95%) derived from epithelial
cells. Clinical signs and symptoms of ovarian epithelial carcinoma are the same as of any other ovarian
carcinoma and include. Bloating or abdominal distension Difficulty eating, nausea, anorexia, or early
satiety Pelvic or abdominal pain Urinary urgency and frequency Vaginal bleeding If not diagnosed
early, ovarian cancer may have an acute presentation, including. Ascites Bowel obstruction Pleural
effusion Venous thromboembolism The initial diagnosis is based on a suggestive history, convincing
physical examination, imaging, and serum tumor markers. Physical examination is consistent with the
presence of an adnexal mass. A pelvic or transvaginal ultrasound reveals an adnexal mass with a solid
component that is not hyperechoic and is often nodular or papillary. Doppler demonstration of flow in
the solid component, irregularly thick septations, and ascites. Elevated serum CA 125 levels,
accompanied by the abovementioned physical and imaging findings, point towards ovarian cancer.
Following a histopathological diagnosis of ovarian cancers, the histological subtypes of ovarian
epithelial carcinomas have been described. High-grade serous carcinoma (HGSC; 70 to 80 per cent)
Endometrioid carcinoma (10 per cent) Clear cell carcinomas (10 per cent) Mucinous carcinoma (3 per
cent) Low-grade serous carcinoma (LGSC; <5 per cent) Brenner tumours Sero-mucinous tumours
• Clinical signs and symptoms of ovarian epithelial carcinoma are the same as of any other ovarian
carcinoma and include. Bloating or abdominal distension Difficulty eating, nausea, anorexia, or early
satiety Pelvic or abdominal pain Urinary urgency and frequency Vaginal bleeding
• Bloating or abdominal distension
• Difficulty eating, nausea, anorexia, or early satiety
• Pelvic or abdominal pain
• Urinary urgency and frequency
• Vaginal bleeding
• If not diagnosed early, ovarian cancer may have an acute presentation, including. Ascites Bowel
obstruction Pleural effusion Venous thromboembolism
• Ascites

Page 22

938
• Bowel obstruction
• Pleural effusion
• Venous thromboembolism
• The initial diagnosis is based on a suggestive history, convincing physical examination, imaging, and
serum tumor markers. Physical examination is consistent with the presence of an adnexal mass. A
pelvic or transvaginal ultrasound reveals an adnexal mass with a solid component that is not
hyperechoic and is often nodular or papillary. Doppler demonstration of flow in the solid component,
irregularly thick septations, and ascites. Elevated serum CA 125 levels, accompanied by the
abovementioned physical and imaging findings, point towards ovarian cancer.
• Physical examination is consistent with the presence of an adnexal mass.
• A pelvic or transvaginal ultrasound reveals an adnexal mass with a solid component that is not
hyperechoic and is often nodular or papillary. Doppler demonstration of flow in the solid component,
irregularly thick septations, and ascites.
• Elevated serum CA 125 levels, accompanied by the abovementioned physical and imaging findings,
point towards ovarian cancer.
• Following a histopathological diagnosis of ovarian cancers, the histological subtypes of ovarian
epithelial carcinomas have been described. High-grade serous carcinoma (HGSC; 70 to 80 per cent)
Endometrioid carcinoma (10 per cent) Clear cell carcinomas (10 per cent) Mucinous carcinoma (3 per
cent) Low-grade serous carcinoma (LGSC; <5 per cent) Brenner tumours Sero-mucinous tumours
• High-grade serous carcinoma (HGSC; 70 to 80 per cent)
• Endometrioid carcinoma (10 per cent)
• Clear cell carcinomas (10 per cent)
• Mucinous carcinoma (3 per cent)
• Low-grade serous carcinoma (LGSC; <5 per cent)
• Brenner tumours
• Sero-mucinous tumours
The surface epithelial tumour is the most common ovarian malignancy (95%) derived from epithelial cel
ls.
• Clinical signs and symptoms of ovarian epithelial carcinoma are the same as of any other ovarian
carcinoma and include. Bloating or abdominal distension Difficulty eating, nausea, anorexia, or early
satiety Pelvic or abdominal pain Urinary urgency and frequency Vaginal bleeding
• Bloating or abdominal distension
• Difficulty eating, nausea, anorexia, or early satiety
• Pelvic or abdominal pain
• Urinary urgency and frequency
• Vaginal bleeding
• If not diagnosed early, ovarian cancer may have an acute presentation, including. Ascites Bowel
obstruction Pleural effusion Venous thromboembolism
• Ascites
• Bowel obstruction

Page 23

939
• Pleural effusion
• Venous thromboembolism
• The initial diagnosis is based on a suggestive history, convincing physical examination, imaging, and
serum tumor markers. Physical examination is consistent with the presence of an adnexal mass. A
pelvic or transvaginal ultrasound reveals an adnexal mass with a solid component that is not
hyperechoic and is often nodular or papillary. Doppler demonstration of flow in the solid component,
irregularly thick septations, and ascites. Elevated serum CA 125 levels, accompanied by the
abovementioned physical and imaging findings, point towards ovarian cancer.
• Physical examination is consistent with the presence of an adnexal mass.
• A pelvic or transvaginal ultrasound reveals an adnexal mass with a solid component that is not
hyperechoic and is often nodular or papillary. Doppler demonstration of flow in the solid component,
irregularly thick septations, and ascites.
• Elevated serum CA 125 levels, accompanied by the abovementioned physical and imaging findings,
point towards ovarian cancer.
• Following a histopathological diagnosis of ovarian cancers, the histological subtypes of ovarian
epithelial carcinomas have been described. High-grade serous carcinoma (HGSC; 70 to 80 per cent)
Endometrioid carcinoma (10 per cent) Clear cell carcinomas (10 per cent) Mucinous carcinoma (3 per
cent) Low-grade serous carcinoma (LGSC; <5 per cent) Brenner tumours Sero-mucinous tumours
• High-grade serous carcinoma (HGSC; 70 to 80 per cent)
• Endometrioid carcinoma (10 per cent)
• Clear cell carcinomas (10 per cent)
• Mucinous carcinoma (3 per cent)
• Low-grade serous carcinoma (LGSC; <5 per cent)
• Brenner tumours
• Sero-mucinous tumours
Clinical signs and symptoms of ovarian epithelial carcinoma are the same as of any other ovarian carci
noma and include.
• Bloating or abdominal distension
• Difficulty eating, nausea, anorexia, or early satiety
• Pelvic or abdominal pain
• Urinary urgency and frequency
• Vaginal bleeding
Bloating or abdominal distension
Difficulty eating, nausea, anorexia, or early satiety
Pelvic or abdominal pain
Urinary urgency and frequency
Vaginal bleeding
If not diagnosed early, ovarian cancer may have an acute presentation, including.
• Ascites

Page 24

940
• Bowel obstruction
• Pleural effusion
• Venous thromboembolism
Ascites
Bowel obstruction
Pleural effusion
Venous thromboembolism
The initial diagnosis is based on a
suggestive history, convincing physical examination, imaging, and serum tumor markers.
• Physical examination is consistent with the presence of an adnexal mass.
• A pelvic or transvaginal ultrasound reveals an adnexal mass with a solid component that is not
hyperechoic and is often nodular or papillary. Doppler demonstration of flow in the solid component,
irregularly thick septations, and ascites.
• Elevated serum CA 125 levels, accompanied by the abovementioned physical and imaging findings,
point towards ovarian cancer.
Physical examination is consistent with the presence of an adnexal mass.
A pelvic or transvaginal ultrasound reveals an adnexal mass with a solid component that is not hyperec
hoic and is often nodular or papillary. Doppler demonstration of flow in the solid component, irregularly
thick septations, and ascites.
Elevated serum CA 125 levels, accompanied by the abovementioned physical and imaging findings, po
int towards ovarian cancer.
Following a histopathological diagnosis of ovarian cancers, the histological subtypes of ovarian epitheli
al carcinomas have been described.
• High-grade serous carcinoma (HGSC; 70 to 80 per cent)
• Endometrioid carcinoma (10 per cent)
• Clear cell carcinomas (10 per cent)
• Mucinous carcinoma (3 per cent)
• Low-grade serous carcinoma (LGSC; <5 per cent)
• Brenner tumours
• Sero-mucinous tumours
High-grade serous carcinoma (HGSC; 70 to 80 per cent)
Endometrioid carcinoma (10 per cent)
Clear cell carcinomas (10 per cent)
Mucinous carcinoma (3 per cent)
Low-grade serous carcinoma (LGSC; <5 per cent)
Brenner tumours
Sero-mucinous tumours
Incorrect Options:

Page 25

941
Option A - Dysgerminoma:
• Dysgerminoma is a germ-cell ovarian tumour characterised by elevated serum alkaline phosphatase
and LDH levels. Histopathologically, it is composed of undifferentiated germ cells, large vesicular cells
with clear cytoplasm, well-defined cell boundaries, and centrally placed regular nuclei; the overall
appearance is sometimes described as resembling "fried eggs".
• Histopathologically, it is composed of undifferentiated germ cells, large vesicular cells with clear
cytoplasm, well-defined cell boundaries, and centrally placed regular nuclei; the overall appearance is
sometimes described as resembling "fried eggs".
Dysgerminoma is a
germ-cell ovarian tumour characterised by elevated serum alkaline phosphatase and LDH levels.
• Histopathologically, it is composed of undifferentiated germ cells, large vesicular cells with clear
cytoplasm, well-defined cell boundaries, and centrally placed regular nuclei; the overall appearance is
sometimes described as resembling "fried eggs".
Histopathologically, it is composed of undifferentiated germ cells, large vesicular cells with clear cytopl
asm, well-defined cell boundaries, and centrally placed regular nuclei; the overall appearance is someti
mes described as resembling "fried eggs".
Option C - Teratoma:
• Teratoma or dermoid cyst is a benign germ-cell tumour characterized by mature tissues arising from
more than one germ-cell layer.
Teratoma or dermoid cyst is a
benign germ-cell tumour characterized by mature tissues arising from more than one germ-cell layer.
Option D - Endodermal Sinus Tumor:
• Endodermal sinus tumor is also known as the yolk-sac tumor of the ovary and is characterized by
elevated serum AFP levels.
• Histologically, these neoplasms consist of tubules or spaces lined by single layers of flattened
cuboidal cells, reticular stroma, and scattered globules. Invaginated papillary structures with central
vessels (Schiller-Duval bodies) are found within some spaces.
Endodermal sinus tumor is also known as the yolk-sac tumor of the ovary and is characterized by elev
ated serum AFP levels.
Histologically, these neoplasms consist of tubules or spaces lined by single layers of flattened cuboidal
cells, reticular stroma, and scattered globules. Invaginated papillary structures with central vessels (Sc
hiller-Duval bodies) are found within some spaces.

Page 26

942
Male Genital System
1. What is the appropriate grade of tumor in a 60-year-old male with difficulty urinating, frequent
urination, blood in urine, an enlarged prostate with areas of induration with ill-defined margins on DRE,
and a PSA of 8.2 ng/ml, based on the following pathology findings? 75% 15% 10%

75%

15%

10%

A. Gleason Grade IV
B. (3+4) Gleason Grade III
C. (4+3) Gleason Grade III
D. Gleason Grade V
----------------------------------------
2. What is the likely diagnosis for a 30-year-old male with infertility, absent sperm in the ejaculate
during semen analysis, and ultrasonography revealing loculated cysts in the testes, with the given

943
biopsy findings?

A. Leydig cell tumor


B. Choriocarcinoma
C. Teratoma
D. Yolk sac tumor
----------------------------------------
3. What histological findings would you expect in a 25-year-old male patient with rapidly enlarging
testicular swelling, significant weight loss, and elevated β-HCG levels who underwent orchidectomy?

A. It shows Syncytiotrophoblasts and cytotrophoblasts.


B. It shows a central capillary with visceral and parietal layers of cells.
C. Cells are polyhedral with watery-appearing cytoplasm.
D. Cells in an alveolar or tubular pattern.
----------------------------------------
4. Which statement is accurate regarding a 2-year-old male child with a testicular mass and elevated
AFP levels and a biopsy revealing the given histological findings? Assertion: The tumor has structures
resembling endodermal sinuses called Glomerular Bodies. Reason: It has a characteristic unit that
consists of a mesodermal core with a central capillary and a visceral and parietal layer of cells.

Page 2

944
A. Both Assertion and Reason are true, and Reason is the correct explanation for Assertion.
B. Both Assertion and Reason are true, but Reason is not the correct explanation for Assertion.
C. Assertion is true, but Reason is false.
D. Both Assertion and Reason are false.
----------------------------------------
5. Which tumor marker is most likely to turn positive in a 32-year-old male with a radiosensitive tumor
spreading via lymphatics to the para-aortic lymph nodes, and the given gross specimen and histology
findings?

A. AFP(Alpha-Fetoprotein)
B. hCG(HumanChorionicGonadotropin)
C. CA125
D. PLAP (Placental Alkaline Phosphatase)

Page 3

945
----------------------------------------
6. Which testicular tumor is least likely to develop in a 25-year-old male with a history of cryptorchidism
and Germ cell Neoplasia In Situ detected during a routine examination?
A. Sertoli cell tumor
B. Spermatocytic Seminoma
C. Leydig cell tumor
D. Endodermal sinus tumor
----------------------------------------
7. A 32-year-old male presents with infertility concerns, and testicular tissue analysis is planned. Which
fixative would be most appropriate for preserving the morphology of germ cells and facilitating the
identification of any cellular arrests?
A. 10% Neutral Buffered Formalin (NBF)
B. Bouin's Fixative (Picric acid)
C. Frozen section without fixation
D. 70% Ethanol
----------------------------------------
8. Consider the following statements regarding verrucous carcinoma of penis, and choose the option. It
is an exophytic well-differentiated variant of squamous cell carcinoma. It is locally invasive, but rarely
metastasizes.
A. Both statements are true.
B. Statement 1 is true, but statement 2 is false.
C. Statement 1 is false, but statement 2 is true.
D. Both statements are false.
----------------------------------------
9. What is the correct statement regarding the diagnosis for a 25-year-old sexually active male with
non-pruritic reddish-brown papular lesions on the penile shaft and the given examination and
histological findings?

Page 4

946
A. It is a benign condition
B. The distinguishing point between the diagnosis and Bowen disease is the “younger age of the
patient”.
C. It is not usually involved with HPV infection.
D. It is not indistinguishable from Bowen disease.
----------------------------------------
10. What is the likely diagnosis of the primary tumor in a 40-year-old male with blood-stained sputum,
5kg weight loss in the last month, chest X-ray showing cannonball opacities, elevated serum hCG,
normal AFP, and CEA, normal colonoscopy and upper gastrointestinal endoscopy, no palpable
testicular mass and lung biopsy findings given below?

A. Endodermal sinus tumor


B. Seminoma
C. Choriocarcinoma
D. Teratoma
----------------------------------------
11. Which is the likely diagnosis for a 34-year-old man with gynecomastia, erectile dysfunction,
elevated testosterone, and estradiol levels, and a palpable testicular mass within the body of the right
testis showing histopathological features of closely packed cells with eosinophilic cytoplasm, bland
nuclei, small nucleoli, and eosinophilic cytoplasmic inclusions following orchiectomy?
A. Leydig cell tumour
B. Testicular choriocarcinomas

Page 5

947
C. Spermatocytic tumour
D. Yolk sac tumour
----------------------------------------
12. Which of the following is not true for seminomas?
A. Most common type of germ cell tumour
B. Serum HCG levels are elevated
C. Schiller-Duval bodies are commonly seen
D. Spermatocytic seminomas are usually slow growing
----------------------------------------
13. A 28-year-old male presents with a painless lump in his left testicle. On examination, the lump is
firm and non-tender, raising suspicion of testicular cancer. Which of the following is NOT used as a
tumor marker in suspected testicular malignancy?
(or)
Which of the following is NOT used as a tumor marker in suspected testicular malignancy?
A. AFP
B. LDH
C. hCG
D. CEA
----------------------------------------
14. Which of the following tumour markers is not useful in the immunohistochemistry analysis of
prostatic carcinoma?
A. NKX3.1
B. AMACR
C. TMPRSS2
D. Ki-67
----------------------------------------
15. What is the condition in which a 5-year-old boy presents with only one testis palpable in the scrotum
during a physical examination before starting elementary school, with a palpable mass detected in the
left inguinal region?
A. Cryptorchidism
B. Hydrocele
C. Orchitis
D. Varicocele
----------------------------------------
16. What's the likely diagnosis for a 3-year-old boy with a scrotal mass and elevated serum
α-fetoprotein levels?
A. Choriocarcinoma

Page 6

948
B. Endodermal sinus (yolk sac) tumour
C. Cryptorchidism
D. Teratoma
----------------------------------------

Correct Answers
Question Correct Answer

Question 1 1
Question 2 3
Question 3 1
Question 4 1
Question 5 4
Question 6 2
Question 7 2
Question 8 1
Question 9 2
Question 10 3
Question 11 1
Question 12 3
Question 13 4
Question 14 4
Question 15 1
Question 16 2

Solution for Question 1:


Correct Option A - Gleason Grade IV:
• From images, 75% show Distinct glands with infiltrative margins, 15% show cribriform pattern, and
10% show single cells.
Given that the Gleason pattern is scored as:
1. Small uniform glands
2. More stroma between the glands
3. Distinctly infiltrative margins
4. Irregular masses of neoplastic glands (a. Cribriform b. Hypernephroid)
5. Only occasional gland formation (a. Single cells b. Comedonecrosis)
The numbering for grading would be:
• (3+ 4 or 5).

Page 7

949
• Given that 5 is the greater number of the two, 3+5=8.
• Grade IV would be the designated grade.
Incorrect Options:
Option B - (3+4) Gleason Grade III:
• This option suggests a Gleason score of 7, which would be correct if pattern 5 were not present.
However, since pattern 5 is present in 10% of the tumor, the Gleason score should be higher.
Option C - (4+3) Gleason Grade III:
• Similar to option B, this suggests a Gleason score of 7, but in a different arrangement. Again, it
doesn't account for the presence of pattern 5.
Option D - Gleason Grade V:
• This option suggests a Gleason score of 10, which would require the presence of patterns adding up
to 10. In this case, the sum of the two most prevalent patterns is 8 (3 + 5).

Solution for Question 2:


Correct Option C - Teratoma:
The given slide shows:
• Multiple nodules of hyaline cartilage and squamous epithelium are surrounded by simple cysts lined
by columnar epithelium.
• Based on the provided histological findings of multiple nodules of hyaline cartilage and squamous
epithelium surrounded by simple cysts lined by columnar epithelium, the likely diagnosis is a teratoma.
• Teratomas are often multinodular, and their cut surface varies from multicystic to solid. Nodules of
translucent, white cartilage may be seen. The cysts may be filled with clear or mucoid fluid or
keratinous material.

Page 8

950
Incorrect Options:
Option A - Leydig cell tumor:
• Leydig cell tumors typically present with sheets or nests of polygonal cells with eosinophilic
cytoplasm. Rod-shaped Crystalloids of Reinke. They are not associated with the characteristic
histological features described in the question, such as multiple nodules of hyaline cartilage and
squamous epithelium.
Option B - Choriocarcinoma:
• Choriocarcinoma is characterized by the presence of syncytiotrophoblasts and cytotrophoblasts. The
histological features described in the question, including hyaline cartilage, squamous epithelium, and
simple cysts with columnar epithelium, are not consistent with choriocarcinoma.
Option D - Yolk sac tumor:
• Yolk sac tumors often exhibit a variety of patterns, including glandular, papillary, and solid structures.
However, the presence of multiple nodules of hyaline cartilage and squamous epithelium, as described
in the question, is not typical for yolk sac tumors.

Solution for Question 3:


Correct Option A - It shows Syncytiotrophoblasts and cytotrophoblasts:
• The likely diagnosis in this patient with elevated beta-HCG levels and a tumor showing areas of
hemorrhage and necrosis is choriocarcinoma of the testes.
• Choriocarcinoma is a highly malignant germ cell tumor and patients often present with constitutional
symptoms such as significant weight loss and features of metastatic disease. Microscopically, it is
characterized by the presence of syncytiotrophoblasts and cytotrophoblasts. Syncytiotrophoblasts are
multinucleated, syncytial giant cells, while cytotrophoblasts are mononucleated cells with clear or
eosinophilic cytoplasm.
Incorrect Options:
Option B - It shows a central capillary with visceral and parietal layers of cells:

Page 9

951
• This description is characteristic of a Schiller-Duval body, a structure that consists of a central vessel
surrounded by tumor cells. This cell-vessel structure is contained in a cystic space. It is found in
approximately 50% of yolk sac tumors, not choriocarcinoma.
Option C - Cells are polyhedral with watery-appearing cytoplasm:
• This description is typical of seminoma, not choriocarcinoma. Seminomas are a different subtype of
testicular germ cell tumors.
Option D - Cells in an alveolar or tubular pattern:
• This pattern is characteristic of embryonal carcinoma, not choriocarcinoma. Embryonal carcinoma is
another subtype of testicular germ cell tumor.

Solution for Question 4:


Correct Options A
- Both Assertion and Reason are true, and Reason is the correct explanation for Assertion:
• In a 2-year-old child with a testicular tumor and characteristic findings of glomerular bodies on HPE,
the likely diagnosis is a yolk sac tumor. Glomerular bodies are the characteristic unit mentioned in the
Reason statement, with a mesodermal core, central capillary, and visceral and parietal layers of cells
They are also known as Schiller-Duval bodies.
Incorrect Options:
Option B - Both Assertion and Reason are true, but Reason is not the correct explanation for Assertion:
• This option is incorrect and the correct answer has been explained above.
Option C - Assertion is true, but Reason is false:
• This option is incorrect and the correct answer has been explained above.
Option D - Both Assertion and Reason are false:
• This option is incorrect and the correct answer has been explained above.

Solution for Question 5:


Correct Option D - PLAP (Placental Alkaline Phosphatase):
• The characteristics mentioned in the question, including the patient's age falls within the 2nd and 3rd
decades, a radiosensitive tumor, and lymphatic spread to the para-aortic group of lymph nodes, are
indicative of a seminomatous testicular tumor
• The given histology shows “Fried egg appearance” which is a characteristic of Seminomas.
• Seminoma, a type of testicular germ cell tumor, is known to produce certain tumor markers. One of
these markers is PLAP (Placental Alkaline Phosphatase). Therefore, in the case of a painless, slowly
progressive testicular mass, particularly if it is identified as a seminoma on HPE, PLAP is most likely to
be elevated as a tumor marker.
Incorrect Options:
Option A - AFP (Alpha-Fetoprotein):

Page 10

952
• AFP is a tumor marker associated with non-seminomatous germ cell tumors, particularly yolk sac
tumors. In seminomas, AFP is typically not elevated.
Option B - hCG (Human Chorionic Gonadotropin):
• hCG is another tumor marker associated with non-seminomatous germ cell tumors, especially
choriocarcinoma. In pure seminomas, hCG is generally not elevated.
Option C - CA125:
• CA125 is not a specific marker for testicular tumors. It is more commonly associated with ovarian
cancer.

Solution for Question 6:


Correct Option B - Spermatocytic Seminoma:
• Spermatocytic Seminoma is generally considered a distinct entity from classic seminoma and is not
believed to arise from precursor lesions like GCNIS. In contrast, the other tumor types listed (Sertoli cell
tumor, Leydig cell tumor, and Endodermal sinus tumor) may have associations with GCNIS or similar
precursor lesions.
Incorrect Options:
Option A - Sertoli Cell Tumor:
• Sertoli cell tumors can be associated with GCNIS or similar precursor lesions, making them more
likely in the presence of germ cell neoplasia.
Option C - Leydig Cell Tumor:
• Leydig cell tumors may arise independently of germ cell neoplasia, but they can also occur in
association with GCNIS, albeit less frequently than germ cell tumors.
Option D - Endodermal Sinus Tumor:
• Prepubertal endodermal sinus tumors occur in infants and young children; they are not associated
with germ cell neoplasia in situ (GCNIS).
• While Post-pubertal type endodermal sinus tumors are GCNIS-associated tumors and occur in adults.

Solution for Question 7:


Correct Option B - Bouin's Fixative (Picric acid):
• Bouin's Fixative (Picric acid) is the most appropriate choice for preserving the morphology of germ
cells and aiding in the identification of cellular arrests in testicular tissue analysis for infertility concerns.
Bouin's Fixative is specifically designed to maintain the delicate structures of cells, particularly germ
cells, making it an ideal choice for reproductive tissue examination. This fixative is known for its ability
to provide excellent preservation of cellular details, which is crucial when assessing fertility-related
issues.
Incorrect Options:
Option A - 10% Neutral Buffered Formalin (NBF):

Page 11

953
• While NBF is a commonly used fixative, it may not be the best choice for testicular tissues in infertility
analysis. Formalin can cause significant shrinkage of tissues and may not preserve the delicate
morphology of germ cells as effectively as other fixatives.
Option C - Frozen section without fixation:
• Frozen section without fixation is a rapid method often used for intraoperative assessments. However,
for detailed examination of germ cell morphology, it may not provide the optimal preservation compared
to formalin or Bouin's fixative.
Option D - 70% Ethanol:
• Ethanol is not a standard fixative but rather a dehydration agent. It is not suitable for preserving
cellular morphology in the same way that formalin or specialized fixatives like Bouin's would. Ethanol is
more commonly used in the later stages of tissue processing.

Solution for Question 8:


Correct Option A - Both statements are true:
1. It is an exophytic well-differentiated variant of squamous cell carcinoma.
• This statement is true. Verrucous carcinoma is indeed an exophytic, well-differentiated variant of
squamous cell carcinoma. It is characterized by a slow-growing, locally invasive tumor with a verrucous
or papillomatous appearance. The tumor tends to grow outward, forming a mass with a pebbly or
cauliflower-like surface.
2. It is locally invasive but rarely metastasizes.
• This statement is also true. Verrucous carcinoma is known for its locally invasive behavior, often
infiltrating surrounding tissues. However, it has a low metastatic potential, meaning that it tends to
remain localized without spreading to distant organs. This is in contrast to more typical squamous cell
carcinomas, which may have a higher propensity for metastasis.
• Therefore, in this case, both statements are accurate, and the correct answer is A) Both statements
are true.
Incorrect Options:
Option B - Statement 1 is true, but statement 2 is false:
• This option suggests that the lesion is an exophytic well-differentiated variant of squamous cell
carcinoma (which is true, as it describes verrucous carcinoma) but incorrectly states that it rarely
metastasizes. In reality, verrucous carcinoma does have a low metastatic potential.
Option C - Statement 1 is false, but statement 2 is true:
• This option implies that the lesion is not an exophytic well-differentiated variant of squamous cell
carcinoma, which is incorrect. Verrucous carcinoma does fit this description. However, it correctly
states that verrucous carcinoma is locally invasive but rarely metastasizes.
Option D - Both statements are false:
• This option contradicts the actual characteristics of verrucous carcinoma. As explained earlier,
verrucous carcinoma is an exophytic well-differentiated variant of squamous cell carcinoma, and it is
locally invasive with a low metastatic potential.

Page 12

954
Solution for Question 9:
Correct Option B - The distinguishing point between the diagnosis and Bowen disease is the “younger
age of the patient”:
• The given image and the patient's age confirm a diagnosis of “Bowenoid papulosis”.
• Bowenoid papulosis and Bowen disease are histologically indistinguishable from each other. The
distinguishing feature of Bowenoid papulosis from Bowen is that younger sexually active patients
are diagnosed with Bowenoid papulosis and Bowen is diagnosed in patients who are older than 35
years of age.
Incorrect Options:
Option A - It is a benign condition:
• Bowenoid papulosis is a premalignant condition.
• Carcinoma In-Situ (CIS) penis includes Bowen disease, Bowenoid papulosis and Erythroplasia of
Queyrat.
Option C - It is not usually involved with HPV infection:
• Be it Bowen or Bowenoid papulosis, both conditions are strongly associated with high-risk HPV, most
commonly HPV-16.
Option D - It is not indistinguishable from Bowen disease:
• As explained in the correct option. Bowenoid papulosis is indistinguishable from Bowen disease when
it comes to histology. The diagnosis of Bowenoid papulosis is made if the patient is younger than 35
years.

Solution for Question 10:


Correct Option C - Choriocarcinoma:
• The above-given case history and biopsy findings are diagnostic of choriocarcinoma.
• Testicular choriocarcinoma comes under the classification of non-seminoma germ cell tumours of the
testes.
• The choriocarcinomas secrete an excessive amount of hCG, which is also elevated in this patient.
• Testicular choriocarcinoma is a malignant and aggressive neoplasm and shows a tendency to
metastasize early.
• It usually metastasizes into the lungs or brain. The dissemination into the lungs creates spherical
nodular opacities (cannonball metastases) affecting the lungs, as shown below:

Page 13

955
• The histology of choriocarcinoma, which is shown in the specimen above, reveals
syncytiotrophoblasts and cytotrophoblasts.

• The syncytiotrophoblasts are large, multinucleated cells with vacuolization and amphophilic
cytoplasm.
• They typically have multiple large, smudged-appearing pleomorphic nuclei.
• The cytotrophoblasts are small to medium in size, with clear cytoplasm, and may show nuclear
pleomorphism.
• The background shows extensive necrosis and haemorrhage.

Incorrect Options:
Option A - Endodermal sinus tumour:
• Endodermal sinus tumours are known as yolk-sac tumours.

Page 14

956
• These tumours contain primitive glomerular-like structures that are known as Schiller-Duval bodies.
• They do not contain syncytiotrophoblasts and cytotrophoblasts.
• The tumour marker is usually alpha-fetoprotein rather than hCG.
Option B - Seminoma:
• Seminomas are the most common germ-cell tumours of the testes.
• They have very little tendency to metastasize, and the cannonball pulmonary metastases that are
present in this patient are not typical of seminomas.
• Moreover, instead of the syncytiotrophoblasts and cytotrophoblasts, these tumour show: Fibrous
septae that divide the tumour into lobules Cells having an abundance of watery cytoplasm Fried egg
cell appearance (a clear cytoplasm with a round nucleus, appearing like a fried egg)
• Fibrous septae that divide the tumour into lobules
• Cells having an abundance of watery cytoplasm
• Fried egg cell appearance (a clear cytoplasm with a round nucleus, appearing like a fried egg)
• Fibrous septae that divide the tumour into lobules
• Cells having an abundance of watery cytoplasm
• Fried egg cell appearance (a clear cytoplasm with a round nucleus, appearing like a fried egg)
Option D - Teratoma:
• Teratoma consists of tissue that is derived from different germinal layers.
• The tissues can be mature (differentiated) like cartilage, bone, and teeth, or they can be
undifferentiated mesenchymal elements.
• The lack of these pathological findings in the image above rules out teratomas.

Solution for Question 11:


• Leydig-cell tumours are a sex-cord stromal type of testicular tumour.
• They usually develop at 5–10 years of age or in middle adulthood (30–60 years).
• Tumours of Leydig cells may secrete androgens, estrogens, and corticosteroids.
• The most common clinical presentation of these tumours is testicular swelling.
• However, children usually present with precocious puberty due to excess testosterone secreted by the
tumour.
• Adults (like the patient described above in the clinical scenario) usually present with gynecomastia,
loss of libido, erectile dysfunction and infertility.
• The microscopic findings of Leydig-cell tumours are: Large, closely packed cells with eosinophilic
cytoplasm, bland nuclei, and small nucleoli. Reinke crystals are pale-staining, cylindrical, eosinophilic
cytoplasmic inclusions that are pathognomonic for Leydig cell tumours and are found in up to 30% of
patients. These findings can be seen in the diagram shown below:
• Large, closely packed cells with eosinophilic cytoplasm, bland nuclei, and small nucleoli.

Page 15

957
• Reinke crystals are pale-staining, cylindrical, eosinophilic cytoplasmic inclusions that are
pathognomonic for Leydig cell tumours and are found in up to 30% of patients.
• These findings can be seen in the diagram shown below:


• Large, closely packed cells with eosinophilic cytoplasm, bland nuclei, and small nucleoli.
• Reinke crystals are pale-staining, cylindrical, eosinophilic cytoplasmic inclusions that are
pathognomonic for Leydig cell tumours and are found in up to 30% of patients.
• These findings can be seen in the diagram shown below:

Page 16

958
Incorrect Options:
Option B - Testicular choriocarcinomas:
• Testicular choriocarcinomas are characterized by the presence of syncytiotrophoblasts and
cytotrophoblasts.
• These are germ cell tumours and do not cause an elevation in sex hormones (which are elevated in
the patient described above).
• Moreover, the histological findings of these tumours (syncytiotrophoblasts and cytotrophoblasts) are
distinct from those described in the question above.
Option C - Spermatocytic tumour:
• Spermatocytic germ cell tumours of the testes usually affect older men (unlike this patient).
• Their distinct histopathological appearance comprises three types of cells: smaller cells resembling
secondary spermatocytes, medium-sized cells, and scattered giant cells.
Option D - Yolk sac tumour:
• Yolk-sac tumours are also known as Endodermal sinus tumours.
• These tumours contain primitive glomerular-like structures that are known as Schiller-Duval bodies
(that are different from the histopathological findings described in the question).
• Moreover, unlike this patient, the yolk-sac tumours develop at a very early age.

Solution for Question 12:


Correct Option C - Schiller-Duval bodies are commonly seen:
• Schiller-Duval body is a cellular structure characterised by central blood vessel surrounded by tumour
cells, as shown in the image below:

• Schiller-Duval bodies are seen in endodermal sinus tumours.


• Endodermal sinus tumour is also known as a yolk sac tumour.

Page 17

959
• It is the most common malignant germ cell testicular tumour in children.
• Schiller Duval bodies are not seen in seminomas.
• All other options are correct regarding seminomas
Incorrect Options:
Option A - Most common type of germ cell tumour:
• Seminomas are the most common type of germ cell tumours. They are common among men of age
15-35.
Option B - Serum hCG levels are elevated:
• Beta-hCG levels are elevated in seminomas. Hence, this statement is correct about seminomas.
Option D - Spermatocytic seminomas are usually slow growing:
• Spermatocytic seminomas are uncommon, slow-growing tumours.
• They are generally seen in older individuals (usually over 65 years old).

Solution for Question 13:


Correct Option D - CEA:
CEA (Carcinoembryonic antigen) is used as a
tumor marker for many cancers like colorectal cancer but not testicular tumors.
• The most common age of presentation is 15-35.
• Cryptorchidism and family history are the risk factors.
• The mass is usually asymptomatic, and patients often notice it after a trauma.
• Testicular tumors usually secrete polypeptide hormones and certain enzymes.
• Biological markers include hCG, AFP, and lactate dehydrogenase, which are valuable in diagnosing
and managing testicular cancer.
• Marked increased levels of serum AFP or hCG levels are produced by yolk sac tumor and
choriocarcinoma elements, respectively.
Incorrect Options:
Option A - AFP:
• Alpha-fetoprotein is a tumor marker seen in non-seminoma tumors.
• Hence, it is measured in the diagnostic workup of a testicular mass.
Option B - LDH:
• Lactate dehydrogenase, although less specific than hCG and AFP, is elevated in testicular tumors.
• Elevation of the lactate dehydrogenase provides a tool for assessing tumor burden.
Option C - hCG:
• Beta-hCG is raised in both seminomas and non-seminomas.
• Hence, it is measured in the diagnostic workup of a testicular mass.

Page 18

960
Solution for Question 14:
Correct Option D - Ki-67:
• The diagnosis of prostatic carcinoma is confirmed in histopathology. If any suspicion is found on
histopathological analysis, then immunohistochemistry is ordered to cross-check the diagnosis, and
various markers help assist in this procedure. Here are a few markers used for establishing the
diagnosis of CA prostate on immunohistochemistry. AMACR EGR/TMPRSS2 NKX3.1
• Here are a few markers used for establishing the diagnosis of CA prostate on immunohistochemistry.
AMACR EGR/TMPRSS2 NKX3.1
• AMACR
• EGR/TMPRSS2
• NKX3.1
• Ki-67 is a nuclear protein associated with ribosomal RNA synthesis and may be necessary for
cell-cycle proliferation. It makes Ki-67 an important molecular prognostic marker rather than a
diagnostic marker for prostatic carcinoma and has very little to no proven benefit for the diagnosis of
prostatic carcinoma. Ki-67 is used as a marker for HPV infection in cervical intraepithelial neoplasia.
• Ki-67 is used as a marker for HPV infection in cervical intraepithelial neoplasia.
• Here are a few markers used for establishing the diagnosis of CA prostate on immunohistochemistry.
AMACR EGR/TMPRSS2 NKX3.1
• AMACR
• EGR/TMPRSS2
• NKX3.1
• AMACR
• EGR/TMPRSS2
• NKX3.1
• Ki-67 is used as a marker for HPV infection in cervical intraepithelial neoplasia.
Incorrect Options:
Option A - NKX3.1:
• NK3 homeobox 1(NKX3.1), a validated prostate-specific marker, exhibits distinctive nuclear staining in
most high-grade prostatic adenocarcinomas.
• Its absence in biopsy specimens, where it typically lines benign prostatic epithelium, signals a
non-benign (malignant/carcinomatous) process, suggesting prostatic adenocarcinoma as a differential
diagnosis.

Page 19

961
• Prostatic adenocarcinoma cells (arrows) show strong nuclear staining for NKX3.1.
• In the large benign prostatic glands (arrowheads), secretory cells also show strong nuclear staining
for NKX3.1, and some benign basal cells also show weaker nuclear staining.
Option B - AMACR:
• Alpha-methylacyl-CoA racemase (AMACR) also known as P504S, is a marker commonly used in
immunohistochemistry for prostate cancer diagnosis.
• Increased expression of AMACR is associated with prostate cancer.
Option C - TMPRSS2:
• Transmembrane protease serine 2 (TMPRSS2) is a protein associated with prostate cancer,
especially in the context of gene fusions with the ERG gene.
• It is commonly evaluated in immunohistochemistry analysis of prostate cancer

Solution for Question 15:


Correct Option A - Cryptorchidism:
• Only One palpable testis in the scrotal sac and a palpable mass in the inguinal region suggest
Cryptorchidism.
• It is associated with an increased incidence of germ cell tumors of the testes, even if the testis is
surgically moved from its ectopic location back to the scrotum.
• The descent of the testes occurs in two phases:
1. Transabdominal phase
• During this phase, the testes lie within the lower abdomen or brim of the pelvis.
• Mullerian inhibiting substances regulate this phase.
2. Inguinoscrotal Phase

Page 20

962
• In this phase, the testes descend through the inguinal canal into the scrotum.
• Testes' descent may arrest anywhere between the pathway, but the most common site is in the
inguinal canal.
Incorrect Options:
Option B - Hydrocele:
• A hydrocele is a serous fluid collection in the scrotum.
• A testicle's thin sheath can get clogged with fluid, causing a scrotal enlargement known as a
hydrocele.
• Hydrocele is frequent in babies and typically goes away untreated by age 1.
• Typically, hydroceles are painless.
• The most typical sign is typically an enlarged scrotum.
• However, pain or discomfort may also occur depending on the severity of the swelling.
• In adults, there may be a feeling of heaviness in the scrotum.
Option C - Orchitis:
• Orchitis (inflammation of the testis) can result from bacterial or viral infection.
• Bacterial or viral infections can cause orchitis, or the cause can be unknown.
• Orchitis is often the result of a bacterial infection, such as a sexually transmitted infection (STI).
• In some cases, the mumps virus can cause orchitis.
Option D - Varicocele:
• A varicocele is caused by dilation of the veins of the spermatic cord, and the term “bag of worms” aptly
describes the abnormality.
• A varicocele is an enlargement of the veins within the loose bag of skin that holds the testicles
(scrotum).
• These veins transport oxygen-depleted blood from the testicles.
• A varicocele occurs when blood pools in the veins rather than circulating efficiently out of the scrotum.

Solution for Question 16:


Correct Option B - Endodermal sinus (yolk sac) tumor:
• Endodermal sinus (yolk sac) tumor is the testis' most common malignant germ cell tumor in infancy
and early childhood.
• It is characterized by an elevated serum α-fetoprotein and histologically stainable α1 -antitrypsin.
Endodermal Sinus Tumor:
• In this age range, choriocarcinoma is less prevalent and often leads to a rise in hCG. Although
hepatocellular carcinomas can produce -fetoprotein, this is improbable in this patient's case.
• Teratomas include several germ layers and various tissues, including hair, teeth, and sebaceous
tissue, and are derived from the testicular stroma.

Page 21

963
• They generate androgens and estrogens and frequently present with premature puberty.
• Unlike teratomas in women, the majority of teratomas in men are cancerous.
Incorrect Options:
Option A - Choriocarcinoma:
• Trophoblastic cells give rise to a malignant, quickly expanding tumor (cells that help an embryo attach
to the uterus and help form the placenta).
• After a sperm fertilizes an egg, almost all choriocarcinomas develop in the uterus; however, a tiny
percentage develop in the testis or the ovary.
• Choriocarcinoma symptoms, which have migrated to the lungs, include coughing, breathing problems,
and occasionally chest pain.
• The majority of the time, choriocarcinoma can be cured. Although it can occasionally result in
difficulties, the outlook for patients with this illness is often very bright.
Option C - Cryptorchidism :
• Cryptorchidism refers to undescended testes and does not typically present with increased serum
alpha-fetoprotein.
• While it may be associated with an increased risk of testicular cancer later in life, it is not a likely
diagnosis for a 3-year-old with an abnormal scrotal lump and elevated alpha-fetoprotein.
Option D - Teratoma:
• A germ cell tumour may contain several tissue types, such as bone, muscle, and hair. A teratoma
develops when there are disruptions during your cells' differentiation process.
• Teratomas might be mature or immature depending on how natural the cells seem under a
microscope.
• Sometimes immature and mature cells coexist in teratomas.
• Although malignant degeneration is quite rare, the cyst should be removed entirely, and if immature
elements are found, the patient should undergo a standard staging procedure.
• Possible symptoms include: A visible mass in the tailbone area. Abdominal pain. Constipation.
Dysuria (painful urination).
• A visible mass in the tailbone area.
• Abdominal pain.
• Constipation.
• Dysuria (painful urination).
• Pubic swelling.
• A visible mass in the tailbone area.
• Abdominal pain.
• Constipation.
• Dysuria (painful urination).

Page 22

964
Breast Tumor
1. Choose the incorrect statement regarding a 28-year-old woman who presents with a painless,
well-defined, mobile mass in her left breast and the given FNAC and histology findings:

A. The lifetime risk of development of malignancy is 3%


B. FNAC shows bare nuclei
C. A mammogram will show popcorn calcification
D. If this occurs pericanalicularly, the ducts are compressed
----------------------------------------
2. What differentiates the condition of a 64-year-old woman with a rapidly growing, painless breast
mass, well-circumscribed and mobile, showing a large, lobulated mass with cystic and solid
components on imaging, inconclusive FNAC, and histopathological examination revealing hypercellular
stroma with leaf-like projections lined by epithelial and myoepithelial cells from fibroadenoma?
(or)
Which of the given features differentiate a rapidly growing, well-circumscribed breast mass with
hypercellular stroma and leaf-like projections on microscopy from fibroadenoma in a 64-year-old
woman?
A. Higher cellularity
B. Higher mitotic rates and nuclear pleomorphism
C. Stromal overgrowth and infiltrative borders
D. All of the above
----------------------------------------

965
3. What subtype of breast cancer is suspected in a 48-year-old woman with microcalcifications in the
ductal distribution of the left breast and an extended family history of breast and ovarian cancer on her
mother’s side, based on the biopsy image below?

A. Solid
B. Cribriform
C. Micropapillary
D. Comedo-necrosis
----------------------------------------
4. Which of the following are components of the Modified Bloom & Richardson Score for grading
invasive carcinoma?
A. Tubules, Necrosis, Mitotic count
B. Cellular atypia, Nuclear size, Hyper cellularity
C. Tubule formation, Nuclear pleomorphism, Mitotic count
D. Typical cells, Ductal components preserved, Stromal abnormality
----------------------------------------
5. A 55-year-old woman presents with a painless, palpable mass in her left breast, She also complains
of loss of appetite and recent unintentional weight loss. Mammography reveals a well-defined, round
mass with radiolucent centers. An excision biopsy of the mass is performed and it is soft and rubbery in
consistency and its appearance resembles that of gray gelatin. The histology of the mass is given
below. What is the most likely diagnosis?

A. Tubular carcinoma.
B. Colloid carcinoma

Page 2

966
C. Papillary carcinoma
D. Lobular carcinoma
----------------------------------------
6. What is the most likely histological subtype of breast cancer in a 50-year-old woman who undergoes
a breast biopsy(image given below) due to the discovery of a breast mass and immunohistochemical
staining shows the tumor cells are negative for estrogen and progesterone receptors, as well as
HER2/neu?
(or)
What is the most likely histological subtype of breast cancer in a 50-year-old woman with a breast
mass, histopathology showing pushing borders, syncytial growth pattern, and negative staining for
estrogen, progesterone receptors, and HER2/neu?

A. Invasive Ductal Carcinoma


B. Lobular Carcinoma in Situ
C. Medullary Carcinoma
D. Inflammatory Breast Cancer
----------------------------------------
7. Match the following to their expected IHC: A. Mucinous carcinoma 1. ER -, PR-, HER-2 - B. Invasive
lobular carcinoma 2. ER+, PR+, HER-2 - C. Apocrine carcinoma 3. ER+, PR+, HER-2+ D. Medullary
carcinoma breast 4. ER-, PR-, HER-2+
A. Mucinous carcinoma 1. ER -, PR-, HER-2 -
B. Invasive lobular carcinoma 2. ER+, PR+, HER-2 -
C. Apocrine carcinoma 3. ER+, PR+, HER-2+
D. Medullary carcinoma breast 4. ER-, PR-, HER-2+

A. A-2, B-1, C-4, D-3


B. A-4, B-3, C-2, D-1
C. A-2, B-3, C-4, D-1
D. A-3, B-2, C-4, D-1
----------------------------------------
8. What's the likely diagnosis for a 57-year-old woman presenting with left breast pain and redness,
which on examination were swollen, tender, and erythematous, showing a peau d'orange appearance

Page 3

967
and a completely retracted left nipple, despite no prior medical visits in the past decade?
(or)
What is the most likely cause of the left nipple being entirely retracted, and the breast having significant
erythematous and dimpling (peau d'orange)?
A. Granulomatous mastitis
B. Micropapillary carcinoma
C. Fibrocystic disease of the breast
D. Inflammatory carcinoma
----------------------------------------
9. What's the likely diagnosis for a 25-year-old woman with a painless, 0.5 cm rubbery mass in the
upper right quadrant of her breast that can move in horizontal and vertical planes, with normal
mammography and no significant medical history?
A. Fibrocystic disease
B. Fibroadenoma
C. Medullary carcinoma
D. Colloid carcinoma
----------------------------------------
10. What's the most likely diagnosis for a 56-year-old woman presenting with bloody nipple discharge,
a 0.6 cm subareolar breast nodule that is surgically excised, and its histological findings are given
below?

A. Ductal carcinoma in situ


B. Intraductal papilloma
C. Lobular carcinoma in situ
D. Medullary carcinoma
----------------------------------------
11. A 56-year-old female patient visits an outpatient clinic. She was concerned about a mass in her left
breast that she had palpated the previous week. An ultrasound revealed a high risk of malignancy. A
mastectomy was performed following biopsy confirmation. HER2/neu immunohistochemical staining
(shown in the image) is performed. Which of the following genetic mechanisms best accounts for the
staining intensity in this specimen?

Page 4

968
(or)
What genetic mechanism best explains the staining intensity of the HER2/neu immunohistochemical
staining (shown in the image)?

A. Gene amplification
B. Insertional mutagenesis
C. Chromosomal non-homologous crossing over
D. Single nucleotide polymorphism
----------------------------------------
12. What's the likely pathologic diagnosis for a 56-year-old woman with an irregular nodularity in her
right breast, varying breast densities on mammography in both breasts and the given histology in both
breasts upon biopsy?

A. Colloid carcinoma
B. Lobular carcinoma in situ
C. Phyllodes tumour
D. Medullary carcinoma
----------------------------------------
13. What is the likely finding associated with a 47-year-old woman with multiple small areas of
increased density visualized on mammography, no palpable mass, findings of fine needle aspiration
are suspicious of malignancy, and histology of excisional biopsy given below?

Page 5

969
A. Opposite breast involvement
B. Absent family history of breast cancer
C. Concomitant Paget disease of the nipple
D. Negative estrogen receptor assay
----------------------------------------
14. A 57-year-old female is presented in opd with a mass in her right breast. She was diagnosed with
breast cancer. BRCA genes are responsible for breast cancer. The BRCA1 gene is located on which
chromosome
A. 17q21
B. 13q12
C. 12q13
D. 21q17
----------------------------------------

Correct Answers
Question Correct Answer

Question 1 4
Question 2 4
Question 3 4
Question 4 3
Question 5 2
Question 6 3
Question 7 3
Question 8 4
Question 9 2
Question 10 2
Question 11 1

Page 6

970
Question 12 2
Question 13 1
Question 14 1

Solution for Question 1:


Correct Option D - If this occurs pericanalicularly, the ducts are compressed:
• Pericanalicular fibroadenoma causes the ducts to stay open. Ducts are compressed in intracanalicular
fibrodenoma.
Pericanalicular fibroadenoma causes the ducts to stay open. Ducts are compressed in intracanalicular
fibrodenoma.
Incorrect Options:
Option A - The lifetime risk of development of malignancy is 3%:
• Fibroadenoma without complex features has only a 3% lifetime risk of turning into cancer.
Fibroadenoma with complex features has a 5-7% risk.
Fibroadenoma without complex features has only a
3% lifetime risk of turning into cancer. Fibroadenoma with complex features has a 5-7% risk.
Option B - FNAC shows bare nuclei:
• This image shows: Branching sheets of cohesive cells forming the antler horn pattern. Bipolar bare
nuclei of myoepithelial cells are seen (FNA, Pap stain, 20x).
• Staghorn or antler horn pattern is a characteristic formation in which the bare nuclei are seen in
FNAC.
This image shows: Branching sheets of cohesive cells forming the antler horn pattern. Bipolar bare nuc
lei of myoepithelial cells are seen (FNA, Pap stain, 20x).
Staghorn or antler horn pattern is a characteristic formation in which the bare nuclei are seen in FNAC.
Option C - A mammogram will show popcorn calcification:
• Fibroadenomas may show popcorn-like calcifications on imaging. This refers to the appearance of
small, rounded calcifications resembling popped popcorn kernels.
Fibroadenomas may show popcorn-like calcifications on imaging. This refers to the appearance of sma
ll, rounded calcifications resembling popped popcorn kernels.

Solution for Question 2:


Correct Option D - All of the above:
• All of the given features - higher cellularity, higher mitotic rates, nuclear pleomorphism, stromal
overgrowth, and infiltrative borders - can differentiate a rapidly growing, well-circumscribed breast mass
with hypercellular stroma and leaf-like projections from fibroadenoma. Phyllodes tumors, unlike
fibroadenomas, often exhibit increased cellularity, higher mitotic activity, nuclear pleomorphism, and
stromal overgrowth. Additionally, infiltrative borders may be observed, indicating a more aggressive
behavior compared to the typically well-circumscribed nature of fibroadenomas.

Page 7

971
Solution for Question 3:
Correct Option D- Comedo-necrosis:
• The age of the patient, microcalcification detected in the ductal distribution of the left breast on
mammography, and extended family history of breast and ovarian cancer leads to the suspicion of
DCIS.
• The image shows DCIS (Ductal Carcinoma In Situ) of Comedo pattern characterized by central
necrosis in the involved ducts, as seen in histopathological examination at 100× magnification using
Hematoxylin and Eosin (H&E;) staining.
Incorrect Options:
Option A - Solid:
• Ducts are filled with cells, making them solid as shown in the image.

Option B - Cribriform:
• In this type of DCIS Histology: Shows round regular “cookie cutter” spaces containing calcifying
secretory material.
• Shows round regular “cookie cutter” spaces containing calcifying secretory material.
• Shows round regular “cookie cutter” spaces containing calcifying secretory material.

Page 8

972
Option C - Micropapillary:
• In this form of DCIS shows papillary projections and lacks fibrovascular cores

Solution for Question 4:


Correct Option C - Tubule formation, Nuclear pleomorphism, Mitotic count:
• Tubule formation: This refers to the extent to which cancer cells form small duct-like structures
(tubules) similar to normal breast tissue. Higher-grade tumors often have fewer well-formed tubules,
while lower-grade tumors may have more.
• Nuclear pleomorphism: This component evaluates the variation in size, shape, and staining
characteristics of the cell nuclei. Higher-grade tumors typically exhibit more significant nuclear
pleomorphism, indicating increased abnormality of the cell nuclei.

Page 9

973
• Mitotic count: This assesses the number of actively dividing cells (mitoses) in a given microscopic
field. A higher mitotic count is associated with a more rapidly dividing and potentially more aggressive
tumor.
Tubule formation: This refers to the extent to which cancer cells form small duct-like structures (tubules
) similar to normal breast tissue. Higher-grade tumors often have fewer well-formed tubules, while lowe
r-grade tumors may have more.
Nuclear pleomorphism: This component evaluates the variation in size, shape, and staining characteris
tics of the cell nuclei. Higher-grade tumors typically exhibit more significant nuclear pleomorphism, indi
cating increased abnormality of the cell nuclei.
Mitotic count: This assesses the number of actively dividing cells (mitoses) in a
given microscopic field. A higher mitotic count is associated with a
more rapidly dividing and potentially more aggressive tumor.
Incorrect Options:
Option A - Tubules, Necrosis, Mitotic count:
• Tubules and Mitotic count are correct components, but necrosis is not a component of the Modified
Bloom & Richardson Score. Necrosis is cell death, and while it may be present in tumors, it is not one
of the parameters used in this particular grading system.
Tubules and Mitotic count are correct components, but necrosis is not a
component of the Modified Bloom & Richardson Score. Necrosis is cell death, and while it may be pres
ent in tumors, it is not one of the parameters used in this particular grading system.
Options A, B and D are incorrect.

Solution for Question 5:


Correct Option B - Colloid carcinoma :
• The combination of clinical features, mammography findings, gross appearance, and histology
strongly points towards colloid carcinoma as the most likely diagnosis in this scenario.
• Colloid carcinoma of the breast is a type of cancer where tumor cells are spread throughout pools of
extracellular mucin. This histology type is sometimes described as "cellular islands in lakes of mucin".
The tumor cells generally have low-grade nuclei and are surrounded by clear mucin-filled spaces.
The combination of clinical features, mammography findings, gross appearance, and histology strongly
points towards colloid carcinoma as the most likely diagnosis in this scenario.
Colloid carcinoma of the breast is a type of cancer where tumor cells are spread throughout pools of ex
tracellular mucin. This histology type is sometimes described as "cellular islands in lakes of mucin". Th
e tumor cells generally have low-grade nuclei and are surrounded by clear mucin-filled spaces.
Incorrect Options:
Option A - Tubular Carcinoma:
• Tubular carcinoma is a subtype of invasive ductal carcinoma. It typically presents as small,
well-defined masses and is associated with a good prognosis.
• Unlike colloid carcinoma, tubular carcinoma does not usually present with the characteristic gross
appearance of gray gelatin. The histology of tubular carcinoma shows small, well-formed tubules.
Option C - Papillary Carcinoma:

Page 10

974
• Papillary carcinoma of the breast is characterized by finger-like projections of tumor tissue. It often
presents as a well-defined mass and may have cystic components.
• The described gross appearance resembling gray gelatin and histological features do not align with
the typical characteristics of papillary carcinoma.
Option D - Lobular Carcinoma:
• Lobular carcinoma originates in the lobules of the breast and can have a diffuse growth pattern.
• The presented case describes a well-defined round mass, and the gross appearance resembling gray
gelatin is not typical for lobular carcinoma. Additionally, lobular carcinoma may not always present as a
palpable mass.

Solution for Question 6:


Correct Option C - Medullary Carcinoma:
• Medullary carcinoma of the breast is characterized by distinct histopathological features, including
pushing borders, a syncytial growth pattern (where tumor cells are densely packed), and a prominent
lymphocytic infiltrate. Additionally, it is often associated with a lack of expression of estrogen and
progesterone receptors and HER2/neu (Triple Negative)
Medullary carcinoma of the breast is characterized by distinct histopathological features, including pus
hing borders, a syncytial growth pattern (where tumor cells are densely packed), and a
prominent lymphocytic infiltrate. Additionally, it is often associated with a
lack of expression of estrogen and progesterone receptors and HER2/neu (Triple Negative)
Incorrect Options:
Option A - Invasive Ductal Carcinoma: Invasive Ductal Carcinoma (IDC) is the most common type of br
east cancer. It often presents with irregular borders and may or may not show a syncytial growth patter
n. However, the negative staining for estrogen and progesterone receptors, as well as HER2/neu, is no
t typical for IDC.
Option B - Lobular Carcinoma in Situ: Lobular Carcinoma In Situ (LCIS) is a
non-invasive condition and not a subtype of invasive breast cancer. It is characterized by abnormal cell
s within the lobules but lacks the invasive features seen in this patient's histopathology.
Option D - Inflammatory Breast Cancer: Inflammatory Breast Cancer (IBC) is a rare and aggressive for
m of breast cancer that typically presents with redness, swelling, and warmth of the breast, resembling
an inflammation. The histopathological features described in the question, including well-defined borde
rs and syncytial growth patterns, are not characteristic of IBC.

Solution for Question 7:


Correct Option C - A-2, B-3, C-4, D-1:
A. Mucinous carcinoma
2. ER+, PR+, HER-2 -
B. Invasive lobular carcinoma
3. ER+, PR+, HER-2+

Page 11

975
C. Apocrine carcinoma
4. ER-, PR-, HER-2+
D. Medullary carcinoma breast
1. ER -, PR-, HER-2 -

Solution for Question 8:


Correct Option D - Inflammatory carcinoma:
• Inflammatory carcinoma is caused by carcinoma plugging lymphovascular spaces, resulting in
swelling and nipple retraction.
• Although it is called inflammatory carcinoma, there is no inflammation present.
• This is usually of high grade and has distant metastasis.
• It has a very poor prognosis.
• Common symptoms are Pain Erythema Dimpling of skin (peau d'orange) Skin thickening Usually, no
palpable mass is present
• Pain
• Erythema
• Dimpling of skin (peau d'orange)
• Skin thickening
• Usually, no palpable mass is present
• "Peau d'orange" refers to the appearance of skin that resembles the texture of an orange peel. This
effect is caused by cutaneous lymphatic edema, the presence of sweat ducts in the affected skin area
prevents swelling, resulting in a dimpled appearance similar to that of orange skin.
• The presentation is similar to breast infections, resulting in delayed diagnosis.
Inflammatory carcinoma is caused by carcinoma plugging lymphovascular spaces, resulting in swelling
and nipple retraction.
Although it is called inflammatory carcinoma, there is no inflammation present.
This is usually of high grade and has distant metastasis.
It has a very poor prognosis.
Common symptoms are
• Pain
• Erythema
• Dimpling of skin (peau d'orange)
• Skin thickening
• Usually, no palpable mass is present
Pain

Page 12

976
Erythema
Dimpling of skin (peau d'orange)
Skin thickening
Usually, no palpable mass is present
"Peau d'orange" refers to the appearance of skin that resembles the texture of an orange peel. This eff
ect is caused by cutaneous lymphatic edema, the presence of sweat ducts in the affected skin area pre
vents swelling, resulting in a dimpled appearance similar to that of orange skin.
The presentation is similar to breast infections, resulting in delayed diagnosis.
Incorrect Options:
Option A - Granulomatous mastitis:
• It is a rare breast disease that can manifest in case of systemic granulomatous disease (Sarcoidosis,
Tuberculosis, and so on).
• The presence of granulomas adjacent to lobules is the histologic pattern, which may contain lipids.
• Histologically, the pattern resembles cystic neutrophilic granulomatous mastitis.
• It should be treated with antibiotics and, in some cases, steroids.
It is a rare breast disease that can manifest in case of systemic granulomatous disease (Sarcoidosis,
Tuberculosis, and so on).
The presence of granulomas adjacent to lobules is the histologic pattern, which may contain lipids.
Histologically, the pattern resembles cystic neutrophilic granulomatous mastitis.
It should be treated with antibiotics and, in some cases, steroids.
Option B - Micropapillary carcinoma:
• Micropapillary carcinoma forms a cell ball that floats in intercellular fluid.
• It forms a structure similar to true papillae.
Micropapillary carcinoma forms a cell ball that floats in intercellular fluid.
It forms a structure similar to true papillae.
Option C - Fibrocystic disease of the breast:
• Fibrocystic disease is characterised by the formation of cysts and stromal fibrosis.
• Smaller cysts then combine to form larger cysts.
• It is a benign breast condition.
Fibrocystic disease is characterised by the formation of cysts and stromal fibrosis.
Smaller cysts then combine to form larger cysts.
It is a benign breast condition.

Solution for Question 9:

Page 13

977
Correct Option B - Fibroadenoma:
• It is a common benign breast tumour in women.
• Although the pathogenesis of fibroadenoma is unknown, two-thirds of patients have MED12
mutations.
• These tumours are most common in young females.
• Because fibroadenoma responds to oestrogen levels, it can grow during the period of high oestrogen
(pregnancy) and regress after menopause.
• Characteristics of fibroadenoma are Well circumscribed Rubbery Greyish Mobile
• Well circumscribed
• Rubbery
• Greyish
• Mobile
• Because of the rapid growth and infarction of tumours during pregnancy may resemble carcinoma.
• Fibroadenoma can increase the risk of carcinoma when complex features are present Cyst > 0.3 cm
Sclerosing adenosis Epithelial calcifications Papillary apocrine change
• Cyst > 0.3 cm
• Sclerosing adenosis
• Epithelial calcifications
• Papillary apocrine change
• The majority of patients who receive cyclosporin after a kidney transplant are at an increased risk of
developing multiple fibroadenomas.
• Histopathology Pericanalicular pattern - epithelium is surrounded by stroma Intracanalicular pattern -
compressed and distorted lumen by stroma
• Pericanalicular pattern - epithelium is surrounded by stroma
• Intracanalicular pattern - compressed and distorted lumen by stroma
It is a common benign breast tumour in women.
Although the pathogenesis of fibroadenoma is unknown, two-thirds of patients have MED12 mutations.
These tumours are most common in young females.
Because fibroadenoma responds to oestrogen levels, it can grow during the period of high oestrogen (
pregnancy) and regress after menopause.
Characteristics of fibroadenoma are
• Well circumscribed
• Rubbery
• Greyish
• Mobile
Well circumscribed
Rubbery

Page 14

978
Greyish
Mobile
Because of the rapid growth and infarction of tumours during pregnancy may resemble carcinoma.
Fibroadenoma can increase the risk of carcinoma when complex features are present
• Cyst > 0.3 cm
• Sclerosing adenosis
• Epithelial calcifications
• Papillary apocrine change
Cyst > 0.3 cm
Sclerosing adenosis
Epithelial calcifications
Papillary apocrine change
The majority of patients who receive cyclosporin after a
kidney transplant are at an increased risk of developing multiple fibroadenomas.
Histopathology
• Pericanalicular pattern - epithelium is surrounded by stroma
• Intracanalicular pattern - compressed and distorted lumen by stroma
Pericanalicular pattern - epithelium is surrounded by stroma
Intracanalicular pattern - compressed and distorted lumen by stroma
Incorrect Options:
Option A - Fibrocystic disease:
• It is a benign breast condition palpated as "lumpy bumpy."
• It causes multiple cysts to form with stromal fibrosis.
• Smaller cysts combine to form larger ones.
• Fibrosis is caused by material leakage caused by cyst rupture.
It is a benign breast condition palpated as "lumpy bumpy."
It causes multiple cysts to form with stromal fibrosis.
Smaller cysts combine to form larger ones.
Fibrosis is caused by material leakage caused by cyst rupture.
Option C - Medullary carcinoma:
• The majority of medullary carcinomas are caused by hypermethylation of the BRCA 1 promoter, which
results in decreased gene expression in approximately 67% of patients.
• It is not associated with the familial BRCA1 mutation.
• The infiltration of T lymphocytes is the most common feature of these tumours. As a result, the
outcome is determined by the host's immune response and antigen burden.
• It has a better prognosis.

Page 15

979
The majority of medullary carcinomas are caused by hypermethylation of the BRCA 1
promoter, which results in decreased gene expression in approximately 67% of patients.
It is not associated with the familial BRCA1 mutation.
The infiltration of T lymphocytes is the most common feature of these tumours. As a
result, the outcome is determined by the host's immune response and antigen burden.
It has a better prognosis.
Option D - Colloid carcinoma:
• It has a soft or rubbery consistency and the appearance of pale grey-blue gelatin.
• Tumours appear as small islands of cells surrounded by mucin.
It has a soft or rubbery consistency and the appearance of pale grey-blue gelatin.
Tumours appear as small islands of cells surrounded by mucin.

Solution for Question 10:


Correct Option B - Intraductal papilloma:
• The papilloma grows within the duct lumen.
• The given histology shows cuboidal and myoepithelial cell-lined vascular connective tissue cores
projecting into the major lactiferous duct which confirms the diagnosis of intraductal papilloma
• It is a benign breast tumor.
• It is connected to the duct wall by fibrovascular branches.
• Epithelial hyperplasia and apocrine metaplasia are also associated with it.
• A large intraductal papilloma is a solitary papilloma found in the lactiferous sinuses.
• Smaller papillomas are typically multiple and found in deeper ducts.
• Symptoms of intraductal papilloma are Bloody discharge (due to torsion of the stalk) Serous discharge
(due to intermittent blockage) Normal breast secretions (due to irritation of the duct by papilloma)
• Bloody discharge (due to torsion of the stalk)
• Serous discharge (due to intermittent blockage)
• Normal breast secretions (due to irritation of the duct by papilloma)
• Small palpable masses or calcifications detected on mammograms are the most common
presentation.
The papilloma grows within the duct lumen.
The given histology shows cuboidal and myoepithelial cell-lined vascular connective tissue cores proje
cting into the major lactiferous duct which confirms the diagnosis of intraductal papilloma
It is a benign breast tumor.
It is connected to the duct wall by fibrovascular branches.
Epithelial hyperplasia and apocrine metaplasia are also associated with it.
A large intraductal papilloma is a solitary papilloma found in the lactiferous sinuses.

Page 16

980
Smaller papillomas are typically multiple and found in deeper ducts.
Symptoms of intraductal papilloma are
• Bloody discharge (due to torsion of the stalk)
• Serous discharge (due to intermittent blockage)
• Normal breast secretions (due to irritation of the duct by papilloma)
Bloody discharge (due to torsion of the stalk)
Serous discharge (due to intermittent blockage)
Normal breast secretions (due to irritation of the duct by papilloma)
Small palpable masses or calcifications detected on mammograms are the most common presentation.
Incorrect Options:
Option A - Ductal carcinoma in situ:
• Carcinoma cells are restricted to ducts.
• It lacks myoepithelial cells.
• Ductal carcinoma in situ can have a variety of growth patterns.
• Growth patterns in ductal carcinoma are Comedo DCIS Cribriform DCIS Micropapillary DCIS Papillary
DCIS
• Comedo DCIS
• Cribriform DCIS
• Micropapillary DCIS
• Papillary DCIS
Carcinoma cells are restricted to ducts.
It lacks myoepithelial cells.
Ductal carcinoma in situ can have a variety of growth patterns.
Growth patterns in ductal carcinoma are
• Comedo DCIS
• Cribriform DCIS
• Micropapillary DCIS
• Papillary DCIS
Comedo DCIS
Cribriform DCIS
Micropapillary DCIS
Papillary DCIS
Option C - Lobular carcinoma in situ:
• Lobular carcinoma in situ does not cross the epithelial basement membrane.
• Most of the time, it is an incidental finding.
• It is caused by the loss of E-cadherin function (the CDH1 gene).

Page 17

981
• Mucin-containing signet ring cells can be seen in pathology.
Lobular carcinoma in situ does not cross the epithelial basement membrane.
Most of the time, it is an incidental finding.
It is caused by the loss of E-cadherin function (the CDH1 gene).
Mucin-containing signet ring cells can be seen in pathology.
Option D - Medullary carcinoma:
• Breast cancer with a medullary pattern is caused primarily by BRCA 1 gene hypermethylation.
• Gene expression is reduced as a result of hypermethylation.
• It is not associated with familial mutation of the BRCA 1 gene.
• It has a good prognosis.
Breast cancer with a medullary pattern is caused primarily by BRCA 1 gene hypermethylation.
Gene expression is reduced as a result of hypermethylation.
It is not associated with familial mutation of the BRCA 1 gene.
It has a good prognosis.

Solution for Question 11:


Correct Option A - Gene amplification:
• HER2/neu overexpression is found in 10% to 35% of primary breast tumours and is mostly due to
gene amplification on chromosome 17q.
• The amplification may involve adjacent genes that may promote the growth of the tumour.
• Breast cancer due to HER/neu overexpression may be ER-positive or negative.
• It functions as a tyrosine kinase receptor, promoting cell proliferation and inhibiting apoptosis by
activating the P13K - AKT pathway.
• It is also associated with the p54 gene mutation (Li Fraumeni syndrome)
• The gene expression is associated with Transcripts that HER2 encodes Other amplified genes
Pro-growth genes amplification Proliferation
• Transcripts that HER2 encodes
• Other amplified genes
• Pro-growth genes amplification
• Proliferation
• It is usually diagnosed by HER2 immunohistochemistry (overexpression) or in situ hybridisation
(amplification)
• More than half of the patients with HER2-positive carcinoma can achieve remission by using
HER2-blocking antibodies.
HER2/neu overexpression is found in 10% to 35% of primary breast tumours and is mostly due to gene
amplification on chromosome 17q.

Page 18

982
The amplification may involve adjacent genes that may promote the growth of the tumour.
Breast cancer due to HER/neu overexpression may be ER-positive or negative.
It functions as a
tyrosine kinase receptor, promoting cell proliferation and inhibiting apoptosis by activating the P13K -
AKT pathway.
It is also associated with the p54 gene mutation (Li Fraumeni syndrome)
The gene expression is associated with
• Transcripts that HER2 encodes
• Other amplified genes
• Pro-growth genes amplification
• Proliferation
Transcripts that HER2 encodes
Other amplified genes
Pro-growth genes amplification
Proliferation
It is usually diagnosed by HER2 immunohistochemistry (overexpression) or in situ hybridisation (amplifi
cation)
More than half of the patients with HER2-positive carcinoma can achieve remission by using HER2-blo
cking antibodies.
Incorrect Options:
Option B - Insertional mutagenesis:
• It is the insertion of foreign DNA into a chromosome, which can either inhibit or promote adjacent
genes.
• It alters the phenotype of the cell.
• It has no role in the HER2/neu mutation.
It is the insertion of foreign DNA into a
chromosome, which can either inhibit or promote adjacent genes.
It alters the phenotype of the cell.
It has no role in the HER2/neu mutation.
Option C - Chromosomal non-homologous crossing over:
• Non - homologous crossing over means exchanging a section of chromosome between
non-homologous chromosomes.
• This mutation does not cause overexpression of the HER/neu gene.
Non - homologous crossing over means exchanging a
section of chromosome between non-homologous chromosomes.
This mutation does not cause overexpression of the HER/neu gene.
Option D - Single nucleotide polymorphism:
• It is the variation in the position of a single nucleotide between individuals.

Page 19

983
• It is not linked to the HER2/neu mutation.
It is the variation in the position of a single nucleotide between individuals.
It is not linked to the HER2/neu mutation.

Solution for Question 12:


Correct Option B - Lobular carcinoma in situ:
• Lobular carcinoma in situ is caused by the asynchronous growth of ducts and lobules.
• It is caused by the loss of the E - Cadherin gene, CDH1 (which produces a protein that helps in cell
adhesion).
• Because E-cadherin also functions as a tumor suppressor protein, so mutations can result in
carcinoma development.
• Sometimes, a mutation in another protein called catenin is required for E-cadherin to function.
• Because LCIS and invasive lobular carcinoma share the same mutation, the risk of developing
invasive lobular carcinoma increases.
• It is mostly unrelated to calcification and is discovered incidentally
• It is usually bilateral.
• Morphology of LCIS shows Uniform cells with oval or round nuclei (due to E- cadherin loss) Signet
ring cells
• Uniform cells with oval or round nuclei (due to E- cadherin loss)
• Signet ring cells
• It almost always expresses ER and PR but not HER2.
• Prophylactic bilateral mastectomy, tamoxifen, and close clinical monitoring are all part of the treatment
plan.
Lobular carcinoma in situ is caused by the asynchronous growth of ducts and lobules.
It is caused by the loss of the E - Cadherin gene, CDH1 (which produces a
protein that helps in cell adhesion).
Because E-cadherin also functions as a
tumor suppressor protein, so mutations can result in carcinoma development.
Sometimes, a mutation in another protein called catenin is required for E-cadherin to function.
Because LCIS and invasive lobular carcinoma share the same mutation, the risk of developing invasive
lobular carcinoma increases.
It is mostly unrelated to calcification and is discovered incidentally
It is usually bilateral.
Morphology of LCIS shows
• Uniform cells with oval or round nuclei (due to E- cadherin loss)
• Signet ring cells
Uniform cells with oval or round nuclei (due to E- cadherin loss)

Page 20

984
Signet ring cells
It almost always expresses ER and PR but not HER2.
Prophylactic bilateral mastectomy, tamoxifen, and close clinical monitoring are all part of the treatment
plan.
Incorrect Options
Option A - Colloid carcinoma:
• Colloid carcinoma is a soft rubbery mass.
• It appears to be floating cells in a lake of mucin.
Colloid carcinoma is a soft rubbery mass.
It appears to be floating cells in a lake of mucin.
Option C - Phyllodes tumor:
• Phyllodes tumors are caused by a mutation in the MED12 gene, which causes stoma overgrowth.
• It does not match the histological presentation.
Phyllodes tumors are caused by a mutation in the MED12 gene, which causes stoma overgrowth.
It does not match the histological presentation.
Option D - Medullary carcinoma:
• Medullary carcinoma is caused by BRCA1 gene hypermethylation.
• The histological presentation does not explain this tumor
Medullary carcinoma is caused by BRCA1 gene hypermethylation.
The histological presentation does not explain this tumor

Solution for Question 13:


Correct Option A - Opposite breast involvement:
• In the biopsy the tumor cells present back to back, in an Indian file pattern. There are no palpatory
findings indicating a hard mass and the histology shows no desmoplasia.
• LCIS is often considered a marker of increased risk rather than an invasive cancer. Women with LCIS
have an increased risk of developing invasive breast cancer, not only in the same breast but also in the
opposite breast.
Incorrect Options:
Option B - Absent family history of breast cancer: LCIS can occur in women with or without a
family history of breast cancer. It's not strictly associated with the absence of a family history.
Option C - Concomitant Paget disease of the nipple: LCIS is not typically associated with Paget diseas
e of the nipple. These are distinct breast conditions with different clinical and histological features.
Option D - Negative estrogen receptor assay: LCIS is often associated with hormone receptor-positive
status, particularly estrogen receptor (ER)-positive. It is more commonly ER-positive than ER-negative.
• In summary, the most likely finding associated with LCIS is an increased risk of developing invasive
breast cancer, including in the opposite breast. Individuals with LCIS need to undergo regular breast

Page 21

985
cancer screening and consider risk-reducing strategies in consultation with their healthcare providers.

Solution for Question 14:


Correct Option A - 17q21:
• The BRCA1 and BRCA2 (BReast CAncer genes 1 and 2, respectively) generate proteins that aid in
repairing DNA damage.
• Each of these genes is present in two copies, one from each parent.
• About 3% of all breast cancers and 80% to 90% of "single gene" familial breast cancers are caused by
mutations in BRCA1 and BRCA2.
• BRCA1 is located on 17q21
• BRCA2 is located on chromosome 13q12.
The BRCA1 and BRCA2 (BReast CAncer genes 1
and 2, respectively) generate proteins that aid in repairing DNA damage.
Each of these genes is present in two copies, one from each parent.
About 3% of all breast cancers and 80% to 90% of "single gene" familial breast cancers are caused by
mutations in BRCA1 and BRCA2.
BRCA1 is located on 17q21
BRCA2 is located on chromosome 13q12.
Incorrect Options:
Option B - 13q12:
• BRCA2 is located on chromosome 13q12.
Option C - 12q13:
• Chromosome 12 spans about 133 million base pairs (the building material of DNA) and represents
between 4 and 4.5 percent of the total DNA in cells.
• Chromosome 12 contains the Homeobox C gene cluster.
Chromosome 12 spans about 133 million base pairs (the building material of DNA) and represents bet
ween 4 and 4.5 percent of the total DNA in cells.
Chromosome 12 contains the Homeobox C gene cluster.
Option D - 17q17:
• Chromosome 17 contains the Homeobox B gene cluster.
Chromosome 17 contains the Homeobox B gene cluster.

Page 22

986
Inflammatory Lesions of Breast
1. What is the pathogenesis of the tender, swollen right breast in a 26-year-old breastfeeding woman
with erythema, warmth, and pain during breastfeeding and a history of similar episodes during a
previous pregnancy with the given FNAC and special staining findings?

A. Tuberculosis-induced granuloma
B. Hypersensitivity to antigens expressed during lactation
C. Localized infection by Corynebacterium.
D. Local Staph infection due to breastfeeding.
----------------------------------------
2. What's the correct statement about the condition of a 32-year-old breastfeeding woman with a
painful, swollen left breast, erythema, and localized warmth, but no fever or systemic symptoms, and a
tender, palpable mass in the upper outer quadrant of the left breast, based on FNAC results?

987
A. Cellulitis is a common symptom of Staphylococcal infection.
B. During weaning, the causative organism can enter through cracks in the nipple.
C. Surgical drainage is necessary for the treatment.
D. Streptococcus is a less common cause of mastitis.
----------------------------------------
3. What is the relative risk of developing cancer in a 55-year-old woman with a periareolar mass, skin
retractions, and white nipple discharge and the biopsy findings shown below?
(or)
What is the relative risk of developing cancer in a 55-year-old woman with a periareolar mass, skin
retractions, and white nipple discharge, as confirmed by the biopsy result shown below?

A. 1-3%
B. 5-7%
C. 0-1%
D. 4-5%
----------------------------------------
4. What's the recommended treatment for a 38-year-old woman with a recurrent, painful mass under
her left breast's areola, occasional discharge, a history of 15 years of chewing tobacco, examination
revealing well-defined, erythematous nodules with draining sinuses and the given biopsy findings?
(or)
What is the recommended mode of treatment for a subareolar mass that occasionally presents with a
discharging sinus at the edge of the areola, and a biopsy shows squamous metaplasia of the duct
involved?

Page 2

988
A. Antibiotics like cephalexin, clindamycin, or co-trimoxazole
B. Simple incision and drainage
C. En Bloc surgical removal
D. Antibiotic and steroid therapy
----------------------------------------
5. Choose the correct statement regarding ■■■■■■a 45-year-old post-menopausal woman who came
with a complaint of a lump in her breast and radiological findings of dense breast with cysts, and the
given images of mammogram and biopsy.

A. Diagnosis is confirmed by the disappearance of the mass after FNAC


B. Cysts rupture into adjacent stroma causing chronic inflammation and fibrosis
C. There is an increase in the number of acini per lobule
D. All the above

Page 3

989
----------------------------------------
6. Match the Proliferative breast disease to their respective histology: A. Complex sclerosing lesion 1.
B. Papilloma 2. C. Epithelial Hyperplasia 3. D. Sclerosing Adenosis 4 .

A. Complex sclerosing lesion 1.

B. Papilloma 2.

C. Epithelial Hyperplasia 3.

Page 4

990
D. Sclerosing Adenosis 4.

A. A-2, B-1, C-4, D-3


B. A-2, B-3, C-4, D-1
C. A-2, B-4, C-1, D-3
D. A-3, B-1, C-4, D-2
----------------------------------------
7. What is the lifetime risk for developing cancer in a 45-year-old woman with fibroadenoma,
microcalcifications, absent E-cadherin expression, and atypical lobular hyperplasia on core needle
biopsy?
(or)
What is the lifetime risk of developing cancer in a case of atypical lobular hyperplasia?

A. 15%
B. 28%
C. 8%
D. 5%
----------------------------------------
8. What risk factor does a 25-year-old male with bilateral breast enlargement, despite regular exercise
and diet, likely have, as indicated by palpable glandular tissue beneath the areolas?
(or)
Which of the following is a risk factor for gynecomastia in males?
A. Anabolic Steroids

Page 5

991
B. 45 XO karyotype
C. Fibrocystic changes
D. BRCA 2 gene mutation
----------------------------------------
9. What diagnosis corresponds to a 36-year-old nulliparous woman with enlarged and nodular breasts,
lumpy/bumpy on examination, mammogram showing focal calcifications and the given breast biopsy
findings?

A. Ductal carcinoma in situ


B. Fibroadenoma
C. Fibrocystic change
D. Granulomatous mastitis
----------------------------------------
10. The risk of breast cancer is about 4 to 5 times higher than average in women having which of the
following conditions?
A. Papilloma
B. Atypical ductal hyperplasia
C. Complex fibroadenoma
D. Sclerosing adenosis
----------------------------------------
11. What medical history is probable for a 45-year-old woman with persistent non-bloody nipple
discharge, a painful erythematous subareolar mass, thick white discharge, unremarkable
mammography, and biopsy indicating keratinizing squamous metaplasia of nipple ducts?
(or)
Which medical history is most likely associated with Zuska's disease?
A. Diabetes mellitus
B. Smoking history
C. Recent childbirth
D. Chronic use of oral contraceptives
----------------------------------------

Page 6

992
12. What's the likely condition for a 40-year-old chronic smoker who's been breastfeeding for a month
and presents with a painful, reddish subareolar mass in her right breast since 1 month?
A. Carcinoma breast
B. Periductal mastitis
C. Fat necrosis
D. Granulomatous mastitis
----------------------------------------
13. What is the diagnosis for a 26-year-old breastfeeding woman with nipple redness, pain, and
purulent discharge, which shows Staphylococcus aureus on culture, but no palpable mass or
fluctuation?
A. Breast abscess
B. Lobular carcinoma
C. Acute Mastitis
D. Breast cysts
----------------------------------------

Correct Answers
Question Correct Answer

Question 1 1
Question 2 4
Question 3 3
Question 4 3
Question 5 4
Question 6 1
Question 7 1
Question 8 1
Question 9 3
Question 10 2
Question 11 2
Question 12 2
Question 13 3

Solution for Question 1:


Correct Option A - Tuberculosis-induced granuloma:
The first image shows FNAC suggestive of granulomatous mastitis (showing multinucleated giant cell)
and the second image shows acid-fast bacilli in ZN stain.

Page 7

993
• The pathogenesis underlying the recurrent episodes of a tender, swollen right breast with erythema,
warmth, and pain during breastfeeding in a lactating woman, as evidenced by FNAC (showing
multinucleated giant cells) and special staining (ZN staining showing acid-fast bacilli), is indicative of
localized infection by mycobacterium, suggesting Tuberculosis-induced granuloma.
The pathogenesis underlying the recurrent episodes of a
tender, swollen right breast with erythema, warmth, and pain during breastfeeding in a lactating woman
, as evidenced by FNAC (showing multinucleated giant cells) and special staining (ZN staining showing
acid-fast bacilli), is indicative of localized infection by mycobacterium, suggesting Tuberculosis-induce
d granuloma.
Incorrect Options:
Option B - Hypersensitivity to antigens expressed during lactation :
• This is speculated to be the pathogenesis of granulomatous mastitis in cases where polyangiitis,
sarcoidosis, and tuberculosis have been ruled out.
• However, the presentation described, including the presence of multinucleated giant cells and
acid-fast bacilli, is more suggestive of an infectious process like tuberculosis rather than a
hypersensitivity reaction.
This is speculated to be the pathogenesis of granulomatous mastitis in cases where polyangiitis, sarcoi
dosis, and tuberculosis have been ruled out.
However, the presentation described, including the presence of multinucleated giant cells and acid-fast
bacilli, is more suggestive of an infectious process like tuberculosis rather than a
hypersensitivity reaction.
Option C - Localized infection by Corynebacterium:
• While Corynebacterium species can be associated with breast infections, the specific findings of
acid-fast bacilli seen in the ZN staining are not typical of Corynebacterial infections.
• Corynebacterium species are not commonly associated with acid-fast staining but with gram staining.
They are gram-positive bacilli.
While Corynebacterium species can be associated with breast infections, the specific findings of acid-f
ast bacilli seen in the ZN staining are not typical of Corynebacterial infections.
Corynebacterium species are not commonly associated with acid-fast staining but with gram staining. T
hey are gram-positive bacilli.
Option D - Local Staph infection due to breastfeeding :
• Staphylococcal infections, including Staphylococcus aureus, are common causes of mastitis in
breastfeeding women. However, the presence of multinucleated giant cells and acid-fast bacilli, as
indicated by the FNAC and special staining, is not characteristic of a Staphylococcal infection.
• Staph infections typically present with neutrophils and do not show acid-fast bacilli.
Staphylococcal infections, including Staphylococcus aureus, are common causes of mastitis in breastf
eeding women. However, the presence of multinucleated giant cells and acid-fast bacilli, as indicated b
y the FNAC and special staining, is not characteristic of a Staphylococcal infection.
Staph infections typically present with neutrophils and do not show acid-fast bacilli.

Solution for Question 2:

Page 8

994
Correct Option D - Streptococcus is a less common cause of mastitis:
• The FNAC of an acute mastitis breast shows an accumulation of acute inflammatory cells including
neutrophils, histiocytes, and occasional plasma cells.
• Most commonly due to Staph. aureus and cause single or multiple abscesses in the breast. In case of
a Streptococcal infection, there is cellulitis of the breast.
The FNAC of an acute mastitis breast shows an accumulation of acute inflammatory cells including ne
utrophils, histiocytes, and occasional plasma cells.
Most commonly due to Staph. aureus and cause single or multiple abscesses in the breast. In case of
a Streptococcal infection, there is cellulitis of the breast.
Incorrect Options:
Option A - Cellulitis is a common symptom of Staphylococcal infection:
• This statement is not accurate. Staphylococcal infection, especially Staphylococcus aureus,
commonly causes acute mastitis, which may present with localized inflammation, abscess formation,
and not necessarily cellulitis.
This statement is not accurate. Staphylococcal infection, especially Staphylococcus aureus, commonly
causes acute mastitis, which may present with localized inflammation, abscess formation, and not nec
essarily cellulitis.
Option B - During weaning, the causative organism can enter through cracks in the nipple:
• Mastitis is more commonly associated with breastfeeding and can occur due to milk stasis, blocked
ducts, or bacterial infection. Cracks and fissures in the nipple can contribute to the development of
mastitis, but it is not limited to the weaning period and is more common during the first month of
breastfeeding.
Mastitis is more commonly associated with breastfeeding and can occur due to milk stasis, blocked du
cts, or bacterial infection. Cracks and fissures in the nipple can contribute to the development of mastiti
s, but it is not limited to the weaning period and is more common during the first month of breastfeedin
g.
Option C - Surgical drainage is necessary for the treatment:
• Surgical drainage is not typically the first-line management for uncomplicated cases of mastitis. Initial
management usually involves antibiotic therapy, warm compresses, and continued expression of milk
from the breast. Surgical drainage may be considered in cases of severe abscess formation.
Surgical drainage is not typically the first-line management for uncomplicated cases of mastitis. Initial
management usually involves antibiotic therapy, warm compresses, and continued expression of milk f
rom the breast. Surgical drainage may be considered in cases of severe abscess formation.

Solution for Question 3:


Correct Option C - 0-1%:
• The given histology shows: Dilated ducts with foamy macrophages and periductal scarring; seen in
late stage of mammary duct ectasia.
• The symptoms, age of presentation, and histology support the diagnosis of Duct Ectasia.
• In the context of Duct ectasia: Relative Risk: 0-1% (minimal to no increased risk). Absolute Risk: 3%
(the actual likelihood).

Page 9

995
• Relative Risk: 0-1% (minimal to no increased risk).
• Absolute Risk: 3% (the actual likelihood).
• Therefore, C) 0-1% would be the right answer.
The given histology shows: Dilated ducts with foamy macrophages and periductal scarring; seen in late
stage of mammary duct ectasia.
The symptoms, age of presentation, and histology support the diagnosis of Duct Ectasia.
In the context of Duct ectasia:
• Relative Risk: 0-1% (minimal to no increased risk).
• Absolute Risk: 3% (the actual likelihood).
Relative Risk: 0-1% (minimal to no increased risk).
Absolute Risk: 3% (the actual likelihood).
Therefore, C) 0-1% would be the right answer.
Incorrect Options:
Option A - 1-3%:
• This option falls within a range that might be associated with an absolute lifetime risk of developing
cancer in a non-proliferative breast change.
This option falls within a
range that might be associated with an absolute lifetime risk of developing cancer in a
non-proliferative breast change.
Option B - 5-7%:
• This option falls within a range that might be associated with an absolute lifetime risk of developing
cancer in a proliferative breast change without atypia.
This option falls within a
range that might be associated with an absolute lifetime risk of developing cancer in a
proliferative breast change without atypia.
Option D - 4-5%:
• This option falls within a range that might be associated with an absolute lifetime risk of developing
cancer in a proliferative breast change with atypia.
This option falls within a
range that might be associated with an absolute lifetime risk of developing cancer in a
proliferative breast change with atypia.

Solution for Question 4:


Correct Option C - En Bloc surgical removal:
• The images show the histology of a lactiferous duct: This lactiferous duct has been almost completely
replaced by squamous epithelium. There is only a small area with glandular epithelial cells. Closer view
of a duct space with normal ductal epithelium and the lumen filled with keratin from SMOLD involving

Page 10

996
this duct upstream (magnification 10x).
• Given the clinical presentation and histological findings described, the recommended mode of
treatment would be consistent with managing a condition known as squamous metaplasia of lactiferous
ducts (SMOLD).
• In most cases of squamous metaplasia of the lactiferous duct, En Bloc surgical removal of the
involved duct and contagious fistula tract is curative.
Incorrect Options:
Option A - Easily treated with antibiotics like cephalexin, clindamycin, or co-trimoxazole:
• This would be an appropriate treatment if the diagnosis was Acute mastitis caused by
Staphylococcus. In the case of SMOLD antibiotics only have a therapeutic role if there is evidence of
secondary bacterial infection.
This would be an appropriate treatment if the diagnosis was Acute mastitis caused by Staphylococcus.
In the case of SMOLD antibiotics only have a
therapeutic role if there is evidence of secondary bacterial infection.
Option B - Simple incision and drainage:
• In a SMOLD, a simple incision drains the abscess cavity, but the offending keratinizing epithelium
remains and recurrences tend to become common.
In a SMOLD, a simple incision drains the abscess cavity, but the offending keratinizing epithelium rema
ins and recurrences tend to become common.
Option D - Antibiotic and steroid therapy is required:
• Steroid therapy would be an appropriate mode of treatment for granulomatous mastitis in a person
with an underlying autoimmune condition such as sarcoidosis. Otherwise, antibiotic+steroid therapy
would not be a treatment of choice in SMOLD.
Steroid therapy would be an appropriate mode of treatment for granulomatous mastitis in a person with
an underlying autoimmune condition such as sarcoidosis. Otherwise, antibiotic+steroid therapy would
not be a treatment of choice in SMOLD.

Solution for Question 5:


Correct Option D - All the above:
• The mammogram showing dense breast with cysts and the given histology confirms the diagnosis of
non-proliferative fibrocystic change of the breast
The mammogram showing dense breast with cysts and the given histology confirms the diagnosis of n
on-proliferative fibrocystic change of the breast

Page 11

997
Incorrect Options:
Option A - Diagnosis is confirmed by the disappearance of the mass after FNAC:
• This statement is true regarding non-proliferative fibrocystic change. This is not curative as the “bump”
often recurs.
This statement is true regarding non-proliferative fibrocystic change. This is not curative as the “bump”
often recurs.
Option B - Cysts rupture into adjacent stroma causing chronic inflammation and fibrosis:
• This explains the pathogenesis of fibrosis in non-proliferative fibrocystic disease of breast.
This explains the pathogenesis of fibrosis in non-proliferative fibrocystic disease of breast.
Option C - There is an increase in the number of acini per lobule:
• This statement is also true regarding fibrocystic disease of breast. It is known as adenosis.
This statement is also true regarding fibrocystic disease of breast. It is known as adenosis.

Solution for Question 6:


Correct Option A - A-2, B-1, C-4, D-3
Option A - Complex sclerosing lesion:
• Low power image demonstrating typical architecture of radial scars: sclerotic central core with
entrapped glands radiating outward with varying degrees of proliferative and cystic changes.

Page 12

998
Option B - Papilloma:
• The image shows an intraductal papilloma arising in a small duct. Such lesions are microscopic and
not detected on palpation.

Option C - Epithelial Hyperplasia:


• The image shows an increased number of both luminal and myoepithelial cell types. Irregular lumen. It
is usually an incidental finding.

Page 13

999
Option D - Sclerosing Adenosis:
• The involved Terminal Ductal Lobular Unit (TDLU) is enlarged and the acini are compressed and
distorted by dense stroma. Calcifications are present in some of the lumen.

Solution for Question 7:


Correct Option A - 15%:
• The pathologist reports the lesion to be an atypical lobular hyperplasia.
• The image shows monomorphic small, round, discohesive cells within the fibroadenoma.

Page 14

1000
• The cells present in the biopsy are similar to the cells of lobular carcinoma in situ. However, the
pathologist mentions atypical lobular hyperplasia in the report. This suggests that the level of
involvement might not have been sufficient to confirm a diagnosis of lobular carcinoma in situ.
• Atypical lobular hyperplasia also shows loss of E-cadherin expression.
• The absolute lifetime risk of the development of cancer in ALH ranges from 13-17%.

Incorrect Options:
Option B - 28%:
• In the case of a CIS (Carcinoma In Situ) this would be the right option as the absolute lifetime risk of
the development of cancer in CIS ranges between 25-30%
Option C - 8%:
• This option does not align with the values in case the of ALH
Option D - 5%:
• This option does not align with the values in case the of ALH

Solution for Question 8:


Correct Option A - Anabolic Steroids:
• Gynecomastia is defined as male breast enlargement.
• It is a benign condition caused by an imbalance of estrogen and androgens in males.
• Estrogen stimulates breast tissue proliferation, while androgens inhibit it.
• Gynecomastia develops due to connective tissue proliferation and hyperplasia of the epithelial cells
that line the ducts.
• Lobule formation is extremely rare.
• It can manifest in young people due to anabolic steroid use.

Page 15

1001
• Adult and older people can also suffer from gynecomastia due to multiple causes Alcohol Marijuana
Heroin Antiviral therapy Anabolic steroids Cirrhosis Hypoestrogenism.
• Alcohol
• Marijuana
• Heroin
• Antiviral therapy
• Anabolic steroids
• Cirrhosis
• Hypoestrogenism.
Gynecomastia is defined as male breast enlargement.
It is a benign condition caused by an imbalance of estrogen and androgens in males.
Estrogen stimulates breast tissue proliferation, while androgens inhibit it.
Gynecomastia develops due to connective tissue proliferation and hyperplasia of the epithelial cells tha
t line the ducts.
Lobule formation is extremely rare.
It can manifest in young people due to anabolic steroid use.
Adult and older people can also suffer from gynecomastia due to multiple causes
• Alcohol
• Marijuana
• Heroin
• Antiviral therapy
• Anabolic steroids
• Cirrhosis
• Hypoestrogenism.
Alcohol
Marijuana
Heroin
Antiviral therapy
Anabolic steroids
Cirrhosis
Hypoestrogenism.
Incorrect Options:
Option B - 45 XO karyotype:
• Turner syndrome is associated with the 45 XO karyotype.
• It is not associated with gynecomastia.

Page 16

1002
• Gynecomastia is occasionally associated with Klinefelter syndrome (47 XXY karyotype) Testicular
tumors (Leydig cell, Sertoli cell tumors)
• Klinefelter syndrome (47 XXY karyotype)
• Testicular tumors (Leydig cell, Sertoli cell tumors)
Turner syndrome is associated with the 45 XO karyotype.
It is not associated with gynecomastia.
Gynecomastia is occasionally associated with
• Klinefelter syndrome (47 XXY karyotype)
• Testicular tumors (Leydig cell, Sertoli cell tumors)
Klinefelter syndrome (47 XXY karyotype)
Testicular tumors (Leydig cell, Sertoli cell tumors)
Option C - Fibrocystic Changes:
• Fibrocystic changes include cyst formation and stromal fibrosis.
• It is a benign condition that does not raise the risk of gynecomastia or carcinoma.
Fibrocystic changes include cyst formation and stromal fibrosis.
It is a benign condition that does not raise the risk of gynecomastia or carcinoma.
Option D - BRCA 2 gene mutation:
• It is a common gene mutation associated with male breast cancer.
• It is linked to approximately 35% of breast cancers.
• It contributes to the repair of double-stranded DNA breaks.
It is a common gene mutation associated with male breast cancer.
It is linked to approximately 35% of breast cancers.

Solution for Question 9:


Correct Option C - Fibrocystic change:
• Fibrocystic change is a common benign breast condition
• It is a non-proliferative disease that does not raise the risk of developing carcinoma.
• Fibrocystic changes are Small cysts form due to lobule dilation, which coalesces into larger cysts
Cystic dilation of ducts Fibrosis of stroma Adenosis
• Small cysts form due to lobule dilation, which coalesces into larger cysts
• Cystic dilation of ducts
• Fibrosis of stroma
• Adenosis
• Cysts that haven't ruptured could contain Turbid fluid Semitranslucent brown fluid Blue-coloured fluid
(blue dome)

Page 17

1003
• Turbid fluid
• Semitranslucent brown fluid
• Blue-coloured fluid (blue dome)
• Cysts have either flattened atrophic epithelium or metaplastic apocrine cells as their inner lining.
• Apocrine metaplasia looks like normal sweat gland epithelium and has much eosinophilic cytoplasm.
• Although calcifications are common when the cyst is solitary and firm, it should be investigated further.
• The aspiration of fluid confirms the diagnosis.
• Fibrosis is caused by material leakage into the stroma caused by cyst rupture.
• Adenosis is the increase in acinic per lobule that normally occurs during pregnancy.
Fibrocystic change is a common benign breast condition
It is a non-proliferative disease that does not raise the risk of developing carcinoma.
Fibrocystic changes are
• Small cysts form due to lobule dilation, which coalesces into larger cysts
• Cystic dilation of ducts
• Fibrosis of stroma
• Adenosis
Small cysts form due to lobule dilation, which coalesces into larger cysts
Cystic dilation of ducts
Fibrosis of stroma
Adenosis
Cysts that haven't ruptured could contain
• Turbid fluid
• Semitranslucent brown fluid
• Blue-coloured fluid (blue dome)
Turbid fluid
Semitranslucent brown fluid
Blue-coloured fluid (blue dome)
Cysts have either flattened atrophic epithelium or metaplastic apocrine cells as their inner lining.
Apocrine metaplasia looks like normal sweat gland epithelium and has much eosinophilic cytoplasm.
Although calcifications are common when the cyst is solitary and firm, it should be investigated further.
The aspiration of fluid confirms the diagnosis.
Fibrosis is caused by material leakage into the stroma caused by cyst rupture.
Adenosis is the increase in acinic per lobule that normally occurs during pregnancy.
Incorrect Options:
Option A - Ductal carcinoma in situ:

Page 18

1004
• Ductal carcinoma in situ arises from ducts and does not invade the basement membrane.
• The most common types are comedo, micropapillary, papillary, and cribriform ductal carcinoma in
situ.
Ductal carcinoma in situ arises from ducts and does not invade the basement membrane.
The most common types are comedo, micropapillary, papillary, and cribriform ductal carcinoma in situ.
Option B - Fibroadenoma:
• Fibroadenoma is a benign breast condition.
• It usually manifests as a mobile rubbery mass in the breast's upper outer quadrant.
• It is prevalent in young females.
Fibroadenoma is a benign breast condition.
It usually manifests as a mobile rubbery mass in the breast's upper outer quadrant.
It is prevalent in young females.
Option D - Granulomatous mastitis:
• It is a common symptom of systemic granulomatous diseases like sarcoidosis and tuberculosis.
• Histologically, it appears as granulomas with lipid globules.
• It is treated with antibiotics and steroids.
It is a common symptom of systemic granulomatous diseases like sarcoidosis and tuberculosis.
Histologically, it appears as granulomas with lipid globules.
It is treated with antibiotics and steroids.

Solution for Question 10:


Correct Option B - Atypical ductal hyperplasia:
• Breast cells can develop a precancerous condition called atypical hyperplasia.
• "Atypical hyperplasia" refers to a buildup of aberrant cells in the breast's milk ducts and lobules.
Despite not being cancer, atypical hyperplasia raises the possibility of breast cancer.
• Surgical excision is recommended for all occurrences of atypical ductal hyperplasia (ADH) found on
core needle biopsies for malignancy diagnosis and treatment of lesions.
• The excision of all ADH lesions may lead to overtreatment, which results in invasive surgeries for
benign lesions in many women.
Breast cells can develop a precancerous condition called atypical hyperplasia.
"Atypical hyperplasia" refers to a buildup of aberrant cells in the breast's milk ducts and lobules. Despit
e not being cancer, atypical hyperplasia raises the possibility of breast cancer.
Surgical excision is recommended for all occurrences of atypical ductal hyperplasia (ADH) found on co
re needle biopsies for malignancy diagnosis and treatment of lesions.
The excision of all ADH lesions may lead to overtreatment, which results in invasive surgeries for benig
n lesions in many women.

Page 19

1005
Incorrect Options:
Option A - Papilloma:
• A papilloma is a benign (non-cancerous) tumour arising from an epithelial surface, usually known to
grow outwardly.
• This means that they refrain from spreading widely or growing aggressively. Even though these
tissues are found throughout the body, the growths only appear in specific types of those tissues.
• When papillomas reach the skin, they are frequently referred to as warts and verrucae.
• Whether or not papillomas need to be treated depends on factors such as their size, if there is more
than one, and if they're causing symptoms.
• Papilloma is not cancer and is very unlikely to develop into cancer. However the papilloma cells
should be examined under the microscope after removal.
A papilloma is a
benign (non-cancerous) tumour arising from an epithelial surface, usually known to grow outwardly.
This means that they refrain from spreading widely or growing aggressively. Even though these tissues
are found throughout the body, the growths only appear in specific types of those tissues.
When papillomas reach the skin, they are frequently referred to as warts and verrucae.
Whether or not papillomas need to be treated depends on factors such as their size, if there is more th
an one, and if they're causing symptoms.
Papilloma is not cancer and is very unlikely to develop into cancer. However the papilloma cells should
be examined under the microscope after removal.
Option C - Complex fibroadenoma:
• A type of benign (not cancer) tumor that usually forms in the breast and is made up of fibrous
(connective) and glandular tissue. In complex fibroadenomas, other abnormal tissue changes are also
present.
• Most fibroadenomas don't need to be treated. But doctors might recommend removing them in some
cases, especially if they keep growing or change the shape of the breast.
• Sometimes fibroadenomas stop growing or even shrink on their own without any treatment.
• Simple fibroadenomas have a uniform appearance. Complex fibroadenomas typically have more
diverse tissue and are bigger and irregularly shaped.
• The standard of care for large fibroadenomas and phyllodes tumors is surgery.
A type of benign (not cancer) tumor that usually forms in the breast and is made up of fibrous (connecti
ve) and glandular tissue. In complex fibroadenomas, other abnormal tissue changes are also present.
Most fibroadenomas don't need to be treated. But doctors might recommend removing them in some c
ases, especially if they keep growing or change the shape of the breast.
Sometimes fibroadenomas stop growing or even shrink on their own without any treatment.
Simple fibroadenomas have a uniform appearance. Complex fibroadenomas typically have more divers
e tissue and are bigger and irregularly shaped.
The standard of care for large fibroadenomas and phyllodes tumors is surgery.
Option D - Sclerosing adenosis:
• A benign (not cancerous) condition in which scar-like fibrous tissue is found in the breast lobules (the
glands that make milk).

Page 20

1006
• In sclerosing adenosis, the lobules are larger than normal. This may result in a breast lump that may
be large enough to feel.
• Sclerosing adenosis is a benign breast condition due to normal ageing.
• Breasts comprise lobules (milk-producing glands) and ducts (tubes that carry milk to the nipple),
surrounded by glandular, fibrous and fatty tissue.
A benign (not cancerous) condition in which scar-like fibrous tissue is found in the breast lobules (the gl
ands that make milk).
In sclerosing adenosis, the lobules are larger than normal. This may result in a
breast lump that may be large enough to feel.
Sclerosing adenosis is a benign breast condition due to normal ageing.
Breasts comprise lobules (milk-producing glands) and ducts (tubes that carry milk to the nipple), surrou
nded by glandular, fibrous and fatty tissue.

Solution for Question 11:


Option B: Smoking history
• The description of a sterile discharge, palpable subareolar mass, and the biopsy findings showing
keratinizing squamous metaplasia are characteristic of Zuska's disease or (SMOLD- Squamous
Metaplasia Of Lactiferous Ducts).
• More than 90% of the afflicted are smokers.
• It has also been suggested that a relative deficiency of vitamin A associated with smoking or toxic
substances in tobacco smoke alters the differentiation of ductal epithelium.
Option A: Diabetes mellitus
• Diabetes mellitus is not typically associated with squamous metaplasia of the nipple ducts.
• It is more commonly linked to conditions such as diabetic mastopathy, which involves fibrous masses
in the breast, rather than squamous metaplasia.
Option C: Recent childbirth
• While hormonal changes during pregnancy and lactation can affect the breast tissue, recent childbirth
is not specifically associated with squamous metaplasia of the nipple ducts.
• This condition is more commonly linked to factors like smoking.
Option D: Chronic use of oral contraceptives
• Long-term use of oral contraceptives is not typically associated with squamous metaplasia of the
nipple ducts.
• Breast changes related to oral contraceptives usually involve benign conditions like fibroadenomas or
cysts.

Solution for Question 12:


Correct Option B - Periductal mastitis:

Page 21

1007
• She is most likely suffering from periductal mastitis.
• Periductal mastitis occurs when the ducts (tubes) under the nipple become inflamed and infected.
• Aka Zuska's disease /Recurrent subareolar abscess/Squamous metaplasia of lactiferous ducts
• Smoking has been implicated as a risk factor. Smokers → relative Vit A deficiency → Squamous
metaplasia → Obstruction → Dilation of duct
• Smokers → relative Vit A deficiency → Squamous metaplasia → Obstruction → Dilation of duct
• Features: Subareolar mass Pain and redness Nipple retraction Fistula formation May mimic
malignancy
• Subareolar mass
• Pain and redness
• Nipple retraction
• Fistula formation
• May mimic malignancy
She is most likely suffering from periductal mastitis.
Periductal mastitis occurs when the ducts (tubes) under the nipple become inflamed and infected.
Aka Zuska's disease /Recurrent subareolar abscess/Squamous metaplasia of lactiferous ducts
Smoking has been implicated as a risk factor.
• Smokers → relative Vit A deficiency → Squamous metaplasia → Obstruction → Dilation of duct
Smokers → relative Vit A deficiency → Squamous metaplasia → Obstruction → Dilation of duct
Features:
• Subareolar mass
• Pain and redness
• Nipple retraction
• Fistula formation
• May mimic malignancy
Subareolar mass
Pain and redness
Nipple retraction
Fistula formation
May mimic malignancy
Incorrect Options:
Option A - Carcinoma breast:
• Breast carcinomas in situ can be classified as either ductal carcinoma in situ (DCIS) or Paget disease
of the nipple.
• DCIS is a condition when the lining of a breast duct has atypical cells.
• Carcinoma can be In situ or invasive depending on an intact basement membrane (in the former) and
a breech in basement membrane causing spread to the adjacent structures (in the latter).

Page 22

1008
• Symptoms: Thickening or swelling of part of the breast New lump in the breast or underarm (armpit).
Irritation or dimpling of breast skin.
• Thickening or swelling of part of the breast
• New lump in the breast or underarm (armpit).
• Irritation or dimpling of breast skin.
Breast carcinomas in situ can be classified as either ductal carcinoma in situ (DCIS) or Paget disease
of the nipple.
DCIS is a condition when the lining of a breast duct has atypical cells.
Carcinoma can be In situ or invasive depending on an intact basement membrane (in the former) and a
breech in basement membrane causing spread to the adjacent structures (in the latter).
Symptoms:
• Thickening or swelling of part of the breast
• New lump in the breast or underarm (armpit).
• Irritation or dimpling of breast skin.
Thickening or swelling of part of the breast
New lump in the breast or underarm (armpit).
Irritation or dimpling of breast skin.
Option C - Fat necrosis:
• Fat necrosis is the death of fat tissue resulting from trauma or surgical complications.
• It can lead to the formation of hard lumps resembling tumors beneath the skin.
• Despite its tumor-like appearance, fat necrosis is harmless and typically resolves on its own.
• Trauma to the breast tissue, such as during a biopsy or surgery, is a common cause of fat necrosis.
Option D - Granulomatous mastitis:
• Granulomatous lobular mastitis (GM) is a persistent, inflammatory, and frequently idiopathic breast
disease.
• It can mimic cancer and is challenging to diagnose, causing long-term pain and distress for affected
patients despite being benign.
• Granulomatous mastitis can be either idiopathic or secondary to various conditions such as
tuberculosis, infections, sarcoidosis, and granulomatosis with polyangiitis.

Solution for Question 13:


Correct Option C - Acute Mastitis:
• Acute mastitis is an inflammatory condition of the breast tissue that is often associated with infection,
typically by bacteria such as Staphylococcus aureus. It can occur in breastfeeding mothers due to
factors like milk stasis, cracked nipples, or poor breastfeeding technique. This infection can lead to
symptoms such as breast pain, redness, swelling, and the presence of purulent discharge from the
nipple. The culture of the discharge indicating Staphylococcus aureus is consistent with a bacterial
infection commonly seen in acute mastitis.

Page 23

1009
• Options a, b, and d are less likely in the context of purulent nipple discharge and a positive culture for
Staphylococcus aureus:
Acute mastitis is an inflammatory condition of the breast tissue that is often associated with infection, ty
pically by bacteria such as Staphylococcus aureus. It can occur in breastfeeding mothers due to factor
s like milk stasis, cracked nipples, or poor breastfeeding technique. This infection can lead to symptom
s such as breast pain, redness, swelling, and the presence of purulent discharge from the nipple. The c
ulture of the discharge indicating Staphylococcus aureus is consistent with a
bacterial infection commonly seen in acute mastitis.
Options a, b, and d are less likely in the context of purulent nipple discharge and a
positive culture for Staphylococcus aureus:
Incorrect Options:
Option A - Breast abscess: A breast abscess is a
collection of pus within the breast tissue. It may occur as a
complication of untreated or severe mastitis. A breast abscess usually presents as a discrete tender m
ass that is either tense or shows fluctuation. However, the current symptoms described primarily sugge
st acute mastitis rather than a fully developed abscess.
Option B - Lobular carcinoma: Breast cancer, including lobular carcinoma, may present with nipple dis
charge, but it is less commonly associated with purulent discharge. Breast cancer diagnoses would req
uire further evaluation and imaging studies.
Option D - Breast cysts: While breast cysts can cause nipple discharge, it is usually serous or clear, no
t purulent like what is typically seen in acute mastitis.

Page 24

1010
Basics of CNS and Brain Tumors
1. What is the most likely diagnosis in a 22-year-old patient with hearing loss complaints shows
sensorineural hearing loss on audiometry and multiple meningiomas and bilateral vestibular
schwannomas on head MRI.
A. Neurofibromatosis type 2 (NF2)
B. Tuberous sclerosis
C. Von Hippel Lindau (VHL) disease
D. Sturge-Weber syndrome
----------------------------------------
2. A 45-year-old patient presented with a history of seizures. Imaging showed a lesion in the temporal
lobe. Based on the characteristic histological features, what is the most likely diagnosis?

A. Glioblastoma
B. Pilocytic astrocytoma
C. Oligodendroglioma
D. Medulloblastoma
----------------------------------------
3. Which of the following is the most common primary benign brain tumor in children?
A. Pilocytic astrocytoma
B. Glioblastoma
C. Medulloblastoma
D. Ependymoma
----------------------------------------
4. What is the likely diagnosis for a 64-year-old patient admitted for seizures, with an MRI suggestive of
a tumor, a stereotactic brain biopsy revealing a histological pattern given below, and high vascularity in
the tumor cells?

1011
A. Glioblastoma
B. Meningiomas
C. Cerebellopontine angle schwannoma
D. Ependymoma
----------------------------------------
5. What is the likely diagnosis for a 15-year-old girl with developmental delay, absence of menarche,
frequent headaches, whose ophthalmic exam shows bitemporal heteronymous hemianopia and her CT
Brain scan reveals a suprasellar mixed tumor with occasional calcifications, and the following
histological findings?

A. Lactotroph adenoma
B. Pituitary carcinoma
C. Papillary craniopharyngioma
D. Adamantinomatous craniopharyngioma
----------------------------------------
6. Which of the following is a characteristic histological feature of ependymoma ?
A. Serpentine necrosis
B. Homer-Wright Pseudorosettes
C. Perivascular pseudorosettes
D. None of the above
----------------------------------------

Page 2

1012
7. Which of the following statements is true regarding Meningiomas?
A. Radiation and Neurofibromatosis are risk factors
B. Their characteristic histological feature is psammoma bodies
C. They are progesterone receptor positive.
D. All of the above
----------------------------------------
8. A 40-year-old woman has a painless, slowly growing mass on her forearm. The mass is well-defined
and shows a typical dumbbell shape upon examination. Microscopic analysis shows spindle cells
arranged in a distinct pattern with nuclei aligned in rows, leaving gaps without nuclei. The patient has
no neurological symptoms and genetic testing confirms involvement of the NF2 gene. What is the likely
diagnosis, and what microscopic feature is characteristic of this tumor?
(or)
What is the likely diagnosis for a painless, slowly growing forearm mass in a 40-year-old woman with a
well-defined dumbbell shape, absence of neurological symptoms, and involvement of the NF2 gene,
and what microscopic feature is characteristic of this tumor?
A. Rhabdomyosarcoma; Alveolar pattern
B. Schwannoma; Verocay bodies
C. Meningioma; Psammoma bodies
D. Ependymoma; Perivascular pseudorosettes
----------------------------------------
9. What is the key feature distinguishing the disease in a 44-year-old man with involuntary facial
grimacing and finger movements, reminiscent of his mother's symptoms that started around the same
age and progressed to dancing movements, arm, and leg writhing, coma, and death, similar to his
maternal grandfather's experience?
A. Degeneration of upper and lower motor neurons
B. Dopamine depletion and depigmentation of the substantia nigra
C. Increased number of trinucleotide repeats in a gene on chromosome 4
D. Neurofibrillary tangles and amyloid plaques in the cerebral cortex
----------------------------------------
10. Choose the correct statement regarding the pathogenesis of Amyotrophic Lateral Sclerosis.
A. Loss of upper motor neurons in the cerebral cortex and lower motor neurons in the spinal cord and
brainstem.
B. Atrophy of the caudate nucleus
C. Loss of dopaminergic neurons from the substantia nigra
D. Deposition of Tau protein
----------------------------------------
11. Which of the following nuclei are commonly affected in Alzheimer's disease?
A. Nucleus Basalis of Meynert
B. Superior salivary nucleus

Page 3

1013
C. Ventromedial nucleus of the thalamus
D. Midline raphe nucleus
----------------------------------------
12. What are the chromosomes involved in early-onset Alzheimer's disease?
A. 1, 14, 21
B. 1, 13, 21
C. 3, 14, 20
D. 3, 14, X
----------------------------------------
13. What disease presents a greater risk for a 52-year-old male exhibiting memory loss and personality
changes, leading to death, as indicated by a brain biopsy given below?

A. Klinefelter syndrome
B. Noonan syndrome
C. Down's syndrome
D. Turner syndrome
----------------------------------------
14. Which of the following protein deposition/defect is associated with corresponding
neurodegenerative disease?
A. Tau protein is associated with amyotrophic lateral sclerosis
B. FUS protein is associated with Huntington's disease
C. Polyglutamine aggregates are associated with Alzheimer's disease
D. Alpha-synuclein is associated with Parkinson's disease
----------------------------------------
15. Choose the correct option that defines the substitution at codon 178 of PrP in Fatal Familial
Insomnia (FFI)?
A. Aspargine is replaced by valine
B. Aspargine is replaced by aspartate
C. Aspartate is replaced by Aspargine

Page 4

1014
D. Methionine replaces Aspargine
----------------------------------------
16. Which protein is primarily associated with the formation of Lewy bodies?
A. LRRK2
B. PINK
C. Alpha-synuclein
D. Parkin
----------------------------------------
17. Which is not a feature of adamantinomatous variant of craniopharyngioma?
A. Nests and cords of squamous epithelium embedded in the spongy reticulum
B. Compact, lamellar keratin formation (“wet keratin”)
C. Cysts containing cholesterol-rich fluid that resembles ‘machine oil’
D. Papillae lined by well-differentiated squamous epithelium
----------------------------------------

Correct Answers
Question Correct Answer

Question 1 1
Question 2 3
Question 3 1
Question 4 1
Question 5 4
Question 6 3
Question 7 4
Question 8 2
Question 9 3
Question 10 1
Question 11 1
Question 12 1
Question 13 3
Question 14 4
Question 15 2
Question 16 3
Question 17 4

Page 5

1015
Solution for Question 1:
Correct Option A - Neurofibromatosis type 2:
• NF2 is characterized by the presence of bilateral eighth-nerve schwannomas, multiple meningiomas,
and ependymomas of the spinal cord, among other tumors. The presence of a specific type of cataract,
known as juvenile posterior subcapsular lenticular opacity, is another characteristic feature of NF2.
• The condition is caused by mutations in the NF2 gene, located on chromosome 22q. This gene
encodes for the neurofibromin 2 protein, also known as merlin protein, which acts as a tumor
suppressor. Mutations in the NF2 gene lead to the loss of function of the merlin protein, contributing to
the development of tumors observed in NF2 patients.
The condition is caused by mutations in the NF2 gene, located on chromosome 22q. This gene encode
s for the neurofibromin 2 protein, also known as merlin protein, which acts as a tumor suppressor. Mut
ations in the NF2 gene lead to the loss of function of the merlin protein, contributing to the development
of tumors observed in NF2 patients.
Incorrect Options:
Option B - Tuberous sclerosis:
• Tuberous sclerosis is an autosomal dominant disorder with variable expression.
• Due to mutation in tumor suppressor genes TSC1 on chromosome 9 (hamartin) and TSC2 on
chromosome 16 (tuberin).
• TSC is characterized by the presence of hamartomas in CNS and skin, angiofibroma, mitral
regurgitation, and leaf spots, cardiac rhabdomyoma, intellectual disability, renal angiomyolipoma,
seizures, and shagreen patches are seen in patients with disease.
• There is an increased incidence of subependymal giant cell astrocytomas and ungual fibromas.
Angiofibromas:

Option C - Von Hippel Lindau (VHL) disease:


• VHL disease is characterized by the development of multiple tumors and cysts in various organs,
including the brain, spinal cord, kidneys, and pancreas. While VHL can involve the central nervous
system, it typically presents with hemangioblastomas in the cerebellum, brainstem, and spinal cord,

Page 6

1016
rather than bilateral vestibular schwannomas.
• It is an autosomal dominant disorder due to the deletion of the VHL gene on chromosome 3p. VHL
ubiquitinates hypoxia-inducible factor 1a.
Option D - Sturge-Weber syndrome:
• Sturge-Weber syndrome is a congenital nonhereditary anomaly of neural crest derivatives.
• It is a neurocutaneous disorder characterized by a port-wine stain on the face, glaucoma, ipsilateral
leptomeningeal angioma with calcifications which can cause seizures, intellectual disability and
episcleral hemangioma causing increased intraocular pressure and early-onset glaucoma.
• Somatic mosaicism of an activating mutation in one copy of the GNAQ gene.
• Capillary vascular malformation gives port-wine stain (nevus flammeus or non-neoplastic birthmark) in
CN V1/V2 distribution.

Solution for Question 2:


Correct Option C - Oligodendroglioma:
• The likely diagnosis in this patient is oligodendroglioma:
• Oligodendroglioma is a type of brain tumor (glioma) that arises from oligodendrocytes.
• Most commonly presents in the fourth and fifth decades.
• The lesion is mostly found in the cerebral hemispheres (mainly in the temporal or frontal lobes).

• The tumors are composed of spherical nuclei containing finely granular chromatin (similar to normal
oligodendrocytes) surrounded by a clear halo of vacuolated cytoplasm, giving a 'fried egg' appearance.
• The tumor typically contains a delicate network of anastomosing capillaries ('chicken-wire capillaries).
• Perineuronal Satellitosis is a feature of oligodendroglioma, it refers to the presence of small,
neoplastic cells clustered around neuronal cell bodies.
• Perinuclear halos giving rise to a 'fried egg' appearance Perineuronal satellitosis.

Page 7

1017
Perinuclear halos giving rise to a 'fried egg' appearance
Perineuronal satellitosis.

Page 8

1018
Incorrect Options:
Option A - Glioblastoma:
• Glioblastoma is a Grade IV astrocytoma.
• It is a common, highly malignant primary brain tumor with one-year median survival.
• It is found in the cerebral hemispheres.
• It can cross the corpus callosum "butterfly glioma" associated with EGFR amplification.
• It is of astrocyte origin, glial fibrillary acidic protein (GFAP) positive.
• Glioblastoma typically shows areas of necrosis, nuclear atypia, and mitotic activity not seen in the
provided image
• "Pseudopalisading" pleomorphic tumor cells with central border areas of necrosis, hemorrhage,
and/or microvascular proliferation are seen in this tumor.

Option B - Pilocytic astrocytoma:

Page 9

1019
• Pilocytic astrocytoma is a low-grade astrocytoma. Most common primary brain tumor in childhood.
• It has a benign course and a good prognosis
• Usually well-circumscribed and often found in the posterior fossa (e.g., cerebellum), but supratentorial
can also be found.
• Histology shows it is of astrocyte origin, GFAP positive, with bipolar neoplastic cells with hairlike
projections.
• Associated with microcysts and Rosenthal fibres (eosinophilic, corkscrew fibres)
• Pilocytic astrocytoma typically presents with a more compact, less cellular pattern with Rosenthal
fibers and eosinophilic granular bodies, which are not evident in the image provided in the scenario.
Option D - Medulloblastoma:
• Medulloblastoma is the most common malignant brain tumor in childhood.
• It commonly involves the cerebellum and can compress the 4th ventricle causing non-communicating
hydrocephalus, headaches, and papilledema.
• It can involve the cerebellar vermis causing truncal ataxia.
• It can send "drop metastases" to the spinal cord.
• Histology shows it is a form of primitive neuroectodermal tumor (PNET).
• "Homer-Wright rosette" (small blue cells surrounding the central area of the neuropil)
• It is positive for synaptophysin.

Solution for Question 3:


Correct Option A - Pilocytic astrocytoma:
• The most common primary benign brain tumor in children is Pilocytic Astrocytoma
• Pilocytic astrocytoma, typically presents as a cystic mass, often found in the cerebellum or third
ventricle.
• Microscopically, it is characterized by bipolar cells with hair-like processes, along with the presence of
Rosenthal fibers and eosinophilic granular bodies.
• This tumor is associated with NF-1 and BRAF mutations, and it rarely exhibits p53 mutations.
• Overall, pilocytic astrocytoma is a grade I astrocytoma with distinct histopathological features and a
generally favorable prognosis.
Incorrect Options:
Option B - Glioblastoma:
• Glioblastoma is a Grade IV astrocytoma.
• It is a common, highly malignant primary brain tumor with one-year median survival.
• It is found in the cerebral hemispheres.
• It can cross the corpus callosum, so is called a "butterfly glioma"
• It is of astrocyte origin, glial fibrillary acidic protein(GFAP) positive , and is associated with EGFR
amplification.

Page 10

1020
• Pseudopalisading of tumor cells around areas of necrosis, pleomorphic tumor cells, and
central area of necrosis, hemorrhage, and/or microvascular proliferation are classic hallmarks of
glioblastoma multiforme.
Option C - Medulloblastoma:
• Most common malignant brain tumor in childhood.
• It commonly involves the cerebellum and can compress the 4th ventricle causing non-communicating
hydrocephalus causing, headaches and papilledema.
• It can involve the cerebellar vermis causing truncal ataxia.
• It can send "drop metastases" to the spinal cord.
• Histology shows it is a form of primitive neuroectodermal tumor (PNET) and it stains positive for
synaptophysin.
• Homer-Wright rosettes are considered a histological hallmark of medulloblastomas.
Option D - Ependymoma:
• Most commonly found in the fourth ventricle.
• Histologically, it is of ependymal origin.
• Characteristic "peri-vascular pseudorosettes" are seen.
• Rod-shaped blepharoblasts (basal ciliary bodies) are found near the nucleus

Solution for Question 4:


Correct Option A - Glioblastoma:
• Glioblastoma is a Grade IV astrocytoma.
• It is a common, highly malignant primary brain tumor. Patients with this tumor have a median survival
of one year.
• It is found in the cerebral hemispheres.
• It can cross the corpus callosum “butterfly glioma” associated with EGFR amplification.
• It is of astrocyte origin, and hence it is glial fibrillary acidic protein (GFAP) positive.
• The histological image in the scenario shows serpentine necrosis surrounded by pseudopalisading
tumor cells, image also shows a glomerular body.

Page 11

1021
• Pseudopalisading, i.e. necrosis surrounded by tumour cells.

• Glomeruloid body, i.e. endothelial cell proliferation


Incorrect Options:
Option B - Meningiomas:
• Meningiomas typically have a distinct histological appearance with characteristic whorled architecture
and psammoma bodies, which do not match the given histology.
• Microscopy reveals Spindle cells are concentrically arranged in a whorled pattern; with laminated
calcifications called psammoma bodies.
Option C: Cerebellopontine angle schwannoma
• Schwannomas arising in the cerebellopontine angle are usually encapsulated tumors composed of
Schwann cells. They typically exhibit a biphasic pattern with Antoni A and Antoni B areas, which is
different from the histological pattern seen in glioblastomas

Page 12

1022
Option D - Ependymoma:
• Ependymomas arise from ependymal cells lining the ventricular system. They often exhibit
perivascular pseudorosettes and characteristic rod-shaped blepharoblasts, which are not typically
observed in glioblastomas.

Solution for Question 5:


Correct Option D - Adamantinomatous Craniopharyngioma:
• The presenting features of the patient are suggestive of a pituitary pathology causing compression
symptoms.
• The CT Brain showing a suprasellar mixed tumor with occasional calcifications is consistent with
adamantinomatous craniopharyngioma, which commonly presents as a solid-cystic lesion with
dystrophic calcifications.

• Adamatinomatous craniopharyngioma is characterized by nests or cords of stratified squamous
epithelium within a spongy "reticulum," with palisading of the squamous epithelium at the periphery.
Compact, lamellar keratin formation (known as "wet keratin") is a distinctive feature. Dystrophic
calcification, cyst formation containing cholesterol-rich fluid, and fibrosis are commonly observed.

Incorrect Options:
Option A - Lactotroph adenoma:
• Lactotroph adenomas typically present with symptoms related to hyperprolactinemia, such as
galactorrhea and menstrual irregularities. They are characterized histologically by chromophobe cells
Option B - Pituitary carcinoma:
• Pituitary carcinomas are extremely rare and are characterized by invasive growth and distant
metastasis. They usually present with aggressive symptoms and are diagnosed based on histological
evidence of invasion into surrounding tissues or distant metastasis
Option C - Papillary craniopharyngiomas:

Page 13

1023
• Papillary craniopharyngiomas are a subtype of craniopharyngioma characterized by papillary
structures lined by columnar epithelium. This subtype is less common than the adamantinomatous type
and typically occurs in adults. However, the histological description provided in the scenario matches
more closely with the features of the adamantinomatous subtype.

Solution for Question 6:


Correct Option C -Perivascular pseudorosettes:
• Ependymomas are tumors derived from ependymal cells, which line the ventricles of the brain and the
central canal of the spinal cord.
• A characteristic histological feature of ependymomas is the presence of perivascular pseudorosettes,
where tumor cells form a rosette-like arrangement around blood vessels.
• These perivascular pseudorosettes are a key diagnostic feature observed under the microscope.

• The image shows a perivascular pseudorosette.

Incorrect Options:
Option A - Serpentine necrosis:
• Serpentine necrosis refers to a particular pattern of necrosis observed in some tumors, particularly
glioblastomas.
• It is characterized by a winding or serpentine-shaped necrotic area within the tumor tissue.
• This pattern is not typically associated with ependymomas, although necrosis may be observed in
some cases.
Option B - Homer-Wright pseudorosettes:
• Homer-Wright pseudorosettes are a histological feature commonly seen in medulloblastomas.
• These pseudorosettes consist of tumor cells arranged around a central space, resembling a rosette.

Page 14

1024
• While ependymomas may exhibit rosette-like structures, they are typically perivascular
pseudorosettes rather than Homer-Wright pseudorosettes.
Option D - None of the above:
• This is not true as option C is the right answer.

Solution for Question 7:


Correct Option D - All of the above:
• Radiation exposure and neurofibromatosis are indeed established risk factors for meningiomas.
Radiation exposure, especially to the head, is a well-known risk factor, and individuals with
neurofibromatosis type 2 (NF2) have a higher predisposition to develop meningiomas.
• Psammoma bodies are indeed characteristic histological features often observed in meningiomas.
These are concentrically laminated calcific structures found within the tumor tissue and are considered
a hallmark of meningiomas.

• Meningiomas commonly express progesterone receptors, which can be demonstrated
immunohistochemically. This hormone receptor positivity is utilized in diagnostic pathology and may
have implications for treatment options, such as hormonal therapy.

Solution for Question 8:


Correct Option B - Schwannoma; Verocay bodies:
The likely diagnosis in this patient is Schwannoma and the characteristic microscopic feature is the pre
sence of Verocay bodies.
Schwannoma Characteristics:

Page 15

1025
• Clinical Presentation: Slowly enlarging, painless mass with a classic dumbbell-shaped appearance is
a characteristic feature of schwannomas.
• Microscopic Examination: The presence of spindle cells arranged in a distinctive pattern with tumor
nuclei aligned in palisading rows leaving anuclear zones is consistent with schwannomas. This is
called “Antoni A area”.
• Verocay Bodies: These are characteristic structures seen in schwannomas under microscopic
examination. Verocay bodies consist of alternating hypercellular and hypocellular areas, representing
the compacted spindle cell palisades.

Association with NF2 Gene:


• The confirmation of NF2 gene involvement through genetic testing aligns with the hereditary nature of
schwannomas, which is a common feature in individuals with neurofibromatosis type 2 (NF2).
Incorrect Options:
Option A - Rhabdomyosarcoma; Alveolar pattern:
• Rhabdomyosarcoma is a malignant tumor derived from skeletal muscle cells.
• The presented case describes a slowly enlarging, painless mass on the forearm, which is not
characteristic of rhabdomyosarcoma.
• The microscopic features mentioned, such as spindle cells with palisading nuclei and anuclear zones,
are not typical of rhabdomyosarcoma, which exhibits a different histological pattern.
Option C - Meningioma; Psammoma bodies:
• Meningiomas are typically dura-based intracranial tumors and are not commonly found in the forearm.
• Psammoma bodies, calcified structures, are characteristic of meningiomas but are not mentioned in
the case description.
Option D - Ependymoma; Perivascular pseudorosettes:
• Ependymomas are tumors arising from ependymal cells lining the ventricles of the brain and the
spinal canal.
• The clinical presentation and location described (forearm) are not typical for ependymomas.

Page 16

1026
• Perivascular pseudorosettes, a characteristic feature of ependymomas, are not mentioned in the
microscopic examination.

Solution for Question 9:


Correct Option C - Increased number of trinucleotide repeats in a gene on chromosome 4:
This is a case of Huntington's disease.
• It is an autosomal dominant disorder
• It causes fatal, progressive degeneration and atrophy of the Striatum (Caudate nucleus and
Putamen).
• It is characterised by an increased number of trinucleotide repeats (CAG) in the Huntington gene on
the short arm of chromosome 4.
Incorrect Options:
Option A - Degeneration of upper and lower motor neurons:
• Degeneration of the upper and lower motor neurons is characteristic of amyotrophic lateral sclerosis.
• ALS initially presents with asymmetric weakness in hands, muscle cramping, and fasciculations,
progressing to widespread muscle weakness and respiratory involvement.
• Variants include progressive muscular atrophy, primary lateral sclerosis, and bulbar ALS, each with
distinct clinical features and prognoses.
• ALS can also involve cortical regions, leading to frontotemporal dementia, with genetic and
pathological overlap between ALS and frontotemporal lobar degeneration.
Option B - Dopamine depletion and depigmentation of the substantia nigra:
• Dopamine depletion and depigmentation of the substantia nigra are characteristic of Parkinson's
disease.
• Clinical features include tremors, rigidity, akinesia/bradykinesia, and postural instability.
• Dementia is a common feature of late disease.
Option D - Neurofibrillary tangles and amyloid plaques in the cerebral cortex:
• Neurofibrillary tangles and amyloid plaques are found in Alzheimer's disease.
• Alzheimer’s disease is the most common cause of dementia and movement disorders are not seen

Solution for Question 10:


Correct Option A - Loss of upper motor neurons in the cerebral cortex and lower motor neurons in the s
pinal cord and brainstem:
• This refers to amyotrophic lateral sclerosis/Lou Gehrig's disease
• Loss of upper motor neurons in the cerebral cortex and lower motor neurons in the spinal cord and
brainstem.

Page 17

1027
• The diagnosis is based on clinical criteria in which there are both Upper and lower motor neuron signs
with progressive disease.
Incorrect Options:
Option B - Atrophy of the caudate nucleus:
• Degeneration & atrophy of the caudate nucleus is associated with Huntington's disease.
• It is an autosomal dominant disorder
• It is a fatal, progressive degeneration and atrophy of the Striatum (Caudate nucleus and Putamen).
• Characterised by an increased number of trinucleotide repeats (CAG) in the Huntington gene on the
short arm of chromosome 4.
Option C - Loss of dopaminergic neurons from the substantia nigra:
• Parkinson's disease is characterized by a degenerative loss of dopaminergic neurons in the
substantia nigra of the basal ganglia.
• In Parkinson's disease, histology reveals loss of pigmented neurons in the substantia nigra and round,
eosinophilic inclusions of an α-synuclein protein (Lewy bodies) in affected neurons.
• Clinical features are tremors, rigidity, akinesia/bradykinesia and postural instability.
• Dementia is a common feature of the late disease.

Option D - Deposition of Tau protein:


• Tau is a microtubule-associated protein
• Neurofibrillary tangles are intracellular aggregates of fibres composed of hyperphosphorylated tau
protein
• These are seen in Alzheimer's disease.
• Alzheimer’s disease is the most common cause of dementia.

Solution for Question 11:


Correct Option A - Nucleus Basalis of Meynert:
• The Nucleus Basalis of Meynert is commonly affected in Alzheimer's disease.
• Histologically, Alzheimer's disease is characterized by, a significant loss of cholinergic neurons in the
Nucleus Basalis of Meynert, contributing to the cognitive decline observed in affected individuals. This
loss of neurons is associated with the formation of characteristic amyloid plaques and neurofibrillary
tangles in the brain, which are hallmark pathological features of Alzheimer's disease.
Incorrect Options:
Options B, C and D are incorrect.

Solution for Question 12:

Page 18

1028
Correct Option A - 1, 14, 21:
• The chromosomes involved with early onset Alzheimer’s disease are 1,14 and 21.
• Young-onset (also called early-onset) Alzheimer's is an uncommon form of dementia that
affects people younger than age 65.
• About 5% to 6% of people with Alzheimer's disease develop symptoms before age 65.
• Implicated genes in early-onset Alzheimer's disease include presenilin 1 and 2 (on chr 14 & 1) and
APP gene (on chr 21)
Incorrect Options:
Options B, C and D are incorrect. Refer to the explanation of Option A.

Solution for Question 13:


Correct Option C - Down's syndrome:
• The image shows neurofibrillary tangles.
• Neurofibrillary tangles are abnormal aggregates of hyperphosphorylated Tau protein found inside
nerve cells (neurons). These tangles are a hallmark neuropathological feature of Alzheimer's disease.
• Individuals with Down's syndrome have an extra copy of chromosome 21, which carries the gene for
amyloid precursor protein (APP). This genetic abnormality leads to an increased production of
amyloid-beta peptide, a key component of amyloid plaques found in the brains of individuals with
Alzheimer's disease. Consequently, individuals with Down's syndrome are at a higher risk of
developing Alzheimer's pathology and symptoms at an earlier age compared to the general population.
Therefore, Down's syndrome is a significant risk factor for Alzheimer's disease.
Incorrect Options:
Options A, B and D are incorrect. Refer to the explanation of Option C.

Solution for Question 14:


Correct Option D - Alpha-synuclein is associated with Parkinson's disease:
• Parkinson's disease is characterized by the accumulation of abnormal protein aggregates, primarily
composed of alpha-synuclein, within nerve cells of certain brain regions. These aggregates are known
as Lewy bodies and Lewy neurites, and their presence in the brain is a pathological hallmark of
Parkinson's disease.
• α-synuclein is a small protein normally found in presynaptic nerve terminals, where it plays a role in
regulating neurotransmitter release. In Parkinson's disease, α-synuclein misfolds and aggregates,
leading to neuronal dysfunction and cell death, contributing to the symptoms of the disease.
• Tau protein is associated with other neurodegenerative disorders such as Alzheimer's disease, while
FUS protein is implicated in amyotrophic lateral sclerosis (ALS), and polyglutamine aggregates are
linked to Huntington's disease.
Options A, B and C are incorrect. Refer to the correct option.

Page 19

1029
Solution for Question 15:
Correct Option B - Aspargine is replaced by aspartate:
• In Fatal Familial Insomnia (FFI), a rare genetic prion disease, there is a mutation in the PRNP gene,
which encodes the prion protein (PrP).
• In FFI, the amino acid asparagine is replaced by aspartate at codon 178.
• Individuals with FFI experience severe sleep disturbances, autonomic dysfunction, and psychiatric
symptoms including hallucinations, depression, anxiety, etc.
Incorrect Options:
Option A - Aspargine is replaced by valine:
• This is incorrect because the given mutation involves the substitution of aspartate (D) for asparagine
(N), not valine.
Option C - Aspartate is replaced by Aspargine:
• This is incorrect. The mutation involves the substitution of asparagine (N) for aspartate (D), not the
other way around.
Option D - Methionine replaces Aspargine:
• This option is incorrect as the mutation described involves the substitution of asparagine (N) for
aspartate (D), not methionine.

Solution for Question 16:


Correct Option C - Alpha-synuclein:
Alpha-synuclein and Lewy Bodies:
• Alpha-synuclein is a protein that is normally present in the brain.
• In Parkinson's disease, there is an abnormal accumulation of alpha-synuclein, leading to the formation
of aggregates known as Lewy bodies.

• The image on the left shows a normal mid-brain section and the image on the right shows that of
Parkinson's patient (showing less pigmentation corresponding to the area of substantia nigra due to
degeneration of neurons in this part).

Page 20

1030
Incorrect Options:
Option A - LRRK2, Option B - PINK and Option D - Parkin:
• LRRK2, PINK1, and Parkin are genes associated with Parkinson's disease, but they are not primarily
linked to the formation of Lewy bodies.
• Mutations in LRRK2 increase the risk of Parkinson's and may involve other pathological processes,
while PINK1 mutations are also implicated in certain cases of the disease.
• Parkin mutations are associated with dysfunction in the ubiquitin-proteasome system and
mitochondrial quality control, contributing to familial forms of Parkinson's, but not directly to Lewy body
formation.

Solution for Question 17:


Correct Option D - Papillae lined by well-differentiated squamous epithelium:
• The papillary variant is associated with papillae that are lined by well-differentiated squamous cell
epithelium
Craniopharyngioma
Description:
• A relatively benign dysontogenetic tumor arising from a remnant of the Rathke pouch (ectodermal
derivative).
Epidemiology
• Bimodal distribution: 5–14 years; second peak at 50–75 years.
• Most common childhood supratentorial tumor.
Clinical features:
• The tumor arises in the suprasellar region and can extend into the intrasellar region.
• Compression of the pituitary gland due to intrasellar extension → hypopituitarism.

Page 21

1031
• Hypogonadotropic hypogonadism
• Compression of the ventromedial hypothalamic nucleus → hyperphagia and obesity.
• Compression of the optic chiasm → bitemporal hemianopsia
• Compression of interventricular foramina and/or aqueduct → obstructive hydrocephalus
Diagnostics
• Imaging: Suprasellar calcified cyst with a lobulated contour
• Biopsy: Cholesterol crystals found in a motor oil-like fluid on gross examination
Histological variants
Adamantinomatous (common)
• Reticular epithelial cells
• Frequently associated with calcifications
• Cysts and keratin nodules
Papillary
• Metaplastic squamous cells
• Calcifications and cysts are rare.
• No keratin nodules
Treatment
• Resection Adjuvant radiotherapy In the case of hypopituitarism: hormone replacement therapy
Incorrect Options:
Option A - Nests and cords of squamous epithelium embedded in the spongy reticulum:
• The adamantinomatous variant comprises nests and cords of the squamous cell epithelium.
Option B - Compact, lamellar keratin formation ("wet keratin"):
• Wet keratin is one of the features of the adanmantinomatous variant of the craniopharyngioma.
Option C - Cysts containing cholesterol-rich fluid that resembles ‘machine oil’:
• Dystrophic calcification, cyst formation containing cholesterol-rich fluid, and fibrosis are common
findings in adamantinomatous variant of craniopharyngioma.

Page 22

1032
Thyroid Gland Pathology
1. What potential co-existing condition might be present in a 50-year-old woman with a firm, fixed,
painless anterior neck mass, difficulty swallowing, hoarse voice, and a hard, woody enlargement of the
thyroid gland extending beyond its usual borders, based on the provided histopathology?

A. Systemic Sclerosis
B. Sjögren's syndrome
C. Mikulicz syndrome
D. Jod-Basedow phenomenon
----------------------------------------
2. What is true regarding a 28-year-old woman presenting with fatigue, weight gain, cold intolerance,
diffuse painless thyroid enlargement, elevated TSH, low free T4, positive anti-thyroid peroxidase
antibodies, and the given thyroid gland histology?
(or)
In a 28-year-old postpartum woman presenting with fatigue, weight gain, and a painless diffusely
enlarged thyroid, along with an elevated TSH, low free T4, and positive anti-thyroid peroxidase
antibodies, with given histological finding on biopsy, choose the correct statement regarding her
diagnosis.

A. The patient usually gives a history of an upper respiratory tractinfections


B. The patient gives a family history of autoimmune diseases
C. The patient mostly shows symptoms consistent with hyperthyroidism
D. All the statements are true

1033
----------------------------------------
3. What is the likely diagnosis for a 45-year-old woman with unilateral neck swelling and difficulty
swallowing, revealing a 5cm spherical mass on the right side of her neck on examination, a single cold
nodule with relatively normal thyroid parenchyma on imaging, and an excision biopsy showing more
than 75% of the following characteristics?

A. RAS mutation
B. TSH receptor mutation
C. GNAS mutation
D. EZH1 mutation.
----------------------------------------
4. What genetic mutation is most likely associated with a 38-year-old woman presenting with weight
loss, palpitations, heat intolerance, a single spherical well-demarcated thyroid mass, multiple nodules
with increased radioactive iodine uptake, and biopsy revealing follicular growth pattern with oxyphil
cells?
A. RET proto-oncogene mutation
B. RAS mutation
C. BRAF mutation
D. GNAS mutation
----------------------------------------
5. Which mutation is not associated with a 32-year-old woman who has a gradually increasing painless
neck mass revealing a firm irregular mass in the right thyroid lobe on examination and the following
FNAC findings?

Page 2

1034
A. RET/PTC fusion gene
B. NTRK1 mutation
C. CASR mutation
D. BRAF mutation.
----------------------------------------
6. What mutation is most expected in a 45-year-old male with a painless neck mass, elevated calcitonin
levels, and thyroid involvement with Acal amyloidosis?
(or)
In a 45-year-old male with a painless neck mass, a firm nodule, elevated calcitonin, and Acal
amyloidosis on microscopy, What is the most likely genetic mutation?
A. RET
B. PTEN
C. RAS
D. MEN
----------------------------------------
7. In a 23-year-old female with prolonged history of constipation, hypothyroidism, irregular menstrual
cycles, and elevated anti-TPO antibodies, which histological changes are not seen in this disease?
A. Lymphocytic infiltrate with germinal center formation
B. Follicular destruction and atrophy
C. Hürthle cell metaplasia
D. Orphan Annie eye nuclei
----------------------------------------
8. What is the probable diagnosis for a 45-year-old woman who experiences fatigue and unintended
weight gain, has irregular menstruation, a diffuse mass on the front of her neck, increased TSH,
decreased T3 and T4 levels, and the subsequent biopsy results?
(or)
In a 45-year-old female with fatigue, unintentional weight gain, menstrual irregularities, and a diffuse
anterior neck mass, a thyroid biopsy shows histologic features as given below. What is the most likely
cause of her current symptoms?

A. Hashimoto thyroiditis

Page 3

1035
B. Granulomatous thyroiditis
C. Papillary carcinoma of thyroid
D. Reidel's thyroiditis
----------------------------------------
9. What feature is associated with the diagnosis of the 22-year-old male presenting with a diffusely
enlarged and tender thyroid, recent upper respiratory tract infection, elevated T3 and T4, low TSH,
elevated leukocytes and TSR with the following biopsy findings?
(or)
Which of the following is the feature of De-Quervain thyroiditis?

A. In the early phase, follicles are disrupted and replaced by neutrophils forming microabscesses
B. Orphan Annie nuclei are seen on histology
C. Bacterial infection is a trigger in most patients
D. It is the most common cause of painful, multinodular thyroid
----------------------------------------
10. Which of the following is a histological feature seen in patients with Grave’s disease?
A. Sheets of Polygonal cells in an amyloid stroma
B. Lymphocytic infiltration with germinal centres
C. Empty appearing nuclei with central clearing
D. A pale colloid in the follicular lumen with scalloped margins
----------------------------------------
11. Which of the following tumor markers is least likely to be positive incase of medullary carcinoma
thyroid?
A. MUC1
B. S-100
C. NSE
D. All of the above
----------------------------------------
12. Which of the following is a characteristic biomarker in patients with medullary carcinoma of the
thyroid?

Page 4

1036
A. Calcitonin
B. Thyroglobulin
C. Thyroxine T4
D. Triiodothyronine T3
----------------------------------------

Correct Answers
Question Correct Answer

Question 1 3
Question 2 2
Question 3 1
Question 4 4
Question 5 3
Question 6 1
Question 7 4
Question 8 1
Question 9 1
Question 10 4
Question 11 1
Question 12 1

Solution for Question 1:


Correct Option C - Mikulicz syndrome:
The histological image shows:
• Thyroid parenchyma is replaced by hyalinized fibrous tissue. Inflammatory infiltrate can be quite
prominent and usually consists of plasma cells (mostly IgG-4 producing), lymphocytes, and
macrophages. Numerous eosinophils may also be seen.
• The clinical presentation and histopathological findings described in the scenario are indicative of
Riedel's thyroiditis, a rare form of chronic thyroiditis characterized by fibrosis and infiltration of the
thyroid gland and surrounding tissues. Riedel's thyroiditis can be associated with a broader condition
known as IgG4-related disease, which involves immune-mediated fibroinflammatory lesions in various
organs.
• Mikulicz syndrome is a manifestation of IgG4-related disease that specifically affects the lacrimal and
salivary glands. It is characterized by bilateral enlargement of the lacrimal and salivary glands,
resembling Sjögren's syndrome. The syndrome can coexist with other manifestations of IgG4-related
disease, such as Riedel's thyroiditis.
• Clinical Features: Firm, fixed, painless anterior neck mass; difficulty swallowing; hoarse voice.
• Treatment:

Page 5

1037
• Related Diseases:
• Association with IgG4: Linked to IgG4-related disease characterized by fibrosis from IgG4-producing
plasma cells.
• Diagnostic Challenge: Mistaken for thyroid cancer due to hard consistency; requires careful biopsy for
accurate diagnosis.
• Management Approach: Use immunomodulatory agents - Rituximab, tamoxifen, steroids, to reduce
inflammation and fibrosis.
• Prognosis: Variable; often chronic, requiring long-term management.
Incorrect Options:
Option A - Systemic Sclerosis: Systemic sclerosis primarily involves fibrosis of the skin and internal org
ans, including the esophagus, lungs, and kidneys. While it may affect the thyroid gland, the described
woody enlargement and fibrous replacement of thyroid parenchyma with prominent inflammatory infiltr
ate, especially with IgG-4 producing plasma cells, are more characteristic of Riedel's thyroiditis and IgG
4-related disease than systemic sclerosis.
Option B - Sjögren's Syndrome: Sjögren's syndrome is characterized by dry eyes and dry mouth due to
lymphocytic infiltration of the salivary and lacrimal glands. The scenario describes a thyroid-specific pa
thology (Riedel's thyroiditis) rather than the characteristic involvement of salivary and lacrimal glands s
een in Sjögren's syndrome.
Option D - Jod-Basedow Phenomenon: Jod-Basedow phenomenon refers to the development of hyper
thyroidism in individuals with an underlying thyroid disorder (such as nodular goiter)

Solution for Question 2:


Correct Option B - Patients give a family history of autoimmune diseases:
The image shows:
• Subacute Lymphocytic thyroiditis: There is nonspecific lymphocytic infiltration with almost normal
thyroid follicles.
• Subacute Lymphocytic thyroiditis is also known as Postpartum Thyroiditis.
• This clinical presentation is consistent with postpartum thyroiditis, which is an autoimmune thyroid
disorder that typically occurs in the first year after childbirth. The presence of anti-thyroid peroxidase
antibodies and the family history of autoimmune diseases are commonly associated with autoimmune
thyroid conditions. The condition often presents with symptoms of hypothyroidism initially, and then
may eventually normalize or lead to persistent hypothyroidism.
• Features Onset Typically occurs within the first year after childbirth. History Commonly associated
with autoimmune diseases, family history of thyroid disorders, or other autoimmune conditions.
Histology Biopsy may show almost normal follicles and unspecific lymphocytic infiltration. Fine Needle
Aspiration Cytology (FNAC) FNAC may show evidence of lymphocytic infiltration. Symptoms 1.
Hypothyroid Phase: Fatigue, weight gain, constipation, cold intolerance. 2. Recovery Phase: May return
to normal thyroid function or develop persistent hypothyroidism. Treatment 1. Supportive Care: For
symptoms during hypothyroid phases. 2. Thyroid Hormone Replacement: If persistent hypothyroidism
ensues. Monitoring thyroid function is crucial. Prognosis Generally favorable, with many women
returning to normal thyroid function. Persistent hypothyroidism may require long-term thyroid hormone
replacement.

Page 6

1038
Features
Onset
Typically occurs within the first year after childbirth.
History
Commonly associated with autoimmune diseases, family history of thyroid disorders, or other autoimm
une conditions.
Histology
Biopsy may show almost normal follicles and unspecific lymphocytic infiltration.
Fine Needle Aspiration Cytology (FNAC)
FNAC may show evidence of lymphocytic infiltration.
Symptoms
1. Hypothyroid Phase: Fatigue, weight gain, constipation, cold intolerance.
2. Recovery Phase: May return to normal thyroid function or develop persistent hypothyroidism.
Treatment
1. Supportive Care: For symptoms during hypothyroid phases.
2. Thyroid Hormone Replacement: If persistent hypothyroidism ensues. Monitoring thyroid function is c
rucial.
Prognosis
Generally favorable, with many women returning to normal thyroid function. Persistent hypothyroidism
may require long-term thyroid hormone replacement.

Incorrect Options:
Option A - The patient usually gives a history of an upper respiratory tract infections: This statement is
not characteristic of postpartum thyroiditis. Postpartum thyroiditis is typically associated with childbirth
and an autoimmune response, not upper respiratory tract infections. History of viral upper respiratory in
fection before developing thyroiditis is seen in De Quervain thyroiditis
Option C - Patients mostly show symptoms consistent with hyperthyroidism: The typical presentation of
postpartum thyroiditis involves an initial phase of hypothyroidism, and then potentially returning to nor
mal thyroid function or progressing to persistent hypothyroidism. The statement is incorrect because th
e symptoms are not predominantly consistent with hyperthyroidism.
Option D - All the statements are true: This option is incorrect because Option A and Option C
are not true. Patients with postpartum thyroiditis do not usually give a history of upper respiratory tract i
nfections, and their symptoms are not primarily consistent with hyperthyroidism.

Solution for Question 3:


Correct Option A - RAS mutation:
• The likely diagnosis is Hurtle cell adenoma.

Page 7

1039
• The clinical question image displays Hurthle cells, also referred to as Oncocytic cells, Oxyphyllic cells,
or Askanasy cells.
• Follicular adenoma with over 75% of Hurthle cells, is considered Hurtle cell adenoma.
• Hurthle cell tumor is a type of follicular cell carcinoma and RAS mutation is common in both follicular
adenoma and follicular carcinoma.
• Feature Follicular Adenoma Follicular Carcinoma Genetic Mutation RAS mutation is common in
non-functioning follicular adenomas, Whereas TSHR and GNAS mutations are seen in Toxic
adenomas. Can also exhibit RAS mutations. Invasion Typically lacks capsular or vascular invasion.
Characterized by capsular and vascular invasion. Fine Needle Aspiration Cytology (FNAC) Difficult to
distinguish on FNAC alone. Differentiation often requires a biopsy. Histological Appearance
Microscopically appears as a well-circumscribed nodule. Microscopic features may show invasion
beyond the thyroid capsule.
Feature
Follicular Adenoma
Follicular Carcinoma
Genetic Mutation
RAS mutation is common in non-functioning follicular adenomas, Whereas TSHR and GNAS mutations
are seen in Toxic adenomas.
Can also exhibit RAS mutations.
Invasion
Typically lacks capsular or vascular invasion.
Characterized by capsular and vascular invasion.
Fine Needle Aspiration Cytology (FNAC)
Difficult to distinguish on FNAC alone.
Differentiation often requires a biopsy.
Histological Appearance
Microscopically appears as a well-circumscribed nodule.
Microscopic features may show invasion beyond the thyroid capsule.

Incorrect Options:
Options B, C and D
are more commonly associated with toxic adenomas, rather than follicular carcinomas.

Solution for Question 4:


Correct Option D - GNAS mutation:
• This is a case of clinically apparent thyrotoxicosis since the patient gives a history of weight loss,
palpitations and heat intolerance.

Page 8

1040
• Given the clinical presentation of a single spherical mass with multiple nodules showing increased
radioactive iodine uptake, along with the histological findings of a follicular growth pattern and the
presence of oxyphil cells, the most likely diagnosis is a toxic thyroid adenoma
• Toxic thyroid adenomas are characterized by the presence of a single, hyperfunctioning nodule (or
mass) in the thyroid gland. The GNAS mutation is commonly associated with these adenomas. This
mutation leads to constitutive activation of the TSH receptor, promoting unregulated thyroid hormone
production and secretion.

Incorrect Options:
Option A - RET proto-oncogene mutation: The RET proto-oncogene mutation is associated with multipl
e endocrine neoplasia type 2 (MEN2), particularly medullary thyroid carcinoma (MTC), and is not com
monly associated with the development of toxic thyroid adenomas.
Option B - RAS mutation: RAS mutations are associated with thyroid neoplasms, including both non-fu
nctioning follicular adenoma and carcinomas. GNAS mutation is more specifically linked to the pathoge
nesis of toxic thyroid adenomas.
Option C - BRAF mutation: BRAF mutations are commonly associated with papillary thyroid carcinoma
but are less likely to be implicated in toxic thyroid adenomas, which are characterized by hyperfunctioni
ng nodules rather than malignant transformation.

Solution for Question 5:


Correct Option C - CASR mutation:
• The features described, along with the cytological findings, are suggestive of papillary thyroid
carcinoma

• It is associated with Autosomal-Dominant Hypoparathyroidism. A gain of function mutation in the
calcium-sensing receptor (CASR) gene is seen in Autosomal-Dominant Hypoparathyroidism. Loss of
function mutation of CASR may cause familial parathyroid adenomas.
• Papillary Carcinoma Thyroid Genetic Mutations BRAF V600E mutation: Most common mutation in
PTC. RET-PTC fusion: Resulting from a translocation (t(10:17)). NTRK mutation: Associated with PTC.
Other Risk Factors History of Hashimoto's thyroiditis. Long-standing thyroglossal cyst. History of
radiation exposure (may manifest years later). Clinical Features Anterior neck nodule.
Lymphadenopathy. Lymphatic spread (except for follicular carcinoma). Microscopic Characteristics
Fingerlike projections (true papilla). True papillae have blood vessels (fibrovascular core). Optically
clear nucleus. (Orphan Annie eye nuclei Intranuclear cytoplasmic inclusions (INCI). Nuclear groove or
coffee bean nucleus. Presence of psammoma bodies.
• BRAF V600E mutation: Most common mutation in PTC.
• RET-PTC fusion: Resulting from a translocation (t(10:17)).
• NTRK mutation: Associated with PTC.
• History of Hashimoto's thyroiditis.
• Long-standing thyroglossal cyst.
• History of radiation exposure (may manifest years later).

Page 9

1041
• Anterior neck nodule.
• Lymphadenopathy.
• Lymphatic spread (except for follicular carcinoma).
• Fingerlike projections (true papilla).
• True papillae have blood vessels (fibrovascular core).
• Optically clear nucleus. (Orphan Annie eye nuclei
• Intranuclear cytoplasmic inclusions (INCI).
• Nuclear groove or coffee bean nucleus.
• Presence of psammoma bodies.

Papillary Carcinoma Thyroid


Genetic Mutations
• BRAF V600E mutation: Most common mutation in PTC.
• RET-PTC fusion: Resulting from a translocation (t(10:17)).
• NTRK mutation: Associated with PTC.
Other Risk Factors
• History of Hashimoto's thyroiditis.
• Long-standing thyroglossal cyst.
• History of radiation exposure (may manifest years later).
Clinical Features
• Anterior neck nodule.
• Lymphadenopathy.
• Lymphatic spread (except for follicular carcinoma).
Microscopic Characteristics

Page 10

1042
• Fingerlike projections (true papilla).
• True papillae have blood vessels (fibrovascular core).
• Optically clear nucleus. (Orphan Annie eye nuclei
• Intranuclear cytoplasmic inclusions (INCI).
• Nuclear groove or coffee bean nucleus.
• Presence of psammoma bodies.

Incorrect Options:
Option A - RET/PTC fusion gene, Option B - NTRK1 mutation, and Option D - BRAF mutation:
• RET/PTC fusion gene and NTRK1 mutation are genetic alterations that can be associated with thyroid
cancer, particularly papillary thyroid carcinoma (PTC). These mutations can lead to abnormal activation
of signaling pathways promoting thyroid cell proliferation.
• BRAF mutation is commonly found in papillary thyroid carcinoma, especially the V600E mutation. It
plays a role in the MAP kinase signaling pathway, contributing to the development and progression of
thyroid cancer.

Solution for Question 6:


Correct Option A - RET:
• Medullary thyroid carcinoma (MTC) is associated with elevated calcitonin levels and Acal amyloidosis
involving the thyroid. The most common and specific mutation associated with medullary thyroid
carcinoma is the RET proto-oncogene mutation.
• Medullary Carcinoma Thyroid Definition Tumor arising from parafollicular C cells of the thyroid
Hormone Production Produces and releases calcitonin Tumor Markers Calcitonin (primary marker),
Carcinoembryonic antigen (CEA) Genetic Mutations Most common: RET proto-oncogene Syndromes
Associated MEN 2a and MEN 2b RET Gene Location Located on chromosome 10 (10q11.2)
Histological Findings Amyloid deposits (Acal amyloidosis), Congo red-positive Polarized Microscopic
Appearance Apple green birefringence under polarized microscopy Neuroendocrine Characteristics
Expresses neuroendocrine markers: Synaptophysin, Chromogranin, NSE
Medullary Carcinoma Thyroid
Definition
Tumor arising from parafollicular C cells of the thyroid
Hormone Production
Produces and releases calcitonin
Tumor Markers
Calcitonin (primary marker), Carcinoembryonic antigen (CEA)
Genetic Mutations
Most common: RET proto-oncogene
Syndromes Associated

Page 11

1043
MEN 2a and MEN 2b
RET Gene Location
Located on chromosome 10 (10q11.2)
Histological Findings
Amyloid deposits (Acal amyloidosis), Congo red-positive
Polarized Microscopic Appearance
Apple green birefringence under polarized microscopy
Neuroendocrine Characteristics
Expresses neuroendocrine markers: Synaptophysin, Chromogranin, NSE

Incorrect Options:
Option B - PTEN: PTEN mutations are associated with Cowden syndrome, which may include thyroid l
esions, but it is not specifically linked to medullary thyroid carcinoma.
Option C - RAS: RAS mutations are more commonly associated with follicular thyroid carcinoma and p
apillary thyroid carcinoma, not medullary thyroid carcinoma.
Option D - MEN: MEN (Multiple Endocrine Neoplasia) is a
group of inherited disorders, and while MEN 2 is associated with medullary thyroid carcinoma, the spec
ific mutation involved is RET. Simply stating MEN without specifying the subtype does not provide the
most accurate answer.

Solution for Question 7:


Correct Option D - Orphan Annie's eye nuclei:
• Considering the clinical scenario: A history of Constipation, Menstrual Irregularities, and Dry skin is a
classic manifestation of hypothyroidism. Laboratory tests indicating hypothyroidism, such as elevated
thyroid-stimulating hormone (TSH) levels and low levels of free thyroxine (T4), along with elevated
levels of anti-thyroid peroxidase (anti-TPO) antibodies support the diagnosis of Hashimoto's thyroiditis.
• All of the above histological features are of Hashimoto thyroiditis except Orphan Annie eye nuclei; It is
seen in papillary carcinoma of the thyroid.
Histologic features of Hashimoto thyroiditis:
• Hashimoto thyroiditis is an autoimmune disorder causing chronic inflammation of the thyroid gland.
• It leads to diffuse enlargement of the thyroid gland known as goiter.
• Histologically, there is infiltration of the thyroid parenchyma by lymphocytes and plasma cells.
• Atrophic thyroid follicles are seen, with epithelial cells exhibiting metaplastic changes termed Hürthle
cells.
• Fine-needle aspiration biopsy reveals Hürthle cells along with a heterogeneous population of
lymphocytes.

Page 12

1044
The image displays thyroid parenchyma with a dense lymphocytic infiltrate and germinal centers. Resid
ual thyroid follicles lined with eosinophilic Hurthle cells are also visible.
Incorrect Options:
Option A - Lymphocytic infiltrate with the germinal center formation:
• Histologically, Hashimoto thyroiditis is characterized by lymphocytic infiltration that leads to the
development of germinal centers.
• There is extensive infiltration of the parenchyma by a mononuclear inflammatory infiltrate containing
small lymphocytes, plasma cells, and well-developed germinal centres.
Option B - Follicular destruction and atrophy:
• The autoimmune process leads to follicular destruction in Hashimoto's thyroiditis.
• This leads to thyroid follicular atrophy.
• These events are evident in the histologic examination of the thyroid gland.
Option C - Hürthle cell metaplasia:
• A morphologic finding indicates the transformation of follicular cells to large cells with abundant
eosinophilic and granular cytoplasm (Hurthle cells) in the thyroid gland.
• It is one of the histologic features of Hashimoto's thyroiditis.

Solution for Question 8:


Correct Option A - Hashimoto thyroiditis:
• Fatigue, unintentional weight gain and menstrual irregularities point towards
hypothyroidism, Increased thyroid-stimulating hormone (TSH) levels and decreased triiodothyronine
(T3) and thyroxine (T4) levels indicate primary hypothyroidism

Page 13

1045
• The image above shows thyroid parenchyma containing a dense lymphocytic infiltrate with germinal
centers. Residual thyroid follicles lined by deeply eosinophilic Hürthle cells are also seen.
• These findings are consistent with Hashimoto's thyroiditis.

Hashimoto thyroiditis:
• Also called chronic autoimmune thyroiditis.
• The most common cause of hypothyroidism in iodine-sufficient regions.
• Associated with HLA-DR3 (differs by ethnicity) and increased risk of primary thyroid lymphoma
(typically diffuse large B-cell lymphoma).
Findings:
• Moderately enlarged, non-tender thyroid.
• It may be preceded by a transient hyperthyroid state ("Hashitoxicosis") due to follicular rupture and
thyroid hormone release.
Serology:
• Positive anti-thyroid peroxidase (anti-microsomal) and anti-thyroglobulin antibodies.
Histology:
• Hürthle cells and lymphoid aggregates with germinal centres
Incorrect Options:
Option B - Subacute granulomatous thyroiditis:
• Also called de Quervain thyroiditis.
• Usually, a self-limited disease.
• Often preceded by viral infection.
• Findings are raised ESR, jaw pain, and very tender thyroid (de Quervain is associated with pain).
• Histology shows granulomatous inflammation.

Page 14

1046
Option C - Papillary carcinoma of the thyroid:
• Most common thyroid cancer.

• Histology shows empty-appearing nuclei with central clearing ("Orphan Annie" eyes), psammoma
bodies and nuclear grooves.
• Papillary carcinoma: most prevalent, palpable lymph nodes.
• It has a good prognosis.

Option D - Reidel's thyroiditis:


• Also called invasive fibrous thyroiditis.
• It may be associated with IgG4-related disease (e.g., autoimmune pancreatitis, retroperitoneal
fibrosis, noninfectious aortitis), a variant of autoimmune thyroiditis, or part of a systemic fibrosing

Page 15

1047
disorder.
• Hypothyroidism occurs in 1 ⁄3 of patients.
• Histology: thyroid replaced by fibrous tissue and inflammatory infiltrate (Shown below).

Solution for Question 9:


Correct Option A
- In the early phase, follicles are disrupted and replaced by neutrophils forming microabscesses:
• The history of upper respiratory tract infection, followed by thyroid gland enlargement, elevated T3
and T4 but low TSH, and the biopsy showing a prominent multinucleated giant cell indicate
Granulomatous thyroiditis.

• De Quervain's thyroiditis, also known as subacute granulomatous thyroiditis or giant cell thyroiditis, is
a self-limiting inflammatory disorder of the thyroid gland. In the early phase of the disease, thyroid
follicles are disrupted, which are replaced by neutrophils, forming microabscesses. This acute
inflammatory process is responsible for the symptoms of fever, neck pain, and tenderness in patients
with de Quervain's thyroiditis.
• Though initially the patient is in a hyperthyroid state, the thyroid function returns to normal in 6 to 8
weeks of recovery from the inflammatory state.

Page 16

1048
Incorrect Options:
Option B - Orphan Annie nuclei are seen on histology: Orphan Annie nuclei are associated with papillar
y thyroid carcinoma, not De Quervain's thyroiditis.
Option C - Bacterial infection is a
trigger in most patients: De Quervain's thyroiditis is usually preceded by a
viral upper respiratory tract infection, not a bacterial infection.
Option D - It is the most common cause of painful, multinodular thyroid: De Quervain's thyroiditis typica
lly presents with a diffuse, tender thyroid gland rather than a multinodular thyroid.

Solution for Question 10:


Correct Option D - Pale colloid in the follicular lumen with scalloped margins:
• In Graves' disease, the histological feature commonly observed is the presence of a pale colloid in the
follicular lumen with scalloped margins. This occurs due to the increased activity of thyroid hormone
production, leading to the depletion of colloid material within the thyroid follicles.
• Due to continuous stimulation by autoantibody, the thyroid gland is usually symmetrically enlarged
due to diffuse hypertrophy and hyperplasia of follicular epithelial cells. Follicular epithelial cells in
untreated cases are tall and more crowded than usual on histology. This crowding may result in the
formation of small papillae, which project into the follicular lumen and encroach on the colloid,
sometimes filling the follicles. The colloid within the follicular lumen is pale, with scalloped margins.

Page 17

1049
Incorrect Options:
Option A - Sheets of Polygonal cells in an amyloid stroma:
• Sheets of Polygonal cells in an amyloid stroma are usually seen in medullary thyroid carcinoma, the
tumour of Parafollicular 'C-cells' of the thyroid gland.
• They produce calcitonin, which can lead to hypocalcemia.
• It is associated with MEN2A and MEN2B (RET mutation).
Option B - Lymphocytic infiltration with germinal centers:
• Lymphocytic infiltration with germinal centers is usually seen in Hashimoto's Thyroiditis.
• It is the most common cause of hypothyroidism in iodine-sufficient countries.
• Other important features are present anti-Thyroid peroxidase antibodies on serology and Hurthle’s
cells on histology.
Option C - Empty appearing nuclei with central clearing:
• Empty appearing nuclei with central clearing (Orphan Annie eyes) are seen in papillary carcinoma of
the thyroid
• The other important features on histology are Psammoma bodies and nuclear grooves.

Solution for Question 11:


Correct Option A - MUC1:
• MUC1, also known as mucin-1, is a glycoprotein that is not typically associated with medullary thyroid
carcinoma (MTC). It is more commonly expressed in adenocarcinomas of various organs, such as the
breast, lung, and pancreas. Therefore, it would be the least likely to be positive in the context of MTC.
• Medullary thyroid carcinoma (MTC) is a neuroendocrine tumor arising from the parafollicular C cells of
the thyroid gland. These tumors often express neuroendocrine markers such as Synaptophysin,
Chromogranin, NSE and, S-100

Page 18

1050
Incorrect Options:
Refer to the Correct Option

Solution for Question 12:


Correct Option A – Calcitonin:
• Calcitonin is a characteristic biomarker in patients with medullary carcinoma of the thyroid. This
hormone is produced by the parafollicular cells (C cells) of the thyroid gland. In medullary carcinoma,
these C cells become malignant and produce excessive amounts of calcitonin. Elevated levels of
calcitonin are indicative of medullary thyroid carcinoma, although they can also be elevated in other
conditions, such as C-cell hyperplasia

Calcitonin is a characteristic biomarker in patients with medullary carcinoma of the thyroid. This hormo
ne is produced by the parafollicular cells (C cells) of the thyroid gland. In medullary carcinoma, these C
cells become malignant and produce excessive amounts of calcitonin. Elevated levels of calcitonin are
indicative of medullary thyroid carcinoma, although they can also be elevated in other conditions, such
as C-cell hyperplasia

Incorrect Options:
Option B – Thyroglobulin:
• Thyroglobulin is a protein produced by normal thyroid cells and is a marker for well-differentiated
thyroid cancers such as papillary and follicular carcinoma. However, it is not specific to medullary
carcinoma of the thyroid.
Option C - Thyroxine (T4):
• Thyroxine (T4) is a hormone produced by the thyroid gland that regulates metabolism. While thyroid
hormones play a role in thyroid cancer, they are not specific biomarkers for medullary carcinoma.

Page 19

1051
Option D - Triiodothyronine (T3):
• Triiodothyronine (T3) is another thyroid hormone involved in regulating metabolism. Like thyroxine, it
is not specific to medullary carcinoma and therefore not a characteristic biomarker for this type of
thyroid cancer.

Page 20

1052
Parathyroid Gland Pathology
1. What's causing seizures in a 3-month-old infant with muscle spasms, recurrent infections, feeding
issues, dysmorphic face, cleft palate, and absent thymic shadow on X-ray?
(or)
Which of the following is the cause of a seizure in an infant with severe muscle spasms, a cleft palate,
recurrent infection, dysmorphic face and no thymic shadow on an X-ray of the chest?
A. Hyperkalemia
B. Hypocalcemia
C. Hyponatremia
D. Hypokalemia
----------------------------------------
2. Which of the following genes is involved in the pathogenesis of adrenal neuroblastoma ?
A. NMYC
B. RET
C. GNAS
D. UPS8
----------------------------------------
3. What underlying disorder could cause a 72-year-old woman with abdominal pain, neuromuscular
weakness, reduced serum calcium, elevated parathyroid hormone, type-2 diabetes mellitus,
hypertension, and four symmetrically enlarged parathyroid glands?
(or)
Which of the following underlying conditions could be the cause of endocrinopathy, which includes
symmetrically enlarged parathyroid glands and elevated parathyroid hormone and reduced serum
calcium?
A. Hyperaldosteronism
B. Chronic Liver Disease
C. Myasthenia
D. Renal Insufficiency
----------------------------------------
4. A 30-year-old male patient has been diagnosed with prolactinoma, gastrinoma, and parathyroid
adenoma. Which of the following genetic mutations is likely present in this patient?
A. BRCA1
B. RB
C. MENIN
D. VHL
----------------------------------------

1053
5. What is the most likely cause of upper abdominal pain in a 41-year-old woman with a history of
hyperparathyroidism, microcytic and hypochromic anemia, and a bleeding mucosal defect in the antrum
observed during gastroscopy?
A. Decreased Serum Calcium levels
B. Decreased Serum levels of PTH
C. Decreased Secretion of Gastrin
D. Increased Secretion of Gastrin
----------------------------------------
6. What's the likely diagnosis for a 55-year-old man with jaw and left arm pain for six months, end-stage
renal disease on dialysis, and an X-ray showing multiple small bone cysts with a pathologic fracture in
the left arm?
A. Chronic osteomyelitis
B. Marble bone disease
C. Osteitis fibrosa cystica
D. Osteoid osteoma
----------------------------------------

Correct Answers
Question Correct Answer

Question 1 2
Question 2 1
Question 3 4
Question 4 3
Question 5 4
Question 6 3

Solution for Question 1:


Correct Option B - Hypocalcemia:
• An absent thymic shadow along with recurrent infections and muscle spasms are likely indicatives for
DiGeorge syndrome
• DiGeorge syndrome is caused by a microdeletion in 22nd chromosomes (22q11 microdeletion).
• It causes failure in the development of 3rd and 4th branchial pouches, resulting in the following
anomalies: Agenesis/ hypoplasia of thymus & parathyroid glands Congenital heart defects Dysmorphic
facies
• Agenesis/ hypoplasia of thymus & parathyroid glands
• Congenital heart defects
• Dysmorphic facies

Page 2

1054
• As a result of parathyroid agenesis, patients with DiGeorge syndrome exhibit hypocalcemia, which
manifests as increased neuromuscular excitability.
• Symptoms may range from: Mild tingling in hands & feet Severe muscle cramps Convulsions
• Mild tingling in hands & feet
• Severe muscle cramps
• Convulsions
• Agenesis/ hypoplasia of thymus & parathyroid glands
• Congenital heart defects
• Dysmorphic facies
• Mild tingling in hands & feet
• Severe muscle cramps
• Convulsions

• (A) shows a normal thymic shadow. The bottom radiograph (B) shows the absence of the thymic
shadow in an infant
Incorrect Options:
Option A - Hyperkalemia:
• Hyperkalemia rarely causes symptoms in the CNS, and seizures do not occur.
• High potassium levels can cause palpitations, shortness of breath, chest pain, nausea, or vomiting.
Option C - Hyponatremia:
• Hyponatremia is strongly associated with seizures but not associated with DiGeorge syndrome.
• They are usually generalized tonic-clonic, but partial seizures may also occur.
Option D - Hypokalemia:
• Hypokalemia rarely causes symptoms in the CNS, and seizures do not occur.

Page 3

1055
• Low potassium levels can make muscles feel weak, twitch, or even become paralyzed, and cause
arrhythmia.

Solution for Question 2:


Correct Option A - NMYC:
• NMYC, is a gene involved in the development and progression of neuroblastoma, including adrenal
neuroblastoma. Neuroblastoma is a cancer that develops from immature nerve cells found in several
areas of the body, including the adrenal glands.
• The NMYC gene encodes a transcription factor that plays a critical role in regulating the expression of
genes involved in cell proliferation, differentiation, and survival. Amplification or overexpression of
NMYC is commonly associated with aggressive forms of neuroblastoma, including adrenal
neuroblastoma.
The NMYC gene encodes a transcription factor that plays a critical role in regulating the expression of
genes involved in cell proliferation, differentiation, and survival. Amplification or overexpression of NMY
C is commonly associated with aggressive forms of neuroblastoma, including adrenal neuroblastoma.

Incorrect Options:
Options B, C and D are incorrect.

Solution for Question 3:


Correct Option D - Renal Insufficiency:
Considering the clinical scenario:
• Both diabetes mellitus and hypertension are well-known risk factors for the development of chronic
kidney disease (CKD).
• Markedly Reduced Serum Calcium and Elevated Parathyroid Hormone (PTH) Levels: These
laboratory findings are consistent with secondary hyperparathyroidism. In renal insufficiency, there is
reduced excretion of phosphate, leading to phosphate retention and subsequent hypocalcemia.
Hypocalcemia stimulates the parathyroid glands to produce and release more parathyroid hormone
(PTH) in an attempt to increase serum calcium levels.
• In cases where all four glands are symmetrically enlarged, the underlying cause is often secondary to
chronic kidney disease (renal insufficiency). Chronic elevation of PTH leads to the symmetric
enlargement of all four parathyroid glands, known as parathyroid hyperplasia.
Markedly Reduced Serum Calcium and Elevated Parathyroid Hormone (PTH) Levels: These laboratory
findings are consistent with secondary hyperparathyroidism. In renal insufficiency, there is reduced ex
cretion of phosphate, leading to phosphate retention and subsequent hypocalcemia. Hypocalcemia sti
mulates the parathyroid glands to produce and release more parathyroid hormone (PTH) in an attempt
to increase serum calcium levels.
In cases where all four glands are symmetrically enlarged, the underlying cause is often secondary to c
hronic kidney disease (renal insufficiency). Chronic elevation of PTH leads to the symmetric enlargeme
nt of all four parathyroid glands, known as parathyroid hyperplasia.

Page 4

1056
Incorrect Options:
Option A - Hyperaldosteronism:
• Primary hyperaldosteronism Conn’s syndrome, either due to adrenal adenoma or bilateral adrenal
hyperplasia.
• It is characterized by increased aldosterone and decreased renin.
• Clinical features include the following: Hypertension Decreased or normal potassium Metabolic
alkalosis
• Hypertension
• Decreased or normal potassium
• Metabolic alkalosis
• It does not affect calcium homeostasis
• Primary hyperaldosteronism does not directly cause edema due to the aldosterone escape
mechanism.
• It leads to treatment-resistant hypertension.
• Hypertension
• Decreased or normal potassium
• Metabolic alkalosis
It does not affect calcium homeostasis
Primary hyperaldosteronism does not directly cause edema due to the aldosterone escape mechanism
.
Option B - Chronic Liver Disease:
• Chronic liver disease is a progressive deterioration of liver functions.
• Liver functions include the production of clotting factors and other proteins, harmful detoxifying
products of metabolism, and the excretion of bile.
• This continuous process of inflammation, destruction, and regeneration of liver parenchyma leads to
fibrosis and cirrhosis.
• Liver cirrhosis is the final stage of chronic liver disease.
Option C - Myasthenia gravis:
• Myasthenia gravis is a neuromuscular disorder caused by autoantibodies against acetylcholine
receptors, and it is not related to parathyroid dysfunction.

Solution for Question 4:


• The diagnosis in this male patient with prolactinoma, gastrinoma, and parathyroid adenoma is likely
Multiple Endocrine Neoplasia (MEN) syndrome type 1. This disease is caused due to the mutation of
the MEN (MENIN) gene.
• Patients with MEN syndrome type 1 & 2 have gene mutations that make them susceptible to
neoplasia or hyperplasia in multiple organs.

Page 5

1057
The diagnosis in this male patient with prolactinoma, gastrinoma, and parathyroid adenoma is likely Mu
ltiple Endocrine Neoplasia (MEN) syndrome type 1. This disease is caused due to the mutation of the
MEN (MENIN) gene.
Patients with MEN syndrome type 1 & 2
have gene mutations that make them susceptible to neoplasia or hyperplasia in multiple organs.
• All MEN syndromes have an autosomal dominant inheritance.
• It has the following types: MEN1 MEN2A MEN 2B
• MEN1
• MEN2A
• MEN 2B
• MEN1 It is caused by a mutation of MEN1 (menin, a tumor suppressor gene on chromosome 11). It is
characterized by pituitary tumors (prolactin or Growth hormone), pancreatic endocrine tumors
(Zollinger-Ellison syndrome, insulinomas, VIPomas, glucagonomas), and parathyroid adenomas.
• It is characterized by pituitary tumors (prolactin or Growth hormone), pancreatic endocrine tumors
(Zollinger-Ellison syndrome, insulinomas, VIPomas, glucagonomas), and parathyroid adenomas.
• MEN2A is characterized by parathyroid hyperplasia, medullary thyroid carcinoma, and
pheochromocytoma.
• It is associated with gain of function mutation in the RET gene
• It is associated with gain of function mutation in the RET gene
• MEN2B is characterized by medullary thyroid cancer, pheochromocytoma, mucosal neuromas, and
marfanoid habitus with features of long axial skeletal and hyperextensible joints. It is also associated
with gain of function mutation in the RET gene
• It is also associated with gain of function mutation in the RET gene
• MEN1
• MEN2A
• MEN 2B
• It is characterized by pituitary tumors (prolactin or Growth hormone), pancreatic endocrine tumors
(Zollinger-Ellison syndrome, insulinomas, VIPomas, glucagonomas), and parathyroid adenomas.
• It is associated with gain of function mutation in the RET gene
• It is also associated with gain of function mutation in the RET gene
Incorrect Options:
Option A - BRCA1:
• Mutation in BRCA1 causes the following: Breast cancer Ovarian cancer Pancreatic cancer Prostate
cancer
• Breast cancer
• Ovarian cancer
• Pancreatic cancer
• Prostate cancer
• Breast cancer

Page 6

1058
• Ovarian cancer
• Pancreatic cancer
• Prostate cancer
Option B - RB:
• Mutation in RB causes: Retinoblastoma Osteosarcoma
• Retinoblastoma
• Osteosarcoma
• Retinoblastoma
• Osteosarcoma
Option D - VHL:
• Mutation in VHL causes Von-Hippel-Lindau disease
• Therefore this option is incorrect

Solution for Question 5:


Correct Option D - Increased Secretion of Gastrin:
• Given clinical features point towards Peptic ulcer disease, secondary to hyperparathyroidism.
• The incidence of peptic ulcer disease is increased in patients with hyperparathyroidism because
hypercalcemia increases serum gastrin, thereby stimulating gastric acid secretion.
• Peptic ulcers in the context of MEN-1 may be secondary to Zollinger-Ellison syndrome.
Incorrect Options:
Options A, B, and C are incorrect. Refer to the explanation of Option D

Solution for Question 6:


Correct Option C - Osteitis Fibrosa Cystica:
• Given the patient's history of end-stage renal disease and renal dialysis, along with the presentation of
pain in the jaw and left arm, the most likely diagnosis is osteitis fibrosa cystica. This condition is a
skeletal manifestation of hyperparathyroidism secondary to chronic kidney disease (CKD).
• This disease occurs due to hyperparathyroidism.
• In patients with CKD, the kidneys are unable to excrete phosphate efficiently, leading to
hyperphosphatemia and secondary hyperparathyroidism. Excessive secretion of parathyroid hormone
(PTH) results in bone resorption, leading to the formation of cystic lesions within the bones, particularly
in areas rich in trabecular bone, such as the jaw and long bones. Pathologic fractures can occur due to
the weakened bone structure.
• Osteitis fibrosa cystica is a severe bone deformity that causes "brown tumors" (non-neoplastic).

Page 7

1059
• Clinical Features include: Bone pain Bone cysts Pathologic fractures Localised bone swellings (due to
brown tumors).
• Bone pain
• Bone cysts
• Pathologic fractures
• Localised bone swellings (due to brown tumors).
• Bone pain
• Bone cysts
• Pathologic fractures
• Localised bone swellings (due to brown tumors).
• Histopathology shows: Brown tumor comprises numerous giant cells with interstitial hemorrhage,
hemosiderin, and vascularity.
• Brown tumor comprises numerous giant cells with interstitial hemorrhage, hemosiderin, and
vascularity.
• Brown tumor comprises numerous giant cells with interstitial hemorrhage, hemosiderin, and
vascularity.
Incorrect Options:
Option A - Chronic Osteomyelitis:
• Chronic osteomyelitis typically presents with localized bone pain, fever, and inflammation, and is
unlikely in this case given the patient's clinical history and findings.
Option B - Marble Bone Disease:
• Marble bone disease, also known as osteopetrosis, is characterized by abnormally dense and brittle
bones, which would not present with cystic lesions.
Option D - Osteoid Osteoma:
• Osteoid osteoma is a benign bone tumor characterized by a painful nidus surrounded by reactive
bone formation, which is not consistent with the cystic lesions described in this patient.

Page 8

1060
Adrenal and Pituitary Gland Pathology
1. What's the likely diagnosis for a 55-year-old man with jaw and left arm pain for six months, end-stage
renal disease on dialysis, and an X-ray showing multiple small bone cysts with a pathologic fracture in
the left arm?
A. Chronic osteomyelitis
B. Marble bone disease
C. Osteitis fibrosa cystica
D. Osteoid osteoma
----------------------------------------
2. A 47-year-old male with no previous medical history presents to his physician complaining of
episodic headaches, sweating, palpitations, and tremors. The symptoms started a few years ago, have
become more frequent, and can last between a few seconds and an hour. The episodes often occur
when the patient is stressed or during exercises. He has no history of serious illnesses,
hospitalizations, or trauma. He is not on any medications. His blood pressure measured at the clinic
was 230/130 mm Hg. Radiological imaging was done, which is shown below. Which of the following is
the least likely feature of the most probable diagnosis?

A. 90% are malignant


B. 95% occur in the abdomen
C. They secrete catecholamines
D. They arise from sympathetic ganglia
----------------------------------------
3. What findings might be expected in a 35-year-old woman, six months postpartum, who experienced
severe cervical lacerations during delivery, resulting in hemorrhagic shock, and now presents with
continued amenorrhea, weight loss, and muscle weakness?
(or)
When giving birth, a woman who suffered serious cervical lacerations experiences hemorrhagic shock.
Now she complaints of ongoing amenorrhea. Which of the following conclusions will be shown by
further investigation?
A. Decreased serum cortisol
B. Hyperestrogenism
C. Hyperglycemia

1061
D. Increased serum-free thyroxine
----------------------------------------
4. What is the most likely cause of a 34-year-old woman presenting with unexplained weight gain,
central obesity, "moon-faced" appearance, purple-colored stretch marks, increased hair growth in male
distribution patterns, elevated blood pressure, and elevated blood sugar, with a smooth, homogeneous
lesion in the left adrenal gland on computed tomography?
A. Adrenal insufficiency
B. Ectopic production of adrenocorticotropin
C. Hyperproduction of adrenal glucocorticoids
D. Hyperproduction of hypothalamic corticotropin-releasing factor
----------------------------------------
5. What's the likely diagnosis for a 34-year-old man with hypertension, persistent hypokalemia despite
oral potassium supplements, elevated serum sodium, normal glucose, normal weight, and a 3-cm mass
in the right adrenal gland on computed tomography?
(or)
A 34-year-old has hypertension and persistent hypokalemia. Serum sodium is 149 mEq/L; potassium,
3.3 mEq/L; and a 3-cm mass in the right adrenal gland. What is the most likely diagnosis?
A. Conn syndrome
B. Addison disease
C. Cushing syndrome
D. Sipple syndrome
----------------------------------------
6. Which deficiency is suggested by ambiguous genitalia with an enlarged clitoris resembling a penis,
along with hyponatremia, hyperkalemia, and hypotension in a newborn?
A. 11-Hydroxylase
B. 17-Hydroxylase
C. 21-Hydroxylase
D. Insulin
----------------------------------------
7. What is the most likely diagnosis in an 18-year-old female with fever, hypotension, obtunded mental
status, petechial/purpuric hemorrhages, neutrophils engulfing gram-negative diplococci, low sodium,
high potassium, and coagulation tests revealing increased prothrombin time, activated partial
thromboplastin time, fibrin-fibrinogen split products?
A. Conn syndrome
B. Hyper prolactinoma
C. Neuroblastoma
D. Waterhouse-Friderichsen syndrome
----------------------------------------
8. Which of the following diagnostic procedures can be used to diagnose pheochromocytoma?

Page 2

1062
A. Serum C-peptide
B. Serum calcitonin
C. Urinary vanillylmandelic acid
D. Serum aldosterone
----------------------------------------
9. Which hormones are likely to be elevated in laboratory studies for a 60-year-old woman with small
cell carcinoma of the lung, presenting with symptoms including rounding of the face, upper truncal
obesity, muscle weakness, thin skin with abdominal striae, multiple purpuric skin lesions, and a blood
pressure of 176/96 mm Hg?
A. Aldosterone
B. Corticotropin
C. Epinephrine
D. Prolactin
----------------------------------------
10. Which of the following indicated good prognosis in neuroblastoma?
A. Age of the patient is 5 years
B. Presence of TRK A
C. Presence of TRK B
D. Absence of NMYC
----------------------------------------
11. Which of the following is the most reliable feature of malignant transformation of
pheochromocytoma?
(or)
Which of the following is the most reliable feature of malignant transformation of pheochromocytoma?
A. Presence of mitotic figures
B. Presence of metastasis to other organs
C. Vascular/ capsular invasion
D. All of the above
----------------------------------------
12. Which of the following best describes the pattern of arrangement of cells in Pheochromocytoma?
A. Alveolar Pattern
B. Sheet-like Pattern
C. Nesting Pattern
D. Storiform Pattern
----------------------------------------
13. What is the most likely diagnosis for a 32-year-old female with amenorrhea, galactorrhea, visual
field defects, and a hypophyseal mass impinging on the optic chiasma?

Page 3

1063
A. Prolactinoma
B. Somatotropic adenoma
C. Corticotropic adenoma
D. Craniopharyngioma
----------------------------------------
14. Which of the following is likely to be increased in a 46-year-old man exhibiting recent-onset
diabetes mellitus alongside facial coarseness, frontal bossing, thickened nose, prognathism,
macroglossia, enlarged extremities with sausage-like fingers, and inability to wear his wedding ring?
A. Corticotropin
B. Dopamine
C. Insulin like growth factor-I
D. Prolactin
----------------------------------------
15. What conditions is a 44-year-old female patient with a history of recurrent asthmatic exacerbations
at increased risk for when started on oral corticosteroids?
A. Osteopetrosis
B. Dislocation
C. Osteomalacia
D. Osteoporosis
----------------------------------------

Correct Answers
Question Correct Answer

Question 1 3
Question 2 1
Question 3 1
Question 4 3
Question 5 1
Question 6 3
Question 7 4
Question 8 3
Question 9 2
Question 10 2
Question 11 2
Question 12 3
Question 13 1

Page 4

1064
Question 14 3
Question 15 4

Solution for Question 1:


Correct Option C - Osteitis Fibrosa Cystica:
• Given the patient's history of end-stage renal disease and renal dialysis, along with the presentation of
pain in the jaw and left arm, the most likely diagnosis is osteitis fibrosa cystica. This condition is a
skeletal manifestation of hyperparathyroidism secondary to chronic kidney disease (CKD).
• This disease occurs due to hyperparathyroidism.
• In patients with CKD, the kidneys are unable to excrete phosphate efficiently, leading to
hyperphosphatemia and secondary hyperparathyroidism. Excessive secretion of parathyroid hormone
(PTH) results in bone resorption, leading to the formation of cystic lesions within the bones, particularly
in areas rich in trabecular bone, such as the jaw and long bones. Pathologic fractures can occur due to
the weakened bone structure.
• Osteitis fibrosa cystica is a severe bone deformity that causes "brown tumors" (non-neoplastic).
• Clinical Features include: Bone pain Bone cysts Pathologic fractures Localised bone swellings (due to
brown tumors).
• Bone pain
• Bone cysts
• Pathologic fractures
• Localised bone swellings (due to brown tumors).
• Bone pain
• Bone cysts
• Pathologic fractures
• Localised bone swellings (due to brown tumors).
• Histopathology shows: Brown tumor comprises numerous giant cells with interstitial hemorrhage,
hemosiderin, and vascularity.
• Brown tumor comprises numerous giant cells with interstitial hemorrhage, hemosiderin, and
vascularity.
• Brown tumor comprises numerous giant cells with interstitial hemorrhage, hemosiderin, and
vascularity.
Incorrect Options:
Option A - Chronic Osteomyelitis:
• Chronic osteomyelitis typically presents with localized bone pain, fever, and inflammation, and is
unlikely in this case given the patient's clinical history and findings.
Option B - Marble Bone Disease:
• Marble bone disease, also known as osteopetrosis, is characterized by abnormally dense and brittle
bones, which would not present with cystic lesions.
Option D - Osteoid Osteoma:

Page 5

1065
• Osteoid osteoma is a benign bone tumor characterized by a painful nidus surrounded by reactive
bone formation, which is not consistent with the cystic lesions described in this patient.

Solution for Question 2:


Correct Option A - 90% are malignant:
• This statement is incorrect. 90% of pheochromocytomas are benign only 10% are malignant
Pheochromocytoma:
• A pheochromocytoma is a catecholamine-secreting tumor that typically develops in the adrenal
medulla.
• They are usually benign (∼ 90% of cases) but may also be malignant.
• Classic clinical features are due to excess sympathetic nervous system stimulation and involve
episodic blood pressure crises with paroxysmal headaches, diaphoresis, palpitations, and tremors.
• They may also be asymptomatic or manifest with persistent hypertension.
• Elevated catecholamine metabolites in the plasma or urine confirm the diagnosis, while imaging
studies in patients with positive biochemistry are used to determine the tumor's location.

• Surgical resection is the treatment of choice but is only carried out once alpha-adrenergic blockade
with phenoxybenzamine has become effective.
• Pheochromocytoma-rule of 10 10% malignant 10% Bilateral 10% extra-adrenal 10% calcified 10%
children 10% of adrenal pheochromocytomas are not associated
• 10% malignant
• 10% Bilateral
• 10% extra-adrenal
• 10% calcified
• 10% children
• 10% of adrenal pheochromocytomas are not associated

Page 6

1066
• 10% malignant
• 10% Bilateral
• 10% extra-adrenal
• 10% calcified
• 10% children
• 10% of adrenal pheochromocytomas are not associated
Incorrect Options:
Option B - 95% occur in the abdomen:
• Almost 95% of tumours are abdominal in location since 90% arise from the adrenal medulla.
Option C - They secrete catecholamines.:
• These tumors secrete catecholamines like epinephrine, norepinephrine, dopamine.
Option D - They arise from sympathetic ganglia:
• Extra-adrenal pheochromocytomas occur in the sympathetic ganglia in the neck, mediastinum,
bladder, and organ of Zukercandl [aortic bifurcation]

Solution for Question 3:


Correct Option A - Decreased serum cortisol:
• Given history is strongly suggestive of panhypopituitarism due to ischemic necrosis of the pituitary,
occurring as sequelae to childbirth complicated by hemorrhagic shock (Sheehan syndrome).
• This syndrome is characterized by deficiencies in gonadotropin, corticotropin, and thyrotropin, both
clinically and in the laboratory.
Sheehan syndrome:

Page 7

1067
• Sheehan syndrome (i.e., postpartum hypopituitarism) is a rare but potentially life-threatening
complication.
• The pituitary gland is enlarged in pregnancy and prone to infarction from hypovolemic shock.
• Damage to the pituitary can be mild or severe and can affect the secretion of one, several, or all of its
hormones.
• A common presentation is a combination of failure to lactate postpartum and amenorrhea or
oligomenorrhea. Still, any manifestations of hypopituitarism (e.g., hypotension, hyponatremia,
hypothyroidism) can occur anytime from the immediate postpartum period to years after birth.
Sheehan syndrome
Pathogenesis
• Obstetric hemorrhage complicated by hypotension leads to postpartum pituitary infarction
Clinical features
• Lactation failure (↓ prolactin)
• Amenorrhea, hot flashes, vaginal atrophy (↓FSH, LH)
• Fatigue, bradycardia (↓ TSH)
• Anorexia, weight loss, hypotension (↓ ACTH)
• Decreased lean body mass (↓ growth hormone)
Incorrect Options:
Option B - Hyperestrogenism:
• Sheehan syndrome leads to hypogonadotropic hypogonadism.
• This results in low estrogen, usually manifested as a lack of menstruation.
Option C - Hyperglycemia:
• Because of the lack of ACTH, hypoglycemia may occur in the Sheehan syndrome.
• However, hyperglycemia is not observed.
Option D - Increased serum-free thyroxine:
• There is decreased TSH in Sheehan syndrome.'
• This results in low thyroxin levels in the serum

Solution for Question 4:


Correct Option C - Hyperproduction of adrenal glucocorticoids:
• The given illustration demonstrates a well-circumscribed adrenal cortical adenoma that results in
hypersecretion of adrenal glucocorticoids [Cushing syndrome].
• Hypothalamic, pituitary, & ectopic causes of Cushing syndrome result in bilateral adreno-cortical
hyperplasia.
Hypercortisolism:
• Hypercortisolism results from excess glucocorticoids (iatrogenic Cushing syndrome).

Page 8

1068
• The adrenal gland's autonomous overproduction of cortisol is the cause of primary hypercortisolism
(e.g., due to an adrenal adenoma or carcinoma). Adrenocorticotropic hormone (ACTH) is produced
excessively by either a pituitary adenoma (Cushing disease) or ectopic, paraneoplastic foci, leading to
secondary hypercortisolism (e.g., in patients with small cell lung cancer).
• The typical clinical characteristics are central obesity, thin, easily bruised skin, abdominal striae,
secondary Hypertension, hyperglycemia, and proximal muscular weakness.
• To treat endogenous hypercortisolism, the source of excessive cortisol or ACTH production is typically
surgically removed (adrenalectomy or transsphenoidal hypophysectomy).
• In addition to surgery, pharmacological therapy to normalize cortisol levels, such as metyrapone, may
be utilized as a backup plan if surgery is unsuccessful.
Incorrect Options:
Option A - Adrenal insufficiency:
• Adrenal insufficiency would typically present with weight loss rather than unexplained weight gain.
• Symptoms might include fatigue, weakness, and hypotension, which are not consistent with the
described clinical findings.
Option B - Ectopic production of adrenocorticotropin:
• Ectopic ACTH production would result in hypertrophy of the adrenal gland.
• The adrenal enlargement would be bilateral.
Option D - Hyperproduction of hypothalamic corticotropin-releasing factor:
• CRH release from the hypothalamus would result in bilateral enlargement of adrenal glands

Solution for Question 5:


Correct Option A - Conn syndrome:
• Combination of hypertension, persistent hypokalemia & slightly elevated serum sodium→
hyperaldosteronism and mass in the adrenal gland suggests Conn syndrome (primary
hyperaldosteronism).
• Primary Hyperaldosteronism, sometimes referred to as Conn syndrome, is an excess of aldosterone
caused by autonomous overproduction.
• It is typically due to adrenal hyperplasia (most commonly bilateral) or adrenal adenoma (typically
unilateral).
• Primary Hyperaldosteronism is a common cause of secondary Hypertension, occurring in > 5–12% of
hypertensive patients.
• High systemic aldosterone levels result in increased renal sodium reabsorption and potassium
secretion, which lead to water retention and hypokalemia.
• Patients are often asymptomatic and found to have Hypertension at routine health checks.
• Hypertension due to primary Hyperaldosteronism is often resistant to pharmacotherapy, and patients
may have other signs suggestive of secondary Hypertension, such as onset before the age of 30 or
after the age of 55.

Page 9

1069
• If symptoms are present, these are usually manifestations of hypokalemia (e.g., headache, muscle
weakness, and polyuria).
• Initial laboratory values in primary Hyperaldosteronism classically show hypokalemia, hypernatremia,
metabolic alkalosis, high plasma aldosterone concentration (PAC), and low plasma renin activity (PRA).
• The plasma aldosterone-to-renin ratio is the confirmatory test.
• Treating unilateral disease involves surgical resection, whereas bilateral disease is managed
medically with aldosterone antagonists (e.g., spironolactone, eplerenone).
Incorrect Options:
Option B - Addison's disease:
• It is characterized by a low level of aldosterone and glucocorticoids.
• It is caused by autoimmune adrenalitis.
• It can lead to hypotension that is often refractory to the fluid administration.
• Treatment includes fludrocortisone.
Option C - Cushing syndrome:
• It is characterized by pink striae, buffalo hump, and Hypertension.
• Other features include poor wound healing and hyperglycemia.
• It can have some features due to mineralocorticoid activities but has other features as well.
Option D - Sipple syndrome:
• A rare genetic disorder affects the endocrine glands and causes a type of thyroid cancer called
medullary thyroid cancer, pheochromocytoma, and parathyroid gland cancer.
• It may also cause benign (noncancerous) tumors in the parathyroid glands and adrenal glands.

Solution for Question 6:


Correct Option C - 21-Hydroxylase:
Congenital adrenal hyperplasia:
• Congenital adrenal hyperplasia (CAH) encompasses a group of autosomal recessive defects in the
enzymes responsible for cortisol, aldosterone, and, in rare cases, androgen synthesis.
• All forms of CAH are characterized by low levels of cortisol, high levels of ACTH, and adrenal
hyperplasia.
• The exact clinical manifestations depend on the enzyme defect.
• A deficiency of 21β-hydroxylase causes the most common form of CAH. It manifests with hypotension
(because of decreased aldosterone synthesis), ambiguous genitalia, virilization (in the female
genotype), and/or precocious puberty (in both males and females).
• Certain rare forms of CAH (e.g., 11β-hydroxylase and 17α-hydroxylase deficiencies) manifest with
symptoms of mineralocorticoid excess (e.g., hypertension) and therefore require spironolactone
(aldosterone receptor inhibitor) in addition to glucocorticoid replacement.
Incorrect Options:

Page 10

1070
Option A - 11-Hydroxylase:
• The deficiency of 11 hydroxylases can cause ambiguous genitalia.
• However, hypertension occurs despite hypoaldosteronism because deoxycorticosterone possesses
mineralocorticoid (aldosterone-like) activity.
Option B - 17-Hydroxylase:
• Usually presents with hypertension symptoms
• Normal female external genitalia at birth and delayed puberty (primary amenorrhea) or sexual
infantilism
• Male pseudohermaphroditism: female external genitalia with a blind-ending vagina and
intra-abdominal testes at birth, Delayed puberty, or sexual infantilism
Option D - Insulin:
• Insulin deficiency results in diabetes
• Maternal diabetes leads to fetal macrosomia
• This may result in multiple birth injuries and shoulder dystocia.
• However, ambiguous genitalia are not seen.

Solution for Question 7:


Correct Option D - Waterhouse-Friderichsen syndrome:
• The patient presents with signs & symptoms of Waterhouse Friderichsen syndrome, a devastating
consequence of disseminated meningococcal infection.
• The clinical presentation of Waterhouse-Friderichsen syndrome includes sudden onset of fever,
hypotension, altered mental status, and petechial or purpuric hemorrhages due to disseminated
intravascular coagulation (DIC).
• It is characterized by hemorrhagic destruction of the adrenals complicated by disseminated
intravascular coagulation.
Waterhouse-Friderichsen syndrome:
Epidemiology:
• Predominantly affects small children and asplenic individuals
Etiology:
• Hemorrhagic adrenal insufficiency.
• A dangerous complication of several diseases but most commonly associated with meningococcal
meningitis
• Rarer causes include DIC, endotoxic shock, and septicemia due to other pathogens (e.g., S.
pneumoniae)
Clinical features:
• Acute adrenal gland failure
• Fever

Page 11

1071
• Myalgia
• Nonblanching, petechial rash (mostly on trunk and legs); in severe cases, even purpura fulminans
with extensive necrosis of the skin
• Severe malaise
• Hypotension or even shock
• Disseminated intravascular coagulation
• Respiratory failure
Treatment:
• Glucocorticoid replacement (main treatment)
• Supportive measures for shock, electrolyte abnormalities, hypoglycemia, and DIC
• Treatment of underlying cause
Incorrect Options:
Option A - Conn syndrome:
• Characterized by severe Hypertension
• There is muscle weakness secondary to low potassium
• The aldosterone level is usually high.
• The renin is low due to feedback inhibition
Option B - Hyper prolactinoma:
• Presents with irregular menstruation.
• Milky discharge occurs from the breasts when not pregnant or breastfeeding.
• Painful intercourse due to vaginal dryness and acne and excessive body and facial hair growth can be
observed
Option C - Neuroblastoma:
• The most common primary childhood malignancy
• Arises from the adrenal glands and sympathetic ganglia
• Can metastasize to orbits and the central nervous system
• Hypertension can occur due to increased levels of catecholamines.

Solution for Question 8:


Correct Option C - Urinary vanillylmandelic acid:
• Urinary vanillylmandelic acid & metanephrines are metabolites of epinephrine & norepinephrine:
Markedly elevated in pheochromocytoma.
Pheochromocytoma:
• A pheochromocytoma is a catecholamine-secreting tumor that typically develops in the adrenal
medulla.

Page 12

1072
• Pheochromocytomas are usually benign (∼ 90% of cases) but may also be malignant.
• Classic clinical features are due to excess sympathetic nervous system stimulation and involve
episodic blood pressure crises with paroxysmal headaches, diaphoresis, heart palpitations, and
tremors.
• Elevated catecholamine metabolites in the plasma (fractionated metanephrine) or urine (VMA) confirm
the diagnosis while imaging studies in patients with positive biochemistry are used to determine the
tumor's location.
• Surgical resection is the treatment of choice but is only carried out once alpha-adrenergic blockade
with phenoxybenzamine has become effective.
Incorrect Options:
Option A - Serum C-peptide:
• It is released along with insulin in equal quantity from the pancreas
• The main clinical use of C-peptide is to diagnose insulinoma with elevated C-peptide
Option B - Serum calcitonin:
• Used to differentiate between causes of hypocalcemia.
• Calcitonin is produced by the thyroid gland, which lowers calcium levels.
Option D - Serum aldosterone:
• Serum aldosterone, is used in the diagnosis of primary aldosteronism, which is characterized by
excess aldosterone production from the adrenal glands. It is not specific for pheochromocytoma.
• Serum aldosterone is high in Conn syndrome.

Solution for Question 9:


Correct Option B - Corticotropin:
• Given features point towards Cushing syndrome.
• This patient most likely has a paraneoplastic condition associated with corticotropin (ACTH)-producing
small cell carcinoma of the lung [i.e., ectopic secretion of ACTH]
Hypercortisolism:
• Hypercortisolism results from excess glucocorticoids .
• The adrenal gland's autonomous overproduction of cortisol causes primary hypercortisolism (e.g., due
to an adrenal adenoma or carcinoma). Adrenocorticotropic hormone (ACTH) is produced excessively
by either a pituitary adenoma (Cushing disease) or ectopic, paraneoplastic foci, leading to secondary
hypercortisolism (e.g., in patients with small cell lung cancer).
• The typical clinical characteristics are central obesity, thin, easily bruised skin, abdominal striae,
secondary hypertension, hyperglycemia, and proximal muscular weakness.
• To treat endogenous hypercortisolism, the source of excessive cortisol or ACTH production is typically
surgically removed (adrenalectomy or transsphenoidal hypophysectomy).
• In addition to surgery, pharmacological therapy to normalize cortisol levels, such as metyrapone, may
be utilized as a backup plan if surgery is unsuccessful.

Page 13

1073
Incorrect Options:
Option A - Aldosterone:
• Aldosterone is high in patients with Conn syndrome
• This leads to elevated blood pressure and persistent hypokalemia.
• Mostly occurs secondary to adrenal hyperplasia.
Option C - Epinephrine:
• Epinephrine levels are high in patients with pheochromocytoma.
• This results in hypertension.
Option D - Prolactin:
• Hyperprolactinemia is due to elevated prolactin levels
• Mostly due to pituitary adenoma.
• Amenorrhea is a common complaint.
• Lack of lactation is another common problem.

Solution for Question 10:


Correct Option B - Presence of TRK A:
• The presence of TRK A is considered in neuroblastoma. TRK (tropomyosin receptor kinase) genes
encode for receptors involved in neurotrophin signaling pathways, which play crucial roles in neuronal
development and survival. In neuroblastoma, the presence of TRK A indicates a differentiated and
more mature neuronal phenotype, which is associated with a better prognosis. Conversely, the
absence of TRK A or the presence of TRK B is often associated with less differentiated, more
aggressive neuroblastoma tumors and poorer prognosis.
• Prognostic Factors Good Bad Age <18 months >18 months Differentiation Gangliocytic or
Schwannian Others MKI (Mitosis-Karyorrhexis Index <200/5000 cells >200/5000 cells DNA Ploidy
Hyperdiploid Hypodiploid TRK A Present Absent TRK B Absent Present Chromosome 1p loss -
Present Chromosome 11q loss - Present NMYC - Amplified
The presence of TRK A is considered in neuroblastoma. TRK (tropomyosin receptor kinase) genes enc
ode for receptors involved in neurotrophin signaling pathways, which play crucial roles in neuronal dev
elopment and survival. In neuroblastoma, the presence of TRK A indicates a
differentiated and more mature neuronal phenotype, which is associated with a better prognosis.
Conversely, the absence of TRK A or the presence of TRK B is often associated with less differentiate
d, more aggressive neuroblastoma tumors and poorer prognosis.

Incorrect Options:
Options A, C and, D are indicative of a bad prognosis and is explained under option B

Solution for Question 11:

Page 14

1074
Correct Option B - Presence of metastasis to other organs:
• There is no histologic feature that reliably predicts malignant pheochromocytoma.
• Cellular & nuclear pleomorphism, including the presence of giant cells & mitotic figures, are often seen
in benign pheochromocytomas, while cellular monotony is paradoxically associated with aggressive
tumours.
• Even capsular and vascular invasion may be encountered in benign lesions.
• Therefore, the definitive diagnosis of malignancy in pheochromocytomas is based exclusively on the
presence of metastases.

Incorrect Options:
Option A - Presence of mitotic figures:
• The presence of mitotic figures is characteristic of any malignancy.
• However, these findings cannot predict the metastatic transformation in pheochromocytoma.
Option C - Vascular/ capsular invasion:
• Vascular invasion or capsular invasion does not predict the metastatic transformation of the
pheochromocytoma
• Rather, metastasis to other organs is the most reliable predictor.
Option D - All of the above:
• The most reliable predictor of malignant transformation of pheochromocytoma is metastasis to other
organs.

Solution for Question 12:


Correct Option C - Nesting Pattern:
• The pattern of arrangement of cells in pheochromocytoma is typically described as a Nesting Pattern.
• In a nesting pattern, the tumor cells arrange themselves into nests or clusters, often resembling a
"nesting" or "Zellballen" pattern. These nests can vary in size and shape and are surrounded by a
fibrovascular stroma. This pattern is typical of neuroendocrine tumors, where tumor cells tend to
aggregate into cohesive clusters, delineated by delicate fibrovascular septa. This pattern is often
observed in tumors such as carcinoid tumors, neuroendocrine carcinomas, and paragangliomas.

• Other features of pheochromocytoma include, the cytoplasm exhibiting a finely granular appearance,
particularly evident with silver stains, attributed to granules containing catecholamines. Nuclei typically
display a round to ovoid shape, with a stippled "salt-and-pepper" chromatin pattern characteristic of
neuroendocrine tumors. Electron microscopy reveals varying numbers of membrane-bound,
electron-dense secretory granules. Immunoreactivity for neuroendocrine markers such as
chromogranin and synaptophysin is observed in chief cells, while peripheral sustentacular cells stain
positively with antibodies against S-100.

Page 15

1075
Incorrect Options:
Option A - Alveolar Pattern:
• Cells arranged in small, round, or polygonal patterns resembling the alveoli of the lung.
Option B - Sheet-like Pattern:
• Cells arranged in solid sheets without forming any specific structures.
Option D - Storiform Pattern:
• Cells arranged in a cartwheel-like or radial pattern with cells radiating outward.

Solution for Question 13:


Correct Option A - Prolactinoma:
• The clinical presentation of amenorrhea, galactorrhea, and visual field defects, along with a
hypophyseal mass observed on magnetic resonance imaging (MRI) impinging on the optic chiasma,
strongly suggests the diagnosis of a prolactinoma.
• Amenorrhea is the outcome of hyperprolactinemia, which leads to feedback inhibition of GnRH.
• clinical features include galactorrhea, decreased libido, and infertility.
• The tumor is typically small in size; however, approximately 10% of cases reach a significant size,
potentially affecting adjacent structures. Bitemporal hemianopia, resulting from pressure on the optic
chiasm, is a frequent manifestation in affected individuals.
Incorrect Options:
Option B - Somatotropic adenoma:
• A somatotropic adenoma is a type of pituitary adenoma that produces excess growth hormone (GH).
These adenomas arise from the somatotroph cells of the anterior pituitary gland.

Page 16

1076
• It causes acromegaly in adults and gigantism in children.
Option C - Corticotropic adenoma:
• Corticotropic adenoma, refers to adrenocorticotropic hormone (ACTH)-secreting adenomas, which
usually cause Cushing's disease, characterized by features such as central obesity, moon facies, and
muscle weakness, rather than the symptoms described.
Option D - Craniopharyngioma:
• Craniopharyngioma, is a benign tumor arising from remnants of Rathke's pouch and typically presents
with symptoms related to mass effect, such as headaches, visual disturbances, and hormonal
deficiencies. While it can cause hypopituitarism, it typically does not present with the specific symptoms
of amenorrhea, galactorrhea, and visual field defects described in the scenario.

Solution for Question 14:


Correct Option C - Insulin-like growth factor-I:
• The clinical presentation described in the scenario, including the recent onset of diabetes mellitus,
facial changes (coarseness, frontal bossing, thickening of the nose, prognathism, and macroglossia),
and enlargement of extremities with sausage-like fingers, is characteristic of acromegaly, a condition
caused by increased secretion of growth hormone (GH) and subsequent elevated levels of insulin-like
growth factor-I (IGF-I).
• GH, produced by the anterior pituitary gland, stimulates the production of IGF-I in the liver and other
tissues. Excess GH and IGF-I lead to the clinical features of acromegaly, including characteristic facial
changes, enlarged hands and feet, joint pain, and organ enlargement.
• Acromegaly is characterized by several physical characteristics, including frontal bossing, thickening
of the nose, prognathism, enlargement and increased prominence of the jaw, macroglossia, and
enlarged extremities with sausage-like fingers.
• Acromegaly is a disorder that is caused by a pituitary somatotropic adenoma, which is the second
most common sort of pituitary adenoma that is still functional.
• Pituitary adenoma can also cause acromegaly, pituitary adenomas can cause various other medical
conditions, including diabetes mellitus, impaired glucose tolerance, and gigantism in children.
• Patients can also present with bitemporal hemianopsia due to the pressure effects of the adenoma.
Incorrect Options:
Option A - Corticotropin:
• Corticotropin (adrenocorticotropic hormone, ACTH), is associated with conditions such as Cushing's
disease when secreted in excess, but it does not cause the specific features described in the scenario.
Option B - Dopamine:
• Dopamine, is a neurotransmitter and hormone involved in various physiological processes, but
increased activity of dopamine is not typically associated with the clinical features described in the
scenario.
Option D - Prolactin:
• Prolactin, when elevated, can cause symptoms such as galactorrhea and menstrual irregularities, but
it does not typically lead to the characteristic features of acromegaly described in the scenario.

Page 17

1077
Solution for Question 15:
Correct Option D - Osteoporosis:
• Long-term use of oral corticosteroids, such as prednisone, can lead to a variety of adverse effects,
including bone loss and increased risk of osteoporosis. Corticosteroids interfere with the normal bone
remodeling process by inhibiting bone formation and promoting bone resorption.
• This imbalance results in decreased bone mineral density and increased susceptibility to fractures. It
also causes avascular necrosis of bones
• Therefore, patients on long-term oral corticosteroid therapy should be monitored closely for signs of
osteoporosis, and preventive measures, such as calcium and vitamin D supplementation and lifestyle
modifications, should be considered to minimize the risk of bone complications.
Incorrect Options:
Options A, B, and C are incorrect. Refer to the explanation of Option D.

Page 18

1078
Previous Year Questions
1. What is the probable diagnosis for a 19-year-old female with a neck swelling that shifts during
swallowing? Ultrasound reveals a 2 cm x 2 cm mass in the left lobe of the thyroid, reaching the strap
muscles. Refer to the provided histopathology images.

A. Medullary CA thyroid
B. Follicular CA thyroid
C. Thyroglossal cyst
D. Papillary CA thyroid
----------------------------------------
2. The following is a histopathological image of thyroid pathology. What is the diagnosis ?

A. Medullary carcinoma of thyroid


B. Papillary carcinoma of thyroid
C. Follicular carcinoma of thyroid
D. Anaplastic carcinoma of thyroid
----------------------------------------
3. Neurosecretory bodies are seen in all except
A. Medullary thyroid carcinoma
B. Pituitary adenoma
C. Paraganglioma

1079
D. Adrenal cortical tumor
----------------------------------------
4. A patient presents with midline swelling in the neck. A biopsy revealing Orphan Annie's eye nuclei is
shown in the image below. What is the diagnosis?

A. Follicular carcinoma
B. Papillary carcinoma
C. Toxic nodular goitre
D. Medullary carcinoma
----------------------------------------
5. A middle-aged woman presents with thyroid swelling. Blood tests show elevated TSH levels and the
presence of thyroid peroxidase antibodies. Histopathological examination revealed the presence of
Hurthle cells. What is the most likely diagnosis?
A. Hashimoto thyroiditis
B. Graves’ disease
C. Follicular carcinoma of thyroid
D. Medullary carcinoma of thyroid
----------------------------------------
6. Which of the following statements accurately describe the impact of Glucose-6-Phosphate
Dehydrogenase deficiency? Decrease in reduced glutathione Decrease NADPH Decreased lipid
peroxidation Decreased RBC membrane damage
A. Only 1 is correct
B. 1&2 are correct
C. 2&3 are correct
D. 1,2 & 4 are correct
----------------------------------------
7. What is the probable diagnosis for a patient who has a swelling in the middle of their neck, along with
weight loss, and a thyroid biopsy showing the presence of 'Orphan Annie eye nuclei'?
A. Follicular thyroid carcinoma
B. Papillary thyroid carcinoma

Page 2

1080
C. Anaplastic thyroid carcinoma
D. Medullary thyroid carcinoma
----------------------------------------
8. What is the likely diagnosis for a patient who presents with a swelling in the midline of the neck,
based on the biopsy findings which reveal orphan Annie's eye nuclei as depicted in the image
provided?

A. Follicular carcinoma
B. Papillary carcinoma
C. Toxic nodular goiter
D. Medullary carcinoma
----------------------------------------
9. What is the most probable diagnosis for a male patient, 34 years of age, presenting at the outpatient
department with intermittent headaches, palpitations, and excessive sweating? During the physical
examination, the patient's chest exhibits pectus excavatum and an increased ratio of extremities to
body size. Additionally, there is observed growth on the tongue and a palpable mass on the left flank.
A. MEN 1
B. MEN 2A
C. MEN2B
D. Neurofibromatosis type 2
----------------------------------------
10. Pheochromocytoma produces all except?
A. Nor-epinephrine
B. Secretin
C. Vasoactive intestinal polypeptide
D. Somatostatin
----------------------------------------
11. In the scenario presented, a patient visited the outpatient department (OPD) exhibiting elongated
hands and feet, as well as a protruding lower jaw. Which gene among the options provided is most
likely associated with this condition?
A. GATA1

Page 3

1081
B. PIT1
C. TPIT
D. SF1
----------------------------------------
12. What is the most suitable biopsy specimen for a thyroid biopsy?
A. 15 follicular cell clusters contain 10-12 cells
B. 10 follicular cell clusters contain 6-8 cells
C. 3 follicular cell clusters contain 10-15 cells
D. 6 follicular cell clusters contain 10-15 cells
----------------------------------------

Correct Answers
Question Correct Answer

Question 1 4
Question 2 1
Question 3 4
Question 4 2
Question 5 1
Question 6 2
Question 7 2
Question 8 2
Question 9 3
Question 10 2
Question 11 2
Question 12 4

Solution for Question 1:


Correct Option D - Papillary CA thyroid:
• Papillary carcinoma of the thyroid is the most common type of thyroid cancer, accounting for
approximately 80% of cases. It typically presents as a solitary nodule on the thyroid gland and may
extend into the surrounding tissues. The histopathology image provided shows the characteristic
features of papillary carcinoma, including papillary structures lined by tall columnar cells with nuclear
features such as nuclear clearing, nuclear grooves, and intranuclear cytoplasmic inclusions (Orphan
Annie eye nuclei).
Incorrect Options:
Option A - Medullary carcinoma of the thyroid: Medullary carcinoma of the thyroid is a
type of thyroid cancer that arises from the parafollicular C cells, which produce calcitonin. The histopat
hology image provided does not show the characteristic features of medullary carcinoma, such as nest

Page 4

1082
s or clusters of polygonal cells with amyloid deposits.
Option B - Follicular carcinoma of the thyroid: Follicular carcinoma of the thyroid is a type of thyroid can
cer that arises from the follicular cells of the thyroid gland. The histopathology image provided does not
show the characteristic features of follicular carcinoma, such as invasion of the tumor capsule or blood
vessels.
Option C - Thyroglossal cyst: A thyroglossal cyst is a congenital anomaly that occurs due to the persist
ence of remnants of the thyroglossal duct. It typically presents as a midline neck swelling that moves o
n deglutition. However, the histopathology image provided does not show the features of a
thyroglossal cyst, such as a cystic structure lined by stratified squamous or columnar epithelium.

Solution for Question 2:


Correct Option A - Medullary carcinoma of thyroid:
• The histopathological image shows round, polygonal, or spindle cells in nests or follicles indicating
medullary carcinoma of the thyroid. Medullary carcinoma of the thyroid is a rare type of thyroid cancer
that arises from the parafollicular C cells of the thyroid gland. It is typically associated with elevated
levels of calcitonin and can be sporadic or familial.
Incorrect Options:
Option B - Papillary Carcinoma of Thyroid: Papillary carcinoma is the most common type of thyroid can
cer, accounting for the majority of cases. It originates from the follicular cells of the thyroid gland and is
characterized by distinctive nuclear features. It often presents as a slow-growing tumor with a
favorable prognosis.
Option C - Follicular Carcinoma of Thyroid: Follicular carcinoma is another type of thyroid cancer that a
rises from the follicular cells of the thyroid gland. It is less common than papillary carcinoma but has a
higher potential for spreading to distant sites (metastasis), thus shows more frequent mitoses.
Option D - Anaplastic Carcinoma of Thyroid: Anaplastic carcinoma is a highly aggressive and rare form
of thyroid cancer. It is characterized by undifferentiated malignant cells that rapidly grow and invade su
rrounding structures. Anaplastic carcinoma has a
poor prognosis and typically requires aggressive treatment.

Solution for Question 3:


Correct Option D - Adrenal cortical tumor:
• Neurosecretory bodies are specialized cells that are found in the nervous system and are responsible
for the synthesis and secretion of neurohormones.
• They are typically located in the hypothalamus.
• Neurosecretory cells have both neural and endocrine functions, as they can receive and process
neural signals, as well as release hormones into the bloodstream.
Incorrect Options:
• Option A, B and C are associated with the presence of neurosecretory bodies.

Page 5

1083
Solution for Question 4:
Correct Option B - Papillary carcinoma:
• Papillary carcinoma is a type of thyroid cancer that is characterized by the presence of specific
nuclear features, including 'Orphan Annie's eye nuclei'.
• This term refers to nuclei that are enlarged, irregularly shaped, and optically clear, giving them the
appearance of an 'Orphan Annie's eye'.
Incorrect Options:
Options A, C, and D do not exhibit the nuclear features of 'Orphan Annie's eye nuclei'.

Solution for Question 5:


Correct Option A - Hashimoto thyroiditis:
• Hashimoto thyroiditis, also known as chronic lymphocytic thyroiditis, is an autoimmune disorder
characterized by chronic inflammation of the thyroid gland.
• It is the most common cause of hypothyroidism in areas with sufficient iodine intake.
• In Hashimoto thyroiditis, thyroid peroxidase antibodies (anti-TPO antibodies) are present.
• Histopathological examination often reveals the presence of Hurthle cells, which are enlarged,
eosinophilic follicular cells with abundant cytoplasm.
Incorrect Options:
Options B, C, and D do not present with Hurthle cells on histopathological examination.

Solution for Question 6:


Correct Option B:
Decrease in reduced glutathione: Glucose-6-phosphate dehydrogenase (G6PD) deficiency leads to a d
ecrease in the production of NADPH, which is required for the regeneration of reduced glutathione (GS
H). GSH is an important antioxidant that helps protect cells from oxidative damage. In G6PD deficiency
, the decreased NADPH levels result in reduced regeneration of GSH, leading to a decrease in its conc
entration. This reduction in reduced glutathione makes the cells more susceptible to oxidative stress.
Decrease in NADPH : G6PD deficiency results in a decrease in the production of NADPH, as G6PD is i
nvolved in the pentose phosphate pathway, which generates NADPH. NADPH is essential for various c
ellular functions, including maintaining the reduced state of glutathione and providing reducing equivale
nts for biosynthetic reactions and detoxification processes. The decreased levels of NADPH in G6PD d
eficiency can disrupt these cellular processes, leading to increased susceptibility to oxidative stress an
d impaired cellular function.
Incorrect Option:
Option C. Decreased lipid peroxidation:

Page 6

1084
G6PD deficiency does not directly cause a decrease in lipid peroxidation. In fact, G6PD deficiency can
result in increased susceptibility to oxidative stress and reactive oxygen species (ROS) production. The
decreased levels of NADPH and reduced glutathione can compromise the cell's antioxidant defense s
ystem, leading to increased lipid peroxidation and oxidative damage.
Option D. Decreased RBC membrane damage:
G6PD deficiency does not lead to a decrease in RBC membrane damage. In fact, G6PD-
deficient red blood cells (RBCs) are more susceptible to oxidative stress and hemolysis. The
deficiency impairs the ability of RBCs to regenerate reduced glutathione and counteract
oxidative damage, making them more vulnerable to hemolysis and membrane damage in the
presence of oxidative stressors.

Solution for Question 7:


Correct Option: B
The likely diagnosis based on the presence of "Orphan Annie eye nuclei" is Papillary thyroid carcinoma
.
Explanation of the options:
Option A: Follicular thyroid carcinoma: Follicular thyroid carcinoma typically does not exhibit "Orphan A
nnie eye nuclei." Instead, it is characterized by follicular cell differentiation and the presence of follicular
patterns.
Option B: Papillary thyroid carcinoma: Papillary thyroid carcinoma is associated with the presence of "
Orphan Annie eye nuclei." These nuclei are named after the comic strip character "Little Orphan Annie"
due to their characteristic appearance. They are enlarged, irregularly shaped nuclei with ground glass
or optically clear chromatin.
Option C: Anaplastic thyroid carcinoma: Anaplastic thyroid carcinoma is a highly aggressive and undiff
erentiated form of thyroid cancer. It typically does not exhibit "Orphan Annie eye nuclei." Instead, it is c
haracterized by marked cellular pleomorphism and loss of cellular differentiation.
Option D: Medullary thyroid carcinoma: Medullary thyroid carcinoma arises from the parafollicular C cel
ls of the thyroid gland and is associated with the production of calcitonin. It does not typically exhibit "O
rphan Annie eye nuclei."
Therefore, based on the presence of "Orphan Annie eye nuclei," the likely diagnosis is Papillary thyroid
carcinoma. It is the most common type of thyroid cancer and is characterized by the presence of nucle
ar features such as "Orphan Annie eye nuclei" and nuclear grooves.

Solution for Question 8:


Correct Option B - Papillary carcinoma:
• Papillary thyroid carcinoma (PTC) is an epithelial malignancy showing evidence of follicular cell
differentiation and a set of distinctive nuclear features. Papillary thyroid carcinoma is the predominant

Page 7

1085
form of thyroid cancer, accounting for 80 to 85% of all thyroid cancer cases. The diagnosis of papillary
carcinoma has as its basis the nuclear morphology of a thyroid neoplasm. The existence of multiple
architectural variants proves the irrelevance of architecture. The following constellation of features
characterizes the diagnostic nuclear morphology:
• Enlarged and elongated nuclei with crowding and overlap
• Irregular nuclear contour
• Chromatin clearing with peripheral margination of chromatin, giving rise to what has been described
as Orphan Annie Eye nuclei
• Multiple micro-nucleoli located immediately underneath the nuclear membrane
• Nuclear grooves resulting from an irregularity of nuclear contour seen in 2 dimensions
• Intranuclear cytoplasmic pseudo inclusions from the accumulation of cytoplasm in prominent nuclear
grooves
• Presence of psammoma bodies
Incorrect Choices:
Option A - Follicular carcinoma: Follicular carcinoma is a malignant thyroid neoplasm showing follicular
cell differentiation but lacking the diagnostic features of papillary carcinoma. It generally occurs in pati
ents with a
higher mean age than those with follicular adenoma. Follicular carcinoma usually manifests as a
solitary thyroid mass or, less commonly, as a metastatic tumor, particularly in bone. A
follicular carcinoma tends to be more cellular with a
thick irregular capsule, and often with areas of necrosis and more frequent mitoses.
Option C - Toxic nodular goiter: Plummer disease, also known as toxic multi-nodular goiter (MNG), is a
hormonally active multi-nodular goiter with hyperthyroidism. Toxic nodular goiter involves an enlarged t
hyroid gland that contains a small rounded mass or masses called nodules, which produce too much th
yroid hormone. Radioactive iodine is a treatment for an overactive thyroid gland. The dose of radioactiv
e iodine is taken orally. The thyroid takes up the radioactive iodine, which destroys cells in the thyroid.
The treatment lowers or eliminates hormone production and may decrease the size of the goiter.
Option D - Medullary carcinoma: Medullary carcinoma of the thyroid is cancer of the thyroid gland that
starts in cells that release a hormone called calcitonin. These cells are called "C" cells. Medullary carci
noma of the thyroid (MTC) accounts for less than 5% of all thyroid cancers. MTC is a
neuroendocrine tumor that arises from the C cells of the thyroid, which do not accumulate radioiodine b
ut do secrete calcitonin (Ctn). MTC can start as a lump in the throat. The tumor growing in the thyroid c
an make our voice hoarse by blocking our vocal cords or it can make it hard to breathe by blocking our
windpipe.

Solution for Question 9:


Correct option: C
• The clinical features described in the patient, including episodic headaches, palpitations, sweating,
pectus excavatum, high extremities-to-body ratio, tongue growth, and a flank mass, are characteristic
of multiple endocrine neoplasia type 2B (MEN 2B).
• MEN 2B is an autosomal dominant genetic syndrome caused by a mutation in the RET
proto-oncogene. It is associated with the development of multiple endocrine tumors, such as medullary
thyroid carcinoma (MTC), pheochromocytoma, and parathyroid adenomas. MEN 2B is also

Page 8

1086
characterized by distinctive physical features, including marfanoid habitus (high extremities-to-body
ratio), mucosal neuromas (growth on the tongue), and gastrointestinal ganglioneuromas.
Incorrect options
Optio A: MEN 1: MEN 1 (multiple endocrine neoplasia type 1) is characterized by the development of t
umors in multiple endocrine glands, including the parathyroid glands, pancreas, and pituitary gland. It d
oes not typically present with the specific physical features described in the patient.
Option B: MEN 2A: MEN 2A is another form of multiple endocrine neoplasia caused by RET gene muta
tions. It is characterized by the development of medullary thyroid carcinoma, pheochromocytoma, and
primary hyperparathyroidism. The physical features associated with MEN 2B, such as marfanoid habitu
s and mucosal neuromas, are not typically seen in MEN 2A.
Option D: Neurofibromatosis type 2: Neurofibromatosis type 2 is a genetic disorder characterized by th
e development of bilateral vestibular schwannomas (acoustic neuromas) and other central nervous sys
tem tumors. It does not typically present with the specific physical features described in the patient.

Solution for Question 10:


Correct option: B
• Pheochromocytomas are tumors that arise from chromaffin cells of the adrenal medulla, resulting in
excessive production of catecholamines such as epinephrine and norepinephrine. These tumors can
produce various hormones and vasoactive substances. However, they do not produce secretin.
Secretin is a hormone produced by the S cells of the duodenum, it is not related to
pheochromocytomas.
Incorrect options:
Option A: Nor-epinephrine: Pheochromocytomas produce and release norepinephrine, which is a
catecholamine and a main product of these tumors.
Option C: Vasoactive intestinal polypeptide: Although not as common as norepinephrine and epinephri
ne, pheochromocytomas can also produce and release vasoactive intestinal polypeptide (VIP). VIP is a
neuropeptide involved in various physiological processes, including vasodilation and regulation of intes
tinal motility.
Option D: Somatostatin: Pheochromocytomas can produce and release somatostatin, which is another
neuropeptide hormone involved in the regulation of various processes, including hormone secretion an
d neurotransmission.

Solution for Question 11:


Correct Option B:
The clinical features described in the scenario, including long hands and feet (acromegaly) and a
protruding lower jaw (prognathism), are characteristic of a
condition called acromegaly. Acromegaly is most commonly caused by a growth hormone-secreting pit
uitary adenoma, which leads to excessive secretion of growth hormone (GH). The gene most commonl
y implicated in this condition is the PIT1 gene.

Page 9

1087
The PIT1 gene (also known as POU1F1) encodes a transcription factor that plays a crucial role in the d
evelopment and function of the anterior pituitary gland. Mutations in the PIT1 gene can lead to dysregul
ation of pituitary hormone production, including excessive GH secretion. This, in turn, causes the chara
cteristic features of acromegaly.
Incorrect Options:
Option A. GATA1: The GATA1 gene is associated with a
different condition called Diamond-Blackfan anemia, which is characterized by a failure of red blood cel
l production. It is not directly related to the clinical features described in the scenario.
Option C. TPIT: The TPIT gene is associated with a condition called isolated adrenocorticotropic horm
one (ACTH) deficiency. It is not directly related to the clinical features described in the scenario.
Option D. SF1: The SF1 gene (also known as NR5A1) is associated with a condition called 46,XY disor
ders of sex development (DSD). It is involved in the development of the gonads and adrenal glands. S
F1 mutations can cause a variety of intersex disorders, but they are not directly related to the clinical fe
atures described in the scenario.

Solution for Question 12:


Correct Option D:
When performing a thyroid biopsy, the most appropriate specimen is obtained when there are 6 follicul
ar cell clusters containing 10-15 cells. This provides an adequate representation of the thyroid tissue fo
r examination and evaluation.
The thyroid is composed of follicles, which are the functional units of the gland. Each follicle is lined by
follicular cells and contains colloid, which is rich in thyroglobulin. Therefore, a
biopsy specimen that includes multiple follicular cell clusters allows for a
comprehensive assessment of the thyroid tissue.
Incorrect Options:
Option A. 15 follicular cell clusters contain 10-12 cells: This option describes a larger number of follicul
ar cell clusters but with fewer cells in each cluster. While it may still provide some information, it may n
ot provide a representative sample of the thyroid tissue.
Option B. 10 follicular cell clusters contain 6-8 cells: This option describes a smaller number of follicular
cell clusters with fewer cells in each cluster. This may not provide enough material for a
comprehensive evaluation of the thyroid.
Option C. 3 follicular cell clusters contain 10-15 cells: This option describes a
smaller number of follicular cell clusters with a variable number of cells in each cluster. Again, this may
not provide enough material for an accurate assessment of the thyroid tissue.

Page 10

1088
Diagnosis of Genetic Disorders & Miscellaneous
1. A 30-year-old male patient is diagnosed with chronic myelogenous leukaemia. Chromosomal
analysis for Philadelphia chromosome shows the translocation of part of chromosome 9 to
chromosome 22. Based on the position of the centromere, chromosome 22 is regarded as:
(or)
What is the position of the centromere in chromosome 22?
A. Telocentric
B. Metacentric
C. Submetacentric
D. Acrocentric
----------------------------------------
2. Which technique is shown in the image for detecting apoptotic cell fraction?

A. In-situ DNA nick end labelling


B. Allele-specific PCR (ASPCR)
C. Fluorescent in situ hybridisation
D. Periodic acid–Schiff
----------------------------------------
3. What's the mode of inheritance for polycystic kidney disease in this patient's family based on the
pedigree chart given below?

A. Autosomal Recessive

1089
B. Autosomal Dominant
C. X-Linked Recessive
D. X-Linked Dominant
----------------------------------------
4. Which of the following is a component of triple test screening for Down syndrome?
A. FSH
B. Oxytocin
C. Human Chorionic Gonadotropin (HCG)
D. Inhibin Alpha
----------------------------------------
5. Which of the following describes the inheritance of the ABO blood groups in humans?
(or)
Which of the following describes the inheritance of the ABO blood groups in humans?
A. Codominance
B. Mitochondrial inheritance
C. Allelic exclusion
D. Sex-linked Inheritance
----------------------------------------
6. Which inheritance pattern does osteogenesis imperfecta follow?
A. Autosomal dominant
B. Autosomal recessive
C. x-linked recessive
D. Co-dominance
----------------------------------------
7. For which types of disorders is karyotyping performed?
A. Chromosomal disorders
B. Autosomal recessive disorders
C. Autosomal dominant disorders
D. Linkage disorders
----------------------------------------
8. How many copies will be formed once a DNA molecule has undergone three successful PCR
cycles?
A. Double the number of copies
B. Three times the number of DNA molecule
C. Four times the number of DNA molecule
D. Eight times the number of copies

Page 2

1090
----------------------------------------
9. What is the location of gene responsible for causing Wilm's tumor?
A. Chromosome 1
B. Chromosome 10
C. Chromosome 11
D. Chromosome 12
----------------------------------------
10. What genetic abnormality is expected in a 1-year-old child presenting with a high-pitched cat-like
cry, microcephaly, hypertelorism, micrognathia, epicanthal folds, low-set ears, hypotonia, and
echocardiography findings of patent ductus arteriosus?
A.
B.
C.
D.
----------------------------------------
11. Which of the following mechanisms mechanisms contribute to the earlier and more severe onset of
Huntington's disease across successive generations?
A. A shift from trinucleotide repeats to pentanucleotide repeats
B. An increase in the number of trinucleotide repeats in successive generations
C. Defects in membrane receptors and transport systems.
D. Imprinting variability in successive generations.
----------------------------------------
12. Identity the mode of inheritance:

A. Autosomal dominant
B. Autosomal recessive
C. X – linked dominant
D. X – linked recessive
----------------------------------------

Page 3

1091
13. What is the mode of inheritance in Diabetes mellitus?
A. Polygenic pattern
B. Autosomal dominant
C. X linked recessive
D. X linked dominant
----------------------------------------
14. What is the diagnosis for a child with an X-linked pattern of inheritance who has trouble standing up
from a laying or sitting posture, learning difficulties, calf muscle hypertrophy, and a waddling gait?
A. Myotonic dystrophy
B. Duchenne muscular dystrophy
C. Familial hypercholesterolemia
D. G6PD
----------------------------------------
15. Which step in MLPA (Multiplex Ligation dependent Probe Amplification) aids in the detection of
mutations in the CFTR gene, contributing to the diagnosis of cystic fibrosis?
A. Denaturation process
B. Annealing using multiple probes
C. Ligation of two probes
D. Amplification for probe analysis
----------------------------------------
16. How many Barr bodies are observed in a patient with complaint of primary amenorrhea and
examination findings of low hairline and widely spaced nipples and a harsh systolic murmur in the left
infraclavicular area on auscultation?
(or)
How many Barr bodies are found in Turner syndrome?
A. 0
B. 1
C. 2
D. None of the above
----------------------------------------
17. Barr body can be seen in which phase of the cell cycle?
A. Interphase
B. Metaphase
C. G1 phase
D. Telophase
----------------------------------------
18. Mark the correct statement regarding microsatellites:

Page 4

1092
A. Repeat size of more than 10 to 15 nucleotides
B. More prone to variation
C. Instability of this structure is not found in colon cancer
D. RNA repeats present
----------------------------------------
19. In a child with 22q11 deletion syndrome which of the following specific manifestations can be
expected?
A. Hypercalcemia
B. Conotruncal abnormalities
C. Thymic hyperplasia
D. Dysmorphogenesis of the 1st and 2nd pharyngeal pouches
----------------------------------------
20. In a patient with a suspected miRNA processing disorder, which enzyme is primarily responsible for
cleaving the precursor microRNA into a duplex structure within the cytoplasm?
A. Drosha
B. DGCR8
C. Exportin5
D. DICER
----------------------------------------
21. Which technology serves as the basis for the "Feluda Test," a diagnostic method used for the
detection of SARS-CoV-2 in COVID-19 patients?
A. PCR (Polymerase Chain Reaction)
B. CRISPR-Cas9 (Clustered Regularly Interspaced Short Palindromic Repeats)
C. ELISA (Enzyme-Linked Immunosorbent Assay)
D. Western Blotting
----------------------------------------
22. Which of the following is the most commonly used technique in karyotyping?
A. G banding
B. Q banding
C. C banding
D. R banding
----------------------------------------

Correct Answers
Question Correct Answer

Question 1 4

Page 5

1093
Question 2 1
Question 3 2
Question 4 3
Question 5 1
Question 6 1
Question 7 1
Question 8 4
Question 9 3
Question 10 1
Question 11 2
Question 12 4
Question 13 1
Question 14 2
Question 15 3
Question 16 1
Question 17 1
Question 18 2
Question 19 2
Question 20 4
Question 21 2
Question 22 1

Solution for Question 1:


Correct Option D - Acrocentric:
• The position of the centromere varies in different chromosomes. Various types of chromosomes
based on the position of the centromere include: Metacentric chromosomes Telocentric chromosomes
Acrocentric chromosomes Submetacentric chromosomes
• Metacentric chromosomes
• Telocentric chromosomes
• Acrocentric chromosomes
• Submetacentric chromosomes
• Humans usually have five pairs of acrocentric autosomes, including chromosomes 13, 14, 15, 21 and
22, and one acrocentric sex chromosome, the Y chromosome.
• Metacentric chromosomes
• Telocentric chromosomes
• Acrocentric chromosomes

Page 6

1094
• Submetacentric chromosomes

Incorrect Options:
Option A - Telocentric:
• Telocentric chromosomes possess a centromere at one end of the chromosome and therefore display
only one arm at the cytological level.
• They are not naturally present in humans but can form through cellular chromosomal errors.
• Telocentric chromosomes exist naturally in many species, like house mice, where all chromosomes
are telocentric except the Y chromosome.
Option B - Metacentric:
• The metacentric chromosome is the one in which the centromere is positioned midway between the
two chromosomal ends.
• This results in equal lengths of arms of chromosomes.
• These include chromosomes 1, 3, 16, 19 and 20.
Option C - Submetacentric:
• A submetacentric chromosome is one in which the centromere is positioned slightly away from the
center resulting in one chromosome arm being shorter than the other.
• This type of chromosome is L-shaped.

Solution for Question 2:


Correct Option A - In-situ DNA nick end labelling:
• Apoptosis has been extensively studied in the context of cancer cell chemosensitivity and
radiosensitivity and DNA fragmentation being the marker of apoptosis. Accordingly, the TUNEL assay
has become the gold standard for detecting cell death through apoptosis.
• It is an In-situ method for detecting areas of DNA which are nicked during apoptosis.

Page 7

1095
• Terminal deoxynucleotidyl transferase-mediated dUTP–biotin nick end labelling (TUNEL) is a method
for detecting apoptotic cells that exhibit DNA fragmentation.
• Considering that genomic DNA breaks occur during the early and late stages of apoptosis, the TUNEL
staining technique remains widely used as a measure of apoptotic cell death.
• Advantages include the broad availability of TUNEL assay kits and relative ease of performance for
apoptosis analysis.
Incorrect Options:
Option B - Allele-specific PCR (ASPCR):
• The fraction of apoptotic cells is not measured by allele-specific PCR.
• The allele-specific polymerase chain reaction (ASPCR) procedure analyses and detect known genetic
polymorphisms.
• It has already been used in DNA-based diagnostic techniques for diagnosing infectious and genetic
diseases.
Option C - Fluorescent in situ hybridisation (FISH):
• The fraction of apoptotic cells is not measured by fluorescent in situ hybridisation.
• Fluorescence in situ hybridisation (FISH) is a technique for locating and detecting a specific DNA
sequence on a chromosome.
• FISH involves using fluorescent DNA probes to target specific chromosomal locations in the nucleus,
generating coloured signals that can be detected using a fluorescent microscope.
Option D - Periodic acid–Schiff stain:
• The fraction of apoptotic cells is not measured by Periodic acid–Schiff
• Periodic acid–Schiff (PAS) is a staining method to detect polysaccharides such as glycogen,
glycoproteins, mucins and glycolipids in tissues.

Solution for Question 3:


Correct Option B - Autosomal Dominant:
• Polycystic kidney disease is an inherited disease that involves bilateral renal cysts.
• Adult polycystic kidney disease has an autosomal dominant type of polycystic kidney disease.
• Autosomal recessive disease occurs in infancy.
• Individuals with ADPKD have mutated genes on an autosomal chromosome.
• Affected parents have a 50% chance of transferring the mutated gene to their child.
• In autosomal dominant inheritance, males and females are affected.
Incorrect Options:
Option A - Autosomal Recessive:
• Although polycystic kidney disease may be inherited as an autosomal recessive disorder, that occurs
in infancy and not in adults.

Page 8

1096
• In an autosomal recessive type inheritance pattern, both copies of the gene must be mutated for an
individual to express the disease.
• Two carriers have a 25% risk of transferring disease to their offspring.
Option C - X-Linked Recessive:
• X-linked recessive conditions result from mutations in genes on the X chromosome.
• Males are much more frequently affected as compared to females.
Option D - X-Linked Dominant:
• X-linked dominant conditions are rare genetic disorders caused by dominant alleles on the X
chromosome. Affected females transmit the condition to both sons and daughters, while affected males
pass it to all daughters but no sons, unless the mother is also affected.
• Example: Vitamin D-resistant rickets, Alport syndrome.

Solution for Question 4:


Correct Option C - Human Chorionic Gonadotropin (HCG):
• The triple test screening for Down syndrome includes HCG, alpha-fetoprotein (AFP), and
unconjugated estriol, so human chorionic gonadotropin is the correct option.
• The triple test does not provide a definitive diagnosis but helps identify women who may benefit from
additional testing.
• It can detect up to 69% of Down syndrome pregnancies
• A high level of HCG and a low level of AFP and estriol suggest a positive triple test for Down
syndrome.

Aneuploidies
AFP
Estriol

Page 9

1097
hCG
Inhibin A
Down's syndrome
Low
High
Turner's syndrome
Decreased
Very high
Edward's syndrome
Unchanged
Very low
Patau's syndrome
Increased
Normal
Incorrect Options:
Options A, B and D are incorrect.

Solution for Question 5:


Correct Option A - Codominance:
Non-Mendelian Inheritance:
• In non-Mendelian inheritance, the resulting phenotype of the offspring is different from the phenotypes
of either parent. Codominance and incomplete dominance are examples.
• Co-dominance: Co-dominance is a genetic phenomenon where two different alleles of a gene are
both fully expressed in the phenotype of heterozygous individuals. Example: ABO blood group,
histocompatibility antigens etc.
Incorrect Options:
Option B - Mitochondrial inheritance:
• Example: MELAS syndrome
Option C - Allelic exclusion:
• Example: Immunoglobulin expression in B lymphocytes
Option D - Sex-linked Inheritance:
• Examples: Hemophilia, red-green color blindness, Duchenne muscular dystrophy, Fragile X syndrome
etc.,

Page 10

1098
Solution for Question 6:
Correct Option A - Autosomal dominant:
• Autosomal dominant disorders are expressed when an individual inherits one copy of the mutated
gene from an affected parent. This means that at least one parent of an affected individual is usually
affected themselves.
• Both males and females can be affected, and they have an equal chance of transmitting the condition
to their offspring.
• When an affected individual marries an unaffected person, each child has a 50% chance of inheriting
the mutated gene and developing the disorder.
• Example: Achondroplasia Huntington disease Neurofibromatosis type 1 and 2 Osteogenesis
imperfecta Autosomal dominant polycystic kidney disease Myotonic muscular dystrophy etc
• Achondroplasia
• Huntington disease
• Neurofibromatosis type 1 and 2
• Osteogenesis imperfecta
• Autosomal dominant polycystic kidney disease
• Myotonic muscular dystrophy etc
• Achondroplasia
• Huntington disease
• Neurofibromatosis type 1 and 2
• Osteogenesis imperfecta
• Autosomal dominant polycystic kidney disease
• Myotonic muscular dystrophy etc
Incorrect Options:
Option B - Autosomal recessive:
• Examples: Cystic fibrosis Wilson disease Sickle cell disease etc
• Cystic fibrosis
• Wilson disease
• Sickle cell disease etc
• Cystic fibrosis
• Wilson disease
• Sickle cell disease etc
Option C - X-linked recessive:
• Example: Hemophilia Duchenne Muscular Dystrophy Lesch Nyhan Syndrome
• Hemophilia
• Duchenne Muscular Dystrophy
• Lesch Nyhan Syndrome

Page 11

1099
• Hemophilia
• Duchenne Muscular Dystrophy
• Lesch Nyhan Syndrome
Option D - Co-dominance:
• In codominant inheritance, two different versions (alleles) of a gene are expressed, and each version
makes a slightly different protein.
• Both alleles influence the genetic trait or determine the characteristics of the genetic condition.
• Example: ABO blood grouping. Sickle cell disease
• ABO blood grouping.
• Sickle cell disease
• ABO blood grouping.
• Sickle cell disease

Solution for Question 7:


Correct Option A - Chromosomal disorders:
• Karyotyping is used to assess the chromosomal disorders.
Karyotyping, the primary tool in cytogenetics, involves arresting dividing cells in metaphase and stainin
g chromosomes. Chromosomes appear as two chromatids connected at the centromere, arranged by l
ength to create a
karyotype, including sex chromosomes. Staining methods like Giemsa stain, particularly G
banding, enable the identification of chromosomes based on distinctive banding patterns.
Incorrect Options:
Option B - Autosomal recessive disorders:
• The pedigree analysis helps to determine the autosomal recessive disorders
• Examples of autosomal recessive disorders include cystic fibrosis, sickle cell anaemia, and
Tay-Sachs disease
Option C - Autosomal dominant disorders:
• A careful analysis of the disease history via the pedigree helps identify whether the disorder is
dominant or recessive, autosomal or X-linked.
Option D - Linkage disorders:
• A test cross is another ideal method to know whether the genes are linked.
• Any deviation from the ratio of progenies, as expected by the law of independent assortment, is to be
verified for linkage.
• A test cross is one with one of the parents being homozygous recessive.

Page 12

1100
Solution for Question 8:
Correct Option D - Eight times the number of copies:
• The total copies of the particle after the 'n' cycles of the Polymerase Chain Reaction are given by the
formula 2 to the power n times of the original copies. So, the number of copies after 3 cycles would be
23 =8 times the original copies.
• While PCR is used to amplify DNA, RT – PCR (reverse transcriptase PCR) can be used to investigate
mRNA.
Incorrect Options:
Option A - Double the number of copies:
• Using the formula 2 to the power of n, the answer is 8.
Option B - Three times the number of DNA molecule:
• Using the formula 2 to the power n, the answer is 8.
Option C - Four times the number of DNA molecule:
Using the formula 2 to the power n, the answer is 8.

Solution for Question 9:


Correct Option C - Chromosome 11:
• Gene for Wilm’s tumour is located on Chromosome 11.
• Two genes associated with Wilms tumour WT1 gene (located on Ch 11p13) WT2 gene (located on Ch
11p15)
• WT1 gene (located on Ch 11p13)
• WT2 gene (located on Ch 11p15)
• It is the most common primary renal malignancy of childhood and is usually diagnosed before age 10.
It is usually noticed as a mass or swelling in the abdomen.
• The child usually presents with fever, pain abdomen, hematuria, or high blood pressure.
• Approximately 90 per cent of Wilm's tumour cases occur due to somatic mutations
• WT1 gene (located on Ch 11p13)
• WT2 gene (located on Ch 11p15)
Incorrect Options:
Option A - Chromosome 1:
• Diseases caused by chromosome 1 defect: Charcot-Marie-Tooth disease (CMT) [PMP22 gene in
CMT type 1A] Spinal muscular atrophy (rare form known as SMA type 4 can be caused by mutations in
the gene DYNC1H1 located on chromosome 1)
• Charcot-Marie-Tooth disease (CMT) [PMP22 gene in CMT type 1A]
• Spinal muscular atrophy (rare form known as SMA type 4 can be caused by mutations in the gene
DYNC1H1 located on chromosome 1)

Page 13

1101
• Charcot-Marie-Tooth disease (CMT) [PMP22 gene in CMT type 1A]
• Spinal muscular atrophy (rare form known as SMA type 4 can be caused by mutations in the gene
DYNC1H1 located on chromosome 1)
Option B - Chromosome 10:
• Diseases caused by chromosome 10 defect: Retinitis pigmentosa (RP) Cowden syndrome
• Retinitis pigmentosa (RP)
• Cowden syndrome
• Retinitis pigmentosa (RP)
• Cowden syndrome
Option D - Chromosome 12:
• Diseases caused by chromosome 12 defect: Achondrogenesis type 2 Collagenopathy, types II and XI
Episodic ataxia Hereditary hemorrhagic telangiectasia
• Achondrogenesis type 2
• Collagenopathy, types II and XI
• Episodic ataxia
• Hereditary hemorrhagic telangiectasia
• Achondrogenesis type 2
• Collagenopathy, types II and XI
• Episodic ataxia
• Hereditary hemorrhagic telangiectasia

Solution for Question 10:


Correct Option A - 5p-:
• Based on the presenting complaint of high-pitched cat-like cry associated with mental retardation and
slow development and clinical findings of microcephaly, hypertelorism, micrognathia, epicanthal folds,
low-set ears, and hypotonia, the most likely diagnosis is cri du chat syndrome.

• The karyotypic analysis of this syndrome would show the deletion of the 5p chromosome (5p-).
• Cri-du-chat syndrome is an autosomal deletion syndrome that occurs due to a partial deletion/removal
of the p chromosome (5p-).

Page 14

1102
Incorrect Options:
Option B - Downs syndrome: Trisomy 21:

3 copies of chromosome 21 is seen in the above karyotyping.


Option C - Edwards syndrome: Trisomy 18:

Page 15

1103
3 copies of chromosome 18 is seen in the above karyotyping.
Option D - Klinefelter syndrome: 47 XXY

2 X chromosomes and 1 Y chromosome is seen in this karyotyping

Solution for Question 11:


Correct Option B - An increase in the number of trinucleotide repeats in successive generations:
• Based on the history of developmental delay and family history of autosomal disease associated with
a mutation in chromosome 4 and athetoid movements and dementia with hypertonicity, the most likely
diagnosis of the father and his child is Huntington's disease.
• An increase in trinucleotide repeats in successive generations is responsible for this disorder's earlier
and debilitating appearance.

Page 16

1104
• Huntington's disease is generally an adult-onset, autosomal dominant inherited disorder associated
with cell loss in the basal ganglia and cortex.
• Clinical Features: The clinical features of Huntington's disease include a movement disorder, a
cognitive disorder, and a behavioural disorder. Chorea is the commonest movement disorder. Dystonia
and parkinsonian features include bradykinesia, rigidity, and postural instability. In advanced disease,
akinetic-rigid syndrome. Other late features are spasticity, clonus, and extensor plantar responses.
Dysarthria and dysphagia are also common
• The clinical features of Huntington's disease include a movement disorder, a cognitive disorder, and a
behavioural disorder.
• Chorea is the commonest movement disorder.
• Dystonia and parkinsonian features include bradykinesia, rigidity, and postural instability.
• In advanced disease, akinetic-rigid syndrome.
• Other late features are spasticity, clonus, and extensor plantar responses.
• Dysarthria and dysphagia are also common
• Workup: Genetic testing reported the CAG repeat number for each allele.
• Genetic testing reported the CAG repeat number for each allele.
• The clinical features of Huntington's disease include a movement disorder, a cognitive disorder, and a
behavioural disorder.
• Chorea is the commonest movement disorder.
• Dystonia and parkinsonian features include bradykinesia, rigidity, and postural instability.
• In advanced disease, akinetic-rigid syndrome.
• Other late features are spasticity, clonus, and extensor plantar responses.
• Dysarthria and dysphagia are also common
• Genetic testing reported the CAG repeat number for each allele.
Incorrect Options:
Options A, C and D are incorrect.

Solution for Question 12:


Correct Option D - X-linked recessive:
• In any situation when the previous generation of the affected individual is not affected, the condition is
recessive.
• In X-linked recessive disease, the female won't have the disease, but she will transmit it to her son.
• If there is selective male-to-male transmission, it is most likely an X-linked condition.
• Men are affected in X-linked recessive disease.
• Example: colour blindness
Incorrect Options:
Option A - Autosomal dominant:

Page 17

1105
• Transmitted even if one parent is affected.
• One copy of the mutated gene from one parent can cause the genetic condition

Option B - Autosomal recessive:


• In autosomal recessive inheritance, a genetic disorder occurs when the child inherits a single mutated
copy of the gene from each parent.
• The parents usually do not have the condition.

Option C - X-linked dominant:


• The X-linked dominant pattern of inheritance is rare.
• Both male and female children of an affected female can have the trait

Page 18

1106
Solution for Question 13:
Correct Option A - Polygenic pattern:
• A polygenic pattern occurs when two or more genes control one characteristic. An example is
Diabetes mellitus.
• Type 1 and type 2 diabetes, the most prevalent types, are polygenic; they are caused by an alteration,
or fault, in numerous genes.
• Polygenic forms of diabetes can also develop due to environmental factors, such as obesity, in the
case of type 2 diabetes.
• Some examples of polygenic inheritance are: Human skin colour Eye colour Height Weight
Intelligence Atherosclerosis Rheumatoid arthritis, systemic lupus erythematosus, and multiple sclerosis
• Human skin colour
• Eye colour
• Height
• Weight
• Intelligence
• Atherosclerosis
• Rheumatoid arthritis, systemic lupus erythematosus, and multiple sclerosis
• Human skin colour
• Eye colour
• Height
• Weight
• Intelligence

Page 19

1107
• Atherosclerosis
• Rheumatoid arthritis, systemic lupus erythematosus, and multiple sclerosis
Incorrect Options:
Option B - Autosomal dominant:
• An example is Familial hypercholesterolemia.
Option C - X-linked recessive:
• An example is G6pD.
Option D - X linked dominant:
• An example is Vit D resistant rickets

Solution for Question 14:


Correct Option B - Duchenne muscular dystrophy:
• Based on the symptoms of difficulty rising from a lying or sitting position, learning disabilities, calf
muscle hypertrophy and waddling gait on examination, and X-linked pattern, the child has Duchenne
muscular dystrophy.
• It has an X-linked recessive pattern of inheritance characterized by progressive muscle loss
• It is a multi-systemic condition affecting many parts of the body, which results in the deterioration of
the skeletal, heart, and lung muscles.
• Clinical features: Frequent falls Waddling gait Walking on the toes Learning disabilities Difficulty rising
from a lying or sitting position Hypertrophy of calf muscles Muscle pain and stiffness Trouble running
and jumping
• Frequent falls
• Waddling gait
• Walking on the toes
• Learning disabilities
• Difficulty rising from a lying or sitting position
• Hypertrophy of calf muscles
• Muscle pain and stiffness
• Trouble running and jumping
• Frequent falls
• Waddling gait
• Walking on the toes
• Learning disabilities
• Difficulty rising from a lying or sitting position
• Hypertrophy of calf muscles
• Muscle pain and stiffness

Page 20

1108
• Trouble running and jumping
Incorrect Options:
Option A - Myotonic dystrophy:
• It is an autosomal dominant disease.
Option C - Familial hypercholesterolemia:
• Familial hypercholesterolemia is an autosomal dominant disease.
• One copy of a mutated gene from one parent can cause the genetic condition.
Option D - G6PD:
• G6PD follows an X-linked recessive inheritance, a genetic condition that occurs when the child
inherits one mutated copy of a gene from each parent.
• But then G6PD presents with anemia, episodic hemolysis, jaundice, hemoglobinuria...

Solution for Question 15:


Correct Option C - Ligation of two probes:
• The ligation of 2 probes is the crucial step in MLPA as suggested by the name Multiplex
Ligation-dependent Probe Amplification.
• Steps involved in MLPA (Multiplex Ligation-dependent Probe Amplification) Step Description
Denaturation Separation of double-stranded DNA into single strands to prepare for further processing.
Annealing Use of multiple probes to bind to specific DNA sequences within the sample. Ligation Joining
of two adjacent probes if properly annealed, indicating specific DNA sequences. Amplification
Multiplication of ligated probes for further analysis, enabling detection of mutations.
Steps involved in MLPA (Multiplex Ligation-dependent Probe Amplification)
Step
Description
Denaturation
Separation of double-stranded DNA into single strands to prepare for further processing.
Annealing
Use of multiple probes to bind to specific DNA sequences within the sample.
Ligation
Joining of two adjacent probes if properly annealed, indicating specific DNA sequences.
Amplification
Multiplication of ligated probes for further analysis, enabling detection of mutations.
Here's how this step is significant in mutation detection within the CFTR gene:
Incorrect Options:
Option A - Denaturation process: This step involves the separation of the double-stranded DNA into sin
gle strands, which is an initial preparatory step. Denaturation helps to make the DNA accessible for furt
her processing, but it doesn’t directly contribute to mutation detection.

Page 21

1109
Option B - Annealing using multiple probes: In this step, multiple probes specifically designed to target
particular sequences within the CFTR gene attach or hybridize to their complementary sequences in th
e patient's DNA sample. While this step is crucial for targeting specific gene regions associated with cy
stic fibrosis, it alone doesn't identify mutations; rather, it prepares the DNA for further steps.
Option D - Amplification for probe analysis: After the ligation step, the resulting products, if formed, are
amplified. This amplification creates multiple copies of the ligated probes, which are then analyzed for t
he presence or absence of specific sequences. Although essential for obtaining a quantifiable amount
of the DNA sequence of interest, the amplification step itself doesn't directly detect mutations in the CF
TR gene.

Solution for Question 16:


Correct Option A – 0:
• The case mentioned above has clinical features of primary amenorrhea and examination findings
suggesting Turner Syndrome. Its karyotype is 45X0, and the number of Barr bodies is 0 in this case.
• Barr body, or X chromatin, is an inactive X that appears as a darkly stained tiny mass in contact with
the nuclear membrane known as in the interphase nucleus: Number of Barr bodies = number of X
chromosome -1, Turner syndrome patients (45X0), number of Barr body = 1-1=0. Turner syndrome is
the most common cause of primary amenorrhea.
• Number of Barr bodies = number of X chromosome -1,
• Turner syndrome patients (45X0), number of Barr body = 1-1=0. Turner syndrome is the most
common cause of primary amenorrhea.
• Turner's syndrome is a chromosomal disorder affecting around 1 in 2,500 females.
• It is caused by either the presence of only one sex chromosome (X) or a deletion of the short arm of
one of the X chromosomes.
• Number of Barr bodies = number of X chromosome -1,
• Turner syndrome patients (45X0), number of Barr body = 1-1=0. Turner syndrome is the most
common cause of primary amenorrhea.

Page 22

1110
Incorrect Options:
Turner Syndrome has no barr bodies hence options B, C and, D are incorrect.

Solution for Question 17:


Correct Option A - Interphase:
• Barr bodies are condensed, inactive X chromosomes present in the nuclei of female mammalian cells.
They are typically seen during interphase, specifically in the G0 and G1 phases when the cell is not
actively dividing.
Incorrect Options:
Option B - Metaphase:
• In the mitotic stage, known as metaphase, chromosomes are at their second-most compacted and
coil-like state.
• These chromosomes, carrying genetic information, align in the cell's equator before being separated
into each of the two daughter cells.
Option C - G1 phase:
• The G■ phase, gap 1 phase, or growth 1 phase is the first of four cell cycle phases
• The cell synthesizes mRNA and proteins in preparation for subsequent steps leading to mitosis.
Option D - Telophase:
• Telophase is the fifth and final step of mitosis. It is a process that separates the duplicated genetic
material.
• Once the duplicated, paired chromosomes have been split apart and brought to the cell's poles, or
opposite sides, telophase begins.

Page 23

1111
Solution for Question 18:
Correct Option B - More prone to variation:
• Microsatellites are 1-6 bp repeats dispersed throughout DNA:-size <1kb.
• Microsatellite instability (MSI) is a hypermutable phenotype caused by DNA mismatch repair activity
loss.
• In patients with HNPCC, cells from resected tumors show microsatellite instability, whereas normal
cells from the individual do not show microsatellite instability. (MSI is a hallmark of Lynch syndrome)
Incorrect Options:
Option A - Repeat size of more than 10 to 15 nucleotides:
• Microsatellites are 1-6 bp repeats dispersed throughout DNA but usually in the non-coding
region:-size <1kb.
• Therefore this option is incorrect
Option C - Instability of this structure is not found in colon cancer:
• Microsatellite instability is found in colon cancer.
• The APC/-catenin pathway is activated in the classic adenoma-carcinoma sequence; the
microsatellite instability pathway is linked to DNA mismatch repair failures and mutation accumulation
in microsatellite repeat areas of the genome.
Option D - RNA repeats present:
• In microsatellites, DNA repeats are seen.
• microsatellites instability is a hypermutable phenotype caused by DNA mismatch repair activity loss.

Solution for Question 19:


Correct Option B - Conotruncal abnormalities:
• DiGeorge syndrome is characterized by 22q11.2 microdeletion.
• Conotruncal abnormalities are a characteristic feature of the syndrome.
• The acronym CATCH22 is often used as a mnemonic to remember key features associated with
DiGeorge syndrome. C - Cardiac Abnormalities: (conotruncal abnormalities) Commonly includes
conotruncal anomalies such as tetralogy of Fallot, interrupted aortic arch, and ventricular septal
defects. A - Abnormal Facies: Facial dysmorphogenesis, including low-set ears and hooded eyes,
along with abnormalities of the palate. T - Thymic Hypoplasia or Aplasia: Results in immune system
dysfunction due to underdevelopment or absence of the thymus, leading to T-cell deficiency. C - Cleft
Palate: Palatal abnormalities, including cleft palate or velopharyngeal insufficiency. H - Hypocalcemia:
Due to hypoparathyroidism, which can lead to seizures and other symptoms related to low calcium
levels. 22 - Chromosome 22q11.2 Deletion: The underlying genetic cause of DiGeorge syndrome is a
deletion of a segment of chromosome 22 at position q11.2.
• C - Cardiac Abnormalities: (conotruncal abnormalities) Commonly includes conotruncal anomalies
such as tetralogy of Fallot, interrupted aortic arch, and ventricular septal defects.
• Commonly includes conotruncal anomalies such as tetralogy of Fallot, interrupted aortic arch, and
ventricular septal defects.

Page 24

1112
• A - Abnormal Facies: Facial dysmorphogenesis, including low-set ears and hooded eyes, along with
abnormalities of the palate.
• Facial dysmorphogenesis, including low-set ears and hooded eyes, along with abnormalities of the
palate.
• T - Thymic Hypoplasia or Aplasia: Results in immune system dysfunction due to underdevelopment or
absence of the thymus, leading to T-cell deficiency.
• Results in immune system dysfunction due to underdevelopment or absence of the thymus, leading to
T-cell deficiency.
• C - Cleft Palate: Palatal abnormalities, including cleft palate or velopharyngeal insufficiency.
• Palatal abnormalities, including cleft palate or velopharyngeal insufficiency.
• H - Hypocalcemia: Due to hypoparathyroidism, which can lead to seizures and other symptoms
related to low calcium levels.
• Due to hypoparathyroidism, which can lead to seizures and other symptoms related to low calcium
levels.
• 22 - Chromosome 22q11.2 Deletion: The underlying genetic cause of DiGeorge syndrome is a
deletion of a segment of chromosome 22 at position q11.2.
• The underlying genetic cause of DiGeorge syndrome is a deletion of a segment of chromosome 22 at
position q11.2.
• High risk of development of ADHD and Schizophrenia.
• C - Cardiac Abnormalities: (conotruncal abnormalities) Commonly includes conotruncal anomalies
such as tetralogy of Fallot, interrupted aortic arch, and ventricular septal defects.
• Commonly includes conotruncal anomalies such as tetralogy of Fallot, interrupted aortic arch, and
ventricular septal defects.
• A - Abnormal Facies: Facial dysmorphogenesis, including low-set ears and hooded eyes, along with
abnormalities of the palate.
• Facial dysmorphogenesis, including low-set ears and hooded eyes, along with abnormalities of the
palate.
• T - Thymic Hypoplasia or Aplasia: Results in immune system dysfunction due to underdevelopment or
absence of the thymus, leading to T-cell deficiency.
• Results in immune system dysfunction due to underdevelopment or absence of the thymus, leading to
T-cell deficiency.
• C - Cleft Palate: Palatal abnormalities, including cleft palate or velopharyngeal insufficiency.
• Palatal abnormalities, including cleft palate or velopharyngeal insufficiency.
• H - Hypocalcemia: Due to hypoparathyroidism, which can lead to seizures and other symptoms
related to low calcium levels.
• Due to hypoparathyroidism, which can lead to seizures and other symptoms related to low calcium
levels.
• 22 - Chromosome 22q11.2 Deletion: The underlying genetic cause of DiGeorge syndrome is a
deletion of a segment of chromosome 22 at position q11.2.
• The underlying genetic cause of DiGeorge syndrome is a deletion of a segment of chromosome 22 at
position q11.2.

Page 25

1113
• Commonly includes conotruncal anomalies such as tetralogy of Fallot, interrupted aortic arch, and
ventricular septal defects.
• Facial dysmorphogenesis, including low-set ears and hooded eyes, along with abnormalities of the
palate.
• Results in immune system dysfunction due to underdevelopment or absence of the thymus, leading to
T-cell deficiency.
• Palatal abnormalities, including cleft palate or velopharyngeal insufficiency.
• Due to hypoparathyroidism, which can lead to seizures and other symptoms related to low calcium
levels.
• The underlying genetic cause of DiGeorge syndrome is a deletion of a segment of chromosome 22 at
position q11.2.
Incorrect Options:
Option A - Hypercalcemia:
• The patient suffering from DiGeorge syndrome will have hypocalcemia.
• Hypercalcemia is not a feature of DiGeorge syndrome.
Option C - Thymic hyperplasia:
• There is thymic hypoplasia in DiGeorge syndrome resulting in T-cell deficits instead of hyperplasia.
• Therefore this option is incorrect
Option D - Dysmorphogenesis of the 1st and 2nd pharyngeal pouches:
• In DiGeorge syndrome, there is facial dysmorphogenesis, including low-set ears and hooded eyes,
along with abnormalities of the palate.
• Therefore this option is incorrect

Solution for Question 20:


Correct Option D - DICER:
• DICER is an enzyme that plays a pivotal role in microRNA (miRNA) processing. In the cytoplasm, the
single-looped precursor miRNA is cleaved by DICER, resulting in the formation of a duplex structure,
which consists of two single-stranded miRNA molecules. This process is essential for the generation of
functional miRNAs that can regulate gene expression through the RNA-induced silencing complex
(RISC).
• In patients with a suspected miRNA processing disorder, a malfunction or deficiency in DICER
function could disrupt the normal processing of miRNA, potentially leading to dysregulation of gene
expression and associated clinical implications: Relation to Leukemia (CLL) 13q deletion in Chronic
Lymphocytic Leukemia (CLL) results in the loss of good miRNA 15 and miRNA 16, contributing to
cancer development. Impact on Hematological Malignancies ONCOmirs, or oncogenic miRNAs, play a
role in the development of hematological malignancies.

Relation to Leukemia (CLL)

Page 26

1114
13q deletion in Chronic Lymphocytic Leukemia (CLL) results in the loss of good miRNA 15 and miRNA
16, contributing to cancer development.
Impact on Hematological Malignancies
ONCOmirs, or oncogenic miRNAs, play a role in the development of hematological malignancies.
Incorrect Options:
Option A - Drosha: Drosha is an enzyme involved in the initial step of microRNA biogenesis within the
nucleus. Along with the RNA-binding protein DGCR8, Drosha cleaves the primary miRNA transcript to
produce the precursor miRNA, which has a
hairpin structure. Drosha acts in the nucleus, while DICER functions in the cytoplasm.
Option B - DGCR8: DGCR8, also known as DiGeorge syndrome critical region gene 8, is a
protein that forms a complex with Drosha in the nucleus. Together, Drosha and DGCR8 are responsibl
e for the initial cleavage of the primary miRNA transcript, generating the precursor miRNA. However, D
GCR8's role is primarily in the nucleus during the early stages of miRNA biogenesis.
Option C - Exportin5: Exportin5 is a protein involved in transporting the precursor miRNA from the nucl
eus to the cytoplasm. It recognizes and binds to the precursor miRNA in the nucleus and assists in its t
ransport across the nuclear membrane into the cytoplasm. However, Exportin5 is not directly involved i
n the enzymatic cleavage of the precursor miRNA into a duplex structure

Solution for Question 21:


Correct Option B - CRISPR-Cas9 (Clustered Regularly Interspaced Short Palindromic Repeats):
• CRISPR-Cas9 is a powerful tool in molecular biology that allows precise editing of genes.
• The Feluda Test uses this technology to detect genetic material specific to SARS-CoV-2. The test
involves the use of CRISPR-Cas9 to identify the genetic sequences of the virus present in a patient
sample, providing a rapid and accurate means of detecting the virus. This approach has been
considered for its potential to offer an efficient and sensitive diagnostic tool for COVID-19.
• Nobel Prize Winners Jennifer Doudna and Emmanuelle Charpentier received the Nobel Prize for their
CRISPR-Cas9 study. CRISPR Function A bacterial defense mechanism against viruses. Mechanism of
Action Takes snapshots of invading viruses, forming a memory for future defense. CRISPR Formation
Developed after the bacterium collects virus "screenshots." Enzyme Action Cas9 acts as a molecular
scissor, cutting and removing viral segments from the cell. Application in Humans Genome editing for
virus protection, used in treating diseases like HIV and muscular dystrophy.
Nobel Prize Winners
Jennifer Doudna and Emmanuelle Charpentier received the Nobel Prize for their CRISPR-Cas9 study.
CRISPR Function
A bacterial defense mechanism against viruses.
Mechanism of Action
Takes snapshots of invading viruses, forming a memory for future defense.
CRISPR Formation
Developed after the bacterium collects virus "screenshots."
Enzyme Action

Page 27

1115
Cas9 acts as a molecular scissor, cutting and removing viral segments from the cell.
Application in Humans
Genome editing for virus protection, used in treating diseases like HIV and muscular dystrophy.
Incorrect Options:
Option A - PCR (Polymerase Chain Reaction): PCR is a
widely used molecular technique for amplifying DNA. It is a sensitive method used for detecting the pre
sence of DNA, including that of viruses. RT-PCR is used for the detection of COVID-19 using nasophar
yngeal swabs.
Option C - ELISA (Enzyme-Linked Immunosorbent Assay): ELISA is an assay technique that is used to
detect the presence of antigens or antibodies in a sample. It is commonly used in serological tests to d
etect antibodies produced by the immune system in response to a
pathogen, but it's not the primary method used for detecting viral RNA, such as that of SARS-CoV-2.
Option D - Western Blotting: This method is used to detect proteins, not viruses directly, making it less
suitable for diagnosing viral infections like COVID-19.

Solution for Question 22:


Correct Option A - G banding:
• Giemsa-banding, which includes staining with Giemsa, is the most common technique used for
karyotyping.
• Various Staining methods have been developed to identify individual chromosomes based on the
distinctive and reliable pattern of alternating light and dark bands.
• The methods of dye-based chromosome banding are G- (Giemsa), R- (reverse), C- (centromere), and
Q- (quinacrine) banding.
• Bands that show strong staining are referred to as positive bands; weakly staining bands are negative
bands
• The most important application of banding is in the identification of individual chromosomes.
Incorrect Options:
Option B - Q banding:
• The fluorescent pattern was obtained using quinacrine for staining.
Option C - C banding:
• C-banding, or centromere banding, is a cytogenetic staining technique used to visualize the
constitutive heterochromatin regions near the centromeres of chromosomes.
• It involves treating chromosomes with a special dye, often Giemsa stain, which preferentially stains
the heterochromatic regions.
Option D - R banding:
• R-banding, or reverse banding, is a cytogenetic staining technique that involves a reversal of the
traditional G-banding process.

Page 28

1116
Diagnosis of Tumors and Paraneoplastic Syndromes
1. Which of the following immunohistochemical markers are incorrectly matched to their origin? Desmin
– Carcinoma Vimentin – Sarcoma Leukocyte common antigen – Lymphoma S – 100 – Melanoma
A. 1 Only
B. 1 & 4
C. 3 Only
D. 1 & 3
----------------------------------------
2. The biopsy sample of Paget’s disease of the breast is tested for Immunohistochemical markers, as
shown in the image below. The principle of the stain used is that it forms a brown precipitate at the site
of the target antigen. Which of the following is the stain being used?

A. Oil Red O
B. Papanicolaou Stain
C. 3,3'-Diaminobenzidine
D. Periodic Acid-Schiff
----------------------------------------
3. A 57-year-old male is diagnosed with a lung cancer. Serum calcium is 13.0 mg/dL (normal 8.5 to
10.2 mg/dL). Which of the following substances is likely increased, leading to this metabolic
abnormality?
A. Adrenocorticotropic hormone-like substance
B. Erythropoietin
C. Vassopressin
D. Parathyroid hormone-related protein
----------------------------------------
4. A biopsy of a lung mass is done, and the histopathological finding is shown below. On
immunohistochemical studies of this biopsy specimen, which of the following tumor markers would
likely show strong expression?
(or)

1117
Which of the following tumour markers would show increased expression in squamous cell carcinoma
of the lung?

A. Alpha-fetoprotein
B. Calretinin
C. Carcinoembryonic antigen
D. Cytokeratin
----------------------------------------
5. A 2cm submucosal nodule is detected in the appendix of a patient. Biopsy shows island cells with
scant, pink granular cytoplasm and a round to oval nucleus with a “salt and pepper” chromatin pattern
(shown in image). On electron microscopy, numerous neurosecretory granules are seen in the
cytoplasm. Urinary 5-hydroxyindoleacetic acid (5-HIAA) and serum chromogranin A are elevated.
Which of the following can be seen in patients with this tumour?
(or)
Which of the following can be seen in Carcinoid tumour?

A. Hemoptysis
B. Flushing and wheezing
C. Fatigue
D. Anorexia
----------------------------------------
6. Which of the following is the most common location for carcinoid tumors?
A. Heart

Page 2

1118
B. Small intestine
C. Liver
D. Kidney
----------------------------------------
7. Which of the following tumor markers is associated with medullary thyroid cancer?
A. HCG
B. Calcitonin
C. CA 19-9
D. Neuron-specific enolase (NSE)
----------------------------------------
8. Which of the following is a specific marker for small cell carcinoma of the lung?
A. Chromogranin
B. Cytokeratin
C. Desmin
D. Vimentin
----------------------------------------
9. Which of the following can be potentially used to determine treatment response in patients with
colorectal cancer?
A. Alpha-fetoprotein
B. Carcinoembryonic antigen
C. Chorionic gonadotropin
D. Chromogranin
----------------------------------------
10. Which of the following options correctly matches the tumour marker paired with the associated
disease?
A. Chromagranin–neuroendocrine tumors
B. Vimentin–Melanoma
C. Leucocyte specific antigen– Sarcoma
D. S100– Lymphoma
----------------------------------------
11. Which of the following is the most common fixative used in electron microscopy?
A. Glutaraldehyde
B. Formalin
C. Picric acid
D. Absolute alcohol
----------------------------------------

Page 3

1119
Correct Answers
Question Correct Answer

Question 1 1
Question 2 3
Question 3 4
Question 4 4
Question 5 2
Question 6 2
Question 7 2
Question 8 1
Question 9 2
Question 10 1
Question 11 1

Solution for Question 1:


Correct Option A - 1 Only:

• Desmin – Carcinoma: This is incorrect because Desmin is typically associated with muscle
differentiation and is commonly used to identify tumors of muscle origin such as rhabdomyosarcoma.
Therefore, Desmin is not typically expressed in carcinomas. Vimentin – Sarcoma: This is correct.
Vimentin is a mesenchymal marker commonly expressed in sarcomas. Leukocyte commonantigen –
Lymphoma: This is correct. Leukocyte common antigen (CD45) is commonly expressed in lymphomas.
S-100 – Melanoma: This is correct. S-100 is frequently expressed in melanoma, as well as in other
tumors originating from neural crest cells.

Desmin – Carcinoma: This is incorrect because Desmin is typically associated with muscle differentiati
on and is commonly used to identify tumors of muscle origin such as rhabdomyosarcoma. Therefore, D
esmin is not typically expressed in carcinomas.
Vimentin – Sarcoma: This is correct. Vimentin is a
mesenchymal marker commonly expressed in sarcomas.
Leukocyte commonantigen –
Lymphoma: This is correct. Leukocyte common antigen (CD45) is commonly expressed in lymphomas.
S-100 – Melanoma: This is correct. S-100 is frequently expressed in melanoma, as well as in other tum
ors originating from neural crest cells.
INCORRECT OPTIONS:
Options B, C, and D are incorrect, and the explanation is provided above.

Page 4

1120
Solution for Question 2:
Correct Option C) 3,3'-Diaminobenzidine (DAB):
• DAB is a chromogen commonly used in immunohistochemistry (IHC).
• In IHC, antibodies are used to detect specific antigens or proteins in tissue samples.
• The secondary antibody is often linked to an enzyme, such as horseradish peroxidase (HRP).
• When DAB is used as a substrate for HRP, a reaction occurs at the site of the target antigen.
• This reaction results in the formation of a brown precipitate, providing a visible marker for the location
of the antigen.
• DAB is widely used for its ability to produce a sharp, brown color, making it easy to identify and
visualize specific proteins or cellular structures under a microscope.
Incorrect Options:
Option A - Oil Red O:
• Oil Red O is a stain used to detect lipids or fats. It stains lipid droplets in tissues, producing a red
color. It is not used in immunohistochemistry, and it doesn't involve the formation of a brown precipitate
at the site of the target antigen.
Option B - Papanicolaou Stain:
• Papanicolaou stain, commonly known as Pap stain, is used for cytology rather than
immunohistochemistry. It is utilized for the examination of cells collected from cervical smears and
other cytological specimens. This stain involves multiple steps and produces a range of colors but
doesn't involve the formation of a brown precipitate.
Option D - Periodic Acid-Schiff (PAS):
• PAS stain is used to highlight structures containing glycogen, mucin, and other carbohydrate-rich
macromolecules. It does not involve the formation of a brown precipitate at the site of the target antigen
in immunohistochemistry.

Solution for Question 3:


Correct Option D - Parathyroid hormone-related protein:
• Elevated serum calcium levels, known as hypercalcemia, are a known paraneoplastic syndrome
associated with certain cancers (breast Ca, Renal Ca, adult T cell leukemia/lymphoma), including lung
cancer. In lung cancer, hypercalcemia is often attributed to the secretion of parathyroid
hormone-related protein (PTHrP), TGF alpha, TNF, and IL-1 by tumor cells.
Elevated serum calcium levels, known as hypercalcemia, are a
known paraneoplastic syndrome associated with certain cancers (breast Ca, Renal Ca, adult T cell leu
kemia/lymphoma), including lung cancer. In lung cancer, hypercalcemia is often attributed to the secret
ion of parathyroid hormone-related protein (PTHrP), TGF alpha, TNF, and IL-1 by tumor cells.

Incorrect Options A, B and C are incorrect and the correct option is explained above.

Page 5

1121
Solution for Question 4:
Correct Option D - Cytokeratin:
• The biopsy shows well-differentiated squamous cell carcinoma with prominent keratinization with
keratin pearl formation, the most likely type of lung cancer is squamous cell carcinoma (SqCC), which
tends to occur centrally and can cause bronchial obstruction.

• The tumor marker most likely to show strong expression in squamous cell carcinoma of the lung is
cytokeratin.
• Cytokeratin is an intermediate filament.

Incorrect Options:
Option A - Alpha-fetoprotein:
• Alpha-fetoprotein is a marker for hepatocellular carcinoma and yolk sac carcinoma.
Option B - Calretinin:
• Calretinin is a marker for mesothelioma.
Option C - Carcinoembryonic antigen:
• Carcinoembryonic antigen is a marker for colonic carcinoma and many other malignancies (pancreatic
Ca, lung Ca).

Solution for Question 5:


Correct Option B - Flushing and wheezing:
• The clinical characteristics of flushing and wheezing are suggestive of carcinoid syndrome, which can
occur due to the secretion of vasoactive substances such as serotonin by carcinoid tumors. In this
case, the presence of nests of cells with neurosecretory granules suggests a carcinoid tumor, which
commonly arises in the gastrointestinal tract, including the appendix. Hemoptysis, fatigue, and anorexia

Page 6

1122
are not typically associated with carcinoid tumors.
• Because of the excessive production of serotonin, carcinoid syndrome is associated with facial
flushing and bronchospasm, leading to wheezing.
Carcinoid tumor and carcinoid syndrome:
Carcinoid Syndrome Features
1. Flushing
• Sudden redness and warmth of the face and neck
• Episodic, triggered by various factors
2. Diarrhea
• Chronic, watery diarrhea
• Associated with abdominal cramping and urgency
3. Wheezing
• Wheezing or difficulty breathing
• Often due to airway constriction
4. Dyspnea
• Shortness of breath or difficulty breathing
• May occur during flushing and wheezing episodes
5. Tachycardia
• Rapid heart rate
• Commonly associated with other symptoms
6. Telangiectasia
• Small, visible blood vessels (telangiectasias)
• Often seen on the face
7. Abdominal Pain
• Abdominal pain or cramping may occur in some cases
• Symptoms can vary in severity

Incorrect Options:
Options A, C and D are incorrect and the correct option is explained above.

Solution for Question 6:


Correct Option B - Small intestine:
• The small intestine is the most common location of the carcinoid tumor
Carcinoid syndrome and neuroendocrine tumor:

Page 7

1123
• Well-differentiated neuroendocrine tumors (NETs) are rare neoplasms that can arise at several body
sites.
• Most originate in the gastrointestinal tract (55 %) and lungs (30%); they rarely arise in the
genitourinary tract.
• Episodic flushing is the clinical hallmark of carcinoid syndrome and occurs in 85% of patients.
• The typical flush associated with midgut NETs (jejunum, ileum, cecum, appendix) begins suddenly
and often lasts 20 to 30 seconds.
• It primarily involves the face, neck, and upper chest, which become red to violaceous or purple and is
associated with a mild burning sensation.
• Severe flushes are accompanied by hypotension and tachycardia. As the disease progresses, the
episodes may last longer, and the flushing may be more diffuse.
• Diarrhoea is a major complaint in most patients with carcinoid syndrome due to rapid intestinal transit
time.
• Right-sided valvular heart disease affects as many as 40% of patients.
• A minority may experience bronchospasm.
Incorrect Options:
Option A – Heart:
• Although the secondary effects of the carcinoid tumor can be affected by the heart, it is not a site of
carcinoid origination.
Option C – Liver:
• The tumor first metastasizes to the liver from the intestine.
• Here, it can produce multiple lesions.
• However, the liver is not the primary site of carcinoid syndrome.
Option D – Kidney:
• Carcinoid tumor does not arise from the kidney.

Solution for Question 7:


Correct Option B - Calcitonin:
Medullary thyroid cancer
• Medullary thyroid cancer is a neuroendocrine tumor derived from the thyroid gland's para-follicular C
cells.
• The medullary carcinomas secrete calcitonin from the para-follicular cells in over-normal amounts.
• Because tumor markers aren't specific, they can not be applied to confirm a diagnosis. A biopsy is
used to confirm the diagnosis.
Incorrect Options:
Option A - HCG:
• HCG is the human chorionic gonadotropin secreted by the cytotrophoblasts and syncytiotrophoblasts.

Page 8

1124
• Elevated levels are seen in patients with choriocarcinoma, and non-seminomatous germ cell tumors.
Option C - CA 19-9:
• Pancreatic cancer is a disease of older adults, mostly occurring between 60 and 80 years of age. The
strongest environmental influence is smoking which doubles the risk; another risk factor is chronic
pancreatitis.
• Pancreatic cancer is associated with CA 19-9.
Option D – NSE:
• Neuroblastoma is the most common extracranial solid tumor in childhood.
• It occurs most commonly during the first five years of life and may arise during infancy.
• Neuroblastoma is associated with neuron-specific enolase.

Solution for Question 8:


Correct Option A – Chromogranin:
• Small cell cancer of the lung originates from neuroendocrine progenitor cells.
• It can secrete polypeptide hormones.
• Neuroendocrine markers such as chromogranin, synaptophysin, and CD57 are present in 75% of
cases.
Incorrect Options:
Option B - Cytokeratin:
• Cytokeratin is a sensitive and specific marker for epithelial tumors such as squamous cell carcinoma.
Option C – Desmin:
• Desmin is a particular marker for muscle tumors such as rhabdomyosarcomas.
Option D – Vimentin:
• Vimentin is a marker for mesenchymal tumors such as sarcomas and many other tumors such as
endometrial, renal cell, and meningioma.

Solution for Question 9:


Correct Option B - Carcinoembryonic antigen:
• Carcinoembryonic antigen (CEA) is associated with carcinomas of the colon, pancreas, stomach, and
breast. CEA can be used to assess responsiveness to treatment. Successful resection of the
underlying tumor results in markers such as CEA disappearing from the serum. Persistent elevation of
CEA post resection, warrants further investigation for the presence of the underlying tumor.
Carcinoembryonic antigen (CEA) is associated with carcinomas of the colon, pancreas, stomach, and
breast.
CEA can be used to assess responsiveness to treatment. Successful resection of the underlying tumor
results in markers such as CEA disappearing from the serum. Persistent elevation of CEA post resecti

Page 9

1125
on, warrants further investigation for the presence of the underlying tumor.
Incorrect Options:
Option A - Alpha-fetoprotein:
• Alpha-fetoprotein is associated with hepatocellular carcinoma and yolk sac tumors
Option C - Chorionic gonadotropin:
• Chorionic gonadotropin is associated with choriocarcinoma.
Option D - Chromogranin:
• Neuroendocrine tumors express chromogranin

Solution for Question 10:


Correct Option A - Chromogranin - Neuroendocrine tumors:
• Chromogranin is a tumour marker that is often associated with neuroendocrine tumors.
Neuroendocrine tumors are a group of tumors that originate from cells of the endocrine
(hormone-producing) and nervous systems.
Incorrect Options:
Option B - Vimentin - Melanoma:
• Vimentin is a type of intermediate filament protein and is not a specific tumor marker for melanoma.
Vimentin is a marker for mesenchymal tumors such as sarcomas and many other tumors such as
endometrial, renal cell, and meningioma. Melanoma is a type of skin cancer that arises from
melanocytes, and markers like S-100, HMB-45, and Melan-A are more commonly associated with
melanoma.
Option C - Leucocyte specific antigen - Sarcoma:
• Leucocyte-specific antigens are a group of cell surface molecules present on white blood cells, but
they are not specific tumor markers for sarcomas. Sarcomas are a diverse group of malignant tumors
that arise from mesenchymal tissues like bone, cartilage, muscle, and fat. Specific markers for
sarcomas include CD99, CD34, vimentin, and desmin.
Option D - S100 - Lymphoma:
• S100 is a calcium-binding protein and is not a specific tumour marker for lymphoma. Lymphomas are
a group of cancers that affect the lymphatic system, and markers such as CD20, CD30, and CD15 are
more commonly associated with lymphoma. S100 is a tumor marker of melanoma, LCH, cartilage
tumors, neurogenic tumors

Solution for Question 11:


Correct Option A - Glutaraldehyde:
• Glutaraldehyde (2-2.5% Glutaraldehyde) is a widely used fixative in electron microscopy. It is a highly
effective fixative for preserving cellular ultrastructure, especially for delicate structures like cell
membranes and organelles. Glutaraldehyde works by cross-linking proteins and other biomolecules in
the cells, preventing their degradation and preserving their structural integrity for electron microscopy

Page 10

1126
analysis.
Incorrect Options:
Option B - Formalin:
• Formalin (10% neutral buffered formalin (NBF)) is a common fixative used in light microscopy, but it is
not the primary choice for electron microscopy. It is mainly used for routine histological processing,
where the preservation of ultrastructural details is not a priority. Formalin fixation is not ideal for electron
microscopy because it can cause significant distortion and damage to delicate cellular structures.
Option C - Picric acid:
• Picric acid is a fixative used in histology, especially for tissues with high lipid content, such as the
brain. It is not commonly used in electron microscopy because it can interfere with the quality of
electron micrographs and cause artifacts.
Option D - Absolute alcohol:
• Absolute alcohol is often used as a dehydrating agent in the tissue preparation process for electron
microscopy but is not used as the primary fixative. Dehydration is typically performed after the fixation
step to remove water from the tissues before embedding in resin for sectioning and imaging.

Page 11

1127
Pulmonary Hypertension,Lung Tumors
1. Which of the following is a characteristic histological finding seen in patients with Bronchiolitis
obliterans?
A. Presence of epithelial hyperplasia in bronchioles
B. Fibrous plugs in bronchioles
C. Normal bronchiolar architecture without any abnormalities
D. Diffuse alveolar damage with hyaline membrane formation
----------------------------------------
2. Inactivating mutations of which of the following is responsible for causing primary pulmonary
hypertension?
A. BMPR1
B. BMPR2
C. SMAD2
D. SMAD4
----------------------------------------
3. In a 2-hour-old preterm newborn with maternal pre-eclampsia, presenting with cyanosis, tachypnea,
and diffuse bilateral ground-glass opacities on chest X-ray, which cell likely produces the deficient
substance?
A. Type 1 pneumocytes
B. Type 2 pneumocytes
C. Alveolar macrophages
D. All of the above
----------------------------------------
4. In a 58-year-old chronic smoker with a persistent cough, unintentional weight loss, muscle weakness
improving with activity, and a central lung mass positive for chromogranin on biopsy, which statement
regarding the underlying condition is false?
A. The tumor shows positive staining for Neuron-specific enolase and Synaptophysin.
B. The tumor displays an amplification of L Myc
C. The tumor marker p40 is considered to be confirmatory
D. None of the above
----------------------------------------
5. Which of the following statements is incorrect regarding small cell carcinoma of lungs?
A. It is classified as a neuroendocrine tumor due to its origin from neuroendocrine cells in the bronchial
epithelium
B. It is known for its distinctive "salt and pepper" appearance of chromatin
C. Horners syndrome is very often associated with this tumor
D. The Azzopardi Effect in this tumor involves the molding and smudging of tumor cells.

1128
----------------------------------------
6. In a 55-year-old female nonsmoker with a peripherally located mass in the right lung and positive
immunohistochemistry for CK 7 and TTF1, which of the following is correct regarding the underlying
condition?

A. Pathogenesis involves loss of function mutation of KRAS, ALK, and EGFR1


B. The tumor shows positive staining for MUC1 and NAPSIN A
C. Atypical Adenomatous Hyperplasia is not a predisposing factor
D. All of the above
----------------------------------------
7. Which of the following is an important risk factor for this condition depicted in the image given below
of gross examination during autopsy?

A. Silica exposure
B. Tobacco smoking.
C. Asbestos exposure
D. Marble dust exposure
----------------------------------------
8. Which of the following is responsible for causing the condition depicted in the histopathological
image given below?

Page 2

1129
A. Hypoxia
B. High altitude
C. Left ventricular failure
D. BMPR2 mutation
----------------------------------------
9. What is the primary role of IL-8 in the pathogenesis of acute respiratory distress syndrome?
A. Activating macrophages
B. Recruiting neutrophils
C. Enhancing surfactant production
D. Damaging endothelial cells
E. Endothelial cell activation
----------------------------------------
10. Which of the following is not associated with the development of Acute Respiratory Distress
Syndrome (ARDS)?
A. Near drowning
B. Gastric aspiration
C. Chronic lung injury
D. Transfusion-Related Acute Lung Injury (TRALI)
----------------------------------------
11. Match the following: 1 Squamous cell carcinoma A Loss of chromosome 3p 2 Adenocarcinoma B
Loss of chromosome 3 and 17p 3 Oat cell carcinoma C Gain of function of EFGR
1 Squamous cellcarcinoma A Loss of chromosome 3p
2 Adenocarcinoma B Loss of chromosome 3 and 17p
3 Oat cell carcinoma C Gain of function of EFGR

A. 1-A, 2-C, 3-B


B. 1-B, 2-C, 3-A
C. 1-C, 2-B, 3-A
D. 1-B, 2-A, 3-C

Page 3

1130
----------------------------------------
12. Which of the following is the most aggressive lung tumor?
A. Large cell carcinoma
B. Small cell carcinoma
C. Adenocarcinoma
D. Squamous cell carcinoma
----------------------------------------
13. Which of the following histopathological findings is a hallmark feature of adenocarcinoma-in-situ?
A. Azzopardi effect
B. Salt and pepper appearance
C. Keratin pearls
D. Lepidic pattern
----------------------------------------
14. Which genetic mutation is most likely to be present in lung cancer among non-smokers?
A. KRAS mutation
B. TP53 mutation
C. EGFR mutation
D. ALK mutation
----------------------------------------
15. Which radiological finding is expected in the most common type of benign lung tumor?
A. Signet ring sign
B. Crazy pavement pattern
C. Eggshell calcification
D. Coin lesion
----------------------------------------
16. What immunohistochemical marker is typically positive in both Sugar Tumors in the lungs and
angiomyolipoma (AML) of the kidney?
A. CD20
B. CD34
C. HMB 45
D. AE1/AE3
----------------------------------------
17. Mark the correct statement about carcinoid tumors:
A. Coat button lesion on bronchoscopy
B. Electron microscopy shows Neurosecretory granule
C. Nesting of cells observed on microscopy

Page 4

1131
D. All of the above
----------------------------------------
18. Which of the following is the most common risk factor associated with malignant mesothelioma?
A. Smoking
B. Radon exposure
C. Asbestosis
D. Silicosis
----------------------------------------
19. Which of the following is not a feature of adenocarcinoma lung?
A. Peripheral tumor
B. Associated with asbestos exposure
C. Calretinin positive
D. Non-branching microvilli on electron microscopy
----------------------------------------

Correct Answers
Question Correct Answer

Question 1 2
Question 2 2
Question 3 2
Question 4 3
Question 5 3
Question 6 2
Question 7 3
Question 8 4
Question 9 2
Question 10 3
Question 11 2
Question 12 2
Question 13 4
Question 14 3
Question 15 4
Question 16 3
Question 17 4
Question 18 3

Page 5

1132
Question 19 3

Solution for Question 1:


Correct Option B - Fibrous plugs in bronchioles.
• Bronchiolitis Obliterans with Organizing Pneumonia (BOOP) / Cryptogenic Organizing Pneumonia
(COP) typically arises in response to lung infection or inflammatory lung injury, commonly linked with
viral or bacterial pneumonia, inhaled toxins, medications, connective tissue disorders, and
graft-versus-host disease in stem cell transplant patients.
• The condition is fibrotic, with fibrous plugs found in bronchioles, alveoli, and alveolar sacs.
• This fibrous plus on histological examination shows Masson bodies, Masson bodies are composed of
loosely organized connective tissue. They consist of fibrin, collagen, and inflammatory cells, such as
macrophages and fibroblasts, along with other cellular debris.

Incorrect Options:
Options A, C, and D are incorrect.

Solution for Question 2:


Correct Option B - BMPR2:
• Primary pulmonary hypertension (PPH) is associated with mutations in the bone morphogenetic
protein receptor type 2 (BMPR2) gene. When BMPR2 is defective, it leads to dysregulation in the
signaling pathway, resulting in failure of smooth muscle apoptosis, which contributes to an increase in
smooth muscle and the development of plexiform lesions in the pulmonary vasculature.
Incorrect Options:
Options A, C, and D are incorrect as they are associated with Juvenile Rectal Polyposis.

Page 6

1133
Solution for Question 3:
Correct Option B - Type 2 pneumocytes:
• The newborn is likely suffering from Neonatal respiratory distress syndrome (RDS).
• Clinical Presentation: Preterm neonates (less than 32 weeks GA), present with cyanosis (bluish
discoloration), tachypnea (rapid breathing), nasal flaring, and intercostal retractions. These signs
suggest respiratory distress
• Chest X-ray Findings: The chest X-ray shows diffuse bilateral ground-glass opacities. This indicates
that the lungs have a hazy appearance due to the accumulation of fluid and cellular debris in the
alveoli, which impairs gas exchange.
• Pathogenesis: Neonatal respiratory distress syndrome (RDS) stems from surfactant deficiency,
particularly in immature lungs, increasing surface tension within small airways and alveoli, thereby
reducing lung compliance. LaPlace's law, represented as P=2T/R, illustrates the relationship between
pressure (P), surface tension (T), and radius (R), where increased surface tension demands more
pressure to maintain alveolar shape. Reduced surfactant leads to widespread atelectasis, impairing gas
exchange, and damaging respiratory epithelium via cytokine-mediated inflammation. Pulmonary edema
exacerbates surfactant inactivation as protein-rich fluid leaks into alveoli
• Cell Type Function Type 1 Pneumocytes Facilitate gas exchange by forming a thin barrier across
which oxygen and carbon dioxide diffuse Type 2 Pneumocytes Produce and secrete surfactant, which
reduces surface tension in the alveoli and prevents collapse Act as stem cells, capable of
differentiating into type 1 pneumocytes Dust Cells (Alveolar Macrophages) Phagocytose (engulf and
digest) particles, bacteria, and debris present in the alveoli Contribute to the immune defense of the
lungs by removing pathogens and foreign material
• Facilitate gas exchange by forming a thin barrier across which oxygen and carbon dioxide diffuse
• Produce and secrete surfactant, which reduces surface tension in the alveoli and prevents collapse
• Act as stem cells, capable of differentiating into type 1 pneumocytes
• Phagocytose (engulf and digest) particles, bacteria, and debris present in the alveoli
• Contribute to the immune defense of the lungs by removing pathogens and foreign material
Cell Type
Function
Type 1 Pneumocytes
• Facilitate gas exchange by forming a thin barrier across which oxygen and carbon dioxide diffuse
Type 2 Pneumocytes
• Produce and secrete surfactant, which reduces surface tension in the alveoli and prevents collapse

• Act as stem cells, capable of differentiating into type 1 pneumocytes


Dust Cells (Alveolar Macrophages)
• Phagocytose (engulf and digest) particles, bacteria, and debris present in the alveoli
• Contribute to the immune defense of the lungs by removing pathogens and foreign material

Page 7

1134
Incorrect Options:
Option A - Type 1 pneumocytes:
• Type 1 pneumocytes are responsible for gas exchange in the lungs, as they form the thin barrier
across which oxygen and carbon dioxide diffuse. They do not produce surfactant. Their main function is
to provide a large surface area for efficient gas exchange rather than producing surfactant.
Option C - Alveolar macrophages:
• Alveolar macrophages, also known as dust cells are specialized immune cells located in the alveoli.
They play a crucial role in phagocytosing (engulfing and digesting) particles, bacteria, and debris that
may enter the lungs. While they contribute to the overall health of the lungs by clearing pathogens and
foreign material, they do not produce surfactants.
Option D - All of the above:
• This option suggests that all the cells mentioned (Type 1 pneumocytes, Type 2 pneumocytes, or dust
cells) are responsible for producing surfactant in the newborn. This is not true

Solution for Question 4:


Correct Option C - The tumor marker p40 is considered to be confirmatory:
• The patient is probably suffering from squamous cell and small cell carcinoma of the lung.
• Smoking history is a significant risk factor for small cell carcinoma of the lung.
• Clinical features: persistent cough, unintentional loss of weight, paraneoplastic syndrome:
Lambert-Eaton Myasthenic Syndrome Lambert-Eaton Myasthenic Syndrome: Small cell lung
carcinoma (SCLC) cells often express voltage-gated calcium channels (VGCCs) on their surface. The
expression of VGCC by tumor cells triggers the immune system to produce autoantibodies directed
against these channels. The autoantibodies produced against VGCCs in the tumor tissue can
cross-react with presynaptic VGCC antigens at the neuromuscular junction, leading to impaired
neurotransmitter release and muscle weakness characteristic of Lambert-Eaton Myasthenic Syndrome
(LEMS).
• Lambert-Eaton Myasthenic Syndrome:
• Small cell lung carcinoma (SCLC) cells often express voltage-gated calcium channels (VGCCs) on
their surface. The expression of VGCC by tumor cells triggers the immune system to produce
autoantibodies directed against these channels. The autoantibodies produced against VGCCs in the
tumor tissue can cross-react with presynaptic VGCC antigens at the neuromuscular junction, leading to
impaired neurotransmitter release and muscle weakness characteristic of Lambert-Eaton Myasthenic
Syndrome (LEMS).
• Pathogenesis: SCLC carcinogenesis can arise from pathways disrupting DNA repair mechanisms,
with prevalent mutations including loss of the RB1 tumor suppressor gene and TP53 mutations
reducing cancer cell apoptosis. Additionally, nearly all SCLC tumors exhibit a deletion in chromosome
3p's short arm, encompassing the tumor suppressor gene FHIT.
• Biopsy of the lung mass revealing poorly differentiated nests of cells that stain positive for
chromogranin is characteristic of small cell lung cancer (SCLC).
• Small cell lung cancer being a neuroendocrine tumor, shows a Zellballen pattern (nesting) of cells that
stain positive for chromogranin, Synaptophysin, and Neuron-specific enolase (NSE).
• Lambert-Eaton Myasthenic Syndrome:

Page 8

1135
• Small cell lung carcinoma (SCLC) cells often express voltage-gated calcium channels (VGCCs) on
their surface. The expression of VGCC by tumor cells triggers the immune system to produce
autoantibodies directed against these channels. The autoantibodies produced against VGCCs in the
tumor tissue can cross-react with presynaptic VGCC antigens at the neuromuscular junction, leading to
impaired neurotransmitter release and muscle weakness characteristic of Lambert-Eaton Myasthenic
Syndrome (LEMS).

( Zellballen pattern (nesting) of cells).


• On arriving at a diagnosis of Small cell lung carcinoma, Option A: The tumor showing positive staining
for Neuron-specific enolase and Synaptophysin is true. These markers are commonly used in the
diagnosis of neuroendocrine tumors, including SCLC. Option B: The tumor displaying an amplification
of L Myc is also true. L-Myc amplification is a common genetic alteration seen in SCLC. Option C: The
tumor marker p40 is not considered to be confirmatory for SCLC. While p40 staining is useful in
distinguishing squamous cell carcinoma from other lung cancers.
• Option A: The tumor showing positive staining for Neuron-specific enolase and Synaptophysin is true.
These markers are commonly used in the diagnosis of neuroendocrine tumors, including SCLC.
• Option B: The tumor displaying an amplification of L Myc is also true. L-Myc amplification is a common
genetic alteration seen in SCLC.
• Option C: The tumor marker p40 is not considered to be confirmatory for SCLC. While p40 staining is
useful in distinguishing squamous cell carcinoma from other lung cancers.
• Option A: The tumor showing positive staining for Neuron-specific enolase and Synaptophysin is true.
These markers are commonly used in the diagnosis of neuroendocrine tumors, including SCLC.
• Option B: The tumor displaying an amplification of L Myc is also true. L-Myc amplification is a common
genetic alteration seen in SCLC.
• Option C: The tumor marker p40 is not considered to be confirmatory for SCLC. While p40 staining is
useful in distinguishing squamous cell carcinoma from other lung cancers.
Incorrect Options:
All options have been explained under the correct option.

Page 9

1136
Solution for Question 5:
Correct Option C - Horners syndrome is very often associated with this tumor:

SCLC (Small Cll lung Carcer)


• History: Small-cell lung carcinoma is strongly associated with smoking.
• Symptoms: The patient usually presents with a persistent cough, dyspnea, chest pain, anorexia,
unintentional weight loss and paraneoplastic syndromes (eg: LEMS, SIADH, etc.)
• Physical Examination: Clubbing, cyanosis, and lymphadenopathy may be observed.
• Imaging Findings: Chest X-ray usually reveals a centrally located mass arising from the bronchi.
• Histological Findings in SCLC: "Salt and Pepper" Appearance of Chromatin: This refers to the
characteristic appearance of small cell carcinoma cells under the microscope, with dark specks (salt)
and pale regions (pepper) of chromatin. The image shows mostly spindle-shaped/oval cells with scanty
cytoplasm, ill-defined borders, finely granular chromatin, and inconspicuous nucleoli.
• "Salt and Pepper" Appearance of Chromatin: This refers to the characteristic appearance of small cell
carcinoma cells under the microscope, with dark specks (salt) and pale regions (pepper) of chromatin.

• The image shows mostly spindle-shaped/oval cells with scanty cytoplasm, ill-defined borders, finely
granular chromatin, and inconspicuous nucleoli.
• "Salt and Pepper" Appearance of Chromatin: This refers to the characteristic appearance of small cell
carcinoma cells under the microscope, with dark specks (salt) and pale regions (pepper) of chromatin.

• The image shows mostly spindle-shaped/oval cells with scanty cytoplasm, ill-defined borders, finely
granular chromatin, and inconspicuous nucleoli.

Horner's syndrome is not typically associated with small cell carcinoma of the lung. Horner's syndrome
is a combination of signs caused by disruption of the sympathetic nerve pathway to the eye and is ofte
n associated with tumors affecting the sympathetic nerves in the neck or chest. In the context of lung c
ancer, it is more commonly linked to Pancoast tumors, which are a type of non-small cell lung cancer r

Page 10

1137
ather than small cell carcinoma. All other statements are true regarding small cell carcinoma.
Incorrect Options:
Option A - It is classified as a
neuroendocrine tumor due to its origin from neuroendocrine cells in the bronchial epithelium.:
• Small cell carcinoma of the lung is characterized by the expression of various neuroendocrine
markers. These include chromogranin, synaptophysin, and neuron-specific enolase (NSE).
Option B - It is known for its distinctive "salt and pepper" appearance of chromatin:

• Microscopically, small cell carcinoma is known for its distinctive "salt and pepper" appearance of
chromatin. This refers to the irregular and stippled distribution of chromatin within the tumor cells when
viewed under a microscope. This feature is a result of the high nuclear-cytoplasmic ratio and the
hyperchromatic nuclei of the tumor cells.
Option D - The Azzopardi Effect in this tumor involves the molding and smudging of tumor cells:
• This phenomenon is associated with the penetration of DNA particles into blood vessels. Feulgen
stain, a special stain for DNA, can be used to highlight this process, emphasizing the DNA transfer from
tumor cells into blood vessels. The staining technique helps visualize the genetic material and
contributes to understanding the biology of the tumor.

Solution for Question 6:


Correct Option B - The tumor shows positive staining for MUC1 and NAPSIN A:
• The patient is likely suffering from adenocarcinoma carcinoma of the lung.
• History: While adenocarcinoma can occur in smokers, it is more commonly seen in nonsmokers.
• Clinical Presentation: symptoms like cough, hemoptysis, or weight loss, lung adenocarcinoma may
present with locoregional spread symptoms, including superior vena cava obstruction, phrenic nerve
palsy, Horner syndrome, compression of the brachial plexus, or pericardial effusion.
• Pathogenesis: Loss of chr3, chr17, CDKN2A, p53 mutation and amplification of FGFR1

Page 11

1138
• Chest X-ray: Usually presents as a peripherally located mass.
• Histology: typically exhibits glandular structures.

• Immunohistochemistry: CK 7, TTF1(thyroid transcription factor 1), MUC1, NAPSIN A (Napsin A
aspartic peptidase) POSITIVE.

Incorrect Options:
Option A - Pathogenesis Involves Loss of Function Mutation of KRAS, ALK, and EGFR1: This stateme
nt is not true. Adenocarcinoma of the lung, especially in nonsmokers, is often associated with gain of fu
nction mutations such as mutations in KRAS, ALK, and EGFR.
Option C - Atypical Adenomatous Hyperplasia is Not a Predisposing Factor: Atypical Adenomatous Hy
perplasia (AAH) and Adenocarcinoma In Situ (AAH) are considered a
precursor lesion to adenocarcinoma of the lung. Therefore, this statement is not true.

Solution for Question 7:


Correct Option C - Asbestos exposure:

Page 12

1139
• The image shows a thick, firm, white pleural tumor tissue that ensheathes the lung. This is indicative
of malignant mesothelioma.
• The most important risk factor for mesothelioma is asbestos exposure.
• Asbestos fibers are mineral fibers that, when inhaled, irritate the pleura. The long and thin structure of
asbestos fiber is particularly hazardous as it facilitates penetration of the lung tissue, causing repeated
injury to the mesothelial cells lining the pleura. Asbestos fibers disrupt mitosis by severing the mitotic
spindles resulting in aneuploidy.
Incorrect Options:
Options A, B and D are incorrect.

Solution for Question 8:


Correct Option D - BMPR2 mutation:
• The given histopathological image shows plexiform lesions, which are a characteristic feature of
primary pulmonary hypertension. It is caused by a defect in the BMPR2 (Bone morphogenic protein
receptor - 2) gene defect, because of which apoptosis in smooth muscle does not take place, and there
is an increase in smooth muscle mass.
Incorrect Options:
Options A, B, and C are causes of secondary pulmonary hypertension, not primary.

Solution for Question 9:


Correct Option B - Recruiting neutrophils:
• IL-8 (Interleukin-8) primarily acts as a chemoattractant, recruiting neutrophils from the bloodstream to
the site of inflammation within the lungs. Neutrophils are key mediators of the inflammatory response,

Page 13

1140
and their recruitment by IL-8 contributes to the amplification of the inflammatory cascade in ARDS.

Incorrect Options:
Options A, C, and D are not involved with IL-8.

Solution for Question 10:


Correct Option C - Chronic lung injury:
• ARDS is characterized by the acute onset of respiratory symptoms following various precipitating
factors such as sepsis, trauma, pneumonia, aspiration of gastric contents, near drowning, and
transfusion-related acute lung injury (TRALI). Chronic lung injury does not lead to acute respiratory lung
syndrome.
Incorrect Options:
Options A, B, and D can lead to acute respiratory distress syndrome.

Solution for Question 11:


Correct Option B - 1-B, 2-C, 3-A:

Solution for Question 12:


Correct Option B - Small cell carcinoma:
• Small cell carcinoma of the lung is considered the most aggressive type of lung tumor. It is
characterized by rapid growth, early metastasis, and a poor prognosis. It is a centrally located
neuroendocrine carcinoma associated with smoking.
Incorrect Options:
Options A, C, and D are not the most aggressive lung tumors.

Solution for Question 13:


Correct Option D -Lepidic pattern:
• Adenocarcinoma-in-situ (AIS) is a non-invasive tumor of the lung, where atypical cells proliferate
along the alveolar lining without invasion into the surrounding tissue. It shows a lepidic pattern or
butterfly on fence pattern on histology.

Page 14

1141
Incorrect Options:
Option A - Azzopardi effect:
• The Azzopardi effect refers to incrustation of blood vessel wall with basophilic nuclear material. It is a
feature of small cell carcinoma.

Option B - Salt and pepper appearance:


• The salt and pepper appearance of chromatin refers to the pattern seen in neuroendocrine tumors,
such as carcinoid tumors, where the nuclei of tumor cells appear speckled with dark (pepper-like) and
light (salt-like) chromatin. It is also seen in small-cell carcinoma.

Page 15

1142
Option C - Keratin pearls:
• Keratin pearls are a histopathological feature typically seen in squamous cell carcinoma of the lung.
They consist of concentric layers of keratinized squamous epithelial cells.

Solution for Question 14:


Correct Option C - EGFR mutation:
• The most common type of lung cancer that occurs in nonsmokers is adenocarcinoma, and EGFR
(Epidermal Growth Factor Receptor) mutations are most likely the cause of lung cancer among
non-smokers. These mutations result in the activation of the EGFR pathway, leading to uncontrolled
cell growth and proliferation.
Incorrect Options:

Page 16

1143
Options A, B, and D are not commonly associated with lung cancer in non-smokers.

Solution for Question 15:


Correct Option D - Coin lesion:
• The most common benign tumor of the lungs is pulmonary hamartoma. Pulmonary hamartomas
typically present as solitary, well-defined nodules on radiological imaging, often referred to as "coin
lesions."
Incorrect Options:
Option A - Signet ring sign: The signet ring sign is seen in the case of bronchiectasis.
Option B - Crazy pavement pattern: The crazy pavement pattern is seen in usual interstitial pneumonia
/idiopathic pulmonary fibrosis.
Option C - Eggshell calcification: Eggshell calcification refers to peripheral calcification of lymph nodes
and can be seen in sarcoidosis, silicosis, and tuberculosis.

Solution for Question 16:


Correct Option - C HMB 45:
• HMB 45 is a specific immunohistochemical marker that is typically positive in Perivascular Epithelioid
Cell Tumors, i.e., both Sugar Tumors in the lungs and angiomyolipoma (AML) of the kidney.
Incorrect Option s:
Options A, B, and D are not positive in both sugar tumors and angiomyolipoma.

Solution for Question 17:


Correct Option D - All of the above:
• All of the given statements about carcinoid tumors are correct.
Option A - Coat button lesion on bronchoscopy:
• Carcinoid tumors, particularly bronchial carcinoids, often present as coat button lesions on
bronchoscopy.

Page 17

1144
Option B - Electron microscopy shows Neurosecretory granules:
• Carcinoid tumors are neuroendocrine tumors, and electron microscopy can reveal neurosecretory
granules within the tumor cells.

Option C - Nesting of cells observed on microscopy:


• Microscopically, carcinoid tumors typically exhibit a nesting growth pattern of cells.

Page 18

1145
Solution for Question 18:
Correct Option C - Asbestosis:
• Asbestos exposure is the primary risk factor for the development of mesothelioma, particularly
occupational exposure in industries such as construction, shipbuilding, and insulation manufacturing.
Asbestos fibers can be inhaled and lodged in the pleura, leading to chronic inflammation and,
eventually, the development of mesothelioma.
Incorrect Options:
Options A, B, and D are not associated with the causation of malignant mesothelioma.

Solution for Question 19:


Correct Option C - Calretinin positive:
• Adenocarcinoma of the lung typically presents as a peripheral tumor arising in the outer regions of the
lung. Calretinin is a marker that is commonly used in the diagnosis of malignant mesothelioma.
Adenocarcinoma of the lung is typically negative for calretinin staining.

Incorrect Options:
Option A - Peripheral tumor: Adenocarcinomas often arise in the periphery of the lung tissue.
Option B - Associated with asbestos exposure: Adenocarcinoma of the lung is associated with asbesto
s exposure.
Option D - Non-branching microvilli on electron microscopy: The adenocarcinoma of the lungs exhibits
non-branching microvilli on electron microscopy.

Page 19

1146
Page 20

1147
Previous Year Questions
1. In the WHO classification of astrocytoma, which specific grade is glioblastoma categorized as?
A. I
B. II
C. III
D. IV
----------------------------------------
2. Which of the following histological findings are associated with the diagnosis in a 40-year-old female
who presents with frequent headaches, and is found to have raised intracranial pressure and a tumor
located in the dura?
A. Fried egg appearance
B. Flexner - Wintersteiner rosettes
C. Psammoma bodies with whorling of tumor cells
D. Homer - Wright rosettes
----------------------------------------
3. A 24-year-old presented with swelling in the wrist joint. Histopathological examination of the swelling
revealed spindle cells with palisading of the nuclei and Verocay bodies. Which of the following is the
most likely condition?
A. Schwannoma
B. Neurofibroma
C. Neuroma
D. Glioma
----------------------------------------
4. In the most common variant of Guillain-Barre syndrome, determine the region that will be impacted
based on an electron microscopic image of a sliced nerve fiber.

A. 1
B. 2
C. 3

1148
D. 4
----------------------------------------
5. What is the probable diagnosis for a 40-year-old woman who presents with seizures and has
undergone a CT-guided biopsy, revealing the presence of 'psammoma bodies' in the histopathological
report?

A. Astrocytoma
B. Medulloblastoma
C. Meningioma
D. Ependymoma
----------------------------------------
6. Tau protein inclusions are involved in:
A. Alzheimer’s disease
B. Huntington disease
C. Amyotrophic lateral sclerosis
D. CNS lymphoma
----------------------------------------
7. Which of the following is the correct statement regarding Huntington’s chorea?
A. It is a trinucleotide repeat disorder
B. There is a loss of function type of mutation
C. There are abnormal repeats of CUG
D. Abnormality is seen due to mutation in chromosome 6
----------------------------------------
8. Which of the following is a risk factor for Alzheimer’s disease?
A. Apo E1
B. Apo E2
C. Apo E3
D. Apo E4
----------------------------------------

Page 2

1149
Correct Answers
Question Correct Answer

Question 1 4
Question 2 3
Question 3 1
Question 4 2
Question 5 3
Question 6 1
Question 7 1
Question 8 4

Solution for Question 1:


• In the WHO (World Wellbeing Association) evaluating framework for astrocytoma, glioblastoma falls
under Grade IV. This is the highest grade, and it means that the tumor is very bad and growing quickly.
Glioblastoma is otherwise called a grade IV astrocytoma.
• Based on how the tumor cells look under a microscope, the WHO grades astrocytoma, with higher
grades indicating a more malignant and aggressive tumor. Astrocytoma of grade I and grade II are
low-grade tumors that typically develop slowly and are less likely to spread to other brain regions.
Anaplastic astrocytoma, also known as Grade III tumors, are more aggressive than Grades I and II
tumors, while glioblastomas, also known as Grade IV gliomas, are the most aggressive and grow the
fastest.
Incorrect choices:
• Option a. &Option; b. option c. are incorrect as GLIOBLASTOMA is grade IV astrocytoma.

Solution for Question 2:


• The most probable diagnosis in this case scenario would be Meningioma.
• It is a benign intracranial, extra-axial tumor that originates from the meningothelial cells of the
arachnoid matter.
• The most common sites are the parasagittal region of the cerebral hemisphere and the olfactory
groove.
• The characteristic microscopic feature of meningioma is the whorled pattern of meningothelial cells
and psammoma bodies.

Page 3

1150
Incorrect Choices:
• Option a. Fried egg appearance: It is the microscopic feature of oligodendroglioma. It arises from
oligodendrocytic cells. Hence they are not dural-based tumors. 3 C’s of oligodendroglioma- Clear halo
of cytoplasm (Fried egg appearance), Calcification, Chicken-wire appearance of anastomosing
capillaries. Symptoms include headache, visual disturbances, seizures, etc.
• 3 C’s of oligodendroglioma- Clear halo of cytoplasm (Fried egg appearance), Calcification,
Chicken-wire appearance of anastomosing capillaries. Symptoms include headache, visual
disturbances, seizures, etc.
• Option b. Flexner-Wintersteiner rosettes: It is typically seen in retinoblastoma, which is the most
common malignant ocular tumor found in children. On microscopic examination, the tumor cells are
arranged in rosettes which are of 2 types- One is the Flexner Wintersteiner rosette, where the nuclei of
the tumor cells are arranged around and away from the lumen. Another is the Homer-Wright rosette,
where the tumor cells are arranged around a central neurofibrillary structure.
• On microscopic examination, the tumor cells are arranged in rosettes which are of 2 types- One is the
Flexner Wintersteiner rosette, where the nuclei of the tumor cells are arranged around and away from
the lumen.
• Another is the Homer-Wright rosette, where the tumor cells are arranged around a central
neurofibrillary structure.
• Option d. Homer-Wright rosettes: This is seen in both neuroblastoma and medulloblastoma. The
tumor cells are arranged radially around a central fibrillar eosinophilic material which stains positively
with silver impregnation, which suggests that they are young nerve fibrils. Neuroblastoma is seen in
children < 5 years of age. They present with abdominal distension, fever, and weight loss.
Medulloblastoma is seen in individuals more than 20 years of age. They typically present with
headaches, nausea, vomiting, ataxia, and seizures. However, they are not dural-based tumors.
• 3 C’s of oligodendroglioma- Clear halo of cytoplasm (Fried egg appearance), Calcification,
Chicken-wire appearance of anastomosing capillaries. Symptoms include headache, visual
disturbances, seizures, etc.
• On microscopic examination, the tumor cells are arranged in rosettes which are of 2 types- One is the
Flexner Wintersteiner rosette, where the nuclei of the tumor cells are arranged around and away from
the lumen.

Page 4

1151
• Another is the Homer-Wright rosette, where the tumor cells are arranged around a central
neurofibrillary structure.

Solution for Question 3:


Correct Option A - Schwannoma:
• Schwannomas are benign tumors derived from Schwann cells.
• Histopathologically, schwannomas exhibit spindle cells arranged in a palisading pattern, with Verocay
bodies present.
• Verocay bodies are characteristic histological features of schwannomas, representing areas of
nuclear palisading and fibrillary cytoplasmic processes.
Incorrect Options:
• Options B, C, and D do not have the histopathological findings described in the question.
Option B - Neurofibroma:
• Neurofibromas are benign tumors that arise from peripheral nerves.
• They consist of a mixture of Schwann cells, fibroblasts, and other cellular elements.
• Histopathologically, neurofibromas display a more heterogeneous appearance compared to
schwannomas, with a variety of cell types and an absence of Verocay bodies.
Option C - Neuroma:
• Neuromas refer to abnormal nerve tissue growth resulting from nerve injury or trauma.
• They are characterized by disorganized nerve fibres and can cause pain, tingling, and other sensory
disturbances.
Option D - Glioma:
• Gliomas are tumors that originate from glial cells.

Solution for Question 4:


Correct Option B:
In AIDP, the most common variant of GBS, the primary site of damage is the myelin sheath. Myelin is t
he fatty substance that surrounds and insulates the axons, allowing for efficient conduction of nerve im
pulses. In GBS, the immune system mistakenly attacks and damages the myelin sheath, leading to de
myelination. This results in impaired nerve conduction and the characteristic symptoms of GBS, such a
s muscle weakness, sensory disturbances, and loss of reflexes.
Incorrect Option:
Option A. 1. Schwann cells: Schwann cells are the supporting cells of the peripheral nervous system a
nd play a crucial role in forming the myelin sheath around the axons. While Schwann cells can be invol
ved in GBS, they are not the primary site of damage in the most common variant, acute inflammatory d
emyelinating polyradiculoneuropathy (AIDP).

Page 5

1152
Option C. 3. Myelin: While axonal damage can occur in GBS, it is more commonly associated with vari
ants such as acute motor axonal neuropathy (AMAN) or acute motor-sensory axonal neuropathy (AMS
AN). In the most common AIDP variant, the primary pathology is focused on the myelin sheath rather t
han the axon itself.
Option D. 4. Mesaxon: The mesaxon refers to the region where the inner and outer layers of the Schw
ann cell membrane meet. It is involved in the formation of the myelin sheath. Although the mesaxon ca
n be affected in GBS, it is not the primary area of damage in the most common AIDP variant.

Solution for Question 5:


Correct Option: C
The most likely diagnosis based on the presence of "psammoma bodies" in the histopathological findin
gs is Meningioma.
Explanation of the options:
Option A: Astrocytoma: Astrocytoma is a type of brain tumor that arises from astrocytes, which are star
-shaped cells in the brain. Psammoma bodies are not typically associated with astrocytomas.
Option B: Medulloblastoma: Medulloblastoma is a type of malignant brain tumor that primarily affects c
hildren. Psammoma bodies are not commonly observed in medulloblastomas.
Option C: Meningioma: Meningioma is a type of brain tumor that arises from the meninges, the protecti
ve membranes surrounding the brain and spinal cord. Psammoma bodies are characteristic microscopi
c calcifications commonly seen in meningiomas, particularly in certain subtypes such as psammomato
us meningioma. The presence of psammoma bodies is highly suggestive of a meningioma.
Option D: Ependymoma: Ependymomas are tumors that develop from ependymal cells, which line the
ventricles of the brain and the central canal of the spinal cord. Psammoma bodies are not a
typical feature of ependymomas.
Given the presence of psammoma bodies in the histopathological findings, the most likely diagnosis in
this case is Meningioma.

Solution for Question 6:


Correct Option A:Alzheimer’s disease
• The fundamental abnormality in Alzheimer’s disease is the accumulation of two proteins (Aβ and tau)
in specific brain regions, in the forms of plaques and tangles.
Incorrect options:
Option B: Huntington disease is caused by CAG trinucleotide repeat expansions in a gene located on 4
p16.3 that encodes the protein huntingtin. The disorder is characterized by involuntary jerky movement
s of all parts of the body; writhing movements of the extremities are typical.
Option C: Amyotrophic lateral sclerosis results from the death of lower motor neurons in the spinal cord
and brain stem as well as upper motor neurons (Betz cells) in the motor cortex. The loss of lower moto
r neurons results in the denervation of muscles, muscular atrophy (the “amyotrophy” of the condition),

Page 6

1153
weakness, and fasciculations, while the loss of upper motor neurons results in paresis, hyperreflexia, a
nd spasticity, along with Babinski sign.
Option D: Primary CNS lymphoma occurs mostly as diffuse large B-cell lymphomas. The diagnosis is c
onfirmed by immunohistochemistry for B cell markers such as CD20.

Solution for Question 7:


Correct Option A - It is a trinucleotide repeat disorder:
• Huntington's chorea, also known as Huntington's disease, is an inherited neurodegenerative disorder.
• It is caused by a specific mutation in the huntingtin (HTT) gene, which is located on chromosome 4.
• Huntington's chorea is a trinucleotide repeat disorder involving the expansion of a repeated
trinucleotide sequence (CAG) within the HTT gene.
• The expanded repeat leads to the production of an abnormal form of the huntingtin protein, which
causes the characteristic symptoms of the disease.
Incorrect Options:
Option B - There is a loss of function type of mutation: Huntington's chorea is characterized by a
gain-of-function mutation caused by the expansion of the CAG trinucleotide repeat.
Option C - There are abnormal repeats of CUG: Huntington's chorea is a
trinucleotide repeat disorder involving the expansion of a
repeated trinucleotide sequence (CAG) within the HTT gene.
Option D - Abnormality is seen due to mutation in chromosome 6: Huntington's chorea is caused by a
specific mutation in the huntingtin (HTT) gene, which is located on chromosome 4.

Solution for Question 8:


Correct Option D - Apo E4:
• The ApoE4 variant of the Apo E gene is considered a risk factor for Alzheimer's disease.
• The presence of ApoE4 is associated with a higher likelihood of developing Alzheimer's disease
and contribute to an earlier onset of the condition.
Incorrect Options:
Options A, B, and C are not a risk factor for Alzheimer’s disease.

Page 7

1154
Skin, Bones and Joints
1. Which of the following is a common cytogenic abnormality seen in synovial sarcomas?
A. t(x;18)
B. t(11;22)
C. t(12;22)
D. t(9;22)
----------------------------------------
2. What is the likely diagnosis for a 35-year-old woman presenting with painless nodules in the pinna of
the ear and extensor surface of right forearm, along with recurrent episodes of fever and toe pain, with
histopathological findings of tissue sample below?

A. Rheumatoid arthritis
B. Cysticercosis
C. Gout
D. Chondrocalcinosis
----------------------------------------
3. Which of the following sarcomas is inaccurately matched with its causative gene?
A. SS18-SSX1-Synovial sarcoma
B. COLA 1 -PDGFB- Dermatofibrosarcoma protuberans
C. EWS-FLI1 - Clear cell sarcoma
D. PAX3-F0X01 - Rhabdomyosarcoma
----------------------------------------
4. Which one of the following bone tumors is most common in the age of 15 years?
A. Ewing’s Sarcoma
B. Osteosarcoma
C. Osteoclastoma
D. Chondrosarcoma
----------------------------------------

1155
5. What is the likely diagnosis for a 7-year-old boy with a painful left thigh swelling for two months,
examination reveals an irregular tender swelling measuring 5 x 4cm in the anterior thigh compartment,
with a 'moth-eaten' destructive lesion on the femur on X-ray with an elevation of the overlying
periosteum giving an ‘onion-peel’ appearance, and histopathology indicating the following
characteristics?

A. Chondrosarcoma
B. Ewing sarcoma
C. Giant cell tumour
D. Aneurysmal bone cyst
----------------------------------------
6. What is the most likely diagnosis for a 40-year-old man presenting with weakness in arms and legs,
difficulty opening his hand after a handshake, difficulty extending his arm after flexing it, marked muscle
atrophy in limbs, ptosis, fixed facial expression, testicular atrophy, mild diabetes, and a muscle biopsy
showing atrophy of type I fibers, hypertrophy of type II fibers, and centrally located nuclei in numerous
fibers?
(or)
Which of the following is the most likely diagnosis in a patient with mild diabetes, a muscle biopsy
revealing the atrophy of type I fibres, hypertrophy of type II fibres, and numerous fibres with centrally
located nuclei?
A. Dermatomyositis
B. Duchenne muscular dystrophy
C. Nemaline myopathy
D. Myotonic dystrophy
----------------------------------------
7. Which sign is not relevant in the condition of a 16-year-old male presenting with swelling and pain in
his right knee which increases at night with no history of any significant trauma, a palpable mass over
the distal femur, and limited range of motion, as shown in the biopsy?

Page 2

1156
A. Codman's triangle
B. Sunburst appearance
C. Periosteal reaction
D. Chicken wire calcification
----------------------------------------
8. Which of the following mutations are commonly associated with enchondroma?
A. TP53 and RB1
B. Isocitrate dehydrogenase 1 (IDH 1) and Isocitrate dehydrogenase 2 (IDH 2)
C. BRCA1 and BRCA2
D. APC and beta-catenin
----------------------------------------
9. Which of the following responds the most to Denosumab therapy?
A. Ewing's sarcoma
B. Giant cell osteosarcoma
C. Osteochondroma
D. Fibrosarcoma
----------------------------------------
10. The formation of liposarcoma is likely due to the amplification of which gene?
A. EWS
B. MDM2
C. ABL
D. USP6
----------------------------------------
11. A 42-year-old female presents with a pigmented lesion on her right thigh, which was initially small
but has grown larger and darker in the last three months. On examination, the lesion is asymmetrical
with irregular borders and has a maximum diameter of 7mm. The biopsy and subsequent
histopathology reveal epitheloid cells having fine granules in the cytoplasm, invading the basement
membrane. Immunohistochemistry showed HMB45, S-100 positivity. Which of the following is the most
likely diagnosis?

Page 3

1157
A. Melanoma
B. Synovial sarcoma
C. Rhabdomyosarcoma
D. Neurofibroma
----------------------------------------
12. A 57-year-old male with a history of intermittent painful oral blisters on the lips and gingiva which
stay for a week, regress spontaneously, and then recur. Recently, he noticed similar lesions on his
torso and limbs. A biopsy of the affected area shows a fishnet pattern on immunofluorescence. Which
of the following findings does not correlate with this type of lesion?
(or)
Which finding does not correlate with the lesions described in a 57-year-old male presenting with
painful oral blisters that recur intermittently and recently appeared on his torso and limbs, showing a
fishnet pattern on immunofluorescence?
A. Antibodies against desmoglein 1
B. Antibodies against desmoglein 3
C. Antibodies against hemidesmosomes
D. Involvement of skin and mucous membranes both
----------------------------------------
13. A patient presents with skin lesions characterized by silvery scales and pink plaques. Upon removal
of the scales, pinpoint bleeds are observed. What clinical sign is demonstrated when the scales are
removed?
A. Nikolsky sign
B. Darier sign
C. Auspitz sign
D. Koebner phenomenon
----------------------------------------
14. What genetic mutation is primarily associated with Hailey-Hailey Disease (HHD), a rare genetic skin
disorder characterized by widespread keratinocyte acantholysis?
A. TP53 gene mutation
B. ATP2C1 gene mutation
C. COL7A1 gene mutation
D. FBN1 gene mutation
----------------------------------------
15. What is the underlying cause of Gorlin syndrome?
A. Mutations in the TP53 gene
B. Mutations in the PTCH1 gene
C. Mutations in the BRCA1 gene
D. Mutations in the APC gene

Page 4

1158
----------------------------------------

Correct Answers
Question Correct Answer

Question 1 1
Question 2 3
Question 3 3
Question 4 2
Question 5 2
Question 6 4
Question 7 4
Question 8 2
Question 9 2
Question 10 2
Question 11 1
Question 12 3
Question 13 3
Question 14 2
Question 15 2

Solution for Question 1:


Correct Option A - t(x;18):
• Synovial sarcoma, is a malignant soft tissue tumor classified as a tumor of uncertain origin.
• Synovial sarcoma is a misnomer because it does not necessarily involve the synovial joints.
• It can affect the upper or lower limbs, head, neck, trunk, and chest.
• Synovial sarcomas are characterized by characteristic chromosomal translocation (x;18)(p11;q11)
that creates fusion gene products SS18-SSX1, SS18-SSX2, or SS18-SSX4.
• The diagnosis of synovial sarcomas is made by biopsy and subsequent histopathological and
immunohistochemical examination.
• On histopathology, synovial sarcomas are either monophasic or biphasic.
• Monophasic tumors manifest uniform spindle cells with scant cytoplasm and dense chromatin growing
in short, tightly packed fascicles.
• Biphasic tumors show gland-like structures in addition to the spindle cells.
• On immunohistochemistry, these tumor stains positively with cytokeratins, EMA, BCL2, and CD-99.
Incorrect Options:
Option B - t(11;22):

Page 5

1159
• t(11;22) (q24;q12) is usually associated with Ewing sarcoma resulting in fusion gene product
EWS-FLI1.
• t(11;22)(p13;q12) is usually associated with the Desmoplastic small round-cell tumor resulting in
fusion gene product EWS-WT1.
Option C - t(12;22):
• t(12;22)(q13;q12) is usually associated with Clear-cell sarcoma resulting in fusion gene product
EWS-ATF1
Option D - t(9;22):
• t(9;22)(q22;q12) is usually associated with Extraskeletal myxoid chondrosarcoma resulting in fusion
gene product EWS-CHN.

Solution for Question 2:


Correct Option C - Gout:
• The patient has a history of recurrent episodes of toe pain with fever and now develops multiple
painless nodules on the pinna and right forearms.
• Gout flare is usually monoarticular and commonly involves joints of the lower limb such as the first
metatarsophalangeal joint of the big toe.
• The distinctive patterns of gout are (1) acute arthritis, (2) chronic tophaceous arthritis, (3)
accumulation of tophi in extraarticular sites, and (4) urate (gouty) nephropathy.
• The histopathological image of the gouty tophus, which is shown in the question, reveals a central
aggregate of urate crystals (not stained by H and E staining) that is surrounded by reactive fibroblasts,
mononuclear inflammatory cells, and giant cells.

• Gout is a condition characterized by acute or chronic destruction of joints resulting from the deposition
of uric acid.
• The recurrent episodes of acute gout eventually result in chronic gouty arthritis and the formation of
tophi.

Page 6

1160
• Tophi are pathognomonic for gout. Tophi are painless hard nodules that develop in the bones or soft
tissues. Soft tissue tophi usually develop in the following sites: Pinna of the external ear Tendon
sheaths (such as Achilles tendon) Synovial bursa
• Pinna of the external ear
• Tendon sheaths (such as Achilles tendon)
• Synovial bursa
• Bone tophi usually affect elbows, knees, and extensor surfaces of forearms.
• Pinna of the external ear
• Tendon sheaths (such as Achilles tendon)
• Synovial bursa
Incorrect Options:

Page 7

1161
Option A - Rheumatoid arthritis:
• Although rheumatoid arthritis can result in the development of painless subcutaneous swellings
(rheumatoid nodules), they do not show inflammatory reactions centered around the urate crystals.
• Patients with rheumatoid arthritis also present with symmetrical joint involvement unlike in gout.
Option B - Cysticercosis:
• Cysticercosis results from the infection of Taenia solium. The cysticerci can accumulate in the
subcutaneous tissues forming subcutaneous swellings, but a chronic history of joint pain and fevers is
unlikely. Moreover, these cysticerci are composed of cystic lesions with an invaginated scolex.
Option D - Chondrocalcinosis:
• Chondrocalcinosis is associated with calcium pyrophosphate crystal deposition that is amorphous and
basophilic on histology, unlike the unstained urate crystals seen in gout

Solution for Question 3:


Correct Option C - EWS-FLI1 - Clear cell sarcoma:
• Also Known As “melanoma of soft parts"
• Clear-cell sarcoma (CCS) is a rare soft tissue tumour that generally arises in the soft tissue of the
limbs.
• CCS often contains melanocytic properties and thus resembles malignant melanoma (MM). Despite
their shared properties, CCS can be distinguished from MM by the presence of a specific fusion
oncogene, EWS/ATF1, which results from a reciprocal translocation, t(12;22)(q13;q12)
• In clear cell sarcoma, there is a fusion between a gene called EWSR1 and a region called ATF1,
creating a fusion gene, i.e., EWSR1/ATF1.
• In the CCS of the GI tract, this can also happen with the gene CREB1 forming EWSR1/CREB1 fusion.
Incorrect Options:
Option A - SS18-SSX1-Synovial sarcoma:
• Synovial sarcoma is a highly aggressive mesenchymal cancer affecting adolescents and young
adults.
• It is caused by the fusion of oncoproteins SS18-SSX1, SS18-SSX2, or rarely, SS18-SSX4.
Option B - COLA 1 -PDGFB- Dermatofibrosarcoma protuberans:
• Dermatofibrosarcoma protuberans is caused by the sporadic translocation between chromosomes 17
and 22 (t:17;22).
• The COL1A1 gene on chromosome 17 gets relocated with the PDGFB gene on chromosome 22,
causing the disease.
Option D - PAX3-F0X01 - Rhabdomyosarcoma:
• In rhabdomyosarcoma, there is the translocation of a gene called PAX3 from chromosome 2 to
chromosome 13. This moves PAX3 adjacent to a gene known as FOXO1, forming a PAX/FOX01 fusion
gene.
• The PAX genes have an important role in embryonic muscle tissue cell growth and are usually shut
down once they're no longer needed.

Page 8

1162
• This translocation adjacent to the FOXO1 gene keeps the PAX gene active, leading to tumour
formation.

Solution for Question 4:


Correct Option B - Osteosarcoma:
• Osteosarcoma is a rare primary malignant bone tumor characterized by an osteoid matrix produced
by malignant cells.
• There is a bimodal age distribution for the incidence of osteosarcoma. Among children, the peak
incidence is between 13 and 16 years of age, which appears to coincide with the adolescent growth
spurt.
• Osteosarcoma also occurs at an earlier age in female children, corresponding to their more advanced
skeletal age and earlier adolescent growth spurt than male children.
• In adults, the peak incidence occurs over the age of 65.
• Among Children, osteosarcoma occurs more frequently in Black individuals and other ethnicities than
White individuals.
• Among adults, in contrast to children, osteosarcoma is more common in White individuals than in
Black individuals or individuals of other ethnicities.
• Tumours according to increasing age group Ewing’s Sarcoma Osteosarcoma Osteoclastoma
Chondrosarcoma
• Ewing’s Sarcoma
• Osteosarcoma
• Osteoclastoma
• Chondrosarcoma
• Ewing’s Sarcoma
• Osteosarcoma
• Osteoclastoma
• Chondrosarcoma
Incorrect Options:
Option A - Ewing’s Sarcoma:
• The Ewing sarcoma most often arises in the long bones of the extremities (predominantly the femur,
but also the tibia, fibula, and humerus) and the bones of the pelvis. The spine, hands, feet, and skull are
affected less often.
• Primary bone tumours are responsible for 6 percent of all childhood cancers. Although rare, Ewing
sarcoma is the 2nd most common primary bone malignancy affecting children and adolescents after
osteosarcoma.
Option C - Osteoclastoma:
• Giant cell tumor, also known as osteoclastoma, is a relatively rare, benign, locally aggressive
osteolytic skeletal neoplasm of young adults.

Page 9

1163
• It usually occurs after skeletal maturity, with a peak incidence in patients in their 20s and 30s and a
slight female predominance. The disease is rare before the age of 20. Patients who develop GCTB
before epiphyseal closure tend to have a higher incidence of vertebral primary tumors and
multicentricity than those who develop the lesion after skeletal maturity.
Option D - Chondrosarcoma:
• Chondrosarcomas are more common after 30 years of age.

Solution for Question 5:


Correct Option B - Ewing Sarcoma:
• The clinical presentation of a mass from the bony origin associated with soft tissue swelling, an
‘onion-peel’ appearance on plain radiograph and characteristic histopathological features is consistent
with Ewing sarcoma. The characteristic histopathological features of Ewing sarcoma include Sheets of
uniform, small, round, blue cells with hyperchromatic nuclei and scant cytoplasm,(as seen in the
image) which is clear because of the presence of abundant glycogen. Extensive necrosis, with
preservation of viable tumor around blood vessels. Nuclear pleomorphism, palisading, and the
formation of rosettes or pseudorosettes are typically absent. Invasion of blood vessels can be seen.
Mitotic figures are infrequent, and aneuploidy is uncommon in flow cytometry studies. The radiographic
appearance of an Ewing sarcoma includes
• The characteristic histopathological features of Ewing sarcoma include Sheets of uniform, small,
round, blue cells with hyperchromatic nuclei and scant cytoplasm,(as seen in the image) which is clear
because of the presence of abundant glycogen. Extensive necrosis, with preservation of viable tumor
around blood vessels. Nuclear pleomorphism, palisading, and the formation of rosettes or
pseudorosettes are typically absent. Invasion of blood vessels can be seen. Mitotic figures are
infrequent, and aneuploidy is uncommon in flow cytometry studies.

• Sheets of uniform, small, round, blue cells with hyperchromatic nuclei and scant cytoplasm,(as seen
in the image) which is clear because of the presence of abundant glycogen.
• Extensive necrosis, with preservation of viable tumor around blood vessels.
• Nuclear pleomorphism, palisading, and the formation of rosettes or pseudorosettes are typically
absent.
• Invasion of blood vessels can be seen.
• Mitotic figures are infrequent, and aneuploidy is uncommon in flow cytometry studies.
• The radiographic appearance of an Ewing sarcoma includes
• A series of finely destructive ‘moth-eaten’ lesions that become confluent over time, as shown.
• The characteristic histopathological features of Ewing sarcoma include Sheets of uniform, small,
round, blue cells with hyperchromatic nuclei and scant cytoplasm,(as seen in the image) which is clear
because of the presence of abundant glycogen. Extensive necrosis, with preservation of viable tumor
around blood vessels. Nuclear pleomorphism, palisading, and the formation of rosettes or
pseudorosettes are typically absent. Invasion of blood vessels can be seen. Mitotic figures are
infrequent, and aneuploidy is uncommon in flow cytometry studies.

Page 10

1164
• Sheets of uniform, small, round, blue cells with hyperchromatic nuclei and scant cytoplasm,(as seen
in the image) which is clear because of the presence of abundant glycogen.
• Extensive necrosis, with preservation of viable tumor around blood vessels.
• Nuclear pleomorphism, palisading, and the formation of rosettes or pseudorosettes are typically
absent.
• Invasion of blood vessels can be seen.
• Mitotic figures are infrequent, and aneuploidy is uncommon in flow cytometry studies.
• The radiographic appearance of an Ewing sarcoma includes

• Sheets of uniform, small, round, blue cells with hyperchromatic nuclei and scant cytoplasm,(as seen
in the image) which is clear because of the presence of abundant glycogen.
• Extensive necrosis, with preservation of viable tumor around blood vessels.
• Nuclear pleomorphism, palisading, and the formation of rosettes or pseudorosettes are typically
absent.
• Invasion of blood vessels can be seen.
• Mitotic figures are infrequent, and aneuploidy is uncommon in flow cytometry studies.

Page 11

1165
• The overlying periosteum is displaced or elevated due to the underlying tumour resulting in the clinical
signs of a Codman’s triangle. The characteristic periosteal reaction produces layers of reactive bone
deposited in an "onion peel" appearance.
• Ewing sarcomas tend to be large and, in long bones, are metaphyseal or diaphyseal in location.

Incorrect Options:
Option A - Chondrosarcoma:
• Chondrosarcomas are a heterogenous group of malignant bone tumors characterized by chondroid
(cartilaginous) matrix production. Chondrosarcomas are metaphyseal except clear cell
chondrosarcoma (epiphyseal) The radiographic findings include a mixed radiolucent and sclerotic
appearance, with the mineralized chondroid matrix appearing as a punctate or ring-and-arc pattern of
calcifications that may coalesce to form a more radiopaque flocculent pattern of calcification (the
so-called chondroid type of calcification) The histopathological features include low cellularity,
chondroid matrix, and absent mitoses.
• Chondrosarcomas are metaphyseal except clear cell chondrosarcoma (epiphyseal)

Page 12

1166
• The radiographic findings include a mixed radiolucent and sclerotic appearance, with the mineralized
chondroid matrix appearing as a punctate or ring-and-arc pattern of calcifications that may coalesce to
form a more radiopaque flocculent pattern of calcification (the so-called chondroid type of calcification)
• The histopathological features include low cellularity, chondroid matrix, and absent mitoses.
• Chondrosarcomas are metaphyseal except clear cell chondrosarcoma (epiphyseal)
• The radiographic findings include a mixed radiolucent and sclerotic appearance, with the mineralized
chondroid matrix appearing as a punctate or ring-and-arc pattern of calcifications that may coalesce to
form a more radiopaque flocculent pattern of calcification (the so-called chondroid type of calcification)
• The histopathological features include low cellularity, chondroid matrix, and absent mitoses.
Option C - Giant Cell Tumor:
• Giant cell tumours of bone are relatively rare, benign, but locally aggressive osteolytic skeletal
neoplasms of young adults. Histopathological findings include sheets of round to oval, polygonal or
elongated mononuclear cells that are interspersed with uniformly distributed, large osteoclast giant
cells. On a plain radiograph, a giant cell tumour of the bone appears as an expansile, eccentrically
placed lytic area resulting from intratumoral haemorrhage. The lesion normally involves the epiphysis
and the adjacent metaphysis.
• Histopathological findings include sheets of round to oval, polygonal or elongated mononuclear cells
that are interspersed with uniformly distributed, large osteoclast giant cells.
• On a plain radiograph, a giant cell tumour of the bone appears as an expansile, eccentrically placed
lytic area resulting from intratumoral haemorrhage. The lesion normally involves the epiphysis and the
adjacent metaphysis.
• Histopathological findings include sheets of round to oval, polygonal or elongated mononuclear cells
that are interspersed with uniformly distributed, large osteoclast giant cells.
• On a plain radiograph, a giant cell tumour of the bone appears as an expansile, eccentrically placed
lytic area resulting from intratumoral haemorrhage. The lesion normally involves the epiphysis and the
adjacent metaphysis.
Option D - Aneurysmal bone cyst:
• Aneurysmal bone cysts are nonmalignant expansile vascular lesions that consist of blood-filled
channels. They grow rapidly and destroy the bone.
• On radiographs, aneurysmal bone cysts appear as aggressive, expansile, lytic metaphyseal lesions
with an "eggshell" sclerotic rim. They may have a "soap bubble" appearance secondary to reinforcing
the remaining trabeculae that support the bone structure.

Solution for Question 6:


Correct Option D - Myotonic dystrophy:
• Based on the clinical features and biopsy findings, the most likely diagnosis is myotonic dystrophy.
• Myotonic dystrophy is an autosomal dominant disease characterized by the weakness of skeletal
muscles, associated with cataracts, endocrinopathy, and cardiomyopathy.
• Myotonic dystrophy usually results from expansions of CTG triplet repeats in the 3′-noncoding region
of the myotonic dystrophy protein kinase (DMPK) gene.

Page 13

1167
• The patient’s symptoms of weakness in extremities and muscle atrophy, along with an inability to
open his hand for a handshake or to extend his arm (referred to as myotonia), are suggestive of
myotonic dystrophy. The ptosis and a fixed facial expression are due to the weakness of the eyelid and
facial muscles, respectively.
• The patient also suffers from insulin resistance which is associated with myotonic dystrophy.
• The histopathological findings of myotonic dystrophy include type 1 myofibre atrophy with type 2
myofibres hypertrophy and numerous Fibres with centrally located nuclei.
Incorrect Options:
Option A - Dermatomyositis:
• Dermatomyositis can cause progressive muscle weakness. However, it also causes skin changes like
heliotrope rash, periorbital Oedema, and erythematous eruption or patches over the knuckles, elbows,
and knees (known as Gottron papules).
• Histopathological findings of dermatomyositis include infiltration of mononuclear inflammatory cells
and perifascicular atrophy.
Option B - Duchenne muscular dystrophy:
• Duchene muscular dystrophy is an X-linked progressive muscular dystrophy that usually presents in
the first decades of life. The histopathological findings comprise segmental myofibre degeneration and
regeneration associated with an admixture of atrophic myofibres. This is followed by fatty replacement
of the muscle tissue.
Option C - Nemaline myopathy:
• Nemaline myopathy is a congenital myopathy and causes mild focus weakness of skeletal muscles.
The pathological findings include threadlike structures (nemaline bodies) predominantly in type I fibres.

Solution for Question 7:


Correct Option D - Chicken wire calcification:
• The clinical presentation of a 16-year-old male with increasing pain and swelling in the right knee,
along with a palpable mass over the distal femur and limited range of motion, raises suspicion for a
bone tumor. The biopsy revealing malignant osteoids further supports the suspicion of a bone tumor,
specifically osteosarcoma.
• Nocturnal pain is a characteristic feature and is thought to result from increased intraosseous
pressure due to tumor growth, leading to ischemia and pain at night when the limb is at rest.

• The image (which shows malignant osteoids) suggests the possibility of osteosarcoma.
• Osteosarcoma is the most common primary malignant bone tumor, typically arising from the
metaphysis of long bones, especially around the knee joint in adolescents and young adults.

Page 14

1168
• Chicken wire calcification is not typically associated with osteosarcoma. This term is more commonly
linked to chondrosarcoma, another type of malignant bone tumor.
Incorrect Options:
Option A - Codman's triangle:
• Codman's triangle is a type of periosteal reaction,often associated with aggressive bone lesions and is
seen in osteosarcoma. It represents the elevation of the periosteum from the cortex due to the growing
tumor.
Option B - Sunburst appearance:
• The sunburst appearance on imaging, caused by the radiating spicules of bone, is characteristic of
osteosarcoma. It reflects the aggressive, invasive nature of the tumor.
Option C - Periosteal reaction:
• Periosteal reaction is commonly seen in osteosarcoma. It is a response of the periosteum to the
underlying tumor, and it can manifest as new bone formation.

Solution for Question 8:


Correct Option B - Isocitrate dehydrogenase 1 (IDH 1) and Isocitrate dehydrogenase 2 (IDH 2):
• Chondroma is a benign tumor of hyaline cartilage that occurs in bones of endochondral origin. Tumors
can arise within the medullary cavity, where they are termed enchondromas, or on the bone surface,
where they are called juxtacortical chondromas. Enchondromas are the most common intraosseous
cartilage tumor; often diagnosed between 20 and 50 years of age. They are typically solitary
metaphyseal lesions of tubular bones of the hands and feet.
• Enchondromas are benign cartilage tumors that commonly occur in the hands and feet. Isocitrate
dehydrogenase mutations, particularly IDH1 and IDH2, have been identified in enchondromas. These
mutations are involved in the abnormal accumulation of cartilage cells in these tumors. The presence of
mutations in IDH1 and IDH2 can be relevant in the diagnosis and understanding of enchondromas.
Incorrect Options:

Page 15

1169
Option A
- P53 gene mutation: P53 mutations are commonly associated with various cancers, but they are not a
characteristic mutation in enchondromas. Enchondromas are more closely linked to mutations in isocitr
ate dehydrogenase genes (IDH1 and IDH2).
Option C - BRCA1 gene mutation: BRCA1 mutations are typically associated with an increased risk of
breast and ovarian cancers. There is no established connection between BRCA1 mutations and encho
ndromas.
Option D - APC gene mutation: APC mutations are commonly associated with familial adenomatous po
lyposis (FAP) and colorectal cancer. There is no established link between APC mutations and enchond
romas.

Solution for Question 9:


Correct B - Giant cell osteosarcoma:
• Denosumab, an anti-RANKL drug, is primarily used for the treatment of Giant Cell Tumor (GCT) of
bone. Therefore, giant cell osteosarcoma, which also exhibits osteoclast-like giant cells, is expected to
respond most to Denosumab therapy.
Incorrect Options:
Option A - Ewing's Sarcoma: Ewing's sarcoma is not typically treated with Denosumab. The primary tre
atment for Ewing's sarcoma involves a
combination of surgery, chemotherapy, and sometimes radiation therapy.
Option C - Osteochondroma: Osteochondromas are benign bone tumors, and they do not typically req
uire Denosumab therapy. Treatment for osteochondromas may involve surgical removal if they cause s
ymptoms or complications.
Option D - Fibrosarcoma: Fibrosarcoma is a
malignant (cancerous) tumor of the connective tissue. Denosumab is not a
standard treatment for fibrosarcoma.

Solution for Question 10:


Correct Option B - MDM2:
• The amplification of MDM2 (Mouse Double Minute 2 homolog) gene is associated with the formation
of liposarcoma. MDM2 (p53 inhibitor) amplification is a characteristic genetic alteration in liposarcoma,
particularly in the well-differentiated and dedifferentiated subtypes. This amplification contributes to the
dysregulation of the p53 tumor suppressor pathway, promoting tumorigenesis.
Incorrect Options:
Option A - ETV6: ETV6 is not commonly associated with liposarcoma. ETV6-NTRK3 fusion gene is ass
ociated with Infantile fibrosarcoma
Option C - ABL: The ABL gene is associated with chronic myeloid leukemia (CML) when it forms the B
CR-ABL fusion gene due to the Philadelphia chromosome translocation (9;22). However, it is not linked
to liposarcoma.

Page 16

1170
Option D
- USP6: USP6 (Ubiquitin-Specific Peptidase 6) rearrangements are associated with nodular fasciitis, a
benign proliferative lesion of fibroblasts. It is not implicated in liposarcoma development.

Solution for Question 11:


Correct Option A - Melanoma:
• The patient’s clinical features and biopsy findings are diagnostic of melanoma.
• Melanoma is a malignant tumor that arises from melanocytes. It is the most lethal of all skin cancers
and is strongly linked to acquired mutations caused by exposure to UV radiation.
• The most frequent mutations in melanoma affect cell cycle control, pro-growth pathways, and
telomerase. Some of the more common mutations are as follows: Mutations that disrupt cell cycle
control genes. The CDKN2A Mutations that activate pro-growth signaling pathways. The PI3K/AKT
pathway
• Mutations that disrupt cell cycle control genes. The CDKN2A
• Mutations that activate pro-growth signaling pathways. The PI3K/AKT pathway
• The clinical features of melanoma that differentiate it from a benign lesion and are present in this
patient are:
• A-Asymmetry B- Border irregularity C- Colour variation D- Change of diameter > 6mm E- Evolution
• A-Asymmetry
• B- Border irregularity
• C- Colour variation
• D- Change of diameter > 6mm
• E- Evolution
• The histopathological picture manifests as large epitheloid cells with large nuclei having irregular
contours, chromatin that is characteristically clumped at the periphery of the nuclear membrane, and
prominent eosinophilic nucleoli. These cells invade the basement membrane.
• Immunohistochemistry is positive for HMB45 and S-100.
• Mutations that disrupt cell cycle control genes. The CDKN2A
• Mutations that activate pro-growth signaling pathways. The PI3K/AKT pathway
• A-Asymmetry
• B- Border irregularity
• C- Colour variation
• D- Change of diameter > 6mm
• E- Evolution
Incorrect Options:
Option B - Synovial sarcoma:

Page 17

1171
• Synovial sarcoma does not stain positive for HMB45 and S-100. Rather it stains positive for
cytokeratins, EMA, BCL2, and CD-99.
Option C - Rhabdomyosarcoma:
• Rhabdomyosarcoma is a malignant mesenchymal tumor of primitive skeletal muscle cells
(rhabdomyoblasts). The pathological findings reveal rhabdomyoblasts that stain positive for desmin,
MyoD1, and myogenin.
Option D - Neurofibroma:
• Neurofibroma is a benign nerve sheath tumor of peripheral nerves. It stains positive for S-100, CD-34,
and SOX10.

Solution for Question 12:


Correct Option C - Antibodies against hemidesmosomes:
• The clinical presentation of intermittent and recurring painful oral or mucosal blisters followed by
similar cutaneous lesions and a fish-net pattern on immunofluorescence is consistent with pemphigus
vulgaris. Desmoglein is a cadherin-like adhesion molecule that maintains tissue integrity and facilitates
cell-cell communication. The manifestations of pemphigus vulgaris are hypothesized to be induced by
the binding of the autoantibodies to epithelial cell surface antigens, both desmoglein 1 and desmoglein
3. Hemidesmosomes are multiprotein complexes that facilitate the stable adhesion of the basal
epithelial cells to the underlying basement membrane. Antibodies against hemidesmosomes are
characteristic and pathognomonic of bullous pemphigoid, not pemphigus vulgaris.
• Desmoglein is a cadherin-like adhesion molecule that maintains tissue integrity and facilitates cell-cell
communication.
• The manifestations of pemphigus vulgaris are hypothesized to be induced by the binding of the
autoantibodies to epithelial cell surface antigens, both desmoglein 1 and desmoglein 3.
• Hemidesmosomes are multiprotein complexes that facilitate the stable adhesion of the basal epithelial
cells to the underlying basement membrane.
• Antibodies against hemidesmosomes are characteristic and pathognomonic of bullous pemphigoid,
not pemphigus vulgaris.
• Desmoglein is a cadherin-like adhesion molecule that maintains tissue integrity and facilitates cell-cell
communication.
• The manifestations of pemphigus vulgaris are hypothesized to be induced by the binding of the
autoantibodies to epithelial cell surface antigens, both desmoglein 1 and desmoglein 3.
• Hemidesmosomes are multiprotein complexes that facilitate the stable adhesion of the basal epithelial
cells to the underlying basement membrane.
• Antibodies against hemidesmosomes are characteristic and pathognomonic of bullous pemphigoid,
not pemphigus vulgaris.
Incorrect Options:
Option A - Antibodies against desmoglein 1:
• IgG autoantibodies against desmoglein 1 are detected by ELISA and are strongly associated with
cutaneous pemphigus vulgaris and pemphigus foliaceus.
Option B - Antibodies against desmoglein 3:

Page 18

1172
• IgG autoantibodies against desmoglein 3 are detected by ELISA and are strongly associated with
mucosal pemphigus vulgaris.
Option D - Involvement of skin and mucosal membranes both:
• Pemphigus vulgaris is characterized by intermittent and recurrent painful mucosal blisters in the oral
cavity, followed by an aggressive cutaneous presentation. Very rarely, it presents as isolated mucosal
or cutaneous lesions only.

Solution for Question 13:


Correct Options C - Auspitz sign:
• This sign is specifically associated with psoriasis. When the characteristic silvery scales of psoriasis
are removed, pinpoint bleeding occurs due to the presence of tiny blood vessels in the dermal papillae.
Incorrect Options:
Option A - Nikolsky sign: This sign is associated with pemphigus vulgaris, a potentially serious autoim
mune skin disorder. It involves the separation of the epidermis (outer layer of the skin) from the underly
ing layers when slight pressure is applied.
Option B - Darier sign: This sign is observed in patients with urticaria pigmentosa, a
type of mastocytosis. When the skin is rubbed or scratched, it forms welts or hives.
Option D - Koebner phenomenon: This is a response observed in certain skin conditions, including pso
riasis, where new skin lesions develop at the site of trauma or injury to previously unaffected skin.

Solution for Question 14:


Correct Option B - ATP2C1 gene mutation:
• Hailey-Hailey Disease (HHD) is primarily linked to mutations in the ATP2C1 gene. The ATP2C1 gene
is involved in regulating a Ca2+-dependent signaling pathway, influencing cell-to-cell adhesion and
epidermal differentiation. Mutations in ATP2C1 lead to disruptions in calcium signaling, causing
widespread keratinocyte acantholysis in intertriginous areas.
Incorrect Options:
• The other options (A, C, D) are associated with different genetic conditions and are not specifically
linked to Hailey-Hailey Disease.

Solution for Question 15:


Correct Option B - Mutations in the PTCH1 gene:
• Gorlin syndrome is primarily caused by mutations in the PTCH1 gene. The PTCH1 gene encodes a
transmembrane receptor that recognizes sonic hedgehog signaling proteins. Mutations in this gene
lead to tumorigenesis, resulting in the development of multiple basal cell carcinomas (BCCs) and other
neoplasms.

Page 19

1173
• Gorlin syndrome: Feature Description Basal Cell Carcinomas (BCCs) Numerous BCCs, starting in
childhood. Translucent papules with telangiectasias or acrochordons (skin tags). Skeletal Abnormalities
Musculoskeletal anomalies in 60% to 75% of patients, including bifid or splayed ribs, frontal bossing,
and more. Ophthalmologic Abnormalities Hypertelorism, congenital blindness, cataracts, strabismus,
and colobomas of the retina or iris may occur. Odontogenic Keratocysts Pain, swelling, and drainage of
jaw cysts. Malignant transformation of jaw cysts reported. Mutation Mutations in the patched 1 (PTCH1)
gene on chromosome 9q.
Gorlin syndrome:
Feature
Description

Basal Cell Carcinomas (BCCs)


Numerous BCCs, starting in childhood. Translucent papules with telangiectasias or acrochordons (skin
tags).
Skeletal Abnormalities
Musculoskeletal anomalies in 60% to 75% of patients, including bifid or splayed ribs, frontal bossing, a
nd more.
Ophthalmologic Abnormalities
Hypertelorism, congenital blindness, cataracts, strabismus, and colobomas of the retina or iris may occ
ur.
Odontogenic Keratocysts
Pain, swelling, and drainage of jaw cysts. Malignant transformation of jaw cysts reported.
Mutation
Mutations in the patched 1 (PTCH1) gene on chromosome 9q.
Incorrect Options:
Option A - Mutations in the TP53 gene: While TP53 mutations are associated with various cancers, the
y are not the underlying cause of Gorlin syndrome.
Option C - Mutations in the BRCA1 gene: BRCA1 mutations are associated with an increased risk of br
east and ovarian cancers but are not linked to Gorlin syndrome.
Option D - Mutations in the APC gene: Mutations in the APC gene are associated with familial adenom
atous polyposis (FAP), a condition characterized by the development of numerous colorectal polyps. A
PC mutations are not related to Gorlin syndrome.

Page 20

1174
Previous Year Questions
1. What is the involvement of the following in the development of rheumatoid arthritis?
A. IgE mediated
B. Defective cellular and humoral immunity
C. Autoimmunity
D. Chronic microbial infections
----------------------------------------
2. Regarding this condition, which of the following statements is true? An 8-year-old boy is experiencing
progressive muscle weakness and difficulties with walking. During examination, pseudohypertrophy of
the calf muscles was observed.
A. Abnormal collagen
B. Absent dystrophin
C. Expansions of CTG triplet repeats
D. Defective fibrillin
----------------------------------------
3. A child presents with multiple skin lesions over the inner thigh and genital areas. It initially started as
macule and later developed to become nodules. Some nodules have central umbilication. The histology
of the lesion is shown below. What would be found on biopsy?

A. Multinucleate giant cells (Tzanck cells)


B. Donovan bodies
C. Henderson – Patterson bodies
D. Koilocytes
----------------------------------------
4. A woman presents with lesions on the inner thighs and peri – anal region. They are nodular, 4-6 mm
in size and appear pale. The histopathological image shows multiple intracytoplasmic inclusion bodies
consistent with Henderson – Patterson bodies. The diagnosis is:

1175
A. Trichodysplasia spinulosa
B. Condyloma acuminata
C. Molluscum contagiosum
D. Donovanosis
----------------------------------------
5. In terms of the sequence, how are the bands arranged in the Sarcomere from the outermost region
to the innermost region?
(or)
What is the order of the band in Sarcomere from the periphery to center?
A. Z-M-A-H
B. Z-H-M-A
C. Z-A-H-M
D. Z-H-A-M
----------------------------------------
6. Which of the following antibodies are specific for Scleroderma?
A. Anti centromere antibody
B. Anti La antibody
C. Anti Jo-1 antibody
D. Anti U1 RNP antibodies
----------------------------------------
7. What is the likely diagnosis based on the provided X-ray image, along with normal clinical
examination, hematological, and urinary test results, in a 65-year-old male patient?

Page 2

1176
A. Multiple myeloma
B. Langerhans cell histiocytosis
C. Metastasis
D. Hyperparathyroidism
----------------------------------------
8. What is the diagnosis for a woman who had a total knee replacement surgery and is now
experiencing knee pain due to implant loosening, as indicated by the histopathological slide showing
foreign body giant cells and the highlighted appearance in the image?

A. Ochronisis
B. Metallosis
C. Malignant melanoma
D. Rosal dorfiman disease
----------------------------------------
9. What glycoprotein is found in the highest concentration in basement membranes?
A. Fibronectin
B. Laminin
C. Integrin
D. Syndecan
----------------------------------------

Page 3

1177
10. A 36 year old male with skin – colored umbilicated papules. Microscopy is as shown below. The
most likely diagnosis is?

A. Herpes virus
B. Verruca vulgaris
C. Molluscum contagiosum
D. Lichen planus
----------------------------------------
11. Which of the subsequent choices is not classified as an intermediate filament?
A. Vimentin
B. Tubulin
C. Desmin
D. Neurofilament
----------------------------------------

Correct Answers
Question Correct Answer

Question 1 3
Question 2 2
Question 3 3
Question 4 3
Question 5 3
Question 6 1
Question 7 2
Question 8 2
Question 9 2
Question 10 3
Question 11 2

Page 4

1178
Solution for Question 1:
• The pathogenesis of rheumatoid arthritis starts with an antigenic exposure (like an infectious agent) in
an individual who is genetically predisposed, causing activation of CD4+ T-cells thus releasing
cytokines like TNF, IL-1,6, and interferon, which in turn activates endothelial cells, B lymphocytes, and
macrophages. Activation of B lymphocytes results in the release of IgM antibodies against IgG
(Rheumatoid factor).
• This immune complex triggers inflammatory damage to the blood vessels, synovium, and collagen.
Further activation of macrophages releases more cytokines that damage the joints and the blood
vessels of cartilages, termed pannus formation. As a result of the destruction of bone and cartilage,
fibrosis takes place, which produces visible joint deformities.
Incorrect Choices:
• Option a. IgE mediated: Rheumatoid arthritis is not IgE mediated. It is IgM mediated.
• Option b. Defective cellular and humoral immunity: The exact etiopathogenesis of rheumatoid arthritis
is unknown; however, certain genetic factors, environmental factors, and autoimmunity play a major
role in initiating inflammation.
• Option d. Chronic microbial infections: Certain microbial agents like viruses and mycoplasma have
been considered potential triggers of rheumatoid arthritis. Chronic infections will likely trigger the
immune system and lead to an autoimmune response. However, it has not been proven and would
require more studies to establish an exact relationship between rheumatoid arthritis and chronic
infections.

Solution for Question 2:


• The diagnosis in the given case scenario is Duchenne’s muscular dystrophy. It is an X-linked
recessive disorder. It is usually seen by 5 years of age. It occurs due to a mutation in the dystrophin
gene which leads to weakness initially in the proximal muscles, later involving the distal muscles.
Clinical features include pseudohypertrophy of calf muscles, waddling gait, and lumbar lordosis. Other
systems are involved causing cardiomegaly and reduced intelligence. Death usually occurs by 20 years
of age due to paralysis of the muscles of respiration.
Incorrect Choices:
• Option a. Abnormal collagen: Abnormal collagen is found in a few diseases like scurvy, Ehler danlos
syndrome, osteogenesis imperfecta, and Marfan syndrome. However, pseudohypertrophy of calf
muscles is a characteristic finding of Duchenne muscular dystrophy.
• Option c. Expansion of CAG triplet repeats: It is a genetic mutation that is usually seen in Huntington's
disease and, type 1 myotonic dystrophy, type 8 spinocerebellar ataxia. However, It is unrelated to
Duchenne muscular dystrophy.
• Option d. Defective fibrillin: Defective fibrillin is seen in Marfan's syndrome. These patients are tall with
long slender arms, long fingers, and toes, ocular lens dislocations, aortic dissection/aneurysm, and
scoliosis.

Solution for Question 3:

Page 5

1179
Correct Option C - Henderson – Patterson bodies:
• Henderson-Patterson bodies are large, eosinophilic, intracytoplasmic inclusion bodies that are
typically seen in molluscum contagiosum. These inclusion bodies represent viral particles and are often
surrounded by a halo or clear zone. They are named after the two scientists who first described them in
1952.
• Henderson-Patterson bodies are formed during the replication cycle of the poxvirus, which causes
molluscum contagiosum. The virus infects the skin cells and replicates inside them, resulting in the
formation of these inclusion bodies. These bodies are an important diagnostic feature of molluscum
contagiosum and can be visualized using light microscopy.
• It is worth noting that Henderson-Patterson bodies can also be seen in other poxvirus infections,
including smallpox (Variola virus) and monkeypox (Monkeypox virus). However, these infections are
rare and have been largely eradicated.
Incorrect Options:
Option A - Multinucleate giant cells (Tzanck cells): Multinucleate giant cells are characteristic of herpes
virus infections and can be seen in conditions like herpes simplex, herpes zoster, and varicella. Howev
er, they are not typically seen in molluscum contagiosum.
Option B - Donovan bodies: Donovan bodies are seen in infections caused by Klebsiella granulomatis,
which causes granuloma inguinale. This disease typically presents with painless ulcers in the genital ar
ea and is more common in tropical regions.
Option D - Koilocytes: Koilocytes are characteristic of human papillomavirus (HPV) infection and can b
e seen in conditions like genital warts and cervical dysplasia. However, they are not typically seen in m
olluscum contagiosum.

Solution for Question 4:


Correct Option C - Molluscum contagiosum:
• The clinical presentation of nodular, pale lesions on the inner thighs and peri-anal region is highly
suggestive of molluscum contagiosum. Henderson-Patterson bodies are eosinophilic inclusion bodies
that are pathognomonic for molluscum contagiosum virus infection. The virus infects the skin and
causes characteristic skin lesions that are often described as pearl-like, dome-shaped, and umbilicated.
The lesions are usually painless and self-limited but can persist for months to years. Treatment options
include cryotherapy, curettage, and topical or oral antiviral medications.

Incorrect Options:
Option A - Trichodysplasia spinulosa: Trichodysplasia spinulosa is a rare viral infection that affects imm
unocompromised individuals, especially those with organ transplants. It presents as follicular papules a
nd spiny nodules on the face, neck, and upper extremities. Histologically, it is characterized by hyperke
ratosis, follicular plugging, and hair shaft abnormalities. Henderson-Patterson bodies are not seen in tri
chodysplasia spinulosa.
Option B - Condyloma acuminata: Condyloma acuminata, also known as genital warts, is a sexually tra
nsmitted infection caused by human papillomavirus (HPV). It presents as flesh-colored, cauliflower-like
growths on the genital and perianal areas. Histologically, it is characterized by hyperkeratosis, acantho
sis, and koilocytosis (abnormal squamous cells with a
clear halo around the nucleus). Henderson-Patterson bodies are not seen in condyloma acuminata.

Page 6

1180
Option D - Donovanosis: Donovanosis, also known as granuloma inguinale, is a bacterial infection cau
sed by Klebsiella granulomatis. It presents as painless nodules or ulcers on the genital and perianal ar
eas. Histologically, it is characterized by granulomatous inflammation with Donovan bodies (large, intra
cytoplasmic, rod-shaped bacteria). Henderson-Patterson bodies are not seen in donovanosis.

Solution for Question 5:


The correct option is 3. Z-A-H-M.
Let's go through the order of the bands in the sarcomere from the periphery to the center:
Z-line (Z): The Z-line (also known as Z-disk or Z-band) marks the boundary of a
sarcomere. It is the periphery of the sarcomere and acts as an anchor for actin filaments.
A-band (A): The A-band is the region in the sarcomere where thick myosin filaments and overlapping t
hin actin filaments are present. It extends the entire length of the thick filaments.
H-zone (H): The H-zone is a region in the center of the sarcomere where only thick myosin filaments ar
e present. It is located between the overlapping actin filaments and shortens during muscle contraction
.
M-line (M): The M-line is the center of the sarcomere and contains proteins that stabilize the arrangem
ent of the thick myosin filaments.
Now, let's compare the options:
Z-M-A-H: This option is incorrect as it places the M-line before the A-band, which is not the correct ord
er.
Z-H-M-A: This option is incorrect as it places the H-zone before the A-band, which is not the correct or
der.
Z-A-H-M: This option is correct as it correctly orders the components of the sarcomere from the periph
ery to the center: Z-line, A-band, H-zone, and M-line.
Z-H-A-M: This option is incorrect as it places the H-zone before the A-band and the M-line before the H
-zone, which is not the correct order.
So, the correct option is 3. Z-A-H-M, which represents the correct order of the bands in the sarcomere f
rom the periphery to the center.

Solution for Question 6:


Correct Option: A: Anti-centromere:
• Anti-centromere antibody is specific for scleroderma, particularly for the limited cutaneous form, also
known as CREST syndrome (Calcinosis, Raynaud's phenomenon, Esophageal dysmotility,
Sclerodactyly, Telangiectasia).
• Scleroderma involves abnormal immune system activation, leading to inflammation and fibrosis of the
skin and internal organs. Anti-centromere antibodies suggest an autoimmune response targeting
centromere proteins, which are involved in chromosome segregation during cell division.
Incorrect options

Page 7

1181
Option B: Anti-La Antibody
• Anti-La antibody is not specific for scleroderma. It is associated with other autoimmune diseases like
Sjögren's syndrome, where it targets the La antigen found in the nucleus and cytoplasm of cells.
• Sjögren's syndrome is characterized by lymphocytic infiltration of exocrine glands, leading to dryness
of the eyes and mouth. The presence of anti-La antibodies contributes to the autoimmune attack on
these glands.
Option C: Anti-Jo-1 Antibody
• Anti-Jo-1 antibody is associated with dermatomyositis, not scleroderma. Dermatomyositis is an
autoimmune inflammatory muscle disease characterized by muscle weakness and skin manifestations,
such as a distinctive rash such as Gottron papules and Heliotrope rash
• Dermatomyositis involves inflammatory infiltrates in the muscles and skin. Anti-Jo-1 antibody targets
histidyl-tRNA synthetase, contributing to muscle inflammation and damage.
Option D:Anti-U1 RNP Antibody
• Anti-U1 RNP antibody is associated with mixed connective tissue disease (MCTD), not specifically
scleroderma.
• MCTD is characterized by overlapping features of several autoimmune diseases, including systemic
lupus erythematosus (SLE), scleroderma, polymyositis, and rheumatoid arthritis. Anti-U1 RNP
antibodies target components of the U1 small nuclear ribonucleoprotein particle, leading to immune
complex deposition and tissue damage.

Solution for Question 7:


Correct option B. Langerhans cell histiocytosis.
• Langerhans cell histiocytosis (LCH) is a rare disorder characterized by the proliferation and
accumulation of Langerhans cells, a type of immune cell, in various tissues. LCH can affect multiple
organs, including the bone, skin, and lymph nodes. In the context of the given X-ray showing a
punched-out lytic lesion in the cranium, LCH becomes a probable diagnosis.
Incorrect options:
Option A . Multiple myeloma: Multiple myeloma is a
type of blood cancer that involves the abnormal proliferation of plasma cells, a type of white blood cell,
in the bone marrow. Multiple myeloma is usually associated with abnormal hematological and urinary t
est results, which were reported as normal in this case.
Option C . Metastasis: Metastatic lesions can be seen in various organs, including the bone. However,
the X-ray findings alone cannot definitively indicate metastasis. A detailed clinical history and further in
vestigations, such as imaging studies and biopsy, are necessary to identify the primary tumor and confi
rm metastasis.
Option D . Hyperparathyroidism: Hyperparathyroidism is a condition characterized by excessive secreti
on of parathyroid hormone (PTH) by the parathyroid glands, leading to high levels of calcium in the blo
od. While hyperparathyroidism can cause bone changes, such as osteopenia or osteoporosis, the typic
al X-ray appearance associated with hyperparathyroidism is not a punched-out lytic lesion.

Page 8

1182
Solution for Question 8:
Correct Option B:
Metallosis refers to a condition characterized by the deposition of metallic debris, typically from implant
s or prosthetic devices, in the surrounding tissues. It occurs as a result of wear and corrosion of metalli
c implants, leading to the release of metal particles into the adjacent tissues. In this case, the histopath
ological slide showing foreign body giant cells along with the appearance of metallic debris is consisten
t with metallosis.
Metallosis can occur in various types of joint replacements, including total knee replacements. When th
e implants become loose or undergo mechanical wear, the metal components rub against each other, l
eading to the release of metal particles. These metal particles can trigger an inflammatory response, c
ausing the formation of foreign body giant cells, which are characteristic histopathological findings in m
etallosis.
Incorrect Options:
Option A. Ochronosis: Ochronosis is a
condition characterized by the deposition of homogentisic acid, a metabolic byproduct, in connective tis
sues. It typically presents with dark pigmentation of the tissues, particularly in the cartilage and skin. O
chronosis is not associated with the presence of foreign body giant cells or metallic debris.
Option C. Malignant melanoma: Malignant melanoma is a
type of skin cancer that arises from melanocytes. It typically presents as a pigmented lesion on the skin
and can metastasize to other sites. The histopathological features of malignant melanoma include aty
pical melanocytes, invasion of the dermis, and various architectural and cytological characteristics that
distinguish it from other conditions. The presence of foreign body giant cells and metallic debris is not i
ndicative of malignant melanoma.
Option D. Rosal dorfiman disease: Rosai-Dorfman disease, also known as sinus histiocytosis with mas
sive lymphadenopathy, is a rare disorder characterized by the proliferation of histiocytes, a type of imm
une cell, primarily in the lymph nodes. It typically presents with painless lymph node enlargement and c
an involve extranodal sites. The histopathological features of Rosai-Dorfman disease include the prese
nce of large histiocytes with abundant cytoplasm and engulfed lymphocytes. The findings in the given c
ase do not correspond to Rosai-Dorfman disease.

Solution for Question 9:


Correct option B
• Laminin is the most abundant glycoprotein found in basement membranes. It is a large protein
complex composed of three different polypeptide chains (α, β, and γ) that form a cross-shaped
structure. Laminin plays a crucial role in the structure and function of basement membranes, including
cell adhesion, migration, and signaling.
Incorrect options:
Option A: Fibronectin: Fibronectin is another glycoprotein present in the extracellular matrix, but it is no
t the most abundant glycoprotein in basement membranes.
Option C: Integrin: Integrins are cell surface receptors that mediate cell adhesion to the extracellular m
atrix but are not glycoproteins themselves.
Option D: Syndecan: Syndecan is a transmembrane heparan sulfate proteoglycan and is not the most
abundant glycoprotein in basement membranes.

Page 9

1183
Solution for Question 10:
Correct Option C:
Molluscum contagiosum is the correct answer. It is a viral skin infection caused by the molluscum cont
agiosum virus (MCV). Molluscum contagiosum presents as skin-coloured or pearly umbilicated papules
. Microscopically, it shows characteristic intracytoplasmic inclusions called molluscum bodies. These b
odies consist of aggregates of viral particles surrounded by host cell reaction. The given clinical presen
tation of skin-coloured umbilicated papules along with the provided microscopic image of molluscum bo
dies strongly suggests molluscum contagiosum.
Incorrect Option:
Option A: Herpes virus. Herpes virus infections typically present with grouped vesicles or ulcers and ar
e associated with a burning or tingling sensation. The microscopic appearance of herpes virus infection
would show multinucleated giant cells and intranuclear inclusions called Cowdry bodies. This descripti
on does not match the clinical and microscopic findings provided, so herpes virus is an incorrect option
.
Option B: Verruca vulgaris, also known as common warts, is caused by the human papillomavirus (HP
V). Verruca vulgaris presents as rough, hyperkeratotic papules with a characteristic cauliflower-like app
earance. Microscopically, verruca vulgaris would show hyperkeratosis, acanthosis, and presence of koi
locytes (enlarged keratinocytes with perinuclear clearing). The given clinical presentation and microsco
pic image do not correspond to verruca vulgaris, so this option is incorrect.
Option D: Lichen planus is a chronic inflammatory skin condition characterized by pruritic, polygonal, fl
at-topped papules. The microscopic findings in lichen planus include band-like lymphocytic infiltrate at t
he dermo-epidermal junction, known as sawtooth appearance. Lichen planus is not associated with the
clinical or microscopic features described in the case, so this option is incorrect.

Solution for Question 11:


Correct Option B.
Tubulin is not an intermediate filament. It is a protein that polymerizes to form microtubules, which are i
nvolved in various cellular processes such as intracellular transport, cell division, and maintenance of c
ell structure.
Incorrect Option:
Option A: Vimentin is an intermediate filament protein found in various types of cells, including mesenc
hymal cells, fibroblasts, and endothelial cells. It provides structural support to the cytoskeleton and help
s maintain cell shape and integrity.
Option C: Desmin is an intermediate filament protein primarily found in muscle cells, including skeletal,
cardiac, and smooth muscle cells. It provides structural support to muscle fibers and plays a
role in maintaining muscle integrity.
Option D: Neurofilament is an intermediate filament protein specifically found in neurons. It contributes
to the structural organization of neuronal axons and provides mechanical support to the axons, allowin
g for efficient signal transmission.

Page 10

1184
Therefore, the correct option is B, Tubulin, as it is not an intermediate filament protein. Intermediate fila
ments are a distinct type of cytoskeletal component, while tubulin is a
protein involved in the formation of microtubules.

Page 11

1185
Introduction to Hematology, RBC Inclusions and
Shapes
1. What sample is obtained using this instrument?

A. Bone Marrow Biopsy.


B. Lumbar Tap.
C. Bone Marrow Aspirate.
D. Tissue Biopsy.
----------------------------------------
2. Which of the following instruments is used for bone marrow biopsy?
A.
B.
C.
D.
----------------------------------------
3. A 45-year-old woman undergoes a bone marrow biopsy for diagnostic evaluation. Which of the
following statements accurately describes the components in processing this sample?
A. Fixation with 10% EDTA and decalcification with 10% NBF.
B. Both fixation and decalcification involve the use of 10% NBF.
C. Fixation with 10% NBF and decalcification with 10-15% EDTA
D. Fixation and decalcification are not required in bone marrow biopsy processing
----------------------------------------
4. A 15-year-old patient undergoes a bone marrow evaluation. The pathologist reports that the slide
shows 75% cellular component. Consider this information and choose the correct comment on this
finding.
A. Cellularity is normal in this case.
B. The slide shows less than appropriate cellularity.
C. The slide shows excessive cellularity due to underlying conditions.

1186
D. Additional information is required for an interpretation.
----------------------------------------
5. In G6PD deficiency, the characteristic Heinz bodies on peripheral blood smear are visualized using a
sub-type of which of the following stains?
A. Giemsa Stain
B. Romanowsky stain.
C. Periodic Acid-Schiff
D. Supravital stain
----------------------------------------
6. The ESR of a patient is being analyzed using both methods shown in the image below. Choose the
correct option regarding these methods.

A. The Westergren is a 110 mm tube.


B. The sample for Wintrobe should be collected in a red vacutainer.
C. The ideal anticoagulant for the Westergren method is EDTA.
D. Wintrobe’s Tube has bidirectional graduation.
----------------------------------------
7. Match the images to their respective causes: A. Post-splenectomy finding. 1. B. Megaloblastic
Anemia 2. C. α-Thalassemia 3. D. Hb-C 4.

A. Post-splenectomy finding. 1.

Page 2

1187
B. Megaloblastic Anemia 2.

C. α-Thalassemia 3.

D. Hb-C 4.

A. A-3, B-1, C-2, D-4


B. A-3, B-4, C-2, D-1
C. A-1, B-3, C-2, D-4
D. A-3, B-2, C-1, D-4
----------------------------------------
8. Match the types of cells with the appropriate peripheral smears they are seen in. A. Degmacytes 1.
B. Drepanocytes 2. C. Schistocytes 3. D. Codocytes 4.

Page 3

1188
A. Degmacytes 1.

B. Drepanocytes 2.

C. Schistocytes 3.

D. Codocytes 4.

A. A-1, B-3, C-4, D-2


B. A-2, B-4, C-3, D-1

Page 4

1189
C. A-2, B-3, C-4, D-1
D. A-3, B-2, C-4, D-1
----------------------------------------
9. Which of the following indices can be used to differentiate between microcytic, normocytic, and
macrocytic anemia?
A. Hematocrit
B. Hemoglobin
C. Mean corpuscular hemoglobin concentration [MCHC]
D. Mean corpuscular volume
----------------------------------------
10. What will be the rectified reticulocyte count in a patient with Hb of 5gms/dl and an absolute
reticulocyte count of 9%?
A. 4.5%
B. 6%
C. 1%
D. 3%
----------------------------------------
11. Where does hematopoiesis primarily occur during the initial three months of fetal development?
A. Yolk sac
B. Liver
C. Spleen
D. Bone marrow
----------------------------------------
12. In obese patients, which site is preferred for bone marrow aspiration?
A. Posterior Superior Iliac Spine (PSIS)
B. Anterior Superior Iliac Spine (ASIS)
C. Anteromedial surface of the Tibia
D. Sternum
----------------------------------------
13. Which of the following conditions is not associated with dry tap during bone marrow aspiration?
A. Aplastic Anemia
B. Myelofibrosis
C. Hairy cell Leukemia
D. Chronic Myeloid Leukemia (CML)
----------------------------------------

Page 5

1190
14. Which stage of erythropoiesis is characterized by the absence of a nucleus but the presence of
hemoglobin?
A. Pronormoblast
B. Early Normoblast
C. Reticulocyte
D. Late normoblast
----------------------------------------
15. What does the Reticulocyte Production Index (RPI) signify if the reticulocyte count is 8%,
hemoglobin is 9gm/dL, and the maturation time is 1.5 days?
A. Increased bone marrow activity
B. Normal bone marrow activity
C. Decreased bone marrow activity
D. No correlation with bone marrow activity
----------------------------------------
16. What occurs during Stage 2 of the Erythrocyte Sedimentation Rate (ESR)?
A. Rouleaux formation
B. Settling of red blood cell aggregates
C. Packing of red blood cell aggregates
D. Hemolysis of red blood cells
----------------------------------------
17. Match the following: 1 Mean corpuscular volume A Hb × 10/RBC 2 Mean corpuscular hemoglobin B
PCV × 10/RBC 3 Mean corpuscular hemoglobin concentration C Hb (100mg of blood) × 100/PVC
1 Mean corpuscular volume A Hb × 10/RBC
2 Mean corpuscular hemoglobin B PCV × 10/RBC
3 Mean corpuscular hemoglobin concentration C Hb (100mg of blood) × 100/PVC

A. 1-A, 2-C, 3-B


B. 1-B, 2-C, 3-A
C. 1-C, 2-A, 3-B
D. 1-B, 2-A, 3-C
----------------------------------------
18. What condition does Mean Corpuscular Hemoglobin Concentration (MCHC) of 45g/dL signify?
A. Hereditary spherocytosis
B. Thalassemia
C. Hemolytic anemia
D. Iron deficiency anemia
----------------------------------------

Page 6

1191
19. What does RDW (Red Cell Distribution Width) measure?
A. Variation in the size of all blood cells
B. Variation in the size of red blood cells
C. Size of red blood cells
D. Width of red blood cells
----------------------------------------
20. Mark the incorrect statement:
A. Heinz bodies can be stained by Romanowsky stain
B. Pappenheimer bodies are seen in sideroblastic anemia
C. Howell Jolly's body appears as a single dot
D. Howell Jolly bodies are seen in megaloblastic anemia
----------------------------------------

Correct Answers
Question Correct Answer

Question 1 3
Question 2 4
Question 3 3
Question 4 2
Question 5 4
Question 6 4
Question 7 1
Question 8 3
Question 9 4
Question 10 4
Question 11 1
Question 12 2
Question 13 4
Question 14 3
Question 15 1
Question 16 2
Question 17 4
Question 18 1
Question 19 2
Question 20 1

Page 7

1192
Solution for Question 1:
Correct Option C - Bone Marrow Aspirate:
• The instrument shown in the image is a Klima needle. The Klima needle is a specific type of needle
used for bone marrow aspiration. In medical procedures, bone marrow aspiration involves using a
needle to withdraw a liquid sample of the bone marrow from the marrow space.

• Needle is advanced into the marrow cavity with the stylet locked in place.
• The stylet is then removed, and a syringe is used to aspirate the marrow.
• Part Description Handle Provides a grip for the healthcare professional, designed for easy
manipulation and control. Needle Cannula Tubular portion inserted into the bone to reach the marrow
space; has a beveled or trocar tip for penetration. Stylet Solid, inner rod inside the needle cannula;
used during initial insertion, often removed before sample collection. Trocar Sharply pointed,
cone-shaped end on some Klima needles; assists in piercing the bone during initial penetration. Luer
Lock Connector Standard connector at the proximal end of the needle; allows easy attachment and
detachment of syringes or devices.

Part
Description
Handle
Provides a grip for the healthcare professional, designed for easy manipulation and control.
Needle Cannula
Tubular portion inserted into the bone to reach the marrow space; has a
beveled or trocar tip for penetration.
Stylet
Solid, inner rod inside the needle cannula; used during initial insertion, often removed before sample c
ollection.
Trocar

Page 8

1193
Sharply pointed, cone-shaped end on some Klima needles; assists in piercing the bone during initial pe
netration.
Luer Lock Connector
Standard connector at the proximal end of the needle; allows easy attachment and detachment of syrin
ges or devices.
Incorrect Options:
The Klima needle is not used for the procedures described in Options A, B, and D. Hence these option
s are incorrect.
Jamshidi needle is commonly used for bone marrow biopsy.

Solution for Question 2:


Correct Option D-

The Jamshidi needle is specifically designed for bone marrow biopsy procedures. It is a
specialized instrument used by healthcare professionals to obtain a
core biopsy sample from the bone marrow for diagnostic purposes.
Steps Involved in bone marrow aspiration:
Description
Insertion of the Jamshidi Needle
Insert the Jamshidi needle through the prepared skin and into the bone, using a
rotating motion for penetration. The trocar tip aids in initial bone penetration.
Removal of Stylet
Once the needle is appropriately positioned in the marrow space, remove the stylet. This allows for the
collection of the bone marrow biopsy sample.
Aspiration and Core Biopsy

Page 9

1194
Attach a syringe to the Luer lock connector and aspirate bone marrow fluid. Perform a
core biopsy by applying a rotating or twisting motion to the needle with gentle, steady pressure.
Needle Removal
Withdraw the Jamshidi needle carefully once the biopsy sample is obtained.
Pressure and Dressing
Apply pressure to the biopsy site to control bleeding. Dress the site with a sterile dressing.
Incorrect Options:
Option A -

Westerman Jensen Soft Tissue Biopsy Needle, luer lock fitting:


• Similar to the Franklin Modified Silverman needle, this needle is designed for soft tissue biopsies and
is not intended for bone marrow biopsy.
Option B -

Page 10

1195
Franklin Modified Silverman Soft Tissue Biopsy Needle:
• This needle is typically used for soft tissue biopsies and is not specifically designed for bone marrow
biopsy.
Option C -

• Sahal needle is mainly used for bone marrow aspiration and can also be used to obtain a bone
marrow biopsy but the Jamshidi needle is widely used. It would be a more appropriate answer in this
case.

Solution for Question 3:


Correct Option C - Fixation with 10% NBF and decalcification with 10-15% EDTA:
• This option reflects the standard approach in bone marrow biopsy processing where fixation is
typically performed with 10% neutral buffered formalin (NBF), and decalcification is achieved with
10-15% ethylenediaminetetraacetic acid (EDTA).
• STEPS INVOLVED IN BONE MARROW PROCESSING:

• Fixation preserves the cellular and structural components, while decalcification removes mineral
deposits from bone, allowing for better sectioning and analysis.

Page 11

1196
Incorrect Options:
Options A, B, and D are incorrect. Refer to the explanation of the correct answer.

Solution for Question 4:


Correct Option B - The slide shows less than appropriate cellularity:
• The term "cellularity" refers to the proportion of cellular elements in a tissue sample. In the context of
bone marrow evaluation, it reflects the ratio of cellular components (such as hematopoietic cells) to
non-cellular components (e.g., fat).
• Considering the formula “Ideal Cellularity = 100 – Age”. The ideal cellularity for a 15-year-old would be
85%.
Incorrect Options:
Options A, C, and D are incorrect. Refer to the explanation of the correct answer.

Solution for Question 5:


Correct Option D - Supravital stain:
• Heinz's body is a characteristic finding seen in G6PD deficiency. On Romanowsky stains the RBCs
appear as helmet/bite cells. This deficient area is stained by supravital stains demonstrating Heinz's
bodies.
• Heinz's body is demonstrated with Brilliant Cresyl Blue which is a supravital stain. It is also visualized
by crystal violet stain. The denatured hemoglobin picks up the supravital stain.

Page 12

1197
• Supravital Stains: New Methylene Blue and Brilliant Cresyl Blue.
Incorrect Options:
Options A, B and C are not commonly used to visualize Heinz bodies.

Solution for Question 6:


Correct Option D - Wintrobe’s Tube has bidirectional graduation:
• Westergren Method: Uses a 200 mm long tube. The ideal anticoagulant for the Westergren method is
sodium citrate, not EDTA.

• Uses a 200 mm long tube.
• The ideal anticoagulant for the Westergren method is sodium citrate, not EDTA.
• Wintrobe Method: Uses a tube that is shorter (around 110 mm) compared to the Westergren method.
The sample for Wintrobe is often collected in a blue vacutainer. Wintrobe's Tube has bidirectional
graduation, meaning that the scale is marked on both sides of the tube.

• Uses a tube that is shorter (around 110 mm) compared to the Westergren method.
• The sample for Wintrobe is often collected in a blue vacutainer.
• Wintrobe's Tube has bidirectional graduation, meaning that the scale is marked on both sides of the
tube.

• Uses a 200 mm long tube.
• The ideal anticoagulant for the Westergren method is sodium citrate, not EDTA.

Page 13

1198

• Uses a tube that is shorter (around 110 mm) compared to the Westergren method.
• The sample for Wintrobe is often collected in a blue vacutainer.
• Wintrobe's Tube has bidirectional graduation, meaning that the scale is marked on both sides of the
tube.

Solution for Question 7:


Correct Option A - A-3, B-1, C-2, D-4:
A. Post Splenectomy finding.

Page 14

1199
The image shows Howell-jolly bodies. Howell-Jolly bodies are small, round remnants of nuclear materi
al found in circulating erythrocytes. It is a
characteristic finding in megaloblastic anemia and in post-splenectomy patients.
3.

B. Megaloblastic Anemia
The image shows a Cabot ring. These are arginine-rich rings seen as a characteristic finding in megalo
blastic anemia. They take up different shapes like figures of 8, round, etc.
1.

C. α Thalassemia
The image shows golf ball inclusions, seen in thalassemia (3-α gene deletion variant). The stain used i
s the New Methylene Blue stain.
2.

Page 15

1200
D. Hb-C
The image shows Crystal forms (Hb-C), it is caused due to point mutation occurring at 5q6 subunit of h
emoglobin where glutamic acid is replaced by valine.
4.

Solution for Question 8:


Correct Option C - A-2, B-3, C-4, D-1:
A. Degmacytes

Page 16

1201
The image shows Degmacytes or bite cells (a portion of the cell margin is removed in a
bite-like fashion). They are also known as helmet cells, with a
portion of the cell membrane resembling a
helmet. They are seen in the Romanowsky stained slide of G6PD deficient patient.
1.

B. Drepanocytes
The image shows sickle cells or drepanocytes. Drepanocytes typically have a
characteristic crescent or sickle shape, resembling a curved or elongated structure with pointed ends.
The degree of curvature can vary, with some cells showing a
more pronounced sickle shape than others. They are typically seen in sickle cell anemia.
2.

C. Schistocytes.
The image shows Fragmented RBCs also known as schistocytes. Schistocytes have an irregular, frag
mented appearance rather than the smooth, biconcave shape of normal erythrocytes. They are seen in

Page 17

1202
Micro Angiopathic Hemolytic Anemia, Hemolytic Uremic Syndrome and Thrombotic thrombocytopenic
Purpura.
3.

D. Codocytes
The image shows, a few target cells or codocytes, having a Bull's-eye appearance with a
dark center and a surrounding ring. They are seen in thalassemia and sickle cell anemia.
4.

Solution for Question 9:


Correct Option D - Mean corpuscular volume:

Page 18

1203
• MCV can be used to determine if anemia is microcytic, normocytic or macrocytic.
Anemia is divided into three types based on MCV:
• Microcytic anemia, normocytic anemia and macrocytic anemia.
• If MCV is less than 80 fL, it is called microcytic anemia.
• If MCV is between 80 and 100 fL, it is called Normocytic anemia.
• If the MCV is more than 100 fL, it is a macrocytic anemic.
• Normocytic is further classified into hemolytic and non-hemolytic anemias, while hemolytic anemia is
further divided into extravascular hemolysis or intravascular hemolysis.
• Macrocytic anemia is divided into megaloblastic and non-megaloblastic anemia.
Incorrect Options:
Options A, B and C are incorrect. Refer to the explanation of the correct answer.

Solution for Question 10:


Correct Option D - 3%:
Data given :
• Patient Hb = 5mg/dl
• Patient HCT = Hb × 3

• Therefore, HCT= 5 × 3
• Resulting value of HCT = 15 L/L
But normal HCT = 45 L/L
• Corrected reticulocyte count = Reticulocyte % × (patient HCT/normal HCT)
• Corrected reticulocyte count = 9 % × (calculated HCT(15)/ normal HCT (45))
• Corrected reticulocyte count = 3 %
Incorrect Options:
Options A, B and C are incorrect. Refer to the explanation of Option D

Solution for Question 11:


Correct Option A- Yolk sac:
• Hematopoiesis initially occurs in the yolk sac during early fetal development, typically from 1 to 3
months of gestation.
Incorrect Options:
Option B- Liver: Hematopoiesis occurs in the liver during the third month of intrauterine life.

Page 19

1204
Option C- Spleen: Hematopoiesis does not occur in the spleen.
Option D- Bone marrow: Hematopoiesis occurs in the bone marrow after the fourth month of intrauterin
e life till puberty.

Solution for Question 12:


Correct Option B- Anterior Superior Iliac Spine (ASIS):
• In obese patients, accessing the Posterior Superior Iliac Spine (PSIS) for bone marrow aspiration or
biopsy may be challenging due to increased tissue depth. Therefore, the Anterior Superior Iliac Spine
(ASIS) is preferred for its accessibility and ability to obtain cellular marrow.
Incorrect Options:
Options A, C, and D: They are not preferred sites in obese patients, as explained above.

Solution for Question 13:


Correct Option D- Chronic Myeloid Leukemia (CML):
• Chronic Myeloid Leukemia (CML) typically does not present with a dry tap during bone marrow
aspiration. Dry tap is more commonly associated with conditions such as aplastic anemia,
myelofibrosis, hairy cell leukemia, and AML M7.
Incorrect Options:
Options A, B, and C: They are associated with dry tap.

Solution for Question 14:


Correct Option C- Reticulocyte:
• Reticulocytes are the stage of erythropoiesis where the nucleus is ejected. Reticulocytes contain
hemoglobin but lack a nucleus. It is also the first cell of erythropoiesis in the peripheral circulation.
Incorrect Options:
Options A, B, and D: Pronormoblasts, early and late normoblasts, contain a nucleus.

Solution for Question 15:


Correct Option A- Increased bone marrow activity:
RPI = CRC / Maturation Time
So,
CRC = RC × Hb of patient/ Hb of a normal person

Page 20

1205
CRC=8×9/15
CRC= 4.8
RPI= 4.8/1.5
RPI= 3.2
As RPI >3, it signifies increased bone marrow activity.
Incorrect Options:
Options B, C, and D: They are incorrect, as explained above.

Solution for Question 16:


Correct Option B- Settling of red blood cell aggregates:
• During Stage 2 of ESR, the small aggregates of red blood cells formed in Stage 1 settle down and
form a larger group, eventually settling at the bottom of the tube.
Incorrect Options:
Options A, C, and D: They do not occur in Stage 2 of ESR.

Solution for Question 17:


Correct Option D: 1-B, 2-A, 3-C
Incorrect Options:
Options A, B, and C: They are incorrectly matched.

Solution for Question 18:


Correct Option A- Hereditary spherocytosis:
• Mean Corpuscular Hemoglobin Concentration (MCHC) is a measure of the concentration of
hemoglobin in the entire hematocrit. An increased level of MCHC level, i.e., 45g/dL, is associated with
hereditary spherocytosis, a genetic disorder characterized by abnormal red blood cells that are
spherical in shape instead of the normal biconcave disc shape.
Incorrect Options:
Options B, C, and D: Thalassemia, Hemolytic anemia, and Iron deficiency anemia are not associated w
ith an increased MCHC.

Solution for Question 19:


Correct Option B- Variation in the size of all blood cells:

Page 21

1206
• RDW (Red Cell Distribution Width) measures the variation in the size of red blood cells. An elevated
RDW indicates anisocytosis, which means there is a wide range of sizes of red blood cells present.
Incorrect Options:
Options A, C, and D: They are incorrect, as explained above.

Solution for Question 20:


Correct Option A- Heinz bodies can be stained by Romanowsky stain:
• Heinz bodies are denatured hemoglobin molecules that occur in red blood cells due to oxidative
damage. They are typically seen in conditions such as G6PD deficiency or thalassemia. Heinz's bodies
cannot be stained by Romanowsky stains. Instead, they are best visualized using supravital stains such
as crystal violet.
Incorrect Options:
Options B, C, and D: They are correct statements.

Page 22

1207
Microcytic Hypochromic Anemia
1. A chronic alcoholic is brought to the emergency department, unconscious. His blood investigations
showed the following parameters: MCV 62fl, MCH 18 pg. His peripheral smear is shown below:
Consider the following options regarding the likely pathogenesis of his condition and pick the correct
option.

A. Mitochondrial poisoning
B. Vit B6 deficiency
C. Decreased serum iron
D. Low transferrin saturation.
----------------------------------------
2. Of the following choose the parameter that differentiates iron deficiency anemia from anemia of
chronic disease?
A. Serum iron
B. Serum transferrin saturation
C. Total Iron binding capacity
D. None of the above
----------------------------------------
3. The peripheral smear of a young female patient with iron deficiency anemia is shown below. Identify
the cell labeled in this smear.

A. Basophil

1208
B. Neutrophil
C. Eosinophil
D. Monocyte
----------------------------------------
4. Which of the following can lead to iron deficiency anemia?
A. Phenytoin toxicity
B. Fish tapeworm infection
C. Hookworm infection
D. Blind loop syndrome
----------------------------------------
5. Which of the following can be used to increase iron absorption in iron deficient patients?
A. Phytates
B. Tannates
C. Plant foods
D. Vitamin C
----------------------------------------
6. Which of the following responses describes the changes seen after iron replacement in patients with
iron deficiency anemia?
A. Reticulocytosis after 48- 72 hours of oral replacement therapy
B. Erythroid Hyperplasia after 12- 24 hours of oral replacement therapy
C. Repletion of stores after 12- 24 hours of oral replacement therapy
D. Subjective improvement and increased appetite after 12- 24 days of oral replacement therapy.
----------------------------------------
7. The complete blood count of a patient is shown below. Haemoglobin Mean corpuscular volume
Platelet count Leukocyte count RDW 10.3 g/dL 74 µm3 280,000/mm3 6,000/mm3 30 fl Which is the
most appropriate next step in evaluating this patient's illness?
HaemoglobinMean corpuscular volumePlatelet countLeukocyte countRDW 10.3 g/dL74 µm3280,000/mm36,000/mm330 fl

A. Haemoglobin electrophoresis
B. Ferritin levels
C. Serum TSH measurement
D. Vitamin B12levels
----------------------------------------
8. Which of the following plays a significant role in the pathogenesis of Anemia Of Chronic Disease?
A. IL5
B. IL6
C. IL12

Page 2

1209
D. IL4
----------------------------------------
9. Which laboratory findings would be expected in patients with Iron deficiency anemia (IDA)?
A. Hyperbilirubinemia
B. Increased serum ferritin
C. Low plasma transferrin saturation
D. Positive Direct Coombs test
----------------------------------------
10. What is the best parameter for assessing body iron stores?
A. Serum iron
B. Serum TIBC
C. Serum ferritin
D. Serum transferrin
----------------------------------------
11. Which of the following options best describes the pathogenesis of Anemia of Chronic Disease?
A. Clonal stem cell defect
B. Red blood cell membrane defects
C. Impaired utilization of iron from storage sites
D. Synthesis of structurally abnormal globin chains
----------------------------------------
12. A 4-year-old boy is referred to the medicine outpatient department for the evaluation of anemia. The
laboratory studies show an RBC count of 4.8(x1000000 per mm3) and MCV of 72 fl. What is the most
likely value of the Mentzer Index in this patient?
A. 12
B. 14
C. 11
D. 15
----------------------------------------
13. Which transporter is responsible for the absorption of iron into duodenal cells?
A. Ceruloplasmin
B. Divalent Metal Transporter-1
C. Ferroportin
D. Hephaestin
----------------------------------------
14. What is the role of hepcidin in iron regulation?
A. It promotes iron absorption by activating ferroportin

Page 3

1210
B. It inhibits iron absorption by inhibiting ferroportin
C. It promotes the release of iron from ferritin
D. It increases iron binding to transferrin
----------------------------------------
15. Which type of anemia is characterized by very high levels of serum iron, transferrin saturation, and
ferritin, along with low total iron binding capacity?
A. Sideroblastic anemia
B. Iron Deficiency Anemia (IDA)
C. Anemia of Chronic Disease (AOCD)
D. Both A and C
----------------------------------------
16. What are the characteristic blue-colored bodies seen in peripheral blood smears in patients with
sideroblastic anemia?
A. Heinz bodies
B. Pappenheimer bodies
C. Howell Jolly bodies
D. Ringed sideroblasts
----------------------------------------

Correct Answers
Question Correct Answer

Question 1 1
Question 2 3
Question 3 1
Question 4 3
Question 5 4
Question 6 1
Question 7 2
Question 8 2
Question 9 3
Question 10 3
Question 11 3
Question 12 4
Question 13 2
Question 14 2
Question 15 1

Page 4

1211
Question 16 2

Solution for Question 1:


Correct Option A - Mitochondrial poisoning:
Considering the lab findings in the scenario :
• The patient's mean corpuscular volume (MCV) is significantly decreased (62 fl), indicating microcytic
red blood cells. The mean corpuscular hemoglobin (MCH) is also decreased (18 pg),
indicating hypochromic RBCs

• Sideroblastic anemia in chronic alcoholics is often associated with mitochondrial poisoning,


particularly due to the toxic effects of alcohol on the mitochondria in the bone marrow. This toxicity can
impair heme synthesis, leading to the characteristic ringed sideroblasts seen in sideroblastic anemia.
The accumulation of iron in the mitochondria contributes to the formation of these ringed sideroblasts.
• Causes of sideroblastic anemia:
• Congenital causes: Due to enzyme deficiency.
• Acquired causes:
• Vit B6 deficiency
• Lead poisoning.
• Alcohol poisoning.
• Peripheral smear:
• RBC shows basophilic stippling.
• These stipplings are called “PappenHeimer Bodies”.
• The stain used is Perl’s/ Prussian Blue.

• Feature Sideroblastic Anaemia Serum iron Very high Transferrin Saturation Very high Ferritin Very
high TIBC Low
Causes of sideroblastic anemia:

Page 5

1212
Feature
Sideroblastic Anaemia
Serum iron
Very high
Transferrin Saturation
Ferritin
TIBC
Low
Incorrect Options:
Option B - Vit B6 deficiency: It is important to note that while vitamin B6 deficiency can contribute to Si
deroblastic anemia, it is not the primary cause of Sideroblastic anemia associated with chronic alcohol
consumption. Sideroblastic anemia in alcoholics is more directly linked to mitochondrial toxicity.
Option C - Decreased serum iron: Sideroblastic anemia is characterized by the presence of ringed side
roblasts in the bone marrow, indicating an excess of iron within the mitochondria of developing red bloo
d cells. In Sideroblastic anemia, rather than total body iron deficiency, there is often an increase in total
body iron, but it is not properly utilized in heme synthesis.
Option D - Low transferrin saturation: In sideroblastic anemia, there may be an increase in total body ir
on, but the problem lies in the inefficient use of iron in heme synthesis rather than a
low transferrin saturation. Therefore, low transferrin saturation is not a
characteristic feature of sideroblastic anemia.

Solution for Question 2:


Correct Option C - Total Iron binding capacity:
• In iron deficiency anemia, TIBC is typically elevated as the body attempts to compensate for low iron
levels by increasing the capacity to bind iron.

Page 6

1213
• In contrast, in anemia of chronic disease, TIBC may be normal or decreased due to the altered iron
metabolism associated with inflammation and the retention of iron within macrophages.
Incorrect Options:
Option A - Serum iron■■■■■■■:
• In iron deficiency anemia, serum iron levels are typically decreased due to the insufficient availability
of iron for erythropoiesis.
• In anemia of chronic disease, serum iron levels may also be decreased due to the sequestration of
iron within macrophages and decreased absorption, but this is not a consistent finding. Iron levels can
sometimes be normal or even elevated due to increased release of iron from stores.
Option B - Serum transferrin saturation■■■■■■■:
• In iron deficiency anemia, serum transferrin saturation is typically decreased due to the reduced
availability of iron for binding to transferrin.
• In anemia of chronic disease, serum transferrin saturation may be low-normal despite low serum iron
levels, reflecting the altered iron metabolism associated with inflammation.
Option D - None of the above■■■■■■■:
• This option is incorrect as option C is the correct answer.

Solution for Question 3:


Correct Option A - Basophil:
• The labeled cell in this patient’s smear is a basophil.
• Reactive basophilia is sometimes seen in iron deficiency anemia.
• Basophils are the least common granulocytes in the blood, making up less than 1% of all circulating
leukocytes.
• Basophils stain purple with dense blue cytoplasmic granules overlaying and almost obscuring the
nucleus.
• They have a role in allergic reactions and helminth infestations.
Incorrect Options:
Options B, C and D are incorrect. Refer to the explanation of the correct answer.

Solution for Question 4:


Correct Option C - Hookworm infection:
• Iron deficiency anemia presents with fatigue, conjunctival pallor, angular cheilitis, koilonychia,
decreased Haemoglobin, decreased hematocrit, decreased MCV, and increased RDW.
• The common laboratory findings in IDA include decreased ferritin, increased transferrin, decreased
transferrin saturation, and decreased mean corpuscular Haemoglobin (hypochromic anemia).

Page 7

1214
• Iron deficiency anemia results from depleted iron stores, either due to chronic loss (e.g., GI bleeding,
menorrhagia), inadequate intake, decreased absorption (e.g., atrophic gastritis, IBD), or increased
demand (e.g., during pregnancy).
• In children, hookworm infection can cause iron deficiency anemia due to chronic blood loss.
Incorrect Options:
Options A, B and D are associated with megaloblastic anemia.

Solution for Question 5:


Correct Option D - Vitamin C:
The concomitant use of vitamin C
increases the reabsorption of iron. Hence, prescribing ascorbic acid or vitamin C
in iron deficient patients is recommended to enhance the reabsorption of iron from the gut.

Incorrect Options:
Option A -Phytates:
• Phytates decrease iron absorption rather than increasing iron reabsorption.
• Therefore, this option is also ruled out
Option B - Tannates:
• Tannates (found in tea) decrease iron absorption rather than increasing iron reabsorption.
• Therefore, this option is also ruled out
Option C - Plant foods:
• Plant foods (non-heme iron) decrease iron absorption rather than increasing iron reabsorption.
• Therefore, this option is also ruled out

Solution for Question 6:


Correct Option A - Reticulocytosis after 12- 24 hours of oral replacement therapy:
• The response to iron replacement therapy includes: Initially, a rapid subjective improvement, with a
disappearance or marked reduction of fatigue, lassitude, and other nonspecific symptoms, before any
improvement in anaemia is observed. The earliest hematologic evidence of response to treatment is an
increase in reticulocytes on 48th to 72nd hours after supplementation therapy, the concentration of
reticulocytes peaks around 5-7 days. There is steady improvement in Haemoglobin and normalization
of red cell indices.
• Initially, a rapid subjective improvement, with a disappearance or marked reduction of fatigue,
lassitude, and other nonspecific symptoms, before any improvement in anaemia is observed.
• The earliest hematologic evidence of response to treatment is an increase in reticulocytes on 48th to
72nd hours after supplementation therapy, the concentration of reticulocytes peaks around 5-7 days.

Page 8

1215
• There is steady improvement in Haemoglobin and normalization of red cell indices.
• These responses and their respective timelines are summarized in the table below:
• Initially, a rapid subjective improvement, with a disappearance or marked reduction of fatigue,
lassitude, and other nonspecific symptoms, before any improvement in anaemia is observed.
• The earliest hematologic evidence of response to treatment is an increase in reticulocytes on 48th to
72nd hours after supplementation therapy, the concentration of reticulocytes peaks around 5-7 days.
• There is steady improvement in Haemoglobin and normalization of red cell indices.
Time after Iron Administration
Response
12-24 hr
Replacement of intracellular iron enzymes; subjective improvement; decreased irritability; increased ap
petite; increased serum iron
36-48 hr
Initial bone marrow response; erythroid hyperplasia
48-72 hr
Reticulocytosis, peaking at 5-7 days
4-30 days
Increase in hemoglobin level; increase in mean corpuscular volume; increase in ferritin
1-3 mon
Repletion of stores

Incorrect Options:
Option B - Erythroid Hyperplasia after 12- 24 hours of oral replacement therapy:
• Erythroid Hyperplasia after 36- 48 hours of oral replacement therapy,
• Therefore, this option is incorrect.
Option C - Repletion of stores after 12- 24 hours of oral replacement therapy:
• Repletion of stores occurs after 1- 3 months of oral replacement therapy.
• Therefore, this option is incorrect.
Option D
- Subjective improvement and increased appetite after 12- 24 days of oral replacement therapy:
• Subjective improvement and increased appetite after 12- 24 hours of oral replacement therapy
• Therefore, this option is incorrect.

Solution for Question 7:


Correct Option B - Ferritin levels:

Page 9

1216
• This patient is most likely suffering from microcytic anemia.
• Iron deficiency is the most common cause of microcytic anemia, especially in women, due to blood
loss during menstruation, which can gradually decrease the body's iron stores.
• Ferritin measurement is the first-line test for assessing the body's iron stores and screening for iron
deficiency anemia (IDA).
• The changes associated with IDA include: Decreased ferritin Increased transferrin Decreased
transferrin saturation Elevated RDW
• Decreased ferritin
• Increased transferrin
• Decreased transferrin saturation
• Elevated RDW
• Further workup would be needed to ascertain whether abnormal bleeding or malabsorption may be
causing the iron deficiency.
• The peripheral blood film shows anisocytosis, poikilocytosis, and small pale red blood cells.
• The treatment is oral iron replacement therapy.
• Decreased ferritin
• Increased transferrin
• Decreased transferrin saturation
• Elevated RDW

Incorrect Options:
Options A, C and D are incorrect. Refer to the explanation of the correct answer.

Solution for Question 8:


Correct Option B - IL6:
• In conditions such as chronic infections, autoimmune disorders, and malignancies, there is a
dysregulation of the immune system leading to increased production of IL-6.
• IL-6 stimulates the production of hepcidin, a key regulator of iron metabolism. Hepcidin acts by
inhibiting ferroportin, the protein responsible for exporting iron from cells into the bloodstream. This
inhibition of ferroportin leads to decreased iron absorption from the gut and decreased release of iron
from macrophages, resulting in decreased availability of iron for erythropoiesis.
• Additionally, IL-6 can directly suppress erythropoiesis by inhibiting the production of erythropoietin, the
hormone responsible for stimulating red blood cell production in the bone marrow.
Incorrect Options:
Options A, C and D are incorrect.

Page 10

1217
Solution for Question 9:
Correct Option C - Low plasma transferrin saturation:
• The earliest and specific parameter to be affected is serum ferritin.
• Serum ferritin is decreased in IDA.
• The other findings include the following: Decreased plasma transferrin saturation. ↑ TIBC ↓ Serum
iron levels Free erythrocyte protoporphyrin ↑↑↑ CBC: ↓Hb, MCH ↓, MCV ↓, MCHC ↓ ↑ Transferrin
levels
• Decreased plasma transferrin saturation.
• ↑ TIBC
• ↓ Serum iron levels
• Free erythrocyte protoporphyrin ↑↑↑
• CBC: ↓Hb, MCH ↓, MCV ↓, MCHC ↓
• ↑ Transferrin levels
• Transferrin: Transferrin is the carrier protein responsible for transporting iron in the blood. It binds to
iron ions and delivers them to cells and tissues throughout the body.
• Transferrin Saturation: Transferrin saturation is a measure of the amount of iron bound to transferrin
relative to the total iron-binding capacity of transferrin. It provides insight into the body's ability to
transport and deliver iron, with lower levels indicating reduced iron availability for binding and higher
levels suggesting adequate iron binding capacity.
• Decreased plasma transferrin saturation.
• ↑ TIBC
• ↓ Serum iron levels
• Free erythrocyte protoporphyrin ↑↑↑
• CBC: ↓Hb, MCH ↓, MCV ↓, MCHC ↓
• ↑ Transferrin levels
Transferrin: Transferrin is the carrier protein responsible for transporting iron in the blood. It binds to iro
n ions and delivers them to cells and tissues throughout the body.
Transferrin Saturation: Transferrin saturation is a measure of the amount of iron bound to transferrin rel
ative to the total iron-binding capacity of transferrin. It provides insight into the body's ability to transpor
t and deliver iron, with lower levels indicating reduced iron availability for binding and higher levels sug
gesting adequate iron binding capacity.

Incorrect Options:
Options A, B and D are incorrect.

Solution for Question 10:


Correct Option C - Serum ferritin:

Page 11

1218
• Serum ferritin is a protein complex responsible for iron storage.
• It is also an acute phase reactant that increases in response to systemic inflammation.
• Iron in the body is stored in the form of ferritin [ubiquitous protein-iron complex] at the highest levels in
macrophages of the liver, spleen, and bone marrow.
• Serum ferritin is the choice parameter for determining iron deficiency (decreased ferritin) and iron
overload (increased ferritin)

Incorrect Options:
Options A,B and ,D are incorrect, refer to option C

Solution for Question 11:


Option C - Impaired utilization of iron from storage sites:
• Anemia of chronic disease (ACD) is caused by decreased erythropoiesis despite adequate iron
stores.
• Anemia of chronic disease (ACD) is associated with chronic diseases as the name suggests. The
chronic disease process causes chronic inflammation with increased acute phase reactants such as
hepcidin and cytokines such as IL-6.
• The increase in hepcidin and IL-6 increases intracellular iron retention in macrophages and decreases
intestinal iron absorption.
• Consequently, serum iron levels and transferrin saturation decrease.
• Furthermore, the involved cytokines increase the transcription of acute-phase reactant ferritin,
resulting in high serum ferritin levels.
• Finally, these cytokines also inhibit erythropoiesis in the bone marrow, resulting in a low reticulocyte
count.
• The later phase of ACD may be accompanied by iron deficiency and manifest with microcytic anemia.

Incorrect Options:
Options A, B and D are incorrect and do not describe the correct underlying mechanism in patients wit
h anemia of chronic disease.

Solution for Question 12:


Correct Option D - 15:
• The Mentzer index is an excellent and sensitive tool to differentiate iron deficiency anemia from
beta-thalassemia.
• The Mentzer index is calculated by dividing the MCV by the RBC count.

Page 12

1219
• Thalassemia is said to be more likely if the mean corpuscular volume ratio to the red blood cell count
is lesser than 13.
• However, iron-deficiency anemia is more likely if the result is greater than 13.
• In this case, the Mentzer index is =72/4.8 = 15, suggesting that the patient is likely suffering from iron
deficiency anemia

Incorrect Options:
Options A, B, and C are incorrect. The correct option is explained above.

Solution for Question 13:


Correct Option B- Divalent Metal Transporter-1:
• Divalent Metal Transporter-1 (DMT1) is the transporter responsible for the absorption of iron into
duodenal cells. It is primarily located on the apical membrane of enterocytes in the duodenum. DMT1
facilitates the uptake of divalent metal ions, including iron, from the intestinal lumen into the
enterocytes.
Incorrect Options:
Options A, C, and D: They are not responsible for iron absorption into the duodenum.

Solution for Question 14:


Correct Option B- It inhibits iron absorption by inhibiting ferroportin:
• Hepcidin is the master regulator of iron metabolism. It primarily acts by inhibiting iron absorption by
inhibiting ferroportin, and it also inhibits the release of iron from ferritin.
Incorrect Options:
Options A, C, and D: They are not the roles of hepcidin.

Solution for Question 15:


Correct Option A- Sideroblastic anemia:
• Sideroblastic anemia is characterized by defective heme synthesis, resulting in excessive iron
accumulation in the mitochondria of erythroid precursors. This leads to increased levels of serum iron,
transferrin saturation, and ferritin due to iron overload. However, despite the high levels of iron, the total
iron binding capacity (TIBC) is low because the body's ability to bind and transport iron is impaired.
Incorrect Options:
Option B- Iron Deficiency Anemia (IDA):
• Iron deficiency anemia is characterized by low levels of serum iron, transferrin saturation, and ferritin,
along with high TIBC.

Page 13

1220
Option C- Anemia of Chronic Disease (AOCD):
• Anemia of chronic disease is characterized by low levels of serum iron and transferrin saturation, with
high ferritin levels and low TIBC.
Option D- Both A and C:
• Anemia of Chronic Disease does not correspond to the given iron profile.

Solution for Question 16:


Correct Option B- Pappenheimer bodies:
• Pappenheimer bodies are small, blue-staining, iron-containing granules that can be visualized in
peripheral blood smears stained with special stains like Perl's or Prussian blue. These bodies represent
iron deposits. The blue coloration results from the staining of iron within these granules.

Incorrect Options:
Option A- Heinz bodies:
• Heinz bodies are seen in G6PD deficiency, not in sideroblastic anemia.

Page 14

1221
Option C- Howell Jolly bodies:
• Howell Jolly bodies are seen in megaloblastic anemia.

Option D- Ringed sideroblasts:


• Ringed sideroblasts are characterized by iron present in the mitochondria, which lie in the perinuclear
region. They are a hallmark feature of sideroblastic anemia observed in bone marrow aspirate or
biopsy; they are not seen in peripheral blood smears.

Page 15

1222
Page 16

1223
Macrocytic Hypochromic Anemia
1. What is the most likely diagnosis based on the findings seen on this peripheral blood smear?

A. Megaloblastic anemia
B. Sickle cell disease
C. Iron deficiency anemia
D. Autoimmune hemolytic anemia
----------------------------------------
2. Of the following which is the most common site of folate absorption?
A. Ceacum
B. Jejunum
C. Duodenum
D. None of the above
----------------------------------------
3. A 60-year-old male patient comes to the physician for his annual physical exam. He reports no
complaints other than occasional fatigue. Physical examination shows conjunctival pallor. His
peripheral blood smear picture is given below. What is the most likely diagnosis for this patient?

A. Microcytic hypochromic anemia due to iron deficiency


B. Macrocytic anemia
C. Thalassemia

1224
D. Spherocytosis
----------------------------------------
4. Which of the following is most commonly associated with macrocytic anemia without megaloblastic
changes?
A. Alcoholism
B. Anemia of chronic disease
C. Iron deficiency
D. Renal disease
----------------------------------------
5. Which of the following organisms is associated with macrocytic hypochromic anemia?
A. Ancylostoma duodenale
B. Diphyllobothrium latum
C. Ascariasis lumdricoides
D. None of the above
----------------------------------------
6. Which of the following statements regarding FIGLU (Formiminoglutamic Acid) is true?
A. FIGLU is a marker for vitamin B12 deficiency.
B. FIGLU is a marker for folate deficiency.
C. FIGLU is a marker for iron deficiency.
D. FIGLU is a marker for vitamin D deficiency.
----------------------------------------
7. What are the Howell Jolly bodies remnants of?
A. DNA
B. RNA
C. Protein
D. Amino acid
----------------------------------------
8. Which receptor in the terminal ileum facilitates the absorption of vitamin B12 bound to intrinsic
factors?
A. Cubilin receptors
B. Gastric receptors
C. Enteroendocrine receptors
D. Parietal cell receptors
----------------------------------------
9. Vitamin B12 is essential to produce which nucleotide that is required for DNA synthesis and nuclear
maturation?

Page 2

1225
A. Adenine
B. Cytosine
C. Guanine
D. Thymidine
----------------------------------------
10. Which of the following antibodies in pernicious anemia inhibits the binding of vitamin B12 to intrinsic
factors?
A. Type 1
B. Type 2
C. Type 3
D. Type 4
----------------------------------------
11. Which type of cancer poses a notably increased risk in individuals with pernicious anemia?
A. Renal cell carcinoma
B. Pancreatic adenocarcinoma
C. Gastric adenocarcinoma
D. Prostate cancer
----------------------------------------
12. What is the myeloid/erythroid ratio likely to be observed in bone marrow with erythroid hyperplasia?
A. 1:5
B. 5:1
C. 1:3
D. 3:1
----------------------------------------
13. Which biochemical findings are likely to be seen in pernicious anemia? Decreased folic acid
Decreased homocysteine Increased serum bilirubin Anti-IF body present Decreased Methymalonyl
CoA
A. 1, 2 and 5
B. 1, 3 and 4
C. 1, 2, 4 and 5
D. All of the above
----------------------------------------
14. Which chromosome is primarily involved in controlling folate metabolism?
A. Chromosome 21
B. Chromosome 7
C. Chromosome 18

Page 3

1226
D. Chromosome 27
----------------------------------------

Correct Answers
Question Correct Answer

Question 1 1
Question 2 2
Question 3 2
Question 4 1
Question 5 2
Question 6 2
Question 7 1
Question 8 1
Question 9 4
Question 10 1
Question 11 3
Question 12 3
Question 13 2
Question 14 1

Solution for Question 1:


Correct Option A - Megaloblastic Anaemia:
Cabot Rings:
• The peripheral blood smear showing Cabot rings points towards a diagnosis of megaloblastic anemia.
• Megaloblastic anemia is caused by the deficiency of vitamin B12 or folic acid → defective DNA
synthesis, → megaloblasts.
• Cabot rings are thin, thread-like, red to violet, loop, or figure-of-eight-shaped inclusions found in
reticulocytes or sometimes megaloblasts.
• These rings are composed of nuclear material and are considered the remnants of mitotic spindles
that were mishandled during abnormal cell division.

Page 4

1227
Incorrect Options:
Options B, C and D are not associated with Cabot's ring on peripheral smear.

Solution for Question 2:


CORRECT OPTION
Option B- Jejunum :
• Folate is primarily absorbed in the jejunum, which is the middle part of the small intestine.
• The jejunum is well-equipped with villi and microvilli, providing a large surface area for efficient
absorption of nutrients, including folate.
• Folate absorption primarily occurs in the small intestine, particularly in the jejunum.
INCORRECT OPTIONS:
Options A, C and D are incorrect. Refer to the explanation of the correct answer.

Solution for Question 3:


Correct Option B - Macrocytic Anaemia:
• This patient has the signs and symptoms suggesting anemia.
• The peripheral smear findings of hypersegmented neutrophils and macro-ovalocytes → macrocytic
anemia.

Page 5

1228
• Macrocytic anemia or megaloblastic anemia is characterized by impairment of DNA synthesis leading
to ineffective hematopoiesis.
• This abnormal hematopoiesis results in characteristic morphologic changes, including abnormally
large erythroid precursors and red blood cells and the hypersegmented neutrophils as seen on this
patient's peripheral blood smear.
• Megaloblastic anemia is caused by folic acid or vitamin B12 deficiency.
• Vitamin B12 and folic acid are required as coenzymes for thymidylate synthesis, which is an essential
component for DNA synthesis.
• Defective DNA synthesis → delayed nuclear maturation → Macro polymorphonuclear cells with
nuclear hypersegmentation (five or more lobules instead of the normal three or four lobules).
Incorrect Options:
Option A - Microcytic hypochromic anemia due to iron deficiency:
• Iron deficiency initially causes normocytic, normochromic anemia and, if severe, causes microcytic
hypochromic anemia.
• The peripheral blood smear shows small red blood cells with central pallor and a thin peripheral rim of
hemoglobin (hypochromia).
Option C - Thalassemia:
• This elderly patient is less likely to have thalassemia as it is usually diagnosed early.
• The peripheral blood smear may show microcytic hypochromic red blood cells with occasional target
cells.
Option D - Spherocytosis:
• Spherocytosis is an inherited disorder characterized by defective red cell membranes resulting in
fragile red blood cells prone to hemolysis.
• It is characterized by jaundice and hepatomegaly, which are not seen in this patient.
• The peripheral blood smear shows spherocytes, which are small, hyperchromic cells without central
pallor.

Page 6

1229
Solution for Question 4:
Correct Option A - Alcoholism:
• Macrocytic anemia without megaloblastic changes can be seen in alcoholism, liver disease, and drugs
(e.g., 5-FU).
B. Option B -Anemiaof Chronic Disease:
• Anemia of chronic disease is associated with chronic inflammation (e.g., autoimmune conditions or
endocarditis) or cancer.
• It appears as microcytic anemia.
C. Option C - Iron Deficiency:
• Iron deficiency is a common cause of anemia.
• Iron deficiency causes microcytic anemia with a mean corpuscular volume of < 80 µm3.
Option D - Renal Disease:
• Renal disease can cause anemia of chronic disease, which is characterized by microcytosis.

Solution for Question 5:


Correct Option B - Diphyllobothrium latum:
• Diphyllobothrium latum, a fish tapeworm, can lead to macrocytic hypochromic anemia due to its ability
to compete with the host for vitamin B12 absorption in the intestine, leading to vitamin B12 deficiency.
• This deficiency can result in the production of large, immature red blood cells (macrocytes) with
decreased hemoglobin content (hypochromic).
Incorrect Options:

Page 7

1230
Option A- Ancylostoma duodenale (hookworm):
• Hookworm infection, caused by Ancylostoma duodenale, typically leads to microcytic hypochromic
anemia rather than macrocytic hypochromic anemia.
• This is because hookworms feed on blood in the small intestine, leading to chronic blood loss and iron
deficiency anemia characterized by small, pale red blood cells (microcytic) with decreased hemoglobin
content (hypochromic).
Option C - Ascariasis lumbricoides (roundworm):
• While ascariasis can lead to nutritional deficiencies and anemia, it does not typically result in the
characteristic features of macrocytic hypochromic anemia.
Option D - None of the above:
• This option is incorrect because Diphyllobothrium latum is associated with macrocytic hypochromic
anemia, as explained earlier.

Solution for Question 6:


Correct Option B - FIGLU is a marker for folate deficiency:
• FIGLU (Formiminoglutamic Acid) is a metabolite that accumulates in the urine in cases of folate
deficiency. When folate is deficient, the conversion of histidine to glutamate is impaired, leading to the
accumulation of FIGLU. Therefore, the presence of FIGLU in the urine serves as a marker for folate
deficiency.
Incorrect Options:
Options A, C and D are incorrect, and the correct option is explained above.

Solution for Question 7:


Correct Option A - DNA:
• Howell-Jolly bodies are RBC nuclei leftovers that are generally eliminated by the spleen.
• Therefore, they are noticed in individuals who have had a splenectomy or have functional asplenia
(e.g., from sickle cell disease).
• It is also seen in the following conditions. Amyloidosis Severe hemolytic anemia Megaloblastic anemia
Hereditary spherocytosis Myelodysplastic syndrome (MDS)
• Amyloidosis
• Severe hemolytic anemia
• Megaloblastic anemia
• Hereditary spherocytosis
• Myelodysplastic syndrome (MDS)
• Amyloidosis
• Severe hemolytic anemia

Page 8

1231
• Megaloblastic anemia
• Hereditary spherocytosis
• Myelodysplastic syndrome (MDS)
Incorrect Options:
Options B, C and D are incorrect. Refer to the explanation of Option A

Solution for Question 8:


Correct Option A- Cubilin receptors:
• Cubilin receptors are located in the terminal ileum and play a crucial role in the absorption of vitamin
B12 bound to intrinsic factors. Intrinsic factor is secreted by the parietal cells of the stomach and binds
to vitamin B12, forming a complex that travels to the terminal ileum for absorption. Cubilin receptors
recognize and bind to this complex, facilitating its absorption into the bloodstream.
Incorrect Options:
Options B, C, and D

Solution for Question 9:


Correct Option D-Thymidine:
• Vitamin B12 plays a crucial role in the production of thymidine, which is essential for DNA synthesis
and nuclear maturation. Thymidine is one of the four nucleotides that constitute DNA, and its synthesis
requires the involvement of Vitamin B12 in various biochemical pathways. Without sufficient Vitamin
B12, the synthesis of thymidine is impaired, leading to disruptions in DNA replication and cellular
maturation processes.
Incorrect Options:
Options A, B, and C: They are not produced with the help of vitamin B12.

Solution for Question 10:


Correct Option A- Type 1:
• In pernicious anemia, Type 1 antibodies inhibit the binding of vitamin B12 to intrinsic factors. These
antibodies interfere with the formation of the vitamin B12-intrinsic factor complex, which is necessary
for the absorption of vitamin B12 in the ileum. Consequently, this leads to vitamin B12 deficiency.
Incorrect Options:
Option B- Type 2: Type 2
antibodies inhibit the binding of the vitamin B12-intrinsic factor complex to ileal cubilin receptors.
Option C- Type 3: Type 3 antibodies are directed against parietal cells in the gastric mucosa, so the int
rinsic factor is not released.

Page 9

1232
Option D- Type 4: Type 4 antibodies do not exist.

Solution for Question 11:


Correct Option C- Gastric adenocarcinoma:
• Pernicious anemia involves autoimmune destruction of the gastric parietal cells by type 3 antibodies,
leading to a deficiency in intrinsic factor secretion, which is necessary for vitamin B12 absorption. This
condition can result in chronic atrophic gastritis, which is a known precursor to gastric adenocarcinoma.
Incorrect Options:
Options A, B, and D: They are not at increased risk in case of pernicious anemia.

Solution for Question 12:


Correct Option C-1:3:
• In bone marrow with erythroid hyperplasia, the myeloid/erythroid ratio is expected to be reversed
compared to normal(3:1). This means that there is a predominance of erythroid cells over myeloid cells.
Therefore, the M/E ratio is 1:3, indicating that there are three times more erythroid cells than myeloid
cells in the bone marrow.

Solution for Question 13:


Correct Option B- 1, 3 and 4:
1: Decreased folic acid: Pernicious anemia leads to a deficiency of both vitamin B12 and folic acid.
3: Increased serum bilirubin: In pernicious anemia, the destruction of red blood cells (hemolysis) may o
ccur, leading to an increase in serum bilirubin levels due to the breakdown of hemoglobin.
4: Anti-IF antibody present: Pernicious anemia is an autoimmune disorder characterized by the presen
ce of antibodies against intrinsic factor (IF) or parietal cells. These antibodies interfere with the absorpti
on of vitamin B12, contributing to the development of anemia.
Incorrect Options:
2: Decreased homocysteine: In pernicious anemia, there is impaired conversion of homocysteine to me
thionine due to the deficiency of vitamin B12. This leads to an accumulation of homocysteine in the blo
od, resulting in increased levels, not decreased.
5: Decreased Methylmalonyl CoA: Vitamin B12 is essential for the conversion of methylmalonyl CoA to
succinyl CoA. In pernicious anemia, where there is a deficiency of vitamin B12, this conversion is impa
ired, resulting in increased levels of methylmalonyl CoA, not decreased.

Solution for Question 14:

Page 10

1233
Correct Option A- Chromosome 21:
• Folate metabolism is primarily regulated by Chromosome 21. Deficiency in folate is evident in
alcoholics and pregnant women, manifesting similar symptoms to those of vitamin B12 deficiency,
excluding neurological manifestations.
Incorrect Options:
Options B, C, and D: They are incorrect, as explained above.

Page 11

1234
Hemolytic Anemias
1. Identify the type of RBC shown in the peripheral smear below

A. Macrocyte
B. Acanthocyte
C. Dacrocyte
D. Elliptocyte
----------------------------------------
2. Which of the following abnormalities is expected in a patient with hereditary spherocytosis?
A. Bilirubinuria
B. High mean corpuscular volume
C. Increased direct (conjugated) serum bilirubin
D. Polychromatophilic erythrocytes on peripheral blood smear
----------------------------------------
3. A 26-year-old male pilot presents with complaints of dizziness, palpitations, and blood in the urine.
He had just received primaquine chemoprophylaxis for Plasmodium vivax malaria. Physical
examination reveals signs of pallor and jaundice. Laboratory investigations reveal anemia with
hemoglobinuria. Special studies will likely reveal an abnormality in which of the following?
A. Duffy antigen
B. G6PD deficiency
C. Intrinsic factor
D. PIG-A
----------------------------------------
4. A 40-year-old male presented to the emergency department with complaints of excessive fatigue,
headache, mild abdominal pain, and dark-colored urine for three days. He also gives a history of
erectile dysfunction. CBC reveals a WBC count of 2.3 x109/L with hemoglobin at 10.2 grams/dL and a
platelet count of 135000 cells/mm3. Flow cytometry is shown below. Which type of acquired hemolytic
anemia best describes the above scenario?

1235
A. Paroxysmal nocturnal hemoglobinuria
B. Paroxysmal cold hemoglobinuria
C. Autoimmune hemolytic anemia
D. Microangiopathic hemolytic anemia
----------------------------------------
5. Which defects play a role in the pathophysiology of hereditary spherocytosis?
A. DNA synthesis
B. Erythrocyte cytoskeleton
C. Erythrocyte maturation
D. Enzyme Deficiency
----------------------------------------
6. A 17-year-old female patient presented to the hematology clinic with fatigue, shortness of breath, and
yellowing of eyes and skin. The patient had a history of viral infection three weeks ago. Physical
examination reveals icterus, and splenomegaly. A complete blood count (CBC) reveals moderate
normocytic anemia. The peripheral blood smear is shown in the image. Which of the following is the
most likely diagnosis in this patient?

A. Hereditary elliptocytosis
B. Sickle cell disease
C. Glucose-6- phosphate dehydrogenase deficiency
D. Hemoglobin C (HbC) disease

Page 2

1236
----------------------------------------
7. Which is the most severe variant of G6PD deficiency?
A. G6PD-B
B. G6PD-
C. G6PD Mediterranean
D. None of the above
----------------------------------------
8. Which of the following stains can be used to detect Heinz bodies in patients with G6PD deficiency?
A. Giemsa stain
B. Brilliant cresyl blue
C. Leishman
D. Wright
----------------------------------------
9. Which of the following conditions is most likely associated with cold agglutinin hemolytic anemia?
A. Systemic lupus erythematosus
B. Post Epstein-Barr virus infection
C. Chronic lymphocytic leukemia
D. Babesiosis
----------------------------------------
10. Cold agglutinin antibodies act against which of the following RBC antigens?
A. P antigen
B. I antigen
C. Le antigen
D. Rh antigen
----------------------------------------
11. Which of the following cell markers exhibit decreased activity/absence in patients with paroxysmal
nocturnal hemoglobinuria?
A. CD45 and CD59
B. CD45 and CD55
C. CD55 and CD59
D. None of the above
----------------------------------------
12. Which of the following diseases is associated with warm autoimmune hemolytic anemia?
A. Mycoplasma pneumonia infection
B. Waldenström macroglobulinemia
C. Systemic lupus erythematosus

Page 3

1237
D. EBV infection
----------------------------------------
13. A middle aged male pateient with a history of working in a benzene factory presents with complaints
of fatigue. His bone marrow biopsy is shown in the image. Which of the following is the most likely
underlying mechanism in the development of this patient’s anemia?

A. Damage to stem cells


B. Decreased erythropoietin production by the kidneys
C. Folate deficiency
D. Impaired globin chain synthesis
----------------------------------------
14. Evan's syndrome involves which of the following type of hemolytic anemia?
A. Cold IgG auto immune hemolytic anemia
B. Cold IgM auto immune hemolytic anemia
C. Warm Autoimmune Hemolytic Anemia
D. None of the above
----------------------------------------
15. Which of the following infections may be implicated in the cause of pure red cell aplasia?
A. Flavivirus
B. Herpes Simplex virus
C. Cytomegalovirus
D. Parvovirus
----------------------------------------
16. What type of antibody is Donath-Landsteiner antibody ?
A. IgE
B. IgM
C. IgG
D. None of the above
----------------------------------------

Page 4

1238
17. Which of the following is not caused by an extra-corpuscular defect?
A. Autoimmune hemolytic anemia
B. March hemoglobinuria
C. Microangiopathic hemolytic anemia
D. Paroxysmal nocturnal hemoglobinuria
----------------------------------------
18. Which of the following tests is used as a confirmatory test for Hereditary Spherocytosis (HS)?
A. Osmotic fragility test using Pink's solution
B. EMA (Eosin 5' Maleimide) test
C. Direct Coombs test
D. Schilling test
----------------------------------------
19. Paroxysmal cold hemoglobinuria results in:
A. Intravascular hemolysis
B. Extravascular hemolysis
C. Both intravascular and extravascular hemolysis
D. No hemolysis
----------------------------------------
20. Which investigation is considered the gold standard for diagnosing Aplastic Anemia?
A. Peripheral blood smear
B. Bone marrow aspirate
C. Hemoglobin Electrophoresis
D. Bone marrow biopsy
----------------------------------------

Correct Answers
Question Correct Answer

Question 1 2
Question 2 4
Question 3 2
Question 4 1
Question 5 2
Question 6 1
Question 7 3
Question 8 2

Page 5

1239
Question 9 2
Question 10 2
Question 11 3
Question 12 3
Question 13 1
Question 14 3
Question 15 4
Question 16 3
Question 17 4
Question 18 2
Question 19 1
Question 20 4

Solution for Question 1:


Correct Option B - Acanthocyte:
• Acanthocytes or spur cells are spiculated red cells with projections of varying size and surface
distribution (irregularly spiculated cells).

• Acanthocytes are usually seen in abetalipoproteinemia, neuroacanthocytosis, and chronic liver
disease.
• Abetalipoproteinemia (ABL) is characterized by an inability to secrete triglyceride-rich lipoproteins
failing to absorb essential fatty acids that leads to deficiencies of fat-soluble vitamins and lipid defects in
plasma membranes, often producing acantholytic red blood cells in peripheral blood smear.

Incorrect Options:

Page 6

1240
Option A - Macrocyte:
• Macrocytes are abnormal red blood cells with overly increased size and are usually formed due to
intrinsic deficiencies of vitamin B12 or folate.
• Other causes of macrocytic anemia are liver disease, hypothyroidism, and immunosuppressive drugs.
Option C - Dacrocyte:
• It is a type of poikilocyte shaped like a teardrop. Red blood cells are deformed in this way when
mechanically squeezed out from the marrow due to an infiltrative process.
• These teardrop cells are usually found in diseases with bone marrow fibrosis, like myelofibrosis,
myelodysplastic disorders, acute leukemias, and autoimmune diseases.
Option D - Elliptocyte:
• Elliptocytes are oval-shaped elongated erythrocytes, sometimes referred to as pencil cells.
• They are commonly seen in hereditary elliptocytosis, megaloblastic anemia, sideroblastic anemia,
severe iron deficiency anemia, etc.

Solution for Question 2:


Correct Option D - Polychromatophilic erythrocytes on peripheral blood smear:
• Hereditary spherocytosis (HS) is primarily caused by defects in erythrocyte cytoskeleton proteins,
particularly spectrin, ankyrin, band 3 protein, and other structural proteins. These defects lead to a loss
of membrane stability and deformability, resulting in the characteristic spherical shape of affected red
blood cells. This altered morphology makes them more susceptible to trapping and destruction in the
spleen, leading to hemolytic anemia.
• A family history of a similar condition with resultant splenectomy is an important ecological marker of
hereditary spherocytosis.

Page 7

1241
• The peripheral blood smear shows the presence of spherocytes which can be found in hereditary
spherocytosis as well as warm-mediated autoimmune haemolytic anaemia. However, negative direct
Coombs test helps differentiates it from warm-mediated hemolytic anemia.

• Polychromatophilic erythrocytes are nucleated, immature erythrocytes that are an expected finding in
any hemolytic anemia.

Incorrect Options:
Option A - Bilirubinuria:
• In hereditary spherocytosis, the level of unconjugated, not conjugated bilirubin rises, but due to its
hydrophobic nature, it cannot be excreted in the urine resulting in no bilirubinuria.
Option B - High mean corpuscular volume:
• Mean corpuscular volume is used to help classify anemia based on red cell morphology.
• In hereditary spherocytosis, the MCV is normal due to the spheroidal shape of RBCs.
Option C - Increased direct (conjugated) serum bilirubin:
• An expected finding in hereditary spherocytosis is an increase in indirect (unconjugated) serum
bilirubin, not direct (conjugated).
• This is because of the increased catabolic degradation of hemoglobin due to rapid hemolysis.

Solution for Question 3:


Correct Option B - G6PD deficiency:
• This is a case of glucose-6-phosphate dehydrogenase deficiency
• The signs and symptoms indicate an ongoing hemolytic crisis due to oxidative stress caused by the
antimalarial drug primaquine.
• Investigation results reveal anemia with hemoglobinuria, which further establishes the diagnosis.

Page 8

1242
• Special investigations include the fluorescent spot test for the screening of individuals with a family
history and spectrophotometric assay of GPD activity for the confirmation of diagnosis.
• G6PD deficiency due to genetic mutations results in reduced glutathione activity of antioxidation which
leads to RBCs being more susceptible to damage when triggered by oxidative stress. This results in
denatured hemoglobin and intravascular hemolysis.
• Conditions resulting in oxidative stress are infections like pneumonia, sepsis, antimalarial drugs,
antituberculosis drugs, sulfonamides, aspirin, nitrofurantoin, and some foods like fava beans.
Incorrect Options:
Options A, C, and D are incorrect. Refer to the explanation of the correct answer.

Solution for Question 4:


Correct Option A - Paroxysmal nocturnal hemoglobinuria:
• This is a case of paroxysmal nocturnal hemoglobinuria
• The flow cytometry results showing the absence of CD55 and CD59 are crucial for diagnosing
Paroxysmal Nocturnal Hemoglobinuria (PNH).
• PNH is the only acquired cause of intra-corpuscular hemolysis caused by a defect in the PIGA gene
leading to dysfunctional red cell membrane protein.
CD55 and CD59
• CD55 (DAF): Function: CD55 regulates the activation of the complement system by accelerating the
decay of C3 and C5 convertases, thus preventing the formation of the membrane attack complex
(MAC). Role in PNH: In PNH, mutations in the PIG-A gene result in deficient or absent expression of
CD55 on the surface of affected red blood cells (RBCs). Without CD55, RBCs are unable to regulate
complement activation effectively, making them more susceptible to complement-mediated lysis.
• Function: CD55 regulates the activation of the complement system by accelerating the decay of C3
and C5 convertases, thus preventing the formation of the membrane attack complex (MAC).
• Role in PNH: In PNH, mutations in the PIG-A gene result in deficient or absent expression of CD55 on
the surface of affected red blood cells (RBCs). Without CD55, RBCs are unable to regulate
complement activation effectively, making them more susceptible to complement-mediated lysis.
• CD59 (MACIF): Function: CD59 inhibits the formation of the membrane attack complex (MAC) by
binding to the C5b-8 complex and preventing the incorporation of C9, thereby blocking the formation of
the pore-forming complex that leads to cell lysis. Role in PNH: Similar to CD55, mutations in the PIG-A
gene lead to deficient or absent expression of CD59 on the surface of PNH RBCs. This deficiency
allows unregulated assembly of the MAC on RBC membranes, resulting in increased susceptibility to
complement-mediated hemolysis, a hallmark feature of PNH.
• Function: CD59 inhibits the formation of the membrane attack complex (MAC) by binding to the C5b-8
complex and preventing the incorporation of C9, thereby blocking the formation of the pore-forming
complex that leads to cell lysis.
• Role in PNH: Similar to CD55, mutations in the PIG-A gene lead to deficient or absent expression of
CD59 on the surface of PNH RBCs. This deficiency allows unregulated assembly of the MAC on RBC
membranes, resulting in increased susceptibility to complement-mediated hemolysis, a hallmark
feature of PNH.
CD55 (DAF):

Page 9

1243
• Function: CD55 regulates the activation of the complement system by accelerating the decay of C3
and C5 convertases, thus preventing the formation of the membrane attack complex (MAC).
• Role in PNH: In PNH, mutations in the PIG-A gene result in deficient or absent expression of CD55 on
the surface of affected red blood cells (RBCs). Without CD55, RBCs are unable to regulate
complement activation effectively, making them more susceptible to complement-mediated lysis.
CD59 (MACIF):
• Function: CD59 inhibits the formation of the membrane attack complex (MAC) by binding to the C5b-8
complex and preventing the incorporation of C9, thereby blocking the formation of the pore-forming
complex that leads to cell lysis.
• Role in PNH: Similar to CD55, mutations in the PIG-A gene lead to deficient or absent expression of
CD59 on the surface of PNH RBCs. This deficiency allows unregulated assembly of the MAC on RBC
membranes, resulting in increased susceptibility to complement-mediated hemolysis, a hallmark
feature of PNH.
Incorrect Options:
Option B - Paroxysmal cold hemoglobinuria:
• The absence of cold exposure-related symptoms and the flow cytometry findings of CD55 and CD59
deficiency do not support a diagnosis of PCH.
Option C - Autoimmune hemolytic anemia:
• While AIHA can also present with hemolysis and anemia, the absence of autoantibodies and the
presence of CD55 and CD59 deficiency on flow cytometry make AIHA less likely.
Option D - Microangiopathic hemolytic anemia:
• Microangiopathic hemolytic anemia (MAHA) typically presents with schistocytes on peripheral blood
smear, indicative of mechanical destruction of RBCs within small blood vessels. Since no mention of
schistocytes is made in the scenario, and given the flow cytometry findings, MAHA is not the most
appropriate diagnosis.

Solution for Question 5:


Correct Option B - Erythrocyte cytoskeleton:
• Hereditary spherocytosis (HS) is primarily caused by defects in erythrocyte cytoskeleton proteins,
particularly spectrin, ankyrin, band 3 protein, and other structural proteins. These defects lead to a loss
of membrane stability and deformability, resulting in the characteristic spherical shape of affected red
blood cells. This altered morphology makes them more susceptible to trapping and destruction in the
spleen, leading to hemolytic anemia.
• Clinical findings include splenomegaly, pigmented gall stones, aplastic anemia, etc.
• Peripheral smear shows small round RBCs with a lack of central pallor.
• Anisocytosis and several hyperchromic spherocytes are seen.
• Howell-Jolly bodies can also be seen.
• There is a decreased surface area of the RBCs
• MCHC is usually increased. Premature removal of RBCs by the spleen leads to extravascular
hemolysis.

Page 10

1244
• The osmotic fragility test measures erythrocyte resistance to hemolysis when exposed to hypotonic
saline solutions.

• Decreased mean fluorescence of RBCs in the EMA binding test.

Incorrect Options:
Option A - DNA synthesis:
• DNA synthesis is defective in vitamin B12 deficiency or folate deficiency
• They usually present as macrocytic anemia.
Option C - Erythrocyte maturation:
• Abnormal erythroid maturation is not the cause of spherocytes.
Option D - Enzyme Deficiency:
• G6PD deficiency is an example of an enzyme deficiency causing anemia
• It is characterized by the presents of cells and Heinz bodies on smear.

Solution for Question 6:


Correct Option A - Hereditary elliptocytosis:
• This is a case of hereditary elliptocytosis (HE)
• Hereditary elliptocytosis is also known as ovalocytosis
• HE is an inherited disorder associated with a defect in the erythrocyte cytoskeleton; the most
commonly involved defect is abnormal polymerization of spectrin molecules.
• HE Variants include a defect in the self-assembly of spectrin, spectrin-ankyrin binding, protein 4.1,
and glycoprotein C.

Page 11

1245
• Diagnosis: Blood smear: A blood smear shows the elliptical shape of red blood cells. Osmotic fragility
test and direct protein assaying by gel electrophoresis can also be used for diagnosis.
• Blood smear: A blood smear shows the elliptical shape of red blood cells.
• Osmotic fragility test and direct protein assaying by gel electrophoresis can also be used for
diagnosis.
• Blood smear: A blood smear shows the elliptical shape of red blood cells.
• Osmotic fragility test and direct protein assaying by gel electrophoresis can also be used for
diagnosis.
Incorrect Options:
Option B - Sickle cell disease:
• SCD leads to both extravascular and intravascular hemolysis.
• Blood smears reveal sickle cells and target cells.
Option C - Glucose-6- phosphate dehydrogenase (G6PD) deficiency:
• G6PD deficiency led to a decreased synthesis of nicotinamide adenine dinucleotide phosphate
(NADPH).
• A decrease in NADPH leads to a decrease in reduced glutathione, which increases RBC susceptibility
to oxidants.
• On blood smear RBC with Heinz bodies and bite cells are visible.
Option D - Hemoglobin C (HbC) disease:
• Hemoglobin C disease is caused by a change in amino acids: glutamic acid to lysine, in the beta chain
of hemoglobin.
• HbC disease leads to extravascular hemolysis in inpatients.
• Blood smear in HBC reveals Hb crystals inside red blood cells and target cells.

Solution for Question 7:


Correct Option C - G6PD Mediterranean:
• The half-life of G6PD-Mediterranean is more markedly abnormal compared to the other
variants, hence making it the most severe variant of G6PD deficiency anemia.
• G6PD Variant Prevalence Severity G6PD B Common normal variant Not associated with hemolysis
G6PD- About 10% of American blacks Moderately reduced half-life G6PD Mediterranean Prevalent in
Middle Eastern populations Markedly abnormal half-life
Incorrect Options:
Options A, B and D are incorrect. correct option is explained above.

Solution for Question 8:


Correct Option B - Brilliant cresyl blue:

Page 12

1246
• Heinz bodies are typically difficult to visualize with Romanowsky dyes and may appear as pink-brown
nodular structures. They stain dark purple-blue and are better visualized using supravital stains.
• Supravital dyes include new methylene blue, brilliant cresyl blue, crystal violet, methyl violet, Nile blue,
and Hoechst stain.

Incorrect Options:
Options A, C and D are not typically used to stain Heinz bodies.

Solution for Question 9:


Correct Option B - Post Epstein-Barr virus (EBV) infection:
• Cold agglutinin hemolytic anemia is mostly idiopathic. In some cases, Mycoplasma pneumonia, EBV
infection, non-Hodgkin lymphoma, and Waldenstrom macroglobulinemia can be important etiological
factors for cold AIHA.
• Cold-sensitive antibodies, mostly IgM antibodies, bind to RBCs at cold ambient temperatures of
0–4°C.
• They can cause abnormal complement activation leading to acute intravascular and extravascular
hemolysis.
• Raynaud phenomenon results from IgM binding to red cells in vascular beds with temperatures below
30°C (e.g., exposed fingers, toes, and ears).
Incorrect Options:
Options A, C and D are associated with warm antibody mediated hemolytic anemia.

Solution for Question 10:


Correct Option B - I antigen:

Page 13

1247
• Autoantibodies in cold agglutinin disease are mostly IgM antibodies that act against the I antigen of
RBCs by attaching to it at cold temperatures resulting in clumping/ agglutination, which leads to
acrocyanosis and extravascular hemolysis.
Incorrect Options:
Option A - P antigen:
• The Donath-Landsteiner antibody is responsible for complement-mediated hemolysis in paroxysmal
cold hemoglobinuria, a rare form of AHA in adults, which is characterized by recurrent episodes of
massive hemolysis following cold exposure.
• It is an IgG antibody that binds to the P Antigen of RBCs at 40 C resulting in complement activation
and intravascular hemolysis in paroxysmal cold hemoglobinuria.
Options C, and D have no role in cold agglutinin disease.

Solution for Question 11:


Correct Option C - CD55 and CD59■■■:
• CD55 (also known as Decay Accelerating Factor or DAF) and CD59 (also known as Membrane Attack
Complex Inhibitor or MAC-IP) are cell surface proteins that regulate the complement system. They
prevent the formation of the membrane attack complex (MAC), which is a part of the body's immune
response that can lead to the lysis of red blood cells (hemolysis).
• CD55 and CD59 deficiency renders red blood cells susceptible to complement-mediated lysis, leading
to intravascular hemolysis, as evidenced by the presence of hemoglobinuria in patients with
paroxysmal nocturnal hemoglobinuria.
Incorrect Options:
Options A, B and D are incorrect, refer to the explanation of the correct answer.

Solution for Question 12:


Correct Option C - Systemic lupus erythematosus (SLE):
• Warm autoimmune hemolytic anemia (AIHA) is primarily associated with autoimmune disorders such
as systemic lupus erythematosus (SLE). In SLE, autoantibodies attack self antigens including
P-antigen on red blood cells.
• This autoimmune reaction leads to the production of autoantibodies, particularly IgG antibodies, that
bind to red blood cells at body temperature (hence the term "warm" AIHA). The binding of these
antibodies to red blood cells causes their destruction by the immune system, resulting in anemia.
Incorrect Options:
Options A, B, and D are usually associated with cold auto-immune hemolytic anemia.

Solution for Question 13:

Page 14

1248
Correct Option A - Damage to stem cells:
• Patients with aplastic anemia present with severe pancytopenia and clinical symptoms related to
various cytopenias, including fatigue (anemia), fever (neutropenia), and petechiae (thrombocytopenia).
• This is caused by damage to bone marrow stem cells which is caused by a toxic environmental factor
in this case, as the patient has a history of working in a benzene factory.
• The bone marrow biopsy, seen in the above picture, is mainly aplastic, consisting largely of fat cells
and fibrous stroma & lacking normal hematopoietic activity suggesting the diagnosis of aplastic anemia.
• Damage to the stem cells due to prolonged benzene exposure usually results in suppression of
erythropoiesis by abnormal mitotic cells and apoptosis likely mediated by expanded production of
inhibitory cytokines and nitric oxide.
Incorrect Options:
Option B - Decreased erythropoietin production by the kidneys:
• This is usually seen in anemia due to any chronic inflammatory, infectious, or malignant disease.
Option C - Folate deficiency:
• The peripheral blood smear in patients with folate deficiency shows macrocytic RBCs and
hyper-segmented neutrophils.
Option D - Impaired globin chain synthesis:
• Impaired or absent globin chain synthesis in beta-thalassemia results in excessive unbound globin
chains that can precipitate in mature and immature RBCs leading to ineffective erythropoiesis and
hemolysis.
• Peripheral smear shows microcytic, hypochromic, and nucleated RBCs, target cells, and poikilocytes.

Solution for Question 14:


Correct Option C - Warm Autoimmune Hemolytic Anemia:
• Evan's syndrome is characterized by the simultaneous or sequential presence of autoimmune
hemolytic anemia (AIHA) and immune thrombocytopenic purpura (ITP), both of which are mediated by
autoantibodies against red blood cells and platelets, respectively.
• In Evan's syndrome, warm autoimmune hemolytic anemia is typically the type of hemolytic anemia
observed.
INCORRECT OPTIONS:
Options A, B, and C are incorrect and the correct option is explained above.

Solution for Question 15:


Correct Option D - Parvovirus:
• Pure red cell aplasia is a marrow failure due to erythroid precursor suppression.

Page 15

1249
• Outside of cases associated with B19 parvovirus infections (that infect and destroy RBC precursors),
the etiology is likely autoimmune.
• It can be associated with drug exposures, autoimmune diseases, and neoplasms (e.g., large granular
lymphocytic leukemia or thymoma).
• Diagnostic criteria for pure red cell aplasia are: Giant normoblasts in bone marrow Severe anaemia
Reticulocyte count < 1% Normocellular marrow with mature erythroblasts <0.5%.
• Giant normoblasts in bone marrow
• Severe anaemia
• Reticulocyte count < 1%
• Normocellular marrow with mature erythroblasts <0.5%.
• The major difference between PRCA and aplastic anemia is that, in PRCA, only the red blood cell line
is affected, while the white blood cells and platelets remain at normal levels. In aplastic anemia, all
three blood cell types are typically affected.
• Giant normoblasts in bone marrow
• Severe anaemia
• Reticulocyte count < 1%
• Normocellular marrow with mature erythroblasts <0.5%.
Incorrect Options:
Options A, B, and C are incorrect. Refer to the explanation of Option A

Solution for Question 16:


Correct Option C - IgG:
• The Donath-Landsteiner antibody is an IgG antibody. This antibody is unique in that it is responsible
for the pathogenesis of paroxysmal cold hemoglobinuria (PCH), a type of autoimmune hemolytic
anemia.
• Unlike warm autoimmune hemolytic anemia (AIHA), which is typically mediated by IgG antibodies,
PCH is characterized by the presence of IgG antibodies that bind to red blood cells in cold
temperatures (0-4°C), leading to their destruction upon rewarming in the central circulation.
• This phenomenon causes episodic hemoglobinuria, often accompanied by symptoms like abdominal
pain and fatigue.
Incorrect Options:
Options A, B, and D are incorrect. Refer to the explanation of the correct answer.

Solution for Question 17:


Correct Option D - Paroxysmal nocturnal hemoglobinuria:

Page 16

1250
• Paroxysmal nocturnal hemoglobinuria occurs as a result of an acquired intra-corpuscular defect. It is
not caused by an extra-corpuscular defect.

Incorrect Options:
• Options A, B, and C are caused by an extra-corpuscular defect.

Solution for Question 18:


Correct Option B - EMA (Eosin 5' Maleimide) test:
• The EMA(Eosin 5' Maleimide) test is a confirmatory diagnostic test for Hereditary Spherocytosis,
where the spherocytes are labeled with a fluorescent dye. The osmotic fragility test using Pink's
solution is a screening test for HS, it is not the confirmatory test for the condition.
Incorrect Options:
• Options A, C, and D are incorrect, as explained above.

Solution for Question 19:


Correct Option A - Intravascular hemolysis:
• Paroxysmal cold hemoglobinuria (PCH) is an autoimmune hemolytic anemia characterized by the
presence of Donath-Landsteiner antibodies, which bind to red blood cells (RBCs) in cold temperatures,
leading to their destruction upon rewarming. This process primarily occurs within the blood vessels,
resulting in intravascular hemolysis. As a result, hemoglobin is released into the bloodstream, leading
to hemoglobinuria.
Incorrect Options:
• Options B, C, and D are not seen in paroxysmal cold hemoglobinuria.

Solution for Question 20:


Correct Option D - Bone marrow biopsy:
• Bone marrow biopsy is considered the gold standard for diagnosing Aplastic Anemia. The cellularity of
the bone marrow is reduced in aplastic anemia, and there is also an increase in the fat content in the
bone marrow.
Incorrect Options:
• Options A, B, and C are not gold-standard techniques for diagnosing aplastic anemia.

Page 17

1251
Sickle Cell Anemia and Thalassemia
1. Which of the following diseases are patients with sickle cell trait protected against?
A. P.vivax
B. P. falciparum
C. P. ovale
D. All of the above
----------------------------------------
2. Which of the following does not influence the degree of sickling of red cells?
A. Hereditary persistence of fetal hemoglobin
B. Intracellular pH
C. Intake of primaquine
D. Co-existing α-thalassemia
----------------------------------------
3. Which of the following mutations is most likely associated with severe beta-thalassemia in a patient
with 2 β0 alleles?
A. Splicing mutation
B. Promoter mutation
C. Chain Termination mutation
D. Silent mutation
----------------------------------------
4. Diagnosis of thalassemia major is based on the detection of which of the following hemoglobin
subtypes?
A. HbA
B. HbA2
C. HbF
D. HbS
----------------------------------------
5. Which of the following features precipitates the sickling of RBCs?
A. Hyperoxemia
B. Infections
C. Alkalosis
D. None of the above
----------------------------------------
6. A 22-year-old female patient visits the clinic for a routine health visit. Her peripheral blood smear
shows microcytic hypochromic red blood cells. Based on certain suspicion, her blood sample is taken
and subjected to a test shown in the image below. Which of the following is associated with this

1252
patient’s underlying condition?

A. The affected individual has HbF as the predominant hemoglobin.


B. It is a definitive test for the suspected condition.
C. Most individuals develop severe anemia as a clinical manifestation
D. Affected individuals rarely require blood transfusions.
----------------------------------------
7. Sickle cell disease is associated with which type of renal cell carcinoma?
A. Medullary
B. Papillary
C. Chromophobe
D. Clear cell
----------------------------------------
8. A 32-year-old female visits the clinic for a routine health visit. She has no active complaints and feels
good. She has no chronic medical complaints and takes no medications. On investigations, her Hb is
13.0 gm/dl, HbF levels are 95%, and HbA2 is 1.5%. Which of the following is the most likely working
diagnosis?
A. Hereditary persistence of fetal hemoglobin
B. Beta homozygous thalassemia
C. Thalassemia intermedia
D. Beta heterozygous thalassemia
----------------------------------------
9. What is the cause of bone pain in patients with sickle cell anemia?
A. Bone infarction
B. Osteoporosis
C. Osteomalacia
D. Periosteal reaction
----------------------------------------
10. Which of the following statements is true regarding the most severe form of αThalassemia, causing
hydrops-fetalis and still birth?

Page 2

1253
A. Hb Bart’s cannot bind oxygen
B. The excess alpha-globin form insoluble precipitates
C. Hb Bart’s cannot release oxygen to fetal tissues
D. The excess beta-globin form insoluble precipitates
----------------------------------------
11. Which of the following is associated with β-Thalassemia major?
A. Onset at puberty
B. No need for blood transfusions
C. Mild to moderate anaemia
D. Hepatosplenomegaly
----------------------------------------
12. Which of the following is associated with thalassemia major?
A. Transfusion dependency
B. Elfin facies
C. Effective erythropoiesis
D. Macrocytic anemia
----------------------------------------
13. A couple visits the genetic clinic for counseling purposes. The father has beta thalassemic trait
whereas the mother has a normal genotype. Which of the following is the risk of having a child with
beta-thalassemia major?
A. 50%
B. 25%
C. 5%
D. 0%
----------------------------------------
14. Which amino acid substitution characterizes the mutation in sickle cell anemia?
A. Glutamic acid to valine
B. Valine to glutamic acid
C. Alanine to lysine
D. Glycine to proline
----------------------------------------
15. Which type of crisis in sickle cell anemia can cause auto-splenectomy?
A. Aplastic crisis
B. Hemolytic crisis
C. Sequestration crisis
D. Vaso-occlusive crisis

Page 3

1254
----------------------------------------
16. Which of the following is seen in both Iron Deficiency Anemia from β Thalassemia Minor?
A. Normal Red Cell Distribution Width
B. Microcytic hypochromic anemia
C. More RBCs
D. Mentzer index <13
----------------------------------------
17. What genetic constitution is associated with the given histological findings?

A. AA/A-
B. AA/--
C. A-/--
D. --/--
----------------------------------------

Correct Answers
Question Correct Answer

Question 1 2
Question 2 3
Question 3 3
Question 4 3
Question 5 2
Question 6 4
Question 7 1
Question 8 1
Question 9 1
Question 10 3
Question 11 4

Page 4

1255
Question 12 1
Question 13 4
Question 14 1
Question 15 3
Question 16 2
Question 17 3

Solution for Question 1:


Correct Option B - P. falciparum:
• Sickle cell trait (heterozygosity for HbS) confers a protective effect against falciparum malaria. This is
due to the fact that Plasmodium falciparum cannot effectively invade and replicate in sickle-shaped red
blood cells, leading to reduced susceptibility to this particular type of malaria. However, sickle cell trait
does not provide protection against other species of Plasmodium such as P. vivax or P. ovale.
Incorrect Options:
Options A, C and D are incorrect, and the correct option is explained above.

Solution for Question 2:


Correct Option C - Intake of primaquine:
• Intake of primaquine triggers hemolysis in G6PD deficiency and is unrelated to sickle cell
disease/sickling.
• Sickle cell anemia is a type of intrinsic hemolytic anemia that results in both extravascular and
intravascular hemolysis.
• Sickle cell disease is caused by a point mutation in which glutamic acid is replaced by valine at
position 6 of the beta chain, generating a sticky patch on the surface of HbS.
• The sticky patch on the surface of adjacent HbS molecules causes their polymerization [In
dehydration & deoxygenation], forming long fibrous precipitates.
• HbA, HbF can also weakly interact with HbS & prevent its polymerization (HbF >> HbA). Therefore
individuals with the co-inherited conditions of hereditary persistence of fetal Hb (HPFH) have less
severe disease due to reduced sickling.
• The mainstay of treatment is an inhibitor of DNA synthesis, hydroxyurea.
Incorrect Options:
Option A - Hereditary persistence of fetal hemoglobin:
• HbF is protective against sickle cell disease.
• Hydroxyurea used in the management of sickle cell disease since it increases the concentration of
HbF.
Option B - Intracellular pH:
• A decrease in intracellular pH reduces the oxygen affinity of hemoglobin.

Page 5

1256
• Thereby increasing the fraction of deoxygenated HbS at any given oxygen tension and augmenting
the tendency for sickling.
Option D - Co-existing α-thalassemia:
• Co-existing α-thalassemia can reduce the severity of sickling. α-Thalassemia results in decreased
production of α-globin chains, leading to an imbalance in α- and β-globin chain production. This
imbalance can reduce the intracellular concentration of HbS and mitigate sickling.

Solution for Question 3:


Correct Option C - Chain Termination mutation:
In β-thalassemia major, the underlying genetic defect often involves chain termination mutations, which
result in the premature termination of β-globin chain synthesis. This leads to a deficiency in β-globin ch
ains, causing ineffective erythropoiesis and subsequent microcytic hypochromic anemia.
While splicing mutations and promoter mutations can also contribute to β-thalassemia, chain terminatio
n mutations are particularly associated with the severe form of the disease. Silent mutations, on the oth
er hand, do not typically lead to significant alterations in protein structure or function and are less com
monly associated with β-thalassemia major.
• The β chains are encoded by a single β-globin gene located on chromosome 11.
• Most people inheriting any two β0 and β+ alleles have β-thalassemia major.
• Chain termination mutation → ß0
Incorrect Options:
Option A - Splicing mutation:
• Splicing mutations occur in regions of DNA involved in the process of splicing, where introns are
removed from pre-mRNA transcripts to produce mature mRNA. In β-thalassemia, splicing mutations
can affect the splicing of β-globin mRNA, resulting in reduced or absent production of β-globin chains.
• Splicing mutation → ß+>>>ß0
• Mutations involving splice sites are more associated with the β+ allele.
Option B - Promoter mutation:
• Promoter mutations occur in the regulatory regions of genes known as promoters, which control the
initiation of transcription. In β-thalassemia, promoter mutations can disrupt the normal transcription of
the β-globin gene, leading to reduced synthesis of β-globin chains.
• Mutations involving the promotor region are more commonly associated with the β+ allele.
Option D - Silent mutations:
• Silent mutations are DNA sequence changes that do not result in any change to the amino acid
sequence of the encoded protein. These mutations typically occur in the third position of a codon,
where multiple codons can code for the same amino acid (degeneracy of the genetic code). Silent
mutations may occur in the β-globin gene without affecting the production or function of β-globin chains.
They are not typically associated with the pathogenesis of β-thalassemia major.
• These mutations yield the same protein as seen in otherwise healthy adults.
• Thus, are not associated with any evident abnormalities.

Page 6

1257
Solution for Question 4:
Correct Option C - HbF:
• In thalassemia major, fetal hemoglobin (HbF or α2 γ2) is markedly elevated due to the impaired
production of adult hemoglobin. Elevated HbF levels are a key diagnostic feature of thalassemia major.
• HbF predominance can be confirmed through various diagnostic methods, such as HPLC
(High-Performance Liquid Chromatography) or capillary electrophoresis.
• Additionally, in thalassemia trait, there is an increase in HbA2 levels (>3.5%), which can aid in
confirming the diagnosis. However, for diagnosing thalassemia major, the significant elevation of HbF is
the characteristic finding
In thalassemia major, fetal hemoglobin (HbF or α2 γ2) is markedly elevated due to the impaired produc
tion of adult hemoglobin. Elevated HbF levels are a key diagnostic feature of thalassemia major.
HbF predominance can be confirmed through various diagnostic methods, such as HPLC (High-Perfor
mance Liquid Chromatography) or capillary electrophoresis.
Additionally, in thalassemia trait, there is an increase in HbA2 levels (>3.5%), which can aid in confirmi
ng the diagnosis. However, for diagnosing thalassemia major, the significant elevation of HbF is the ch
aracteristic finding
Incorrect Options:
Options A, B, and D are incorrect. Refer to the explanation of the correct answer.

Solution for Question 5:


Correct Option B – Infection:
• Sickle-cell anemia is an autosomal recessive condition that results in the synthesis of an abnormal
hemoglobin chain termed HbS.
• Symptoms in homozygotes don't tend to develop until 4-6 months when the abnormal HbS molecules
take over from fetal hemoglobin.
• Precipitating factors include: Increase in concentration of the HbS. Acidosis Hypoxia Dehydration
Exercise infections Sluggish circulation in organs leads to prolonged exposure of red blood cells to low
oxygen tension, increasing the risk of sickling
• Increase in concentration of the HbS.
• Acidosis
• Hypoxia
• Dehydration
• Exercise
• infections
• Sluggish circulation in organs leads to prolonged exposure of red blood cells to low oxygen tension,
increasing the risk of sickling

Page 7

1258
• Increase in concentration of the HbS.
• Acidosis
• Hypoxia
• Dehydration
• Exercise
• infections
• Sluggish circulation in organs leads to prolonged exposure of red blood cells to low oxygen tension,
increasing the risk of sickling
Incorrect Options:
Option A – Hyperoxemia:
• Conditions that precipitate the sickling of HbS include hypoxia.
• Low oxygen levels also cause sickling.
• Hyperoxemia does not precipitate the condition.
Option C – Alkalosis:
• Acidosis precipitates the sickling of HbS, not alkalosis.
• Alkalosis does not precipitate the condition.
Option D - None of the above:
• Infection is the precipitating factor for Sickle Cell disease.

Solution for Question 6:


Correct Option D - Affected individuals rarely require blood transfusions:
• The picture above shows the NESTROF test (Naked Eye Single Tube Red Cell Osmotic Fragility test),
which is done to screen for the thalassemia trait.
• In the affected patient, the black line will not be visible as hemolysis doesn’t occur. [i.e. osmotic
fragility is decreased].

• Thalassemia trait is characterized by red cell indices and blood smear revealing a microcytic
hypochromic picture with an elevated red cell count.
• Transfusions are generally not required in these patients.

Page 8

1259
Incorrect Options:
Option A - The affected individual has HbF as the predominant hemoglobin:
• High-Pressure Liquid Chromatography (HPLC) shows HbA predominantly and elevated HbA2 levels
(>3.5%) in individuals affected by the thalassemia trait.
• Therefore, this option is incorrect.
Option B - It is a definitive test for the suspected condition:
• NESTROF is a screening test, not definitive for confirmation of the diagnosis of thalassemia trait.
• Hb electrophoresis can be helpful in further confirmation of the disorder.
Option C - Most individuals develop severe anemia as the clinical manifestation:
• Individuals with beta thalassemia trait are usually asymptomatic and have normal or mildly reduced
Hb levels.
• Therefore, this option is incorrect

Solution for Question 7:


Correct Option A – Medullary:
• Renal medullary carcinoma are tumors that arise from the thick ascending limb of the Loop of Henle in
the medulla and are characterized by biallelic loss of SMARCB1 (INI1). Patients with these tumors most
frequently have sickle cell trait or disease. The most common mechanisms of SMARCB1 loss are
deletion and inactivating translocation.
• RMC is one of the most aggressive renal carcinomas.
Incorrect Options:
Options B, C and D are not typically associated with sickle cell trait/disease.

Page 9

1260
Solution for Question 8:
Correct Option A - Hereditary persistence of fetal hemoglobin:
• This asymptomatic patient with elevated HbF levels, and normal Hb levels, most likely has Hereditary
persistence of fetal hemoglobin (HPFH).
• It is a benign condition in which fetal hemoglobin (HbF) production is increased and continues into
adulthood.
• It has the following features: Patient remains asymptomatic. No anaemia is seen. No splenomegaly is
seen. No need for blood transfusions
• Patient remains asymptomatic.
• No anaemia is seen.
• No splenomegaly is seen.
• No need for blood transfusions
• Patient remains asymptomatic.
• No anaemia is seen.
• No splenomegaly is seen.
• No need for blood transfusions
Incorrect Options:
Option B - Beta thalassemia major:
• It is characterized by the following features: Severe anaemia with Hb levels < 6 gm/dl. Chipmunk
facies Hepatosplenomegaly Transient aplastic crisis Repeated need of transfusion
• Severe anaemia with Hb levels < 6 gm/dl.
• Chipmunk facies
• Hepatosplenomegaly
• Transient aplastic crisis
• Repeated need of transfusion
• Severe anaemia with Hb levels < 6 gm/dl.
• Chipmunk facies
• Hepatosplenomegaly
• Transient aplastic crisis
• Repeated need of transfusion
Option C - Thalassemia intermedia:
• It is characterized by mild to moderate anemia with Hb levels between 6-10 gm/dl. It is characterized
by an increase in both HbF and HbA2 levels.
• In this case, the patient’s Hb levels are within the normal range, effectively ruling it out.
Option D - Beta thalassemia minor:
• It is characterized by asymptomatic patients having relatively normal levels of Hb. It is characterized
by HbA2 levels elevated more than 3.5%

Page 10

1261
• The increased levels of HbF noticed in this patient are typically not seen.

Solution for Question 9:


Correct Option A - Bone infarction:
• A vaso-occlusive crisis involving the bone can result in bone pain in patients suffering from sickle cell
anemia. This can also lead to bone infarctions.
• If bones of the hand & feet are involved (dactylitis), the complication is termed "hand-foot syndrome".
• The involvement of long bones can result in avascular necrosis of the femur.
• When the vertebral column is involved, the result is an 'H' shaped vertebra, also known as codfish
vertebra or fish mouth vertebra.
Incorrect Options:
Options B, C and D are incorrect. Refer to the explanation of option A

Solution for Question 10:


Correct Option C - Hb Bart’s cannot release oxygen to fetal tissues:
• Hydrops fetalis is seen in the most severe form of α-thalassemia, caused by the deletion of all four
α-globin genes.

• It is confirmed by an ultrasound examination showing the following features: Foetal ascites


Widespread skin edema A thicker placenta
• Foetal ascites
• Widespread skin edema
• A thicker placenta

Page 11

1262
• Hb in these fetuses have tetramers of γ-globin chains (hemoglobin Barts), which have a high affinity
for oxygen.
• This high affinity for oxygen results in decreased oxygen delivery to the tissues.
• Survival in the early development of these fetuses is due to the expression of ζ chains, an embryonic
globin that pairs with γ chains to form a functional ζ 2 γ 2 Hb tetramer.
• Foetal ascites
• Widespread skin edema
• A thicker placenta
Incorrect Options:
Option A - Hb Bart’s cannot bind oxygen:
• Hb Bart’s has a very high affinity for oxygen.
Option B - The excess alpha-globin form insoluble precipitates:
• Tetramers of γ-globin chains form insoluble precipitates leading to the formation of Hb Bart’s.
Option D - The excess beta-globin form insoluble precipitates:
• Excess γ-globin chains form insoluble precipitates rather than beta-globin chains.

Solution for Question 11:


Correct Option D – Hepatosplenomegaly:
• Hepatosplenomegaly is a prominent feature of β-Thalassemia major.
• It occurs secondary to extramedullary hematopoiesis taking place in these patients.
Incorrect Options:
Option A - Onset at puberty:
• The onset of β-Thalassemia major occurs particularly in childhood rather than puberty.
• Therefore this option is incorrect
Option B - No need for blood transfusions:
• There is a need for repeated transfusions in these patients, which may lead to a variety of side effects,
such as: Transfusion reactions Secondary hemochromatosis
• Transfusion reactions
• Secondary hemochromatosis
• Transfusion reactions
• Secondary hemochromatosis
Option C - Mild to moderate anaemia:
• Severe anemia with a Hb level of 6-7g/dl is seen in patients with thalassemia major.
• Therefore this option is incorrect

Page 12

1263
Solution for Question 12:
Correct Option A - Transfusion dependency:
• Thalassemia major patients are dependent on blood transfusions.
• Thalassemia major patients are bound to receive lifelong transfusions for survival as the body is
unable to produce the adequate number of globin chains for the formation of hemoglobin in the body.
• Repeated blood transfusions are associated with many side effects, such as: Transfusion reactions
Secondary hemochromatosis
• Transfusion reactions
• Secondary hemochromatosis
• Transfusion reactions
• Secondary hemochromatosis
Incorrect Options:
Option B - Elfin facies:
• It is a feature of William's syndrome associated with supravalvular aortic stenosis and hypercalcemia.
• Chipmunk facies are seen in cases of thalassemia major.
Option C - Effective erythropoiesis:
• Thalassemia major is associated with ineffective erythropoiesis secondary to ineffective beta-globin
chain synthesis.
• Therefore this option is incorrect
Option D - Macrocytic anemia:
• Thalassemia major is characterized by microcytic anemia.
• Macrocytic anemia is seen in the following conditions: Folate deficiency B12 deficiency
Hypothyroidism Liver disease
• Folate deficiency
• B12 deficiency
• Hypothyroidism
• Liver disease
• Folate deficiency
• B12 deficiency
• Hypothyroidism
• Liver disease

Solution for Question 13:

Page 13

1264
Correct Option D - 0%:
• The normal percentage of HbA2 ranges from 1.5 to 3%.
• In the thalassemia trait (β thalassemia minor), the HbA2 level may be elevated (3.5-7.5%).
• The wife, in this case, has a normal genotype (β, β), whereas the husband has thalassemia – trait
(β+, β).
• β-Thalassemia is an autosomal recessive disease.

• So, none of the offspring will have thalassemia major (β+ β+), risk of having a child with thalassemia
major is 0%.
Incorrect Options:
Options A, B and C are incorrect. Refer to the explanation of the correct answer.

Solution for Question 14:


Correct Option A - Glutamic acid to valine:
• In sickle cell anemia, a point mutation occurs at the β 6 subunit of hemoglobin, where glutamic acid is
replaced by valine. This substitution results in the formation of sickled hemoglobin (HbS).
Incorrect Options:
• Options B, C, and D are not seen in sickle cell anemia.

Solution for Question 15:


Correct Option C - Sequestration crisis:
• A sequestration crisis in sickle cell anemia involves the entrapment of sickle cells within the spleen,
leading to splenic damage and multiple splenic infarcts. This condition eventually results in a decrease

Page 14

1265
in spleen size, leading to auto-splenectomy.
Incorrect Options:
• Options A, B, and D do not lead to auto-splenectomy.

Solution for Question 16:


Correct Option B - Microcytic hypochromic anemia:
• Microcytic hypochromic anemia is a common characteristic shared by both Iron Deficiency Anemia
and β Thalassemia Minor. It results in reduced oxygen-carrying capacity of the blood, leading to
symptoms such as fatigue, weakness, and shortness of breath.

Incorrect Options:
Option A - Normal Red Cell Distribution Width (RDW):
• RDW is typically increased in both Iron Deficiency Anemia and normal in β Thalassemia Minor.
Option C - More RBCs:
• Iron Deficiency Anemia is typically associated with a decreased number of red blood cells. In contrast,
β Thalassemia Minor may show an increased number of red blood cells.
Option D - Mentzer index <13:
• The Mentzer index is calculated by dividing the mean corpuscular volume (MCV) by the red blood cell
count (RBC). In Iron Deficiency Anemia, the Mentzer index is usually greater than 13. In β Thalassemia
Minor, the Mentzer index is typically less than 13.

Solution for Question 17:


Correct Option C - A-/-- :
• The genetic constitution A-/-- is associated with the formation of golf ball appearances in the red blood
cells. In this condition, there is a decreased production of alpha-globin chains, resulting in excess
beta-globin chains. These excess beta chains form unstable tetramers, leading to the characteristic golf
ball appearance in the red blood cells.
Incorrect Options:
• Options A, B, and D do not lead to the formation of golf ball appearances.

Page 15

1266
Basics of WBC and Flow Cytometry
1. A peripheral blood smear examination post splenectomy would show which of the following findings?
A. Dohle bodies
B. Hypersegmented neutrophils
C. Spherocytes
D. Howell-Jolly bodies
----------------------------------------
2. Which of the following is predictive of a good prognosis in a patient with CLL?
A. Notch mutation
B. Elevated levels of β2M
C. A microRNA 15 & 16 deletion
D. ZAP-70 positivity
----------------------------------------
3. Match the dot blot analysis to their appropriate diagnosis: 1) a. B-ALL 2) b. AML 3) c. PNH

1) a. B-ALL

2) b. AML

1267
3) c. PNH

A. 1-b, 2-c, 3-a


B. 1-c, 2-b, 3-a
C. 1-a, 2-c, 3-b
D. 1-b, 2-a, 3-c
----------------------------------------
4. What is the characteristic mutation that causes May Hegglin anomaly?
A. MYH 19
B. LYST
C. MYH 9
D. WAS
----------------------------------------
5. Which of the following is a marker commonly associated with hematopoietic stem cells?
A. CD 19
B. CD 34+
C. CD 3
D. CD 56
----------------------------------------
6. Match the following: 1 May hegglin anomaly A 2 Large cytoplasmic granules B 3 Alder Reily anomaly
C 4 Pelger huet anomaly D

1 May hegglin anomaly A

Page 2

1268
2 Large cytoplasmic granules B

3 Alder Reily anomaly C

4 Pelger huet anomaly D

A. 1-C, 2-A, 3-D, 4-B


B. 1-D, 2-A, 3-B, 4-C
C. 1-B, 2-D, 3-C, 4-A
D. 1-C, 2-D, 3-A, 4-B
----------------------------------------
7. What is the absolute count range for eosinophils in a healthy individual?
A. 2000-7000/mm3
B. 1000-3000/mm3
C. 200-1000/mm3
D. 50-500/mm3

Page 3

1269
----------------------------------------
8. Which white blood cell type is likely to be increased in polycythemia vera?
A. Neutrophil
B. Basophil
C. Lymphocyte
D. Monocyte
----------------------------------------
9. Which characteristic is unique to myeloblasts compared to lymphoblasts?
A. Presence of Auer rods
B. Positive staining with PAS
C. Acid phosphatase positivity
D. Both B and C
----------------------------------------

Correct Answers
Question Correct Answer

Question 1 4
Question 2 3
Question 3 1
Question 4 3
Question 5 2
Question 6 1
Question 7 4
Question 8 2
Question 9 1

Solution for Question 1:


Option D - Howell-Jolly bodies:
• Howell-Jolly bodies are remnants of RBC nuclei normally removed by the spleen.

• Thus, they are seen in patients who have undergone splenectomy or have functional asplenia (e.g.,
from sickle cell disease). Target cells (shown below) are another consequence of splenectomy.

Page 4

1270
Changes after splenectomy:
• Presence of Howell-Jolly Bodies: Howell-Jolly bodies, and less commonly Heinz bodies, may be
observed in blood smears post-splenectomy.
• Howell-Jolly bodies are typically found in patients post-splenectomy.
• They are nuclear remnants in erythrocytes.
• Their presence indicates the absence of normal splenic filtration.
• Howell-Jolly bodies are less commonly seen in individuals with intact spleens.
Presence of Howell-Jolly Bodies: Howell-Jolly bodies, and less commonly Heinz bodies, may be obser
ved in blood smears post-splenectomy.
Incorrect Options:
Option A - Dohle bodies:

Page 5

1271
• Döhle bodies are small, round, or oval, pale blue-grey structures usually found at the periphery of the
neutrophil.
• They consist of ribosomes and endoplasmic reticulum.
• They are seen in bacterial infections but also following tissue damage, including burns, inflammation,
administration of G-CSF, neutrophilic leukemoid reactions, and during pregnancy.
Option B - Hypersegmented neutrophils:
• The presence of hypersegmented neutrophils is an important diagnostic feature of megaloblastic
anemias.
• Neutrophil hypersegmentation can be defined as neutrophils with six or more lobes or more than 3%
of neutrophils with at least five lobes.
Option C - Spherocytes:
• Spherocytes are spherical red blood cells without an area of central pallor and are usually slightly
smaller in size than the average red cell.
• They are seen in hereditary spherocytosis and autoimmune hemolytic anemia.

Solution for Question 2:


Correct Option C - A microRNA 15 & 16 deletion:
• The microRNA 15 and 16 genes are located on chromosome 13q14.
• Deletions in this region are frequently observed in CLL.
• Patients with CLL and deletion in microRNAs 15 & 16 tend to have a more indolent course and better
prognosis.
Incorrect Options:
Option A - Notch Mutation:
• Notch mutations in association with CLL are associated with a poor prognosis.
Option B - Elevated Levels of β2M:
• Elevated beta-2 macroglobulin (β2M) levels are generally associated with a more advanced or
aggressive disease course in CLL and other hematological malignancies.
Option D - ZAP-70 Positivity:
• ZAP-70 positivity is generally associated with a less favorable prognosis in CLL. It indicates a more
aggressive disease course.

Solution for Question 3:


Correct Option A - 1-b, 2-c, 3-a:
1)

Page 6

1272
b. AML
• CD-34 on the X-axis is a blast cell marker, Y-axis plots CD13 and CD33 which are myeloid cell
markers.
• Most of the cells are plotted in the double positive quadrant and hence the diagnosis has to be AML.
2)

c. PNH
• X-axis CD55, Y-axis CD 59.
• Most of the cells are plotted in double negative quadrant (CD55- & CD59-). Hence indicative of PNH
(some cells are double positive.
3)

Page 7

1273
a. B-ALL
• TdT on the Y-axis is a blast marker and CD10 on the X-axis is a B-Cell marker.
• The double positive quadrant indicates most of the cells (86.07%). The diagnosis has to be B-ALL

Solution for Question 4:


Correct Option C - MYH 9:
• May-Hegglin anomaly causes inclusions in neutrophils, that are often mistaken for a Dohle body. On
electron microscopy, these inclusions are devoid of cell organelles, unlike the Dohle body which
consists of endoplasmic reticulum.
• May-Hegglin anomaly is an autosomal dominant disorder characterized by thrombocytopenia (low
platelet count), giant platelets, and inclusion bodies in granulocytes.

• The characteristic mutation associated with the May-Hegglin anomaly is in the MYH9 gene
(non-muscle myosin heavy chain 9).

Page 8

1274
Incorrect Options:
Options A, B, and D are incorrect. Refer to the explanation of the correct answer.

Solution for Question 5:


Correct Option B – CD 34+:
• CD34+ is a cell surface glycoprotein commonly expressed on hematopoietic stem and progenitor
cells.
Incorrect Options:
Option A – CD 19: CD19 is a cell surface marker found on mature B
cells, not on hematopoietic stem cells.
Option C – CD 3: CD3 is a complex of proteins found on the surface of T lymphocytes and is a
marker for T lymphocytes. It is not a marker for hematopoietic stem cells.
Option D – CD 56: CD56 is a cell adhesion molecule found on natural killer (NK) cells and a
subset of T cells. It is not specific to hematopoietic stem cells.

Solution for Question 6:


Correct Option A - 1-C, 2-A, 3-D, 4-B:
1. May-Hegglin anomaly (MYH9 gene):

Page 9

1275
• May-Hegglin anomaly is characterized by giant platelets, low platelet count, and the presence of
inclusions in granulocytes resembling Döhle bodies. It is associated with mutations in the MYH9 gene.
2. Large cytoplasmic granules:

• This refers to Chediak-Higashi syndrome, which is caused by a defect in the LYST gene. It is
characterized by CNS alterations (C), hemorrhages (H), decreased immunity (DI), albinism (A), and
coarse cytoplasmic granules (K).
3. Alder-Reilly anomaly:

Page 10

1276
• Alder-Reilly anomaly is characterized by large cytoplasmic granules and is associated with
mucopolysaccharidoses.
4. Pelger-Huet anomaly:

• It refers to hyposegmented neutrophils.

Incorrect Options:
• Options B, C, and D are incorrectly matched.

Solution for Question 7:


Correct Option D - 50-500/mm3:

Page 11

1277
• Eosinophils are a type of white blood cell involved in the immune response, particularly against
parasitic infections and allergic reactions. In a healthy individual, the normal absolute count range for
eosinophils is typically between 50 and 500 cells per cubic millimeter (mm³) of blood.

Incorrect Options:
• Options A, B, and C do not represent the normal range of eosinophils in healthy individuals.

Solution for Question 8:


Correct Option B - Basophil:
• Polycythemia vera is a disorder characterized by an abnormal increase in red blood cells in the bone
marrow. While it primarily affects red blood cells, it can also lead to elevated levels of other blood cell
types, including basophils. Increased basophil count is also seen in chronic myelogenous leukemia.
Incorrect Options:
• Options A, C, and D are not raised in polycythemia vera.

Solution for Question 9:


Correct Option A - Presence of Auer rods:
• Auer rods are needle-like cytoplasmic inclusions that are unique to myeloblasts. They are composed
of aggregated azurophilic granules.

Incorrect Options:
Option B -
Positive staining with PAS: Periodic acid-Schiff (PAS) staining is seen in lymphoblasts, it is not a
unique characteristic of myeloblasts.

Page 12

1278
Option C - Acid phosphatase positivity: Acid phosphatase is seen for T-lymphoblasts; it is not a
feature of myeloblasts.
Option D - Both B and C are incorrect as explained above.

Page 13

1279
ALL & AML
1. Which of the following translocations is most commonly seen in childhood B-ALL?
A. t (15; 17)
B. t (8;14)
C. t(14;18)
D. t(9;22)
----------------------------------------
2. What is the most likely diagnosis in a patient whose cytogenetic analysis of leukocytes shows
reciprocal translocation of chromosomes 15 and 17?
A. Acute lymphoblastic leukemia
B. Acute megakaryocytic leukemia
C. Acute promyelocytic leukemia
D. Chronic myelogenous leukemia
----------------------------------------
3. Which AML subtype is positive for all 3 stains (Myeloperoxidase, Chloroacetic esterase and
Nonspecific esterase)?
A. AML M2
B. AML M3
C. AML M0
D. AML M4
----------------------------------------
4. A 60-year-old female patient came to the clinic with right eye upper and lower lid swelling for 1
month. The patient is currently being treated for acute myeloid leukemia. Ultrasound revealed a
well-circumscribed mass in the orbit's lateral and lower quadrants. The patient underwent orbitotomy for
biopsy. Histopathological examination of the biopsy specimen revealed Arbiskov's cells. Which of the
following is the most likely diagnosis?
A. Myelodysplastic syndrome
B. Multiple myeloma
C. Granulocytic sarcoma
D. Leukemia cutis
----------------------------------------
5. Which subtype of AML is most commonly associated with Down syndrome?
A. AML-M3
B. AML-M5
C. AML-M7
D. AML-M6
----------------------------------------

1280
6. Which is a favorable prognostic marker in acute lymphoblastic leukemia (ALL)?
A. Hypodiploidy
B. 20 -30 years of age
C. Low WBC count
D. t(17, 23)
----------------------------------------
7. Which of the given cells display Dot or Blot positivity for PAS?
A. Monocytes
B. Monoblast
C. Erythroblast
D. Lymphoblast
----------------------------------------
8. Acid phosphatase cytochemical staining is used to diagnose which of the following hematological
neoplasm?
A. Chronic myeloid leukemia
B. Acute myeloid leukemia
C. B-cell acute lymphoblastic leukemia
D. T-cell acute lymphoblastic leukemia
----------------------------------------
9. What is the expected molecular mass of the BCR-ABL fusion protein in the case of ALL?
A. 170 kDa
B. 190 kDa
C. 210 kDa
D. 230 kDa
----------------------------------------

Correct Answers
Question Correct Answer

Question 1 4
Question 2 3
Question 3 4
Question 4 3
Question 5 3
Question 6 3
Question 7 4

Page 2

1281
Question 8 4
Question 9 2

Solution for Question 1:


Correct Option D - t(9;22):
Considering the given scenario:
• Translocation t(9;22) is the most common chromosomal rearrangement in childhood B-ALL.
• Factors associated with a poor prognosis include a Philadelphia translocation t(9;22) and age < 1 year
or > 10 years.
• 85% of children with ALL achieve complete remission with chemotherapy.

Other Options:
Option A - t (15; 17):
• Translocation t(15;17) is associated with acute myeloid leukemia (particularly acute promyelocytic
leukemia, or APL), the most common kind of acute leukemia in adults
• This translocation alters the retinoic acid receptor, preventing myeloblast differentiation from occurring
under physiologic levels of retinoic acid.
• High doses of all-trans retinoic acid may induce remission, causing malignant cells to mature.
Option B - t (8;14):
• Translocation t(8;14) is associated with Burkitt lymphoma, an aggressive non-Hodgkin lymphoma
typically manifesting in children and young adults as a rapidly expanding jaw mass.
Option C - t(14;18):
• Translocation t(14;18) is associated with follicular lymphoma and diffuse large B-cell lymphoma, the
most common non-Hodgkin lymphoma in adults.

Solution for Question 2:


Correct Option C - Acute promyelocytic leukemia:
• A chromosomal t(15;17) translocation is characteristic of acute promyelocytic leukemia (M3).
Acute Myelogenous Leukemia:
• Acute myeloid leukemia (AML) is the most common acute leukemia in adults.
• AML comprises a heterogeneous group of cancers of blood cells that arise from genetic and
epigenetic changes in hematopoietic stem and/or progenitor cells that exhibit abnormal growth and
differentiation.
• Most patients with AML present with findings of cytopenias (e.g., fatigue, infections, bleeding), but
some exhibit only laboratory abnormalities.
• Others present with a clinical emergency, including hyperleukocytosis/leukostasis, tumor lysis
syndrome (TLS), disseminated intravascular coagulation (DIC), or involvement of the central nervous

Page 3

1282
system (CNS), and require urgent management.
• (APL)M3-AML is the most common type of AML associated with DIC (Acute promyelocytic leukemia).
• TREATMENT All-trans retinoic acid (ATRA), a vitamin A analog, causes neoplastic promyelocytes to
differentiate into neutrophils, rapidly lowering the risk of DIC. As2O3 is another useful agent (arsenic
trioxide)
• All-trans retinoic acid (ATRA), a vitamin A analog, causes neoplastic promyelocytes to differentiate
into neutrophils, rapidly lowering the risk of DIC.
• As2O3 is another useful agent (arsenic trioxide)
• All-trans retinoic acid (ATRA), a vitamin A analog, causes neoplastic promyelocytes to differentiate
into neutrophils, rapidly lowering the risk of DIC.
• As2O3 is another useful agent (arsenic trioxide)
Incorrect Options:
Options A, B, and D are incorrect, they are not associated with t(15:17) translocation mutation.

Solution for Question 3:


Correct Option C - AML M4:
• AML M4 refers to acute myelomonocytic leukemia. This subtype is characterized by the presence of
both myeloid and monocytic elements. It's known for being positive for all three stains:
• Myeloperoxidase (MPO): This stain is used to identify myeloid cells, as myeloid precursors and blasts
are typically positive for MPO.
• Chloroacetate esterase: This stain is also used to identify myeloid cells, particularly myeloid
precursors.
• Nonspecific esterase: This stain helps identify monocytic cells, as monocytes and monocytic
precursors are usually positive for nonspecific esterase.
AML M4 refers to acute myelomonocytic leukemia. This subtype is characterized by the presence of bo
th myeloid and monocytic elements. It's known for being positive for all three stains:
Myeloperoxidase (MPO): This stain is used to identify myeloid cells, as myeloid precursors and blasts
are typically positive for MPO.
Chloroacetate esterase: This stain is also used to identify myeloid cells, particularly myeloid precursors
.
Nonspecific esterase: This stain helps identify monocytic cells, as monocytes and monocytic precursor
s are usually positive for nonspecific esterase.
Incorrect Options:
OPTIONS A, B, and C are incorrect. and the correct option is explained above.

Solution for Question 4:

Page 4

1283
Correct Option C - Granulocytic sarcoma:
• Granulocytic sarcoma is also known as Choloroma or myeloblastoma.
• It is a rare extramedullary neoplasm of myeloid cells/ immature granulocytic cells.
• It mostly occurs in individuals with a history of acute myeloid leukemia and acute myeloproliferative
disorder.
• Soft tissue, subcutaneous tissue, orbit, peritoneum, central nervous system, and lymph nodes are the
most common locations.
• It presents with signs and symptoms of mass effect and dysfunction of the organ affected.
• Chloroma of the orbit can manifest as exophthalmos, ptosis, lacrimal gland swelling, conjunctival
mass, and redness.
• Diagnosis is made by biopsy of the lesion.
• Arbiskov cells are modified monocytes seen in Chloroma.

Incorrect Options:
Options A, B, and D are incorrect. Refer to the explanation of Option C

Solution for Question 5:


Correct Option C - AML-M7:
• Acute megakaryocytic leukemia (M7) is a rare subtype of acute myeloid leukemia associated with
Down syndrome.
• It occurs due to the arrest of hematopoietic precursors in the earliest stages of development.
• Children with trisomy 21 or Down syndrome have a 10-20-fold increase in developing acute myeloid
leukemia.
• Acute myeloid leukemia affecting children with Down syndrome is known as myeloid leukemia of
Down syndrome.

Page 5

1284
• Trisomy 21 contributes to the malignant transformation of hematopoietic cells.
• It can be observed directly after birth and is characterized by an elevated white blood cell count.
• It is almost always diagnosed before 5 years of age.
• If untreated, it is significantly associated with early death.
• The genetic mutation in Down syndrome that increases the susceptibility to leukemia also increases
their response to chemotherapy, favoring their prognosis with a survival rate of 80-100%.
Incorrect Options:
Options A, B and D are not commonly associated with Down syndrome.

Solution for Question 6:


Correct Option C - Low WBC count:
• Favourable prognostic markers in ALL include Age between 2 and 10 years A low WBC count
Hyperdiploidy Trisomy of chromosomes 4, 7, and 10 The presence of a t(12;21)
• Age between 2 and 10 years
• A low WBC count
• Hyperdiploidy
• Trisomy of chromosomes 4, 7, and 10
• The presence of a t(12;21)
• Age between 2 and 10 years
• A low WBC count
• Hyperdiploidy
• Trisomy of chromosomes 4, 7, and 10
• The presence of a t(12;21)
Incorrect Options:
Option A - Hypodiploidy:
• Hyperdiploidy is associated with a good prognosis, not hypoploidy
Option B - 20- 30 years of age:
• Favourable prognostic markers in ALL include ages between 2 and 10 years.
Option D - t(17, 23):
• The t(12;21) translocation is associated with a good prognosis.

Solution for Question 7:


Correct Option D - Lymphoblast:

Page 6

1285
• PAS staining is a technique used in pathology to detect the presence of glycogen and other
carbohydrates in tissues.
• In the context of leukemia, strong PAS positivity in lymphoblasts, particularly associated with blast cell
vacuoles, is a feature more commonly seen in Acute Lymphoblastic Leukemia (ALL).
• Lymphoblasts show Dot, Blot, or block positivity to PAS staining.
• In certain glycogen storage disorders erythrocytes show PAS positivity.
Incorrect Options:
OPTIONS A, B and C are incorrect and the correct option is explained above.

Solution for Question 8:


Correct Option D - T-cell acute lymphoblastic leukemia:
• The principal diagnostic use of cytochemical observable acid phosphatase is detecting T-cell ALL and
hairy leukemia.
• Acid phosphatase staining is an early differentiating characteristic in T cells
• Almost all T-lineage leukemias, both acute and chronic, show significant activity.
Incorrect Options:
Options A, B and C are incorrect. Refer to the explanation of Option D.

Solution for Question 9:


Correct Option B - 190 kDa :
• In ALL (Acute Lymphoblastic Leukemia), the BCR-ABL fusion protein typically weighs around 190
kDa.
• In CML (Chronic Myeloid Leukemia), the BCR-ABL fusion protein typically weighs around 210 kDa.
• In CNL (Chronic Neutrophilic Leukemia), the BCR-ABL fusion protein typically weighs around 230
kDa.
Incorrect Options:
OPTIONS A, C, and D are incorrect.

Page 7

1286
CLL & CML
1. Which of the following is an expected finding in CML?
A. Auer rods
B. Basophilia
C. Increased LAP score
D. Bone marrow fibrosis
----------------------------------------
2. Choose the appropriate description of cells seen in the bone marrow aspirate of a patient with CML.
A. Garden Party appearance.
B. Convent girl appearance.
C. Parachute appearance.
D. None of the above
----------------------------------------
3. Which chromosomal translocation will be present in CML?
A. t(8;14)
B. t(11;18)
C. t(14;18)
D. t(9;22)
----------------------------------------
4. Which of the following genetic mutations can be expected in a patient with chronic myeloid
leukemia?
A. BCR-ABL1 fusion/translocation
B. KIT mutation
C. CSF3FR mutation
D. JAK2 mutation
----------------------------------------
5. The loss of which of the following components results in the formation of smudge cells in CLL?
A. Desmin
B. Vimentin
C. Cytokeratin
D. None of the above
----------------------------------------
6. Histopathological examination of the lymph node of a patient with CLL shows which of the following?
A. A monomorphic lymphoid proliferation with admixed proliferation centers

1287
B. A polymorphous population of lymphocytes, plasma cells, eosinophils, and scattered large
binucleated cells
C. A predominantly follicular pattern with variably sized follicles effacing nodal architecture
D. The diffuse proliferation of medium to large lymphoid cells with a high mitotic rate.
----------------------------------------
7. A 57-year-old man with inguinal and cervical lymphadenopathy is admitted to the hospital The serum
proteins are within normal limits, but the WBC count is 25,000/mL, with many small abnormal
lymphocytes. HPE of the cervical lymph node biopsy is shown below. What is the diagnosis?

A. Burkitt lymphoma
B. Hodgkin lymphoma
C. Small lymphocytic lymphoma
D. Reactive follicular hyperplasia
----------------------------------------
8. Which of the following is considered a non-radiogenic form of cancer?
(or)
Which of the following is considered a non-radiogenic form of cancer?
A. Acute myeloid leukemia
B. Chronic myeloid leukemia
C. Acute lymphoblastic leukemia
D. Chronic lymphocytic leukemia
----------------------------------------
9. A 64-year-old female patient presents to the clinic with massive left axillary lymphadenopathy. Upon
further evaluation, she was diagnosed with leukemia and the patient is treated with local field radiation
to the axilla. A week after radiotherapy, the lymph nodes that were unirradiated and away from the
radiation field started regressing. This effect is most likely associated with which of the following
leukemia?
A. Chronic myeloid leukemia
B. Chronic lymphocytic leukemia
C. Acute lymphoblastic leukemia
D. Acute myelogenous leukemia

Page 2

1288
----------------------------------------
10. A 40-year-old woman was previously diagnosed with leukemia, and her blood smear is shown
below. The flow cytometry test results revealed that the sample was positive for CD 5, CD 23, and CD
200. She is on treatment, and now she complains of feeling feverish every day and experiencing rapid
unintentional weight loss. Which of the following is the most likely explanation for her current
symptoms?

A. Richter transformation
B. Progression of CLL
C. Development of secondary infections
D. Immunodeficiency-associated hemolytic anemia
----------------------------------------
11. Which of the following conditions show a decreased LAP score?
A. Neutrophilia
B. Paroxysmal Nocturnal Hemoglobinuria
C. Leukemoid Reaction
D. Downs Syndrome
----------------------------------------

Correct Answers
Question Correct Answer

Question 1 2
Question 2 1
Question 3 4
Question 4 1
Question 5 2
Question 6 1
Question 7 3
Question 8 4

Page 3

1289
Question 9 2
Question 10 1
Question 11 2

Solution for Question 1:


Correct Option B - Basophilia:
• Chronic myeloid leukemia (CML) is a myeloproliferative disorder characterized by the presence of the
Philadelphia chromosome (a translocation between chromosomes 9 and 22) resulting in the formation
of the BCR-ABL fusion gene. This gene produces a constitutively active tyrosine kinase, leading to the
uncontrolled proliferation of myeloid cells.
• In CML, basophilia is a common laboratory finding. Basophilia refers to an increase in basophilic
granulocytes in the peripheral blood. Elevated basophil count is one of the hallmarks of CML.
Incorrect Options:
Option A - Auer rods: Auer rods are needle-shaped cytoplasmic inclusions found in the blasts of acute
myeloid leukemia (AML) and are not specific to CML.
Option C - Increased LAP score: In CML, the leukocyte alkaline phosphatase (LAP) score is decreased
, not increased. It is due to defective lymphocytes being unable to produce LAP
Option D - Bone marrow fibrosis: CML typically does not cause significant bone marrow fibrosis in its c
hronic phase. Fibrosis is more commonly seen in other myeloproliferative disorders like primary myelofi
brosis.

Solution for Question 2:


Correct Option A - Garden Party appearance:
• In the context of Chronic Myeloid Leukemia (CML), the characteristic appearance associated with the
bone marrow aspirate is often referred to as the "Garden Party" or "Myeloid Bulge" appearance. CML is
classically defined by the presence of metamyelocytes, myelocytes and other progenitor cells, giving
the bone marrow a crowded and garden-like appearance.

• The image above is the bone marrow aspirate seen in the case of Chronic Myeloid Leukemia.

Page 4

1290
Incorrect Options:
Option B - Convent girl appearance: This is attributed to the uniform appearance of lymphocytes (small
with condensed chromatin and scanty cytoplasm) in CLL.
Option C - Parachute appearance: This is another description provided for Smudge cells or disrupted t
umor cells, which also happen to be a characteristic of CLL.
Option D - None of the above: This option would not apply as the answer is Option A

Solution for Question 3:


Correct Option D - t(9;22):
Chronic Myeloid Leukemia (CML) is associated with the Philadelphia chromosome, that results from th
e translocation between chromosomes 9 and 22, [t(9;22)], giving rise to shortened chromosome 22, w
hich is referred to as the Philadelphia chromosome (Ph chromosome).

Page 5

1291
Incorrect Options:
Option A - t(8;14):
• This translocation is present in Burkitt lymphoma, a neoplasm of mature B cells.
Option B - t(11;18):
• This translocation leads to Marginal zone lymphoma (it is also a neoplasm of mature B cells).
• Mostly associated with chronic inflammation (e.g., Sjogren syndrome, chronic gastritis);
Option C - t(14;18):
• This translocation is present in Follicular lymphoma.

Solution for Question 4:


Correct Option A - BCR-ABL1 fusion/translocation:
• The genetic mutation expected in a patient with Chronic Myeloid Leukemia (CML) is the BCR-ABL1
fusion/translocation, which results from the translocation between chromosomes 9 and 22, also known
as the Philadelphia chromosome.

Page 6

1292
• The BCR-ABL is not an inheritable mutation but a somatic mutation
Incorrect Options:
Option B - KIT mutation:
• Systemic mastocytosis is a rare disease characterized by KIT-driven expansion and accumulation of
neoplastic mast cells in various tissues.
• Mutations of c-KIT are the hallmark of systemic mastocytosis.
Option C - CSF3FR mutation:
• CSF3FR mutation is associated with familial chronic neutrophilic leukemia.
• Therefore this option is incorrect
Option D - JAK2 mutation:
• JAK2 mutation is associated with the development of essential thrombocythemia.

Solution for Question 5:


Correct Option B - Vimentin:
Smudge cells, also known as basket cells or Gumprecht shadows, are fragile lymphocytes that rupture
during slide preparation, leaving a characteristic smudged appearance. Smudge cells are a
hallmark feature of CLL.
• (arrows point to smudge cells)
• The loss of vimentin content in leukemic cells can result in the formation of smudge cells observed in
patients with this disease.

Page 7

1293
Incorrect Options:
Options A, C and D are incorrect and the correct option is explained above.

Solution for Question 6:


Correct Option A - A monomorphic lymphoid proliferation with admixed proliferation centers:
• The histological feature of most CLL consists of the proliferation of small mature B cells with
irregularly condensed chromatin.
• Lymph node biopsy is suggestive of a pseudo follicular pattern with proliferation centers.
• The formation of proliferation centers (pseudo follicles) is due to the nodular expansion of
paraimmunoblast and prolymphocytes admixed with small lymphocytes.
• Paraimmunoblast is a large cell with a round nucleus, an enlarged central nucleolus with basophilic
cytoplasm, and dispersed chromatin.
• Prolymphocytes are small to medium mature lymphocytes with clumped chromatin.
• The below image shows the lymph node histological features
• Image A demonstrated a vaguely nodular (pseudofollicular pattern).
• Image B demonstrates the predominance of small lymphocytes with scattered large cells
(prolymphocytes and para immunoblasts).

Page 8

1294
Incorrect Options:
Options B, C, and D are incorrect. Refer to the explanation of the correct answer.

Solution for Question 7:


Correct Option C - Small lymphocytic lymphoma:
The likely diagnosis based on the clinical features and the histological appearance of the lymph node b
iopsy is SLL. The histological appearance of the lymph node biopsy shows small, mature lymphocytes
characteristic of SLL.
Incorrect Options:
Options A, B and D are incorrect. Refer to the explanation of the correct answer and the LO.

Solution for Question 8:


Correct Option D - Chronic lymphocytic leukemia:
• Malignances that are ‘non-radiogenic’ are said not to be caused by ionizing radiation.
• Radiation is not an etiological factor for CLL.
• The association between external exposure to ionizing radiation and CLL is weak.
• Leukemias, including acute myeloid leukemia (AML), acute lymphocytic leukemia, chronic myeloid
leukemia, and myelodysplastic syndrome (MDS), are radiation-associated hematopoietic neoplasms.
Incorrect Options:
Options A, B and C are radiogenic neoplasms.

Page 9

1295
Solution for Question 9:
Correct Option B - Chronic lymphocytic leukemia:
• Radiation therapy is a highly effective treatment option for numerous cancers.
• It is a local therapy that induces lethal chromosomal aberrations resulting in cell cycle arrest and
subsequent cell death.
• Ionizing radiation affects the nuclear and cytoplasmic components of the cells.
• It results in increased vascular permeability, local inflammation, and altered cytokine levels.
• In addition to the local effect of radiation therapy on tumor cells, the abscopal effect is seen.
• The abscopal effect is the disappearance or shrinkage of tumors at sites that are not directly targeted
by radiation therapy.
• Radiation therapy leads to the release of tumor antigens engulfed by antigen-presenting cells and
presented to T cells.
• It leads to the activation of T cell-mediated immune response.
• Cytokines are also released by the tumor cells that transport the cytotoxic T cells to the remotely
located diseased site.
• This effect is associated with chronic lymphocytic leukemia.

Incorrect Options:
Options A, C and D are not associated with the abscopal effect and are incorrect.

Solution for Question 10:


Correct Option A - Richter transformation:
This patient’s symptoms are suggestive of Richter's transformation.
• Richter transformation or Richter syndrome is a rare complication of chronic lymphocytic leukemia.

Page 10

1296
• This syndrome is characterized by the sudden transformation of CLL into a significantly aggressive
form of large-cell lymphoma.
• The most common symptoms include sudden clinical deterioration, characterized by an increase in
lymphadenopathy, usually painless areas of swelling in the neck, axilla, spleen, and groin.
• Worsening B symptoms (unexplained weight loss, fever, night sweats).
• The gold standard for diagnosing Richter transformation is histological documentation with open
biopsy.
• Diagnosis by flow cytometry positive for CD19/20, CD 5/23.
• Fludarabine is the drug of choice for management.
• The image below demonstrates enlarged cells with pleomorphic nuclei, less condensed chromatin,
prominent nucleoli, and scant cytoplasm.

(The image shows the findings consistent with Richter transformation)


Incorrect Options:
Options B, C and D are incorrect. Refer to the explanation of the correct answer.

Solution for Question 11:


Correct Option B - Paroxysmal Nocturnal Hemoglobinuria:
• In Paroxysmal Nocturnal Hemoglobinuria, there is a deficiency of the enzyme alkaline phosphatase
(AP) on the surface of blood cells, leading to a decreased LAP score. This is a characteristic feature of
PNH and distinguishes it from other conditions.
Incorrect Options:
Option A- Neutrophilia:
• Neutrophilia refers to an elevated number of neutrophils in the blood, which can occur in response to
various conditions, including infections, inflammation, stress, and certain medications. Neutrophilia is

Page 11

1297
not typically associated with changes in the leukocyte alkaline phosphatase (LAP) score.
Option C - Leukemoid Reaction:
• A benign condition characterized by a marked increase in the number of white blood cells, often
resembling leukemia but due to a reactive cause rather than a malignant process. It leads to an
increased LAP score.
Option D - Down syndrome:
• Down syndrome is associated with an increased LAP score.

Page 12

1298
Chronic Myeloproliferative Disorders and
Myelodysplastic Syndromes
1. The complete blood count of a 25-year-old woman with fatigue, is shown below: Hemoglobin: 18.5
g/dL RBC count: 6.2 x 106/µL TLC: 8.5 x 103/µL What is the likely diagnosis in this patient?
A. Polycythemia vera
B. Juvenile myelomonocytic leukemia
C. Acute myeloid leukemia
D. Thalassemia
----------------------------------------
2. Which of the following mutations is associated with essential thrombocytosis?
A. Janus Kinase
B. STAT -3
C. SMAD4
D. Bruton tyrosine kinase
----------------------------------------
3. Which mutation is associated with myelodysplastic syndrome?
A. SF4B1
B. SF3B1
C. SF3B2
D. SF4B2
----------------------------------------
4. A 45-year-old female patient came with complaints of throbbing sensation in her hands and feet. Her
CBC showed a platelet count of 5.5L/cubic mm, but other parameters were normal. A peripheral smear
reveals the feature shown below. Which is the most likely mutation seen in her case?

A. Monosomy 7
B. MPL
C. Deletion 5q

1299
D. ALK gene
----------------------------------------
5. Which of the following is not a characteristic feature seen in patients with myelofibrosis?
A. Fibrotic obliteration of bone marrow spaces.
B. Dacrocytes are seen
C. Erythromelalgia
D. JAK2 mutation
----------------------------------------
6. A 72-year-old male came with complaints of recurrent infections, nose bleeds, and generalized
weakness. Labs showed pncytopenia.Bone marrow aspiration was done. A certain characteristic
feature was noted in the patient's neutrophils. An image of this finding is shown below. Identify the
image and the diagnosis.

A. Pseudo-Pegler-Huet cells, Myelodysplastic syndrome.


B. May-hegglin Anomaly, Thrombocytopenia.
C. Pseudo-Pegler-Huet cells with Dohle bodies, Myelodysplastic syndrome
D. Howell-Jolly bodies, megaloblastic anemia.
----------------------------------------

Correct Answers
Question Correct Answer

Page 2

1300
Question 1 1
Question 2 1
Question 3 2
Question 4 2
Question 5 3
Question 6 3

Solution for Question 1:


Correct Option A - Polycythemia vera:
• In polycythemia vera, patients typically present with symptoms related to hyperviscosity due to
increased red blood cell mass. This can manifest as headaches, dizziness, blurred vision, and fatigue.
Additionally, symptoms of bleeding and thrombosis are common due to altered blood flow dynamics.
The elevated hemoglobin and RBC count seen in this patient's lab results are consistent with
polycythemia vera.
• Activating JAK2 mutations are seen in: Nearly all cases of polycythemia vera (Approx. 50% of cases
of primary myelofibrosis and essential thrombocythemia).
• Nearly all cases of polycythemia vera (Approx. 50% of cases of primary myelofibrosis and essential
thrombocythemia).
• Symptoms include: Ruddy complexion Cyanosis Headaches Dizziness Pruritus, abdominal pain
(peptic ulcers)
• Ruddy complexion
• Cyanosis
• Headaches
• Dizziness
• Pruritus, abdominal pain (peptic ulcers)
• Diagnostic tests: Hypertension Complete blood picture: Increased RBCs (Elevated hemoglobin)
Hyperuricemia with symptomatic gout in 5-10% (due to high cell turnover) Bone marrow aspiration or
biopsy Specific gene testing
• Hypertension
• Complete blood picture: Increased RBCs (Elevated hemoglobin) Hyperuricemia with symptomatic
gout in 5-10% (due to high cell turnover)
• Increased RBCs (Elevated hemoglobin)
• Hyperuricemia with symptomatic gout in 5-10% (due to high cell turnover)
• Bone marrow aspiration or biopsy
• Specific gene testing
• Examples of myeloproliferative neoplasms: Chronic myeloid leukemia Polycythemia vera Primary
myelofibrosis Essential thrombocythemia\ Chronic eosinophilic leukemia
• Chronic myeloid leukemia
• Polycythemia vera

Page 3

1301
• Primary myelofibrosis
• Essential thrombocythemia\
• Chronic eosinophilic leukemia
• Nearly all cases of polycythemia vera (Approx. 50% of cases of primary myelofibrosis and essential
thrombocythemia).
• Ruddy complexion
• Cyanosis
• Headaches
• Dizziness
• Pruritus, abdominal pain (peptic ulcers)
• Hypertension
• Complete blood picture: Increased RBCs (Elevated hemoglobin) Hyperuricemia with symptomatic
gout in 5-10% (due to high cell turnover)
• Increased RBCs (Elevated hemoglobin)
• Hyperuricemia with symptomatic gout in 5-10% (due to high cell turnover)
• Bone marrow aspiration or biopsy
• Specific gene testing
• Increased RBCs (Elevated hemoglobin)
• Hyperuricemia with symptomatic gout in 5-10% (due to high cell turnover)
• Chronic myeloid leukemia
• Polycythemia vera
• Primary myelofibrosis
• Essential thrombocythemia\
• Chronic eosinophilic leukemia
Incorrect Options:
Options B, C and D are incorrect. Refer to the explanation of the correct answer.

Solution for Question 2:


Correct Option A - Janus Kinase:
• Essential thrombocythemia (ET) is a myeloproliferative disorder characterized by elevated platelet
counts, usually resulting from mutations in JAK2 (Janus Kinase, MPL, or CALR genes, leading to
dysregulated signaling pathways.
• This condition manifests clinically with elevated platelet count, thrombosis, and hemorrhage as major
complications, along with erythromelalgia, a characteristic symptom.
• ET typically has an indolent course with long asymptomatic periods and a median survival time of 12
to 15 years.

Page 4

1302
• Bone marrow findings include increased megakaryocytes with abnormal morphology.
Incorrect Options:
Option B - STAT -3:
• STAT-3 mutation is not typically associated with essential thrombocythemia. STAT-3 is involved in
various cellular processes, but mutations in this gene are not commonly linked to myeloproliferative
disorders such as essential thrombocythemia.
Option C - SMAD4:
• SMAD4 mutation is associated with juvenile polyposis syndrome and certain cancers but is not
implicated in essential thrombocythemia.
Option D - Bruton tyrosine kinase:
• Bruton tyrosine kinase (BTK) mutation is associated with X-linked agammaglobulinemia, a primary
immunodeficiency disorder. BTK plays a crucial role in B-cell development and function but is not
known to be involved in essential thrombocythemia.

Solution for Question 3:


Correct Option B - SF3B1:
• SF3B1 is associated with myelodysplastic syndrome (MDS). Mutations in this gene are commonly
found in patients with MDS, particularly in cases of refractory anemia with ring sideroblasts. SF3B1 is
involved in RNA splicing, and mutations in this gene contribute to the pathogenesis of MDS by
disrupting normal RNA processing and leading to abnormal hematopoiesis.
Incorrect Options:
Options A, C and D are incorrect and the correct option is explained above.

Solution for Question 4:


Correct Option B - MPL:
• Presenting symptoms like erythromelalgia (throbbing sensation in her hands and feet), Lab findings of
thrombocytosis, along with the peripheral smear showing staghorn megakaryocyte (shown in the
image) is indicative of essential thrombocythemia.

• Mutations in the MPL gene are commonly associated with essential thrombocythemia. The MPL gene
encodes for the thrombopoietin receptor, and mutations can lead to increased platelet production.

Page 5

1303
Incorrect Options:
Options A, C, and D are incorrect. and the correct option is explained above.

Solution for Question 5:


Correct Option C - Erythromelalgia:
Anemia, is common in myelofibrosis due to ineffective hematopoiesis.
• Enlargement of the spleen is a hallmark feature of myelofibrosis due to extramedullary hematopoiesis.
• The image shows cloud-like megakaryocyte seen in patients with myelofibrosis.

• Erythromelalgia is not typically associated with this diagnosis. Erythromelalgia, characterized by
episodes of redness, warmth, and painful swelling in the extremities, is more commonly associated with
conditions like essential thrombocythemia and polycythemia vera

Page 6

1304
Incorrect Options:
Option A, B & D - (Fibrotic obliteration of bone marrow spaces, Dacrocytes are seen & JAK2 mutation):
• The other options (A, B, and D) are more in line with features of myelofibrosis, a type of
myeloproliferative disorder that can arise as a progression of essential thrombocythemia. In
myelofibrosis, there is fibrotic obliteration of bone marrow spaces (Option A), the presence of abnormal,
teardrop-shaped red blood cells called dacrocytes (Option B), and often, the presence of JAK2
mutation (Option D).

Solution for Question 6:


Correct Option C - Pseudo-Pegler-Huet cells with Dohle bodies, Myelodysplastic syndrome:
• The image shows Pseudo-Pegler-Huet and Dohle bodies.
• Given the patient's clinical presentation of recurrent infections, nose bleeds, generalized weakness,
and pancytopenia, along with the specific features observed in the neutrophils (Pseudo-Pelger-Huet
cells and Dohle bodies), the most likely diagnosis is myelodysplastic syndrome (MDS).
Incorrect Options:
Option A - Pseudo-Pelger-Huet cells, Myelodysplastic syndrome:
• While Pseudo-Pelger-Huet cells are associated with myelodysplastic syndromes, the presence of
Dohle bodies further refines the diagnosis in this case.
Option B - May-Hegglin Anomaly, Thrombocytopenia:
• May-Hegglin Anomaly is a rare inherited disorder characterized by the presence of large platelets and
inclusion bodies in the cytoplasm of neutrophils. The inclusion bodies are called "Döhle-like bodies,"
and they resemble Dohle bodies. In May-Hegglin Anomaly, the Döhle-like bodies are associated with a
platelet disorder and not myelodysplastic syndrome.
Option D - Howell-Jolly bodies, megaloblastic anemia:

Page 7

1305
• Howell-Jolly bodies are nuclear remnants found in red blood cells and are associated with conditions
affecting erythropoiesis, such as megaloblastic anemia. The clinical presentation and the neutrophil
abnormalities described in the question are not characteristic of megaloblastic anemia.

Page 8

1306
Plasma Cell Dyscrasia
1. A 68-year-old patient presents with complaints of shortness of breath, swelling in his feet, and back
pain persisting for one month. Urine analysis shows light chain protein excretion. An image of his bone
marrow aspirate is shown below. Which of the following is true regarding this disease?

A. It is a plasma cell neoplasm characterized by lytic bone lesions, hypercalcemia, renal failure, and
acquired immune abnormalities
B. It involves rearrangements of genes such as cyclin D1 and cyclin D3, as well as deletions of
chromosome 17p
C. The malignant cells produce cytokines like IL-6
D. All the above.
----------------------------------------
2. Which of the following cytokines is involved in the proliferation and survival of tumor cells in multiple
myeloma?
A. IL-1
B. IL-6
C. IL-2
D. IL-5
----------------------------------------
3. Which of the following constitutes the proteins excreted in the urine of patients with multiple
myeloma?
A. Alpha globulins
B. Light chain globulins
C. Beta globulins
D. Albumin
----------------------------------------
4. Which of the following is likely responsible for the M-spike seen on serum electrophoresis in a patient
with multiple myeloma?
A. IgM
B. IgA

1307
C. IgG
D. IgD
----------------------------------------
5. What is the mechanism involved in bony lysis in patients with multiple myeloma?
A. Notch Signaling
B. RANKL pathway
C. Macrophage-Colony Stimulating Factor (M-CSF) Pathway
D. Cathepsin K Pathway
----------------------------------------
6. A 64-year-old male with multiple myeloma has a bone marrow aspiration done. Images from this test
are shown below. Which of the following is responsible for the changes seen in these cells?

A. Immunoglobulins
B. Cholesterol
C. Phospholipids
D. Lipoproteins
----------------------------------------
7. Which of the following is the correct statement regarding MGUS and multiple myeloma?
A. MGUS and multiple myeloma present with bone pains and renal dysfunction.
B. Bence-jones proteinuria is characteristic of MGUS but not for multiple myeloma.
C. On Bone Marrow biopsy, MGUS has <10% plasma cells, whereas multiple myeloma has more than
10%.
D. MGUS does not progress to multiple myeloma.
----------------------------------------
8. Which of the following mutations is associated with Waldenstrom macroglobulinemia?
A. MYD 88
B. MYD 99
C. MYD 77
D. MYD 66

Page 2

1308
----------------------------------------

Correct Answers
Question Correct Answer

Question 1 4
Question 2 2
Question 3 2
Question 4 3
Question 5 2
Question 6 1
Question 7 3
Question 8 1

Solution for Question 1:


Correct Option D- All the above:
• Based on the clinical presentation described, the patient is likely suffering from multiple myeloma.
• The renal symptoms including swelling in legs, could be due to renal amyloidosis in association with
MM, back pain may be due to metastatic involvement of the spine (lytic lesions in the spine) and
constipation may be due to hypercalcemia affecting gastrointestinal motility.
• The image shows a MOTT cell or mulberry cell characterized by multiple grapelike cytoplasmic
droplets.
• The cytoplasmic intra-cytoplasmic inclusions are called Russel Bodies, and the intranuclear
cytoplasmic inclusions are called Dutcher bodies.

Considering the diagnosis of Multiple myeloma all the statements are true:

Page 3

1309
Solution for Question 2:
Correct Option B- IL-6:

• The image above demonstrates the punched-out lesions (salt and pepper skull) seen in a case of
Multiple Myeloma.
• Multiple myeloma is one of the most common lymphoid malignancies.
• RANKL (Receptor Activator of Nuclear Factor Kappa-B Ligand) is a protein that plays a crucial role in
bone remodeling by stimulating osteoclast formation and activity. In multiple myeloma, RANKL is
overexpressed, often as a result of IL-6 stimulation. This overexpression leads to the activation of
osteoclasts, cells responsible for bone resorption.
• It usually presents in old age with the characteristic features as follows: Bone pains/ pathologic
fractures Confusion (due to hypercalcemia) Weakness and lethargy (due to hypercalcemia) Recurrent
bacterial infections Renal insufficiency/dysfunction (Bence-jones proteinuria/renal failure)
• Bone pains/ pathologic fractures
• Confusion (due to hypercalcemia)
• Weakness and lethargy (due to hypercalcemia)
• Recurrent bacterial infections
• Renal insufficiency/dysfunction (Bence-jones proteinuria/renal failure)
• Myeloma cell proliferation occurs due to the following: Dysregulation of D cyclins Interleukin 6(IL-6)
MYC translocations
• Dysregulation of D cyclins
• Interleukin 6(IL-6)
• MYC translocations
• Fibroblasts and macrophages produce IL-6 in the stroma of bone marrow.
• MYC translocations are responsible for aggressive disease.

Page 4

1310
• Bone pains/ pathologic fractures
• Confusion (due to hypercalcemia)
• Weakness and lethargy (due to hypercalcemia)
• Recurrent bacterial infections
• Renal insufficiency/dysfunction (Bence-jones proteinuria/renal failure)
• Dysregulation of D cyclins
• Interleukin 6(IL-6)
• MYC translocations
Incorrect Options:
Option A- IL-1:
• While IL-1 is involved in inflammation and immune responses, it is not specifically implicated in the
pathogenesis of multiple myeloma.
• Interleukin 1 (IL-1) is responsible for recruiting leukocytes and inducing inflammation.
• It is a mediator in autoimmune diseases and lymphomas.
• Anakinra, an IL-1 antagonist, is used as a treatment for rheumatoid arthritis.
Option C- IL-2:
• IL-2 is primarily known for its role in T cell proliferation and activation, and it is not directly associated
with the pathophysiology of multiple myeloma.
• Interleukin 2 (IL-2) promotes the growth and development of immune cells (B and T lymphocytes)
during the initiation of the immune response.
• It also prevents autoimmune diseases by differentiating immature T cells from regulatory T cells.
Option D- IL-5:
• IL-5 is mainly associated with eosinophil activation and is not directly involved in the proliferation and
survival of plasma cells, which are the hallmark of multiple myeloma.

Page 5

1311
• Interleukin 5 (IL-5) is responsible for the maturation of eosinophils in the bone marrow and their
release in the blood.

Solution for Question 3:


Correct Option B- Light chain globulins:
• In patients with multiple myeloma, the presence of abnormal monoclonal immunoglobulins (M
proteins) produced by the malignant plasma cells can lead to the excretion of light chains in the urine.
These light chains, also known as Bence Jones proteins, are part of the immunoglobulin structure and
are filtered by the kidneys, leading to their presence in the urine.
• The plasma cells in multiple myeloma produce and release only one ofkappa and lambda (Κ and λ)
chains.
• These Bence-jones proteins, Tamm-Horsfall protein, albumin, and complete immunoglobulins form
obstructive casts in distal convoluted tubules and the collecting ducts.
• Light chain deposition in the glomeruli or the interstitium, referred to as amyloid or linear deposits,
causes renal damage.
Incorrect Options:
Options A, C and D are incorrect and the correct option is explained above.

Solution for Question 4:


Correct Option C- IgG:

• M-protein is a monoclonal immunoglobulin in the blood.
• M-proteins are complete proteins with high molecular weight that cannot pass through the membranes
and hence, are restricted to the plasma and extracellular fluid.
• They cannot be excreted in the urine without glomerular damage.
• The most frequent M protein produced and seen in multiple myeloma is IgG (55% of patients) followed
by IgA (25% of cases).
• IgM, E, and D are rarely produced by myeloma cells.

Page 6

1312
Incorrect Options:
• IgG is most commonly responsible for the ‘M protein’ seen in patients with multiple myeloma.
• IgM is common in Waldenström Macroglobulinemia (WM)
Options A, B, and D are incorrect. Refer to the explanation of Option C.

Solution for Question 5:


Correct Option B- RANKL pathway:
• In multiple myeloma, bony lysis and the formation of lytic and punched-out lesions involve the
dysregulation of the RANKL (Receptor Activator of Nuclear Factor κB Ligand) pathway. Normal Bone
Remodeling: In normal bone remodeling, there is a balance between bone formation by osteoblasts
and bone resorption by osteoclasts. RANKL-RANK-OPG System: RANKL is a cytokine that activates
its receptor RANK on the surface of osteoclast precursors. This binding promotes osteoclast
differentiation, activation, and survival. Osteoprotegerin (OPG) is a decoy receptor that competes with
RANK for binding to RANKL, preventing excessive osteoclast activation. Dysregulation in Multiple
Myeloma: In multiple myeloma, plasma cells (malignant cells) in the bone marrow produce increased
amounts of RANKL. The overexpression of RANKL leads to an imbalance in the RANKL-RANK-OPG
system, favoring excessive osteoclast activation and bone resorption.
• Normal Bone Remodeling: In normal bone remodeling, there is a balance between bone formation by
osteoblasts and bone resorption by osteoclasts.
• In normal bone remodeling, there is a balance between bone formation by osteoblasts and bone
resorption by osteoclasts.
• RANKL-RANK-OPG System: RANKL is a cytokine that activates its receptor RANK on the surface of
osteoclast precursors. This binding promotes osteoclast differentiation, activation, and survival.
Osteoprotegerin (OPG) is a decoy receptor that competes with RANK for binding to RANKL, preventing
excessive osteoclast activation.
• RANKL is a cytokine that activates its receptor RANK on the surface of osteoclast precursors. This
binding promotes osteoclast differentiation, activation, and survival.

Page 7

1313
• Osteoprotegerin (OPG) is a decoy receptor that competes with RANK for binding to RANKL,
preventing excessive osteoclast activation.
• Dysregulation in Multiple Myeloma: In multiple myeloma, plasma cells (malignant cells) in the bone
marrow produce increased amounts of RANKL. The overexpression of RANKL leads to an imbalance
in the RANKL-RANK-OPG system, favoring excessive osteoclast activation and bone resorption.
• In multiple myeloma, plasma cells (malignant cells) in the bone marrow produce increased amounts of
RANKL.
• The overexpression of RANKL leads to an imbalance in the RANKL-RANK-OPG system, favoring
excessive osteoclast activation and bone resorption.
• Normal Bone Remodeling: In normal bone remodeling, there is a balance between bone formation by
osteoblasts and bone resorption by osteoclasts.
• In normal bone remodeling, there is a balance between bone formation by osteoblasts and bone
resorption by osteoclasts.
• RANKL-RANK-OPG System: RANKL is a cytokine that activates its receptor RANK on the surface of
osteoclast precursors. This binding promotes osteoclast differentiation, activation, and survival.
Osteoprotegerin (OPG) is a decoy receptor that competes with RANK for binding to RANKL, preventing
excessive osteoclast activation.
• RANKL is a cytokine that activates its receptor RANK on the surface of osteoclast precursors. This
binding promotes osteoclast differentiation, activation, and survival.
• Osteoprotegerin (OPG) is a decoy receptor that competes with RANK for binding to RANKL,
preventing excessive osteoclast activation.
• Dysregulation in Multiple Myeloma: In multiple myeloma, plasma cells (malignant cells) in the bone
marrow produce increased amounts of RANKL. The overexpression of RANKL leads to an imbalance
in the RANKL-RANK-OPG system, favoring excessive osteoclast activation and bone resorption.
• In multiple myeloma, plasma cells (malignant cells) in the bone marrow produce increased amounts of
RANKL.
• The overexpression of RANKL leads to an imbalance in the RANKL-RANK-OPG system, favoring
excessive osteoclast activation and bone resorption.
• In normal bone remodeling, there is a balance between bone formation by osteoblasts and bone
resorption by osteoclasts.
• RANKL is a cytokine that activates its receptor RANK on the surface of osteoclast precursors. This
binding promotes osteoclast differentiation, activation, and survival.
• Osteoprotegerin (OPG) is a decoy receptor that competes with RANK for binding to RANKL,
preventing excessive osteoclast activation.
• In multiple myeloma, plasma cells (malignant cells) in the bone marrow produce increased amounts of
RANKL.
• The overexpression of RANKL leads to an imbalance in the RANKL-RANK-OPG system, favoring
excessive osteoclast activation and bone resorption.
Incorrect Options:
Option A- Notch Signaling:
• Notch signaling is involved in bone remodeling, but it is not the primary pathway associated with
osteoclast activation and bone resorption in multiple myeloma.
• The dysregulation of Notch signaling is not a prominent feature heret.

Page 8

1314
Option C- Macrophage-Colony Stimulating Factor (M-CSF) Pathway:
• M-CSF is essential for osteoclast formation and survival, but it acts upstream of RANKL in the
osteoclast differentiation pathway.
• In multiple myeloma, RANKL overexpression is a more direct contributor to increased osteoclast
activation and subsequent bone resorption.
Option D- Cathepsin K Pathway:
• Cathepsin K is a cysteine protease involved in the degradation of bone matrix proteins during
osteoclast activity.
• While it plays a role in bone resorption, it is not the primary pathway dysregulated in multiple
myeloma-associated osteolysis.
• The overexpression of RANKL is a more direct mechanism in this context.

Solution for Question 6:


Correct Option A- Immunoglobulins:
• The images above show Mott cells.
• Mott cells are atypical plasma cells that contain cytoplasmic bodies called the Russel bodies.

• Russel Bodies contain immunoglobulins.
• Other multiple plasma cell morphologies associated with multiple myeloma include: Flame cell
(distinctive pinkish hue in the outer rim of cytoplasm)
• Flame cell (distinctive pinkish hue in the outer rim of cytoplasm)

• Flame cell (distinctive pinkish hue in the outer rim of cytoplasm)

Page 9

1315
• Plasma cells containing Dutcher bodies (nuclear inclusions containing immunoglobulins)

Incorrect Options:
Options B, C and D are incorrect, Refer to the explanation of Option A

Solution for Question 7:


Correct Option C- On Bone Marrow biopsy, MGUS has <10% plasma cells, whereas multiple myeloma
has more than 10%:
• MGUS is a plasma cell dyscrasia in which plasma cells secrete a myeloma protein.
• It is usually asymptomatic. However, some individuals may experience a rash, numbness, or tingling.

Page 10

1316
• It is usually identified when an individual undergoes evaluation for some other disorder or as a part of
standard screening.
• It is differentiated from multiple myeloma by the percentage of plasma cells on bone marrow biopsy. It
has less than 10% of plasma cells, whereas multiple myeloma has more than 10% of plasma cells.
• Patients are diagnosed with MGUS when: The monoclonal paraprotein band is less than 3g/dl Plasma
cells in the bone marrow are less than 10% There is no evidence of bone lesions, anemia,
hypercalcemia, or chronic kidney disease (unlike multiple myeloma) There is no evidence of any other
B-cell proliferative disease.
• The monoclonal paraprotein band is less than 3g/dl
• Plasma cells in the bone marrow are less than 10%
• There is no evidence of bone lesions, anemia, hypercalcemia, or chronic kidney disease (unlike
multiple myeloma)
• There is no evidence of any other B-cell proliferative disease.
• MGUS can transform into multiple myeloma or similar lymphoproliferative disorders occurs at a rate of
1-2% per year.
• Therefore, a patient with MGUS is followed up once every 6 months or a year with serum
electrophoresis to look for transformation into multiple myeloma.
• The monoclonal paraprotein band is less than 3g/dl
• Plasma cells in the bone marrow are less than 10%
• There is no evidence of bone lesions, anemia, hypercalcemia, or chronic kidney disease (unlike
multiple myeloma)
• There is no evidence of any other B-cell proliferative disease.
Incorrect Options:
Option A- MGUS and multiple myeloma present with bone pains and renal dysfunction:
• Multiple myeloma may present with bone pains, pathologic fractures, symptoms of hypercalcemia
(confusion, weakness, lethargy), renal dysfunction and recurrent bacterial infections, whereas MGUS is
usually asymptomatic.
• Patients with MGUS are usually asymptomatic but may present with rash, numbness, or tingling.
Option B- Bence-jones proteinuria is characteristic of MGUS but not for multiple myeloma:
• Bence-jones proteinuria is characteristic of multiple myeloma.
• As there is no renal dysfunction, no proteinuria is seen in MGUS.
Option D- MGUS does not progress to multiple myeloma:
• MGUS leads to multiple myeloma at a rate of 1-2 % per year.
• Patients above 50 years of age have a transformation rate of 3%, while those above 70 years have a
transformation rate of 5%.

Solution for Question 8:


Correct Option A- MYD 88:

Page 11

1317
• Waldenstrom macroglobulinemia/Lymphoplasmacytic lymphoma is a plasma cell dyscrasias that
presents in the sixth or seventh decade.
• Bone marrow aspirate showing an abundance of medium to large-size lymphocytes, plasma cells, and
mast cells
• The plasma cell component mostly secretes IgM, which results in hyperviscosity syndrome (visual
impairment, neurologic problems, bleeding, and cryoglobulinemia).
• Almost all cases are associated with MYDD 88 acquired mutations. MYD is responsible for encoding
the adapter protein that participates in signaling events activating NF-kB. It also potentiates
downstream signals of the BCR complex. NF-kB and BCR complex promotes the growth and survival
of the tumor cells.
• The symptoms of hyperviscosity syndrome are relieved by plasmapheresis, which reduces the IgM
levels.
Incorrect Options:
Option B- MYD 99:
• MYD 99 mutation has no link to Waldenstrom macroglobulinemia.
Option C- MYD 77:
• MYD 77 mutation is not linked to Waldenstrom macroglobulinemia.
Option D - MYD 66:
• MYD 66 mutation is not associated with Waldenstrom macroglobulinemia.

Page 12

1318
Hodgkin's & Non- Hodgkin's Lymphoma
1. Which protein marker would expected to be positive in follicular lymphoma?
A. ABL
B. BAX
C. BCL-2
D. Myc
----------------------------------------
2. Which of the following investigations would confirm a diagnosis of Hodgkins lymphoma?
A. Lymph node biopsy
B. Peripheral blood film
C. Computed tomography (CT) scan
D. Positron emission tomography (PET)
----------------------------------------
3. A 72-year-old man presents with complaints of a 4-month history of fatigue, weight loss, and bilateral
cervical lymphadenopathy. His labs show low hemoglobin, platelets, lymphocytosis, and LDH of
350IU/L. His FISH report shows chromosome 14;18 translocations. An excisional biopsy of the right
axillary lymph node shows the following findings: Which of the following non-Hodgkin lymphomas is
most likely to cause this condition?

A. Follicular lymphoma
B. Burkitt lymphoma
C. Diffuse large cell lymphoma
D. Chronic lymphocytic leukaemia
----------------------------------------
4. Which marker is expected to be expressed in polyps of patients with mantle cell lymphoma?
A. CD 23
B. BCL6
C. Cyclin D1
D. MYC gene

1319
----------------------------------------
5. What is the likely diagnosis for a patient with nausea, vomiting, hyperuricemia and hypocalcemia
who recently started treatment for Burkitt's lymphoma?
A. Sepsis
B. Tumor Lysis syndrome
C. Vit-D deficiency
D. Hypomagnesemia
----------------------------------------
6. Which of the following statements is true regarding anaplastic large-cell lymphoma?
A. t(2;5) translocation imparts a worsened prognosis
B. ALK protein immunostaining is seen in a minority of cases
C. This tumour occurs exclusively in children
D. Hallmark cells are a characteristic feature
----------------------------------------
7. Which of the following statements about mycosis fungoides is correct?
A. The disease eventually disseminates to lymph nodes and internal organs
B. The neoplastic cells commonly display cell markers of CD19 and CD20
C. The skin rash usually disappears over time
D. This disease is caused by chronic fungal infection in the skin
----------------------------------------
8. Which of the following translocations is associated with Burkitt lymphoma?
A. t(15;17)
B. t(14;18)
C. t(8;14)
D. t(11;14)
----------------------------------------
9. Which of the following genes is involved in causing the most common non-Hodgkins lymphoma
globally?
A. BCL 2
B. BCL 6
C. SOX-11
D. BRAF
----------------------------------------
10. Post-transplant lymphoma occurs due to which of the following cells?
A. T Cell
B. B Cell

Page 2

1320
C. NK Cell
D. Monocyte
----------------------------------------
11. What are the characteristics of Sezary syndrome?
A. Generalized exfoliative erythroderma
B. Circulating Lutzner cells
C. Lymphocytes that express CCR4 and CCR10.
D. All of the above
----------------------------------------
12. Which of the following tumor markers indicates a positive diagnosis of Mantle Cell Lymphoma?
A. SOX11
B. MYD88
C. Annexin V
D. BCL-2
----------------------------------------
13. Activation of which of the following factors is characterized by the presence of the given
characteristic cells (as shown in the image below) in lymphoid tissue?

A. TNF-α
B. NF-kB
C. EGF
D. TNF- β
----------------------------------------
14. Which of the following subtypes of Hodgkin's lymphoma is associated with a lymphohistocytic
variant of Reed Sternberg cells?
A. Lymphocyte-rich HL
B. Lymphocyte predominant HL
C. Nodular sclerosis HL
D. Mixed cellularity HL

Page 3

1321
----------------------------------------
15. Which of the following variants of Hodgkins Lymphoma is associated with HIV?
A. Lymphocyte-rich
B. Lymphocyte-depleted
C. Nodular lymphocyte-predominant
D. Nodular sclerosis
----------------------------------------
16. Which of the following is a marker for myeloid cancers?
A. S100
B. HMB45
C. CD45
D. Cytokeratin
----------------------------------------
17. Which of the following is a characteristic feature of hairy cell leukemia?
A. Sudan black positive
B. Myeloperoxidase positive
C. Low leukocyte alkaline phosphatase levels
D. Tartrate-resistant acid phosphatase positive
----------------------------------------

Correct Answers
Question Correct Answer

Question 1 3
Question 2 1
Question 3 1
Question 4 3
Question 5 2
Question 6 4
Question 7 1
Question 8 3
Question 9 2
Question 10 2
Question 11 4
Question 12 1
Question 13 2

Page 4

1322
Question 14 2
Question 15 2
Question 16 3
Question 17 4

Solution for Question 1:


Correct Option C - BCL-2:
• BCL-2 immunohistochemical marker is expected to be positive in patients with follicular lymphoma..
• Follicular lymphoma is a germinal-centre B cell-derived clonal lymphoid tumour.
• T(14;18) and (q32;q21) are the most prevalent cytogenetic translocations seen. They are seen
in almost 80% of cases.
• The BCL2 gene is translocated to the Ig H locus on the 14th chromosome, leading to a subsequent
overproduction of BCL-2 protein.
• The BCL-2 protein works as an inhibitor of apoptosis, leading to cancerous growth.
Incorrect Options:
Option A - ABL:
• It is seen in chronic myeloid leukemia (CML).
• Translocation (9,22) involves the ABL and BCR genes.
Option B - BAX:
• BAX gene (ABL-2 associated gene) is proapoptotic.
• It is defective in colorectal carcinoma and acute lymphoblastic leukemia.
Option D - Myc:
• It is seen in Burkitt lymphoma.
• The genetic defect is in t(8;14) and is associatedwith an increased expression of c-Myc.

Solution for Question 2:


Correct Option A - Lymph node biopsy:
• The confirmatory test for lymphoma is a histologic diagnosis.
• On microscopy Hodgkin's lymphoma shows reed Sternberg cells-large B cells with two mirror image
nuclei or nuclear lobes, each containing an inclusion-like nucleolus surrounded by a clear zone (Owl
eye nucleus).

Page 5

1323
Incorrect Options:
Option B - Peripheral blood film:
• Blood films are useful in investigating lymphoma in advanced stages of the disease, i.e., stage 4 when
neoplastic cells spill into the blood.
• In advanced stages, immunophenotyping of peripheral blood lymphocytes can be done to confirm the
diagnosis.
Option C - Computed tomography (CT) scan:
• CT scan is a nonspecific investigation that may show a mass or lesion responsible for the patient's
symptoms but won’t confirm the diagnosis.
• It is used for staging rather than the diagnosis of lymphomas.
Option D - Positron emission tomography (PET):
• PET scan measures the metabolic activity of the tissues and can be used to stage the disease and
monitor the response to treatment.
• Diagnosis of lymphoma is histological, and a PET scan would not be useful in making an early
diagnosis of lymphoma.

Solution for Question 3:


Correct Option A - Follicular lymphoma:
• Elevated lymphocytes, LDH, t(14; 18), and centrocytes in the biopsy sample suggest follicular
lymphoma, the most typical form of indolent (low-grade) NHL.
• Chromosome 14;18 translocations are seen in up to 90% of follicular lymphomas that fuse the BCL2
gene on chromosome 18 to the IgH locus on chromosome 14.
• These tumors immunophenotypically express pan-B cell markers CD19, CD20, CD10, and BCL6.

Page 6

1324
• It mainly occurs in adults older than 50 years of age and usually manifests as painless, generalized
lymphadenopathy and easy fatigability. It usually involves lymph nodes, bone marrow, and spleen.

• Histopathologically, small lymphoid cells with irregular or cleaved nuclear outlines (centrocytes) mixed
with a population of larger cells with nucleoli (centroblasts) are found. In most follicular lymphomas,
centrocytes are in the majority.
Incorrect Options:
Option B - Burkitt lymphoma:
• Burkitt lymphoma is an aggressive (high-grade) non-Hodgkin lymphoma (NHL) that commonly occurs
in adolescents and young adults with characteristic jaw lymphadenopathy.
• A translocation involving the MYC gene on chromosome 8 is commonly seen, but variant
translocations on chromosomes 2, 14, and 22 are also observed.
Option C - Diffuse large B cell lymphoma:
• 30% of tumors have a 14;18 translocation involving the BCL2 gene.
• Morphologically, the neoplastic B cells are large with a round or oval nucleus and pale or basophilic
cytoplasm unlike the presentation described here.
Option D - Chronic lymphocytic leukemia:
• Chronic lymphocytic leukemia (CLL) are indolent NHLs commonly occurring in older adults with a 2:1
male predominance.
• CLL's most common genetic anomalies are deletions of 11q, 13q, and 17p and trisomy 12q.
• The lymph nodes are diffusely effaced by small lymphocytes, predominantly with irregular nuclei and
scant cytoplasm.

Solution for Question 4:


Option C:

Page 7

1325
Mantle cell lymphoma is a moderately aggressive NHL commonly occurring in the fifth to sixth decade
of life with male predominance.
• Mantle cell lymphomas have an (11; 14) translocation involving the IGH locus on chromosome 14 and
the cyclin D1 locus on chromosome 11 that leads to overexpression of cyclin D1.
• These lymphomas express high levels of cyclin D1, CD19, and CD20 with moderately high levels of
surface Ig (usually IgM and IgD with κ or λ light chain).
• It is usually CD5 positive and CD23 negative, which helps to distinguish it from CLL/SLL.
• It most commonly presents with painless lymphadenopathy.
• Symptoms related to the involvement of the spleen and gut also are common.
Incorrect Options:
Options A - CD23: CD23 is a B-cell marker, but it is typically not expressed in mantle cell lymphoma. It
is more commonly associated with other B-cell disorders, such as chronic lymphocytic leukemia (CLL).
Options B - CL6: BCL6 is a transcription factor and is not a specific marker for mantle cell lymphoma. It
is more commonly associated with germinal center B-cell lymphomas, including follicular lymphoma an
d diffuse large B-cell lymphoma. Option D
- MYC gene: The MYC gene is involved in various lymphomas, particularly Burkitt lymphoma, it is not a
specific marker for mantle cell lymphoma.

Solution for Question 5:


Correct Option B - Tumor Lysis syndrome:
• Tumor lysis syndrome (TLS) is a potentially life-threatening medical emergency that can occur as a
result of cancer treatment. It is often associated with Burkitt’s Lymphoma. Patients can present with
nausea, vomiting, and altered mental status. Lab investigations reveal hyperuricemia, hyperkalemia,
hyperphosphatemia, hypermagnesemia, and hypocalcemia. This is due to increased destruction of
neoplastic cells.
Incorrect Options:
Options A, C and D are incorrect.

Solution for Question 6:


Option D - Hallmark cells are a characteristic feature:
• CD30 positivity and the presence of Hallmark cells are characteristic features of anaplastic large-cell
lymphoma.
• Hallmark cells are scattered cells with kidney or doughnut-shaped nuclei and an eosinophilic region
adjacent to the nucleus.

• Anaplastic large-cell lymphoma is a type of diffuse large-cell lymphoma.

Page 8

1326
• Hallmark cells are seen in all types of ALCL.

Incorrect Options:
Option A - t(2;5) translocation imparts a worsened prognosis:
• Most cases of ALCL show a t(2;5) translocation.
• It correlates with ALK positivity and improved survival chances.
Option B - ALK protein immunostaining is seen in a minority of cases:
• The presence or absence of ALK protein divides ALCL into ALC positive and ALC negative types.
• ALK-positive ALCL is reported often.
• AKL-negative ACLC has a poor prognosis.
Option C - This tumour occurs exclusively in children:
• The median age of ALCL is 55 years old.
• It is rare in children.

Solution for Question 7:


Correct Option A - The disease eventually disseminates to lymph nodes and internal organs:
Mycosis fungoides presents as a pruritic skin rash on non-sun exposed areas and later progresses to h
ard and raised erythematous plagues, histologically composed of atypical CD4+T cells with cerebriform
nuclei. It is a
malignant condition and will likely spread to lymph nodes and internal organs if not treated.
• Clinical Presentation: Patients present with a pruritic rash that initially appears as scaly,
erythematous, eczematoid patches distributed asymmetrically over the chest and abdomen. Later, the
rash changes to hard and raised plaques. Mycosis fungoides (MF) is the most common subtype of
cutaneous T-cell lymphoma (CTCL).

Page 9

1327
• Patients present with a pruritic rash that initially appears as scaly, erythematous, eczematoid patches
distributed asymmetrically over the chest and abdomen. Later, the rash changes to hard and raised
plaques.
• Mycosis fungoides (MF) is the most common subtype of cutaneous T-cell lymphoma (CTCL).
• Histopathological Findings: The plaque biopsy reveals atypical CD4+ T cells with cerebriform nuclei.
Cerebriform or convoluted nuclei are characteristic of Sézary cells, which are malignant T lymphocytes
seen in MF.
• The plaque biopsy reveals atypical CD4+ T cells with cerebriform nuclei.
• Cerebriform or convoluted nuclei are characteristic of Sézary cells, which are malignant T
lymphocytes seen in MF.
Clinical Presentation:
• Patients present with a pruritic rash that initially appears as scaly, erythematous, eczematoid patches
distributed asymmetrically over the chest and abdomen. Later, the rash changes to hard and raised
plaques.
• Mycosis fungoides (MF) is the most common subtype of cutaneous T-cell lymphoma (CTCL).
Histopathological Findings:
• The plaque biopsy reveals atypical CD4+ T cells with cerebriform nuclei.
• Cerebriform or convoluted nuclei are characteristic of Sézary cells, which are malignant T
lymphocytes seen in MF.
Incorrect Options:
Option B - The neoplastic cells most commonly display cell markers of CD19 and CD20:
• The neoplastic cells in MF typically do not express CD19 and CD20.
• These markers are usually seen in B-Cell lymphoma.
Option C - The skin rash most commonly disappears over time:
• The skin rash associated with MF may persist over time and can progress to more advanced stages
rather than disappearing spontaneously.
Option D - This disease is caused by a chronic fungal infection in the skin:
• MF is not caused by chronic fungal infection in the skin. It is a form of non-Hodgkin lymphoma
involving T lymphocytes.
• The cause of mycosis fungiodes is unknown.

Solution for Question 8:


Correct Option C - t(8;14):
• The likely translocation seen in patients with Burkitt lymphoma is t(8;14).
• Burkitt lymphoma is an aggressive B-cell non-Hodgkin lymphoma most common in children and
young adults, which has two types endemic and sporadic.
• The endemic (African) form is characterized by the involvement of the face, especially the maxilla or
mandible and is associated with EBV.

Page 10

1328
• The sporadic (Western) form usually involves the abdominal organs such as the ileocecum and
peritoneum.
• The tumor cells are mature B cells that express surface IgM, CD19, CD20, CD10, and BCL6, a
phenotype consistent with a germinal centre B-cell origin.
• All forms of Burkitt lymphoma are associated with translocations of the MYC gene on chromosome 8.
• MYC translocation partner is usually IGH locus [t(8;14)], but may also be Ig κ [t(2;8)] or Ig λ [t(8;22)]
light chain loci.
• In Burkitt lymphoma, involved tissues are effaced by a diffuse infiltrate of intermediate-sized lymphoid
cells. Cells have a high mitotic index; numerous apoptotic cells are observed. The apoptotic remnants
are phagocytosed by many tingible body macrophages, which have abundant clear cytoplasm
scattered in between tumor cells giving rise to a 'starry-sky' pattern.
Incorrect Options:
Option A - t(15;17):
• This translocation is associated with acute promyelocytic leukemia (APL), which typically presents
with features such as bleeding tendencies, coagulopathy, and bone marrow involvement.
Option B - t(14;18):
• This translocation is associated with follicular lymphoma, a low-grade B-cell lymphoma characterized
by painless lymphadenopathy and bone marrow involvement.
Option D - t(11;14):
• This translocation is associated with mantle cell lymphoma, which typically presents with
lymphadenopathy, splenomegaly, and bone marrow involvement.

Solution for Question 9:


Correct Option B - BCL 6:
• The most common non-Hodgkin's lymphoma (NHL) worldwide is Large B-cell lymphoma
• Diffuse large B-cell lymphoma (DLBCL) commonly shows BCL6 gene rearrangements, which can
contribute to the pathogenesis of the disease. BCL6 rearrangements often lead to dysregulated
expression of the BCL6 protein, which plays a role in the development and progression of DLBCL.
Incorrect Options:
Options A, C and D are not typically associated with Large B-cell lymphoma.

Solution for Question 10:


Correct Option B - B Cell
• Post-transplant lymphomas commonly originate from B cells.
• After transplantation, immunosuppressive therapy suppresses the recipient's immune system to
prevent rejection of the transplanted organ or cells. This suppression can lead to the reactivation of
latent infections and impair the immune surveillance against malignancies.

Page 11

1329
• B cells are particularly susceptible to transformation in this context due to the dysregulation of immune
surveillance mechanisms.
• Post-transplant lymphomas can manifest as various subtypes, including diffuse large B-cell lymphoma
(DLBCL), Burkitt lymphoma, and primary central nervous system lymphoma (PCNSL), among others.
• Post-Transplant Lymphoproliferative Disorder (PTLD) and EBV: EBV as a Causative Agent:
• The majority of PTLD cases are associated with Epstein-Barr Virus (EBV).
• In immunocompetent individuals, primary EBV infection is usually self-limiting without significant
complications.
• After EBV infection, the virus may remain latent in B-cell lymphocytes, expressing a partial EBV
genome.
• Alternatively, it may lead to viral replication and B-cell lysis.
• In immunocompetent patients, cytotoxic T-cells control B-cell proliferation.
• PTLD in Immunocompromised Patients:
• In immunocompromised individuals (post-transplant recipients), unchecked B-cell activation and
continuous proliferation occur, leading to PTLD.
• Immunosuppressive therapy, given to prevent organ rejection in transplant recipients, can impair
immune surveillance against EBV-infected B cells.
Post-Transplant Lymphoproliferative Disorder (PTLD) and EBV:
EBV as a Causative Agent:
PTLD in Immunocompromised Patients:
INCORRECT OPTIONS:

Options A, C and D are incorrect and the correct option is explained above.

Solution for Question 11:

Page 12

1330
Correct Option D - All of the above:
• (All the options are true about Sezary syndrome)
• Option A) Generalized exfoliative erythroderma: Sezary syndrome typically presents with intense,
widespread erythroderma, which refers to a generalized redness and scaling of the skin affecting most
of the body surface area. Option B) Circulating Lutzner cells: Lutzner cells are atypical lymphocytes that
are characteristic of Sezary syndrome. They are found circulating in the blood and are considered a
diagnostic feature of the disease. Option C) Lymphocytes that express CCR4 and CCR10: Sezary
syndrome is associated with the expression of certain chemokine receptors on the surface of malignant
lymphocytes. Specifically, these cells express CCR4 and CCR10, which are involved in their migration
to the skin and lymph nodes.
(All the options are true about Sezary syndrome)
Option A) Generalized exfoliative erythroderma: Sezary syndrome typically presents with intense, wide
spread erythroderma, which refers to a
generalized redness and scaling of the skin affecting most of the body surface area.
Option B) Circulating Lutzner cells: Lutzner cells are atypical lymphocytes that are characteristic of Sez
ary syndrome. They are found circulating in the blood and are considered a
diagnostic feature of the disease.
Option C) Lymphocytes that express CCR4 and CCR10: Sezary syndrome is associated with the expre
ssion of certain chemokine receptors on the surface of malignant lymphocytes. Specifically, these cells
express CCR4 and CCR10, which are involved in their migration to the skin and lymph nodes.

Solution for Question 12:


Correct Option A - SOX11:
• SOX11 is a tumor marker that indicates a positive diagnosis of Mantle Cell Lymphoma. It is a
transcription factor that is also present in the cells of mantle cell lymphoma and can be used for the
diagnosis of MCL
Incorrect Options:
Options B, C and D are not characteristically associated with mantle cell lymphoma.

Solution for Question 13:


Correct Option B - NF-kB:
• The picture shows a prominent Reed-Sternberg (RS cell) typical of classical Hodgkin lymphoma.

• NF-kappa B is a transcription factor that plays a key role in regulating the expression of genes
involved in inflammation, cell survival, and immune responses.
• In Hodgkin's lymphoma, the activation of NF-kappa B is thought to contribute to the survival and
proliferation of Reed-Sternberg cells.

Page 13

1331
NF-kappa B is a transcription factor that plays a key role in regulating the expression of genes involved
in inflammation, cell survival, and immune responses.
In Hodgkin's lymphoma, the activation of NF-kappa B
is thought to contribute to the survival and proliferation of Reed-Sternberg cells.
Incorrect Options:
Options A, C and D are incorrect. and the correct option is explained above.

Solution for Question 14:


Correct Option B - Lymphocyte-predominant HL:
• Nodular Lymphocyte Predominance HL is characterized by the effacement of lymph nodes by nodules
composed mainly of small lymphocytes, with variable numbers of macrophages.
• In this variant, the classic Reed-Sternberg cells are typically scarce, and instead, the tumor contains
L&H; (lymphohistocytic) variants, which are cells with multilobed nuclei resembling popcorn kernels.
• Unlike in other HL subtypes, eosinophils and plasma cells are usually scant or absent in Nodular
Lymphocyte Predominance HL.
• (Lymphocyte-predominant HL)

Page 14

1332
Incorrect Options:
Options A, C and D are incorrect. Refer to the explanation of Option A

Solution for Question 15:


Correct Option B - Lymphocyte-depleted:
• It has the strongest association with HIV.
• Lymphocyte depleted Hodgkin lymphoma occurs predominantly in older adults, in HIV-positive
individuals of any age.
• It is the least common subtype of Hodgkin lymphoma and accounts for less than 5% of cases.
• These cells are positive for CD15 and CD30. These cells are infected with EBV in 90% of cases.
• The lymph node biopsy shows a pleomorphic or reticular variant of Reed-Sternberg cells with reactive
cells.
• The patient usually presents with advanced-stage and systemic symptoms such as fever, night
sweats and unexplained weight loss.
• It has the worst prognosis.
Incorrect Options:
Options A, C and D are incorrect.

Solution for Question 16:

Page 15

1333
Correct Option C - CD45:
• Myeloid malignancies are a group of clonal disorders of the hematopoietic stem or progenitor cells.
• These include: Acute myeloid leukemia Chronic myeloid leukemia Polycythemia vera Essential
thrombocythemia Primary myelofibrosis Mastocytosis Myelodysplastic syndromes
• Acute myeloid leukemia
• Chronic myeloid leukemia
• Polycythemia vera
• Essential thrombocythemia
• Primary myelofibrosis
• Mastocytosis
• Myelodysplastic syndromes
• Common leukocyte antigen, also known as CD45, is a pan-leukocyte marker expressed on all cells of
hematopoietic origin, including myeloid cells.
• It is commonly used to diagnose hematopoietic neoplasms, including myeloid leukemias.
• Acute myeloid leukemia
• Chronic myeloid leukemia
• Polycythemia vera
• Essential thrombocythemia
• Primary myelofibrosis
• Mastocytosis
• Myelodysplastic syndromes
Correct Options:
Option A - S100:
• S100 are a group of low molecular weight proteins.
• It is a marker of malignant melanoma and schwannoma.
• S100 is a protein marker used in immunohistochemistry to identify cells of neural origin, such as
melanomas, Schwannomas, and Langerhans cell histiocytosis. It is not specific to myeloid cancers.
Option B - HMB45:
• Human melanoma black 45 (HMB45) is a membrane protein.
• It is a marker of melanoma.
• HMB45 is an immunohistochemical marker used to identify melanocytic tumors, especially
melanomas. It is not specific to myeloid cancers.
Option D - Cytokeratin:
• Cytokeratins are proteins found on epithelial cells.
• They are markers for epithelial cell tumors.
• They are intermediate filament proteins found in epithelial cells. They are used as markers for
carcinomas of epithelial origin, such as lung, breast, and colon cancers. They are not specific to

Page 16

1334
myeloid cancers.

Solution for Question 17:


Correct Option D - Tartrate-resistant acid phosphatase positive:
• The enzyme tartrate-resistant acid phosphatase (TRAP) is useful in the differential diagnosis of hairy
cell leukemia (HCL).
• The test is positive in 95% of patients with hairy cell leukemia.
• In some cases of splenic marginal zone lymphoma, TRAP is positive.
Incorrect Options:
Option A - Sudan black positive:
• Sudan black stain is positive in acute myeloid and acute lymphoid leukemia.
• It is not positive in hairy cell leukemia.
Option B - Myeloperoxidase positive:
• Myeloperoxidase is positive in acute myeloid leukemia.
• It is not positive in hairy cell leukemia.
Option C - Low leukocyte alkaline phosphatase levels:
• The LAP (leukocyte alkaline phosphatase) score is low in chronic myeloid leukemia and paroxysmal
nocturnal hemoglobinuria.
• However, the LAP score frequently rises when CML progresses to a blast crisis or accelerated phase.
• LAP is usually increased in hairy cell leukemia.

Page 17

1335
Platelets
1. A 55-year-old male presented with complaints of fever, dizziness, headache, and intermittent
bleeding from the nose for three days. He also complained of a red-purple rash on his upper and lower
extremities. He is currently on chemotherapy for colon carcinoma. Physical examination shows signs of
pallor and jaundice. CBC shows hemoglobin at 7.8g/dl and platelet count at 30,000/mm3. Peripheral
blood smear reveals the characteristic finding as shown below. Coagulation studies show mildly
prolonged PT and aPTT. ADAMTS13 activity is at 9%. CT-brain reveals no abnormality. Given this
scenario, the following histopathological finding is associated with which of the following?

A. Thrombotic thrombocytopenic purpura (TTP)


B. Hereditary spherocytosis
C. Hemoglobin C disease
D. Paroxysmal nocturnal hemoglobinuria
----------------------------------------
2. A 3-year-old male presents with complaints of the passage of dark-colored urine with reduced urine
output for two days. Physical examination shows signs of pallor, jaundice, and a distended abdomen.
He is tachycardic and tachypneic. Investigations show anemia, thrombocytopenia, and elevated blood
urea nitrogen and serum creatinine. Urinalysis shows hemoglobinuria and proteinuria. The peripheral
blood smear shows the presence of helmet cells. Which of the following will be the most likely
diagnosis?
A. Hemolytic uremic syndrome
B. Polysplenia syndrome
C. Spherocytosis
D. Acanthocytosis
----------------------------------------
3. Four patients have presented to the pathological laboratory for a complete blood count. Patient A has
been diagnosed with Henoch Schonlein purpura, Patient B has Wiskott Aldrich syndrome Patient C has
Immune thrombocytopenia, Patient D has dengue fever. Which of the following patients will have a
normal platelet count?
A. Henoch Schonlein purpura
B. Wiskott Aldrich syndrome
C. Immune thrombocytopenia

1336
D. Dengue fever
----------------------------------------
4. Which of the following laboratory findings will be seen in a patient with vonWillebrand’s disease?
A. Normal platelet count and normal bleeding time
B. Normal platelet count and increased bleeding time
C. Decreased platelet count and increased bleeding time
D. Normal platelet count and decreased bleeding time
----------------------------------------
5. A 37-year-old male presents to the clinic complaining of easy bruising and bleeding while brushing
his teeth. His platelet count is 2,50,000 per microliter of blood, and his bleeding time is 8 minutes. On
Aggregometry the sample fails to aggregate on addition of ADP, collagen, and epinephrine. This
disease is characterized by which of the following?
A. An inherited defect in glycoprotein GpIIb-IIIa
B. An inherited defect in glycoprotein GpIb-IIa
C. Autosomal dominant inheritance
D. X-linked recessive inheritance
----------------------------------------
6. Which platelet-related thrombogenic process is most directly harmed in von Willebrand factor
deficiency?
A. Platelet adhesion
B. Formation of fibrinogen bridges
C. Release reaction
D. Fibrinolysis
----------------------------------------
7. Which of the following diseases involve a defective Gp1b/IX?
A. Bernard Soulier syndrome
B. ITP
C. Glanzmann thrombasthenia
D. Gray platelet syndrome
----------------------------------------
8. Which of the following statements are true regarding immune thrombocytopenic purpura? A. Most of
the patients respond to immunosuppressive doses of glucocorticoids. B. Relapse is rare. C.
Splenectomy is the treatment of choice for relapse. D. A minority of patients have refractory forms that
are difficult to treat.
A. A, B, and C are true
B. A, C, D are true
C. A, B, D are true
D. B, C, D are true

Page 2

1337
----------------------------------------
9. Which of the following statements regarding Factor XIII is not true?
A. It is also known as the Laki Lorand Factor
B. Urea clot solubility test : Clot is soluble in deficient factor 13
C. In Factor XIII deficient patient BT, CT are normal
D. None of the above
----------------------------------------
10. A 10-year-old male presents to the clinic with a history of recurrent nosebleeds and easy bruising.
His mother reports a family history of similar bleeding tendencies. Laboratory tests reveal a prolonged
activated partial thromboplastin time (aPTT), while the bleeding time (BT) and prothrombin time (PT)
are within normal limits. Given the clinical scenario consider the following statements: The scenario is
least indicative of Von Willebrand Disease. Bleeding time is a sufficient rule to put VWD as a differential
diagnosis.
A. 1 and 2 are true
B. 1 is true and 2 is false
C. 1 and 2 are False
D. 2 is true and 1 is false
----------------------------------------

Correct Answers
Question Correct Answer

Question 1 1
Question 2 1
Question 3 1
Question 4 2
Question 5 1
Question 6 1
Question 7 1
Question 8 2
Question 9 4
Question 10 1

Solution for Question 1:


Correct Option A - Thrombotic thrombocytopenic purpura (TTP):
• The presenting symptoms are consistent with the classic pentad of TTP, which includes fever,
neurological symptoms (such as dizziness and headache), thrombocytopenia (manifested by bleeding),
and microangiopathic hemolytic anemia. While not all patients with TTP will present with all five
components, the combination of fever, neurological symptoms, and bleeding raises suspicion for TTP.

Page 3

1338
• The presence of a red-purple rash on the upper and lower extremities is characteristic of purpura,
which is commonly seen in TTP due to small vessel thrombosis and resultant hemorrhage.
• Pallor suggests anemia, which is a common feature of TTP due to hemolysis. Jaundice may occur
due to the increased breakdown of red blood cells and subsequent elevation of bilirubin levels.
• The presence of schistocytes on the peripheral blood smear confirms microangiopathic hemolytic
anemia manifested by low Hb and jaundice, a hallmark of TTP.
• The mildly prolonged PT and aPTT may be seen in TTP due to the consumption of coagulation factors
and platelets in microthrombi formation.
• ADAMTS13 activity is significantly decreased at 9%, which is consistent with severe deficiency seen
in TTP. ADAMTS13 is an enzyme responsible for cleaving large von Willebrand factor (vWF) multimers.

• TTP Is thrombotic microangiopathy in which the formation of autoantibodies or genetic mutations


results in the deficiency of ADAMTS13, causing the decreased breakdown of the von Willebrand factor.
This results in increased platelet adhesion, erythrocyte fragmentation, schistocyte formation,
microthrombosis, and hemolytic anemia.
• Deficiency of ADAMTS13 leads to the accumulation of ultra-large vWF multimers, promoting platelet
aggregation and microthrombi formation in small blood vessels.
• The above picture shows schistocytes which are usually a feature of microangiopathic hemolytic
anemia, as shown in the picture below with the arrow marks.

Incorrect Options:
Option B - Hereditary spherocytosis:
• It is the most common congenital hemolytic disorder, which is caused, in most cases, by an autosomal
dominant gene defect resulting in defects in RBC membrane protein. Peripheral smear reveals
sphere-shaped RBCs with decreased membrane stability, resulting in extravascular hemolysis.
• This condition typically presents with anemia, jaundice, and splenomegaly. While anemia and
jaundice are present in both TTP and hereditary spherocytosis, splenomegaly is not mentioned in the
clinical scenario. Additionally, hereditary spherocytosis is characterized by the presence of spherocytes
on the peripheral blood smear, which is not described in this case. Therefore, hereditary spherocytosis

Page 4

1339
is less likely.
• It is mostly seen in infancy or childhood and is often asymptomatic.
Option C - Hemoglobin C disease:
• This condition is characterized by the presence of hemoglobin C on hemoglobin electrophoresis and
may present with mild hemolytic anemia. However, it typically does not cause thrombocytopenia,
microangiopathic hemolytic anemia, or neurological symptoms such as dizziness and headache.
Therefore, hemoglobin C disease is an unlikely diagnosis in this case.
Option D - Paroxysmal nocturnal hemoglobinuria (PNH):
• PNH is characterized by intravascular hemolysis, leading to hemoglobinuria, thrombosis, and
cytopenias. While both TTP and PNH can present with hemolytic anemia and thrombocytopenia, the
absence of hemoglobinuria and the presence of severely decreased ADAMTS13 activity favor TTP
over PNH. Additionally, the presence of schistocytes on the peripheral blood smear is more indicative
of TTP than PNH.

Solution for Question 2:


Correct Option A - Hemolytic uremic syndrome:
• Hemolytic Uremic Syndrome is a condition characterized by the triad of microangiopathic hemolytic
anemia, thrombocytopenia, and acute kidney injury. It is commonly associated with a prodrome of
gastrointestinal symptoms, such as diarrhea, particularly in cases caused by Shiga toxin-producing
Escherichia coli (STEC) infections.
• Clinical Presentation: The patient presents with signs of hemolytic anemia (pallor, jaundice), renal
dysfunction (reduced urine output, elevated BUN and creatinine), and thrombocytopenia. These
findings are consistent with HUS.
• Urinalysis: The presence of hemoglobinuria and proteinuria on urinalysis is suggestive of intravascular
hemolysis and kidney injury, which are characteristic features of HUS.
• Peripheral Blood Smear: The presence of helmet cells (schistocytes) on the peripheral blood smear
indicates mechanical hemolysis, which is typical in HUS.
Incorrect Options:
Option B - Polysplenia syndrome:
• Polysplenia syndrome is a rare congenital condition characterized by multiple small spleens, not
typically associated with the symptoms and findings described in the case.
Option C - Spherocytosis:
• Hereditary spherocytosis is a congenital hemolytic anemia caused by defects in red blood cell
membrane proteins. While it can cause anemia and jaundice, it is not typically associated with the
acute kidney injury and thrombocytopenia seen in HUS.
Option D - Acanthocytosis:
• Acanthocytosis refers to the presence of abnormally shaped red blood cells (acanthocytes) and is
associated with conditions like neuroacanthocytosis syndromes. It is not typically associated with the
constellation of symptoms seen in HUS
• Given the clinical presentation, laboratory findings, and peripheral blood smear results, Hemolytic
Uremic Syndrome (HUS) is the most likely diagnosis.

Page 5

1340
Given the clinical presentation, laboratory findings, and peripheral blood smear results, Hemolytic Ure
mic Syndrome (HUS) is the most likely diagnosis.

Solution for Question 3:


Correct Option A - Henoch Schonlein purpura:
• All other options have decreased platelet count except Henoch Schonlein purpura.
• Henoch-Schönlein purpura is a systemic immune disorder of unknown cause.
• It is characterized by purpuric rash, colicky abdominal pain, polyarthralgia, and acute
glomerulonephritis with IgA deposition in the glomerular mesangium, but the platelet count is normal.
• All these changes result from the deposition of circulating immune complexes within blood vessels
throughout the body and within the glomerular mesangial regions.
• Most common age group affected are children, 3-8 years old
• It occurs in adults with severe renal manifestations.
Option B - Wiskott Aldrich syndrome:
• WAS is a rare X-linked recessive disorder characterized by dermatitis, thrombocytopenia (low platelet
count), immunological insufficiency (Low IgM levels, normal IgG levels, and elevated IgA & IgE levels),
progressive loss of T cells in the peripheral blood, and bloody diarrhea (secondary to
thrombocytopenia).
• It is also known as an eczema-thrombocytopenia-immunodeficiency syndrome.
Option C - Immune thrombocytopenia:
• ITP is described as isolated thrombocytopenia (platelet count < 1L) with normal white blood cells and
hemoglobin in the presence of a widespread purpuric rash.
• ITP used to be called idiopathic thrombocytopenic purpura or immune thrombocytopenic purpura.
• Primary ITP is defined as ITP that does not have a secondary etiology or underlying condition.
• Secondary ITP is described as ITP with an underlying cause or condition, which can be drug-induced
or caused by a systemic ailment (e.g. SLE, HIV, CVID (Common Variable Immunodeficiency), etc.).
Option D - Dengue fever:
• Dengue fever is a tropical mosquito-borne illness caused by the dengue virus.
• Symptoms appear three to fourteen days following infection. These symptoms may include a high
temperature, headache, vomiting, muscular and joint problems, and a distinctive itch and rash on the
skin.
• In most cases, recovery takes two to seven days. In a tiny percentage of patients, the sickness
progresses to dengue hemorrhagic fever, which causes bleeding due low levels of platelets and blood
plasma leakage, or to dengue shock syndrome.

Solution for Question 4:


Correct Option B - Normal platelet count and increased bleeding time:

Page 6

1341
• Von Willebrand disease is a bleeding disorder caused by a lack of the pro-von Willebrand factor,
either qualitatively or quantitatively.
• People affected may experience severe bruising, prolonged bleeding from mucosal areas, and
prolonged bleeding following mild trauma.
• Deficiency of Von Willebrand factor (VWF), which promotes platelet adherence and activation, causes
↓ Platelet adhesion ( ↑BT ) ↓ Intrinsic pathway activity ( ↑ aPTT) ↓ Factor VIII in plasma, (this is due to
instability the instability of factor VIII secondary to vWD)
• ↓ Platelet adhesion ( ↑BT )
• ↓ Intrinsic pathway activity ( ↑ aPTT)
• ↓ Factor VIII in plasma, (this is due to instability the instability of factor VIII secondary to vWD)
• Von Willebrand Disease is of 3 types: Type 1&3 are quantitative defects in the factor and Type 2 is
qualitative defects in the factor.
• Type 1&3 are quantitative defects in the factor and Type 2 is qualitative defects in the factor.
• Clinical features include; Mucosal bleeding → Petechiae /purpura /epistaxis / melena Tissue bleeding
• Mucosal bleeding → Petechiae /purpura /epistaxis / melena
• Tissue bleeding
• Treatment includes; Desmopressin for the mild form Cryoprecipitate for the severe form
• Desmopressin for the mild form
• Cryoprecipitate for the severe form
• ↓ Platelet adhesion ( ↑BT )
• ↓ Intrinsic pathway activity ( ↑ aPTT)
• ↓ Factor VIII in plasma, (this is due to instability the instability of factor VIII secondary to vWD)
• Type 1&3 are quantitative defects in the factor and Type 2 is qualitative defects in the factor.
• Mucosal bleeding → Petechiae /purpura /epistaxis / melena
• Tissue bleeding
• Desmopressin for the mild form
• Cryoprecipitate for the severe form
Incorrect Options:
Options A, C, and D are incorrect. Refer to the explanation of Option B.

Solution for Question 5:


Correct Option A - Inherited defect in glycoprotein GpIIb-IIIa:
• Glanzmann thrombasthenia is characterized by a defect in glycoprotein GpIIb-IIIa, which is essential
for platelet aggregation. In this condition, platelets fail to aggregate properly due to the dysfunctional
GpIIb-IIIa complex, resulting in impaired clot formation and increased bleeding tendency. Therefore, the
inability of platelets to aggregate in response to various agonists strongly suggests Glanzmann
thrombasthenia as the underlying diagnosis.

Page 7

1342
• The patient's presentation of easy bruising and bleeding, along with laboratory findings of a normal
platelet count but prolonged bleeding time, suggests a platelet dysfunction disorder rather than a
quantitative platelet disorder like thrombocytopenia.
• The key clincher in the question is the failure of platelet aggregation on aggregometry in response to
multiple agonists (ADP, collagen, and epinephrine). This indicates a defect in platelet function rather
than a deficiency in specific platelet receptors or mediators. ADP and collagen aggregation tests are
done for aggregation defects.

Incorrect Options:
Options: B, C, and D are incorrect

Solution for Question 6:


Option A - Platelet adhesion:
• Endothelial injury leads to transient vasoconstriction and endothelin release from the damaged cells.
VWF binds to the exposed collagen. Platelets bind vWF via the GP Ib receptor at the site of injury.

Page 8

1343
• Von Willebrand disease is characterized by a decrease in the levels of von Willebrand factor.
• Von Willebrand factor (vWF) is stored in specialized endothelial cell organelles called Weibel-Palade
bodies. These structures are found within endothelial cells lining blood vessels and are responsible for
storing and releasing vWF when needed
• Signs and symptoms of Von Willebrand disease include large bruises, epistaxis, menorrhagia,
bleeding gums, and heavy bleeding after cuts.
• Decreased vWF leads to platelet plug formation defects due to platelet-to-vWF adhesion.
• A deficiency of Von Willebrand factor (VWF) causes decreased platelet adhesion and increases the
bleeding time of the patient.
• Intrinsic coagulation defect leads to an increase in aPTT as vWF carries and protects factor 8.
• Platelet count and PT remain normal.
• There is no coagulation with the ristocetin coagulation assay.
• Treatment includes desmopressin, which releases vWF stored in the endothelium.
Incorrect Options: Option B - Formation of fibrinogen bridges:
• Low fibrinogen levels may lead to defects in the formation of fibrinogen bridges.
• Fibrinogen binds glycoprotein IIb/IIIa and links the platelets.
Option C - Release reaction:
• Platelets are reactive to different stimuli and release various mediators stored in their granules. This is
known as the release reaction.
• It is an important step in primary hemostasis.
Option D - Fibrinolysis:
• Plasmin is an important enzyme that activates fibrinolysis.
• Defects in fibrinolysis are seen in liver cirrhosis, amyloidosis, acute promyelocytic leukemia, some
solid tumors

Page 9

1344
Solution for Question 7:
Correct Option A - Bernard Soulier Syndrome:
• Bernard-Soulier syndrome involves a defective Gp1b/IX complex. This syndrome is a rare inherited
bleeding disorder .
• Patients with Bernard-Soulier syndrome often have prolonged bleeding times, low hemoglobin levels
due to excessive bleeding, and abnormal platelet aggregation responses to various agonists, including
ADP, collagen, and epinephrine.
• The defect affects the Gp1b/IX complex, which is responsible for platelet adhesion to damaged blood
vessel walls.
Incorrect Options:
Options B, C and D are incorrect.

Solution for Question 8:


Correct Option B - A, C, and D are true:

Chronic ITP:
Immune thrombocytopenic purpura (ITP) involves low platelet counts due to autoantibodies, primarily o
f the IgG class, targeting platelet membrane glycoproteins. Specifically glycoproteins IIb–IIIa or Ib–IX,
with about 80% of patients showing antibodies bound to the platelet surface.
• Duration: Longer duration history, often persistent over time.
• Severity: Generally less severe compared to acute ITP.
• Population: Typically seen in adults.
Clinical Features (C/F):
• Petechiae: Small, red or purple spots on the skin caused by minor bleeding from broken capillaries.
• Purpura: Larger areas of bleeding under the skin, appearing as purple or red patches.
• Hemorrhagic bullae: Blisters filled with blood due to bleeding into the skin layers.
• Gum bleeding: Bleeding from the gums, often seen during brushing or eating.
• Hematuria: Presence of blood in the urine, visible or detected through laboratory tests.
• Melena: Black, tarry stools resulting from the presence of blood in the digestive tract.
Diagnosis:
• ITP is a diagnosis of exclusion, meaning other potential causes of thrombocytopenia need to be ruled
out.
• Laboratory findings include an increased bleeding time and decreased platelet count.
• Prothrombin time (PT) and activated partial thromboplastin time (aPTT) are typically normal.

Page 10

1345
• Coombs test may be positive due to the presence of autoantibodies against platelets.
Autoantibodies are often directed against platelet membrane glycoproteins IIb-IIIa or Ib-IX.
Rx –
Initial treatment
Glucocorticoids ± IVIG/ Anti D ± platelets (if bleeding)
Relapsed/ Persistent ITP
Rituximab/ Anti D/ IVIG/ Thrombopoietin receptor agonist
Worsening ITP
Splenectomy
Incorrect Options:
Options A, C and D are incorrect. Refer to the explanation of Option B

Solution for Question 9:


Correct Option D - None of the above:

• Option A states that factor XIII is also known as the Laki Lorand Factor. This is true. Factor XIII is
indeed referred to as the Laki Lorand Factor after the scientists who discovered it.
• Option B mentions the urea clot solubility test, stating that the clot is soluble in deficient factor XIII.
This is also true. In the urea clot solubility test, a clot formed in the presence of calcium ions and
thrombin is dissolved by urea. If factor XIII is deficient, the clot will be more easily dissolved by urea,
indicating a positive test.
• Option D suggests that in patients deficient in factor XIII, bleeding time (BT) and clotting time (CT) are
normal. This is also accurate. Factor XIII deficiency primarily affects the stability of the fibrin clot rather
than its formation. Therefore, BT and CT are typically normal in factor XIII deficiency.

Page 11

1346
Option A states that factor XIII is also known as the Laki Lorand Factor. This is true. Factor XIII is indee
d referred to as the Laki Lorand Factor after the scientists who discovered it.
Option B mentions the urea clot solubility test, stating that the clot is soluble in deficient factor XIII. This
is also true. In the urea clot solubility test, a clot formed in the presence of calcium ions and thrombin i
s dissolved by urea. If factor XIII is deficient, the clot will be more easily dissolved by urea, indicating a
positive test.
Option D suggests that in patients deficient in factor XIII, bleeding time (BT) and clotting time (CT) are
normal. This is also accurate. Factor XIII deficiency primarily affects the stability of the fibrin clot rather
than its formation. Therefore, BT and CT are typically normal in factor XIII deficiency.
Incorrect Options:
Refer to the explanation under the correct option.

Solution for Question 10:


Correct Option A - 1 and 2 are true:
• 1-The scenario is least indicative of Von Willebrand Disease (VWD) because the bleeding time (BT) is
within normal limits, which is more characteristic of hemophilia.
• 2-Bleeding time (BT) is mentioned to be within normal limits. This is relevant in distinguishing between
hemophilia and VWD, as a normal BT is more consistent with hemophilia.
Incorrect Options:
Option B - 1 is true and 2 is false:
• This option is incorrect because both statements 1 and 2 are true. The scenario is least indicative of
Von Willebrand Disease, and bleeding time is indeed relevant in distinguishing between hemophilia and
VWD.
Option B - 1 and 2 are False:
• This option is incorrect because both statements 1 and 2 are true. The scenario is least indicative of
Von Willebrand Disease, and bleeding time is indeed relevant in distinguishing between hemophilia and
VWD.
Option D - 2 is true and 1 is false:
• This option is incorrect because both statements 1 and 2 are true. The scenario is least indicative of
Von Willebrand Disease, and bleeding time is indeed relevant in distinguishing between hemophilia and
VWD.

Page 12

1347
Previous Year Questions
1. Why is blood stored in citrate-phosphate-dextrose preferred over acid-citrate-dextrose for hypoxic
patients?
A. It has less P50
B. It is less acidic
C. The fall in 2,3-DPG is less
D. None of the above
----------------------------------------
2. What could be the possible cause for a 24-year-old man who was involved in a road traffic accident
(RTA) and arrived at the emergency department after 6 hours with a blood pressure of 90/60 mm Hg,
heart rate of 110 beats per minute, and oxygen saturation (SpO2) of 92%? The patient received a blood
transfusion, but after some time, he developed a fever, increased respiratory rate, and started bleeding
from the intravenous (IV) and nasogastric (NG) tube insertion sites. Additionally, his SpO2 levels began
to decline.
A. Transfusion reaction
B. Acute adrenal crisis
C. Fat embolism
D. Hemorrhagic shock
----------------------------------------
3. What is the longest time frame within which TRALI can manifest after a blood transfusion?
A. 12 hours
B. 24 hours
C. 6 hours
D. 48 hours
----------------------------------------
4. What is the color of the vacutainer that contains sodium fluoride?
A. Red
B. Grey
C. Blue
D. Yellow
----------------------------------------
5. In your ER, a patient has been admitted with a critical bleeding condition. The blood group of the
patient is unidentified. Which blood groups' fresh frozen plasma can be administered to this patient?
A. O
B. AB
C. B
D. A

1348
----------------------------------------
6. A CKD patient had to undergo dialysis and his Hb was 5.2 gm / dL. So two blood transfusions were
to be given. The first bag was completed in 2 hours. The second was started and in the midway, he
developed shortness of breath and hypertension. Vitals were BP- 180/120 mmHg and pulse rate –
110/min. what is the cause ?
A. Allergic
B. Transfusion related circulatory overload
C. Transfusion – related acute lung injury
D. Febrile non – hemolytic transfusion reaction
----------------------------------------
7. A patient who receives recurrent transfusions for thalassemia develops fever and chills during this
time. There is no evidence of hemolysis. Which of the following measure can decrease the rate of
transfusion reactions?
A. Leukoreduced RBCs
B. Antibiotics
C. Irradiation
D. Washed RBCs
----------------------------------------
8. What does the vacutainer tube depicted in the provided image contain?

A. Na-EDTA
B. Trisodium citrate
C. NAF-heparin
D. Clot activator
----------------------------------------
9. What condition is associated with an extended duration of bleeding?
A. Hemophilia A
B. Hemophilia B
C. Von Willebrand disease
D. Both (b) and (c)

Page 2

1349
----------------------------------------
10. Leiden mutation is of factor:
A. V
B. VIII
C. I
D. X
----------------------------------------
11. In the emergency department, a child presented with a sizable lesion on their forearm. As a surgery
resident, your decision is to remove the lesion in the emergency operation theater. Which of the
subsequent investigations will assess the complete coagulation pathway?
A. Activated partial thromboplastin time
B. Prothrombin time
C. Thromboelastography
D. Bleeding time
----------------------------------------
12. Which test evaluates the extrinsic pathway?
A. APTT
B. PT
C. TT
D. Bleeding time
----------------------------------------
13. Which of the following is not a haemostatic agent?
A. Zeolite
B. Chitosan
C. Boric acid
D. Kaolin
----------------------------------------
14. Which of the following is the major source of plasma von – Willebrand factor ?
A. Platelets
B. Lymphocytes
C. Macrophages
D. Endothelial cells
----------------------------------------
15. Which of the following deficiencies in clotting factors will have no impact on clotting in the living
organism?
A. Factor VII

Page 3

1350
B. Factor V
C. Factor XII
D. Factor IX
----------------------------------------
16. Please arrange the following clotting factors in the sequence that they follow in the coagulation
cascade: 13 12 10 5 9
A. 2,3,4,1,5
B. 2,5,3,4,1
C. 1,2,5,3,4
D. 2,3,5,4,1
----------------------------------------
17. Which of the following conditions is most likely to be the cause of the severe injuries, ruptured
spleen, low blood pressure (80/60 mm Hg), and metabolic acidosis in a 24-year-old male who was
brought to the emergency department after a motorcycle accident?
A. Cardiogenic shock
B. Septic shock
C. Hypovolemic shock
D. Neurogenic shock
----------------------------------------
18. What condition can be avoided by administering irradiated red blood cells?
A. Graft versus host disease
B. HLA Alloimmunization
C. Immunomodulation
D. Transfusion Related Acute Lung Injury (TRALI)
----------------------------------------
19. What staining technique is used to identify acid mucin?
A. PAS
B. Alcian blue
C. Masson trichrome
D. PTAH
----------------------------------------
20. What does the following image show ?

Page 4

1351
A. Autophagy
B. Emperipolesis
C. Phagocytosis
D. Amoebic autophagy
----------------------------------------
21. Which of the following statements is accurate concerning liquid-based cytology?
A. Urine specimen is used
B. Sanger sequencing is used
C. Study of fluid in tumour microenvironment
D. DNA is collected in microvessels
----------------------------------------
22. Which of the following is not a crosslinking fixative?
A. Glutaraldehyde
B. Formaldehyde
C. Methanol
D. Osmium tetraoxide
----------------------------------------
23. Stellate granuloma is found in:
A. Cat scratch disease
B. Leprosy
C. Coccidomycosis
D. Histoplasmosis
----------------------------------------
24. What is the term used to describe the type of cell in the body that has the ability to give rise to the
highest number of other cells?
A. Totipotent
B. Multipotent

Page 5

1352
C. Pluripotent
D. Lineage stem cells
----------------------------------------
25. What is the fixation technique employed to examine fat in histopathology?
A. IHC
B. Frozen section
C. Liquid paraffin
D. Formalin fixed
----------------------------------------
26. A patient was transfused 4 units of blood 2 hours ago, and now she complains of chest discomfort.
The chest X-ray shows bilateral fissural thickening. Which of the following is the most probable cause?
A. ABO incompatibility reactions
B. Fluid overload
C. Hypocalcemia
D. Graft-versus-host disease
----------------------------------------
27. A young patient presents with a large retroperitoneal hemorrhage. He also gives a frequent history
of intermittent knee swelling during strenuous exercise. There is no history of any mucosal bleeding.
Which of the following deficiency is responsible for the condition mentioned above?
A. Factors VIII and IX
B. GP1b/IX
C. von Willebrand factor
D. Lupus anticoagulant
----------------------------------------

Correct Answers
Question Correct Answer

Question 1 3
Question 2 1
Question 3 3
Question 4 2
Question 5 1
Question 6 2
Question 7 1
Question 8 4
Question 9 3

Page 6

1353
Question 10 1
Question 11 3
Question 12 2
Question 13 3
Question 14 4
Question 15 3
Question 16 2
Question 17 3
Question 18 1
Question 19 2
Question 20 2
Question 21 1
Question 22 4
Question 23 1
Question 24 1
Question 25 2
Question 26 2
Question 27 1

Solution for Question 1:


Correct Option C - The fall in 2,3-DPG is less:
• 2,3-DPG is a molecule that binds to hemoglobin and helps facilitate the release of oxygen to tissues.
• When blood is stored, the levels of 2,3-DPG gradually decrease over time.
• A lower concentration of 2,3-DPG causes a leftward shift of the oxygen dissociation curve, resulting in
decreased oxygen release to tissues.
• This can be detrimental for hypoxic patients who already have compromised oxygen delivery.
Incorrect Options:
Options A, B, and D are incorrect. Refer to the explanation of the correct answer.

Solution for Question 2:


Correct Option A:
• Transfusion reaction refers to an adverse immune response that occurs following a blood transfusion.
It can result from incompatibility between the donor's blood and the recipient's blood, leading to
activation of the recipient's immune system. Transfusion reactions can manifest with various symptoms
depending on the type of reaction.

Page 7

1354
• In this case, the patient's symptoms of fever, tachypnea, bleeding from IV and NG tube sites, and
falling SpO2 levels are suggestive of a transfusion reaction. Febrile and respiratory symptoms can be
seen in febrile non-hemolytic transfusion reactions, which are immune-mediated reactions caused by
cytokine release. Bleeding from the sites of IV and NG tube insertion can occur due to
thrombocytopenia resulting from immune-mediated destruction of platelets. Falling SpO2 levels can be
attributed to pulmonary edema, which can occur in severe transfusion reactions.
Incorrect Options:
Option B: Acute adrenal crisis: Acute adrenal crisis, also known as adrenal insufficiency or Addisonian
crisis, occurs due to a sudden decrease in cortisol production by the adrenal glands. It can present with
symptoms such as severe fatigue, hypotension, electrolyte abnormalities, and metabolic disturbances.
While hypotension can be a feature of acute adrenal crisis, the bleeding manifestations and falling Sp
O2 levels described in the case are not consistent with this diagnosis.
Option C: Fat embolism: Fat embolism occurs when fat globules enter the bloodstream and obstruct s
mall blood vessels, leading to tissue ischemia. It typically occurs after long bone fractures or extensive
soft tissue trauma. Common symptoms include respiratory distress, petechial rash, and neurological ab
normalities. While fat embolism can cause respiratory symptoms, the bleeding manifestations describe
d in the case are not characteristic of this condition.
Option D: Hemorrhagic shock: Hemorrhagic shock refers to a state of inadequate tissue perfusion due
to significant blood loss. It is characterized by hypotension, tachycardia, and signs of tissue hypoperfus
ion. While bleeding is a hallmark of hemorrhagic shock, the presence of other symptoms such as fever,
falling SpO2 levels, and bleeding from IV and NG tube sites suggests an immune-mediated reaction ra
ther than pure hemorrhage.

Solution for Question 3:


Correct Option C: 6 hours
• Transfusion-related acute lung injury (TRALI) is a potentially severe complication that can occur after
blood transfusion.
• It is characterized by the sudden onset of respiratory distress and non-cardiogenic pulmonary edema.
• TRALI typically manifests within 6 hours after blood transfusion.
Incorrect Options:
Option A: 12 hours- TRALI is more likely to occur within the first 6
hours after blood transfusion rather than within 12 hours.
Option B: 24 hours- TRALI is less likely to occur after 6
hours and is typically observed within the early post-transfusion period, rather than within 24 hours.
Option D: 48 hours- TRALI is highly unlikely to occur after 6
hours and is very uncommon beyond the first day after blood transfusion.

Solution for Question 4:


Correct Option B.

Page 8

1355
• Vacutainers are tubes used for blood collection and storage. They come in different colors, each
indicating the type of additive or anticoagulant present in the tube. Sodium fluoride is commonly used
as an additive in vacutainers to prevent glycolysis (the breakdown of glucose) in blood samples.
• The vacutainer tube that contains sodium fluoride as an additive typically has a grey color. The
grey-top vacutainer is specifically designed for glucose and lactate determinations, as sodium fluoride
acts as a glycolysis inhibitor. It is important to use the appropriate color-coded vacutainer tube for
specific tests to ensure accurate results.
Incorrect Option
Option A. Red:
• The red-top vacutainer tube is typically used for blood collection without any additive or anticoagulant.
It is commonly used for serum separation and various laboratory tests that require a clot to form. The
absence of any additive in the red-top tube allows the blood to clot, and the resulting serum can be
used for analysis.
Option C. Blue:
• The blue-top vacutainer tube is commonly used for coagulation tests. It contains sodium citrate as an
anticoagulant. Sodium citrate prevents blood from clotting by binding to calcium ions, thereby
maintaining the blood in its liquid form. The blue color of the tube indicates the presence of sodium
citrate.
Option D. Yellow:
• The yellow-top vacutainer tube is typically used for blood culture collection. It contains a specific
culture media to support the growth of microorganisms if there is an infection suspected. The yellow
color of the tube indicates its use for blood culture purposes.

Solution for Question 5:


Correct Option A: Fresh frozen plasma (FFP) from blood group O
can be given to patients with unknown blood groups as a universal plasma donor. Blood group O
individuals are considered universal plasma donors because their plasma lacks both A and B antigens
on red blood cells, making it compatible with recipients of any ABO blood type without causing an ABO
incompatibility reaction. This is why FFP from blood group O
is often used in emergency situations when the patient's blood type is unknown.
It's important to note that while FFP from blood group O is compatible with all blood types, it only contai
ns plasma components and is not suitable for red blood cell transfusions. If red blood cells are needed,
their compatibility with the patient's blood type should be determined separately.

Incorrect Options:
Option B. AB: Fresh frozen plasma (FFP) from blood group AB can be given to individuals with blood g
roup AB because they can receive plasma from any ABO blood type. However, in this scenario where t
he patient's blood group is unknown, it's not the preferred choice. If the patient's blood group happens t
o be A, B, or O, there could be a potential issue with ABO compatibility.
Option C. Blood group B: Individuals with blood group B have B
antigens on their red blood cells and have anti-A antibodies in their plasma. If a

Page 9

1356
patient with an unknown blood group receives FFP from blood group B, there is a
risk of transfusion reaction if the patient has blood group A
and the anti-A antibodies in the donated plasma react with the patient's A antigens.
Option D. Blood group A: Individuals with blood group A have A
antigens on their red blood cells and have anti-B antibodies in their plasma. If a
patient with an unknown blood group receives FFP from blood group A, there is a risk of transfusion re
action if the patient has blood group B, and the anti-B antibodies in the donated plasma react with the p
atient's B antigens.

Solution for Question 6:


Correct Option B:
Transfusion-related circulatory overload (TACO) occurs when a patient receives blood or fluid too quick
ly. In the given scenario, the patient undergoing dialysis received two blood transfusions. overwhelming
the cardiovascular system's capacity to handle the increased volume. Symptoms of TACO include sho
rtness of breath, hypertension, elevated blood pressure, and fluid overload. These symptoms are consi
stent with the presentation of the patient in the scenario.
Incorrect Options:
Option A: Allergic reaction: An allergic reaction to the blood transfusion is an incorrect option. While all
ergic reactions can occur during transfusions, they typically present with symptoms such as itching, ras
h, hives, or anaphylaxis. Shortness of breath and hypertension are not typical symptoms of an allergic r
eaction.
Option C: Transfusion-related acute lung injury (TRALI): Transfusion-related acute lung injury is an inc
orrect option. TRALI is a rare but potentially serious complication of blood transfusion characterized by
acute respiratory distress and pulmonary edema. While shortness of breath is a symptom of TRALI, hy
pertension and elevated blood pressure are not typically associated with this condition.
Option D: Febrile non-hemolytic transfusion reaction: Febrile non-hemolytic transfusion reaction is an i
ncorrect option. This reaction is characterized by fever and chills during or after a blood transfusion, us
ually due to antibodies in the recipient's blood reacting to donor leukocytes. Hypertension and shortnes
s of breath are not typical symptoms of this reaction.

Solution for Question 7:


Correct Option A.
• Leukoreduced RBCs: Option A is the correct option.
• Leukoreduced red blood cells (RBCs) refer to blood products that have undergone a process to
remove or reduce the number of white blood cells (leukocytes).
• White blood cells can trigger immune responses and cause transfusion reactions, including febrile
non-hemolytic transfusion reactions (FNHTRs).
• It reduces risk of non hemolytic transfusion reaction.
• Reduce risk of alloimmunization
• Reduce risk CMV transmission

Page 10

1357
• By reducing the number of leukocytes in the transfused blood, the likelihood of developing fevers and
chills can be decreased.
Incorrect Options:
Option B: Antibiotics: Antibiotics are an incorrect option. Antibiotics are medications used to treat bacte
rial infections and are not directly related to preventing transfusion reactions. They are not effective in d
ecreasing the rate of transfusion reactions in the scenario described.
Option C: Irradiation: Irradiation is an incorrect option. Irradiation refers to the process of exposing bloo
d products to radiation, which helps to prevent graft-versus-host disease (GVHD) by inactivating donor
lymphocytes. While irradiation is important in certain situations, such as transfusions from a blood relati
ve or when blood products are intended for immunocompromised recipients, it is not directly related to
preventing fevers and chills in transfusion reactions without evidence of hemolysis.
Option D: Washed RBCs: Option D
is an incorrect option. Washed red blood cells (RBCs) are blood products that have undergone a washi
ng process to remove plasma, platelets, and other substances. This technique is primarily used to rem
ove substances that may cause allergic reactions or other complications in certain individuals with kno
wn sensitivities. However, it is not specifically effective in decreasing the rate of transfusion reactions c
haracterized by fever and chills in the absence of hemolysis.

Solution for Question 8:


Correct Option D:
• The vacutainer tube shown in the image contains a clot activator. A clot activator is a substance
added to the tube to promote blood clotting. It initiates the clotting process by activating coagulation
factors, which leads to the formation of a clot. This type of tube is commonly used for collecting blood
samples for serum separation, where the blood is allowed to clot, and then the serum is separated from
the clot by centrifugation.
Incorrect Options:
Option A: Na-EDTA: Na-EDTA stands for sodium ethylenediaminetetraacetic acid. It is an anticoagulan
t commonly used in purple-top tubes for collecting blood samples for hematological tests. It prevents bl
ood from clotting by binding to calcium ions, which are necessary for the coagulation process.
Option B: Trisodium citrate: Trisodium citrate is an anticoagulant commonly used in blue-top tubes for c
oagulation studies, such as prothrombin time (PT) and activated partial thromboplastin time (aPTT). It
acts by binding to calcium ions and inhibiting the coagulation cascade.
Option C: NAF-heparin: NAF-heparin stands for lithium heparin with sodium fluoride. It is a combination
anticoagulant used in gray-top tubes for collecting blood samples for glucose testing. Sodium fluoride i
nhibits glycolysis, preserving blood glucose levels, while heparin prevents clotting.

Solution for Question 9:


Correct Option C: Von Willebrand disease
• Prolonged bleeding time is seen in von Willebrand disease.

Page 11

1358
• Von Willebrand disease is a bleeding disorder caused by a deficiency or dysfunction of von Willebrand
factor (VWF), a protein involved in platelet adhesion and blood clotting.
Incorrect Options:
Option A: Hemophilia A- Hemophilia A is a genetic disorder caused by a
deficiency of clotting factor VIII. APTT is elevated in hemophilia A
Option B: Hemophilia B- Hemophilia B, also known as Christmas disease, is a
genetic disorder caused by a deficiency of clotting factor IX.APTT is elevated in hemophilia B
Option D: Both (b) and (c)- This is incorrect.

Solution for Question 10:


Correct Option A: V
• Leiden mutation is a genetic mutation of factor V, also known as Factor V Leiden. Factor V is a protein
involved in the blood clotting process. The Leiden mutation refers to a specific mutation in the factor V
gene, which leads to an increased risk of abnormal blood clot formation.
Incorrect Options:
Option B: Factor VIII- Factor VIII is another protein involved in the blood clotting process, but it is not a
ssociated with the Leiden mutation.
Option C: Factor I- Factor I, also known as fibrinogen, is a
protein involved in the formation of blood clots, but it is not associated with the Leiden mutation.
Option D: Factor X- Factor X is another protein involved in the blood clotting process, but it is not assoc
iated with the Leiden mutation.

Solution for Question 11:


Correct Option C:
Thromboelastography (TEG) is a global coagulation test that assesses the entire coagulation pathway,
including both the intrinsic and extrinsic pathways, as well as fibrinolysis. It provides a
comprehensive evaluation of clot formation, clot strength, and clot lysis.
TEG measures the viscoelastic properties of blood as it clots, allowing for the assessment of various p
arameters such as clotting time, clot formation time, clot strength (represented by maximum amplitude)
, and fibrinolysis. By analyzing these parameters, TEG provides valuable information about the function
ality of the coagulation system and helps guide clinical decision-making during surgical procedures.
Incorrect Options:
Option A: Activated partial thromboplastin time (aPTT) primarily evaluates the intrinsic pathway of the c
oagulation cascade. It measures the time taken for clot formation when specific activators are added to
the patient's plasma. While aPTT provides information about certain coagulation factors and their inter
actions, it does not assess the entire coagulation pathway.
Option B: Prothrombin time (PT) primarily assesses the extrinsic pathway of the coagulation cascade. I
t measures the time taken for clot formation when tissue factor and calcium are added to the patient's p
lasma. PT is commonly used to evaluate the activity of vitamin K-dependent factors, primarily Factor VI

Page 12

1359
I. While PT provides information about specific factors and their interactions, it does not test the entire
coagulation pathway.
Option D: Bleeding time measures the time taken for bleeding to stop after a standardized skin incision
. It primarily assesses platelet function and primary hemostasis. Bleeding time does not evaluate the co
agulation cascade or provide information about specific factors involved in clot formation.

Solution for Question 12:


Correct Option B.
PT (Prothrombin Time) evaluates the extrinsic pathway of coagulation. It measures the time it takes for
blood to clot in response to activating agents that initiate the extrinsic pathway, primarily involving fact
ors VII, X, V, II (prothrombin), and fibrinogen.
Incorrect Option:
Option A: APTT (Activated Partial Thromboplastin Time) evaluates the intrinsic pathway of coagulation.
It measures the time it takes for blood to clot in response to activating agents that initiate the intrinsic p
athway.
Option C: TT (Thrombin Time) measures the time it takes for fibrinogen to be converted into fibrin by th
e action of thrombin. It evaluates the final common pathway of coagulation and is used to assess the a
ctivity of thrombin.
Option D: Bleeding time assesses the primary hemostasis, specifically the ability of platelets to form a
platelet plug and initiate clot formation. It measures the time it takes for bleeding to stop after a
standardized skin incision.

Solution for Question 13:


Correct Option C.
Boric acid: Boric acid is not commonly used as a hemostatic agent. It is primarily used in various indust
rial and pharmaceutical applications, but it does not have specific hemostatic properties.
Incorrect Option
Option A: Zeolite: Zeolite is a mineral-based compound that has been used as a
hemostatic agent. It works by absorbing water and promoting clot formation at the site of bleeding.
Option B. Chitosan: Chitosan is a biopolymer derived from chitin, which is found in the exoskeleton of c
rustaceans. Chitosan-based products have been used as hemostatic agents due to their ability to prom
ote clotting and accelerate wound healing.
Option D. Kaolin: Kaolin is a naturally occurring clay mineral that has hemostatic properties. It acts by a
ctivating the intrinsic pathway of the coagulation cascade and promoting clot formation.

Solution for Question 14:


Correct Option D.

Page 13

1360
This option is correct. Endothelial cells, which line the blood vessels, are the primary source of plasma
von Willebrand factor. These cells synthesize and release vWF into the bloodstream. von Willebrand fa
ctor produced by endothelial cells plays a
crucial role in blood clotting and platelet adhesion to damaged blood vessel walls
Incorrect Options:
Option A: Platelets: While platelets do contain von Willebrand factor, they are not the major source of p
lasma vWF. Platelets store vWF within their granules and release it upon activation to facilitate platelet
adhesion and clot formation. However, platelet-derived vWF represents a
minor fraction compared to the vWF produced by endothelial cells.
Option B: Lymphocytes: Lymphocytes are a
type of white blood cell involved in the immune response. They do not play a significant role in the prod
uction or release of von Willebrand factor. Lymphocytes are primarily associated with the adaptive imm
une system and are not a major source of plasma vWF.
Option C: Macrophages: Macrophages are phagocytic immune cells involved in the clearance of cellula
r debris and pathogens. While macrophages can produce and release various substances, they are not
a major source of plasma von Willebrand factor. Macrophages are not known for their involvement in v
WF production.

Solution for Question 15:


Correct option C:
• Factor XII, also known as Hageman factor, is not essential for normal clotting in vivo. It is part of the
intrinsic pathway of the coagulation cascade. Activation of Factor XII initiates a series of reactions
leading to the formation of a blood clot. However, in vivo, the absence or deficiency of Factor XII does
not significantly affect normal clotting because other factors and pathways compensate for its
deficiency.
Incorrect Options:
Option A: Deficiency of Factor VII, also known as stable factor, leads to impaired clotting in vivo. It play
s a crucial role in the initiation of the extrinsic pathway of the coagulation cascade.
Option B: Deficiency of Factor V, also known as labile factor, results in impaired clotting in vivo. It serve
s as a cofactor for Factor X in the formation of the prothrombinase complex.
Option D: Deficiency of Factor IX, also known as Christmas factor, leads to impaired clotting in vivo. It i
s a crucial factor in the intrinsic pathway and is required for the activation of Factor X.

Solution for Question 16:


Correct Option B
• The coagulation cascade involves a series of sequential activation and interactions of various clotting
factors, leading to the formation of a stable fibrin clot.
• Factor 2 (Prothrombin) is converted to Factor 2a (Thrombin) through the action of Factor 5 (Labile
factor or Proaccelerin) and Factor 10

Page 14

1361
• Factor 5 is activated to Factor 5a by Factor 10a, which is generated by the tissue factor/Factor 7a
complex. Factor 5a then acts as a cofactor for the conversion of Factor 2 to Factor 2a (Thrombin).
• Factor 3 plays a critical role in initiating the extrinsic pathway of coagulation by forming a complex with
Factor 7, leading to the activation of Factor 7 to Factor 7a. This, in turn, activates Factor 10 to Factor
10a.
• Factor 4 (Calcium ions) is required as a cofactor for the activation of Factors 2 (Prothrombin) and 10
(Stuart-Prower factor) to their active forms.
• Factor 1 (Fibrinogen) is converted to fibrin by the action of Factor 2a (Thrombin). Fibrin then forms a
stable clot by polymerizing and crosslinking.
• Therefore, the correct order of the clotting factors in the coagulation cascade is 2, 5, 3, 4, 1, which
corresponds to option B.
Incorrect options:
Option A. 2, 3, 4, 1, 5: This option incorrectly places Factor 5 (Labile factor) before Factor 3
(Tissue factor) in the sequence of the coagulation cascade.
Option C. 1, 2, 5, 3, 4: This option places Factor 1
(Fibrinogen) at the beginning of the sequence, which is incorrect. Factor 1
is the final product of the coagulation cascade.
Option D. 2, 3, 5, 4, 1: This option incorrectly places Factor 5 (Labile factor) before Factor 3
(Tissue factor) and Factor 4 (Calcium ions) in the sequence of the coagulation cascade.

Solution for Question 17:


Correct Option C:
In the above case, the patient presents to the emergency after suffering severe injuries in a
motorcycle accident. His BP is low and USG showed a ruptured spleen which caused internal bleeding
leading to hypovolemic shock. Hemorrhage, fluid loss, severe burn, perspiration in excess, and diarrhe
a are some of the events that lead to hypovolemic shock that occurs due to significant loss of fluid from
the vascular compartment.
Incorrect Options:
Option A: In cardiac shock, the failure of cardiac pumps lead to the condition.
Option B: There is no evidence of infection and no history of fever hence septic shock is unlikely.
Option D: Neurogenic shock occurs due to spinal cord injuries which is not seen above.

Solution for Question 18:


Option a. Graft versus host disease: Transfusion of irradiated red blood cells (RBCs) can prevent graft
versus host disease (GVHD). GVHD is a
rare but severe complication that occurs when immunocompetent donor T cells in the transfused blood
recognize the recipient's tissues as foreign and mount an immune response against them. This immun
e response can lead to tissue damage and various systemic manifestations.

Page 15

1362
• Irradiation of blood products, including RBCs, is performed to prevent the proliferation of lymphocytes
in the transfused blood. The irradiation process damages the DNA of lymphocytes, rendering them
unable to divide and mount an immune response. By inhibiting the activity of donor lymphocytes,
irradiation reduces the risk of GVHD in susceptible recipients, particularly those who are
immunocompromised or receive blood products from close relatives.
Incorrect Choices:
Option b. HLA Alloimmunization: Transfusion of irradiated RBCs does not prevent HLA (human leukoc
yte antigen) alloimmunization. HLA alloimmunization occurs when a recipient's immune system recogni
zes HLA antigens on transfused RBCs as foreign and produces antibodies against them. Irradiation do
es not affect HLA antigens on RBCs or prevent the recipient's immune response to HLA incompatibility.
Other measures, such as matching HLA types between donor and recipient, are taken to minimize HL
A alloimmunization.
Option c. Immunomodulation: Transfusing irradiated RBCs does not prevent immunomodulation. Immu
nomodulation refers to suppressing or altering the recipient's immune response due to blood transfusio
n. It can manifest as a decrease in the recipient's immune function or an increased susceptibility to infe
ctions or tumors. Irradiation of RBCs is specifically performed to prevent the proliferation of lymphocyte
s and reduce the risk of GVHD, but it does not directly affect other aspects of immunomodulation.
Option d. Transfusion Related Acute Lung Injury (TRALI):
• Transfusion of irradiated RBCs does not prevent transfusion-related acute lung injury
• (TRALI). TRALI is a severe reaction characterized by acute respiratory distress and lung injury
following blood transfusion. It is believed to be caused by the infusion of antibodies or biologically active
substances in blood products, leading to an immune response and lung inflammation. Preventing
TRALI involves careful selection of blood donors, particularly avoiding those with a history of producing
antibodies associated with TRALI. Irradiation of RBCs does not directly address the underlying
mechanisms of TRALI.

Solution for Question 19:


Correct Option: B
• Alcian blue staining is specifically used to detect and visualize acidic mucopolysaccharides, such as
acid mucin. It is often employed to identify mucin-producing cells, mucous glands, and mucin-rich
connective tissues. Alcian blue staining results in blue or bluish-green color, indicating the presence of
acid mucin.
Incorrect Options:
Option A. PAS (Periodic Acid-Schiff): PAS staining is used to detect and highlight the presence of carb
ohydrates, including glycogen and mucopolysaccharides. It is commonly used to stain structures such
as basement membranes, glycogen granules, and fungal organisms. PAS staining results in a
pink-to-magenta coloration.
Option C. Masson trichrome: Masson trichrome staining is a versatile staining technique used to differe
ntiate various tissue components. It involves staining collagen fibers blue, cytoplasm and muscle fibers
red, and nuclei dark red to black. While Masson trichrome staining can help visualize connective tissu
es and collagen, it does not specifically target acid mucin.

Page 16

1363
Option D PTAH (Phosphotungstic acid-hematoxylin): PTAH staining is commonly used to highlight mus
cle fibers and fibrin. It is particularly useful in diagnosing muscle disorders and identifying abnormalities
in muscle structure. PTAH staining does not specifically target acid mucin and would not be the stainin
g of choice for acid mucin detection.

Solution for Question 20:


Correct Option: B
• Emperipolesisis a cellular process in which one cell is engulfed and contained within the cytoplasm of
another cell without being degraded. It is a non-destructive process, unlike phagocytosis or autophagy,
where the engulfed material is typically broken down and degraded.
Incorrect Options:
Option A. Autophagy: Autophagy is a
cellular process that involves the degradation and recycling of cellular components. It is a mechanism
by which cells remove damaged organelles, misfolded proteins, and other cytoplasmic constituents. Au
tophagy plays a
crucial role in maintaining cellular homeostasis and responding to various stress conditions.
Option C. Phagocytosis: Phagocytosis is a process by which cells, such as macrophages and neutroph
ils, engulf and internalize solid particles, such as bacteria, cellular debris, or foreign substances. The e
ngulfed material is then enclosed within a membrane-bound vesicle called a
phagosome, which fuses with lysosomes for degradation and destruction of the engulfed material.
Option D. Amoebic autophagy: Amoebic autophagy refers to the autophagic process observed in amoe
bae, which are single-celled organisms. It involves the engulfment and degradation of cellular compone
nts or particles by the amoebae to obtain nutrients or eliminate unwanted materials.

Solution for Question 21:


Correct Option A:
Liquid-based cytology is a technique used for collecting and preparing cell samples for microscopic exa
mination. It involves the collection of cells from various body fluids, such as cervical or vaginal fluids, re
spiratory secretions, or urine, depending on the specific diagnostic purpose.
Liquid-based cytology is commonly used for cervical cancer screening, where cells are collected from t
he cervix using a brush or spatula. The collected cells are then rinsed in a liquid medium, such as a
preservative solution, to create a suspension of cells. This suspension is used to prepare slides for cyt
ological examination, which involves analyzing the cells under a
microscope for any abnormalities or signs of disease.
Incorrect Options:
Option B. Sanger sequencing is used: Sanger sequencing is a
method used for DNA sequencing, which involves determining the exact order of nucleotides in a DNA
molecule. Liquid-based cytology does not involve DNA sequencing but rather focuses on the collection
and examination of cells.
Option C. Study of fluid in tumor microenvironment: Liquid-based cytology primarily focuses on the exa
mination of individual cells rather than the study of the fluid in the tumor microenvironment. It is more c

Page 17

1364
ommonly used for the screening and diagnosis of various diseases, including cancer.
Option D. DNA is collected in microvessels: Liquid-based cytology does not specifically involve the coll
ection of DNA in microvessels. It primarily focuses on the collection of cells from body fluids, which are
then processed for cytological examination.

Solution for Question 22:


Correct Option D : Osmium tetraoxide
Osmium tetroxide is a strong oxidizing agent and is used primarily as a staining agent in transmission e
lectron microscopy (TEM) for biological specimens. It is not primarily used as a crosslinking fixative. Gl
utaraldehyde, formaldehyde (also known as formalin), and methanol are commonly used crosslinking fi
xatives in biological and histological sample preparation. These fixatives help preserve the structure of
biological tissues by forming chemical crosslinks between proteins and other macromolecules, preventi
ng degradation and maintaining the tissue's integrity for further analysis.
Incorrect Options:
Option A. Glutaraldehyde: Glutaraldehyde is a widely used crosslinking fixative in biological research a
nd electron microscopy. It forms crosslinks between proteins and preserves cell structures effectively.
Option B. Formaldehyde (Formalin): Formaldehyde, commonly known as formalin when dissolved in w
ater, is another common crosslinking fixative used to preserve tissues for histology. It forms crosslinks
between proteins, stabilizing cellular structures.
Option C. Methanol: Methanol is used in combination with formaldehyde in fixatives like Carnoy's soluti
on. It helps in preserving both nuclear and cytoplasmic structures by forming crosslinks.
These fixatives work by preserving the structure of biological specimens, allowing scientists to study tis
sues and cells under various microscopes without significant distortion or degradation.

Solution for Question 23:


Correct option: A
• Stellate granuloma is a histopathological finding characterized by central necrosis surrounded by
histiocytes, giant cells, and lymphocytes arranged in a stellate or star-shaped pattern. It is commonly
associated with cat scratch disease, which is caused by the bacterium Bartonella henselae. In cat
scratch disease, stellate granulomas can be found in the affected lymph nodes, indicating the immune
response to the infection.
Incorrect options:
Option B: Leprosy: Stellate granulomas are not a characteristic finding in leprosy. Leprosy is associate
d with granulomatous inflammation but presents with different histopathological features.
Option C: Coccidiomycosis: Coccidiomycosis is a fungal infection caused by Coccidioides species and
is not typically associated with stellate granulomas.
Option D: Histoplasmosis: Histoplasmosis is a fungal infection caused by Histoplasma capsulatum and
does not typically present with stellate granulomas.

Page 18

1365
Solution for Question 24:
Correct option: A
• Totipotent cells are capable of differentiating into all cell types, including the cells of the embryo and
the extraembryonic tissues. They have the ability to form all cell lineages, both embryonic and
extraembryonic. Totipotent cells are found in the early stages of embryonic development, up to the
4-cell stage. They have the highest developmental potential and can give rise to a complete organism.
Incorrect options:
Option B: Multipotent: Multipotent cells are capable of differentiating into multiple, but not all, cell types.
They can give rise to cells of a closely related cell lineage or tissue type.
Option C: Pluripotent: Pluripotent cells can differentiate into cells of all three germ layers (endoderm, m
esoderm, and ectoderm) but cannot form the extraembryonic tissues.
Option D: Lineage stem cells: This term is not commonly used in the context of cell potency. It may ref
er to stem cells that are capable of differentiating into specific lineages but not all cell types.

Solution for Question 25:


Correct Option B:
• In the histopathological examination, different fixation techniques are used to preserve and prepare
tissue samples for microscopic analysis. When it comes to examining fat in histopathology, the
preferred technique is the frozen section.
• Frozen section is a rapid fixation technique that involves freezing the tissue sample using liquid
nitrogen or a cryostat. This technique allows the preservation of tissue architecture and cellular details,
including fat droplets. By freezing the tissue, the lipids within the fat cells are solidified and can be
visualized under a microscope.
• The frozen section technique is particularly useful when evaluating the presence of fat in various
tissues, such as adipose tissue tumors, lipomas, or the assessment of fat content in organs like the
liver. It provides a quick and reliable method to assess fat distribution and morphology.
Incorrect Options:
Option A. IHC (Immunohistochemistry): This technique is used to detect specific proteins or antigens in
tissue sections using specific antibodies. While IHC can be used to identify certain markers associated
with fat cells, it is not a fixation technique specifically used for examining fat.
Option C. Liquid paraffin: Liquid paraffin is not a fixation technique but rather a mounting medium used
to cover the tissue sections on glass slides. It provides optical clarity and preserves the specimen. It do
es not specifically fix or preserve fat.
Option D. Formalin fixed: Formalin fixation is a widely used technique for preserving tissue samples in
histopathology. While it can preserve overall tissue structure, it is not the preferred method for preservi
ng fat. Formalin fixation can cause partial extraction of fat from the tissue, leading to artifacts and loss
of fat droplets during the processing steps.

Page 19

1366
Solution for Question 26:
Correct Option B - Fluid overload:
• The patient has chest discomfort after transfusing 4 units of blood, and a chest X-ray showing bilateral
fissural thickening is suggestive of fluid overload.
Incorrect Options:
Options A, C, and D are incorrect.

Solution for Question 27:


Correct Option A - Factors VIII and IX:
• The patient's presentation with a large retroperitoneal hemorrhage, a frequent history of intermittent
knee swelling during strenuous exercise, and no history of any mucosal bleeding is suggestive of
hemophillia, i.e., factor VIII and IX deficiency.
Incorrect Options:
Option B
- GP1b/IX: GP1b/IX deficiency, i.e., Bernard-Soulier syndrome, presents with mucosal bleeding.
Option C - von Willebrand factor: von Willebrand factor deficiency presents with mucosal bleeding.
Option D - Lupus anticoagulant: It is seen in women with a history of recurrent abortion.

Page 20

1367
Blood Banking and Blood Transfusion Medicine
1. A 22-year-old male presents to the emergency room after he suffered a road traffic accident. His
blood pressure is 88/67 mmHg. He appears pale, and his extremities are cold. A FAST scan is positive
for free fluid in the peritoneum, suggestive of splenic rupture. An immediate laparotomy is done, and
two packs of blood are transfused intra-operatively. After the transfusion, the patient develops a fever.
The resident suspects a transfusion reaction. Which of the following statements is true regarding the
suspected transfusion reaction?
(or)
Which of the following statements are true regarding the features of febrile non-hemolytic transfusion
reaction (FNHTR)?
A. Occurs within 2 hours of red cells and platelet transfusion
B. Frequency decreases with the storage age of the product
C. The pathogenesis is non-cytokine mediated
D. Frequency is increased by measures that limit donor leukocyte contamination.
----------------------------------------
2. Which blood product carries minimal risk of ABO incompatibility?
A. Fresh frozen plasma
B. Platelet-rich plasma
C. Single donor platelets
D. Cryoprecipitate
----------------------------------------
3. Which of the following complications is most likely to occur, immediately after transfusing several
units of blood?
A. Metabolic alkalosis
B. Metabolic acidosis
C. Respiratory alkalosis
D. Respiratory acidosis
----------------------------------------
4. A 56-year-old male accident victim in the emergency room is in urgent need of blood. Since the
patient is in severe hemorrhagic shock, a blood sample for blood grouping cannot be obtained. The
patient's emergency blood transfusion should include which of the following:
A. RBC corresponding to his red cell group and colloids/ crystalloid
B. Whole blood corresponding to his plasma group
C. O-positive RBC and colloids/crystalloid
D. AB-negative whole-blood
----------------------------------------
5. Choose the correct blood product and its suitable storage temperature.
A. Packed red blood cells are stored at 2°C -6°C

1368
B. Platelets are stored at -18°C
C. Fresh frozen plasma is stored at -12°C
D. Cryoprecipitate is stored at 20°C -24°C
----------------------------------------
6. Of the given options what size needle is best for transfusions of blood products?
A. 26 G needle
B. 18 G needle
C. 22 G needle
D. 24 G needle
----------------------------------------
7. Which of the following can be found in cases of mismatched blood transfusion?
A. Elevated plasma haptoglobin level
B. Negative direct antiglobulin test (DAT)
C. Hemoglobinuria
D. Decreased serum LDH level
----------------------------------------
8. Which of the following blood products is correctly paired with its corresponding shelf life?
A. At 20°C, platelets have a shelf-life of one year
B. Fresh frozen plasma at -18°C is one year
C. Whole blood with CPDA-1 solution at 4 degrees C is 5 days
D. Cryoprecipitate at -18°C is 21 days
----------------------------------------
9. Which of the following clotting factors are present in cryoprecipitate?
A. Factor VIII, von Willebrand factor, fibrinogen (Factor I), Factor XIII, fibronectin
B. Factor VIII, von Willebrand factor, fibrinogen (Factor I), Factor XIII, factor V
C. Factor VIII, von Willebrand factor, factor V, Factor XIII, fibronectin
D. Factor VIII, factor V, fibrinogen (Factor I), Factor XIII, fibronectin
----------------------------------------
10. What is the first blood component to be separated during centrifugation?
A. Red blood cells
B. Platelet concentrates
C. Fresh frozen plasma
D. Cryoprecipitate
----------------------------------------
11. Which of the following statements regarding the Duffy antigen is correct?

Page 2

1369
A. Associated withdifferentiation factor 234 (CD234)
B. Located on the ribosomes of RBCs
C. Glycosylated cytoplasmic protein
D. Associated withdifferentiation factor 123 (CD123)
----------------------------------------
12. After receiving fresh frozen plasma, a patient suddenly starts coughing. She rapidly develops fever
and severe dyspnea, with Spo2 of 60% on room air. Her blood pressure fell to 90/40mmHg. The
patient’s D-dimer levels were within normal limits. Plain radiographs of the chest and echocardiogram
reveal features of noncardiogenic pulmonary edema. What is the likely diagnosis?
A. Febrile non-hemolytic transfusion reaction
B. Transfusion-related acute lung injury (TRALI)
C. Transfusion-associated circulatory overload (TACO)
D. Pulmonary embolism
----------------------------------------
13. A 35-year-old female patient was hospitalized for a blood transfusion as she is a known case of
chronic kidney disease. She was transfused with packed red blood cells, and within eight hours of the
treatment, she developed a cough, with the progression of respiratory symptoms to severe dyspnoea.
On examination, her JVP was elevated, and bilateral pedal edema was observed. Her blood pressure is
160/100 mmHg, and she is tachycardic. The oxygen saturation of the patient is < 60 %. Her B-type
natriuretic peptide (BNP) Levels = 1300 pg/ml. What is the diagnosis of the above condition?
(or)
What is the diagnosis for a 35-year-old female with chronic kidney disease who developed respiratory
symptoms, elevated jugular venous pressure, bilateral pedal edema, hypertension, tachycardia, and
low oxygen saturation following a blood transfusion, with B-type natriuretic peptide levels of 1300
pg/ml?
A. Transfusion-associated lung injury (TRALI)
B. Transfusion-associated circulatory overload (TACO)
C. Febrile non-hemolytic transfusion reaction (FNHTR)
D. Acute respiratory distress syndrome (ARDS)
----------------------------------------
14. What condition is prevented using the instrument given below?

Page 3

1370
A. Febrile non-hemolytic transfusion reaction (FNHTR)
B. Transfusion-associated lung injury (TRALI)
C. Transfusion-associated circulatory overload (TACO)
D. Acute respiratory distress syndrome (ARDS)
----------------------------------------
15. Identify the blood bag given below.

A. Turnaround time bag (TAT)


B. Top and bottom bag (TAB)
C. Penta bag
D. Triple bag
----------------------------------------
16. What is the ratio of red blood cells (RBC), fresh frozen plasma (FFP), and platelets in a massive
blood transfusion protocol?
A. 1: 1:1
B. 2: 1: 3
C. 1:3: 5
D. 3: 2: 1
----------------------------------------
17. Observe the image below and determine the group of the sample used:

A. B+

Page 4

1371
B. A+
C. B-
D. None of the above
----------------------------------------
18. Which of the following is the anticoagulant used in a green vacutainer?
A. Potassium Oxalate
B. K2EDTA
C. Trisodium citrate
D. Sodium Heparin
----------------------------------------
19. Which of the following anticoagulants can be used to carry out the ESR test?
A. Sodium Fluoride
B. Heparin
C. Sodium/Potassium Oxalate
D. Trisodium citrate
----------------------------------------
20. Which of the following statements is true regarding the container in which blood is collected for PT
study
A. Potassium oxalate and sodium fluoride are the additives used.
B. 1:4 is the ideal anticoagulant: blood ratio.
C. The ratio of the blood sample to anticoagulant is 9:1
D. Sample for ESR can never be collected in this tube
----------------------------------------
21. Observe the images and select the proper order of sample collection into the tubes. 1. 2. 3. 4.

1.

Page 5

1372
2.

3.

4.

A. 1,2,3,4.
B. 4,1,2,3.
C. 4,2,1,3.
D. 3,1,4,2.
----------------------------------------

Correct Answers
Question Correct Answer

Question 1 1
Question 2 4

Page 6

1373
Question 3 1
Question 4 3
Question 5 1
Question 6 2
Question 7 3
Question 8 2
Question 9 1
Question 10 1
Question 11 1
Question 12 2
Question 13 2
Question 14 1
Question 15 2
Question 16 1
Question 17 2
Question 18 4
Question 19 4
Question 20 3
Question 21 3

Solution for Question 1:


Correct Option A - Occurs within 2 hours of red cells and platelet transfusion:
• FNHTR is a type of transfusion reaction that typically occurs within 6 hours of receiving a transfusion
of red blood cells or platelets.
• Febrile non-hemolytic transfusion reactions are the most common transfusion reactions, typically
characterized by the development of fever within 6 hours of receiving a transfusion of red cells or
platelets.
• Additionally, the frequency of these reactions tends to increase with the storage age of the blood
products. This is because older blood products may contain higher levels of cytokines and other factors
that can trigger immune responses.
• On the other hand, measures that limit donor leukocyte contamination, such as leukoreduction, can
decrease the frequency of febrile non-hemolytic transfusion reactions. By removing white blood cells
from the blood products, these measures reduce the likelihood of cytokine release and subsequent
febrile reactions.

Incorrect Options:
Options B, C, and D are incorrect.

Page 7

1374
Solution for Question 2:
Correct Option D - Cryoprecipitate:
• Antihemophilic Factor, also called cryoprecipitate, is a portion of plasma, the liquid part of our blood.
Cryoprecipitate is rich in clotting factors, which are proteins that can reduce blood loss by helping to
slow or stop bleeding
• Cryoprecipitate is derived from plasma and contains Fibrinogen Factor VIII Factor XIII Von Willebrand
factor
• Fibrinogen
• Factor VIII
• Factor XIII
• Von Willebrand factor
• It does not contain ABO Abs, so the risk of ABO incompatibility is minimal.
• Fibrinogen
• Factor VIII
• Factor XIII
• Von Willebrand factor

Incorrect Options:
Option A - Fresh frozen plasma:
• Fresh frozen plasma contains anti-a or anti-b Abs according to the donor's blood group.
• Fresh frozen plasma must be ABO compatible with the recipient's red cells.
Option B - Platelet-rich plasma:
• Platelet-rich plasma contains intrinsic platelet ABO antigens and anti-a or anti-b Abs according to the
donor's blood group.
• Therefore, platelet-rich plasma should preferably be ABO-compatible.
Option C - Single donor platelets:
• The platelets bear the intrinsic ABO antigens.
• Therefore, platelet-rich plasma and single donor platelets also should be preferably ABO compatible.

Solution for Question 3:


Correct Option A - Metabolic alkalosis:
• Massive transfusion is defined as transfusing one to two times the patient's normal blood volume or >
50% of blood volume within three hours.

Page 8

1375
• This is the equivalent of 10-20 units in a healthy adult.
• Common abnormality, resulting from Massive transfusion is metabolic alkalosis.
• It results from the conversion of citrate (present in stored blood) to bicarbonate.

Incorrect Options:
Option B - Metabolic acidosis:
• Metabolic acidosis is of two types. Anion gap metabolic acidosis and non-anion gap metabolic
acidosis.
• The causes of anion-gap metabolic acidosis are as follows:
• Ethylene glycol, propylene glycol, methanol, aspirin (late effect) toxicity,
• Chronic acetaminophen use
• Renal failure
• High ketogenic states (diabetic, alcoholic, starvation) etc..
The causes of non-anion gap metabolic acidosis are as follows:
• The causes of non-anion gap metabolic acidosis are as follows:
• Hyperchloremia/hyperalimentation
• Renal tubular acidosis
• Spironolactone
• Saline infusion, etc..
Option C - Respiratory alkalosis:
• Patients who receive multiple transfusions develop metabolic alkalosis due to the conversion of citrate
(present in stored blood) to bicarbonate.
Option D - Respiratory acidosis:
• Respiratory acidosis occurs as a compensatory response to metabolic alkalosis as a result of multiple
blood transfusions.

Solution for Question 4:


Correct Option C - O-positive RBC and colloids/crystalloid:
• If the patient's blood type cannot be determined, red blood cells from Group O can be used. When
utilized in women of childbearing age, to avoid Rh sensitization, Rh antigen in the unpaired blood
should be Rh (–ve).
• However, for older females and males, Rh-positive BG may be used, since Rh-negative blood is
typically in short supply. Sensitization may occur in such instances, although the likelihood of an acute
hemolytic reaction is limited.
Incorrect Options:
Option A - RBC corresponding to his red cell group and colloids/ crystalloid:

Page 9

1376
• The ideal treatment is to transfuse red blood cells corresponding to the recipient's red cell group along
with crystalloids.
• However, in the given scenario patient needs an urgent transfusion and his ABO blood group is
unidentified.
• Therefore, the best option is to transfuse O-positive red blood cells with crystalloids/colloids because
O-positive red blood cells are universal donors as they contain no A & B antigens to which the
recipient's anti-A or anti-B antibodies could react.
Option B - Whole blood corresponding to his plasma group:

• In the given scenario, the patient's ABO group is unidentified. Therefore, the best way to transfuse is
group O red blood cells rather than whole blood because donor plasma contains anti-A & anti-B
antibodies, which can react with the recipient's A or B antigens.
Option D - AB negative whole blood:
• AB-negative whole blood contains A & B antigens and no anti-A & anti-B antibodies.
• When AB blood is transfused to recipients with A, B, or O blood groups, it can precipitate an ABO
incompatibility reaction due to, the recipient's blood anti-A or anti-B antibodies reacting with A & B
donor antigens. Thus, precipitating an AB incompatibility reaction.
• Therefore, transfusion of AB-positive whole blood to either A or B-positive patients carries a high AB
incompatibility transfusion reaction.
• AB-positive blood should only be transfused to AB-positive recipients.
• On the other hand, AB positive is a universal recipient as it contains no anti-A & anti-B antibodies to
react with donor antigens

Solution for Question 5:


Correct Option A - Packed red blood cells are stored at 2°C -6°C:
• Packed red blood cells are spun-down and concentrated packs of red blood cells.

Page 10

1377
• Each unit has volume of approximately 330 mL and has a hematocrit of 50%-70%
• Packed cells are stored in a SAG-M solution (saline-adenine-glucose-mannitol) to increase shelf life to
5 weeks at 2°C -6°

Incorrect Options:
Option B - Platelets are stored at -18°C: Platelets: stored at a
temperature of 20°C -24°C, to limit the risk of sepsis.
Option C - Fresh frozen plasma is stored at -12°C: Fresh frozen plasma is stored at -18°C.
Option D - Cryoprecipitate is stored at 20°C -24°C: Cryoprecipitate is stored at -18°C

Solution for Question 6:


Correct Option B - 18 G needle:
• Ideally, an 18-20 Gauge needle should be used for blood transfusion due to its large bore size. A
large bore speeds up the flow rate and reduces the risk of hemolysis. This is useful for trauma, when
large quantities of blood in a small amount of time need to be transfused.
• Usually, the needle size depends on the size and integrity of a patient's vein.
• The smaller the gauge, the slower the flow rate and the higher the risk of clotting.

Incorrect Options:
Option A - 26 G needle:
• The 26 G cannula is small in size, making it suitable for use in pediatric and neonatal patients. Its thin
gauge helps minimize trauma during insertion.
• Typically, the 26 G cannula is equipped with a butterfly needle, it is ideal for patients with fragile or
brittle capillaries.
Option C - 22 G needle:

Page 11

1378
• It is mostly used for infusions in neonates, pediatric and older adults, and routine blood transfusions.
Option D - 24 G needle:
• Pediatricians may opt to use 24-gauge needles for vaccinating children. The Centers for Disease
Control and Prevention recommends gauges 22, 23, and 24 for both pediatric and adult intramuscular
injections.
• The specific gauge and needle length chosen depends upon the volume of the patient's muscle mass,
the amount of fat at the injection site, and the volume of the liquid for injection.
• 24 Gauge needles are used for spinal injections or catheterization

Solution for Question 7:


Correct Option C – Hemoglobinuria:
• Mismatched blood transfusions, causing acute hemolytic reactions, usually stem from most commonly
stem from an error in patient identification or tube labeling that allows a patient to receive an
ABO-incompatible unit of blood.
• Pre-existing high affinity "natural" IgM antibodies, usually against polysaccharide blood group
antigens A or B, bind to red cells and rapidly induce complement-mediated lysis, intravascular
hemolysis, and hemoglobinuria.
• Fever, shaking chills, and flank pain are clinical manifestations, that appear soon after
transfusion. The direct Coombs test is typically positive unless all of the donor red cells have lysed.
• The signs and symptoms are due to complement activation rather than intravascular hemolysis per
se, as osmotic lysis of red cells produces hemoglobinuria without any of the other symptoms of a
hemolytic reaction.
• In severe cases, the process may rapidly progress to disseminated intravascular coagulation (DIC),
shock, and acute renal failure.

Incorrect Options:
Option A - Elevated plasma haptoglobin level:
• Haptoglobin is a protein in plasma that bind with high affinity to free hemoglobin released from
erythrocytes and thereby inhibits its deleterious oxidative activity.
• The haptoglobin-hemoglobin complex will then be removed by the reticuloendothelial system.
• In hemolysis, free hemoglobin is released into plasma by red blood cell lysis and is bound by
haptoglobin to form a complex. Therefore, plasma haptoglobin is reduced in hemolysis.
Option B - Negative direct antiglobulin test (DAT):
• Direct antiglobulin test (DAT) is also called Direct Coomb's test.
• Direct Coomb's test detects antibodies attached directly to the RBC surface.
• In mismatch transfusion recipient's antibodies bind to the donor's red blood cells causing lysis,
resulting in a positive Coomb's test.
Option D - Decreased serum lactate dehydrogenase (LDH) level:
• Serum LDH levels are increased in hemolysis

Page 12

1379
Solution for Question 8:
Correct Option B - Fresh frozen plasma at -18°C is one year:
• Shelf life is defined as the length of time a product may be stored without becoming unsuitable for use
or consumption. Shelf life depends on the degradation mechanism of the specific product. Most can be
influenced by several factors: exposure to light, heat, and moisture; transmission of gases; mechanical
stresses; and contamination by things such as microorganisms.
• The shelf life of frozen plasma at -18°C is one year.

Incorrect Options:
Option A - At 20°C, platelets have a shelf-life of one year:
• The shelf life of platelets at 20°C is 5 days.
Option C - Whole blood with CPDA-1 solution at 4 degrees C is 5 days:
• The shelf life of whole blood within citrate-phosphate-dextrose (CPD) solution at 4 degrees C is 21
days.
Option D - Cryoprecipitate at -18°C is 21 days:
• The shelf life of cryoprecipitate at -18°C is 1 year.

Solution for Question 9:


Correct Option A - Factor VIII, von Willebrand factor, fibrinogen (Factor I), Factor XIII, fibronectin:
• Antihemophilic Factor, also called cryoprecipitate, is a portion of plasma, the liquid part of our blood.
Cryoprecipitate is rich in clotting factors, which are proteins that can reduce blood loss by helping to
slow or stop bleeding.
The blood clotting proteins found in cryoprecipitate include:
• Fibrinogen
• Factor VIII
• Factor XIII
• Von Willebrand factor
• Fibronectin

Incorrect Options:
Options B, C and D are incorrect.

Page 13

1380
Solution for Question 10:
Correct Option A - Red blood cells:
• Blood is separated into its various components in the laboratory through centrifugation. The physical
force from continuous revolutions pushes the denser, heavier particles to the outer edges of the sample
resulting in three layers of different densities: RBCs, a mixture of WBCs and platelets, and plasma. The
addition of a density separation medium can also cause the isolation of mononuclear cells and
granulocytes from human blood.
• Separation of blood components from one another by centrifugation is possible due to differences in
their specific gravities.
• Red blood cells have the highest density and settle earliest at the bottom.
• This permits the administration of specific replacement therapy as per the patient's requirements and
avoids transfusion of unwanted constituents of blood.

Incorrect Options:
Option B - Platelet concentrates:
• Platelets, having a lower density than red blood cells, settle after RBCs.
• Therefore, these are separated after red blood cells.
Option C - Fresh frozen plasma:
• Fresh frozen plasma has a lower density than red blood cells and platelets and settles after platelets.
• Therefore, these are separated after red blood cells and platelets.
Option D - Cryoprecipitate:
• Cryoprecipitate has the lowest density and is settled at last.
• Therefore, these are separated at the end of centrifugation.

Solution for Question 11:


Correct Option A - Associated with differentiation factor 234 (CD234):
• Duffy antigen/chemokine receptor (DARC), also known as Fy glycoprotein (FY)
• It is a glycosylated membrane protein that acts as a non-specific chemokine receptor.
• Duffy antigen is located on the surface of red blood cells (RBCs).
• The Duffy blood group system is based on polymorphisms in this gene.
• Antibodies formed against the Duffy antigens are of the IgG subclass and are clinically significant as
they can be implicated in acute and delayed hemolytic transfusion reactions as well as hemolytic
disease of the fetus and newborn.
• Patients who form anti-Fya or anti-Feb must receive antigen-negative blood units in the future.
• Associated with CD234 (Cluster of Differentiation 234).
• Human malaria parasites Plasmodium vivax and Plasmodium knowlesi bind to this receptor.
Approximately 68% of Africans lack both Fya and Fyb antigens. Individuals with this unique phenotype

Page 14

1381
are resistant to two malaria species.

Incorrect Options:
Option B - Located on the ribosomes of RBCs: It is located on the surface of red blood cells.
Option C - Glycosylated cytoplasmic protein: It is a glycosylated membrane protein that acts as a
receptor for chemokines.
Option D - Associated with differentiation factor 123 (CD123): It is associated with a
cluster of differentiation 234 (CD234)

Solution for Question 12:


Correct Option B - Transfusion-related acute lung injury (TRALI):
• Transfusion-related acute lung injury (TRALI) is a serious blood transfusion complication
characterized by the sudden onset of non-cardiogenic pulmonary edema accompanied by hypoxia
following the transfusion of blood products.
• Signs and symptoms are sudden development of dyspnea, severe hypoxemia (O2 saturation <90% in
room air), hypotension and fever that develops within 6 hours after transfusion and usually resolves
with supportive care within 48 to 96 hours.
• Antibodies directed toward human leukocyte antigens (HLA) or human neutrophil antigens (HNA)
have been implicated.
• Multiparous women develop these antibodies through exposure to fetal blood; transfusion of blood
components obtained from these donors is thought to carry a higher risk of inducing immune-mediated
TRALI.
• TRALI is more commonly associated with plasma products such as fresh frozen plasma (FFP).
However, it can also occur in recipients of PRBC, both in adult and pediatric patients.
• Supportive care is the mainstay of therapy in TRALI. Oxygen supplementation is employed in all
reported cases of TRALI, and aggressive respiratory support is needed in 72 percent of patients.
Intravenous administration of fluids, as well as vasopressors, are essential for blood pressure support.
The use of diuretics is indicated in transfusion-associated circulatory overload (TACO) and should be
avoided in TRALI. Corticosteroids can be beneficial.
• Oxygen supplementation is employed in all reported cases of TRALI, and aggressive respiratory
support is needed in 72 percent of patients.
• Intravenous administration of fluids, as well as vasopressors, are essential for blood pressure support.
• The use of diuretics is indicated in transfusion-associated circulatory overload (TACO) and should be
avoided in TRALI.
• Corticosteroids can be beneficial.
• Oxygen supplementation is employed in all reported cases of TRALI, and aggressive respiratory
support is needed in 72 percent of patients.
• Intravenous administration of fluids, as well as vasopressors, are essential for blood pressure support.
• The use of diuretics is indicated in transfusion-associated circulatory overload (TACO) and should be
avoided in TRALI.

Page 15

1382
• Corticosteroids can be beneficial.

Incorrect Options:
Option A - Febrile non-hemolytic transfusion reaction:
• It is caused by cytokines created by donor white blood cells (WBCs) accumulating during the storage
of blood products.
• It is more common in children.
• Fever develops within 1–6 hours (due to preformed cytokines) headaches, chills, and flushing.
• The reactions are prevented by the leukoreduction of blood products.
Option C - Transfusion-associated circulatory overload (TACO):
• The distinguishing feature between transfusion-associated circulatory overload (TACO) and
transfusion-related acute lung injury (TRALI) lies in their clinical presentations. Specifically, TACO
typically lacks hypotension and fever, whereas TRALI may manifest with these symptoms.
• TACO is a common transfusion reaction in which pulmonary edema develops primarily due to volume
excess or circulatory overload.
• TACO typically occurs in patients who receive a large volume of a transfused product over a short
period, especially those with underlying cardiovascular or renal disease.
• TACO is characterized by pulmonary hydrostatic (cardiogenic) edema, whereas TRALI presents as
pulmonary permeability edema (noncardiogenic).
Option D - Pulmonary embolism:
• In pulmonary embolism patient presents with symptoms such as substernal or lateral pain, pleuritic
nature, hemoptysis, tachycardia, and hypoxemia.
• History of recent immobilization/surgeries is mostly present.
• However, given this patient's normal D dimer, PE is unlikely. Further, her recent transfusion makes
TRALI more likely.

Solution for Question 13:


Correct Option B - Transfusion-associated circulatory overload (TACO):
• This patient, with CKD, likely has TACO due to her transfusion.
• TACO occurs due to excessive volume overload from rapid or excessive transfusion, leading to
pulmonary edema and subsequent respiratory symptoms. The clinical features align with this diagnosis,
particularly the acute onset of symptoms post-transfusion and signs of volume overload.
• The elevated BNP levels signify cardiac strain, suggesting the presence of HF or fluid overload
• TACO lacks fever, and hypotension and has other peripheral features of fluid overload.
• Immediate discontinuation of the transfusion, supplemental oxygen if needed, and diuretics to remove
excess fluid are the mainstay of treatment.

Incorrect Options:

Page 16

1383
Option A - Transfusion-associated lung injury (TRALI):
• Transfusion-related acute lung injury (TRALI) is a sudden onset of non-cardiogenic pulmonary edema
following transfusion of blood products.
• Dyspnea, severe hypoxemia (O2 saturation <90% in room air), hypotension, and fever develop within
6 hours after transfusion and usually resolves with supportive care within 48 to 96 hours.
• TRALI is more commonly associated with plasma products such as fresh frozen plasma (FFP).
Option C - Febrile non-hemolytic transfusion reaction (FNHTR):
• It is caused by cytokines created by donor white blood cells (WBCs) accumulating during the storage
of blood products.
• It is more common in children.
• Fever develops within 1–6 hours (due to preformed cytokines) headaches, chills, and flushing.
• The reactions are prevented by the leukoreduction of blood products.
Option D - Acute respiratory distress syndrome (ARDS):
• Key diagnostic criteria for ARDS include (1) diffuse bilateral pulmonary infiltrates on chest X-ray, (2)
PaO2 (arterial partial pressure of oxygen in mmHg)/FIO2 (inspired O2 fraction) ≤200 mmHg; and (3)
absence of elevated left atrial pressure (pulmonary capillary wedge pressure ≤18 mmHg).
• ARDS is a severe form of respiratory failure with various causes, including sepsis, trauma, aspiration,
etc. While the patient in this case has severe respiratory distress, the clinical presentation with signs of
volume overload (elevated blood pressure, JVP, bilateral pedal edema) and elevated BNP levels are
more suggestive of volume overload-related conditions like TACO rather than ARDS.

Solution for Question 14:


Correct Option A - Febrile non-hemolytic transfusion reaction (FNHTR):
• The above-given image is of a leukoreduction filter which removes WBCs from the blood products.
There are three well-accepted reasons for leukocyte-reduced blood products:
• 1) Prevention of alloimmunization to foreign HLA antigens
• 2) Prevention of febrile non-hemolytic transfusion reactions
• 3) Prevention of CMV transmission (though efficacy unclear)
• Leukocyte reduction may also play a role in decreasing bacterial contamination of blood products and
prevention of transmission of prions (such as the vector of variant Creutzfeldt-Jakob disease), though
these are less accepted indications.

Incorrect Options:
Option B - Transfusion-associated lung injury (TRALI): Transfusion-associated lung injury is caused by
antibodies directed toward human leukocyte antigens (HLA) or human neutrophil antigens (HNA).
Option C - Transfusion-associated circulatory overload (TACO): Transfusion-associated circulatory ove
rload is caused by the rapid transfusion of a large volume of blood but can also occur during a
single red blood cell transfusion.

Page 17

1384
Option D - Acute respiratory distress syndrome (ARDS): Most cases (>80%) result from sepsis, bacteri
al pneumonia, trauma, multiple blood transfusions, gastric acid aspiration, and drug overdose.

Solution for Question 15:


Correct Option B - Top and bottom bag (TAB):
• The bag given in the image is a TAB (top and bottom) bag.
• The conventional (top & top) systems are intended to collect whole blood, and prepare plasma, red
cell concentrates, and platelet concentrates rapidly and easily, either manually or with the help of blood
separators.
• The top & bottom systems are used to prepare high-quality plasma, red cell concentrates, and platelet
concentrates, ideally by the buffy coat method, which is gentle on platelets.
• The needle to which the bag is attached is known as the mother bag to which the donor blood is
collected; from the top outlet, the plasma goes out, and from the bottom outlet, red blood cells go out.
• After blood donation, component separation should be done within 6 hours.

Incorrect Options:
Option A - Turnaround time bag (TAT): TAT has no bottom outlet, and both outlets are on top.
Option C - Penta bag: A Penta bag is used for pediatric blood transfusion because these patients usual
ly require 100-120 ml of blood transfusion.
Option D - Triple bag: Triple Blood Bag is designed to separate whole blood into three blood componen
ts: red blood cells, platelets, and plasma through the process of centrifugation and extraction. Each ba
g is composed of the following: an anti-coagulant additive solution, a primary bag, a
platelet-transfer bag for 5-day platelet storage, and a donor needle gauge.

Solution for Question 16:


Correct Option A - 1: 1:1
• A fixed-ratio (1:1:1) transfusion strategy is a resuscitation strategy for trauma patients that promotes
the transfusion of red blood cells (RBC), plasma, and platelets (PLT) at a 1:1:1 ratio while minimizing
crystalloid infusion.
• This balanced transfusion strategy aims to correct both the early coagulopathy of trauma and the
volume status of patients in hemorrhagic shock, thus targeting preventable hemorrhage-related deaths.
• Retrospective studies of the 1:1:1 transfusion protocol reported marked reductions in mortality,
hypothesized to be due to a protective effect against dilutional coagulopathy-related mortality.

Incorrect Options:
Options B, C and D are incorrect.

Page 18

1385
Solution for Question 17:
Correct Option B - A+:
• The test depicted in the given image is a Gel Card test for blood grouping.

The principle involved is:


• After mixing the patient's blood sample with specific antibodies against blood group antigens, the red
blood cells (RBCs) agglutinate if they react with the corresponding antibodies. Agglutinated RBCs
cannot pass through the gel due to their size; so they accumulate at the top of the gel column, forming
a visible line. This line indicates a positive reaction, suggesting the presence of the specific blood group
antigen on the patient's RBCs.
• Conversely, if the patient's RBCs do not react with the antibodies and remain unagglutinated, they can
freely pass through the gel and settle at the bottom of the gel column during centrifugation. This results
in the formation of a visible line at the bottom of the gel card, indicating a negative reaction, which
suggests the absence of the specific blood group antigen on the patient's RBCs.
• GIVEN THIS : the sample used in the test shown is A+
Incorrect Options:
Options A, C and D are incorrect.

Solution for Question 18:


Correct Option D) Sodium Heparin :
• Sodium Heparin is the anticoagulant used in a green vacutainer tube.

Page 19

1386
• Green tubes are used for : Osmotic fragility testing, immunophenotyping, Arterial Blood Gas studies.
Incorrect Options:
Options A, B and C are incorrect. Correct option is explained above.

Solution for Question 19:


Correct Option D - Trisodium citrate:
• For Erythrocyte sedimentation rates, blood is anticoagulated with 3.8% trisodium citrate (Blue tube) in
case the Westergren method is being used. Ethylene diamine tetra acetic acid (EDTA) (violet tube) is
used in case the Winthrobe method is used.
• Hence of the given options, the appropriate anticoagulant for ESR would be Trisodium citrate.

Incorrect Options:
Option A - Sodium Fluoride: Sodium Fluoride is used as an anticoagulant additive for Blood Glucose E
stimation in grey tube.
Option B - Heparin: Heparin (green tube) is used as an anticoagulant when the sample is taken for arte
rial blood gas or osmotic fragility test.
Option C - Potassium Oxalate: In the case of glucose estimator tests Potassium oxalate with NaF as a
dditive is used in a Grey tube.

Solution for Question 20:


Correct Option C - The ratio of the blood sample to anticoagulant is 9:1
• The sample for PT is ideally collected in a blue tube and the anticoagulant in a blue tube is trisodium
citrate.
• The ratio of the blood sample to anticoagulant should be kept at 9:1 for an accurate PT or any other
value in coagulation studies.

Incorrect Options:
Option A -Potassium oxalate and sodium fluoride are the additives used :
• This option describes the additives used in the Grey vacutainer. It is used for glucose estimator tests
and not for PT
Option B -1:4 is the ideal anticoagulant: blood ratio.:
• This is the ratio for ESR estimation in a blue tube. For PT study the blood sample to anticoagulant
should be kept at 9:1.
Option D -Sample for ESR can never be collected in this tube:
• This is not true ESR study can be done on the sample collected in a blue tube given the ratio of
anticoagulant: blood ratio in which the sample is sent is 1:4

Page 20

1387
Solution for Question 21:
Correct Options C - 4,2,1,3:
Following a standardized order of draw reduces the risk of contamination between tubes and ensures t
hat additives do not interfere with subsequent tests.
4. Blue vacutainer:
• Anticoagulant used- Trisodium Citrate.
• Used for coagulation studies. Should ideally be filled first out of the given containers.

2. Green vacutainer:
• Anticoagulant used - Heparin.
• Used for ABG studies.
• Ideally to be filled after the blue vacutainer considering the given containers.

Page 21

1388
1. Purple vacutainer:
• Anticoagulant used- K2EDTA.
• Used for CBC, Peripheral smear, HbA1C, etc.
• Ideally to be filled after the green vacutainer considering the given containers.

3. Grey vacutainer:
• Anticoagulant used- K Oxalate, Additive- NaF.
• Used for Glucose estimation studies.
• Ideally to be filled after all the given containers.

Page 22

1389
Page 23

1390
Tissue Processing
1. In the dehydration step of tissue processing, which of the following agents is primarily used to
remove water from the tissue?
A. Xylene
B. Acetone
C. Formalin
D. Glutaraldehyde
----------------------------------------
2. What is the primary purpose of this instrument in the tissue processing workflow?

A. Fixation of tissues
B. Application of paraffin wax
C. Embedding and blocking
D. Removal of water using alcohol
----------------------------------------
3. Identify the process depicted in the image

A. Removal of water from tissues


B. Application of paraffin wax to tissue blocks
C. Shaping and forming paraffin blocks
D. Production of thin, consistent tissue sections for microscopy

1391
----------------------------------------
4. Which of the following statements is most relevant to the device shown?

A. Used in tissue fixation


B. Helps embedding of tissue in paraffin wax
C. Facilitates simultaneous staining of multiple slides
D. Used in section cutting on a microtome
----------------------------------------
5. Of the following choose the purpose of Distrene Dibutyl phthalate xylene (DPX) in histopathology.
A. Tissue fixation
B. Staining enhancement
C. Slide covering and prevention of drying
D. Microtome section cutting
----------------------------------------
6. What is the maximum number of samples that can be analyzed using an Improved Neubauer
Chamber in one go?
A. One
B. Two
C. Three
D. Four
----------------------------------------
7. Which of the following features is incorrectly matched with the corresponding pipette?
A. RBC Pipette: Bead Color: Red Markings: 0.5, 1, 101
B. WBC Pipette: Bead Color: White Markings: 0.5, 1, 11
C. RBC Pipette: Dilution: 1:200
D. WBC Pipette: Dilution: 1:10
----------------------------------------

Correct Answers

Page 2

1392
Question Correct Answer

Question 1 2
Question 2 3
Question 3 4
Question 4 3
Question 5 3
Question 6 2
Question 7 4

Solution for Question 1:


Correct Option B - Acetone:
• In the dehydration step of tissue processing, the goal is to remove water from the tissue, preparing it
for subsequent steps like clearing and embedding. Acetone is commonly used as a dehydrating agent
in this step.
• Acetone is a dehydrating agent commonly used to remove water from tissues. It helps to replace
water within the tissues and is often used in the transition from aqueous solutions to the embedding
medium.
Incorrect Options:
Option A - Xylene: Xylene is used in the clearing step to make tissues transparent. It does not play a
primary role in dehydrating tissues.
Option C - Formalin: Formalin is a
fixative used in the fixation step to preserve tissue structure. It is not a dehydrating agent.
Option D - Glutaraldehyde: Glutaraldehyde is another fixative used in fixation, especially for electron mi
croscopy. It is not primarily used for dehydrating tissues

Solution for Question 2:


Correct Option C - Embedding and blocking:
• Leuckhard's mould is used in the embedding and blocking step of tissue processing. It helps in
shaping and forming the paraffin block that contains the tissue sample. The paraffin block, once
solidified, is then subjected to section cutting, allowing for the production of thin, consistent sections of
the tissue for microscopic examination.
Incorrect Options:
Option A - Fixation of tissues:
• Fixation of tissues is typically done using specific fixatives during the initial step of tissue processing,
not with Leuckhard's mould.
Option B - Application of paraffin wax:

Page 3

1393
• Leuckhard's mould is involved in the embedding and blocking step, where paraffin wax is applied, but
it is not the primary purpose of the mould. Primarily paraffin wax is applied to the tissue during
impregnation.
Option D - Removal of water using alcohol:
• Dehydration, which involves the removal of water using alcohol, is a distinct step in tissue processing
but is not the primary purpose of Leuckhard's mould.

Solution for Question 3:


Correct Option D - Production of thin, consistent tissue sections for microscopy:
• The process depicted in the image is the production of thin, consistent tissue sections for microscopy.
• During section cutting, a microtome is used to cut thin slices from a paraffin-embedded tissue block.
These thin sections are then mounted on slides, stained, and examined under a microscope for
detailed microscopic analysis. This step is crucial in histopathology as it allows pathologists to visualize
the cellular and tissue structures, aiding in the diagnosis of various diseases.
Incorrect Options:
Options: A, B, and C are incorrect, and the correct option is explained above.

Solution for Question 4:


Correct Option C - Facilitates simultaneous staining of multiple slides:
• The device shown, Coplin's jar, is specifically designed for the simultaneous staining of multiple slides
in histopathology. It features slots or grooves to hold individual slides vertically, allowing them to be
immersed uniformly in staining solutions. This design ensures that several slides can undergo the
staining process concurrently, contributing to the efficiency and consistency of staining in histological
procedures
Incorrect Options:
Option A - Used in tissue fixation:
• Coplin's jar is not primarily designed for tissue fixation. Tissue fixation is typically performed using
fixatives like formalin before the staining process.
Option B - Helps embedding of tissue in paraffin wax:
• Coplin's jar is not involved in the embedding process. Embedding is a distinct step where tissues are
placed in molds and covered with paraffin wax.
Option D - Used in section cutting on a microtome:
• Coplin's jar is not used in the section cutting process. Section cutting involves using a microtome to
cut thin slices from paraffin-embedded tissue blocks.

Solution for Question 5:

Page 4

1394
Correct Option C - Slide covering and prevention of drying:
• DPX serves as a mountant, which means it is used to cover stained tissue slides.
• It prevents the stains on the slides from drying out, ensuring the preservation of the stained
specimens.
• The covering provided by DPX allows for clear observation of the tissue under a microscope, aiding in
the analysis of microscopic details.
Incorrect Options:
Option A, B & D - (Tissue fixation, Staining enhancement & Microtome section cutting):
• While other processes in histopathology, such as tissue fixation and staining enhancement, are critical
steps in preparing the slides, DPX specifically functions as a mountant to protect the stained tissues
and facilitate their examination

Solution for Question 6:


Correct Option B - Two:
• An Improved Neubauer Chamber is commonly used for counting cells in hemocytometry. It consists of
two counting chambers, each with a specified grid pattern. Each chamber has a depth of 0.1 mm.
• The maximum number of samples that can be analyzed using an Improved Neubauer Chamber in one
go is two. This is because there are two counting chambers on the slide, and each chamber is used to
count cells independently. Therefore, one can analyze two samples simultaneously by placing one
sample in each counting chamber.
Incorrect Options:
Options A, C and D are incorrect refer to the explanation above.

Solution for Question 7:


Correct Option D - WBC Pipette:, Dilution: 1:10:
• The correct dilution for the WBC pipette is 1:20. The dilution factor represents the ratio of the volume
of blood to the volume of diluting fluid in the pipette. In the case of the WBC Pipette, a dilution factor of
1:20 means that one part of the blood is diluted with 20 parts of the diluting fluid. This is essential for
obtaining an accurate count of white blood cells in the hemogram.
Feature
RBC Pipette
WBC Pipette

Page 5

1395
Bead Color
Red
White
Markings
0.5, 1, 101
0.5, 1, 11
Role of Bead
Identification, Mixing
Uses
Higher dilution (e.g., TLC)
Diluting Fluid

Page 6

1396
Hayem’s or Dacie’s fluid
Turk’s fluid
Dilution Ratio
1:200
1:20

Page 7

1397
Previous Year Questions
1. What is the most likely diagnosis for a 30-year-old male who has been experiencing fatigue for the
past year and has been found to have a significantly enlarged spleen on examination, based on the
following blood test results? Anemia Total leucocyte count - 1,50,000/microL Peripheral blood smear
60% neutrophils-6% Basophils-4% Eosinophils Myeloblasts, myelocytes, and metamyelocytes Myeloid
to erythroid ratio – 18:1
A. ALL
B. AML
C. CML
D. CLL
----------------------------------------
2. What is the probable diagnosis for an 18-year-old male who visited the outpatient department due to
gum bleeding and fever persisting for the last two months? The general examination indicated pallor,
while the systemic examination was normal. Laboratory tests unveiled a hemoglobin level of 3 gm/dL,
TLC count of 1500/microL, and platelet count of 15000/microL. The peripheral smear displayed
macrocytes and a reticulocyte count of 0.5. Bone marrow examination revealed the presence of fatty
streaks and a lack of megakaryocytes, although immature cells were absent. Hb-Hemoglobin
TLC-Total leucocyte counts
A. Idiopathic acquired aplastic anemia
B. Paroxysmal nocturnal hemoglobinuria
C. Myelodysplastic syndrome
D. Tuberculosis
----------------------------------------
3. A 24-year-old male patient arrives with a case of anemia. Both his father and paternal aunt
experienced a similar illness and were effectively treated with splenectomy. The patient's peripheral
blood smear resembles the one depicted in the accompanying illustration. What other abnormalities
can be anticipated?

A. Decreased osmotic fragility


B. Decreased reticulocytes
C. Heinz bodies
D. Howell-jolly bodies

1398
----------------------------------------
4. Which of the following is a possible explanation for the presence of pancytopenia as indicated by the
blood examination results? A. Megaloblastic anemia B. Hairy cell leukemia C. MDS with hypocellular
BM D. APML
A. B,D
B. A,B,C
C. B,C,D
D. A,B,D
----------------------------------------
5. A young boy from high altitude came with fever and weakness and had the following reports-
Hemoglobin 17 g%, TLC is 21,000 with neutrophils 25, lymphocytes 30, eosinophils 5, myelocytes and
metamyelocytes 40 in peripheral smear. Next step in the investigation should be:
A. Philadelphia chromosome
B. JAK mutation
C. Erythropoietin levels
D. Bone marrow biopsy with reticulin stain
----------------------------------------
6. A child presents with recurrent chest infections and abdominal pain. There is a history of delayed
growth and bone problems causing facial changes. On examination, he had icterus and mild
splenomegaly. Electrophoresis is given below. What is the likely diagnosis?

A. Beta thalassemia
B. HbC disease
C. Sickle cell disease
D. Acute coronary disease
----------------------------------------
7. An elderly patient presents with anemia and hemoglobinuria. Investigations reveal increased bilirubin
and lactate dehydrogenase. The peripheral smear image is given below. Which of the following is
associated with this?

Page 2

1399
A. Splenomegaly
B. Mechanical second heart valve
C. Increased HbA2
D. Goitre
----------------------------------------
8.
(or)
A female patient presented with fatigue and a history of piles. Routine complete blood count analysis
showed a hemoglobin of 9 g/dL, an MCV of 60 fL, and an RBC count of 5.2 million. A peripheral smear
is given below. Which of the following is the next best investigation for this patient?
A. HbA2 levels
B. Serum ferritin levels
C. Serum folate levels
D. Serum homocysteine levels
----------------------------------------
9. In which of the following conditions is macrocytic anemia seen?
A. Thalassemia
B. Anemia of chronic disease
C. Sideroblastic anemia
D. Liver disease
----------------------------------------
10. Which of the following stains is used for the detection of hemosiderin deposits?
A. Mason-Fontana stain
B. Oil Red O
C. Sudan black
D. Perls Prussian blue
----------------------------------------

Page 3

1400
11. A child visited the hospital with her mother with complaints of severe fatigue, lethargy, pallor, and
irritation. She has a known case of iron deficiency anemia and is on oral iron supplements. On
reviewing the child, the physician in charge plans to start IV iron supplementation. What is the next best
test to be done in assessing this child?
A. Blood hemoglobin
B. Total iron binding capacity
C. Serum ferritin
D. Hematocrit
----------------------------------------
12. Which of the following is not routinely screened before conducting a blood transfusion?
A. Hepatitis A
B. Hepatitis B
C. Hepatitis C
D. HIV 1 / HIV 2
----------------------------------------
13. Which of the following is a feature of hemolytic anemia?
A. Increased haptoglobin
B. Neutropenia
C. Reticulocytopenia
D. Decreased haptoglobin
----------------------------------------
14. Which of the following correctly represents the effect of the mutation causing sickle cell anemia?
A. Glutamate by valine at the 6th position
B. Valine by glutamate at the 6th position
C. Glutamate by valine at the 5th position
D. Valine by glutamate at the 5th position
----------------------------------------
15. A child presents with intermittent jaundice and splenomegaly. There is a history of similar
complaints in the elder brother. A peripheral smear shows the following finding. How will you investigate
this condition?

Page 4

1401
A. Osmotic fragility test - Hereditary spherocytosis
B. Coombs test - Autoimmune hemolytic anemia
C. Genetic testing - G6PD deficiency
D. Flow cytometry - Paroxysmal nocturnal hemoglobinuria
----------------------------------------
16. Identify the instrument:

A. Klima needle
B. Salah needle
C. Jamshidi needle
D. Islam needle
----------------------------------------
17. A 10-year-old boy presented with fatigue. Investigations revealed haemoglobin: 9 g/dL, MCV: 60 fL,
MCH: 20 pg, and serum ferritin: 185 µg/L. The TLC was elevated and showed predominant
lymphocytes and neutrophils. What is the likely diagnosis for this patient?
A. Thalassemia
B. Anemia of chronic disorder
C. Iron deficiency aenmia
D. Sickle cell anemia
----------------------------------------

Page 5

1402
18. The Hb is 5 g/dL and the absolute reticulocyte count percentage is 9%. What is the correct
reticulocyte count?
A. 5
B. 4.5
C. 3
D. 1.8
----------------------------------------
19. A 35-year-old woman presents with fatigue. Investigations revealed the following: Hb: 5 gm%, low
MCH, and low MCV. The peripheral smear is shown below. What is the diagnosis?

A. Megaloblastic anemia
B. Iron deficiency anemia
C. Thalassemia major
D. Essential thrombocytosis
----------------------------------------
20. The following image shows an instrument used to estimate hemoglobin levels. What does it
measure?

A. Alkaline hematin
B. Acid hematin
C. Oxygemoglobin
D. Deoxyhemoglobin

Page 6

1403
----------------------------------------
21. Iron in tissues is stained by:
A. Masson’s trichrome
B. Prussian blue
C. PAS
D. Fite-Faraco
----------------------------------------
22. All of the following are seen in the anaemia of chronic disease, except:
A. High serum hepcidin
B. Low serum ferritin
C. Chronic infection
D. Normal iron stores
----------------------------------------
23. What is the interpretation of IDA? a. Low serum ferritin b. Low transferrin saturation c. Low serum
iron d. Increased TIBC
A. a, b, c, d
B. a, b
C. c, d
D. a, c, d
----------------------------------------
24. Which of the following cells is shown in the given image?

A. Hairy cell
B. Gaucher cell
C. Sickle cell
D. Target cell
----------------------------------------
25. In iron deficiency anemia, all of the following are increased, except:
A. Transferrin saturation

Page 7

1404
B. RBC protoprophyrin
C. Total iron binding capacity
D. Ferritin soluble receptors
----------------------------------------
26. An 18-year-old male presented to the outpatient department with gum bleeding and fever for the
past 2 months. The general examination showed pallor, and the systemic examination was
unremarkable. Laboratory examination revealed an Hb level of 3 g/dl, a TLC of 1500/microL, and a and
a platelet count of 15000/microL. A peripheral smear shows macrocytes, and the reticulocyte count is
0.5. A bone marrow examination revealed fatty streaks and absent megakaryocytes but no immature
cells. What is the likely diagnosis?
A. Idiopathic acquired aplastic anemia
B. Paroxysmal nocturnal hemoglobinuria
C. Myelodysplastic syndrome
D. Tuberculosis
----------------------------------------
27. A software engineer presents to the outpatient department with easy fatiguability. He gives a history
of sitting in front of the computer for 12-14 hours. He consumes a lot of junk food with very less fruits
and vegetables. Hb is 9 g/dl and MCV is 80 fl. What is the most likely cause of this patient’s anemia?
A. Cyanocobalamin deficiency
B. Iron deficiency
C. Folic acid deficiency
D. Hereditary spherocytosis
----------------------------------------
28. A 65-year-old female patient is diagnosed with pure red cell aplasia and mediastinal mass. Which of
the following is a likely cause?
A. Thymoma
B. Non-Hodgkin lymphoma
C. Bronchogenic carcinoma
D. Germ cell tumor
----------------------------------------
29. A 23-year-old male patient presented with a history of fatigue and tiredness. On investigation, he
was found to have Hb = 9 g/dl and MCV = 101 FL. A peripheral smear examination showed macrocytic
RBC and hypersegmented neutrophils. Which of the following is the most likely etiology?
A. Lead poisoning
B. Iron deficiency anemia
C. Hemolytic anemia
D. Chronic alcoholism
----------------------------------------

Page 8

1405
30. A 52-year-old male patient comes with complaints of abdominal pain, nausea, and vomiting. A
peripheral blood smear reveals macrocytes and hypersegmented neutrophils, and an endoscopy
reveals atrophic gastritis. Which of the following conditions is likely to be seen in this patient?
A. Vitamin B12 deficiency
B. Folate deficiency
C. Pyridoxine deficiency
D. Niacin deficiency
----------------------------------------
31. Which of the following is not likely to be seen in a patient with paroxysmal nocturnal hemoglobinuria
(PNH)?
A. Aplastic anemia
B. Leukemia
C. Thrombosis
D. Hemolysis
----------------------------------------
32. The vacutainer tube shown in the given image contains:

A. Na-EDTA
B. Trisodium citrate
C. NAF- heparin
D. Clot activator
----------------------------------------
33. Which of the following is used in treating sideroblastic anemia?
A. Vitamin B12
B. Vitamin B6
C. Vitamin B1
D. Iron
----------------------------------------
34. A 45-year-old male patient was diagnosed with iron deficiency anemia. Which of the following is
true about intracellular regulation of iron in the above condition?

Page 9

1406
A. Ferritin translation is increased by binding IRP to IRE
B. Ferritin translation is decreased by binding IRP to IRE
C. Transferrin receptor 1 translation is unaffected binding IRP to IRE
D. Transferrin receptor 1 translation is decreased by binding IRP to IRE
----------------------------------------
35. Which of the following is true regarding glucose-6-phosphate dehydrogenase deficiency?
A. Spectrin defect
B. Cells are spherical
C. Cells do not lyse in hypotonic solution
D. Cells lyse in hypertonic solution
----------------------------------------
36. Supravital stains are used in the identification of all except?
A. Reticulocytes
B. Copper in tissue
C. Heinz bodies
D. Ribosomal RNA
----------------------------------------
37. CD-59 is involved in:
A. Paroxysmal nocturnal hemoglobinuria
B. Chediak-Higashi syndrome
C. Essential thrombocythemia
D. Primary myelofibrosis
----------------------------------------
38. Choose the correct option regarding the storage temperature of blood components.
A. RBC = 20-22ºC, Platelet = 2-6ºC, FFP = 18ºC
B. RBC = 2-6ºC, Platelet = 20-24ºC, FFP = -18ºC
C. RBC = 30ºC, FFP = 2-6ºC Platelet = 20-22ºC
D. RBC = 20-22ºC, FFP = Below -30ºC, Platelet = 2-6ºC
----------------------------------------
39. What is correct regarding transfusion of the blood received from the blood bank?
A. It should be started within 4 hours of receiving it from the blood bank
B. It should be completed within 4 hours of receiving it from the blood bank
C. No such time restraint and can be done anytime
D. It should be completed within 6 hours of receiving it from the blood bank
----------------------------------------

Page 10

1407
40. Which of the following is correct about anaemia of chronic disease?
A. High ferritin
B. Increased percentage of saturation of transferrin
C. Increased total iron binding capacity
D. High serum iron
----------------------------------------
41. What is the most sensitive indicator for iron deficiency anaemia?
A. Serum ferritin
B. TIBC
C. Percentage saturation of transferrin
D. Bone marrow iron
----------------------------------------
42. A 12-year-old boy presents to the clinic complaining of severe abdominal pain and headaches for
the past two days. He has a history of myalgia and several episodes of vomiting. His parents
self-medicated with paracetamol, but the patient had no relief. A dengue NS-1 antigen test was
ordered, which came out positive. What is the first line of defence for the body in the given scenario?
A. Natural Killer cells
B. T cell
C. Histiocyte
D. Macrophage
----------------------------------------
43. Which of the following factors hinders the absorption of iron?
A. Vitamin C
B. Phytates
C. Oxalate
D. Myoglobin
----------------------------------------
44. What is the diagnosis for an individual whose serum alkaline phosphatase is normal, PTH is
normal, and vitamin D3 is normal with elevated serum calcium?
A. Vitamin D intoxication
B. Hyperparathyroidism
C. Multiple myeloma
D. Nutritional rickets
----------------------------------------
45. In iron deficiency anemia, all of the following are reduced, except:
A. Total iron binding capacity
B. Percentage saturation of transferrin

Page 11

1408
C. Hemoglobin level
D. Serum ferritin
----------------------------------------
46. Rees and Ecker diluting fluid is used in:
A. Wintrobe tube
B. WBC pipette
C. RBC pipette
D. Hemoglobin pipette
----------------------------------------
47. What is your diagnosis with respect to the findings shown in the peripheral smear given below?

A. Iron deficiency anemia


B. B12 deficiency
C. Hereditary spherocytosis
D. Acute myeloid leukemia
----------------------------------------
48. Which of the following are features of iron deficiency anaemia?
A. TIBC increased, ferritin decreased, transferrin saturation increased
B. TIBC increased, soluble transferrin receptor decreased, ferritin increased
C. TIBC increased, transferrin saturation decreased, ferritin decreased
D. TIBC increased, transferrin saturation decreased, ferritin increased
----------------------------------------
49. Which of the following is true regarding the use of the instrument given in the image below?

Page 12

1409
A. Contraindicated if coagulopathy
B. Breath holding is required
C. Procedure cannot be done in both lateral and prone positions
D. Diagnosing an infiltrate or granulomatous disease
----------------------------------------
50. The JAK2 mutation is most commonly associated with:
A. Polycythemia vera
B. Essential thrombocytosis
C. CML
D. PMF
----------------------------------------
51. A 20-year-old female with easy fatiguability and pallor presented to the OPD. The image of her
hand is given below. What is the most likely diagnosis?

A. Aplastic anemia
B. Vitamin B12 deficiency
C. Iron deficiency anemia
D. Hypoalbuminemia
----------------------------------------
52. A 9-month-old boy presents with progressive pallor and abdominal distention. On investigations,
hepatosplenomegaly and malar eminence are seen, and his peripheral blood smear is as shown below.

Page 13

1410
He has a low Hb; the Mentzer index is found to be less than 13. What diagnosis would this signify for
this patient?

A. Megaloblastic anemia
B. Iron deficiency anemia
C. Thalassemia
D. Pernicious anemia
----------------------------------------
53. A 40-year-old male presented to OPD with complaints of itching during baths. His hemoglobin was
20%, his platelet count was 89000/mL, and his WBC count was 30,000/mL. What is the most likely
diagnosis?
A. Primary myelofibrosis
B. Chronic myeloid leukemia
C. Polycythemia vera
D. Essential thrombocytosis
----------------------------------------

Correct Answers
Question Correct Answer

Question 1 3
Question 2 1
Question 3 1
Question 4 2
Question 5 3
Question 6 1
Question 7 2
Question 8 2
Question 9 4
Question 10 4

Page 14

1411
Question 11 3
Question 12 1
Question 13 4
Question 14 1
Question 15 1
Question 16 1
Question 17 2
Question 18 3
Question 19 2
Question 20 2
Question 21 2
Question 22 2
Question 23 1
Question 24 1
Question 25 1
Question 26 1
Question 27 2
Question 28 1
Question 29 4
Question 30 1
Question 31 2
Question 32 4
Question 33 2
Question 34 2
Question 35 2
Question 36 2
Question 37 1
Question 38 2
Question 39 2
Question 40 1
Question 41 1
Question 42 1
Question 43 2
Question 44 3
Question 45 1

Page 15

1412
Question 46 3
Question 47 2
Question 48 3
Question 49 1
Question 50 1
Question 51 3
Question 52 3
Question 53 3

Solution for Question 1:


Correct Option C:
In the above case, there is splenomegaly and fatigue. The above blood work points towards a
probable diagnosis of CML. There is leukocytosis exceeding 100000 cells/mm3 and blasts are < 10%.
Incorrect Options:
Option A: AML- Usually seen in >
65 years age group and initial symptoms include fever, fatigue and weight loss’
Option B: ALL- Unlikely as the history is not acute and is mostly seen in children. TLC is usually within
normal limits and blast cells >20%.
Option D: CLL- is common in the pediatric age group, there is fatigue and weight loss.

Solution for Question 2:


Correct Option A:
In the above case, the patient presents with bleeding of gums and fever for 2 months, along with pallor,
pancytopenia, and bone marrow examination revealed fatty streaks and absent megakaryocytes but n
o immature cells. It points towards a diagnosis of idiopathic acquired aplastic anemia.
Incorrect Options:
Option B: Paroxysmal nocturnal hemoglobinuria is unlikely as no symptoms of myoglobinuria, passing r
eddish urine present. Bone marrow biopsy should show a cellular marrow.
Option C: Myelodysplastic syndrome is unlikely as reticulocyte count will be high, with complete marro
w fibrosis. Bone marrow biopsy should show hypercellularity.
Option D: Tuberculosis is unlikely as it has the clinical picture is suggestive of microcytic anemia, anem
ia of chronic disease with the presence of evening rise of fever.

Solution for Question 3:


The correct option is:

Page 16

1413
Option A. Decreased osmotic fragility
Explanation:
This patient likely has hereditary spherocytosis, a genetic disorder characterized by a
deficiency in spectrin or other proteins in the red blood cell membrane, leading to a spherical shape of t
he red blood cells (spherocytes). It is an inherited disorder that often presents with anemia, jaundice, a
nd splenomegaly.
The additional abnormalities seen in hereditary spherocytosis include:
Spherocytes are less able to withstand osmotic stress, making them more prone to hemolysis in hypot
onic solutions. This results in an increased fragility of the red blood cells when exposed to low osmolari
ty solutions.
Incorrect Options:
Option B. Decreased reticulocytes: This is incorrect. In hereditary spherocytosis, reticulocyte count is ty
pically elevated due to the bone marrow's compensatory response to hemolysis. Increased reticulocyte
s represent an attempt by the bone marrow to replace the prematurely destroyed red blood cells.
Option C. Heinz bodies: Heinz bodies are seen in conditions like G6PD deficiency, where oxidative stre
ss damages hemoglobin, leading to the formation of intracellular inclusions. Heinz bodies are not a
typical feature of hereditary spherocytosis.
Option D. Howell-jolly bodies: Howell-jolly bodies are small, round remnants of nuclear material that ar
e usually removed by the spleen. In hereditary spherocytosis, splenectomy is often performed to allevia
te hemolysis and anemia. However, Howell-jolly bodies are more commonly seen in individuals who ha
ve undergone splenectomy due to the absence of spleen function, not specifically in hereditary spheroc
ytosis itself.

Solution for Question 4:


Correct Option B:
Incorrect Options:
Option A. Megaloblastic anemia and MDS with hypocellular BM (incorrect):
• This option does not include hairy cell leukemia, which can also cause pancytopenia. Therefore, it is
not the correct option.
Option C. Megaloblastic anemia, MDS with hypocellular BM, and APML (incorrect):
• This option includes acute promyelocytic leukemia (APML), which is a subtype of acute myeloid
leukemia characterized by abnormal promyelocytes. While APML can cause pancytopenia, it is not
specifically associated with megaloblastic anemia. Therefore, this option is not correct.
Option D. Megaloblastic anemia, Hairy cell leukemia, and APML (incorrect):
• This option includes APML but does not include MDS with hypocellular BM. Therefore, it is not the
correct option.

Solution for Question 5:

Page 17

1414
Correct Option C
• The clinical presentation of a young boy from a high altitude with fever, weakness, high hemoglobin,
and increased TLC with a predominance of neutrophils and myeloid cells suggests reactive or
secondary polycythemia.
• In this context, it is important to assess the erythropoietin levels.
• In reactive polycythemia, EPO levels are expected to be low or normal due to an appropriate
physiological response to hypoxia.
• If EPO levels are elevated, it would raise concerns for primary polycythemia such as polycythemia
vera or an EPO-secreting tumor
Incorrect Option:
Option A: Philadelphia chromosome is a
genetic abnormality associated with chronic myeloid leukemia (CML), a type of blood cancer. However,
the given clinical presentation with high hemoglobin, increased total leukocyte count (TLC), and prese
nce of myelocytes and metamyelocytes in the peripheral smear does not suggest CML. Therefore, inve
stigating the Philadelphia chromosome is not indicated in this case.
Option B: JAK mutation refers to mutations in the Janus kinase (JAK) genes, which can be seen in cert
ain myeloproliferative neoplasms (MPNs) such as polycythemia vera, essential thrombocythemia, and
primary myelofibrosis. While the peripheral smear findings of increased myelocytes and metamyelocyt
es may raise suspicion for an MPN, the high haemoglobin level and low leukocyte count are not consis
tent with these disorders. Therefore, investigating JAK mutation is not the most appropriate next step.
Option D: Bone marrow biopsy with reticulin stain is a more invasive and specific investigation that may
be considered if the initial workup, including EPO levels, is inconclusive or if there are additional findin
gs suggestive of a
myeloproliferative disorder or myelodysplastic syndrome. However, in the given clinical scenario of a y
oung boy with high haemoglobin, increased TLC with myeloid predominance, and weakness, the more
appropriate initial step is to assess EPO levels before proceeding to a bone marrow biopsy.
Therefore, the correct option is C, to measure the erythropoietin levels.

Solution for Question 6:


Correct Option A - Beta thalassemia:
• The patient's presentation with recurrent chest infections, abdominal pain, and a history of delayed
growth and bone problems with icterus and mild splenomegaly on examination and the electrophoresis
results suggest beta thalassemia.
Incorrect Options:
Option B - HbC disease: Hemoglobin C is a benign hemoglobinopathy. It is caused by a mutation in the
beta-globin chain of hemoglobin. People with HbC disease have normal growth and development and
a normal life expectancy.
Option C
- Sickle cell disease: The electrophoresis result in sickle cell disease shows elevated levels of HbS.

Page 18

1415
Option D - Acute coronary disease: ST-elevation myocardial infarction (STEMI), non-ST elevation myo
cardial infarction (NSTEMI), and unstable angina are all categorized under the umbrella term acute cor
onary syndrome (ACS).

Solution for Question 7:


Correct Option B - Mechanical second heart valve:
• The patient's presentation with anemia, hemoglobinuria, and increased bilirubin and lactate
dehydrogenase with schistocytes on a peripheral smear suggests microangiopathic hemolytic anemia
or prosthetic cardiac valve.

Incorrect Options:

Page 19

1416
Options A, C, and D are incorrect.

Solution for Question 8:


Correct Option B - Serum ferritin levels:
• The patient's presentation with fatigue, a history of piles, low hemoglobin, and MCV with a peripheral
smear showing anisocytosis, poikilocytosis, and target cells is suggestive of microcytic hypochromic
anemia.
• Serum ferritin level is used to diagnose iron deficiency, which is the most common cause of microcytic
hypochromic anemia.
• While a normal or high level of serum ferritin does not necessarily rule out iron deficiency anemia, a
low level is highly suggestive.
Incorrect Options:
Options A, C, and D are not used to diagnose iron deficiency, which is the most common cause of micr
ocytic hypochromic anemia.

Solution for Question 9:


Correct Option D - Liver disease:
• There are several potential pathological mechanisms of macrocytic anemia associated with liver
disease.
• First, patients with liver disease are more likely to have vitamin B12 or folate deficiencies, which
directly result in macrocytic anemia.
• Vitamin B12 and folate coenzymes are required for thymidylate and purine synthesis; thus, their
deficiencies result in retarded DNA synthesis and eventually will develop into macrocytic anemia.
• Second, macrocytic anemia in liver disease may be due to an increased deposition of cholesterol on
the membranes of circulating RBCs.
• This deposition effectively increases the surface area of the erythrocyte.
Incorrect Options:
• In options A, B, and C, macrocytic anemia is not seen.

Solution for Question 10:


Correct Option D - Perls Prussian Blue:
• Hemosiderin is an iron-containing pigment that is produced by the breakdown of hemoglobin.
• It is commonly found in macrophages and can accumulate in various tissues in the body under certain
conditions, such as hemorrhage, inflammation, and iron overload disorders.
• Perls Prussian blue stain is a commonly used stain for the detection of hemosiderin.

Page 20

1417
• The stain is based on the reaction of potassium ferrocyanide and hydrochloric acid with iron in
hemosiderin, which produces a blue color.
• The staining is often performed on tissue sections from biopsy or autopsy specimens.
Incorrect Options:
Option A - Mason-Fontana stain:
• This stain is based on the oxidation of melanin by potassium permanganate, which results in the
formation of brown or black pigment.
• The stain is commonly used for the detection of melanin pigment in tissues, such as the skin, hair, and
eyes.
Option B - Oil Red O:
• This stain is a lipophilic dye that is used to detect lipids, such as triglycerides and cholesterol esters.
• The dye binds to the lipid droplets in tissues, producing a bright red color.
• The stain is commonly used for the detection of fat in frozen sections of tissue samples.
Option C - Sudan black:
• This stain is a lipophilic dye that is used to detect lipids in tissues.
• The stain binds to the lipid droplets in tissues, producing a dark blue-black color.
• The stain is commonly used for the detection of fat in frozen sections of tissue samples.

Solution for Question 11:


Correct Option C - Serum ferritin:
• Serum ferritin is considered the most sensitive and specific test for the diagnosis of iron deficiency
anemia.
• It reflects the amount of iron stored in the body and is considered an indirect measure of iron stores.
• A low serum ferritin level is the earliest indicator of iron deficiency anemia, and it can be used to
monitor the effectiveness of iron therapy.
Incorrect Options:
• Options A, B, and D are not specific to iron deficiency anemia.

Solution for Question 12:


Correct Option A - Hepatitis A:
• Before a blood transfusion, screening for infectious diseases such as hepatitis B, hepatitis C, HIV,
malaria, and syphilis is mandatory.
Incorrect Options:
• Options B, C, and D are incorrect. Refer to the explanation of the correct answer.

Page 21

1418
Solution for Question 13:
Correct Option D - Decreased Haptoglobin:
• In hemolytic anemia, a complete blood count will reveal reduced hemoglobin, an increased
reticulocyte count, neutrophilia with a shift to the left, and thrombocytosis.
• A peripheral smear shows large, nucleated polychromatophilic red cells and an increased count of
reticulocytes.
• Haptoglobin levels become depleted in the presence of large amounts of free hemoglobin.
• A decrease in serum haptoglobin is seen in intravascular hemolysis, i.e., paroxysmal nocturnal
hemoglobinuria and G6PD deficiency.
Incorrect Options:
• Options A, B, and C are incorrect. Refer to the explanation of the correct answer.

Solution for Question 14:


Correct Option A - Glutamate by valine at the 6th position:
• In sickle cell anemia, glutamate is replaced by valine at the 6th position of the beta-globin chain of
hemoglobin.
• This alters hemoglobin's structural integrity and stability, resulting in hemolytic anemia.
Incorrect Options:
• Options B, C, and D are incorrect.

Solution for Question 15:


Correct Option A - Osmotic fragility test- Hereditary spherocytosis:
• A child with complaints of intermittent jaundice and splenomegaly and a history of similar complaints
in the elderly brother with the presence of spherocytes in the peripheral smear suggests hereditary
spherocytosis.
• The osmotic fragility test is used for the diagnosis of hereditary spherocytosis.
Incorrect Options:
Option B - Coombs test- Autoimmune hemolytic anemia: There is a history of similar complaints in the
elderly brother, i.e., the condition is hereditary. Autoimmune hemolytic anemia is autoimmune, not here
ditary.
Option C - Genetic testing- G6PD deficiency: Spherocytes are not seen in G6PD deficiency.
Option D - Flow cytometry- Paroxysmal nocturnal hemoglobinuria: Spherocytes are not seen in paroxy
smal nocturnal hemoglobinuria.

Page 22

1419
Solution for Question 16:
Correct Option A - Klima needle:
• A Klima needle is used for bone marrow aspiration.
Incorrect Options:
• Options B, C, and D are incorrect.

Solution for Question 17:


Correct Option B - Anemia of chronic disorder:
• Anemia of chronic disorder, also known as anemia of chronic inflammation, is a type of anemia that
occurs in the setting of chronic diseases, infections, or inflammatory conditions.
• It is characterized by a normocytic or mildly microcytic anemia with low iron availability due to
inflammatory processes.
• In this case, the patient's low hemoglobin, low MCV, and low MCH indicate microcytic and
hypochromic anemia.
• Additionally, the elevated total leukocyte count (TLC) with a predominance of lymphocytes and
neutrophils is consistent with the chronic inflammatory state commonly seen in anemia of chronic
disorders.
Incorrect Options:
Option A - Thalassemia:
• Thalassemia is a genetic disorder characterized by abnormal production of hemoglobin.
• It typically presents with microcytic and hypochromic anemia.
• In this case, the patient's low hemoglobin, low MCV, and low MCH.
• Thalassemia usually presents with a normal or increased MCH.
Option C - Iron deficiency anemia:
• Iron-deficiency anemia is characterized by microcytic and hypochromic anemia.
• While the boy's low hemoglobin, low MCV, and low MCH are suggestive of microcytic anemia, the
normal serum ferritin level is not consistent with iron deficiency.
• Iron-deficiency anemia would typically present with a low serum ferritin level.
Option D - Sickle cell anemia:
• Sickle cell anemia is a genetic disorder characterized by abnormal hemoglobin structure, leading to
the production of sickle-shaped red blood cells.
• It is associated with chronic hemolytic anemia and presents with a low hemoglobin level.
• In this case, the patient's low hemoglobin, low MCV, and low MCH indicate microcytic and
hypochromic anemia. which is not characteristic of sickle cell anemia.

Page 23

1420
Solution for Question 18:
Correct Option C - 3:
• The correct reticulocyte count can be calculated using the following formula: Reticulocyte count =
(Absolute reticulocyte count percentage) x (Actual Hb / Normal Hb)
• The reticulocyte count calculated = 9 × (5/15) = 3
Incorrect Options:
• Options A, B, and D are incorrect.

Solution for Question 19:


Correct Option B - Iron deficiency anemia:
• The laboratory findings and peripheral smear suggest iron deficiency anemia.
• Iron deficiency anemia is characterized by microcytic, hypochromic, and decreased hemoglobin
levels.
• The peripheral smear can show hypochromic, microcytic red blood cells with anisocytosis and
poikilocytosis.
Incorrect Options:
Option A - Megaloblastic anemia:
• Megaloblastic anemia is characterized by macrocytic red blood cells.
Option C - Thalassemia major:
• Thalassemia major is a genetic disorder characterized by a deficiency in alpha or beta globin chains,
leading to anemia with small, pale, and abnormal red blood cells.
• This patient has a low MCV, which is characteristic of iron deficiency anaemia.
Option D - Essential thrombocytosis:
• Essential thrombocytosis is a disorder characterized by an increase in platelets.

Solution for Question 20:


Correct Option B - Acid hematin:
• The image shows a device called the Sahli hemoglobinometer, which is used to estimate hemoglobin
levels in blood.
• The hemoglobinometer measures the formation of acid hematin by adding a dilute acid to a small
amount of blood, which is then compared to a standard color chart to determine the hemoglobin
concentration.
• Acid hematin is a derivative of hemoglobin that is formed by the reaction of hemoglobin with an acid.

Page 24

1421
Incorrect Options:
• Options A, C, and D are incorrect.

Solution for Question 21:


Correct Option B - Prussian blue:
• Iron in tissues is stained by Prussian blue.
• Prussian blue staining is based on the ability of Fe3+ ions to react with ferrocyanide ions to form an
insoluble blue pigment.
• When tissue sections are treated with a solution of potassium ferrocyanide and hydrochloric acid, any
iron present in the tissue reacts with the ferrocyanide to produce the blue pigment.
• The resulting stain is specific for iron and is commonly used to detect iron deposition in diseases such
as hemochromatosis and hemosiderosis.
Incorrect Options:
Option A - Masson's trichrome: Masson's trichrome is a histological staining method that is used to diff
erentiate between collagen and muscle fibers in tissue sections.
Option C - PAS: Periodic acid-Schiff (PAS) staining is used to detect the presence of carbohydrates an
d glycogen in tissues.
Option D - Fite-Faraco: File-Faraco staining is a
method used to visualize fibrin and fibrinoid deposits in tissue sections.

Solution for Question 22:


Correct Option B - Low serum ferritin:
• Anemia of chronic disease is a type of anemia that occurs in people with chronic medical conditions
such as chronic infections, inflammatory diseases, and cancer.
• In anemia of chronic disease, the body's response to chronic inflammation or infection leads to
decreased production of red blood cells and anemia.
• The iron levels in the body are typically normal or high in anemia of chronic disease.
• This is because the body's response to chronic inflammation or infection increases the production of a
protein called hepcidin, which inhibits the absorption of iron from the diet and sequesters iron in the
body's stores, leading to increased ferritin levels.
Incorrect Options:
Option A - High serum hepcidin:
• Hepcidin is a hormone produced by the liver and is a key regulator of iron metabolism.
• In anemia of chronic disease, there is an increase in serum hepcidin levels.
• Hepcidin acts by reducing iron absorption from the gut and inhibiting iron release from macrophages,
ultimately leading to decreased iron availability for red blood cell production.

Page 25

1422
Option C - Chronic infection:
• Chronic infection is one of the underlying causes of anemia of chronic disease.
• Inflammatory mediators produced during chronic infection can disrupt iron metabolism and lead to
anemia.
Option D - Normal iron stores:
• In anemia of chronic disease, there is a disruption in iron utilization and distribution, leading to
functional iron deficiency.
• This means that although iron stores may appear normal or even elevated, there is a decreased
availability of iron for erythropoiesis.

Solution for Question 23:


Correct Option A - a, b, c, d:
Incorrect Options:
• Options B, C, and D are incorrect. Refer to the explanation of the correct answer.

Solution for Question 24:


Correct Option A - Hairy cell:
• The image given in the question is of a hairy cell with distinct cytoplasmic projections resembling hairy
projections.
• These cells have abundant cytoplasm with hair-like extensions, giving them a characteristic fried egg
appearance.
• They typically have a rounded or oval nucleus and pale blue cytoplasm.
Incorrect Options:
Option B - Gaucher cell:
• Gaucher cells are large, lipid-laden macrophages that are characteristic of Gaucher disease.
• These cells have a distinctive crinkled tissue paper or wrinkled silk appearance.
Option C - Sickle cell:
• Sickle cells seen in sickle cell anemia are red blood cells that are abnormally shaped due to the
presence of an abnormal form of hemoglobin called hemoglobin S.
• These cells have a characteristic crescent or sickle shape.
Option D - Target cell:
• Target cells are red blood cells with a distinctive appearance characterized by a dark central area
(bull's eye or target-like appearance) surrounded by a lighter ring of hemoglobin.
• Target cells are observed in thalassemia and liver diseases.

Page 26

1423
Solution for Question 25:
Correct option A: Transferrin saturation:
Incorrect options: Options B, C, and D are incorrect. Refer to the explanation of the correct answer.

Solution for Question 26:


Correct Option A - Idiopathic acquired aplastic anemia:
• The patient's presentation with gum bleeding and fever for the past 2 months with low Hb, leucocyte
count, and platelet count with peripheral smear showing macrocytes and bone marrow biopsy showing
fatty streaks and absent megakaryocytes but no immature cells is suggestive of idiopathic acquired
aplastic anemia.
• This condition is characterized by pancytopenia due to bone marrow failure.
• The absence of immature cells indicates a primary defect in the bone marrow's ability to produce
blood cells.
Incorrect Options:
Option B - Paroxysmal nocturnal hemoglobinuria:
• It is a rare acquired disorder characterized by hemolytic anemia and hemoglobinuria.
• The peripheral smear shows evidence of hemolysis, such as schistocytes and spherocytes.
Option C - Myelodysplastic syndrome:
• It is a group of disorders characterized by ineffective blood cell production in the bone marrow, leading
to cytopenias and dysplastic changes in the blood cells.

Page 27

1424
• While macrocytosis and cytopenias may be present in myelodysplastic syndrome, the absence of
megakaryocytes on bone marrow examination suggests a different diagnosis.
Option D - Tuberculosis:
• Tuberculosis is an infectious disease caused by Mycobacterium tuberculosis and primarily affects the
lungs.

Solution for Question 27:


Correct Option B - Iron deficiency:
• Iron deficiency is a common cause of anemia and presents as microcytic anemia, i.e., MCV < 80 fl,
but can present as normocytic anemia, i.e., MCV = 80 - 100 fl in the early stages.
Incorrect Options:
Option A - Cyanocobalamin deficiency:
• Cyanocobalamin (vitamin B12) deficiency presents as macrocytic anemia, i.e., MCV > 100 fl.
Option C - Folic acid deficiency:
• Folic acid deficiency presents as macrocytic anemia, i.e., MCV > 100 fl
Option D - Hereditary spherocytosis:
• Hereditary spherocytosis is an inherited condition characterized by spherocytes on peripheral smears.
• It presents with hemolytic anemia, jaundice, and splenomegaly.

Solution for Question 28:


Correct Option A - Thymoma:
• Thymoma is a tumor that originates from the thymus gland located in the mediastinum.
• Thymomas are commonly associated with autoimmune diseases and paraneoplastic syndromes,
such as pure red cell aplasia.
• Pure red cell aplasia is a rare condition characterized by a reduction in or absence of red blood cell
precursors in the bone marrow.
Incorrect Options:
Options B, C, and D are not associated with pure red cell aplasia.

Solution for Question 29:


Correct Option D - Chronic alcoholism:
• Chronic alcoholism is associated with several hematological abnormalities, including macrocytic
anemia.

Page 28

1425
• The most common cause of macrocytic anemia in chronic alcoholism is folate deficiency.
• Alcohol interferes with folate absorption and utilization, leading to reduced folate levels and impaired
DNA synthesis.
• This results in the production of macrocytes and hypersegmented neutrophils.
Incorrect Options:
Option A - Lead poisoning:
• Lead poisoning typically causes microcytic anemia rather than macrocytic anemia.
• Lead poisoning can also lead to basophilic stippling on a peripheral smear examination.
Option B - Iron Deficiency anemia:
• Iron deficiency anemia is characterized by microcytic and hypochromic cells rather than macrocytic
cells.
Option C - Hemolytic anemia:
• Hemolytic anemia is a condition characterized by the premature destruction of red blood cells,
resulting in a low hemoglobin level.
• A peripheral smear examination in hemolytic anemia shows signs of red blood cell destruction, such
as schistocytes or spherocytes.

Solution for Question 30:


Correct Option A - Vitamin B12 deficiency:
• Vitamin B12 deficiency can lead to a condition known as megaloblastic anemia, characterized by the
presence of macrocytes and hypersegmented neutrophils on the peripheral blood smear.
• It can result from various causes, including pernicious anemia, malabsorption due to atrophic gastritis,
or inadequate dietary intake.
Incorrect Options:
Option B - Folate deficiency:
• Folate deficiency can cause megaloblastic anemia similar to vitamin B12 deficiency, with macrocytes
and hypersegmented neutrophils.
• However, in this question, the patient's endoscopy revealing atrophic gastritis suggests vitamin B12
deficiency rather than folate deficiency.
Option C - Pyridoxine deficiency:
• Pyridoxine deficiency does not present with the characteristic peripheral blood smear findings as
described in this question.
Option D - Niacin deficiency:
• Niacin deficiency can lead to a condition known as pellagra, which is characterized by the classic triad
of dermatitis, diarrhea, and dementia.

Page 29

1426
Solution for Question 31:
Correct Option B - Leukemia:
• Leukemia is not likely to be seen in a patient with paroxysmal nocturnal hemoglobinuria.
• PNH is a rare acquired hematopoietic stem cell disorder characterized by a deficiency of
glycosylphosphatidylinositol (GPI)-anchored proteins on the surface of blood cells, particularly red
blood cells.
• A mutation in the PIGA gene causes this deficiency.
• While PNH is associated with various complications, including hemolysis, aplastic anemia, and
thrombosis, it does not lead to the development of leukemia.
Incorrect Options:
Option A - Aplastic anemia:
• Aplastic anemia is commonly associated with paroxysmal nocturnal hemoglobinuria.
• In PNH, the bone marrow fails to produce an adequate number of blood cells, including RBCs, white
blood cells, and platelets.
• This bone marrow failure can manifest as aplastic anemia, characterized by a decrease in all three
blood cell lineages.
• Aplastic anemia is a frequent complication of PNH and can contribute to the clinical manifestations
and morbidity associated with the condition.
Option C - Thrombosis:
• Thrombosis is a common feature of paroxysmal nocturnal hemoglobinuria.
• PNH is associated with the presence of abnormal RBCs lacking GPI-anchored proteins, such as
CD55 and CD59, which protect against complement-mediated hemolysis.
• The absence of these proteins leads to chronic intravascular hemolysis, resulting in the release of free
hemoglobin and activation of the complement system.
• The activation of the complement system and subsequent formation of complement-mediated
microthrombi can lead to thrombosis in various organs, including the liver, kidneys, and brain.
Option D - Hemolysis:
• Hemolysis is a hallmark feature of paroxysmal nocturnal hemoglobinuria.
• The absence of GPI-anchored proteins on the surface of RBCs renders them susceptible to
complement-mediated destruction.
• The complement system is activated by the presence of abnormal RBCs, leading to intravascular
hemolysis and the release of free hemoglobin into the circulation.
• The hemolysis in PNH is often paroxysmal, meaning it occurs in episodes and is exacerbated during
periods of stress or at night.

Solution for Question 32:


Correct Option D - Clot activator:
• The vacutainer tube shown in the image contains a clot activator.

Page 30

1427
• A clot activator is a substance added to the tube to promote blood clotting.
• It enhances the blood coagulation process, causing the blood sample to clot more quickly.
• This type of tube is commonly used for obtaining serum samples for various laboratory tests.
Incorrect Options:
Option A - Na-EDTA:
• Na-EDTA stands for sodium ethylenediaminetetraacetic acid.
• It is an anticoagulant used to prevent blood from clotting.
• It is commonly used for tests that require whole blood or plasma samples.
Option B - Trisodium citrate:
• Trisodium citrate is another anticoagulant used to prevent blood clotting.
• It is commonly used for coagulation tests, such as prothrombin time (PT) and activated partial
thromboplastin time (aPTT).
• The tube used for collecting samples with trisodium citrate is usually a blue-top tube.
Option C - NAF-heparin:
• NAF stands for sodium fluoride and is commonly used as a preservative for glucose testing.
• Heparin is an anticoagulant that inhibits blood clotting.
• A NAF-heparin tube is typically used for glucose and lactate measurements.

Solution for Question 33:


Correct Option B - Vitamin B6:
• The treatment of sideroblastic anemia involves the use of vitamin B6 (pyridoxine).
• Sideroblastic anemia is a condition characterized by defective heme synthesis in the bone marrow,
leading to the accumulation of iron within the mitochondria of red blood cell precursors (sideroblasts).
• Vitamin B6 is essential for the normal function of enzymes involved in heme synthesis.
• It is a cofactor for the enzyme aminolevulinic acid synthase, which catalyzes the first step in heme
synthesis.
• By providing an adequate supply of vitamin B6, the activity of this enzyme is restored, promoting
proper heme synthesis and alleviating the symptoms of sideroblastic anemia.
Incorrect Options:
• Options A, C, and D are incorrect.

Solution for Question 34:


Correct Option B - Ferritin translation is decreased by binding IRP to IRE:

Page 31

1428
• In iron deficiency anemia, the intracellular regulation of iron involves the interaction between iron
regulatory proteins (IRPs) and iron-responsive elements (IREs) present in the mRNA molecules of key
iron-related proteins.
• Ferritin is a protein that serves as an intracellular storage form of iron.
• Its translation is regulated by the binding of IRP to the IRE present in the mRNA of ferritin.
• When intracellular iron levels are low, IRP binds to the IRE in the ferritin mRNA, preventing its
translation.
• This leads to a decrease in the production of ferritin, which helps conserve iron and prevent its
storage when it is limited.
Incorrect Options:
Option A - Ferritin translation is increased by binding IRP to IRE:
• In iron deficiency anemia, the binding of IRP to the IRE in the mRNA of ferritin leads to a decrease in
ferritin translation.
• This is to conserve iron and prevent its storage when it is limited.
Option C - Transferrin receptor 1 translation is unaffected by binding IRP to IRE:
• Transferrin receptor 1 (TfR1) is a protein involved in the uptake of iron into cells.
• Its translation is also regulated by the interaction between IRP and IRE.
• In iron deficiency, IRP binds to the IRE in the mRNA of TfR1, stabilizing the mRNA and increasing its
translation.
• This results in an increased production of TfR1, which enhances the uptake of iron into cells to
compensate for the deficiency.
Option D - Transferrin receptor 1 translation is decreased by binding IRP to IRE:
• In iron deficiency anemia, the binding of IRP to the IRE in the mRNA of TfR1 leads to an increased
translation of TfR1.
• This helps facilitate the increased uptake of iron into cells to overcome the deficiency.

Solution for Question 35:


Correct Option B - Cells are Spherical:
• Glucose-6-phosphate dehydrogenase (G6PD) deficiency is an inherited enzyme deficiency that
affects red blood cells.
• It is characterized by a deficiency of the G6PD enzyme, which plays a role in protecting red blood
cells from oxidative damage.
• In G6PD deficiency, the red blood cells are more susceptible to oxidative stress and may undergo
hemolysis.
• One of the morphological characteristics associated with G6PD deficiency is the presence of spherical
or round-shaped red blood cells.
• This is due to the loss of membrane deformability and the increased rigidity of the cells.
Incorrect Options:

Page 32

1429
Option A - Spectrin defect:
• Spectrin is a structural protein that helps maintain the shape and flexibility of red blood cells.
• Spectrin deficiency is the most common deficiency found in hereditary spherocytosis.
Option C - Cells do not lyse in hypotonic solution:
• Hypotonic solutions can cause the swelling and lysis of red blood cells, which can be more
pronounced in G6PD-deficient cells.
Option D - Cells lyse in hypertonic solution:
• Hypotonic solutions can cause the swelling and lysis of red blood cells, which can be more
pronounced in G6PD-deficient cells.

Solution for Question 36:


Correct Option B - Copper in tissue:
• Supravital stains are used to stain living cells.
• Copper is a metal that is non-living, so supravital stains cannot be used to detect copper.
Incorrect Options:
• Options A, C, and D can be identified using supravital stains.

Solution for Question 37:


Correct Option A - Paroxysmal Nocturnal Hemoglobinuria:
• CD59 is a regulatory protein that inhibits the complement system, specifically membrane attack
complex (MAC) formation.
• In PNH, there is a deficiency of CD59 on the surface of red blood cells, making them susceptible to
complement-mediated lysis.
Incorrect Options:
• Options B, C, and D are incorrect.

Solution for Question 38:


Correct Option B - RBC = 2-6ºC, Platelet = 20-24ºC, FFP = -18ºC:
• Red blood cells: The storage temperature mentioned for RBCs is 2-6ºC.
• Platelets: Platelets are stored at room temperature 20-24ºC.
• Fresh Frozen Plasma: FFP is typically stored at a temperature of -18ºC or colder.
Incorrect Options:
• Options A, C, and D are incorrect.

Page 33

1430
Solution for Question 39:
Correct Option B - It should be completed within 4 hours of receiving it from the blood bank:
• The transfusion should be completed within 4 hours of receiving the blood from the blood bank.
• The purpose of this time restriction is to ensure the safety and effectiveness of the transfusion by
minimizing the risk of bacterial contamination and maintaining the quality of the blood product.
Incorrect Options:
• Options A, C, and D are incorrect.

Solution for Question 40:


Correct Option A - High Ferritin:
Incorrect Options:
• Options B, C, and D are incorrect.

Solution for Question 41:


Correct Option A - Serum Ferritin:
• The most sensitive indicator for iron deficiency anemia is serum ferritin.
• Ferritin is a protein that stores iron in the body.
• In cases of iron deficiency, the iron stores are depleted, leading to low levels of serum ferritin.
Incorrect Options:
• Options B, C, and D are incorrect.

Solution for Question 42:


Correct Option A - Natural Killer cells:
• In the given case of a positive dengue NS-1 antigen test, the first line of defense of the body would
involve the activation of NK cells.
• NK cells are a type of lymphocyte that plays a crucial role in the innate immune response against viral
infections.
• They can recognize and eliminate virus-infected cells without prior exposure or specific recognition of
the virus.
Incorrect Options:

Page 34

1431
• Options B, C, and D are not the primary first-line defense against viral infections in the given case.

Solution for Question 43:


Correct Option B - Phytates:
• Phytates, also known as phytic acid, are naturally occurring compounds found in many plant-based
foods, such as whole grains, legumes, and nuts.
• Phytates have the ability to bind to dietary iron and form insoluble complexes, which can inhibit the
absorption of iron in the intestines.
• This interference with iron absorption can contribute to iron deficiency in individuals who rely heavily
on plant-based diets or consume a high amount of phytate-rich foods.
Incorrect Options:
Option A - Vitamin C:
• Vitamin C, also known as ascorbic acid, actually enhances the absorption of non-heme iron, which is
the form of iron found in plant-based foods.
• Vitamin C can help convert non-heme iron into a more absorbable form, increasing iron absorption.
Option C - Oxalate:
• Oxalate, found in foods like spinach, rhubarb, and beet greens, can bind to calcium and form insoluble
crystals.
• Oxalates interfere with calcium absorption.
Option D - Myoglobin:
• Myoglobin is an iron-containing protein found in muscle tissue.

Solution for Question 44:


Correct Option C - Multiple Myeloma:
• Multiple myeloma can cause elevated serum calcium levels.
• It is a malignancy of plasma cells and can lead to bone destruction and an increased release of
calcium into the bloodstream.
Incorrect Options:
Option A - Vitamin D intoxication: Vitamin D intoxication can lead to elevated serum calcium levels.
Option B - Hyperparathyroidism: Hyperparathyroidism is characterized by excessive production of para
thyroid hormone, which can lead to increased calcium levels.
Option D - Nutritional rickets: Nutritional rickets is a condition characterized by vitamin D
deficiency, which can lead to low calcium levels.

Page 35

1432
Solution for Question 45:
Correct Option A - Total iron binding capacity:
Incorrect Options:
• Options B, C, and D are incorrect. Refer to the explanation of the correct answer.

Solution for Question 46:


Correct Option C - RBC pipette:
• The Rees and Ecker diluting fluid is a specific type of diluent used in the RBC pipette for performing a
manual RBC count.
• The RBC pipette is a calibrated glass tube used in hematology to measure the concentration of red
blood cells in a blood sample.
• The Rees and Ecker diluting fluid is added to the RBC pipette along with a measured volume of blood
sample.
• The diluting fluid helps in lysing the red blood cells, releasing their contents, and allowing for accurate
counting and measurement.
• It also provides the appropriate environment for the red blood cells to maintain their shape and
integrity during the counting process.
Incorrect Options:
• Rees and Ecker diluting fluid is not used in options A, B, and D.

Solution for Question 47:


Correct Option B - B12 deficiency:
• The presence of macro-ovalocytes and hypersegmented neutrophils are characteristic findings of
megaloblastic anemia, which is commonly associated with vitamin B12 deficiency.
• In B12 deficiency, impaired DNA synthesis leads to delayed maturation of red blood cells and white
blood cells, resulting in the observed morphological abnormalities.
Incorrect Options:
Option A - Iron deficiency anemia: Iron deficiency anemia typically presents with microcytic and hypoch
romic red blood cells.
Option C - Hereditary spherocytosis: Hereditary spherocytosis is a
congenital disorder characterized by spherocytes with increased fragility.
Option D - Acute myeloid leukemia: Auer rods are cytoplasmic inclusions found only in the leukaemic c
ells of some cases of acute myeloid leukaemia.

Page 36

1433
Solution for Question 48:
Correct Option C - TIBC increased, transferrin saturation decreased, ferritin decreased:
Incorrect Options:
• Options A, B, and D are incorrect. Refer to the explanation of the correct answer.

Solution for Question 49:


Correct Option A - Contraindicated if coagulopathy:
• The given image is of a bone marrow biopsy needle.
• Performing a bone marrow biopsy is contraindicated in cases where coagulopathy is present.
Incorrect Options:
Option B - Breath holding is required:
• Breath holding is not required during a bone marrow biopsy.
• The procedure involves the insertion of a needle into the bone marrow cavity to obtain a sample, and
patients are usually instructed to lie still and cooperate to minimize movement during the procedure.
Option C - Procedure cannot be done in both lateral and prone positions:
• A bone marrow biopsy can be performed with the patient in either a lateral or prone position.
• The choice of position depends on the preference of the physician and patient comfort.
Option D - Diagnosing an infiltrate or granulomatous disease:
• A bone marrow biopsy is a diagnostic procedure used to evaluate various conditions, including
hematologic disorders, metastatic cancer, infections, and bone marrow failure syndromes.
• While an infiltrate or granulomatous disease may be detected through a bone marrow biopsy, it is not
specific to this procedure alone.

Solution for Question 50:


Correct Option A - Polycythemia vera:
• The JAK2 mutation is most commonly associated with polycythemia vera, a myeloproliferative
neoplasm characterized by an overproduction of red blood cells.
• In polycythemia vera, the JAK2 mutation leads to the constitutive activation of the JAK-STAT signaling
pathway, which results in uncontrolled proliferation and survival of red blood cell precursors.
• This causes an increase in the number of red blood cells in the bloodstream, leading to thickening of
the blood and potential complications such as thrombosis.
Incorrect Options:
Option B - Essential thrombocytosis:
• While JAK2 mutations can also occur in ET, they are less common compared to polycythemia vera.

Page 37

1434
• ET is characterized by the overproduction of platelets in the bone marrow, leading to an increased risk
of abnormal blood clotting.
Option C - CML:
• The majority of CML is associated with the Philadelphia chromosome, which results from a reciprocal
translocation between chromosomes 9 and 22.
• This translocation leads to the formation of the BCR-ABL fusion gene, which drives the uncontrolled
proliferation of myeloid cells.
Option D - PMF:
• While JAK2 mutations can also occur in PMF, they are less common compared to polycythemia vera.
• Primary myelofibrosis is characterized by the excessive production of abnormal blood cells.

Solution for Question 51:


Correct Option C - Iron deficiency anemia:
• The image given shows koilonychia, which is a deformity of the nails where the central portion of the
nail is depressed and the lateral aspects of the nail are elevated.
• The patient's presentation with easy fatiguability and pallor with koilonychia is suggestive of iron
deficiency anemia.
Incorrect Options:
Option A - Aplastic anemia:
• Aplastic anemia is a condition characterized by bone marrow failure, leading to a decrease in the
production of all blood cells, including red blood cells, white blood cells, and platelets.
Option B - Vitamin B12 deficiency:
• Vitamin B12 deficiency can cause megaloblastic anemia and can present with additional neurological
symptoms like numbness and tingling.
Option D - Hypoalbuminemia:
• Hypoalbuminemia refers to low levels of albumin.
• The appearance of the paired, narrow, white bands on the fingernails is seen in hypoalbuminemia.

Solution for Question 52:


Correct Option C - Thalassemia:
• The peripheral smear shows a microcytic hypochromic picture with target cells.
• The Mentzer index is calculated as MCV / RBC count.
• If the result is >13, iron deficiency anemia is the probable diagnosis, and a value < 13 indicates the
likelihood of β-thalassemia.
• This, along with the clinical history, suggests that the child is suffering from β-thalassemia.

Page 38

1435
Incorrect Options:
Option A - Megaloblastic anemia:
• The two principal causes of megaloblastic anemia are folate deficiency and vitamin B12 deficiency.
• In the peripheral blood, the earliest change is the appearance of hypersegmented neutrophils and
macroovalocytes.
• The bone marrow is markedly hypercellular and contains numerous megaloblasts and giant
metamyelocytes.
Option B - Iron deficiency anemia:
• Iron deficiency is the most common nutritional deficiency in the world.
• Diagnostic criteria include anemia, hypochromic and microcytic red cell indices, low serum ferritin and
iron levels, low transferrin saturation, and increased total iron-binding capacity.
Option D - Pernicious anemia:
• The most frequent cause of vitamin B12 deficiency is pernicious anemia, which results from an
autoimmune attack on the gastric mucosa that suppresses the production of intrinsic factors.

Solution for Question 53:


Correct Option C - Polycythemia Vera:
• The clinical presentation of a 40-year-old male with complaints of itching during baths and laboratory
findings of increased hemoglobin, decreased platelet count, and increased white blood cell count is
suggestive of polycythemia vera.
• Polycythemia vera is a myeloproliferative neoplasm characterized by the overproduction of red blood
cells.
• It is usually associated with an increase in hemoglobin and hematocrit.
• Patients with polycythemia vera may experience symptoms such as pruritus (itching), especially after
a warm bath or shower, due to increased histamine release caused by increased basophils in the
blood.
Incorrect Options:
Option A - Primary myelofibrosis:
• Primary myelofibrosis, also known as idiopathic myelofibrosis, is a myeloproliferative neoplasm
characterized by bone marrow fibrosis, extramedullary hematopoiesis, and cytopenias.
• It typically presents with anemia, splenomegaly, and other constitutional symptoms.
Option B - Chronic myeloid leukemia:
• Chronic myeloid leukemia (CML) is a myeloproliferative neoplasm characterized by the Philadelphia
chromosome, resulting from the reciprocal translocation between chromosomes 9 and 22.
• CML typically presents with an increased white blood cell count, especially neutrophils, and
splenomegaly.
Option D - Essential thrombocytosis:

Page 39

1436
• Essential thrombocytosis, also known as primary thrombocythemia, is a myeloproliferative neoplasm
characterized by the overproduction of platelets.
• It typically presents with an increased platelet count but does not usually cause significant changes in
hemoglobin or white blood cell count.

Page 40

1437
Previous Year Questions
1. What is the involvement of the following in the development of rheumatoid arthritis?
A. IgE mediated
B. Defective cellular and humoral immunity
C. Autoimmunity
D. Chronic microbial infections
----------------------------------------
2. Regarding this condition, which of the following statements is true? An 8-year-old boy is experiencing
progressive muscle weakness and difficulties with walking. During examination, pseudohypertrophy of
the calf muscles was observed.
A. Abnormal collagen
B. Absent dystrophin
C. Expansions of CTG triplet repeats
D. Defective fibrillin
----------------------------------------
3. A child presents with multiple skin lesions over the inner thigh and genital areas. It initially started as
macule and later developed to become nodules. Some nodules have central umbilication. The histology
of the lesion is shown below. What would be found on biopsy?

A. Multinucleate giant cells (Tzanck cells)


B. Donovan bodies
C. Henderson – Patterson bodies
D. Koilocytes
----------------------------------------
4. A woman presents with lesions on the inner thighs and peri – anal region. They are nodular, 4-6 mm
in size and appear pale. The histopathological image shows multiple intracytoplasmic inclusion bodies
consistent with Henderson – Patterson bodies. The diagnosis is:

1438
1439
1440
1441
10. A 36 year old male with skin – colored umbilicated papules. Microscopy is as shown below. The
most likely diagnosis is?

A. Herpes virus
B. Verruca vulgaris
C. Molluscum contagiosum
D. Lichen planus
----------------------------------------
11. Which of the subsequent choices is not classified as an intermediate filament?
A. Vimentin
B. Tubulin
C. Desmin
D. Neurofilament
----------------------------------------

Correct Answers
Question Correct Answer

Question 1 3
Question 2 2
Question 3 3
Question 4 3
Question 5 3
Question 6 1
Question 7 2
Question 8 2
Question 9 2
Question 10 3
Question 11 2

Page 4

1442
Solution for Question 1:
• The pathogenesis of rheumatoid arthritis starts with an antigenic exposure (like an infectious agent) in
an individual who is genetically predisposed, causing activation of CD4+ T-cells thus releasing
cytokines like TNF, IL-1,6, and interferon, which in turn activates endothelial cells, B lymphocytes, and
macrophages. Activation of B lymphocytes results in the release of IgM antibodies against IgG
(Rheumatoid factor).
• This immune complex triggers inflammatory damage to the blood vessels, synovium, and collagen.
Further activation of macrophages releases more cytokines that damage the joints and the blood
vessels of cartilages, termed pannus formation. As a result of the destruction of bone and cartilage,
fibrosis takes place, which produces visible joint deformities.
Incorrect Choices:
• Option a. IgE mediated: Rheumatoid arthritis is not IgE mediated. It is IgM mediated.
• Option b. Defective cellular and humoral immunity: The exact etiopathogenesis of rheumatoid arthritis
is unknown; however, certain genetic factors, environmental factors, and autoimmunity play a major
role in initiating inflammation.
• Option d. Chronic microbial infections: Certain microbial agents like viruses and mycoplasma have
been considered potential triggers of rheumatoid arthritis. Chronic infections will likely trigger the
immune system and lead to an autoimmune response. However, it has not been proven and would
require more studies to establish an exact relationship between rheumatoid arthritis and chronic
infections.

Solution for Question 2:


• The diagnosis in the given case scenario is Duchenne’s muscular dystrophy. It is an X-linked
recessive disorder. It is usually seen by 5 years of age. It occurs due to a mutation in the dystrophin
gene which leads to weakness initially in the proximal muscles, later involving the distal muscles.
Clinical features include pseudohypertrophy of calf muscles, waddling gait, and lumbar lordosis. Other
systems are involved causing cardiomegaly and reduced intelligence. Death usually occurs by 20 years
of age due to paralysis of the muscles of respiration.
Incorrect Choices:
• Option a. Abnormal collagen: Abnormal collagen is found in a few diseases like scurvy, Ehler danlos
syndrome, osteogenesis imperfecta, and Marfan syndrome. However, pseudohypertrophy of calf
muscles is a characteristic finding of Duchenne muscular dystrophy.
• Option c. Expansion of CAG triplet repeats: It is a genetic mutation that is usually seen in Huntington's
disease and, type 1 myotonic dystrophy, type 8 spinocerebellar ataxia. However, It is unrelated to
Duchenne muscular dystrophy.
• Option d. Defective fibrillin: Defective fibrillin is seen in Marfan's syndrome. These patients are tall with
long slender arms, long fingers, and toes, ocular lens dislocations, aortic dissection/aneurysm, and
scoliosis.

Solution for Question 3:

Page 5

1443
Option B: Anti-La Antibody
• Anti-La antibody is not specific for scleroderma. It is associated with other autoimmune diseases like
Sjögren's syndrome, where it targets the La antigen found in the nucleus and cytoplasm of cells.
• Sjögren's syndrome is characterized by lymphocytic infiltration of exocrine glands, leading to dryness
of the eyes and mouth. The presence of anti-La antibodies contributes to the autoimmune attack on
these glands.
Option C: Anti-Jo-1 Antibody
• Anti-Jo-1 antibody is associated with dermatomyositis, not scleroderma. Dermatomyositis is an
autoimmune inflammatory muscle disease characterized by muscle weakness and skin manifestations,
such as a distinctive rash such as Gottron papules and Heliotrope rash
• Dermatomyositis involves inflammatory infiltrates in the muscles and skin. Anti-Jo-1 antibody targets
histidyl-tRNA synthetase, contributing to muscle inflammation and damage.
Option D:Anti-U1 RNP Antibody
• Anti-U1 RNP antibody is associated with mixed connective tissue disease (MCTD), not specifically
scleroderma.
• MCTD is characterized by overlapping features of several autoimmune diseases, including systemic
lupus erythematosus (SLE), scleroderma, polymyositis, and rheumatoid arthritis. Anti-U1 RNP
antibodies target components of the U1 small nuclear ribonucleoprotein particle, leading to immune
complex deposition and tissue damage.

Solution for Question 7:


Correct option B. Langerhans cell histiocytosis.
• Langerhans cell histiocytosis (LCH) is a rare disorder characterized by the proliferation and
accumulation of Langerhans cells, a type of immune cell, in various tissues. LCH can affect multiple
organs, including the bone, skin, and lymph nodes. In the context of the given X-ray showing a
punched-out lytic lesion in the cranium, LCH becomes a probable diagnosis.
Incorrect options:
Option A . Multiple myeloma: Multiple myeloma is a
type of blood cancer that involves the abnormal proliferation of plasma cells, a type of white blood cell,
in the bone marrow. Multiple myeloma is usually associated with abnormal hematological and urinary t
est results, which were reported as normal in this case.
Option C . Metastasis: Metastatic lesions can be seen in various organs, including the bone. However,
the X-ray findings alone cannot definitively indicate metastasis. A detailed clinical history and further in
vestigations, such as imaging studies and biopsy, are necessary to identify the primary tumor and confi
rm metastasis.
Option D . Hyperparathyroidism: Hyperparathyroidism is a condition characterized by excessive secreti
on of parathyroid hormone (PTH) by the parathyroid glands, leading to high levels of calcium in the blo
od. While hyperparathyroidism can cause bone changes, such as osteopenia or osteoporosis, the typic
al X-ray appearance associated with hyperparathyroidism is not a punched-out lytic lesion.

Page 8

1444
Solution for Question 10:
Correct Option C:
Molluscum contagiosum is the correct answer. It is a viral skin infection caused by the molluscum cont
agiosum virus (MCV). Molluscum contagiosum presents as skin-coloured or pearly umbilicated papules
. Microscopically, it shows characteristic intracytoplasmic inclusions called molluscum bodies. These b
odies consist of aggregates of viral particles surrounded by host cell reaction. The given clinical presen
tation of skin-coloured umbilicated papules along with the provided microscopic image of molluscum bo
dies strongly suggests molluscum contagiosum.
Incorrect Option:
Option A: Herpes virus. Herpes virus infections typically present with grouped vesicles or ulcers and ar
e associated with a burning or tingling sensation. The microscopic appearance of herpes virus infection
would show multinucleated giant cells and intranuclear inclusions called Cowdry bodies. This descripti
on does not match the clinical and microscopic findings provided, so herpes virus is an incorrect option
.
Option B: Verruca vulgaris, also known as common warts, is caused by the human papillomavirus (HP
V). Verruca vulgaris presents as rough, hyperkeratotic papules with a characteristic cauliflower-like app
earance. Microscopically, verruca vulgaris would show hyperkeratosis, acanthosis, and presence of koi
locytes (enlarged keratinocytes with perinuclear clearing). The given clinical presentation and microsco
pic image do not correspond to verruca vulgaris, so this option is incorrect.
Option D: Lichen planus is a chronic inflammatory skin condition characterized by pruritic, polygonal, fl
at-topped papules. The microscopic findings in lichen planus include band-like lymphocytic infiltrate at t
he dermo-epidermal junction, known as sawtooth appearance. Lichen planus is not associated with the
clinical or microscopic features described in the case, so this option is incorrect.

Solution for Question 11:


Correct Option B.
Tubulin is not an intermediate filament. It is a protein that polymerizes to form microtubules, which are i
nvolved in various cellular processes such as intracellular transport, cell division, and maintenance of c
ell structure.
Incorrect Option:
Option A: Vimentin is an intermediate filament protein found in various types of cells, including mesenc
hymal cells, fibroblasts, and endothelial cells. It provides structural support to the cytoskeleton and help
s maintain cell shape and integrity.
Option C: Desmin is an intermediate filament protein primarily found in muscle cells, including skeletal,
cardiac, and smooth muscle cells. It provides structural support to muscle fibers and plays a
role in maintaining muscle integrity.
Option D: Neurofilament is an intermediate filament protein specifically found in neurons. It contributes
to the structural organization of neuronal axons and provides mechanical support to the axons, allowin
g for efficient signal transmission.

Page 10

1445
A. Kikuchi disease
B. Infectious mononucleosis
C. Hodgkins lymphoma
D. Burkitt lymphoma
----------------------------------------
28. What is the most likely diagnosis for a 60-year-old male patient who has presented with anemia and
hepatosplenomegaly? The peripheral smear revealed teardrop cells, nucleated red blood cells, and
abnormally large platelets, while the bone marrow biopsy findings are provided below.

A. Polycythemia vera
B. Essential thrombocythemia
C. Primary myelofibrosis
D. Chronic myeloid leukemia
----------------------------------------
29. What is the most probable diagnosis for a 35-year-old man who is experiencing nosebleeds,
bleeding gums, and multiple ecchymoses, with laboratory findings showing a hemoglobin level of 8.5, a
platelet count of 12,000, and a total leukocyte count of 52,000 with 83% blasts that are CD33+?

Page 9

1446
A. Acute myeloid leukemia
B. Acute lymphoblastic leukemia
C. Chronic lymphocytic leukemia /small lymphocytic lymphoma
D. Myelodysplastic syndrome
----------------------------------------
30. What is the type of cell that the cell displayed below originates from?

A. NK cells
B. CD4 cells
C. B cells
D. CD8 cells
----------------------------------------
31. All of the following statements are true regarding neutrophil extracellular trapping (NET), except?
A. It is detected in blood during sepsis
B. It is produced in response to bacterial infection
C. Mitochondrial DNA is seen
D. It is chromatin with antibacterial enzymes
----------------------------------------
32. A 56-year-old man presents with dragging pain in the abdomen. On examination, there is massive
splenomegaly. A peripheral smear shows leukocytesis with increased myelocytes, metamyelocytes,
and basophils. Which of the following translocations is seen in this condition?

Page 10

1447
A. t(9;22)
B. t(8;21)
C. t(15;17)
D. t(8;14)
----------------------------------------
33. An African child is found to have a mass in the jaw. A biopsy of the mass was done, and it appears
as shown in the image below. The child also had a translocation, t(8;14). What is the probable
diagnosis?

A. Acute myeloid leukemia


B. Follicular lymphoma
C. Multiple myeloma
D. Burkitt lymphoma
----------------------------------------
34. An elderly male went for a medical examination. Laboratory investigations show a very high TLC.
The peripheral smear of the patient is given below. What is the appropriate next step for this patient?
A. Stem cell transplantation
B. Bone marrow biopsy
C. Flow cytometry
D. Start high-dose chemotherapy
----------------------------------------

Correct Answers
Question Correct Answer

Question 1 3
Question 2 1
Question 3 2
Question 4 2
Question 5 2

Page 11

1448
Question 6 3
Question 7 3
Question 8 1
Question 9 1
Question 10 1
Question 11 2
Question 12 4
Question 13 2
Question 14 2
Question 15 1
Question 16 1
Question 17 3
Question 18 4
Question 19 3
Question 20 3
Question 21 3
Question 22 3
Question 23 1
Question 24 2
Question 25 4
Question 26 4
Question 27 4
Question 28 3
Question 29 1
Question 30 3
Question 31 3
Question 32 1
Question 33 4
Question 34 3

Solution for Question 1:


• Acute Myeloid Leukemia (AML) is a type of cancer that affects the bone marrow and blood. It is
characterized by the rapid growth of abnormal white blood cells, which can interfere with the production
of normal blood cells, leading to anemia, bleeding, and infections.
• Based on the given information, the most likely diagnosis is Acute Myeloid Leukemia M5 (AML M5),
also known as acute monocytic leukemia. This is suggested by the presence of bleeding gums, gingival
hyperplasia, hepatosplenomegaly, and bone marrow biopsy findings, which indicate 100% cellularity

Page 12

1449
with many blast cells that are peroxidase negative and non-specific esterase positive.
Incorrect choices:
• Option a. Acute Myeloid Leukemia M2 (AML M2): This subtype of AML accounts for approximately
25% of cases and is characterized by the presence of abnormal promyelocytes in the bone marrow and
blood.
• Option b. Acute Myeloid Leukemia M0 (AML M0): This subtype of AML is relatively rare and accounts
for less than 5% of cases. It is characterized by the absence of surface markers on the blast cells,
which makes it difficult to diagnose. However, these cells can be identified by their morphology and
genetic abnormalities. This subtype is also known as minimally differentiated AML.
• Option c. Acute Myeloid Leukemia M7 (AML M7): This subtype of AML is relatively rare and accounts
for 1% of cases. It is characterized by the presence of abnormal megakaryocytes in the bone marrow,
which can lead to thrombocytopenia (low platelet count) and bleeding.

Solution for Question 2:


• OPTION A -In Multiple Myeloma, bone marrow shows hypercellularity due to the presence of
myeloma plasma cells that are more than 30%. Myeloma plasma cells are large oval-shaped cells
having abundant pale blue cytoplasm with round to oval nuclei eccentrically placed with perinuclear
clearing. Nuclear chromatin has a characteristic spoke wheel appearance.
• Plasma bodies can be seen in the image.
• these plasma cells produce proteins that are known as antibodies.
Incorrect Choices:
• Option b. CLL: Chronic Lymphocytic leukemia is a tumor composed of small B monomorphic
lymphocytes. The age at presentation and gender predominance are similar to multiple myeloma
(50-60 years; males); however, patients with CLL usually present with fatigue, anorexia, weight loss,
and generalized lymphadenopathy. Peripheral smear shows lymphocytosis with > 50% of the white
cells. Mature lymphocytes are seen with scanty cytoplasm, round nuclei with soccer ball type of
chromatin, and absent nucleoli. Bone marrow shows hypercellularity due to mature lymphocyte
infiltration.
• The age at presentation and gender predominance are similar to multiple myeloma (50-60 years;
males); however, patients with CLL usually present with fatigue, anorexia, weight loss, and generalized
lymphadenopathy.
• Peripheral smear shows lymphocytosis with > 50% of the white cells.
• Mature lymphocytes are seen with scanty cytoplasm, round nuclei with soccer ball type of chromatin,
and absent nucleoli.
• Bone marrow shows hypercellularity due to mature lymphocyte infiltration.
• The age at presentation and gender predominance are similar to multiple myeloma (50-60 years;
males); however, patients with CLL usually present with fatigue, anorexia, weight loss, and generalized
lymphadenopathy.
• Peripheral smear shows lymphocytosis with > 50% of the white cells.
• Mature lymphocytes are seen with scanty cytoplasm, round nuclei with soccer ball type of chromatin,
and absent nucleoli.

Page 13

1450
• Bone marrow shows hypercellularity due to mature lymphocyte infiltration.

• Option c. ALL: Acute lymphoblastic leukemia is a group of neoplasm that contains lymphoblasts.
There are two varieties- B lymphoblastic (85%) and T lymphoblastic leukemia (15%). It is the most
common hematological malignancy occurring in children between 1-5 years of age. Symptoms of bone
marrow failure (anemia, neutropenia, and thrombocytopenia) are seen. Peripheral smear shows a
marked increase in WBC count with >20% lymphoblasts, and thrombocytopenia is observed.
Lymphoblasts are larger than lymphocytes, with a high N: C ratio, a nucleus with condensed chromatin,
small or absent nucleoli, and scanty basophilic agranular cytoplasm. Cytochemistry of lymphoblasts
shows PAS-positive, MPO-negative, and Sudan black B-negative.
• It is the most common hematological malignancy occurring in children between 1-5 years of age.
• Symptoms of bone marrow failure (anemia, neutropenia, and thrombocytopenia) are seen.
• Peripheral smear shows a marked increase in WBC count with >20% lymphoblasts, and
thrombocytopenia is observed.
• Lymphoblasts are larger than lymphocytes, with a high N: C ratio, a nucleus with condensed
chromatin, small or absent nucleoli, and scanty basophilic agranular cytoplasm.
• Cytochemistry of lymphoblasts shows PAS-positive, MPO-negative, and Sudan black B-negative.
• It is the most common hematological malignancy occurring in children between 1-5 years of age.
• Symptoms of bone marrow failure (anemia, neutropenia, and thrombocytopenia) are seen.
• Peripheral smear shows a marked increase in WBC count with >20% lymphoblasts, and
thrombocytopenia is observed.
• Lymphoblasts are larger than lymphocytes, with a high N: C ratio, a nucleus with condensed
chromatin, small or absent nucleoli, and scanty basophilic agranular cytoplasm.
• Cytochemistry of lymphoblasts shows PAS-positive, MPO-negative, and Sudan black B-negative.

Page 14

1451
• Option d. CML: Chronic myelogenous leukemia is a myeloproliferative neoplasm of pluripotent
hematopoietic stem cells with excess production of myeloid series resulting in leukocytosis and marked
splenomegaly. The Philadelphia chromosome is present in > 90% of cases. It is a shortened
chromosome 22 due to the reciprocal translocation between chromosome 9 and 22-t. Patients with
CML present with non-specific symptoms like weakness, fatigue, weight loss, and moderate to severe
splenomegaly. On peripheral smear, marked leukocytosis with WBC count more than 1,00,000/cu mm,
shift to the left, basophilia, eosinophilia, thrombocytosis. Bone marrow shows hypercellularity, and
reduced erythropoiesis, with normal or increased megakaryocytes
• The Philadelphia chromosome is present in > 90% of cases. It is a shortened chromosome 22 due to
the reciprocal translocation between chromosome 9 and 22-t.
• Patients with CML present with non-specific symptoms like weakness, fatigue, weight loss, and
moderate to severe splenomegaly.
• On peripheral smear, marked leukocytosis with WBC count more than 1,00,000/cu mm, shift to the
left, basophilia, eosinophilia, thrombocytosis.
• Bone marrow shows hypercellularity, and reduced erythropoiesis, with normal or increased
megakaryocytes
• The Philadelphia chromosome is present in > 90% of cases. It is a shortened chromosome 22 due to
the reciprocal translocation between chromosome 9 and 22-t.
• Patients with CML present with non-specific symptoms like weakness, fatigue, weight loss, and
moderate to severe splenomegaly.
• On peripheral smear, marked leukocytosis with WBC count more than 1,00,000/cu mm, shift to the
left, basophilia, eosinophilia, thrombocytosis.
• Bone marrow shows hypercellularity, and reduced erythropoiesis, with normal or increased
megakaryocytes

Page 15

1452
Solution for Question 3:
• The given microscopic picture is diagnostic of Hodgkin’s disease due to the presence of Reed
Sternberg cells.
• Classic Reed Sternberg cells are large, with bilobed nuclei appearing as a mirror image of each
other.
• Nucleus has a prominent, eosinophilic, inclusion-like nucleolus with a clear halo surrounding it, with
abundant cytoplasm.
• The age of presentation is around 20-30 years, with male predominance.
• They present with painless enlargement of lymph nodes over the neck, axilla, or groin, pel-Ebstein
type of fever, night sweats, and weight loss.
Incorrect Choices:
• Option a. Acute myeloid leukemia: In AML, there is anemia, thrombocytopenia, and marked elevation
of WBC count. The majority of these WBCs are blasts. Bone marrow examination shows hypercellular
marrow with 20% myeloblasts and reduced megakaryocytes. It is seen in adults around 50 years of
age. They present with pallor, shortness of breath, bleeding gums, petechiae, and respiratory
infections.
• Option c. Human papillomavirus (HPV) is a common sexually transmitted infection that affects both
men and women. There are over 100 different types of HPV, and some strains can cause genital warts.
In contrast, others can lead to various types of cancer, including cervical cancer, anal cancer, and
oropharyngeal cancer.
• Option d. Chronic lymphocytic leukemia: Chronic lymphocytic leukemia (CLL) is a type of cancer that
affects the blood and bone marrow. It is a slow-growing cancer that develops in the white blood cells,
specifically the lymphocytes, which are important cells in the immune system that help fight infections.

Page 16

1453
Solution for Question 4:
• The given image shows Reed Sternberg cells, which are neoplastic cells that represent the diagnostic
feature of Hodgkin’s disease.
• Hodgkin lymphoma is a malignant tumor of the lymphatic system. There are several variants of Reed
Sternberg cells.
• Class RS cell is binucleated with owl-eyed nuclei having a mirror image appearance.
• Patients with Hodgkin lymphoma present with painless enlargement of lymph nodes present in the
neck, axilla, and groin, night sweats, low-grade fever, unexplained weight loss, and fatigue.
Incorrect Choices:
• Option a. Faggot cells: These cells are found in acute promyelocytic leukemia in a hyper granular
form. The cytoplasm contains multiple Auer rods, which resemble a bundle of sticks.

• Option c. Mott cells: These cells are typically seen in multiple myeloma. They have intracytoplasmic
inclusions which contain antibodies. They can be visualized using toluidine blue or wright-Giemsa
stains. If Mott cells are present in the bone marrow, it is a diagnostic feature of multiple myeloma.

Page 17

1454
• Option d. Sézary-Lutzner cells: These cells are also called Sézary cells, and their presence in skin or
blood is a diagnostic feature of mycosis fungoides (cutaneous T cell lymphoma). They have a
cerebriform nucleus and a large pale cytoplasm. They are characterized by generalized rash and lymph
node swelling.

Solution for Question 5:


Correct Option B - Chronic lymphocytic leukemia:
• CLL is a type of leukemia characterized by the accumulation of mature lymphocytes in the blood and
bone marrow. On a peripheral smear, CLL typically shows an increased number of small, mature
lymphocytes with a characteristic smudge appearance. The given peripheral smear image may show
an increased number of small lymphocytes along with smudge cells, which is consistent with CLL.
Incorrect Options:

Page 18

1455
Option A - Acute lymphoblastic leukemia (ALL): ALL is a
type of leukemia characterized by the proliferation of immature lymphoblasts. On a
peripheral smear, ALL typically shows a high number of blast cells with reduced numbers of mature cel
ls. In the given scenario, the image is not indicative of the high number of blast cells seen in ALL.
Option C - Acute myeloid leukemia (AML): AML is a type of leukemia characterized by the proliferation
of abnormal myeloid precursor cells, known as myeloblasts. On a peripheral smear, AML typically sho
ws an increased number of myeloblasts along with reduced numbers of mature cells. The given periph
eral smear image does not show an abundance of myeloblasts, making AML unlikely.
Option D - Chronic myelogenous leukemia (CML): CML is a type of leukemia characterized by the pres
ence of the Philadelphia chromosome, which results in the abnormal fusion of the BCR and ABL genes
. On a peripheral smear, CML typically shows an increased number of mature and immature granulocyt
es, including myelocytes and metamyelocytes. The given peripheral smear image does not exhibit the
abundance of granulocytes seen in CML.

Solution for Question 6:


Correct Option C: Flow cytometry from peripheral blood
• In the given scenario of an old man with a high WBC count of 65,000 and the presence of smudge
cells on peripheral blood smear, the next step in the evaluation would be to determine the underlying
cause of these findings. The most appropriate next step would be Flow cytometry from peripheral blood
Incorrect options:
Option A: Cytogenetic study: Cytogenetic studies involve analyzing the chromosomes of cells to detect
any chromosomal abnormalities. While this test may be useful in certain cases, it is not the initial step i
n evaluating a patient with a high WBC count and smudge cells.
Option B: Bone marrow biopsy: Bone marrow biopsy is a procedure to collect a sample of bone marro
w for examination. It is commonly performed in cases of unexplained cytopenias or suspected hematol
ogical malignancies. However, in the given scenario, the peripheral blood findings indicate a
need for further evaluation before proceeding to a bone marrow biopsy.
Option D: X-ray crystallography: X-ray crystallography is a
technique used to determine the atomic and molecular structure of a
crystal. It is not applicable in this context and is not a standard diagnostic tool for evaluating a
high WBC count and smudge cells.

Solution for Question 7:


Correct Option C - FISH and PCR:
• The patient's presentation with fatigue, massive splenomegaly, anemia, leukocytosis with increased
myeloid cells, and an elevated myeloid-to-erythroid ratio is suggestive of a myeloproliferative disorder.
• FISH (fluorescence in situ hybridization) and PCR (polymerase chain reaction) are molecular
techniques used to detect specific chromosomal abnormalities, such as the Philadelphia chromosome
[t(9;22)], which is characteristic of CML. These tests can provide definitive evidence for the presence of
the BCR-ABL1 fusion gene, confirming the diagnosis of CML.
Incorrect Options:

Page 19

1456
Option A - Immunophenotyping:
• Immunophenotyping is a technique that uses antibodies against specific cell surface markers to
identify and characterize different cell types.
• It is commonly used in the diagnosis of hematological malignancies.
Option B - LAP score:
• LAP (leukocyte alkaline phosphatase) score is a laboratory test used to assess the activity of the
alkaline phosphatase enzyme in neutrophils.
• It can help differentiate between reactive leukocytosis and chronic myeloproliferative disorders such
as CML.
• In CML, the LAP score is usually low or absent.
Option D - Flow cytometry:
• Flow cytometry is a powerful technique that allows the analysis of multiple characteristics of individual
cells in a heterogeneous population.
• It is commonly used to detect and characterize abnormal cell populations in hematological
malignancies.
• Flow cytometry can help identify aberrant antigen expression patterns, aiding in the diagnosis of
specific diseases.

Solution for Question 8:


Correct Option A - Leukemia:
• Benzene is a known occupational carcinogen that can increase the risk of developing various types of
cancer, with leukemia being the most strongly associated.
• Repeated exposure to benzene can lead to the development of hematopoietic malignancies,
particularly acute myeloid leukemia (AML) and acute lymphoblastic leukemia (ALL).
Incorrect Options:
Option B, C, and D
• Benzene exposure is not a risk factor for options B, C, and D.

Solution for Question 9:


Correct Option A - β-2 Microglobulin:
• Elevated levels of β2M are associated with more advanced disease, a higher tumor burden, and a
poorer prognosis in multiple myeloma.
Incorrect Options:
Options B, C, and D are not a poor prognostic marker of multiple myeloma.

Page 20

1457
Solution for Question 10:
Correct Option A - Follicular lymphoma:
• Follicular lymphoma, a type of non-Hodgkin lymphoma, is characterized by the presence of atypical
cells within the lymph nodes, resulting in the effacement of the normal lymph node architecture.
• The cells typically have an indented nucleus and prominent nucleolus and express certain markers
such as CD10 and BCL2.
• It often presents with painless lymphadenopathy and systemic symptoms like fatigue.
Incorrect Options:
Option B - Burkitt lymphoma:
• Burkitt lymphoma is a high-grade B-cell lymphoma characterized by rapidly dividing abnormal B-cells.
• It typically presents with rapidly growing tumors, often involving the abdomen or jaw.
• The lymph nodes may be involved, but it typically does not cause diffuse effacement of the lymph
node architecture
Option C - Mycosis fungoides:
• Mycosis fungoides is a type of cutaneous T-cell lymphoma that primarily affects the skin.
• It commonly presents with skin lesions, such as patches, plaques, or tumors, rather than cervical
lymphadenopathy.
Option D - Chronic myeloid leukemia:
• Chronic myeloid leukemia is characterized by the overproduction of mature and immature
granulocytes.
• The TLC and DLC were normal, which is not consistent with chronic myeloid leukemia.

Solution for Question 11:


• A characteristic finding on electron microscopy in Langerhans cell histiocytosis (LCH) is the presence
of Birbeck granules. Birbeck granules are rod-shaped, tennis racket-like structures that have a central
linear striation. They are considered to be specific ultrastructural markers for Langerhans cells, which
are the abnormal proliferating cells in LCH.
• Birbeck granules are thought to be derived from the endosomal compartment and are composed of a
protein called langerin (CD207). Langerin is a unique marker for Langerhans cells and is involved in
antigen presentation and immune response regulation.
Incorrect Choices:
• Option a. Histiocytes: Histiocytes are a broad term that encompasses various cell types, including
Langerhans cells. While Langerhans cells are a type of histiocyte involved in LCH, the presence of
histiocytes alone is not a characteristic finding on electron microscopy.
• Option c. Eosinophils: Eosinophils are a type of white blood cell involved in allergic reactions and
parasitic infections. They are not specific to Langerhans cell histiocytosis, and their presence would not
be a characteristic finding on electron microscopy in LCH.
• Option d. Giant cells: Giant cells are large, multinucleated cells that can be found in various
pathological conditions, including infections and certain granulomatous diseases. While giant cells can

Page 21

1458
be present in Langerhans cell histiocytosis, their presence alone is not a characteristic finding on
electron microscopy.

Solution for Question 12:


Correct Option D:
In the given scenario, the next investigation of choice for a child suspected to have chloroma (also kno
wn as granulocytic sarcoma or extramedullary myeloid tumor) is a bone marrow biopsy. Chloroma is a l
ocalized extramedullary tumor composed of immature myeloid cells, which can occur in patients with a
cute myeloid leukemia (AML) or myelodysplastic syndrome (MDS).
A bone marrow biopsy involves the extraction of a small sample of bone marrow tissue from the hipbon
e or sternum. This procedure allows for the examination of the cellular composition, morphology, and a
rchitecture of the bone marrow. In the case of suspected chloroma, a bone marrow biopsy helps confir
m the diagnosis by revealing the presence of abnormal myeloid cells infiltrating the bone marrow or pro
viding insights into the underlying hematologic malignancy.
Incorrect Option:
Option A. Platelet count: Platelet count is not the next investigation of choice for a child suspected to h
ave chloroma. While chloroma can be associated with abnormal platelet counts, determining the platel
et count alone would not provide a definitive diagnosis or further information regarding the underlying c
ondition. The peripheral smear mentioned in the question likely indicated the presence of abnormal cell
s but does not provide sufficient information to confirm the diagnosis or guide management.
Option B. WBC count: Similar to the platelet count, evaluating the white blood cell (WBC) count alone i
s not the next investigation of choice for suspected chloroma. While chloroma involves the proliferation
of myeloid cells, a complete evaluation of the bone marrow is necessary to confirm the diagnosis and d
etermine the extent of myeloid cell involvement. The WBC count may be elevated or altered in the peri
pheral blood, but it does not provide specific information about chloroma or its underlying cause.
Option C. Flow cytometry: Flow cytometry is a valuable tool for analyzing cell surface markers and char
acteristics of various cell populations. However, it may not be the initial investigation of choice in this p
articular scenario. Flow cytometry can be helpful in characterizing the immunophenotype of the abnorm
al cells observed in chloroma and can aid in the diagnosis and classification of hematologic malignanci
es. However, a bone marrow biopsy is typically performed first to confirm the presence of chloroma an
d assess the overall bone marrow status before utilizing flow cytometry for further characterization.

Solution for Question 13:


Correct Option B:
Massive splenomegaly refers to a significant spleen enlargement, usually extending well below the left
costal margin. It is commonly associated with various hematological disorders. However, aplastic anem
ia is not typically associated with massive splenomegaly.
Aplastic anemia is a bone marrow failure disorder characterized by reduced production of red blood cel
ls, white blood cells, and platelets. It is caused by destroying or suppressing hematopoietic stem cells i
n the bone marrow. While the bone marrow may be hypocellular (reduced cellularity), splenomegaly is

Page 22

1459
not characteristic of aplastic anemia. In fact, the spleen is usually not significantly enlarged or may eve
n be slightly reduced in size.
Incorrect Option:
Option A. Chronic myeloid leukemia (CML): CML is a myeloproliferative neoplasm characterized by the
presence of the Philadelphia chromosome, resulting in the overproduction of mature and immature my
eloid cells. Massive splenomegaly is a
common feature of CML due to infiltration of the spleen by abnormal myeloid cells.
Option C. Hairy cell leukemia: Hairy cell leukemia is a
rare B-cell lymphoproliferative disorder characterized by the presence of abnormal B
lymphocytes with hair-like projections. Massive splenomegaly is a hallmark feature of hairy cell leukemi
a, with the spleen often being several times larger than normal due to infiltration by the abnormal lymph
ocytes.
Option D. Lymphoma: Lymphomas are a group of cancers arising from lymphocytes, which are immun
e system cells. Depending on the type and stage of lymphoma, splenomegaly can be present, ranging
from moderate to massive enlargement. Lymphoma involvement in the spleen leads to the infiltration o
f abnormal lymphocytes or lymphomatous masses within the splenic tissue.

Solution for Question 14:


Correct Option: B
The likely component of the solution used in the autopsy to produce the observed color change in the h
eart is Triphenyl tetrazolium chloride.
Explanation of the options:
Option A: Formalin: Formalin is a
commonly used fixative in histopathology to preserve tissues. However, it does not cause a
color change in the heart tissue.
Option B: Triphenyl tetrazolium chloride: Triphenyl tetrazolium chloride (TTC) is a chemical that is used
to assess tissue viability. It is often used to differentiate viable tissue from infarcted tissue. TTC is redu
ced by dehydrogenase enzymes in metabolically active cells, resulting in the formation of a red formaz
an compound. Viable tissue appears red, while infarcted tissue appears pale or white. Therefore, the o
bserved color change in the heart suggests the presence of Triphenyl tetrazolium chloride in the solutio
n.
Option C: Ethanol: Ethanol is a
commonly used alcohol for tissue fixation and preservation. However, it does not cause a
color change in the heart tissue.
Option D: Glutaraldehyde: Glutaraldehyde is a
fixative commonly used for electron microscopy. It does not cause a
color change in the heart tissue similar to the one described in the scenario.
Therefore, based on the observed color change, the likely component of the solution used in the autop
sy is Triphenyl tetrazolium chloride, which is used to assess tissue viability and differentiates viable tiss
ue from infarcted tissue.

Page 23

1460
Solution for Question 15:
Correct Option: A
The best investigation for this patient with suspected multiple myeloma is:
Serum electrophoresis showing IgG
Explanation of each option:
Option A: Serum electrophoresis showing IgG: This test is used to detect abnormal levels of immunogl
obulins in the blood. In multiple myeloma, there is an overproduction of abnormal monoclonal immunog
lobulins, which can be detected through serum electrophoresis. It helps in confirming the diagnosis and
determining the type and quantity of abnormal immunoglobulins present.
Option B: Serum levels of CA 15.3: CA 15.3 is a tumor marker primarily used for monitoring breast can
cer. It is not specific to multiple myeloma and is not considered a diagnostic tool for this condition.
Option C: Whole body scan: While imaging studies such as whole-body scans may be useful in assess
ing the extent of multiple myeloma involvement, they are not typically the initial investigation for diagno
sis. Imaging modalities like skeletal survey (X-rays) or more sensitive techniques like MRI or PET/CT s
cans may be used to evaluate bone lesions in multiple myeloma.
Option D: CT head with contrast: A CT scan of the head with contrast is not necessary for the initial ev
aluation of multiple myeloma. In this patient, the presence of punched-out lytic lesions on skull radiogra
ph already suggests bone involvement, which is common in multiple myeloma.
Therefore, the most appropriate investigation in this case is serum electrophoresis showing IgG, as it h
elps confirm the diagnosis and assess the abnormal immunoglobulin levels associated with multiple my
eloma.

Solution for Question 16:


Correct Option: A
• Based on the information provided, the likely diagnosis of the patient is Multiple Myeloma.
• Multiple Myeloma is a malignancy characterized by the presence of abnormal plasma cells in the bone
marrow. These abnormal plasma cells produce monoclonal (identical) immunoglobulins or
immunoglobulin fragments, also known as monoclonal gammopathy. The image showing the presence
of monoclonal plasma cells in the bone marrow supports the diagnosis of multiple myeloma.
The other options listed are less likely based on the given information:
Option B: ALL (Acute Lymphoblastic Leukemia): ALL is a malignancy characterized by the proliferation
of immature lymphoid cells in the bone marrow. It typically affects children and young adults rather tha
n older individuals like the patient described.
Option C: Tuberculosis: Tuberculosis is a bacterial infection caused by Mycobacterium tuberculosis. It
primarily affects the lungs but can also involve other organs. The presence of monoclonal plasma cells
in the bone marrow is not consistent with the diagnosis of tuberculosis.
Option D: Sickle Cell Disease: Sickle Cell Disease is a genetic disorder characterized by abnormal he
moglobin leading to the deformation of red blood cells. It does not involve the proliferation of abnormal
plasma cells as seen in the bone marrow of the patient.

Page 24

1461
Solution for Question 17:
Correct Option: C
• The clinical scenario described, is most commonly associated with Acute Lymphoblastic Leukemia
(ALL).
• Most frequently occurs in children; less common in adults (worse prognosis).
• T-cell ALL can present as mediastinal mass (presenting as SVC-like syndrome). Associated with
Down syndrome.
• Peripheral blood and bone marrow have increased lymphoblasts.
• TdT+ (marker of pre-T and pre-B cells), CD10+ (marker of pre-B cells).
Most frequently occurs in children; less common in adults (worse prognosis).
T-cell ALL can present as mediastinal mass (presenting as SVC-like syndrome). Associated with Down
syndrome.
Peripheral blood and bone marrow have increased lymphoblasts.
TdT+ (marker of pre-T and pre-B cells), CD10+ (marker of pre-B cells).
Explanation for the options:
Option A: Acute Myeloblastic Leukemia (AML): AML typically presents with symptoms such as fatigue,
recurrent infections, bleeding tendencies, and bone pain. It is less commonly associated with hepatospl
enomegaly and sternal tenderness.
Option B: Chronic Lymphoblastic Leukemia (CLL): CLL usually occurs in older individuals and is chara
cterized by lymphocytosis, lymphadenopathy, and splenomegaly. It is less likely to present with hepato
megaly, sternal tenderness, and petechiae in a 10-year-old patient.
Option C: Acute Lymphoblastic Leukemia (ALL): ALL is the most common childhood leukemia. It often
presents with nonspecific symptoms such as fever, weight loss, fatigue, and bone pain. Hepatospleno
megaly, sternal tenderness, and petechiae can also be seen in ALL.
Option D: Chronic Myeloid Leukemia (CML): CML typically presents in adults and is characterized by a
n elevated white blood cell count, splenomegaly, and a characteristic genetic abnormality called the Ph
iladelphia chromosome. It is less common in children and may not present with the described symptom
s.
Based on the age of the patient, clinical presentation, and associated findings, Acute Lymphoblastic Le
ukemia (ALL) is the most likely diagnosis.

Solution for Question 18:


Correct Choice: D
Explanation:

Page 25

1462
It is an aggressive non-Hodgkin B-cell lymphoma. BL occurs in patients with Epstein-Barr virus (EBV) i
nfection, HIV as concomitant infection by EBV, and chromosomal translocations causing oncogene c-
myc overexpression. BL is classified into 3 clinical groups by the WHO: endemic, sporadic, and immun
odeficiency-related. These symptoms later two types of Burkitt lymphoma include:
• Painless swellings: The most common symptom is one or more painless swellings. They can grow
very quickly. These swellings are enlarged lymph nodes.
• Heavy sweating at night
• High temperatures come and go with no obvious cause
• Losing a lot of weight (more than one-tenth of your total weight)
Incorrect Choices:
Option A. Hodgkin’s disease: This is a cancer of the lymphatic system in which the immunity of the bod
y is compromised. In this, lymphocytes multiply without control which results in swollen LNs and growth
in the body. It can be treated with chemotherapy, RT, and stem-cell transplantation.The primary type o
f HL is painless. The characteristic feature is the presence of Reed-Sternberg cells which are large, ab
normal lymphocytes containing two nucleus (owl eye).
Option B. Acute lymphocytic leukemia: It is the malignancy of the blood and bone marrow. As it is acut
e in nature, this disease progresses very rapidly in which immature blood cells are produced. ALL is ca
used by stem cells DNA mutation. Symptoms include:
• pale skin
• tiredness
• breathlessness
• having repeated infections over a short space of time
• unusual and frequent bleeding
Option C. Chloroma: A rare type of cancer that is made up of myeloblasts (a type of immature white blo
od cell) and forms outside the bone marrow and blood. The tumor cells may look green when viewed u
nder a microscope. This most commonly affects people with acute myeloid leukemia or a myeloprolifer
ative disorder. It is also known as the extramedullary myeloid tumor, granulocytic sarcoma, and myeloi
d sarcoma. Chloroma most commonly occurs in the bones, lungs, CNS, skin, LNs, bowel, and soft tiss
ues in the head, neck, and breasts. The symptoms are very nonspecific like fatigue, weight loss, loss of
appetite, or pain.

Solution for Question 19:


Correct Choice: C
Explanation:
Multiple myeloma is a type of cancer that develops in a type of white blood cell known as a plasma cell.
Healthy plasma cells aid in the fight against infections by producing antibodies that recognize and dest
roy bacteria. Cancerous plasma cells grow in the bone marrow and drive out healthy blood cells in mult
iple myeloma. Instead of producing beneficial antibodies, cancer cells create aberrant proteins that can
lead to difficulties. Multiple myeloma symptoms and signs can vary, and early in the disease, there ma
y be none. When signs and symptoms do occur, they can include:
• Bone pain, especially in your spine or chest

Page 26

1463
• Nausea
• Constipation
• Loss of appetite
• Mental fogginess or confusion
• Fatigue
• Frequent infections
• Weight loss
• Weakness or numbness in your legs
• Excessive thirst
Incorrect Choices:
Option A. CML: Chronic myeloid leukaemia (CML) is a blood and bone marrow malignancy. CML incre
ases the quantity of white blood cells in the blood. CML is typically detected during its chronic phase, w
hen therapy is highly beneficial for the majority of patients. CML is also known as chronic myelogenous
leukaemia, chronic granulocytic leukaemia, and chronic myelocytic leukaemia. CML is one of the four
most common kinds of leukaemia. Chronic myeloid leukaemia (CML) patients may not exhibit any sym
ptoms at initially. Patients are frequently diagnosed with CML during a regular physical exam or blood t
est. CML signs and symptoms usually appear gradually. Those with symptoms often report experiencin
g:
• Weakness
• Fatigue
• Shortness of breath during basic everyday activities
• Fever
• Bone pain
• Unexplained weight loss
• Pain or a feeling of fullness below the ribs on the left side, due to an enlarged spleen
• Night sweats
Option B. CLL: Chronic lymphocytic leukemia (CLL) is a blood and bone marrow malignancy that norm
ally progresses slowly. CLL is one of the most frequent kinds of adult leukemia. It usually happens at or
after middle age, and it seldom happens in youngsters. Chronic lymphocytic leukemia is a kind of canc
er in which the bone marrow produces an abnormally large number of lymphocytes (a type of white blo
od cell). Red blood cells, white blood cells, and platelets can all be affected by leukemia. CLL does not
produce any symptoms at first and can be detected through a
normal blood test. Later, signs and symptoms may occur.
• Painless swelling of the lymph nodes in the neck, underarm, stomach, or groin.
• Weakness or feeling tired.
• Pain or a feeling of fullness below the ribs.
• Fever and infection.
• Easy bruising or bleeding.
• Petechiae (flat, pinpoint, dark-red spots under the skin caused by bleeding).
• Weight loss for no known reason.

Page 27

1464
• Drenching night sweats.
Option D. Hairy cell leukemia: Hairy cell leukemia is a
cancer of the white blood cells. The white blood cells help fight off germs. There are a few different typ
es of white blood cells. The white blood cells involved in hairy cell leukemia are called B cells. B
cells are also called B lymphocytes. In hairy cell leukemia, the body makes too many B
cells. The cells don't look like healthy B
cells. Instead, they've changed to become leukemia cells. The leukemia cells look "hairy" under a micr
oscope. Hairy cell leukemia might not cause symptoms. When it causes symptoms, hairy cell leukemia
might cause:
• A feeling of fullness in your belly that may make it uncomfortable to eat more than a little at a time
• Fatigue
• Easy bruising
• Recurring infections
• Weakness
• Losing weight without trying

Solution for Question 20:


Correct Choice: C
Explanation:
• A type of cancer of the blood and bone marrow is Acute lymphocytic leukemia (ALL). Acute
lymphoblastic leukemia (acute lymphocytic leukemia, ALL) is a malignant (clonal) disease of the bone
marrow in which early lymphoid precursors proliferate and replace the normal hematopoietic cells of the
marrow. ALL is the most common type of cancer and leukemia in children. Bone marrow aspirates
smear from a person with precursor B-cell ALL. The large purple cells are lymphoblasts present within
the bone marrow in persons affected with ALL. These lymphoblasts are the cancerous cells in ALL.
Normal lymphoblasts develop into mature, infection-fighting B-cells or T-cells, also called lymphocytes.
• Most frequently occurs in children; less common in adults (worse prognosis).
• T-cell ALL can present as a mediastinal mass (presenting as SVC-like syndrome).
• Associated with Down syndrome.
• Peripheral blood and bone marrow have increased lymphoblasts
Most frequently occurs in children; less common in adults (worse prognosis).
T-cell ALL can present as a mediastinal mass (presenting as SVC-like syndrome).
Associated with Down syndrome.
Peripheral blood and bone marrow have increased lymphoblasts

Incorrect Choices:
Option A. AML: Acute myeloid leukemia (AML) is a
cancer of the blood and bone marrow. It is the most common type of acute leukemia in adults. If a
person has AML then their bone marrow makes a large number of abnormal blood cells. Leukemia ma

Page 28

1465
y affect red blood cells, white blood cells, and platelets. In AML, the myeloid stem cells usually become
a type of immature white blood cell called myeloblasts (or myeloid blasts). The myeloblasts in AML are
abnormal and do not become healthy white blood cells. Sometimes in AML, too many stem cells beco
me abnormal red blood cells or platelets. These abnormal white blood cells, red blood cells, or platelets
are also called leukemia cells or blasts. Common symptoms are infection, anemia, or easy bleeding m
ay occur.
Option B. Aplastic anemia: When our bone marrow becomes incapable of making enough new blood c
ells it causes aplastic anemia. Aplastic anemia is a rare but serious blood condition. It can develop quic
kly or slowly, and it can be mild or serious. Many people with this condition present with minimal sympt
oms but with low blood counts. The most common symptoms include:
• Bleeding/bruising easily.
• Heavy menstrual bleeding.
• Dyspnea (shortness of breath).
• Fatigue.
• Frequent infections.
• There are two types of aplastic anemia:
• Inherited aplastic anemia occurs because of a random gene mutation.
• Acquired aplastic anemia occurs because of an immune system problem.
Option D. Juvenile myelomonocytic leukemia: Juvenile myelomonocytic leukemia (JMML) is a rare can
cer of the blood that affects young children. JMML happens when types of white blood cells called mon
ocytes and myelocytes do not mature normally. Bone marrow transplantation (hematopoietic stem cell t
ransplantation) is the only known successful treatment for JMML. Children may also be treated with ch
emotherapy before a bone marrow transplant.

Solution for Question 21:


Correct Option C: Drug toxicity
• The most common cause of agranulocytosis is drug toxicity.
• Agranulocytosis refers to a severe decrease in the number of granulocytes (a type of white blood cell)
in the blood.
• Various medications, such as certain antibiotics (e.g., penicillin, sulfonamides), antithyroid drugs (e.g.,
propylthiouracil, methimazole), anticonvulsants (e.g. Carbamazepine), and nonsteroidal
anti-inflammatory drugs (NSAIDs), can cause agranulocytosis as a rare adverse reaction.
Incorrect Options:
Option A: Marrow infiltration- While marrow infiltration (e.g., by cancer cells) can cause a decrease in t
he production of granulocytes and other blood cells, it is not the most common cause of agranulocytosi
s.
Option B: Autoimmune- Autoimmune disorders, such as autoimmune neutropenia, can lead to a decre
ase in the number of granulocytes. However, autoimmune causes are not the most common cause of a
granulocytosis.
Option D: Radiation exposure- Radiation exposure can affect bone marrow function and lead to a decr
ease in blood cell production, including granulocytes. However, radiation exposure is not the most com

Page 29

1466
mon cause of agranulocytosis.

Solution for Question 22:


Correct Option C: Lymphocyte predominant
• Mononuclear Reed-Sternberg (RS) cells are characteristic cells seen in the lymphocyte-rich subtype
of Hodgkin lymphoma.
Incorrect Options:
Option A: Mixed cellular type-Classical RS cells (classical Owl eye) is typically seen in this subtype.
Option B: Nodular sclerosis- Lacunar cells is typically seen in this subtype.
Option D: Classical Hodgkin lymphoma- CD 15, 30 and PAX 5.

Solution for Question 23:


Correct Option:
Option A: Iron deficiency anemia: Among the options provided, Iron deficiency anemia is the condition l
east likely to result in a dry bone marrow aspiration.
Incorrect Options:
Option B: Follicular lymphoma: Follicular lymphoma primarily involves lymph nodes and does not often
result in significant bone marrow infiltration. However, in advanced stages or aggressive transformation
, bone marrow involvement can occur, leading to a dry tap.
Option C: Multiple myeloma: In Multiple myeloma bone marrow aspiration more challenging and potenti
ally resulting in a dry tap.
Option D: Acute leukemia: It can also cause dry bone marrow taps.

Solution for Question 24:


Correct Option B:
Based on the given image, the correct identification of the marked cells is as follows:
Lymphocyte - 2: Lymphocytes are characterized by their large, round nucleus and minimal cytoplasm. I
n the given image, the cell marked as "2" matches the morphological features of a lymphocyte.
Monocyte - 4: Monocytes are larger immune cells with a kidney-shaped nucleus and abundant cytoplas
m. The cell marked as "4" in the image matches the characteristics of a monocyte.
Eosinophil - 1: Eosinophils are granulocytes characterized by their bilobed nucleus and distinct granule
s within the cytoplasm. The cell marked as "1" in the image exhibits the features of an eosinophil.
Basophil - 3: Basophils are another type of granulocyte with segmented nuclei and numerous granules
in the cytoplasm. The cell marked as "3" in the image shows the characteristics of a basophil.

Page 30

1467
Incorrect Options:
Option A: This option incorrectly identifies the lymphocyte as cell 1, eosinophil as cell 2, basophil as cel
l 3, and monocyte as cell 4.
Option C: This option incorrectly identifies the lymphocyte as cell 1, monocyte as cell 2, eosinophil as c
ell 3, and basophil as cell 4.
Option D: This option incorrectly identifies the lymphocyte as cell 2, monocyte as cell 1, eosinophil as c
ell 3, and basophil as cell 4

Solution for Question 25:


Correct Option D:
The t(11;14) translocation is characteristic of mantle cell lymphoma (MCL), a type of B-cell non-Hodgki
n lymphoma. To confirm the diagnosis of MCL, specific markers are evaluated.
Sox11 Sox11 is a
transcription factor that is highly expressed in mantle cell lymphoma. It is considered a
specific marker for MCL and is commonly used in its diagnosis. Therefore, this option is correct.
Cyclin D1 Cyclin D1 is another marker used in the diagnosis of mantle cell lymphoma. The t(11;14) tra
nslocation results in overexpression of the Cyclin D1 protein, which plays a
crucial role in the pathogenesis of MCL. Therefore, this option is correct.
CD10 CD10, also known as CALLA, is a cell surface marker that is commonly used in the diagnosis of
other types of B-cell lymphomas, such as follicular lymphoma and Burkitt lymphoma. However, it is not
specific to mantle cell lymphoma. Therefore, this option is incorrect.
CD200 CD200 is an immunoregulatory protein that is often evaluated in the diagnosis of certain hemat
ological malignancies. It is not specific to mantle cell lymphoma and is not commonly used as a
diagnostic marker for this condition. Therefore, this option is incorrect.
Incorrect Options:
Option A : A,B is incorrect choice
Option B :B, Dis incorrect choice
Option C : A,C is incorrect choice

Solution for Question 26:


Correct Option D:
• Nodular lymphocyte–predominant Hodgkin lymphoma (NLPHL) is a subtype of Hodgkin lymphoma
(HL) that has distinct characteristics.
• Poor prognosis compared to classical HL: This statement is not true. NLPHL is generally associated
with a better prognosis compared to classical HL. It tends to have a more indolent clinical course and a
lower rate of relapse. However, it is important to note that each individual case may vary, and the
prognosis can be influenced by several factors.
Incorrect Options:

Page 31

1468
Option A: EBV negative: It is true that NLPHL is typically negative for Epstein-Barr virus (EBV) infection
. Unlike classical Hodgkin lymphoma (cHL), which is often associated with EBV infection, NLPHL is ge
nerally EBV negative.
Option B: CD30 negative: This statement is not true. NLPHL is typically positive for CD30, which is a m
arker commonly expressed on the surface of Reed-Sternberg cells in cHL. However, in NLPHL, the ch
aracteristic neoplastic cells are referred to as lymphocytic and histiocytic (L&H;) cells rather than Reed-
Sternberg cells.
Option C: CD20 positive: This statement is true. NLPHL is typically positive for CD20, which is a
B-cell marker. CD20 positivity is a characteristic feature of NLPHL and helps distinguish it from cHL, w
here the neoplastic cells are usually CD20 negative.

Solution for Question 27:


Correct Option D: Burkitt lymphoma
• The histopathological picture is of a Classical Burkitt Lymphoma involving a lymph node. The most
characteristic finding is a “starry sky” pattern.
• At high-power magnification, the neoplastic cells are of intermediate size, similar to the size of benign
histiocyte nuclei.
• The neoplastic cells have round nuclear contours, multiple nucleoli, frequent mitoses, and basophilic
cytoplasm. Macrophages with engulfed pyknotic nuclei and nuclear debris are also present.
Incorrect options:
Option A: Kikuchi disease is a Subacute necrotizing regional lymphadenopathy. The patients are youn
g adults, predominantly women, most commonly in Japan and neighboring Asian countries. The lymph
adenopathy is painless and associated with moderate fever, chills, myalgia, sore throat, and skin rashe
s. Histopathology shows Patches of necrosis, follicles with reactive germinal centers &
areas of histiocyte proliferation
Option B: Infectious Mononucleosis Lymphadenitis is Acute lymphadenitis caused by infection with Eps
tein-Barr virus (EBV). The characteristic triad of symptoms includes fever, pharyngitis, and cervical or g
eneralized lymphadenopathy. Histologically, the lymph node architecture is regionally distorted. The pa
racortical hyperplasia imparts a mottled (“moth-eaten”) pattern.
Option C: Hodgkin lymphoma (HL) is characterized by a heterogeneous cellularity comprising a
minority of specific neoplastic cells—the Hodgkin cells and the Reed-Sternberg cells—and a
majority of reactive non-neoplastic cells.

Solution for Question 28:


Correct Option C: Primary myelofibrosis
• The above features are suggestive of primary myelofibrosis. Splenomegaly is a well-established
clinical feature of MF with 85% or more of MF patients presenting with palpable splenomegaly at the
time of diagnosis. Characteristic laboratory findings in MF may include peripheral blood
leukoerythroblastosis, dacryocytosis, and teardrop-shaped red blood cells. Bone marrow biopsy shows

Page 32

1469
marked marrow fibrosis with dilated sinusoids containing megakaryocyte clusters.
Incorrect Options:
Option A: Polycythemia vera (PV) presents as an expansion of the red blood cell mass. An Absolute er
ythrocytosis is a feature that separates PV from other Myeloproliferative neoplasms.
Option B: Essential thrombocythemia is a bone marrow proliferation characterized primarily by an elev
ation in peripheral blood platelets, usually over 1,000 × 109/L.
Option D: Chronic myeloid leukemia is primarily a proliferation of granulocytic cells. The peripheral bloo
d findings are those of leukocytosis with granulocytes at all stages of maturation.

Solution for Question 29:


Correct Option A: Acute myeloid leukemia
• The above peripheral smear picture shows typical myeloblasts with delicate nuclear chromatin, three
to five nucleoli, fine azurophilic cytoplasmic granules & Auer rods. symptoms with complaints related to
anemia, neutropenia, and thrombocytopenia, most notably fatigue, fever, and spontaneous mucosal
and cutaneous bleeding along with CD33+ myeloid blasts more than 20% indicates the patient is
suffering from Acute myeloid leukemia.
Incorrect Options:
Option B: in Acute lymphoblastic leukemia, the marrow is hypercellular and packed with lymphoblasts.
In both B- and T-ALL, the tumor cells have scant basophilic cytoplasm and nuclei with delicate, finely st
ippled chromatin and small nucleoli.in contrast to myeloblasts are CD33-ve, myeloperoxidase-negative,
and often contain periodic acid-Schiff–positive glycogen granules.
Option C: Chronic Lymphocytic Leukemia/Small Lymphocytic Lymphoma is an indolent, slowly growing
tumor. The foci of mitotically active cells called proliferation centers, are pathognomonic for CLL/SLL. I
n most patients, there is an absolute lymphocytosis. These circulating cells are fragile and during the pr
eparation of smears many are disrupted, producing characteristic smudge cells. They are CD33-ve, C
D20+, and CD5+.
Option D: The term myelodysplastic syndrome (MDS) refers to a group of clonal stem cell disorders ch
aracterized by maturation defects that are associated with ineffective hematopoiesis and a high risk of t
ransformation to AML.the marrow usually is hypercellular or normocellular, but the peripheral blood sho
ws one or more cytopenias.common abnormalities include megaloblastoid erythroid precursors,ring sid
eroblasts,granulocyte precursors with abnormal granules.

Solution for Question 30:


Correct Option C: B cells
• The cell shown above is a tumor giant cell, the Reed-Sternberg (RS) cell.
• The sine qua non of Hodgkin lymphoma is the Reed-Sternberg (RS) cell.
• Hodgkin lymphoma is a neoplasm that arises from germinal center B cells.
Incorrect Options:

Page 33

1470
Option A: NK (Natural Killer)cells are innate immune cells.NK cells are lymphocytes that arise from the
same common lymphoid progenitor that gives rise to T lymphocytes and B lymphocytes.
Option B: CD4 and CD8 are expressed on distinct T-cell subsets and act as coreceptors during T-cell a
ctivation.CD4 is expressed on 50% to 60% of T cells, whereas CD8 is expressed on about 40% of T
cells. The CD4- and CD8-expressing T cells—called CD4+ and CD8+ cells, respectively
Option D: CD4 and CD8 are expressed on distinct T-cell subsets and act as coreceptors during T-cell a
ctivation.CD4 is expressed on 50% to 60% of T cells, whereas CD8 is expressed on about 40% of T
cells. The CD4- and CD8-expressing T cells—called CD4+ and CD8+ cells, respectively

Solution for Question 31:


Correct Option C - Mitochondrial DNA is seen:
• Activated neutrophils create neutrophil extracellular trapping (NET), which are web-like structures
made of chromatin and antimicrobial enzymes in response to bacterial, viral, and fungal infections.
• These structures support host defences by trapping and eliminating germs.
• In NET, mitochondrial DNA is rarely found.
Incorrect Options:
Option A - It is detected in blood during sepsis: NET is seen in the blood during sepsis and in other infl
ammatory and autoimmune disorders.
Option B - It is produced in response to bacterial infection: NET is produced in response to bacterial, vi
ral, and fungal infection in the body as well as other inflammatory stimuli.
Option D - It is chromatin with antibacterial enzymes: NET is a
process in which neutrophils release their chromatin and associated antimicrobial proteins to form a
web-like structure that can trap and kill microbes.

Solution for Question 32:


Correct Option A - t(9;22):
• The patient's presentation with massive splenomegaly and a peripheral smear showing leukocytesis
with increased myelocytes, metamyelocytes, and basophils is suggestive of chronic myleoid leukemia.
• The translocation commonly seen in chronic myleoid leukemia is t(9;22), also known as the
Philadelphia chromosome.
Incorrect Options:
Option B - t(8;21): t(8;21) = RUNX1-RUNX1T1 fusion is seen in AML M2 - AML with maturation.
Option C - t(15;17): t(15;17) = PML-RARA fusion is seen in AML M3 - Acute promyelocytic leukemia.
Option D - t(8;14): t(8;14) is seen in ALL and Burkitt lymphoma.

Page 34

1471
Solution for Question 33:
Correct Option D - Burkitt lymphoma:
• An African child with jaw swelling and a translocation, t(8;14) with a biopsy showing a starry sky
appearance is a feature of Burkitt lymphoma. Sky is the tumor cells & stars are tingible body
macrophages.
Incorrect Options:
Options A, B, and C are incorrect. Refer to the explanation of the correct answer.

Solution for Question 34:


Correct Option C - Flow cytometry:
• An elderly patient with a very high TLC and a peripheral smear showing smudge cells is suggestive of
chronic lymphoid leukemia.
• In chronic lymphoid leukemia, CD 5+ and CD 23+: To check this flow cytometry is used.

Smudge Cells
Incorrect Options:
Option A - Stem cell transplantation: Stem cell transplantation is not a
treatment option for chronic lymphocytic leukemia.
Option B - Bone marrow biopsy: Bone marrow biopsy is one of the diagnostic criteria for chronic lymph
ocytic leukemia, but flow cytometry should be done before that.
Option D - Start high-dose chemotherapy: High-dose chemotherapy should not be started before doing
flow cytometry.

Page 35

1472

You might also like